Você está na página 1de 1040

University of the Cordilleras College of Law First Year C S.Y.

2013 - 2014

Persons and Family Relations Cases

Block C First Years S.Y. 2013 - 2014 University of the Cordilleras College of Law

This compilation is a collaboration work of the first year law students of the University of the Cordilleras.

Collaborators: Artates, Derick Jackson Bandoy, Mary Grace Bangasan, Roxanne C. Bautista, Eugenice Ivy Gwyn U. Bawagan, Ma. Danica Kaye Bocyag, Cathy Budaden, June Mae R. Caampued, John Matley M. Candelaria, Michelle Dulce M. Castro April Jacob P. Ci-O, Purita A. Cungihan, LV Katrina B. Dote, Jillian Jen Esnara, Jonathan Estudillo, Alemar Roli E. Gansoen, Gretchen B. Garcia, Janine R. Labiaga, Jalimgi Lai, Lovely Angel Mendoza, Ma. Teresa A. Nepomuceno, Isaiah M. Puguon, Mark Reuel L. Ramirez, Francisco Ignacio V. Reyes, Jona B. Rodriguera, Eduard Angelo Q. Tiwaken, Romeo Jr. K Treptor, Ronic Albert D. Ventura, Maria Teresita R. Vitug, Jane Erika

University of the Cordilleras College of Law First Year C S.Y. 2013 - 2014

Table of Contents
I. INTRODUCTION ......................................................................................................................1 A. CONSTITUTIONAL PROVISIONS B. PRELIMINARY TITLE TAADA VS. TUVERA 136 SCRA 27 CASE DIGEST PHIL. ASSOCIATION OF SERVICE EXPORTERS, INC. VS. TORRES 212 SCRA 299 CASE DIGEST DE JESUS VS. COMMISSION ON AUDIT, 249 SCRA 152 CASE DIGEST PVB EMPLOYEES UNION VS. VEGA, 360 SCRA 33 CASE DIGEST DE ROY VS. COURT OF APPEALS, 157 SCRA 575 CASE DIGEST UNCIANO PARAMEDICAL COLLEGE, INC. VS. COURT OF APPEALS, 221 SCRA 285 CASE DIGEST ALONZO VS. PADUA, 150 SCRA 259 CASE DIGEST PEOPLE VS. JABINAL, 55 SCRA 607 CASE DIGEST CUI VS. ARELLANO UNIVERSITY, 2 SCRA 205 CASE DIGEST ASAALI VS. COMMISSIONER OF CUSTOMS 26 SCRA 205 CASE DIGEST VAN DORN VS. ROMILLO, 139 SCRA 139 CASE DIGEST PILAPIL VS. IBAY-SOMERA, 174 SCRA 653 CASE DIGEST LLORENTE VS COURT OF APPEALS, 345 SCRA 592 CASE DIGEST ROEHR VS RODRIGUEZ, 404 SCRA 495 CASE DIGEST AZNAR VS. GARCIA, 7 SCRA 95 CASE DIGEST BELLIS VS. BELLIS, 20 SCRA 358 CASE DIGEST TESTATE ESTATE OF BOHONAN VS. BOHONAN, 106 PHIL 997 CASE DIGEST 2 3 3 8 9 14 15 18 19 22 23 25 26 31 32 36 37 40 41 44 45 49 50 53 54 59 60 66 67 72 73 80 82 85 86 89

II. HUMAN RELATIONS ........................................................................................................... 90 ALBENSON VS. COURT OF APPEALS 217 SCRA 16 CASE DIGEST CUSTODIO VS COURT OF APPEALS 2533 SCRA 483 CASE DIGEST BARONS MARKETING CORP. VS COURT OF APPEALS 286 SCRA 96 CASE DIGEST i 91 97 98 102 104 109

Table of Contents
GLOBE MACKAY VS COURT OF APPEALS 176 SCRA 778 CASE DIGEST RCPI VS DIONELA 143 SCRA 657 CASE DIGEST UE VS JADER 325 SCRA 804 CASE DIGEST RAMA VS COURT OF APPEALS 148 SCRA 496 CASE DIGEST PE VS PE 5 SCRA 200 CASE DIGEST HERMOSISIMA VS COURT OF APPEALS 109 PHIL 629 CASE DIGEST ESTOPA VS PIANSAY 109 PHIL 640 WASSMER VS VELEZ 10 PHIL 1440 CASE DIGEST CONSTANTINO VS MENDEZ 209 SCRA 18 CASE DIGEST GASHEM-SHOOKAT VS COURT OF APPEALS 219 SCRA 115 CASE DIGEST 110 116 117 1200 121 125 126 130 131 133 134 137 138 139 141 142 145 146 154

III. INDEPENDENT CIVIL ACTION ....................................................................................... 155 RUIZ VS UCOL 153 SCRA 16 PADILLA VS COURT OF APPEALS 129 SCRA 558 DULAY VS COURT OF APPEALS 243 SCRA 220 156 160 167

IV. PREJUDICIAL QUESTIONS.............................................................................................. 173 QUIAMBAO VS OSORIO 158 SCRA 674 BOBIS VS BOBIS 336 SCRA 747 BELTRAN VS PEOPLE 334 SCRA 106 TE VS COURT OF APPEALS 346 SCRA 327 V. 174 177 180 183

CIVIL PERSONALITY ........................................................................................................ 188 189 194 196 198 199

LIMJOCO VS INTERSTATE ESTATE OF FRAGANTE 80 PHIL 776 GELUZ VS COURT OF APPEALS 2 SCRA 132 QUIMIGING VS. ICAO, 34 SCRA 132 CASE DIGEST JOAQUIN VS NAVARRO 93 SCRA 257

VI. MARRIAGE ........................................................................................................................ 205 DE LORIA VS FELIX 104 PHIL 1 CASE DIGEST PEOPLE VS MENDOZA 95 PHIL 845 CASE DIGEST ALVADO VS CITY OF TACLOBAN 139 SCRA 230 CASE DIGEST TRINIDAD VS COURT OF APPEALS 289 SCRA 188 ii 206 209 210 212 213 216 217

Table of Contents
CASE DIGEST WEIGEL VS SEMPIO-DY 143 SCRA 499 CASE DIGEST APIAG VS CANTERO 268 SCRA 198 CASE DIGEST BALOGBOG VS COURT OF APPEALS 269 SCRA 259 CASE DIGEST TOLENTINO VS PARAS 122 SCRA 525 CASE DIGEST NAVARRO VS DOMAGTOY 259 SCRA 129 CASE DIGEST ARAES VS OCCIANO 380 SCRA 402 CASE DIGEST BESO VS JUDGE DAGUMAN 323 SCRA 566 CASE DIGEST COSCA VS PALAYPAYON JR. 237 SCRA 249 CASE DIGEST REPUBLIC VS COURT OF APPEALS 236 SCRA 257 CASE DIGEST QUITA VS COURT OF APPEALS 300 SCRA 406 CASE DIGEST GARCIA VS RECIO 366 SCRA 437 CASE DIGEST MORENO VS MORENO 246 SCRA 120 CASE DIGEST PEOPLE VS BORROMEO 133 SCRA 106 CASE DIGEST SEGUISABAL VS CABRERA 106 SCRA 67 CASE DIGEST NIAL VS BAYADOG 328 SCRA 122 CASE DIGEST BORJA-MANZANO VS SANCHEZ 354 SCRA 1 CASE DIGEST YAP VS YAP 145 SCRA 299 CASE DIGEST SANTOS VS COURT OF APPEALS 240 SCRA 20Y CASE DIGEST CHI MING TSOI VS COURT OF APPEALS 266 SCRA 324 CASE DIGEST REPUBLIC VS MOLINA 268 SCRA 198 CASE DIGEST HERNANDEZ VS COURT OF APPEALS 320 SCRA 76 CASE DIGEST MARCOS VS MARCOS 343 SCRA 755 CASE DIGEST ZAMORA VS COURT OF APPEALS 515 SCRA 19 CASE DIGEST SERMONIA VS COURT OF APPEALS 233 SCRA 15 CASE DIGEST BIENVENIDO VS COURT OF APPEALS 237 SCRA 676 CASE DIGEST iii 228 229 230 231 238 239 243 244 246 247 250 252 255 256 261 262 274 275 278 279 282 283 289 290 293 294 307 308 310 311 315 316 318 319 323 324 339 340 345 346 363 364 370 371 376 377 381 382 385 386 390

Table of Contents
TERRE VS TERRE 211 SCRA 6 CASE DIGEST BUCCAT VS BUCCAT 72 PHIL 21 CASE DIGEST AQUINO VS DELIZO 109 PHIL 21 CASE DIGEST ANAYA VS PALAROAN 36 SCRA 97 CASE DIGEST REPUBLIC VS NOLASCO 220 SCRA 20 CASE DIGEST TUASON VS COURT OF APPEALS 256 SCRA 15 CASE DIGEST ATIENZA VS BRILLANTE JR. 243 SCRA 32 CASE DIGEST DOMINGO VS COURT OF APPEALS 226 SCRA 572 CASE DIGEST BELTRAN VS PEOPLE 334 SCRA 106 CASE DIGEST MERCADO VS TAN 337 SCRA 122 CASE DIGEST CARATING-SIAYNGCO VS SIAYNGCO 441 SCRA 422 CASE DIGEST SIN VS SIN 355 SCRA 28 CASE DIGEST CHOA VS CHOA 392 SCRA 198 CASE DIGEST REPUBLIC VS IYOY 470 SCRA 508 CASE DIGEST PESCA VS PESCA 356 SCRA 588 CASE DIGEST FERRARIS VS FERRARIS 495 SCRA 396 DEDEL VS COURT OF APPEALS 421 SCRA 461 CASE DIGEST TONGOL VS TONGOL 537 SCRA 135 CASE DIGEST ANTONIO VS REYES 484 SCRA 353 CASE DIGEST VILLANUEVA VS COURT OF APPEALS 505 SCRA 564 CASE DIGEST NGO-TE VS YU-TE G.R. NO. 161 SCRA 793 CASE DIGEST 392 395 396 397 398 401 402 405 406 411 412 416 417 419 420 429 430 433 434 441 442 449 450 453 454 462 463 471 472 475 476 480 483 484 491 492 504 506 510 511 528

VII. LEGAL SEPERATION ...................................................................................................... 530 GOITIA VS CAMPOS RUEDA35 PHIL 252 CASE DIGEST ARROYO VS VASQUEZ DE ARROYO 42 PHIL 54 CASE DIGEST PEOPLE VS SCHNECKENBURGER, ET AL. 73 SCRA 413 CASE DIGEST LAPERAL VS REPUBLIC 6 SCRA 357 iv 531 536 537 540 541 543 544

Table of Contents
CASE DIGEST TENCHAVEZ VS ESCANO 15 SCRA 355 CASE DIGEST MACADANGDANG VS COURT OF APPEALS 108 SCRA 314 CASE DIGEST LAPUZ-SY VS EUFEMIO 43 SCRA 177 CASE DIGEST PCETE VS CARRIAGO 231 SCRA 321 CASE DIGEST ALCANTARA VS ALCANTARA 531 SCRA 446 CASE DIGEST 546 547 553 554 561 562 565 566 570 571 575

VIII. RIGHTS AND OBLIGATIONS BETWEEN HUSBAND AND WIFE .............................. 576 PELAYO VS IAURON 12 PHIL 453 NANCY GO AND ALEX GO VS COURT OF APPEALS 272 SCRA 752 NARAG VS NARAG 291 SCRA 451 MANGONON VS COURT OF APPEALS 494 SCRA 1 577 579 582 590

IX. PROPERTY RELATIONS BETWEEN HUSBAND AND WIFE ........................................ 597 JOCSON VS COURT OF APPEALS 170 SCRA 333 CASE DIGEST MARIANO VS COURT OF APPEALS 174 SCRA 59 CASE DIGERST STASA, INC. VS COURT OF APPEALS 182 SCRA 879 VILLANUEVA VS IAC 192 SCRA 21 CASE DIGEST BELCODERO VS COURT OF APPEALS 227 SCRA 303 CASE DIGEST AYALA DEVELOPMENT CORP. VS COURT OF APPEALS 286 SCRA 272 CASE DIGEST TODA VS COURT OF APPEALS 183 SCRA 713 CASE DIGEST X. 598 603 605 608 609 613 616 618 621 622 629 630 636

PROPERTY REGIME OF UNIONS WITHOUT MARRIAGE ............................................ 637 638 640 644 645 649 650 654 656 663 664 670 671 676

ACRE VS YUTTIKKI 534 SCRA 224 ENRICO VS HEIRS OF SPOUSES MEDINACELI 534 SCRA 418 CASE DIGEST: VALDEZ VS RTC 260 SCRA 221 CASE DIGEST: AGAPAY VS PALANG 276 SCRA 340 CASE DIGEST: TUMLOS VS FERNANDEZ 330 SCRA 718 CASE DIGEST: MALILIN JR. VS CASTILLO 351 SCRA 127 CASE DIGEST: CARIO VS CARIO CASE DIGEST v

Table of Contents
CARIO VS CARIO SAGUID VS COURT OF APPEALS 403 SCRA 678 CASE DIGEST: HEIRS OF MARCELINO VS HEIRS OF FORTUNATO DORONIO 541 SCRA 479 CASE DIGEST 676 678 682 684 693

XI. THE FAMILY HOME ......................................................................................................... 694 SANDEJAS VS IGNACIO JR. 541 SCRA 61 CASE DIGEST MODEQUILI VS BREVA 185 SCRA 766 CASE DIGEST MANACOP VS COURT OF APPEALS 215 SCRA 773 CASE DIGEST GUERRERO VS RTC, ILOCOS NORTE 229 SCRA 274 695 704 705 708 709 713 714

XII. PATERNITY AND FILIATION ........................................................................................ 717 JAO VS. CA CASE DIGEST UYGUANGCO VS. CA CASE DIGEST ARUEGO VS. CA CASE DIGEST: PEOPLE VS. MALAPO SAYSON VS. CA CASE DIGEST TAYAG VS. CA CASE DIGEST BENITEZ-BADUA VS. CA CASE DIGEST FERNANDEZ VS CA DE SANTOS VS. HON. ANGELES MARQUINO VS INTERMEDIATE APPELLATE COURT 233 SCRA 130 CASE DIGEST: MARQUINO VS INTERMEDIATE APPELLATE COURT 233 SCRA 130 RODRIGUEZ VS COURT OF APPEALS 245 SCRA 150 CASE DIGEST: RODRIGUEZ VS COURT OF APPEALS 245 SCRA 150 DE SANTOS VS ANGELES 251 SCRA 206 CASE DIGEST: PE LIM VS COURT OF APPEALS 270 SCRA 1 CASE DIGEST: MOSSESGELD VS COURT OF APPEALS 300 SCRA 464 CASE DIGEST: LEONARDO VS COURT OF APPEALS CASE DIGEST: JISON VS COURT OF APPEALS 286 SCRA 495 CASE DIGEST: ESTATE OF ROGELIO G. ONG VS DIAZ 540 SCRA 480 vi 718 723 724 727 728 733 734 741 745 746 752 753 759 760 765 770 774 774 775 779 779 780 804 805 808 809 811 812 815 816 833 835

Table of Contents
CASE DIGEST: 843

XIII. ADOPTION ...................................................................................................................... 844 SSS VS AGUAS 483 SCRA 383 CASE DIGEST: CERVANTES VS FAJARDO 169 SCRA 575 CASE DIGEST: REPUBLIC VS VERGARA 270 SCRA 206 CASE DIGEST REPUBLIC VS COURT OF APPEALS 205 SCRA 356 CASE DIGEST: REPUBLIC VS COURT OF APPEALS 209 SCRA 189 CASE DIGEST: REPUBLIC VS COURT OF APPEALS 227 SCRA 401 CASE DIGEST: REPUBLIC VS TOLEDANO 233 SCRA 9 CASE DIGEST: 845 852 853 855 856 858 859 863 864 870 871 873 874 877

XIV. SUPPORT ......................................................................................................................... 878 DAVID VS COURT OF APPEALS 250 SCRA 82 CASE DIGEST ESTATE OF RUIZ VS COURT OF APPEALS 252 SCRA 541 879 882 883

XV. PARENTAL AUTHORITY ................................................................................................ 888 GAMBOA HIRSCH VS COURT OF APPEALS 527 SCRA 380 ESPIRITU V. COURT OF APPEALS, 242 SCRA 362 SANTOS, SR. VS. CA SAGALA-ESLAO VS. CA SANDEJAS VS IGNACIO JR. 541 SCRA 61 SY VS. COURT OF APPEALS PEREZ VS. CA DAVID VS. CA 889 891 898 902 906 915 922 927

XVI. SUMMARY OF JUDICIAL PROCEEDING IN THE FAMILY LAW ............................. 930 REPUBLIC OF THE PHILIPPINES VS COURT OF APPEALS AND MAXIMO WONG REPUBLIC OF THE PHILIPPINES, VS HON. JOSE R. HERNANDEZ REPUBLIC VS COURT OF APPEALS LEONOR VS COURT OF APPEALS 931 937 947 950

XVII. LATEST JURISPRUDENCE .......................................................................................... 955 GERBERT R. CORPUZ VS. DAISYLYN TIROL STO. TOMAS CASE DIGEST RICARDO P. TORING VS. TERESITA M. TORING CASE DIGEST vii 956 963 965 973

Table of Contents
MYRNA P. ANTONE VS. LEO R. BERONILLA, CASE DIGEST ENRIQUE AGRAVIADOR VS. ERLINDA AMPARO-AGRAVIADOR AND REPUBLIC OF THE PHILIPPINES, CASE DIGEST MARABLE VS MARABLE CASE DIGEST ALAIN M. DIO , VS. MA. CARIDAD L. DIO CASE DIGEST DANILO A. AURELIO VS. VIDA MA. CORAZON P. AURELIO CASE DIGEST JUANITA TRINIDAD RAMOS,ET AL. V.DANILO PANGILINANET AL. BEN-HUR NEPOMUCENO V. ARCHBENCEL ANN LOPEZ, REPRESENTED BY HERMOTHER ARACELI LOPEZ MA. VIRGINIA V. REMO VS. THE HONORABLE SECRETARY OF FOREIGN AFFAIRS INSURANCE OF THE PHILIPPINE ISLANDS CORP VS SPOUSES GREGORIO 975 984 985 996 998 1003 1005 1010 1011 1015 1017 1020 1024 1028

viii

I.

Introduction

University of the Cordilleras College of Law First Year C S.Y. 2013 - 2014

A. Constitutional Provisions
Section 12. The State recognizes the sanctity of family life and shall protect and strengthen the family as a basic autonomous social institution. It shall equally protect the life of the mother and the life of the unborn from conception. The natural and primary right and duty of parents in the rearing of the youth for civic efficiency and the development of moral character shall receive the support of the Government. Section 1. The State recognizes the Filipino family as the foundation of the nation. Accordingly, it shall strengthen its solidarity and actively promote its total development. Section 2. Marriage, as an inviolable social institution, is the foundation of the family and shall be protected by the State. Section 14. The State recognizes the role of women in nation-building, and shall ensure the fundamental equality before the law of women and men. Section 1. No person shall be deprived of life, liberty, or property without due process of law, nor shall any person be denied the equal protection of the laws.

University of the Cordilleras College of Law First Year C S.Y. 2013 - 2014

B. Preliminary Title
Taada vs. Tuvera 136 SCRA 27 G.R. No. L-63915 April 24, 1985 Full Case LORENZO M. TAADA, ABRAHAM F. SARMIENTO, and MOVEMENT OF ATTORNEYS FOR BROTHERHOOD, INTEGRITY AND NATIONALISM, INC. [MABINI], petitioners, vs. HON. JUAN C. TUVERA, in his capacity as Executive Assistant to the President, HON. JOAQUIN VENUS, in his capacity as Deputy Executive Assistant to the President , MELQUIADES P. DE LA CRUZ, in his capacity as Director, Malacaang Records Office, and FLORENDO S. PABLO, in his capacity as Director, Bureau of Printing, respondents. ESCOLIN, J.: Invoking the people's right to be informed on matters of public concern, a right recognized in Section 6, Article IV of the 1973 Philippine Constitution, as well as the principle that laws to be valid and enforceable must be published in the Official Gazette or otherwise effectively promulgated, petitioners seek a writ of mandamus to compel respondent public officials to publish, and/or cause the publication in the Official Gazette of various presidential decrees, letters of instructions, general orders, proclamations, executive orders, letter of implementation and administrative orders. Specifically, the publication of the following presidential issuances is sought: a] Presidential Decrees Nos. 12, 22, 37, 38, 59, 64, 103, 171, 179, 184, 197, 200, 234, 265, 286, 298, 303, 312, 324, 325, 326, 337, 355, 358, 359, 360, 361, 368, 404, 406, 415, 427, 429, 445, 447, 473, 486, 491, 503, 504, 521, 528, 551, 566, 573, 574, 594, 599, 644, 658, 661, 718, 731, 733, 793, 800, 802, 835, 836, 923, 935, 961, 1017-1030, 1050, 1060-1061, 1085, 1143, 1165, 1166, 1242, 1246, 1250, 1278, 1279, 1300, 1644, 1772, 1808, 1810, 1813-1817, 1819-1826, 1829-1840, 1842-1847. b] Letter of Instructions Nos.: 10, 39, 49, 72, 107, 108, 116, 130, 136, 141, 150, 153, 155, 161, 173, 180, 187, 188, 192, 193, 199, 202, 204, 205, 209, 211-213, 215-224, 226-228, 231-239, 241-245, 248, 251, 253-261, 263-269, 271-273, 275-283, 285-289, 291, 293, 297-299, 301303, 309, 312-315, 325, 327, 343, 346, 349, 357, 358, 362, 367, 370, 382, 385, 386, 396397, 405, 438-440, 444- 445, 473, 486, 488, 498, 501, 399, 527, 561, 576, 587, 594, 599, 600, 602, 609, 610, 611, 612, 615, 641, 642, 665, 702, 712-713, 726, 837-839, 878-879, 881, 882, 939-940, 964,997,1149-1178,1180-1278. c] General Orders Nos.: 14, 52, 58, 59, 60, 62, 63, 64 & 65. d] Proclamation Nos.: 1126, 1144, 1147, 1151, 1196, 1270, 1281, 1319-1526, 1529, 1532, 1535, 1538, 1540-1547, 1550-1558, 1561-1588, 1590-1595, 1594-1600, 1606-1609, 1612-1628, 1630-1649, 1694-1695, 1697-1701, 1705-1723, 1731-1734, 1737-1742, 1744, 1746-1751, 1752, 1754, 1762, 1764-1787, 1789-1795, 1797, 1800, 1802-1804, 1806-1807, 1812-1814, 1816, 1825-1826, 1829, 1831-1832, 1835-1836, 1839-1840, 1843-1844, 1846-1847, 1849, 1853-1858, 1860, 1866, 1868, 1870, 1876-1889, 1892, 1900, 1918, 1923, 1933, 1952, 1963, 1965-1966, 1968-1984, 1986-2028, 2030-2044, 2046-2145, 2147-2161, 2163-2244. e] Executive Orders Nos.: 411, 413, 414, 427, 429-454, 457- 471, 474-492, 494-507, 509-510, 522, 524-528, 531-532, 536, 538, 543-544, 549, 551-553, 560, 563, 567-568, 570, 574, 593, 594, 598-604, 609, 611- 647, 649-677, 679-703, 705-707, 712-786, 788-852, 854-857. f] Letters of Implementation Nos.: 7, 8, 9, 10, 11-22, 25-27, 39, 50, 51, 59, 76, 80-81, 92, 94, 95, 107, 120, 122, 123. g] Administrative Orders Nos.: 347, 348, 352-354, 360- 378, 380-433, 436-439. The respondents, through the Solicitor General, would have this case dismissed outright on the ground that petitioners have no legal personality or standing to bring the instant petition. The view is submitted that in the absence of any showing that petitioners are personally and directly affected or prejudiced by 3

University of the Cordilleras College of Law First Year C S.Y. 2013 - 2014 the alleged non-publication of the presidential issuances in question said petitioners are without the requisite legal personality to institute this mandamus proceeding, they are not being "aggrieved parties" within the meaning of Section 3, Rule 65 of the Rules of Court, which we quote: SEC. 3.Petition for Mandamus.When any tribunal, corporation, board or person unlawfully neglects the performance of an act which the law specifically enjoins as a duty resulting from an office, trust, or station, or unlawfully excludes another from the use a rd enjoyment of a right or office to which such other is entitled, and there is no other plain, speedy and adequate remedy in the ordinary course of law, the person aggrieved thereby may file a verified petition in the proper court alleging the facts with certainty and praying that judgment be rendered commanding the defendant, immediately or at some other specified time, to do the act required to be done to Protect the rights of the petitioner, and to pay the damages sustained by the petitioner by reason of the wrongful acts of the defendant. Upon the other hand, petitioners maintain that since the subject of the petition concerns a public right and its object is to compel the performance of a public duty, they need not show any specific interest for their petition to be given due course. The issue posed is not one of first impression. As early as the 1910 case of Severino vs. Governor General, this Court held that while the general rule is that "a writ of mandamus would be granted to a private individual only in those cases where he has some private or particular interest to be subserved, or some particular right to be protected, independent of that which he holds with the public at large," and "it is for the public officers exclusively to apply for the writ when public rights are to be subserved [Mithchell vs. Boardmen, 79 M.e., 469]," nevertheless, "when the question is one of public right and the object of the mandamus is to procure the enforcement of a public duty, the people are regarded as the real party in interest and the relator at whose instigation the proceedings are instituted need not show that he has any legal or special interest in the result, it being sufficient to show that he is a citizen and as such interested in the execution of the laws [High, Extraordinary Legal Remedies, 3rd ed., sec. 431]. Thus, in said case, this Court recognized the relator Lope Severino, a private individual, as a proper party to the mandamus proceedings brought to compel the Governor General to call a special election for the position of municipal president in the town of Silay, Negros Occidental. Speaking for this Court, Mr. Justice Grant T. Trent said: We are therefore of the opinion that the weight of authority supports the proposition that the relator is a proper party to proceedings of this character when a public right is sought to be enforced. If the general rule in America were otherwise, we think that it would not be applicable to the case at bar for the reason 'that it is always dangerous to apply a general rule to a particular case without keeping in mind the reason for the rule, because, if under the particular circumstances the reason for the rule does not exist, the rule itself is not applicable and reliance upon the rule may well lead to error' No reason exists in the case at bar for applying the general rule insisted upon by counsel for the respondent. The circumstances which surround this case are different from those in the United States, inasmuch as if the relator is not a proper party to these proceedings no other person could be, as we have seen that it is not the duty of the law officer of the Government to appear and represent the people in cases of this character. The reasons given by the Court in recognizing a private citizen's legal personality in the aforementioned case apply squarely to the present petition. Clearly, the right sought to be enforced by petitioners herein is a public right recognized by no less than the fundamental law of the land. If petitioners were not allowed to institute this proceeding, it would indeed be difficult to conceive of any other person to initiate the same, considering that the Solicitor General, the government officer generally empowered to represent the people, has entered his appearance for respondents in this case.

University of the Cordilleras College of Law First Year C S.Y. 2013 - 2014 Respondents further contend that publication in the Official Gazette is not a sine qua non requirement for the effectivity of laws where the laws themselves provide for their own effectivity dates. It is thus submitted that since the presidential issuances in question contain special provisions as to the date they are to take effect, publication in the Official Gazette is not indispensable for their effectivity. The point stressed is anchored on Article 2 of the Civil Code: Art. 2. Laws shall take effect after fifteen days following the completion of their publication in the Official Gazette, unless it is otherwise provided, The interpretation given by respondent is in accord with this Court's construction of said article. In a long line of decisions, this Court has ruled that publication in the Official Gazette is necessary in those cases where the legislation itself does not provide for its effectivity date-for then the date of publication is material for determining its date of effectivity, which is the fifteenth day following its publication-but not when the law itself provides for the date when it goes into effect. Respondents' argument, however, is logically correct only insofar as it equates the effectivity of laws with the fact of publication. Considered in the light of other statutes applicable to the issue at hand, the conclusion is easily reached that said Article 2 does not preclude the requirement of publication in the Official Gazette, even if the law itself provides for the date of its effectivity. Thus, Section 1 of Commonwealth Act 638 provides as follows: Section 1. There shall be published in the Official Gazette [1] all-important legislative acts and resolutions of a public nature of the, Congress of the Philippines; [2] all executive and administrative orders and proclamations, except such as have no general applicability; [3] decisions or abstracts of decisions of the Supreme Court and the Court of Appeals as may be deemed by said courts of sufficient importance to be so published; [4] such documents or classes of documents as may be required so to be published by law; and [5] such documents or classes of documents as the President of the Philippines shall determine from time to time to have general applicability and legal effect, or which he may authorize so to be published The clear object of the above-quoted provision is to give the general public adequate notice of the various laws which are to regulate their actions and conduct as citizens. Without such notice and publication, there would be no basis for the application of the maxim "ignorantia legis non excusat." It would be the height of injustice to punish or otherwise burden a citizen for the transgression of a law of which he had no notice whatsoever, not even a constructive one. Perhaps at no time since the establishment of the Philippine Republic has the publication of laws taken so vital significance that at this time when the people have bestowed upon the President a power heretofore enjoyed solely by the legislature. While the people are kept abreast by the mass media of the debates and deliberations in the Batasan Pambansaand for the diligent ones, ready access to the legislative recordsno such publicity accompanies the law-making process of the President. Thus, without publication, the people have no means of knowing what presidential decrees have actually been promulgated, much less a definite way of informing themselves of the specific contents and texts of such decrees. As the Supreme Court of Spain ruled: "Bajo la denominaciongenerica de leyes, se comprendentambien los reglamentos, Reales decretos, Instrucciones, Circulares y Reales ordines dictadas de conformidad con lasmismaspor el Gobierno en uso de supotestad. The very first clause of Section I of Commonwealth Act 638 reads: "There shall be published in the Official Gazette ...." The word "shall" used therein imposes upon respondent officials an imperative duty. That duty must be enforced if the Constitutional right of the people to be informed on matters of public concern is to be given substance and reality. The law itself makes a list of what should be published in the Official Gazette. Such listing, to our mind, leaves respondents with no discretion whatsoever as to what must be included or excluded from such publication.

University of the Cordilleras College of Law First Year C S.Y. 2013 - 2014 The publication of all presidential issuances "of a public nature" or "of general applicability" is mandated by law. Obviously, presidential decrees that provide for fines, forfeitures or penalties for their violation or otherwise impose a burden or. the people, such as tax and revenue measures, fall within this category. Other presidential issuances which apply only to particular persons or class of persons such as administrative and executive orders need not be published on the assumption that they have been circularized to all concerned. It is needless to add that the publication of presidential issuances "of a public nature" or "of general applicability" is a requirement of due process. It is a rule of law that before a person may be bound by law, he must first be officially and specifically informed of its contents. As Justice Claudio Teehankee said in Peralta vs. COMELEC: In a time of proliferating decrees, orders and letters of instructions which all form part of the law of the land, the requirement of due process and the Rule of Law demand that the Official Gazette as the official government repository promulgate and publish the texts of all such decrees, orders and instructions so that the people may know where to obtain their official and specific contents. The Court therefore declares that presidential issuances of general application, which have not been published, shall have no force and effect. Some members of the Court, quite apprehensive about the possible unsettling effect this decision might have on acts done in reliance of the validity of those presidential decrees which were published only during the pendency of this petition, have put the question as to whether the Court's declaration of invalidity apply to P.D.s which had been enforced or implemented prior to their publication. The answer is all too familiar. In similar situations in the past this Court had taken the pragmatic and realistic course set forth in Chicot County Drainage District vs. Baxter Bank to wit: The courts below have proceeded on the theory that the Act of Congress, having been found to be unconstitutional, was not a law; that it was inoperative, conferring no rights and imposing no duties, and hence affording no basis for the challenged decree. Norton v. Shelby County, 118 U.S. 425, 442; Chicago, 1.& L. Ry. Co. v. Hackett, 228 U.S. 559, 566. It is quite clear, however, that such broad statements as to the effect of a determination of unconstitutionality must be taken with qualifications. The actual existence of a statute, prior to such a determination, is an operative fact and may have consequences which cannot justly be ignored. The past cannot always be erased by a new judicial declaration. The effect of the subsequent ruling as to invalidity may have to be considered in various aspects-with respect to particular conduct, private and official. Questions of rights claimed to have become vested, of status, of prior determinations deemed to have finality and acted upon accordingly, of public policy in the light of the nature both of the statute and of its previous application, demand examination. These questions are among the most difficult of those which have engaged the attention of courts, state and federal and it is manifest from numerous decisions that an all-inclusive statement of a principle of absolute retroactive invalidity cannot be justified. Consistently with the above principle, this Court in Rutter vs. Esteban sustained the right of a party under the Moratorium Law, albeit said right had accrued in his favor before said law was declared unconstitutional by this Court. Similarly, the implementation/enforcement of presidential decrees prior to their publication in the Official Gazette is "an operative fact which may have consequences which cannot be justly ignored. The past cannot always be erased by a new judicial declaration ... that an all-inclusive statement of a principle of absolute retroactive invalidity cannot be justified." From the report submitted to the Court by the Clerk of Court, it appears that of the presidential decrees sought by petitioners to be published in the Official Gazette, only Presidential Decrees Nos. 1019 to 1030, inclusive, 1278, and 1937 to 1939, inclusive, have not been so published. Neither the subject matters nor the texts of these PDs can be ascertained since no copies thereof are available. But whatever their subject matter may be, it is undisputed that none of these unpublished PDs has ever been implemented or enforced by the government. In Pesigan vs. Angeles, the Court, through Justice Ramon Aquino, ruled that 6

University of the Cordilleras College of Law First Year C S.Y. 2013 - 2014 "publication is necessary to apprise the public of the contents of [penal] regulations and make the said penalties binding on the persons affected thereby. The cogency of this holding is apparently recognized by respondent officials considering the manifestation in their comment that "the government, as a matter of policy, refrains from prosecuting violations of criminal laws until the same shall have been published in the Official Gazette or in some other publication, even though some criminal laws provide that they shall take effect immediately. WHEREFORE, the Court hereby orders respondents to publish in the Official Gazette all unpublished presidential issuances which are of general application, and unless so published, they shall have no binding force and effect. SO ORDERED.

University of the Cordilleras College of Law First Year C S.Y. 2013 - 2014 Case Digest EFFECTIVITY OF LAWS: TAADA VS. TUVERA G.R. No. L-63915 Decided on: April 24, 1985 Ponente: ESCOLIN, J. FACTS: Petitioners Lorenzo Tanada and others, seek a writ of mandamus to compel respondent public officials to publish, and/or cause the publication in the Official Gazette of various presidential decrees, letters of instructions, general orders, proclamations, executive orders, letters of implementation and administrative orders. Respondents, through the Solicitor General would have this case dismissed outright on the ground that petitioners have no legal personality or standing to bring the instant petition. The view is submitted that in the absence of any showing that the petitioner are personally and directly affected or prejudiced by the alleged nonpublication of the presidential issuances in question. CONTENTION OF THE PETITIONER: Said laws needs publication requirement. CONTENTION OF THE RESPONDENT: They argued that while publication was necessary as a rule, it was not so when it was otherwise provided, as when decrees themselves provides for their own effectivity dates. (Effective immediately upon approval) ISSUE: Whether the presidential decrees in question which contain special provisions as to the date they are to take effect, publication in the Official Gazette is not indispensable for their effectivity? RULING: Publication in the Official Gazette is necessary in those cases where the legislation itself does not provide for its effectivity date, for then the date of publication is material for determining its date of effectivity, which is the 15th day following its publication, but not when the law itself provides for the date when it goes into effect. Article 2 does not preclude the requirement of publication in the Official Gazette, even if the law itself provides for the date of its effectivity. The publication of all presidential issuances of a public nature or of general applicability is mandated by law. Obviously, presidential decrees that provide for fines, forfeitures or penalties for their violation or otherwise impose burdens on the people, such as tax revenue measures, fall within this category. Other presidential issuances which apply only to particular persons or class of persons such as administrative and executive orders need not be published on the assumption that they have been circularized to all concern. The Court therefore declares that presidential issuances of general application, which have not been published, shall have no force and effect.

University of the Cordilleras College of Law First Year C S.Y. 2013 - 2014 Phil. Association of Service Exporters, Inc. vs. Torres 212 SCRA 299 G.R. No. 101279 August 6, 1992 Full Case PHILIPPINE ASSOCIATION OF SERVICE EXPORTERS, INC., petitioner, vs. HON. RUBEN D. TORRES, as Secretary of the Department of Labor & Employment, and JOSE N. SARMIENTO, as Administrator of the PHILIPPINE OVERSEAS EMPLOYMENT ADMINISTRATION,respondents. GRIO-AQUINO, J.: This petition for prohibition with temporary restraining order was filed by the Philippine Association of Service Exporters (PASEI, for short), to prohibit and enjoin the Secretary of the Department of Labor and Employment (DOLE) and the Administrator of the Philippine Overseas Employment Administration (or POEA) from enforcing and implementing DOLE Department Order No. 16, Series of 1991 and POEA Memorandum Circulars Nos. 30 and 37, Series of 1991, temporarily suspending the recruitment by private employment agencies of Filipino domestic helpers for Hong Kong and vesting in the DOLE, through the facilities of the POEA, the task of processing and deploying such workers. PASEI is the largest national organization of private employment and recruitment agencies duly licensed and authorized by the POEA, to engaged in the business of obtaining overseas employment for Filipino landbased workers, including domestic helpers. On June 1, 1991, as a result of published stories regarding the abuses suffered by Filipino housemaids employed in Hong Kong, DOLE Secretary Ruben D. Torres issued Department Order No. 16, Series of 1991, temporarily suspending the recruitment by private employment agencies of "Filipino domestic helpers going to Hong Kong" (p. 30, Rollo). The DOLE itself, through the POEA took over the business of deploying such Hong Kong-bound workers. In view of the need to establish mechanisms that will enhance the protection for Filipino domestic helpers going to Hong Kong, the recruitment of the same by private employment agencies is hereby temporarily suspended effective 1 July 1991. As such, the DOLE through the facilities of the Philippine Overseas Employment Administration shall take over the processing and deployment of household workers bound for Hong Kong, subject to guidelines to be issued for said purpose. In support of this policy, all DOLE Regional Directors and the Bureau of Local Employment's regional offices are likewise directed to coordinate with the POEA in maintaining a manpower pool of prospective domestic helpers to Hong Kong on a regional basis. For compliance.(Emphasis ours; p. 30, Rollo.) Pursuant to the above DOLE circular, the POEA issued Memorandum Circular No. 30, Series of 1991, dated July 10, 1991, providing GUIDELINES on the Government processing and deployment of Filipino domestic helpers to Hong Kong and the accreditation of Hong Kong recruitment agencies intending to hire Filipino domestic helpers. Subject: Guidelines on the Temporary Government Processing and Deployment of Domestic Helpers to Hong Kong. Pursuant to Department Order No. 16, series of 1991 and in order to operationalize the temporary government processing and deployment of domestic helpers (DHs) to Hong Kong resulting from the temporary suspension of recruitment by private employment agencies for said skill and host market, the following guidelines and mechanisms shall govern the implementation of said policy. I. Creation of a joint POEA-OWWA Household Workers Placement Unit (HWPU)

University of the Cordilleras College of Law First Year C S.Y. 2013 - 2014 An ad hoc, one-stop Household Workers Placement Unit [or HWPU] under the supervision of the POEA shall take charge of the various operations involved in the Hong Kong-DH industry segment: The HWPU shall have the following functions in coordination with appropriate units and other entities concerned: 1. Negotiations with and Accreditation of Hong Kong Recruitment Agencies 2. Manpower Pooling 3. Worker Training and Briefing 4. Processing and Deployment 5. Welfare Programs II. Documentary Requirements and Other Conditions for Accreditation of Hong Kong Recruitment Agencies or Principals Recruitment agencies in Hong Kong intending to hire Filipino DHs for their employers may negotiate with the HWPU in Manila directly or through the Philippine Labor Attache's Office in Hong Kong. xxx xxx xxx X. Interim Arrangement All contracts stamped in Hong Kong as of June 30 shall continue to be processed by POEA until 31 July 1991 under the name of the Philippine agencies concerned. Thereafter, all contracts shall be processed with the HWPU. Recruitment agencies in Hong Kong shall submit to the Philippine Consulate General in Hong kong a list of their accepted applicants in their pool within the last week of July. The last day of acceptance shall be July 31 which shall then be the basis of HWPU in accepting contracts for processing. After the exhaustion of their respective pools the only source of applicants will be the POEA manpower pool. For strict compliance of all concerned. (pp. 31-35, Rollo.) On August 1, 1991, the POEA Administrator also issued Memorandum Circular No. 37, Series of 1991, on the processing of employment contracts of domestic workers for Hong Kong. TO: All Philippine and Hong Kong Agencies engaged in the recruitment of Domestic helpers for Hong Kong Further to Memorandum Circular No. 30, series of 1991 pertaining to the government processing and deployment of domestic helpers (DHs) to Hong Kong, processing of employment contracts which have been attested by the Hong Kong Commissioner of Labor up to 30 June 1991 shall be processed by the POEA Employment Contracts Processing Branch up to 15 August 1991 only. Effective 16 August 1991, all Hong Kong recruitment agent/s hiring DHs from the Philippines shall recruit under the new scheme which requires prior accreditation which the POEA. Recruitment agencies in Hong Kong may apply for accreditation at the Office of the Labor Attache, Philippine Consulate General where a POEA team is posted until 31 August 1991. Thereafter, those who failed to have themselves accredited in Hong Kong may proceed to the POEA-OWWA Household Workers Placement Unit in Manila for accreditation before their recruitment and processing of DHs shall be allowed. 10

University of the Cordilleras College of Law First Year C S.Y. 2013 - 2014 Recruitment agencies in Hong Kong who have some accepted applicants in their pool after the cut-off period shall submit this list of workers upon accreditation. Only those DHs in said list will be allowed processing outside of the HWPU manpower pool. For strict compliance of all concerned. (Emphasis supplied, p. 36, Rollo.) On September 2, 1991, the petitioner, PASEI, filed this petition for prohibition to annul the aforementioned DOLE and POEA circulars and to prohibit their implementation for the following reasons: 1. That the respondents acted with grave abuse of discretion and/or in excess of their rule-making authority in issuing said circulars; 2. That the assailed DOLE and POEA circulars are contrary to the Constitution, are unreasonable, unfair and oppressive; and 3. That the requirements of publication and filing with the Office of the National Administrative Register were not complied with. There is no merit in the first and second grounds of the petition. Article 36 of the Labor Code grants the Labor Secretary the power to restrict and regulate recruitment and placement activities. Art. 36. Regulatory Power. The Secretary of Labor shall have the power to restrict and regulate the recruitment and placement activities of all agencies within the coverage of this title [Regulation of Recruitment and Placement Activities] and is hereby authorized to issue orders and promulgate rules and regulations to carry out the objectives and implement the provisions of this title. (Emphasis ours.) On the other hand, the scope of the regulatory authority of the POEA, which was created by Executive Order No. 797 on May 1, 1982 to take over the functions of the Overseas Employment Development Board, the National Seamen Board, and the overseas employment functions of the Bureau of Employment Services, is broad and far-ranging for: 1. Among the functions inherited by the POEA from the defunct Bureau of Employment Services was the power and duty: "2. To establish and maintain a registration and/or licensing system to regulate private sector participation in the recruitment and placement of workers, locally and overseas, . . ." (Art. 15, Labor Code, Emphasis supplied). (p. 13, Rollo.) 2. It assumed from the defunct Overseas Employment Development Board the power and duty: 3. To recruit and place workers for overseas employment of Filipino contract workers on a government to government arrangement and in such other sectors as policy may dictate . . . (Art. 17, Labor Code.) (p. 13, Rollo.) 3. From the National Seamen Board, the POEA took over: 2. To regulate and supervise the activities of agents or representatives of shipping companies in the hiring of seamen for overseas employment; and secure the best possible terms of employment for contract seamen workers and secure compliance therewith. (Art. 20, Labor Code.) The vesture of quasi-legislative and quasi-judicial powers in administrative bodies is not unconstitutional, unreasonable and oppressive. It has been necessitated by "the growing complexity of the modern society" (Solid Homes, Inc. vs. Payawal, 177 SCRA 72, 79). More and more administrative bodies are necessary to help in the regulation of society's ramified activities. "Specialized in the particular field assigned to

11

University of the Cordilleras College of Law First Year C S.Y. 2013 - 2014 them, they can deal with the problems thereof with more expertise and dispatch than can be expected from the legislature or the courts of justice" (Ibid.). It is noteworthy that the assailed circulars do not prohibit the petitioner from engaging in the recruitment and deployment of Filipino landbased workers for overseas employment. A careful reading of the challenged administrative issuances discloses that the same fall within the "administrative and policing powers expressly or by necessary implication conferred" upon the respondents (People vs. Maceren, 79 SCRA 450). The power to "restrict and regulate conferred by Article 36 of the Labor Code involves a grant of police power (City of Naga vs. Court of Appeals, 24 SCRA 898). To "restrict" means "to confine, limit or stop" (p. 62, Rollo) and whereas the power to "regulate" means "the power to protect, foster, promote, preserve, and control with due regard for the interests, first and foremost, of the public, then of the utility and of its patrons" (Philippine Communications Satellite Corporation vs. Alcuaz, 180 SCRA 218). The Solicitor General, in his Comment, aptly observed: . . . Said Administrative Order [i.e., DOLE Administrative Order No. 16] merely restricted the scope or area of petitioner's business operations by excluding therefrom recruitment and deployment of domestic helpers for Hong Kong till after the establishment of the "mechanisms" that will enhance the protection of Filipino domestic helpers going to Hong Kong. In fine, other than the recruitment and deployment of Filipino domestic helpers for Hongkong, petitioner may still deploy other class of Filipino workers either for Hongkong and other countries and all other classes of Filipino workers for other countries. Said administrative issuances, intended to curtail, if not to end, rampant violations of the rule against excessive collections of placement and documentation fees, travel fees and other charges committed by private employment agencies recruiting and deploying domestic helpers to Hongkong. [They are reasonable, valid and justified under the general welfare clause of the Constitution, since the recruitment and deployment business, as it is conducted today, is affected with public interest. xxx xxx xxx The alleged takeover [of the business of recruiting and placing Filipino domestic helpers in Hongkong] is merely a remedial measure, and expires after its purpose shall have been attained. This is evident from the tenor of Administrative Order No. 16 that recruitment of Filipino domestic helpers going to Hongkong by private employment agencies are hereby " temporarily suspended effective July 1, 1991." The alleged takeover is limited in scope, being confined to recruitment of domestic helpers going to Hongkong only. xxx xxx xxx . . . the justification for the takeover of the processing and deploying of domestic helpers for Hongkong resulting from the restriction of the scope of petitioner's business is confined solely to the unscrupulous practice of private employment agencies victimizing applicants for employment as domestic helpers for Hongkong and not the whole recruitment business in the Philippines. (pp. 62-65,Rollo.) The questioned circulars are therefore a valid exercise of the police power as delegated to the executive branch of Government. Nevertheless, they are legally invalid, defective and unenforceable for lack of power publication and filing in the Office of the National Administrative Register as required in Article 2 of the Civil Code, Article 5 of the Labor Code and Sections 3(1) and 4, Chapter 2, Book VII of the Administrative Code of 1987 which provide: Art. 2. Laws shall take effect after fifteen (15) days following the completion of their publication in the Official Gazatte, unless it is otherwise provided. . . . (Civil Code.) 12

University of the Cordilleras College of Law First Year C S.Y. 2013 - 2014 Art. 5. Rules and Regulations. The Department of Labor and other government agencies charged with the administration and enforcement of this Code or any of its parts shall promulgate the necessary implementing rules and regulations. Such rules and regulations shall become effective fifteen (15) days after announcement of their adoption in newspapers of general circulation. (Emphasis supplied, Labor Code, as amended.) Sec. 3.Filing. (1) Every agency shall file with the University of the Philippines Law Center, three (3) certified copies of every rule adopted by it. Rules in force on the date of effectivity of this Code which are not filed within three (3) months shall not thereafter be the basis of any sanction against any party or persons. (Emphasis supplied, Chapter 2, Book VII of the Administrative Code of 1987.) Sec. 4.Effectivity. In addition to other rule-making requirements provided by law not inconsistent with this Book, each rule shall become effective fifteen (15) days from the date of filing as above provided unless a different date is fixed by law, or specified in the rule in cases of imminent danger to public health, safety and welfare, the existence of which must be expressed in a statement accompanying the rule. The agency shall take appropriate measures to make emergency rules known to persons who may be affected by them. (Emphasis supplied, Chapter 2, Book VII of the Administrative Code of 1987). Once, more we advert to our ruling in Taada vs. Tuvera, 146 SCRA 446 that: . . . Administrative rules and regulations must also be published if their purpose is to enforce or implement existing law pursuant also to a valid delegation. (p. 447.) Interpretative regulations and those merely internal in nature, that is, regulating only the personnel of the administrative agency and not the public, need not be published. Neither is publication required of the so-called letters of instructions issued by administrative superiors concerning the rules or guidelines to be followed by their subordinates in the performance of their duties. (p. 448.) We agree that publication must be in full or it is no publication at all since its purpose is to inform the public of the content of the laws. (p. 448.) For lack of proper publication, the administrative circulars in question may not be enforced and implemented. WHEREFORE, the writ of prohibition is GRANTED. The implementation of DOLE Department Order No. 16, Series of 1991, and POEA Memorandum Circulars Nos. 30 and 37, Series of 1991, by the public respondents is hereby SUSPENDED pending compliance with the statutory requirements of publication and filing under the aforementioned laws of the land. SO ORDERED.

13

University of the Cordilleras College of Law First Year C S.Y. 2013 - 2014 Case Digest EFFECTIVITY OF LAWS: PHILIPPINE ASSOCIATION OF SERVICE EXPORTERS, INC. vs. HON. RUBEN D. TORRES G.R. No. 101279 Decided on: August 6, 1992 Ponente: GRIO-AQUINO, J.: FACTS: Department of Labor and Employment (DOLE) Secretary Ruben Torres issued Department Order No. 16 Series of 1991 temporarily suspending the recruitment by private employment agencies of Filipino domestic helpers going to Hong Kong. Pursuant to the circular, the Philippine Overseas Employment Administration (POEA) issued Memorandum Circular No. 30, Series of 1991, providing guidelines on the Government processing and deployment of Filipino domestic helpers to Hong Kong and the accreditation of Hong Kong recruitment agencies intending to hire Filipino domestic helpers. As a result, DOLE, through the POEA, took over the business of deploying Hong Kong bound workers. Petitioner Philippine Association of Service Exporters (PASEI), filed a petition for prohibition to annul the aforementioned order and to prohibit implementation. CONTENTION OF THE PETITIONER: The requirements of publication and filing with the Office of the National Administrative Register were not complied with. Hence, said order in invalid. CONTENTION OF THE RESPONDENT: The circulars are valid exercise of the police power as delegated to the executive branch of Government. ISSUES: Whether or not the requirements of publication was complied with. RULING: No. The orders and circulars issued are invalid and unenforceable. The reason is the lack of proper publication and filing in the Office of the National Administrative Registrar as required in Article 2 of the Civil Code. Once, more we advert to our ruling in Taada vs. Tuvera, 146 SCRA 446 that: Administrative rules and regulations must also be published if their purpose is to enforce or implement existing law pursuant also to a valid delegation.

14

University of the Cordilleras College of Law First Year C S.Y. 2013 - 2014 De Jesus vs. Commission on Audit, 249 SCRA 152 G.R. No. 109023. August 12, 1998 Full Case RODOLFO S. DE JESUS, EDELWINA DE PARUNGAO, VENUS M. POZON AND other similarly situated personnel of the LOCAL WATER UTILITIES ADMINISTRATION (LWUA), petitioners, vs. COMMISSION ON AUDIT AND LEONARDO L. JAMORALIN in his capacity as COA-LWUA Corporate Auditor respondents. PURISIMA, J.: The pivotal issue raised in this petition is whether or not the petitioners are entitled to the payment of honoraria which they were receiving prior to the effectivity of Rep. Act 6758. Petitioners are employees of the Local Water Utilities Administration (LWUA). Prior to July 1, 1989, they were receiving honoraria as designated members of the LWUA Board Secretariat and the PreQualification, Bids and Awards Committee. On July 1, 1989, Republic Act No. 6758 (Rep. Act 6758), entitled An Act Prescribing A Revised Compensation and Position Classification System in the Government and For Other Purposes, took effect. Section 12 of said law provides for the consolidation of allowances and additional compensation into standardized salary rates. Certain additional compensations, however, were exempted from consolidation. Section 12, Rep. Act 6758, reads Sec. 12. - Consolidation of Allowances and Compensation.- Allowances, except for representation and transportation allowances; clothing and laundry allowances; subsistence allowance of marine officers and crew on board government vessels and hospital personnel; hazard pay; allowances of foreign services personnel stationed abroad; and such other additional compensation not otherwise specified herein as may be determined by the DBM, shall be deemed included in the standardized salary rates herein prescribed. Such other additional compensation, whether in cash or in kind, being received by incumbents as of July 1, 1989 not integrated into the standardized salary rates shall continue to be authorized. To implement Rep. Act 6758, the Department of Budget and Management (DBM) issued Corporate Compensation Circular No. 10 (DBM-CCC No. 10), discontinuing without qualification effective November 1, 1989, all allowances and fringe benefits granted on top of basic salary.

Paragraph 5.6 of DBM-CCC No. 10 provides: Payment of other allowances/fringe benefits and all other forms of compensation granted on top of basic salary, whether in cash or in kind, xxx shall be discontinued effective November 1, 1989. Payment made for such allowances/fringe benefits after said date shall be considered as illegal disbursement of public funds. Pursuant to the aforesaid Law and Circular, respondent Leonardo Jamoralin, as corporate auditor, disallowed on post audit, the payment of honoraria to the herein petitioners. Aggrieved, petitioners appealed to the COA, questioning the validity and enforceability of DBM-CCC No. 10. More specifically, petitioners contend that DBM-CCC No. 10 is inconsistent with the provisions of Rep. Act 6758 (the law it is supposed to implement) and, therefore, void. And it is without force and effect because it was not published in the Official Gazette; petitioners stressed.

15

University of the Cordilleras College of Law First Year C S.Y. 2013 - 2014 In its decision dated January 29, 1993, the COA upheld the validity and effectivity of DBM-CCC No. 10 and sanctioned the disallowance of petitioners honoraria. Undaunted, petitioners found their way to this court via the present petition, posing the questions: (1) Whether or not par. 5.6 of DBM-CCC No. 10 can supplant or negate the express provisions of Sec. 12 of Rep. Act 6758 which it seeks to implement; and (2) Whether or not DBM-CCC No. 10 is legally effective despite its lack of publication in the Official Gazette. Petitioners are of the view that par. 5.6 of DBM-CCC No. 10 prohibiting fringe benefits and allowances effective November 1, 1989, is violative of Sec. 12 of Rep. Act 6758 which authorizes payment of additional compensation not integrated into the standardized salary which incumbents were enjoying prior to July 1, 1989. To buttress petitioners stance, the Solicitor General presented a Manifestation and Motion in Lieu of Comment, opining that Sec. 5.6 of DBM-CCC No. 10 is a nullity for being inconsistent with and repugnant to the very law it is intended to implement. The Solicitor General theorized, that: xxx following the settled principle that implementing rules must necessarily adhere to and not depart from the provisions of the statute it seeks to implement, it is crystal clear that Section 5.6 of DBM-CCC No. 10 is a patent nullity. An implementing rule can only be declared valid if it is in harmony with the provisions of the legislative act and for the sole purpose of carrying into effect its general provisions. When an implementing rule is inconsistent or repugnant to the provisions of the statute it seeks to interpret, the mandate of the statute must prevail and must be followed. Respondent COA, on the other hand, pointed out that to allow honoraria without statutory, presidential or DBM authority, as in this case, would run counter to Sec. 8, Article IX-B of the Constitution which proscribes payment of additional or double compensation, unless specifically authorized by law. Therefore, the grant of honoraria or like allowances requires a specific legal or statutory authority. And DBM-CCC No. 10 need not be published for it is merely an interpretative regulation of a law already published; COA concluded. In his Motion for Leave to intervene, the DBM Secretary asserted that the honoraria in question are considered included in the basic salary, for the reason that they are not listed as exceptions under Sec. 12 of Rep. Act 6758. Before resolving the other issue - whether or not Paragraph 5.6 of DBM-CCC No. 10 can supplant or negate the pertinent provisions of Rep. Act 6758 which it seeks to implement, we have to tackle first the other question whether or not DBM-CCC No. 10 has legal force and effect notwithstanding the absence of publication thereof in the Official Gazette. This should take precedence because should we rule that publication in the Official Gazette or in a newspaper of general circulation in the Philippines is sine qua non to the effectiveness or enforceability of DBM-CCC No. 10, resolution of the first issue posited by petitioner would not be necessary. The applicable provision of law requiring publication in the Official Gazette is found in Article 2 of the New Civil Code of the Philippines, which reads: Art. 2. Laws shall take effect after fifteen days following the completion of their publication in the Official Gazette, unless it is otherwise provided. This Code shall take effect one year after such publication. In Tanada v. Tuvera, 146 SCRA 453, 454, this Court succinctly construed the aforecited provision of law in point, thus: We hold therefore that all statutes, including those of local application and private laws, shall be published as a condition for their effectivity, which shall begin fifteen days after publication

16

University of the Cordilleras College of Law First Year C S.Y. 2013 - 2014 unless a different effectivity, which shall begin fifteen days after publication unless a different effectivity date is fixed by the legislature. Covered by this rule are presidential decrees and executive orders promulgated by the President in the exercise of legislative powers whenever the same are validly delegated by the legislature or, at present, directly conferred by the Constitution. Administrative rules and regulations must also be published if their purpose is to enforce or implement existing law pursuant to a valid delegation. Interpretative regulations and those merely internal in nature, that is, regulating only the personnel of the administrative agency and not the public, need not be published. Neither is publication required of the socalled letters of instructions issued by administrative superiors concerning the rules or guidelines to be followed by their subordinates in the performance of their duties. Accordingly, even the charter of a city must be published notwithstanding that it applies to only a portion of the national territory and directly affects only the inhabitants of that place. All presidential decrees must be published, including even, say, those naming a public place after a favored individual or exempting him from certain prohibitions or requirements. The circulars issued by the Monetary Board must be published if they are meant not merely to interpret but to fill in the details of the Central Bank Act which that body is supposed to enforce. The same ruling was reiterated in the case of Philippine Association of Service Exporters, Inc. vs. Torres, 212 SCRA 299 [1992]. On the need for publication of subject DBM-CCC No. 10, we rule in the affirmative. Following the doctrine enunciated in Tanada, publication in the Official Gazette or in a newspaper of general circulation in the Philippines is required since DBM-CCC No. 10 is in the nature of an administrative circular the purpose of which is to enforce or implement an existing law. Stated differently, to be effective and enforceable, DBM-CCC No. 10 must go through the requisite publication in the Official Gazette or in a newspaper of general circulation in the Philippines. In the present case under scrutiny, it is decisively clear that DBM-CCC No. 10, which completely disallows payment of allowances and other additional compensation to government officials and employees, starting November 1, 1989, is not a mere interpretative or internal regulation. It is something more than that. And why not, when it tends to deprive government workers of their allowances and additional compensation sorely needed to keep body and soul together. At the very least, before the said circular under attack may be permitted to substantially reduce their income, the government officials and employees concerned should be apprised and alerted by the publication of subject circular in the Official Gazette or in a newspaper of general circulation in the Philippines - to the end that they be given amplest opportunity to voice out whatever opposition they may have, and to ventilate their stance on the matter. This approach is more in keeping with democratic precepts and rudiments of fairness and transparency. In light of the foregoing disquisition on the ineffectiveness of DBM-CCC No. 10 due to its nonpublication in the Official Gazette or in a newspaper of general circulation in the country, as required by law, resolution of the other issue at bar is unnecessary. WHEREFORE, the Petition is hereby GRANTED, the assailed Decision of respondent Commission on Audit is SET ASIDE, and respondents are ordered to pass on audit the honoraria of petitioners. No pronouncement as to costs. SO ORDERED.

17

University of the Cordilleras College of Law First Year C S.Y. 2013 - 2014 Case Digest EFFECTIVITY OF LAWS: RODOLFO S. DE JESUS vs. COMMISSION ON AUDIT G.R. No. 109023 Decided on: August 12, 1998 Ponente: PURISIMA, J., FACTS: Petitioners are employees of the Local Water Utilities Administration (LWUA). Prior to July 1, 1989, they were receiving honoraria as designated members of the LWUA Board Secretariat and the Pre-Qualification, Bids and Awards Committee. Republic Act No. 6758 entitled An Act Prescribing A Revised Compensation and Position Classification System in the Government and For Other Purposes, took effect. Section 12 of said law provides for the consolidation of allowances and additional compensation into standardized salary rates. Certain additional compensations, however, were exempted from consolidation. Department of Budget and Management (DBM) issued Corporate Compensation Circular No. 10 (DBM-CCC No. 10), discontinuing without qualification effective November 1, 1989, all allowances and fringe benefits granted on top of basic salary. Pursuant to the aforesaid Law and Circular, respondent Leonardo Jamoralin, as corporate auditor, disallowed on post audit, the payment of honoraria to the herein petitioners. Petitioners appealed to the COA, questioning the validity and enforceability of DBM-CCC No. 10 but were denied. Hence the instant petition. CONTENTION OF THE PETITIONER: DBM-CCC No. 10 is inconsistent with the provisions of Rep. Act 6758 (the law it is supposed to implement) and, therefore, void. And it is without force and effect because it was not published in the Official Gazette. CONTENTION OF THE RESPONDENT: DBM-CCC No. 10 need not be published for it is merely an interpretative regulation of a law already published. ISSUE: Whether or not DBM-CCC No. 10 is legally effective despite its lack of publication in the Official Gazette. RULING: No. Following the doctrine enunciated in Tanada, publication in the Official Gazette or in a newspaper of general circulation in the Philippines is required since DBM-CCC No. 10 is in the nature of an administrative circular the purpose of which is to enforce or implement an existing law. Stated differently, to be effective and enforceable, DBM-CCC No. 10 must go through the requisite publication in the Official Gazette or in a newspaper of general circulation in the Philippines.

18

University of the Cordilleras College of Law First Year C S.Y. 2013 - 2014 PVB Employees Union vs. Vega, 360 SCRA 33 G.R. No. 105364. June 28, 2001 Full Case PHILIPPINE VETERANS BANK EMPLOYEES UNION-N.U.B.E. and PERFECTO vs. FERNANDEZ, petitioners, vs. HONORABLE BENJAMIN VEGA, Presiding Judge of Branch 39 of the REGIONAL TRIAL COURT of Manila, the CENTRAL BANK OF THE PHILIPPINES and THE LIQUIDATOR OF THE PHILIPPINE VETERANS BANK, respondents KAPUNAN, J.: May a liquidation court continue with liquidation proceedings of the Philippine Veterans Bank (PVB) when Congress had mandated its rehabilitation and reopening? This is the sole issue raised in the instant Petition for Prohibition with Petition for Preliminary Injunction and application for Ex Parte Temporary Restraining Order. The antecedent facts of the case are as follows: Sometime in 1985, the Central Bank of the Philippines (Central Bank, for brevity) filed with Branch 39 of the Regional Trial Court of Manila a Petition for Assistance in the Liquidation of the Philippine Veterans Bank, the same docketed as Case No. SP-32311. Thereafter, the Philippine Veterans Bank Employees Union-N.U.B.E., herein petitioner, represented by petitioner Perfecto V. Fernandez, filed claims for accrued and unpaid employee wages and benefits with said court in SP-32311. After lengthy proceedings, partial payments of the sums due to the employees were made. However, due to the piecemeal hearings on the benefits, many remain unpaid. On March 8, 1991, petitioners moved to disqualify the respondent judge from hearing the above case on grounds of bias and hostility towards petitioners. On January 2, 1992, the Congress enacted Republic Act No. 7169 providing for the rehabilitation of the Philippine Veterans Bank. Thereafter, petitioners filed with the labor tribunals their residual claims for benefits and for reinstatement upon reopening of the bank. Sometime in May 1992, the Central Bank issued a certificate of authority allowing the PVB to reopen.[6] Despite the legislative mandate for rehabilitation and reopening of PVB, respondent judge continued with the liquidation proceedings of the bank. Moreover, petitioners learned that respondents were set to order the payment and release of employee benefits upon motion of another lawyer, while petitioners claims have been frozen to their prejudice. Hence, the instant petition. Petitioners argue that with the passage of R.A. 7169, the liquidation court became functus officio, and no longer had the authority to continue with liquidation proceedings. In a Resolution, dated June 8, 1992, the Supreme Court resolved to issue a Temporary Restraining Order enjoining the trial court from further proceeding with the case. On June 22, 1992, VOP Security & Detective Agency (VOPSDA) and its 162 security guards filed a Motion for Intervention with prayer that they be excluded from the operation of the Temporary Restraining Order issued by the Court. They alleged that they had filed a motion before Branch 39 of the RTC of Manila, in SP-No. 32311, praying that said court order PVB to pay their backwages and salary differentials by authority of R.A. No 6727, Wage Orders No. NCR-01 and NCR-01-Ad and Wage Orders No. NCR-02 and NCR-02-A; and, that said court, in an Order dated June 5, 1992, approved therein 19

University of the Cordilleras College of Law First Year C S.Y. 2013 - 2014 movants case and directed the bank liquidator or PVB itself to pay the back wages and differentials in accordance with the computation incorporated in the order. Said interveners likewise manifested that there was an error in the computation of the monetary benefits due them. On August 18, 1992, petitioners, pursuant to the Resolution of this Court, dated July 6, 1992, filed their Comment opposing the Motion for Leave to File Intervention and for exclusion from the operation of the T.R.O. on the grounds that the movants have no legal interest in the subject matter of the pending action; that allowing intervention would only cause delay in the proceedings; and that the motion to exclude the movants from the T.R.O. is without legal basis and would render moot the relief sought in the petition. On September 3, 1992, the PVB filed a Petition-In-Intervention praying for the issuance of the writs of certiorari and prohibition under Rule 65 of the Rules of Court in connection with the issuance by respondent judge of several orders involving acts of liquidation of PVB even after the effectivity of R.A. No. 7169. PVB further alleges that respondent judge clearly acted in excess of or without jurisdiction when he issued the questioned orders. We find for the petitioners. Republic Act No. 7169 entitled An Act To Rehabilitate The Philippine Veterans Bank Created Under Republic Act No. 3518, Providing The Mechanisms Therefor, And For Other Purposes, which was signed into law by President Corazon C. Aquino on January 2, 1992 and which was published in the Official Gazette on February 24, 1992, provides in part for the reopening of the Philippine Veterans Bank together with all its branches within the period of three (3) years from the date of the reopening of the head office. The law likewise provides for the creation of a rehabilitation committee in order to facilitate the implementation of the provisions of the same. Pursuant to said R.A. No. 7169, the Rehabilitation Committee submitted the proposed Rehabilitation Plan of the PVB to the Monetary Board for its approval. Meanwhile, PVB filed a Motion to Terminate Liquidation of Philippine Veterans Bank dated March 13, 1992 with the respondent judge praying that the liquidation proceedings be immediately terminated in view of the passage of R.A. No. 7169. On April 10, 1992, the Monetary Board issued Monetary Board Resolution No. 348 which approved the Rehabilitation Plan submitted by the Rehabilitation Committee. Thereafter, the Monetary Board issued a Certificate of Authority allowing PVB to reopen. On June 3, 1992, the liquidator filed A Motion for the Termination of the Liquidation Proceedings of the Philippine Veterans Bank with the respondent judge. As stated above, the Court, in a Resolution dated June 8, 1992, issued a temporary restraining order in the instant case restraining respondent judge from further proceeding with the liquidation of PVB. On August 3, 1992, the Philippine Veterans Bank opened its doors to the public and started regular banking operations. Clearly, the enactment of Republic Act No. 7169, as well as the subsequent developments has rendered the liquidation court functus officio. Consequently, respondent judge has been stripped of the authority to issue orders involving acts of liquidation. Liquidation, in corporation law, connotes a winding up or settling with creditors and debtors. It is the winding up of a corporation so that assets are distributed to those entitled to receive them. It is the process of reducing assets to cash, discharging liabilities and dividing surplus or loss. On the opposite end of the spectrum is rehabilitation which connotes a reopening or reorganization. Rehabilitation contemplates a continuance of corporate life and activities in an effort to restore and reinstate the corporation to its former position of successful operation and solvency.

20

University of the Cordilleras College of Law First Year C S.Y. 2013 - 2014 It is crystal clear that the concept of liquidation is diametrically opposed or contrary to the concept of rehabilitation, such that both cannot be undertaken at the same time. To allow the liquidation proceedings to continue would seriously hinder the rehabilitation of the subject bank. Anent the claim of respondents Central Bank and Liquidator of PVB that R.A. No. 7169 became effective only on March 10, 1992 or fifteen (15) days after its publication in the Official Gazette; and, the contention of interveners VOP Security, et. al. that the effectivity of said law is conditioned on the approval of a rehabilitation plan by the Monetary Board, among others, the Court is of the view that both contentions are bereft of merit. While as a rule, laws take effect after fifteen (15) days following the completion of their publication in the Official Gazette or in a newspaper of general circulation in the Philippines, the legislature has the authority to provide for exceptions, as indicated in the clause unless otherwise provided. In the case at bar, Section 10 of R.A. No. 7169 provides: Sec. 10. Effectivity. - This Act shall take effect upon its approval. Hence, it is clear that the legislature intended to make the law effective immediately upon its approval. It is undisputed that R.A. No. 7169 was signed into law by President Corazon C. Aquino on January 2, 1992. Therefore, said law became effective on said date. Assuming for the sake of argument that publication is necessary for the effectivity of R.A. No. 7169, then it became legally effective on February 24, 1992, the date when the same was published in the Official Gazette, and not on March 10, 1992, as erroneously claimed by respondents Central Bank and Liquidator. WHEREFORE, in view of the foregoing, the instant petition is hereby GIVEN DUE COURSE and GRANTED. Respondent Judge is hereby PERMANENTLY ENJOINED from further proceeding with Civil Case No. SP- 32311. SO ORDERED.

21

University of the Cordilleras College of Law First Year C S.Y. 2013 - 2014 Case Digest EFFECTIVITY OF LAWS: Philippine Veterans Bank Employees Union vs. Vega G.R. No. 105364 Decided on: June 28, 2001 Ponente: KAPUNAN, J.: FACTS: In 1985, Central Bank of the Philippines filed a petition for assistance in the liquidation of the Philippine Veterans Bank (PVB). Thereafter, the PVB employees union, herein petitioner, filed claim for accrued and unpaid employee wages and benefits. On January 2, 1992, RA 7169 (An Act to Rehabilitate the PVB) which was signed into law by Pres. Corazon Aquino and which was published in the Official Gazette on February 24, 1992. Thereafter, petitioners filed with the labor tribunals their residual claims for benefits and for reinstatement upon reopening of the bank. In May 1992, Central Bank issued a certificate of authority allowing the PVB to reopen despite the late mandate for rehabilitation and reopening, respondent Judge Vega continued with the liquidation proceedings of the bank alleging further that RA 7169 became effective only on March 10, 1992 or 15 days after its publication in the Official Gazette on February 24, 1992. CONTENTION OF THE PETITIONER: With the passage of R.A. 7169, the liquidation court became functus officio, and no longer had the authority to continue with liquidation proceedings. ISSUE: Whether or not RA 7169 became effective on January 2, 1992. RULING: The Supreme Court upheld that while as a rule laws take effect after 15 days following completion of their publication in the Official Gazette or in a newspaper of general circulation in the Philippines, the legislature has the authority to provide for exceptions as indicated in the clause unless otherwise provided. Citing Tanada vs Tuvera, this clause refers to the date of effectivity and not to the requirement of publication, which cannot in any event be omitted. The reason is that such omission would affect due process in so far as it would deny the public knowledge of the laws that are supposed to govern it.

22

University of the Cordilleras College of Law First Year C S.Y. 2013 - 2014 De Roy vs. Court of Appeals, 157 SCRA 575 G.R. No. 80718 January 29, 1988 Full Case FELIZA P. DE ROY and VIRGILIO RAMOS, petitioners, vs. COURT OF APPEALS and LUIS BERNAL, SR., GLENIA BERNAL, LUIS BERNAL, JR., HEIRS OF MARISSA BERNAL, namely, GLICERIA DELA CRUZ BERNAL and LUIS BERNAL, SR., respondents. CORTES, J.: This special civil action for certiorari seeks to declare null and void two (2) resolutions of the Special First Division of the Court of Appeals in the case of Luis Bernal, Sr., et al. v. Felisa Perdosa De Roy, et al., CA-G.R. CV No. 07286. The first resolution promulgated on 30 September 1987 denied petitioners motion for extension of time to file a motion for reconsideration and directed entry of judgment since the decision in said case had become final; and the second Resolution dated 27 October 1987 denied petitioners motion for reconsideration for having been filed out of time. At the outset, this Court could have denied the petition outright for not being verified as required by Rule 65 section 1 of the Rules of Court. However, even if the instant petition did not suffer from this defect, this Court, on procedural and substantive grounds, would still resolve to deny it. The facts of the case are undisputed. The firewall of a burned-out building owned by petitioners collapsed and destroyed the tailoring shop occupied by the family of private respondents, resulting in injuries to private respondents and the death of Marissa Bernal, a daughter. Private respondents had been warned by petitioners to vacate their shop in view of its proximity to the weakened wall but the former failed to do so. On the basis of the foregoing facts, the Regional Trial Court. First Judicial Region, Branch XXXVIII, presided by the Hon. Antonio M. Belen, rendered judgment finding petitioners guilty of gross negligence and awarding damages to private respondents. On appeal, the decision of the trial court was affirmed in toto by the Court of Appeals in a decision promulgated on August 17, 1987, a copy of which was received by petitioners on August 25, 1987. On September 9, 1987, the last day of the fifteen-day period to file an appeal, petitioners filed a motion for extension of time to file a motion for reconsideration, which was eventually denied by the appellate court in the Resolution of September 30, 1987. Petitioners filed their motion for reconsideration on September 24, 1987 but this was denied in the Resolution of October 27, 1987. This Court finds that the Court of Appeals did not commit a grave abuse of discretion when it denied petitioners motion for extension of time to file a motion for reconsideration, directed entry of judgment and denied their motion for reconsideration. It correctly applied the rule laid down in Habaluyas Enterprises, Inc. v. Japzon, [G.R. No. 70895, August 5, 1985,138 SCRA 461, that the fifteen-day period for appealing or for filing a motion for reconsideration cannot be extended. In its Resolution denying the motion for reconsideration, promulgated on July 30, 1986 (142 SCRA 208), this Court en banc restated and clarified the rule, to wit: Beginning one month after the promulgation of this Resolution, the rule shall be strictly enforced that no motion for extension of time to file a motion for reconsideration may be filed with the Metropolitan or Municipal Trial Courts, the Regional Trial Courts, and the Intermediate Appellate Court. Such a motion may be filed only in cases pending with the Supreme Court as the court of last resort, which may in its sound discretion either grant or deny the extension requested. (at p. 212) Lacsamana v. Second Special Cases Division of the intermediate Appellate Court, [G.R. No. 73146-53, August 26, 1986, 143 SCRA 643], reiterated the rule and went further to restate and clarify the modes and periods of appeal.

23

University of the Cordilleras College of Law First Year C S.Y. 2013 - 2014 Bacaya v. Intermediate Appellate Court, [G.R. No. 74824, Sept. 15, 1986,144 SCRA 161], stressed the prospective application of said rule, and explained the operation of the grace period, to wit: In other words, there is a one-month grace period from the promulgation on May 30, 1986 of the Courts Resolution in the clarificatory Habaluyas case, or up to June 30, 1986, within which the rule barring extensions of time to file motions for new trial or reconsideration is, as yet, not strictly enforceable. Since petitioners herein filed their motion for extension on February 27, 1986, it is still within the grace period, which expired on June 30, 1986, and may still be allowed. This grace period was also applied in Mission v. Intermediate Appellate Court [G.R. No. 73669, October 28, 1986, 145 SCRA 306]. In the instant case, however, petitioners motion for extension of time was filed on September 9, 1987, more than a year after the expiration of the grace period on June 30, 1986. Hence, it is no longer within the coverage of the grace period. Considering the length of time from the expiration of the grace period to the promulgation of the decision of the Court of Appeals on August 25, 1987, petitioners cannot seek refuge in the ignorance of their counsel regarding said rule for their failure to file a motion for reconsideration within the reglementary period. Petitioners contend that the rule enunciated in the Habaluyas case should not be made to apply to the case at bar owing to the non-publication of the Habaluyas decision in the Official Gazette as of the time the subject decision of the Court of Appeals was promulgated. Contrary to petitioners view, there is no law requiring the publication of Supreme Court decisions in the Official Gazette before they can be binding and as a condition to their becoming effective. It is the bounden duty of counsel as lawyer in active law practice to keep abreast of decisions of the Supreme Court particularly where issues have been clarified, consistently reiterated, and published in the advance reports of Supreme Court decisions (G. R. s) and in such publications as the Supreme Court Reports Annotated (SCRA) and law journals. This Court likewise finds that the Court of Appeals committed no grave abuse of discretion in affirming the trial courts decision holding petitioner liable under Article 2190 of the Civil Code, which provides that the proprietor of a building or structure is responsible for the damage r esulting from its total or partial collapse, if it should be due to the lack of necessary repairs. Nor was there error in rejecting petitioners argument that private respondents had the last clear chance to avoid the accident if only they heeded the warning to vacate the tailoring shop and, therefore, petitioners prior negligence should be disregarded, since the doctrine of last clear chance, which has been applied to vehicular accidents, is inapplicable to this case. WHEREFORE, in view of the foregoing, the Court Resolved to DENY the instant petition for lack of merit. SO ORDERED.

24

University of the Cordilleras College of Law First Year C S.Y. 2013 - 2014 Case Digest PUBLICATION OF SUPREME COURT DECISIONS De Roy vs Court of Appeals G.R. No. 80718 Decided on: January 29, 1988 Ponente: CORTES, J.: FACTS: The firewall of a burned out building owned by petitioners collapsed and destroyed the tailoring shop occupied by the family of the private respondents resulting in injuries to private respondents had been warned by petitioners to vacate their shop in view of its proximity to the weakened wall but the former failed to do. In the RTC, petitioners were found guilty of gross negligence. On the last day of the 15 days period to file an appeal, petitioners filed a motion for reconsideration which was again denied. The Supreme Court finds that Court of Appeal did not commit a grave abuse of discretion when it denied petitioners motion for reconsideration. It correctly applied the rule laid down in HabulayasvsJapzon. Counsel for petitioner contends that the said case should not be applied non-publication in the Official Gazette. CONTENTION OF THE PETITIONER: The rule enunciated in the Habaluyas case should not be made to apply to the case at bar owing to the non-publication of the Habaluyas decision in the Official Gazette as of the time the subject decision of the Court of Appeals was promulgated. CONTENTION OF THE RESPONDENT: There is no law requiring the publication of Supreme Court decisions in the Official Gazette before they can be binding and as a condition to their becoming effective. ISSUE: Whether or not Supreme Court decisions must be published in the Official Gazette before they can be binding. RULING: No. There is no law requiring the publication of Supreme Court decision in the Official Gazette before they can be binding and as a condition to their becoming effective. It is bounden duty of counsel as lawyer in active law practice to keep abreast of decisions of the Supreme Court particularly where issues have been clarified, consistently reiterated and published in the advance reports of Supreme Court decisions and in such publications as the SCRA and law journals.

25

University of the Cordilleras College of Law First Year C S.Y. 2013 - 2014 Unciano Paramedical College, Inc. vs. Court of Appeals, 221 SCRA 285 G.R. No. 100335. April 7, 1993 Full Case UNCIANO PARAMEDICAL COLLEGE, INC. (now UNCIANO COLLEGES & GENERAL HOSPITAL, INC.); MIRANDO C. UNCIANO, SR., DOMINADOR SANTOS AND EDITHA MORA, petitioners, vs. THE COURT OF APPEALS, Honorable LOURDES K. TAYAO-JAGUROS, in her capacity as Presiding Judge, Regional Trial Court, Branch 21, Manila; ELENA VILLEGAS thru VICTORIA VILLEGAS; and TED MAGALLANES thru JACINTA MAGALLANES, respondents. NOCON, J p: This is a petition for review on certiorari seeking reversal of the decision 1 of public respondent Court of Appeals dated February 7, 1991, in CA-G.R. SP No. 21020; and its resolution dated June 3, 1991. The antecedent facts are, as follows: On April 16, 1990, private respondents Elena Villegas and Ted Magallanes, thru their mothers, Victoria Villegas and Jacinta Magallanes, respectively, filed before the Regional Trial Court, National Capital Judicial Region, Branch 21, a petition for injunction and damages with prayer for a writ of preliminary mandatory injunction against petitioners Unciano Paramedical College, Inc. (now Unciano Colleges and General Hospital, Inc.), Mirando C. Unciano, Sr., Dominador Santos, Editha Mora, Dr. Evelyn Moral and LaureanaVitug, docketed as Civil Case No. 9052745. Among other things, they alleged therein that: "6.01. Around the latter part of July 1989, the above-named students initiated a petition proposing to the school authorities the organization of a student council in the school. They solicited support of their petition from the studentry by asking the students to endorse the same with their signatures. They were able to get at least 180 signatures. "6.02. On August 18, 1989, Elena Villegas and a certain student named Solomon Barroa were summoned to the Office of Dr. Moral and were admonished not to proceed with the proposal because, according to her, the school does not allow and had never allowed such an organization. "6.03. On September 12, 1989, when news leaked out that the above-named students would be barred from enrolment, they sought confirmation with respondent Dr. Moral, Dean of Discipline, who told them 'it's not true unless you violate the rules and regulations of the school and if you still insist with your student council.' "6.04. On October 28, 1989, in compliance with an announcement to see the Dean of Nursing, the above-named students met with Dean Vitug and Dr. Moral who informed them that they would be barred from enrolment for the second semester because they supposedly harassed a female student, invited an outsider to the school to speak before the students, and also because the school has an arrangement with the Department of Education, Culture and Sports not to allow their students to put up a student council. Dr. Moral advised them to get their Honorable Dismissal, and warned them that if she herself were to give it, it would be marked `expelled.' "6.05. On November 6, 1989, the students again approached Dr. Moral who informed them that they were no longer allowed to enroll because they are allegedly members of the National Union of Students of the Philippines (NUSP) and the League of Filipino Students (LFS), officers of the student organization they organized, and, moreover 'drug addicts.' The students asked for proof of these accusations but were not given any, and were told by Dr. Moral that the school has people investigating for (sic) them but she did not disclose their identities nor provide any proof to support her allegations.

26

University of the Cordilleras College of Law First Year C S.Y. 2013 - 2014 "6.06. On November 13, 1989, a few days after petitioners retained the services of counsel FREE LEGAL ASSISTANCE GROUP (FLAG), counsel sent a letter to Mr. MirandoUnciano, President of the College, demanding that the constitutional requirements of due process be complied with prior to unilaterally dismissing the students, and requesting that a conference be held prior to 17 November 1989, as the enrollment deadline was fast approaching . . .: "6.07. On 17 November 1989, acceding to the demand, a meeting was held, attended by Dr. Moral, Dean Vitug, Mr. Rustico Lopez, the students, and their counsel. Due, however, to the inability of Dr. Moral to resolve the problem in the absence of the College President and their legal counsel, the meeting was reset to November 22, 1989 upon Dr. Moral's request. However, notice was sent to the students' counsel from Unciano Paramedical College resetting the meeting to November 27, 1989 stating that the President will attend personally therein . . . "6.08. On 27 November 1989, due to the absence of the school's legal counsel and the President who allegedly just arrived from the United States, Dr. Moral again requested that the meeting be reset. A verbal altercation occurred between the parties due to the delaying tactics of the school officials and the failure to resolve the problem by their continuous refusal to discuss the merits of the accusations against the students. The meeting, attended by Dr. Moral, Dean Vitug and Dean Dominador Santos, ended with the school officials' request that it be reset for 29 November 19B9 and that the students bring their parents or guardian with them at said meeting. The students agreed to this request and their counsel prepared a written summary of the matters discussed and agreed during the meeting. The school officials refused to sign it, however . . . "6.09. On 29 November 1989, the students were informed that the President had unilaterally refused to allow them to enroll and it was up to their parents to request or appeal to the school officials to change their decision. Mrs. Victoria Villegas and Mrs. Jacinta Magallanes wrote to the school officials to request that their children be allowed to enroll . . . Dr. Moral informed them that the Board of Trustees will have to decide on these requests. "6.10. On 11 December 1989, the students were informed that the Board of Trustees had refused to grant the parents' request." On May 16, 1990, the trial court issued a temporary restraining order effective May 17, 1990, enjoining petitioner school from not enrolling private respondents in its College of Nursing and setting the hearing for the issuance of the writ of preliminary injunction on June 4, 1990. Petitioners filed an opposition to the prayer for a preliminary mandatory injunction on the ground that private respondents are not entitled thereto and have no clear legal right to the relief demanded. On the same date, the trial court issued an order, the pertinent parts of which, read: "xxxxxxxxx "It is the opinion of the Court that there will be irreparable injury to the petitioners if they are not allowed to enroll. At least they will miss another semester. "On the other hand, the injuries mentioned by Dr. Unciano, in particular the withdrawal of the other students and the school will lose money if the petitioners are allowed to enroll is still a speculation, and may not take place. "In view thereof, the Court hereby GRANTS the petition for issuance of a preliminary mandatory injunction, ordering the respondents to allow petitioners to enroll for the first semester of school year 1990-1991, upon filing by petitioners of a bond in the amount of P2,000.00 each. xxx xxx xxx "SO ORDERED." On June 11. 1990, the writ of preliminary mandatory injunction was issued. 27

University of the Cordilleras College of Law First Year C S.Y. 2013 - 2014 On June 13, 1990, petitioners' motion for reconsideration of the Order of June 4, 1990 was denied. Elevating the matter to the Court of Appeals in a petition for certiorari and prohibition with preliminary injunction, the same was dismissed on February 7, 1991 for lack of merit. Said the court: "The arguments advanced in support of the petition are mainly anchored on the decision of the Supreme Court in the case of ALCUAZ, et al. vs. Philippine School of Business Administration, Quezon City Branch (PSBA), et al., L-76353, May 2, 1988; 161 SCRA 7 where it was held that 'It is beyond dispute that a student once admitted by the school is considered enrolled for one semester. It is provided in Paragraph 137 (of the) Manual of Regulations for Private Schools, that when a college student registers in a school, it is understood that he is enrolling for the entire semester. Likewise, it is provided in the Manual that the 'written contracts' required for college teachers are for 'one semester.' It is thus evident that after the close of the first semester, the PSBA-QC no longer has any existing contract either with the students or with the intervening teachers . . . "However, in the more recent case of Ariel Non, et al. vs. Hon. Sancho Dames II, et al., G.R. No. 89317, May 20, 1990 (185 SCRA 523), the Supreme Court, abandoned and overruled its decision in Alcuaz and declared thus: The Court, in Alcuaz, anchored its decision on the 'termination of contract' theory. But it must be repeatedly emphasized that the contract between the school and the student is not an ordinary contract. It is imbued with public interest, considering the high priority given by the Constitution to education and the grant to the State of supervisory and regulatory powers over all educational institutions [See Art. XIV, Secs. 1-2, 4(1).] 'Respondent school cannot justify its actions by relying on Paragraph 137 of the Manual of Regulations for Private School which provides that '(w)hen a student registers in a school, it is understood that he is enrolling for the entire semester for collegiate courses,' which the Court in Alcuaz construed as authority for schools to refuse enrolment to a student on the ground that his contract, which has a term of one semester, has already expired. 'The 'termination of contract' theory does not even find support in the Manual. Paragraph 137 merely clarifies that a college student enrolls for the entire semester. It serves to protect schools wherein tuition fees are collected and paid on an instalment basis, i.e. collection and payment of the down payment upon enrollment and the balance before examinations. Thus, even if a student does not complete the semester for which he was enrolled, but has stayed on for more than two weeks, he may be required to pay his tuition fees for the whole semester before he is given his credentials for transfer. This is the import of Paragraph 137, subsumed under Section VII on Tuition and Other Fees, which in its totality provides: '137. When a student registers in a school, it is understood that he is enrolling for the entire school year for elementary and secondary courses, and for the entire semester for collegiate courses. A student who transfers or otherwise withdraws, in writing, within two weeks after the beginning of classes and who has already paid the pertinent tuition and other school fees in full or for any length of time longer than one month may be charged ten per cent of the total amount due for the term if he withdraws within the first week of classes, or twenty per cent if within the second week of classes, regardless of whether or not he has actually attended classes. The student may be charged all the school fees in full if he withdraws any time after the second week of classes. However, if the transfer or withdrawal is due to a justifiable reason, the student shall be charged the pertinent fees only up to and including the last month of attendance.' 'Clearly, in no way may Paragraph 137 be construed to mean that the student shall be enrolled for only one semester, and that after the semester is over his re-enrolment is dependent solely on the sound discretion of the school. On the contrary, the Manual recognizes the right of the student to be enrolled in his course for the entire period he is expected to complete it. Thus, Paragraph 107 states:

28

University of the Cordilleras College of Law First Year C S.Y. 2013 - 2014 'Every student has the right to enroll in any school, college or university upon meeting its specific requirement and reasonable regulation: Provided, that except in the case of academic delinquency and violation of disciplinary regulation, the student is presumed to be qualified for enrolment for the entire period he is expected to his (sic) complete his course without prejudice to his right to transfer.' 'This 'presumption' has been translated into a right in Batas Pambansa Blg. 232, the 'Education Act of 1982.' Section 9 of this act provides: 'SEC. 9.Rights of Students in School. In addition to other rights, and subject to the limitations prescribed by law and regulations, students and pupils in all schools shall enjoy the following rights: xxx xxx xxx 12. The right to freely choose their field of study subject to existing curricula and to continue their course therein up to graduation, except in cases of academic deficiency, or violation of disciplinary regulations.' On June 3, 1991, the motion for reconsideration was denied, again, for lack of merit. Hence, the present petition. Petitioners raise this lone issue: "WHETHER OR NOT THE NON DOCTRINE SHOULD BE APPLIED RETROACTIVELY TO GOVERN AND INVALIDATE THE LEGAL EFFECTS OF INCIDENTS THAT TOOK PLACE PRIOR TO ITS ADOPTION AND WHICH INCIDENTS WERE PROPER AND VALID UNDER THE ALCUAZ DOCTRINE PREVAILING AT THE TIME SAID INCIDENTS TOOK PLACE." Petitioners argue that under the then prevailing Alcuaz doctrine which was promulgated on May 2, 1988, the contract between them and private respondents was validly terminated upon the end of the first semester of school year 1989-1990. Although said doctrine was later abandoned in Non, et al. v. Dames II, et al., supra, this case was promulgated much later, or on May 20, 1990, when the termination of the contract between them had long become fait accompli. Settled is the rule that when a doctrine of this Court is overruled and a different view is adopted, the new doctrine is applied prospectively, and should not apply to parties who relied on the old doctrine and acted on the faith thereof, conformably with the case of People v. Jabinal, G.R. No. L-30061, 55 SCRA 607 (1974). Thus, the writ of preliminary mandatory injunction was issued by the trial court with grave abuse of discretion. We agree with the arguments of petitioners. The ruling in the Non case should not be given a retroactive effect to cases that arose before its promulgation on May 20, 1990, as in this case, which was filed on April 16, 1990. If it were otherwise, it would result in oppression to petitioners and other schools similarly situated who relied on the ruling in the Alcuaz case, promulgated on May 2, 1988, which recognized the termination of contract theory. We had an opportunity to resolve a similar issue in National Service Corporation, et al. v. NLRC. In this case, petitioner claimed that as a government corporation (by virtue of its being a subsidiary of the National Investment and Development Corporation, a subsidiary wholly owned by the Philippine National Bank, which in turn is a government owned corporation), the terms and conditions of employment of its employees are governed by the civil service law, rules and regulations. In support thereof, petitioner cited the ruling in National Housing Corporation v. Juco, that employees of government owned or controlled corporations are governed by the civil service law, rules and regulations, we rejected this claim of petitioner and held that: "It would appear that, in the interest of justice, the holding in said case should not be given retroactive effect, that is, to cases that arose before its promulgation on 17 January 1985. To do otherwise would be oppressive to Credo and other employees similarly situated, because under the same 1973 Constitution but prior to the ruling in National Housing Corporation vs. Juco, this Court had recognized the 29

University of the Cordilleras College of Law First Year C S.Y. 2013 - 2014 applicability of the Labor Code to, and the authority of the NLRC to exercise jurisdiction over, disputes involving terms and conditions of employment in government-owned or controlled corporations, among them, the National Service Corporation (NASECO)." In addition, We reiterate Our earlier pronouncement in the case of People v. Jabinal, supra, that it is a settled rule that when a doctrine of this Court is overruled and a different view is adopted, the new doctrine should be applied prospectively, and should not apply to parties who had relied on the old doctrine and acted on the faith thereof. Coming now to the question on the propriety of the issuance of the writ of preliminary mandatory injunction, the case of Capitol Medical Center, Inc., et al. v. Court of Appeals, et al. discussed exhaustively the purpose in issuing said writ: "The sole object of a preliminary injunction, whether prohibitory or mandatory, is to preserve the status quo until the merits of the case can be heard. The status quo is the last actual peaceable uncontested status which preceded the controversy (Rodulfa vs. Alfonso, 76 Phil. 225). It may only be resorted to by a litigant for the preservation or protection of his rights or interests and for no other purpose during the pendency of the principal action (Calo vs. Roldan, 76 Phil. 445). It should only be granted if the party asking for it is clearly entitled thereto (Climaco vs. Macaraeg, 4 SCRA 930; Subido vs. Gopengco, 27 SCRA 455; Police Commission vs. Bello, 37 SCRA 230). Inasmuch as a mandatory injunction tends to do more than to maintain the status quo, it is generally improper to issue such an injunction prior to the final hearing (Manila Electric Railroad and Light Co. vs. Del Rosario, 22 Phil. 433). It may, however, issue 'in cases of extreme urgency; where the right is very clear; where considerations of relative inconvenience bear strongly in complainant's favor where there is a willful and unlawful invasion of plaintiff's right against his protest and remonstrance, the injury being a continuing one and where the effect of the mandatory injunction is rather to re-establish and maintain a pre-existing continuing relation between the parties, recently and arbitrarily interrupted by the defendant, than to establish a new relation. Indeed, the writ should not be denied the complainant when he makes out a clear case, free from doubt and dispute.' (Commissioner of Customs vs. Cloribel, et al., SCRA 235.)." In the present case, the contract between the parties was validly terminated upon the end of the first semester of school year 1989-1990, or in October, 1989. This is the status quo. The trial court gravely abused its discretion in issuing the writ of preliminary mandatory injunction which ordered petitioners to allow private respondents "to enroll for the first semester of school year 1990-1991." Guided by the Capitol case, certainly, this writ will not restore the status quo but will go a step backward, then restore the condition preceding the status quo. Private respondents do not possess any clear legal right to reenroll, corollarily, petitioners are not obliged legally to re-admit them. WHEREFORE, the petition is hereby GRANTED. The decision of the Court of Appeals dated February 7, 1991 and its resolution dated June 3, 1991 are SET ASIDE. The orders of the trial court dated June 4, 1990 and June 13, 1990 and the writ of preliminary mandatory injunction are likewise SET ASIDE. SO ORDERED.

30

University of the Cordilleras College of Law First Year C S.Y. 2013 - 2014 Case Digest PROSPECTIVITY OF LAWS: UNCIANO PARAMEDICAL COLLEGE vs. COURT OF APPEALS G.R. No. 100335 Decided on: April 7, 1993 Ponente: NOCON, J.: FACTS: Private respondents Elena Villegas and Ted Magallanes (nursing students), thru their mothers, filed a petition for injunction and damages with prayer for a writ of preliminary mandatory injunction against petitioners Unciano Paramedical College, Inc. They alleged that they were not admitted for the second semester for organizing a student body council which was not allowed by the school which was allegedly a violation of the school regulations. The school argued that under the Alcuaz Doctrine, schools have the discretion to admit students for the second semester because upon enrolment of a student in the first semester, the contract was for that semester only. However a new doctrine was later on promulgated which abandoned the Alcuaz doctrine which is now the NON doctrine. CONTENTION OF THE PETITIONER: Under the Alcuaz Doctrine, schools have the discretion to admit students for the second semester because upon enrolment of a student in the first semester, the contract was for that semester only. CONTENTION OF THE RESPONDENT: The new doctrine should be applied retroactively. ISSUE: Whether or not the new Non doctrine be applied retroactively to the case. RULING: No. The new doctrine shall be applied prospectively and should not apply to parties who relied on the old doctrine and acted on faith thereof. Although said doctrine was later abandoned in Non, et al. v. Dames II, et al., supra, this case was promulgated much later, or on May 20, 1990, when the termination of the contract between them had long become fait accompli. Settled is the rule that when a doctrine of this Court is overruled and a different view is adopted, the new doctrine is applied prospectively, and should not apply to parties who relied on the old doctrine and acted on the faith thereof Thus, the writ of preliminary mandatory injunction was issued by the trial court with grave abuse of discretion. The ruling in the Non case should not be given a retroactive effect to cases that arose before its promulgation on May 20, 1990, as in this case, which was filed on April 16, 1990. If it were otherwise, it would result in oppression to petitioners and other schools similarly situated who relied on the ruling in the Alcuaz case, promulgated on May 2, 1988, which recognized the termination of contract theory.

31

University of the Cordilleras College of Law First Year C S.Y. 2013 - 2014 Alonzo vs. Padua, 150 SCRA 259 G.R. No. 72873 May 28, 1987 Full Case CARLOS ALONZO and CASIMIRA ALONZO, petitioners, vs. INTERMEDIATE APPELLATE COURT and TECLA PADUA, respondents. CRUZ, J p: The question is sometimes asked, in serious inquiry or in curious conjecture, whether we are a court of law or a court of justice. Do we apply the law even if it is unjust or do we administer justice even against the law? Thus queried, we do not equivocate. The answer is that we do neither because we are a court both of law and of justice. We apply the law with justice for that is our mission and purpose in the scheme of our Republic. This case is an illustration. Five brothers and sisters inherited in equal pro indiviso shares a parcel of land registered in 'the name of their deceased parents under OCT No. 10977 of the Registry of Deeds of Tarlac. On March 15, 1963, one of them, Celestino Padua, transferred his undivided share of the herein petitioners for the sum of P550.00 by way of absolute sale. One year later, on April 22, 1964, Eustaquia Padua, his sister, sold her own share to the same vendees, in an instrument denominated "Con Pacto de Retro Sale," for the sum of P 440.00. By virtue of such agreements, the petitioners occupied, after the said sales, an area corresponding to twofifths of the said lot, representing the portions sold to them. The vendees subsequently enclosed the same with a fence. In 1975, with their consent, their son Eduardo Alonzo and his wife built a semi-concrete house on a part of the enclosed area. On February 25, 1976, Mariano Padua, one of the five coheirs, sought to redeem the area sold to the spouses Alonzo, but his complaint was dismissed when it appeared that he was an American citizen. On May 27, 1977, however, Tecla Padua, another co-heir, filed her own complaint invoking the same right of redemption claimed by her brother. The trial court * also dismiss this complaint, now on the ground that the right had lapsed, not having been exercised within thirty days from notice of the sales in 1963 and 1964. Although there was no written notice, it was held that actual knowledge of the sales by the co-heirs satisfied the requirement of the law. In truth, such actual notice as acquired by the co-heirs cannot be plausibly denied. The other co-heirs, including Tecla Padua, lived on the same lot, which consisted of only 604 square meters, including the portions sold to the petitioners. Eustaquia herself, who had sold her portion, was staying in the same house with her sister Tecla, who later claimed redemption petition. Moreover, the petitioners and the private respondents were close friends and neighbors whose children went to school together. It is highly improbable that the other co-heirs were unaware of the sales and that they thought, as they alleged, that the area occupied by the petitioners had merely been mortgaged by Celestino and Eustaquia. In the circumstances just narrated, it was impossible for Tecla not to know that the area occupied by the petitioners had been purchased by them from the other co-heirs. Especially significant was the erection thereon of the permanent semi-concrete structure by the petitioners' son, which was done without objection on her part or of any of the other co-heirs. The only real question in this case, therefore, is the correct interpretation and application of the pertinent law as invoked, interestingly enough, by both the petitioners and the private respondents. This is Article 1088 of the Civil Code, providing as follows: Art. 1088. Should any of the heirs sell his hereditary rights to a stranger before the partition, any or all of the co-heirs may be subrogated to the rights of the purchaser by reimbursing him for the price of the sale,

32

University of the Cordilleras College of Law First Year C S.Y. 2013 - 2014 provided they do so within the period of one month from the time they were notified in writing of the sale by the vendor. In reversing the trial court, the respondent court ** declared that the notice required by the said article was writtennotice and that actual notice would not suffice as a substitute. Citing the same case of De Conejero v. Court of Appeals applied by the trial court, the respondent court held that that decision, interpreting a like rule in Article 1623, stressed the need for written notice although no particular form was required. Thus, according to Justice J.B.L. Reyes, who was the ponente of the Court, furnishing the co-heirs with a copy of the deed of sale of the property subject to redemption would satisfy the requirement for written notice. "So long, therefore, as the latter (i.e., the redemptioner) is informed in writing of the sale and the particulars thereof," he declared, "the thirty days for redemption start running. " In the earlier decision of Butte v. UY, " the Court, speaking through the same learned jurist, emphasized that the written notice should be given by the vendor and not the vendees, conformably to a similar requirement under Article 1623, reading as follows: Art. 1623. The right of legal pre-emption or redemption shall not be exercised except within thirty days from the notice in writing by the prospective vendor, or by the vendors, as the case may be. The deed of sale shall not be recorded in the Registry of Property, unless accompanied by an affidavit of the vendor that he has given written notice thereof to all possible redemptioners. The right of redemption of co-owners excludes that of the adjoining owners. As "it is thus apparent that the Philippine legislature in Article 1623 deliberately selected a particular method of giving notice, and that notice must be deemed exclusive," the Court held that notice given by the vendees and not the vendor would not toll the running of the 30-day period. The petition before us appears to be an illustration of the Holmes dictum that "hard cases make bad laws" as the petitioners obviously cannot argue against the fact that there was really no written notice given by the vendors to their co-heirs. Strictly applied and interpreted, Article 1088 can lead to only one conclusion, to wit, that in view of such deficiency, the 30 day period for redemption had not begun to run, much less expired in 1977. But as has also been aptly observed, we test a law by its results; and likewise, we may add, by its purposes. It is a cardinal rule that, in seeking the meaning of the law, the first concern of the judge should be to discover in its provisions the intent of the lawmaker. Unquestionably, the law should never be interpreted in such a way as to cause injustice as this is never within the legislative intent. An indispensable part of that intent, in fact, for we presume the good motives of the legislature, is to render justice. Thus, we interpret and apply the law not independently of but in consonance with justice. Law and justice are inseparable, and we must keep them so. To be sure, there are some laws that, while generally valid, may seem arbitrary when applied in a particular case because of its peculiar circumstances. In such a situation, we are not bound, because only of our nature and functions, to apply them just the same, in slavish obedience to their language. What we do instead is find a balance between the word and the will, that justice may be done even as the law is obeyed. As judges, we are not automatons. We do not and must not unfeelingly apply the law as it is worded, yielding like robots to the literal command without regard to its cause and consequence. "Courts are apt to err by sticking too closely to the words of a law," so we are warned, by Justice Holmes again, "where these words import a policy that goes beyond them." While we admittedly may not legislate, we nevertheless have the power to interpret the law in such a way as to reflect the will of the legislature. While we may not read into the law a purpose that is not there, we nevertheless have the right to read out of it the reason for its enactment. In doing so, we defer not to "the letter that killeth" but to "the spirit that vivifieth," to give effect to the law maker'swill. 33

University of the Cordilleras College of Law First Year C S.Y. 2013 - 2014 The spirit, rather than the letter of a statute determines its construction, hence, a statute must be read according to its spirit or intent. For what is within the spirit is within the letter but although it is not within the letter thereof, and that which is within the letter but not within the spirit is not within the statute. Stated differently, a thing which is within the intent of the lawmaker is as much within the statute as if within the letter; and a thing which is within the letter of the statute is not within the statute unless within the intent of the lawmakers. In requiring written notice, Article 1088 seeks to ensure that the redemptioner is properly notified of the sale and to indicate the date of such notice as the starting time of the 30-day period of redemption. Considering the shortness of the period, it is really necessary, as a general rule, to pinpoint the precise date it is supposed to begin, to obviate any problem of alleged delays, sometimes consisting of only a day or two. The instant case presents no such problem because the right of redemption was invoked not days but years after the sales were made in 1963 and 1964. The complaint was filed by Tecla Padua in 1977, thirteen years after the first sale and fourteen years after the second sale. The delay invoked by the petitioners extends to more than a decade, assuming of course that there was a valid notice that tolled the running of the period of redemption. Was there a valid notice? Granting that the law requires the notice to be written, would such notice be necessary in this case? Assuming there was a valid notice although it was not in writing. would there be any question that the 30-day period for redemption had expired long before the complaint was filed in 1977? In the face of the established facts, we cannot accept the private respondents' pretense that they were unaware of the sales made by their brother and sister in 1963 and 1964. By requiring written proof of such notice, we would be closing our eyes to the obvious truth in favor of their palpably false claim of ignorance, thus exalting the letter of the law over its purpose. The purpose is clear enough: to make sure that the redemptioners are duly notified. We are satisfied that in this case the other brothers and sisters were actually informed, although not in writing, of the sales made in 1963 and 1964, and that such notice was sufficient. Now, when did the 30-day period of redemption begin? While we do not here declare that this period started from the dates of such sales in 1963 and 1964, we do say that sometime between those years and 1976, when the first complaint for redemption was filed, the other co-heirs were actually informed of the sale and that thereafter the 30-day period started running and ultimately expired. This could have happened any time during the interval of thirteen years, when none of the co-heirs made a move to redeem the properties sold. By 1977, in other words, when Tecla Padua filed her complaint, the right of redemption had already been extinguished because the period for its exercise had already expired. The following doctrine is also worth noting: While the general rule is, that to charge a party with laches in the assertion of an alleged right it is essential that he should have knowledge of the facts upon which he bases his claim, yet if the circumstances were such as should have induced inquiry, and the means of ascertaining the truth were readily available upon inquiry, but the party neglects to make it, he will be chargeable with laches, the same as if he had known the facts. It was the perfectly natural thing for the co-heirs to wonder why the spouses Alonzo, who were not among them, should enclose a portion of the inherited lot and build thereon a house of strong materials. This definitely was not the act of a temporary possessor or a mere mortgagee. This certainly looked like an act of ownership. Yet, given this unseemly situation, none of the co-heirs saw fit to object or at least inquire, to ascertain the facts, which were readily available. It took all of thirteen years before one of them chose to claim the right of redemption, but then it was already too late.

34

University of the Cordilleras College of Law First Year C S.Y. 2013 - 2014 We realize that in arriving at our conclusion today, we are deviating from the strict letter of the law, which the respondent court understandably applied pursuant to existing jurisprudence. The said court acted properly as it had no competence to reverse the doctrines laid down by this Court in the above-cited cases. In fact, and this should be clearly stressed, we ourselves are not abandoning the De Conejero and Buttle doctrines. What we are doing simply is adopting an exception to the general rule, in view of the peculiar circumstances of this case. The co-heirs in this case were undeniably informed of the sales although no notice in writing was given them. And there is no doubt either that the 30-day period began and ended during the 14 years between the sales in question and the filing of the complaint for redemption in 1977, without the co-heirs exercising their right of redemption. These are the justifications for this exception. More than twenty centuries ago, Justinian defined justice "as the constant and perpetual wish to render everyone his due." That wish continues to motivate this Court when it assesses the facts and the law in every case brought to it for decision. Justice is always an essential ingredient of its decisions. Thus when the facts warrants, we interpret the law in a way that will render justice, presuming that it was the intention of the lawmaker, to begin with, that the law be dispensed with justice. So we have done in this case. WHEREFORE, the petition is granted. The decision of the respondent court is REVERSED and that of the trial court is reinstated, without any pronouncement as to costs. It is so ordered.

35

University of the Cordilleras College of Law First Year C S.Y. 2013 - 2014 Case Digest COMPUTATION OF PERIODS: CARLOS ALONZO vs. INTERMEDIATE APPELLATE COURT and TECLA PADUA G.R. No. 72873 Decided on: May 28, 1987 Ponente: CRUZ, J.: FACTS: Five brothers and sisters inherited in equal pro indiviso shares a parcel of land registered in 'the name of their deceased parents. On March 15, 1963, one of them, Celestino Padua, transferred his undivided share of the herein petitioners for the sum of P550.00 by way of absolute sale. One year later, on April 22, 1964, Eustaquia Padua, his sister, sold her own share to the same vendees, in an instrument denominated "Con Pacto de Retro Sale," for the sum of P 440.00. By virtue of such agreements, the petitioners occupied, after the said sales, an area corresponding to two-fifths of the said lot, representing the portions sold to them. The vendees subsequently enclosed the same with a fence. In 1975, with their consent, their son Eduardo Alonzo and his wife built a semi-concrete house on a part of the enclosed area. On February 25, 1976, Mariano Padua, one of the five coheirs, sought to redeem the area sold to the spouses Alonzo, but his complaint was dismissed when it appeared that he was an American citizen. On May 27, 1977, however, Tecla Padua, another co-heir, filed her own complaint invoking the same right of redemption claimed by her brother. The trial court * also dismiss this complaint, now on the ground that the right had lapsed, not having been exercised within thirty days from notice of the sales in 1963 and 1964. Although there was no written notice, it was held that actual knowledge of the sales by the co-heirs satisfied the requirement of the law. ISSUE: When did the 30-day period of redemption begin? RULING: While we do not here declare that this period started from the dates of such sales in 1963 and 1964, we do say that sometime between those years and 1976, when the first complaint for redemption was filed, the other co-heirs were actually informed of the sale and that thereafter the 30-day period started running and ultimately expired. This could have happened any time during the interval of thirteen years, when none of the co-heirs made a move to redeem the properties sold. By 1977, in other words, when Tecla Padua filed her complaint, the right of redemption had already been extinguished because the period for its exercise had already expired. The co-heirs in this case were undeniably informed of the sales although no notice in writing was given them. And there is no doubt either that the 30-day period began and ended during the 14 years between the sales in question and the filing of the complaint for redemption in 1977, without the co-heirs exercising their right of redemption. These are the justifications for this exception.

36

University of the Cordilleras College of Law First Year C S.Y. 2013 - 2014 People vs. Jabinal, 55 SCRA 607 G.R. No. L-30061 February 27, 1974 Full Case THE PEOPLE OF THE PHILIPPINES, plaintiff-appellees, vs. JOSE JABINAL Y CARMEN, defendant-appellant. ANTONIO, J.: Appeal from the judgment of the Municipal Court of Batangas (provincial capital), Batangas, in Criminal Case No. 889, finding the accused guilty of the crime of Illegal Possession of Firearm and Ammunition and sentencing him to suffer an indeterminate penalty ranging from one (1) year and one (1) day to two (2) years imprisonment, with the accessories provided by law, which raises in issue the validity of his conviction based on a retroactive application of Our ruling in People v. Mapa. The complaint filed against the accused reads: That on or about 9:00 o'clock, p.m., the 5th day of September, 1964, in the poblacion, Municipality of Batangas, Province of Batangas, Philippines, and within the jurisdiction of this Honorable Court, the above-named accused, a person not authorized by law, did then and there wilfully, unlawfully and feloniously keep in his possession, custody and direct control a revolver Cal. .22, RG8 German Made with one (1) live ammunition and four (4) empty shells without first securing the necessary permit or license to possess the same. At the arraignment on September 11, 1964, the accused entered a plea of not guilty, after which trial was accordingly held. The accused admitted that on September 5, 1964, he was in possession of the revolver and the ammunition described in the complaint, without the requisite license or permit. He, however, claimed to be entitled to exoneration because, although he had no license or permit, he had an appointment as Secret Agent from the Provincial Governor of Batangas and an appointment as Confidential Agent from the PC Provincial Commander, and the said appointments expressly carried with them the authority to possess and carry the firearm in question. Indeed, the accused had appointments from the above-mentioned officials as claimed by him. His appointment from Governor Feliciano Leviste, dated December 10, 1962, reads: Reposing special trust and confidence in your civic spirit, and trusting that you will be an effective agent in the detection of crimes and in the preservation of peace and order in the province of Batangas, especially with respect to the suppression of trafficking in explosives, jueteng, illegal cockfighting, cattle rustling, robbery and the detection of unlicensed firearms, you are hereby appointed a SECRET AGENT of the undersigned, the appointment to take effect immediately, or as soon as you have qualified for the position. As such Secret Agent, your duties shall be those generally of a peace officer and particularly to help in the preservation of peace and order in this province and to make reports thereon to me once or twice a month. It should be clearly understood that any abuse of authority on your part shall be considered sufficient ground for the automatic cancellation of your appointment and immediate separation from the service. In accordance with the decision of the Supreme Court in G.R. No. L-12088 dated December 23, 1959, you will have the right to bear a firearm, particularly described below, for use in connection with the performance of your duties. By virtue hereof, you may qualify and enter upon the performance of your duties by taking your oath of office and filing the original thereof with us.

Very truly yours, 37

University of the Cordilleras College of Law First Year C S.Y. 2013 - 2014 (Sgd.) FELICIANO LEVISTE Provincial Governor FIREARM AUTHORIZED TO CARRY: Kind: ROHM-Revolver Make: German SN: 64 Cal: .22 On March 15, 1964, the accused was also appointed by the PC Provincial Commander of Batangas as Confidential Agent with duties to furnish information regarding smuggling activities, wanted persons, loose firearms, subversives and other similar subjects that might affect the peace and order condition in Batangas province, and in connection with these duties he was temporarily authorized to possess a ROHM revolver, Cal. .22 RG-8 SN-64, for his personal protection while in the performance of his duties. The accused contended before the court a quo that in view of his above-mentioned appointments as Secret Agent and Confidential Agent, with authority to possess the firearm subject matter of the prosecution, he was entitled to acquittal on the basis of the Supreme Court's decision in People vs. Macarandang and People vs. Lucero. The trial court, while conceding on the basis of the evidence of record the accused had really been appointed Secret Agent and Confidential Agent by the Provincial Governor and the PC Provincial Commander of Batangas, respectively, with authority to possess and carry the firearm described in the complaint, nevertheless held the accused in its decision dated December 27, 1968, criminally liable for illegal possession of a firearm and ammunition on the ground that the rulings of the Supreme Court in the cases of Macarandang and Lucero were reversed and abandoned in People vs. Mapa, supra. The court considered as mitigating circumstances the appointments of the accused as Secret Agent and Confidential Agent. Let us advert to Our decisions in People v. Macarandang, supra, People v. Lucero, supra, and People v. Mapa, supra. In Macarandang, We reversed the trial court's judgment of conviction against the accused because it was shown that at the time he was found to possess a certain firearm and ammunition without license or permit, he had an appointment from the Provincial Governor as Secret Agent to assist in the maintenance of peace and order and in the detection of crimes, with authority to hold and carry the said firearm and ammunition. We therefore held that while it is true that the Governor has no authority to issue any firearm license or permit, nevertheless, section 879 of the Revised Administrative Code provides that "peace officers" are exempted from the requirements relating to the issuance of license to possess firearms; and Macarandang's appointment as Secret Agent to assist in the maintenance of peace and order and detection of crimes, sufficiently placed him in the category of a "peace officer" equivalent even to a member of the municipal police who under section 879 of the Revised Administrative Code are exempted from the requirements relating to the issuance of license to possess firearms. In Lucero, We held that under the circumstances of the case, the granting of the temporary use of the firearm to the accused was a necessary means to carry out the lawful purpose of the batallion commander to effect the capture of a Huk leader. In Mapa, expressly abandoning the doctrine in Macarandang, and by implication, that in Lucero, We sustained the judgment of conviction on the following ground: The law is explicit that except as thereafter specifically allowed, "it shall be unlawful for any person to ... possess any firearm, detached parts of firearms or ammunition therefor, or any instrument or implement used or intended to be used in the manufacture of firearms, parts of firearms, or ammunition." (Sec. 878, as amended by Republic Act No. 4, Revised Administrative Code.) The next section provides that "firearms and ammunition regularly and lawfully issued to officers, soldiers, sailors, or marines [of the Armed Forces of the Philippines], the Philippine Constabulary, guards in the employment of the Bureau of Prisons, municipal police, provincial governors, lieutenant governors, provincial treasurers, municipal treasurers, municipal mayors, and guards of provincial prisoners and jails," are not covered "when such firearms are in possession of such officials and public servants for use in the performance of their official duties." (Sec. 879, Revised Administrative Code.) The law cannot be any clearer. No provision is made for a secret agent. As such he is not exempt. ... .

38

University of the Cordilleras College of Law First Year C S.Y. 2013 - 2014 It will be noted that when appellant was appointed Secret Agent by the Provincial Government in 1962, and Confidential Agent by the Provincial Commander in 1964, the prevailing doctrine on the matter was that laid down by Us in People v. Macarandang (1959) and People v. Lucero (1958). Our decision in People v. Mapa reversing the aforesaid doctrine came only in 1967. The sole question in this appeal is: Should appellant be acquitted on the basis of Our rulings in Macarandang and Lucero, or should his conviction stand in view of the complete reversal of the Macarandang and Lucero doctrine in Mapa? The Solicitor General is of the first view, and he accordingly recommends reversal of the appealed judgment. Decisions of this Court, although in themselves not laws, are nevertheless evidence of what the laws mean, and this is the reason why under Article 8 of the New Civil Code "Judicial decisions applying or interpreting the laws or the Constitution shall form a part of the legal system ..." The interpretation upon a law by this Court constitutes, in a way, a part of the law as of the date that law originally passed, since this Court's construction merely establishes the contemporaneous legislative intent that law thus construed intends to effectuate. The settled rule supported by numerous authorities is a restatement of legal maxim "legis interpretatio legis vim obtinet" the interpretation placed upon the written law by a competent court has the force of law. The doctrine laid down in Lucero and Macarandang was part of the jurisprudence, hence of the law, of the land, at the time appellant was found in possession of the firearm in question and when he arraigned by the trial court. It is true that the doctrine was overruled in the Mapa case in 1967, but when a doctrine of this Court is overruled and a different view is adopted, the new doctrine should be applied prospectively, and should not apply to parties who had relied on the old doctrine and acted on the faith thereof. This is especially true in the construction and application of criminal laws, where it is necessary that the punishability of an act be reasonably foreseen for the guidance of society. It follows, therefore, that considering that appellant conferred his appointments as Secret Agent and Confidential Agent and authorized to possess a firearm pursuant to the prevailing doctrine enunciated in Macarandang and Lucero, under which no criminal liability would attach to his possession of said firearm in spite of the absence of a license and permit therefor, appellant must be absolved. Certainly, appellant may not be punished for an act which at the time it was done was held not to be punishable. WHEREFORE, the judgment appealed from is hereby reversed, and appellant is acquitted, with costs de oficio.

39

University of the Cordilleras College of Law First Year C S.Y. 2013 - 2014 Case Digest JUDICIAL DECISIONS- FORM PART OF THE LEGAL SYSTEM People vs. Jabinal G.R. No. L-30061 Decided on: February 27, 1974 Ponente: ANTONIO, J.: FACTS: Jabinal was found guilty of the crime of Illegal Possession of Firearm and Ammunition. The accused admitted that on September 5, 1964, he was in possession of the revolver and the ammunition described in the complaint, without the requisite license or permit. He, however, claimed to be entitled to exoneration because, although he had no license or permit, he had an appointment as Secret Agent from the Provincial Governor of Batangas and an appointment as Confidential Agent from the PC Provincial Commander, and the said appointments expressly carried with them the authority to possess and carry the firearm in question. The accused contended before the court a quo that in view of his above-mentioned appointments as Secret Agent and Confidential Agent, with authority to possess the firearm subject matter of the prosecution, he was entitled to acquittal on the basis of the Supreme Courts decision in People vs. Macarandang(1959) and People vs. Lucero(1958) and not on the basis of the latest reversal and abandonment in People vs. Mapa (1967). CONTENTION OF THE ACCUSED: He must be acquitted basing from the Supreme Courts decision in People vs. Macarandang(1959) and People vs. Lucero(1958). CONTENTION OF THE STATE: The rulings of the Supreme Court in the cases of Macarandang and Lucero were reversed and abandoned in People vs. Mapa, supra. ISSUE: Whether or not appellant should be acquitted on the basis of the courts rulings in Macarandang and Lucero, or should his conviction stand in view of the complete reversal of the MAcarandang and Lucero doctrine in Mapa. RULING: Decisions of this Court, under Article 8 of the New Civil Code states that Judicial decisions applying or interpreting the laws or the Constitution shall form a part of the legal system The settled rule supported by numerous authorities is a restatement of legal maxim legis interpretation legis vim obtinet" the interpretation placed upon the written law by a competent court has the force of law. Appellant was appointed as Secret Agent and Confidential Agent and authorized to possess a firearm pursuant to the prevailing doctrine enunciated in Macarandang and Lucero under which no criminal liability would attach to his possession of said firearm in spite of the absence of a license and permit therefor, appellant must be absolved. Certainly, appellant may not be punished for an act which at the time it was done was held not to be punishable.

40

University of the Cordilleras College of Law First Year C S.Y. 2013 - 2014 Cui vs. Arellano University, 2 SCRA 205 G.R. No. L-15127; May 30, 1961 Full Case EMETERIO CUI, plaintiff-appellant, vs. ARELLANO UNIVERSITY, defendant-appellee. CONCEPCION, J.: Appeal by plaintiff Emeterio Cui from a decision of the Court of First Instance of Manila, absolving defendant Arellano University from plaintiff's complaint, with costs against the plaintiff, and dismissing defendant's counter claim, for insufficiency of proof thereon. In the language of the decision appealed from: The essential facts of this case are short and undisputed. As established by the agreement of facts Exhibits X and by the respective oral and documentary evidence introduced by the parties, it appears conclusive that plaintiff, before the school year 1948-1949 took up preparatory law course in the defendant University. After finishing his preparatory law course plaintiff enrolled in the College of Law of the defendant from the school year 1948-1949. Plaintiff finished his law studies in the defendant university up to and including the first semester of the fourth year. During all the school years in which plaintiff was studying law in defendant law college, Francisco R. Capistrano, brother of the mother of plaintiff, was the dean of the College of Law and legal counsel of the defendant university. Plaintiff enrolled for the last semester of his law studies in the defendant university but failed to pay his tuition fees because his uncle Dean Francisco R. Capistrano having severed his connection with defendant and having accepted the deanship and chancellorship of the College of Law of Abad Santos University, plaintiff left the defendant's law college and enrolled for the last semester of his fourth year law in the college of law of the Abad Santos University graduating from the college of law of the latter university. Plaintiff, during all the time he was studying law in defendant university was awarded scholarship grants, for scholastic merit, so that his semestral tuition fees were returned to him after the ends of semester and when his scholarship grants were awarded to him. The whole amount of tuition fees paid by plaintiff to defendant and refunded to him by the latter from the first semester up to and including the first semester of his last year in the college of law or the fourth year, is in total P1,033.87. After graduating in law from Abad Santos University he applied to take the bar examination. To secure permission to take the bar he needed the transcripts of his records in defendant Arellano University. Plaintiff petitioned the latter to issue to him the needed transcripts. The defendant refused until after he had paid back the P1,033 87 which defendant refunded to him as above stated. As he could not take the bar examination without those transcripts, plaintiff paid to defendant the said sum under protest. This is the sum which plaintiff seeks to recover from defendant in this case. Before defendant awarded to plaintiff the scholarship grants as above stated, he was made to sign the following contract covenant and agreement: "In consideration of the scholarship granted to me by the University, I hereby waive my right to transfer to another school without having refunded to the University (defendant) the equivalent of my scholarship cash. (Sgd.) Emeterio Cui". It is admitted that, on August 16, 1949, the Director of Private Schools issued Memorandum No. 38, series of 1949, on the subject of "Scholarship," addressed to "All heads of private schools, colleges and universities," reading:

41

University of the Cordilleras College of Law First Year C S.Y. 2013 - 2014 1. School catalogs and prospectuses submitted to this, Bureau show that some schools offer full or partial scholarships to deserving students for excellence in scholarship or for leadership in extra-curricular activities. Such inducements to poor but gifted students should be encouraged. But to stipulate the condition that such scholarships are good only if the students concerned continue in the same school nullifies the principle of merit in the award of these scholarships. 2. When students are given full or partial scholarships, it is understood that such scholarships are merited and earned. The amount in tuition and other fees corresponding to these scholarships should not be subsequently charged to the recipient students when they decide to quit school or to transfer to another institution. Scholarships should not be offered merely to attract and keep students in a school. 3. Several complaints have actually been received from students who have enjoyed scholarships, full or partial, to the effect that they could not transfer to other schools since their credentials would not be released unless they would pay the fees corresponding to the period of the scholarships. Where the Bureau believes that the right of the student to transfer is being denied on this ground, it reserves the right to authorize such transfer. that defendant herein received a copy of this memorandum; that plaintiff asked the Bureau of Private Schools to pass upon the issue on his right to secure the transcript of his record in defendant University, without being required to refund the sum of P1,033.87; that the Bureau of Private Schools upheld the position taken by the plaintiff and so advised the defendant; and that, this notwithstanding, the latter refused to issue said transcript of records, unless said refund were made, and even recommended to said Bureau that it issue a written order directing the defendant to release said transcript of record, "so that the case may be presented to the court for judicial action." As above stated, plaintiff was, accordingly, constrained to pay, and did pay under protest, said sum of P1,033.87, in order that he could take the bar examination in 1953. Subsequently, he brought this action for the recovery of said amount, aside from P2,000 as moral damages, P500 as exemplary damages, P2,000 as attorney's fees, and P500 as expenses of litigation. In its answer, defendant reiterated the stand it took, vis-a-vis the Bureau of Private Schools, namely, that the provisions of its contract with plaintiff are valid and binding and that the memorandum above-referred to is null and void. It, likewise, set up a counterclaim for P10,000.00 as damages, and P3,000 as attorney's fees. The issue in this case is whether the above quoted provision of the contract between plaintiff and the defendant, whereby the former waived his right to transfer to another school without refunding to the latter the equivalent of his scholarships in cash, is valid or not. The lower court resolved this question in the affirmative, upon the ground that the aforementioned memorandum of the Director of Private Schools is not a law; that the provisions thereof are advisory, not mandatory in nature; and that, although the contractual provision "may be unethical, yet it was more unethical for plaintiff to quit studying with the defendant without good reasons and simply because he wanted to follow the example of his uncle." Moreover, defendant maintains in its brief that the aforementioned memorandum of the Director of Private Schools is null and void because said officer had no authority to issue it, and because it had been neither approved by the corresponding department head nor published in the official gazette. We do not deem it necessary or advisable to consider as the lower court did, the question whether plaintiff had sufficient reasons or not to transfer from defendant University to the Abad Santos University. The nature of the issue before us, and its far reaching effects, transcend personal equations and demand a determination of the case from a high impersonal plane. Neither do we deem it essential to pass upon the validity of said Memorandum No. 38, for, regardless of the same, we are of the opinion that the stipulation in question is contrary to public policy and, hence, null and void. The aforesaid memorandum merely incorporates a sound principle of public policy. As the Director of Private Schools correctly pointed, out in his letter, Exhibit B, to the defendant,

42

University of the Cordilleras College of Law First Year C S.Y. 2013 - 2014 There is one more point that merits refutation and that is whether or not the contract entered into between Cui and Arellano University on September 10, 1951 was void as against public policy. In the case of Zeigel vs. Illinois Trust and Savings Bank, 245 Ill. 180, 19 Ann. Case 127, the court said: 'In determining a public policy of the state, courts are limited to a consideration of the Constitution, the judicial decisions, the statutes, and the practice of government officers.' It might take more than a government bureau or office to lay down or establish a public policy, as alleged in your communication, but courts consider the practices of government officials as one of the four factors in determining a public policy of the state. It has been consistently held in America that under the principles relating to the doctrine of public policy, as applied to the law of contracts, courts of justice will not recognize or uphold a transaction which its object, operation, or tendency is calculated to be prejudicial to the public welfare, to sound morality or to civic honesty (Ritter vs. Mutual Life Ins. Co., 169 U.S. 139; Heding vs. Gallaghere 64 L.R.A. 811; Veazy vs. Allen, 173 N.Y. 359). If Arellano University understood clearly the real essence of scholarships and the motives which prompted this office to issue Memorandum No. 38, s. 1949, it should have not entered into a contract of waiver with Cui on September 10, 1951, which is a direct violation of our Memorandum and an open challenge to the authority of the Director of Private Schools because the contract was repugnant to sound morality and civic honesty. And finally, in Gabriel vs. Monte de Piedad, Off. Gazette Supp. Dec. 6, 1941, p. 67 we read: 'In order to declare a contract void as against public policy, a court must find that the contract as to consideration or the thing to be done, contravenes some established interest of society, or is inconsistent with sound policy and good morals or tends clearly to undermine the security of individual rights. The policy enunciated in Memorandum No. 38, s. 1949 is sound policy. Scholarship are awarded in recognition of merit not to keep outstanding students in school to bolster its prestige. In the understanding of that university scholarships award is a business scheme designed to increase the business potential of an education institution. Thus conceived it is not only inconsistent with sound policy but also good morals. But what is morals? Manresa has this definition. It is good customs; those generally accepted principles of morality which have received some kind of social and practical confirmation. The practice of awarding scholarships to attract students and keep them in school is not good customs nor has it received some kind of social and practical confirmation except in some private institutions as in Arellano University. The University of the Philippines which implements Section 5 of Article XIV of the Constitution with reference to the giving of free scholarships to gifted children, does not require scholars to reimburse the corresponding value of the scholarships if they transfer to other schools. So also with the leading colleges and universities of the United States after which our educational practices or policies are patterned. In these institutions scholarships are granted not to attract and to keep brilliant students in school for their propaganda mine but to reward merit or help gifted students in whom society has an established interest or a first lien. (Emphasis supplied.) WHEREFORE, the decision appealed from is hereby reversed and another one shall be entered sentencing the defendant to pay to the plaintiff the sum of P1,033.87, with interest thereon at the legal rate from September 1, 1954, date of the institution of this case, as well as the costs, and dismissing defendant's counterclaim. It is so ordered.

43

University of the Cordilleras College of Law First Year C S.Y. 2013 - 2014 Case Digest EMETERIO CUI vs. ARELLANO UNIVERSITY G.R. No. L-15127 Decided on: May 30, 1961 Ponente: CONCEPCION,J .: FACTS: Emeterio Cui enrolled in the defendant university where plaintiff finished his law studies in the up to and including the first semester of the fourth year. During all the school years in which plaintiff was studying law in defendant Law College, he was awarded scholarship grants and his semestral tuition fees were returned to him after ends of the semester. Plaintiff left the defendant's law college and enrolled for the last semester of his fourth year law in the college of law of the Abad Santos University graduating from the college of law of the latter university. He applied to take the bar examination in which he needed the transcripts of his records in defendant Arellano University. The defendant refused until after he had paid back the P1,033 87, noting the contract that he signed which stated that in consideration of the scholarship granted to him by the University, he waives his right to transfer to another school without having refunded to the defendant the equivalent of the scholarship cash and followed by Memorandum No. 38 that the Director of Private Schools issued. ISSUE: Whether or not the contract between Cui and the respondent university, whereby the former waives his right to transfer to another school without having refunded to the defendant the equivalent of the scholarship cash valid or not? RULING: The contract of waiver between the plaintiff and respondent on September 10, 1951, is a direct violation of Memorandum No. 38 and hence null and void. The contract wascontrary to sound policy and civic honesty. The policy enunciated in Memorandum No.38, s. 1949 is sound policy. When students are given full or partial scholarships, it isunderstood that such scholarships are merited and earned. The amount in tuition andother fees corresponding to these scholarships should not be subsequently charged to therecipient students when they decide to quit school or to transfer to another institution.Scholarships should not be offered merely to attract and keep students in a school.

44

University of the Cordilleras College of Law First Year C S.Y. 2013 - 2014 ASAALI vs. COMMISSIONER OF CUSTOMS 26 SCRA 205 G.R. No. L-24170 ; December 16, 1968 Full Case ILLUH ASAALI, HATIB ABDURASID, INGKOH BANTALA, BASOK INGKIN, and MOHAMMAD BANTALLA, petitioners, vs. Commisioner of Customs FERNANDO, J.: The policy relentlessly adhered to and unhesitatingly pursued to minimize, if not to do away entirely, with the evil and corruption that smuggling brings in its wake would be frustrated and set at naught if the action taken by respondent Commissioner of Customs in this case, as affirmed by the Court of Tax Appeals, were to be set aside and this appeal from the decision of the latter were to succeed. Fortunately, the controlling principles of law do not call for a contrary conclusion. It cannot be otherwise if the legitimate authority vested in the government were not to be reduced to futility and impotence in the face of an admittedly serious malady, that at times has assumed epidemic proportions. The principal question raised by petitioners, owners of five sailing vessels and the cargo loaded therein declared forfeited by respondent Commissioner of Customs for smuggling, is the validity of their interception and seizure by customs officials on the high seas, the contention being raised that importation had not yet begun and that the seizure was effected outside our territorial waters.. Why such a plea could not be given the least credence without doing violence to common sense and placing the law in disrepute would be apparent from a statement of the case and the findings of facts as set forth in the decision now under review, of the Court of Tax Appeals, dated November 19, 1964, the opinion being penned by the late Associate Judge Augusto M. Luciano. His opinion starts thus: This is an appeal from the decision of the Acting Commissioner of Cu stoms in Customs Case No. 113, dated September 26, 1961, (Jolo Seizure Identification Cases Nos. 38, 39, 40, 41 & 42) decreeing the forfeiture of five (5) sailing vessels (kumpits) named Iroc -Iroc, Lahat-lahat, Liberal Wing III, Sulu Area Command, and Business, with their respective cargoes of blue seal cigarettes and rattan chairs for violation of Section 1363(a) of the Revised Administrative Code and Section 20 of Republic Act No. 426 in relation with Section 1363(f) of the Revised Administrative Code. The facts according to the above opinion are not controverted. Thus: It appears that on September 10, 1950, at about noon time, a customs patrol team on board Patrol Boat ST-23 intercepted the five (5) sailing vessels in question on the high seas, between British North Borneo and Sulu while they were heading towards Tawi-tawi, Sulu. After ordering the vessels to stop, the customs officers boarded and found on board, 181 cases of Herald cigarettes, 9 cases of Camel cigarettes, and some pieces of rattan chairs. The sailing vessels are all of Philippine registry, owned and manned by Filipino residents of Sulu, and of less than thirty (30) tons burden. They came from Sandakan, British North Borneo, but did not possess any permit from the Commissioner of Customs to engage in the importation of merchandise into any port of the Sulu sea, as required by Section 1363(a) of the Revised Administrative Code. Their cargoes were not covered by the required import license under Republic Act No. 426, otherwise known as the Import Control Law. Respondent Commissioner of Customs, as noted at the outset, affirmed the decision rendered by the Collector of Customs of Jolo, who found cause for forfeiture under the law of the vessels and the cargo contained therein. He was, as also already made known, sustained by the Court of Tax Appeals. Hence this petition for review. The first two errors assigned by petitioners would impugn the jurisdiction of the Bureau of Customs to institute seizure proceedings and thereafter to declare the forfeiture of the vessels in question and their cargo. They would justify their stand thus: In the light of the fact that the vessels involved with the articles laden therein were apprehended and seized on the high seas, beyond the territorial waters of the 45

University of the Cordilleras College of Law First Year C S.Y. 2013 - 2014 Philippines, the said vessels could not have touched any place or port in the Philippines, whether a port or place of entry or not, consequently, the said vessels could not have been engaged in the importation of the articles laden therein into any Philippine port or place, whether a port or place of entry or not, to have incurred the liability of forfeiture under Section 1363(a) of the Revised Administrative Code. Such a contention was advanced by petitioners before the Court of Tax Appeals. It met the repudiation that it deserved. Thus: We perfectly see the point of the petitioners but considering the circumstances surrounding the apprehension of the vessels in question, we believe that Section 1363(a) of the Revised Administrative Code should be applied to the case at bar. It has been established that the five vessels came from Sandakan, British North Borneo, a foreign port, and when intercepted, all of them were heading towards Tawi-tawi, a domestic port within the Sulu sea. Laden with foreign manufactured cigarettes, they did not possess the import license required by Republic Act No. 426, nor did they carry a permit from the Commissioner of Customs to engage in importation into any port in the Sulu sea. Their course announced loudly their intention not merely to skirt along the territorial boundary of the Philippines but to come within our limits and land somewhere in Tawi-tawi towards which their prows were pointed. As a matter of fact, they were about to cross our aquatic boundary but for the intervention of a customs patrol which, from all appearances, was more than eager to accomplish its mission. The sense of realism and the vigorous language employed by the late Judge Luciano in rejecting such a plea deserve to be quoted. Thus: To entertain even for a moment the thought that these vessels were probably not bound for a Philippine port would be too much a concession even for a simpleton or a perennial optimist. It is quite irrational for Filipino sailors manning five Philippine vessels to sneak out of the Philippines and go to British North Borneo, and come a long way back laden with highly taxable goods only to turn about upon reaching the brink of our territorial waters and head for another foreign port. 1. We find no plausible reason not to accept in its entirety such a conclusion reached by the Court of Tax Appeals. Nor, even if the persuasive element in the above view were not so overwhelming, could we alter the decisive facts as found by it. For it is now beyond question that its finding, if supported by substantial evidence, binds us, only questions of law being for us to resolve. Where the issue raised belongs to the former category, we lack the power of review. Moreover, for understandable reasons, we feel extreme reluctance to substitute our own discretion for that of the Court of Tax Appeals in its appreciation of the relevant facts and its appraisal of their significance. As we had occasion to state in a relatively recent decision: Nor as a matter of principle is it advis able for this Court to set aside the conclusion reached by an agency such as the Court of Tax Appeals which is, by the very nature of its function, dedicated exclusively to the study and consideration of tax problems and has necessarily developed an expertise on the subject, , there has been an abuse or improvident exercise of its authority. 2. We thus could rest our decision affirming that of the Court of Tax Appeals on the above consideration. It might not be amiss however to devote some degree of attention to the legal points raised in the above two assignment of errors, discussed jointly by petitioners-appellants, alleging the absence of jurisdiction, the deprivation of property without due process of law and the abatement of liability consequent upon the repeal of Republic Act No. 426. Not one of the principles of law relied upon suffices to call for reversal of the action taken by the respondent Commissioner of Customs, even if the facts presented a situation less conclusive against the pretension of petitioners-appellants. From the apprehension and seizure of the vessels in question on the high seas beyond the territorial waters of the Philippines, the absence of jurisdiction of Commissioner of Customs is predicated. Such contention of petitioners-appellants is without merit. It is unquestioned that all vessels seized are of Philippine registry. The Revised Penal Code leaves no doubt as to its applicability and enforceability not only within the Philippines, its interior waters and maritime zone, but also outside of its jurisdiction against those committing offense while on a Philippine 46

University of the Cordilleras College of Law First Year C S.Y. 2013 - 2014 ship The principle of law that sustains the validity of such a provision equally supplies a firm foundation for the seizure of the five sailing vessels found thereafter to have violated the applicable provisions of the Revised Administrative Code. Moreover, it is a well settled doctrine of International Law that goes back to Chief Justice Marshalls opinion in Church v. Hubbart,10 an 1804 decision, that a state has the right to protect itself and its revenues, a right not limited to its own territory but extending to the high seas. In the language of Chief Justice Marshall: The authority of a nation within its own territory is absolute and exclusive. The seizure of a vessel within the range of its cannon by a foreign force is an invasion of that territory, and is a hostile act which it is its duty to repel. But its power to secure itself from injury may certainly be exercised beyond the limits of its territory. The question asked in the brief of petitioners-appellants as to whether the seizure of the vessels in question and the cargoes on the high seas and thus beyond the territorial waters of the Philippines was legal must be answered in the affirmative. 4. The next question raised is the alleged denial of due process arising from such forfeiture and seizure. The argument on the alleged lack of validity of the action taken by the Commissioner of Customs is made to rest on the fact that the alleged offense imputed to petitioners-appellants is a violation of Section 1363(a) and not Section 1363(f). The title of Section 1363 is clear, Property subject to forfeiture under customs laws. The first subsection thereof, (a) cover any vessel including cargo unlawfully engaged in the importation of merchandise except a port of entry. Subsection (f) speaks of any merchandise of any prohibited importation, the importation of which is effected or attempted contrary to law and all other merchandise which in the opinion of the Collector of Customs have been used are or were intended to be used as instrument in the importation or exportation of the former. From the above recital of the legal provisions relied upon, it would appear most clearly that the due process question raised is insubstantial. Certainly, the facts on which the seizure was based were not unknown to petitioners-appellants. On those facts the liability of the vessels and merchandise under the above terms of the statute would appear to be undeniable. The action taken then by the Commissioner of Customs was in accordance with law. How could there be a denial of due process? There was nothing arbitrary about the manner in which such seizure and forfeiture were effected. The right to a hearing of petitioners-appellants was respected. They could not have been unaware of what they were doing. It would be an affront to reason if under the above circumstances they could be allowed to raise in all seriousness a due process question. Such a constitutional guaranty, basic and fundamental, certainly should not be allowed to lend itself as an instrument for escaping a liability arising from ones own nefarious acts. 5. Petitioners-appellants would further assail the validity of the action taken by the respondent Commissioner of Customs by the plea that the repeal of Republic Act No. 426 abated whatever liability could have been incurred thereunder. This argument raised before the Court of Tax Appeals was correctly held devoid of any persuasive force. The decision under review cited our opinion in Golay-Buchel & Cie v. Commissioner of Customs to the effect that the expiration of the Import Control Law did not produce the effect of declaring legal the importation of goods which were illegally imported and the seizure and forfeiture thereof as ordered by the Collector of Customs illegal or null and void. Roxas v. Sayoc announced that principle earlier. Thus: Herein, we are concerned with the effect of the expiration of a law, not with the abrogation of a law, and we hold the view that once the Commissioner of Customs has acquired jurisdiction over the case, the mere expiration of Republic Act No. 650 will not divest him of his jurisdiction thereon duly acquired while said law was still in force. In other words, we believe that despite the expiration of Republic Act No. 650 the Commissioner of Customs retained his jurisdiction over the case and could continue to take cognizance thereof until its final determination, for the main question brought in by the appeal from the decision of the Collector of Customs was the legality or illegality of the decision of the Collector of Customs, and that question could not have been abated by the mere expiration of Republic Act No. 650. We firmly believe that the expiration of Republic Act No. 47

University of the Cordilleras College of Law First Year C S.Y. 2013 - 2014 650 could not have produced the effect (1) of declaring legal the importation of the cotton counterpanes which were illegally imported, and (2) of declaring the seizure and forfeiture ordered by the Collector of Customs illegal or null and void; in other words it could not have the effect of annulling or setting aside the decision of the Collector of Customs which was rendered while the law was in force and which should stand until it is revoked by the appellate tribunal. As late as 1965, in Bombay Dept. Store v. Commissioner of Customs, we had occasion to reaffirm the doctrine in the above two decisions, the present Chief Justice, speaking for the Court, stating that such expiration of the period of effectivity of Republic Act No. 650 did not have the effect of de priving the Commissioner of Customs of the jurisdiction, acquired by him prior thereto, to act on cases of forfeiture pending before him, which are in the nature of proceeding in rem. It is thus most evident that the Court of Tax Appeals had not in any wise refused to adhere faithfully to controlling legal principles when it sustained the action taken by respondent Commissioner of Customs. It would be a reproach and a reflection on the law if on the facts as they had been shown to exist, the seizure and forfeiture of the vessels and cargo in question were to be characterized as outside the legal competence of our government and violative of the constitutional rights of petitioners-appellants. Fortunately, as had been made clear above, that would be an undeserved reflection and an unwarranted reproach. The vigor of the war against smuggling must not be hampered by a misreading of international law concepts and a misplaced reliance on a constitutional guaranty that has not in any wise been infringed. WHEREFORE, the decision of respondent Court of Tax Appeals of November 19, 1964, is affirmed. With costs against petitioners-appellants.

48

University of the Cordilleras College of Law First Year C S.Y. 2013 - 2014 Case Digest

REPEAL OF LAWS: ASAALI vs. COMMISSIONER OF CUSTOMS 26 SCRA 205 Decided On: December 16, 1968 Ponente: FERNANDO J. FACTS: Five vessels were seized in the high seas by customs carrying smuggled cigarettes and rattan chairs which is contrary to the importation law R.A. 426. Later on R.A.426 was repealed by R.A. 650. The accused then claimed that customs has no longer jurisdiction over it and that the crime committed was abated. ISSUE: Is the contention of the accused correct? HELD: No. Jurisdiction is determined by law in force at the commission of the act and continuous until the termination of the case except only when the repeal: 1. Expressly prohibit the exercise of jurisdiction. 2. When the provision penalizing the act is repealed and the act is no longer punishable. When the court failed to provide a counsel when the accused did not waive his right to counsel

49

University of the Cordilleras College of Law First Year C S.Y. 2013 - 2014 Van Dorn vs. Romillo, 139 SCRA 139 G.R. No. L-68470 October 8, 1985 Full Case ALICE REYES VAN DORN, petitioner, vs. HON. MANUEL V. ROMILLO, JR., as Presiding Judge of Branch CX, Regional Trial Court of the National Capital Region Pasay City and RICHARD UPTON respondents. MELENCIO-HERRERA, J.:\ In this Petition for certiorari and Prohibition, petitioner Alice Reyes Van Dorn seeks to set aside the Orders, dated September 15, 1983 and August 3, 1984, in Civil Case No. 1075-P, issued by respondent Judge, which denied her Motion to Dismiss said case, and her Motion for Reconsideration of the Dismissal Order, respectively. The basic background facts are that petitioner is a citizen of the Philippines while private respondent is a citizen of the United States; that they were married in Hongkong in 1972; that, after the marriage, they established their residence in the Philippines; that they begot two children born on April 4, 1973 and December 18, 1975, respectively; that the parties were divorced in Nevada, United States, in 1982; and that petitioner has re-married also in Nevada, this time to Theodore Van Dorn. Dated June 8, 1983, private respondent filed suit against petitioner in Civil Case No. 1075-P of the Regional Trial Court, Branch CXV, in Pasay City, stating that petitioners business in Ermita, Manila, (the Galleon Shop, for short), is conjugal property of the parties, and asking that petitioner be ordered to render an accounting of that business, and that private respondent be declared with right to manage the conjugal property. Petitioner moved to dismiss the case on the ground that the cause of action is barred by previous judgment in the divorce proceedings before the Nevada Court wherein respondent had acknowledged that he and petitioner had no community property as of June 11, 1982. The Court below denied the Motion to Dismiss in the mentioned case on the ground that the property involved is located in the Philippines so that the Divorce Decree has no bearing in the case. The denial is now the subject of this certiorari proceeding. Generally, the denial of a Motion to Dismiss in a civil case is interlocutory and is not subject to appeal. certiorari and Prohibition are neither the remedies to question the propriety of an interlocutory order of the trial Court. However, when a grave abuse of discretion was patently committed, or the lower Court acted capriciously and whimsically, then it devolves upon this Court in a certiorari proceeding to exercise its supervisory authority and to correct the error committed which, in such a case, is equivalent to lack of jurisdiction. Prohibition would then lie since it would be useless and a waste of time to go ahead with the proceedings. We consider the petition filed in this case within the exception, and we have given it due course. For resolution is the effect of the foreign divorce on the parties and their alleged conjugal property in the Philippines. Petitioner contends that respondent is estopped from laying claim on the alleged conjugal property because of the representation he made in the divorce proceedings before the American Court that they had no community of property; that the Galleon Shop was not established through conjugal funds, and that respondents claim is barred by prior judgment. For his part, respondent avers that the Divorce Decree issued by the Nevada Court cannot prevail over the prohibitive laws of the Philippines and its declared national policy; that the acts and declaration of a foreign Court cannot, especially if the same is contrary to public policy, divest Philippine Courts of jurisdiction to entertain matters within its jurisdiction.

50

University of the Cordilleras College of Law First Year C S.Y. 2013 - 2014 For the resolution of this case, it is not necessary to determine whether the property relations between petitioner and private respondent, after their marriage, were upon absolute or relative community property, upon complete separation of property, or upon any other regime. The pivotal fact in this case is the Nevada divorce of the parties. The Nevada District Court, which decreed the divorce, had obtained jurisdiction over petitioner who appeared in person before the Court during the trial of the case. It also obtained jurisdiction over private respondent who, giving his address as No. 381 Bush Street, San Francisco, California, authorized his attorneys in the divorce case, Karp &Gradt Ltd., to agree to the divorce on the ground of incompatibility in the understanding that there were neither community property nor community obligations. As explicitly stated in the Power of Attorney he executed in favor of the law firm of KARP & GRAD LTD., 336 W. Liberty, Reno, Nevada, to represent him in the divorce proceedings: xxx xxx xxx You are hereby authorized to accept service of Summons, to file an Answer, appear on my behalf and do an things necessary and proper to represent me, without further contesting, subject to the following: 1. That my spouse seeks a divorce on the ground of incompatibility. 2. That there is no community of property to be adjudicated by the Court. 3. Ihat there are no community obligations to be adjudicated by the court. xxx xxx xxx There can be no question as to the validity of that Nevada divorce in any of the States of the United States. The decree is binding on private respondent as an American citizen. For instance, private respondent cannot sue petitioner, as her husband, in any State of the Union. What he is contending in this case is that the divorce is not valid and binding in this jurisdiction, the same being contrary to local law and public policy. It is true that owing to the nationality principle embodied in Article 15 of the Civil Code, only Philippine nationals are covered by the policy against absolute divorces the same being considered contrary to our concept of public police and morality. However, aliens may obtain divorces abroad, which may be recognized in the Philippines, provided they are valid according to their national law. In this case, the divorce in Nevada released private respondent from the marriage from the standards of American law, under which divorce dissolves the marriage. As stated by the Federal Supreme Court of the United States in Atherton vs. Atherton, 45 L. Ed. 794, 799: The purpose and effect of a decree of divorce from the bond of matrimony by a court of competent jurisdiction are to change the existing status or domestic relation of husband and wife, and to free them both from the bond. The marriage tie when thus severed as to one party, ceases to bind either. A husband without a wife, or a wife without a husband, is unknown to the law. When the law provides, in the nature of a penalty, that the guilty party shall not marry again, that party, as well as the other, is still absolutely freed from the bond of the former marriage. Thus, pursuant to his national law, private respondent is no longer the husband of petitioner. He would have no standing to sue in the case below as petitioners husband entitled to exercise control over conjugal assets. As he is bound by the Decision of his own countrys Court, which validly exercised jurisdiction over him, and whose decision he does not repudiate, he is estopped by his own representation before said Court from asserting his right over the alleged conjugal property. To maintain, as private respondent does, that, under our laws, petitioner has to be considered still married to private respondent and still subject to a wifes obligations under Article 109, et. seq. of the Civil Code cannot be just. Petitioner should not be obliged to live together with, observe respect and fidelity, and render support to private respondent. The latter should not continue to be one of her heirs with possible 51

University of the Cordilleras College of Law First Year C S.Y. 2013 - 2014 rights to conjugal property. She should not be discriminated against in her own country if the ends of justice are to be served. WHEREFORE, the Petition is granted, and respondent Judge is hereby ordered to dismiss the Complaint filed in Civil Case No. 1075-P of his Court. Without costs. SO ORDERED.

52

University of the Cordilleras College of Law First Year C S.Y. 2013 - 2014 Case Digest NATIONALITY PRINCIPLE DIVORCE VAN DORN VS. ROMILLO JR. G.R. No. L-68470 Decided on: October 8, 1985 Ponente: MELENCIO-HERRERA, J.: FACTS: Petitioner Alice Reyes is a citizen of the Philippines while private respondent is a citizen of the United States. They were married in Hongkong. Thereafter, they established their residence in the Philippines and begot two children. Subsequently, they were divorced in Nevada, United States, and that petitioner has re-married also in Nevada, this time to Theodore Van Dorn. Private respondent filed suit against petitioner, stating that petitioners business in Manila is their conjugal property; that petitioner he ordered to render accounting of the business and that private respondent be declared to manage the conjugal property. Petitioner moved to dismiss the case contending that the cause of action is barred by the judgment in the divorce proceedings before the Nevada Court. The denial now is the subject of the certiorari proceeding. CONTENTION OF THE PETITIONER: Respondent is estopped from laying claim on the alleged conjugal property because of the representation he made in the divorce proceedings before the American Court that they had no community of property. CONTENTION OF THE RESPONDENT: The Divorce Decree issued by the Nevada Court cannot prevail over the prohibitive laws of the Philippines and its declared national policy; that the acts and declaration of a foreign Court cannot, especially if the same is contrary to public policy, divest Philippine Courts of jurisdiction to entertain matters within its jurisdiction. ISSUE: Whether or not the divorce obtained by the parties is binding only to the alien spouse. RULING: Is it true that owing to the nationality principle embodied in Article 15 of the Civil Code, only Philippine nationals are covered by the policy against absolute divorces the same being considered contrary to our concept of public policy and morality. However, aliens may obtain divorces abroad, which may be recognized in the Philippines, provided they are valid according to their national law. In this case, the divorce in Nevada released private respondent from the marriage from the standards of American Law, under which divorce dissolves the marriage. Thus, pursuant to his national law, private respondent is no longer the husband petitioner. He would have no standing to sue in the case below as petitioners husband entitled to exercise control over conjugal assets. As he is bound by the decision of his own countrys court, which validly exercised jurisdiction over him, and whose decision he does not repudiate, he is stopped by his own representation before said court from asserting his right over the alleged conjugal property.

53

University of the Cordilleras College of Law First Year C S.Y. 2013 - 2014 Pilapil vs. Ibay-Somera, 174 SCRA 653 G.R. No. 80116 June 30, 1989 Full Case IMELDA MANALAYSAY PILAPIL, petitioner, vs. HON. CORONA IBAY-SOMERA, in her capacity as Presiding Judge of the Regional Trial Court of Manila, Branch XXVI; HON. LUIS C. VICTOR, in his capacity as the City Fiscal of Manila; and ERICH EKKEHARD GEILING, respondents. REGALADO, J.: An ill-starred marriage of a Filipina and a foreigner which ended in a foreign absolute divorce, only to be followed by a criminal infidelity suit of the latter against the former, provides us the opportunity to lay down a decisional rule on what hitherto appears to be an unresolved jurisdictional question. On September 7, 1979, petitioner Imelda Manalaysay Pilapil, a Filipino citizen, and private respondent Erich Ekkehard Geiling, a German national, were married before the Registrar of Births, Marriages and Deaths at Friedensweiler in the Federal Republic of Germany. The marriage started auspiciously enough, and the couple lived together for some time in Malate, Manila where their only child, Isabella Pilapil Geiling, was born on April 20, 1980. Thereafter, marital discord set in, with mutual recriminations between the spouses, followed by a separation de facto between them. After about three and a half years of marriage, such connubial disharmony eventuated in private respondent initiating a divorce proceeding against petitioner in Germany before the Schoneberg Local Court in January, 1983. He claimed that there was failure of their marriage and that they had been living apart since April, 1982. Petitioner, on the other hand, filed an action for legal separation, support and separation of property before the Regional Trial Court of Manila, Branch XXXII, on January 23, 1983 where the same is still pending as Civil Case No. 83-15866. On January 15, 1986, Division 20 of the Schoneberg Local Court, Federal Republic of Germany, promulgated a decree of divorce on the ground of failure of marriage of the spouses. The custody of the child was granted to petitioner. The records show that under German law said court was locally and internationally competent for the divorce proceeding and that the dissolution of said marriage was legally founded on and authorized by the applicable law of that foreign jurisdiction. On June 27, 1986, or more than five months after the issuance of the divorce decree, private respondent filed two complaints for adultery before the City Fiscal of Manila alleging that, while still married to said respondent, petitioner "had an affair with a certain William Chia as early as 1982 and with yet another man named Jesus Chua sometime in 1983". Assistant Fiscal Jacinto A. de los Reyes, Jr., after the corresponding investigation, recommended the dismissal of the cases on the ground of insufficiency of evidence. However, upon review, the respondent city fiscal approved a resolution, dated January 8, 1986, directing the filing of two complaints for adultery against the petitioner. The complaints were accordingly filed and were eventually raffled to two branches of the Regional Trial Court of Manila. The case entitled "People of the Philippines vs. Imelda Pilapil and William Chia", docketed as Criminal Case No. 8752435, was assigned to Branch XXVI presided by the respondent judge; while the other case, "People of the Philippines vs. Imelda Pilapil and James Chua", docketed as Criminal Case No. 87-52434 went to the sala of Judge Leonardo Cruz, Branch XXV, of the same court. On March 14, 1987, petitioner filed a petition with the Secretary of Justice asking that the aforesaid resolution of respondent fiscal be set aside and the cases against her be dismissed. A similar petition was filed by James Chua, her co-accused in Criminal Case No. 87-52434. The Secretary of Justice, through the Chief State Prosecutor, gave due course to both petitions and directed the respondent city fiscal to

54

University of the Cordilleras College of Law First Year C S.Y. 2013 - 2014 inform the Department of Justice "if the accused have already been arraigned and if not yet arraigned, to move to defer further proceedings" and to elevate the entire records of both cases to his office for review. Petitioner thereafter filed a motion in both criminal cases to defer her arraignment and to suspend further proceedings thereon. As a consequence, Judge Leonardo Cruz suspended proceedings in Criminal Case No. 87-52434. On the other hand, respondent judge merely reset the date of the arraignment in Criminal Case No. 87-52435 to April 6, 1987. Before such scheduled date, petitioner moved for the cancellation of the arraignment and for the suspension of proceedings in said Criminal Case No. 87-52435 until after the resolution of the petition for review then pending before the Secretary of Justice. A motion to quash was also filed in the same case on the ground of lack of jurisdiction, which motion was denied by the respondent judge in an order dated September 8, 1987. The same order also directed the arraignment of both accused therein, that is, petitioner and William Chia. The latter entered a plea of not guilty while the petitioner refused to be arraigned. Such refusal of the petitioner being considered by respondent judge as direct contempt, she and her counsel were fined and the former was ordered detained until she submitted herself for arraignment. Later, private respondent entered a plea of not guilty. On October 27, 1987, petitioner filed this special civil action for certiorari and prohibition, with a prayer for a temporary restraining order, seeking the annulment of the order of the lower court denying her motion to quash. The petition is anchored on the main ground that the court is without jurisdiction "to try and decide the charge of adultery, which is a private offense that cannot be prosecuted de officio (sic), since the purported complainant, a foreigner, does not qualify as an offended spouse having obtained a final divorce decree under his national law prior to his filing the criminal complaint." On October 21, 1987, this Court issued a temporary restraining order enjoining the respondents from implementing the aforesaid order of September 8, 1987 and from further proceeding with Criminal Case No. 87-52435. Subsequently, on March 23, 1988 Secretary of Justice Sedfrey A. Ordoez acted on the aforesaid petitions for review and, upholding petitioner's ratiocinations, issued a resolution directing the respondent city fiscal to move for the dismissal of the complaints against the petitioner. We find this petition meritorious. The writs prayed for shall accordingly issue. Under Article 344 of the Revised Penal Code, the crime of adultery, as well as four other crimes against chastity, cannot be prosecuted except upon a sworn written complaint filed by the offended spouse. It has long since been established, with unwavering consistency, that compliance with this rule is a jurisdictional, and not merely a formal, requirement. While in point of strict law the jurisdiction of the court over the offense is vested in it by the Judiciary Law, the requirement for a sworn written complaint is just as jurisdictional a mandate since it is that complaint which starts the prosecutory proceeding and without which the court cannot exercise its jurisdiction to try the case. Now, the law specifically provides that in prosecutions for adultery and concubinage the person who can legally file the complaint should be the offended spouse, and nobody else. Unlike the offenses of seduction, abduction, rape and acts of lasciviousness, no provision is made for the prosecution of the crimes of adultery and concubinage by the parents, grandparents or guardian of the offended party. The so-called exclusive and successive rule in the prosecution of the first four offenses above mentioned do not apply to adultery and concubinage. It is significant that while the State, as parens patriae, was added and vested by the 1985 Rules of Criminal Procedure with the power to initiate the criminal action for a deceased or incapacitated victim in the aforesaid offenses of seduction, abduction, rape and acts of lasciviousness, in default of her parents, grandparents or guardian, such amendment did not include the crimes of adultery and concubinage. In other words, only the offended spouse, and no other, is authorized by law to initiate the action therefor. Corollary to such exclusive grant of power to the offended spouse to institute the action, it necessarily follows that such initiator must have the status, capacity or legal representation to do so at the time of the filing of the criminal action. This is a familiar and express rule in civil actions; in fact, lack of legal capacity to sue, as a ground for a motion to dismiss in civil cases, is determined as of the filing of the complaint or petition. 55

University of the Cordilleras College of Law First Year C S.Y. 2013 - 2014 The absence of an equivalent explicit rule in the prosecution of criminal cases does not mean that the same requirement and rationale would not apply. Understandably, it may not have been found necessary since criminal actions are generally and fundamentally commenced by the State, through the People of the Philippines, the offended party being merely the complaining witness therein. However, in the so-called "private crimes" or those which cannot be prosecuted de oficio, and the present prosecution for adultery is of such genre, the offended spouse assumes a more predominant role since the right to commence the action, or to refrain therefrom, is a matter exclusively within his power and option. This policy was adopted out of consideration for the aggrieved party who might prefer to suffer the outrage in silence rather than go through the scandal of a public trial. Hence, as cogently argued by petitioner, Article 344 of the Revised Penal Code thus presupposes that the marital relationship is still subsisting at the time of the institution of the criminal action for, adultery. This is a logical consequence since the raison d'etre of said provision of law would be absent where the supposed offended party had ceased to be the spouse of the alleged offender at the time of the filing of the criminal case. 21 In these cases, therefore, it is indispensable that the status and capacity of the complainant to commence the action be definitely established and, as already demonstrated, such status or capacity must indubitably exist as of the time he initiates the action. It would be absurd if his capacity to bring the action would be determined by his status before or subsequent to the commencement thereof, where such capacity or status existed prior to but ceased before, or was acquired subsequent to but did not exist at the time of, the institution of the case. We would thereby have the anomalous spectacle of a party bringing suit at the very time when he is without the legal capacity to do so. To repeat, there does not appear to be any local precedential jurisprudence on the specific issue as to when precisely the status of a complainant as an offended spouse must exist where a criminal prosecution can be commenced only by one who in law can be categorized as possessed of such status. Stated differently and with reference to the present case, the inquiry ;would be whether it is necessary in the commencement of a criminal action for adultery that the marital bonds between the complainant and the accused be unsevered and existing at the time of the institution of the action by the former against the latter. American jurisprudence, on cases involving statutes in that jurisdiction which are in parimateria with ours, yields the rule that after a divorce has been decreed, the innocent spouse no longer has the right to institute proceedings against the offenders where the statute provides that the innocent spouse shall have the exclusive right to institute a prosecution for adultery. Where, however, proceedings have been properly commenced, a divorce subsequently granted can have no legal effect on the prosecution of the criminal proceedings to a conclusion. In the cited Loftus case, the Supreme Court of Iowa held that 'No prosecution for adultery can be commenced except on the complaint of the husband or wife.' Section 4932, Code. Though Loftus was husband of defendant when the offense is said to have been committed, he had ceased to be such when the prosecution was begun; and appellant insists that his status was not such as to entitle him to make the complaint. We have repeatedly said that the offense is against the unoffending spouse, as well as the state, in explaining the reason for this provision in the statute; and we are of the opinion that the unoffending spouse must be such when the prosecution is commenced. (Emphasis supplied.) We see no reason why the same doctrinal rule should not apply in this case and in our jurisdiction, considering our statutory law and jural policy on the matter. We are convinced that in cases of such nature, the status of the complainant vis-a-vis the accused must be determined as of the time the complaint was filed. Thus, the person who initiates the adultery case must be an offended spouse, and by this is meant that he is still married to the accused spouse, at the time of the filing of the complaint. In the present case, the fact that private respondent obtained a valid divorce in his country, the Federal Republic of Germany, is admitted. Said divorce and its legal effects may be recognized in the Philippines 56

University of the Cordilleras College of Law First Year C S.Y. 2013 - 2014 insofar as private respondent is concerned in view of the nationality principle in our civil law on the matter of status of persons. Thus, in the recent case of Van Dorn vs. Romillo, Jr., et al., after a divorce was granted by a United States court between Alice Van Dorn a Filipina, and her American husband, the latter filed a civil case in a trial court here alleging that her business concern was conjugal property and praying that she be ordered to render an accounting and that the plaintiff be granted the right to manage the business. Rejecting his pretensions, this Court perspicuously demonstrated the error of such stance, thus: There can be no question as to the validity of that Nevada divorce in any of the States of the United States. The decree is binding on private respondent as an American citizen. For instance, private respondent cannot sue petitioner, as her husband, in any State of the Union. ... It is true that owing to the nationality principle embodied in Article 15 of the Civil Code, only Philippine nationals are covered by the policy against absolute divorces the same being considered contrary to our concept of public policy and morality. However, aliens may obtain divorces abroad, which may be recognized in the Philippines, provided they are valid according to their national law. ... Thus, pursuant to his national law, private respondent is no longer the husband of petitioner. He would have no standing to sue in the case below as petitioner's husband entitled to exercise control over conjugal assets... Under the same considerations and rationale, private respondent, being no longer the husband of petitioner, had no legal standing to commence the adultery case under the imposture that he was the offended spouse at the time he filed suit. The allegation of private respondent that he could not have brought this case before the decree of divorce for lack of knowledge, even if true, is of no legal significance or consequence in this case. When said respondent initiated the divorce proceeding, he obviously knew that there would no longer be a family nor marriage vows to protect once a dissolution of the marriage is decreed. Neither would there be a danger of introducing spurious heirs into the family, which is said to be one of the reasons for the particular formulation of our law on adultery, since there would thenceforth be no spousal relationship to speak of. The severance of the marital bond had the effect of dissociating the former spouses from each other, hence the actuations of one would not affect or cast obloquy on the other. The afore cited case of United States vs. Mata cannot be successfully relied upon by private respondent. In applying Article 433 of the old Penal Code, substantially the same as Article 333 of the Revised Penal Code, which punished adultery "although the marriage be afterwards declared void", the Court merely stated that "the lawmakers intended to declare adulterous the infidelity of a married woman to her marital vows, even though it should be made to appear that she is entitled to have her marriage contract declared null and void, until and unless she actually secures a formal judicial declaration to that effect". Definitely, it cannot be logically inferred therefrom that the complaint can still be filed after the declaration of nullity because such declaration that the marriage is void ab initio is equivalent to stating that it never existed. There being no marriage from the beginning, any complaint for adultery filed after said declaration of nullity would no longer have a leg to stand on. Moreover, what was consequently contemplated and within the purview of the decision in said case is the situation where the criminal action for adultery was filed before the termination of the marriage by a judicial declaration of its nullity ab initio. The same rule and requisite would necessarily apply where the termination of the marriage was effected, as in this case, by a valid foreign divorce. Private respondent's invocation of Donio-Teves, et al. vs. Vamenta, hereinbefore cited, must suffer the same fate of inapplicability. A cursory reading of said case reveals that the offended spouse therein had duly and seasonably filed a complaint for adultery, although an issue was raised as to its sufficiency but which was resolved in favor of the complainant. Said case did not involve a factual situation akin to the one at bar or any issue determinative of the controversy herein.

57

University of the Cordilleras College of Law First Year C S.Y. 2013 - 2014 WHEREFORE, the questioned order denying petitioner's motion to quash is SET ASIDE and another one entered DISMISSING the complaint in Criminal Case No. 87-52435 for lack of jurisdiction. The temporary restraining order issued in this case on October 21, 1987 is hereby made permanent. SO ORDERED.

58

University of the Cordilleras College of Law First Year C S.Y. 2013 - 2014 Case Digest NATIONALITY PRINCIPLE PILAPIL vs. HON. IBAY-SOMERA G.R. No. 80116 Decided on: June 30, 1989 Ponente: REGALADO, J.: FACTS: Petitioner Imelda Pilapil, a Filipino citizen, and private respondent Erich Geiling, a German national, were married in Germany. After about three and a half years of marriage, such connubial disharmony eventuated in Geiling initiating a divorce proceeding against Pilapil in Germany. The Local Court, Federal Republic of Germany, promulgated a decree of divorce on the ground of failure of marriage of the spouses. More than five months after the issuance of the divorce decree, Geiling filed two complaints for adultery before the City Fiscal of Manila alleging in one that, while still married to saidGeiling, Pilapil had an affair with a certain William Chia. The Assistant Fiscal, after the corresponding investigation, recommended the dismissal of the cases on the ground of insufficiency of evidence. However, upon review, the respondent city fiscal Victor approved a resolution directing the filing of 2 complaint for adultery against the petitioner. Pilapil appealed, hence this petition. CONTENTION OF THE PETITIONER: The court is without jurisdiction since the purported complainant, a foreigner, does not qualify as an offended spouse having obtained a final divorce decree under his national law prior to his filing the criminal complaint. CONTENTION OF THE RESPONDENT: Private respondent argued that he could not have brought this case before the decree of divorce for lack of knowledge. ISSUE: Did Geiling have legal capacity at the time of the filing of the complaint for adultery, considering that it was done after obtaining a divorce decree? RULING: No. The fact that private respondent obtained a valid divorce in his country, the Federal Republic of Germany, is admitted. Said divorce and its legal effects may be recognized in the Philippines insofar as private respondent is concerned in view of the nationality principle in our civil law on the matter of status of persons Under the same considerations and rationale, private respondent, being no longer the husband of petitioner, had no legal standing to commence the adultery case under the imposture that he was the offended spouse at the time he filed suit. It is true that owing to the nationality principle embodied in Article 15 of the Civil Code, only Philippine nationals are covered by the policy against absolute divorces the same being considered contrary to our concept of public policy and morality. However, aliens may obtain divorces abroad, which may be recognized in the Philippines, provided they are valid according to their national law. ... Thus, pursuant to his national law, private respondent is no longer the husband of petitioner. He would have no standing to sue in the case below as petitioner's husband entitled to exercise control over conjugal assets.

59

University of the Cordilleras College of Law First Year C S.Y. 2013 - 2014 Llorente vs Court of Appeals, 345 SCRA 592 G.R. No. 124371 November 23, 2000 Full Case PAULA T. LLORENTE, petitioner, vs. COURT OF APPEALS and ALICIA F. LLORENTE, respondents. PARDO, J.: The Case The case raises a conflict of laws issue. What is before us is an appeal from the decision of the Court of Appeals modifying that of the Regional Trial Court, Camarines Sur, Branch 35, Iriga City declaring respondent Alicia F. Llorente (herinafter referred to as "Alicia"), as co-owners of whatever property she and the deceased Lorenzo N. Llorente (hereinafter referred to as "Lorenzo") may have acquired during the twenty-five (25) years that they lived together as husband and wife. The Facts The deceased Lorenzo N. Llorente was an enlisted serviceman of the United States Navy from March 10, 1927 to September 30, 1957. On February 22, 1937, Lorenzo and petitioner Paula Llorente (hereinafter referred to as "Paula") were married before a parish priest, Roman Catholic Church, in Nabua, Camarines Sur. Before the outbreak of the Pacific War, Lorenzo departed for the United States and Paula stayed in the conjugal home in barrio Antipolo, Nabua, Camarines Sur. On November 30, 1943, Lorenzo was admitted to United States citizenship and Certificate of Naturalization No. 5579816 was issued in his favor by the United States District Court, Southern District of New York. Upon the liberation of the Philippines by the American Forces in 1945, Lorenzo was granted an accrued leave by the U. S. Navy, to visit his wife and he visited the Philippines. He discovered that his wife Paula was pregnant and was "living in" and having an adulterous relationship with his brother, Ceferino Llorente. On December 4, 1945, Paula gave birth to a boy registered in the Office of the Registrar of Nabua as "Crisologo Llorente," with the certificate stating that the child was not legitimate and the line for the fathers name was left blank. Lorenzo refused to forgive Paula and live with her. In fact, on February 2, 1946, the couple drew a written agreement to the effect that (1) all the family allowances allotted by the United States Navy as part of Lorenzos salary and all other obligations for Paulas daily maintenance and support would be suspended; (2) they would dissolve their marital union in accordance with judicial proceedings; (3) they would make a separate agreement regarding their conjugal property acquired during their marital life; and (4) Lorenzo would not prosecute Paula for her adulterous act since she voluntarily admitted her fault and agreed to separate from Lorenzo peacefully. The agreement was signed by both Lorenzo and Paula and was witnessed by Paulas father and stepmother. The agreement was notarized by Notary Public Pedro Osabel. Lorenzo returned to the United States and on November 16, 1951 filed for divorce with the Superior Court of the State of California in and for the County of San Diego. Paula was represented by counsel, John Riley, and actively participated in the proceedings. On November 27, 1951, the Superior Court of the State of California, for the County of San Diego found all factual allegations to be true and issued an interlocutory judgment of divorce.

60

University of the Cordilleras College of Law First Year C S.Y. 2013 - 2014 On December 4, 1952, the divorce decree became final. In the meantime, Lorenzo returned to the Philippines. On January 16, 1958, Lorenzo married Alicia F. Llorente in Manila. Apparently, Alicia had no knowledge of the first marriage even if they resided in the same town as Paula, who did not oppose the marriage or cohabitation. From 1958 to 1985, Lorenzo and Alicia lived together as husband and wife. Their twenty-five (25) year union produced three children, Raul, Luz and Beverly, all surnamed Llorente. On March 13, 1981, Lorenzo executed a Last Will and Testament. The will was notarized by Notary Public Salvador M. Occiano, duly signed by Lorenzo with attesting witnesses Francisco Hugo, Francisco Neibres and Tito Trajano. In the will, Lorenzo bequeathed all his property to Alicia and their three children, to wit: "(1) I give and bequeath to my wife ALICIA R. FORTUNO exclusively my residential house and lot, located at San Francisco, Nabua, Camarines Sur, Philippines, including ALL the personal properties and other movables or belongings that may be found or existing therein; "(2) I give and bequeath exclusively to my wife Alicia R. Fortuno and to my children, Raul F. Llorente, Luz F. Llorente and Beverly F. Llorente, in equal shares, all my real properties whatsoever and wheresoever located, specifically my real properties located at Barangay AroAldao, Nabua, Camarines Sur; Barangay Paloyon, Nabua, Camarines Sur; Barangay Baras, Sitio Puga, Nabua, Camarines Sur; and Barangay Paloyon, Sitio Nalilidong, Nabua, Camarines Sur; "(3) I likewise give and bequeath exclusively unto my wife Alicia R. Fortuno and unto my children, Raul F. Llorente, Luz F. Llorente and Beverly F. Llorente, in equal shares, my real properties located in Quezon City Philippines, and covered by Transfer Certificate of Title No. 188652; and my lands in Antipolo, Rizal, Philippines, covered by Transfer Certificate of Title Nos. 124196 and 165188, both of the Registry of Deeds of the province of Rizal, Philippines; "(4) That their respective shares in the above-mentioned properties, whether real or personal properties, shall not be disposed of, ceded, sold and conveyed to any other persons, but could only be sold, ceded, conveyed and disposed of by and among themselves; "(5) I designate my wife ALICIA R. FORTUNO to be the sole executor of this my Last Will and Testament, and in her default or incapacity of the latter to act, any of my children in the order of age, if of age; "(6) I hereby direct that the executor named herein or her lawful substitute should served (sic) without bond; "(7) I hereby revoke any and all my other wills, codicils, or testamentary dispositions heretofore executed, signed, or published, by me; "(8) It is my final wish and desire that if I die, no relatives of mine in any degree in the Llorentes Side should ever bother and disturb in any manner whatsoever my wife Alicia R. Fortunato and my children with respect to any real or personal properties I gave and bequeathed respectively to each one of them by virtue of this Last Will and Testament." On December 14, 1983, Lorenzo filed with the Regional Trial Court, Iriga, Camarines Sur, a petition for the probate and allowance of his last will and testament wherein Lorenzo moved that Alicia be appointed Special Administratrix of his estate. On January 18, 1984, the trial court denied the motion for the reason that the testator Lorenzo was still alive. On January 24, 1984, finding that the will was duly executed, the trial court admitted the will to probate. 61

University of the Cordilleras College of Law First Year C S.Y. 2013 - 2014 On June 11, 1985, before the proceedings could be terminated, Lorenzo died. On September 4, 1985, Paula filed with the same court a petition for letters Paula contended (1) that she was Lorenzos surviving spouse, (2) that the various property were acquired during their marriage, (3) that Lorenzos will disposed of all his property in favor of Alicia and her children, encroaching on her legitime and 1/2 share in the conjugal propertyof administration over Lorenzos estate in her favor.. On December 13, 1985, Alicia filed in the testate proceeding (Sp. Proc. No. IR-755), a petition for the issuance of letters testamentary. On October 14, 1985, without terminating the testate proceedings, the trial court gave due course to Paulas petition in Sp. Proc. No. IR-888. On November 6, 13 and 20, 1985, the order was published in the newspaper "Bicol Star". On May 18, 1987, the Regional Trial Court issued a joint decision, thus: "Wherefore, considering that this court has so found that the divorce decree granted to the late Lorenzo Llorente is void and inapplicable in the Philippines, therefore the marriage he contracted with Alicia Fortunato on January 16, 1958 at Manila is likewise void. This being so the petition of Alicia F. Llorente for the issuance of letters testamentary is denied. Likewise, she is not entitled to receive any share from the estate even if the will especially said so her relationship with Lorenzo having gained the status of paramour which is under Art. 739 (1). "On the other hand, the court finds the petition of Paula Titular Llorente, meritorious, and so declares the intrinsic disposition of the will of Lorenzo Llorente dated March 13, 1981 as void and declares her entitled as conjugal partner and entitled to one-half of their conjugal properties, and as primary compulsory heir, Paula T. Llorente is also entitled to one-third of the estate and then one-third should go to the illegitimate children, Raul, Luz and Beverly, all surname (sic) Llorente, for them to partition in equal shares and also entitled to the remaining free portion in equal shares. "Petitioner, Paula Llorente is appointed legal administrator of the estate of the deceased, Lorenzo Llorente. As such let the corresponding letters of administration issue in her favor upon her filing a bond in the amount (sic) of P100,000.00 conditioned for her to make a return to the court within three (3) months a true and complete inventory of all goods, chattels, rights, and credits, and estate which shall at any time come to her possession or to the possession of any other person for her, and from the proceeds to pay and discharge all debts, legacies and charges on the same, or such dividends thereon as shall be decreed or required by this court; to render a true and just account of her administration to the court within one (1) year, and at any other time when required by the court and to perform all orders of this court by her to be performed. "On the other matters prayed for in respective petitions for want of evidence could not be granted. "SO ORDERED." In time, Alicia filed with the trial court a motion for reconsideration of the afore quoted decision. On September 14, 1987, the trial court denied Alicias motion for reconsideration but modified its earlier decision, stating that Raul and Luz Llorente are not children "legitimate or otherwise" of Lorenzo since they were not legally adopted by him. Amending its decision of May 18, 1987, the trial court declared Beverly Llorente as the only illegitimate child of Lorenzo, entitling her to one-third (1/3) of the estate and one-third (1/3) of the free portion of the estate. On September 28, 1987, respondent appealed to the Court of Appeals. On July 31, 1995, the Court of Appeals promulgated its decision, affirming with modification the decision of the trial court in this wise: 62

University of the Cordilleras College of Law First Year C S.Y. 2013 - 2014 "WHEREFORE, the decision appealed from is hereby AFFIRMED with the MODIFICATION that Alicia is declared as co-owner of whatever properties she and the deceased may have acquired during the twenty-five (25) years of cohabitation. "SO ORDERED." On August 25, 1995, petitioner filed with the Court of Appeals a motion for reconsideration of the decision. On March 21, 1996, the Court of Appeals, denied the motion for lack of merit. Hence, this petition. The Issue Stripping the petition of its legalese and sorting through the various arguments raised,36 the issue is simple. Who are entitled to inherit from the late Lorenzo N. Llorente? We do not agree with the decision of the Court of Appeals. We remand the case to the trial court for ruling on the intrinsic validity of the will of the deceased. The Applicable Law The fact that the late Lorenzo N. Llorente became an American citizen long before and at the time of: (1) his divorce from Paula; (2) marriage to Alicia; (3) execution of his will; and (4) death, is duly established, admitted and undisputed. Thus, as a rule, issues arising from these incidents are necessarily governed by foreign law. The Civil Code clearly provides: "Art. 15. Laws relating to family rights and duties, or to the status, condition and legal capacity of persons are binding upon citizens of the Philippines, even though living abroad. "Art. 16. Real property as well as personal property is subject to the law of the country where it is situated. "However, intestate and testamentary succession, both with respect to the order of succession and to the amount of successional rights and to the intrinsic validity of testamentary provisions, shall be regulated by the national law of the person whose succession is under consideration, whatever may be the nature of the property and regardless of the country wherein said property may be found." (emphasis ours) True, foreign laws do not prove themselves in our jurisdiction and our courts are not authorized to take judicial notice of them. Like any other fact, they must be alleged and proved. While the substance of the foreign law was pleaded, the Court of Appeals did not admit the foreign law. The Court of Appeals and the trial court called to the fore the renvoi doctrine, where the case was "referred back" to the law of the decedents domicile, in this case, Philippine law. We note that while the trial court stated that the law of New York was not sufficiently proven, in the same breath it made the categorical, albeit equally unproven statement that "American law follows the domiciliary theory hence, Philippine law applies when determining the validity of Lorenzos will.38 First, there is no such thing as one American law. The "national law" indicated in Article 16 of the Civil Code cannot possibly apply to general American law. There is no such law governing the validity of testamentary provisions in the United States. Each State of the union has its own law applicable to its citizens and in force only within the State. It can therefore refer to no other than the law of the State of

63

University of the Cordilleras College of Law First Year C S.Y. 2013 - 2014 which the decedent was a resident. Second, there is no showing that the application of the renvoi doctrine is called for or required by New York State law. The trial court held that the will was intrinsically invalid since it contained dispositions in favor of Alice, who in the trial courts opinion was a mere paramour. The trial court threw the will out, leaving Alice, and her two children, Raul and Luz, with nothing. The Court of Appeals also disregarded the will. It declared Alice entitled to one half (1/2) of whatever property she and Lorenzo acquired during their cohabitation, applying Article 144 of the Civil Code of the Philippines. The hasty application of Philippine law and the complete disregard of the will, already probated as duly executed in accordance with the formalities of Philippine law, is fatal, especially in light of the factual and legal circumstances here obtaining. Validity of the Foreign Divorce In Van Dorn v. Romillo, Jr. we held that owing to the nationality principle embodied in Article 15 of the Civil Code, only Philippine nationals are covered by the policy against absolute divorces, the same being considered contrary to our concept of public policy and morality. In the same case, the Court ruled that aliens may obtain divorces abroad, provided they are valid according to their national law. Citing this landmark case, the Court held in Quita v. Court of Appeals, that once proven that respondent was no longer a Filipino citizen when he obtained the divorce from petitioner, the ruling in Van Dorn would become applicable and petitioner could "very well lose her right to inherit" from him. In Pilapil v. Ibay-Somera, we recognized the divorce obtained by the respondent in his country, the Federal Republic of Germany. There, we stated that divorce and its legal effects may be recognized in the Philippines insofar as respondent is concerned in view of the nationality principle in our civil law on the status of persons. For failing to apply these doctrines, the decision of the Court of Appeals must be reversed. We hold that the divorce obtained by Lorenzo H. Llorente from his first wife Paula was valid and recognized in this jurisdiction as a matter of comity. Now, the effects of this divorce (as to the succession to the estate of the decedent) are matters best left to the determination of the trial court. Validity of the Will The Civil Code provides: "Art. 17. The forms and solemnities of contracts, wills, and other public instruments shall be governed by the laws of the country in which they are executed. "When the acts referred to are executed before the diplomatic or consular officials of the Republic of the Philippines in a foreign country, the solemnities established by Philippine laws shall be observed in their execution." (underscoring ours) The clear intent of Lorenzo to bequeath his property to his second wife and children by her is glaringly shown in the will he executed. We do not wish to frustrate his wishes, since he was a foreigner, not covered by our laws on "family rights and duties, status, condition and legal capacity." Whether the will is intrinsically valid and who shall inherit from Lorenzo are issues best proved by foreign law which must be pleaded and proved. Whether the will was executed in accordance with the formalities required is answered by referring to Philippine law. In fact, the will was duly probated. As a guide however, the trial court should note that whatever public policy or good customs may be involved in our system of legitimes, Congress did not intend to extend the same to the succession of foreign nationals. Congress specifically left the amount of successional rights to the decedent's national law. 64

University of the Cordilleras College of Law First Year C S.Y. 2013 - 2014 Having thus ruled, we find it unnecessary to pass upon the other issues raised. The Fallo WHEREFORE, the petition is GRANTED. The decision of the Court of Appeals in CA-G. R. SP No. 17446 promulgated on July 31, 1995 is SET ASIDE. In lieu thereof, the Court REVERSES the decision of the Regional Trial Court and RECOGNIZES as VALID the decree of divorce granted in favor of the deceased Lorenzo N. Llorente by the Superior Court of the State of California in and for the County of San Diego, made final on December 4, 1952. Further, the Court REMANDS the cases to the court of origin for determination of the intrinsic validity of Lorenzo N. Llorentes will and determination of the parties successional rights allowing proof of foreign law with instructions that the trial court shall proceed with all deliberate dispatch to settle the estate of the deceased within the framework of the Rules of Court. No costs. SO ORDERED.

65

University of the Cordilleras College of Law First Year C S.Y. 2013 - 2014 Case Digest Nationality Principle Llorente vs. CA G.R. No. 124371 Decided on: November 23, 2000 Ponente: Pardo, J. FACTS: Lorenzo and petitioner Paula Llorente were married before a parish priest. Before the outbreak of war, Lorenzo departed for the United States and Paula was left at the conjugal home. Lorenzo was naturalized by the United State. After the liberation of the Philippines he went home and visited his wife to which he discovered that his wife was pregnant and was having an adulterous relationship with his brother, Ceferino Llorente. Lorenzo returned to the US and filed for divorce. Lorenzo married Alicia Llorente; they lived together for 25 years and begot 3 children. Lorenzo on his last will and testament bequeathed all his property to Alicia and their 3 children. Paula filed a petition for letters administration over Lorenzos estate. Paula contended (1) that she was Lorenzos surviving spouse, (2) that the various property were acquired during their marriage, (3) that Lorenzos will disposed of all his property in favor of Alicia and her children, encroaching on her legitime and 1/2 share in the conjugal property. The RTC ruled in favor of Paula. On appeal, the decision was modified declaring Alicia as co-owner of whatever properties they have acquired. Hence, this petition to the Supreme Court. ISSUES: Whether or not the divorce obtained by Lorenzo capacitated him to remarry. Who are entitled to inherit from the late Lorenzo Llorente? HELD: In Van Dorn vs Ramillo Jr. the Supreme Court held that owing to the nationality principle embodied in Article 15 of the Civil Code, only Philippine nationals are covered by the policy against absolute divorce. In the same case, the Court ruled that aliens may obtain divorce abroad provided that they are valid according to their national law. The Supreme Court held that divorce obtained by Lorenzo from his first wife Paula was valid and recognized in this jurisdiction as a matter of comity. The Supreme Court remanded the case to the court of origin for the determination of the intrinsic validity of Lorenzos will and determine the successional rights allowing proof of foreign law. The deceased is not covered by our laws on family rights and duties, status, condition and legal capacity since he was a foreigner.

66

University of the Cordilleras College of Law First Year C S.Y. 2013 - 2014 Roehr vs Rodriguez, 404 SCRA 495 G.R. No. 142820 June 20, 2003 Full Case WOLFGANG O. ROEHR, petitioner, vs. MARIA CARMEN D. RODRIGUEZ, HON. JUDGE JOSEFINA GUEVARA-SALONGA, Presiding Judge of Makati RTC, Branch 149, respondents. QUISUMBING, J.: At the core of the present controversy are issues of (a) grave abuse of discretion allegedly committed by public respondent and (b) lack of jurisdiction of the regional trial court, in matters that spring from a divorce decree obtained abroad by petitioner. In this special civil action for certiorari, petitioner assails (a) the order dated September 30, 1999 of public respondent Judge Josefina Guevara-Salonga, Presiding Judge of Makati Regional Trial Court, Branch 149, in Civil Case No. 96-1389 for declaration of nullity of marriage, and (b) the order dated March 31, 2000 denying his motion for reconsideration. The assailed orders partially set aside the trial courts order dismissing Civil Case No. 96-1389, for the purpose of resolving issues relating to the property settlement of the spouses and the custody of their children. Petitioner Wolfgang O. Roehr, a German citizen and resident of Germany, married private respondent Carmen Rodriguez, a Filipina, on December 11, 1980 in Hamburg, Germany. Their marriage was subsequently ratified on February 14, 1981 in Tayasan, Negros Oriental. Out of their union were born Carolynne and Alexandra Kristine on November 18, 1981 and October 25, 1987, respectively. On August 28, 1996, private respondent filed a petition for declaration of nullity of marriage before the Regional Trial Court (RTC) of Makati City. On February 6, 1997, petitioner filed a motion to dismiss, but it was denied by the trial court in its order dated May 28, 1997. On June 5, 1997, petitioner filed a motion for reconsideration, but was also denied in an order dated August 13, 1997. On September 5, 1997, petitioner filed a petition for certiorari with the Court of Appeals. On November 27, 1998, the appellate court denied the petition and remanded the case to the RTC. Meanwhile, petitioner obtained a decree of divorce from the Court of First Instance of HamburgBlankenese, promulgated on December 16, 1997. The decree provides in part: [T]he Court of First Instance, Hamburg-Blankenese, Branch 513, has ruled through Judge van Buiren of the Court of First Instance on the basis of the oral proceedings held on 4 Nov. 1997: The marriage of the Parties contracted on 11 December 1980 before the Civil Registrar of HamburgAltona is hereby dissolved. The parental custody for the children Carolynne Roehr, born 18 November 1981 Alexandra Kristine Roehr, born on 25 October 1987 is granted to the father. The litigation expenses shall be assumed by the Parties.

67

University of the Cordilleras College of Law First Year C S.Y. 2013 - 2014 In view of said decree, petitioner filed a Second Motion to Dismiss on May 20, 1999 on the ground that the trial court had no jurisdiction over the subject matter of the action or suit as a decree of divorce had already been promulgated dissolving the marriage of petitioner and private respondent. On July 14, 1999, Judge Guevara-Salonga issued an order granting petitioners motion to dismiss. Private respondent filed a Motion for Partial Reconsideration, with a prayer that the case proceed for the purpose of determining the issues of custody of children and the distribution of the properties between petitioner and private respondent. On August 18, 1999, an Opposition to the Motion for Partial Reconsideration was filed by the petitioner on the ground that there is nothing to be done anymore in the instant case as the marital tie between petitioner Wolfgang Roehr and respondent Ma. Carmen D. Rodriguez had already been severed by the decree of divorce promulgated by the Court of First Instance of Hamburg, Germany on December 16, 1997 and in view of the fact that said decree of divorce had already been recognized by the RTC in its order of July 14, 1999, through the implementation of the mandate of Article 26 of the Family Code, endowing the petitioner with the capacity to remarry under the Philippine law. On September 30, 1999, respondent judge issued the assailed order partially setting aside her order dated July 14, 1999 for the purpose of tackling the issues of property relations of the spouses as well as support and custody of their children. The pertinent portion of said order provides: Acting on the Motion for Partial Reconsideration of the Order dated July 14, 1999 filed by petitioner thru counsel which was opposed by respondent and considering that the second paragraph of Article 26 of the Family Code was included as an amendment thru Executive Order 227, to avoid the absurd situation of a Filipino as being still married to his or her alien spouse though the latter is no longer married to the Filipino spouse because he/she had obtained a divorce abroad which is recognized by his/her national law, and considering further the effects of the termination of the marriage under Article 43 in relation to Article 50 and 52 of the same Code, which include the dissolution of the property relations of the spouses, and the support and custody of their children, the Order dismissing this case is partially set aside with respect to these matters which may be ventilated in this Court. SO ORDERED. (Emphasis supplied.) Petitioner filed a timely motion for reconsideration on October 19, 1999, which was denied by respondent judge in an order dated March 31, 2000. Petitioner ascribes lack of jurisdiction of the trial court and grave abuse of discretion on the part of respondent judge. He cites as grounds for his petition the following: 1. Partially setting aside the order dated July 14, 1999 dismissing the instant case is not allowed by 1997 Rules of Civil Procedure. 2. Respondent Maria Carmen Rodriguez by her motion for Partial Reconsideration had recognized and admitted the Divorce Decision obtained by her ex-husband in Hamburg, Germany. 3. There is nothing left to be tackled by the Honorable Court as there are no conjugal assets alleged in the Petition for Annulment of Marriage and in the Divorce petition, and the custody of the children had already been awarded to Petitioner Wolfgang Roehr. Pertinent in this case before us are the following issues: 1. Whether or not respondent judge gravely abused her discretion in issuing her order dated September 30, 1999, which partially modified her order dated July 14, 1999; and 2. Whether or not respondent judge gravely abused her discretion when she assumed and retained jurisdiction over the present case despite the fact that petitioner has already obtained a divorce decree from a German court. 68

University of the Cordilleras College of Law First Year C S.Y. 2013 - 2014 On the first issue, petitioner asserts that the assailed order of respondent judge is completely inconsistent with her previous order and is contrary to Section 3, Rule 16, Rules of Civil Procedure, which provides: Sec. 3. Resolution of motion - After the hearing, the court may dismiss the action or claim, deny the motion, or order the amendment of the pleading. The court shall not defer the resolution of the motion for the reason that the ground relied upon is not indubitable. In every case, the resolution shall state clearly and distinctly the reasons therefor. (Emphasis supplied.) Petitioner avers that a courts action on a motion is limited to dismissing the action or claim, denying the motion, or ordering the amendment of the pleading. Private respondent, on her part, argues that the RTC can validly reconsider its order dated July 14, 1999 because it had not yet attained finality, given the timely filing of respondents motion for reconsideration. Pertinent to this issue is Section 3 in relation to Section 7, Rule 37 of the 1997 Rules of Civil Procedure, which provides: Sec. 3. Action upon motion for new trial or reconsideration.The trial court may set aside the judgment or final order and grant a new trial, upon such terms as may be just, or may deny the motion. If the court finds that excessive damages have been awarded or that the judgment or final order is contrary to the evidence or law, it may amend such judgment or final order accordingly. Sec. 7. Partial new trial or reconsideration.If the grounds for a motion under this Rule appear to the court to affect the issues as to only a part, or less than all of the matters in controversy, or only one, or less than all, of the parties to it, the court may order a new trial or grant reconsideration as to such issues if severable without interfering with the judgment or final order upon the rest. (Emphasis supplied.) It is clear from the foregoing rules that a judge can order a partial reconsideration of a case that has not yet attained finality. Considering that private respondent filed a motion for reconsideration within the reglementary period, the trial court's decision of July 14, 1999 can still be modified. Moreover, in Saado v. Court of Appeals, we held that the court could modify or alter a judgment even after the same has become executory whenever circumstances transpire rendering its decision unjust and inequitable, as where certain facts and circumstances justifying or requiring such modification or alteration transpired after the judgment has become final and executory and when it becomes imperative in the higher interest of justice or when supervening events warrant it. In our view, there are even more compelling reasons to do so when, as in this case, judgment has not yet attained finality. Anent the second issue, petitioner claims that respondent judge committed grave abuse of discretion when she partially set aside her order dated July 14, 1999, despite the fact that petitioner has already obtained a divorce decree from the Court of First Instance of Hamburg, Germany. In Garcia v. Recio, Van Dorn v. Romillo, Jr.,20 and Llorente v. Court of Appeals, we consistently held that a divorce obtained abroad by an alien may be recognized in our jurisdiction, provided such decree is valid according to the national law of the foreigner. Relevant to the present case is Pilapil v. Ibay-Somera, where this Court specifically recognized the validity of a divorce obtained by a German citizen in his country, the Federal Republic of Germany. We held in Pilapil that a foreign divorce and its legal effects may be recognized in the Philippines insofar as respondent is concerned in view of the nationality principle in our civil law on the status of persons. In this case, the divorce decree issued by the German court dated December 16, 1997 has not been challenged by either of the parties. In fact, save for the issue of parental custody, even the trial court recognized said decree to be valid and binding, thereby endowing private respondent the capacity to remarry. Thus, the present controversy mainly relates to the award of the custody of their two children, Carolynne and Alexandra Kristine, to petitioner. 69

University of the Cordilleras College of Law First Year C S.Y. 2013 - 2014 As a general rule, divorce decrees obtained by foreigners in other countries are recognizable in our jurisdiction, but the legal effects thereof, e.g. on custody, care and support of the children, must still be determined by our courts. Before our courts can give the effect of res judicata to a foreign judgment, such as the award of custody to petitioner by the German court, it must be shown that the parties opposed to the judgment had been given ample opportunity to do so on grounds allowed under Rule 39, Section 50 of the Rules of Court (now Rule 39, Section 48, 1997 Rules of Civil Procedure), to wit: SEC. 50. Effect of foreign judgments. - The effect of a judgment of a tribunal of a foreign country, having jurisdiction to pronounce the judgment is as follows: (a) In case of a judgment upon a specific thing, the judgment is conclusive upon the title to the thing; (b) In case of a judgment against a person, the judgment is presumptive evidence of a right as between the parties and their successors in interest by a subsequent title; but the judgment may be repelled by evidence of a want of jurisdiction, want of notice to the party, collusion, fraud, or clear mistake of law or fact. It is essential that there should be an opportunity to challenge the foreign judgment, in order for the court in this jurisdiction to properly determine its efficacy. In this jurisdiction, our Rules of Court clearly provide that with respect to actions in personam, as distinguished from actions in rem, a foreign judgment merely constitutes prima facie evidence of the justness of the claim of a party and, as such, is subject to proof to the contrary. In the present case, it cannot be said that private respondent was given the opportunity to challenge the judgment of the German court so that there is basis for declaring that judgment as res judicata with regard to the rights of petitioner to have parental custody of their two children. The proceedings in the German court were summary. As to what was the extent of private respondents participation in the proceedings in the German court, the records remain unclear. The divorce decree itself states that neither has she commented on the proceedings nor has she given her opinion to the Social Services Office. Unlike petitioner who was represented by two lawyers, private respondent had no counsel to assist her in said proceedings. More importantly, the divorce judgment was issued to petitioner by virtue of the German Civil Code provision to the effect that when a couple lived separately for three years, the marriage is deemed irrefutably dissolved. The decree did not touch on the issue as to who the offending spouse was. Absent any finding that private respondent is unfit to obtain custody of the children, the trial court was correct in setting the issue for hearing to determine the issue of parental custody, care, support and education mindful of the best interests of the children. This is in consonance with the provision in the Child and Youth Welfare Code that the childs welfare is always the paramount consideration in all questions concerning his care and custody. On the matter of property relations, petitioner asserts that public respondent exceeded the bounds of her jurisdiction when she claimed cognizance of the issue concerning property relations between petitioner and private respondent. Private respondent herself has admitted in Par. +14 of her petition for declaration of nullity of marriage dated August 26, 1996 filed with the RTC of Makati, subject of this case, that: "[p]etitioner and respondent have not acquired any conjugal or community property nor have they incurred any debts during their marriage." Herein petitioner did not contest this averment. Basic is the rule that a court shall grant relief warranted by the allegations and the proof. Given the factual admission by the parties in their pleadings that there is no property to be accounted for, respondent judge has no basis to assert jurisdiction in this case to resolve a matter no longer deemed in controversy. In sum, we find that respondent judge may proceed to determine the issue regarding the custody of the two children born of the union between petitioner and private respondent. Private respondent erred, however, in claiming cognizance to settle the matter of property relations of the parties, which is not at issue.

70

University of the Cordilleras College of Law First Year C S.Y. 2013 - 2014 WHEREFORE, the orders of the Regional Trial Court of Makati, Branch 149, issued on September 30, 1999 and March 31, 2000 are AFFIRMED with MODIFICATION. We hereby declare that the trial court has jurisdiction over the issue between the parties as to who has parental custody, including the care, support and education of the children, namely Carolynne and Alexandra Kristine Roehr. Let the records of this case be remanded promptly to the trial court for continuation of appropriate proceedings. No pronouncement as to costs. SO ORDERED. Bellosillo, (Chairman), and Callejo, Sr., JJ., concur. Austria-Martinez, J., on official leave.

71

University of the Cordilleras College of Law First Year C S.Y. 2013 - 2014 Case Digest Nationality Principle Roehr vs. Rodriguez G.R. No. 142820 Decided on: June 20, 2003 Ponente: Quisumbing, J. Facts: Petitioner Wolfgang, a German citizen and resident of Germany, married private respondent Carmen, a Filipina, on 11 December 1980 in Hamburg, Gemany. Early 1981, the marriage was ratified in Tayasan, Negros Oriental. They had two daughters, Carolyne and Alexandria Kristine. Private respondent filed a petition for the declaration of nullity of marriage before the Regional Trial Court of Makati on 28 August 1996. Petitioner filed a motion to dismiss but was denied by trial court. A motion for reconsideration was filed by private respondent but was again denied by the trial court. In 1997, petitioner obtained a decree of divorce from the Court of First Instance of HamburgBlankenese and granting the custody of the children to the father. An Opposition to the Motion for Partial Reconsideration was filed by the petitioner on the ground that there is nothing to be done anymore in the instant case as the marital tie between petitioner Wolfgang Roehr and respondent Ma. Carmen D. Rodriguez had already been severed by the decree of divorce promulgated by the Court of First Instance of Hamburg, Germany on December 16, 1997 and in view of the fact that said decree of divorce had already been recognized by the RTC in its order of July 14, 1999, through the implementation of the mandate of Article 26 of the Family Code,10 endowing the petitioner with the capacity to remarry under the Philippine law. It was June 14, 1999 when public respondent issued an order granting the petitioners motion to dismiss, but was partially set aside on September 1999 for the purpose of tackling issues regarding property relations of the spouses as well as support and custody of their children. Petitioner assailed for the trial courts lack of jurisdiction, and grave abuse of discretion on the part of the respondent judge. Issue: Whether or not the Philippine courts can determine the legal effects of a decree of divorce from a foreign country. Held: Yes. Our courts can determine the legal effects of a divorce obtained from a foreign country such as those concerning with support and custody of the children. In this case, the decree did not touch as to who the offending spouse was. The trial court was correct in setting the issue for hearing to determine the issue of parental custody, care, support and education of the best interests of the children. After all, the childs welfare is always the paramount consideration in all questions concerning his care and custody. WHEREFORE, the orders of the Regional Trial Court of Makati, Branch 149, issued on September 30, 1999 and March 31, 2000 are AFFIRMED with MODIFICATION. We hereby declare that the trial court has jurisdiction over the issue between the parties as to who has parental custody, including the care, support and education of the children, namely Carolyne and Alexandra Kristine Roehr. Let the records of this case be remanded promptly to the trial court for continuation of appropriate proceedings. No pronouncement as to costs. SO ORDERED.

72

University of the Cordilleras College of Law First Year C S.Y. 2013 - 2014 Aznar vs. Garcia, 7 SCRA 95 G.R. No. L-16749 January 31, 1963 Full Case ADOLFO C. AZNAR, Executor and LUCY CHRISTENSEN, Heir of the deceased, Executor and Heirappellees, vs. HELEN CHRISTENSEN GARCIA, oppositor-appellant. LABRADOR, J.: This is an appeal from a decision of the Court of First Instance of Davao, Hon. Vicente N. Cusi, Jr., presiding, in Special Proceeding No. 622 of said court, dated September 14, 1949, approving among things the final accounts of the executor, directing the executor to reimburse Maria Lucy Christensen the amount of P3,600 paid by her to Helen Christensen Garcia as her legacy, and declaring Maria Lucy Christensen entitled to the residue of the property to be enjoyed during her lifetime, and in case of death without issue, one-half of said residue to be payable to Mrs. Carrie Louise C. Borton, etc., in accordance with the provisions of the will of the testator Edward E. Christensen. The will was executed in Manila on March 5, 1951 and contains the following provisions: 3. I declare ... that I have but ONE (1) child, named MARIA LUCY CHRISTENSEN (now Mrs. Bernard Daney), who was born in the Philippines about twenty-eight years ago, and who is now residing at No. 665 Rodger Young Village, Los Angeles, California, U.S.A. 4. I further declare that I now have no living ascendants, and no descendants except my above named daughter, MARIA LUCY CHRISTENSEN DANEY. xxx xxx xxx

7. I give, devise and bequeath unto MARIA HELEN CHRISTENSEN, now married to Eduardo Garcia, about eighteen years of age and who, notwithstanding the fact that she was baptized Christensen, is not in any way related to me, nor has she been at any time adopted by me, and who, from all information I have now resides in Egpit, Digos, Davao, Philippines, the sum of THREE THOUSAND SIX HUNDRED PESOS (P3,600.00), Philippine Currency the same to be deposited in trust for the said Maria Helen Christensen with the Davao Branch of the Philippine National Bank, and paid to her at the rate of One Hundred Pesos (P100.00), Philippine Currency per month until the principal thereof as well as any interest which may have accrued thereon, is exhausted.. xxx xxx xxx

12. I hereby give, devise and bequeath, unto my well-beloved daughter, the said MARIA LUCY CHRISTENSEN DANEY (Mrs. Bernard Daney), now residing as aforesaid at No. 665 Rodger Young Village, Los Angeles, California, U.S.A., all the income from the rest, remainder, and residue of my property and estate, real, personal and/or mixed, of whatsoever kind or character, and wheresoever situated, of which I may be possessed at my death and which may have come to me from any source whatsoever, during her lifetime: .... It is in accordance with the above-quoted provisions that the executor in his final account and project of partition ratified the payment of only P3,600 to Helen Christensen Garcia and proposed that the residue of the estate be transferred to his daughter, Maria Lucy Christensen. Opposition to the approval of the project of partition was filed by Helen Christensen Garcia, insofar as it deprives her (Helen) of her legitime as an acknowledged natural child, she having been declared by Us in G.R. Nos. L-11483-84 an acknowledged natural child of the deceased Edward E. Christensen. The legal grounds of opposition are (a) that the distribution should be governed by the laws of the Philippines, and 73

University of the Cordilleras College of Law First Year C S.Y. 2013 - 2014 (b) that said order of distribution is contrary thereto insofar as it denies to Helen Christensen, one of two acknowledged natural children, one-half of the estate in full ownership. In amplification of the above grounds it was alleged that the law that should govern the estate of the deceased Christensen should not be the internal law of California alone, but the entire law thereof because several foreign elements are involved, that the forum is the Philippines and even if the case were decided in California, Section 946 of the California Civil Code, which requires that the domicile of the decedent should apply, should be applicable. It was also alleged that Maria Helen Christensen having been declared an acknowledged natural child of the decedent, she is deemed for all purposes legitimate from the time of her birth. The court below ruled that as Edward E. Christensen was a citizen of the United States and of the State of California at the time of his death, the successional rights and intrinsic validity of the provisions in his will are to be governed by the law of California, in accordance with which a testator has the right to dispose of his property in the way he desires, because the right of absolute dominion over his property is sacred and inviolable (In re McDaniel's Estate, 77 Cal. Appl. 2d 877, 176 P. 2d 952, and In re Kaufman, 117 Cal. 286, 49 Pac. 192, cited in page 179, Record on Appeal). Oppositor Maria Helen Christensen, through counsel, filed various motions for reconsideration, but these were denied. Hence, this appeal. The most important assignments of error are as follows: I THE LOWER COURT ERRED IN IGNORING THE DECISION OF THE HONORABLE SUPREME COURT THAT HELEN IS THE ACKNOWLEDGED NATURAL CHILD OF EDWARD E. CHRISTENSEN AND, CONSEQUENTLY, IN DEPRIVING HER OF HER JUST SHARE IN THE INHERITANCE. II THE LOWER COURT ERRED IN ENTIRELY IGNORING AND/OR FAILING TO RECOGNIZE THE EXISTENCE OF SEVERAL FACTORS, ELEMENTS AND CIRCUMSTANCES CALLING FOR THE APPLICATION OF INTERNAL LAW. III THE LOWER COURT ERRED IN FAILING TO RECOGNIZE THAT UNDER INTERNATIONAL LAW, PARTICULARLY UNDER THE RENVOI DOCTRINE, THE INTRINSIC VALIDITY OF THE TESTAMENTARY DISPOSITION OF THE DISTRIBUTION OF THE ESTATE OF THE DECEASED EDWARD E. CHRISTENSEN SHOULD BE GOVERNED BY THE LAWS OF THE PHILIPPINES. IV THE LOWER COURT ERRED IN NOT DECLARING THAT THE SCHEDULE OF DISTRIBUTION SUBMITTED BY THE EXECUTOR IS CONTRARY TO THE PHILIPPINE LAWS. V THE LOWER COURT ERRED IN NOT DECLARING THAT UNDER THE PHILIPPINE LAWS HELEN CHRISTENSEN GARCIA IS ENTITLED TO ONEHALF (1/2) OF THE ESTATE IN FULL OWNERSHIP. There is no question that Edward E. Christensen was a citizen of the United States and of the State of California at the time of his death. But there is also no question that at the time of his death he was domiciled in the Philippines, as witness the following facts admitted by the executor himself in appellee's brief:

74

University of the Cordilleras College of Law First Year C S.Y. 2013 - 2014 In the proceedings for admission of the will to probate, the facts of record show that the deceased Edward E. Christensen was born on November 29, 1875 in New York City, N.Y., U.S.A.; his first arrival in the Philippines, as an appointed school teacher, was on July 1, 1901, on board the U.S. Army Transport "Sheridan" with Port of Embarkation as the City of San Francisco, in the State of California, U.S.A. He stayed in the Philippines until 1904. In December, 1904, Mr. Christensen returned to the United States and stayed there for the following nine years until 1913, during which time he resided in, and was teaching school in Sacramento, California. Mr. Christensen's next arrival in the Philippines was in July of the year 1913. However, in 1928, he again departed the Philippines for the United States and came back here the following year, 1929. Some nine years later, in 1938, he again returned to his own country, and came back to the Philippines the following year, 1939. Wherefore, the parties respectfully pray that the foregoing stipulation of facts be admitted and approved by this Honorable Court, without prejudice to the parties adducing other evidence to prove their case not covered by this stipulation of facts. 1wph1.t Being an American citizen, Mr. Christensen was interned by the Japanese Military Forces in the Philippines during World War II. Upon liberation, in April 1945, he left for the United States but returned to the Philippines in December, 1945. Appellees Collective Exhibits "6", CFI Davao, Sp. Proc. 622, as Exhibits "AA", "BB" and "CC-Daney"; Exhs. "MM", "MM-l", "MM-2-Daney" and p. 473, t.s.n., July 21, 1953.) In April, 1951, Edward E. Christensen returned once more to California shortly after the making of his last will and testament (now in question herein) which he executed at his lawyers' offices in Manila on March 5, 1951. He died at the St. Luke's Hospital in the City of Manila on April 30, 1953. (pp. 2-3) In arriving at the conclusion that the domicile of the deceased is the Philippines, we are persuaded by the fact that he was born in New York, migrated to California and resided there for nine years, and since he came to the Philippines in 1913 he returned to California very rarely and only for short visits (perhaps to relatives), and considering that he appears never to have owned or acquired a home or properties in that state, which would indicate that he would ultimately abandon the Philippines and make home in the State of California. Sec. 16. Residence is a term used with many shades of meaning from mere temporary presence to the most permanent abode. Generally, however, it is used to denote something more than mere physical presence. (Goodrich on Conflict of Laws, p. 29) As to his citizenship, however, We find that the citizenship that he acquired in California when he resided in Sacramento, California from 1904 to 1913, was never lost by his stay in the Philippines, for the latter was a territory of the United States (not a state) until 1946 and the deceased appears to have considered himself as a citizen of California by the fact that when he executed his will in 1951 he declared that he was a citizen of that State; so that he appears never to have intended to abandon his California citizenship by acquiring another. This conclusion is in accordance with the following principle expounded by Goodrich in his Conflict of Laws. The terms "'residence" and "domicile" might well be taken to mean the same thing, a place of permanent abode. But domicile, as has been shown, has acquired a technical meaning. Thus one may be domiciled in a place where he has never been. And he may reside in a place where he has no domicile. The man with two homes, between which he divides his time, certainly resides in each one, while living in it. But if he went on business which would require his presence for several weeks or months, he might properly be said to have sufficient connection with the place to be called a resident. It is clear, however, that, if he treated his settlement as continuing only for the particular business in hand, not giving up his former "home," he could not be a domiciled New Yorker. Acquisition of a domicile of choice requires the exercise of intention as well as physical presence. "Residence simply requires bodily presence of an inhabitant in a given place, while domicile requires bodily presence in that place and also an intention to 75

University of the Cordilleras College of Law First Year C S.Y. 2013 - 2014 make it one's domicile." Residence, however, is a term used with many shades of meaning, from the merest temporary presence to the most permanent abode, and it is not safe to insist that any one use et the only proper one. (Goodrich, p. 29) The law that governs the validity of his testamentary dispositions is defined in Article 16 of the Civil Code of the Philippines, which is as follows: ART. 16. Real property as well as personal property is subject to the law of the country where it is situated. However, intestate and testamentary successions, both with respect to the order of succession and to the amount of successional rights and to the intrinsic validity of testamentary provisions, shall be regulated by the national law of the person whose succession is under consideration, whatever may be the nature of the property and regardless of the country where said property may be found. The application of this article in the case at bar requires the determination of the meaning of the term "national law" is used therein. There is no single American law governing the validity of testamentary provisions in the United States, each state of the Union having its own private law applicable to its citizens only and in force only within the state. The "national law" indicated in Article 16 of the Civil Code above quoted can not, therefore, possibly mean or apply to any general American law. So it can refer to no other than the private law of the State of California. The next question is: What is the law in California governing the disposition of personal property? The decision of the court below, sustains the contention of the executor-appellee that under the California Probate Code, a testator may dispose of his property by will in the form and manner he desires, citing the case of Estate of McDaniel, 77 Cal. Appl. 2d 877, 176 P. 2d 952. But appellant invokes the provisions of Article 946 of the Civil Code of California, which is as follows: If there is no law to the contrary, in the place where personal property is situated, it is deemed to follow the person of its owner, and is governed by the law of his domicile. The existence of this provision is alleged in appellant's opposition and is not denied. We have checked it in the California Civil Code and it is there. Appellee, on the other hand, relies on the case cited in the decision and testified to by a witness. (Only the case of Kaufman is correctly cited.) It is argued on executor's behalf that as the deceased Christensen was a citizen of the State of California, the internal law thereof, which is that given in the abovecited case, should govern the determination of the validity of the testamentary provisions of Christensen's will, such law being in force in the State of California of which Christensen was a citizen. Appellant, on the other hand, insists that Article 946 should be applicable, and in accordance therewith and following the doctrine of the renvoi, the question of the validity of the testamentary provision in question should be referred back to the law of the decedent's domicile, which is the Philippines. The theory of doctrine of renvoi has been defined by various authors, thus: The problem has been stated in this way: "When the Conflict of Laws rule of the forum refers a jural matter to a foreign law for decision, is the reference to the purely internal rules of law of the foreign system; i.e., to the totality of the foreign law minus its Conflict of Laws rules?" On logic, the solution is not an easy one. The Michigan court chose to accept the renvoi, that is, applied the Conflict of Laws rule of Illinois which referred the matter back to Michigan law. But once having determined the the Conflict of Laws principle is the rule looked to, it is difficult to see why the reference back should not have been to Michigan Conflict of Laws. This would have resulted in the "endless chain of references" which has so often been criticized be legal writers. The opponents of the renvoi would have looked merely to the internal law of Illinois, thus rejecting the renvoi or the reference back. Yet there seems no compelling logical reason why the original reference should be the internal law rather than to 76

University of the Cordilleras College of Law First Year C S.Y. 2013 - 2014 the Conflict of Laws rule. It is true that such a solution avoids going on a merry-go-round, but those who have accepted the renvoi theory avoid this inextricabilis circulas by getting off at the second reference and at that point applying internal law. Perhaps the opponents of the renvoi are a bit more consistent for they look always to internal law as the rule of reference. Strangely enough, both the advocates for and the objectors to the renvoi plead that greater uniformity will result from adoption of their respective views. And still more strange is the fact that the only way to achieve uniformity in this choice-of-law problem is if in the dispute the two states whose laws form the legal basis of the litigation disagree as to whether the renvoi should be accepted. If both reject, or both accept the doctrine, the result of the litigation will vary with the choice of the forum. In the case stated above, had the Michigan court rejected the renvoi, judgment would have been against the woman; if the suit had been brought in the Illinois courts, and they too rejected the renvoi, judgment would be for the woman. The same result would happen, though the courts would switch with respect to which would hold liability, if both courts accepted the renvoi. The Restatement accepts the renvoi theory in two instances: where the title to land is in question, and where the validity of a decree of divorce is challenged. In these cases the Conflict of Laws rule of the situs of the land, or the domicile of the parties in the divorce case, is applied by the forum, but any further reference goes only to the internal law. Thus, a person's title to land, recognized by the situs, will be recognized by every court; and every divorce, valid by the domicile of the parties, will be valid everywhere. (Goodrich, Conflict of Laws, Sec. 7, pp. 13-14.) X, a citizen of Massachusetts, dies intestate, domiciled in France, leaving movable property in Massachusetts, England, and France. The question arises as to how this property is to be distributed among X's next of kin. Assume (1) that this question arises in a Massachusetts court. There the rule of the conflict of laws as to intestate succession to movables calls for an application of the law of the deceased's last domicile. Since by hypothesis X's last domicile was France, the natural thing for the Massachusetts court to do would be to turn to French statute of distributions, or whatever corresponds thereto in French law, and decree a distribution accordingly. An examination of French law, however, would show that if a French court were called upon to determine how this property should be distributed, it would refer the distribution to the national law of the deceased, thus applying the Massachusetts statute of distributions. So on the surface of things the Massachusetts court has open to it alternative course of action: (a) either to apply the French law is to intestate succession, or (b) to resolve itself into a French court and apply the Massachusetts statute of distributions, on the assumption that this is what a French court would do. If it accepts the socalled renvoi doctrine, it will follow the latter course, thus applying its own law. This is one type of renvoi. A jural matter is presented which the conflict-of-laws rule of the forum refers to a foreign law, the conflict-of-laws rule of which, in turn, refers the matter back again to the law of the forum. This is renvoi in the narrower sense. The German term for this judicial process is 'Ruckverweisung.'" (Harvard Law Review, Vol. 31, pp. 523-571.) After a decision has been arrived at that a foreign law is to be resorted to as governing a particular case, the further question may arise: Are the rules as to the conflict of laws contained in such foreign law also to be resorted to? This is a question which, while it has been considered by the courts in but a few instances, has been the subject of frequent discussion by textwriters and essayists; and the doctrine involved has been descriptively designated by them as the "Renvoyer" to send back, or the "Ruchversweisung", or the "Weiterverweisung", since an affirmative answer to the question postulated and the operation of the adoption of the foreign law in toto would in many cases result in returning the main controversy to be decided according to the law of the forum. ... (16 C.J.S. 872.) Another theory, known as the "doctrine of renvoi", has been advanced. The theory of the doctrine of renvoi is that the court of the forum, in determining the question before it, must take into account the whole law of the other jurisdiction, but also its rules as to conflict of laws, and then apply the law to the

77

University of the Cordilleras College of Law First Year C S.Y. 2013 - 2014 actual question which the rules of the other jurisdiction prescribe. This may be the law of the forum. The doctrine of the renvoi has generally been repudiated by the American authorities. (2 Am. Jur. 296) The scope of the theory of renvoi has also been defined and the reasons for its application in a country explained by Prof. Lorenzen in an article in the Yale Law Journal, Vol. 27, 1917-1918, pp. 529-531. The pertinent parts of the article are quoted herein below: The recognition of the renvoi theory implies that the rules of the conflict of laws are to be understood as incorporating not only the ordinary or internal law of the foreign state or country, but its rules of the conflict of laws as well. According to this theory 'the law of a country' means the whole of its law. xxx xxx xxx

Von Bar presented his views at the meeting of the Institute of International Law, at Neuchatel, in 1900, in the form of the following theses: (1) Every court shall observe the law of its country as regards the application of foreign laws. (2) Provided that no express provision to the contrary exists, the court shall respect: (a) The provisions of a foreign law which disclaims the right to bind its nationals abroad as regards their personal statute, and desires that said personal statute shall be determined by the law of the domicile, or even by the law of the place where the act in question occurred. (b) The decision of two or more foreign systems of law, provided it be certain that one of them is necessarily competent, which agree in attributing the determination of a question to the same system of law. xxx xxx xxx

If, for example, the English law directs its judge to distribute the personal estate of an Englishman who has died domiciled in Belgium in accordance with the law of his domicile, he must first inquire whether the law of Belgium would distribute personal property upon death in accordance with the law of domicile, and if he finds that the Belgian law would make the distribution in accordance with the law of nationality that is the English law he must accept this reference back to his own law. We note that Article 946 of the California Civil Code is its conflict of laws rule, while the rule applied in In re Kaufman, Supra, its internal law. If the law on succession and the conflict of laws rules of California are to be enforced jointly, each in its own intended and appropriate sphere, the principle cited In re Kaufman should apply to citizens living in the State, but Article 946 should apply to such of its citizens as are not domiciled in California but in other jurisdictions. The rule laid down of resorting to the law of the domicile in the determination of matters with foreign element involved is in accord with the general principle of American law that the domiciliary law should govern in most matters or rights which follow the person of the owner. When a man dies leaving personal property in one or more states, and leaves a will directing the manner of distribution of the property, the law of the state where he was domiciled at the time of his death will be looked to in deciding legal questions about the will, almost as completely as the law of situs is consulted in questions about the devise of land. It is logical that, since the domiciliary rules control devolution of the personal estate in case of intestate succession, the same rules should determine the validity of an attempted testamentary dispostion of the property. Here, also, it is not that the domiciliary has effect beyond the borders of the domiciliary state. The rules of the domicile are recognized as controlling by the Conflict of Laws rules at the situs property, and the reason for the recognition as in the case of intestate succession, is the general convenience of the doctrine. The New York court has said on the point: 'The general principle that a dispostiton of a personal property, valid at the domicile of the owner, is valid anywhere, is one of the universal application. It had its origin in that international comity which was one of the first fruits of civilization, and it this age, when business intercourse and the process of

78

University of the Cordilleras College of Law First Year C S.Y. 2013 - 2014 accumulating property take but little notice of boundary lines, the practical wisdom and justice of the rule is more apparent than ever. (Goodrich, Conflict of Laws, Sec. 164, pp. 442-443.) Appellees argue that what Article 16 of the Civil Code of the Philippines pointed out as the national law is the internal law of California. But as above explained the laws of California have prescribed two sets of laws for its citizens, one for residents therein and another for those domiciled in other jurisdictions. Reason demands that We should enforce the California internal law prescribed for its citizens residing therein, and enforce the conflict of laws rules for the citizens domiciled abroad. If we must enforce the law of California as in comity we are bound to go, as so declared in Article 16 of our Civil Code, then we must enforce the law of California in accordance with the express mandate thereof and as above explained, i.e., apply the internal law for residents therein, and its conflict-of-laws rule for those domiciled abroad. It is argued on appellees' behalf that the clause "if there is no law to the contrary in the place where the property is situated" in Sec. 946 of the California Civil Code refers to Article 16 of the Civil Code of the Philippines and that the law to the contrary in the Philippines is the provision in said Article 16 that the national law of the deceased should govern. This contention can not be sustained. As explained in the various authorities cited above the national law mentioned in Article 16 of our Civil Code is the law on conflict of laws in the California Civil Code, i.e., Article 946, which authorizes the reference or return of the question to the law of the testator's domicile. The conflict of laws rule in California, Article 946, Civil Code, precisely refers back the case, when a decedent is not domiciled in California, to the law of his domicile, the Philippines in the case at bar. The court of the domicile can not and should not refer the case back to California; such action would leave the issue incapable of determination because the case will then be like a football, tossed back and forth between the two states, between the country of which the decedent was a citizen and the country of his domicile. The Philippine court must apply its own law as directed in the conflict of laws rule of the state of the decedent, if the question has to be decided, especially as the application of the internal law of California provides no legitime for children while the Philippine law, Arts. 887(4) and 894, Civil Code of the Philippines, makes natural children legally acknowledged forced heirs of the parent recognizing them. The Philippine cases (In re Estate of Johnson, 39 Phil. 156; Riera vs. Palmaroli, 40 Phil. 105; Miciano vs. Brimo, 50 Phil. 867; Babcock Templeton vs. Rider Babcock, 52 Phil. 130; and Gibbs vs. Government, 59 Phil. 293.) cited by appellees to support the decision can not possibly apply in the case at bar, for two important reasons, i.e., the subject in each case does not appear to be a citizen of a state in the United States but with domicile in the Philippines, and it does not appear in each case that there exists in the state of which the subject is a citizen, a law similar to or identical with Art. 946 of the California Civil Code. We therefore find that as the domicile of the deceased Christensen, a citizen of California, is the Philippines, the validity of the provisions of his will depriving his acknowledged natural child, the appellant, should be governed by the Philippine Law, the domicile, pursuant to Art. 946 of the Civil Code of California, not by the internal law of California.. WHEREFORE, the decision appealed from is hereby reversed and the case returned to the lower court with instructions that the partition be made as the Philippine law on succession provides. Judgment reversed, with costs against appellees.

79

University of the Cordilleras College of Law First Year C S.Y. 2013 - 2014 Case Digest Nationality Principle Internal and Conflict Rule Aznar vs Garcia G.R. No. L-16749 Decided on: January 31, 1963 Ponente: LABRADOR, J.: FACTS: Edward Christensen, though born in New York, migrated to California, where he resided (and consequently was considered a California citizen) for a period of 9 years. In 1913, he came to the Philippines where he became a domiciliary until the time of his death . However, during the entire period of his residence in this country he had always considered himself a citizen of California. In his will executed on March 5, 1951, he instituted an acknowledged natural daughter, Maria Lucy Christensen as her only heir, but left a legacy of sum of money in favor of Helen Christensen Garcia (who in a decision rendered by the Supreme Court had been declared another acknowledged natural daughter of his). Counsel for the acknowledged natural daughter Helen Claims that under Art. 16, par. 2 of the Civil Code, California law should be applied; that under California law, the matter is referred back to the law of the domicile; that therefore Philippine law is ultimately applicable; that finally, the share of Helen must be increased in view of the successional rights of illegitimate children under Philippine law. Upon the other hand, consel for the child Maria Lucy contends that inasmuch as it is clear that under Art. 16 par.2 of our Civil Code, the national law of the deceased must apply, our courts must immediately apply the internal law of California on the matter, that under California law there are no compulsory heirs and consequently a testator could dispose of any property possessed by him in absolute dominion and that finally, illegitimate children not being entitled to anything under california law, the will of the deceased giving the bulk of the property to Maria Lucy must remain undisturbed. ISSUE: What law should govern? Philippine law or California law? HELD: WHEREFORE, the decision apealed from is hereby reversed and the case returned to the lower court with instructions that the partition be made as the Philippine law on succession provides. The law that governs the validity of his testamentary dispositions is defined in Article 16 of the Civil Code of the Philippines, which is as follows: ART. 16. Real property as well as personal property is subject to the law of the country where it is situated. However, intestate and testamentary successions, both with respect to the order of succession and to the amount of successional rights and to the intrinsic validity of testamentary provisions, shall be regulated by the national law of the person whose succession is under consideration, whatever may be the nature of the property and regardless of the country where said property may be found. The application of this article in the case at bar requires the determination of the meaning of the term national law is used therein. The next question is: What is the law in California governing the disposition of personal property? The decision of CFI Davao, sustains the contention of the executor-appellee that under the California Probate Code, a testator may dispose of his property by will in the form and manner he desires. But HELEN invokes the provisions of Article 946 of the Civil Code of California, which is as follows: If there is no law to the contrary, in the place where personal property is situated, it is deemed to follow the person of its owner, and is governed by the law of his domicile. 80

University of the Cordilleras College of Law First Year C S.Y. 2013 - 2014 It is argued on executors behalf that as the deceased Christensen was a citizen of the State of California, the internal law thereof, which is that given in the Kaufman case, should govern the determination of the validity of the testamentary provisions of Christensens will, such law being in force in the State of California of which Christensen was a citizen. Appellant, on the other hand, insists that Article 946 should be applicable, and in accordance therewith and following the doctrine of the renvoi, the question of the validity of the testamentary provision in question should be referred back to the law of the decedents domicile, which is the Philippines. We note that Article 946 of the California Civil Code is its conflict of laws rule, while the rule applied in In re Kaufman, its internal law. If the law on succ ession and the conflict of laws rules of California are to be enforced jointly, each in its own intended and appropriate sphere, the principle cited In re Kaufman should apply to citizens living in the State, but Article 946 should apply to such of its citizens as are not domiciled in California but in other jurisdictions. The rule laid down of resorting to the law of the domicile in the determination of matters with foreign element involved is in accord with the general principle of American law that the domiciliary law should govern in most matters or rights which follow the person of the owner. Appellees argue that what Article 16 of the Civil Code of the Philippines pointed out as the national law is the internal law of California. But as above explained the laws of California have prescribed two sets of laws for its citizens, one for residents therein and another for those domiciled in other jurisdictions. It is argued on appellees (Aznar and LUCY) behalf that the clause if there is no law to the contrary in the place where the property is situated in Sec. 946 of the California Civil Code refers to Article 16 of the Civil Code of the Philippines and that the law to the contrary in the Philippines is the provision in said Article 16 that the national law of the deceased should govern. This contention cannot be sustained. As explained in the various authorities cited above, the national law mentioned in Article 16 of our Civil Code is the law on conflict of laws in the California Civil Code, i.e., Article 946, which authorizes the reference or return of the question to the law of the testators domicile. The conflict of laws rule in California, Article 946, Civil Code, precisely refers back the case, when a decedent is not domiciled in California, to the law of his domicile, the Philippines in the case at bar. The court of the domicile cannot and should not refer the case back to California; such action would leave the issue incapable of determination because the case will then be like a football, tossed back and forth between the two states, between the country of which the decedent was a citizen and the country of his domicile. The Philippine court must apply its own law as directed in the conflict of laws rule of the state of the decedent, if the question has to be decided, especially as the application of the internal law of California provides no legitime for children while the Philippine law, Arts. 887(4) and 894, Civil Code of the Philippines, makes natural children legally acknowledged forced heirs of the parent recognizing them. We therefore find that as the domicile of the deceased Edward, a citizen of California, is the Philippines, the validity of the provisions of his will depriving his acknowledged natural child, the appellant HELEN, should be governed by the Philippine Law, the domicile, pursuant to Art. 946 of the Civil Code of California, not by the internal law of California..

81

University of the Cordilleras College of Law First Year C S.Y. 2013 - 2014 Bellis vs. Bellis, 20 SCRA 358 G.R. No. L-23678 June 6, 1967 Full Case TESTATE ESTATE OF AMOS G. BELLIS, deceased. PEOPLE'S BANK and TRUST COMPANY, executor. MARIA CRISTINA BELLIS and MIRIAM PALMA BELLIS, oppositors-appellants, vs. EDWARD A. BELLIS, ET AL., heirs-appellees. Vicente R. Macasaet and Jose D. Villena for oppositors appellants. Paredes, Poblador, Cruz and Nazareno for heirs-appellees E. A. Bellis, et al. Quijano and Arroyo for heirs-appellees W. S. Bellis, et al. J. R. Balonkita for appellee People's Bank & Trust Company. Ozaeta, Gibbs and Ozaeta for appellee A. B. Allsman. BENGZON, J.P., J.: This is a direct appeal to Us, upon a question purely of law, from an order of the Court of First Instance of Manila dated April 30, 1964, approving the project of partition filed by the executor in Civil Case No. 37089 therein.1wph1.t The facts of the case are as follows: Amos G. Bellis, born in Texas, was "a citizen of the State of Texas and of the United States." By his first wife, Mary E. Mallen, whom he divorced, he had five legitimate children: Edward A. Bellis, George Bellis (who pre-deceased him in infancy), Henry A. Bellis, Alexander Bellis and Anna Bellis Allsman; by his second wife, Violet Kennedy, who survived him, he had three legitimate children: Edwin G. Bellis, Walter S. Bellis and Dorothy Bellis; and finally, he had three illegitimate children: Amos Bellis, Jr., Maria Cristina Bellis and Miriam Palma Bellis. On August 5, 1952, Amos G. Bellis executed a will in the Philippines, in which he directed that after all taxes, obligations, and expenses of administration are paid for, his distributable estate should be divided, in trust, in the following order and manner: (a) $240,000.00 to his first wife, Mary E. Mallen; (b) P120,000.00 to his three illegitimate children, Amos Bellis, Jr., Maria Cristina Bellis, Miriam Palma Bellis, or P40,000.00 each and (c) after the foregoing two items have been satisfied, the remainder shall go to his seven surviving children by his first and second wives, namely: Edward A. Bellis, Henry A. Bellis, Alexander Bellis and Anna Bellis Allsman, Edwin G. Bellis, Walter S. Bellis, and Dorothy E. Bellis, in equal shares.1wph1.t Subsequently, or on July 8, 1958, Amos G. Bellis died a resident of San Antonio, Texas, U.S.A. His will was admitted to probate in the Court of First Instance of Manila on September 15, 1958. The People's Bank and Trust Company, as executor of the will, paid all the bequests therein including the amount of $240,000.00 in the form of shares of stock to Mary E. Mallen and to the three (3) illegitimate children, Amos Bellis, Jr., Maria Cristina Bellis and Miriam Palma Bellis, various amounts totalling P40,000.00 each in satisfaction of their respective legacies, or a total of P120,000.00, which it released from time to time according as the lower court approved and allowed the various motions or petitions filed by the latter three requesting partial advances on account of their respective legacies. On January 8, 1964, preparatory to closing its administration, the executor submitted and filed its "Executor's Final Account, Report of Administration and Project of Partition" wherein it reported, inter alia, the satisfaction of the legacy of Mary E. Mallen by the delivery to her of shares of stock amounting to $240,000.00, and the legacies of Amos Bellis, Jr., Maria Cristina Bellis and Miriam Palma Bellis in the amount of P40,000.00 each or a total of P120,000.00. In the project of partition, the executor pursuant to the "Twelfth" clause of the testator's Last Will and Testament divided the residuary estate into seven equal portions for the benefit of the testator's seven legitimate children by his first and second marriages.

82

University of the Cordilleras College of Law First Year C S.Y. 2013 - 2014 On January 17, 1964, Maria Cristina Bellis and Miriam Palma Bellis filed their respective oppositions to the project of partition on the ground that they were deprived of their legitimes as illegitimate children and, therefore, compulsory heirs of the deceased. Amos Bellis, Jr. interposed no opposition despite notice to him, proof of service of which is evidenced by the registry receipt submitted on April 27, 1964 by the executor.1 After the parties filed their respective memoranda and other pertinent pleadings, the lower court, on April 30, 1964, issued an order overruling the oppositions and approving the executor's final account, report and administration and project of partition. Relying upon Art. 16 of the Civil Code, it applied the national law of the decedent, which in this case is Texas law, which did not provide for legitimes. Their respective motions for reconsideration having been denied by the lower court on June 11, 1964, oppositors-appellants appealed to this Court to raise the issue of which law must apply Texas law or Philippine law. In this regard, the parties do not submit the case on, nor even discuss, the doctrine of renvoi, applied by this Court in Aznar v. Christensen Garcia, L-16749, January 31, 1963. Said doctrine is usually pertinent where the decedent is a national of one country, and a domicile of another. In the present case, it is not disputed that the decedent was both a national of Texas and a domicile thereof at the time of his death.2 So that even assuming Texas has a conflict of law rule providing that the domiciliary system (law of the domicile) should govern, the same would not result in a reference back (renvoi) to Philippine law, but would still refer to Texas law. Nonetheless, if Texas has a conflicts rule adopting the situs theory (lex rei sitae) calling for the application of the law of the place where the properties are situated, renvoi would arise, since the properties here involved are found in the Philippines. In the absence, however, of proof as to the conflict of law rule of Texas, it should not be presumed different from ours.3 Appellants' position is therefore not rested on the doctrine of renvoi. As stated, they never invoked nor even mentioned it in their arguments. Rather, they argue that their case falls under the circumstances mentioned in the third paragraph of Article 17 in relation to Article 16 of the Civil Code. Article 16, par. 2, and Art. 1039 of the Civil Code, render applicable the national law of the decedent, in intestate or testamentary successions, with regard to four items: (a) the order of succession; (b) the amount of successional rights; (e) the intrinsic validity of the provisions of the will; and (d) the capacity to succeed. They provide that ART. 16. Real property as well as personal property is subject to the law of the country where it is situated. However, intestate and testamentary successions, both with respect to the order of succession and to the amount of successional rights and to the intrinsic validity of testamentary provisions, shall be regulated by the national law of the person whose succession is under consideration, whatever may he the nature of the property and regardless of the country wherein said property may be found. ART. 1039. Capacity to succeed is governed by the law of the nation of the decedent. Appellants would however counter that Art. 17, paragraph three, of the Civil Code, stating that Prohibitive laws concerning persons, their acts or property, and those which have for their object public order, public policy and good customs shall not be rendered ineffective by laws or judgments promulgated, or by determinations or conventions agreed upon in a foreign country. prevails as the exception to Art. 16, par. 2 of the Civil Code afore-quoted. This is not correct. Precisely, Congress deleted the phrase, "notwithstanding the provisions of this and the next preceding article" when they incorporated Art. 11 of the old Civil Code as Art. 17 of the new Civil Code, while reproducing without substantial change the second paragraph of Art. 10 of the old Civil Code as Art. 16 in the new. It must have been their purpose to make the second paragraph of Art. 16 a specific provision in itself which must be applied in testate and intestate succession. As further indication of this legislative intent, 83

University of the Cordilleras College of Law First Year C S.Y. 2013 - 2014 Congress added a new provision, under Art. 1039, which decrees that capacity to succeed is to be governed by the national law of the decedent. It is therefore evident that whatever public policy or good customs may be involved in our System of legitimes, Congress has not intended to extend the same to the succession of foreign nationals. For it has specifically chosen to leave, inter alia, the amount of successional rights, to the decedent's national law. Specific provisions must prevail over general ones. Appellants would also point out that the decedent executed two wills one to govern his Texas estate and the other his Philippine estate arguing from this that he intended Philippine law to govern his Philippine estate. Assuming that such was the decedent's intention in executing a separate Philippine will, it would not alter the law, for as this Court ruled in Miciano v. Brimo, 50 Phil. 867, 870, a provision in a foreigner's will to the effect that his properties shall be distributed in accordance with Philippine law and not with his national law, is illegal and void, for his national law cannot be ignored in regard to those matters that Article 10 now Article 16 of the Civil Code states said national law should govern. The parties admit that the decedent, Amos G. Bellis, was a citizen of the State of Texas, U.S.A., and that under the laws of Texas, there are no forced heirs or legitimes. Accordingly, since the intrinsic validity of the provision of the will and the amount of successional rights are to be determined under Texas law, the Philippine law on legitimes cannot be applied to the testacy of Amos G. Bellis. Wherefore, the order of the probate court is hereby affirmed in toto, with costs against appellants. So ordered.

84

University of the Cordilleras College of Law First Year C S.Y. 2013 - 2014 Case Digest Nationality principle Bellis vs. Bellis G.R. No. L-23678 Decided on: June 6, 1967 Ponente: BENGZON, J.P., J.: FACTS: Amos G. Bellis was a citizen and resident of Texas at the time of his death. He executed a will in the Philippines, in which he directed that after all taxes, obligations, and expenses of administration are paid for, his distributable estate should be divided, in trust, in the following order and manner a) $240,000.00 to his first wife MARY E. MALLEN b) $120,000.00 to his three illegitimate childrenAMOS BELLIS, JR., MARIA CRISTINA BELLIS, MIRIAM PALMA BELLIS,or $40,000.00 each, and c) After foregoing the two items have been satisfied, the remainder shall go to his seven surviving children by his first and second wives EDWARD A. BELLIS, HENRY A. BELLIS, ALEXANDER BELLIS, and ANNA BELLIS-ALLSMAN, EDWARD G. BELLIS, WA LTER S. BELLIS, and DOROTHY E. BELLIS in equal shares. MARIA CRISTINA BELLIS and MIRIAM PALMA BELLIS filed their respective oppositions to the project of partition on the ground that they were deprived of their legitimes as illegitimate children and, therefore, compulsory heirs of the deceased. The LOWER COURT issued an order overruling the oppositions and approving the executors final account, report and administration, and project of partition. Relying upon Article 16 of the Civil Code, it applied the national law of the decedent, which in this case is which did not provide for legitimes. ISSUE: Which law must apply in executing the will of the deceased Texas Law or Philippine Law? RULING: The said illegitimate children are not entitled to their legitimes under the Texas Law(which is the national law of the deceased), here are no legitimes. The renvoi doctrinecannot be applied. Said doctrine is usually pertinent where the decedent is a national of one country ad a domiciliary of another. In the said case, it is not disputed that the deceased was both a national of Texas and a domicile thereof at the time of his death. Article 16, Paragraph 2 of Civil code render applicable the national law of the decedent, in intestate and testamentary successions, with regard to four items: (a) the order of succession, (b) the amount of successional rights, (c) the intrinsic validity of provisions of will, and (d) the capacity to succeed. They provide that: ART.16 Real property as well as personal property is subject to the law of the country to where it is situated. However, intestate and testamentary successions, both with respect to the order of successions and to the amount of successional rights and to the intrinsic validity of testamentary provisions, shall be regulated by the national law of the person whose succession is under consideration, whatever may be the nature of the property and regardless of the country wherein said property may be found.

85

University of the Cordilleras College of Law First Year C S.Y. 2013 - 2014 Testate Estate of Bohonan vs. Bohonan, 106 Phil 997 G.R. No. L-12105 January 30, 1960 Full Case TESTATE ESTATE OF C. O. BOHANAN, deceased. PHILIPPINE TRUST CO., executor-appellee, vs. MAGDALENA C. BOHANAN, EDWARD C. BOHANAN, and MARY LYDIA BOHANAN, oppositors-appellants. LABRADOR, J.: Appeal against an order of the Court of First Instance of Manila, Hon. Ramon San Jose, presiding, dismissing the objections filed by Magdalena C. Bohanan, Mary Bohanan and Edward Bohanan to the project of partition submitted by the executor and approving the said project. On April 24, 195 0, the Court of First Instance of Manila, Hon. Rafael Amparo, presiding, admitted to probate a last will and testament of C. O. Bohanan, executed by him on April 23, 1944 in Manila. In the said order, the court made the following findings: According to the evidence of the opponents the testator was born in Nebraska and therefore a citizen of that state, or at least a citizen of California where some of his properties are located. This contention in untenable. Notwithstanding the long residence of the decedent in the Philippines, his stay here was merely temporary, and he continued and remained to be a citizen of the United States and of the state of his pertinent residence to spend the rest of his days in that state. His permanent residence or domicile in the United States depended upon his personal intent or desire, and he selected Nevada as his homicide and therefore at the time of his death, he was a citizen of that state. Nobody can choose his domicile or permanent residence for him. That is his exclusive personal right. Wherefore, the court finds that the testator C. O. Bohanan was at the time of his death a citizen of the United States and of the State of Nevada and declares that his will and testament, Exhibit A, is fully in accordance with the laws of the state of Nevada and admits the same to probate. Accordingly, the Philippine Trust Company, named as the executor of the will, is hereby appointed to such executor and upon the filing of a bond in the sum of P10,000.00, let letters testamentary be issued and after taking the prescribed oath, it may enter upon the execution and performance of its trust. (pp. 26-27, R.O.A.). It does not appear that the order granting probate was ever questions on appeal. The executor filed a project of partition dated January 24, 1956, making, in accordance with the provisions of the will, the following adjudications: (1) one-half of the residuary estate, to the Farmers and Merchants National Bank of Los Angeles, California, U.S.A. in trust only for the benefit of testator's grandson Edward George Bohanan, which consists of several mining companies; (2) the other half of the residuary estate to the testator's brother, F.L. Bohanan, and his sister, Mrs. M. B. Galbraith, share and share alike. This consist in the same amount of cash and of shares of mining stock similar to those given to testator's grandson; (3) legacies of P6,000 each to his (testator) son, Edward Gilbert Bohana, and his daughter, Mary Lydia Bohanan, to be paid in three yearly installments; (4) legacies to Clara Daen, in the amount of P10,000.00; Katherine Woodward, P2,000; Beulah Fox, P4,000; and Elizabeth Hastings, P2,000; It will be seen from the above that out of the total estate (after deducting administration expenses) of P211,639.33 in cash, the testator gave his grandson P90,819.67 and one-half of all shares of stock of several mining companies and to his brother and sister the same amount. To his children he gave a legacy of only P6,000 each, or a total of P12,000. The wife Magadalena C. Bohanan and her two children question the validity of the testamentary provisions disposing of the estate in the manner above indicated, claiming that they have been deprived of the legitimate that the laws of the form concede to them. The first question refers to the share that the wife of the testator, Magdalena C. Bohanan, should be entitled to received. The will has not given her any share in the estate left by the testator. It is argued that it was error for the trial court to have recognized the Reno divorce secured by the testator from his 86

University of the Cordilleras College of Law First Year C S.Y. 2013 - 2014 Filipino wife Magdalena C. Bohanan, and that said divorce should be declared a nullity in this jurisdiction, citing the case of Querubin vs. Querubin, 87 Phil., 124, 47 Off. Gaz., (Sup, 12) 315, Cousins Hiz vs. Fluemer, 55 Phil., 852, Ramirez vs. Gmur, 42 Phil., 855 and Gorayeb vs. Hashim, 50 Phil., 22. The court below refused to recognize the claim of the widow on the ground that the laws of Nevada, of which the deceased was a citizen, allow him to dispose of all of his properties without requiring him to leave any portion of his estate to his wife. Section 9905 of Nevada Compiled Laws of 1925 provides: Every person over the age of eighteen years, of sound mind, may, by last will, dispose of all his or her estate, real and personal, the same being chargeable with the payment of the testator's debts. Besides, the right of the former wife of the testator, Magdalena C. Bohanan, to a share in the testator's estafa had already been passed upon adversely against her in an order dated June 19, 1955, (pp. 155-159, Vol II Records, Court of First Instance), which had become final, as Magdalena C. Bohanan does not appear to have appealed therefrom to question its validity. On December 16, 1953, the said former wife filed a motion to withdraw the sum of P20,000 from the funds of the estate, chargeable against her share in the conjugal property, (See pp. 294-297, Vol. I, Record, Court of First Instance), and the court in its said error found that there exists no community property owned by the decedent and his former wife at the time the decree of divorce was issued. As already and Magdalena C. Bohanan may no longer question the fact contained therein, i.e. that there was no community property acquired by the testator and Magdalena C. Bohanan during their converture. Moreover, the court below had found that the testator and Magdalena C. Bohanan were married on January 30, 1909, and that divorce was granted to him on May 20, 1922; that sometime in 1925, Magdalena C. Bohanan married Carl Aaron and this marriage was subsisting at the time of the death of the testator. Since no right to share in the inheritance in favor of a divorced wife exists in the State of Nevada and since the court below had already found that there was no conjugal property between the testator and Magdalena C. Bohanan, the latter can now have no longer claim to pay portion of the estate left by the testator. The most important issue is the claim of the testator's children, Edward and Mary Lydia, who had received legacies in the amount of P6,000 each only, and, therefore, have not been given their shares in the estate which, in accordance with the laws of the forum, should be two-thirds of the estate left by the testator. Is the failure old the testator to give his children two-thirds of the estate left by him at the time of his death, in accordance with the laws of the forum valid? The old Civil Code, which is applicable to this case because the testator died in 1944, expressly provides that successional rights to personal property are to be earned by the national law of the person whose succession is in question. Says the law on this point: Nevertheless, legal and testamentary successions, in respect to the order of succession as well as to the extent of the successional rights and the intrinsic validity of their provisions, shall be regulated by the national law of the person whose succession is in question, whatever may be the nature of the property and the country in which it is found. (par. 2, Art. 10, old Civil Code, which is the same as par. 2 Art. 16, new Civil Code.) In the proceedings for the probate of the will, it was found out and it was decided that the testator was a citizen of the State of Nevada because he had selected this as his domicile and his permanent residence. (See Decision dated April 24, 1950, supra). So the question at issue is whether the estementary dispositions, especially hose for the children which are short of the legitime given them by the Civil Code of the Philippines, are valid. It is not disputed that the laws of Nevada allow a testator to dispose of all his properties by will (Sec. 9905, Complied Nevada Laws of 1925, supra). It does not appear that at time of the hearing of the project of partition, the above-quoted provision was introduced in evidence, as it was the executor's duly to do. The law of Nevada, being a foreign law can only be proved in our courts in the form and manner provided for by our Rules, which are as follows:

87

University of the Cordilleras College of Law First Year C S.Y. 2013 - 2014 SEC. 41. Proof of public or official record. An official record or an entry therein, when admissible for any purpose, may be evidenced by an official publication thereof or by a copy tested by the officer having the legal custody of the record, or by his deputy, and accompanied, if the record is not kept in the Philippines, with a certificate that such officer has the custody. . . . (Rule 123). We have, however, consulted the records of the case in the court below and we have found that during the hearing on October 4, 1954 of the motion of Magdalena C. Bohanan for withdrawal of P20,000 as her share, the foreign law, especially Section 9905, Compiled Nevada Laws. was introduced in evidence by appellant's (herein) counsel as Exhibits "2" (See pp. 77-79, VOL. II, and t.s.n. pp. 24-44, Records, Court of First Instance). Again said laws presented by the counsel for the executor and admitted by the Court as Exhibit "B" during the hearing of the case on January 23, 1950 before Judge Rafael Amparo (se Records, Court of First Instance, Vol. 1). In addition, the other appellants, children of the testator, do not dispute the above-quoted provision of the laws of the State of Nevada. Under all the above circumstances, we are constrained to hold that the pertinent law of Nevada, especially Section 9905 of the Compiled Nevada Laws of 1925, can be taken judicial notice of by us, without proof of such law having been offered at the hearing of the project of partition. As in accordance with Article 10 of the old Civil Code, the validity of testamentary dispositions are to be governed by the national law of the testator, and as it has been decided and it is not disputed that the national law of the testator is that of the State of Nevada, already indicated above, which allows a testator to dispose of all his property according to his will, as in the case at bar, the order of the court approving the project of partition made in accordance with the testamentary provisions, must be, as it is hereby affirmed, with costs against appellants. Paras, Bengzon, C.J., Padilla, Bautista Angelo and Endencia, JJ., concur. Barrera, J., concurs in the result.

88

University of the Cordilleras College of Law First Year C S.Y. 2013 - 2014 Case Digest Nationality Principle Testate Estate of C.O Bohanan vs. Bohanan G.R. No. L-12105 Decided on: January 30, 1960 Ponente: LABRADOR, J.: FACTS: C.O. Bohanan was born in Nebraska and therefore a citizen of that state. Notwithstanding his long residence in the Philippines, he continued and remained to be a citizen of the United States and of the state of his pertinent residence to spend the rest of his days in that state. His permanent residence or domicile in the United States depended upon his personal intent or desire, and he selected Nevada as his domicile and therefore at the time of his death, he was a citizen of that state. ISSUES: The oppositors, Magadalena C. Bohanan and her two children, question the validity of the executor/testator C.O. Bohanans last will and testament, claiming that they havebeen deprived of the legitimate that the laws of the form concede to them. Another, is the claim of the testator's children, Edward and Mary Lydia Bohanan, who had received legacies in the amount of PHP 6, 000 each only, and, therefore, have not been given their shares in the estate which, in accordance with the laws, should be two-thirds of the estate left by the testator. RULING: The first issue refers to the share that the wife of the testator, Magdalena C. Bohanan, should be entitled to receive. The will has not given her any share in the estate left by the testator. It is argued that it was error for the trial court to have recognized the Reno divorce secured by the testator from his Filipino wife Magdalena C. Bohanan, and that said divorce should be declared a nullity in this jurisdiction. The court refused to recognize the claim of the widow on the ground that the laws of Nevada, of which the deceased was a citizen, allow him to dispose of all of his properties without requiring him to leave any portion of his estate to his former (or divorced) wife. No right to share in the inheritance in favor of a divorced wife exists in the State of Nevada, thus the oppositor can no longer claim portion of the estate left by the testator. With regards the second issue, the old Civil Code, which is applicable to this case because the testator died in 1944, expressly provides that successional rights to personal property are to be earned by the national law of the person whose succession is in question, thus the two-third rule is not enforceable. Wherefore, the court finds that the testator C. O. Bohanan was at the time of his death a citizen of the United States and of the State of Nevada and declares that his will and testament is fully in accordance with the laws of the state of Nevada and admits the same to probate. As in accordance with Article 10 of the old Civil Code, the validity of testamentary dispositions are to be governed by the national law of the testator, and as it has been decided and it is not disputed that the national law of the testator is that of the State of Nevada which allows a testator to dispose of all his property according to his will, as in the case at bar, the order of the court approving the project of partition made in accordance with the testamentary provisions, must be, as it is hereby affirmed, with costs against appellants.

89

University of the Cordilleras College of Law First Year C S.Y. 2013 - 2014

II. Human Relations

90

University of the Cordilleras College of Law First Year C S.Y. 2013 - 2014

Albenson vs. Court of Appeals 217 SCRA 16


G.R. No. 88694 January 11, 1993 Full Case ALBENSON ENTERPRISES CORP., JESSE YAP, AND BENJAMIN MENDIONA, petitioners, vs. THE COURT OF APPEALS AND EUGENIO S. BALTAO, respondents. Puruganan, Chato, Chato & Tan for petitioners. Lino M. Patajo, Francisco Ma. Chanco, Ananiano Desierto and Segundo Mangohig for private respondent. BIDIN, J.: This petition assails the decision of respondent Court of Appeals in CA-GR CV No. 14948 entitled "Eugenio S. Baltao, plaintiff-appellee vs. Albenson Enterprises Corporation, et al, defendants-appellants", which modified the judgment of the Regional Trial Court of Quezon City, Branch XCVIII in Civil Case No. Q-40920 and ordered petitioner to pay private respondent, among others, the sum of P500,000.00 as moral damages and attorney's fees in the amount of P50,000.00. The facts are not disputed. In September, October, and November 1980, petitioner Albenson Enterprises Corporation (Albenson for short) delivered to Guaranteed Industries, Inc. (Guaranteed for short) located at 3267 V. Mapa Street, Sta. Mesa, Manila, the mild steel plates which the latter ordered. As part payment thereof, Albenson was given Pacific Banking Corporation Check No. 136361 in the amount of P2,575.00 and drawn against the account of E.L. Woodworks (Rollo, p. 148). When presented for payment, the check was dishonored for the reason "Account Closed." Thereafter, petitioner Albenson, through counsel, traced the origin of the dishonored check. From the records of the Securities and Exchange Commission (SEC), Albenson discovered that the president of Guaranteed, the recipient of the unpaid mild steel plates, was one "Eugenio S. Baltao." Upon further inquiry, Albenson was informed by the Ministry of Trade and Industry that E.L. Woodworks, a single proprietorship business, was registered in the name of one "Eugenio Baltao". In addition, upon verification with the drawee bank, Pacific Banking Corporation, Albenson was advised that the signature appearing on the subject check belonged to one "Eugenio Baltao." After obtaining the foregoing information, Albenson, through counsel, made an extrajudicial demand upon private respondent Eugenio S. Baltao, president of Guaranteed, to replace and/or make good the dishonored check. Respondent Baltao, through counsel, denied that he issued the check, or that the signature appearing thereon is his. He further alleged that Guaranteed was a defunct entity and hence, could not have transacted business with Albenson. On February 14, 1983, Albenson filed with the Office of the Provincial Fiscal of Rizal a complaint against Eugenio S. Baltao for violation of Batas Pambansa Bilang 22. Submitted to support said charges was an affidavit of petitioner Benjamin Mendiona, an employee of Albenson. In said affidavit, the abovementioned circumstances were stated. It appears, however, that private respondent has a namesake, his son Eugenio Baltao III, who manages a business establishment, E.L. Woodworks, on the ground floor of the Baltao Building, 3267 V. Mapa Street, Sta. Mesa, Manila, the very same business address of Guaranteed. On September 5, 1983, Assistant Fiscal Ricardo Sumaway filed an information against Eugenio S. Baltao for Violation of Batas Pambansa Bilang 22. In filing said information, Fiscal Sumaway claimed that he had given Eugenio S. Baltao opportunity to submit controverting evidence, but the latter failed to do so and therefore, was deemed to have waived his right. Respondent Baltao, claiming ignorance of the complaint against him, immediately filed with the Provincial Fiscal of Rizal a motion for reinvestigation, alleging that it was not true that he had been given 91

University of the Cordilleras College of Law First Year C S.Y. 2013 - 2014 an opportunity to be heard in the preliminary investigation conducted by Fiscal Sumaway, and that he never had any dealings with Albenson or Benjamin Mendiona, consequently, the check for which he has been accused of having issued without funds was not issued by him and the signature in said check was not his. On January 30, 1984, Provincial Fiscal Mauro M. Castro of Rizal reversed the finding of Fiscal Sumaway and exonerated respondent Baltao. He also instructed the Trial Fiscal to move for dismissal of the information filed against Eugenio S. Baltao. Fiscal Castro found that the signature in PBC Check No. 136361 is not the signature of Eugenio S. Baltao. He also found that there is no showing in the records of the preliminary investigation that Eugenio S. Baltao actually received notice of the said investigation. Fiscal Castro then castigated Fiscal Sumaway for failing to exercise care and prudence in the performance of his duties, thereby causing injustice to respondent who was not properly notified of the complaint against him and of the requirement to submit his counter evidence. Because of the alleged unjust filing of a criminal case against him for allegedly issuing a check which bounced in violation of Batas Pambansa Bilang 22 for a measly amount of P2,575.00, respondent Baltao filed before the Regional Trial Court of Quezon City a complaint for damages against herein petitioners Albenson Enterprises, Jesse Yap, its owner, and Benjamin Mendiona, its employee. In its decision, the lower court observed that "the check is drawn against the account of "E.L. Woodworks," not of Guaranteed Industries of which plaintiff used to be President. Guaranteed Industries had been inactive and had ceased to exist as a corporation since 1975. . . . . The possibility is that it was with Gene Baltao or Eugenio Baltao III, a son of plaintiff who had a business on the ground floor of Baltao Building located on V. Mapa Street, that the defendants may have been dealing with . . . ." ( Rollo, pp. 41-42). The dispositive portion of the trial court 's decision reads: WHEREFORE, judgment is hereby rendered in favor of plaintiff and against defendants ordering the latter to pay plaintiff jointly and severally: 1. actual or compensatory damages of P133,350.00; 2. moral damages of P1,000,000.00 (1 million pesos); 3. exemplary damages of P200,000.00; 4. attorney's fees of P100,000.00; 5 costs. Defendants' counterclaim against plaintiff and claim for damages against Mercantile Insurance Co. on the bond for the issuance of the writ of attachment at the instance of plaintiff are hereby dismissed for lack of merit. (Rollo, pp. 38-39). On appeal, respondent court modified the trial court's decision as follows: WHEREFORE, the decision appealed from is MODIFIED by reducing the moral damages awarded therein from P1,000,000.00 to P500,000.00 and the attorney's fees from P100,000.00 to P50,000.00, said decision being hereby affirmed in all its other aspects. With costs against appellants. (Rollo, pp. 50-51) Dissatisfied with the above ruling, petitioners Albenson Enterprises Corp., Jesse Yap, and Benjamin Mendiona filed the instant Petition, alleging that the appellate court erred in: 1. Concluding that private respondent's cause of action is not one based on malicious prosecution but one for abuse of rights under Article 21 of the Civil Code notwithstanding the fact that the basis of a civil action for malicious prosecution is Article 2219 in relation to Article 21 or Article 2176 of the Civil Code . . . .

92

University of the Cordilleras College of Law First Year C S.Y. 2013 - 2014 2. Concluding that "hitting at and in effect maligning (private respondent) with an unjust criminal case was, without more, a plain case of abuse of rights by misdirection" and "was therefore, actionable by itself," and which "became inordinately blatant and grossly aggravated when . . . (private respondent) was deprived of his basic right to notice and a fair hearing in the so-called preliminary investigation . . . . " 3. Concluding that petitioner's "actuations in this case were coldly deliberate and calculated", no evidence having been adduced to support such a sweeping statement. 4. Holding the petitioner corporation, petitioner Yap and petitioner Mendiona jointly and severally liable without sufficient basis in law and in fact. 5. Awarding respondents 5.1. P133,350.00 as actual or compensatory damages, even in the absence of sufficient evidence to show that such was actually suffered. 5.2. P500,000.00 as moral damages considering that the evidence in this connection merely involved private respondent's alleged celebrated status as a businessman, there being no showing that the act complained of adversely affected private respondent's reputation or that it resulted to material loss. 5.3. P200,000.00 as exemplary damages despite the fact that petitioners were duly advised by counsel of their legal recourse. 5.4. P50,000.00 as attorney's fees, no evidence having been adduced to justify such an award (Rollo, pp. 4-6). Petitioners contend that the civil case filed in the lower court was one for malicious prosecution. Citing the case of Madera vs. Lopez (102 SCRA 700 [1981]), they assert that the absence of malice on their part absolves them from any liability for malicious prosecution. Private respondent, on the other hand, anchored his complaint for Damages on Articles 19, 20, and 21 of the Civil Code. Article 19, known to contain what is commonly referred to as the principle of abuse of rights, sets certain standards which may be observed not only in the exercise of one's rights but also in the performance of one's duties. These standards are the following: to act with justice; to give everyone his due; and to observe honesty and good faith. The law, therefore, recognizes the primordial limitation on all rights: that in their exercise, the norms of human conduct set forth in Article 19 must be observed. A right, though by itself legal because recognized or granted by law as such, may nevertheless become the source of some illegality. When a right is exercised in a manner which does not conform with the norms enshrined in Article 19 and results in damage to another, a legal wrong is thereby committed for which the wrongdoer must be held responsible. Although the requirements of each provision is different, these three (3) articles are all related to each other. As the eminent Civilist Senator Arturo Tolentino puts it: "With this article (Article 21), combined with articles 19 and 20, the scope of our law on civil wrongs has been very greatly broadened; it has become much more supple and adaptable than the Anglo-American law on torts. It is now difficult to conceive of any malevolent exercise of a right which could not be checked by the application of these articles" (Tolentino, 1 Civil Code of the Philippines 72). There is however, no hard and fast rule which can be applied to determine whether or not the principle of abuse of rights may be invoked. The question of whether or not the principle of abuse of rights has been violated, resulting in damages under Articles 20 and 21 or other applicable provision of law, depends on the circumstances of each case. (Globe Mackay Cable and Radio Corporation vs. Court of Appeals, 176 SCRA 778 [1989]). The elements of an abuse of right under Article 19 are the following: (1) There is a legal right or duty; (2) which is exercised in bad faith; (3) for the sole intent of prejudicing or injuring another. Article 20 speaks of the general sanction for all other provisions of law which do not especially provide for their own sanction (Tolentino, supra, p. 71). Thus, anyone who, whether willfully or negligently, in the exercise of his legal right or duty, causes damage to another, shall indemnify his victim for injuries suffered thereby. Article 21 deals with acts contra bonus mores, and has the following elements: 1) There is an act which is 93

University of the Cordilleras College of Law First Year C S.Y. 2013 - 2014 legal; 2) but which is contrary to morals, good custom, public order, or public policy; 3) and it is done with intent to injure. Thus, under any of these three (3) provisions of law, an act which causes injury to another may be made the basis for an award of damages. There is a common element under Articles 19 and 21, and that is, the act must be intentional. However, Article 20 does not distinguish: the act may be done either "willfully", or "negligently". The trial court as well as the respondent appellate court mistakenly lumped these three (3) articles together, and cited the same as the bases for the award of damages in the civil complaint filed against petitioners, thus: With the foregoing legal provisions (Articles 19, 20, and 21) in focus, there is not much difficulty in ascertaining the means by which appellants' first assigned error should be resolved, given the admitted fact that when there was an attempt to collect the amount of P2,575.00, the defendants were explicitly warned that plaintiff Eugenio S. Baltao is not the Eugenio Baltao defendants had been dealing with (supra, p. 5). When the defendants nevertheless insisted and persisted in filing a case a criminal case no less against plaintiff, said defendants ran afoul of the legal provisions (Articles 19, 20, and 21 of the Civil Code) cited by the lower court and heretofore quoted (supra). Defendants, not having been paid the amount of P2,575.00, certainly had the right to complain. But that right is limited by certain constraints. Beyond that limit is the area of excess, of abuse of rights. (Rollo, pp. 44-45). Assuming, arguendo, that all the three (3) articles, together and not independently of each one, could be validly made the bases for an award of damages based on the principle of "abuse of right", under the circumstances, We see no cogent reason for such an award of damages to be made in favor of private respondent. Certainly, petitioners could not be said to have violated the aforestated principle of abuse of right. What prompted petitioners to file the case for violation of Batas Pambansa Bilang 22 against private respondent was their failure to collect the amount of P2,575.00 due on a bounced check which they honestly believed was issued to them by private respondent. Petitioners had conducted inquiries regarding the origin of the check, and yielded the following results: from the records of the Securities and Exchange Commission, it was discovered that the President of Guaranteed (the recipient of the unpaid mild steel plates), was one "Eugenio S. Baltao"; an inquiry with the Ministry of Trade and Industry revealed that E.L. Woodworks, against whose account the check was drawn, was registered in the name of one "Eugenio Baltao"; verification with the drawee bank, the Pacific Banking Corporation, revealed that the signature appearing on the check belonged to one "Eugenio Baltao". In a letter dated December 16, 1983, counsel for petitioners wrote private respondent demanding that he make good the amount of the check. Counsel for private respondent wrote back and denied, among others, that private respondent ever transacted business with Albenson Enterprises Corporation; that he ever issued the check in question. Private respondent's counsel even went further: he made a warning to defendants to check the veracity of their claim. It is pivotal to note at this juncture that in this same letter, if indeed private respondent wanted to clear himself from the baseless accusation made against his person, he should have made mention of the fact that there are three (3) persons with the same name, i.e.: Eugenio Baltao, Sr., Eugenio S. Baltao, Jr. (private respondent), and Eugenio Baltao III (private respondent's son, who as it turned out later, was the issuer of the check). He, however, failed to do this. The last two Baltaos were doing business in the same building Baltao Building located at 3267 V. Mapa Street, Sta. Mesa, Manila. The mild steel plates were ordered in the name of Guaranteed of which respondent Eugenio S. Baltao is the president and delivered to Guaranteed at Baltao building. Thus, petitioners had every reason to believe that the Eugenio Baltao who issued the bouncing check is respondent Eugenio S. Baltao when their counsel wrote respondent to make good the amount of the check and upon refusal, filed the complaint for violation of BP Blg. 22. Private respondent, however, did nothing to clarify the case of mistaken identity at first hand. Instead, private respondent waited in ambush and thereafter pounced on the hapless petitioners at a time he thought was propitious by filing an action for damages. The Court will not countenance this devious scheme. 94

University of the Cordilleras College of Law First Year C S.Y. 2013 - 2014 The criminal complaint filed against private respondent after the latter refused to make good the amount of the bouncing check despite demand was a sincere attempt on the part of petitioners to find the best possible means by which they could collect the sum of money due them. A person who has not been paid an obligation owed to him will naturally seek ways to compel the debtor to pay him. It was normal for petitioners to find means to make the issuer of the check pay the amount thereof. In the absence of a wrongful act or omission or of fraud or bad faith, moral damages cannot be awarded and that the adverse result of an action does not per se make the action wrongful and subject the actor to the payment of damages, for the law could not have meant to impose a penalty on the right to litigate (Rubio vs. Court of Appeals, 141 SCRA 488 [1986]). In the case at bar, private respondent does not deny that the mild steel plates were ordered by and delivered to Guaranteed at Baltao building and as part payment thereof, the bouncing check was issued by one Eugenio Baltao. Neither had private respondent conveyed to petitioner that there are two Eugenio Baltaos conducting business in the same building he and his son Eugenio Baltao III. Considering that Guaranteed, which received the goods in payment of which the bouncing check was issued is owned by respondent, petitioner acted in good faith and probable cause in filing the complaint before the provincial fiscal. To constitute malicious prosecution, there must be proof that the prosecution was prompted by a sinister design to vex and humiliate a person, and that it was initiated deliberately by the defendant knowing that his charges were false and groundless. Concededly, the mere act of submitting a case to the authorities for prosecution does not make one liable for malicious prosecution. (Manila Gas Corporation vs. Court of Appeals, 100 SCRA 602 [1980]). Still, private respondent argues that liability under Articles 19, 20, and 21 of the Civil Code is so encompassing that it likewise includes liability for damages for malicious prosecution under Article 2219 (8). True, a civil action for damages for malicious prosecution is allowed under the New Civil Code, more specifically Articles 19, 20, 26, 29, 32, 33, 35, and 2219 (8) thereof. In order that such a case can prosper, however, the following three (3) elements must be present, to wit: (1) The fact of the prosecution and the further fact that the defendant was himself the prosecutor, and that the action was finally terminated with an acquittal; (2) That in bringing the action, the prosecutor acted without probable cause; (3) The prosecutor was actuated or impelled by legal malice (Lao vs. Court of Appeals, 199 SCRA 58, [1991]). Thus, a party injured by the filing of a court case against him, even if he is later on absolved, may file a case for damages grounded either on the principle of abuse of rights, or on malicious prosecution. As earlier stated, a complaint for damages based on malicious prosecution will prosper only if the three (3) elements aforecited are shown to exist. In the case at bar, the second and third elements were not shown to exist. It is well-settled that one cannot be held liable for maliciously instituting a prosecution where one has acted with probable cause. "Probable cause is the existence of such facts and circumstances as would excite the belief, in a reasonable mind, acting on the facts within the knowledge of the prosecutor, that the person charged was guilty of the crime for which he was prosecuted. In other words, a suit will lie only in cases where a legal prosecution has been carried on without probable cause. The reason for this rule is that it would be a very great discouragement to public justice, if prosecutors, who had tolerable ground of suspicion, were liable to be sued at law when their indictment miscarried" (Que vs. Intermediate Appellate Court, 169 SCRA 137 [1989]). The presence of probable cause signifies, as a legal consequence, the absence of malice. In the instant case, it is evident that petitioners were not motivated by malicious intent or by sinister design to unduly harass private respondent, but only by a well-founded anxiety to protect their rights when they filed the criminal complaint against private respondent. To constitute malicious prosecution, there must be proof that the prosecution was prompted by a sinister design to vex and humiliate a person, that it was initiated deliberately by the defendant knowing that his charges were false and groundless. Concededly, the mere act of submitting a case to the authorities for prosecution does not make one liable for malicious prosecution. Proof and motive that the institution of the action was prompted by a sinister design to vex and humiliate a person must be clearly and preponderantly established to entitle the victims to damages (Ibid.). In the case at bar, there is no proof of a sinister design on the part of petitioners to vex or humiliate private respondent by instituting the criminal case against him. While petitioners may have been negligent

95

University of the Cordilleras College of Law First Year C S.Y. 2013 - 2014 to some extent in determining the liability of private respondent for the dishonored check, the same is not so gross or reckless as to amount to bad faith warranting an award of damages. The root of the controversy in this case is founded on a case of mistaken identity. It is possible that with a more assiduous investigation, petitioners would have eventually discovered that private respondent Eugenio S. Baltao is not the "Eugenio Baltao" responsible for the dishonored check. However, the record shows that petitioners did exert considerable effort in order to determine the liability of private respondent. Their investigation pointed to private respondent as the "Eugenio Baltao" who issued and signed the dishonored check as the president of the debtor-corporation Guaranteed Enterprises. Their error in proceeding against the wrong individual was obviously in the nature of an innocent mistake, and cannot be characterized as having been committed in bad faith. This error could have been discovered if respondent had submitted his counter-affidavit before investigating fiscal Sumaway and was immediately rectified by Provincial Fiscal Mauro Castro upon discovery thereof, i.e., during the reinvestigation resulting in the dismissal of the complaint. Furthermore, the adverse result of an action does not per se make the act wrongful and subject the actor to the payment of moral damages. The law could not have meant to impose a penalty on the right to litigate, such right is so precious that moral damages may not be charged on those who may even exercise it erroneously. And an adverse decision does not ipso facto justify the award of attorney's fees to the winning party (Garcia vs. Gonzales, 183 SCRA 72 [1990]). Thus, an award of damages and attorney's fees is unwarranted where the action was filed in good faith. If damage results from a person's exercising his legal rights, it is damnum absque injuria (Ilocos Norte Electric Company vs. Court of Appeals, 179 SCRA 5 [1989]). Coming now to the claim of private respondent for actual or compensatory damages, the records show that the same was based solely on his allegations without proof to substantiate the same. He did not present proof of the cost of the medical treatment which he claimed to have undergone as a result of the nervous breakdown he suffered, nor did he present proof of the actual loss to his business caused by the unjust litigation against him. In determining actual damages, the court cannot rely on speculation, conjectures or guesswork as to the amount. Without the actual proof of loss, the award of actual damages becomes erroneous (Guilatco vs. City of Dagupan, 171 SCRA 382 [1989]). Actual and compensatory damages are those recoverable because of pecuniary loss in business, trade, property, profession, job or occupation and the same must be proved, otherwise, if the proof is flimsy and unsubstantiated, no damages will be given (Rubio vs. Court of Appeals, 141 SCRA 488 [1986]). For these reasons, it was gravely erroneous for respondent court to have affirmed the award of actual damages in favor of private respondent in the absence of proof thereof. Where there is no evidence of the other party having acted in wanton, fraudulent or reckless, or oppressive manner, neither may exemplary damages be awarded (Dee Hua Liong Electrical Equipment Corporation vs. Reyes, 145 SCRA 488 [1986]). As to the award of attorney's fees, it is well-settled that the same is the exception rather than the general rule. Needless to say, the award of attorney's fees must be disallowed where the award of exemplary damages is eliminated (Article 2208, Civil Code; Agustin vs. Court of Appeals, 186 SCRA 375 [1990]). Moreover, in view of the fact that there was no malicious prosecution against private respondent, attorney's fees cannot be awarded him on that ground. In the final analysis, there is no proof or showing that petitioners acted maliciously or in bad faith in the filing of the case against private respondent. Consequently, in the absence of proof of fraud and bad faith committed by petitioners, they cannot be held liable for damages (Escritor, Jr. vs. Intermediate Appellate Court, 155 SCRA 577 [1987]). No damages can be awarded in the instant case, whether based on the principle of abuse of rights, or for malicious prosecution. The questioned judgment in the instant case attests to the propensity of trial judges to award damages without basis. Lower courts are hereby cautioned anew against awarding unconscionable sums as damages without bases therefor. WHEREFORE, the petition is GRANTED and the decision of the Court of Appeals in C.A. G.R. C.V. No. 14948 dated May 13, 1989, is hereby REVERSED and SET ASIDE. Costs against respondent Baltao. SO ORDERED. 96

University of the Cordilleras College of Law First Year C S.Y. 2013 - 2014 Case Digest Albenson vs. Court of Appeals G.R. No. 88694 Decided On: January 11, 1993 Ponente: BIDIN, J.: Facts: Albenson Ent. delivered mild steel plates to Guaranteed Industries Inc. A Pacific Banking Corporation Check was paid and drawn against the account of EL Woodworks. Check was later dishonored for the reason Account Closed. Company traced source of check and later d i s c o v e r e d t h a t t h e s i g n a t u r e b e l o n g e d t o o n e E u g e n i o B a l t a o . A l b e n s o n m a d e a n extrajudical demand upon Baltao but later denied that he issued the check or that the signature was his. Company filed a complaint against Baltao for violation of BP 22. It was later discovered that private respondent had son: Eugene Baltao III, who manages the business establishment, EL Woodworks. No effort from the father to inform Albenson of such information. Rather the father filed complaint for damages against Albenson. Issue: Whether there is indeed cause for the damages against Albenson Enterprise. Ruling: Based on Art 19, 20, 21 of the civil code, petitioners didnt have the intent to cause damage to the respondent or enrich themselves but just to collect what was due to them. There was no abuse of right on the part of Albenson on accusing Baltao of BP 22.Albenson Corp. honestly believed that it was private respondent who issued check based on the following inquiries: -SEC records showed that president to Guaranteed was Eugene Baltao -Bank said signature belonged to EB -Eugene Baltao did not do his part in clarifying that there were in fact 3 Eugene Baltaos: Jr., Sr. and the III. There was no malicious prosecution on the part of Albenson: there must be proof that: -the prosecution was prompted by a sinister design to vex and h u m i l i a t e a person and -that damages was initiated deliberately by defendant knowing that his charges were false and groundless. Elements of abuse of right under Article 19: 1. There is a legal right or duty 2. Exercised in bad fait 3. for the sole intent of prejudicing or injuring another. Elements under Article 21: 1. There is an act which is legal 2. But which is contrary to morals, good custom, public order or public policy 3. It is done with intent to injure a person who has not been paid an obligation owed to him will naturally seek ways to compel the debtor to pay him. It was normal for petitioners to find means to make the issuer of the check pay the amount thereof. In the absence of a wrongful act or omission or of fraud or bad faith, moral damages cannot be awarded and that the adverse result of an action does not per se make the action wrongful and subject the actor to the payment of damages, for the law could not have meant to impose a penalty on the right to litigate WHEREFORE, the petition is GRANTED and the decision of the Court of Appeals in C.A. G.R.C.V. No. 14948 dated May 13, 1989, is hereby REVERSED and SET ASIDE. Costs against respondent Baltao.

97

University of the Cordilleras College of Law First Year C S.Y. 2013 - 2014 Custodio vs Court of Appeals 2533 SCRA 483 G.R. No. 116100 February 9, 1996 Full Case SPOUSES CRISTINO and BRIGIDA CUSTODIO and SPOUSES LITO and MARIA CRISTINA SANTOS, petitioners, vs. COURT OF APPEALS, HEIRS OF PACIFICO C. MABASA and REGIONAL TRIAL COURT OF PASIG, METRO MANILA, BRANCH 181, respondents. REGALADO, J.: This petition for review on certiorari assails the decision of respondent Court of Appeals in CA-G.R. CV No. 29115, promulgated on November 10, 1993, which affirmed with modification the decision of the trial court, as well as its resolution dated July 8, 1994 denying petitioner's motion for reconsideration. On August 26, 1982, Civil Case No. 47466 for the grant of an easement of right of way was filed by Pacifico Mabasa against Cristino Custodio, Brigida R. Custodio, Rosalina R. Morato, Lito Santos and Maria Cristina C. Santos before the Regional Trial Court of Pasig and assigned to Branch 22 thereof. The generative facts of the case, as synthesized by the trial court and adopted by the Court of Appeals, are as follows: Perusing the record, this Court finds that the original plaintiff Pacifico Mabasa died during the pendency of this case and was substituted by Ofelia Mabasa, his surviving spouse [and children]. The plaintiff owns a parcel of land with a two-door apartment erected thereon situated at Interior P. Burgos St., Palingon, Tipas, Tagig, Metro Manila. The plaintiff was able to acquire said property through a contract of sale with spouses Mamerto Rayos and Teodora Quintero as vendors last September 1981. Said property may be described to be surrounded by other immovables pertaining to defendants herein. Taking P. Burgos Street as the point of reference, on the left side, going to plaintiff's property, the row of houses will be as follows: That of defendants Cristino and Brigido Custodio, then that of Lito and Maria Cristina Santos and then that of Ofelia Mabasa. On the right side (is) that of defendant Rosalina Morato and then a Septic Tank (Exhibit "D"). As an access to P. Burgos Street from plaintiff's property, there are two possible passageways. The first passageway is approximately one meter wide and is about 20 meters distan(t) from Mabasa's residence to P. Burgos Street. Such path is passing in between the previously mentioned row of houses. The second passageway is about 3 meters in width and length from plaintiff Mabasa's residence to P. Burgos Street; it is about 26 meters. In passing thru said passageway, a less than a meter wide path through the septic tank and with 5-6 meters in length, has to be traversed. When said property was purchased by Mabasa, there were tenants occupying the remises and who were acknowledged by plaintiff Mabasa as tenants. However, sometime in February, 1982, one of said tenants vacated the apartment and when plaintiff Mabasa went to see the premises, he saw that there had been built an adobe fence in the first passageway making it narrower in width . Said adobe fence was first constructed by defendants Santoses along their property which is also along the first passageway. Defendant Morato constructed her adobe fence and even extended said fence in such a way that the entire passageway was enclosed. (Exhibit "1-Santoses and Custodios, Exh. "D" for plaintiff, Exhs. "1-C", "1-D" and "1-E") And it was then that the remaining tenants of said apartment vacated the area. Defendant Ma. Cristina Santos testified that she constructed said fence because there was an incident when her daughter was dragged by a bicycle pedalled by a son of one of the tenants in said apartment along the first passageway. She also mentioned some other inconveniences of having (at) the front of her house a pathway such as when some of the tenants were drunk and would bang their doors and windows. Some of their footwear were even lost. . . . (Emphasis in original text; corrections in parentheses supplied) On February 27, 1990, a decision was rendered by the trial court, with this dispositive part: Accordingly, judgment is hereby rendered as follows: 98

University of the Cordilleras College of Law First Year C S.Y. 2013 - 2014 1) Ordering defendants Custodios and Santoses to give plaintiff permanent access ingress and egress, to the public street; 2) Ordering the plaintiff to pay defendants Custodios and Santoses the sum of Eight Thousand Pesos (P8,000) as indemnity for the permanent use of the passageway. The parties to shoulder their respective litigation expenses. Not satisfied therewith, therein plaintiff represented by his heirs, herein private respondents, went to the Court of Appeals raising the sole issue of whether or not the lower court erred in not awarding damages in their favor. On November 10, 1993, as earlier stated, the Court of Appeals rendered its decision affirming the judgment of the trial court with modification, the decretal portion of which disposes as follows: WHEREFORE, the appealed decision of the lower court is hereby AFFIRMED WITH MODIFICATION only insofar as the herein grant of damages to plaintiffs-appellants. The Court hereby orders defendants-appellees to pay plaintiffs-appellants the sum of Sixty Five Thousand (P65,000) Pesos as Actual Damages, Thirty Thousand (P30,000) Pesos as Moral Damages, and Ten Thousand (P10,000) Pesos as Exemplary Damages. The rest of the appealed decision is affirmed to all respects. On July 8, 1994, the Court of Appeals denied petitioner's motion for reconsideration.Petitioners then took the present recourse to us, raising two issues, namely, whether or not the grant of right of way to herein private respondents is proper, and whether or not the award of damages is in order. With respect to the first issue, herein petitioners are already barred from raising the same. Petitioners did not appeal from the decision of the court a quo granting private respondents the right of way, hence they are presumed to be satisfied with the adjudication therein. With the finality of the judgment of the trial court as to petitioners, the issue of propriety of the grant of right of way has already been laid to rest. For failure to appeal the decision of the trial court to the Court of Appeals, petitioners cannot obtain any affirmative relief other than those granted in the decision of the trial court. That decision of the court below has become final as against them and can no longer be reviewed, much less reversed, by this Court. The rule in this jurisdiction is that whenever an appeal is taken in a civil case, an appellee who has not himself appealed may not obtain from the appellate court any affirmative relief other than what was granted in the decision of the lower court. The appellee can only advance any argument that he may deem necessary to defeat the appellant's claim or to uphold the decision that is being disputed, and he can assign errors in his brief if such is required to strengthen the views expressed by the court a quo. These assigned errors, in turn, may be considered by the appellate court solely to maintain the appealed decision on other grounds, but not for the purpose of reversing or modifying the judgment in the appellee's favor and giving him other affirmative reliefs. However, with respect to the second issue, we agree with petitioners that the Court of Appeals erred in awarding damages in favor of private respondents. The award of damages has no substantial legal basis. A reading of the decision of the Court of Appeals will show that the award of damages was based solely on the fact that the original plaintiff, Pacifico Mabasa, incurred losses in the form of unrealized rentals when the tenants vacated the leased premises by reason of the closure of the passageway. However, the mere fact that the plaintiff suffered losses does not give rise to a right to recover damages. To warrant the recovery of damages, there must be both a right of action for a legal wrong inflicted by the defendant, and damage resulting to the plaintiff therefrom. Wrong without damage, or damage without wrong, does not constitute a cause of action, since damages are merely part of the remedy allowed for the injury caused by a breach or wrong. There is a material distinction between damages and injury. Injury is the illegal invasion of a legal right; damage is the loss, hurt, or harm which results from the injury; and damages are the recompense or compensation awarded for the damage suffered. Thus, there can be damage without injury in those instances in which the loss or harm was not the result of a violation of a legal duty. These situations are often called damnum absque injuria. In order that a plaintiff may maintain an action for the injuries of which he complains, he must establish that such injuries resulted from a breach of duty which the defendant owed to the plaintiff a concurrence 99

University of the Cordilleras College of Law First Year C S.Y. 2013 - 2014 of injury to the plaintiff and legal responsibility by the person causing it. The underlying basis for the award of tort damages is the premise that an individual was injured in contemplation of law. Thus, there must first be the breach of some duty and the imposition of liability for that breach before damages may be awarded; it is not sufficient to state that there should be tort liability merely because the plaintiff suffered some pain and suffering. Many accidents occur and many injuries are inflicted by acts or omissions which cause damage or loss to another but which violate no legal duty to such other person, and consequently create no cause of action in his favor. In such cases, the consequences must be borne by the injured person alone. The law affords no remedy for damages resulting from an act which does not amount to a legal injury or wrong. In other words, in order that the law will give redress for an act causing damage, that act must be not only hurtful, but wrongful. There must be damnum et injuria. If, as may happen in many cases, a person sustains actual damage, that is, harm or loss to his person or property, without sustaining any legal injury, that is, an act or omission which the law does not deem an injury, the damage is regarded as damnum absque injuria. In the case at bar, although there was damage, there was no legal injury. Contrary to the claim of private respondents, petitioners could not be said to have violated the principle of abuse of right. In order that the principle of abuse of right provided in Article 21 of the Civil Code can be applied, it is essential that the following requisites concur: (1) The defendant should have acted in a manner that is contrary to morals, good customs or public policy; (2) The acts should be willful; and (3) There was damage or injury to the plaintiff. The act of petitioners in constructing a fence within their lot is a valid exercise of their right as owners, hence not contrary to morals, good customs or public policy. The law recognizes in the owner the right to enjoy and dispose of a thing, without other limitations than those established by law. It is within the right of petitioners, as owners, to enclose and fence their property. Article 430 of the Civil Code provides that "(e)very owner may enclose or fence his land or tenements by means of walls, ditches, live or dead hedges, or by any other means without detriment to servitudes constituted thereon." At the time of the construction of the fence, the lot was not subject to any servitudes. There was no easement of way existing in favor of private respondents, either by law or by contract. The fact that private respondents had no existing right over the said passageway is confirmed by the very decision of the trial court granting a compulsory right of way in their favor after payment of just compensation. It was only that decision which gave private respondents the right to use the said passageway after payment of the compensation and imposed a corresponding duty on petitioners not to interfere in the exercise of said right. Hence, prior to said decision, petitioners had an absolute right over their property and their act of fencing and enclosing the same was an act which they may lawfully perform in the employment and exercise of said right. To repeat, whatever injury or damage may have been sustained by private respondents by reason of the rightful use of the said land by petitioners is damnum absque injuria. A person has a right to the natural use and enjoyment of his own property, according to his pleasure, for all the purposes to which such property is usually applied. As a general rule, therefore, there is no cause of action for acts done by one person upon his own property in a lawful and proper manner, although such acts incidentally cause damage or an unavoidable loss to another, as such damage or loss is damnum absque injuria. When the owner of property makes use thereof in the general and ordinary manner in which the property is used, such as fencing or enclosing the same as in this case, nobody can complain of having been injured, because the incovenience arising from said use can be considered as a mere consequence of community life. The proper exercise of a lawful right cannot constitute a legal wrong for which an action will lie, although the act may result in damage to another, for no legal right has been invaded. One may use any lawful means to accomplish a lawful purpose and though the means adopted may cause damage to another, no cause of action arises in the latter's favor. An injury or damage occasioned thereby is damnum absque injuria. The courts can give no redress for hardship to an individual resulting from action reasonably calculated to achieve a lawful means.

100

University of the Cordilleras College of Law First Year C S.Y. 2013 - 2014 WHEREFORE, under the compulsion of the foregoing premises, the appealed decision of respondent Court of Appeals is hereby REVERSED and SET ASIDE and the judgment of the trial court is correspondingly REINSTATED.

101

University of the Cordilleras College of Law First Year C S.Y. 2013 - 2014 Case Digest Custodio vs Court of Appeals 2533 SCRA 483 G.R. No. 116100 Decided on: February 9, 1996 Ponente: Regalado, J.: Facts: The respondent (Pacifico Mabasa) owns a parcel of land with a two-door apartment erected thereon situated at Interior P. Burgos St., Palingon, Tipas, Tagig, Metro Manila. Said property may be described to be surrounded by other immovables pertaining to respondents herein. As an access to P. Burgos Street from respondents property, there are two possible passageways. The first passageway is approximately one meter wide and is about 20 meters distan(t) from Mabasas residence to P. Burgos Street. Such path is passing in between the previously mentioned row of houses of the petitioners The second passageway is about 3 meters in width and length from Mabasas residence to P. Burgos Street; it is about 26 meters. In passing thru said passageway, a less than a meter wide path through the septic tank and with 5-6 meters in length, has to be traversed. When said property was purchased by Mabasa, there were tenants occupying the remises and who were acknowledged by Mabasa as tenants. However, sometime in February, 1982, one of said tenants vacated the apartment and when Mabasa went to see the premises, he saw that there had been built an adobe fence in the first passageway making it narrower in width. Said adobe fence was first constructed by Petitioners Santoses along their property which is also along the first passageway. Petitioner Morato constructed her adobe fence and even extended said fence in such a way that the entire passageway was enclosed. And it was then that the remaining tenants of said apartment vacated the area. Petitioner Ma. Cristina Santos testified that she constructed said fence because of some other inconveniences of having (at) the front of her house a pathway such as when some of the tenants were drunk and would bang their doors and windows. Trial court rendered a decision ordering the Petitioners Custodios and Santoses to give Respondent Mabasa permanent access ingress and egress, to the public street and Mabasa to pay the Custodios and Santoses the sum of Eight Thousand Pesos (P8,000) as indemnity for the permanent use of the passageway. Respondent Mabasa went to the CA raising the sole issue of whether or not the lower court erred in not awarding damages in their favor. The CA rendered its decision affirming the judgment of the trial court with modification only insofar as the. grant of damages to Mabasa The motion for reconsideration filed by the petitioners was denied. Issues: Whether the grant of right of way to herein private respondent Mabasa is proper. Whether the award of damages is in order. Held: No. Herein petitioners are already barred from raising the same. Petitioners did not appeal from the decision of the court a quo granting private respondents the right of way, hence they are presumed to be satisfied with the adjudication therein. With the finality of the judgment of the trial court as to petitioners, the issue of propriety of the grant of right of way has already been laid to rest. No. A reading of the decision of the CA will show that the award of damages was based solely on the fact that the original plaintiff, Pacifico Mabasa, incurred losses in the form of unrealized rentals when the tenants vacated the leased premises by reason of the closure of the passageway.However, the mere fact that the plaintiff suffered losses does not give rise to a right to recover damages. There is a material distinction between damages and injury. Injury is the illegal invasion of a legal right; damage is the loss, hurt, or harm which results from the injury; and damages are the recompense or compensation awarded for the damage suffered. Thus, there can be damage without injury in those instances in which the loss or harm was not the result of a violation of a 102

University of the Cordilleras College of Law First Year C S.Y. 2013 - 2014 legal duty. (damnum absque injuria). In order that a plaintiff may maintain an action for the injuries of which he complains, he must establish that such injuries resulted from a breach of duty which the defendant owed to the plaintiff a concurrence of injury to the plaintiff and legal responsibility by the person causing it (damnum et injuria.) In the case at bar, although there was damage, there was no legal injury. The act of petitioners in constructing a fence within their lot is a valid exercise of their right as owners, hence not contrary to morals, good customs or public policy. The law recognizes in the owner the right to enjoy and dispose of a thing, without other limitations than those established by law. It is within the right of petitioners, as owners, to enclose and fence their property. Article 430 of the Civil Code provides that (e)very owner may enclose or fence his land or tenements by means of walls, ditches, live or dead hedges, or by any other means without detriment to servitudes constituted thereon. At the time of the construction of the fence, the lot was not subject to any servitudes. There was no easement of way existing in favor of private respondents, either by law or by contract. The fact that private respondents had no existing right over the said passageway is confirmed by the very decision of the trial court granting a compulsory right of way in their favor after payment of just compensation. Hence, prior to said decision, petitioners had an absolute right over their property and their act of fencing and enclosing the same was an act which they may lawfully perform in the employment and exercise of said right. To repeat, whatever injury or damage may have been sustained by private respondents by reason of the rightful use of the said land by petitioners is damnum absque injuria.

103

University of the Cordilleras College of Law First Year C S.Y. 2013 - 2014 Barons Marketing Corp. vs Court of Appeals 286 SCRA 96 G.R. No. 126486 February 9, 1998 Full Case BARONS MARKETING CORP., petitioner, vs. COURT OF APPEALS and PHELPS DODGE PHILS., INC. respondents. KAPUNAN, J.: The instant petition raises two issues: (1) whether or not private respondent is guilty of abuse of right; and (2) whether or not private respondent is entitled to interest and attorney's fees. The facts are undisputed: On August 31, 1973, plaintiff [Phelps Dodge, Philippines, Inc. private respondent herein] appointed defendant [petitioner Barons Marketing, Corporation] as one of its dealers of electrical wires and cables effective September 1, 1973 (Exh. A). As such dealer, defendant was given by plaintiff 60 days credit for its purchases of plaintiff's electrical products. This credit term was to be reckoned from the date of delivery by plaintiff of its products to defendant (Exh. 1). During the period covering December 1986 to August 17, 1987, defendant purchased, on credit, from plaintiff various electrical wires and cables in the total amount of P4,102,438.30 (Exh. B to K). These wires and cables were in turn sold, pursuant to previous arrangements, by defendant to MERALCO, the former being the accredited supplier of the electrical requirements of the latter. Under the sales invoices issued by plaintiff to defendant for the subject purchases, it is stipulated that interest at 12% on the amount due for attorney's fees and collection (Exh. BB). On September 7, 1987, defendant paid plaintiff the amount of P300,000.00 out of its total purchases as abovestated (Exh. S), thereby leaving an unpaid account on the aforesaid deliveries of P3,802,478.20. On several occasions, plaintiff wrote defendant demanding payment of its outstanding obligations due plaintiff (Exhs. L, M, N, and P). In response, defendant wrote plaintiff on October 5, 1987 requesting the latter if it could pay its outstanding account in monthly installments of P500,000.00 plus 1% interest per month commencing on October 15, 1987 until full payment (Exh. O and O-4). Plaintiff, however, rejected defendant's offer and accordingly reiterated its demand for the full payment of defendant's account (Exh. P). On 29 October 1987, private respondent Phelps Dodge Phils., Inc. filed a complaint before the Pasig Regional Trial Court against petitioner Barons Marketing Corporation for the recovery of P3,802,478.20 representing the value of the wires and cables the former had delivered to the latter, including interest. Phelps Dodge likewise prayed that it be awarded attorney's fees at the rate of 25% of the amount demanded, exemplary damages amounting to at least P100,000.00, the expenses of litigation and the costs of suit. Petitioner, in its answer, admitted purchasing the wires and cables from private respondent but disputed the amount claimed by the latter. Petitioner likewise interposed a counterclaim against private respondent, alleging that it suffered injury to its reputation due to Phelps Dodge's acts. Such acts were purportedly calculated to humiliate petitioner and constituted an abuse of rights. After hearing, the trial court on 17 June 1991 rendered its decision, the dispositive portion of which reads: WHEREFORE, from all the foregoing considerations, the Court finds Phelps Dodge Phils., Inc. to have preponderantly proven its case and hereby orders Barons Marketing, Inc. to pay Phelps Dodge the following: 1. P3,108,000.00 constituting the unpaid balance of defendant's purchases from plaintiff and interest thereon at 12% per annum computed from the respective expiration of the 60 day credit term, vis-a-vis the various sales invoices and/or delivery receipts; 2. 25% of the preceding obligation for and as attorney's fees; 104

University of the Cordilleras College of Law First Year C S.Y. 2013 - 2014 3. P10,000.00 as exemplary damages; 4. Costs of suit. Both parties appealed to respondent court. Private respondent claimed that the trial court should have awarded it the sum of P3,802,478.20, the amount which appeared in the body of the complaint and proven during the trial rather than P3,1081000.00 The latter amount appears in petitioner's prayer supposedly as a result of a typographical error. On the other hand, petitioner reiterated its claims for damages as a result of "creditor's abuse." It also alleged that private respondent failed to prove its cause of action against it. On 25 June 1996, the Court of Appeals rendered a decision modifying the decision of the trial court, thus: WHEREFORE, from all the foregoing considerations, the Court finds Phelps Dodge Phils., Inc. to have preponderantly proven its case and hereby orders Barons Marketing, Inc. to pay Phelps Dodge the following: 1. P3,802,478.20 constituting the unpaid balance of defendant's purchases from plaintiff and interest thereon at 12% per annum computed from the respective expiration of the 60 day credit term, vis-a-vis the various sales invoices and/or delivery receipts; and 2. 5% of the preceding obligation for and as attorney's fees. No costs. Petitioner Barons Marketing is now before this Court alleging that respondent court erred when it held (1) private respondent Phelps Dodge not guilty of "creditor's abuse," and (2) petitioner liable to private respondent for interest and attorney's fees. I Petitioner does not deny private respondent's rights to institute an action for collection and to claim full payment. Indeed, petitioner's right to file an action for collection is beyond cavil. 5 Likewise, private respondent's right to reject petitioner's offer to pay in installments is guaranteed by Article 1248 of the Civil Code which states: Art. 1248. Unless there is an express stipulation to that effect, the creditor cannot be compelled partially to receive the prestations in which the obligation consists. Neither may the debtor be required to make partial payments. However, when the debt is in part liquidated and in part unliquidated, the creditor may demand and the debtor may effect the payment of the former without waiting for the liquidation of the latter. Under this provision, the prestation, i.e., the object of the obligation, must be performed in one act, not in parts. Tolentino concedes that the right has its limitations: Partial Prestations. Since the creditor cannot be compelled to accept partial performance, unless otherwise stipulated, the creditor who refuses to accept partial prestations does not incur in delay or mora accipiendi, except when there is abuse of right or if good faith requires acceptance. Indeed, the law, as set forth in Article 19 of the Civil Code, prescribes a "primordial limitation on all rights" by setting certain standards that must be observed in the exercise thereof. Thus: Art. 19. Every person must, in the exercise of his rights and in the performance of his duties, act with justice, give everyone his due, and observe honesty and good faith. 105

University of the Cordilleras College of Law First Year C S.Y. 2013 - 2014 Petitioner now invokes Article 19 and Article 21 of the Civil Code, claiming that private respondent abused its rights when it rejected petitioner's offer of settlement and subsequently filed the action for collection considering: . . . that the relationship between the parties started in 1973 spanning more than 13 years before the complaint was filed, that the petitioner had been a good and reliable dealer enjoying a good credit standing during the period before it became delinquent in 1987, that the relationship between the parties had been a fruitful one especially for the private respondent, that the petitioner exerted its outmost efforts to settle its obligations and avoid a suit, that the petitioner did not evade in the payment of its obligation to the private respondent, and that the petitioner was just asking a small concession that it be allowed to liquidate its obligation to eight (8) monthly installments of P500,000.00 plus 1% interest per month on the balance which proposal was supported by post-dated checks. Expounding on its theory, petitioner states: In the ordinary course of events, a suit for collection of a sum of money filed in court is done for the primary purpose of collecting a debt or obligation. If there is an offer by the debtor to pay its debt or obligation supported by post-dated checks and with provision for interests, the normal response of a creditor would be to accept the offer of compromise and not file the suit for collection. It is of common knowledge that proceedings in our courts would normally take years before an action is finally settled. It is always wiser and more prudent to accept an offer of payment in installment rather than file an action in court to compel the debtor to settle his obligation in full in a single payment. xxx xxx xxx . . . Why then did private respondent elect to file a suit for collection rather than accept petitioner's offer of settlement, supported by post-dated checks, by paying monthly installments of P500,000.00 plus 1% per month commencing on October 15, 1987 until full payment? The answer is obvious. The action of private respondent in filling a suit for collection was an abuse of right and exercised for the sole purpose of prejudicing and injuring the petitioner. Petitioner prays that the Court order private respondent to pay petitioner moral and exemplary damages, attorney's fees, as well as the costs of suit. It likewise asks that it be allowed to liquidate its obligation to private respondent, without interests, in eight equal monthly installments. Petitioner's theory is untenable. Both parties agree that to constitute an abuse of rights under Article 19 the defendant must act with bad faith or intent to prejudice the plaintiff. They cite the following comments of Tolentino as their authority: Test of Abuse of Right. Modern jurisprudence does not permit acts which, although not unlawful, are anti-social. There is undoubtedly an abuse of right when it is exercised for the only purpose of prejudicing or injuring another. When the objective of the actor is illegitimate, the illicit act cannot be concealed under the guise of exercising a right. The principle does not permit acts which, without utility or legitimate purpose cause damage to another, because they violate the concept of social solidarity which considers law as rational and just. Hence, every abnormal exercise of a right, contrary to its socio-economic purpose, is an abuse that will give rise to liability. The exercise of a right must be in accordance with the purpose for which it was established, and must not be excessive or unduly harsh; there must be no intention to injure another. Ultimately, however, and in practice, courts, in the sound exercise of their discretion, will have to determine all the facts and circumstances when the exercise of a right is unjust, or when there has been an abuse of right. The question, therefore, is whether private respondent intended to prejudice or injure petitioner when it rejected petitioner's offer and filed the action for collection. We hold in the negative. It is an elementary rule in this jurisdiction that good faith is presumed and that the burden of proving bad faith rests upon the party alleging the same. In the case at bar, petitioner has failed to prove bad faith on the part of private respondent. Petitioner's allegation that private respondent 106

University of the Cordilleras College of Law First Year C S.Y. 2013 - 2014 was motivated by a desire to terminate its agency relationship with petitioner so that private respondent itself may deal directly with Meralco is simply not supported by the evidence. At most, such supposition is merely speculative. Moreover, we find that private respondent was driven by very legitimate reasons for rejecting petitioner's offer and instituting the action for collection before the trial court. As pointed out by private respondent, the corporation had its own "cash position to protect in order for it to pay its own obligations." This is not such "a lame and poor rationalization" as petitioner purports it to be. For if private respondent were to be required to accept petitioner's offer, there would be no reason for the latter to reject similar offers from its other debtors. Clearly, this would be inimical to the interests of any enterprise, especially a profit-oriented one like private respondent. It is plain to see that what we have here is a mere exercise of rights, not an abuse thereof Under these circumstances, we do not deem private respondent to have acted in a manner contrary to morals, good customs or public policy as to violate the provisions of Article 21 of the Civil Code. Consequently, petitioner's prayer for moral and exemplary damages must thus be rejected. Petitioner's claim for moral damages is anchored on Article 2219 (10) of the Civil Code which states: Art. 2219. Moral damages may be recovered in the following and analogous cases: xxx xxx xxx (10) Acts and actions referred to in articles 21, 26, 27, 28, 29, 30, 32, 34, and 35. xxx xxx xxx Having ruled that private respondent's acts did not transgress the provisions of Article 21, petitioner cannot be entitled to moral damages or, for that matter, exemplary damages. While the amount of exemplary damages need not be proved, petitioner must show that he is entitled to moral, temperate or compensatory damages before the court may consider the question of whether or not exemplary damages should be awarded. 13 As we have observed above; petitioner has failed to discharge this burden. It may not be amiss to state that petitioner's contract with private respondent has the force of law between them. 14 Petitioner is thus bound to fulfill what has been expressly stipulated therein. 15 In the absence of any abuse of right, private respondent cannot be allowed to perform its obligation under such contract in parts. Otherwise, private respondent's right under Article 1248 will be negated, the sanctity of its contract with petitioner defiled. The principle of autonomy of contracts 16 must be respected. II Under said contract, petitioner is liable to private respondent for the unpaid balance of its purchases from private respondent plus 12% interest. Private respondent's sales invoices expressly provide that: . . . Interest at 12% per annum will be charged on all overdue account plus 25% on said amount for attorney's fees and collection. . . . It may also be noted that the above stipulation, insofar as it provides for the payment of "25% on said amount for attorney's fees and collection (sic)," constitutes what is known as a penal clause. 18 Petitioner is thus obliged to pay such penalty in addition to the 12% annual interest, there being an express stipulation to that effect. Petitioner nevertheless urges this Court to reduce the attorney's fees for being "grossly excessive," "considering the nature of the case which is a mere action for collection of a sum of money." It may be pointed out however that the above penalty is supposed to answer not only for attorney's fees but for collection fees as well. Moreover: . . . the attorneys' fees here provided is not, strictly speaking, the attorneys' fees recoverable as between attorney and client spoken of and regulated by the Rules of Court. Rather, the attorneys' fees here are in the nature of liquidated damages and the stipulation therefor is aptly called a penal clause. It has been said that so long as such stipulation does not contravene law, morals, or 107

University of the Cordilleras College of Law First Year C S.Y. 2013 - 2014 public order, it is strictly binding upon defendant. The attorneys' fees so provided are awarded in favor of the litigant, not his counsel. It is the litigant, not counsel, who is the judgment creditor entitled to enforce the judgment by execution. 19 Nonetheless, courts are empowered to reduce such penalty if the same is "iniquitous or unconscionable." Article 1229 of the Civil Code states thus: Art. 1229. The judge shall equitably reduce the penalty when the principal obligation has been partly or been irregularly complied with by the debtor. Even if there has no performance, the penalty may also be reduced by the courts if it is iniquitous or unconscionable . (Emphasis supplied.) The sentiments of the law are echoed in Article 2227 of the same Code: Art. 2227. Liquidated damages, whether intended as an indemnity or a penalty, shall be equitably reduced if they are iniquitous or unconscionable. It is true that we have upheld the reasonableness of penalties in the form of attorney's fees consisting of twenty-five percent (25%) of the principal debt plus interest. 20 In the case at bar, however, the interest alone runs to some four and a half million pesos (P4.5M), even exceeding the principal debt amounting to almost four million pesos (P4.0M). Twenty five percent (25%) of the principal and interest amounts to roughly two million pesos (P2M). In real terms, therefore, the attorney's fees and collection fees are manifestly exorbitant. Accordingly, we reduce the same to ten percent (10%) of the principal. Private respondent, however, argues that petitioner failed to question the award of attorney's fees on appeal before respondent court and raised the issue only in its motion for reconsideration. Consequently, petitioner should be deemed to have waived its right to question such award. Private respondent's attempts to dissuade us from reducing the penalty are futile. The Court is clothed with ample authority to review matters, even if they are not assigned as errors in their appeal, if it finds that their consideration is necessary in arriving at a just decision of the case. 21 WHEREFORE, the decision of the Court of Appeals is hereby MODIFIED in that the attorney's and collection fees are reduced to ten percent (10%) of the principal but is AFFIRMED in all other respects. SO ORDERED.

108

University of the Cordilleras College of Law First Year C S.Y. 2013 - 2014 Case Digest Barons Marketing Corp. vs. CA G.R. No. 126486 Decided on: February 9, 1998 Ponente: KAPUNAN, J.: Facts: On August 31, 1973, plaintiff [Phelps Dodge, Philippines, Inc. private respondent herein] appointed defendant [petitioner Barons Marketing, Corporation] as one of its dealers of electrical wires and cables effective September 1, 1973. Defendant was given by plaintiff 60 days credit for its purchases of plaintiff's electrical products. During the period covering December 1986 to August 17, 1987, defendant purchased, on credit, from plaintiff various electrical wires and cables in the total amount of P4,102,438.30 and were sold to MERALCO. Under the sales invoices, it is stipulated that interest at 12% on the amount due for attorney's fees and collection defendant paid plaintiff the amount of P300,000.00 out of its total purchases leaving an unpaid amount of P3,802,478.20. plaintiff wrote defendant demanding payment. Defendant wrote plaintiff on October 5, 1987 requesting the latter if it could pay its outstanding account in monthly installments of P500,000.00 plus 1% interest per month commencing on October 15, 1987 until full payment. Plaintiff, however, rejected defendant's offer. private respondent Phelps Dodge Phils., Inc. filed a complaint before the Pasig Regional Trial Court against petitioner Barons Marketing Corporation for the recovery of P3,802,478.20. Petitioner, in its answer, admitted purchasing the wires and cables from private respondent but disputed the amount claimed by the latter. Petitioner likewise interposed a counterclaim against private respondent, alleging that it suffered injury to its reputation due to Phelps Dodge's acts. Such acts were purportedly calculated to humiliate petitioner and constituted an abuse of rights. The trial court ordered Barons Marketing, Inc. to pay Phelps Dodge in the amount of P3,108,000.00. Both parties appealed. Private respondent contends that the amount should be P3,802,478.20 and that there has been a clear typographical error in the decision. The Court of appeals modified the decision ruling in favour of private respondent. Thus, this appeal. Petitioner now alleges that its rights guaranteed by Arts. 19 and 20 of the Civil Code have been abused. Issue: Whether or not private respondent intended to prejudice or injure petitioner when it rejected petitioner's offer and filed the action for collection. Held: We hold in the negative. It is an elementary rule in this jurisdiction that good faith is presumed and that the burden of proving bad faith rests upon the party alleging the same. 12 In the case at bar, petitioner has failed to prove bad faith on the part of private respondent. Petitioner's allegation that private respondent was motivated by a desire to terminate its agency relationship with petitioner so that private respondent itself may deal directly with Meralco is simply not supported by the evidence. At most, such supposition is merely speculative. Moreover, we find that private respondent was driven by very legitimate reasons for rejecting petitioner's offer and instituting the action for collection before the trial court. As pointed out by private respondent, the corporation had its own "cash position to protect in order for it to pay its own obligations." This is not such "a lame and poor rationalization" as petitioner purports it to be. For if private respondent were to be required to accept petitioner's offer, there would be no reason for the latter to reject similar offers from its other debtors. Clearly, this would be inimical to the interests of any enterprise, especially a profit-oriented one like private respondent. It is plain to see that what we have here is a mere exercise of rights, not an abuse thereof Under these circumstances, we do not deem private respondent to have acted in a manner contrary to morals, good customs or public policy as to violate the provisions of Article 21 of the Civil Code.

109

University of the Cordilleras College of Law First Year C S.Y. 2013 - 2014 Globe Mackay vs Court of Appeals 176 SCRA 778 G.R. No. 81262 August 25, 1989 Full Case GLOBE MACKAY CABLE AND RADIO CORP., and HERBERT C. HENDRY, petitioners, vs. THE HONORABLE COURT OF APPEALS and RESTITUTO M. TOBIAS, respondents. Atencia & Arias Law Offices for petitioners. Romulo C. Felizmena for private respondent. CORTES, J.: Private respondent Restituto M. Tobias was employed by petitioner Globe Mackay Cable and Radio Corporation (GLOBE MACKAY) in a dual capacity as a purchasing agent and administrative assistant to the engineering operations manager. In 1972, GLOBE MACKAY discovered fictitious purchases and other fraudulent transactions for which it lost several thousands of pesos. According to private respondent it was he who actually discovered the anomalies and reported them on November 10, 1972 to his immediate superior Eduardo T. Ferraren and to petitioner Herbert C. Hendry who was then the Executive Vice-President and General Manager of GLOBE MACKAY. On November 11, 1972, one day after private respondent Tobias made the report, petitioner Hendry confronted him by stating that he was the number one suspect, and ordered him to take a one week forced leave, not to communicate with the office, to leave his table drawers open, and to leave the office keys. On November 20, 1972, when private respondent Tobias returned to work after the forced leave, petitioner Hendry went up to him and called him a "crook" and a "swindler." Tobias was then ordered to take a lie detector test. He was also instructed to submit specimen of his handwriting, signature, and initials for examination by the police investigators to determine his complicity in the anomalies. On December 6,1972, the Manila police investigators submitted a laboratory crime report (Exh. "A") clearing private respondent of participation in the anomalies. Not satisfied with the police report, petitioners hired a private investigator, retired Col. Jose G. Fernandez, who on December 10, 1972, submitted a report (Exh. "2") finding Tobias guilty. This report however expressly stated that further investigation was still to be conducted. Nevertheless, on December 12, 1972, petitioner Hendry issued a memorandum suspending Tobias from work preparatory to the filing of criminal charges against him. On December 19,1972, Lt. Dioscoro V. Tagle, Metro Manila Police Chief Document Examiner, after investigating other documents pertaining to the alleged anomalous transactions, submitted a second laboratory crime report (Exh. "B") reiterating his previous finding that the handwritings, signatures, and initials appearing in the checks and other documents involved in the fraudulent transactions were not those of Tobias. The lie detector tests conducted on Tobias also yielded negative results. Notwithstanding the two police reports exculpating Tobias from the anomalies and the fact that the report of the private investigator, was, by its own terms, not yet complete, petitioners filed with the City Fiscal of Manila a complaint for estafa through falsification of commercial documents, later amended to just estafa. Subsequently five other criminal complaints were filed against Tobias, four of which were for estafa through Falsification of commercial document while the fifth was for of Article 290 of' the Revised Penal Code (Discovering Secrets Through Seizure of Correspondence).lwph1.t Two of these complaints were refiled with the Judge Advocate General's Office, which however, remanded them to the fiscal's office. All of the six criminal complaints were dismissed by the fiscal. Petitioners appealed four of the fiscal's resolutions dismissing the criminal complaints with the Secretary of Justice, who, however, affirmed their dismissal. In the meantime, on January 17, 1973, Tobias received a notice (Exh. "F") from petitioners that his employment has been terminated effective December 13, 1972. Whereupon, Tobias filed a complaint for illegal dismissal. The labor arbiter dismissed the complaint. On appeal, the National Labor Relations Commission (NLRC) reversed the labor arbiter's decision. However, the Secretary of Labor, acting on 110

University of the Cordilleras College of Law First Year C S.Y. 2013 - 2014 petitioners' appeal from the NLRC ruling, reinstated the labor arbiter's decision. Tobias appealed the Secretary of Labor's order with the Office of the President. During the pendency of the appeal with said office, petitioners and private respondent Tobias entered into a compromise agreement regarding the latter's complaint for illegal dismissal. Unemployed, Tobias sought employment with the Republic Telephone Company (RETELCO). However, petitioner Hendry, without being asked by RETELCO, wrote a letter to the latter stating that Tobias was dismissed by GLOBE MACKAY due to dishonesty. Private respondent Tobias filed a civil case for damages anchored on alleged unlawful, malicious, oppressive, and abusive acts of petitioners. Petitioner Hendry, claiming illness, did not testify during the hearings. The Regional Trial Court (RTC) of Manila, Branch IX, through Judge Manuel T. Reyes rendered judgment in favor of private respondent by ordering petitioners to pay him eighty thousand pesos (P80,000.00) as actual damages, two hundred thousand pesos (P200,000.00) as moral damages, twenty thousand pesos (P20,000.00) as exemplary damages, thirty thousand pesos (P30,000.00) as attorney's fees, and costs. Petitioners appealed the RTC decision to the Court of Appeals. On the other hand, Tobias appealed as to the amount of damages. However, the Court of Appeals, an a decision dated August 31, 1987 affirmed the RTC decision in toto. Petitioners' motion for reconsideration having been denied, the instant petition for review on certiorari was filed. The main issue in this case is whether or not petitioners are liable for damages to private respondent. Petitioners contend that they could not be made liable for damages in the lawful exercise of their right to dismiss private respondent. On the other hand, private respondent contends that because of petitioners' abusive manner in dismissing him as well as for the inhuman treatment he got from them, the Petitioners must indemnify him for the damage that he had suffered. One of the more notable innovations of the New Civil Code is the codification of "some basic principles that are to be observed for the rightful relationship between human beings and for the stability of the social order." [REPORT ON THE CODE COMMISSION ON THE PROPOSED CIVIL CODE OF THE PHILIPPINES, p. 39]. The framers of the Code, seeking to remedy the defect of the old Code which merely stated the effects of the law, but failed to draw out its spirit, incorporated certain fundamental precepts which were "designed to indicate certain norms that spring from the fountain of good conscience" and which were also meant to serve as "guides for human conduct [that] should run as golden threads through society, to the end that law may approach its supreme ideal, which is the sway and dominance of justice" (Id.) Foremost among these principles is that pronounced in Article 19 which provides: Art. 19. Every person must, in the exercise of his rights and in the performance of his duties, act with justice, give everyone his due, and observe honesty and good faith. This article, known to contain what is commonly referred to as the principle of abuse of rights, sets certain standards which must be observed not only in the exercise of one's rights but also in the performance of one's duties. These standards are the following: to act with justice; to give everyone his due; and to observe honesty and good faith. The law, therefore, recognizes a primordial limitation on all rights; that in their exercise, the norms of human conduct set forth in Article 19 must be observed. A right, though by itself legal because recognized or granted by law as such, may nevertheless become the source of some illegality. When a right is exercised in a manner which does not conform with the norms enshrined in Article 19 and results in damage to another, a legal wrong is thereby committed for which the wrongdoer must be held responsible. But while Article 19 lays down a rule of conduct for the government of human relations and for the maintenance of social order, it does not provide a remedy for its violation. Generally, an action for damages under either Article 20 or Article 21 would be proper. Article 20, which pertains to damage arising from a violation of law, provides that: Art. 20. Every person who contrary to law, wilfully or negligently causes damage to another, shall indemnify the latter for the same.

111

University of the Cordilleras College of Law First Year C S.Y. 2013 - 2014 However, in the case at bar, petitioners claim that they did not violate any provision of law since they were merely exercising their legal right to dismiss private respondent. This does not, however, leave private respondent with no relief because Article 21 of the Civil Code provides that: Art. 21. Any person who wilfully causes loss or injury to another in a manner that is contrary to morals, good customs or public policy shall compensate the latter for the damage. This article, adopted to remedy the "countless gaps in the statutes, which leave so many victims of moral wrongs helpless, even though they have actually suffered material and moral injury" [ Id.] should "vouchsafe adequate legal remedy for that untold number of moral wrongs which it is impossible for human foresight to provide for specifically in the statutes" [Id. it p. 40; See also PNB v. CA, G.R. No. L27155, May 18,1978, 83 SCRA 237, 247]. In determining whether or not the principle of abuse of rights may be invoked, there is no rigid test which can be applied. While the Court has not hesitated to apply Article 19 whether the legal and factual circumstances called for its application [See for e.g., Velayo v. Shell Co. of the Phil., Ltd., 100 Phil. 186 (1956); PNB v. CA, supra; Grand Union Supermarket, Inc. v. Espino, Jr., G.R. No. L-48250, December 28, 1979, 94 SCRA 953; PAL v. CA, G.R. No. L-46558, July 31,1981,106 SCRA 391; United General Industries, Inc, v. Paler G.R. No. L-30205, March 15,1982,112 SCRA 404; Rubio v. CA, G.R. No. 50911, August 21, 1987, 153 SCRA 183] the question of whether or not the principle of abuse of rights has been violated resulting in damages under Article 20 or Article 21 or other applicable provision of law, depends on the circumstances of each case. And in the instant case, the Court, after examining the record and considering certain significant circumstances, finds that all petitioners have indeed abused the right that they invoke, causing damage to private respondent and for which the latter must now be indemnified. The trial court made a finding that notwithstanding the fact that it was private respondent Tobias who reported the possible existence of anomalous transactions, petitioner Hendry "showed belligerence and told plaintiff (private respondent herein) that he was the number one suspect and to take a one week vacation leave, not to communicate with the office, to leave his table drawers open, and to leave his keys to said defendant (petitioner Hendry)" [RTC Decision, p. 2; Rollo, p. 232]. This, petitioners do not dispute. But regardless of whether or not it was private respondent Tobias who reported the anomalies to petitioners, the latter's reaction towards the former upon uncovering the anomalies was less than civil. An employer who harbors suspicions that an employee has committed dishonesty might be justified in taking the appropriate action such as ordering an investigation and directing the employee to go on a leave. Firmness and the resolve to uncover the truth would also be expected from such employer. But the highhanded treatment accorded Tobias by petitioners was certainly uncalled for. And this reprehensible attitude of petitioners was to continue when private respondent returned to work on November 20, 1972 after his one week forced leave. Upon reporting for work, Tobias was confronted by Hendry who said. "Tobby, you are the crook and swindler in this company." Considering that the first report made by the police investigators was submitted only on December 10, 1972 [See Exh. A] the statement made by petitioner Hendry was baseless. The imputation of guilt without basis and the pattern of harassment during the investigations of Tobias transgress the standards of human conduct set forth in Article 19 of the Civil Code. The Court has already ruled that the right of the employer to dismiss an employee should not be confused with the manner in which the right is exercised and the effects flowing therefrom. If the dismissal is done abusively, then the employer is liable for damages to the employee [Quisaba v. Sta. Ines-Melale Veneer and Plywood Inc., G.R. No. L-38088, August 30, 1974, 58 SCRA 771; See also Philippine Refining Co., Inc. v. Garcia, G.R. No. L-21871, September 27,1966, 18 SCRA 107] Under the circumstances of the instant case, the petitioners clearly failed to exercise in a legitimate manner their right to dismiss Tobias, giving the latter the right to recover damages under Article 19 in relation to Article 21 of the Civil Code. But petitioners were not content with just dismissing Tobias. Several other tortious acts were committed by petitioners against Tobias after the latter's termination from work. Towards the latter part of January, 1973, after the filing of the first of six criminal complaints against Tobias, the latter talked to Hendry to protest the actions taken against him. In response, Hendry cut short Tobias' protestations by telling him to just confess or else the company would file a hundred more cases against him until he landed in jail. Hendry added that, "You Filipinos cannot be trusted." The threat unmasked petitioner's bad faith in the various actions taken against Tobias. On the other hand, the scornful remark about Filipinos as well as Hendry's earlier statements about Tobias being a "crook" and "swindler" are clear violations of 'Tobias' personal dignity [See Article 26, Civil Code]. 112

University of the Cordilleras College of Law First Year C S.Y. 2013 - 2014 The next tortious act committed by petitioners was the writing of a letter to RETELCO sometime in October 1974, stating that Tobias had been dismissed by GLOBE MACKAY due to dishonesty. Because of the letter, Tobias failed to gain employment with RETELCO and as a result of which, Tobias remained unemployed for a longer period of time. For this further damage suffered by Tobias, petitioners must likewise be held liable for damages consistent with Article 2176 of the Civil Code. Petitioners, however, contend that they have a "moral, if not legal, duty to forewarn other employers of the kind of employee the plaintiff (private respondent herein) was." [Petition, p. 14; Rollo, p. 15]. Petitioners further claim that "it is the accepted moral and societal obligation of every man to advise or warn his fellowmen of any threat or danger to the latter's life, honor or property. And this includes warning one's brethren of the possible dangers involved in dealing with, or accepting into confidence, a man whose honesty and integrity is suspect" [Id.]. These arguments, rather than justify petitioners' act, reveal a seeming obsession to prevent Tobias from getting a job, even after almost two years from the time Tobias was dismissed. Finally, there is the matter of the filing by petitioners of six criminal complaints against Tobias. Petitioners contend that there is no case against them for malicious prosecution and that they cannot be "penalized for exercising their right and prerogative of seeking justice by filing criminal complaints against an employee who was their principal suspect in the commission of forgeries and in the perpetration of anomalous transactions which defrauded them of substantial sums of money" [Petition, p. 10, Rollo, p. 11]. While sound principles of justice and public policy dictate that persons shall have free resort to the courts for redress of wrongs and vindication of their rights [Buenaventura v. Sto. Domingo, 103 Phil. 239 (1958)], the right to institute criminal prosecutions can not be exercised maliciously and in bad faith [Ventura v. Bernabe, G.R. No. L-26760, April 30, 1971, 38 SCRA 5871.] Hence, in Yutuk V. Manila Electric Co., G.R. No. L-13016, May 31, 1961, 2 SCRA 337, the Court held that the right to file criminal complaints should not be used as a weapon to force an alleged debtor to pay an indebtedness. To do so would be a clear perversion of the function of the criminal processes and of the courts of justice. And in Hawpia CA, G.R. No. L-20047, June 30, 1967. 20 SCRA 536 the Court upheld the judgment against the petitioner for actual and moral damages and attorney's fees after making a finding that petitioner, with persistence, filed at least six criminal complaints against respondent, all of which were dismissed. To constitute malicious prosecution, there must be proof that the prosecution was prompted by a design to vex and humiliate a person and that it was initiated deliberately by the defendant knowing that the charges were false and groundless [Manila Gas Corporation v. CA, G.R. No. L-44190, October 30,1980, 100 SCRA 602]. Concededly, the filing of a suit by itself, does not render a person liable for malicious prosecution [Inhelder Corporation v. CA, G.R. No. 52358, May 301983122 SCRA 576]. The mere dismissal by the fiscal of the criminal complaint is not a ground for an award of damages for malicious prosecution if there is no competent evidence to show that the complainant had acted in bad faith [Sison v. David, G.R. No. L-11268, January 28,1961, 1 SCRA 60]. In the instant case, however, the trial court made a finding that petitioners acted in bad faith in filing the criminal complaints against Tobias, observing that: xxx Defendants (petitioners herein) filed with the Fiscal's Office of Manila a total of six (6) criminal cases, five (5) of which were for estafa thru falsification of commercial document and one for violation of Art. 290 of the Revised Penal Code "discovering secrets thru seizure of correspondence," and all were dismissed for insufficiency or lack of evidence." The dismissal of four (4) of the cases was appealed to the Ministry of Justice, but said Ministry invariably sustained the dismissal of the cases. As above adverted to, two of these cases were refiled with the Judge Advocate General's Office of the Armed Forces of the Philippines to railroad plaintiffs arrest and detention in the military stockade, but this was frustrated by a presidential decree transferring criminal cases involving civilians to the civil courts. xxx To be sure, when despite the two (2) police reports embodying the findings of Lt. Dioscoro Tagle, Chief Document Examiner of the Manila Police Department, clearing plaintiff of participation or involvement in the fraudulent transactions complained of, 113

University of the Cordilleras College of Law First Year C S.Y. 2013 - 2014 despite the negative results of the lie detector tests which defendants compelled plaintiff to undergo, and although the police investigation was "still under follow-up and a supplementary report will be submitted after all the evidence has been gathered," defendants hastily filed six (6) criminal cases with the city Fiscal's Office of Manila, five (5) for estafa thru falsification of commercial document and one (1) for violation of Art. 290 of the Revised Penal Code, so much so that as was to be expected, all six (6) cases were dismissed, with one of the investigating fiscals, Asst. Fiscal de Guia, commenting in one case that, "Indeed, the haphazard way this case was investigated is evident. Evident likewise is the flurry and haste in the filing of this case against respondent Tobias," there can be no mistaking that defendants would not but be motivated by malicious and unlawful intent to harass, oppress, and cause damage to plaintiff. xxx [RTC Decision, pp. 5-6; Rollo, pp. 235-236]. In addition to the observations made by the trial court, the Court finds it significant that the criminal complaints were filed during the pendency of the illegal dismissal case filed by Tobias against petitioners. This explains the haste in which the complaints were filed, which the trial court earlier noted. But petitioners, to prove their good faith, point to the fact that only six complaints were filed against Tobias when they could have allegedly filed one hundred cases, considering the number of anomalous transactions committed against GLOBE MACKAY. However, petitioners' good faith is belied by the threat made by Hendry after the filing of the first complaint that one hundred more cases would be filed against Tobias. In effect, the possible filing of one hundred more cases was made to hang like the sword of Damocles over the head of Tobias. In fine, considering the haste in which the criminal complaints were filed, the fact that they were filed during the pendency of the illegal dismissal case against petitioners, the threat made by Hendry, the fact that the cases were filed notwithstanding the two police reports exculpating Tobias from involvement in the anomalies committed against GLOBE MACKAY, coupled by the eventual dismissal of all the cases, the Court is led into no other conclusion than that petitioners were motivated by malicious intent in filing the six criminal complaints against Tobias. Petitioners next contend that the award of damages was excessive. In the complaint filed against petitioners, Tobias prayed for the following: one hundred thousand pesos (P100,000.00) as actual damages; fifty thousand pesos (P50,000.00) as exemplary damages; eight hundred thousand pesos (P800,000.00) as moral damages; fifty thousand pesos (P50,000.00) as attorney's fees; and costs. The trial court, after making a computation of the damages incurred by Tobias [See RTC Decision, pp. 7-8; Rollo, pp. 154-1551, awarded him the following: eighty thousand pesos (P80,000.00) as actual damages; two hundred thousand pesos (P200,000.00) as moral damages; twenty thousand pesos (P20,000.00) as exemplary damages; thirty thousand pesos (P30,000.00) as attorney's fees; and, costs. It must be underscored that petitioners have been guilty of committing several actionable tortious acts, i.e., the abusive manner in which they dismissed Tobias from work including the baseless imputation of guilt and the harassment during the investigations; the defamatory language heaped on Tobias as well as the scornful remark on Filipinos; the poison letter sent to RETELCO which resulted in Tobias' loss of possible employment; and, the malicious filing of the criminal complaints. Considering the extent of the damage wrought on Tobias, the Court finds that, contrary to petitioners' contention, the amount of damages awarded to Tobias was reasonable under the circumstances. Yet, petitioners still insist that the award of damages was improper, invoking the principle of damnum absque injuria. It is argued that "[t]he only probable actual damage that plaintiff (private respondent herein) could have suffered was a direct result of his having been dismissed from his employment, which was a valid and legal act of the defendants-appellants (petitioners herein).lwph1.t " [Petition, p. 17; Rollo, p. 18]. According to the principle of damnum absque injuria, damage or loss which does not constitute a violation of a legal right or amount to a legal wrong is not actionable [Escano v. CA, G.R. No. L-47207, September 25, 1980, 100 SCRA 197; See also Gilchrist v. Cuddy 29 Phil, 542 (1915); The Board of Liquidators v. Kalaw, G.R. No. L-18805, August 14, 1967, 20 SCRA 987]. This principle finds no application in this case. It bears repeating that even granting that petitioners might have had the right to dismiss Tobias from work, the abusive manner in which that right was exercised amounted to a legal wrong for which petitioners must now be held liable. Moreover, the damage incurred by Tobias was not

114

University of the Cordilleras College of Law First Year C S.Y. 2013 - 2014 only in connection with the abusive manner in which he was dismissed but was also the result of several other quasi-delictual acts committed by petitioners. Petitioners next question the award of moral damages. However, the Court has already ruled in Wassmer v. Velez, G.R. No. L-20089, December 26, 1964, 12 SCRA 648, 653, that [p]er express provision of Article 2219 (10) of the New Civil Code, moral damages are recoverable in the cases mentioned in Article 21 of said Code." Hence, the Court of Appeals committed no error in awarding moral damages to Tobias. Lastly, the award of exemplary damages is impugned by petitioners. Although Article 2231 of the Civil Code provides that "[i]n quasi-delicts, exemplary damages may be granted if the defendant acted with gross negligence," the Court, in Zulueta v. Pan American World Airways, Inc., G.R. No. L- 28589, January 8, 1973, 49 SCRA 1, ruled that if gross negligence warrants the award of exemplary damages, with more reason is its imposition justified when the act performed is deliberate, malicious and tainted with bad faith. As in the Zulueta case, the nature of the wrongful acts shown to have been committed by petitioners against Tobias is sufficient basis for the award of exemplary damages to the latter. WHEREFORE, the petition is hereby DENIED and the decision of the Court of Appeals in CA-G.R. CV No. 09055 is AFFIRMED. SO ORDERED.

115

University of the Cordilleras College of Law First Year C S.Y. 2013 - 2014 Case Digest Globe Mackay vs. CA G.R. No. 81262 Decided on: August 25, 1989 Ponente: CORTES, J.: Facts: 10 November 1972, herein private respondent Restituto Tobias, a purchasing agent and administrative assistant to the engineering operations manager, discovered fictitious purchases and other fraudulent transactions, which caused Globe Mackay Cable and Radio Corp loss of several thousands of pesos. He reported it to his immediate superior Eduardo T. Ferraren and to the Executive Vice President and General Manager Herbert Hendry. A day after the report, Hendry told Tobias that he was number one suspect and ordered him one week forced leave. When Tobias returned to work after said leave, Hendry called him a crook and a swindler, ordered him to take a lie detector test, and to submit specimen of his handwriting, signature and initials for police investigation. Moreover, petitioners hired a private investigator. Private investigation was still incomplete; the lie detector tests yielded negative results; reports from Manila police investigators and from the Metro Manila Police Chief Document Examiner are in favor of Tobias. Petitioners filed with the Fiscals Office of Manila a total of six (6) criminal cases against private respondent Tobias, but were dismissed. Tobias received a notice of termination of his employment from petitioners in January 1973, effective December 1972. He sought employment with the Republic Telephone Company (RETELCO); but Hendry wrote a letter to RETELCO stating that Tobias was dismissed by Globe Mackay due to dishonesty. Tobias, then, filed a civil case for damages anchored on alleged unlawful, malicious, oppressive, and abusive acts of petitioners. The Regional Trial Court of Manila, Branch IX, through Judge Manuel T. Reyes rendered judgment in favor of private respondent, ordering petitioners to pay him eighty thousand pesos (P80,000.00) as actual damages, two hundred thousand pesos (P200,000.00) as moral damages, twenty thousand pesos (P20,000.00) as exemplary damages, thirty thousand pesos (P30,000.00) as attorney's fees, and costs; hence, this petition for review on certiorari. Issue: Whether or not petitioners are liable for damages to private respondent. Held: Yes. The Court, after examining the record and considering certain significant circumstances, finds that all petitioners have indeed abused the right that they invoke, causing damage to private respondent and for which the latter must now be indemnified: when Hendry told Tobias to just confess or else the company would file a hundred more cases against him until he landed in jail; his (Hendry) scornful remarks about Filipinos ("You Filipinos cannot be trusted.) as well as against Tobias (crook, and swindler); the writing of a letter to RETELCO stating that Tobias was dismissed by Globe Mackay due to dishonesty; and the filing of six criminal cases by petitioners against private respondent. All these reveal that petitioners are motivated by malicious and unlawful intent to harass, oppress, and cause damage to private respondent. The imputation of guilt without basis and the pattern of harassment during the investigations of Tobias transgress the standards of human conduct set forth in Article 19 of the Civil Code. WHEREFORE, the petition is DENIED and the decision of the Court of Appeals in CA-G.R. CV No. 09055 is AFFIRMED.

116

University of the Cordilleras College of Law First Year C S.Y. 2013 - 2014 RCPI vs Dionela 143 SCRA 657 G.R. No. L-44748 August 29, 1986 Full Case RADIO COMMUNICATIONS OF THE PHILS., INC. (RCPI). petitioner, vs. COURT OF APPEALS and LORETO DIONELA, respondents. O. Pythogoras Oliver for respondents. PARAS, J.: Before Us, is a Petition for Review by certiorari of the decision of the Court of Appeals, modifying the decision of the trial court in a civil case for recovery of damages against petitioner corporation by reducing the award to private respondent Loreto Dionela of moral damages from P40,000 to Pl5,000, and attorney's fees from P3,000 to P2,000. The basis of the complaint against the defendant corporation is a telegram sent through its Manila Office to the offended party, Loreto Dionela, reading as follows: 176 AS JR 1215PM 9 PAID MANDALUYONG JUL 22-66 LORETO DIONELA CABANGAN LEGASPI CITY WIRE ARRIVAL OF CHECK FER LORETO DIONELA-CABANGAN-WIRE ARRIVAL OF CHECK-PER 115 PM SA IYO WALANG PAKINABANG DUMATING KA DIYAN-WALA-KANG PADALA DITO KAHIT BULBUL MO (p. 19, Annex "A") Plaintiff-respondent Loreto Dionela alleges that the defamatory words on the telegram sent to him not only wounded his feelings but also caused him undue embarrassment and affected adversely his business as well because other people have come to know of said defamatory words. Defendant corporation as a defense, alleges that the additional words in Tagalog was a private joke between the sending and receiving operators and that they were not addressed to or intended for plaintiff and therefore did not form part of the telegram and that the Tagalog words are not defamatory. The telegram sent through its facilities was received in its station at Legaspi City. Nobody other than the operator manned the teletype machine which automatically receives telegrams being transmitted. The said telegram was detached from the machine and placed inside a sealed envelope and delivered to plaintiff, obviously as is. The additional words in Tagalog were never noticed and were included in the telegram when delivered. The trial court in finding for the plaintiff ruled as follows: There is no question that the additional words in Tagalog are libelous. They clearly impute a vice or defect of the plaintiff. Whether or not they were intended for the plaintiff, the effect on the plaintiff is the same. Any person reading the additional words in Tagalog will naturally think that they refer to the addressee, the plaintiff. There is no indication from the face of the telegram that the additional words in Tagalog were sent as a private joke between the operators of the defendant. The defendant is sued directly not as an employer. The business of the defendant is to transmit telegrams. It will open the door to frauds and allow the defendant to act with impunity if it can escape liability by the simple expedient of showing that its employees acted beyond the scope of their assigned tasks.

117

University of the Cordilleras College of Law First Year C S.Y. 2013 - 2014 The liability of the defendant is predicated not only on Article 33 of the Civil Code of the Philippines but on the following articles of said Code: ART. 19.- Every person must, in the exercise of his rights and in the performance of his duties, act with justice, give everyone his due, and observe honesty and good faith. ART. 20.-Every person who, contrary to law, wilfully or negligently causes damage to another, shall indemnify the latter for the same. There is sufficient publication of the libelous Tagalog words. The office file of the defendant containing copies of telegrams received are open and held together only by a metal fastener. Moreover, they are open to view and inspection by third parties. It follows that the plaintiff is entitled to damages and attorney's fees. The plaintiff is a businessman. The libelous Tagalog words must have affected his business and social standing in the community. The Court fixes the amount of P40,000.00 as the reasonable amount of moral damages and the amount of P3,000.00 as attorney's fee which the defendant should pay the plaintiff. (pp. 15-16, Record on Appeal) The respondent appellate court in its assailed decision confirming the aforegoing findings of the lower court stated: The proximate cause, therefore, resulting in injury to appellee, was the failure of the appellant to take the necessary or precautionary steps to avoid the occurrence of the humiliating incident now complained of. The company had not imposed any safeguard against such eventualities and this void in its operating procedure does not speak well of its concern for their clientele's interests. Negligence here is very patent. This negligence is imputable to appellant and not to its employees. The claim that there was no publication of the libelous words in Tagalog is also without merit. The fact that a carbon copy of the telegram was filed among other telegrams and left to hang for the public to see, open for inspection by a third party is sufficient publication. It would have been otherwise perhaps had the telegram been placed and kept in a secured place where no one may have had a chance to read it without appellee's permission. The additional Tagalog words at the bottom of the telegram are, as correctly found by the lower court, libelous per se, and from which malice may be presumed in the absence of any showing of good intention and justifiable motive on the part of the appellant. The law implies damages in this instance (Quemel vs. Court of Appeals, L-22794, January 16, 1968; 22 SCRA 44). The award of P40,000.00 as moral damages is hereby reduced to P15,000.00 and for attorney's fees the amount of P2,000.00 is awarded. (pp. 22-23, record) After a motion for reconsideration was denied by the appellate court, petitioner came to Us with the following: ASSIGNMENT OF ERRORS I The Honorable Court of Appeals erred in holding that Petitioner-employer should answer directly and primarily for the civil liability arising from the criminal act of its employee. II The Honorable Court of Appeals erred in holding that there was sufficient publication of the alleged libelous telegram in question, as contemplated by law on libel. III

118

University of the Cordilleras College of Law First Year C S.Y. 2013 - 2014 The Honorable Court of Appeals erred in holding that the liability of petitioner-companyemployer is predicated on Articles 19 and 20 of the Civil Code, Articles on Human Relations. IV The Honorable Court of Appeals erred in awarding Atty's. fees. (p. 4, Record) Petitioner's contentions do not merit our consideration. The action for damages was filed in the lower court directly against respondent corporation not as an employer subsidiarily liable under the provisions of Article 1161 of the New Civil Code in relation to Art. 103 of the Revised Penal Code. The cause of action of the private respondent is based on Arts. 19 and 20 of the New Civil Code (supra). As well as on respondent's breach of contract thru the negligence of its own employees. Petitioner is a domestic corporation engaged in the business of receiving and transmitting messages. Everytime a person transmits a message through the facilities of the petitioner, a contract is entered into. Upon receipt of the rate or fee fixed, the petitioner undertakes to transmit the message accurately. There is no question that in the case at bar, libelous matters were included in the message transmitted, without the consent or knowledge of the sender. There is a clear case of breach of contract by the petitioner in adding extraneous and libelous matters in the message sent to the private respondent. As a corporation, the petitioner can act only through its employees. Hence the acts of its employees in receiving and transmitting messages are the acts of the petitioner. To hold that the petitioner is not liable directly for the acts of its employees in the pursuit of petitioner's business is to deprive the general public availing of the services of the petitioner of an effective and adequate remedy. In most cases, negligence must be proved in order that plaintiff may recover. However, since negligence may be hard to substantiate in some cases, we may apply the doctrine of RES IPSA LOQUITUR (the thing speaks for itself), by considering the presence of facts or circumstances surrounding the injury. WHEREFORE, premises considered, the judgment of the appellate court is hereby AFFIRMED. SO ORDERED.

119

University of the Cordilleras College of Law First Year C S.Y. 2013 - 2014 Case Digest RCPI vs. Dionela G.R. No. L-44748 Decided on: August 29, 1986 Ponente: PARAS, J.: Facts: The basis of the complaint against the defendant corporation is a telegram sent through its Manila Office to the offended party, Loreto Dionela, which reads: SA IYO WALANG PAKINABANG DUMATING KA DIYAN-WALA-KANG PADALA DITO KAHIT BULBUL MO. Plaintiffrespondent Loreto Dionela alleges that the defamatory words on the telegram sent to him not only wounded his feelings but also caused him undue embarrassment and affected adversely his business as well because other people have come to know of said defamatory words. Defendant corporation as a defense, alleges that the additional words in Tagalog was a private joke between the sending and receiving operators and that they were not addressed to or intended for plaintiff and therefore did not form part of the telegram and that the Tagalog words are not defamatory. The Trial Court rule in favor of Dionela. On appeal, the CA affirmed the Trial Courts decision and likewise denied a motion for reconsideration. Issues: Whether or not Court of Appeals erred in holding that the liability of petitioner-companyemployer is predicated on Articles 19 and 20 of the Civil Code, Articles on Human Relations. Held: The cause of action of the private respondent is based on Arts. 19 and 20 of the New Civil Code (supra). As well as on respondent's breach of contract thru the negligence of its own employees. Petitioner is a domestic corporation engaged in the business of receiving and transmitting messages. Everytime a person transmits a message through the facilities of the petitioner, a contract is entered into. Upon receipt of the rate or fee fixed, the petitioner undertakes to transmit the message accurately. There is no question that in the case at bar, libelous matters were included in the message transmitted, without the consent or knowledge of the sender. There is a clear case of breach of contract by the petitioner in adding extraneous and libelous matters in the message sent to the private respondent. As a corporation, the petitioner can act only through its employees. Hence the acts of its employees in receiving and transmitting messages are the acts of the petitioner. To hold that the petitioner is not liable directly for the acts of its employees in the pursuit of petitioner's business is to deprive the general public availing of the services of the petitioner of an effective and adequate remedy. In most cases, negligence must be proved in order that plaintiff may recover. However, since negligence may be hard to substantiate in some cases, we may apply the doctrine of RES IPSA LOQUITUR (the thing speaks for itself), by considering the presence of facts or circumstances surrounding the injury.

120

University of the Cordilleras College of Law First Year C S.Y. 2013 - 2014 UE vs Jader 325 SCRA 804 G.R. No. 132344 February 17, 2000 Full Case UNIVERSITY OF THE EAST, petitioner, vs. ROMEO A. JADER, respondent. YNARES-SANTIAGO, J.: May an educational institution be held liable for damages for misleading a student into believing that the latter had satisfied all the requirements for graduation when such is not the case? This is the issue in the instant petition for review premised on the following undisputed facts as summarized by the trial court and adopted by the Court of Appeals (CA),1 to wit: Plaintiff was enrolled in the defendants' College of Law from 1984 up to 1988. In the first semester of his last year (School year 1987-1988), he failed to take the regular final examination in Practice Court I for which he was given an incomplete grade (Exhibits "2", also Exhibit "H"). He enrolled for the second semester as fourth year law student (Exhibit "A") and on February 1, 1988 he filed an application for the removal of the incomplete grade given him by Professor Carlos Ortega (Exhibits "H-2", also Exhibit "2") which was approved by Dean Celedonio Tiongson after payment of the required fee. He took the examination on March 28, 1988. On May 30, 1988, Professor Carlos Ortega submitted his grade. It was a grade of five (5). (Exhibits "H-4", also Exhibits "2-L", "2-N").1wphi1.nt In the meantime, the Dean and the Faculty Members of the College of Law met to deliberate on who among the fourth year students should be allowed to graduate. The plaintiff's name appeared in the Tentative List of Candidates for graduation for the Degree of Bachelor of Laws (LL.B) as of Second Semester (1987-1988) with the following annotation: JADER ROMEO A. Def. Conflict of Laws x-1-87-88, Practice Court I Inc., 1-87-88 C-1 to submit transcript with S.O. (Exhibits "3", "3-C-1", "3-C-2"). The 35th Investitures & Commencement Ceremonies for the candidates of Bachelor of Laws was scheduled on the 16th of April 1988 at 3:00 o'clock in the afternoon, and in the invitation for that occasion the name of the plaintiff appeared as one of the candidates. (Exhibits "B", "B-6", "B-6A"). At the foot of the list of the names of the candidates there appeared however the following annotation: This is a tentative list Degrees will be conferred upon these candidates who satisfactorily complete requirements as stated in the University Bulletin and as approved of the Department of Education, Culture and Sports (Exhibit "B-7-A"). The plaintiff attended the investiture ceremonies at F. dela Cruz Quadrangle, U.E., Recto Campus, during the program of which he went up the stage when his name was called, escorted by her (sic) mother and his eldest brother who assisted in placing the Hood, and his Tassel was turned from left to right, and he was thereafter handed by Dean Celedonio a rolled white sheet of paper symbolical of the Law Diploma. His relatives took pictures of the occasion (Exhibits "C" to "C-6", "D-3" to "D-11"). He tendered a blow-out that evening which was attended by neighbors, friends and relatives who wished him good luck in the forthcoming bar examination. There were pictures taken too during the blow-out (Exhibits "D" to "D-1"). He thereafter prepared himself for the bar examination. He took a leave of absence without pay from his job from April 20, 1988 to September 30, 1988 (Exhibit "G") and enrolled at the pre-bar

121

University of the Cordilleras College of Law First Year C S.Y. 2013 - 2014 review class in Far Eastern University. (Exhibits "F" to "F-2"). Having learned of the deficiency he dropped his review class and was not able to take the bar examination.2 Consequently, respondent sued petitioner for damages alleging that he suffered moral shock, mental anguish, serious anxiety, besmirched reputation, wounded feelings and sleepless nights when he was not able to take the 1988 bar examinations arising from the latter's negligence. He prayed for an award of moral and exemplary damages, unrealized income, attorney's fees, and costs of suit. In its answer with counterclaim, petitioner denied liability arguing mainly that it never led respondent to believe that he completed the requirements for a Bachelor of Laws degree when his name was included in the tentative list of graduating students. After trial, the lower court rendered judgment as follows: WHEREFORE, in view of the foregoing judgment is hereby rendered in favor of the plaintiff and against the defendant ordering the latter to pay plaintiff the sum of THIRTY FIVE THOUSAND FOUR HUNDRED SEVENTY PESOS (P35,470.00) with legal rate of interest from the filing of the complaint until fully paid, the amount of FIVE THOUSAND PESOS (P5,000.00) as attorney's fees and the cost of suit. Defendant's counterclaim is, for lack of merit, hereby dismissed. SO ORDERED.3 which on appeal by both parties was affirmed by the Court of Appeals (CA) with modification. The dispositive portion of the CA decision reads: WHEREFORE, in the light of the foregoing, the lower Court's Decision is hereby AFFIRMED with the MODIFICATION that defendant-appellee, in addition to the sum adjudged by the lower court in favor of plaintiff-appellant, is also ORDERED to pay plaintiff-appellant the amount of FIFTY THOUSAND (P50,000.00) PESOS for moral damages. Costs against defendant-appellee. SO ORDERED.4 Upon the denial of its motion for reconsideration, petitioner UE elevated the case to this Court on a petition for review under Rule 45 of the Rules of Court, arguing that it has no liability to respondent Romeo A. Jader, considering that the proximate and immediate cause of the alleged damages incurred by the latter arose out of his own negligence in not verifying from the professor concerned the result of his removal exam. The petition lacks merit. When a student is enrolled in any educational or learning institution, a contract of education is entered into between said institution and the student. The professors, teachers or instructors hired by the school are considered merely as agents and administrators tasked to perform the school's commitment under the contract. Since the contracting parties are the school and the student, the latter is not duty-bound to deal with the former's agents, such as the professors with respect to the status or result of his grades, although nothing prevents either professors or students from sharing with each other such information. The Court takes judicial notice of the traditional practice in educational institutions wherein the professor directly furnishes his/her students their grades. It is the contractual obligation of the school to timely inform and furnish sufficient notice and information to each and every student as to whether he or she had already complied with all the requirements for the conferment of a degree or whether they would be included among those who will graduate. Although commencement exercises are but a formal ceremony, it nonetheless is not an ordinary occasion, since such ceremony is the educational institution's way of announcing to the whole world that the students included in the list of those who will be conferred a degree during the baccalaureate ceremony have satisfied all the requirements for such degree. Prior or subsequent to the ceremony, the school has the obligation to promptly inform the student of any problem involving the latter's grades and performance and also most importantly, of the procedures for remedying the same. Petitioner, in belatedly informing respondent of the result of the removal examination, particularly at a time when he had already commenced preparing for the bar exams, cannot be said to have acted in good faith. Absence of good faith must be sufficiently established for a successful prosecution by the aggrieved 122

University of the Cordilleras College of Law First Year C S.Y. 2013 - 2014 party in a suit for abuse of right under Article 19 of the Civil Code. Good faith connotes an honest intention to abstain from taking undue advantage of another, even though the forms and technicalities of the law, together with the absence of all information or belief of facts, would render the transaction unconscientious.5 It is the school that has access to those information and it is only the school that can compel its professors to act and comply with its rules, regulations and policies with respect to the computation and the prompt submission of grades. Students do not exercise control, much less influence, over the way an educational institution should run its affairs, particularly in disciplining its professors and teachers and ensuring their compliance with the school's rules and orders. Being the party that hired them, it is the school that exercises general supervision and exclusive control over the professors with respect to the submission of reports involving the students' standing. Exclusive control means that no other person or entity had any control over the instrumentality which caused the damage or injury.6 The college dean is the senior officer responsible for the operation of an academic program, enforcement of rules and regulations, and the supervision of faculty and student services. 7 He must see to it that his own professors and teachers, regardless of their status or position outside of the university, must comply with the rules set by the latter. The negligent act of a professor who fails to observe the rules of the school, for instance by not promptly submitting a student's grade, is not only imputable to the professor but is an act of the school, being his employer. Considering further, that the institution of learning involved herein is a university which is engaged in legal education, it should have practiced what it inculcates in its students, more specifically the principle of good dealings enshrined in Articles 19 and 20 of the Civil Code which states: Art. 19. Every person must, in the exercise of his rights and in the performance of his duties, act with justice, give everyone his due, and observe honesty and good faith. Art. 20. Every person who, contrary to law, wilfully or negligently causes damage to another, shall indemnify the latter for the same. Art. 19 was intended to expand the concept of torts by granting adequate legal remedy for the untold number of moral wrongs which is impossible for human foresight to provide specifically in statutory law.8 In civilized society, men must be able to assume that others will do them no intended injury that others will commit no internal aggressions upon them; that their fellowmen, when they act affirmatively will do so with due care which the ordinary understanding and moral sense of the community exacts and that those with whom they deal in the general course of society will act in good faith. The ultimate thing in the theory of liability is justifiable reliance under conditions of civilized society.9 Schools and professors cannot just take students for granted and be indifferent to them, for without the latter, the former are useless. Educational institutions are duty-bound to inform the students of their academic status and not wait for the latter to inquire from the former. The conscious indifference of a person to the rights or welfare of the person/persons who may be affected by his act or omission can support a claim for damages. 10 Want of care to the conscious disregard of civil obligations coupled with a conscious knowledge of the cause naturally calculated to produce them would make the erring party liable.11 Petitioner ought to have known that time was of the essence in the performance of its obligation to inform respondent of his grade. It cannot feign ignorance that respondent will not prepare himself for the bar exams since that is precisely the immediate concern after graduation of an LL.B. graduate. It failed to act seasonably. Petitioner cannot just give out its student's grades at any time because a student has to comply with certain deadlines set by the Supreme Court on the submission of requirements for taking the bar. Petitioner's liability arose from its failure to promptly inform respondent of the result of an examination and in misleading the latter into believing that he had satisfied all requirements for the course. Worth quoting is the following disquisition of the respondent court: It is apparent from the testimony of Dean Tiongson that defendant-appellee University had been informed during the deliberation that the professor in Practice Court I gave plaintiff-appellant a failing grade. Yet, defendant-appellee still did not inform plaintiff-appellant of his failure to complete the requirements for the degree nor did they remove his name from the tentative list of candidates for graduation. Worse, defendant-appellee university, despite the knowledge that plaintiff-appellant failed in Practice Court I, again included plaintiff-appellant's name in the "tentative list of candidates for graduation which was prepared after the deliberation and which became the basis for the commencement rites program. Dean Tiongson reasons out that plaintiff123

University of the Cordilleras College of Law First Year C S.Y. 2013 - 2014 appellant's name was allowed to remain in the tentative list of candidates for graduation in the hope that the latter would still be able to remedy the situation in the remaining few days before graduation day. Dean Tiongson, however, did not explain how plaintiff appellant Jader could have done something to complete his deficiency if defendant-appellee university did not exert any effort to inform plaintiff-appellant of his failing grade in Practice Court I.12 Petitioner cannot pass on its blame to the professors to justify its own negligence that led to the delayed relay of information to respondent. When one of two innocent parties must suffer, he through whose agency the loss occurred must bear it.13 The modern tendency is to grant indemnity for damages in cases where there is abuse of right, even when the act is not illicit.14 If mere fault or negligence in one's acts can make him liable for damages for injury caused thereby, with more reason should abuse or bad faith make him liable. A person should be protected only when he acts in the legitimate exercise of his right, that is, when he acts with prudence and in good faith, but not when he acts with negligence or abuse.15 However, while petitioner was guilty of negligence and thus liable to respondent for the latter's actual damages, we hold that respondent should not have been awarded moral damages. We do not agree with the Court of Appeals' findings that respondent suffered shock, trauma and pain when he was informed that he could not graduate and will not be allowed to take the bar examinations. At the very least, it behooved on respondent to verify for himself whether he has completed all necessary requirements to be eligible for the bar examinations. As a senior law student, respondent should have been responsible enough to ensure that all his affairs, specifically those pertaining to his academic achievement, are in order. Given these considerations, we fail to see how respondent could have suffered untold embarrassment in attending the graduation rites, enrolling in the bar review classes and not being able to take the bar exams. If respondent was indeed humiliated by his failure to take the bar, he brought this upon himself by not verifying if he has satisfied all the requirements including his school records, before preparing himself for the bar examination. Certainly, taking the bar examinations does not only entail a mental preparation on the subjects thereof; there are also prerequisites of documentation and submission of requirements which the prospective examinee must meet. WHEREFORE, the assailed decision of the Court of Appeals is AFFIRMED with MODIFICATION. Petitioner is ORDERED to PAY respondent the sum of Thirty-five Thousand Four Hundred Seventy Pesos (P35,470.00), with legal interest of 6% per annum computed from the date of filing of the complaint until fully paid; the amount of Five Thousand Pesos (P5,000.00) as attorney's fees; and the costs of the suit. The award of moral damages is DELEIED.1wphi1.nt SO ORDERED.

124

University of the Cordilleras College of Law First Year C S.Y. 2013 - 2014 Case Digest University of the East vs. Jader G.R. No. 132344 Decided on; February 17, 2000 Ponente: YNARES-SNTIAGO, J.: Facts: Petitioner was enrolled in the defendants College of Law. He failed to take the regular examination in Practice Court 1 for which he was given an incomplete grade. He enrolled for the second semester as a fourth year student, and filed an application for the removal of the incomplete grade which was approved by the Dean. In the meantime, the faculty members and the Dean met to deliberate who among the fourth year students should be allowed to graduate. The plaintiffs name appeared on the tentative list, he also attended the investiture ceremonies to which he tendered blowout afterwards. He thereafter prepared himself for the bar examination and took review classes. However, he was not able to take the bar examination because his academic requirements is not complete. Consequently, respondent sued petitioner for damages alleging that he suffered moral shock besmirched reputation, wounded feelings, sleepless nights, when he was not able to take the 1988 bar examinations arising from the latters negligence. He prayed for an award of moral damages, unrealized income, attorneys fees and cost of suit. Issue: Whether or not an educational institution be held liable for damages for misleading a student into believing that the latter had satisfied all the requirements for graduation when such is not the case. Held: The Supreme Court held that UE is liable for damages. It is the contractual obligation of the school to timely inform and furnish sufficient notice and information to each and every student as to where he or she had already complied with the entire requirement for the conferment of a degree or whether they should be included among those who will graduate. The school cannot be said to have acted in good faith. Absence of good faith must be sufficiently established for a successful prosecution by the aggrieved party in suit for abuse of right under Article 19 of the Civil Code.

125

University of the Cordilleras College of Law First Year C S.Y. 2013 - 2014 Rama vs Court of Appeals 148 SCRA 496 G.R. No. L-44484 March 16, 1987 Full Case OSMUNDO G. RAMA, petitioner, vs. COURT OF APPEALS, JOSE ABALA, MELCHOR ABANGAN, EUTIQUIO ALEGRADO, EMIGDIO BLANCO, ISABELO CABUENAS, CESAR CAMILLO, JOSE CENIZA, ANDRES CAMPANA, FIDEL CORONEL, MARCIANO CUESTAS, IGNACIO DACLIZON, ROLAND ENRIQUEZ, DIONISIO FLORES, PATERNO FLORES, MODESTO GERALDE, CENON GESIN, LEONCIO GUMBOC, CLAUDIO LEGASPI, INOCENCIO LLANOS, HIPOLITO MANUBAG, MAURICIO MANACAP, CONSTANCIO MAMAYAGA, BIENVENIDO MATIS, MODESTO NAMONG, CATALINO OCHIA, CECILIO QUIJANO, HILARIO DE LOS SANTOS, FELICIANO SACARES, ENRIQUE SAROMINES, ALFONSO TABAY, ANGEL TEVES, SR., JAIME TRANI, RODULFO VERANO, VICENTE VILLARCA, DOROTEO ARMAS, ISABELO ABAPO, GREGORIO ABASTILLAS, RAFAEL ABASTILLAS, LORETO ALICAWAY, CIRIACO BARILLO, MIGUEL BINOLINAO, CELERINO BUTAY, IGNACIO BELLEZA, ANATOLIO BINOYA, ZACARIAS BUCARIZA, FERNANDO CASTRO, MARCIANO DE LA CERNA, VERANO BADANA, DONATO CABANERO, ANECITO DE LA CERNA, DIOSDADO CAETE, GABRIEL CAETE, ERIBERTO DACALOS, NONILO DE CASTILLA, SERGIO DAYANAN, FLAVIANO DEIPARINE, BERNARDO GAMBOA, ISMAEL GANTUANGCO, CESAR HERNANDEZ, JORGE JACA, GORGONIO JACALAN, SEVERIANO LANGBID, TOMAS LANGBID, DIOSDADO LASTIMADO, PABLO LUNA, MAXIMO LARIOSA, VICENTE LAPAZ, RICARDO MAGALLON, EMILIANO MATARIO, RAMON PADRIGA, NICANOR OPURA, ALBERTO MINTILLOSA, RUFINO REPONTE, BLAS PARDILLO, ESMAEL REGUDUS, MARCELIANO DELOS SANTOS, CANDIDO RUFLO, LUIS SALAPA, PEDRO SACEL, FRISCO SACEL, MIGUEL SARAMOSING, JULIAN VELOSO, BERNARDO TALLO, ARQUIPO YRAY, PATRICIO VILLARMIA, VICENTE VILLAMORA and LEONCIO ZABALA, respondents. ALAMPAY, J.: During the incumbency of Rene Espina as provincial governor of Cebu, Osmundo G. Rama as vicegovernor and Pablo P. Garcia, Reynaldo M. Mendiola and Valerians S. Carillo as members of the Sangguniang Panlalawigan, said officials adopted Resolution No. 990 which appropriated funds "for the maintenance and repair of provincial roads and bridges and for the operation and maintenance of the office of the provincial engineer and for other purposes." (L-44591, Rollo, pp. 34-37). In said resolution, the provincial government of Cebu under the aforementioned officials, declared its policy "to mechanize the maintenance and repair of all roads and bridges of the province (including provincial roads and bridges receiving national aid "JJ"), to economize in the expenditure of its Road and Bridge Fund for the maintenance and repair of provincial roads and bridges receiving national aid "JJ" and to adopt a more comprehensive, systematic, efficient, progressive and orderly operation and maintenance of the Office of the Provincial Engineer." To implement said policy, the provincial board resolved to abolish around thirty positions * the salaries of which were paid from the "JJ" Road and Bridge Fund thus doing away with the caminero (pick-shovelwheelbarrow) system Consequently around 200 employees of the province were eased out of their respective jobs and, to implement the mechanization program in the maintenance of roads and bridges, the provincial government purchased heavy equipment worth P4,000,000.00. However, contrary to its declared policy to economize the provincial administration later on hired around one thousand new employees, renovated the office of the provincial engineer and provided the latter with a Mercedes-Benz car (Decision in CA-G.R. No. 49328-R, L-44591, Rollo, p. 37). Aggrieved by these turn of events, the employees whose positions were abolished filed separate petitions for mandamus, damages and attorneys fees aimed at the annulment of Resolution No. 990, their reinstatement and the recovery of damages The aforementioned provincial officials who, together with the provincial auditor, provincial treasurer, provincial engineer and the province of Cebu, were named respondents in said action, were sued "both in their official and personal" capacities as a result of their alleged "unjust, oppressive, illegal and malicious' acts (Petition, Record in Civil Case No. R-10704, p. 3).

126

University of the Cordilleras College of Law First Year C S.Y. 2013 - 2014 In Civil Case No. R-10704, the Court of First Instance of Cebu declared Resolution No. 990 nun and void and ordered the respondent officials to re-create the positions abolished, to provide funds therefore, to reinstate the 56 petitioners headed by Jose Abala, and to pay them back salaries. For "lack of legal and factual basis," no damages were awarded to petitioners and no pronouncement as to attorney's fees were made as the petitioners had agreed to pay their lawyers 30% of whatever amount they would receive as back salaries (L-44591, Rollo, pp. 33-34). All the parties appealed to the Court of Appeals (CA-G.R. No. 49328-R). Eventually, said appellate court, through its First Division, affirmed the lower court's decision with the modification that respondents were ordered to pay jointly and severally in their "individual and personal capacity" P1,000.00 moral damages to each of the petitioners considering that the case involved a quasi-delict (L-44591 Rollo, p. 54). From that decision, Osmundo G. Rama, interposed an appeal' to this Court (G.R. No. L-44484). Espina, Garcia,' Mendiola and Carillo then filed their own petition for review (G.R. No. L-44591). But before Espina, et al. could file said petition, the province of Cebu and its Sangguniang Panlalawigan filed their own petition for review questioning that portion of the appellate court's decision which ordered the reinstatement with back salaries of the dismissed employees. Said petition, which was docketed as G.R. No. L-44572, was dismissed by this Court for lack of merit in the resolution of October 25, 1976. Entry of judgment was made on November 24, 1976. Meanwhile, dismissed employees Froilan Frondoso and Jeremias Luna, who also had filed their own petition for mandamus in the Court of First Instance of Cebu, elevated their case to the Court of Appeals (CA-G.R. No. SP-04649). In its decision, the Court of Appeals' Ninth Division followed the ruling of its First Division in CA-G. R. No. 48328-R, held that the wrong committed by the respondent Public officials was a quasi-delict and ordered the reinstatement with back salaries of Frondoso and Luna and the payment in solidum by respondent public officials of P1,000.00 each to Frondoso and Luna as moral damages plus P1,000.00 as attorney's fees. With the exception of Rama, the respondent public officials appealed to the Court (G.R. No. L-44842). Subsequently, the Cebu Assistant Provincial Attorney, representing the Province of Cebu and its Sangguniang Panlalawigan, also appealed to this Court from that decision (G.R. No. L-44894). On March 28, 1977, this Court resolved to consolidate G.R. Nos. L- 44484, L-44842, L-44591 and L44894 considering that said cases involve the same issues and factual background (L44591, Rollo, p. 344). Thereafter, Frondoso and Luna filed a motion to dismiss L-44894 and L-44842. They alleged that as the petition in L44572 had been dismissed on October 25, 1976; said two cases should likewise be dismissed because they, together with the private respondents in L-44572 who, like them, were also permanent appointees to their respective positions, "were separated from the service on the same date by the same petitioners" L-44894 Rollo, p. 140) and therefore, the petitions in L-44894 and L-44842 were barred by the rule of stare decision The motion to dismiss, however, was noted in the resolution of February 17, 1978, it appearing that said two cases had already been submitted for decision (L-44894 Rollo, p. 148; L-44842 Rollo, p. 139). Frondoso and Luna filed another motion to dismiss L-44894 but after the petitioners had filed their comment thereon, said motion to dismiss was also noted in the resolution of February 22, 1981 (L-44894 Rollo, p. 186). We find, however, that Frondoso's and Luna's contention that L-44894 should be dismissed is meritorious. The issues raised in L-44894 and L- 44572 are the same. In fact, the prayer in the petition in L-44894 is virtually a verbatim reiteration of that in L-44572. The allegation of petitioner province of Cebu and its Sangguniang Panlalawigan that the question of jurisdiction was not raised in L-44572 (L44894 Rollo, p. 150) cannot successfully save L-44894 from dismissal. In their petition, the province of Cebu and its Sangguniang Panlalawigan merely argued that the Court of Appeals did not acquire jurisdiction over the case, considering that Frondoso and Luna's appeal was perfected after the expiration of the reglementary period and that their brief was filed one month too late. However, the trend of the rulings of this Court in matters pertaining to the timeliness of the perfection of an appeal is to afford every party-litigant amplest opportunity to present their case "for the proper and just determination of his cause, freed from the constraints of technicalities." (Rodriguez vs. Court of Appeals,

127

University of the Cordilleras College of Law First Year C S.Y. 2013 - 2014 L-37522, November 28, 1975, 68 SCRA 262). Applying the above ruling to this case, the Court of Appeals may not, therefore, be faulted for assuming jurisdiction over the appeal of Frondoso and Luna. Hence, with respect to L-44894, this Court is bound by the dismissal of L- 44572 and so L-44894 should likewise be dismissed, as it is hereby dismissed. Proceeding now to resolve the issue, common to L-44484, L-44591 and L-44842, which is whether or not Espina, Rama, Garcia, Mendiola and Carillo are personally liable for damages for adopting a resolution which abolished positions to the detriment of the occupants thereof, this Court has held that, at least, in principle, a public officer by virtue of his office alone, is not immune from damages in his personal capacity arising from illegal acts done in bad faith. A different rule would sanction the use of public office as a tool of oppression (Tabuena vs. Court of Appeals, L-16290, October 31, 1961, 3 SCRA 413). Thus, in Correa vs. CFI of Bulacan, L-46096, July 30, 1979, 92 SCRA 312, We held personally liable a mayor who illegally dismissed policemen even if he had relinquished his position. Therein, We held that: A public officer who commits a tort or other wrongful act, done in excess or beyond the scope of his duty, is not protected by Ms office and is personally liable therefor like any private individual (Palma vs. Graciano, 99 Phil. 72, 74; Carreon vs. Province of Pampanga, 99 Phil. 808). This principle of personal liability has been applied to cases where a public officer removes another officer or discharges an employee wrongfully, the reported cases saying that by reason of noncompliance with the requirements of law in respect to removal from office, the officials were acting outside of their official authority (Stiles vs. Lowell 233 Mass. 174, 123 NE 615, 4 ALR 1365, cited in 63 Am. Jur. 2d. 770). We hold that the petitioners in the instant three cases are personally liable for damages because of their precipitate dismissal of provincial employees through an ostensibly legal means. The Court of Appeals, whose factual findings are binding on this Court, found that the provincial employees concerned were "eased out because of their party affiliation." i.e., they belonged to the Liberal Party whose presidential candidate then was Sergio Osmena Jr. (CA Decision in G.R. No. 49328-R, p. 6, L-44591, Rollo, p. 38). Such act of the petitioners reflected their malicious intent to do away with the followers of the rival political party so as to accommodate their own proteges who, it turned out, even outnumbered the dismissed employees. Indeed, municipal officers are liable for damages if they act maliciously or wantonly and if the work which they perform is done rather to injure an individual than to discharge a public duty (56 Am. Jur. 2d 334, citing Yearly V. Fink 43 Pa 212). As we have held in Vda de Laig vs. Court of Appeals, L-26882, April 5, 1978, 82 SCRA 294, 307-308, a public officer is civilly liable for failure to observe honesty and good faith in the performance of their duties as public officers or for wilfully or negligently causing damage to another (Article 20, Civil Code) or for wilfully causing loss or injury to another in a manner that is contrary to morals, good customs and/or public policy (Article 21, New Civil Code). Neither can petitioners shield themselves from liability by invoking the ruling in the cases of Carino vs. Agricultural Credit and Cooperative Financing Administration L-23966, May 22, 1969, 28 SCRA 268. In those cases, the erring public officials were sued in their official capacities whereas in the instant cases, petitioners were specifically sued in their personal capacities. For their part, the dismissed employees are entitled to damages because they have suffered a special and peculiar injury from the wrongful act of which they complain Mechem, A Treatise on the Law of Public Offices and Officers, p. 391). It is an undeniable fact that the dismissed employees who were holding such positions as foremen, watchmen and drivers, suffered the uncertainties of the unemployed when they were plucked out of their positions. That not all of them testified as to the extent of damages they sustained on account of their separation from their government jobs, cannot be used as a defense by the petitioners. Suffice it to state that considering the positions they were holding, the dismissed employees concerned belong to a low-salaried group, who, if deprived of wages would generally incur considerable economic hardships. Justice demands that they be recompensed for the predicament they were placed in, apart from the back salaries which they are entitled to as a matter of right. We are inclined to agree that the amount of

128

University of the Cordilleras College of Law First Year C S.Y. 2013 - 2014 P1,000.00 damages granted to each of them by the Court of Appeals was fixed by that court judiciously and is a reasonable sum (Article 2216, Civil Code). Petitioner Rama's protestations that when he eventually became the governor of Cebu, he reinstated most of the dismissed employees through provincial board Resolution No. 392 (L-44484 Rollo, p. 16) cannot erase the fact that he had a hand in the adoption of Resolution No. 990. His subsequent benevolent act cannot sufficiently make up for the damage suffered by the dismissed employees during their period of unemployment. Apropos the practice of victorious politicians to remove government employees who did not support them in their campaign for office, this Court has said: "There are altogether too many cases of this nature, wherein local elective officials, upon assumption to office, wield their new-found power indiscriminately by replacing employees with their own proteges regardless of the laws and regulations governing the civil service. Victory at the polls should not be taken as authority for the commission of such illegal acts." (Nemenzo vs. Sabillano, L-20977, September 7, 1968, 25 SCRA 1.) WHEREFORE, in L-44894, the petition for review on certiorari is hereby dismissed for lack of merit. In L-44484, L-44591 and L-44842, the decision of the First and Ninth Divisions of the Court of Appeals are hereby AFFIRMED with costs against the petitioners. SO ORDERED.

129

University of the Cordilleras College of Law First Year C S.Y. 2013 - 2014 Case Digest Rama vs. CA G.R. No. L-44484 Decided on: March 16, 1987 Ponente: ALAMPAY, J.: Facts: A resolution was passed by herein petitioner Rama, then Vice Governor or Cebu, with corespondents Mandeola and Castillo, members Sangguniang Panlunsod of Cebu, to mechanize the maintenance and repair of all roads and bridges of the province, to economize in the expenditure of its Road and Bridges (R&B) Fund, etc. To implement such policy, the Provincial Board resolved to abolish around 30 positions and 200 employees were dismissed the salaries of whom were derived from the R&B fund. The Local Government, however, bought heavy equipment worth 4 million pesos, hired around 1000 new employees, renovated the office of the provincial engineer and provided him with a Mercedes Benz. Upon petition by herein respondents (dismissed employees), the then CFI of Cebu declared said Resolution null and void and ordered the reinstatements of 56 dismissed employees and pay their back wages. Upon appeal by both parties, then CA affirmed the lower courts decision, plus an award of moral damages of P1000 for each of the employees, considering that the case involved quasi-delict. The CA found that the employees were dismissed because of their different political affiliations that they were identified with the Liberal Party of Sergio Osmea Jr. ISSUE: Whether or not petitioners-public officials (Gov. Espina, Rama, Mendiola and Carillo) are personally liable for damages for adopting a resolution which abolished positions to the detriment of the occupants thereof. HELD: In principle, a public officer by virtue of his office alone is not immune from damages in his personal capacity arising from illegal acts done in bad faith. A different rule sould sanction the use of public office as a tool of oppression. [Tabuena vs. CA, 8 SCRA 413,1961]. Thus, in Correa vs. CFI of Bulacan, 92 SCRA 312 (1979), a mayor was held liable for illegally dismissing a policeman even if he had relinquished his position. The SC in that case held that a public officer who commits a tort or other wrongful act, done in excess or beyond the scope of his duty is not protected by his office and is personally liable thereof like any private individual. This personal liability has been applied to cases where a public officer removes another officer or discharges an employee wrongfully, the reported cases saying that by reason of non-compliance with the requirements of law in respect to removal from office, the officials were acting outside their official authority. The officials in these consolidated cases are personally liable for damages because their precipitate dismissal of provincial employees through an ostensibly legal means. Such act of the petitioners of dismissing employees who are of rival political party, to recommend their own protges who even outnumbered the dismissed employees, reflected the petitioners malicious intent. Municipal officers are liable for damages if they act maliciously or wantonly, and if the work which they perform is done rather to injure an individual than to discharge a public duty. A public officer is civilly liable for failure to observe honestly and in good faith in the performance of their duties as public officers or for willfully or negligently causing loss or injury to another (Art. 20, CC) or for willfully causing loss or injury to another in a manner that is contrary to morals, good custom and/or public policy (Art. 21, CC). Dismissed employees are entitled to damages because they suffered a special and peculiar injury from the wrongful act. The dismissed employees who were holding such positions as foreman, watchman and drivers belonged to a low-salaried group, who, if deprived of wages, would generally incur considerable economic hardships.

130

University of the Cordilleras College of Law First Year C S.Y. 2013 - 2014 Pe vs Pe 5 SCRA 200 G.R. No. L-17396 May 30, 1962 Full Case CECILIO PE, ET AL., plaintiffs-appellants, vs. ALFONSO PE, defendant-appellee. BAUTISTA ANGELO, J.: Plaintiffs brought this action before the Court of First Instance of Manila to recover moral, compensatory, exemplary and corrective damages in the amount of P94,000.00 exclusive of attorneys fees and expenses of litigation. Defendant, after denying some allegations contained in the complaint, set up as a defense that the facts alleged therein, even if true, do not constitute a valid cause of action. After trial, the lower court, after finding that defendant had carried on a love affair with one Lolita Pe, an unmarried woman, being a married man himself, declared that defendant cannot be held liable for moral damages it appearing that plaintiffs failed to prove that defendant, being aware of his marital status, deliberately and in bad faith tried to win Lolitas affection. So it rendered decision dismissing the complaint. Plaintiffs brought this case on appeal before this Court on the ground that the issues involved are purely of law. The facts as found by the trial court are: Plaintiffs are the parents, brothers and sisters of one Lolita Pe. At the time of her disappearance on April 14, 1957, Lolita was 24 years old and unmarried. Defendant is a married man and works as agent of the La Perla Cigar and Cigarette Factory. He used to stay in the town of Gasan, Marinduque, in connection with his aforesaid occupation. Lolita was staying with her parents in the same town. Defendant was an adopted son of a Chinaman named PeBeco, a collateral relative of Lolitas father. Because of such fact and the similarity in their family name, defendant became close to the plaintiffs who regarded him as a member of their family. Sometime in 1952, defendant frequented the house of Lolita on the pretext that he wanted her to teach him how to pray the rosary. The two eventually fell in love with each other and conducted clandestine trysts not only in the town of Gasan but also in Boac where Lolita used to teach in a barrio school. They exchanged love notes with each other the contents of which reveal not only their infatuation for each other but also the extent to which they had carried their relationship. The rumors about their love affairs reached the ears of Lolitas parents sometime, in 1955, and since then defendant was forbidden from going to their house and from further seeing Lolita. The plaintiffs even filed deportation proceedings against defendant who is a Chinese national. The affair between defendant and Lolita continued nonetheless. Sometime in April, 1957, Lolita was staying with her brothers and sisters at their residence at 54-B Espaa Extension, Quezon City. On April 14, 1957, Lolita disappeared from said house. After she left, her brothers and sisters checked up her thing and found that Lolitas clothes were gone. However, plaintiffs found a note on a crumpled piece of paper inside Lolitas aparador. Said note, written on a small slip of paper approximately 4 by 3 in size, was in a handwriting recognized to be that of defendants. In English it reads: Honey, suppose I leave here on Sunday night, and thats 13th of this month and we will have a date on the 14th, thats Monday morning at 10 a.m.

Reply Love The disappearance of Lolita was reported to the police authorities and the NBI but up to the present there is no news or trace of her whereabouts. The present action is based on Article 21 of the New Civil Code which provides:

131

University of the Cordilleras College of Law First Year C S.Y. 2013 - 2014 Any person who wilfully causes loss or injury to another in a manner which is contrary to morals, good customs or public policy shall compensate the latter for the damage. There is no doubt that the claim of plaintiffs for damages is based on the fact that defendant, being a married man, carried on a love affair with Lolita Pe thereby causing plaintiffs injury in a manner contrary to morals, good customs and public policy. But in spite of the fact that plaintiffs have clearly established that in illicit affair was carried on between defendant and Lolita which caused great damage to the name and reputation of plaintiffs who are her parents, brothers and sisters, the trial court considered their complaint not actionable for the reason that they failed to prove that defendant deliberately and in bad faith tried to win Lolitas affection Thus, the trial court said: In the absence of proof on this point, the court may not presume that it was the defendant who deliberately induced such relationship. We cannot be unmindful of the uncertainties and sometimes inexplicable mysteries of the human emotions. It is a possibility that the defendant and Lolita simply fell in love with each other, not only without any desire on their part, but also against their better judgment and in full consciousness of what it will bring to both of them. This is specially so with respect to Lolita, being an unmarried woman, falling in love with defendant who is a married man. We disagree with this view. The circumstances under which defendant tried to win Lolitas affection cannot lead, to any other conclusion than that it was he who, thru an ingenious scheme or trickery, seduced the latter to the extent of making her fall in love with him. This is shown by the fact that defendant frequented the house of Lolita on the pretext that he wanted her to teach him how to pray the rosary. Because of the frequency of his visits to the latters family who was allowed free access because he was a collateral relative and was considered as a member of her family, the two eventually fell in love with each other and conducted clandestine love affairs not only in Gasan but also in Boac where Lolita used to teach in a barrio school. When the rumors about their illicit affairs reached the knowledge of her parents, defendant was forbidden from going to their house and even from seeing Lolita. Plaintiffs even filed deportation proceedings against defendant who is a Chinese national. Nevertheless, defendant continued his love affairs with Lolita until she disappeared from the parental home. Indeed, no other conclusion can be drawn from this chain of events than that defendant not only deliberately, but through a clever strategy, succeeded in winning the affection and love of Lolita to the extent of having illicit relations with her. The wrong he has caused her and her family is indeed immeasurable considering the fact that he is a married man. Verily, he has committed an injury to Lolitas family in a manner contrary to morals, good customs and public policy as contemplated in Article 21 of the new Civil Code. WHEREFORE, the decision appealed from is reversed. Defendant is hereby sentenced to pay the plaintiffs the sum of P5,000.00 as damages and P2,000.00 as attorneys fees and expenses of litigations. Costs against appellee.

132

University of the Cordilleras College of Law First Year C S.Y. 2013 - 2014 Case Digest Pe vs Pe 5 SCRA 200 G.R. No. L-17396 Decided on: May 30, 1962 Ponente: BAUTISTA ANGELO, J.: FACTS: Alfonso Pe, the defendant, was a married man, agent of La Perla Cigar and Cigarette Factory in GasanMarinduque who was treated like a son by CecilioPe, one of the petitioners. Cecilio introduced Alfonso to his children and was given access to visit their house. Alfonso got fond of Lolita, 24 year old single, daughter of Cecilio. The defendant frequented the house of Lolita sometime in 1952 on the pretext that he wanted her to teach him how to pray the rosary. Eventually they fell in love with each other. Plaintiff brought action before lower court of Manila and failed to prove Alfonso deliberately and in bad faith tried to win Lolitas affection. The case on moral damages was dismissed. ISSUE: Whether or not defendant is liable to Lolitas family on the ground of moral, good custom and public policy due to their illicit affair. HELD: Alfonso committed an injury to Lolitas family in a manner contrary to morals, good customs and public policy contemplated in Article 20 of the civil code. The defendant took advantage of the trust of Cecilio and even used the praying of rosary as a reason to get close with Lolita. The wrong caused by Alfonso is immeasurable considering the fact that he is a married man. WHEREFORE, the decision appealed from is reversed. Defendant is hereby sentenced to pay the plaintiffs the sum of P5,000.00 as damages and P2,000.00 as attorney's fees and expenses of litigations. Costs against appellee.

133

University of the Cordilleras College of Law First Year C S.Y. 2013 - 2014 Hermosisima vs Court of Appeals 109 Phil 629 G.R. No. L-14628 September 30, 1960 Full Case FRANCISCO HERMOSISIMA, petitioner, vs. THE HON. COURT OF APPEALS, ET AL., respondents. CONCEPCION, J.: An appeal by certiorari, taken by petitioner Francisco Hermosisima, from a decision of Court of Appeals modifying that of the Court of First Instance of Cebu. On October 4, 1954, Soledad Cagigas, hereinafter referred to as complaint, filed with said of her child, Chris Hermosisima, as natural child and moral damages for alleged breach of promise. Petitioner admitted the paternity of child and expressed willingness to support the latter, but denied having ever promised to marry the complainant. Upon her motion, said court ordered petitioner, on October 27, 1954, to pay, by way of alimony pendente lite, P50.00 a month, which was, on February 16, 1955, reduced to P30.00 a month. In due course, later on, said court rendered a decision the dispositive part of which reads: WHEREFORE, judgment is hereby rendered, declaring the child, Chris Hermosisima, as the natural daughter of defendant, and confirming the order pendente lite, ordering defendant to pay to the said child, through plaintiff, the sum of thirty pesos (P30.00), payable on or before the fifth day of every month sentencing defendant to pay to plaintiff the sum of FOUR THOUSAND FIVE HUNDRED PESOS (P4,500.00) for actual and compensatory damages; the sum of FIVE THOUSAND PESOS (P5,000.00) as moral damages; and the further sum of FIVE HUNDRED PESOS (P500.00) as attorney's fees for plaintiff, with costs against defendant. On appeal taken by petitioner, the Court of Appeals affirmed this decision, except as to the actual and compensatory damages and the moral damages, which were increased to P5,614.25 and P7,000.00, respectively. The main issue before us is whether moral damages are recoverable, under our laws, for breach of promise to marry. The pertinent facts are: Complainant Soledad Cagigas, was born in July 1917. Since 1950, Soledad then a teacher in the Sibonga Provincial High School in Cebu, and petitioner, who was almost ten (10) years younger than she, used to go around together and were regarded as engaged, although he had made no promise of marriage prior thereto. In 1951, she gave up teaching and became a life insurance underwriter in the City of Cebu, where intimacy developed among her and the petitioner, since one evening in 1953, when after coming from the movies, they had sexual intercourse in his cabin on board M/V "Escao," to which he was then attached as apprentice pilot. In February 1954, Soledad advised petitioner that she was in the family way, whereupon he promised to marry her. Their child, Chris Hermosisima, was born on June 17, 1954, in a private maternity and clinic. However, subsequently, or on July 24, 1954, defendant married one Romanita Perez. Hence, the present action, which was commenced on or about October 4, 1954. Referring now to the issue above referred to, it will be noted that the Civil Code of Spain permitted the recovery of damages for breach to marry. Article 43 and 44 of said Code provides: ART. 43. A mutual promise of marriage shall not give rise to an obligation to contract marriage. No court shall entertain any complaint by which the enforcement of such promise is sought. ART. 44. If the promise has been in a public or private instrument by an adult, or by a minor with the concurrence of the person whose consent is necessary for the celebration of the marriage, or if the banns have been published, the one who without just cause refuses to marry shall be obliged to reimburse the other for the expenses which he or she may have incurred by reason of the promised marriage. The action for reimbursement of expenses to which the foregoing article refers must be brought within one year, computed from the day of the refusal to celebrate the marriage.

134

University of the Cordilleras College of Law First Year C S.Y. 2013 - 2014 In as much as these articles were never in force in the Philippines, this Court ruled in De Jesus vs. Syquia (58 Phil., 866), that "the action for breach of promises to marry has no standing in the civil law, apart from the right to recover money or property advanced . . . upon the faith of such promise". The Code Commission charged with the drafting of the Proposed Civil Code of the Philippines deem it best, however, to change the law thereon. We quote from the report of the Code Commission on said Proposed Civil Code: Articles 43 and 44 the Civil Code of 1889 refer to the promise of marriage. But these articles are not enforced in the Philippines. The subject is regulated in the Proposed Civil Code not only as to the aspect treated of in said articles but also in other particulars. It is advisable to furnish legislative solutions to some questions that might arise relative to betrothal. Among the provisions proposed are: That authorizing the adjudication of moral damages, in case of breach of promise of marriage, and that creating liability for causing a marriage engagement to be broken.1awphl.nt Accordingly, the following provisions were inserted in said Proposed Civil Code, under Chapter I, Title III, Book I thereof: Art. 56. A mutual promise to marry may be made expressly or impliedly. Art. 57. An engagement to be married must be agreed directly by the future spouses. Art. 58. A contract for a future marriage cannot, without the consent of the parent or guardian, be entered into by a male between the ages of sixteen and twenty years or by a female between the ages of sixteen and eighteen years. Without such consent of the parents or guardian, the engagement to marry cannot be the basis of a civil action for damages in case of breach of the promise. Art. 59. A promise to marry when made by a female under the age of fourteen years is not civilly actionable, even though approved by the parent or guardian. Art. 60. In cases referred to in the proceeding articles, the criminal and civil responsibility of a male for seduction shall not be affected. Art. 61. No action for specific performance of a mutual promise to marry may be brought. Art. 62. An action for breach of promise to marry may be brought by the aggrieved party even though a minor without the assistance of his parent or guardian. Should the minor refuse to bring suit, the parent or guardian may institute the action. Art. 63. Damages for breach of promise to marry shall include not only material and pecuniary losses but also compensation for mental and moral suffering. Art. 64. Any person, other than a rival, the parents, guardians and grandparents, of the affianced parties, who cause a marriage engagement to be broken shall be liable for damages, both material and moral, to the engaged person who is rejected. Art. 65. In case of breach of promise to marry, the party breaking the engagement shall be obliged to return what he or she has received from the other as gift on account of the promise of the marriage. These article were, however, eliminated in Congress. The reason therefor are set forth in the report of the corresponding Senate Committee, from which we quote: The elimination of this Chapter is proposed. That breach of promise to marry is not actionable has been definitely decide in the case of De Jesus vs. Syquia, 58 Phil., 866. The history of breach of promise suit in the United States and in England has shown that no other action lends itself more readily to abuse by designing women and unscrupulous men. It is this experience which has led to the abolition of the rights of action in the so-called Balm suit in many of the American States. See statutes of: Florida Maryland Nevada 1945 pp. 1342 1344 1945 pp. 1759 1762 1943 p. 75 135

University of the Cordilleras College of Law First Year C S.Y. 2013 - 2014 Maine 1941 pp. 140 141 New Hampshire1941 p. 223 California 1939 p. 1245 Massachusetts 1938 p. 326 Indiana 1936 p. 1009 Michigan 1935 p. 201 New York 1935 Pennsylvania p. 450 The Commission perhaps though that it has followed the more progression trend in legislation when it provided for breach of promise to marry suits. But it is clear that the creation of such causes of action at a time when so many States, in consequence of years of experience are doing away with them, may well prove to be a step in the wrong direction. (Congressional Record, Vol. IV, No. 79, Thursday, May 19, 1949, p. 2352.) The views thus expressed were accepted by both houses of Congress. In the light of the clear and manifest intent of our law making body not to sanction actions for breach of promise to marry, the award of moral damages made by the lower courts is, accordingly, untenable. The Court of Appeals said award: Moreover, it appearing that because of defendant-appellant's seduction power, plaintiff-appellee, overwhelmed by her love for him finally yielded to his sexual desires in spite of her age and self-control, she being a woman after all, we hold that said defendant-appellant is liable for seduction and, therefore, moral damages may be recovered from him under the provision of Article 2219, paragraph 3, of the new Civil Code. Apart from the fact that the general tenor of said Article 2219, particularly the paragraphs preceding and those following the one cited by the Court of Appeals, and the language used in said paragraph strongly indicates that the "seduction" therein contemplated is the crime punished as such in Article as such in Article 337 and 338 of the Revised Penal Code, which admittedly does not exist in the present case, we find ourselves unable to say that petitioner is morally guilty of seduction, not only because he is approximately ten (10) years younger than the complainant who around thirty-six (36) years of age, and as highly enlightened as a former high school teacher and a life insurance agent are supposed to be when she became intimate with petitioner, then a mere apprentice pilot, but, also, because, the court of first instance found that, complainant "surrendered herself" to petitioner because, "overwhelmed by her love" for him, she "wanted to bind" "by having a fruit of their engagement even before they had the benefit of clergy." The court of first instance sentenced petitioner to pay the following: (1) a monthly pension of P30.00 for the support of the child: (2) P4,500, representing the income that complainant had allegedly failed to earn during her pregnancy and shortly after the birth of the child, as actual and compensation damages; (3) P5,000, as moral damages; and (4) P500.00, as attorney's fees. The Court of Appeals added to the second item the sum of P1,114.25 consisting of P144.20, for hospitalization and medical attendance, in connection with the parturiation, and the balance representing expenses incurred to support the child and increased the moral damages to P7,000.00. With the elimination of this award for damages, the decision of the Court of Appeals is hereby affirmed, therefore, in all other respects, without special pronouncement as to cost in this instance. It is so ordered.

136

University of the Cordilleras College of Law First Year C S.Y. 2013 - 2014 Case Digest HERMOSISIMA VS COURT OF APPEALS 103 Phil 629 G.R. No. L-14628 Decided on: September 30, 1960 Ponente: CONCEPCION, J.: FACTS: An appeal by certiorari, on October 4, 1954, Soledad Cagigas, hereinafter referred to as complainant, filed with the said CFI a complaint for the acknowledgment of her child, Chris Hermosisima, as a natural child of said petitioner, as well as for support of said child and moral damages for alleged breach of promise to marry. Petitioner admitted the paternity of the child and expressed willingness to support the latter, but denied having ever promised to marry complainant. Complainant Soledad Cagigas, was born in July 1917, since 1950, Soledad then a teacher and petitioner who was almost ten years younger than her used to go around together and were regarded as engaged, although he made no promise of marriage thereto. In 1951, she gave up teaching and became a life insurance underwriter where intimacy developed between her and petitioner, since one evening in 1953 when after coming from the movies, they had sexual intercourse in his cabin on board MV Escano to which he was then attached as apprentice pilot. In February 1954, Soledad advised petitioner that she was pregnant, whereupon he promised to marry her. However, subsequently, or on July 24, 1954, defendant married one Romanita Perez. ISSUE: Whether or not moral damages are recoverable under our laws for breach of promise to marry. HELD: It appearing that because of the defendant-appellants seductive prowess, plaintiff-appellee overwhelmed by her love for him yielded to his sexual desires in spite of her age and self-control. In the present case, the court is unable to say that petitioner is morally guilty of seduction, not only because he is approximately ten years younger but also because the CFI found that complainant surrendered herself to the petitioner because overwhelmed by her love for him she wanted to bind him by having a fruit of their engagement even before they had the benefit of clergy.

137

University of the Cordilleras College of Law First Year C S.Y. 2013 - 2014 Estopa vs Piansay 109 Phil 640 G.R. No. L-14733 September 30, 1960 Full Case ERLINDA ESTOPA, plaintiff-appellee, vs. LORETA PIANSAY, JR., defendant-appellant. BENGZON, J.: Appeal from the decision of the Negros Occidental court of first instance awarding to plaintiff the sum of P5,000.00 by way of moral damages, P2,000.00 as exemplary damages and P1,000.00 as attorney's fees. As stated by the court below, "this is an action for recovery of moral and exemplary damages and attorney's fees. There is no dispute regarding the facts of this case. The plaintiff Erlinda Estopa, a beautiful girl of twenty-three, residing in Bago, Negros Occidental, with her widowed mother, Felicidad Estopa, stated that she fell in love and submitted herself completely to the defendant Loreta Piansay, Jr., sometime in September, 1957, after a courtship that lasted for a couple of months during which period the defendant consistently promised and succeeded to make her believe in him that he was going to marry her; that sometime in December, 1957, the plaintiff was informed reliably that defendant was backing out from his promise of marriage so she demanded defendant's compliance to his promise in order to vindicate her honor, and plaintiff went to the extent of asking the help of defendant's parents, but all her efforts were in vain. Finally, realizing that her efforts were futile but knowing that her cause was not completely lost, she decided to file her complaint, not to compel defendant to marry her, but to demand from him a compensation for the damages that she sustained." There is no claim for any other kind of damages. In fact, Erlinda Estopa filed no brief here. And her complaint merely alleged "social humiliation, mental anguish, besmirched reputation, wounded feeling and moral shock." We have today decided that in this jurisdiction, under the New Civil Code, the mere breach of a promise to marry is not actionable. (Hermosisima vs. Court of Appeals, Supra, 631); and we have reversed the Cebu court's award for moral damages in breach of promise suit. Consistently with such ruling, Loreta Piansay, Jr. may not be condemned to pay moral damages, in this case. Now, as plaintiff has no right to moral damages, she may not demand exemplary damages. (She lays no claim to temperate or compensatory damages.) While the amount of the exemplary damages need not be proved, the plaintiff must show that he is entitled to moral, temperate or compensatory damages before the court may consider the question of whether or not exemplary damages should be awarded. (Art. 2234, New Civil Code) (Emphasis supplied.) Therefore, as plaintiff is not entitled to any damages at all, there is no reason to require Piansay, Jr. to satisfy attorney's fees. Judgment reversed, defendant absolved from all liability. No costs. Padilla, Bautista Angelo, Reyes, J.B.L., Barrera, Gutierrez David and Paredes, JJ., concur.

138

University of the Cordilleras College of Law First Year C S.Y. 2013 - 2014 Wassmer vs Velez 10 Phil 1440 G.R. No. L-20089 December 26, 1964 Full Case BEATRIZ P. WASSMER, plaintiff-appellee, vs. FRANCISCO X. VELEZ, defendant-appellant. BENGZON, J.P., J.: The facts that culminated in this case started with dreams and hopes, followed by appropriate planning and serious endeavors, but terminated in frustration and, what is worse, complete public humiliation. Francisco X. Velez and Beatriz P. Wassmer, following their mutual promise of love, decided to get married and set September 4, 1954 as the big day. On September 2, 1954 Velez left this note for his brideto-be: Dear Bet Will have to postpone wedding My mother opposes it. Am leaving on the Convair today. Please do not ask too many people about the reason why That would only create a scandal. Paquing But the next day, September 3, he sent her the following telegram: NOTHING CHANGED REST ASSURED RETURNING VERY SOON APOLOGIZE MAMA PAPA LOVE . PAKING Thereafter Velez did not appear nor was he heard from again. Sued by Beatriz for damages, Velez filed no answer and was declared in default. Plaintiff adduced evidence before the clerk of court as commissioner, and on April 29, 1955, judgment was rendered ordering defendant to pay plaintiff P2,000.00 as actual damages; P25,000.00 as moral and exemplary damages; P2,500.00 as attorney's fees; and the costs. On June 21, 1955 defendant filed a "petition for relief from orders, judgment and proceedings and motion for new trial and reconsideration." Plaintiff moved to strike it cut. But the court, on August 2, 1955, ordered the parties and their attorneys to appear before it on August 23, 1955 "to explore at this stage of the proceedings the possibility of arriving at an amicable settlement." It added that should any of them fail to appear "the petition for relief and the opposition thereto will be deemed submitted for resolution." On August 23, 1955 defendant failed to appear before court. Instead, on the following day his counsel filed a motion to defer for two weeks the resolution on defendants petition for relief. The counsel stated that he would confer with defendant in Cagayan de Oro City the latter's residence on the possibility of an amicable element. The court granted two weeks counted from August 25, 1955. Plaintiff manifested on June 15, 1956 that the two weeks given by the court had expired on September 8, 1955 but that defendant and his counsel had failed to appear. Another chance for amicable settlement was given by the court in its order of July 6, 1956 calling the parties and their attorneys to appear on July 13, 1956. This time.however, defendant's counsel informed the court that chances of settling the case amicably were nil. On July 20, 1956 the court issued an order denying defendant's aforesaid petition. Defendant has appealed to this Court. In his petition of June 21, 1955 in the court a quo defendant alleged excusable negligence as ground to set aside the judgment by default. Specifically, it was stated that defendant filed no answer in the belief that an amicable settlement was being negotiated. 139

University of the Cordilleras College of Law First Year C S.Y. 2013 - 2014 A petition for relief from judgment on grounds of fraud, accident, mistake or excusable negligence, must be duly supported by an affidavit of merits stating facts constituting a valid defense. (Sec. 3, Rule 38, Rules of Court.) Defendant's affidavit of merits attached to his petition of June 21, 1955 stated: "That he has a good and valid defense against plaintiff's cause of action, his failure to marry the plaintiff as scheduled having been due to fortuitous event and/or circumstances beyond his control." An affidavit of merits like this stating mere conclusions or opinions instead of facts is not valid. (Cortes vs. Co Bun Kim, L-3926, Oct. 10, 1951; Vaswani vs. P. Tarrachand Bros., L-15800, December 29, 1960.) Defendant, however, would contend that the affidavit of merits was in fact unnecessary, or a mere surplusage, because the judgment sought to be set aside was null and void, it having been based on evidence adduced before the clerk of court. In Province of Pangasinan vs. Palisoc, L-16519, October 30, 1962, this Court pointed out that the procedure of designating the clerk of court as commissioner to receive evidence is sanctioned by Rule 34 (now Rule 33) of the Rules of Court. Now as to defendant's consent to said procedure, the same did not have to be obtained for he was declared in default and thus had no standing in court (Velez vs. Ramas, 40 Phil. 787; Alano vs. Court of First Instance, L-14557, October 30, 1959). In support of his "motion for new trial and reconsideration," defendant asserts that the judgment is contrary to law. The reason given is that "there is no provision of the Civil Code authorizing" an action for breach of promise to marry. Indeed, our ruling in Hermosisima vs. Court of Appeals (L-14628, Sept. 30, 1960), as reiterated in Estopa vs. Biansay (L-14733, Sept. 30, 1960), is that "mere breach of a promise to marry" is not an actionable wrong. We pointed out that Congress deliberately eliminated from the draft of the new Civil Code the provisions that would have it so. It must not be overlooked, however, that the extent to which acts not contrary to law may be perpetrated with impunity, is not limitless for Article 21 of said Code provides that "any person who wilfully causes loss or injury to another in a manner that is contrary to morals, good customs or public policy shall compensate the latter for the damage." The record reveals that on August 23, 1954 plaintiff and defendant applied for a license to contract marriage, which was subsequently issued (Exhs. A, A-1). Their wedding was set for September 4, 1954. Invitations were printed and distributed to relatives, friends and acquaintances (Tsn., 5; Exh. C). The bride-to-be's trousseau, party drsrses and other apparel for the important occasion were purchased (Tsn., 7-8). Dresses for the maid of honor and the flower girl were prepared. A matrimonial bed, with accessories, was bought. Bridal showers were given and gifts received (Tsn., 6; Exh. E). And then, with but two days before the wedding, defendant, who was then 28 years old,: simply left a note for plaintiff stating: "Will have to postpone wedding My mother opposes it ... " He enplaned to his home city in Mindanao, and the next day, the day before the wedding, he wired plaintiff: "Nothing changed rest assured returning soon." But he never returned and was never heard from again. Surely this is not a case of mere breach of promise to marry. As stated, mere breach of promise to marry is not an actionable wrong. But to formally set a wedding and go through all the above-described preparation and publicity, only to walk out of it when the matrimony is about to be solemnized, is quite different. This is palpably and unjustifiably contrary to good customs for which defendant must be held answerable in damages in accordance with Article 21 aforesaid. Defendant urges in his afore-stated petition that the damages awarded were excessive. No question is raised as to the award of actual damages. What defendant would really assert hereunder is that the award of moral and exemplary damages, in the amount of P25,000.00, should be totally eliminated. Per express provision of Article 2219 (10) of the New Civil Code, moral damages are recoverable in the cases mentioned in Article 21 of said Code. As to exemplary damages, defendant contends that the same could not be adjudged against him because under Article 2232 of the New Civil Code the condition precedent is that "the defendant acted in a wanton, fraudulent, reckless, oppressive, or malevolent manner." The argument is devoid of merit as under the above-narrated circumstances of this case defendant clearly acted in a "wanton ... , reckless [and] oppressive manner." This Court's opinion, however, is that considering the particular circumstances of this case, P15,000.00 as moral and exemplary damages is deemed to be a reasonable award. PREMISES CONSIDERED, with the above-indicated modification, the lower court's judgment is hereby affirmed, with costs. 140

University of the Cordilleras College of Law First Year C S.Y. 2013 - 2014 Case Digest WASSMER VS VELEZ 12 SCRA 648 G.R. No. L-20089 Decided on: December 26, 1964 Ponente: BENGZON, J.P., J.: FACTS: Franciso Velez and Beatriz Wassmer, following their mutual promise of love, decided to get married and set September 4, 1954 as the big day. On September 2, 1954 Velez left a note to her that they have to postpone their wedding because his mother opposed it. And on the next day he sent her the following telegram Nothing changed rest assured returning very soon apologize mama papa love Paking. Thereafter Velez did not appear nor was he heard from again, sued by Beatrice for damages, Velez filed no answer and was declared in default. The record reveals that on August 23, 1954, plaintiff and defendant applied for a license to contract marriage, which was subsequently issued. Invitations were printed and distributed to relatives, friends and acquaintances. The bride-to-bes trousseau, party dresses and other apparel for the important occasion were purchased. Dresses for the maid of honor and the flower girl were prepared, but two days before the wedding he never returned and was never heard from again. ISSUE: Whether or not in the case at bar, is a case of mere breach of promise to marry. HELD: Surely this is not a case of mere breach of promise to marry. As stated, mere breach of promise to marry is not an actionable wrong. But to formally set a wedding and go through all the abovedescribed preparation and publicity, only to walk out of it when the matrimony is about to be solemnized, is quite different. This is palpably and unjustifiably contrary to good customs for which defendant must be held answerable in damages in accordance with Article 21 aforesaid. The lower courts judgment is hereby affirmed.

141

University of the Cordilleras College of Law First Year C S.Y. 2013 - 2014 Constantino vs Mendez 209 SCRA 18 G.R. No. 57227 May 14, 1992 Full Case AMELITA CONSTANTINO and MICHAEL CONSTANTINO, the latter represented herein by the former, his mother and natural guardian, petitioners, vs. IVAN MENDEZ and the HONORABLE COURT OF APPEALS, respondents. BIDIN, J.: This is a petition for review on certiorari questioning the decision 1 dated April 30, 1981 of the Court of Appeals in CA-G.R. No. 61552-R which dismissed petitioner's complaint and set aside the resolution 2 dated October 21, 1976 of the then Court of First Instance of Davao, 16th Judicial District, amending the dispositive portion of its decision dated June 21, 1976 and ordering private respondent Ivan Mendez: (1) to acknowledge the minor Michael Constantino as his illegitimate child; (2) to give a monthly support of P300.00 to the minor child; (3) to pay complainant Amelita Constantino the sum of P8,200.00 as actual and moral damages; and (4) to pay attorney's fees in the sum of P5,000 plus costs. It appears on record that on June 5, 1975, petitioner Amelita Constantino filed an action for acknowledgment, support and damages against private respondent Ivan Mendez. The case was filed with the then CFI of Davao, 10th Judicial District and docketed as Civil Case No. 8881. In her complaint, Amelita Constantino alleges, among others, that sometime in the month of August, 1974, she met Ivan Mendez at Tony's Restaurant located at Sta. Cruz, Manila, where she worked as a waitress; that the day following their first meeting, Ivan invited Amelita to dine with him at Hotel Enrico where he was billeted; that while dining, Ivan professed his love and courted Amelita; that Amelita asked for time to think about Ivan's proposal; that at about 11:00 o'clock in the evening, Amelita asked Ivan to bring her home to which the latter agreed, that on the pretext of getting something, Ivan brought Amelita inside his hotel room and through a promise of marriage succeeded in having sexual intercourse with the latter; that after the sexual contact, Ivan confessed to Amelita that he is a married man; that they repeated their sexual contact in the months of September and November, 1974, whenever Ivan is in Manila, as a result of which Amelita got pregnant; that her pleas for help and support fell on deaf ears; that Amelita had no sexual relations with any other man except Ivan who is the father of the child yet to be born at the time of the filing of the complaint; that because of her pregnancy, Amelita was forced to leave her work as a waitress; that Ivan is a prosperous businessman of Davao City with a monthly income of P5,000 to P8,000. As relief, Amelita prayed for the recognition of the unborn child, the payment of actual, moral and exemplary damages, attorney's fees plus costs. In his answer dated August 5, 1975, Ivan admitted that he met Amelita at Tony's Cocktail Lounge but denied having sexual knowledge or illicit relations with her. He prayed for the dismissal of the complaint for lack of cause of action. By way of counterclaim, he further prayed for the payment of exemplary damages and litigation expense including attorney's fees for the filing of the malicious complaint. On September 1, 1975, Amelita Constantino filed a motion for leave to amend the complaint impleading as co-plaintiff her son Michael Constantino who was born on August 3, 1975. In its order dated September 4, 1975, the trial court admitted the amended complaint. On September 11, 1975, Ivan Mendez filed his answer to the amended complaint reiterating his previous answer denying that Michael Constantino is his illegitimate son. After hearing, the trial court rendered a decision dated June 21, 1976, the dispositive portion of which reads, viz: WHEREFORE, in view of the foregoing, judgment is hereby rendered in favor of plaintiff Amelita Constantino and against defendant Ivan Mendez, ordering the latter to pay Amelita Constantino the sum of P8,000.00 by way of actual and moral damages; and, the sum of P3,000.00, as and by way of attorney's fees. The defendant shall pay the costs of this suit. SO ORDERED.

142

University of the Cordilleras College of Law First Year C S.Y. 2013 - 2014 From the above decision, both parties filed their separate motion for reconsideration. Ivan Mendez anchored his motion on the ground that the award of damages was not supported by evidence. Amelita Constantino, on the other hand, sought the recognition and support of her son Michael Constantino as the illegitimate son of Ivan Mendez. In its resolution dated October 21, 1976, the trial court granted Amelita Constantino's motion for reconsideration, and amended the dispositive portion of its decision dated June 21, 1976 to read as follows, viz: WHEREFORE, in view of the foregoing, judgment is hereby rendered in favor of plaintiff Amelita Constantino and plaintiff-minor Michael Constantino, and against defendant Ivan Mendez ordering the latter to pay Amelita Constantino the sum of P8,000.00 by way of actual and moral damages and the sum of P200.00 as and by way of payment of the hospital and medical bills incurred during the delivery of plaintiff-minor Michael Constantino; to recognize as his own illegitimate child the plaintiff-minor Michael Constantino who shall be entitled to all the rights, privileges and benefits appertaining to a child of such status; to give a permanent monthly support in favor of plaintiff Michael Constantino the amount of P300.00; and the sum of P5,000.00 as and by way of attorney's fees. The defendant shall pay the costs of this suit. Let this Order form part of the decision dated June 21, 1976. SO ORDERED. On appeal to the Court of Appeals, the above amended decision was set aside and the complaint was dismissed. Hence, this petition for review. Basically, the issue to be resolved in the case at bar is whether or not the Court of Appeals committed a reversible error in setting aside the decision of the trial court and in dismissing the complaint. Petitioners contend that the Court of Appeals erred in reversing the factual findings of the trial and in not affirming the decision of the trial court. They also pointed out that the appellate court committed a misapprehension of facts when it concluded that Ivan did not have sexual access with Amelita during the first or second week of November, 1976 (should be 1974), the time of the conception of the child. It must be stressed at the outset that factual findings of the trial court have only a persuasive and not a conclusive effect on the Court of Appeals. In the exercise of its appellate jurisdiction, it is the duty of the Court of Appeals to review the factual findings of the trial court and rectify the errors it committed as may have been properly assigned and as could be established by a re-examination of the evidence on record. It is the factual findings of the Court of Appeals, not those of the trial court, that as a rule are considered final and conclusive even on this Court (Hermo v. Hon. Court of Appeals, et al., 155 SCRA 24 [1987]). This being a petition for certiorari under Rule 45 of the Rules of Court, this Court will review only errors of law committed by the Court of Appeals. It is not the function of this Court to re-examine all over again the oral and documentary evidence submitted by the parties unless the findings of facts of the Court of Appeals is not supported by the evidence on record or the judgment is based on misapprehension of facts (Remalante v. Tibe, et al., 158 SCRA 138 [1988]; Hernandez v. Court of Appeals, et al., 149 SCRA 97 [1987]). It is the conclusion of the Court of Appeals, based on the evidence on record, that Amelita Constantino has not proved by clear and convincing evidence her claim that Ivan Mendez is the father of her son Michael Constantino. Such conclusion based on the evaluation of the evidence on record is controlling on this Court as the same is supported by the evidence on record. Even the trial court initially entertained such posture. It ordered the recognition of Michael as the illegitimate son of Ivan only when acting on the motions for reconsideration, it reconsidered, on October 21, 1976, its earlier decision dated June 21, 1976. Amelita's testimony on cross-examination that she had sexual contact with Ivan in Manila in the first or second week of November, 1974 (TSN, December 8, 1975, p. 108) is inconsistent with her response that she could not remember the date of their last sexual intercourse in November, 1974 (Ibid, p. 106). Sexual contact of Ivan and Amelita in the first or second week of November, 1974 is the crucial point that was not even established on direct examination as she merely testified that she had sexual intercourse with Ivan in the months of September, October and November, 1974. Michael Constantino is a full-term baby born on August 3, 1975 (Exhibit 6) so that as correctly pointed out by private respondent's counsel, citing medical science (Williams Obstetrics, Tenth Ed., p. 198) to the 143

University of the Cordilleras College of Law First Year C S.Y. 2013 - 2014 effect that "the mean duration of actual pregnancy, counting from the day of conception must be close to 267 days", the conception of the child (Michael) must have taken place about 267 days before August 3, 1975 or sometime in the second week of November, 1974. While Amelita testified that she had sexual contact with Ivan in November, 1974, nevertheless said testimony is contradicted by her own evidence (Exh. F), the letter dated February 11, 1975, addressed to Ivan Mendez requesting for a conference, prepared by her own counsel Atty. Roberto Sarenas to whom she must have confided the attendant circumstances of her pregnancy while still fresh in her memory, informing Ivan that Amelita is four (4) months pregnant so that applying the period of the duration of actual pregnancy, the child was conceived on or about October 11, 1974. Petitioner's assertion that Ivan is her first and only boyfriend (TSN, December 8, 1975, p. 65) is belied by Exhibit 2, her own letter addressed to Mrs. Mendez where she revealed the reason for her attachment to Ivan who possessed certain traits not possessed by her boyfriend. She also confided that she had a quarrel with her boyfriend because of gossips so she left her work. An order for recognition and support may create an unwholesome atmosphere or may be an irritant in the family or lives of the parties so that it must be issued only if paternity or filiation is established by clear and convincing evidence. The burden of proof is on Amelita to establish her affirmative allegations that Ivan is the father of her son. Consequently, in the absence of clear and convincing evidence establishing paternity or filiation, the complaint must be dismissed. As regards Amelita's claim for damages which is based on Articles 19 3 & 21 4 of the Civil Code on the theory that through Ivan's promise of marriage, she surrendered her virginity, we cannot but agree with the Court of Appeals that more sexual intercourse is not by itself a basis for recovery. Damages could only be awarded if sexual intercourse is not a product of voluntariness and mutual desire. At the time she met Ivan at Tony's Restaurant, Amelita was already 28 years old and she admitted that she was attracted to Ivan (TSN, December 3, 1975, p. 83). Her attraction to Ivan is the reason why she surrendered her womanhood. Had she been induced or deceived because of a promise of marriage, she could have immediately severed her relation with Ivan when she was informed after their first sexual contact sometime in August, 1974, that he was a married man. Her declaration that in the months of September, October and November, 1974, they repeated their sexual intercourse only indicates that passion and not the alleged promise of marriage was the moving force that made her submit herself to Ivan. WHEREFORE, the instant petition is Dismissed for lack of merit. SO ORDERED.

144

University of the Cordilleras College of Law First Year C S.Y. 2013 - 2014 Case Digest Constantino vs Mendez 209 SCRA 18 G.R. No. 57227 Decided on: May 14, 1992 Ponente: BIDIN, J.: FACTS: Michael Constantino, an illegitimate child, as represented by Amelita, her mother, sought monthly support from Ivan Mendez including Amelias complaint on damages. The latter and Amelita met in a restaurant in Manila where she was working as a waitress. Ivan invited him at his hotel and through promise of marriage succeeded in having sexual intercourse with Amelita, afterwards, he admitted being a married man. In spite of that, they repeated their sexual contact. Subsequently, she became pregnant and had to resign from work. Trial court ruled in favor of Amelita providing actual and moral damages, acknowledging Michael as Ivans illegitimate child and giving monthly support to the latter which was set aside by CA. ISSUE: Whether or Not the alleged illegitimate child is entitled for the monthly support. HELD: Amelita Constantino has not proved by clear and convincing evidence her claim that Ivan Mendez is the father of her son Michael Constantino. Sexual contact of Ivan and Amelita in the first or second week of November, 1974 is the crucial point that was not even established on direct examination as she merely testified that she had sexual intercourse with Ivan in the months of September, October and November, 1974. More so, Amelita admitted that she was attracted to Ivan and their repeated sexual intercourse indicated that passion and not alleged promise to marriage was the moving force to submit herself with Ivan. The petition was dismissed for lack of merit.

145

University of the Cordilleras College of Law First Year C S.Y. 2013 - 2014 Gashem-Shookat vs Court of Appeals 219 SCRA 115 G.R. No. 97336 February 19, 1993 Full Case GASHEM SHOOKAT BAKSH, petitioner, vs. HON. COURT OF APPEALS and MARILOU T. GONZALES, respondents. DAVIDE, JR., J.: This is an appeal by certiorari under Rule 45 of the Rules of Court seeking to review and set aside the Decision 1 of the respondent Court of Appeals in CA-G.R. CV No. 24256 which affirmed in toto the 16 October 1939 Decision of Branch 38 (Lingayen) of the Regional Trial Court (RTC) of Pangasinan in Civil Case No. 16503. Presented is the issue of whether or not damages may be recovered for a breach of promise to marry on the basis of Article 21 of the Civil Code of the Philippines. The antecedents of this case are not complicated: On 27 October 1987, private respondent, without the assistance of counsel, filed with the aforesaid trial court a complaint 2 for damages against the petitioner for the alleged violation of their agreement to get married. She alleges in said complaint that: she is twenty-two (22) years old, single, Filipino and a pretty lass of good moral character and reputation duly respected in her community; petitioner, on the other hand, is an Iranian citizen residing at the Lozano Apartments, Guilig, Dagupan City, and is an exchange student taking a medical course at the Lyceum Northwestern Colleges in Dagupan City; before 20 August 1987, the latter courted and proposed to marry her; she accepted his love on the condition that they would get married; they therefore agreed to get married after the end of the school semester, which was in October of that year; petitioner then visited the private respondent's parents in Baaga, Bugallon, Pangasinan to secure their approval to the marriage; sometime in 20 August 1987, the petitioner forced her to live with him in the Lozano Apartments; she was a virgin before she began living with him; a week before the filing of the complaint, petitioner's attitude towards her started to change; he maltreated and threatened to kill her; as a result of such maltreatment, she sustained injuries; during a confrontation with a representative of the barangay captain of Guilig a day before the filing of the complaint, petitioner repudiated their marriage agreement and asked her not to live with him anymore and; the petitioner is already married to someone living in Bacolod City. Private respondent then prayed for judgment ordering the petitioner to pay her damages in the amount of not less than P45,000.00, reimbursement for actual expenses amounting to P600.00, attorney's fees and costs, and granting her such other relief and remedies as may be just and equitable. The complaint was docketed as Civil Case No. 16503. In his Answer with Counterclaim, petitioner admitted only the personal circumstances of the parties as averred in the complaint and denied the rest of the allegations either for lack of knowledge or information sufficient to form a belief as to the truth thereof or because the true facts are those alleged as his Special and Affirmative Defenses. He thus claimed that he never proposed marriage to or agreed to be married with the private respondent; he neither sought the consent and approval of her parents nor forced her to live in his apartment; he did not maltreat her, but only told her to stop coming to his place because he discovered that she had deceived him by stealing his money and passport; and finally, no confrontation took place with a representative of the barangay captain. Insisting, in his Counterclaim, that the complaint is baseless and unfounded and that as a result thereof, he was unnecessarily dragged into court and compelled to incur expenses, and has suffered mental anxiety and a besmirched reputation, he prayed for an award of P5,000.00 for miscellaneous expenses and P25,000.00 as moral damages. After conducting a pre-trial on 25 January 1988, the trial court issued a Pre-Trial Order 4 embodying the stipulated facts which the parties had agreed upon, to wit: 1. That the plaintiff is single and resident (sic) of Baaga, Bugallon, Pangasinan, while the defendant is single, Iranian citizen and resident (sic) of Lozano Apartment, Guilig, Dagupan City since September 1, 1987 up to the present; 2. That the defendant is presently studying at Lyceum Northwestern, Dagupan City, College of Medicine, second year medicine proper; 3. That the plaintiff is (sic) an employee at Mabuhay Luncheonette , Fernandez Avenue, Dagupan City since July, 1986 up to the present and a (sic) high school graduate; 146

University of the Cordilleras College of Law First Year C S.Y. 2013 - 2014 4. That the parties happened to know each other when the manager of the Mabuhay Luncheonette, Johhny Rabino introduced the defendant to the plaintiff on August 3, 1986. After trial on the merits, the lower court, applying Article 21 of the Civil Code, rendered on 16 October 1989 a decision favoring the private respondent. The petitioner was thus ordered to pay the latter damages and attorney's fees; the dispositive portion of the decision reads: IN THE LIGHT of the foregoing consideration, judgment is hereby rendered in favor of the plaintiff and against the defendant. 1. Condemning (sic) the defendant to pay the plaintiff the sum of twenty thousand (P20,000.00) pesos as moral damages. 2. Condemning further the defendant to play the plaintiff the sum of three thousand (P3,000.00) pesos as atty's fees and two thousand (P2,000.00) pesos at (sic) litigation expenses and to pay the costs. 3. All other claims are denied. The decision is anchored on the trial court's findings and conclusions that (a) petitioner and private respondent were lovers, (b) private respondent is not a woman of loose morals or questionable virtue who readily submits to sexual advances, (c) petitioner, through machinations, deceit and false pretenses, promised to marry private respondent, d) because of his persuasive promise to marry her, she allowed herself to be deflowered by him, (e) by reason of that deceitful promise, private respondent and her parents in accordance with Filipino customs and traditions made some preparations for the wedding that was to be held at the end of October 1987 by looking for pigs and chickens, inviting friends and relatives and contracting sponsors, (f) petitioner did not fulfill his promise to marry her and (g) such acts of the petitioner, who is a foreigner and who has abused Philippine hospitality, have offended our sense of morality, good customs, culture and traditions. The trial court gave full credit to the private respondent's testimony because, inter alia, she would not have had the temerity and courage to come to court and expose her honor and reputation to public scrutiny and ridicule if her claim was false. The above findings and conclusions were culled from the detailed summary of the evidence for the private respondent in the foregoing decision, digested by the respondent Court as follows: According to plaintiff, who claimed that she was a virgin at the time and that she never had a boyfriend before, defendant started courting her just a few days after they first met. He later proposed marriage to her several times and she accepted his love as well as his proposal of marriage on August 20, 1987, on which same day he went with her to her hometown of Baaga, Bugallon, Pangasinan, as he wanted to meet her parents and inform them of their relationship and their intention to get married. The photographs Exhs. "A" to "E" (and their submarkings) of defendant with members of plaintiff's family or with plaintiff, were taken that day. Also on that occasion, defendant told plaintiffs parents and brothers and sisters that he intended to marry her during the semestral break in October, 1987, and because plaintiff's parents thought he was good and trusted him, they agreed to his proposal for him to marry their daughter, and they likewise allowed him to stay in their house and sleep with plaintiff during the few days that they were in Bugallon. When plaintiff and defendant later returned to Dagupan City, they continued to live together in defendant's apartment. However, in the early days of October, 1987, defendant would tie plaintiff's hands and feet while he went to school, and he even gave her medicine at 4 o'clock in the morning that made her sleep the whole day and night until the following day. As a result of this live-in relationship, plaintiff became pregnant, but defendant gave her some medicine to abort the fetus. Still plaintiff continued to live with defendant and kept reminding him of his promise to marry her until he told her that he could not do so because he was already married to a girl in Bacolod City. That was the time plaintiff left defendant, went home to her parents, and thereafter consulted a lawyer who accompanied her to the barangay captain in Dagupan City. Plaintiff, her lawyer, her godmother, and a barangay tanod sent by the barangay captain went to talk to defendant to still convince him to marry plaintiff, but defendant insisted that he could not do so because he was already married to a girl in Bacolod City, although the truth, as stipulated by the parties at the pre-trial, is that defendant is still single. Plaintiff's father, a tricycle driver, also claimed that after defendant had informed them of his desire to marry Marilou, he already looked for sponsors for the wedding, started preparing for the reception by looking for pigs and chickens, and even already invited many relatives and friends to the forthcoming wedding. 147

University of the Cordilleras College of Law First Year C S.Y. 2013 - 2014 Petitioner appealed the trial court's decision to the respondent Court of Appeals which docketed the case as CA-G.R. CV No. 24256. In his Brief, he contended that the trial court erred (a) in not dismissing the case for lack of factual and legal basis and (b) in ordering him to pay moral damages, attorney's fees, litigation expenses and costs. On 18 February 1991, respondent Court promulgated the challenged decision affirming in toto the trial court's ruling of 16 October 1989. In sustaining the trial court's findings of fact, respondent Court made the following analysis: First of all, plaintiff, then only 21 years old when she met defendant who was already 29 years old at the time, does not appear to be a girl of loose morals. It is uncontradicted that she was a virgin prior to her unfortunate experience with defendant and never had boyfriend. She is, as described by the lower court, a barrio lass "not used and accustomed to trend of modern urban life", and certainly would (sic) not have allowed "herself to be deflowered by the defendant if there was no persuasive promise made by the defendant to marry her." In fact, we agree with the lower court that plaintiff and defendant must have been sweethearts or so the plaintiff must have thought because of the deception of defendant, for otherwise, she would not have allowed herself to be photographed with defendant in public in so (sic) loving and tender poses as those depicted in the pictures Exhs. "D" and "E". We cannot believe, therefore, defendant's pretense that plaintiff was a nobody to him except a waitress at the restaurant where he usually ate. Defendant in fact admitted that he went to plaintiff's hometown of Baaga, Bugallon, Pangasinan, at least thrice; at (sic) the town fiesta on February 27, 1987 (p. 54, tsn May 18, 1988), at (sic) a beach party together with the manager and employees of the Mabuhay Luncheonette on March 3, 1987 (p. 50, tsn id.), and on April 1, 1987 when he allegedly talked to plaintiff's mother who told him to marry her daughter (pp. 55-56, tsn id.). Would defendant have left Dagupan City where he was involved in the serious study of medicine to go to plaintiff's hometown in Baaga, Bugallon, unless there was (sic) some kind of special relationship between them? And this special relationship must indeed have led to defendant's insincere proposal of marriage to plaintiff, communicated not only to her but also to her parents, and (sic) MaritesRabino, the owner of the restaurant where plaintiff was working and where defendant first proposed marriage to her, also knew of this love affair and defendant's proposal of marriage to plaintiff, which she declared was the reason why plaintiff resigned from her job at the restaurant after she had accepted defendant's proposal (pp. 6-7, tsn March 7, 1988). Upon the other hand, appellant does not appear to be a man of good moral character and must think so low and have so little respect and regard for Filipino women that he openly admitted that when he studied in Bacolod City for several years where he finished his B.S. Biology before he came to Dagupan City to study medicine, he had a common-law wife in Bacolod City. In other words, he also lived with another woman in Bacolod City but did not marry that woman, just like what he did to plaintiff. It is not surprising, then, that he felt so little compunction or remorse in pretending to love and promising to marry plaintiff, a young, innocent, trustful country girl, in order to satisfy his lust on her. and then concluded: In sum, we are strongly convinced and so hold that it was defendant-appellant's fraudulent and deceptive protestations of love for and promise to marry plaintiff that made her surrender her virtue and womanhood to him and to live with him on the honest and sincere belief that he would keep said promise, and it was likewise these (sic) fraud and deception on appellant's part that made plaintiff's parents agree to their daughter's living-in with him preparatory to their supposed marriage. And as these acts of appellant are palpably and undoubtedly against morals, good customs, and public policy, and are even gravely and deeply derogatory and insulting to our women, coming as they do from a foreigner who has been enjoying the hospitality of our people and taking advantage of the opportunity to study in one of our institutions of learning, defendant-appellant should indeed be made, under Art. 21 of the Civil Code of the Philippines, to compensate for the moral damages and injury that he had caused plaintiff, as the lower court ordered him to do in its decision in this case. Unfazed by his second defeat, petitioner filed the instant petition on 26 March 1991; he raises therein the single issue of whether or not Article 21 of the Civil Code applies to the case at bar. It is petitioner's thesis that said Article 21 is not applicable because he had not committed any moral wrong or injury or violated any good custom or public policy; he has not professed love or proposed marriage to the private respondent; and he has never maltreated her. He criticizes the trial court for liberally invoking Filipino customs, traditions and culture, and ignoring the fact that since he is a 148

University of the Cordilleras College of Law First Year C S.Y. 2013 - 2014 foreigner, he is not conversant with such Filipino customs, traditions and culture. As an Iranian Moslem, he is not familiar with Catholic and Christian ways. He stresses that even if he had made a promise to marry, the subsequent failure to fulfill the same is excusable or tolerable because of his Moslem upbringing; he then alludes to the Muslim Code which purportedly allows a Muslim to take four (4) wives and concludes that on the basis thereof, the trial court erred in ruling that he does not posses good moral character. Moreover, his controversial "common law life" is now his legal wife as their marriage had been solemnized in civil ceremonies in the Iranian Embassy. As to his unlawful cohabitation with the private respondent, petitioner claims that even if responsibility could be pinned on him for the live-in relationship, the private respondent should also be faulted for consenting to an illicit arrangement. Finally, petitioner asseverates that even if it was to be assumed arguendo that he had professed his love to the private respondent and had also promised to marry her, such acts would not be actionable in view of the special circumstances of the case. The mere breach of promise is not actionable. On 26 August 1991, after the private respondent had filed her Comment to the petition and the petitioner had filed his Reply thereto, this Court gave due course to the petition and required the parties to submit their respective Memoranda, which they subsequently complied with. As may be gleaned from the foregoing summation of the petitioner's arguments in support of his thesis, it is clear that questions of fact, which boil down to the issue of the credibility of witnesses, are also raised. It is the rule in this jurisdiction that appellate courts will not disturb the trial court's findings as to the credibility of witnesses, the latter court having heard the witnesses and having had the opportunity to observe closely their deportment and manner of testifying, unless the trial court had plainly overlooked facts of substance or value which, if considered, might affect the result of the case. Petitioner has miserably failed to convince Us that both the appellate and trial courts had overlooked any fact of substance or values which could alter the result of the case. Equally settled is the rule that only questions of law may be raised in a petition for review on certiorari under Rule 45 of the Rules of Court. It is not the function of this Court to analyze or weigh all over again the evidence introduced by the parties before the lower court. There are, however, recognized exceptions to this rule. Thus, in Medina vs. Asistio, Jr., 16 this Court took the time, again, to enumerate these exceptions: xxx xxx xxx

(1) When the conclusion is a finding grounded entirely on speculation, surmises or conjectures (Joaquin v. Navarro, 93 Phil. 257 [1953]); (2) When the inference made is manifestly mistaken, absurb or impossible (Luna v. Linatok, 74 Phil. 15 [1942]); (3) Where there is a grave abuse of discretion (Buyco v. People, 95 Phil. 453 [1955]); (4) When the judgment is based on a misapprehension of facts (Cruz v. Sosing, L-4875, Nov. 27, 1953); (5) When the findings of fact are conflicting (Casica v. Villaseca, L9590 Ap. 30, 1957; unrep.) (6) When the Court of Appeals, in making its findings, went beyond the issues of the case and the same is contrary to the admissions of both appellate and appellee (Evangelista v. Alto Surety and Insurance Co., 103 Phil. 401 [1958]); (7) The findings of the Court of Appeals are contrary to those of the trial court (Garcia v. Court of Appeals, 33 SCRA 622 [1970]; Sacay v. Sandiganbayan, 142 SCRA 593 [1986]); (8) When the findings of fact are conclusions without citation of specific evidence on which they are based (Ibid.,); (9) When the facts set forth in the petition as well as in the petitioners main and reply briefs are not disputed by the respondents (Ibid.,); and (10) The finding of fact of the Court of Appeals is premised on the supposed absence of evidence and is contradicted by the evidence on record (Salazar v. Gutierrez, 33 SCRA 242 [1970]). Petitioner has not endeavored to joint out to Us the existence of any of the above quoted exceptions in this case. Consequently, the factual findings of the trial and appellate courts must be respected. And now to the legal issue. The existing rule is that a breach of promise to marry per se is not an actionable wrong. 17 Congress deliberately eliminated from the draft of the New Civil Code the provisions that would have made it so. The reason therefor is set forth in the report of the Senate Committees on the Proposed Civil Code, from which We quote: The elimination of this chapter is proposed. That breach of promise to marry is not actionable has been definitely decided in the case of De Jesus vs. Syquia. 18 The history of breach of promise suits in the 149

University of the Cordilleras College of Law First Year C S.Y. 2013 - 2014 United States and in England has shown that no other action lends itself more readily to abuse by designing women and unscrupulous men. It is this experience which has led to the abolition of rights of action in the so-called Heart Balm suits in many of the American states. . . . This notwithstanding, the said Code contains a provision, Article 21, which is designed to expand the concept of torts or quasi-delict in this jurisdiction by granting adequate legal remedy for the untold number of moral wrongs which is impossible for human foresight to specifically enumerate and punish in the statute books. As the Code Commission itself stated in its Report: But the Code Commission had gone farther than the sphere of wrongs defined or determined by positive law. Fully sensible that there are countless gaps in the statutes, which leave so many victims of moral wrongs helpless, even though they have actually suffered material and moral injury, the Commission has deemed it necessary, in the interest of justice, to incorporate in the proposed Civil Code the following rule: Art. 23. Any person who wilfully causes loss or injury to another in a manner that is contrary to morals, good customs or public policy shall compensate the latter for the damage. An example will illustrate the purview of the foregoing norm: "A" seduces the nineteen-year old daughter of "X". A promise of marriage either has not been made, or can not be proved. The girl becomes pregnant. Under the present laws, there is no crime, as the girl is above nineteen years of age. Neither can any civil action for breach of promise of marriage be filed. Therefore, though the grievous moral wrong has been committed, and though the girl and family have suffered incalculable moral damage, she and her parents cannot bring action for damages. But under the proposed article, she and her parents would have such a right of action. Thus at one stroke, the legislator, if the forgoing rule is approved, would vouchsafe adequate legal remedy for that untold number of moral wrongs which it is impossible for human foresight to provide for specifically in the statutes. Article 2176 of the Civil Code, which defines a quasi-delict thus: Whoever by act or omission causes damage to another, there being fault or negligence, is obliged to pay for the damage done. Such fault or negligence, if there is no pre-existing contractual relation between the parties, is called a quasi-delict and is governed by the provisions of this Chapter. is limited to negligent acts or omissions and excludes the notion of willfulness or intent. Quasi-delict, known in Spanish legal treatises as culpa aquiliana, is a civil law concept while torts is an AngloAmerican or common law concept. Torts is much broader than culpa aquiliana because it includes not only negligence, but international criminal acts as well such as assault and battery, false imprisonment and deceit. In the general scheme of the Philippine legal system envisioned by the Commission responsible for drafting the New Civil Code, intentional and malicious acts, with certain exceptions, are to be governed by the Revised Penal Code while negligent acts or omissions are to be covered by Article 2176 of the Civil Code. In between these opposite spectrums are injurious acts which, in the absence of Article 21, would have been beyond redress. Thus, Article 21 fills that vacuum. It is even postulated that together with Articles 19 and 20 of the Civil Code, Article 21 has greatly broadened the scope of the law on civil wrongs; it has become much more supple and adaptable than the Anglo-American law on torts. In the light of the above laudable purpose of Article 21, We are of the opinion, and so hold, that where a man's promise to marry is in fact the proximate cause of the acceptance of his love by a woman and his representation to fulfill that promise thereafter becomes the proximate cause of the giving of herself unto him in a sexual congress, proof that he had, in reality, no intention of marrying her and that the promise was only a subtle scheme or deceptive device to entice or inveigle her to accept him and to obtain her consent to the sexual act, could justify the award of damages pursuant to Article 21 not because of such promise to marry but because of the fraud and deceit behind it and the willful injury to her honor and reputation which followed thereafter. It is essential, however, that such injury should have been committed in a manner contrary to morals, good customs or public policy. In the instant case, respondent Court found that it was the petitioner's "fraudulent and deceptive protestations of love for and promise to marry plaintiff that made her surrender her virtue and 150

University of the Cordilleras College of Law First Year C S.Y. 2013 - 2014 womanhood to him and to live with him on the honest and sincere belief that he would keep said promise, and it was likewise these fraud and deception on appellant's part that made plaintiff's parents agree to their daughter's living-in with him preparatory to their supposed marriage." In short, the private respondent surrendered her virginity, the cherished possession of every single Filipina, not because of lust but because of moral seduction the kind illustrated by the Code Commission in its example earlier adverted to. The petitioner could not be held liable for criminal seduction punished under either Article 337 or Article 338 of the Revised Penal Code because the private respondent was above eighteen (18) years of age at the time of the seduction. Prior decisions of this Court clearly suggest that Article 21 may be applied in a breach of promise to marry where the woman is a victim of moral seduction. Thus, in Hermosisima vs. Court of Appeals, this Court denied recovery of damages to the woman because: . . . we find ourselves unable to say that petitioner is morally guilty of seduction, not only because he is approximately ten (10) years younger than the complainant who was around thirty-six (36) years of age, and as highly enlightened as a former high school teacher and a life insurance agent are supposed to be when she became intimate with petitioner, then a mere apprentice pilot, but, also, because the court of first instance found that, complainant "surrendered herself" to petitioner because, "overwhelmed by her love" for him, she "wanted to bind" him by having a fruit of their engagement even before they had the benefit of clergy. In Tanjanco vs. Court of Appeals, while this Court likewise hinted at possible recovery if there had been moral seduction, recovery was eventually denied because We were not convinced that such seduction existed. The following enlightening disquisition and conclusion were made in the said case: The Court of Appeals seem to have overlooked that the example set forth in the Code Commission's memorandum refers to a tort upon a minor who had been seduced. The essential feature is seduction, that in law is more than mere sexual intercourse, or a breach of a promise of marriage; it connotes essentially the idea of deceit, enticement, superior power or abuse of confidence on the part of the seducer to which the woman has yielded (U.S. vs. Buenaventura, 27 Phil. 121; U.S. vs. Arlante, 9 Phil. 595). It has been ruled in the Buenaventura case (supra) that To constitute seduction there must in all cases be some sufficient promise or inducement and the woman must yield because of the promise or other inducement. If she consents merely from carnal lust and the intercourse is from mutual desire, there is no seduction (43 Cent. Dig. tit. Seduction, par. 56) She must be induced to depart from the path of virtue by the use of some species of arts, persuasions and wiles, which are calculated to have and do have that effect, and which result in her person to ultimately submitting her person to the sexual embraces of her seducer (27 Phil. 123). And in American Jurisprudence we find: On the other hand, in an action by the woman, the enticement, persuasion or deception is the essence of the injury; and a mere proof of intercourse is insufficient to warrant a recovery. Accordingly it is not seduction where the willingness arises out of sexual desire of curiosity of the female, and the defendant merely affords her the needed opportunity for the commission of the act. It has been emphasized that to allow a recovery in all such cases would tend to the demoralization of the female sex, and would be a reward for unchastity by which a class of adventuresses would be swift to profit. (47 Am. Jur. 662) xxx xxx xxx Over and above the partisan allegations, the fact stand out that for one whole year, from 1958 to 1959, the plaintiff-appellee, a woman of adult age, maintain intimate sexual relations with appellant, with repeated acts of intercourse. Such conduct is incompatible with the idea of seduction. Plainly there is here voluntariness and mutual passion; for had the appellant been deceived, had she surrendered exclusively because of the deceit, artful persuasions and wiles of the defendant, she would not have again yielded to his embraces, much less for one year, without exacting early fulfillment of the alleged promises of marriage, and would have cut short all sexual relations upon finding that defendant did not intend to fulfill his defendant did not intend to fulfill his promise. Hence, we conclude that no case is made under article 21 of the Civil Code, and no other cause of action being alleged, no error was committed by the Court of First Instance in dismissing the complaint. 151

University of the Cordilleras College of Law First Year C S.Y. 2013 - 2014 In his annotations on the Civil Code, Associate Justice Edgardo L. Paras, who recently retired from this Court, opined that in a breach of promise to marry where there had been carnal knowledge, moral damages may be recovered: . . . if there be criminal or moral seduction, but not if the intercourse was due to mutual lust. (Hermosisima vs. Court of Appeals, G.R. No. L-14628, Sept. 30, 1960; Estopa vs. Piansay, Jr., L-14733, Sept. 30, 1960; Batarra vs. Marcos, 7 Phil. 56 (sic); Beatriz Galang vs. Court of Appeals, et al., L-17248, Jan. 29, 1962). (In other words, if the CAUSE be the promise to marry, and the EFFECT be the carnal knowledge, there is a chance that there was criminal or moral seduction, hence recovery of moral damages will prosper. If it be the other way around, there can be no recovery of moral damages, because here mutual lust has intervened). . . . together with "ACTUAL damages, should there be any, such as the expenses for the wedding presentations (See Domalagon v. Bolifer, 33 Phil. 471). Senator Arturo M. Tolentino is also of the same persuasion: It is submitted that the rule in Batarra vs. Marcos, still subsists, notwithstanding the incorporation of the present article in the Code. The example given by the Code Commission is correct, if there was seduction, not necessarily in the legal sense, but in the vulgar sense of deception. But when the sexual act is accomplished without any deceit or qualifying circumstance of abuse of authority or influence, but the woman, already of age, has knowingly given herself to a man, it cannot be said that there is an injury which can be the basis for indemnity. But so long as there is fraud, which is characterized by willfulness (sic), the action lies. The court, however, must weigh the degree of fraud, if it is sufficient to deceive the woman under the circumstances, because an act which would deceive a girl sixteen years of age may not constitute deceit as to an experienced woman thirty years of age. But so long as there is a wrongful act and a resulting injury, there should be civil liability, even if the act is not punishable under the criminal law and there should have been an acquittal or dismissal of the criminal case for that reason. We are unable to agree with the petitioner's alternative proposition to the effect that granting, for argument's sake, that he did promise to marry the private respondent, the latter is nevertheless also at fault. According to him, both parties are in pari delicto; hence, pursuant to Article 1412(1) of the Civil Code and the doctrine laid down in Batarra vs. Marcos, the private respondent cannot recover damages from the petitioner. The latter even goes as far as stating that if the private respondent had "sustained any injury or damage in their relationship, it is primarily because of her own doing, for: . . . She is also interested in the petitioner as the latter will become a doctor sooner or later. Take notice that she is a plain high school graduate and a mere employee . . . (Annex "C") or a waitress (TSN, p. 51, January 25, 1988) in a luncheonette and without doubt, is in need of a man who can give her economic security. Her family is in dire need of financial assistance. (TSN, pp. 51-53, May 18, 1988). And this predicament prompted her to accept a proposition that may have been offered by the petitioner. These statements reveal the true character and motive of the petitioner. It is clear that he harbors a condescending, if not sarcastic, regard for the private respondent on account of the latter's ignoble birth, inferior educational background, poverty and, as perceived by him, dishonorable employment. Obviously then, from the very beginning, he was not at all moved by good faith and an honest motive. Marrying with a woman so circumstances could not have even remotely occurred to him. Thus, his profession of love and promise to marry were empty words directly intended to fool, dupe, entice, beguile and deceive the poor woman into believing that indeed, he loved her and would want her to be his life's partner. His was nothing but pure lust which he wanted satisfied by a Filipina who honestly believed that by accepting his proffer of love and proposal of marriage, she would be able to enjoy a life of ease and security. Petitioner clearly violated the Filipino's concept of morality and brazenly defied the traditional respect Filipinos have for their women. It can even be said that the petitioner committed such deplorable acts in blatant disregard of Article 19 of the Civil Code which directs every person to act with justice, give everyone his due and observe honesty and good faith in the exercise of his rights and in the performance of his obligations. No foreigner must be allowed to make a mockery of our laws, customs and traditions.

152

University of the Cordilleras College of Law First Year C S.Y. 2013 - 2014 The pari delicto rule does not apply in this case for while indeed, the private respondent may not have been impelled by the purest of intentions, she eventually submitted to the petitioner in sexual congress not out of lust, but because of moral seduction. In fact, it is apparent that she had qualms of conscience about the entire episode for as soon as she found out that the petitioner was not going to marry her after all, she left him. She is not, therefore, in pari delicto with the petitioner. Pari delicto means "in equal fault; in a similar offense or crime; equal in guilt or in legal fault." At most, it could be conceded that she is merely in delicto. Equity often interferes for the relief of the less guilty of the parties, where his transgression has been brought about by the imposition of undue influence of the party on whom the burden of the original wrong principally rests, or where his consent to the transaction was itself procured by fraud. In Mangayao vs. Lasud, 37 We declared: Appellants likewise stress that both parties being at fault, there should be no action by one against the other (Art. 1412, New Civil Code). This rule, however, has been interpreted as applicable only where the fault on both sides is, more or less, equivalent. It does not apply where one party is literate or intelligent and the other one is not. (c.f. Bough vs. Cantiveros, 40 Phil. 209). We should stress, however, that while We find for the private respondent, let it not be said that this Court condones the deplorable behavior of her parents in letting her and the petitioner stay together in the same room in their house after giving approval to their marriage. It is the solemn duty of parents to protect the honor of their daughters and infuse upon them the higher values of morality and dignity. WHEREFORE, finding no reversible error in the challenged decision, the instant petition is hereby DENIED, with costs against the petitioner. SO ORDERED.

153

University of the Cordilleras College of Law First Year C S.Y. 2013 - 2014 Case Digest GASHEM SHOOKAT BAKSH VS COURT OF APPEALS 219 SCRA 115 G.R. No. 97336 Decided on: February 19, 1993 Ponente: DAVIDE, JR., J.: FACTS: This is an appeal by certiorari. On October 27, 1987, without the assistance of counsel, private respondent filed with the aforesaid trial court a complaint for damages against petitioner for the alleged violation of their agreement to get married. She alleges in said complaint that she is 20 years old, single, Filipino and a pretty lass of good moral character and reputation duly respected in her country; other petitioner, on the other hand, is an Iranian citizen residing at Lozano Apartments, Guilig, Dagupan City, and is an exchange student, before August 20, 1987 the latter courted and proposed to marry her, she accepted his love on the condition that they get married; they therefore agreed to get married. The petitioner forced her to live with him in the Lozano apartments. She was a virgin at that time; after a week before the filing of complaint, petitioners attitude towards her started to change. He maltreated and threatened to kill her; as a result of the complaint. Petitioner repudiated the marriage agreement and asked her not to live with him anymore and that the petitioner is already married to someone in Bacolod City. Private respondent then prayed for judgment ordering petitioner to pay her damages. On the other hand, petitioner claimed that he never proposed marriage to or agreed to be married with the private respondent and denied all allegations against him. After trial on the merits, the lower court ordered petitioner to pay the private respondent damages. ISSUE: Whether or not Article 21 of the Civil Code applies to the case at bar. HELD: The existing rule is that a breach of promise to marry per se is not an actionable wrong. Notwithstanding, Article 21, which is designed to expand the concepts of torts and quasi-delicts in this jurisdiction by granting adequate legal remedy for the untold number of moral wrongs which is impossible for human foresight to specifically enumerate and punish in the statute books. Article 2176 of the Civil Code, which defines quasi-delicts thus: Whoever by act or omission causes damage to another, there being fault or negligence, is obliged to pay for the damage done. Such fault or negligence, if there is no pre-existing contractual relation between the parties, is called a quasi-delict and is governed by the provisions of this Chapter. In the light of the above laudable purpose of Article 21, the court held that where a mans promise to marry in fact the proximate cause of the acceptance of his love by a woman and his representation to fulfill that promise thereafter becomes the proximate cause of the giving of herself unto him in sexual congress, proof that he had, in reality, no intention of marrying her and that the promise was only subtle scheme or deceptive device to entice or inveigle her to accept him and obtain her consent to sexual act could justify the award of damages pursuant to Article 21 not because of such breach of promise of marriage but because of the fraud and deceit behind it, and the willful injury to her honor and reputation which followed thereafter. It is essential however, that such injury should have been committed in a manner contrary to morals, good customs, or public policy.

154

University of the Cordilleras College of Law First Year C S.Y. 2013 - 2014

III. Independent Civil Action

155

University of the Cordilleras College of Law First Year C S.Y. 2013 - 2014 Ruiz vs Ucol 153 SCRA 16 G.R. No. L-45404 August 7, 1987 Full Case G. JESUS B. RUIZ, petitioner, vs. ENCARNACION UCOL and THE COURT OF APPEALS, respondents. GUTIERREZ, JR., J: This is an appeal from the order of the Court of First Instance of Ilocos Norte dismissing the plaintiffappellant's complaint for damages against defendant-appellee on the ground of res judicata. The issue involved being a pure question of law, the appellate court certified the appeal to us for decision on the merits. The facts are not disputed, Agustina Tagaca, laundrywoman for plaintiff-appellant Atty. Jesus B. Ruiz filed an administrative charge against defendant-appellee Encarnacion Ucol, a midwife in the health center of Sarratt Ilocos Norte. In her answer to the charges, Ucol alleged that Tagaca was merely used as a tool by Atty. Ruiz who wanted to get back at the Ucol's because of a case filed by Encarnacion Ucol's husband against Ruiz. She was also alleged to have made remarks that Atty. Ruiz instigated the complaint and fabricated the charges. The administrative case was dismissed. Ruiz decided to file his own criminal complaint for libel against Ucol based on the alleged libelous portion of Ucol's answer. Upon arraignment, Ucol entered a plea of not guilty. During the proceedings in the libel case, complainant Atty. Ruiz entered his appearance and participated as private prosecutor. After trial, the lower court rendered judgment acquitting Ucol on the ground that her guilt was not established beyond reasonable doubt. No pronouncement was made by the trial court as to the civil liability of the accused. Instead of appealing the civil aspects of the case, Ruiz filed a separate complaint for damages based on the same facts upon which the libel case was founded. Ucol filed a motion to dismiss stating that the action had prescribed and that the cause of action was barred by the decision in the criminal case for libel. The trial court granted the motion to dismiss on the ground of res judicata. As earlier stated, on appeal, the Court of Appeals certified the case to us, the only issue being whether or not the civil action for damages was already barred by the criminal case of libel. Before going into the merit of this appeal, it is noteworthy to mention that there are actually two cases now before us involving the contending parties. Defendant-appellee Ucol filed an "appeal by certiorari" before this Court questioning the dissenting opinion of the Court of Appeals. Ucol prays for a ruling "that the respondent Court of Appeals committed a grave abuse of discretion in not dismissing the present case but instead in ordering the same remanded to the lower court for further proceedings ... ." Any ordinary student in law school should readily know that what comprises a decision which can be the subject of an appeal or a special civil action is the majority opinion of the members of the court, but never the dissenting opinion. Moreover, no decision on appeal has as yet been rendered in this case. The act of the defendant-appellee's counsel in filing such a petition defies logic or reason. It is totally inexplicable how a member of the bar could be so careless or, if the act was deliberate, could have the courage to come before this Court asking us to review a dissenting opinion. Counsel is warned that we do not find his mistake in the slightest bit amusing.

156

University of the Cordilleras College of Law First Year C S.Y. 2013 - 2014 Turning now to the present appeal, plaintiff-appellant Ruiz contends that there can be no res judicata since nowhere in its decision did the trial court pass upon the civil aspect of the criminal case nor did it make any express declaration that the fact on which said case was predicated did not exist. He cites the pertinent provisions of Article 29 of the Civil Code and Rule III, Section 3 subsection (c) of the Rules of Court which respectively provide: ART. 29. When the accused in a criminal prosecution is acquitted on the ground that his guilt has not been proved beyond reasonable doubt, a civil action for damages for the same act or omission may be instituted. ... xxx xxx xxx RULE III, Sec. 3(c) Extinction of the penal action does not carry with it extinction of the civil, unless the extinction proceeds from a declaration in a final judgment that the fact from which the civil might arise did not exist. ... We may also mention Article 33 of the Civil Code which gives an offended party in cases of defamation, among others, the right to file a civil action separate and distinct from the criminal proceedings whether or not a reservation was made to that effect. The plaintiff-appellant's contentions have no merit. The right of the plaintiff-appellant under the above provisions to file the civil action for damages based on the same facts upon which he instituted the libel case is not without limitation. We find the appeal of G. Jesus B. Ruiz without merit. We see no advantage or benefit in adding to the clogged dockets of our trial courts what plainly appears from the records to be a harassment suit. In acquitting Encarnacion Ucol of the libel charge, the trial court made these factual findings: Clearly then, Atty. Ruiz filed the instant Criminal Case against Encarnacion Ucol as retaliation for what he believed was an act of ingratitude to him on the part of her husband. The precipitate haste with which the administrative complaint was filed shows that he was the one personally interested in the matter. All that Agustina Tagaca told him was double hearsay. The incident, if there was, happened between the accused and Ceferino in the absence of Agustina; so that, all that Ceferina allegedly told her, and she in turn told Atty. Ruiz, was undoubtedly double check hearsay; and Atty. Ruiz should therefore check the facts with Ceferino, but he did not do that, and he did not even present Ceferino as a witness. For these reasons, accused has every reason to believe that Atty. Ruiz was the author who concocted the charges in the administrative complaint and had his laundry-woman, complainant Agustina Tagaca, sign it. Agustina has very little education and could hardly speak English, yet the administrative complaint was written in polished English, and who else but Atty. Ruiz could have authored those phrases in the complaint: "The retention of Mrs. Ucol in this government service is inimical to the good intentions of the Department to serve humanity and a disgrace and liability to present administration." As will be shown later on, it appears that it is this complaint signed by Agustina, but authored by Atty. Ruiz, that is libelous and not the respondent's answer; and even, assuming that the administrative complaint may not have been impelled by actual malice, the charge(s) were certainly reckless in the face of proven facts and circumstances. Court actions are not established for parties to give bent to their prejudice. The poor and the humble are, as a general rule, grateful to a fault, that intrigues and ingratitude are what they abhor. (Amended Record on Appeal, pp. 8-10). The findings in the criminal case, therefore, show a pattern of harassment. First, petitioner Ruiz had something to do with the administrative complaint. The complaint was dismissed. Second, he filed a criminal case for libel based on portions of Mrs. Ucol's answer in the administrative case. Third, he acted as private prosecutor in the criminal case actively handling as a lawyer the very case where he was the complainant. And fourth, after the accused was acquitted on the basis of the facts stated above, Atty. Ruiz pursued his anger at the Ucols with implacability by filing a civil action for damages. As stated by the trial judge, "court actions are not established for parties to give bent to their prejudice." This is doubly 157

University of the Cordilleras College of Law First Year C S.Y. 2013 - 2014 true when the party incessantly filing cases is a member of the bar. He should set an example in sobriety and in trying to prevent false and groundless suits. In Roa v. de la Cruz, et al. (107, Phil. 10) this Court ruled: Under the above provisions (Art. 33 of the Civil Code), independently of a criminal action for defamation, a civil suit for the recovery of damages arising therefrom may be brought by the injured party. It is apparent, however, from the use of the words "may be," that the institution of such suit is optional." (An Outline of Philippine Civil Law by J.B.L. Reyes and R.C. Puno, Vol. I, p. 54) In other words, the civil liability arising from the crime charged may still be determined in the criminal proceedings if the offended party does not waive to have it adjudged, or does not reserve his right to institute a separate civil action against the defendant. (The case of Reyes v. de la Rosa (52 Off. Gaz., [15] 6548; 99 Phil., 1013) cited by plaintiff in support of her contention that under Art. 33 of the New Civil Code the injured party is not required to reserve her right to institute the civil action, is not applicable to the present case. There was no showing in that case that the offended party intervened in the prosecution of the offense, and the amount of damages sought to be recovered was beyond the jurisdiction of the criminal court so that a reservation of the civil action was useless or unnecessary.) (Dionisio v. Alvendia, 102 Phil., 443; 55 Off. Gaz., [25]4633.]) In the instant case, it is not disputed that plaintiff Maria C. Roa upon whose initiative the criminal action for defamation against the defendant Segunda de la Cruz was filed did not reserve her right to institute it, subject, always to the direction and control of the prosecuting fiscal. (Section 15 in connection with section 4 of Rule 106, Rules of Court; Lim Tek Goan v. Yatco, 94 Phil., 197). The reason of the law in not permitting the offended party to intervene in the prosecution of the offense if he had waived or reserved his right to institute the civil action is that by such action her interest in the criminal case has disappeared. Its prosecution becomes the sole function of the public prosecutor. (Gorospe, et al., v. Gatmaitan, et al., 98 Phil., 600; 52 Off. Gaz., [15] 2526). The rule, therefore, is that the right of intervention reserved to the injured party is for the sole purpose of enforcing the civil liability born of the criminal act and not of demanding punishment of the accused. (People v. Orais, 65 Phil., 744; People v. Velez, 77 Phil., 1026; People v. Flores, et al., G.R. No. L-7528, December 18,1957; see also U.S. v. Malabon, 1 Phil., 731; U.S. v. Heery, 25 Phil., 600). Plaintiff having elected to claim damages arising from the offense charged in the criminal case through her appearance or intervention as private prosecutor we hold that the final judgment rendered therein constitutes a bar to the present civil action for damages based upon the same cause. (See Tan v. Standard Vacuum Oil Co., et al., 91 Phil., 672; 48 Off. Gaz., [7] 2745.). We are, therefore, constrained to dismiss the present appeal. Atty. Ruiz has more than had his day in court. The then court of first instance acquitted Mrs. Ucol and stated in the dispositive portion of its decision that her guilt was not established beyond reasonable doubt. A review of the court's findings, however, indicates that the disputed Answer of Mrs. Ucol in the administrative case contains no libel. As stated by the trial court, "As will be shown later, it appears that it is this complaint signed by Agustina, but authored by Atty. Ruiz, that is libelous and not the respondent's answer." (Emphasis supplied). The court found the charges against Ucol, if not malicious, at least reckless in the face of proven facts and circumstances. The trial court stated. Analyzing defendant's answer Exh. "5", even with meticulous care, the Court did not find any defamatory imputation which causes dishonor or discredit to the complainant. She was the victim of an unprovoked, unjustified and libelous attack against her honor, honesty, character and reputation; she has a right to selfdefense, which she did in her answer, to protect her honesty and integrity and the very job upon which her family depend for their livelihood. Every sentence in her answer (Exh. "5") is relevant, and constitutes privileged matter. She did not go further than her interest or duties require. She did not go beyond explaining what was said of her in the complaint for the purpose of repairing if not entirely removing the

158

University of the Cordilleras College of Law First Year C S.Y. 2013 - 2014 effects of the charge against her. She had absolutely no motive to libel Atty. Ruiz who, by the way, cast the first stone. ... (Amended Record on Appeal pp. 10-11) WHEREFORE, the appeal filed by appellant Jesus B. Ruiz is DISMISSED for lack of merit. The petition filed by petitioner Encarnacion Ucol is likewise DISMISSED for patent lack of merit. SO ORDERED.

159

University of the Cordilleras College of Law First Year C S.Y. 2013 - 2014 Padilla vs Court of Appeals 129 SCRA 558 G.R. No. L-39999 May 31, 1984 Full Case ROY PADILLA, FILOMENO GALDONES, ISMAEL GONZALGO and JOSE FARLEY BEDENIA, petitioners, vs. COURT OF APPEALS, respondent. GUTIERREZ, JR., J.: This is a petition for review on certiorari of a Court of Appeals' decision which reversed the trial court's judgment of conviction and acquitted the petitioners of the crime of grave coercion on the ground of reasonable doubt but inspite of the acquittal ordered them to pay jointly and severally the amount of P9,000.00 to the complainants as actual damages. The petitioners were charged under the following information: The undersigned Fiscal accused ROY PADILLA, FILOMENO GALDONES, PEPITO BEDENIA, YOLLY RICO, DAVID BERMUNDO, VILLANOAC, ROBERTO ROSALES, VILLANIA, ROMEO GARRIDO, JOSE ORTEGA, JR., RICARDO CELESTINO, REALINGO alias "KAMLON", JOHN DOE alias TATO, and FOURTEEN (14) RICARDO DOES of the crime of GRAVE COERCION, committed as follows: That on or about February 8, 1964 at around 9:00 o'clock in the morning, in the municipality of Jose Panganiban, province of Camarines Norte, Philippines, and within the jurisdiction of this Honorable Court, the above- named accused, Roy Padilla, Filomeno Galdones, Pepito Bedenia, Yolly Rico, David Bermundo, Villanoac, Roberto Rosales, Villania, Romeo Garrido, Jose Ortega, Jr., Ricardo Celestino, Realingo alias Kamlon, John Doe alias Tato, and Fourteen Richard Does, by confederating and mutually helping one another, and acting without any authority of law, did then and there wilfully, unlawfully, and feloniously, by means of threats, force and violence prevent Antonio Vergara and his family to close their stall located at the Public Market, Building No. 3, Jose Panganiban, Camarines Norte, and by subsequently forcibly opening the door of said stall and thereafter brutally demolishing and destroying said stall and the furnitures therein by axes and other massive instruments, and carrying away the goods, wares and merchandise, to the damage and prejudice of the said Antonio Vergara and his family in the amount of P30,000.00 in concept of actual or compensatory and moral damages, and further the sum of P20,000.00 as exemplary damages. That in committing the offense, the accused took advantage of their public positions: Roy Padilla, being the incumbent municipal mayor, and the rest of the accused being policemen, except Ricardo Celestino who is a civilian, all of Jose Panganiban, Camarines Norte, and that it was committed with evident premeditation. The Court of First Instance of Camarines Norte, Tenth Judicial District rendered a decision, the dispositive portion of which states that: IN VIEW OF THE FOREGOING, the Court finds the accused Roy Padilla, Filomeno Galdonez, Ismael Gonzalgo and Jose Parley Bedenia guilty beyond reasonable doubt of the crime of grave coercion, and hereby imposes upon them to suffer an imprisonment of FIVE (5) months and One (1) day; to pay a fine of P500.00 each; to pay actual and compensatory damages in the amount of P10,000.00; moral damages in the amount of P30,000.00; and another P10,000.00 for exemplary damages, jointly and severally, and all the accessory penalties provided for by law; and to pay the proportionate costs of this proceedings. The accused Federico Realingo alias 'Kamlon', David Bermundo, Christopher Villanoac, Godofredo Villania, Romeo Garrido, Roberto Rosales, Ricardo Celestino and Jose Ortega, are 160

University of the Cordilleras College of Law First Year C S.Y. 2013 - 2014 hereby ordered acquitted on grounds of reasonable doubt for their criminal participation in the crime charged. The petitioners appealed the judgment of conviction to the Court of Appeals. They contended that the trial court's finding of grave coercion was not supported by the evidence. According to the petitioners, the town mayor had the power to order the clearance of market premises and the removal of the complainants' stall because the municipality had enacted municipal ordinances pursuant to which the market stall was a nuisance per se. The petitioners stated that the lower court erred in finding that the demolition of the complainants' stall was a violation of the very directive of the petitioner Mayor which gave the stall owners seventy two (72) hours to vacate the market premises. The petitioners questioned the imposition of prison terms of five months and one day and of accessory penalties provided by law. They also challenged the order to pay fines of P500.00 each, P10,000.00 actual and compensatory damages, P30,000.00 moral damages, P10,000.00 exemplary damages, and the costs of the suit. The dispositive portion of the decision of the respondent Court of Appeals states: WHEREFORE, we hereby modify the judgment appealed from in the sense that the appellants are acquitted on ground of reasonable doubt. but they are ordered to pay jointly and severally to complainants the amount of P9,600.00, as actual damages. The petitioners filed a motion for reconsideration contending that the acquittal of the defendantsappellants as to criminal liability results in the extinction of their civil liability. The Court of Appeals denied the motion holding that: xxx xxx xxx ... appellants' acquittal was based on reasonable doubt whether the crime of coercion was committed, not on facts that no unlawful act was committed; as their taking the law into their hands, destructing (sic) complainants' properties is unlawful, and, as evidence on record established that complainants suffered actual damages, the imposition of actual damages is correct. Consequently, the petitioners filed this special civil action, contending that: I THE COURT OF APPEALS COMMITTED A GRAVE ERROR OF LAW OR GRAVELY ABUSED ITS DISCRETION IN IMPOSING UPON PETITIONERS PAYMENT OF DAMAGES TO COMPLAINANTS AFTER ACQUITTING PETITIONERS OF THE CRIME CHARGED FROM WHICH SAID LIABILITY AROSE. II THE COURT OF APPEALS ERRED IN HOLDING IN ITS RESOLUTION DATED DECEMBER 26, 1974 THAT SINCE APPELLANTS' ACQUITTAL WAS BASED ON REASONABLE DOUBT, NOT ON FACTS THAT NO UNLAWFUL ACT WAS COMMITTED, THE IMPOSITION OF ACTUAL DAMAGES IS CORRECT. III THE COURT OF APPEALS COMMITTED A LEGAL INCONSISTENCY, IF NOT PLAIN JUDICIAL ERROR, IN HOLDING IN ITS APPEALED RESOLUTION THAT PETITIONERS COMMITTED AN UNLAWFUL ACT, THAT IS TAKING THE LAW INTO THEIR HANDS, DESTRUCTING (sic) 'COMPLAINANTS' PROPERTIES', AFTER HOLDING IN ITS MAIN DECISION OF NOVEMBER 6,1974 THAT THE ACTS FOR WHICH THEY WERE CHARGED DID NOT CONSTITUTE GRAVE COERCION AND THEY WERE NOT CHARGED OF ANY OTHER CRIME.

161

University of the Cordilleras College of Law First Year C S.Y. 2013 - 2014 IV THE COURT OF APPEALS ERRED IN ORDERING THE PETITIONERS HEREIN, APPELLANTS IN CA-G.R. NO. 13456CR, JOINTLY AND SEVERALLY, TO PAY COMPLAINANTS P9,600.00 IN SUPPOSED ACTUAL DAMAGES. The issue posed in the instant proceeding is whether or not the respondent court committed a reversible error in requiring the petitioners to pay civil indemnity to the complainants after acquitting them from the criminal charge. Petitioners maintain the view that where the civil liability which is included in the criminal action is that arising from and as a consequence of the criminal act, and the defendant was acquitted in the criminal case, (no civil liability arising from the criminal case), no civil liability arising from the criminal charge could be imposed upon him. They cite precedents to the effect that the liability of the defendant for the return of the amount received by him may not be enforced in the criminal case but must be raised in a separate civil action for the recovery of the said amount (People v. Pantig, 97 Phil. 748; following the doctrine laid down in Manila Railroad Co. v. Honorable Rodolfo Baltazar, 49 O.G. 3874; Pueblo contra Abellera, 69 Phil. 623; People v. Maniago 69 Phil. 496; People v. Miranda, 5 SCRA 1067; Aldaba v. Elepafio 116 Phil. 457). In the case before us, the petitioners were acquitted not because they did not commit the acts stated in the charge against them. There is no dispute over the forcible opening of the market stall, its demolition with axes and other instruments, and the carting away of the merchandize. The petitioners were acquitted because these acts were denominated coercion when they properly constituted some other offense such as threat or malicious mischief. The respondent Court of Appeals stated in its decision: For a complaint to prosper under the foregoing provision, the violence must be employed against the person, not against property as what happened in the case at bar. ... xxx xxx xxx The next problem is: May the accused be convicted of an offense other than coercion? From all appearances, they should have been prosecuted either for threats or malicious mischief. But the law does not allow us to render judgment of conviction for either of these offenses for the reason that they were not indicted for, these offenses. The information under which they were prosecuted does not allege the elements of either threats or malicious mischief. Although the information mentions that the act was by means of threats', it does not allege the particular threat made. An accused person is entitled to be informed of the nature of the acts imputed to him before he can be made to enter into trial upon a valid information. We rule that the crime of grave coercion has not been proved in accordance with law. While appellants are entitled to acquittal they nevertheless are liable for the actual damages suffered by the complainants by reason of the demolition of the stall and loss of some of their properties. The extinction of the penal action does not carry with it that of the civil, unless the extinction proceeds from a declaration in a final judgment that the fact from which the civil might arise did not exist. (Rule 111, Sec. 3 (c), Rev. Rules of Court; Laperal v. Aliza, 51 OG.R. 1311, People v. Velez, 44 OG. 1811). In the instant case, the fact from which the civil might arise, namely, the demolition of the stall and loss of the properties contained therein; exists, and this is not denied by the accused. And since there is no showing that the complainants have reserved or waived their right to institute a separate civil action, the civil aspect therein is deemed instituted with the criminal action. (Rule 111, Sec. 1, Rev. Rules of Court). xxx xxx xxx Section 1 of Rule 111 of the Rules of Court states the fundamental proposition that when a criminal action is instituted, the civil action for recovery of civil liability arising from the offense charged is impliedly 162

University of the Cordilleras College of Law First Year C S.Y. 2013 - 2014 instituted with it. There is no implied institution when the offended party expressly waives the civil action or reserves his right to institute it separately. (Morte Sr. v. Alvizo, Jr., 101 SCRA 221). The extinction of the civil action by reason of acquittal in the criminal case refers exclusively to civil liability ex delicto founded on Article 100 of the Revised Penal Code. (Elcano v. Hill, 77 SCRA 98; Virata v. Ochoa, 81 SCRA 472). In other words, the civil liability which is also extinguished upon acquittal of the accused is the civil liability arising from the act as a crime. As easily as 1942, the Supreme Court speaking through Justice Jorge Bocobo in Barredo v. Garcia, et at. 73 Phil. 607 laid down the rule that the same punishable act or omission can create two kinds of civil liabilities against the accused and, where provided by law, his employer. 'There is the civil liability arising from the act as a crime and the liability arising from the same act as a quasi-delict. Either one of these two types of civil liability may be enforced against the accused, However, the offended party cannot recover damages under both types of liability. For instance, in cases of criminal negligence or crimes due to reckless imprudence, Article 2177 of the Civil Code provides: Responsibility for fault or negligence under the preceding article is entirely separate and distinct from the civil liability arising from negligence under the Penal Code. But the plaintiff cannot recover damages twice for the same act or omission of the defendant. Section 3 (c) of Rule 111 specifically provides that: Sec. 3. Other civil actions arising from offenses. In all cases not included in the preceding section the following rules shall be observed: xxx xxx xxx xxx xxx xxx (c) Extinction of the penal action does not carry with it extinction of the civil, unless the extinction proceeds from a declaration in a final judgment that the fact from which the civil might arise did not exist. In other cases, the person entitled to the civil action may institute it in the Jurisdiction and in the manner provided by law against the person who may be liable for restitution of the thing and reparation or indemnity for the damage suffered. The judgment of acquittal extinguishes the liability of the accused for damages only when it includes a declaration that the facts from which the civil might arise did not exist. Thus, the civil liability is not extinguished by acquittal where the acquittal is based on reasonable doubt (PNB v. Catipon, 98 Phil. 286) as only preponderance of evidence is required in civil cases; where the court expressly declares that the liability of the accused is not criminal but only civil in nature (De Guzman v. Alvia, 96 Phil. 558; People v. Pantig, supra) as, for instance, in the felonies of estafa, theft, and malicious mischief committed by certain relatives who thereby incur only civil liability (See Art. 332, Revised Penal Code); and, where the civil liability does not arise from or is not based upon the criminal act of which the accused was acquitted (Castro v. Collector of Internal Revenue, 4 SCRA 1093; See Regalado, Remedial Law Compendium, 1983 ed., p. 623). Article 29 of the Civil Code also provides that: When the accused in a criminal prosecution is acquitted on the ground that his guilt has not been proved beyond reasonable doubt, a civil action for damages for the same act or omission may be instituted. Such action requires only a preponderance of evidence. Upon motion of the defendant, the court may require the plaintiff to file a bond to answer for damages in case the complaint should be found to be malicious. If in a criminal case the judgment of acquittal is based upon reasonable doubt, the court shall so declare. In the absence of any declaration to that effect, it may be inferred from the text of the decision whether or not the acquittal is due to that ground. More recently, we held that the acquittal of the defendant in the criminal case would not constitute an obstacle to the filing of a civil case based on the same acts which led to the criminal prosecution: 163

University of the Cordilleras College of Law First Year C S.Y. 2013 - 2014 ... The finding by the respondent court that he spent said sum for and in the interest of the Capiz Agricultural and Fishery School and for his personal benefit is not a declaration that the fact upon which Civil Case No. V-3339 is based does not exist. The civil action barred by such a declaration is the civil liability arising from the offense charged, which is the one impliedly instituted with the criminal action. (Section 1, Rule III, Rules of Court.) Such a declaration would not bar a civil action filed against an accused who had been acquitted in the criminal case if the criminal action is predicated on factual or legal considerations other than the commission of the offense charged. A person may be acquitted of malversation where, as in the case at bar, he could show that he did not misappropriate the public funds in his possession, but he could be rendered liable to restore said funds or at least to make a proper accounting thereof if he shall spend the same for purposes which are not authorized nor intended, and in a manner not permitted by applicable rules and regulations. (Republic v. Bello, 120 SCRA 203) There appear to be no sound reasons to require a separate civil action to still be filed considering that the facts to be proved in the civil case have already been established in the criminal proceedings where the accused was acquitted. Due process has been accorded the accused. He was, in fact, exonerated of the criminal charged. The constitutional presumption of innocence called for more vigilant efforts on the part of prosecuting attorneys and defense counsel, a keener awareness by all witnesses of the serious implications of perjury, and a more studied consideration by the judge of the entire records and of applicable statutes and precedents. To require a separate civil action simply because the accused was acquitted would mean needless clogging of court dockets and unnecessary duplication of litigation with all its attendant loss of time, effort, and money on the part of all concerned. The trial court found the following facts clearly established by the evidence adduced by both the prosecution and the defense: xxx xxx xxx (9) In the morning of February 8, 1964, then Chief Galdones, complying with the instructions contained in said Memorandum No. 32 of the Mayor, and upon seeing that Antonio Vergara had not vacated the premises in question, with the aid of his policemen, forced upon the store or stall and ordered the removal of the goods inside the store of Vergara, at the same time taking inventory of the goods taken out, piled them outside in front of the store and had it cordoned with a rope, and after all the goods were taken out from the store, ordered the demolition of said stall of Antonio Vergara. Since then up to the trial of this case, the whereabouts of the goods taken out from the store nor the materials of the demolished stall have not been made known. The respondent Court of Appeals made a similar finding that: On the morning of February 8th, because the said Vergaras had not up to that time complied with the order to vacate, the co-accused Chief of Police Galdones and some members of his police force, went to the market and, using ax, crowbars and hammers, demolished the stall of the Vergaras who were not present or around, and after having first inventoried the goods and merchandise found therein, they had them brought to the municipal building for safekeeping. Inspite of notice served upon the Vergaras to take possession of the goods and merchandise thus taken away, the latter refused to do so. The loss and damage to the Vergaras as they evaluated them were: Cost of stall construction P1,300.00 Value of furniture and equipment judgment destroyed 300.00 Value of goods and equipment taken 8,000.00 P9,600.00 164

University of the Cordilleras College of Law First Year C S.Y. 2013 - 2014 It is not disputed that the accused demolished the grocery stall of the complainants Vergaras and carted away its contents. The defense that they did so in order to abate what they considered a nuisance per se is untenable, This finds no support in law and in fact. The couple has been paying rentals for the premises to the government which allowed them to lease the stall. It is, therefore, farfetched to say that the stall was a nuisance per se which could be summarily abated. The petitioners, themselves, do not deny the fact that they caused the destruction of the complainant's market stall and had its contents carted away. They state: On February 8, 1964, despite personal pleas on Vergaras by the Mayor to vacate the passageways of Market Building No. 3, the Vergaras were still in the premises, so the petitioners Chief of Police and members of the Police Force of Jose Panganiban, pursuant to the Mayor' 6 directives, demolished the store of the Vergaras, made an inventory of the goods found in said store, and brought these goods to the municipal building under the custody of the Municipal Treasurer, ... The only supposed obstacle is the provision of Article 29 of the Civil Code, earlier cited, that "when the accused in a criminal prosecution is acquitted on the ground that his guilt has not been proved beyond reasonable doubt, a civil action for damages for the same act or omission may be instituted." According to some scholars, this provision of substantive law calls for a separate civil action and cannot be modified by a rule of remedial law even in the interests of economy and simplicity and following the dictates of logic and common sense. As stated by retired Judge J. Cezar Sangco: ... if the Court finds the evidence sufficient to sustain the civil action but inadequate to justify a conviction in the criminal action, may it render judgment acquitting the accused on reasonable doubt, but hold him civilly liable nonetheless? An affirmative answer to this question would be consistent with the doctrine that the two are distinct and separate actions, and win (a) dispense with the reinstituting of the same civil action, or one based on quasi-delict or other independent civil action, and of presenting the same evidence: (b) save the injured party unnecessary expenses in the prosecution of the civil action or enable him to take advantage of the free services of the fiscal; and (c) otherwise resolve the unsettling implications of permitting the reinstitution of a separate civil action whether based on delict, or quasi-delict, or other independent civil actions. ... But for the court to be able to adjudicate in the manner here suggested, Art. 29 of the Civil Code should be amended because it clearly and expressly provides that the civil action based on the same act or omission may only be instituted in a separate action, and therefore, may not inferentially be resolved in the same criminal action. To dismiss the civil action upon acquittal of the accused and disallow the reinstitution of any other civil action, would likewise render, unjustifiably, the acquittal on reasonable doubt without any significance, and would violate the doctrine that the two actions are distinct and separate. In the light of the foregoing exposition, it seems evident that there is much sophistry and no pragmatism in the doctrine that it is inconsistent to award in the same proceedings damages against the accused after acquitting him on reasonable doubt. Such doctrine must recognize the distinct and separate character of the two actions, the nature of an acquittal on reasonable doubt, the vexatious and oppressive effects of a reservation or institution of a separate civil action, and that the injured party is entitled to damages not because the act or omission is punishable but because he was damaged or injured thereby (Sangco, Philippine Law on Torts and Damages, pp. 288-289). We see no need to amend Article 29 of the Civil Code in order to allow a court to grant damages despite a judgment of acquittal based on reasonable doubt. What Article 29 clearly and expressly provides is a remedy for the plaintiff in case the defendant has been acquitted in a criminal prosecution on the ground that his guilt has not been proved beyond reasonable doubt. It merely emphasizes that a civil action for damages is not precluded by an acquittal for the same criminal act or omission. The Civil Code provision 165

University of the Cordilleras College of Law First Year C S.Y. 2013 - 2014 does not state that the remedy can be availed of only in a separate civil action. A separate civil case may be filed but there is no statement that such separate filing is the only and exclusive permissible mode of recovering damages. There is nothing contrary to the Civil Code provision in the rendition of a judgment of acquittal and a judgment awarding damages in the same criminal action. The two can stand side by side. A judgment of acquittal operates to extinguish the criminal liability. It does not, however, extinguish the civil liability unless there is clear showing that the act from which civil liability might arise did not exist. A different conclusion would be attributing to the Civil Code a trivial requirement, a provision which imposes an uncalled for burden before one who has already been the victim of a condemnable, yet noncriminal, act may be accorded the justice which he seeks. We further note the rationale behind Art. 29 of the Civil Code in arriving at the intent of the legislator that they could not possibly have intended to make it more difficult for the aggrieved party to recover just compensation by making a separate civil action mandatory and exclusive: The old rule that the acquittal of the accused in a criminal case also releases him from civil liability is one of the most serious flaws in the Philippine legal system. It has given rise to numberless instances of miscarriage of justice, where the acquittal was due to a reasonable doubt in the mind of the court as to the guilt of the accused. The reasoning followed is that inasmuch as the civil responsibility is derived from the the criminal offense, when the latter is not proved, civil liability cannot be demanded. This is one of those cases where confused thinking leads to unfortunate and deplorable consequences. Such reasoning fails to draw a clear line of demarcation between criminal liability and civil responsibility, and to determine the logical result of the distinction. The two liabilities are separate and distinct from each other. One affects the social order and the other, private rights. One is for the punishment or correction of the offender while the other is for reparation of damages suffered by the aggrieved party... it is just and proper that, for the purposes of the imprisonment of or fine upon the accused, the offense should be proved beyond reasonable doubt. But for the purpose of indemnifying the complaining party, why should the offense also be proved beyond reasonable doubt? Is not the invasion or violation of every private right to be proved only by preponderance of evidence? Is the right of the aggrieved person any less private because the wrongful act is also punishable by the criminal law? (Code Commission, pp. 45-46). A separate civil action may be warranted where additional facts have to be established or more evidence must be adduced or where the criminal case has been fully terminated and a separate complaint would be just as efficacious or even more expedient than a timely remand to the trial court where the criminal action was decided for further hearings on the civil aspects of the case. The offended party may, of course, choose to file a separate action. These do not exist in this case. Considering moreover the delays suffered by the case in the trial, appellate, and review stages, it would be unjust to the complainants in this case to require at this time a separate civil action to be filed. With this in mind, we therefore hold that the respondent Court of Appeals did not err in awarding damages despite a judgment of acquittal. WHEREFORE, we hereby AFFIRM the decision of the respondent Court of Appeals and dismiss the petition for lack of merit. SO ORDERED.

166

University of the Cordilleras College of Law First Year C S.Y. 2013 - 2014 Dulay vs Court of Appeals 243 SCRA 220 G.R. No. 108017 April 3, 1995 Full Case MARIA BENITA A. DULAY, in her own behalf and in behalf of the minor children KRIZTEEN ELIZABETH, BEVERLY MARIE and NAPOLEON II, all surnamed DULAY, petitioners, vs. THE COURT OF APPEALS, Former Eighth Division, HON. TEODORO P. REGINO, in his capacity as Presiding Judge of the Regional Trial Court National Capital Region, Quezon City, Br. 84, SAFEGUARD INVESTIGATION AND SECURITY CO., INC., and SUPERGUARD SECURITY CORPORATION, respondents. BIDIN, J.: This petition for certiorari prays for the reversal of the decision of the Court of Appeals dated October 29, 1991 in CA-G.R. CV No. 24646 which affirmed the order of the Regional Trial Court dismissing Civil Case No. Q-89-1751, and its resolution dated November 17, 1991 denying herein, petitioner's motion for reconsideration. The antecedent facts of the case are as follows: On December 7, 1988, an altercation between Benigno Torzuela and Atty. Napoleon Dulay occurred at the "Big Bang Sa Alabang," Alabang Village, Muntinlupa as a result of which Benigno Torzuela, the security guard on duty at the said carnival, shot and killed Atty. Napoleon Dulay. Herein petitioner Maria Benita A. Dulay, widow of the deceased Napoleon Dulay, in her own behalf and in behalf of her minor children, filed on February 8, 1989 an action for damages against Benigno Torzuela and herein private respondents Safeguard Investigation and Security Co., Inc., ("SAFEGUARD") and/or Superguard Security Corp. ("SUPERGUARD"), alleged employers of defendant Torzuela. The complaint, docketed as Civil Case No. Q-89-1751 among others alleges the following: 1. . . . Defendants SAFEGUARD INVESTIGATION AND SECURITY CO., INC., (Defendant Safeguard) and SUPERGUARD SECURITY CORPORATION (Defendant Superguard) are corporations duly organized and existing in accordance with Philippine laws, with offices at 10th Floor, Manufacturers Building, Inc., Plaza Santa Cruz, Manila. They are impleaded as alternative defendants for, while the former appears to be the employer of defendant BENIGNO TORZUELA (defendant TORZUELA), the latter impliedly acknowledged responsibility for the acts of defendant TORZUELA by extending its sympathies to plaintiffs. Defendant BENIGNO TORZUELA is of legal age, an employee of defendant SAFEGUARD and/or defendant SUPERGUARD and, at the time of the incident complained of, was under their control and supervision. . . . 3. On December 7, 1988 at around 8:00 a.m., defendant TORZUELA, while he was on duty as security guard at the "Big Bang sa Alabang," Alabang Village, Muntinlupa, Metro Manila shot and killed NAPOLEON V. DULAY with a .38 caliber revolver belonging to defendant SAFEGUARD, and/or SUPERGUARD (per Police Report dated January 7, 1989, copy attached as Annex A); 4. The incident resulting in the death of NAPOLEON V. DULAY was due to the concurring negligence of the defendants. Defendant TORZUELA'S wanton and reckless discharge of the firearm issued to him by defendant SAFEGUARD and/or SUPERGUARD was the immediate and proximate cause of the injury, while the negligence of defendant SAFEGUARD and/or SUPERGUARD consists in its having failed to exercise the diligence of a good father of a family in the supervision and control of its employee to avoid the injury. 167

University of the Cordilleras College of Law First Year C S.Y. 2013 - 2014 xxx xxx xxx (Rollo, pp. 117-118) Petitioners prayed for actual, compensatory, moral and exemplary damages, and attorney's fees. The said Civil Case No. Q-89-1751 was raffled to Branch 84 of the Regional Trial Court of Quezon City, presided by respondent Judge Teodoro Regino. On March 2, 1989, private respondent SUPERGUARD filed a Motion to Dismiss on the ground that the complaint does not state a valid cause of action. SUPERGUARD claimed that Torzuela's act of shooting Dulay was beyond the scope of his duties, and that since the alleged act of shooting was committed with deliberate intent (dolo), the civil liability therefor is governed by Article 100 of the Revised Penal Code, which states: Art. 100. Civil liability of a person guilty of a felony. Every person criminally liable for a felony is also civilly liable. Respondent SUPERGUARD further alleged that a complaint for damages based on negligence under Article 2176 of the New Civil Code, such as the one filed by petitioners, cannot lie, since the civil liability under Article 2176 applies only to quasi-offenses under Article 365 of the Revised Penal Code. In addition, the private respondent argued that petitioners' filing of the complaint is premature considering that the conviction of Torzuela in a criminal case is a condition sine qua non for the employer's subsidiary liability (Rollo, p. 55-59). Respondent SAFEGUARD also filed a motion praying that it be excluded as defendant on the ground that defendant Torzuela is not one of its employees (Rollo, p. 96). Petitioners opposed both motions, stating that their cause of action against the private respondents is based on their liability under Article 2180 of the New Civil Code, which provides: Art. 2180. The obligation imposed by Article 2176 is demandable not only for one's own acts or omissions, but also for those of persons for whom one is responsible. xxx xxx xxx Employers shall be liable for the damages caused by their employees and household helpers acting within the scope of their assigned tasks, even though the former are not engaged in any business or an industry. xxx xxx xxx (Emphasis supplied) Petitioners contended that a suit against alternative defendants is allowed under Rule 3, Section 13 of the Rules of Court. Therefore, the inclusion of private respondents as alternative defendants in the complaint is justified by the following: the Initial Investigation Report prepared by Pat. Mario Tubon showing that Torzuela is an employee of SAFEGUARD; and through overt acts, SUPERGUARD extended its sympathies to petitioners (Rollo, pp. 64 and 98). Meanwhile, an Information dated March 21, 1989 charging Benigno Torzuela with homicide was filed before the Regional Trial Court of Makati and was docketed as Criminal Case No. 89-1896. On April 13, 1989, respondent Judge Regino issued an order granting SUPERGUARD'S motion to dismiss and SAFEGUARD'S motion for exclusion as defendant. The respondent judge held that the complaint did not state facts necessary or sufficient to constitute a quasi-delict since it does not mention any negligence on the part of Torzuela in shooting Napoleon Dulay or that the same was done in the performance of his duties. Respondent judge ruled that mere allegations of the concurring negligence of the defendants (private respondents herein) without stating the facts showing such negligence are mere conclusions of law (Rollo, p. 106). Respondent judge also declared that the complaint was one for damages founded on crimes punishable under Articles 100 and 103 of the Revised Penal Code as 168

University of the Cordilleras College of Law First Year C S.Y. 2013 - 2014 distinguished from those arising from, quasi-delict. The dispositive portion of the order dated April 13, 1989 states: WHEREFORE, this Court holds that in view of the material and ultimate facts alleged in the verified complaint and in accordance with the applicable law on the matter as well as precedents laid down by the Supreme Court, the complaint against the alternative defendants Superguard Security Corporation and Safeguard Investigation and Security Co., Inc., must be and (sic) it is hereby dismissed. (Rollo, p. 110) The above order was affirmed by the respondent court and petitioners' motion for reconsideration thereof was denied. Petitioners take exception to the assailed decision and insist that quasi-delicts are not limited to acts of negligence but also cover acts that are intentional and voluntary, citing Andamo v. IAC (191 SCRA 195 [1990]). Thus, petitioners insist that Torzuela' s act of shooting Napoleon Dulay constitutes a quasi-delict actionable under Article 2176 of the New Civil Code. Petitioners further contend that under Article 2180 of the New Civil Code, private respondents are primarily liable for their negligence either in the selection or supervision of their employees. This liability is independent of the employee's own liability for fault or negligence and is distinct from the subsidiary civil liability under Article 103 of the Revised Penal Code. The civil action against the employer may therefore proceed independently of the criminal action pursuant to Rule 111 Section 3 of the Rules of Court. Petitioners submit that the question of whether Torzuela is an employee of respondent SUPERGUARD or SAFEGUARD would be better resolved after trial. Moreover, petitioners argue that Torzuela's act of shooting Dulay is also actionable under Article 33 of the New Civil Code, to wit: Art. 33. In cases of defamation, fraud, and physical injuries, a civil action for damages, entirely separate and distinct from the criminal action, may be brought by the injured party. Such civil action shall proceed independently of the criminal prosecution, and shall require only a preponderance of evidence. (Emphasis supplied) In the same vein, petitioners cite Section 3, Rule 111 of the Rules of Court which provides: Rule 111. . . . . Sec. 3. When civil action may proceed independently In the cases provided for in Articles 32, 33, 34 and 2176 of the Civil Code of the Philippines, the independent civil action which has been reserved may be brought by the offended party, shall proceed independently of the criminal action, and shall require only a preponderance of evidence. (Emphasis supplied) The term "physical injuries" under Article 33 has been held to include consummated, frustrated and attempted homicide. Thus, petitioners maintain that Torzuela's prior conviction is unnecessary since the civil action can proceed independently of the criminal action. On the other hand, it is the private respondents' argument that since the act was not committed with negligence, the petitioners have no cause of action under Articles 2116 and 2177 of the New Civil Code. The civil action contemplated in Article 2177 is not applicable to acts committed with deliberate intent, but only applies to quasi-offenses under Article 365 of the Revised Penal Code. Torzuela's act of shooting Atty. Dulay to death, aside from being purely personal, was done with deliberate intent and could not have been part of his duties as security guard. And since Article 2180 of the New Civil Code covers only: acts done within the scope of the employee's assigned tasks, the private respondents cannot be held liable for damages. We find for petitioners. It is undisputed that Benigno Torzuela is being prosecuted for homicide for the fatal shooting of Napoleon Dulay. Rule 111 of the Rules on Criminal Procedure provides:

169

University of the Cordilleras College of Law First Year C S.Y. 2013 - 2014 Sec. 1. Institution of criminal and civil actions. When a criminal action is instituted, the civil action for the recovery of civil liability is impliedly instituted with the criminal action, unless the offended party waives the civil action , reserves his right to institute it separately or institutes the civil action prior to the criminal action. Such civil action includes recovery of indemnity under the Revised Penal Code, and damages under Articles 32, 33, 34, and 2176 of the Civil Code of the Philippines arising from the same act or omission of the accused. (Emphasis supplied) It is well-settled that the filing of an independent civil action before the prosecution in the criminal action presents evidence is even far better than a compliance with the requirement of express reservation (Yakult Philippines v. Court of Appeals, 190 SCRA 357 [1990]). This is precisely what the petitioners opted to do in this case. However, the private respondents opposed the civil action on the ground that the same is founded on a delict and not on a quasi-delict as the shooting was not attended by negligence. What is in dispute therefore is the nature of the petitioner's cause of action. The nature of a cause of action is determined by the facts alleged in the complaint as constituting the cause of action (Republic v. Estenzo, 158 SCRA 282 [1988]). The purpose of an action or suit and the law to govern it is to be determined not by the claim of the party filing the action, made in his argument or brief, but rather by the complaint itself, its allegations and prayer for relief. (De Tavera v. Philippine Tuberculosis Society, 112 SCRA 243 [1982]). An examination of the complaint in the present case would show that the plaintiffs, petitioners herein, are invoking their right to recover damages against the private respondents for their vicarious responsibility for the injury caused by Benigno Torzuela's act of shooting and killing Napoleon Dulay, as stated in paragraphs 1 and 2 of the complaint. Article 2176 of the New Civil Code provides: Art. 2176. Whoever by act or omission causes damage to another, there being fault or negligence, is obliged to pay for the damage done. Such fault or negligence, if there is no pre-existing contractual relation between the parties is called a quasi-delict and is governed by the provisions of this Chapter. Contrary to the theory of private respondents, there is no justification for limiting the scope of Article 2176 of the Civil Code to acts or omissions resulting from negligence. Well-entrenched is the doctrine that article 2176 covers not only acts committed with negligence, but also acts which are voluntary and intentional. As far back as the definitive case of Elcano v. Hill (77 SCRA 98 [1977]), this Court already held that: . . . Article 2176, where it refers to "fault or negligence," covers not only acts "not punishable by law" but also acts criminal in character; whether intentional and voluntary or negligent. Consequently, a separate civil action against the offender in a criminal act, whether or not he is criminally prosecuted and found guilty or acquitted, provided that the offended party is not allowed, if he is actually charged also criminally, to recover damages on both scores, and would be entitled in such eventuality only to the bigger award of the two, assuming the awards made in the two cases vary. In other words, the extinction of civil liability referred to in Par. (e) of Section 3, Rule 111, refers exclusively to civil liability founded on Article 100 of the Revised Penal Code, whereas the civil liability for the same act considered as quasidelict only and not as a crime is not extinguished even by a declaration in the criminal case that the criminal act charged has not happened or has not been committed by the accused. Briefly stated, We here hold, in reiteration of Garcia, that culpa aquiliana includes voluntary and negligent acts which may be punishable by law. (Emphasis supplied) The same doctrine was echoed in the case of Andamo v. Intermediate Appellate Court (191 SCRA 195 [1990]), wherein the Court held: Article 2176, whenever it refers to "fault or negligence," covers not only acts criminal in character, whether intentional and voluntary or negligent. Consequently, a civil action lies against the offender in a criminal act, whether or not he is prosecuted or found guilty or acquitted, provided that the offended party is not allowed, (if the tortfeasor is actually also charged criminally), to recover damages on both scores, 170

University of the Cordilleras College of Law First Year C S.Y. 2013 - 2014 and would be entitled in such eventuality only to the bigger award of the two, assuming the awards made in the two cases vary. [citing Virata v. Ochoa, 81 SCRA 472] (Emphasis supplied) Private respondents submit that the word "intentional" in the Andamo case is inaccurate obiter, and should be read as "voluntary" since intent cannot be coupled with negligence as defined by Article 365 of the Revised Penal Code. In the absence of more substantial reasons, this Court will not disturb the above doctrine on the coverage of Article 2176. Private respondents further aver that Article 33 of the New Civil Code applies only to injuries intentionally committed pursuant to the ruling in Marcia v. CA (120 SCRA 193 [1983]), and that the actions for damages allowed thereunder are ex-delicto. However, the term "physical injuries" in Article 33 has already been construed to include bodily injuries causing death (Capuno v. Pepsi-Cola Bottling Co. of the Philippines, 121 Phil. 638 [1965); Carandang v. Santiago, 97 Phil. 94 [1955]). It is not the crime of physical injuries defined in the Revised Penal Code. It includes not only physical injuries but also consummated, frustrated, and attempted homicide (Madeja v. Caro, 126 SCRA 293 [1983]). Although in the Marcia case (supra), it was held that no independent civil action may be filed under Article 33 where the crime is the result of criminal negligence, it must be noted however, that Torzuela, the accused in the case at bar, is charged with homicide, not with reckless imprudence, whereas the defendant in Marcia was charged with reckless imprudence. Therefore, in this case, a civil action based on Article 33 lies. Private respondents also contend that their liability is subsidiary under the Revised Penal Code; and that they are not liable for Torzuela's act which is beyond the scope of his duties as a security guard. It having been established that the instant action is not ex-delicto, petitioners may proceed directly against Torzuela and the private respondents. Under Article 2180 of the New Civil Code as aforequoted, when an injury is caused by the negligence of the employee, there instantly arises a presumption of law that there was negligence on the part of the master or employer either in the selection of the servant or employee, or in supervision over him after selection or both (Layugan v. Intermediate Appellate Court, 167 SCRA 363 [1988]). The liability of the employer under Article 2180 is direct and immediate; it is not conditioned upon prior recourse against the negligent employee and a prior showing of the insolvency of such employee (Kapalaran Bus Lines v. Coronado, 176 SCRA 792 [1989]). Therefore, it is incumbent upon the private respondents to prove that they exercised the diligence of a good father of a family in the selection and supervision of their employee. Since Article 2176 covers not only acts of negligence but also acts which are intentional and voluntary, it was therefore erroneous on the part of the trial court to dismiss petitioner's complaint simply because it failed to make allegations of attendant negligence attributable to private respondents. With respect to the issue of whether the complaint at hand states a sufficient cause of action, the general rule is that the allegations in a complaint are sufficient to constitute a cause of action against the defendants if, admitting the facts alleged, the court can render a valid judgment upon the same in accordance with the prayer therein. A cause of action exist if the following elements are present, namely: (1) a right in favor of the plaintiff by whatever means and under whatever law it arises or is created; (2) an obligation on the part of the named defendant to respect or not to violate such right; and (3) an act or omission on the part of such defendant violative of the right of the plaintiff or constituting a breach of the obligation of the defendant to the plaintiff for which the latter may maintain an action for recovery of damages (Del Bros Hotel Corporation v. CA, 210 SCRA 33 [1992]); Development Bank of the Philippines v. Pundogar, 218 SCRA 118 [1993]) This Court finds, under the foregoing premises, that the complaint sufficiently alleged an actionable breach on the part of the defendant Torzuela and respondents SUPERGUARD and/or SAFEGUARD. It is enough that the complaint alleged that Benigno Torzuela shot Napoleon Dulay resulting in the latter's death; that the shooting occurred while Torzuela was on duty; and that either SUPERGUARD and/or SAFEGUARD was Torzuela's employer and responsible for his acts. This does not operate however, to establish that the defendants below are liable. Whether or not the shooting was actually reckless and wanton or attended by negligence and whether it was actually done within the scope of Torzuela's duties; whether the private respondents SUPERGUARD and/or SAFEGUARD failed to exercise the diligence of 171

University of the Cordilleras College of Law First Year C S.Y. 2013 - 2014 a good father of a family; and whether the defendants are actually liable, are questions which can be better resolved after trial on the merits where each party can present evidence to prove their respective allegations and defenses. In determining whether the allegations of a complaint are sufficient to support a cause of action, it must be borne in mind that the complaint does not have to establish or allege the facts proving the existence of a cause of action at the outset; this will have to be done at the trial on the merits of the case (Del Bros Hotel Corporation v. CA, supra). If the allegations in a complaint can furnish a sufficient basis by which the complaint can be maintained, the same should not be dismissed regardless of the defenses that may be assessed by the defendants (Rava Dev't. Corp. v. CA, 211 SCRA 152 [1992] citing Consolidated Bank & Trust Corporation v. Court of Appeals, 197 SCRA 663 [1991]). To sustain a motion to dismiss for lack of cause of action, the complaint must show that the claim for relief does not exist rather than that a claim has been defectively stated, is ambiguous, indefinite or uncertain (Azur v. Provincial Board, 27 SCRA 50 [1969]). Since the petitioners clearly sustained an injury to their rights under the law, it would be more just to allow them to present evidence of such injury. WHEREFORE, premises considered, the petition for review is hereby GRANTED. The decision of the Court of Appeals as well as the Order of the Regional Trial Court dated April 13, 1989 are hereby REVERSED and SET ASIDE. Civil Case No. Q-89-1751 is remanded to the Regional Trial Court for trial on the merits. This decision is immediately executory. SO ORDERED.

172

University of the Cordilleras College of Law First Year C S.Y. 2013 - 2014

IV. Prejudicial Questions

173

University of the Cordilleras College of Law First Year C S.Y. 2013 - 2014 Quiambao vs Osorio 158 SCRA 674 G.R. No. L-48157 March 16, 1988 Full Case RICARDO QUIAMBAO, petitioner, vs. HON. ADRIANO OSORIO, ZENAIDA GAZA BUENSUCERO, JUSTINA GAZA BERNARDO, and FELIPE GAZA, respondents-appellees, LAND AUTHORITY, intervenor-appellant. FERNAN, J.: This case was certified to Us by the Court of Appeals as one involving pure questions of law pursuant to Section 3, Rule 50 of the Revised Rules of Court. The antecedents are as follows: In a complaint for forcible entry filed by herein private respondents Zenaida Gaza Buensucero, Justina Gaza Bernardo and Felipe Gaza against herein petitioner Ricardo Quiambao before the then Municipal Court of Malabon, Rizal, docketed therein as Civil Case No. 2526, it was alleged that private respondents were the legitimate possessors of a 30,835 sq. m. lot known as Lot No. 4, Block 12, Bca 2039 of the Longos Estate situated at Barrio Longos, Malabon Rizal, by virtue of the Agreement to Sell No. 3482 executed in their favor by the former Land Tenure Administration [which later became the Land Authority, then the Department of Agrarian Reform]; that under cover of darkness, petitioner surreptitiously and by force, intimidation, strategy and stealth, entered into a 400 sq. m. portion thereof, placed bamboo posts "staka" over said portion and thereafter began the construction of a house thereon; and that these acts of petitioner, which were unlawful per se, entitled private respondents to a writ of preliminary injunction and to the ejectment of petitioner from the lot in question. Petitioner filed a motion to dismiss the complaint, and upon denial thereof, filed his Answer to the complaint, specifically denying the material allegations therein and averring that the Agreement upon which private respondents base their prior possession over the questioned lot had already been cancelled by the Land Authority in an Order signed by its Governor, Conrado Estrella. By way of affirmative defense and as a ground for dismissing the case, petitioner alleged the pendency of L.A. Case No. 968, an administrative case before the Office of the Land Authority between the same parties and involving the same piece of land. In said administrative case, petitioner disputed private respondents' right of possession over the property in question by reason of the latter's default in the installment payments for the purchase of said lot. Petitioner asserted that his administrative case was determinative of private respondents' right to eject petitioner from the lot in question; hence a prejudicial question which bars a judicial action until after its termination. After hearing, the municipal court denied the motion to dismiss contained in petitioner's affirmative defenses. It ruled that inasmuch as the issue involved in the case was the recovery of physical possession, the court had jurisdiction to try and hear the case. Dissatisfied with this ruling, petitioner filed before the then Court of First Instance of Rizal, Branch XII, Caloocan City in Civil Case No. C-1576 a petition for certiorari with injunction against public respondent Judge Adriano Osorio of the Municipal Court of Malabon and private respondents, praying for the issuance of a writ of preliminary injunction ordering respondent judge to suspend the hearing in the ejectment case until after the resolution of said petition. As prayed for, the then CFI of Rizal issued a restraining order enjoining further proceedings in the ejectment case. In his answer, respondent municipal judge submitted himself to the sound discretion of the CFI in the disposition of the petition for certiorari. Private respondents, on the other hand, filed a motion to dismiss the petition, maintaining that the administrative case did not constitute a prejudicial question as it involved the question of ownership, unlike the ejectment case which involved merely the question of possession. 174

University of the Cordilleras College of Law First Year C S.Y. 2013 - 2014 Meanwhile, the Land Authority filed an Urgent Motion for Leave to Intervene in Civil Case No. C-1576 alleging the pendency of an administrative case between the same parties on the same subject matter in L.A. Case No. 968 and praying that the petition for certiorari be granted, the ejectment complaint be dismissed and the Office of the Land Authority be allowed to decide the matter exclusively. Finding the issue involved in the ejectment case to be one of prior possession, the CFI dismissed the petition for certiorari and lifted the restraining order previously issued. Petitioner's motion for reconsideration of the dismissal order, adopted in toto by Intervenor Land Authority was denied for lack of merit. Hence, this appeal filed by petitioner Quiambao and intervenor Land Authority with the Court of Appeals, and certified to Us as aforesaid. The instant controversy boils down to the sole question of whether or not the administrative case between the private parties involving the lot subject matter of the ejectment case constitutes a prejudicial question which would operate as a bar to said ejectment case. A prejudicial question is understood in law to be that which arises in a case the resolution of which is a logical antecedent of the issue involved in said case and the cognizance of which pertains to another tribunal. The doctrine of prejudicial question comes into play generally in a situation where civil and criminal actions are pending and the issues involved in both cases are similar or so closely related that an issue must be pre-emptively resolved in the civil case before the criminal action can proceed. Thus, the existence of a prejudicial question in a civil case is alleged in the criminal case to cause the suspension of the latter pending final determination of the former. The essential elements of a prejudicial question as provided under Section 5, Rule 111 of the Revised Rules of Court are: [a] the civil action involves an issue similar or intimately related to the issue in the criminal action; and [b] the resolution of such issue determines whether or not the criminal action may proceed. The actions involved in the case at bar being respectively civil and administrative in character, it is obvious that technically, there is no prejudicial question to speak of. Equally apparent, however, is the intimate correlation between said two [2] proceedings, stemming from the fact that the right of private respondents to eject petitioner from the disputed portion depends primarily on the resolution of the pending administrative case. For while it may be true that private respondents had prior possession of the lot in question, at the time of the institution of the ejectment case, such right of possession had been terminated, or at the very least, suspended by the cancellation by the Land Authority of the Agreement to Sell executed in their favor. Whether or not private respondents can continue to exercise their right of possession is but a necessary, logical consequence of the issue involved in the pending administrative case assailing the validity of the cancellation of the Agreement to Sell and the subsequent award of the disputed portion to petitioner. If the cancellation of the Agreement to Sell and the subsequent award to petitioner are voided, then private respondents would have every right to eject petitioner from the disputed area. Otherwise, private respondent's light of possession is lost and so would their right to eject petitioner from said portion. Faced with these distinct possibilities, the more prudent course for the trial court to have taken is to hold the ejectment proceedings in abeyance until after a determination of the administrative case. Indeed, logic and pragmatism, if not jurisprudence, dictate such move. To allow the parties to undergo trial notwithstanding the possibility of petitioner's right of possession being upheld in the pending administrative case is to needlessly require not only the parties but the court as well to expend time, effort and money in what may turn out to be a sheer exercise in futility. Thus, 1 Am Jur 2d tells us: The court in which an action is pending may, in the exercise of a sound discretion, upon proper application for a stay of that action, hold the action in abeyance to abide the outcome of another pending in another court, especially where the parties and the issues are the same, for there is power inherent in every court to control the disposition of causes on its dockets with economy of time and effort for itself, for counsel, and for litigants. Where the rights parties to the second action cannot be properly determined until the questions raised in the first action are settled the second action should be stayed. 175

University of the Cordilleras College of Law First Year C S.Y. 2013 - 2014 While this rule is properly applicable to instances involving two [2] court actions, the existence in the instant case of the same considerations of Identity of parties and issues, economy of time and effort for the court, the counsels and the parties as well as the need to resolve the parties' right of possession before the ejectment case may be properly determined, justifies the rule's analogous application to the case at bar. Fortich-Celdran, et al. vs. Celdran, et al., 19 SCRA 502, provides another analogous situation. In sustaining the assailed order of the then Court of First Instance of Misamis Oriental ordering the suspension of the criminal case for falsification of public document against several persons, among them the subscribing officer Santiago Catane until the civil case involving the issue of the genuineness of the alleged forged document shall have been decided, this Court cited as a reason therefore its own action on the administrative charges against said Santiago Catane, as follows: It should be mentioned here also that an administrative case filed in this Court against Santiago Catane upon the same charge was held by Us in abeyance, thus: "As it appears that the genuineness of the document allegedly forged by respondent attorneys in Administrative Case No. 77 [Richard Ignacio Celdran vs. Santiago Catane, etc., et al.] is necessarily involved in Civil Case No. R-3397 of the Cebu Court of First Instance, action on the herein complaint about such decision." If a pending civil case may be considered to be in the nature of a prejudicial question to an administrative case, We see no reason why the reverse may not be so considered in the proper case, such as in the petition at bar. Finally, events occuring during the pendency of this petition attest to the wisdom of the conclusion herein reached. For in the Manifestation filed by counsel for petitioner, it was stated that the intervenor Land Authority which later became the Department of Agrarian Reform had promulgated a decision in the administrative case, L.A. Case No. 968 affiriming the cancellation of Agreement to Sell No. 3482 issued in favor of private respondents. With this development, the folly of allowing the ejectment case to proceed is too evident to need further elaboration. WHEREFORE, the instant petition is hereby GRANTED. Civil Case No. 2526 of the then Municipal Court of Malabon, Rizal is hereby ordered DISMISSED. No Costs. SO ORDERED.

176

University of the Cordilleras College of Law First Year C S.Y. 2013 - 2014 Bobis vs Bobis 336 SCRA 747 G.R. No. 138509 July 31, 2000 Full Case IMELDA MARBELLA-BOBIS, petitioner, vs. ISAGANI D. BOBIS, respondent. YNARES-SANTIAGO, J.: On October 21, 1985, respondent contracted a first marriage with one Maria Dulce B. Javier. Without said marriage having been annulled, nullified or terminated, the same respondent contracted a second marriage with petitioner Imelda Marbella-Bobis on January 25, 1996 and allegedly a third marriage with a certain Julia Sally Hernandez. Based on petitioner's complaint-affidavit, an information for bigamy was filed against respondent on February 25, 1998, which was docketed as Criminal Case No. Q98-75611 of the Regional Trial Court, Branch 226, Quezon City. Sometime thereafter, respondent initiated a civil action for the judicial declaration of absolute nullity of his first marriage on the ground that it was celebrated without a marriage license. Respondent then filed a motion to suspend the proceedings in the criminal case for bigamy invoking the pending civil case for nullity of the first marriage as a prejudicial question to the criminal case. The trial judge granted the motion to suspend the criminal case in an Order dated December 29, 1998. Petitioner filed a motion for reconsideration, but the same was denied. Hence, this petition for review on certiorari. Petitioner argues that respondent should have first obtained a judicial declaration of nullity of his first marriage before entering into the second marriage, inasmuch as the alleged prejudicial question justifying suspension of the bigamy case is no longer a legal truism pursuant to Article 40 of the Family Code. The issue to be resolved in this petition is whether the subsequent filing of a civil action for declaration of nullity of a previous marriage constitutes a prejudicial question to a criminal case for bigamy. A prejudicial question is one which arises in a case the resolution of which is a logical antecedent of the issue involved therein. It is a question based on a fact distinct and separate from the crime but so intimately connected with it that it determines the guilt or innocence of the accused. It must appear not only that the civil case involves facts upon which the criminal action is based, but also that the resolution of the issues raised in the civil action would necessarily be determinative of the criminal case. Consequently, the defense must involve an issue similar or intimately related to the same issue raised in the criminal action and its resolution determinative of whether or not the latter action may proceed. Its two essential elements are: (a) the civil action involves an issue similar or intimately related to the issue raised in the criminal action; and (b) the resolution of such issue determines whether or not the criminal action may proceed. A prejudicial question does not conclusively resolve the guilt or innocence of the accused but simply tests the sufficiency of the allegations in the information in order to sustain the further prosecution of the criminal case. A party who raises a prejudicial question is deemed to have hypothetically admitted that all the essential elements of a crime have been adequately alleged in the information, considering that the prosecution has not yet presented a single evidence on the indictment or may not yet have rested its case. A challenge of the allegations in the information on the ground of prejudicial question is in effect a question on the merits of the criminal charge through a non-criminal suit. Article 40 of the Family Code, which was effective at the time of celebration of the second marriage, requires a prior judicial declaration of nullity of a previous marriage before a party may remarry. The clear implication of this is that it is not for the parties, particularly the accused, to determine the validity or invalidity of the marriage. Whether or not the first marriage was void for lack of a license is a matter of 177

University of the Cordilleras College of Law First Year C S.Y. 2013 - 2014 defense because there is still no judicial declaration of its nullity at the time the second marriage was contracted. It should be remembered that bigamy can successfully be prosecuted provided all its elements concur two of which are a previous marriage and a subsequent marriage which would have been valid had it not been for the existence at the material time of the first marriage. In the case at bar, respondent's clear intent is to obtain a judicial declaration of nullity of his first marriage and thereafter to invoke that very same judgment to prevent his prosecution for bigamy. He cannot have his cake and eat it too. Otherwise, all that an adventurous bigamist has to do is to disregard Article 40 of the Family Code, contract a subsequent marriage and escape a bigamy charge by simply claiming that the first marriage is void and that the subsequent marriage is equally void for lack of a prior judicial declaration of nullity of the first. A party may even enter into a marriage aware of the absence of a requisite - usually the marriage license - and thereafter contract a subsequent marriage without obtaining a declaration of nullity of the first on the assumption that the first marriage is void. Such scenario would render nugatory the provisions on bigamy. As succinctly held in Landicho v. Relova: (P)arties to a marriage should not be permitted to judge for themselves its nullity, only competent courts having such authority. Prior to such declaration of nullity, the validity of the first marriage is beyond question. A party who contracts a second marriage then assumes the risk of being prosecuted for bigamy. Respondent alleges that the first marriage in the case before us was void for lack of a marriage license. Petitioner, on the other hand, argues that her marriage to respondent was exempt from the requirement of a marriage license. More specifically, petitioner claims that prior to their marriage, they had already attained the age of majority and had been living together as husband and wife for at least five years. The issue in this case is limited to the existence of a prejudicial question, and we are not called upon to resolve the validity of the first marriage. Be that as it may, suffice it to state that the Civil Code, under which the first marriage was celebrated, provides that "every intendment of law or fact leans toward the validity of marriage, the indissolubility of the marriage bonds." [] Hence, parties should not be permitted to judge for themselves the nullity of their marriage, for the same must be submitted to the determination of competent courts. Only when the nullity of the marriage is so declared can it be held as void, and so long as there is no such declaration the presumption is that the marriage exists. No matter how obvious, manifest or patent the absence of an element is, the intervention of the courts must always be resorted to. That is why Article 40 of the Family Code requires a "final judgment," which only the courts can render. Thus, as ruled in Landicho v. Relova, he who contracts a second marriage before the judicial declaration of nullity of the first marriage assumes the risk of being prosecuted for bigamy, and in such a case the criminal case may not be suspended on the ground of the pendency of a civil case for declaration of nullity. In a recent case for concubinage, we held that the pendency of a civil case for declaration of nullity of marriage is not a prejudicial question. This ruling applies here by analogy since both crimes presuppose the subsistence of a marriage. Ignorance of the existence of Article 40 of the Family Code cannot even be successfully invoked as an excuse. The contracting of a marriage knowing that the requirements of the law have not been complied with or that the marriage is in disregard of a legal impediment is an act penalized by the Revised Penal Code. The legality of a marriage is a matter of law and every person is presumed to know the law. As respondent did not obtain the judicial declaration of nullity when he entered into the second marriage, why should he be allowed to belatedly obtain that judicial declaration in order to delay his criminal prosecution and subsequently defeat it by his own disobedience of the law? If he wants to raise the nullity of the previous marriage, he can do it as a matter of defense when he presents his evidence during the trial proper in the criminal case. The burden of proof to show the dissolution of the first marriage before the second marriage was contracted rests upon the defense, but that is a matter that can be raised in the trial of the bigamy case. In the meantime, it should be stressed that not every defense raised in the civil action may be used as a prejudicial question to obtain the suspension of the criminal action. The lower court, therefore, erred in suspending the criminal case for bigamy. Moreover, when respondent was indicted for bigamy, the fact 178

University of the Cordilleras College of Law First Year C S.Y. 2013 - 2014 that he entered into two marriage ceremonies appeared indubitable. It was only after he was sued by petitioner for bigamy that he thought of seeking a judicial declaration of nullity of his first marriage. The obvious intent, therefore, is that respondent merely resorted to the civil action as a potential prejudicial question for the purpose of frustrating or delaying his criminal prosecution. As has been discussed above, this cannot be done.1awphi1 In the light of Article 40 of the Family Code, respondent, without first having obtained the judicial declaration of nullity of the first marriage, cannot be said to have validly entered into the second marriage. Per current jurisprudence, a marriage though void still needs a judicial declaration of such fact before any party can marry again; otherwise the second marriage will also be void. The reason is that, without a judicial declaration of its nullity, the first marriage is presumed to be subsisting. In the case at bar, respondent was for all legal intents and purposes regarded as a married man at the time he contracted his second marriage with petitioner. Against this legal backdrop, any decision in the civil action for nullity would not erase the fact that respondent entered into a second marriage during the subsistence of a first marriage. Thus, a decision in the civil case is not essential to the determination of the criminal charge. It is, therefore, not a prejudicial question. As stated above, respondent cannot be permitted to use his own malfeasance to defeat the criminal action against him. WHEREFORE, the petition is GRANTED. The order dated December 29, 1998 of the Regional Trial Court, Branch 226 of Quezon City is REVERSED and SET ASIDE and the trial court is ordered to IMMEDIATELY proceed with Criminal Case No. Q98-75611. SO ORDERED.

179

University of the Cordilleras College of Law First Year C S.Y. 2013 - 2014 Beltran vs People 334 SCRA 106 G.R. No. 137567 June 20, 2000 MEYNARDO L. BELTRAN, petitioner, vs. PEOPLE OF THE PHILIPPINES, and HON. JUDGE FLORENTINO TUAZON, JR., being the Judge of the RTC, Brach 139, Makati City, respondents. BUENA, J.: This petition for review, filed under Rule 45 of the 1997 Rules of Civil Procedure, seeks to review and set aside the Order dated January 28, 1999 issued by Judge Florentino A. Tuazon, Jr. of the Regional Trial Court of Makati City, Branch 139 in Special Civil Case No. 98-3056, entitled "Meynardo Beltran vs. People of the Philippines and Hon. Judge Alden Cervantes of the Metropolitan Trial Court of Makati City, Branch 61." The said Order denied petitioner's prayer for the issuance of a writ of preliminary injunction to enjoin Judge Cervantes from proceeding with the trial of Criminal Case No. 236176, a concubinage case against petitioner on the ground that the pending petition for declaration of nullity of marriage filed by petitioner against his wife constitutes a prejudicial question. The antecedent facts of the case are undisputed: Petitioner Meynardo Beltran and wife Charmaine E. Felix were married on June 16, 1973 at the Immaculate Concepcion Parish Church in Cubao, Quezon City. On February 7, 1997, after twenty-four years of marriage and four children, petitioner filed a petition for nullity of marriage on the ground of psychological incapacity under Article 36 of the Family Code before Branch 87 of the Regional Trial Court of Quezon City. The case was docketed as Civil Case No. Q-9730192. In her Answer to the said petition, petitioner's wife Charmaine Felix alleged that it was petitioner who abandoned the conjugal home and lived with a certain woman named Milagros Salting. Charmaine subsequently filed a criminal complaint for concubinage under Article 334 of the Revised Penal Code against petitioner and his paramour before the City Prosecutor's Office of Makati who, in a Resolution dated September 16, 1997, found probable cause and ordered the filing of an Information against them. The case, docketed as Criminal Case No. 236176, was filed before the Metropolitan Trial Court of Makati City, Branch 61.1awphi1 On March 20, 1998, petitioner, in order to forestall the issuance of a warrant for his arrest, filed a Motion to Defer Proceedings Including the Issuance of the Warrant of Arrest in the criminal case. Petitioner argued that the pendency of the civil case for declaration of nullity of his marriage posed a prejudicial question to the determination of the criminal case. Judge Alden Vasquez Cervantes denied the foregoing motion in the Order dated August 31, 1998. Petitioner's motion for reconsideration of the said Order of denial was likewise denied in an Order dated December 9, 1998. In view of the denial of his motion to defer the proceedings in the concubinage case, petitioner went to the Regional Trial Court of Makati City, Branch 139 on certiorari, questioning the Orders dated August 31, 1998 and December 9, 1998 issued by Judge Cervantes and praying for the issuance of a writ of preliminary injunction. In an Order dated January 28, 1999, the Regional Trial Court of Makati denied the petition for certiorari. Said Court subsequently issued another Order dated February 23, 1999, denying his motion for reconsideration of the dismissal of his petition. Undaunted, petitioner filed the instant petition for review. Petitioner contends that the pendency of the petition for declaration of nullity of his marriage based on psychological incapacity under Article 36 of the Family Code is a prejudicial question that should merit the suspension of the criminal case for concubinage filed against him by his wife.

180

University of the Cordilleras College of Law First Year C S.Y. 2013 - 2014 Petitioner also contends that there is a possibility that two conflicting decisions might result from the civil case for annulment of marriage and the criminal case for concubinage. In the civil case, the trial court might declare the marriage as valid by dismissing petitioner's complaint but in the criminal case, the trial court might acquit petitioner because the evidence shows that his marriage is void on ground of psychological incapacity. Petitioner submits that the possible conflict of the courts' ruling regarding petitioner's marriage can be avoided, if the criminal case will be suspended, until the court rules on the validity of marriage; that if petitioner's marriage is declared void by reason of psychological incapacity then by reason of the arguments submitted in the subject petition, his marriage has never existed; and that, accordingly, petitioner could not be convicted in the criminal case because he was never before a married man. Petitioner's contentions are untenable. The rationale behind the principle of prejudicial question is to avoid two conflicting decisions. It has two essential elements: (a) the civil action involves an issue similar or intimately related to the issue raised in the criminal action; and (b) the resolution of such issue determines whether or not the criminal action may proceed. The pendency of the case for declaration of nullity of petitioner's marriage is not a prejudicial question to the concubinage case. For a civil case to be considered prejudicial to a criminal action as to cause the suspension of the latter pending the final determination of the civil case, it must appear not only that the said civil case involves the same facts upon which the criminal prosecution would be based, but also that in the resolution of the issue or issues raised in the aforesaid civil action, the guilt or innocence of the accused would necessarily be determined. Art. 40 of the Family Code provides: The absolute nullity of a previous marriage may be invoked for purposes of remarriage on the basis solely of a final judgment declaring such previous marriage void. In Domingo vs. Court of Appeals, this Court ruled that the import of said provision is that for purposes of remarriage, the only legally acceptable basis for declaring a previous marriage an absolute nullity is a final judgment declaring such previous marriage void, whereas, for purposes of other than remarriage, other evidence is acceptable. The pertinent portions of said Decision read: . . . Undoubtedly, one can conceive of other instances where a party might well invoke the absolute nullity of a previous marriage for purposes other than remarriage, such as in case of an action for liquidation, partition, distribution and separation of property between the erstwhile spouses, as well as an action for the custody and support of their common children and the delivery of the latters' presumptive legitimes. In such cases, evidence needs must be adduced, testimonial or documentary, to prove the existence of grounds rendering such a previous marriage an absolute nullity. These needs not be limited solely to an earlier final judgment of a court declaring such previous marriage void. So that in a case for concubinage, the accused, like the herein petitioner need not present a final judgment declaring his marriage void for he can adduce evidence in the criminal case of the nullity of his marriage other than proof of a final judgment declaring his marriage void. With regard to petitioner's argument that he could be acquitted of the charge of concubinage should his marriage be declared null and void, suffice it to state that even a subsequent pronouncement that his marriage is void from the beginning is not a defense. Analogous to this case is that of Landicho vs. Relova cited in Donato vs. Luna where this Court held that: . . . Assuming that the first marriage was null and void on the ground alleged by petitioner, that fact would not be material to the outcome of the criminal case. Parties to the marriage should not 181

University of the Cordilleras College of Law First Year C S.Y. 2013 - 2014 be permitted to judge for themselves its nullity, for the same must be submitted to the judgment of the competent courts and only when the nullity of the marriage is so declared can it be held as void, and so long as there is no such declaration the presumption is that the marriage exists. Therefore, he who contracts a second marriage before the judicial declaration of nullity of the first marriage assumes the risk of being prosecuted for bigamy. Thus, in the case at bar it must also be held that parties to the marriage should not be permitted to judge for themselves its nullity, for the same must be submitted to the judgment of the competent courts and only when the nullity of the marriage is so declared can it be held as void, and so long as there is no such declaration the presumption is that the marriage exists for all intents and purposes. Therefore, he who cohabits with a woman not his wife before the judicial declaration of nullity of the marriage assumes the risk of being prosecuted for concubinage. The lower court therefore, has not erred in affirming the Orders of the judge of the Metropolitan Trial Court ruling that pendency of a civil action for nullity of marriage does not pose a prejudicial question in a criminal case for concubinage. WHEREFORE, for lack of merit, the instant petition is DISMISSED. SO ORDERED.

182

University of the Cordilleras College of Law First Year C S.Y. 2013 - 2014 Te vs Court of Appeals 346 SCRA 327 G.R. No. 126746 November 29, 2000 ARTHUR TE, petitioner, vs. COURT OF APPEALS, and LILIANA CHOA, respondents. KAPUNAN, J.: Before us is a petition for review on certiorari which seeks to reverse the Decision of the Court of Appeals Tenth Division, dated 31 August 1994 in CA-G.R. SP No. 23971 and CA-G.R. SP No. 261782 and the Resolution dated October 18, 1996 denying petitioners motion for reconsideration. The facts of the case are as follows: Petitioner Arthur Te and private respondent Liliana Choa were married in civil rites on September 14, 1988. They did not live together after the marriage although they would meet each other regularly. Not long after private respondent gave birth to a girl on April 21, 1989, petitioner stopped visiting her.3 On May 20, 1990, while his marriage with private respondent was subsisting, petitioner contracted a second marriage with a certain Julieta Santella (Santella).4 On the basis of a complaint-affidavit filed by private respondent sometime in June 1990, when she learned about petitioners marriage to Santella, an information charging petitioner with bigamy was filed with the Regional Trial Court (RTC) of Quezon City on August 9, 1990. 5 This case was docketed as Criminal Case No. Q-90-14409.6 Meanwhile, on July 20, 1990, petitioner filed in the RTC of Quezon City an action for the annulment of his marriage to private respondent on the ground that he was forced to marry her. He alleged that private respondent concealed her pregnancy by another man at the time of their marriage and that she was psychologically incapacitated to perform her essential marital obligations.7 On November 8, 1990, private respondent also filed with the Professional Regulation Commission (PRC) an administrative case against petitioner and Santella for the revocation of their respective engineering licenses on the ground that they committed acts of immorality by living together and subsequently marrying each other despite their knowledge that at the time of their marriage, petitioner was already married to private respondent. With respect to petitioner, private respondent added that he committed an act of falsification by stating in his marriage contract with Santella that he was still single.8 After the prosecution rested its case in the criminal case for bigamy, petitioner filed a demurrer to evidence with leave of court and motion to inhibit the trial court judge for showing antagonism and animosity towards petitioners counsel during the hearings of said case. The trial court denied petitioners demurrer to evidence in an Order dated November 28, 1990 which stated that the same could not be granted because the prosecution had sufficiently established a prima facie case against the accused.9 The RTC also denied petitioners motion to inhibit for lack of legal basis.10 Petitioner then filed with the Court of Appeals a petition for certiorari, alleging grave abuse of discretion on the part of the trial court judge, Judge Cezar C. Peralejo, for (1) exhibiting antagonism and animosity towards petitioners counsel; (2) violating the requirements of due process by denying petitioners [motion for reconsideration and] demurrer to evidence even before the filing of the same; (3) disregarding and failing to comply with the appropriate guidelines for judges promulgated by the Supreme Court; and (4) ruling that in a criminal case only "prima facie evidence" is sufficient for conviction of an accused. This case was docketed as CA-G.R. SP No. 23971.11 Petitioner also filed with the Board of Civil Engineering of the PRC (PRC Board), where the administrative case for the revocation of his engineering license was pending, a motion to suspend the 183

University of the Cordilleras College of Law First Year C S.Y. 2013 - 2014 proceedings therein in view of the pendency of the civil case for annulment of his marriage to private respondent and criminal case for bigamy in Branches 106 and 98, respectively of the RTC of Quezon City. When the Board denied the said motion in its Order dated July 16, 1991,13 petitioner filed with the Court of Appeals another petition for certiorari, contending that the Board gravely abused its discretion in: (1) failing to hold that the resolution of the annulment case is prejudicial to the outcome of the administrative case pending before it; (2) not holding that the continuation of proceedings in the administrative case could render nugatory petitioners right against self-incrimination in this criminal case for bigamy against him; and (3) making an overly-sweeping interpretation that Section 32 of the Rules and Regulations Governing the Regulation and Practice of Professionals does not allow the suspension of the administrative proceeding before the PRC Board despite the pendency of criminal and/or administrative proceedings against the same respondent involving the same set of facts in other courts or tribunals. This petition was docketed as CA-G.R. SP No. 26178.14 The two petitions for certiorari were consolidated since they arose from the same set of facts. On 31 August 1994, the Court of Appeals, Tenth Division, rendered the assailed decision in the consolidated petitions. The appellate court upheld the RTCs denial of the motion to inhibit due to petitioners failure to show any concrete evidence that the trial court judge exhibited partiality and had prejudged the case. It also ruled that the denial of petitioners moti on to suspend the proceedings on the ground of prejudicial question was in accord with law.15 The Court of Appeals likewise affirmed the RTCs denial of the demurrer to evidence filed by petitioner for his failure to set forth persuasive grounds to support the same, considering that the prosecution was able to adduce evidence showing the existence of the elements of bigamy.16 Neither did the appellate court find grave abuse of discretion on the part of the Boards Order denying petitioners motion to suspend proceedings in the administrative case on the ground of prejud icial question. Respondent court held that no prejudicial question existed since the action sought to be suspended is administrative in nature, and the other action involved is a civil case.17 Petitioner thereafter filed a motion for reconsideration of the decision of the Court of Appeals but the same was denied.18 Hence, petitioner filed the instant petition raising the following issues: I PUBLIC RESPONDENT COMMITTED A SERIOUS ERROR IN REFUSING TO SUSPEND THE LEGAL [CRIMINAL AND ADMINISTRATIVE] PROCEEDINGS DESPITE THE PENDENCY OF THE CIVIL CASE FOR DECLARATION OF NULLITY OF MARRIAGE. II PUBLIC RESPONDENT GRAVELY ABUSED ITS DISCRETION AND COMMITTED AN ERROR OF LAW IN NOT HOLDING THAT THE DEMURRER TO EVIDENCE SHOULD HAVE BEEN GIVEN DUE COURSE. III PUBLIC RESPONDENT COMMITTED A SERIOUS LEGAL ERROR IN NOT HOLDING THAT THE TRIAL JUDGE A QUO SHOULD HAVE INHIBITED HIMSELF.19 The petition has no merit. While the termination of Civil Case No. Q-90-6205 for annulment of petitioners marriage to private respondent has rendered the issue of the propriety of suspending both the criminal case for bigamy before the RTC of Quezon City, Branch 98 and the administrative case for revocation of petitioners engineering license before the PRC Board moot and academic, the Court shall discuss the issue of prejudicial question to emphasize the guarding and controlling precepts and rules.20 184

University of the Cordilleras College of Law First Year C S.Y. 2013 - 2014 A prejudicial question has been defined as one based on a fact distinct and separate from the crime but so intimately connected with it that it determines the guilt or innocence of the accused, and for it to suspend the criminal action, it must appear not only that said case involves facts intimately related to those upon which the criminal prosecution would be based but also that in the resolution of the issue or issues raised in the civil case, the guilt or innocence of the accused would necessarily be determined. 21 The rationale behind the principle of suspending a criminal case in view of a prejudicial question is to avoid two conflicting decisions.22 The Court of Appeals did not err when it ruled that the pendency of the civil case for annulment of marriage filed by petitioner against private respondent did not pose a prejudicial question which would necessitate that the criminal case for bigamy be suspended until said civil case is terminated. The outcome of the civil case for annulment of petitioners marriage to private respondent had no bearing upon the determination of petitioners innocence or guilt in the criminal case for bigamy, because all that is required for the charge of bigamy to prosper is that the first marriage be subsisting at the time the second marriage is contracted.23 Petitioners argument that the nullity of his marriage to private respondent had to be resolved first in the civil case before the criminal proceedings could continue, because a declaration that their marriage was voidab initio would necessarily absolve him from criminal liability, is untenable. The ruling in People vs. Mendoza24 andPeople vs. Aragon25 cited by petitioner that no judicial decree is necessary to establish the invalidity of a marriage which is void ab initio has been overturned. The prevailing rule is found in Article 40 of the Family Code, which was already in effect at the time of petitioners marriage to private respondent in September 1988. Said article states that the absolute nullity of a previous marriage may not be invoked for purposes of remarriage unless there is a final judgment declaring such previous marriage void. Thus, under the law, a marriage, even one which is void or voidable, shall be deemed valid until declared otherwise in a judicial proceeding. 26 In Landicho vs. Relova,27 we held that: Parties to a marriage should not be permitted to judge for themselves its nullity, for this must be submitted to the judgment of competent courts and only when the nullity of a marriage is so declared can it be held as void, and so long as there is no such declaration the presumption of marriage exists.28 It is clear from the foregoing that the pendency of the civil case for annulment of petitioners marriage to private respondent did not give rise to a prejudicial question which warranted the suspension of the proceedings in the criminal case for bigamy since at the time of the alleged commission of the crime, their marriage was, under the law, still valid and subsisting. Neither did the filing of said civil case for annulment necessitate the suspension of the administrative proceedings before the PRC Board. As discussed above, the concept of prejudicial question involves a civil and a criminal case. We have previously ruled that there is no prejudicial question where one case is administrative and the other is civil.29 Furthermore, Section 32 of the Rules and Regulations Governing the Regulation and Practice of Professionals of the PRC Board expressly provides that the administrative proceedings before it shall not be suspended notwithstanding the existence of a criminal and/or civil case against the respondent involving the same facts as the administrative case: The filing or pendency of a criminal and/or civil cases in the courts or an administrative case in another judicial body against an examinee or registered professional involving the same facts as in the administrative case filed or to be filed before the Board shall neither suspend nor bar the proceeding of the latter case. The Board shall proceed independently with the investigation of the case and shall render therein its decision without awaiting for the final decision of the courts or quasi-judicial body. It must also be noted that the allegations in the administrative complaint before the PRC Board are not confined to the issue of the alleged bigamous marriage contracted by petitioner and Santella. Petitioner is also charged with immoral conduct for continued failure to perform his obligations as husband to private respondent and as father to their child, and for cohabiting with Santella without the benefit of 185

University of the Cordilleras College of Law First Year C S.Y. 2013 - 2014 marriage.30 The existence of these other charges justified the continuation of the proceedings before the PRC Board. Petitioner also contends that the Court of Appeals erred in upholding the trial courts denial of his demurrer to evidence in the criminal case for bigamy, arguing that the prosecution failed to establish the existence of both the first and second marriages beyond reasonable doubt. Petitioner claims that the original copy of marriage contract between him and private respondent was not presented, the signatures therein were not properly identified and there was no showing that the requisites of a valid marriage were complied with. He alleges further that the original copy of the marriage contract between him and Santella was not presented, that no proof that he signed said contract was adduced, and that there was no witness presented to show that a second marriage ceremony participated in by him ever took place.31 We are not persuaded. The grant or denial of a demurrer to evidence is left to the sound discretion of the trial court, and its ruling on the matter shall not be disturbed in the absence of a grave abuse of such discretion.32 In this case, the Court of Appeals did not find any grave abuse of discretion on the part of the trial court, which based its denial of the demurrer on two grounds: first, the prosecution established a prima facie case for bigamy against the petitioner; and second, petitioners allegations in the demurrer were insufficient to justify the grant of the same. It has been held that the appellate court will not review in a special civil action for certiorari the prosecutions evidence and decide in advance that such evidence has or has not yet established the guilt of the accused beyond reasonable doubt. 33 In view of the trial courts finding that a prima facie case against petitioner exists, his proper recourse is to adduce evidence in his defense.34 The Court also finds it necessary to correct petitioners misimpression that by denying his demurrer to evidence in view of the existence of a prima facie case against him, the trial court was already making a pronouncement that he is liable for the offense charged. As correctly held by the Court of Appeals, the order of the RTC denying the demurrer was not an adjudication on the merits but merely an evaluation of the sufficiency of the prosecutions evidence to determine whether or no t a full-blown trial would be necessary to resolve the case.35 The RTCs observation that there was a prima facie case against petitioner only meant that the prosecution had presented sufficient evidence to sustain its proposition that petitioner had committed the offense of bigamy, and unless petitioner presents evidence to rebut the same, such would be the conclusion.36 Said declaration by the RTC should not be construed as a pronouncement of petitioners guilt. It was precisely because of such finding that the trial court denied the demurrer, in order that petitioner may present evidence in his defense and allow said court to resolve the case based on the evidence adduced by both parties. Lastly, petitioner contends that his motion to inhibit Judge Peralejo in Criminal Case No. Q-90-14409 should have been granted since said judge exhibited partiality and bias against him in several instances. First, when petitioner manifested that he would file a motion for reconsideration of the denial of his motion to suspend the proceedings in said case, the judge said such motion was dilatory and would be denied even though the motion for reconsideration had not yet been filed. Second, when petitioners counsel manifested that he had just recovered from an accident and was not physically fit for trial, the judge commented that counsel was merely trying to delay the case and required said counsel to produce a medical certificate to support his statement. Third, when petitioner manifested that he was going to file a demurrer to evidence, the judge characterized the same as dilatory and declared that he would deny the same. According to petitioner, the judges hostile attitude towards petitioners counsel as shown in the foregoing instances justified the grant of his motion to inhibit. We agree with the appellate court that the grounds raised by petitioner against Judge Peralejo did not conclusively show that the latter was biased and had prejudged the case.37 In People of the Philippines vs. Court of Appeals,38this Court held that while bias and prejudice have been recognized as valid reasons for the voluntary inhibition of a judge under Section 1, Rule 137, the rudimentary rule is that the mere suspicion that a judge is partial is not enough. There should be clear and convincing evidence to prove the charge of bias and partiality.39

186

University of the Cordilleras College of Law First Year C S.Y. 2013 - 2014 Furthermore, since the grounds raised by petitioner in his motion to inhibit are not among those expressly mentioned in Section 1, Rule 137 of the Revised Rules of Court, the decision to inhibit himself lay within the sound discretion of Judge Peralejo. Said provision of law states: Section 1. Disqualification of judges. No judge or judicial officer shall sit in any case in which he, or his wife or child, is pecuniarily interested as heir, legatee, creditor or otherwise, or in which he is related to either party within the sixth degree of consanguinity or affinity, or to counsel within the fourth degree, computed according to the rules of the civil law, or in which he has been executor, administrator, guardian, trustee or counsel, or in which he has presided in any inferior court when his ruling or decision is the subject of review, without the written consent of all parties in interest, signed by them and entered upon the record. A judge may, in the exercise of his sound discretion, disqualify himself from sitting in the case, for just and valid reasons other than those mentioned above. Thus, it was not mandatory that the judge inhibit himself from hearing and deciding the case. This Court does not find any abuse of discretion by respondent judge in denying petitioners motion to inhibit. The test for determining the propriety of the denial of said motion is whether petitioner was deprived a fair and impartial trial.40 The instances when Judge Peralejo allegedly exhibited antagonism and partiality against petitioner and/or his counsel did not deprive him of a fair and impartial trial. As discussed earlier, the denial by the judge of petitioners motion to suspend the criminal proceeding and the demurrer to evidence are in accord with law and jurisprudence. Neither was there anything unreasonable in the requirement that petitioners counsel submit a medical certificate to support his clai m that he suffered an accident which rendered him unprepared for trial. Such requirement was evidently imposed upon petitioners counsel to ensure that the resolution of the case was not hampered by unnecessary and unjustified delays, in keeping with the j udges duty to disposing of the courts business promptly.41 WHEREFORE, the petition is hereby DENIED for lack of merit. SO ORDERED.

187

University of the Cordilleras College of Law First Year C S.Y. 2013 - 2014

V.

Civil Personality

188

University of the Cordilleras College of Law First Year C S.Y. 2013 - 2014 Limjoco vs Interstate Estate of Fragante 80 Phil 776 G.R. No. L-770 April 27, 1948 Full Case ANGEL T. LIMJOCO, petitioner, vs. INTESTATE ESTATE OF PEDRO O. FRAGRANTE, deceased, respondent. HILADO, J.: Under date of May 21, 1946, the Public Service Commission, through Deputy Commissioner Fidel Ibaez, rendered its decision in case No. 4572 of Pedro O. Fragante, as applicant for a certificate of public convenience to install, maintain and operate an ice plant in San Juan, Rizal, whereby said commission held that the evidence therein showed that the public interest and convenience will be promoted in a proper and suitable manner "by authorizing the operation and maintenance of another ice plant of two and one-half (2-) tons in the municipality of San Juan; that the original applicant Pedro O. Fragante was a Filipino Citizen at the time of his death; and that his intestate estate is financially capable of maintaining the proposed service". The commission, therefore, overruled the opposition filed in the case and ordered "that under the provisions of section 15 of Commonwealth Act No. 146, as amended a certificate of public convenience be issued to the Intestate Estate of the deceased Pedro Fragante, authorizing said Intestate Estate through its Special or Judicial Administrator, appointed by the proper court of competent jurisdiction, to maintain and operate an ice plant with a daily productive capacity of two and one-half (21/2) tons in the Municipality of San Juan and to sell the ice produced from said plant in the said Municipality of San Juan and in the Municipality of Mandaluyong, Rizal, and in Quezon City", subject to the conditions therein set forth in detail (petitioner's brief, pp. 33-34). Petitioner makes four assignments of error in his brief as follows: 1. The decision of the Public Service Commission is not in accordance with law. 2. The decision of the Public Service Commission is not reasonably supported by evidence. 3. The Public Service Commission erred in not giving petitioner and the Ice and Cold Storage Industries of the Philippines, Inc., as existing operators, a reasonable opportunity to meet the increased demand. 4. The decision of the Public Service Commission is an unwarranted departure from its announced policy with respect to the establishment and operation of ice plant. (Pp. 1-2, petitioner's brief.) In his argument petitioner contends that it was error on the part of the commission to allow the substitution of the legal representative of the estate of Pedro O. Fragante for the latter as party applicant in the case then pending before the commission, and in subsequently granting to said estate the certificate applied for, which is said to be in contravention of law. If Pedro O. Fragante had not died, there can be no question that he would have had the right to prosecute his application before the commission to its final conclusion. No one would have denied him that right. As declared by the commission in its decision, he had invested in the ice plant in question P 35,000, and from what the commission said regarding his other properties and business, he would certainly have been financially able to maintain and operate said plant had he not died. His transportation business alone was netting him about P1,440 a month. He was a Filipino citizen and continued to be such till his demise. The commission declared in its decision, in view of the evidence before it, that his estate was financially able to maintain and operate the ice plant. The aforesaid right of Pedro O. Fragante to prosecute said application to its conclusion was one which by its nature did not lapse through his death. Hence, it constitutes a part of the assets of his estate, for which a right was property despite the possibility that in the end the commission might have denied application, although under the facts of the case, the 189

University of the Cordilleras College of Law First Year C S.Y. 2013 - 2014 commission granted the application in view of the financial ability of the estate to maintain and operate the ice plant. Petitioner, in his memorandum of March 19, 1947, admits (page 3) that the certificate of public convenience once granted "as a rule, should descend to his estate as an asset". Such certificate would certainly be property, and the right to acquire such a certificate, by complying with the requisites of the law, belonged to the decedent in his lifetime, and survived to his estate and judicial administrator after his death. If Pedro O. Fragrante had in his lifetime secured an option to buy a piece of land and during the life of the option he died, if the option had been given him in the ordinary course of business and not out of special consideration for his person, there would be no doubt that said option and the right to exercise it would have survived to his estate and legal representatives. In such a case there would also be the possibility of failure to acquire the property should he or his estate or legal representative fail to comply with the conditions of the option. In the case at bar Pedro O. Fragrante's undoubted right to apply for and acquire the desired certificate of public convenience the evidence established that the public needed the ice plant was under the law conditioned only upon the requisite citizenship and economic ability to maintain and operate the service. Of course, such right to acquire or obtain such certificate of public convenience was subject to failure to secure its objective through nonfulfillment of the legal conditions, but the situation here is no different from the legal standpoint from that of the option in the illustration just given. Rule 88, section 2, provides that the executor or administrator may bring or defend actions, among other cases, for the protection of the property or rights of the deceased which survive, and it says that such actions may be brought or defended "in the right of the deceased". Rule 82, section 1, paragraph (a), mentions among the duties of the executor or administrator, the making of an inventory of all goods, chattels, rights, credits, and estate of the deceased which shall come to his possession or knowledge, or to the possession of any other person for him. In his commentaries on the Rules of Court (Volume II, 2nd ed., pages 366, 367) the present chief Justice of this Court draws the following conclusion from the decisions cited by him: Therefore, unless otherwise expressly provided by law, any action affecting the property or rights (emphasis supplied) of a deceased person which may be brought by or against him if he were alive, may likewise be instituted and prosecuted by or against the administrator, unless the action is for recovery of money, debt or interest thereon, or unless, by its very nature, it cannot survive, because death extinguishes the right . . . . It is true that a proceeding upon the application for a certificate of public convenience before the Public Service Commission is not an "action". But the foregoing provisions and citations go to prove that the decedent's rights which by their nature are not extinguished by death go to make up a part and parcel of the assets of his estate which, being placed under the control and management of the executor or administrator, cannot be exercised but by him in representation of the estate for the benefit of the creditors, devisees or legatees, if any, and the heirs of the decedent. And if the right involved happens to consist in the prosecution of an unfinished proceeding upon an application for a certificate of public convenience of the deceased before the Public Service Commission, it is but logical that the legal representative be empowered and entitled in behalf of the estate to make the right effective in that proceeding. Manresa (Vol. III, 6th ed., p. 11) says that No. 10 of article 334 and article 336 of the Civil Code, respectively, consider as immovable and movable things rights which are not material. The same eminent commentator says in the cited volume (p. 45) that article 336 of the Civil Code has been deficiently drafted in that it is not sufficiently expressive of all incorporeal rights which are also property for juridical purposes.

190

University of the Cordilleras College of Law First Year C S.Y. 2013 - 2014 Corpus Juris (Vol. 50, p. 737) states that in the broad sense of the term, property includes, among other things, "an option", and "the certificate of the railroad commission permitting the operation of a bus line", and on page 748 of the same volume we read: However, these terms (real property, as estate or interest) have also been declared to include every species of title, inchoate or complete, and embrace rights which lie in contract, whether executory or executed. (Emphasis supplied.) Another important question raised by petitioner is whether the estate of Pedro O. Fragrante is a "person" within the meaning of the Public Service Act. Words and Phrases, First Series, (Vol. 6, p, 5325), states the following doctrine in the jurisdiction of the State of Indiana: As the estate of the decedent is in law regarded as a person, a forgery committed after the death of the man whose name purports to be signed to the instrument may be prosecuted as with the intent to defraud the estate. Billings vs. State, 107 Ind., 54, 55, 6 N. E. 914, 7 N. E. 763, 57 Am. Rep. 77. The Supreme Court of Indiana in the decision cited above had before it a case of forgery committed after the death of one Morgan for the purpose of defrauding his estate. The objection was urged that the information did not aver that the forgery was committed with the intent to defraud any person. The Court, per Elliott, J., disposed of this objection as follows: . . . The reason advanced in support of this proposition is that the law does not regard the estate of a decedent as a person. This intention (contention) cannot prevail. The estate of the decedent is a person in legal contemplation. "The word "person" says Mr. Abbot, "in its legal signification, is a generic term, and includes artificial as well as natural persons," 2 Abb. Dict. 271; Douglas vs. Pacific, etc. Co., 4 Cal. 304; Planters', etc., Bank vs. Andrews, 8 Port. (Ala.) 404. It said in another work that 'persons are of two kinds: natural and artificial. A natural person is a human being. Artificial persons include (1) a collection or succession of natural persons forming a corporation; (2) a collection of property to which the law attributes the capacity of having rights and duties. The latter class of artificial persons is recognized only to a limited extent in our law. "Examples are the estate of a bankrupt or deceased person." 2 Rapalje & L. Law Dict. 954. Our own cases inferentially recognize the correctness of the definition given by the authors from whom we have quoted, for they declare that it is sufficient, in pleading a claim against a decedent's estate, to designate the defendant as the estate of the deceased person, naming him. Ginn vs. Collins, 43 Ind. 271. Unless we accept this definition as correct, there would be a failure of justice in cases where, as here, the forgery is committed after the death of a person whose name is forged; and this is a result to be avoided if it can be done consistent with principle. We perceive no difficulty in avoiding such a result; for, to our minds, it seems reasonable that the estate of a decedent should be regarded as an artificial person. It is the creation of law for the purpose of enabling a disposition of the assets to be properly made, and, although natural persons as heirs, devises, or creditors, have an interest in the property, the artificial creature is a distinct legal entity. The interest which natural persons have in it is not complete until there has been a due administration; and one who forges the name of the decedent to an instrument purporting to be a promissory note must be regarded as having intended to defraud the estate of the decedent, and not the natural persons having diverse interests in it, since ha cannot be presumed to have known who those persons were, or what was the nature of their respective interest. The fraudulent intent is against the artificial person, the estate and not the natural persons who have direct or contingent interest in it. (107 Ind. 54, 55, 6 N.E. 914-915.) In the instant case there would also be a failure of justice unless the estate of Pedro O. Fragrante is considered a "person", for quashing of the proceedings for no other reason than his death would entail prejudicial results to his investment amounting to P35,000.00 as found by the commission, not counting the expenses and disbursements which the proceeding can be presumed to have occasioned him during his 191

University of the Cordilleras College of Law First Year C S.Y. 2013 - 2014 lifetime, let alone those defrayed by the estate thereafter. In this jurisdiction there are ample precedents to show that the estate of a deceased person is also considered as having legal personality independent of their heirs. Among the most recent cases may be mentioned that of "Estate of Mota vs. Concepcion, 56 Phil., 712, 717, wherein the principal plaintiff was the estate of the deceased Lazaro Mota, and this Court gave judgment in favor of said estate along with the other plaintiffs in these words: . . . the judgment appealed from must be affirmed so far as it holds that defendants Concepcion and Whitaker are indebted to he plaintiffs in the amount of P245,804.69 . . . . Under the regime of the Civil Code and before the enactment of the Code of Civil Procedure, the heirs of a deceased person were considered in contemplation of law as the continuation of his personality by virtue of the provision of article 661 of the first Code that the heirs succeed to all the rights and obligations of the decedent by the mere fact of his death. It was so held by this Court in Barrios vs. Dolor, 2 Phil., 44, 46. However, after the enactment of the Code of Civil Procedure, article 661 of the Civil Code was abrogated, as held in Suiliong & Co. vs. Chio-Taysan, 12 Phil., 13, 22. In that case, as well as in many others decided by this Court after the innovations introduced by the Code of Civil Procedure in the matter of estates of deceased persons, it has been the constant doctrine that it is the estate or the mass of property, rights and assets left by the decedent, instead of the heirs directly, that becomes vested and charged with his rights and obligations which survive after his demise. The heirs were formerly considered as the continuation of the decedent's personality simply by legal fiction, for they might not have been flesh and blood the reason was one in the nature of a legal exigency derived from the principle that the heirs succeeded to the rights and obligations of the decedent. Under the present legal system, such rights and obligations as survive after death have to be exercised and fulfilled only by the estate of the deceased. And if the same legal fiction were not indulged, there would be no juridical basis for the estate, represented by the executor or administrator, to exercise those rights and to fulfill those obligations of the deceased. The reason and purpose for indulging the fiction is identical and the same in both cases. This is why according to the Supreme Court of Indiana in Billings vs. State, supra, citing 2 Rapalje & L. Dictionary, 954, among the artificial persons recognized by law figures "a collection of property to which the law attributes the capacity of having rights and duties", as for instance, the estate of a bankrupt or deceased person. Petitioner raises the decisive question of whether or not the estate of Pedro O. Fragrante can be considered a "citizen of the Philippines" within the meaning of section 16 of the Public Service Act, as amended, particularly the proviso thereof expressly and categorically limiting the power of the commission to issue certificates of public convenience or certificates of public convenience and necessity "only to citizens of the Philippines or of the United States or to corporations, copartnerships, associations, or joint-stock companies constituted and organized under the laws of the Philippines", and the further proviso that sixty per centum of the stock or paid-up capital of such entities must belong entirely to citizens of the Philippines or of the United States. Within the Philosophy of the present legal system, the underlying reason for the legal fiction by which, for certain purposes, the estate of the deceased person is considered a "person" is the avoidance of injustice or prejudice resulting from the impossibility of exercising such legal rights and fulfilling such legal obligations of the decedent as survived after his death unless the fiction is indulged. Substantially the same reason is assigned to support the same rule in the jurisdiction of the State of Indiana, as announced in Billings vs. State, supra, when the Supreme Court of said State said: . . . It seems reasonable that the estate of a decedent should be regarded as an artificial person. it is the creation of law for the purpose of enabling a disposition of the assets to be properly made . . .. Within the framework and principles of the constitution itself, to cite just one example, under the bill of rights it seems clear that while the civil rights guaranteed therein in the majority of cases relate to natural persons, the term "person" used in section 1 (1) and (2) must be deemed to include artificial or juridical persons, for otherwise these latter would be without the constitutional guarantee against being deprived of 192

University of the Cordilleras College of Law First Year C S.Y. 2013 - 2014 property without due process of law, or the immunity from unreasonable searches and seizures. We take it that it was the intendment of the framers to include artificial or juridical, no less than natural, persons in these constitutional immunities and in others of similar nature. Among these artificial or juridical persons figure estates of deceased persons. Hence, we hold that within the framework of the Constitution, the estate of Pedro O. Fragrante should be considered an artificial or juridical person for the purposes of the settlement and distribution of his estate which, of course, include the exercise during the judicial administration thereof of those rights and the fulfillment of those obligations of his which survived after his death. One of those rights was the one involved in his pending application before the Public Service Commission in the instant case, consisting in the prosecution of said application to its final conclusion. As stated above, an injustice would ensue from the opposite course. How about the point of citizenship? If by legal fiction his personality is considered extended so that any debts or obligations left by, and surviving, him may be paid, and any surviving rights may be exercised for the benefit of his creditors and heirs, respectively, we find no sound and cogent reason for denying the application of the same fiction to his citizenship, and for not considering it as likewise extended for the purposes of the aforesaid unfinished proceeding before the Public Service Commission. The outcome of said proceeding, if successful, would in the end inure to the benefit of the same creditors and the heirs. Even in that event petitioner could not allege any prejudice in the legal sense, any more than he could have done if Fragrante had lived longer and obtained the desired certificate. The fiction of such extension of his citizenship is grounded upon the same principle, and motivated by the same reason, as the fiction of the extension of personality. The fiction is made necessary to avoid the injustice of subjecting his estate, creditors and heirs, solely by reason of his death to the loss of the investment amounting to P35,000, which he has already made in the ice plant, not counting the other expenses occasioned by the instant proceeding, from the Public Service Commission of this Court. We can perceive no valid reason for holding that within the intent of the constitution (Article IV), its provisions on Philippine citizenship exclude the legal principle of extension above adverted to. If for reasons already stated our law indulges the fiction of extension of personality, if for such reasons the estate of Pedro O. Fragrante should be considered an artificial or juridical person herein, we can find no justification for refusing to declare a like fiction as to the extension of his citizenship for the purposes of this proceeding. Pedro O. Fragrante was a Filipino citizen, and as such, if he had lived, in view of the evidence of record, he would have obtained from the commission the certificate for which he was applying. The situation has suffered but one change, and that is, his death. His estate was that of a Filipino citizen. And its economic ability to appropriately and adequately operate and maintain the service of an ice plant was the same that it received from the decedent himself. In the absence of a contrary showing, which does not exist here, his heirs may be assumed to be also Filipino citizens; and if they are not, there is the simple expedient of revoking the certificate or enjoining them from inheriting it. Upon the whole, we are of the opinion that for the purposes of the prosecution of said case No. 4572 of the Public Service Commission to its final conclusion, both the personality and citizenship of Pedro O. Fragrante must be deemed extended, within the meaning and intent of the Public Service Act, as amended, in harmony with the constitution: it is so adjudged and decreed. Decision affirmed, without costs. So ordered.

193

University of the Cordilleras College of Law First Year C S.Y. 2013 - 2014 Geluz vs Court of Appeals 2 SCRA 132 G.R. No. L-16439 July 20, 1961 Full Case ANTONIO GELUZ, petitioner, vs. THE HON. COURT OF APPEALS and OSCAR LAZO, respondents. REYES, J.B.L., J.: This petition for certiorari brings up for review question whether the husband of a woman, who voluntarily procured her abortion, could recover damages from physician who caused the same. The litigation was commenced in the Court of First Instance of Manila by respondent Oscar Lazo, the of Nita Villanueva, against petitioner Antonio Geluz, a physician. Convinced of the merits of the complaint upon the evidence adduced, the trial court rendered judgment favor of plaintiff Lazo and against defendant Geluz, ordering the latter to pay P3,000.00 as damages, P700.00 attorney's fees and the costs of the suit. On appeal, Court of Appeals, in a special division of five, sustained the award by a majority vote of three justices as against two, who rendered a separate dissenting opinion. The facts are set forth in the majority opinion as follows: Nita Villanueva came to know the defendant (Antonio Geluz) for the first time in 1948 through her aunt Paula Yambot. In 1950 she became pregnant by her present husband before they were legally married. Desiring to conceal her pregnancy from her parent, and acting on the advice of her aunt, she had herself aborted by the defendant. After her marriage with the plaintiff, she again became pregnant. As she was then employed in the Commission on Elections and her pregnancy proved to be inconvenient, she had herself aborted again by the defendant in October 1953. Less than two years later, she again became pregnant. On February 21, 1955, accompanied by her sister Purificacion and the latter's daughter Lucida, she again repaired to the defendant's clinic on Carriedo and P. Gomez streets in Manila, where the three met the defendant and his wife. Nita was again aborted, of a two-month old fetus, in consideration of the sum of fifty pesos, Philippine currency. The plaintiff was at this time in theprovince of Cagayan, campaigning for his election to the provincial board; he did not know of, nor gave his consent, to the abortion. It is the third and last abortion that constitutes plaintiff's basis in filing this action and award of damages. Upon application of the defendant Geluz we granted certiorari. The Court of Appeals and the trial court predicated the award of damages in the sum of P3,000.06 upon the provisions of the initial paragraph of Article 2206 of the Civil Code of the Philippines. This we believe to be error, for the said article, in fixing a minimum award of P3,000.00 for the death of a person, does not cover the case of an unborn foetus that is not endowed with personality. Under the system of our Civil Code, "la criatura abortiva no alcanza la categoria de persona natural y en consscuencia es un ser no nacido a la vida del Derecho" (Casso-Cervera, "Diccionario de Derecho Privado", Vol. 1, p. 49), being incapable of having rights and obligations. Since an action for pecuniary damages on account of personal injury or death pertains primarily to the one injured, it is easy to see that if no action for such damages could be instituted on behalf of the unborn child on account of the injuries it received, no such right of action could derivatively accrue to its parents or heirs. In fact, even if a cause of action did accrue on behalf of the unborn child, the same was extinguished by its pre-natal death, since no transmission to anyone can take place from on that lacked juridical personality (or juridical capacity as distinguished from capacity to act). It is no answer to invoke the provisional personality of a conceived child (conceptus pro nato habetur) under Article 40 of the Civil 194

University of the Cordilleras College of Law First Year C S.Y. 2013 - 2014 Code, because that same article expressly limits such provisional personality by imposing the condition that the child should be subsequently born alive: "provided it be born later with the condition specified in the following article". In the present case, there is no dispute that the child was dead when separated from its mother's womb. The prevailing American jurisprudence is to the same effect; and it is generally held that recovery cannot had for the death of an unborn child (Stafford vs. Roadway Transit Co., 70 F. Supp. 555; Dietrich vs. Northampton, 52 Am. Rep. 242; and numerous cases collated in the editorial note, 10 ALR, (2d) 639. This is not to say that the parents are not entitled to collect any damages at all. But such damages must be those inflicted directly upon them, as distinguished from the injury or violation of the rights of the deceased, his right to life and physical integrity. Because the parents cannot expect either help, support or services from an unborn child, they would normally be limited to moral damages for the illegal arrest of the normal development of the spes hominis that was the foetus, i.e., on account of distress and anguish attendant to its loss, and the disappointment of their parental expectations (Civ. Code Art. 2217), as well as to exemplary damages, if the circumstances should warrant them (Art. 2230). But in the case before us, both the trial court and the Court of Appeals have not found any basis for an award of moral damages, evidently because the appellee's indifference to the previous abortions of his wife, also caused by the appellant herein, clearly indicates that he was unconcerned with the frustration of his parental hopes and affections. The lower court expressly found, and the majority opinion of the Court of Appeals did not contradict it, that the appellee was aware of the second abortion; and the probabilities are that he was likewise aware of the first. Yet despite the suspicious repetition of the event, he appeared to have taken no steps to investigate or pinpoint the causes thereof, and secure the punishment of the responsible practitioner. Even after learning of the third abortion, the appellee does not seem to have taken interest in the administrative and criminal cases against the appellant. His only concern appears to have been directed at obtaining from the doctor a large money payment, since he sued for P50,000.00 damages and P3,000.00 attorney's fees, an "indemnity" claim that, under the circumstances of record, was clearly exaggerated. The dissenting Justices of the Court of Appeals have aptly remarked that: It seems to us that the normal reaction of a husband who righteously feels outraged by the abortion which his wife has deliberately sought at the hands of a physician would be highminded rather than mercenary; and that his primary concern would be to see to it that the medical profession was purged of an unworthy member rather than turn his wife's indiscretion to personal profit, and with that idea in mind to press either the administrative or the criminal cases he had filed, or both, instead of abandoning them in favor of a civil action for damages of which not only he, but also his wife, would be the beneficiaries. It is unquestionable that the appellant's act in provoking the abortion of appellee's wife, without medical necessity to warrant it, was a criminal and morally reprehensible act, that cannot be too severely condemned; and the consent of the woman or that of her husband does not excuse it. But the immorality or illegality of the act does not justify an award of damage that, under the circumstances on record, have no factual or legal basis. The decision appealed from is reversed, and the complaint ordered dismissed. Without costs. Let a copy of this decision be furnished to the Department of Justice and the Board of Medical Examiners for their information and such investigation and action against the appellee Antonio Geluz as the facts may warrant.

195

University of the Cordilleras College of Law First Year C S.Y. 2013 - 2014 QUIMIGING VS. ICAO, 34 SCRA 132 G.R. No. 26795 July 31, 1970 Full Case CARMEN QUIMIGUING, Suing through her parents, ANTONIO QUIMIGUING and JACOBA CABILIN, plaintiffs-appellants, vs. FELIX ICAO, defendant-appellee. REYES, J.B.L., J.: Appeal on points of law from an order of the Court of First Instance of Zamboanga del Norte (Judge Onofre Sison Abalos, presiding), in its Civil Case No. 1590, dismissing a complaint for support and damages, and another order denying amendment of the same pleading. The events in the court of origin can be summarized as follows: Appellant, Carmen Quimiguing, assisted by her parents, sued Felix Icao in the court below. In her complaint it was averred that the parties were neighbors in Dapitan City, and had close and confidential relations; that defendant Icao, although married, succeeded in having carnal intercourse with plaintiff several times by force and intimidation, and without her consent; that as a result she became pregnant, despite efforts and drugs supplied by defendant, and plaintiff had to stop studying. Hence, she claimed support at P120.00 per month, damages and attorney's fees. Duly summoned, defendant Icao moved to dismiss for lack of cause of action since the complaint did not allege that the child had been born; and after hearing arguments, the trial judge sustained defendant's motion and dismissed the complaint. Thereafter, plaintiff moved to amend the complaint to allege that as a result of the intercourse, plaintiff had later given birth to a baby girl; but the court, sustaining defendant's objection, ruled that no amendment was allowable, since the original complaint averred no cause of action. Wherefore, the plaintiff appealed directly to this Court. We find the appealed orders of the court below to be untenable. A conceived child, although as yet unborn, is given by law a provisional personality of its own for all purposes favorable to it, as explicitly provided in Article 40 of the Civil Code of the Philippines. The unborn child, therefore, has a right to support from its progenitors, particularly of the defendant-appellee (whose paternity is deemed admitted for the purpose of the motion to dismiss), even if the said child is only "en ventre de sa mere;" just as a conceived child, even if as yet unborn, may receive donations as prescribed by Article 742 of the same Code, and its being ignored by the parent in his testament may result in preterition of a forced heir that annuls the institution of the testamentary heir, even if such child should be born after the death of the testator Article 854, Civil Code). ART. 742. Donations made to conceived and unborn children may be accepted by those persons who would legally represent them if they were already born. ART. 854. The preterition or omission of one, some, or all of the compulsory heirs in the direct line, whether living at the time of the execution of the will or born after the death of the testator, shall annul the institution of heir; but the devises and legacies shall be valid insofar as they are not inofficious. If the omitted compulsory heirs should die before the testator, the institution shall be effectual, without prejudice to the right of 'representation.

196

University of the Cordilleras College of Law First Year C S.Y. 2013 - 2014 It is thus clear that the lower court's theory that Article 291 of the Civil Code declaring that support is an obligation of parents and illegitimate children "does not contemplate support to children as yet unborn," violates Article 40 aforesaid, besides imposing a condition that nowhere appears in the text of Article 291. It is true that Article 40 prescribing that "the conceived child shall be considered born for all purposes that are favorable to it" adds further "provided it be born later with the conditions specified in the following article" (i.e., that the foetus be alive at the time it is completely delivered from the mother's womb). This proviso, however, is not a condition precedent to the right of the conceived child; for if it were, the first part of Article 40 would become entirely useless and ineffective. Manresa, in his Commentaries (5th Ed.) to the corresponding Article 29 of the Spanish Civil Code, clearly points this out: Los derechos atribuidos al nasciturus no son simples expectativas, ni aun en el sentido tecnico que la moderna doctrina da a esta figura juridica sino que constituyen un caso de los propiamente Ilamados 'derechos en estado de pendenci'; el nacimiento del sujeto en las condiciones previstas por el art. 30, no determina el nacimiento de aquellos derechos (que ya existian de antemano), sino que se trata de un hecho que tiene efectos declarativos. (1 Manresa, Op. cit., page 271) A second reason for reversing the orders appealed from is that for a married man to force a woman not his wife to yield to his lust (as averred in the original complaint in this case) constitutes a clear violation of the rights of his victim that entitles her to claim compensation for the damage caused. Says Article 21 of the Civil Code of the Philippines: ART. 21. Any person who wilfully causes loss or injury to another in a manner that is contrary to morals, good customs or public policy shall compensate the latter for the damage. The rule of Article 21 is supported by Article 2219 of the same Code: ART 2219. Moral damages may be recovered in the following and analogous cases: (3) Seduction, abduction, rape or other lascivious acts: xxx xxx xxx

(10) Acts and actions referred to in Articles 21, 26, 27, 28 .... Thus, independently of the right to Support of the child she was carrying, plaintiff herself had a cause of action for damages under the terms of the complaint; and the order dismissing it for failure to state a cause of action was doubly in error. WHEREFORE, the orders under appeal are reversed and set aside. Let the case be remanded to the court of origin for further proceedings conformable to this decision. Costs against appellee Felix Icao. So ordered.

197

University of the Cordilleras College of Law First Year C S.Y. 2013 - 2014 Case Digest Quimguing vs Icao 34 SCRA 132 G.R. No. 26795 Decided On: July 31, 1970 Ponente: REYES, J.B.L., J.:

Facts: Carmen Quimiguing, suing through her parents, Antonio and Jacoba Cabilin, sought an appeal from the orders of Zamboanga CFI, which dismissed her complaint for support and damages and request for amendment of complaint. Quimiguing averred that the then already married Felix Icao succeeded in having sexual relations with her through force and intimidation. As a result, she became pregnant despite efforts and drugs supplied by Icao and had to stop studying. She then claimed for monthly support, damages and attorneys fees. The defendant-appellee, however, moved to dismiss in light of Quimiguings failure to allege the fact that a child had been born in her complaint. The lower court dismissed the case and subsequently denied further amendment to the complaint, ruling that no amendment was allowed for failure of the original complaint to state a cause of action. Issue: Whether or not the plaintiff-appellants can ask for support and damages from defendant despite failure to allege fact of birth in complaint. Ruling: Yes. The Court ruled that plaintiff-appellant had right to support of the child she was carrying and an independent cause of action for damages. This is because the Civil Code (Art. 40) recognizes the provisional personality of the unborn child, which includes its right to support from its progenitors, even it is only en ventre de sa mere. Article 742 of the same Code holds that, just as a conceived child, it may receive donations through persons that legally represent it. Readings of Articles 40, 854 of the Civil Code and Article 29 of the Spanish Code also further strengthen the case for reversal of order. Additionally, for a married man to force a woman not his wife to yield to his lust xxx constitutes a clear violation of the rights of his victim that entitles her to claim compensation for damage caused per Article 21 of the Civil Code, a provision supported by Article 2219, which provides moral damages for victims of seduction, abduction, rape or other lascivious acts. Judgment reversed, set aside and remanded for proceedings conformable to the decision; with costs against Icao.

198

University of the Cordilleras College of Law First Year C S.Y. 2013 - 2014 Joaquin vs Navarro 93 SCRA 257 G.R. No. L-5426 May 29, 1953 Full Case RAMON JOAQUIN, petitioner, vs. ANTONIO C. NAVARRO, respondent. TUASON, J.: This three proceedings was instituted in the Court of First Instance of Manila in the summary settlement of states of Joaquin Navarro, Sr., his wife Angela Joaquin de Navarro, Joaquin Navarro, Jr., and Pilar Navarro, deceased. All of them having been heard jointly, Judge Rafael Amparo handed down a single decision which was appealed to the Court of Appeals, whose decision, modifying that the Court of First Instance, in turn was elevated to the Supreme Court for review. The main question represented in the first two courts related to the sequence of the deaths of Joaquin Navarro, Sr., his wife, and their children, all of whom were killed in the massacre of civilians by Japanese troops in Manila in February 1945. The trial court found the deaths of this persons to have accurred in this order: 1st. The Navarro girls, named Pilar, Concepcion and Natividad; 2nd. Joaquin Navarro, Jr.; 3rd. Angela Joaquin de Navarro, and 4th, Joaquin Navarro, Sr. The Court of Appeals concurred with the trial court except that, with regard to Angela Joaquin de Navarro and Joaquin Navarro, Jr., the latter was declared to have survived his mother. It is this modification of the lower court's finding which is now being contested by the petitioner. The importance of the question whether Angela Joaquin de Navarro died before Joaquin Navarro, Jr., or vice versa, lies in the fact that it radically affects the rights of succession of Ramon Joaquin, the present petitioner who was an acknowledged natural child of Angela Joaquin and adopted child of the deceased spouses, and Antonio C. Navarro, respondent, son of Joaquin Navarro, Sr. by first marriage. The facts, which is not disputed, are outlined in the statement in the decision of the Court of Appeals as follows: "On February 6, 1945, while the battle for the liberation of Manila was raging, the spouses Joaquin Navarro, Sr. and Angela Joaquin, together with their three daughters, Pilar, Concepcion, and Natividad, and their son Joaquin Navarro, Jr., and the latter's wife, Adela Conde, sought refuge in the ground floor of the building known as the German Club, at the corner of San Marcelino and San Luis Streets of this City. During their stay, the building was packed with refugees, shells were exploding around, and the Club was set on fire. Simultaneously, the Japanese started shooting at the people inside the building, especially those who were trying to escape. The three daughters were hit and fell of the ground near the entrance; and Joaquin Navarro, Sr., and his son decided to abandon the premises to seek a safer heaven. They could not convince Angela Joaquin who refused to join them; and son Joaquin Navarro, Sr., his son, Joaquin Navarro, Jr., and the latter's wife, Angela Conde, and a friend and former neighbor, Francisco Lopez, dashed out of the burning edifice. As they came out, Joaquin Navarro, Jr. was shot in the head by a Japanese soldier and immediately dropped. The others lay flat on the ground in front of the Club premises to avoid the bullets. Minutes later, the German Club, already on fire, collapsed, trapping many people inside, presumably including Angela Joaquin. "Joaquin Navarro, Sr., Mrs. Joaquin Navarro, Jr., and Francisco Lopez managed to reach an air raid shelter nearby, the stayed there about three days, until February 10, 1915, when they were forced to leave the shelter be- cause the shelling tore it open. They fled toward the St. Theresa Academy in San Marcelino Street, but unfortunately met Japanese Patrols, who fired at the refugees, killing Joaquin Navarro, Sr., and his daughter-in-law.

199

University of the Cordilleras College of Law First Year C S.Y. 2013 - 2014 "At the time of the masaccre, Joaquin Navarro, Sr. was aged 70; his wife Angela Joaquin was about 67 years old; Joaquin Navarro, Jr., about 30; Pilar Navarro was two or three years older than her brother; while the other sisters, Concepcion and Natividad Navarro y Joaquin, were between 23 and 25." The Court of Appeals' finding were all taken from the testimony of Francisco Lopez, who miraculously survived the holocaust, and upon them the Court of Appeals opined that, "as between the mother Angela Joaquin and the son Joaquin Navarro, Jr., the evidence of the survivorship is uncertain and insufficient" and the statutory presumption must be applied. The appellate Court's reasoning for its conclusion is thus stated: "It does not require argument to show that survivorship cannot be established by proof of the death of only one of the parties; but that there must be adequate proof that one was alive when the other had already died. Now in this case before us, the testimony of the sole witness Lopez is to the effect that Joaquin Navarro, Jr. was shot and died shortly after the living the German Club in the company of his father and the witness, and that the burning edified entirely collapsed minutes after the shooting of the son; but there is not a scintilla of evidence, direct or circumstantial, from which we may infer the condition of the mother, Angela Joaquin, during the appreciable interval from the instant his son turned his back to her, to dash out to the Club, until he died. All we can glean from the evidence is that Angela Joaquin was unhurt when her son left her to escape from the German Club; but she could have died almost immediately after, from a variety of causes. She might have been shot by the Japanese, like her daughters, killed by falling beams from the burning edifice, overcome by the fumes, or fatally struck by splinters from the exploding shells. We cannot say for certain. No evidence is available on the point. All we can decide is that no one saw her alive after her son left her aside, and that there is no proof when she died. Clearly, this circumstance alone cannot support a finding that she died later than her son, and we are thus compelled to fall back upon the statutory presumption. Indeed, it could be said that the purpose of the presumption of survivorship would be precisely to afford a solution to uncertainties like these. Hence the son Joaquin Navarro, Jr. aged 30, must be deemed to have survived his mother, Angela Joaquin, who was admittedly above 60 years of age (Rule 123, sec. 69, subsec. (ii), Rules of Court). "The total lack of evidence on how Angela Joaquin died likewise disposes of the question whether she and her deceased children perished in the same calamity. There being no evidence to the contrary, the only guide is the occasion of the deaths, which is identical for all of them; that battle for the liberation of Manila. A second reason is that the law, in declaring that those fallen in the same battle are to be regarded as perishing in the same calamity, could not overlooked that a variety of cause of death can ( and usually do) operate in the source of combats. During the same battle, some may die from wounds, other from gages, fire, or drowning. It is clear that the law disregards episodic details, and treats the battle as an overall cause of death in applying the presumption of survivorship. "We are thus led the conclusion that the order in which the members of the Navarro-Joaquin family met their end is as follows: first, the three daughters Pilar, Concepcion, and Natividad; then the mother Angela Joaquin; then the son Joaquin Navarro, Jr., and days later (of which there is no doubt), the father Joaquin Navarro, Sr." Much space in the briefs is taken in a discussion of whether section 334(37) of Act No. 129, now section 69 (ii) of Rule 123 of the Rules of Court, has repealed article 33 of the civil code of 1889, now article 43 of the New Civil Code. It is the contention of the petitioner that it did not, and that on the assumption that there is total lack of evidence, as the Court of Appeals said, then Angela Joaquin and Joaquin Navarro, Jr. should, under article 33, be held to have died at the same time. The point is not of much if any relevancy and will be left open for the consideration when obsolute necessity there for arises. We say irrelevant because our opinion is that neither of the two provisions is applicable for the reasons to be presently set forth. Rule 123, section 69 (ii) of the Revised Rules of Court, reads:

200

University of the Cordilleras College of Law First Year C S.Y. 2013 - 2014 When two person perish in the same calamity, such as wreck, battle or conflagration, and it is not (1) shown who died first, and there are no (2) particular circumstances from when it can be inferred, the survivorship is presumed from the probabilities resulting from the strength and ages of the sexes, according to the following rules: xxx xxx xxx

Article 33 of the Civil Code of 1889 of the following tenor: Whenever a doubt arises as to which was the first to die to the two or more persons who would inherent one from the other, the persons who alleges the prior death of either must prove the allegation; in the absence of proof the presumption shall be that they died at the same time, and no transmission of rights from one to the other shall take place. Most provisions, as their language plainly implies, are intended as a substitute for lacks and so are not to be available when there are facts. With particular reference to section 69 (ii) of Rule 123, "the situation which it present is one in which the facts are not only unknown but unknowable. By hypothesis, there is no specific evidence as to the time of death . . . ." . . . it is assumed that no evidence can be produced. . . . Since the facts are unknown and unknowable, the law may apply the law of fairness appropriate to the different legal situation that arises." (IX Wigmore on Evidence, 1940 ed., 483.) In In re Wallace's Estate, 220 Pac. 683, which the Court of Appeals cited the applied with the respect to the deaths of the Navarro girls, pointing out that "our rule is taken from the Fourth Division of sec. 1936 of the California Code of Civil Procedure," the Supreme Court of California said: When the statue speaks of "particular circumstances from which it can be inferred" that one died before the other it means that there are circumstances from which the fact of death by one before the other may be inferred as a relation conclusion from the facts proven. The statue does not mean circumstances which would show, or which would tend to show, probably that one died before the other. Grand Lodge A.O.W.W.vs. Miller, 8 Cal. App. 28, 96 Pac. 22. When by circumstantial evidence alone, a party seeks to prove a survivorship contrary to the statutory presumption, the circumstances by which it is sought to prove the survivorship must be such as are competent and sufficient when tested by the general rules of evidence in civil cases. The inference of survivorship cannot rest upon mere surmise, speculation, or conjecture. As was said in Grand Lodge vs. Miller, supra, "if the matter is left to probably, then the statue of the presumption." It is manifest from the language of section 69 (ii) of Rule 123 and of that of the foregoing decision that the evidence of the survivorship need not be direct; it may be indirect, circumstantial, or inferential. Where there are facts, known or knowable, from which a rational conclusion can be made, the presumption does not step in, and the rule of preponderance of evidence controls. Are there particular circumstances on record from which reasonable inference of survivorship between Angela Joaquin and her son can be drawn? Is Francisco Lopez' testimony competent and sufficient for this purpose? For a better appreciation of this issue, it is convenient and necessary to detail the testimony, which was described by the trial court as "disinterested and trustworthy" and by the Court of Appeals as "entitled to credence." Lopez testified: Q. You said you were also heat at that time as you leave the German Club with Joaquin Navarro, Sr., Joaquin Navarro, Jr. and the latter's wife?- A. Yes, sir. Q. Did you fall? A. I fell down. Q. And you said you fell down close to Joaquin Navarro, Jr.? A. Yes, sir. Q. When the German Club collapsed where were you? A. We were out 15 meters away from the building but I could see what was going on. 201

University of the Cordilleras College of Law First Year C S.Y. 2013 - 2014 xxx xxx xxx Q. Could there have been an interval of fifteen minutes between the two events, that is the shooting of Joaquin Navarro, Jr. and the collapse of the German Club? A. Yes, sir, I could not say exactly, Occasions like that, you know, you are confused. Q. Could there (have) been an interval of an hour instead of fifteen minutes? A. Possible, but not probable. Q. Could it have been 40 minutes? A. Yes, sir, about 40 minutes. xxx xxx xxx

Q. You also know that Angela Joaquin is already dead? A. Yes, sir. Q. Can you tell the Honorable Court when did Angela Joaquin die? A. Well, a few minutes after we have dashed out, the German Club, which was burning, collapsed over them, including Mrs. Joaquin Navarro, Sr. xxx xxx xxx

Q. From your testimony it would appear that while you can give positive evidence to the fact that Pilar, Concepcion and Natividad Navarro, and Joaquin Navarro, Jr. died, you can not give the same positive evidence to the fact that Angela Joaquin also died? A. Yes, sir, in the sense that I did not see her actually die, but when the building collapsed over her I saw and I am positive and I did not see her come out of that building so I presumed she died there. xxx xxx xxx

Q. Why did you have to dash out of the German Club, you, Mr. Joaquin Navarro, Sr. and Mr. Joaquin Navarro Jr. and the latter's wife? A. Because the Japanese had set fire to the Club and they were shooting people outside, so we thought of running away rather than be roasted. xxx xxx xxx

Q. You mean to say that before you jumped out of the German Club all the Navarro girls, Pilar, Concepcion, and Natividad, were already wounded? A. to my knowledge, yes. Q. They were wounded? A. Yes, sir. Q. Were they lying on the ground or not? A. On the ground near the entrance, because most of the people who were shot by the Japanese were those who were trying to escape, and as far as I can remember they were among those killed. xxx xxx xxx

Q. So you noticed that they were killed or shot by the Japanese a few minutes before you left the place? A. That is what I think, because those Japanese soldiers were shooting the people inside especially those trying to escape. xxx xxx xxx

Q. And none of them was not except the girls, is that what you mean? A . There were many people shot because they were trying to escape. xxx xxx xxx

Q. How come that these girls were shot when they were inside the building, can you explain that? A. They were trying to escape probably.

202

University of the Cordilleras College of Law First Year C S.Y. 2013 - 2014 It is our opinion that the preceding testimony contains facts quite adequate to solve the problem of survivorship between Angela Joaquin and Joaquin Navarro, Jr. and keep the statutory presumption out of the case. It is believed that in the light of the conditions painted by Lopez, a fair and reasonable inference can be arrived at, namely: that Joaquin Navarro, Jr. died before his mother. While the possibility that the mother died before the son cannot be ruled out, it must be noted that this possibility is entirely speculative and must yield to the more rational deduction from proven facts that it was the other way around. Joaquin Navarro, Jr., it will be recalled, was killed, while running, in front of, and 15 meters from, the German Club. Still in the prime of life, 30, he must have negotiated that distance in five seconds or less, and so died within that interval from the time he dashed out of the building. Now, when Joaquin Navarro, Jr. with his father and wife started to flee from the clubhouse, the old lady was alive and unhurt, so much so that the Navarro father and son tried hard to have her come along. She could have perished within those five or fewer seconds, as stated, but the probabilities that she did seem very remote. True, people in the building were also killed but these, according to Lopez, were mostly refugees who had tried to slip away from it and were shot by Japanese troops. It was not very likely that Mrs. Joaquin Navarro, Sr. made an attempt to escape. She even made frantic efforts to dissuade her husband and son from leaving the place and exposing themselves to gun fire. This determination of Mrs. Angela Joaquin to stay where she was may well give an idea, at the same time, of a condition of relative safety in the clubhouse at the moment her husband, son, and daughter-in-law left her. It strongly tends to prove that, as the situation looked to her, the perils of death from staying were not so imminent. And it lends credence to Mr. Lopez' statement that the collapse of the clubhouse occurred about 40 minutes after Joaquin Navarro the son was shot in the head and dropped dead, and that it was the collapse that killed Mrs. Angela Navarro. The Court of Appeals said the interval between Joaquin Navarro's death and the breaking down of the edifice was "minutes". Even so, it was much longer than five seconds, long enough to warrant the inference that Mrs. Angela Joaquin was still alive when her son expired The Court of Appeals mentioned several causes, besides the collapse of the building, by which Mrs. Navarro could have been killed. All these are speculative , and the probabilities, in the light of the known facts, are against them. Dreading Japanese sharpshooters outside as evidenced by her refusal to follow the only remaining living members of her family, she could not have kept away from protective walls. Besides, the building had been set on fire trap the refugees inside, and there was no necessity for the Japanese to was their ammunition except upon those who tried to leave the premises. Nor was Angela Joaquin likely to have been killed by falling beams because the building was made of concrete and its collapse, more likely than not, was sudden. As to fumes, these do not cause instantaneous death; certainly not within the brief space of five seconds between her son's departure and his death. It will be said that all this is indulging in inferences that are not conclusive. Section 69(ii) of Rule 123 does not require that the inference necessary to exclude the presumption therein provided be certain. It is the "particular circumstances from which it (survivorship) can be inferred" that are required to be certain as tested by the rules of evidence. In speaking of inference the rule cannot mean beyond doubt, for "inference is never certainty, but if may be plain enough to justify a finding of fact." (In re Bohenko's Estate, 4 N.Y.S. 2nd. 427, citing Tortora vs. State of New York, 269 N.Y. 199 N.E. 44; Hart vs. Hudson River Bridge Co., 80 N.Y.). 622.) As the California courts have said, it is enough that "the circumstances by which it is sought to prove the survivorship must be such as are competent and sufficient when tested by the general rules of evidence in civil cases." (In re Wallace's Estate, supra.) "Juries must often reason," says one author, "according to probabilities, drawing an inference that the main fact in issue existed from collateral facts not directly proving, but strongly tending to prove, its existence. The vital question in such cases is the cogency of the proof afforded by the secondary facts. How likely, according to experience, is the existence of the primary fact if certain secondary facts exist?" (1 Moore on Facts, Sec. 596.) The same author tells us of a case where "a jury was justified in drawing the inference that the person who was caught firing a shot at an animal trespassing on his land was the person who fired a shot about an hour before at the same animal also trespassing." That conclusion was not airtight, but rational. In fact, the

203

University of the Cordilleras College of Law First Year C S.Y. 2013 - 2014 circumstances in the illustration leave greater room for another possibility than do the facts of the case at hand. In conclusion the presumption that Angela Joaquin de Navarro died before her son is based purely on surmises, speculations, or conjectures without any sure foundation in the evidence. the opposite theory that the mother outlived her son is deduced from established facts which, weighed by common experience, engender the inference as a very strong probability. Gauged by the doctrine of preponderance of evidence by, which civil cases are decided, this inference ought to prevail. It cannot be defeated as in an instance, cited by Lord Chief Justice Kenyon, "bordering on the ridiculous, where in an action on the game laws it was suggested that the gun with which the defendant fired was not charged with shot, but that the bird might have died in consequence of the fright." (1 Moore on Facts, 63, citing Wilkinson vs. Payne, 4 T. R. 468.) It is said that part of the decision of the Court of Appeals which the appellant impugns, and which has been discussed, involves findings of fact which cannot be disturbed. The point is not, in our judgment, well considered. The particular circumstances from which the parties and the Court of Appeals drew conclusions are, as above seen, undisputed, and this being the case, the correctness or incorrectness of those conclusions raises a question of law, not of fact, which the Supreme Court has jurisdiction to look into. As was said in 1 Moran Commentaries on the Rules of ?Court, 3rd Ed. 856, 857, "Undisputed evidence is one thing, and contradicted evidence is another. An incredible witness does not cease to be such because he is not impeached or contradicted. But when the evidence is purely documentary, the authenticity of which is not questioned and the only issue is the construction to be placed thereon, or where a case is submitted upon an agreement of facts, or where all the facts are stated in the judgment and the issue is the correctness of the conclusions drawn therefrom, the question is one of law which may be reviewed by the Supreme Court." The question of whether upon given facts the operation of the statutory presumption is to be invoked is a question of law. The prohibition against intermeddling with decisions on questions of evidence refers to decisions supported by substantial evidence. By substantial evidence is meant real evidence or at least evidence about which reasonable men may disagree. Findings grounded entirely on speculations, surmises, or conjectures come within the exception to the general rule. We are constrained to reverse the decision under review, and hold that the distribution of the decedents' estates should be made in accordance with the decision of the trial court. This result precludes the necessity of passing upon the question of "reserva troncal" which was put forward on the hypothetical theory that Mrs. Joaquin Navarro's death preceded that of her son. Without costs.

204

University of the Cordilleras College of Law First Year C S.Y. 2013 - 2014

VI. Marriage

205

University of the Cordilleras College of Law First Year C S.Y. 2013 - 2014 De Loria vs Felix 104 Phil 1 G.R. No. L-9005 June 20, 1958 Full Case ARSENIO DE LORIA and RICARDA DE LORIA, petitioners, vs. FELIPE APELAN FELIX, respondent. BENGZON, J.: Review of a decision of the Court of Appeals, involving the central issue of the validity of the marriage in articulo mortis between Matea de la Cruz and Felipe Apelan Felix. It appears that long before, and during the War of the Pacific, these two persons lived together as wife and husband at Cabrera Street, Pasay City. They acquired properties but had no children. In the early part of the liberation of Manila and surrounding territory, Matea became seriously ill. Knowing her critical condition, two young ladies of legal age dedicated to the service of God, named Carmen Ordiales and Judith Vizcarra visited and persuaded her to go to confession. They fetched Father Gerardo Bautista, Catholic parish priest of Pasay. The latter, upon learning that the penitent had been living with Felipe Apelan Felix without benefit of marriage, asked both parties to ratify their union according to the rites of his Church. Both agreed. Whereupon the priest heard the confession of the bed-ridden old woman, gave her Holy Communion, administered the Sacrament of Extreme Unction and then solemnized her marriage with Felipe Apelan Felix in articulo mortis, Carmen Ordiales and Judith Vizcarra acting as sponsors or witnesses. It was then January 29 or 30, 1945. After a few months, Matea recovered from her sickness; but death was not to be denied, and in January 1946, she was interred in Pasay, the same Fr. Bautista performing the burial ceremonies. On May 12, 1952, Arsenio de Loria and Ricarda de Loria filed this complaint to compel defendant to an accounting and to deliver the properties left by the deceased. They are grandchildren of Adriana de la Cruz, sister of Matea, and claim to be the only surviving forced heirs of the latter. Felipe Apelan Felix resisted the action, setting up his rights as widower. They obtained favorable judgment in the court of first instance, but on appeal the Court of Appeals reversed and dismissed the complaint. Their request for review here was given due course principally to consider the legal question-which they amply discussed in their petition and printed brief whether the events which took place in January 1945 constituted, in the eyes of the law, a valid and binding marriage. According to the Court of Appeals: There is no doubt at all in the mind of this Court, that Fr. Gerardo Bautista, solemnized the marriage in articulo mortis of Defendant Apelan Felix and Matea de la Cruz, on January 29 and 30, 1945, under the circumstances set forth in the reverend's testimony in court. Fr. Bautista, a respectable old priest of Pasay City then, had no reason to side one or the other. . . . Notwithstanding this positive evidence on the celebration or performance of the marriage in question, Plaintiffs-Appellees contend that the same was not in articulo mortis, because Matea de la Cruz was not then on the point of death. Fr. Bautista clearly testified, however, that her condition at the time was bad; she was bed-ridden; and according to his observation, she might die at any moment (Exhibit 1), so apprehensive was he about her condition that he decided in administering to her the sacrament of extreme unction, after hearing her confession . . . . . The greatest objection of the Appellees and the trial court against the validity of the marriage under consideration, is the admitted fact that it was not registered. The applicable legal provisions are contained in the Marriage Law of 1929 (Act No. 3613) as amended by Commonwealth Act No. 114 (Nov. 1936) specially sections 1, 3, 20 and 21. There is no question about the officiating priest's authority to solemnize marriage. There is also no question that the parties had legal capacity to contract marriage, and that both declared before Fr. Bautista and Carmen Ordiales and Judith Vizcarra that "they took each other as husband and wife." 206

University of the Cordilleras College of Law First Year C S.Y. 2013 - 2014 The appellants' contention of invalidity rests on these propositions: (a) There was no "marriage contract" signed by the wedded couple the witnesses and the priest, as required by section 3 of the Marriage Law; and (b) The priest filed no affidavit, nor recorded the marriage with the local civil registry. The factual basis of the first proposition no signing may seriously be doubted. The Court of Appeals made no finding thereon. Indeed if anything, its decision impliedly held such marriage contract to have been executed, since it said "the marriage in articulo mortis was a fact", and the only question at issue was whether "the failure of Fr. Bautista to send copies of the certificate of marriage in question to the Local Civil Registrar and to register the said marriage in the Record of Marriages of the Pasay Catholic Church . . . renders the said marriage invalid." And such was the only issue tendered in the court of first instance. (See p. 14, 34, Record on Appeal.) However, we may as well face this second issue: Does the failure to sign the "marriage certificate or contract" constitute a cause for nullity? Marriage contract is the "instrument in triplicate" mentioned in sec. 3 of the Marriage Law which provides: Sec. 3. Mutual Consent. No particular form for the ceremony of marriage is required, but the parties with legal capacity to contract marriage must declare, in the presence of the person solemnizing the marriage and of two witnesses of legal age, that they take each other as husband and wife. This declaration shall be set forth in an instrument in triplicate, signed by signature or mark by the contracting parties and said two witnesses and attested by the person solemnizing the marriage. . . . (Emphasis ours). In the first place, the Marriage Law itself, in sections 28, 29 and 30 enumerates the causes for annulment of marriage. Failure to sign the marriage contract is not one of them. In the second place, bearing in mind that the "essential requisites for marriage are the legal capacity of the contracting parties and their consent" (section 1), the latter being manifested by the declaration of "the parties" "in the presence of the person solemnizing the marriage and of two witnesses of legal age that they take each other as husband and wife" which in this case actually occurred. We think the signing of the marriage contract or certificate was required by the statute simply for the purpose of evidencing the act. No statutory provision or court ruling has been cited making it an essential requisite not the formal requirement of evidentiary value, which we believe it is. The fact of marriage is one thing; the proof by which it may be established is quite another. Certificate and Record. Statutes relating to the solemnization of marriage usually provide for the issuance of a certificate of marriage and for the registration or recording of marriage . . . Generally speaking, the registration or recording of a marriage is not essential to its validity, the statute being addressed to the officials issuing the license, certifying the marriage, and making the proper return and registration or recording. (Sec. 27 American Jurisprudence "Marriage" p. 197198.) Formal Requisites. . . . The general rule, however, is that statutes which direct that a license must be issued and procured, that only certain persons shall perform the ceremony, that a certain number of witnesses shall be present, that a certificate of the marriage shall be signed, returned, and recorded, and that persons violating the conditions shall be guilty of a criminal offense, are addressed to persons in authority to secure publicity and to require a record to be made of the marriage contract. Such statutes do not void common-law marriages unless they do so expressly, even where such marriage are entered into without obtaining a license and are not recorded. It is the purpose of these statutes to discourage deception and seduction, prevent illicit intercourse under the guise of matrimony, and relieve from doubt the status of parties who live together as man and wife, by providing competent evidence of the marriage. . . . (Section 15 American Jurisprudence "Marriage" pp. 188-189.) Emphasis Ours. (See also Corpus Juris Secundum "Marriage" Sec. 33.)

207

University of the Cordilleras College of Law First Year C S.Y. 2013 - 2014 And our law says, "no marriage shall be declared invalid because of the absence of one or several formal requirements of this Act . . . ." (Section 27.) In the third place, the law, imposing on the priest the duty to furnish to the parties copies of such marriage certificate (section 16) and punishing him for its omission (section 41) implies his obligation to see that such "certificate" is executed accordingly. Hence, it would not be fair to visit upon the wedded couple in the form of annulment, Father Bautista's omission, if any, which apparently had been caused by the prevailing disorder during the liberation of Manila and its environs. Identical remarks apply to the priest's failure to make and file the affidavit required by sections 20 and 21. It was the priest's obligation; non-compliance with it, should bring no serious consequences to the married pair, specially where as in this case, it was caused by the emergency. The mere fact that the parish priest who married the plaintiff's natural father and mother, while the latter was in articulo mortis, failed to send a copy of the marriage certificate to the municipal secretary, does not invalidate said marriage, since it does not appear that in the celebration thereof all requisites for its validity were not present, the forwarding of a copy of the marriage certificate not being one of the requisites. (Jones vs. Hortiguela, 64 Phil. 179.) See also Madridejo vs. De Leon, 55 Phil. 1. The law permits in articulo mortis marriages, without marriage license; but it requires the priest to make the affidavit and file it. Such affidavit contains the data usually required for the issuance of a marriage license. The first practically substitutes the latter. Now then, if a marriage celebrated without the license is not voidable (under Act 3613), this marriage should not also be voidable for lack of such affidavit. In line with the policy to encourage the legalization of the union of men and women who have lived publicly in a state of concubinage, (section 22), we must hold this marriage to be valid. The widower, needless to add, has better rights to the estate of the deceased than the plaintiffs who are the grandchildren of her sister Adriana. "In the absence of brothers or sisters and of nephews, children of the former . . . the surviving spouse . . . shall succeed to the entire estate of the deceased. (Art 952, Civil Code.) Wherefore, the Court of Appeals' decision is affirmed, with costs. So ordered. Paras, C. J., Montemayor, Reyes, A., Bautista Angelo, Concepcion, Reyes, J.B.L., Endencia, and Felix, JJ.,concur.

208

University of the Cordilleras College of Law First Year C S.Y. 2013 - 2014 Case Digest ARSENIO DE LORIA and RICARDA DE LORIA vs. FELIPE APELAN FELIX G.R. No. L-9005 June 20, 1958 BENGZON, J.: Facts: Matea de la Cruz and Felipe Apelan Felix lived together as wife and husband at Cabrera Street, Pasay City. They acquired properties but had no children. Matea became seriously ill. Knowing her critical condition Carmen Ordiales and Judith Vizcarra visited and persuaded her to go to confession. They fetched Father Gerardo Bautista, Catholic parish priest of Pasay. The latter ratified the union the two. Whereupon the priest heard the confession of the bed-ridden old woman, gave her Holy Communion, administered the Sacrament of Extreme Unction and then solemnized her marriage with Felipe Apelan Felix in articulo mortis, Carmen Ordiales and Judith Vizcarra acting as sponsors or witnesses. It was then January 29 or 30, 1945. After a few months, Matea recovered from her sickness but died in January 1946. On May 12, 1952, Arsenio and Ricarda de Loria, grandchildren of Mateas sister filed a complaint to c ompel the delivery of the property left the deceased however, Felipe resisted the action setting up his rights as widower. The CFI ruled in favor of the complainants but the CA reversed and dismissed the complaint. They appealed before this Court. The appellants contend that the marriage is invalid because, first, there was no marriage contract signed as required by Sec. 3 of the Marriage Law; and second, the priest did file an affidavit with the Local Civil Registry. Issue: Whether or not the marriage is valid Held: The marriage is valid. On the ground of lack of affidavit and non-registration of marriage, the celebration of the marriage in articulo mortis, where all requisites for the validity were present, the marriage is not voided by the failure of the priest to make and file the affidavit required in Sec. 20 and 21 of the Marriage Law and to register said marriage in the Local Civil Registry. The signing of the marriage contract is a formal requirement for evidentiary value, the omission of which does not render the marriage a nullity. The decision of the Court of Appeals is affirmed.

209

University of the Cordilleras College of Law First Year C S.Y. 2013 - 2014 People vs Mendoza 95 Phil 845 G.R. No. L-5877 September 28, 1954 Full Case THE PEOPLE OF THE PHILIPPINES, plaintiff-appellee, vs. ARTURO MENDOZA, defendant-appellant. PARAS, C.J.: The defendant, Arturo Mendoza, has appealed from a judgment of the Court of First Instance of Laguna, finding him guilty of the crime of bigamy and sentencing him to imprisonment for an indeterminate term of from 6 months and 1 day to 6 years, with costs. The following facts are undisputed: On August 5, 1936, the appellant and Jovita de Asis were married in Marikina, Rizal. On May 14, 1941, during the subsistence of the first marriage, the appellant was married to Olga Lema in the City of Manila. On February 2, 1943, Jovita de Asis died. On August 19, 1949, the appellant contracted another marriage with Carmencita Panlilio in Calamba, Laguna. This last marriage gave rise to his prosecution for and conviction of the crime of bigamy. The appellant contends that his marriage with Olga Lema on May 14, 1941 is null and void and, therefore, non-existent, having been contracted while his first marriage with Jovita de Asis August 5, 1936 was still in effect, and that his third marriage to Carmencita Panlilio on August 19, 1949 cannot be the basis of a charge for bigamy because it took place after the death of Jovita de Asis. The Solicitor General, however, argues that, even assuming that appellant's second marriage to Olga Lema is void, he is not exempt from criminal liability, in the absence of a previous judicial annulment of said bigamous marriage; and the case of People vs. Cotas, 40 Off. Gaz., 3134, is cited. The decision invoked by the Solicitor General, rendered by the Court of Appeals, is not controlling. Said case is essentially different, because the defendant therein, Jose Cotas, impeached the validity of his first marriage for lack of necessary formalities, and the Court of Appeals found his factual contention to be without merit. In the case at bar, it is admitted that appellant's second marriage with Olga Lema was contracted during the existence of his first marriage with Jovita de Asis. Section 29 of the marriage law (act 3613), in force at the time the appellant contracted his second marriage in 1941, provides as follows: Illegal marriages. Any marriage subsequently contracted by any person during the lifetime of the first spouse of such person with any person other than such first spouse shall be illegal and void from its performance, unless: (a) The first marriage was annulled or dissolved; (b) The first spouse had been absent for seven consecutive years at the time of the second marriage without the spouse present having news of the absentee being alive, or the absentee being generally considered as dead and believed to be so by the spouse present at the time of contracting such subsequent marriage, the marriage so contracted being valid in either case until declared null and void by a competent court. This statutory provision plainly makes a subsequent marriage contracted by any person during the lifetime of his first spouse illegal and void from its performance, and no judicial decree is necessary to establish its invalidity, as distinguished from mere annulable marriages. There is here no pretence that appellant's second marriage with Olga Lema was contracted in the belief that the first spouse, Jovita de Asis, has been absent for seven consecutive years or generally considered as dead, so as to render said marriage valid until declared null and void by a competent court.1wphl.nt Wherefore, the appealed judgment is reversed and the defendant-appellant acquitted, with costs de officio so ordered.

210

University of the Cordilleras College of Law First Year C S.Y. 2013 - 2014 Pablo, Bengzon, Jugo, Bautista Angelo, Labrador, Concepcion, and Reyes, J.B.L., JJ., concur. Separate Opinions REYES, J., dissenting: I dissent. Article 349 of the Revised Penal Code punishes with prision mayor "any person who shall contract a second or subsequent marriage before the former marriage has been legally dissolved". Though the logical may say that where the former marriage was void there would be nothing to dissolve still it is not for the spouses to judge whether that marriage was void or not. That judgment is reserved to the courts. As Viada says "La santidad e importancia del matrinonio no permite que los casados juzguen por si mismos de su nulidad; esta ha de someterse precisamente al judicio del Tribunal competente, y cuando este declare la nulidad del matrimonio, y solo entonces, se tendra por nulo; mientras no exista esta declaracion, la presuncion esta siempre a favor de la validez del matrimonio, y de consiguente, el que contrae otro segundo antes de dicha declaracion de nulidad, no puede menos de incurrir la pena de este articulo."(3 Viada, Codigo penal p. 275.) "This is a sound opinion," says Mr. Justice Tuason in the case of People vs. Jose Cotas, (CA), 40 Off. Gaz., 3145, "and is in line with the well-known rule established in cases of adultery, that "until by competent authority in a final judgment the marriage contract is set aside, the offense to the vows taken and the attack on the family exist." Padilla and Montemayor, JJ., concur.

211

University of the Cordilleras College of Law First Year C S.Y. 2013 - 2014 Case Digest THE PEOPLE OF THE PHILIPPINES vs. ARTURO MENDOZA G.R. No. L-5877 September 28, 1954 PARAS, C.J.: Facts: On August 5, 1936, the appellant and Jovita de Asis were married in Marikina, Rizal. On May 14, 1941, during the subsistence of the first marriage, the appellant was married to Olga Lema in the City of Manila. On February 2, 1943, Jovita de Asis died. On August 19, 1949, the appellant contracted another marriage with Carmencita Panlilio in Calamba, Laguna. This last marriage gave rise to his prosecution for and conviction of the crime of bigamy. The appellant contends that his marriage with Olga Lema on May 14, 1941 is null and void and, therefore, non-existent, having been contracted while his first marriage with Jovita de Asis August 5, 1936 was still in effect, and that his third marriage to Carmencita Panlilio on August 19, 1949 cannot be the basis of a charge for bigamy because it took place after the death of Jovita de Asis. The Solicitor General, however, argues that, even assuming that appellant's second marriage to Olga Lema is void, he is not exempt from criminal liability, in the absence of a previous judicial annulment of said bigamous marriage; and the case of People vs. Cotas, 40 Off. Gaz., 3134, is cited. The cited case was essentially different therefore the decision invoked by the Solicitor General is not controlling. Issue: Whether or not there is a need for a judicial annulment of a bigamous marriage Ruling: In the case at bar, it is admitted that appellant's second marriage with Olga Lema was contracted during the existence of his first marriage with Jovita de Asis. Section 29 of the marriage law (act 3613), in force at the time the appellant contracted his second marriage in 1941, provides as follows: Illegal marriages. Any marriage subsequently contracted by any person during the lifetime of the first spouse of such person with any person other than such first spouse shall be illegal and void from its performance, unless: (a) The first marriage was annulled or dissolved; (b) The first spouse had been absent for seven consecutive years at the time of the second marriage without the spouse present having news of the absentee being alive, or the absentee being generally considered as dead and believed to be so by the spouse present at the time of contracting such subsequent marriage, the marriage so contracted being valid in either case until declared null and void by a competent court. This statutory provision plainly makes a subsequent marriage contracted by any person during the lifetime of his first spouse illegal and void from its performance, and no judicial decree is necessary to establish its invalidity, as distinguished from mere annulable marriages. There is here no pretence that appellant's second marriage with Olga Lema was contracted in the belief that the first spouse, Jovita de Asis, has been absent for seven consecutive years or generally considered as dead, so as to render said marriage valid until declared null and void by a competent court.

212

University of the Cordilleras College of Law First Year C S.Y. 2013 - 2014 Alvado vs City of Tacloban 139 SCRA 230 G.R. No. L-49084 October 10, 1985 Full Case MATILDE ALAVADO in her own right and as natural guardian of IDA VILMA, IMELDA AND ROLANDO, all surnamed ALAVADO petitioner, vs. CITY GOVERNMENT OF TACLOBAN (ENGINEER'S OFFICE AND WORKMEN'S COMPENSATION COMMISSION, now the LABOR APPEALS AND REVIEW STAFF), respondents. CUEVAS, J.: Assailed in the instant petition is the decision dated November 29, 1975 of the defunct Workmen's Compensation Commission which dismissed petitioner's death benefits claim for the death of her husband, Ricardo Alavado, a former employee of the City Engineer's Office in Tacloban City. The evidence on record discloses that the late Ricardo A. Alavado was employed as a carpenter-foreman by the City Engineer's Office, Tacloban City with a daily wage of P13.12. His last day of service was on April 19, 1974 since he was on leave from April 23, 1974 to May 23, 1974. On August 6, 1974 when he reported for work, he was no longer under the supervision of respondent city. He suffered severe headache when he was supervising laborers on a construction project in Tolosa, Leyte. He died the following day of CVA-Cerebral Hemorrhage. Petitioner, the surviving spouse, filed a claim for death benefits in her own behalf and in behalf of her minor children. Respondent city filed a notice of controversion of the claimant's right to compensation on December 10, 1974. On March 31, 1975, the hearing officer of Regional Office No. 9 in 'Tacloban City issued an award granting petitioner the sum of P5,200.00 as death benefits and P200.00 as reimbursement of burial expenses. Respondent city appealed. On November 29, 1975, a decision was rendered by the Commission dismissing petitioner's death benefits claim, holding that xxx xxx xxx While it is true that the deceased has suffered from the ailment which resulted in his death while he wall the performance of his work as a Carpenter Foreman this case must be denied on tile ground of lack of filiation between the herein claimant and the deceased. It is a settled rule that the status of dependency of a spouse arises from the fact that a marriage exists. A showing of marital status is essential. In this case the herein claimant Matilde Alvarado presented a marriage certificate issued by the Sto. Nino Parish of Tacloban City as proof of her marriage to t he deceased. This certification is not an authentic proof of marital status. To prove filiation as a spouse and, therefore claim as a dependent 'within the meaning of the Act, the surviving spouse-claimant must show either the original of the marriage contract or the marriage certificate duly issued by the local Civil Registrar of the place where the marriage was solemnized. In the absence thereof, as when the records are destroyed or not available due to fire or other causes, secondary evidence may be presented consisting of an affidavit of the claimant and at least three witnesses to the marriage cohabitation. As to the filiation of the children the same is establish by the presentation of the birth certificate. In this case only the baptismal certificates of all the children were presented in evidence by the claimant. A baptismal certificate is not sufficient because it merely proves the fact that originated its execution, and the date of the same, namely the administration of the sacrament of baptism on the date specified. It is not an authentic proof as to the statements made therein respecting the kinsfolk of the person baptized and the presentation of such baptismal certificate does not prove filiation for the purpose of establishing the status of dependency. Dissatisfied with respondent Commission's decision, claimant spouse filed the instant petition raising the following issues: 213

University of the Cordilleras College of Law First Year C S.Y. 2013 - 2014 I May a marriage certificate attesting to the fact that claimant and deceased were in fact married be considered satisfactory proof of marital status in the absence of any evidence to the contrary? and II Whether or not the respondent commission committed a grave abuse of discretion amounting to lack of jurisdiction on the matter. The petition is impressed with merit. Its grant is therefore in order. While admitting the compensability of the claim, respondent Commission nevertheless dismissed the same due to the alleged failure of petitioner claimant to prove that she was legally married to the deceased. In making the said pronouncement, respondent Commission relied solely on the absence of a copy, or a certified copy of petitioner's marriage contract with the deceased Alavado. What was submitted by her is a mere copy issued by the church authorities where the questioned marriage was solemnized. The said document shows that petitioner claimant and the deceased were married on August 9, 1939. Since then, they lived together as man and wife continuously for a period of 35 years in their conjugal abode up to the time of Alavado's death. Section 5(bb) of Rule 31 of the Rules of Court provides: Sec. 5. Disputable Presumptions.The following presumptions are satisfactory if uncontradicted, but may be contradicted and overcome by other evidence. xxx xxx xxx That a man and a woman deporting themselves as husband and wife have entered into a lawful contract of marriage. xxx xxx xxx Courts look upon this presumption with great favor and it could not be lightly repelled. It may be rebutted only by cogent proof to the contrary or by evidence of a higher than ordinary quality. The rationale behind this presumption could be found in the case of Adong vs. Cheong Seng Gee, which runs this wise The basis of human society throughout the civilized world is that of marriage. Marriage in this jurisdiction is not only a civil contract but it is a new relation, an institution in the maintenance of which the public is deeply interested. Consequently, every intendment of the law leans toward legalizing matrimony. Persons dwelling together in apparent matrimony are presumed, in the absence of any counter-presumption or evidence special to the case, to be in fact married. The reason is that such is the common order of society, and if the parties were not what they thus hold themselves out as being, they would be living in the constant violation of decency and of law. A presumption established by our Code of Civil Procedure is 'that a man and a woman deporting themselves as husband and wife have entered into a lawful contract of marriage. (Sec. 334, No. 28) Semper praesumitur pro matrimonio Always presume marriage. So much so that once a man and a woman have lived as husband and wife and such relationship is not denied nor contradicted the presumption of their being married must be admitted as a fact. Likewise, the declaration of the husband is competent evidence to show the fact of marriage. Similarly a witness, who was present at the time the marriage was solemnized, is a competent witness to establish the existence of said marriage. Indeed, public and open cohabitation as husband and wife, birth and baptismal certificates of children born unto them after the celebration of the questioned marriage, and a statement of such marriage in subsequent document were held to be competent evidence as proof of said marriage. A review of the records of this case failed to disclose any evidence whatsoever which will overthrow the aforementioned presumption in favor of claimant's marriage to the deceased Alavado. But what wrote finish to this issue-legality of the claimant's marriage to the deceased is the marriage certificate submitted later by the claimant. In the said document. the contracting parties appeared to be Ricardo Alavado and 214

University of the Cordilleras College of Law First Year C S.Y. 2013 - 2014 Matilde Valdesco The marriage was solemnized on August 19, 1939 by Fr. Ignacio Mora, priest of Tacloban, Leyte. It is certified to be a true copy of the original issued by the local Civil Registrar of the City of Tacloban. The said document indubitably establishes claimant marriage to the deceased Alavado, In the answer filed by City Fiscal Pedroza for respondent City, he averred that Alavado was on longer an employee of respondent city government at the time of his death; hence the city is not liable to pay compensation benefits. We find respondent city's contention untenable. Such a defense should have been raised before the Commission within the period prescribed by the Workmen's Compensation Act within fourteen (14) days from death or within ten (10) days from knowledge thereof. Having failed to controvert the said claim within the prescribed reglementary period, its compensability is now beyond challenge. Respondent city's failure to controvert the claim within the aforesaid period is a waiver of its right to do so. The Workmen's Compensation Act, being a social legislation, aimed at protecting the rights of the workingmen in consonance with the social justice guarantee of the Constitution, its provision must be interpreted liberally in favor of laborers or workers. This basic mandate should guide all tribunals and agencies in the resolution of cases of this nature more specially those involving poor claimants who have come to court as pauper litigants. WHEREFORE, the decision dated November 29, 1979 of the defunct Workmen's Compensation Commission is hereby SET ASIDE. The award of the Hearing Officer of Regional Office No. 9 of Tacloban City is REINSTATED. SO ORDERED. Makasiar, C.J., Concepcio, Jr., Abad Santos, Plana, Escolin, Relova, Gutierrez, Jr., De la Fuente, Alampay and Patajo, JJ., concur.

215

University of the Cordilleras College of Law First Year C S.Y. 2013 - 2014 Case Digest MATILDE ALAVADO vs. CITY GOVERNMENT OF TACLOBAN G.R. No. L-49084 October 10, 1985 CUEVAS, J.: Facts: Late Ricardo A. Alavado was employed as a carpenter-foreman by the City Engineer's Office, Tacloban City with a daily wage of P13.12. His last day of service was on April 19, 1974 since he was on leave from April 23, 1974 to May 23, 1974. On August 6, 1974 when he reported for work, he was no longer under the supervision of respondent city. He suffered severe headache when he was supervising laborers on a construction project in Tolosa, Leyte. He died the following day of CVA-Cerebral Hemorrhage. The surviving spouse filed a claim for death benefits in her own behalf and in behalf of her minor children. Respondent city filed a notice of controversion of the claimant's right to compensation however, on March 31, 1975, the hearing officer of Regional Office in Tacloban City issued an award granting petitioner the sum of P5,200.00 as death benefits and P200.00 as reimbursement of burial expenses. Respondent city appealed. On November 29, 1975, a decision was rendered by the Commission dismissing petitioner's death benefits claim on the ground of lack of filiation between the claimant and the deceased. The Commission held that the marriage certificate from the Parish where the claimants marriage was solemnized is not a valid proof. It should be the original marriage contract or marriage certificate issued by the Local Civil Registrar. The baptismal certificates of their children which was presented by the claimant, according to the commission was not a valid proof of filiation. Issue: Whether or not a marriage certificate attesting to the fact that claimant and deceased were in fact married is considered satisfactory proof of marital status in the absence of any evidence to the contrary Ruling: Section 5(bb) of Rule 31 of the Rules of Court provides for the disputable presumptions which states that such presumptions are satisfactory if uncontradicted, but may be contradicted and overcome by other evidence. It includes That a man and a woman deporting themselves as husband and wife have entered into a lawful contract of marriage. Courts look upon this presumption with great favor and it could not be lightly repelled. It may be rebutted only by cogent proof to the contrary or by evidence of a higher than ordinary quality. The rationale behind this presumption could be found in the case of Adong vs. Cheong Seng Gee, which runs this wise --- The basis of human society throughout the civilized world is that of marriage. Marriage in this jurisdiction is not only a civil contract but it is a new relation, an institution in the maintenance of which the public is deeply interested. Consequently, every intendment of the law leans toward legalizing matrimony. Persons dwelling together in apparent matrimony are presumed, in the absence of any counter-presumption or evidence special to the case, to be in fact married. The reason is that such is the common order of society, and if the parties were not what they thus hold themselves out as being, they would be living in the constant violation of decency and of law. A presumption established by our Code of Civil Procedure is 'that a man and a woman deporting themselves as husband and wife have entered into a lawful contract of marriage. (Sec. 334, No. 28) Semper praesumitur pro matrimonio Always presume marriage. Likewise, the declaration of the husband is competent evidence to show the fact of marriage. Similarly a witness, who was present at the time the marriage was solemnized, is a competent witness to establish the existence of said marriage. Indeed, public and open cohabitation as husband and wife, birth and baptismal certificates of children born unto them after the celebration of the questioned marriage, and a statement of such marriage in subsequent document were held to be competent evidence as proof of said marriage. Therefore, the decision dated November 29, 1979 is set aside and the award of the earing Office of Tacloban City is reinstated.

216

University of the Cordilleras College of Law First Year C S.Y. 2013 - 2014 Trinidad vs Court of Appeals 289 SCRA 188 [G.R. No. 118904. April 20, 1998] Full Case ARTURIO TRINIDAD, petitioner, Vs COURT OF APPEALS, FELIX TRINIDAD (deceased) and LOURDES TRINIDAD, respondents. PANGANIBAN, J.: In the absence of a marriage contract and a birth certificate, how may marriage and filiation be proven? The Case This is the main question raised in this petition for review on certiorari challenging the Court of Appeals Decision promulgated on December 1, 1994 and Resolution promulgated on February 8, 1995 in CA-GR CV No. 23275, which reversed the decision of the trial court and dismissed petitioners acti on for partition and damages. On August 10, 1978, Petitioner Arturio Trinidad filed a complaint for partition and damages against Private Respondents Felix and Lourdes, both surnamed Trinidad, before the Court of First Instance of Aklan, Branch I. On October 28, 1982, Felix died without issue, so he was not substituted as a party. On July 4, 1989, the trial court rendered a twenty-page decision in favor of the petitioner, in which it ruled: Considering therefore that this court is of the opinion that plaintiff is the legitimate son of Inocentes Trinidad, plaintiff is entitled to inherit the property left by his deceased father which is 1/3 of the 4 parcels of land subject matter of this case. Although the plaintiff had testified that he had been receiving [his] share from said land before and the same was stopped, there was no evidence introduced as to what year he stopped receiving his share and for how much. This court therefore cannot rule on that. In its four-page Decision, Respondent Court reversed the trial court on the ground that petitioner failed to adduce sufficient evidence to prove that his parents were legally married to each other and that acquisitive prescription against him had set in. The assailed Decision disposed: WHEREFORE, the Court REVERSES the appealed decision. In lieu thereof, the Court hereby DISMISSES the [petitioners] complaint and the counterclaim thereto. Without costs. Respondent Court denied reconsideration in its impugned Resolution which reads: The Court DENIES defendants-appellants motion for reconsideration, dated December 15, 1994, for lack of merit. There are no new or substantial matters raised in the motion that merit the modification of the decision. Hence, this petition.

The Facts The assailed Decision recites the factual background of this case, as follows: On August 10, 1978, plaintiff [herein petitioner] filed with the Court of First Instance of Aklan, Kalibo, Aklan, an action for partition of four (4) parcels of land, described therein, claiming that he was the son of the late Inocentes Trinidad, one of three (3) children of Patricio Trinidad, who was the original owner of the parcels of land. Patricio Trinidad died in 1940, leaving the four (4) parcels of land to his three (3) children, Inocentes, Lourdes and Felix. In 1970, plaintiff 217

University of the Cordilleras College of Law First Year C S.Y. 2013 - 2014 demanded from the defendants to partition the land into three (3) equal shares and to give him the one-third (1/3) individual share of his late father, but the defendants refused. In their answer, filed on September 07, 1978, defendants denied that plaintiff was the son of the late Inocentes Trinidad. Defendants contended that Inocentes was single when he died in 1941, before plaintiffs birth. Defendants also denied that plaintiff had lived with them, and claimed that the parcels of land described in the complaint had been in their possession since the death of their father in 1940 and that they had not given plaintiff a share in the produce of the land. Patricio Trinidad and Anastacia Briones were the parents of three (3) children, namely, Inocentes, Lourdes and Felix. When Patricio died in 1940, survived by the above named children, he left four (4) parcels of land, all situated at Barrio Tigayon, Kalibo Aklan. Arturio Trinidad, born on July 21, 1943, claimed to be the legitimate son of the late Inocentes Trinidad. Arturio got married in 1966 to Candelaria Gaspar, at the age of twenty three (23). Sometime after the marriage, Arturio demanded from the defendants that the above-mentioned parcels of land be partitioned into three (3) equal shares and that he be given the one-third (1/3) individual shares of his late father, but defendants refused. In order to appreciate more clearly the evidence adduced by both parties, this Court hereby reproduces pertinent portions of the trial courts decision: EVIDENCE FOR THE PLAINTIFF: Plaintiff presented as his first witness, Jovita Gerardo, 77 years old, (at the time she testified in 1981) who is the barangay captain of barrio Tigayon, Kalibo, Aklan, since 1972. She testified that before being elected as barrio captain she held the position of barrio council-woman for 4 years. Also she was [a member of the] board of director[s] of the Parent-Teachers Association of Tigayon, Kalibo, Aklan. That she knows the plaintiff because they are neighbors and she knows him from the time of his birth. She knows the father of the plaintiff as Inocentes Trinidad and his mother Felicidad Molato; both were already dead, Inocentes having died in 1944 and his wife died very much later. Witness recalls plaintiff was born in 1943 in Barrio Tigayon, Kalibo, Aklan, on July 21, 1943. At the time of the birth of the plaintiff, the house of the witness was about 30 meters away from plaintiffs parents[] house and she used to go there 2 or 3 times a week. That she knows both the defendants as they are also neighbors. That both Felix and Lourdes Trinidad are the uncle and aunt of Arturio because Inocentes Trinidad who is the father of the plaintiff is the brother of the defendants, Felix and Lourdes Trinidad. She testified she also knows that the father of Inocentes, Felix and Lourdes[,] all surnamed Trinidad[,] was Patricio Trinidad who is already dead but left several parcels of land which are the 4 parcels subject of this litigation. That she knows all these [parcels of] land because they are located in Barrio Tigayon. When asked about the adjoining owners or boundaries of the 4 parcels of land, witness answered and mentioned the respective adjoining owners. That she knew these 4 parcels belonged to Patricio Trinidad because said Patricio Trinidad was a native also of Barrio Tigayon. Said Patricio died before the [war] and after his death the land went to his 3 children, namely: Inocentes, Felix and Lourdes. Since then the land was never partitioned or divided among the 3 children of Patricio. A picture, Exhibit A, was shown to the witness for identification and she identified a woman in the picture as the defendant, Lourdes Trinidad. A man with a hat holding a baby was identified by her as Felix Trinidad, the defendant. The other woman in the picture was pointed by the witness as the wife of the plaintiff, Arturio Trinidad. When asked if Arturio Trinidad and Lourdes Trinidad and Felix Trinidad pointed to by her in the picture are the same Arturio, Felix and Lourdes, who are the plaintiff and the defendants in this case, witness answered yes. Another picture marked as Exhibit B was presented to the witness for identification. She testified the woman in this picture as Lourdes Trinidad. In said picture, Lourdes Trinidad was holding a child which witness identified as the child Arturio Trinidad. When asked by the court when xxx the picture [was] taken, counsel for the plaintiff answered, in 1966. When asked if Arturio Trinidad was baptized, witness answered yes, as she had gone to the house of his parents. Witness then identified the certificate of baptism marked as Exhibit C. The name Arturio Trinidad was marked as Exhibit C-1 and the name of Inocentes Trinidad and Felicidad Molato as father and mother respectively, were marked as Exhibit C-

218

University of the Cordilleras College of Law First Year C S.Y. 2013 - 2014 2. The date of birth being July 21, 1943 was also marked. The signature of Monsignor Iturralde was also identified. On cross-examination, witness testified that she [knew] the land in question very well as she used to pass by it always. It was located just near her house but she cannot exactly tell the area as she merely passes by it. When asked if she [knew] the photographer who took the pictures presented as Exhibit A and B, witness answered she does not know as she was not present during the picture taking. However, she can identify everybody in the picture as she knows all of them. At this stage of the trial, Felix Trinidad [died] without issue and he was survived by his only sister, Lourdes Trinidad, who is his co-defendant in this case. Next witness for the plaintiff was ISABEL MEREN who was 72 years old and a widow. She testified having known Inocentes Trinidad as the father of Arturio Trinidad and that Inocentes, Felix and Lourdes are brothers and sister and that their father was Patricio Trinidad who left them 4 parcels of land. That she knew Inocentes Trinidad and Felicidad Molato who are the parents of Arturio, the plaintiff, were married in New Washington, Aklan, by a protestant pastor by the name of Lauriano Lajaylajay. That she knows Felicidad Molato and Lourdes Trinidad very well because as a farmer she also owns a parcel of land [and] she used to invite Felicidad and Lourdes to help her during planting and harvesting season. That she knows that during the lifetime of Inocentes the three of them, Inocentes, Felix and Lourdes possessed and usufructed the 4 parcels they inherited from their father, Patricio. That upon the death of Inocentes, Lourdes Trinidad was in possession of the property without giving the widow of Inocentes any share of the produce. As Lourdes outlived her two brothers, namely: Felix and Inocentes, she was the one possessing and usufructing the 4 parcels of land up to the present. The witness testified that upon the death of Inocentes, Lourdes took Arturio and cared for him when he was still small, about 3 years old, until Arturio grew up and got married. That while Arturio was growing up, he had also enjoyed the produce of the land while he was being taken care of by Lourdes Trinidad. That a misunderstanding later on arose when Arturio Trinidad wanted to get his fathers share but Lourdes Trinidad will not give it to him. Plaintiff, ARTURIO TRINIDAD, himself, was presented as witness. He testified that defendants, Lourdes and Felix Trinidad, are his aunt and uncle, they being the brother and sister of his father. That the parents of his father and the defendants were Patricio Trinidad and Anastacia Briones. That both his father, Inocentes Trinidad, and mother, Felicidad Molato, were already dead having died in Tigayon, his father having died in 1944 and his mother about 25 years ago. As proof that he is the son of Inocentes Trinidad and Felicidad Molato, he showed a certificate of baptism which had been previously marked as Exhibit C. That his birth certificate was burned during World War 2 but he has a certificate of loss issued by the Civil Registrar of Kalibo, Aklan. When he was 14 years old, the defendants invited him to live with them being their nephew as his mother was already dead. Plaintiffs mother died when he was 13 years old. They treated him well and provided for all his needs. He lived with defendants for 5 years. At the age of 19, he left the house of the defendants and lived on his own. He got married at 23 to Candelaria Gaspar and then they were invited by the defendants to live with them. So he and his wife and children lived with the defendants. As proof that he and his family lived with the defendants when the latter invited him to live with them, he presented a picture previously marked as Exhibit B where there appears his aunt, Lourdes Trinidad, carrying plaintiffs daughter, his uncle and his wife. In short, it is a family picture according to him. Another family picture previously marked Exhibit A shows his uncle, defendant Felix Trinidad, carrying plaintiffs son. According to him, these 2 pictures were taken when he and his wife and children were living with the defendants. That a few years after having lived with them, the defendants made them vacate the house for he requested for partition of the land to get his share. He moved out and looked for [a] lawyer to handle his case. He testified there are 4 parcels of land in controversy of which parcel 1 is an upland. Parcel 1 is 1,000 square meters, [has] 10 coconut trees and fruit bearing. The harvest is 100 coconuts every 4 months and the cost of coconuts is P2.00 each. The boundaries are: East-Federico Inocencio; West-Teodulo Dionesio; North-Teodulo Dionesio; and South-Bulalio Briones; located at Tigayon.

219

University of the Cordilleras College of Law First Year C S.Y. 2013 - 2014 Parcel 2 is an upland with an area of 500 square meters; it has only 1 coconut tree and 1 bamboo groove; also located in Tigayon, Kalibo, Aklan. Adjoining owners are: East-Ambrosio Trinidad; North-Federico Inocencio; West-Patricio Trinidad and South-Gregorio Briones. Parcel 3 is about 12,000 square meters and 1/4 of that belongs to Patricio Trinidad, the deceased father of the defendants and Inocentes, the father of the plaintiff. Parcel 4 is a riceland with an area of 5,000 square meters. The harvest is 40 cavans two times a year [sic]. Adjoining owners are: East-Gregorio Briones; West-Bulalio Briones; South-Federico Inocencio and North-Digna Carpio. Parcel 1 is Lot No. 903. Parcel 2 is Lot No. 864 of the cadastral survey of Kalibo and only Lot 864-A with an area of 540 square meters is the subject of litigation. Parcel 3 is Lot No. 979 of the cadastral survey of Kalibo covered by Tax Decl. No. 703310 with reference to one of the owners of the land, Patricio Trinidad married to Anastacia Briones, one-half share. Parcel 4 is covered by Original Certificate of Title No. 22502 RO-174 covering Lot No. 863 of the cadastral survey of Kalibo. The title is in the name of Patricio Trinidad married to Anastacia Briones. Parcel 1 is covered by Tax Decl. No. 11609 in the name of Patricio Trinidad while parcel 2 is covered by Tax Decl. No. 10626 in the name of Anastacia Briones and another Tax Declaration No. 11637 for Parcel 3 in the name of Ambrosio Trinidad while Parcel 4 is covered by Tax Decl. No. 16378 in the name of Patricio Trinidad. On cross-examination, plaintiff testified that during the lifetime of his mother they were getting the share in the produce of the land like coconuts, palay and corn. Plaintiff further testified that his father is Inocentes Trinidad and his mother was Felicidad Molato. They were married in New Washington, Aklan, by a certain Atty. Lajaylajay. When asked if this Atty. Lajaylajay is a municipal judge of New Washington, Aklan, plaintiff answered he does not know because he was not yet born at that time. That he does not have the death certificate of his father who died in 1944 because it was wartime. That after the death of his father, he lived with his mother and when his mother died[,] he lived with his aunt and uncle, the defendants in this case. That during the lifetime of his mother, it was his mother receiving the share of the produce of the land. That both defendants, namely Lourdes and Felix Trinidad, are single and they have no other nephews and nieces. That [petitioners] highest educational attainment is Grade 3. EVIDENCE FOR THE DEFENDANTS: First witness for the defendants was PEDRO BRIONES, 68 years old, unemployed and a resident of Nalook, Kalibo, Aklan. He testified having known the defendants, Felix and Lourdes Trinidad. They being his first cousins because the mother of Lourdes and Felix by the name of Anastacia Briones and his father are sister and brother. That he also knew Inocentes Trinidad being the brother of Felix and Lourdes and he is already dead. According to the witness, Inocentes Trinidad [died] in 1940 and at the time of his death Inocentes Trinidad was not married. That he knew this fact because at the time of the death of Inocentes Trinidad he was then residing with his aunt, Nanay Taya, referring to Anastacia Briones who is mother of the defendants, Felix and Lourdes Trinidad, as well as Inocentes Trinidad. That at the time of the death of Inocentes Trinidad, according to this witness he stayed with his aunt, Anastacia Trinidad, and with his children before 1940 for only 3 months. When asked if he knew Inocentes Trinidad cohabited with anybody before his death, he answered, That I do not know, neither does he kn[o]w a person by the name of Felicidad Molato. Furthermore, when asked if he can recall if during the lifetime of Inocentes Trinidad witness knew of anybody with whom said Inocentes Trinidad had lived as husband and wife, witness, Pedro Briones, answered that he could not recall because he was then in Manila working. That after the war, he had gone back to the house of his aunt, Anastacia, at Tigayon, Kalibo, as he always visit[s] her every Sunday, however, he does not know the plaintiff, Arturio Trinidad. When asked if after the death of Inocentes Trinidad, he knew anybody who has stayed with the defendants who claimed to be a son of Inocentes Trinidad, witness, Pedro Briones, answered: I do not know about that.

220

University of the Cordilleras College of Law First Year C S.Y. 2013 - 2014 On cross examination, witness testified that although he was born in Tigayon, Kalibo, Aklan, he started to reside in Nalook, Kalibo, as the hereditary property of their father was located there. When asked if he was aware of the 4 parcels of land which is the subject matter of this case before the court, witness answered that he does not know. What he knew is that among the 3 children of Patricio Trinidad, Inocentes is the eldest. And that at the time of the death of Inocentes in 1940, according to the witness when cross examined, Inocentes Trinidad was around 65 years old. That according to him, his aunt, Anastacia Briones, was already dead before the war. When asked on cross examination if he knew where Inocentes Trinidad was buried when he died in 1940, witness answered that he was buried in their own land because the Japanese forces were roaming around the place. When confronted with Exhibit A which is the alleged family picture of the plaintiff and the defendants, witness was able to identify the lady in the picture, which had been marked as Exhibit A-1, as Lourdes Trinidad, and the man wearing a hat on the said picture marked as Exhibit 2-A is Felix Trinidad. However, when asked if he knew the plaintiff, Arturio Trinidad, he said he does not know him. Next witness for the defendants was the defendant herself, LOURDES TRINIDAD. She stated that she is 75 years old, single and jobless. She testified that Inocentes Trinidad was her brother and he is already dead and he died in 1941 in Tigayon, Kalibo, Aklan. That before the death of her brother, Inocentes Trinidad, he had gone to Manila where he stayed for a long time and returned to Tigayon in 1941. According to her, upon arrival from Manila in 1941 his brother, Inocentes Trinidad, lived only for 15 days before he died. While his brother was in Manila, witness testified she was not aware that he had married anybody. Likewise, when he arrived in Tigayon in 1941, he also did [not] get married. When asked if she knew one by the name of Felicidad Molato, witness answered she knew her because Felicidad Molato was staying in Tigayon. However, according to her[,] she does not kn[o]w if her brother, Inocentes Trinidad, had lived with Felicidad Molato as husband and wife. When asked if she knew the plaintiff, Arturio Trinidad, she said, Yes, but she denied that Arturio Trinidad had lived with them. According to the witness, Arturio Trinidad did not live with the defendants but he stayed with his grandmother by the name of Maria Concepcion, his mother, Felicidad Molato, having died already. When asked by the court if there had been an instance when the plaintiff had lived with her even for days, witness answered, he did not. When further asked if Arturio Trinidad went to visit her in her house, witness also said, He did not. Upon cross examination by counsel for the plaintiff, Lourdes Trinidad testified that her parents, Anastacia Briones and Patricio Trinidad, had 3 children, namely: Inocentes Trinidad, Felix Trinidad and herself. But inasmuch as Felix and Inocentes are already dead, she is the only remaining daughter of the spouses Patricio Trinidad and Anastacia Briones. Defendant, Lourdes Trinidad, testified that her brother, Felix Trinidad, died without a wife and children, in the same manner that her brother, Inocentes Trinidad, died without a wife and children. She herself testified that she does not have any family of her own for she has [no] husband or children. According to her[,] when Inocentes Trinidad [died] in 1941, they buried him in their private lot in Tigayon because nobody will carry his coffin as it was wartime and the municipality of Kalibo was occupied by the Japanese forces. When further cross-examined that I[t] could not be true that Inocentes Trinidad died in March 1941 because the war broke out in December 1941 and March 1941 was still peace time, the witness could not answer the question. When she was presented with Exhibit A which is the alleged family picture wherein she was holding was [sic] the child of Arturio Trinidad, she answered; Yes. and the child that she is holding is Clarita Trinidad, child of Arturio Trinidad. According to her, she was only requested to hold this child to be brought to the church because she will be baptized and that the baptism took place in the parish church of Kalibo. When asked if there was a party, she answered; Maybe there was. When confronted with Exhibit A-1 which is herself in the picture carrying the child, witness identified herself and explained that she was requested to bring the child to the church and that the picture taken together with her brother and Arturio Trinidad and the latters child was taken during the time when she and Arturio Trinidad did not have a case in court yet. She likewise identified the man with a hat holding a child marked as Exhibit A-2 as her brother, Felix. When asked if the child being carried by her brother, Felix Trinidad, is another child of the plaintiff, witness answered she does not know because her eyes are already blurred. Furthermore, when asked to identify the woman in the picture who was at the right of the child held by her brother, Felix, and who was previously identified by plaintiff, Arturio Trinidad, as his wife, witness answered that she cannot identify because she had a poor eyesight neither can she identify plaintiff, Arturio Trinidad, holding another child in the picture for the same reason. When asked by counsel for the plaintiff if she knows that the one who took this 221

University of the Cordilleras College of Law First Year C S.Y. 2013 - 2014 picture was the son of Ambrosio Trinidad by the name of Julito Trinidad who was also their cousin, witness testified that she does not know. Third witness for the defendants was BEATRIZ TRINIDAD SAYON who testified that she knew Arturio Trinidad because he was her neighbor in Tigayon. In the same manner that she also knew the defendants, Felix and Lourdes, and Inocentes all surnamed Trinidad because they were her cousins. She testified that a few months after the war broke out Inocentes Trinidad died in their lolas house whose names was Eugenia Rufo Trinidad. She further testified that Inocentes Trinidad had lived almost in his lifetime in Manila and he went home only when his father fetched him in Manila because he was already sick. That according to her, about 1 months after his arrival from Manila, Inocentes Trinidad died. She also testified that she knew Felicidad Molato and that Felicidad Molato had never been married to Inocentes Trinidad. According to her, it was in 1941 when Inocentes Trinidad died. According to her she was born in 1928, therefore, she was 13 or 14 years old when the war broke out. When asked if she can remember that it was only in the early months of the year 1943 when the Japanese occupied Kalibo, she said she [was] not sure. She further testified that Inocentes Trinidad was buried in their private lot because Kalibo was then occupied by the Japanese forces and nobody would carry his body to be buried in the Poblacion. For rebuttal evidence, [petitioner] presented ISABEL MEREN, who was 76 years old and a resident of Tigayon. Rebuttal witness testified that xxx she knew both the [petitioner] and the [private respondents] in this case very well as her house is only around 200 meters from them. When asked if it is true that according to Lourdes Trinidad, [Inocentes Trinidad] arrived from Manila in 1941 and he lived only for 15 days and died, witness testified that he did not die in that year because he died in the year 1944, and that Inocentes Trinidad lived with his sister, Lourdes Trinidad, in a house which is only across the street from her house. According to the said rebuttal witness, it is not true that Inocentes Trinidad died single because he had a wife by the name of Felicidad Molato whom he married on May 5, 1942 in New Washington, Aklan. That she knew this fact because she was personally present when couple was married by Lauriano Lajaylajay, a protestant pastor. On cross examination, rebuttal witness testified that when Inocentes Trinidad arrived from Manila he was in good physical condition. That she knew both Inocentes Trinidad and Felicidad Molato to be Catholics but that according to her, their marriage was solemnized by a Protestant minister and she was one of the sponsors. That during the marriage of Inocentes Trinidad and Felicidad Molato, Lourdes Trinidad and Felix Trinidad were also present. When plaintiff, ARTURIO TRINIDAD, was presented as rebuttal witness, he was not able to present a marriage contract of his parents but instead a certification dated September 5, 1978 issued by one Remedios Eleserio of the Local Civil Registrar of the Municipality of New Washington, Aklan, attesting to the fact that records of births, deaths, and marriages in the municipality of New Washington were destroyed during the Japanese time.

Respondent Courts Ruling In finding that petitioner was not a child, legitimate or otherwise, of the late Inocentes Trinidad, Respondent Court ruled: We sustain the appeal on the ground that plaintiff has not adduced sufficient evidence to prove that he is the son of the late Inocentes Trinidad. But the action to claim legitimacy has not prescribed. Plaintiff has not established that he was recognized, as a legitimate son of the late Inocentes Trinidad, in the record of birth or a final judgment, in a public document or a private handwritten instrument, or that he was in continuous possession of the status of a legitimate child. Two witnesses, Pedro Briones and Beatriz Trinidad Sayon, testified for the defendants that Inocentes Trinidad never married. He died single in 1941. One witness, Isabel Maren, testified in rebuttal for the plaintiff, that Inocentes Trinidad married Felicidad Molato in New Washington, Aklan, on May 5, 1942, solemnized by a pastor of the protestant church and that she attended the wedding ceremony (t.s.n. Sept. 6, 1988, p. 4). Hence, there was no preponderant evidence of the marriage, nor of Inocentes acknowledgment of plaintiff as his son, who was born on July 21, 1943. 222

University of the Cordilleras College of Law First Year C S.Y. 2013 - 2014 The right to demand partition does not prescribe (de Castro vs. Echarri, 20 Phil. 23). Where one of the interested parties openly and adversely occupies the property without recognizing the coownership (Cordova vs. Cordova, L-9936, January 14, 1958) acquisitive prescription may set in (Florenz D. Regalado, Remedial Law Compendium, Vol. I, Fifth Revised Edition, 1988, p. 497). Admittedly, the defendants have been in possession of the parcels of land involved in the concept of owners since their father died in 1940. Even if possession be counted from 1964, when plaintiff attained the age of majority, still, defendants possessed the land for more than ten (10) years, thus acquiring ownership of the same by acquisitive prescription (Article 1134, Civil Code of the Philippines).

The Issues Petitioner submits the following issues for resolution: 1. Whether or not petitioner (plaintiff-appellee) has proven by preponderant evidence the marriage of his parents. 2. Whether or not petitioner (plaintiff-appellee) has adduced sufficient evidence to prove that he is the son of the late Inocentes Trinidad, brother of private respondents (defendantsappellants) Felix and Lourdes Trinidad. 3. Whether or not the Family Code is applicable to the case at bar[,] the decision of the Regional Trial Court having been promulgated on July 4, 1989, after the Family Code became effective on August 3, 1988. 4. Whether or not petitioners status as a legitimate child can be attacked collaterally by the private respondents. 5. Whether or not private respondent (defendants-appellants) have acquired ownership of the properties in question by acquisitive prescription. Simply stated, the main issues raised in this petition are: 1. Did petitioner present sufficient evidence of his parents marriage and of his filiation? 2. Was petitioners status as a legitimate child subject to collateral attack in the action for partition? 3. Was his claim time-barred under the rules on acquisitive prescription?

The Courts Ruling The merits of this petition are patent. The partition of the late Patricios real properties requires preponderant proof that petitioner is a co-owner or co-heir of the decedents estate. His right as a coowner would, in turn, depend on whether he was born during the existence of a valid and subsisting marriage between his mother (Felicidad) and his putative father (Inocentes). This Court holds that such burden was successfully discharged by petitioner and, thus, the reversal of the assailed Decision and Resolution is inevitable.

First and Second Issues: Evidence of and Collateral Attack on Filiation At the outset, we stress that an appellate courts assessment of the evidence presented by the parties will not, as a rule, be disturbed because the Supreme Court is not a trier of facts. But in the face of the contradictory conclusions of the appellate and the trial courts, such rule does not apply here. So, we had to meticulously pore over the records and the evidence adduced in this case. Petitioners first burden is to prove that Inocentes and his mother (Felicidad) were validly married, and that he was born during the subsistence of their marriage. This, according to Respondent Court, he failed to accomplish. 223

University of the Cordilleras College of Law First Year C S.Y. 2013 - 2014 This Court disagrees. Pugeda vs. Trias ruled that when the question of whether a marriage has been contracted arises in litigation, said marriage may be proven by relevant evidence. To prove the fact of marriage, the following would constitute competent evidence: the testimony of a witness to the matrimony, the couples public and open cohabitation as husband and wife after the alleged wedlock, the birth and the baptismal certificates of children born during such union, and the mention of such nuptial in subsequent documents. In the case at bar, petitioner secured a certification from the Office of the Civil Registrar of Aklan that all records of births, deaths and marriages were either lost, burned or destroyed during the Japanese occupation of said municipality. This fact, however, is not fatal to petitioners case. Although the marriage contract is considered the primary evidence of the marital union, petitioners failure to present it is not proof that no marriage took place, as other forms of relevant evidence may take its place. In place of a marriage contract, two witnesses were presented by petitioner: Isabel Meren, who testified that she was present during the nuptial of Felicidad and Inocentes on May 5, 1942 in New Washington, Aklan; and Jovita Gerardo, who testified that the couple deported themselves as husband and wife after the marriage. Gerardo, the 77-year old barangay captain of Tigayon and former board member of the local parent-teachers association, used to visit Inocentes and Felicidads house twice or thrice a week, as she lived only thirty meters away. On July 21, 1943, Gerardo dropped by Inocentes house when Felicidad gave birth to petitioner. She also attended petitioners baptismal party held at the same house. Her testimony constitutes evidence of common reputation respecting marriage. It further gives rise to the disputable presumption that a man and a woman deporting themselves as husband and wife have entered into a lawful contract of marriage. Petitioner also presented his baptismal certificate (Exhibit C) in which Inocentes and Felicidad were named as the childs father and mother. On the other hand, filiation may be proven by the following: ART. 265. The filiation of legitimate children is proved by the record of birth appearing in the Civil Register, or by an authentic document or a final judgment. ART. 266. In the absence of the titles indicated in the preceding article, the filiation shall be proved by the continuous possession of status of a legitimate child. ART. 267. In the absence of a record of birth, authentic document, final judgment or possession of status, legitimate filiation may be proved by any other means allowed by the Rules of Court and special laws. Petitioner submitted in evidence a certification that records relative to his birth were either destroyed during the last world war or burned when the old town hall was razed to the ground on June 17, 1956. To prove his filiation, he presented in evidence two family pictures, his baptismal certificate and Gerardos testimony. The first family picture (Exhibit A) shows petitioner (Exhibit A-5) carrying his second daughter and his wife (Exhibit A-4) together with the late Felix Trinidad (Exhibit A-2) carrying petitioners first daughter, and Lourdes Trinidad (Exhibit A-1). Exhibit B is another picture showing Lourdes Trinidad (Exhibit B-1) carrying petitioners first child (Exhibit B-2). These pictures were taken before the case was instituted. Although they do not directly prove petitioners filiation to Inocentes, they show that petitioner was accepted by the private respondents as Inocentes legitimate sonante litem motam. Lourdes denials of these pictures are hollow and evasive. While she admitted that Exhibit B shows her holding Clarita Trinidad, the petitioners daughter, she demurred that she did so only because she was requested to carry the child before she was baptized. When shown Exhibit A, she recognized her late brother -- but not petitioner, his wife and the couples children -- slyly explaining that she could not clearly see because of an alleged eye defect. Although a baptismal certificate is indeed not a conclusive proof of filiation, it is one of the other means allowed under the Rules of Court and special laws to show pedigree, as this Court ruled in Mendoza vs. Court of Appeals: What both the trial court and the respondent court did not take into account is that an illegitimate child is allowed to establish his claimed filiation by any other means allowed by the Rules of Court and special laws, according to the Civil Code, or by evidence of proof in his favor that the defendant is her father, according to the Family Code. Such evidence may consist of his baptismal certificate, a judicial admission, a family Bible in which his name has been entered, common reputation respecting his pedigree, admission by silence, the testimony of witnesses, and other kinds of proof admissible under Rule 130 of the Rules of Court. [Justice Alicia Sempio-Diy, Handbook on the Family Code of the Phil. 1988 ed., p. 246] 224

University of the Cordilleras College of Law First Year C S.Y. 2013 - 2014 Concededly, because Gerardo was not shown to be a member of the Trinidad family by either consanguinity or affinity, her testimony does not constitute family reputation regarding pedigree. Hence, it cannot, by itself, be used to establish petitioners legitimacy. Be that as it may, the totality of petitioners positive evidence clearly preponderates over private respondents self-serving negations. In sum, private respondents thesis is that Inocentes died unwed and without issue in March 1941. Private respondents witness, Pedro Briones, testified that Inocentes died in 1940 and was buried in the estate of the Trinidads, because nobody was willing to carry the coffin to the cemetery in Kalibo, which was then occupied by the Japanese forces. His testimony, however, is far from credible because he stayed with the Trinidads for only three months, and his answers on direct examination were noncommittal and evasive: Q: At the time of his death, can you tell the Court if this Inocentes Trinidad was married or not? A: Not married. Q: A: Q: A: In 1940 at the time of death of Inocentes Trinidad, where were you residing? I was staying with them. When you said them, to whom are you referring to [sic]? My aunt Nanay Taya, Anastacia. xxx xxx

xxx Q:

Will you please tell the Court for how long did you stay with your aunt Anastacia Trinidad and his children before 1940? A: For only three months. Q: Now, you said at the time of his death, Inocentes Trinidad was single. Do you know if he had cohabited with anybody before his death? A: [T]hat I do not know. Q: A: Q: You know a person by the name of Felicidad Molato? No, sir.

Can you recall if during the lifetime of Inocentes Trinidad if you have known of anybody with whom he has lived as husband and wife? A: I could not recall because I was then in Manila working. Q: After the war, do you remember having gone back to the house of your aunt Anastacia at Tigayon, Kalibo, Aklan? A: Yes, sir, Q: A: How often did you go to the house of your aunt? Every Sunday. xxx xxx

xxx Q: A: Q:

You know the plaintiff Arturio Trinidad? I do not know him.

After the death of Inocentes Trinidad, do you know if there was anybody who has stayed with the defendants who claimed to be a son of Inocentes Trinidad? A: I do not know about that. Beatriz Sayon, the other witness of private respondent, testified that, when the Japanese occupied Kalibo in 1941, her father brought Inocentes from Manila to Tigayon because he was sick. Inocentes stayed with their grandmother, Eugenia Roco Trinidad, and died single and without issue in March 1941, one and a half months after his return to Tigayon. She knew Felicidad Molato, who was also a resident of Tigayon, but denied that Felicidad was ever married to Inocentes. Taking judicial notice that World War II did not start until December 7, 1941 with the bombing of Pearl Harbor in Hawaii, the trial court was not convinced that Inocentes died in March 1941. The Japanese forces occupied Manila only on January 2, 1942; thus, it stands to reason that Aklan was not occupied until then. It was only then that local residents were unwilling to bury their dead in the cemetery in Kalibo, because of the Japanese soldiers who were roaming around the area. Furthermore, petitioner consistently used Inocentes surname (Trinidad) without objection from private respondents -- a presumptive proof of his status as Inocentes legitimate child.

225

University of the Cordilleras College of Law First Year C S.Y. 2013 - 2014 Preponderant evidence means that, as a whole, the evidence adduced by one side outweighs that of the adverse party. Compared to the detailed (even if awkwardly written) ruling of the trial court, Respondent Courts holding that petitioner failed to prove his legitimate filiation to Inoc entes is unconvincing. In determining where the preponderance of evidence lies, a trial court may consider all the facts and circumstances of the case, including the witnesses manner of testifying, their intelligence, their means and opportunity of knowing the facts to which they are testifying, the nature of the facts, the probability or improbability of their testimony, their interest or want thereof, and their personal credibility. Applying this rule, the trial court significantly and convincingly held that the weight of evidence was in petitioners favor. It declared: xxx [O]ne thing sure is the fact that plaintiff had lived with defendants enjoying the status of being their nephew xxx before plaintiff [had] gotten married and had a family of his own where later on he started demanding for the partition of the share of his father, Inocentes. The fact that plaintiff had so lived with the defendants xxx is shown by the alleged family pictures, Exhibits A & B. These family pictures were taken at a time when plaintiff had not broached the idea of getting his fathers share. xxxx His demand for the partition of the share of his father provoked the ire of the defendants, thus, they disowned him as their nephew. xxxx In this case, the plaintiff enjoyed the continuous possession of a status of the child of the alleged father by the direct acts of the defendants themselves, which status was only broken when plaintiff demanded for the partition xxx as he was already having a family of his own. xxxx. However, the disowning by the defendant [private respondent herein], Lourdes Trinidad, of the plaintiff [petitioner herein] being her nephew is offset by the preponderance of evidence, among them the testimony of witness, Jovita Gerardo, who is the barrio captain. This witness was already 77 years old at the time she testified. Said witness had no reason to favor the plaintiff. She had been a PTA officer and the court sized her up as a civic minded person. She has nothing to gain in this case as compared to the witness for the defendants who are either cousin or nephew of Lourdes Trinidad who stands to gain in the case for defendant, Lourdes Trinidad, being already 75 years old, has no husband nor children. Doctrinally, a collateral attack on filiation is not permitted. Rather than rely on this axiom, petitioner chose to present evidence of his filiation and of his parents marriage. Hence, there is no more need to rule on the application of this doctrine to petitioners cause.

Third Issue: No Acquisitive Prescription Respondent Court ruled that, because acquisitive prescription sets in when one of the interested parties openly and adversely occupies the property without recognizing the co-ownership, and because private respondents had been in possession -- in the concept of owners -- of the parcels of land in issue since Patricio died in 1940, they acquired ownership of these parcels. The Court disagrees. Private respondents have not acquired ownership of the property in question by acquisitive prescription. In a co-ownership, the act of one benefits all the other co-owners, unless the former repudiates the co-ownership. Thus, no prescription runs in favor of a co-owner or co-heir against his or her co-owners or co-heirs, so long as he or she expressly or impliedly recognizes the co-ownership. In this particular case, it is undisputed that, prior to the action for partition, petitioner, in the concept of a co-owner, was receiving from private respondents his share of the produce of the land in dispute. Until such time, recognition of the co-ownership by private respondents was beyond question. There is no evidence, either, of their repudiation, if any, of the co-ownership of petitioners father Inocentes over the land. Further, the titles of these pieces of land were still in their fathers name. Although private respondents had possessed these parcels openly since 1940 and had not shared with petitioner the produce of the land during the pendency of this case, still, they manifested no repudiation of the co-ownership. In Mariategui vs. Court of Appeals, the Court held: x x x Corollarily, prescription does not run again private respondents with respect to the filing of the action for partition so long as the heirs for whose benefit prescription is invoked, have not expressly or impliedly repudiated the co-ownership. In the other words, prescription of an action for partition does not lie except when the co-ownership is properly repudiated by the co-owner (Del Banco vs. Intermediate Appellate Court, 156 SCRA 55 [1987] citing Jardin vs. Hollasco, 117 SCRA 532 [1982]).

226

University of the Cordilleras College of Law First Year C S.Y. 2013 - 2014 Otherwise stated, a co-owner cannot acquire by prescription the share of the other co-owners absent a clear repudiation of co-ownership duly communicated to the other co-owners (Mariano vs. De Vega, 148 SCRA 342 [1987]). Furthermore, an action to demand partition is imprescriptible and cannot be barred by laches (Del Banco vs. IAC, 156 SCRA 55 (1987). On the other hand, an action for partition may be seen to be at once an action for declaration of coownership and for segregation and conveyance of a determinate portion of the property involved (Roque vs. IAC, 165 SCRA 118 [1988]). Considering the foregoing, Respondent Court committed reversible error in holding that petitioners claim over the land in dispute was time-barred. WHEREFORE, the petition is GRANTED and the assailed Decision and Resolution are REVERSED and SET ASIDE. The trial courts decision dated July 4, 1989 is REINSTATED. No costs. SO ORDERED. Davide, Jr., (Chairman), Bellosillo, Vitug, and Quisumbing, JJ., concur.

227

University of the Cordilleras College of Law First Year C S.Y. 2013 - 2014 Case Digest ARTURIO TRINIDAD vs. COURT OF APPEALS G.R. No. 118904 April 20, 1998 PANGANIBAN, J.: Facts: On August 10, 1978, petitioner filed with the Court of First Instance of Aklan, Kalibo, Aklan, an action for partition of four (4) parcels of land, described therein, claiming that he was the son of the late Inocentes Trinidad, one of three (3) children of Patricio Trinidad, who was the original owner of the parcels of land. Patricio Trinidad died in 1940, leaving the four (4) parcels of land to his three (3) children, Inocentes, Lourdes and Felix. In 1970, plaintiff demanded from the defendants to partition the land into three (3) equal shares and to give him the one-third (1/3) individual share of his late father, but the defendants refused and alleged in their answer filed on September 07, 1978 that plaintiff was not the son of the late Inocentes Trinidad. Defendants denied that plaintiff was the son of the late Inocentes Trinidad. Defendants contended that Inocentes was single when he died in 1941, before plaintiff's birth. Defendants also denied that plaintiff had lived with them, and claimed that the parcels of land described in the complaint had been in their possession since the death of their father in 1940 and that they had not givenplaintiff a share in the produce of the land. Patricio Trinidad and Anastacia Briones were the parents of three (3) children, namely, Inocentes,Lourdes and Felix. When Patricio died in 1940, survived by the above named children, he left four (4) parcels of land, all situated at Barrio Tigayon, Kalibo Aklan. Arturio Trinidad, born on July 21, 1943, claimed to be the legitimate son of the late Inocentes Trinidad. On October 28, 1982, Felix died without issue, so he was not substituted as a party. On July 4, 1989, the trial court rendered a decision in favor of the petitioner. Respondent Court reversed the trial court on the ground that petitioner failed to adduce sufficient evidence to prove that his parents were legally married to each other and that acquisitive prescription against him had set in. Issue: Whether or not evidence of the marriage of Inocentes and Arturios filiation is sufficient. Ruling: The merits of this petition are patent. The partition of the late Patricios real properties requires preponderant proof that petitioner is a co-owner or co-heir of the decedents estate. His right as a co-owner would, in turn, depend on whether he was born during the existence of a valid and subsisting marriage between his mother (Felicidad) and his putative father (Inocentes). This Court holds that such burden was successfully discharged by petitioner and, thus, the reversal of the assailed Decision and Resolution is inevitable. In the absence of a marriage certificate, any of the four can be sufficient proof of marriage: fact of marriage ceremony, open cohabitation of the parties, birth certificate of the child, and other documents. Arturio presented the first 3. For filiation, when the birth certificate cant be produced, other evidence like the baptismal certificate, is admissible. Use of surname without objection is also presumptive evidence of legitimacy.

228

University of the Cordilleras College of Law First Year C S.Y. 2013 - 2014 Weigel vs Sempio-Dy 143 SCRA 499 G.R. No. L-53703 August 19, 1986 Full Case LILIA OLIVA WIEGEL, petitioner, vs. THE HONORABLE ALICIA V. SEMPIO-DIY (as presiding judge of the Juvenile and Domestic Relations Court of Caloocan City) and KARL HEINZ WIEGEL, respondents. PARAS, J.: In an action (Family Case No. 483) filed before the erstwhile Juvenile and Domestic Relations Court of Caloocan City, herein respondent Karl Heinz Wiegel (plaintiff therein) asked for the declaration of Nullity of his marriage (celebrated on July, 1978 at the Holy Catholic Apostolic Christian Church Branch in Makati, Metro Manila) with herein petitioner Lilia Oliva Wiegel (Lilia, for short, and defendant therein) on the ground of Lilia's previous existing marriage to one Eduardo A. Maxion, the ceremony having been performed on June 25, 1972 at our Lady of Lourdes Church in Quezon City. Lilia, while admitting the existence of said prior subsisting marriage claimed that said marriage was null and void, she and the first husband Eduardo A. Maxion having been allegedly forced to enter said marital union. In the pre-trial that ensued, the issue agreed upon by both parties was the status of the first marriage (assuming the presence of force exerted against both parties): was said prior marriage void or was it merely voidable? Contesting the validity of the pre-trial order, Lilia asked the respondent court for an opportunity to present evidence(1) that the first marriage was vitiated by force exercised upon both her and the first husband; and (2) that the first husband was at the time of the marriage in 1972 already married to someone else. Respondent judge ruled against the presentation of evidence because the existence of force exerted on both parties of the first marriage had already been agreed upon. Hence, the present petition for certiorari assailing the following Orders of therespondent Judge(1) the Order dated March 17, 1980 in which the parties were compelled to submit the case for resolution based on "agreed facts;" and (2) the Order dated April 14, 1980, denying petitioner's motion to allow her to present evidence in her favor. We find the petition devoid of merit. There is no need for petitioner to prove that her first marriage was vitiated by force committed against both parties because assuming this to be so, the marriage will not be void but merely viodable (Art. 85, Civil Code), and therefore valid until annulled. Since no annulment has yet been made, it is clear that when she married respondent she was still validly married to her first husband, consequently, her marriage to respondent is VOID (Art. 80, Civil Code). There is likewise no need of introducing evidence about the existing prior marriage of her first husband at the time they married each other, for then such a marriage though void still needs according to this Court a judicial declaration of such fact and for all legal intents and purposes she would still be regarded as a married woman at the time she contracted her marriage with respondent Karl Heinz Wiegel); accordingly, the marriage of petitioner and respondent would be regarded VOID under the law. WHEREFORE, this petition is hereby DISMISSED, for lack of merit, and the Orders complained of are hereby AFFIRMED. Costs against petitioner. SO ORDERED. Feria (Chairman), Fernan Alampay and Gutierrez, Jr., JJ., concur.

229

University of the Cordilleras College of Law First Year C S.Y. 2013 - 2014 Case Digest LILIA OLIVA WIEGEL vs. THE HONORABLE ALICIA V. SEMPIO-DIY and KARL HEINZ WIEGEL G.R. No. L-53703 August 19, 1986 PARAS, J.: Facts: Respondent Karl Heinz Wiegel asked for the declaration of Nullity of his marriage celebrated on July, 1978 at the Holy Catholic Apostolic Christian Church Branch in Makati, Metro Manila with herein petitioner Lilia Oliva Wiegel on the ground of Lilia's previous existing marriage to one Eduardo A. Maxion, the ceremony having been performed on June 25, 1972 at our Lady of Lourdes Church in Quezon City. Lilia, while admitting the existence of said prior subsisting marriage claimed that said marriage was null and void, she and the first husband Eduardo A. Maxion having been allegedly forced to enter said marital union. In the pre-trial that ensued, the issue agreed upon by both parties was the status of the first marriage, assuming the presence of force exerted against both parties: Was said prior marriage void or was it merely voidable? It was also asserted in the pre-trial that Eduardo was already married to someone else even before the marriage with Lilia. Issue: Whether or not there is a need to secure a judicial declaration of nullity of the previous marriage in order to validly enter into a subsequent marriage. Ruling: There is no need for petitioner to prove that her first marriage was vitiated by force committed against both parties because assuming this to be so, the marriage will not be void but merely voidable (Art. 85, Civil Code), and therefore valid until annulled. Since no annulment has yet been made, it is clear that when she married respondent she was still validly married to her first husband, consequently, her marriage to respondent is VOID (Art. 80, Civil Code). There is likewise no need of introducing evidence about the existing prior marriage of her first husband at the time they married each other, for then such a marriage though void still needs according to this Court a judicial declaration of such fact and for all legal intents and purposes she would still be regarded as a married woman at the time she contracted her marriage with respondent Karl Heinz Wiegel; accordingly, the marriage of petitioner and respondent would be regarded VOID under the law.

230

University of the Cordilleras College of Law First Year C S.Y. 2013 - 2014 Apiag vs Cantero 268 SCRA 198 A.M. No. MTJ-95-1070. February 12, 1997 Full Case MARIA APIAG, TERESITA CANTERO SECUROM and GLICERIO CANTERO, complainants, vs. JUDGE ESMERALDO G. CANTERO, respondent. PANGANIBAN, J.: Judges ought to be more learned than witty, more reverend than plausible, and more advised than confident. Above all things, integrity is their portion and proper virtue. The eminent Francis Bacon wrote the foregoing exhortation some 400 years ago. Today, it is still relevant and quotable. By the nature of their functions, judges are revered as models of integrity, wisdom, decorum, competence and propriety. Human as they are, however, magistrates do have their own weaknesses, frailties, mistakes and even indiscretions. In the case before us, respondent Judge Esmeraldo G. Cantero was charged administratively in the twilight of his government service, as a result of a failed love affair that happened some 46 years ago. After an otherwise unblemished record, he would have reached the compulsory retirement age of 70 years on August 8, 1997 had death not intervened a few months ago on September 26, 1996. Notwithstanding his death, this Court still resolved to rule on this case, as it may affect his retirement benefits. Antecedent Facts In a letter-complaint dated November 10, 1993, Maria Apiag Cantero with her daughter Teresita A. Cantero Sacurom and son Glicerio A. Cantero charged the respondent, Judge Esmeraldo G. Cantero of the Municipal Circuit Trial Court of Pinamungajan-Aloquinsan, Cebu, with gross misconduct for allegedly having committed bigamy and falsification of public documents. After receipt of the respondent's Comment, the Court on February 5, 1996, referred this case to Executive Judge Gualberto P. Delgado of the Regional Trial Court of Toledo City, Cebu for investigation, report and recommendation. The latter submitted his Report and Recommendation dated July 26, 1996. Thereafter, the Court referred this case also to the Office of the Court Administrator for evaluation, report and recommendation. According to the complainants: "Sometime in August 11, 1947, defendant (should be respondent) and plaintiff (should be complainant) Maria Apiag, joined together in holy matrimony in marriage after having lived together as husband and wife wherein they begot a daughter who was born on June 19, 1947, whom they named: Teresita A. Cantero; and then on October 29, 1953, Glicerio A. Cantero was born. Thereafter, defendant left the conjugal home without any apparent cause, and leaving the plaintiff Maria Apiag to raise the two children with her meager income as a public school teacher at Hinundayan, Southern Leyte. Plaintiffs suffered a lot after defendant abandoned them for no reason whatsoever. For several years, defendant was never heard of and his whereabout unknown. Few years ago, defendant surfaced at Hinundayan, Southern Leyte, whereupon, plaintiffs begged for support, however, they were ignored by defendant. x x x" On September 21, 1993, complainants, through Atty. Redentor G. Guyala, wrote a letter to respondent as follows: "Judge Esmeraldo Cantero Pinamungajan, Cebu Dear Judge Cantero: We are writing in behalf of your legal wife, Maria Apiag, and your two legitimate children by her, Teresita (Mrs. Sacurom) and Glicerio. 231

University of the Cordilleras College of Law First Year C S.Y. 2013 - 2014 It appears that sometime in the 1950's for reasons known only to you, you left your conjugal home at Hinundayan, Southern Leyte, and abandoned without any means of support your said wife and children. Since then and up to now, they have not seen or heard from you. They would wish now that you do them right by living up to your duty as husband and father to them, particularly that expressly provided under Art. 68 and Art. 195 of the Family Code (Art. 109 and 195 of the Civil Code) in relation to Art. 203 of the same Code. You will please consider this letter as a formal demand for maintenance and support for three of them, and a request that they be properly instituted and named as your compulsory heirs and legal beneficiaries in all legal documents now on file and to be filed with the Supreme Court and other agencies or offices as may be required under applicable laws, such as, the insurance (GSIS) and retirement laws. We hope this matter can be amicably settled among you, your wife and children, without having to resort to judicial recourse. Very truly yours, (SGD.) REDENTOR G. GUYALA" The letter elicited no action or response from the respondent. Subsequently, complainants learned that respondent Judge had another family. In their own words, "x x x The plaintiffs later on learned that defendant has another wife by the name of Nieves C. Ygay, a Public School teacher from Tagao, Pinamungajan, Cebu. According to some documents obtained by plaintiffs, the herein defendant and Nieves C. Ygay have children of their own, named as follows with their date of births: Noralyn Y. Cantero -- May 19, 1968; Ellen Y. Cantero -- February 4, 1970; Erwin Y. Cantero -- April 29, 1979; Onofre Y. Cantero -- June 10, 1977; and Desirie Vic Y. Cantero -- December 2, 1981. It was shocking to the senses that in all of the public documents required of defendant Judge Cantero to be filed with the Supreme Court such as his sworn statement of assets and liabilities, his personal data sheet (SC Form P. 001), income tax returns and his insurance policy with the Government Service Insurance System, defendant misrepresented himself as being married to Nieves C. Ygay, with whom he contracted a second marriage. The truth of the matter is that defendant is married to plaintiff Maria Apiag with whom they have two legitimate children, namely: Teresita A. Cantero and Glicerio A. Cantero." The respondent Judge, in his Comment, explained his side as follows: "x x x I admit the existence and form of Annex 'A' of the said complaint, but vehemently deny the validity of its due execution, for the truth of the matter is that such alleged marriage was only dramatized at the instance of our parents just to shot (sic) their wishes and purposes on the matter, without my consent freely given. As a matter of fact, I was only called by my parents to go home to our town at Hinundayan, Southern Leyte to attend party celebration of my sister's birthday from Iligan City, without patently knowing I was made to appear (in) a certain drama marriage and we were forced to acknowledge our signatures appearing in the duly prepared marriage contract(.) That was 46 years ago when I was yet 20 years of age, and at my second year high school days." Furthermore, Judge Cantero related that: "x x x sometime in the year 1947, when both respondent and complainant, Maria Apiag were still in their early age and in their second year high school days, they were engaged in a lovely affair which resulted to the pregnancy of the said complainant, and then and there gave birth to a child, named Teresita Apiag, having (been) born out of wedlock on June 19, 1947, now Mrs. Teresita Sacurom, one of the complainants. That in order to save name and shame, parents of both the respondent and the complainant came to an agreement to allow the respondent, and the complainant (to) get married in the (sic) name, but not to live together as husband, wife for being close relatives, thereby forcing the respondent to appear in a marriage affair where all the pertinent marriage papers were all ready (sic) prepared (sic), and duly signed by somebody; that after the said affair both respondent and the complainant immediately separated each other (sic) without living together as husband, and wife even for a day, nor having established a conjugal home. From that time respondent and the complainant have never met each other nor having 232

University of the Cordilleras College of Law First Year C S.Y. 2013 - 2014 (sic) communicated (with) each other for the last 40 years; that respondent continued his studies at Cebu City, and eventually became member of the Philippine Bar, having passed the bar examination in the year 1960, that is 14 years after the affair of 1947; that in 1964, respondent was first connected in the government service as Comelec Registrar of the Commission on Elections, assigned at Pinamungajan, Cebu(,) that is 16 years after the affair of 1947; that in the year 1982, respondent was appointed as CLAO lawyer, now PAO, of the Department of Justice, that is 35 years after the after the affair of 1947; and finally, on October 3, 1989, respondent was appointed to the Judiciary as Municipal Circuit Trial Judge (MCTC) of the Municipalities of Pinamungajan and Aloguinsan, province of Cebu, that is 42 years from August 11, 1947; that respondent is (sic) already 32 years in the government service up to the present time with more than 6 years in the Judiciary; that respondent is already 69 years old, having been born on August 8, 1927, and retirable by next year if God willing; that respondent has served in the government service for the last 32 years, faithfully, honestly and judiciously without any complaint whatsoever, except this instant case; that respondent as member of the Judiciary, has live-up (sic) to the standard required by the (sic) member (sic) of the bar and judiciary; that the charges against the respondent were all based or rooted from the incedent (sic) that happened on August 11, 1947 and no other; that the complainants are morally dishonest in filing the instant (case) just now, an elapsed (sic) of almost 42 years and knowing that respondent (is) retirable by next year, 1997; that this actuation is very suspicious, and intriguing; xxx xxx xxx

That complainant Maria Apiag has been living together with another man during her public service as public school teacher and have begotten a child, name (sic) Manuel Apiag and respondent promised (sic) the Honorable Court to furnish a complete paper regarding this case in order to enlighten the Honorable (Court) that, he who seek (sic) justice must seek justice with cleab (sic) hand; That respondent did not file any annullment (sic) or judicial declaration (of nullity) of the alleged marriage because it is the contention and honest belief, all the way, that the said marriage was void from the beginning, and as such nothing is to be voided or nullified, and to do so will be inconsistent with the stand of the respondent; that this instant case (was) simply filed for money consideration as reflected in their letter of demand; (t)hat as a matter of fact, respondent and the complainant have already signed a compromised (sic) agreement, copy of which hereto (sic) attached as Annex '1', stating among other things that respondent will give a monthly allowance to Terecita (sic) Sacurom in the (amount) of P4,000.00 and the complainant will withdraw their complaint from the Supreme Court., and that respondent had already given the said allowance for three consecutive months plus the amount of P25,000.00 for their Attorney to withdraw the case, and that respondent stop (sic) the monthly allowance until such time the complainant will actually withdraw the instant case, and without knowledge of the respondent, complainant proceeded (sic) their complaint after the elapsed (sic) of three (3) years." Relevant portions of said compromise agreement which was executed sometime in March 1994 by Esmeraldo C. Cantero and Teresita C. Sacurom and witnessed by Maria Apiag and Leovegardo Sacurom are reproduced thus: "That this COMPROMISE AGREEMENT is executed and entered into by ESMERALDO C. CANTERO, of legal age, married, Filipino, and with residence and postal address at Pinamungajan, Cebu, Philippines, otherwise called as the FIRST PARTY, and TERESITA C. SACUROM, also of legal age, married, Filipino, representing her mother and her brother, and a residence (sic) of 133-A J. Ramos Street, Caloocan City, after having duly sworn (sic) to in accordance with law do hereby depose and say: 1. That the First Party is presently a Municipal Circuit Trial Judge of Pinamungajan-Aloguinsan, Cebu, is charged by Second Party for Misconduct before the Office of the Court Administrator of the Supreme Court now pending action; 2. That the parties have came (sic) to agreement to have the said case settled amicably in the interest of family unity and reconciliation, and arrived at compromise agreement based on law of equity, as follows: (a) That both parties have agreed voluntarily, the Second Party will get ONE FOURTH (1/4) of the retirement that the First will receive from the GSIS, and the rest of it will be for the First Party;

233

University of the Cordilleras College of Law First Year C S.Y. 2013 - 2014 (b) That the Second Party and his brother will be included as one of the beneficiaries of the First Party, in case of death; (c) That the Second party and his only brother will inherit the properties of the First party inherited from his parents; (d) That the Second Party, representing her brother, is authorized to receive and collect P4,000.00, monthly out of the second check salary of the First Party (The second half salary only); 3. That it was further voluntarily agreed that the Second Party will cause the withdrawal and the outright dismissal of the said pending case filed by her and her mother; 4. That it was also agreed that the above agreement, shall never be effective and enforceable unless the said case will be withdrawn and dismiss (sic) from the Supreme Court, and said dismissal be received by the First Party, otherwise the above-agreement is void from the beginning; and the Second Party must desist from further claining (sic) and filing civil abd (sic) criminal liabilities. 5. That this agreement is executed voluntarily, in good faith, and in the interest of good will and reconciliation and both parties is (sic) duty bound to follow faithfully and religiously." In line with the foregoing, the respondent wrote a letter dated 14 March, 1994 addressed to the Government Service Insurance System (GSIS) designating Teresita Cantero Sacurom and Glicerio Cantero as additional beneficiaries in his life insurance policy.

The Issues The respondent Judge formulated the following "issues": "1. That the first marriage with the complainant, Maria Apiag on August 11, 1947 is void; 2. The absence of his first wife complainant Maria Apiag for more than seven (7) years raise the presumption that she is already dead, that there was no need for any judicial declaration; 3. The charge of Grave Misconduct is not applicable to him because assuming that he committed the offense, he was not yet a member of the judiciary; 4. The crime of Bigamy and Falsification had already prescribed; 5. The charges have no basis in fact and in law."

Report and Recommendation of Investigating Judge and Court Administrator Investigating Judge Gualberto P. Delgado recommended in his report that: "After a careful perusal of the evidence submitted by the parties, this Office finds respondent Guilty of the crime of Grave Misconduct (Bigamy and Falsification of Public Documents) however, considering his length of service in the government, it is recommended that he be suspended for one (1) year without pay." The Office of the Court Administrator also submitted its report recommending respondent Judge's dismissal, as follows: "After a careful review of all the documents on file in this case, we find no cogent reason to disturb the findings of the investigating judge.

234

University of the Cordilleras College of Law First Year C S.Y. 2013 - 2014 Extant from the records of the case and as admitted by respondent, he was married to complainant Maria Apiag on August 11, 1947 and have (sic) two (2) children with her. Respondent's contention that such marriage was in jest and assuming that it was valid, it has lost its validity on the ground that they never met again nor have communicated with each other for the last 40 years cannot be given a (sic) scant consideration. Respondent's argument that he was not yet a lawyer, much more, a member of the bench when he contracted his first marriage with the complainant, is unavailing for having studied law and had become a member of the Bar in 1960, he knows that the marriage cannot be dissolved without a judicial declaration of death. Respondent's second marriage with Nieves Ygay was therefore bigamous for it was contracted during the existence of a previous marriage. We are likewise not persuaded by the assertion of the respondent that he cannot be held liable for misconduct on the ground that he was not yet a lawyer nor a judge when the act(s) complained of were committed. The infraction he committed continued from the time he became a lawyer in 1960 to the time he was appointed as a judge in October 23, 1989. This is a continuing offense (an unlawful act performed continuously or over and over again, Law Dictionary, Robert E. Rothenberg). He can therefore be held liable for his misdeeds. On the charge of falsification, it was shown with clarity in his Personal Data Sheet for Judges, Sworn Statement of Assets, Liabilities and Networth, Income Tax Return (pp. 99-102, rollo), that he had committed a misrepresentation by stating therein that his spouse is Nieves Ygay and (had) eight (8) children (with her) which is far from (the) truth that his wife is Maria Apiag with whom he had two (2) children. Aside from the admission, the untenable line of defense by the respondent presupposes the imposition of an administrative sanction for the charges filed against him. 'A judge's actuation of cohabiting with another when his marriage was still valid and subsisting - his wife having been allegedly absent for four years only constitutes gross immoral conduct' (Abadilla vs. Tabiliran Jr., 249 SCRA 447). It is evident that respondent failed to meet the standard of moral fitness for membership in the legal profession. While deceit employed by respondent, existed prior to his appointment as a x x x Judge, his immoral and illegal act of cohabiting with x x x began and continued when he was already in the judiciary. A judge, in order to promote public confidence in the integrity and impartiality of the judiciary, must behave with propriety at all times, in the performance of his judicial duties and in his everyday life. These are judicial guidepost to(sic) self-evident to be overlooked. No position exacts a greater demand on moral righteousness and uprightness of an individual than a seat in the judiciary (Atienza vs. Brilliantes, Jr., 243 SCRA 32-33). ACCORDINGLY, it is respectfully recommended that respondent judge be DISMISSED from the service with forfeiture of all leave and retirement benefits and with prejudice to re-appointment in any branch, instrumentality or agency of the government, including government-owned and controlled corporations." As earlier indicated, respondent Judge died on September 27, 1996 while this case was still being deliberated upon by this Court.

The Court's Ruling In spite of his death, this Court decided to resolve this case on the merits, in view of the foregoing recommendation of the OCA which, if affirmed by this Court, would mean forfeiture of the death and retirement of the respondent.

Gross Misconduct Not Applicable The misconduct imputed by the complainants against the judge comprises the following: abandonment of his first wife and children, failing to give support, marrying for the second time without having first obtained a judicial declaration of nullity of his first marriage, and falsification of public documents. Misconduct, as a ground for administrative action, has a specific meaning in law. "'Misconduct in office has definite and well understood legal meaning. By uniform legal definition, it is a misconduct such as affects his performance of his duties as an officer and not such only as 235

University of the Cordilleras College of Law First Year C S.Y. 2013 - 2014 affects his character as a private individual. In such cases, it has been said at all times, it is necessary to separate the character of man from the character of an officer. x x x It is settled that misconduct, misfeasance, or malfeasance warranting removal from office of an officer, must have direct relation to and be connected with the performance of official duties x x x .' More specifically, in Buenaventura vs. Benedicto, an administrative proceeding against a judge of the court of first instance, the present Chief Justice defines misconduct as referring 'to a transgression of some established and definite rule of action, more particularly unlawful behavior or gross negligence by the public officer.' That is to abide by the authoritative doctrine as set forth in the leading case of In re Horilleno, a decision penned by Justice Malcolm, which requires that in order for serious misconduct to be shown, there must be 'reliable evidence showing that the judicial acts complained of were corrupt or inspired by an intention to violate the law or were in persistent disregard of wellknown legal rules.'" The acts imputed against respondent Judge Cantero clearly pertain to his personal life and have no direct relation to his judicial function. Neither do these misdeeds directly relate to the discharge of his official responsibilities. Therefore, said acts cannot be deemed misconduct much less gross misconduct in office. For any of the aforementioned acts of Judge Cantero" x x x (t)o warrant disciplinary action, the act of the judge must have a direct relation to the performance of his official duties. It is necessary to separate the character of the man from the character of the officer."

Nullity of Prior Marriage It is not disputed that respondent did not obtain a judicial declaration of nullity of his marriage to Maria Apiag prior to marrying Nieves C. Ygay. He argued however that the first marriage was void and that there was no need to have the same judicially declared void, pursuant to jurisprudence then prevailing. In the en banc case of Odayat vs. Amante, complainant charged Amante, a clerk of court, with oppression, immorality and falsification of public document. The complainant Odayat alleged among others " x x x that respondent is cohabiting with one Beatriz Jornada, with whom he begot many children, even while his spouse Filomena Abella is still alive x x x." In order to rebut the charge of immorality, Amante " x x x presented in evidence the certification (of the) x x x Local Civil Registrar x x x attesting that x x x Filomena Abella was married to one Eliseo Portales on February 16, 1948. Respondent's contention is that his marriage with Filomena Abella was void ab initio, because of her previous marriage with said Eliseo Portales." This Court ruled that "Filomena Abella's marriage with the respondent was void ab initio under Article 80 of the New Civil Code, and no judicial decree is necessary to establish the invalidity of void marriages." Now, per current jurisprudence, "a marriage though void still needs x x x a judicial declaration of such fact" before any party thereto "can marry again; otherwise, the second marriage will also be void." This was expressly provided under Article 40 of the Family Code. However, the marriage of Judge Cantero to Nieves Ygay took place and all their children were born before the promulgation of Wiegel vs. Sempio-Diy and before the effectivity of the Family Code. Hence, the doctrine in Odayat vs. Amante applies in favor of respondent. On the other hand, the charge of falsification will not prosper either because it is based on a finding of guilt in the bigamy charge. Since, as shown in the preceding discussion, the bigamy charge cannot stand, so too must the accusation of falsification fail. Furthermore, the respondent judge's belief in good faith that his first marriage was void shows his lack of malice in filling up these public documents, a valid defense in a charge of falsification of public document, which must be appreciated in his favor.

Personal Conduct of a Judge However, the absence of a finding of criminal liability on his part does not preclude this Court from finding him administratively liable for his indiscretion, which would have merited disciplinary action from this Court had death not intervened. In deciding this case, the Court emphasizes that "(t)he personal behavior of a judge, not only upon the bench but also in his everyday life, should be above reproach and free from the appearance of impropriety. He should maintain high ethical principles and sense of propriety without which he cannot preserve the faith of the people in the judiciary, so indispensable in an orderly society. For the judicial office circumscribes the personal conduct of a judge and imposes a number of restrictions thereon, which he has to observe faithfully as the price he has to pay for accepting 236

University of the Cordilleras College of Law First Year C S.Y. 2013 - 2014 and occupying an exalted position in the administration of justice." It is against this standard that we must gauge the public and private life of Judge Cantero. The conduct of the respondent judge in his personal life falls short of this standard because the record reveals he had two families. The record also shows that he did not attend to the needs, support and education of his children of his first marriage. Such is conduct unbecoming a trial magistrate. Thus, the late Judge Cantero "violated Canon 3 of the Canons of Judicial Ethics which mandates that '[a] judge's official conduct should be free from the appearance of impropriety, and his personal behavior, not only upon the bench and in the performance of judicial duties, but also in his everyday life, should be beyond reproach,' and Canon 2 of the Code of Judicial Conduct which provides that '[a] judge should avoid impropriety and the appearance of impropriety in all activities.'"

A Penalty of Suspension is Warranted Finally, the Court also scrutinized the whole of respondent's record. Other than this case, we found no trace of wrongdoing in the discharge of his judicial functions from the time of his appointment up to the filing of this administrative case, and has to all appearances lived up to the stringent standards embodied in the Code of Judicial Conduct. Considering his otherwise untarnished 32 years in government service, this Court is inclined to treat him with leniency. Man is not perfect. At one time or another, he may commit a mistake. But we should not look only at his sin. We should also consider the man's sincerity in his repentance, his genuine effort at restitution and his eventual triumph in the reformation of his life. This respondent should not be judged solely and finally by what took place some 46 years ago. He may have committed an indiscretion in the past. But having repented for it, such youthful mistake should not forever haunt him and should not totally destroy his career and render inutile his otherwise unblemished record. Indeed, it should not demolish completely what he built in his public life since then. Much less should it absolutely deprive him and/or his heirs of the rewards and fruits of his long and dedicated service in government. For these reasons, dismissal from service as recommended by the Office of the Court Administrator would be too harsh. However, we also cannot just gloss over the fact that he was remiss in attending to the needs of his children of his first marriage -- children whose filiation he did not deny. He neglected them and refused to support them until they came up with this administrative charge. For such conduct, this Court would have imposed a penalty. But in view of his death prior to the promulgation of this Decision, dismissal of the case is now in order. WHEREFORE, premises considered, this case is hereby DISMISSED. SO ORDERED.

237

University of the Cordilleras College of Law First Year C S.Y. 2013 - 2014 Case Digest MARIA APIAG vs. JUDGE ESMERALDO G. CANTERO A.M. No. MTJ-95-1070 February 12, 1997 PANGANIBAN, J.: Facts: The wedding of Maria Apiag (one of the complainants) and Esmeraldo Cantero(respondent-judge) took place on August 11, 1947. They begot two children, Teresita andGlicero (complainants). Thereafter, Esmeraldo left the conjugal home and abandoned his wife and children without any means of support. Later on, the complainants learned that Esmeraldo contracted another marriage with Nieves Ygay and they have 5 children of their own. In all the documents filed by Esmeraldo such as his sworn statement of assets and liabilities, personal data sheet, income tax return, and insurance policy with GSIS, he misrepresented himself as being married to Nieves. Herein complainants charged Esmeraldo with gross misconduct for allegedly having committed bigamy and for falsifying public documents. In his comment, Esmeraldo denied the validity of the marriage alleging that it was dramatized and that his parents called him to appear in a certain drama marriage and was forced to sign a duly prepared marriage contract. He pressed the idea that his consent was not freely given. The fact, however, is undisputed that he and Maria were engaged in a love affair which resulted in the pregnancy of the latter prior to the marriage. It is only for the preservation of the family name that their parents agreed to their marriage but not to live together as husband and wife. To bolster his defense, Esmeraldo alleged that Maria has been living with another man during her public service as a teacher and have begotten a child, named Manuel Apiag. He argued the he who seek justice must seek justice with clean hands. He didnt file any annulment or judicial declaration of the alleged marriage because he believed that said marriage was void from the beginning. Thus, nothing is to be nullified because the marriage never existed. However, in view of the complainants request in their letter to the respondent dated September 21, 1993, both parties have agreed that Teresita shall: (1) get of the retirement that Esmeraldo will receive from GSIS; (2) be included as one of the beneficiaries in case of the latters death; (3) inherit the properties of the latter; and (2) receive and collect Php4000 monthly as support Issue: WON the previous marriage of the judge was valid. Ruling: No. Since the second marriage occurred before the promulgation of Wiegel v Sempio-Diy and before the effectivity of the FC, the doctrine of Odayat v. Amante (will be applied in favor of the respondent. Although there did not exist any grave misconduct (since the acts were committed in relation to the judges personal life), he will still be held administratively liable because of his position as a judge of high principles and ethics. Man is not perfect. At one time or another, he may commit a mistake. But we should not look only at his sin. We should also consider the man's sincerity in his repentance, his genuine effort at restitution and his eventual triumph in the reformation of his life. This respondent should not be judged solely and finally by what took place some 46 years ago. He may have committed an indiscretion in the past. But having repented for it, such youthful mistake should not forever haunt him and should not totally destroy his career and render inutile his otherwise unblemished record. Indeed, it should not demolish completely what he built in his public life since then. Much less should it absolutely deprive him and/or his heirs of the rewards and fruits of his long and dedicated service in government. For these reasons, dismissal from service as recommended by the Office of the Court Administrator would be too harsh. Considering that he was remiss in attending to the needs of the children of his first marriage (whose filiations he did not deny), the court would impose a penalty. But since he is dead, the case will merely be dismissed.

238

University of the Cordilleras College of Law First Year C S.Y. 2013 - 2014 Balogbog vs Court of Appeals 269 SCRA 259 G.R. No. 83598. March 7, 1997 Full Case LEONCIA BALOGBOG and GAUDIOSO BALOGBOG, petitioners, vs HONORABLE COURT OF APPEALS, RAMONITO BALOGBOG and GENEROSO BALOGBOG, respondents. MENDOZA, J.: This is a petition for review of the decision of the Court of Appeals, affirming the decision of the Court of First Instance of Cebu City (Branch IX), declaring private respondents heirs of the deceased Basilio and Genoveva Balogbog entitled to inherit from them. The facts are as follows. Petitioners Leoncia and Gaudioso Balogbog are the children of Basilio Balogbog and Genoveva Arnibal who died intestate in 1951 and 1961, respectively. They had an older brother, Gavino, but he died in 1935, predeceasing their parents. In 1968, private respondents Ramonito and Generoso Balogbog brought an action for partition and accounting against petitioners, claiming that they were the legitimate children of Gavino by Catalina Ubas and that, as such, they were entitled to the one-third share of Gavino in the estate of their grandparents. In their answer, petitioners denied knowing private respondents. They alleged that their brother Gavino died single and without issue in their parents residence at Tag-amakan, Asturias, Cebu. In the beginning they claimed that the properties of the estate had been sold to them by their mother when she was still alive, but they later withdrew this allegation. Private respondents presented Priscilo Y. Trazo, then 81 years old, mayor of the municipality of Asturias from 1928 to 1934, who testified that he knew Gavino and Catalina to be husband and wife and Ramonito to be their first child. On cross-examination, Trazo explained that he knew Gavino and Catalina because they performed at his campaign rallies, Catalina as balitaw dancer and Gavino Balogbog as her guitarist. Trazo said he attended the wedding of Gavino and Catalina sometime in 1929, in which Rev. Father Emiliano Jomao-as officiated and Egmidio Manuel, then a municipal councilor, acted as one of the witnesses. The second witness presented was Matias Pogoy, a family friend of private respondents, who testified that private respondents are the children of Gavino and Catalina. According to him, the wedding of Gavino and Catalina was solemnized in the Catholic Church of Asturias, Cebu and that he knew this because he attended their wedding and was in fact asked by Gavino to accompany Catalina and carry her wedding dress from her residence in Camanaol to the poblacion of Asturias before the wedding day. He testified that Gavino died in 1935 in his residence at Obogon, Balamban, Cebu, in the presence of his wife. (This contradicts petitioners claim made in their answer that Gavino died in the ancestral house at Tag-amakan, Asturias.) Pogoy said he was a carpenter and he was the one who had made the coffin of Gavino. He also made the coffin of the couples son, Petronilo, who died when he was six. Catalina Ubas testified concerning her marriage to Gavino. She testified that after the wedding, she was handed a receipt, presumably the marriage certificate, by Fr. Jomao-as, but it was burned during the war. She said that she and Gavino lived together in Obogon and begot three children, namely, Ramonito, Petronilo, and Generoso. Petronilo died after an illness at the age of six. On crossexamination, she stated that after the death of Gavino, she lived in common law relation with a man for a year and then they separated. Private respondents produced a certificate from the Office of the Local Civil Registrar (Exh. P) that the Register of Marriages did not have a record of the marriage of Gavino and Catalina, another certificate from the Office of the Treasurer (Exh. L) that there was no record of the birth of Ramonito in that office and, for this reason, the record must be presumed to have been lost or destroyed during the war, and a certificate by the Parish Priest of Asturias that there was likewise no record of birth of Ramonito in the church, the records of which were either lost or destroyed during the war. (Exh. M) On the other hand, as defendant below, petitioner Leoncia Balogbog testified that Gavino died single at the family residence in Asturias. She denied that her brother had any legitimate children and stated that she did not know private respondents before this case was filed. She obtained a certificate (Exh. 10) from the Local Civil Registrar of Asturias to the effect that that office did not have a record of the names of 239

University of the Cordilleras College of Law First Year C S.Y. 2013 - 2014 Gavino and Catalina. The certificate was prepared by Assistant Municipal Treasurer Juan Maranga, who testified that there was no record of the marriage of Gavino and Catalina in the Book of Marriages between1925 to 1935. Witness Jose Narvasa testified that Gavino died single in 1935 and that Catalina lived with a certain Eleuterio Keriado after the war, although he did not know whether they were legally married. He added, however, that Catalina had children by a man she had married before the war, although he did not know the names of the children. On crossexamination, Narvasa stated that Leoncia Balogbog, who requested him to testify, was also his bondsman in a criminal case filed by a certain Mr. Cuyos. Ramonito Balogbog was presented to rebut Leoncia Balogbogs testimony. On June 15, 1973, the Court of First Instance of Cebu City rendered judgment for private respondents (plaintiffs below), ordering petitioners to render an accounting from 1960 until the finality of its judgment, to partition the estate and deliver to private respondents one-third of the estate of Basilio and Genoveva, and to pay attorneys fees and costs. Petitioners filed a motion for new trial and/or reconsideration, contending that the trial court erred in not giving weight to the certification of the Office of the Municipal Treasurer of Asturias (Exh. 10) to the effect that no marriage of Gavino and Catalina was recorded in the Book of Marriages for the years 19251935. Their motion was denied by the trial court, as was their second motion for new trial and/or reconsideration based on the church records of the parish of Asturias which did not contain the record of the alleged marriage in that church. On appeal, the Court of Appeals affirmed. It held that private respondents failed to overcome the legal presumption that a man and a woman deporting themselves as husband and wife are in fact married, that a child is presumed to be legitimate, and that things happen according to the ordinary course of nature and the ordinary habits of life. Hence, this petition. We find no reversible error committed by the Court of Appeals. First. Petitioners contend that the marriage of Gavino and Catalina should have been proven in accordance with Arts. 53 and 54 of the Civil Code of 1889 because this was the law in force at the time the alleged marriage was celebrated. Art. 53 provides that marriages celebrated under the Civil Code of 1889 should be proven only by a certified copy of the memorandum in the Civil Registry, unless the books thereof have not been kept or have been lost, or unless they are questioned in the courts, in which case any other proof, such as that of the continuous possession by parents of the status of husband and wife, may be considered, provided that the registration of the birth of their children as their legitimate children is also submitted in evidence. This Court noted long ago, however, that Arts. 42 to 107 of the Civil Code of 1889 of Spain did not take effect, having been suspended by the Governor General of the Philippines shortly after the extension of that code to this country. Consequently, Arts. 53 and 54 never came into force. Since this case was brought in the lower court in 1968, the existence of the marriage must be determined in accordance with the present Civil Code, which repealed the provisions of the former Civil Code, except as they related to vested rights, and the rules on evidence. Under the Rules of Court, the presumption is that a man and a woman conducting themselves as husband and wife are legally married. This presumption may be rebutted only by cogent proof to the contrary. In this case, petitioners claim that the certification presented by private respondents (to the effect that the record of the marriage had been lost or destroyed during the war) was belied by the production of the Book of Marriages by the assistant municipal treasurer of Asturias. Petitioners argue that this book does not contain any entry pertaining to the alleged marriage of private respondents parents. This contention has no merit. In Pugeda v. Trias, the defendants, who questioned the marriage of the plaintiffs, produced a photostatic copy of the record of marriages of the Municipality of Rosario, Cavite for the month of January, 1916, to show that there was no record of the alleged marriage. Nonetheless, evidence consisting of the testimonies of witnesses was held competent to prove the marriage. Indeed, although a marriage contract is considered primary evidence of marriage, the failure to present it is not proof that no marriage took place. Other evidence may be presented to prove marriage. Here, private respondents proved, through testimonial evidence, that Gavino and Catalina were married in 1929; that they had three children, one of whom died in infancy; that their marriage subsisted until 1935 when Gavino died; and that their children, private respondents herein, were recognized by Gavinos family and by the public as the legitimate children of Gavino. Neither is there merit in the argument that the existence of the marriage cannot be presumed because there was no evidence showing in particular that Gavino and Catalina, in the presence of two witnesses, 240

University of the Cordilleras College of Law First Year C S.Y. 2013 - 2014 declared that they were taking each other as husband and wife. An exchange of vows can be presumed to have been made from the testimonies of the witnesses who state that a wedding took place, since the very purpose for having a wedding is to exchange vows of marital commitment. It would indeed be unusual to have a wedding without an exchange of vows and quite unnatural for people not to notice its absence. The law favors the validity of marriage, because the State is interested in the preservation of the family and the sanctity of the family is a matter of constitutional concern. As stated in Adong v. Cheong Seng Gee: The basis of human society throughout the civilized world is that of marriage. Marriage in this jurisdiction is not only a civil contract, but it is a new relation, an institution in the maintenance of which the public is deeply interested. Consequently, every intendment of the law leans toward legalizing matrimony. Persons dwelling together in apparent matrimony are presumed, in the absence of any counter-presumption or evidence special to the case, to be in fact married. The reason is that such is the common order of society, and if the parties were not what they thus hold themselves out as being, they would be living in the constant violation of decency and of law. A presumption established by our Code of Civil Procedure is that a man and a woman deporting themselves as husband and wife have entered into a lawful contract of marriage. (Sec. 334, No. 28) Semper praesumitur pro matrimonio Always presume marriage. (U.S. vs. Villafuerte and Rabano [1905], 4 Phil., 476; Son Cui vs. Guepangco, supra; U.S. vs. Memoracion and Uri [1916], 34 Phil., 633; Teter vs. Teter [1884], 101 Ind., 129.) Second. Petitioners contend that private respondents reliance solely on testimonial evidence to support their claim that private respondents had been in the continuous possession of the status of legitimate children is contrary to Art. 265 of the Civil Code which provides that such status shall be proven by the record of birth in the Civil Register, by an authentic document or by final judgment. But in accordance with Arts. 266 and 267, in the absence of titles indicated in Art. 265, the filiation of children may be proven by continuous possession of the status of a legitimate child and by any other means allowed by the Rules of Court or special laws. Thus the Civil Code provides: ART. 266. In the absence of the titles indicated in the preceding article, the filiation shall be proved by the continuous possession of status of a legitimate child ART. 267. In the absence of a record of birth, authentic document, final judgment or possession of status, legitimate filiation may be proved by any other means allowed by the Rules of Court and special laws. Petitioners contend that there is no justification for presenting testimonies as to the possession by private respondents of the status of legitimate children because the Book of Marriages for the years 19281929 is available. What is in issue, however, is not the marriage of Gavino and Catalina but the filiation of private respondents as their children. The marriage of Gavino and Catalina has already been shown in the preceding discussion. The treasurer of Asturias, Cebu certified that the records of birth of that municipality for the year 1930 could not be found, presumably because they were lost or destroyed during the war (Exh. L). But Matias Pogoy testified that Gavino and Catalina begot three children, one of whom, Petronilo, died at the age of six. Catalina testified that private respondents Ramonito and Generoso are her children by Gavino Balogbog. That private respondents are the children of Gavino and Catalina Balogbog cannot therefore be doubted. Moreover, the evidence in the record shows that petitioner Gaudioso Balogbog admitted to the police of Balamban, Cebu that Ramonito is his nephew. As the Court of Appeals found: Ironically, it is appellant Gaudioso himself who supplies the clincher that tips the balance in favor of the appellees. In an investigation before the Police Investigating Committee of Balamban, Cebu, held on March 8, 1968, conducted for the purpose of inquiring into a complaint filed by Ramonito against a patrolman of the Balamban police force, Gaudioso testified that the complainant in that administrative case is his nephew. Excerpts from the transcript of the proceedings conducted on that date ( Exhs. N, N-1, N-2, N-3 and N-4) read: Atty. Q.A.Kiamco - May it please this investigative body. Do you know the complainant in this Administrative Case No. 1? Yes I know. 241

Q.A.Q.A.- Yes.

University of the Cordilleras College of Law First Year C S.Y. 2013 - 2014 Why do you know him? I know because he is my nephew. Are you in good terms with your nephew, the complainant?

Q.- Do you mean to say that you are close to him? A.Yes. We are close. Q.- Why do you say you are close? A.- We are close because aside from the fact that he is my nephew we were also leaving (sic) in the same house in Butuan City, and I even barrow (sic) from him money in the amount of P300.00, when I return to Balamban, Cebu. xxx Q.A.- Because he is the son of my elder brother. This admission of relationship is admissible against Gaudioso although made in another case. It is considered as a reliable declaration against interest (Rule 130, Section 22). Significantly, Gaudioso did not try to offer any explanation to blunt the effects of that declaration. He did not even testify during the trial. Such silence can only mean that Ramonito is indeed the nephew of Gaudioso, the former being the son of Gavino. WHEREFORE, the decision appealed from is AFFIRMED. SO ORDERED. Regalado, (Chairman), Romero, Puno, and Torres, Jr., JJ., concur xxx Why is Ramonito Balogbog your nephew? xxx

242

University of the Cordilleras College of Law First Year C S.Y. 2013 - 2014 Case Digest LEONCIA BALOGBOG and GAUDIOSO BALOGBOG vs. COURT OF APPEALS G.R. No. 83598 March 7, 1997 MENDOZA, J.: Facts: In 1968, the respondents brought an action for the partition and accounting, claiming that they were the legitimate children of Gavino, the brother of the petitioners who died in 1935 and as such they were entitled to the one third share Gavino in Basilios estate. The petitioners denied knowing the respondents. They alleged that Gavino died single and that they are not aware that he has two sons. The petitioners further questioned the validity of marriage between their brother and Catalina. On the trial of the case, petitioners presented witnesses that will prove the invalidity of the marriage between their brother and Catalina. They were contending that the marriage was not valid because there was no record showing in the Local Civil Registry that a marriage transpired between them. The respondent, on the other hand, averred that records were destroyed during the time war as well as the certificate that there was no record of birth of Ramonito which were presumable lost or destroyed. Issue: Whether or not the marriage between Gavino and Catalina is valid even in the absence of a marriage certificate. Whether or not Ramonito and Generoso are legitimate children of Gavino and Catalina Ruling: Yes. Under the Rules of Court, the presumption is that a man and a woman conducting themselves as husband and wife are legally married. This presumption may be rebutted when there is a proof to the contrary. Although a marriage contract is considered primary evidence of marriage, the failure to present such is not a proof that no marriage took place. Other pieces of evidence may be shown prove the marriage. In the case at bar, testimonial evidence was presented. Yes. The Supreme Court held that the fact the there was no record of birth in the civil registry does not mean that the private respondents were not legitimate children. The legitimacy was proved by the testimonies of the witnesses including Catalina. Moreover, although made in another case, Gaudioso admitted that Ramonito is his nephew.

243

University of the Cordilleras College of Law First Year C S.Y. 2013 - 2014 Tolentino vs Paras 122 SCRA 525 G.R. No. L-43905 May 30, 1983 Full Case SERAFIA G. TOLENTINO, petitioner, vs. HON. EDGARDO L. PARAS, MARIA CLEMENTE and THE LOCAL CIVIL REGISTRAR OF PAOMBONG, BULACAN, respondents. MELENCIO-HERRERA, J.: The reversal of respondent Court's Order, dismissing petitioner's suit for her "declaration ... as the lawful surviving spouse of deceased Amado Tolentino and the correction of the death certificate of the same", is sought in this Petition for Review on Certiorari. The records disclose that Amado Tolentino had contracted a second marriage with private respondent herein, Maria Clemente, at Paombong, Bulacan, on November 1, 1948 (Annex "C", Petition), while his marriage with petitioner, Serafia G. Tolentino, celebrated on July 31, 1943, was still subsisting (Annex "A", Petition). Petitioner charged Amado with Bigamy in Criminal Case No. 2768 of the Court of First Instance of Bulacan, Branch II, which Court, upon Amado's plea of guilty, sentenced him to suffer the corresponding penalty. After Amado had served the prison sentence imposed on him, he continued to live with private respondent until his death on July 25, 1974. His death certificate carried the entry "Name of Surviving Spouse Maria Clemente." In Special Proceedings No. 1587-M for Correction of Entry, petitioner sought to correct the name of the surviving spouse in the death certificate from "Maria Clemente" to "Serafia G. Tolentino", her name. The lower Court dismissed the petition "for lack of the proper requisites under the law" and indicated the need for a more detailed proceeding, Conformably thereto, petitioner filed the case below against private respondent and the Local Civil Registrar of Paombong, Bulacan, for her declaration as the lawful surviving spouse, and the correction of the death certificate of Amado. In an Order, dated October 21, 1976, respondent Court, upon private respondent's instance, dismissed the case, stating: The Motion to Dismiss filed by the defendants in this case, thru counsel Atty. Hernan E. Arceo, for the reasons therein mentioned, is hereby GRANTED. Further: (1) the correction of the entry in the Office of the Local Civil Registrar is not the proper remedy because the issue involved is marital relationship; (2) the Court has not acquired proper jurisdiction because as prescribed under Art. 108, read together with Art. 412 of the Civil Code publication is needed in a case like this, and up to now, there has been no such publication; and (3) in a sense, the subject matter of this case has been aptly discussed in Special Proceeding No. 1587-M, which this Court has already dismissed, for lack of the proper requisites under the law. In view of the above dismissal, all other motions in this case are hereby considered MOOT and ACADEMIC. SO ORDERED. Thus, petitioner's present recourse mainly challenging the grounds relied upon by respondent Court in ordering dismissal. We rule for petitioner. First, for the remedy. Although petitioner's ultimate objective is the correction of entry contemplated in Article 412 of the Civil Code and Rule 108 of the Rules of Court, she initially seeks a judicial declaration that she is the lawful surviving spouse of the deceased, Amado, in order to lay the basis for the correction 244

University of the Cordilleras College of Law First Year C S.Y. 2013 - 2014 of the entry in the death certificate of said deceased. The suit below is a proper remedy. It is of an adversary character as contrasted to a mere summary proceeding. A claim of right is asserted against one who has an interest in contesting it. Private respondent, as the individual most affected; is a party defendant, and has appeared to contest the petition and defend her interests. The Local Civil Registrar is also a party defendant. The publication required by the Court below pursuant to Rule 108 of the Rules of Court is not absolutely necessary for no other parties are involved. After all, publication is required to bar indifferently all who might be minded to make an objection of any sort against the right sought to be established. Besides, even assuming that this is a proceeding under Rule 108, it was the Court that was caned upon to order the publication, but it did not. in the ultimate analysis, Courts are not concerned so much with the form of actions as with their substance. Second, for the merits. Considering that Amado, upon his own plea, was convicted for Bigamy, that sentence furnishes the necessary proof of the marital status of petitioner and the deceased. There is no better proof of marriage than the admission by the accused of the existence of such marriage. The second marriage that he contracted with private respondent during the lifetime of his first spouse is null and void from the beginning and of no force and effect. No judicial decree is necessary to establish the invalidity of a void marriage. It can be safely concluded, then, without need of further proof nor remand to the Court below, that private respondent is not the surviving spouse of the deceased Amado, but petitioner. Rectification of the erroneous entry in the records of the Local Civil Registrar may, therefore, be validly made. Having arrived at the foregoing conclusion, the other issues raised need no longer be discussed. In fine, since there is no question regarding the invalidity of Amado's second marriage with private respondent and that the entry made in the corresponding local register is thereby rendered false, it may be corrected. While document such as death and birth certificates, are public and entries therein are presumed to be correct, such presumption is merely disputable and will have to yield to more positive evidence establishing their inaccuracy. WHEREFORE, the Order, dated October 21, 1975, of respondent Court is hereby set aside and petitioner, Serafia G. Tolentino, hereby declared the surviving spouse of the deceased Amado Tolentino. Let the corresponding correction be made in the latter's death certificate in the records of the Local Civil Registrar of Paombong, Bulacan. No costs. SOORDERED.

245

University of the Cordilleras College of Law First Year C S.Y. 2013 - 2014 Case Digest SERAFIA G. TOLENTINO vs. HON. EDGARDO L. PARAS G.R. No. L-43905 May 30, 1983 MELENCIO-HERRERA, J.: Facts: Amado Tolentino had contracted a second marriage with private respondent herein, Maria Clemente, at Paombong, Bulacan, on November 1, 1948, while his marriage with petitioner, Serafia G. Tolentino, celebrated on July 31, 1943, was still subsisting. Petitioner charged Amado with Bigamy in Criminal Case No. 2768 of the Court of First Instance of Bulacan, Branch II, which Court, upon Amado's plea of guilty, sentenced him to suffer the corresponding penalty. After Amado had served the prison sentence imposed on him, he continued to live with private respondent until his death on July 25, 1974. His death certificate carried the entry "Name of Surviving Spouse Maria Clemente." In Special Proceedings No. 1587-M for Correction of Entry, petitioner sought to correct the name of the surviving spouse in the death certificate from "Maria Clemente" to "Serafia G. Tolentino", her name. The lower Court dismissed the petition "for lack of the proper requisites under the law" and indicated the need for a more detailed proceeding. Conformably thereto, petitioner filed the case below against private respondent and the Local Civil Registrar of Paombong, Bulacan, for her declaration as the lawful surviving spouse, and the correction of the death certificate of Amado. In an Order, dated October 21, 1976, respondent Court, upon private respondent's instance, dismissed the case (1) the correction of the entry in the Office of the Local Civil Registrar is not the proper remedy because the issue involved is marital relationship; (2) the Court has not acquired proper jurisdiction because as prescribed under Art. 108, read together with Art. 412 of the Civil Code publication is needed in a case like this, and up to now, there has been no such publication; and (3) in a sense, the subject matter of this case has been aptly discussed in Special Proceeding No. 1587-M, which this Court has already dismissed, for lack of the proper requisites under the law. Issue: Whether or not the petitioner is the lawful surviving spouse of the deceased. Whether or not the subsequent marriage is null and void. Ruling: We rule for petitioner. The suit below is a proper remedy. It is of an adversary character as contrasted to a mere summary proceeding. A claim of right is asserted against one who has an interest in contesting it. Private respondent, as the individual most affected; is a party defendant, and has appeared to contest the petition and defend her interests. The Local Civil Registrar is also a party defendant. The publication required by the Court below pursuant to Rule 108 of the Rules of Court is not absolutely necessary for no other parties are involved. After all, publication is required to bar indifferently all who might be minded to make an objection of any sort against the right sought to be established. Considering that Amado, upon his own plea, was convicted for Bigamy, that sentence furnishes the necessary proof of the marital status of petitioner and the deceased. There is no better proof of marriage than the admission by the accused of the existence of such marriage. The second marriage that he contracted with private respondent during the lifetime of his first spouse is null and void from the beginning and of no force and effect. No judicial decree is necessary to establish the invalidity of a void marriage. It can be safely concluded, then, without need of further proof nor is remand to the Court below that private respondent not the surviving spouse of the deceased Amado, but petitioner. Rectification of the erroneous entry in the records of the Local Civil Registrar may, therefore, be validly made.

246

University of the Cordilleras College of Law First Year C S.Y. 2013 - 2014 Navarro vs Domagtoy 259 SCRA 129 A.M. No. MTJ-96-1088. July 19, 1996 Full Case RODOLFO G. NAVARRO, complainant, Vs JUDGE HERNANDO C. DOMAGTOY, respondent. ROMERO, J.: The complainant in this administrative case is the Municipal Mayor of Dapa, Surigao del Norte, Rodolfo G. Navarro. He has submitted evidence in relation to two specific acts committed by respondent Municipal Circuit Trial Court Judge Hernando Domagtoy, which, he contends, exhibits gross misconduct as well as inefficiency in office and ignorance of the law. First, on September 27, 1994, respondent judge solemnized the wedding between Gaspar A. Tagadan and Arlyn F. Borga, despite the knowledge that the groom is merely separated from his first wife. Second, it is alleged that he performed a marriage ceremony between Floriano Dador Sumaylo and Gemma G. del Rosario outside his court's jurisdiction on October 27, 1994. Respondent judge holds office and has jurisdiction in the Municipal Circuit Trial Court of Sta. Monica-Burgos, Surigao del Norte. The wedding was solemnized at the respondent judge's residence in the municipality of Dapa, which does not fall within his jurisdictional area of the municipalities of Sta. Monica and Burgos, located some 40 to 45 kilometers away from the municipality of Dapa, Surigao del Norte. In his letter-comment to the Office of the Court Administrator, respondent judge avers that the office and name of the Municipal Mayor of Dapa have been used by someone else, who, as the mayor's "lackey," is overly concerned with his actuations both as judge and as a private person. The same person had earlier filed Administrative Matter No. 94-980-MTC, which was dismissed for lack of merit on September 15, 1994, and Administrative Matter No. OCA-IPI-95-16, "Antonio Adapon v. Judge Hernando C. Domagtoy," which is still pending. In relation to the charges against him, respondent judge seeks exculpation from his act of having solemnized the marriage between Gaspar Tagadan, a married man separated from his wife, and Arlyn F. Borga by stating that he merely relied on the Affidavit issued by the Municipal Trial Judge of Basey, Samar, confirming the fact that Mr. Tagadan and his first wife have not seen each other for almost seven years. With respect to the second charge, he maintains that in solemnizing the marriage between Sumaylo and del Rosario, he did not violate Article 7, paragraph 1 of the Family Code which states that: "Marriage may be solemnized by: (1) Any incumbent member of the judiciary within the court's jurisdiction; and that Article 8 thereof applies to the case in question. The complaint was not referred, as is usual, for investigation, since the pleadings submitted were considered sufficient for a resolution of the case. Since the countercharges of sinister motives and fraud on the part of complainant have not been sufficiently proven, they will not be dwelt upon. The acts complained of and respondent judge's answer thereto will suffice and can be objectively assessed by themselves to prove the latter's malfeasance. The certified true copy of the marriage contract between Gaspar Tagadan and Arlyn Borga states that Tagadan's civil status is "separated." Despite this declaration, the wedding ceremony was solemnized by respondent judge. He presented in evidence a joint affidavit by Maurecio A. Labado, Sr. and Eugenio Bullecer, subscribed and sworn to before Judge Demosthenes C. Duquilla, Municipal Trial Judge of Basey, Samar. The affidavit was not issued by the latter judge, as claimed by respondent judge, but merely acknowledged before him. In their affidavit, the affiants stated that they knew Gaspar Tagadan to have been civilly married to Ida D. Pearanda in September 1983; that after thirteen years of cohabitation and having borne five children, Ida Pearanda left the conjugal dwelling in Valencia, Bukidnon and that she has not returned nor been heard of for almost seven years, thereby giving rise to the presumption that she is already dead. In effect, Judge Domagtoy maintains that the aforementioned joint affidavit is sufficient proof of Ida Pearanda's presumptive death, and ample reason for him to proceed with the marriage ceremony. We do not agree. Article 41 of the Family Code expressly provides: 247

University of the Cordilleras College of Law First Year C S.Y. 2013 - 2014 "A marriage contracted by any person during the subsistence of a previous marriage shall be null and void, unless before the celebration of the subsequent marriage, the prior spouse had been absent for four consecutive years and the spouse present had a well-founded belief that the absent spouse was already dead. In case of disappearance where there is danger of death under the circumstances set forth in the provisions of Articles 391 of the Civil Code, an absence of only two years shall be sufficient. For the purpose of contracting the subsequent marriage under the preceding paragraph, the spouse present must institute a summary proceeding as provided in this Code for the declaration of presumptive death of the absentee, without prejudice to the effect of reappearance of the absent spouse." (Emphasis added.) There is nothing ambiguous or difficult to comprehend in this provision. In fact, the law is clear and simple. Even if the spouse present has a well-founded belief that the absent spouse was already dead, a summary proceeding for the declaration of presumptive death is necessary in order to contract a subsequent marriage, a mandatory requirement which has been precisely incorporated into the Family Code to discourage subsequent marriages where it is not proven that the previous marriage has been dissolved or a missing spouse is factually or presumptively dead, in accordance with pertinent provisions of law. In the case at bar, Gaspar Tagadan did not institute a summary proceeding for the declaration of his first wife's presumptive death. Absent this judicial declaration, he remains married to Ida Pearanda. Whether wittingly, or unwittingly, it was manifest error on the part of respondent judge to have accepted the joint affidavit submitted by the groom. Such neglect or ignorance of the law has resulted in a bigamous, and therefore void, marriage. Under Article 35 of the Family Code, "The following marriage shall be void from the beginning: (4) Those bigamous x x x marriages not falling under Article 41." The second issue involves the solemnization of a marriage ceremony outside the court's jurisdiction, covered by Articles 7 and 8 of the Family Code, thus: "Art. 7. (1) Marriage may be solemnized by:

Any incumbent member of the judiciary within the court's jurisdiction; xxx xxx xxx (Emphasis supplied.)

Art. 8. The marriage shall be solemnized publicly in the chambers of the judge or in open court, in the church, chapel or temple, or in the office of the consul-general, consul or vice-consul, as the case may be, and not elsewhere, except in cases of marriages contracted on the point of death or in remote places in accordance with Article 29 of this Code, or where both parties request the solemnizing officer in writing in which case the marriage may be solemnized at a house or place designated by them in a sworn statement to that effect." Respondent judge points to Article 8 and its exceptions as the justifications for his having solemnized the marriage between Floriano Sumaylo and Gemma del Rosario outside of his court's jurisdiction. As the aforequoted provision states, a marriage can be held outside of the judge's chambers or courtroom only in the following instances: (1) at the point of death, (2) in remote places in accordance with Article 29 or (3) upon request of both parties in writing in a sworn statement to this effect. There is no pretense that either Sumaylo or del Rosario was at the point of death or in a remote place. Moreover, the written request presented addressed to the respondent judge was made by only one party, Gemma del Rosario. More importantly, the elementary principle underlying this provision is the authority of the solemnizing judge. Under Article 3, one of the formal requisites of marriage is the "authority of the solemnizing officer." Under Article 7, marriage may be solemnized by, among others, "any incumbent member of the judiciary within the court's jurisdiction." Article 8, which is a directory provision, refers only to the venue of the marriage ceremony and does not alter or qualify the authority of the solemnizing officer as provided in the preceding provision. Non-compliance herewith will not invalidate the marriage. A priest who is commissioned and allowed by his local ordinary to marry the faithful, is authorized to do so only within the area of the diocese or place allowed by his Bishop. An appellate court Justice or a Justice of this Court has jurisdiction over the entire Philippines to solemnize marriages, regardless of the venue, as long as the requisites of the law are complied with. However, judges who are appointed to 248

University of the Cordilleras College of Law First Year C S.Y. 2013 - 2014 specific jurisdictions, may officiate in weddings only within said areas and not beyond. Where a judge solemnizes a marriage outside his court's jurisdiction, there is a resultant irregularity in the formal requisite laid down in Article 3, which while it may not affect the validity of the marriage, may subject the officiating official to administrative liability. Inasmuch as respondent judge's jurisdiction covers the municipalities of Sta. Monica and Burgos, he was not clothed with authority to solemnize a marriage in the municipality of Dapa, Surigao del Norte. By citing Article 8 and the exceptions therein as grounds for the exercise of his misplaced authority, respondent judge again demonstrated a lack of understanding of the basic principles of civil law. Accordingly, the Court finds respondent to have acted in gross ignorance of the law. The legal principles applicable in the cases brought to our attention are elementary and uncomplicated, prompting us to conclude that respondent's failure to apply them is due to a lack of comprehension of the law. The judiciary should be composed of persons who, if not experts, are at least, proficient in the law they are sworn to apply, more than the ordinary laymen. They should be skilled and competent in understanding and applying the law. It is imperative that they be conversant with basic legal principles like the ones involved in instant case. It is not too much to expect them to know and apply the law intelligently. Otherwise, the system of justice rests on a shaky foundation indeed, compounded by the errors committed by those not learned in the law. While magistrates may at times make mistakes in judgment, for which they are not penalized, the respondent judge exhibited ignorance of elementary provisions of law, in an area which has greatly prejudiced the status of married persons. The marriage between Gaspar Tagadan and Arlyn Borga is considered bigamous and void, there being a subsisting marriage between Gaspar Tagadan and Ida Pearanda. The Office of the Court Administrator recommends, in its Memorandum to the Court, a six-month suspension and a stern warning that a repetition of the same or similar acts will be dealt with more severely. Considering that one of the marriages in question resulted in a bigamous union and therefore void, and the other lacked the necessary authority of respondent judge, the Court adopts said recommendation. Respondent is advised to be more circumspect in applying the law and to cultivate a deeper understanding of the law. IN VIEW OF THE FOREGOING, respondent Judge Hernando C. Domagtoy is hereby SUSPENDED for a period of six (6) months and given a STERN WARNING that a repetition of the same or similar acts will be dealt with more severely. SO ORDERED.

249

University of the Cordilleras College of Law First Year C S.Y. 2013 - 2014 Case Digest RODOLFO G. NAVARRO vs. JUDGE HERNANDO C. DOMAGTOY A.M. No. MTJ-96-1088 July 19, 1996 ROMERO, J.: Facts: On September 27, 1994, respondent judge solemnized the wedding between Gaspar A. Tagadan and Arlyn F. Borga, despite the knowledge that the groom is merely separated from his first wife. It is also alleged that he performed a marriage ceremony between Floriano Dador Sumaylo and Gemma G. del Rosario outside his court's jurisdiction on October 27, 1994. Respondent judge holds office and has jurisdiction in the Municipal Circuit Trial Court of Sta. Monica-Burgos, Surigao del Norte. The wedding was solemnized at the respondent judge's residence in the municipality of Dapa, which does not fall within his jurisdictional area of the municipalities of Sta. Monica and Burgos, located some 40 to 45 kilometers away from the municipality of Dapa, Surigao del Norte. Respondent judge now seeks exculpation from his act of having solemnized the marriage between Gaspar Tagadan, a married man separated from his wife, and Arlyn F. Borga by stating that he merely relied on the Affidavit issued by the Municipal Trial Judge of Basey, Samar, which on the other hand was just acknowledged and not a issued by the latter, confirming the fact that Mr. Tagadan and his first wife have not seen each other for almost seven years because the wife, Ida Penarada left the conjugal dwelling after 13 years of cohabitation thereby giving rise to the presumption the she is dead. With respect to the second charge, he maintains that in solemnizing the marriage between Sumaylo and del Rosario, he did not violate Article 7, paragraph 1 of the Family Code which states that: "Marriage may be solemnized by: (1) Any incumbent member of the judiciary within the court's jurisdiction; and that Article 8 thereof applies to the case in question. Issue: Whether or not there is a need for the declaration of presumptive death in order to contract a subsequent marriage Whether or not the judge is correct in solemnizing the marriage outside the courts jurisdiction Ruling: For the purpose of contracting the subsequent marriage under Art. 41, the spouse present must institute a summary proceeding as provided in this Code for the declaration of presumptive death of the absentee, without prejudice to the effect of reappearance of the absent spouse. In the case at bar, Gaspar Tagadan did not institute a summary proceeding for the declaration of his first wife's presumptive death. Absent this judicial declaration, he remains married to Ida Pearanda. Whether wittingly, or unwittingly, it was manifest error on the part of respondent judge to have accepted the joint affidavit submitted by the groom. Such neglect or ignorance of the law has resulted in a bigamous, and therefore void, marriage. Under Article 35 of the Family Code, "The following marriage shall be void from the beginning: (4) Those bigamous marriages not falling under Article 41." The marriage between Gaspar Tagadan and Arlyn Borga is considered bigamous and void, there being a subsisting marriage between Gaspar Tagadan and Ida Pearanda. On the second issue, Art. 8 of the Family Code provides for the exception where the marriage can be solemnized outside the courts jurisdiction namely (1) at the point of death, (2) in remote places in accordance with Article 29 or (3) upon request of both parties in writing in a sworn statement to this effect. In the case at bar, there is no pretense that either Sumaylo or del Rosario was at the point of death or in a remote place. Moreover, the written request presented addressed to the respondent judge was made by only one party, Gemma del Rosario. Respondent judge's jurisdiction covers the municipalities of Sta. Monica and Burgos, he was not clothed with authority to solemnize a marriage in the municipality of Dapa, Surigao del Norte. By citing Article 8 and the exceptions therein as grounds for the exercise of his misplaced authority, respondent judge again demonstrated a lack of understanding of the basic principles of civil law.

250

University of the Cordilleras College of Law First Year C S.Y. 2013 - 2014 Respondent Judge Hernando C. Domagtoy is hereby SUSPENDED for a period of six (6) months and given a stern warning that a repetition of the same or similar acts will be dealt with more severely.

251

University of the Cordilleras College of Law First Year C S.Y. 2013 - 2014 Araes vs Occiano 380 SCRA 402 A.M. No. MTJ-02-1390 April 11, 2002 Full Case MERCEDITA MATA ARAES, petitioner, vs. JUDGE SALVADOR M. OCCIANO, respondent. PUNO, J.: Petitioner Mercedita Mata Araes charges respondent judge with Gross Ignorance of the Law via a sworn Letter-Complaint dated 23 May 2001. Respondent is the Presiding Judge of the Municipal Trial Court of Balatan, Camarines Sur. Petitioner alleges that on 17 February 2000, respondent judge solemnized her marriage to her late groom Dominador B. Orobia without the requisite marriage license and at Nabua, Camarines Sur which is outside his territorial jurisdiction. They lived together as husband and wife on the strength of this marriage until her husband passed away. However, since the marriage was a nullity, petitioner's right to inherit the "vast properties" left by Orobia was not recognized. She was likewise deprived of receiving the pensions of Orobia, a retired Commodore of the Philippine Navy. Petitioner prays that sanctions be imposed against respondent judge for his illegal acts and unethical misrepresentations which allegedly caused her so much hardships, embarrassment and sufferings. On 28 May 2001, the case was referred by the Office of the Chief Justice to then Acting Court Administrator Zenaida N. Elepao for appropriate action. On 8 June 2001, the Office of the Court Administrator required respondent judge to comment. In his Comment dated 5 July 2001, respondent judge averred that he was requested by a certain Juan Arroyo on 15 February 2000 to solemnize the marriage of the parties on 17 February 2000. Having been assured that all the documents to the marriage were complete, he agreed to solemnize the marriage in his sala at the Municipal Trial Court of Balatan, Camarines Sur. However, on 17 February 2000, Arroyo informed him that Orobia had a difficulty walking and could not stand the rigors of travelling to Balatan which is located almost 25 kilometers from his residence in Nabua. Arroyo then requested if respondent judge could solemnize the marriage in Nabua, to which request he acceded. Respondent judge further avers that before he started the ceremony, he carefully examined the documents submitted to him by petitioner. When he discovered that the parties did not possess the requisite marriage license, he refused to solemnize the marriage and suggested its resetting to another date. However, due to the earnest pleas of the parties, the influx of visitors, and the delivery of provisions for the occasion, he proceeded to solemnize the marriage out of human compassion. He also feared that if he reset the wedding, it might aggravate the physical condition of Orobia who just suffered from a stroke. After the solemnization, he reiterated the necessity for the marriage license and admonished the parties that their failure to give it would render the marriage void. Petitioner and Orobia assured respondent judge that they would give the license to him in the afternoon of that same day. When they failed to comply, respondent judge followed it up with Arroyo but the latter only gave him the same reassurance that the marriage license would be delivered to his sala at the Municipal Trial Court of Balatan, Camarines Sur. Respondent judge vigorously denies that he told the contracting parties that their marriage is valid despite the absence of a marriage license. He attributes the hardships and embarrassment suffered by the petitioner as due to her own fault and negligence. On 12 September 2001, petitioner filed her Affidavit of Desistance dated 28 August 2001 with the Office of the Court Administrator. She attested that respondent judge initially refused to solemnize her marriage due to the want of a duly issued marriage license and that it was because of her prodding and reassurances that he eventually solemnized the same. She confessed that she filed this administrative case out of rage.

252

University of the Cordilleras College of Law First Year C S.Y. 2013 - 2014 However, after reading the Comment filed by respondent judge, she realized her own shortcomings and is now bothered by her conscience. Reviewing the records of the case, it appears that petitioner and Orobia filed their Application for Marriage License on 5 January 2000. It was stamped in this Application that the marriage license shall be issued on 17 January 2000. However, neither petitioner nor Orobia claimed it. It also appears that the Office of the Civil Registrar General issued a Certification that it has no record of such marriage that allegedly took place on 17 February 2000. Likewise, the Office of the Local Civil Registrar of Nabua, Camarines Sur issued another Certification dated 7 May 2001 that it cannot issue a true copy of the Marriage Contract of the parties since it has no record of their marriage. On 8 May 2001, petitioner sought the assistance of respondent judge so the latter could communicate with the Office of the Local Civil Registrar of Nabua, Camarines Sur for the issuance of her marriage license. Respondent judge wrote the Local Civil Registrar of Nabua, Camarines Sur. In a letter dated 9 May 2001, a Clerk of said office, Grace T. Escobal, informed respondent judge that their office cannot issue the marriage license due to the failure of Orobia to submit the Death Certificate of his previous spouse. The Office of the Court Administrator, in its Report and Recommendation dated 15 November 2000, found the respondent judge guilty of solemnizing a marriage without a duly issued marriage license and for doing so outside his territorial jurisdiction. A fine of P5,000.00 was recommended to be imposed on respondent judge. We agree. Under the Judiciary Reorganization Act of 1980, or B.P. 129, the authority of the regional trial court judges and judges of inferior courts to solemnize marriages is confined to their territorial jurisdiction as defined by the Supreme Court. The case at bar is not without precedent. In Navarro vs. Domagtoy,1 respondent judge held office and had jurisdiction in the Municipal Circuit Trial Court of Sta. Monica-Burgos, Surigao del Norte. However, he solemnized a wedding at his residence in the municipality of Dapa, Surigao del Norte which did not fall within the jurisdictional area of the municipalities of Sta. Monica and Burgos. We held that: "A priest who is commissioned and allowed by his local ordinance to marry the faithful is authorized to do so only within the area or diocese or place allowed by his Bishop. An appellate court Justice or a Justice of this Court has jurisdiction over the entire Philippines to solemnize marriages, regardless of the venue, as long as the requisites of the law are complied with. However, judges who are appointed to specific jurisdictions, may officiate in weddings only within said areas and not beyond. Where a judge solemnizes a marriage outside his court's jurisdiction, there is a resultant irregularity in the formal requisite laid down in Article 3, which while it may not affect the validity of the marriage, may subject the officiating official to administrative liability."2 (Emphasis supplied.) In said case, we suspended respondent judge for six (6) months on the ground that his act of solemnizing a marriage outside his jurisdiction constitutes gross ignorance of the law. We further held that: "The judiciary should be composed of persons who, if not experts, are at least, proficient in the law they are sworn to apply, more than the ordinary laymen. They should be skilled and competent in understanding and applying the law. It is imperative that they be conversant with basic legal principles like the ones involved in the instant case. x x x While magistrates may at times make mistakes in judgment, for which they are not penalized, the respondent judge exhibited ignorance of elementary provisions of law, in an area which has greatly prejudiced the status of married persons."3 In the case at bar, the territorial jurisdiction of respondent judge is limited to the municipality of Balatan, Camarines Sur. His act of solemnizing the marriage of petitioner and Orobia in Nabua, Camarines Sur therefore is contrary to law and subjects him to administrative liability. His act may not amount to gross

253

University of the Cordilleras College of Law First Year C S.Y. 2013 - 2014 ignorance of the law for he allegedly solemnized the marriage out of human compassion but nonetheless, he cannot avoid liability for violating the law on marriage. Respondent judge should also be faulted for solemnizing a marriage without the requisite marriage license. InPeople vs. Lara,4 we held that a marriage which preceded the issuance of the marriage license is void, and that the subsequent issuance of such license cannot render valid or even add an iota of validity to the marriage. Except in cases provided by law, it is the marriage license that gives the solemnizing officer the authority to solemnize a marriage. Respondent judge did not possess such authority when he solemnized the marriage of petitioner. In this respect, respondent judge acted in gross ignorance of the law. Respondent judge cannot be exculpated despite the Affidavit of Desistance filed by petitioner. This Court has consistently held in a catena of cases that the withdrawal of the complaint does not necessarily have the legal effect of exonerating respondent from disciplinary action. Otherwise, the prompt and fair administration of justice, as well as the discipline of court personnel, would be undermined.5 Disciplinary actions of this nature do not involve purely private or personal matters. They can not be made to depend upon the will of every complainant who may, for one reason or another, condone a detestable act. We cannot be bound by the unilateral act of a complainant in a matter which involves the Court's constitutional power to discipline judges. Otherwise, that power may be put to naught, undermine the trust character of a public office and impair the integrity and dignity of this Court as a disciplining authority.6 WHEREFORE, respondent Judge Salvador M. Occiano, Presiding Judge of the Municipal Trial Court of Balatan, Camarines Sur, is fined P5,000.00 pesos with a STERN WARNING that a repetition of the same or similar offense in the future will be dealt with more severely. SO ORDERED.

254

University of the Cordilleras College of Law First Year C S.Y. 2013 - 2014 Case Digest MERCEDITA MATA ARAES vs. JUDGE SALVADOR M. OCCIANO A.M. No. MTJ-02-1390 April 11, 2002 PUNO, J.: Facts: Petitioner Mercedita Mata Araes charges respondent judge with Gross Ignorance of the Law via a sworn Letter-Complaint dated 23 May 2001. Respondent is the Presiding Judge of the Municipal Trial Court of Balatan, Camarines Sur. Petitioner alleges that on 17 February 2000, respondent judge solemnized her marriage to her late groom Dominador B. Orobia without the requisite marriage license and at Nabua, Camarines Sur which is outside his territorial jurisdiction. His husband died and because the marriage was a nullity, her right to inherit the properties of Orobia was not recognized as well as her right to receive the pensions of her husband from the Philippine Navy. Now petitioner prays for sanctions to be imposed upon respondent judge due to the hardships and embarrassment caused by the same. On the comment of respondent Judge, he averred that he first refused to solemnized the marriage because it was outside the courts jurisdiction and that there was no valid marriage license but because of human compassion, pleadings of the parties, the influx of visitors, and the delivery of provisions for the occasion, and the difficulty of Orobia to go in the court sala for the marriage, he proceeded to solemnize the marriage out of human compassion. Respondent judge then was reassured that the marriage license will be at his sala but no marriage license came. Upon reading the comment of the respondent Judge, Aranes filed an affidavit of desistance because she realized her shortcomings and was bothered by her conscience. Issue: Whether or not the marriage is valid Ruling: The Office of the Court Administrator, in its Report and Recommendation dated 15 November 2000, found the respondent judge guilty of solemnizing a marriage without a duly issued marriage license and for doing so outside his territorial jurisdiction. A fine of P5,000.00 was recommended to be imposed on respondent judge. In the case at bar, the territorial jurisdiction of respondent judge is limited to the municipality of Balatan, Camarines Sur. His act of solemnizing the marriage of petitioner and Orobia in Nabua, Camarines Sur therefore is contrary to law and subjects him to administrative liability. His act may not amount to gross ignorance of the law for he allegedly solemnized the marriage out of human compassion but nonetheless, he cannot avoid liability for violating the law on marriage. Respondent judge cannot be exculpated despite the Affidavit of Desistance filed by petitioner. This Court has consistently held in a catena of cases that the withdrawal of the complaint does not necessarily have the legal effect of exonerating respondent from disciplinary action. Otherwise, the prompt and fair administration of justice, as well as the discipline of court personnel, would be undermined. Disciplinary actions of this nature do not involve purely private or personal matters. They can not be made to depend upon the will of every complainant who may, for one reason or another, condone a detestable act. We cannot be bound by the unilateral act of a complainant in a matter which involves the Courts constitutional power to discipline judges. Otherwise, that power may be put to naught, undermine the trust character of a public office and impair the integrity and dignity of this Court as a disciplining authority. The marriage is null and void.

255

University of the Cordilleras College of Law First Year C S.Y. 2013 - 2014 Beso vs Judge Daguman 323 SCRA 566 A.M. No. MTJ-99-1211. January 28, 2000 Full Case ZENAIDA S. BESO, complainant, vs Judge JUAN DAGUMAN, MCTC, Sta. Margarita-Tarangan-Pagsanjan, Samar, respondent. YNARES-SANTIAGO, J.: In this administrative complaint, respondent Judge stands charged with Neglect of Duty and Abuse of Authority. In a Complaint-Affidavit dated December 12, 1997, Zenaida S. Beso charged Judge Juan J. Daguman, Jr. with solemnizing marriage outside of his jurisdiction and of negligence in not retaining a copy and not registering the marriage contract with the office of the Local Civil Registrar alleging "a. That on August 28, 1997, I and my fiancee (sic) BERNARDITO A. YMAN got married and our marriage was solemnized by judge (sic) Juan Daguman in his residence in J.P.R. Subdivision in Calbayog City, Samar; xxx b. That the ceremony was attended by PACIFICO MAGHACOT who acted as our principal sponsor and spouses RAMON DEAN and TERESITA DEAN; xxx c. That after our wedding, my husband BERNARDITO YMAN abandoned me without any reason at all; d. That I smell something fishy; so what I did was I went to Calbayog City and wrote the City Civil Registrar to inquire regarding my Marriage Contract; e. That to my surprise, I was informed by the Local Civil Registrar of Calbayog City that my marriage was not registered; xxx f. That upon advisement of the Local Civil Registrar, I wrote Judge Juan Daguman, to inquire; g. That to my second surprise, I was informed by Judge Daguman that all the copies of the Marriage Contract were taken by Oloy (Bernardito A. Yman); h. That no copy was retained by Judge Daguman; i. That I believe that the respondent judge committed acts prejudicial to my interest such as: 1. Solemnizing our marriage outside his jurisdiction; 2. Negligence in not retaining a copy and not registering our marriage before the office of the Local Civil Registrar." The Affidavit-Complaint was thereafter referred to respondent Judge for comment. In his Comment, respondent Judge averred that: 1. The civil marriage of complainant Zenaida Beso and Bernardito Yman had to be solemnized by respondent in Calbayog City though outside his territory as municipal Judge of Sta. Margarita, Samar due to the following and pressing circumstances: 1.1. On August 28, 1997 respondent was physically indisposed and unable to report to his station in Sta. Margarita. In the forenoon of that date, without prior appointment, complainant Beso and Mr. Yman unexpectedly came to the residence of respondent in said City, urgently requesting the celebration of their 256

University of the Cordilleras College of Law First Year C S.Y. 2013 - 2014 marriage right then and there, first, because complainants said she must leave that same day to be able to fly from Manila for abroad as scheduled; second, that for the parties to go to another town for the marriage would be expensive and would entail serious problems of finding a solemnizing officer and another pair of witnesses or sponsors, while in fact former Undersecretary Pacifico Maghacot, Sangguniang Panglunsod [member] Ramon Dean were already with them as sponsors; third, if they failed to get married on August 28, 1997, complainant would be out of the country for a long period and their marriage license would lapse and necessitate another publication of notice; fourth, if the parties go beyond their plans for the scheduled marriage, complainant feared it would complicate her employment abroad; and, last, all other alternatives as to date and venue of marriage were considered impracticable by the parties; 1.2. The contracting parties were ready with the desired cocuments (sic) for a valid marriage, which respondent found all in order. 1.3. Complainant bride is an accredited Filipino overseas worker, who, respondent realized, deserved more than ordinary official attention under present Government policy. 2. At the time respondent solemnized the marriage in question, he believed in good faith that by so doing he was leaning on the side of liberality of the law so that it may be not be too expensive and complicated for citizens to get married. 3. Another point brought up in the complaint was the failure of registration of the duplicate and triplicate copies of the marriage certificate, which failure was also occasioned by the following circumstances beyond the control of respondent: 3.1. After handing to the husband the first copy of the marriage certificate, respondent left the three remaining copies on top of the desk in his private office where the marriage ceremonies were held, intending later to register the duplicate and triplicate copies and to keep the forth (sic) in his office. 3.2. After a few days following the wedding, respondent gathered all the papers relating to the said marriage but notwithstanding diligent search in the premises and private files, all the three last copies of the certificate were missing. Promptly, respondent invited by subpoena xxx Mr. Yman to shed light on the missing documents and he said he saw complainant Beso put the copies of the marriage certificate in her bag during the wedding party. Unfortunately, it was too late to contact complainant for a confirmation of Mr. Ymans claim. 3.3. Considering the futility of contracting complainant now that she is out of the country, a reasonable conclusion can be drawn on the basis of the established facts so far in this dispute. If we believe the claim of complainant that after August 28, 1997 marriage her husband, Mr. Yman, abandoned her without any reason xxx but that said husband admitted "he had another girl by the name of LITA DANGUYAN" xxx it seems reasonably clear who of the two marriage contracting parties probably absconded with the missing copies of the marriage certificate. 3.4. Under the facts above stated, respondent has no other recourse but to protect the public interest by trying all possible means to recover custody of the missing documents in some amicable way during the expected hearing of the above mentioned civil case in the City of Marikina, failing to do which said respondent would confer with the Civil Registrar General for possible registration of reconstituted copies of said documents. The Office of the Court Administrator (OCA) in an evaluation report dated August 11, 1998 found that respondent Judge " committed non-feasance in office" and recommended that he be fined Five Thousand Pesos (P5,000.00) with a warning that the commission of the same or future acts will be dealt with more severely pointing out that: 257

University of the Cordilleras College of Law First Year C S.Y. 2013 - 2014 "As presiding judge of the MCTC Sta. Margarita Tarangnan-Pagsanjan, Samar, the authority to solemnize marriage is only limited to those municipalities under his jurisdiction. Clearly, Calbayog City is no longer within his area of jurisdiction. Additionally, there are only three instances, as provided by Article 8 of the Family Code, wherein a marriage may be solemnized by a judge outside his chamber[s] or at a place other than his sala, to wit: (1) when either or both of the contracting parties is at the point of death; (2) when the residence of either party is located in a remote place; (3) where both of the parties request the solemnizing officer in writing in which case the marriage may be solemnized at a house or place designated by them in a sworn statement to that effect. The foregoing circumstances are unavailing in the instant case. Moreover, as solemnizing officer, respondent Judge neglected his duty when he failed to register the marriage of complainant to Bernardito Yman. Such duty is entrusted upon him pursuant to Article 23 of the Family Code which provides: "It shall be the duty of the person solemnizing the marriage to furnish either of the contracting parties the original of the marriage certificate referred to in Article 6 and to send the duplicate and triplicate copies of the certificates not later than fifteen days after the marriage, to the local civil registrar of the place where the marriage was solemnized. xxx" (underscoring ours) It is clearly evident from the foregoing that not only has the respondent Judge committed non-feasance in office, he also undermined the very foundation of marriage which is the basic social institution in our society whose nature, consequences and incidents are governed by law. Granting that respondent Judge indeed failed to locate the duplicate and triplicate copies of the marriage certificate, he should have exerted more effort to locate or reconstitute the same. As a holder of such a sensitive position, he is expected to be conscientious in handling official documents. His imputation that the missing copies of the marriage certificate were taken by Bernardito Yman is based merely on conjectures and does not deserve consideration for being devoid of proof." After a careful and thorough examination of the evidence, the Court finds the evaluation report of the OCA well-taken. Jimenez v. Republic underscores the importance of marriage as a social institution thus: "[M]arriage in this country is an institution in which the community is deeply interested. The state has surrounded it with safeguards to maintain its purity, continuity and permanence. The security and stability of the state are largely dependent upon it. It is the interest and duty of each and every member of the community to prevent the bringing about of a condition that would shake its foundation and ultimately lead to its destruction." With regard to the solemnization of marriage, Article 7 of the Family Code provides, among others, that "ART. 7. Marriage may be solemnized by: (1) Any incumbent member of the judiciary within the courts jurisdiction; xxx" (Italics ours) In relation thereto, Article 8 of the same statute mandates that:

258

University of the Cordilleras College of Law First Year C S.Y. 2013 - 2014 ART. 8. The marriage shall be solemnized publicly in the chambers of the judge or in open court, in the church, chapel or temple, or in the office of the consul-general, consul or vice-consul, as the case may be, and not elsewhere, except in cases of marriages contracted at the point of death or in remote places in accordance with Article 29 of this Code, or where both parties request the solemnizing officer in writing in which case the marriage may be solemnized at a house or place designated by them in a sworn statement to that effect." (Italics ours) As the above-quoted provision clearly states, a marriage can be held outside the judges chambers or courtroom only in the following instances: 1.] at the point of death; 2.] in remote places in accordance with Article 29, or 3.] upon the request of both parties in writing in a sworn statement to this effect. In this case, there is no pretense that either complainant Beso or her fiance Yman was at the point of death or in a remote place. Neither was there a sworn written request made by the contracting parties to respondent Judge that the marriage be solemnized outside his chambers or at a place other than his sala. What, in fact, appears on record is that respondent Judge was prompted more by urgency to solemnize the marriage of Beso and Yman because complainant was "[a]n overseas worker, who, respondent realized deserved more than ordinary official attention under present Government policy." Respondent Judge further avers that in solemnizing the marriage in question, "[h]e believed in good faith that by doing so he was leaning on the side of liberality of the law so that it may not be too expensive and complicated for citizens to get married." A person presiding over a court of law must not only apply the law but must also live and abide by it and render justice at all times without resorting to shortcuts clearly uncalled for. A judge is not only bound by oath toapply the law; he must also be conscientious and thorough in doing so. Certainly, judges, by the very delicate nature of their office should be more circumspect in the performance of their duties. If at all, the reasons proffered by respondent Judge to justify his hurried solemnization of the marriage in this case only tends to degrade the revered position enjoyed by marriage in the hierarchy of social institutions in the country. They also betray respondents cavalier proclivity on its significance in our culture which is more disposed towards an extended period of engagement prior to marriage and frowns upon hasty, ill-advised and ill-timed marital unions. An elementary regard for the sacredness of laws let alone that enacted in order to preserve so sacrosanct an inviolable social institution as marriage and the stability of judicial doctrines laid down by superior authority should have given respondent judge pause and made him more vigilant in the exercise of his authority and the performance of his duties as a solemnizing officer. A judge is, furthermore, presumed to know the constitutional limits of the authority or jurisdiction of his court. Thus respondent Judge should be reminded that A priest, who is commissioned and allowed by his ordinary to marry the faithful, is authorized to do so only within the area of the diocese or place allowed by his Bishop. An appellate court justice or a Justice of this Court has jurisdiction over the entire Philippines to solemnize marriages, regardless of the venue, as long as the requisites of the law are complied with. However, Judges who are appointed to specific jurisdictions may officiate in weddings only within said areas and not beyond. Where a judge solemnizes a marriage outside his courts jurisdiction, there is a resultant irregularity in the formal requisite laid down in Article 3, which while it may not affect the validity of the marriage, may subject the officiating official to administrative liability. Considering that respondent Judges jurisdiction covers the municipality of Sta. Margarita -TaranganPagsanjan, Samar only, he was not clothed with authority to solemnize a marriage in the City of Calbayog. Furthermore, from the nature of marriage, aside from the mandate that a judge should exercise extra care in the exercise of his authority and the performance of his duties in its solemnization, he is likewise commanded to observe extra precautions to ensure that the event is properly documented in accordance with Article 23 of the Family Code which states in no uncertain terms that ART. 23. - It shall be the duty of the person solemnizing the marriage to furnish either of the contracting parties, the original of the marriage contract referred to in Article 6 and 259

University of the Cordilleras College of Law First Year C S.Y. 2013 - 2014 to send the duplicate and triplicate copies of the certificate not later than fifteen days after the marriage, to the local civil registrar of the place where the marriage was solemnized. Proper receipts shall be issued by the local civil registrar to the solemnizing officer transmitting copies of the marriage certificate. The solemnizing officer shall retain in his file the quadruplicate copy of the marriage certificate, the original of the marriage license and, in proper cases, the affidavit of the contracting party regarding the solemnization of the marriage in a place other than those mentioned in Article 8. (Italics supplied) In view of the foregoing, we agree with the evaluation of the OCA that respondent Judge was less than conscientious in handling official documents. A judge is charged with exercising extra care in ensuring that the records of the cases and official documents in his custody are intact. There is no justification for missing records save fortuitous events. However, the records show that the loss was occasioned by carelessness on respondent Judges part. This Court reiterates that judges must adopt a system of record management and organize their dockets in order to bolster the prompt and efficient dispatch of business. It is, in fact, incumbent upon him to devise an efficient recording and filing system in his court because he is after all the one directly responsible for the proper discharge of his official functions. In the evaluation report, the OCA recommended that respondent Judge be fined Five Thousand Pesos (P5,000.00) and warned that a repetition of the same or similar acts will be dealt with more severely. This Court adopts the recommendation of the OCA. WHEREFORE, in view of all the foregoing, respondent Judge is hereby FINED Five Thousand Pesos (P5,000.00) and STERNLY WARNED that a repetition of the same or similar infractions will be dealt with more severely. SO ORDERED. Davide, Jr., C.J., (Chairman), Puno, Kapunan, and Pardo, JJ., concur.

260

University of the Cordilleras College of Law First Year C S.Y. 2013 - 2014 Case Digest ZENAIDA S. BESO vs. Judge JUAN DAGUMAN A.M. No. MTJ-99-1211 January 28, 2000 YNARES-SANTIAGO, J.: Facts: Zenaida S. Beso charged Judge Juan J. Daguman, Jr. of Sta. Margarita Municipal Trial Court, with solemnizing marriage outside of his jurisdiction and not registering the marriage contract with the office of the Local Civil Registrar. On the other hand, the respondent claimed that he solemnized the marriage of Zenaida S. Beso and Bernardito Yman in Calbayog City Samar due to the urgency of the situation, that Beso, on August 28, 1997, the day of the marriage, said that he will be leaving the country for job purposes and considering her to be an Overseas Filipino Worker and that if not solemnized that day, their marriage license would lapse because she will be working abroad for a long period. Thus, this would necessitate spouses for a new marriage license. The necessary documents that was supposedly to be forwarded by him to the Local Civil Registrar was missing and claimed to be taken by someone. The Office of the Court administration held that the respondent Judge committed non-feasance in office and was fined PHP 5,000 with stern warning. Issues: Whether or not the respondent can validly solemnized the marriage of the complainant outside the courts jurisdiction Whether or not the respondent committed negligence by not retaining a copy and not registering the complainants marriage before the office of the Local Civil Regitrar. Ruling: No. the judge solemnized the marriage outside of his jurisdiction. Article 7 of the Family Code provides that the marriage be solemnized by, any incumbent member of the judiciary with the courts jurisdiction. In relation thereto, according to Article 8 of the Family Code, there are only three instances with which a judge may solemnize a marriage outside of his jurisdiction: (1) When either or both the contracting parties is at the point of death; (2) When the residence of either party is located in a remote place; (3) Where both of the parties request the solemnizing officer in writing in which case the marriage may be solemnized at a house or place designated by them in a sworn statement to that effect. In this case, none of the three instances is present. Yes. The judge committed negligence. Pursuant to Article 23 of the Family Code, such duty to register the marriage is the respondents duty. The same article provides, it shall be the duty of the person solemnizing the marriage to send the duplicate and triplicate copies of the certificate not later than 15 days after the marriage, to the local civil registrar of the place where the marriage was solemnized. Proper receipts shall be issued by the local civil registrar to the solemnizing officer transmitting the copies of the certificate. The solemnizing officer shall retain in his file the quadruplicate copy of the marriage certificate, the original of the marriage license, and in proper cases, the affidavit of the contracting party regarding the solemnization of the marriage in the place other than those mentioned in Article 8. The recommendation of the OCA stands.

261

University of the Cordilleras College of Law First Year C S.Y. 2013 - 2014 Cosca vs Palaypayon Jr. 237 SCRA 249 A.M. No. MTJ-92-721 September 30, 1994 Full Case JUVY N. COSCA, EDMUNDO B. PERALTA, RAMON C. SAMBO, and APOLLO A. VILLAMORA, complainants, vs. HON. LUCIO P. PALAYPAYON, JR., Presiding Judge, and NELIA B. ESMERALDA-BAROY, Clerk of Court II, both of the Municipal Trial Court of Tinambac, Camarines Sur, respondents. PER CURIAM, J.: Complainants Juvy N. Cosca, Edmundo B. Peralta, Ramon C. Sambo, and Apollo Villamora, are Stenographer I, Interpreter I, Clerk II, and Process Server, respectively, of the Municipal Trial Court of Tinambac, Camarines Sur. Respondents Judge Lucio P. Palaypayon, Jr. and Nelia B. Esmeralda-Baroy are respectively the Presiding Judge and Clerk of Court II of the same court. In an administrative complaint filed with the Office of the Court Administrator on October 5, 1992, herein respondents were charged with the following offenses, to wit: (1) illegal solemnization of marriage; (2) falsification of the monthly reports of cases; (3) bribery in consideration of an appointment in the court; (4) non-issuance of receipt for cash bond received; (5) infidelity in the custody of detained prisoners; and (6) requiring payment of filing fees from exempted entities. Pursuant to a resolution issued by this Court respondents filed their respective Comments. A Reply to Answers of Respondents was filed by complainants. The case was thereafter referred to Executive Judge David C. Naval of the Regional Trial Court, Naga City, for investigation report and recommendation. The case was however transferred to First Assistant Executive Judge Antonio N. Gerona when Judge Naval inhibited himself for the reason that his wife is a cousin of respondent Judge Palaypayon, Jr. The contending versions of the parties regarding the factual antecedents of this administrative matter, as culled from the records thereof, are set out under each particular charge against respondents. 1. Illegal solemnization of marriage Complainants allege that respondent judge solemnized marriages even without the requisite marriage license. Thus, the following couples were able to get married by the simple expedient of paying the marriage fees to respondent Baroy, despite the absence of a marriage license, viz.: Alano P. Abellano and Nelly Edralin, Francisco Selpo and Julieta Carrido, Eddie Terrobias and Maria Gacer, Renato Gamay and Maricris Belga, Arsenio Sabater and Margarita Nacario, and Sammy Bocaya and Gina Bismonte. As a consequence, their marriage contracts (Exhibits B, C, D, F, G, and A, respectively) did not reflect any marriage license number. In addition, respondent judge did not sign their marriage contracts and did not indicate the date of solemnization, the reason being that he allegedly had to wait for the marriage license to be submitted by the parties which was usually several days after the ceremony. Indubitably, the marriage contracts were not filed with the local civil registrar. Complainant Ramon Sambo, who prepares the marriage contracts, called the attention of respondents to the lack of marriage licenses and its effect on the marriages involved, but the latter opted to proceed with the celebration of said marriages. Respondent Nelia Baroy claims that when she was appointed Clerk of Court II, the employees of the court were already hostile to her, especially complainant Ramon Sambo who told her that he was filing a protest against her appointment. She avers that it was only lately when she discovered that the court had a marriage Register which is in the custody of Sambo; that it was Sambo who failed to furnish the parties copies of the marriage contract and to register these with the local civil registrar; and that apparently Sambo kept these marriage contracts in preparation for this administrative case. Complainant Sambo, however, claims that all file copies of the marriage contracts were kept by respondent Baroy, but the latter insists that she had instructed Sambo to follow up the submission by the contracting parties of their marriage licenses as part of his duties but he failed to do so. Respondent Judge Palaypayon, Jr. contends that the marriage between Alano P. Abellano and Nelly Edralin falls under Article 34 of the Civil Code, hence it is exempt from the marriage license requirement; that he gave strict instructions to complainant Sambo to furnish the couple a copy of the marriage contract 262

University of the Cordilleras College of Law First Year C S.Y. 2013 - 2014 and to file the same with the civil registrar, but the latter failed to do so; that in order to solve the problem, the spouses subsequently formalized their marriage by securing a marriage license and executing their marriage contract, a copy of which was filed with the civil registrar; that the other five marriages alluded to in the administrative complaint were not illegally solemnized because the marriage contracts were not signed by him and they did not contain the date and place of marriage; that copies of these marriage contracts are in the custody of complainant Sambo; that the alleged marriage of Francisco Selpo and Julieta Carrido, Eddie Terrobias and Maria Emma Gaor, Renato Gamay and Maricris Belga, and of Arsenio Sabater and Margarita Nacario were not celebrated by him since he refused to solemnize them in the absence of a marriage license; that the marriage of Samy Bocaya and Gina Bismonte was celebrated even without the requisite license due to the insistence of the parties in order to avoid embarrassment to their guests but that, at any rate, he did not sign their marriage contract which remains unsigned up to the present. 2. Falsification of monthly report for July, 1991 regarding the number of marriages solemnized and the number of documents notarized. It is alleged that respondent judge made it appear that he solemnized seven (7) marriages in the month of July, 1992, when in truth he did not do so or at most those marriages were null and void; that respondents likewise made it appear that they have notarized only six (6) documents for July, 1992, but the Notarial Register will show that there were one hundred thirteen (113) documents which were notarized during that month; and that respondents reported a notarial fee of only P18.50 for each document, although in fact they collected P20.00 therefor and failed to account for the difference. Respondent Baroy contends, however, that the marriage registry where all marriages celebrated by respondent judge are entered is under the exclusive control and custody of complainant Ramon Sambo, hence he is the only one who should be held responsible for the entries made therein; that the reported marriages are merely based on the payments made as solemnization fees which are in the custody of respondent Baroy. She further avers that it is Sambo who is likewise the custodian of the Notarial Register; that she cannot be held accountable for whatever alleged difference there is in the notarial fees because she is liable only for those payments tendered to her by Sambo himself; that the notarial fees she collects are duly covered by receipts; that of the P20.00 charged, P18.50 is remitted directly to the Supreme Court as part of the Judiciary Development Fund and P150 goes to the general fund of the Supreme Court which is paid to the Municipal Treasurer of Tinambac, Camarines Sur. Respondent theorizes that the discrepancies in the monthly report were manipulated by complainant Sambo considering that he is the one in charge of the preparation of the monthly report. Respondent Judge Palaypayon avers that the erroneous number of marriages celebrated was intentionally placed by complainant Sambo; that the number of marriages solemnized should not be based on solemnization fees paid for that month since not all the marriages paid for are solemnized in the same month. He claims that there were actually only six (6) documents notarized in the month of July, 1992 which tallied with the official receipts issued by the clerk of court; that it is Sambo who should be held accountable for any unreceipted payment for notarial fees because he is the one in charge of the Notarial Register; and that this case filed by complainant Sambo is merely in retaliation for his failure to be appointed as the clerk of court. Furthermore, respondent judge contends that he is not the one supervising or preparing the monthly report, and that he merely has the ministerial duty to sign the same. 3. Bribery in consideration of an appointment in the court Complainants allege that because of the retirement of the clerk of court, respondent judge forwarded to the Supreme Court the applications of Rodel Abogado, Ramon Sambo, and Jessell Abiog. However, they were surprised when respondent Baroy reported for duty as clerk of court on October 21, 1991. They later found out that respondent Baroy was the one appointed because she gave a brand-new air-conditioning unit to respondent judge. Respondent Baroy claims that when she was still in Naga City she purchased an air-conditioning unit but when she was appointed clerk of court she had to transfer to Tinambac and, since she no longer needed the air conditioner, she decided to sell the same to respondent judge. The installation and use thereof by the latter in his office was with the consent of the Mayor of Tinambac. Respondent judge contends that he endorsed all the applications for the position of clerk of court to the Supreme Court which has the sole authority over such appointments and that he had no hand in the 263

University of the Cordilleras College of Law First Year C S.Y. 2013 - 2014 appointment of respondent Baroy. He contends that the air-conditioning unit was bought from his co-respondent on installment basis on May 29, 1992, eight (8) months after Baroy had been appointed clerk of court. He claims that he would not be that naive to exhibit to the public as item which could not be defended as a matter of honor and prestige. 4. Cash bond issued without a receipt It is alleged that in Criminal Case No. 5438, entitled "People vs. Mendeza, et al., "bondswoman Januaria Dacara was allowed by respondent judge to change her property bond to cash bond; that she paid the amount of P1,000.00 but was never issued a receipt therefor nor was it made to appear in the records that the bond has been paid; that despite the lapse of two years, the money was never returned to the bondswoman; and that it has not been shown that the money was turned over to the Municipal Treasurer of Tinambac. Respondent Baroy counters that the cash bond was deposited with the former clerk of court, then turned over to the acting clerk of court and, later, given to her under a corresponding receipt; that the cash bond is deposited with the bank; and that should the bondswoman desire to withdraw the same, she should follow the proper procedure therefor. Respondent judge contends that Criminal Case No. 5438 was achieved for failure of the bondsman to deliver the body of the accused in court despite notice; and that he has nothing to do with the payment of the cash bond as this is the duty of the clerk of court. 5. Infidelity in the custody of prisoners Complainants contend that respondent judge usually got detention prisoners to work in his house, one of whom was Alex Alano, who is accused in Criminal Case No. 5647 for violation of the Dangerous Drugs Act; that while Alano was in the custody of respondent judge, the former escaped and was never recaptured; that in order to conceal this fact, the case was archived pursuant to an order issued by respondent judge dated April 6, 1992. Respondent judge denied the accusation and claims that he never employed detention prisoners and that he has adequate household help; and that he had to order the case archived because it had been pending for more than six (6) months and the accused therein remained at large. 6. Unlawful collection of docket fees Finally, respondents are charged with collecting docket fees from the Rural Bank of Tinambac, Camarines Sur, Inc. although such entity is exempt by law from the payment of said fees, and that while the corresponding receipt was issued, respondent Baroy failed to remit the amount to the Supreme Court and, instead, she deposited the same in her personal account. Respondents Baroy contends that it was Judge-Designate Felimon Montenegro (because respondent judge was on sick leave) who instructed her to demand payment of docket fees from said rural bank; that the bank issued a check for P800.00; that she was not allowed by the Philippine National Bank to encash the check and, instead, was instructed to deposit the same in any bank account for clearing; that respondent deposited the same in her account; and that after the check was cleared, she remitted P400.00 to the Supreme Court and the other P400.00 was paid to the Municipal Treasurer of Tinambac. On the basis of the foregoing contentions, First Vice-Executive Judge Antonio N. Gerona prepared and submitted to us his Report and Recommendations dated May 20, 1994, together with the administrative matter. We have perspicaciously reviewed the same and we are favorably impressed by the thorough and exhaustive presentation and analysis of the facts and evidence in said report. We commend the investigating judge for his industry and perspicacity reflected by his findings in said report which, being amply substantiated by the evidence and supported by logical illations, we hereby approve and hereunder reproduce at length the material portions thereof. xxx xxx xxx

264

University of the Cordilleras College of Law First Year C S.Y. 2013 - 2014 The first charge against the respondents is illegal solemnization of marriage. Judge Palaypayon is charged with having solemnized without a marriage license the marriage of Sammy Bocaya and Gina Besmonte (Exh. A). Alano Abellano and Nelly Edralin (Exh. B), Francisco Selpo and Julieta Carrido (Exh. C), Eddie Terrobias and Maria Emma Gaor (Exh. D), Renato Gamay and Maricris Belga (Exh. F) and Arsenio Sabater and Margarita Nacario (Exh. G). In all these aforementioned marriages, the blank space in the marriage contracts to show the number of the marriage was solemnized as required by Article 22 of the Family Code were not filled up. While the contracting parties and their witnesses signed their marriage contracts, Judge Palaypayon did not affix his signature in the marriage contracts, except that of Abellano and Edralin when Judge Palaypayon signed their marriage certificate as he claims that he solemnized this marriage under Article 34 of the Family Code of the Philippines. In said marriages the contracting parties were not furnished a copy of their marriage contract and the Local Civil Registrar was not sent either a copy of the marriage certificate as required by Article 23 of the Family Code. The marriage of Bocaya and Besmonte is shown to have been solemnized by Judge Palaypayon without a marriage license. The testimonies of Bocay himself and Pompeo Ariola, one of the witnesses of the marriage of Bocaya and Besmonte, and the photographs taken when Judge Palaypayon solemnized their marriage (Exhs. K-3 to K-9) sufficiently show that Judge Palaypayon really solemnized their marriage. Bocaya declared that they were advised by Judge Palaypayon to return after ten (10) days after their marriage was solemnized and bring with them their marriage license. In the meantime, they already started living together as husband and wife believing that the formal requisites of marriage were complied with. Judge Palaypayon denied that he solemnized the marriage of Bocaya and Besmonte because the parties allegedly did not have a marriage license. He declared that in fact he did not sign the marriage certificate, there was no date stated on it and both the parties and the Local Civil Registrar did not have a copy of the marriage certificate. With respect to the photographs which show that he solemnized the marriage of Bocaya and Besmonte, Judge Palaypayon explains that they merely show as if he was solemnizing the marriage. It was actually a simulated solemnization of marriage and not a real one. This happened because of the pleading of the mother of one of the contracting parties that he consent to be photographed to show that as if he was solemnizing the marriage as he was told that the food for the wedding reception was already prepared, visitors were already invited and the place of the parties where the reception would be held was more than twenty (20) kilometers away from the poblacion of Tinambac. The denial made by Judge Palaypayon is difficult to believe. The fact alone that he did not sign the marriage certificate or contract, the same did not bear a date and the parties and the Local Civil Registrar were not furnished a copy of the marriage certificate, do not by themselves show that he did not solemnize the marriage. His uncorroborated testimony cannot prevail over the testimony of Bocaya and Ariola who also declared, among others, that Bocaya and his bride were advised by Judge Palaypayon to return after ten (10) days with their marriage license and whose credibility had not been impeached. The pictures taken also from the start of the wedding ceremony up to the signing of the marriage certificate in front of Judge Palaypayon and on his table (Exhs. K-3, K-3-a, K3-b, K-3-c, K-4, K-4-a, K-4-b, K-4-c, K-4-d, K-5, K-5-a, K-5-b, K-6, K-7, K-8, K-8-a and K-9), cannot possibly be just to show a simulated solemnization of marriage. One or two pictures may convince a person of the explanation of Judge Palaypayon, but not all those pictures. Besides, as a judge it is very difficult to believe that Judge Palaypayon would allows himself to be photographed as if he was solemnizing a marriage on a mere pleading of a person whom he did not even know for the alleged reasons given. It would be highly improper and unbecoming of him to allow himself to be used as an instrument of deceit 265

University of the Cordilleras College of Law First Year C S.Y. 2013 - 2014 by making it appear that Bocaya and Besmonte were married by him when in truth and in fact he did not solemnize their marriage. With respect to the marriage of Abellano and Edralin (Exh. B), Judge Palaypayon admitted that he solemnized their marriage, but he claims that it was under Article 34 of the Family Code, so a marriage license was not required. The contracting parties here executed a joint affidavit that they have been living together as husband and wife for almost six (6) years already (Exh. 12; Exh. AA). In their marriage contract which did not bear any date either when it was solemnized, it was stated that Abellano was only eighteen (18) years, two (2) months and seven (7) days old. If he and Edralin had been living together as husband and wife for almost six (6) years already before they got married as they stated in their joint affidavit, Abellano must ha(ve) been less than thirteen (13) years old when he started living with Edralin as his wife and this is hard to believe. Judge Palaypayon should ha(ve) been aware of this when he solemnized their marriage as it was his duty to ascertain the qualification of the contracting parties who might ha(ve) executed a false joint affidavit in order to have an instant marriage by avoiding the marriage license requirement. On May 23, 1992, however, after this case was already filed, Judge Palaypayon married again Abellano and Edralin, this time with a marriage license (Exh. BB). The explanation given by Judge Palaypayon why he solemnized the marriage of the same couple for the second time is that he did not consider the first marriage he solemnized under Article 34 of the Family Code as (a) marriage at all because complainant Ramon Sambo did not follow his instruction that the date should be placed in the marriage certificate to show when he solemnized the marriage and that the contracting parties were not furnished a copy of their marriage certificate. This act of Judge Palaypayon of solemnizing the marriage of Abellano and Edralin for the second time with a marriage license already only gave rise to the suspicion that the first time he solemnized the marriage it was only made to appear that it was solemnized under exceptional character as there was not marriage license and Judge Palaypayon had already signed the marriage certificate. If it was true that he solemnized the first marriage under exceptional character where a marriage license was not required, why did he already require the parties to have a marriage license when he solemnized their marriage for the second time? The explanation of Judge Palaypayon that the first marriage of Abellano and Edralin was not a marriage at all as the marriage certificate did not state the date when the marriage was solemnized and that the contracting parties were not furnished a copy of their marriage certificate, is not well taken as they are not any of those grounds under Article(s) 35, 36, 37 and 38 of the Family Code which declare a marriage void from the beginning. Even if no one, however, received a copy of the marriage certificate, the marriage is still valid (Jones vs. H(o)rtiguela, 64 Phil. 179). Judge Palaypayon cannot just absolve himself from responsibility by blaming his personnel. They are not the guardian(s) of his official function and under Article 23 of the Family Code it is his duty to furnish the contracting parties (a) copy of their marriage contract. With respect to the marriage of Francisco Selpo and Julieta Carrido (Exh. C), and Arsenio Sabater and Margarita Nacario (Exh. G), Selpo and Carrido and Sabater and Nacarcio executed joint affidavits that Judge Palaypayon did not solemnize their marriage (Exh. 13-A and Exh. 1). Both Carrido and Nacario testified for the respondents that actually Judge Palaypayon did not solemnize their marriage as they did not have a marriage license. On cross-examination, however, both admitted that they did not know who prepared their affidavits. They were just told, Carrido by a certain Charito Palaypayon, and Nacario by a certain Kagawad Encinas, to just go to the Municipal building and sign their joint affidavits there which were already prepared before the Municipal Mayor of Tinambac, Camarines Sur. With respect to the marriage of Renato Gamay and Maricris Belga (Exh. f), their marriage contract was signed by them and by their two (2) witnesses, Atty. Elmer Brioso 266

University of the Cordilleras College of Law First Year C S.Y. 2013 - 2014 and respondent Baroy (Exhs. F-1 and F-2). Like the other aforementioned marriages, the solemnization fee was also paid as shown by a receipt dated June 7, 1992 and signed by respondent Baroy (Exh. F-4). Judge Palaypayon also denied having solemnized the marriage of Gamay and Belga allegedly because there was no marriage license. On her part, respondent Baroy at first denied that the marriage was solemnized. When she was asked, however, why did she sign the marriage contract as a witness she answered that she thought the marriage was already solemnized (TSN, p. 14; 10-28-93). Respondent Baroy was, and is, the clerk of court of Judge Palaypayon. She signed the marriage contract of Gamay and Belga as one of the two principal sponsors. Yet, she wanted to give the impression that she did not even know that the marriage was solemnized by Judge Palaypayon. This is found very difficult to believe. Judge Palaypayon made the same denial of having solemnized also the marriage of Terrobias and Gaor (Exh. D). The contracting parties and their witnesses also signed the marriage contract and paid the solemnization fee, but Judge Palaypayon allegedly did not solemnize their marriage due to lack of marriage license. Judge Palaypayon submitted the affidavit of William Medina, Vice-Mayor of Tinambac, to corroborate his testimony (Exh. 14). Medina, however, did not testify in this case and so his affidavit has no probative value. Judge Palaypayon testified that his procedure and practice have been that before the contracting parties and their witnesses enter his chamber in order to get married, he already required complainant Ramon Sambo to whom he assigned the task of preparing the marriage contract, to already let the parties and their witnesses sign their marriage contracts, as what happened to Gamay and Belga, and Terrobias and Gaor, among others. His purpose was to save his precious time as he has been solemnizing marriages at the rate of three (3) to four (4) times everyday (TSN, p. 12; 2-1-94). This alleged practice and procedure, if true, is highly improper and irregular, if not illegal, because the contracting parties are supposed to be first asked by the solemnizing officer and declare that they take each other as husband and wife before the solemnizing officer in the presence of at least two (2) witnesses before they are supposed to sign their marriage contracts (Art. 6, Family Code). The uncorroborated testimony, however, of Judge Palaypayon as to his alleged practice and procedure before solemnizing a marriage, is not true as shown by the picture taken during the wedding of Bocaya and Besmonte (Exhs. K-3 to K-9) and by the testimony of respondent Baroy herself who declared that the practice of Judge Palaypayon ha(s) been to let the contracting parties and their witnesses sign the marriage contract only after Judge Palaypayon has solemnized their marriage (TSN, p. 53; 10-28-93). Judge Palaypayon did not present any evidence to show also that he was really solemnizing three (3) to four (4) marriages everyday. On the contrary his monthly report of cases for July, 1992 shows that his court had only twenty-seven (27) pending cases and he solemnized only seven (7) marriages for the whole month (Exh. E). His monthly report of cases for September, 1992 shows also that he solemnized only four (4) marriages during the whole month (Exh. 7). In this first charge of having illegally solemnized marriages, respondent Judge Palaypayon has presented and marked in evidence several marriage contracts of other persons, affidavits of persons and certification issued by the Local Civil Registrar (Exhs. 12-B to 12-H). These persons who executed affidavits, however, did not testify in this case. Besides, the marriage contracts and certification mentioned are immaterial as Judge Palaypayon is not charged of having solemnized these marriages illegally also. He is not charged that the marriages he solemnized were all illegal.

267

University of the Cordilleras College of Law First Year C S.Y. 2013 - 2014 The second charge against herein respondents, that of having falsified the monthly report of cases submitted to the Supreme Court and not stating in the monthly report the actual number of documents notarized and issuing the corresponding receipts of the notarial fees, have been sufficiently proven by the complainants insofar as the monthly report of cases for July and September, 1992 are concerned. The monthly report of cases of the MTC of Tinambac, Camarines Sur for July, 1992 both signed by the respondents, show that for said month there were six (6) documents notarized by Judge Palaypayon in his capacity as Ex-Officio Notary Public (Exhs. H to H-1-b). The notarial register of the MTC of Tinambac, Camarines Sur, however, shows that there were actually one hundred thirteen (113) documents notarized by Judge Palaypayon for the said month (Exhs. Q to Q-45). Judge Palaypayon claims that there was no falsification of the monthly report of cases for July, 1992 because there were only six (6) notarized documents that were paid (for) as shown by official receipts. He did not, however, present evidence of the alleged official receipts showing that the notarial fee for the six (6) documetns were paid. Besides, the monthly report of cases with respect to the number of documents notarized should not be based on how many notarized documents were paid of the notarial fees, but the number of documents placed or recorded in the notarial register. Judge Palaypayon admitted that he was not personally verifying and checking anymore the correctness of the monthly reports because he relies on his co-respondent who is the Clerk of Court and whom he has assumed to have checked and verified the records. He merely signs the monthly report when it is already signed by respondent Baroy. The explanation of Judge Palaypayon is not well taken because he is required to have close supervision in the preparation of the monthly report of cases of which he certifies as to their correctness. As a judge he is personally responsible for the proper discharge of his functions (The Phil. Trial Lawyer's Asso. Inc. vs. Agana, Sr., 102 SCRA 517). In Nidera vs. Lazaro, 174 SCRA 581, it was held that "A judge cannot take refuge behind the inefficiency or mismanagement of his court personnel." On the part of respondent Baroy, she puts the blame of the falsification of the monthly report of cases on complainant Sambo whom she allegedly assigned to prepare not only the monthly report of cases, but the preparation and custody of marriage contracts, notarized documents and the notarial register. By her own admission she has assigned to complainant Sambo duties she was supposed to perform, yet according to her she never bother(ed) to check the notarial register of the court to find out the number of documents notarized in a month (TSN, p. 30; 11-23-93). Assuming that respondent Baroy assigned the preparation of the monthly report of cases to Sambo, which was denied by the latter as he claims that he only typed the monthly report based on the data given to him by her, still it is her duty to verify and check whether the report is correct. The explanation of respondent Baroy that Sambo was the one in custody of marriage contracts, notarized documents and notarial register, among other things, is not acceptable not only because as clerk of court she was supposed to be in custody, control and supervision of all court records including documents and other properties of the court (p. 32, Manual for Clerks of Court), but she herself admitted that from January, 1992 she was already in full control of all the records of the court including receipts (TSN, p. 11; 11-23-93). The evidence adduced in this cases in connection with the charge of falsification, however, also shows that respondent Baroy did not account for what happened to the notarial fees received for those documents notarized during the month of July and September, 1992. The evidence adduced in this case also sufficiently show that she received cash bond deposits and she did not deposit them to a bank or to the Municipal Treasurer; and that she only issued temporary receipts for said cash bond deposits.

268

University of the Cordilleras College of Law First Year C S.Y. 2013 - 2014 For July, 1992 there were only six (6) documents reported to have been notarized by Judge Palaypayon although the documents notarized for said month were actually one hundred thirteen (113) as recorded in the notarial register. For September, 1992, there were only five (5) documents reported as notarized for that month, though the notarial register show(s) that there were fifty-six (56) documents actually notarized. The fee for each document notarized as appearing in the notarial register was P18.50. Respondent Baroy and Sambo declared that what was actually being charged was P20.00. Respondent Baroy declared that P18.50 went to the Supreme Court and P1.50 was being turned over to the Municipal Treasurer. Baroy, however, did not present any evidence to show that she really sent to the Supreme Court the notarial fees of P18.50 for each document notarized and to the Municipal Treasurer the additional notarial fee of P1.50. This should be fully accounted for considering that Baroy herself declared that some notarial fees were allowed by her at her own discretion to be paid later. Similarly, the solemnization fees have not been accounted for by Baroy considering that she admitted that even (i)n those instances where the marriages were not solemnized due to lack of marriage license the solemnization fees were not returned anymore, unless the contracting parties made a demand for their return. Judge Palaypayon declared that he did not know of any instance when solemnization fee was returned when the marriage was not solemnized due to lack of marriage license. Respondent Baroy also claims that Ramon Sambo did not turn over to her some of the notarial fees. This is difficult to believe. It was not only because Sambo vehemently denied it, but the minutes of the conference of the personnel of the MTC of Tinambac dated January 20, 1992 shows that on that date Baroy informed the personnel of the court that she was taking over the functions she assigned to Sambo, particularly the collection of legal fees (Exh. 7). The notarial fees she claims that Sambo did not turn over to her were for those documents notarized (i)n July and September, 1992 already. Besides there never was any demand she made for Sambo to turn over some notarial fees supposedly in his possession. Neither was there any memorandum she issued on this matter, in spite of the fact that she has been holding meetings and issuing memoranda to the personnel of the court (Exhs. V, W, FF, FF-1, FF-2, FF-3; Exhs. 4-A (supplement(s), 5-8, 6-S, 7-S and 8-S). It is admitted by respondent Baroy that on October 29, 1991 a cash bond deposit of a certain Dacara in the amount of One Thousand (P1,000.00) Pesos was turned over to her after she assumed office and for this cash bond she issued only a temporary receipt (Exh. Y). She did not deposit this cash bond in any bank or to the Municipal Treasurer. She just kept it in her own cash box on the alleged ground that the parties in that case where the cash bond was deposited informed her that they would settle the case amicably. Respondent Baroy declared that she finally deposited the aforementioned cash bond of One Thousand (P1,000.00) Pesos with the Land Bank of the Philippines (LBP) in February, 1993, after this administrative case was already filed (TSN, pp. 27-28; 12-2293). The Pass Book, however, shows that actually Baroy opened an account with the LBP, Naga Branch, only on March 26, 1993 when she deposited an amount of Two Thousand (P2,000.00) Pesos (Exhs. 8 to 8-1-a). She claims that One Thousand (P1,000.000) Pesos of the initial deposit was the cash bond of Dacara. If it were true, it was only after keeping to herself the cash bond of One Thousand (P1,000.00) Pesos for around one year and five months when she finally deposited it because of the filing of this case. On April 29, 1993, or only one month and two days after she finally deposited the One Thousand (P1,000.00) Pesos cash bond of Dacara, she withdrew it from the bank without any authority or order from the court. It was only on July 23, 1993, or after almost three (3) months after she withdrew it, when she redeposited said cash bond (TSN, p. 6; 1-494). The evidence presented in this case also show that on February 28, 1993 respondent Baroy received also a cash bond of Three Thousand (P3,000.00) Pesos from a certain Alfredo Seprones in Crim. Case No. 5180. For this cash bond deposit, respondent Baroy 269

University of the Cordilleras College of Law First Year C S.Y. 2013 - 2014 issued only an annumbered temporary receipt (Exh. X and X-1). Again Baroy just kept this Three Thousand (P3,000.00) Pesos cash bond to herself. She did not deposit it either (in) a bank or (with) the Municipal Treasurer. Her explanation was that the parties in Crim. Case No. 5180 informed her that they would settle the case amicably. It was on April 26, 1993, or almost two months later when Judge Palaypayon issued an order for the release of said cash bond (Exh. 7). Respondent Baroy also admitted that since she assumed office on October 21, 1991 she used to issue temporary receipt only for cash bond deposits and other payments and collections she received. She further admitted that some of these temporary receipts she issued she failed to place the number of the receipts such as that receipt marked Exhibit X (TSN, p. 35; 11-23-93). Baroy claims that she did not know that she had to use the official receipts of the Supreme Court. It was only from February, 1993, after this case was already filed, when she only started issuing official receipts. The next charge against the respondents is that in order to be appointed Clerk of Court, Baroy gave Judge Palaypayon an air conditioner as a gift. The evidence adduced with respect to this charge, show that on August 24, 1991 Baroy bought an air conditioner for the sum of Seventeen Thousand Six Hundred (P17,600.00) Pesos (Exhs. I and I-1). The same was paid partly in cash and in check (Exhs. I-2 and I-3). When the air conditioner was brought to court in order to be installed in the chamber of Judge Palaypayon, it was still placed in the same box when it was bought and was not used yet. The respondents claim that Baroy sold it to Judge Palaypayon for Twenty Thousand (P20,00.00) Pesos on installment basis with a down payment of Five Thousand (P5,000.00) Pesos and as proof thereof the respondents presented a typewritten receipt dated May 29, 1993 (Exh. 22). The receipt was signed by both respondents and by the Municipal Mayor of Tinambac, Camarines Sur and another person as witness. The alleged sale between respondents is not beyond suspicion. It was bought by Baroy at a time when she was applying for the vacant position of Clerk of Court (to) which she was eventually appointed in October, 1991. From the time she bought the air conditioner on August 24, 1991 until it was installed in the office of Judge Palaypayon it was not used yet. The sale to Judge Palaypayon was only evidenced by a mere typewritten receipt dated May 29, 1992 when this case was already filed. The receipt could have been easily prepared. The Municipal Mayor of Tinambac who signed in the receipt as a witness did not testify in this case. The sale is between the Clerk of Court and the Judge of the same court. All these circumstances give rise to suspicion of at least impropriety. Judges should avoid such action as would subject (them) to suspicion and (their) conduct should be free from the appearance of impropriety (Jaagueta vs. Boncasos, 60 SCRA 27). With respect to the charge that Judge Palaypayon received a cash bond deposit of One Thousand (P1,000.00) Pesos from Januaria Dacara without issuing a receipt, Dacara executed an affidavit regarding this charge that Judge Palaypayon did not give her a receipt for the P1,000.00 cash bond she deposited (Exh. N). Her affidavit, however, has no probative value as she did not show that this cash bond of P1,000.00 found its way into the hands of respondent Baroy who issued only a temporary receipt for it and this has been discussed earlier. Another charge against Judge Palaypayon is the getting of detention prisoners to work in his house and one of them escaped while in his custody and was never found again. To hide this fact, the case against said accused was ordered archived by Judge Palaypayon. The evidence adduced with respect to this particular charge, show that in Crim. Case No. 5647 entitled People vs. Stephen Kalaw, Alex Alano and Allan Adupe, accused Alex Alano and Allan Adupe were arrested on April 12, 1991 and placed in the municipal jail of Tinambac, Camarines Sur (Exhs. 0, 0-1, 0-2 and 0-3; Exh. 25). The evidence presented that Alex Alano was taken by Judge Palaypayon from the municipal jail where said accused was confined and that he escaped while in custody of Judge Palaypayon is solely testimonial, particularly that of David Ortiz, a former utility worker of the MTC of Tinambac.

270

University of the Cordilleras College of Law First Year C S.Y. 2013 - 2014 Herein investigator finds said evidence not sufficient. The complainants should have presented records from the police of Tinambac to show that Judge Palaypayon took out from the municipal jail Alex Alano where he was under detention and said accused escaped while in the custody of Judge Palaypayon. The order, however, of Judge Palaypayon dated April 6, 1992 in Crim. Case No. 5047 archiving said case appears to be without basis. The order states: "this case was filed on April 12, 1991 and the records show that the warrant of arrest (was) issued against the accused, but up to this moment there is no return of service for the warrant of arrest issued against said accused" (Exh. 0-4). The records of said case, however, show that in fact there was a return of the service of the warrant of arrest dated April 12, 1991 showing that Alano and Adupe were arrested (Exh. 0-3). Judge Palaypayon explained that his order dated April 6, 1992 archiving Crim. Case No. 5047 referred only to one of the accused who remained at large. The explanation cannot be accepted because the two other accused, Alano and Adupe, were arrested. Judge Palaypayon should have issued an order for the arrest of Adupe who allegedly jumped bail, but Alano was supposed to be confined in the municipal jail if his claim is true that he did not take custody of Alano. The explanation also of Judge Palaypayon why he ordered the case archived was because he heard from the police that Alano escaped. This explanation is not acceptable either. He should ha(ve) set the case and if the police failed to bring to court Alano, the former should have been required to explain in writing why Alano was not brought to court. If the explanation was that Alano escaped from jail, he should have issued an order for his arrest. It is only later on when he could not be arrested when the case should have been ordered archived. The order archiving this case for the reason that he only heard that Alano escaped is another circumstance which gave rise to a suspicion that Alano might have really escaped while in his custody only that the complainants could not present records or other documentary evidence to prove the same. The last charge against the respondents is that they collected filing fees on collection cases filed by the Rural Bank of Tinambac, Camarines Sur which was supposed to be exempted in paying filing fees under existing laws and that the filing fees received was deposited by respondent Baroy in her personal account in the bank. The evidence presented show that on February 4, 1992 the Rural Bank of Tinambac filed ten (10) civil cases for collection against farmers and it paid the total amount of Four Hundred (P400.00) Pesos representing filing fees. The complainants cited Section 14 of Republic Act 720, as amended, which exempts Rural Banks (from) the payment of filing fees on collection of sums of money cases filed against farmers on loans they obtained. Judge Palaypayon, however, had nothing to do with the payment of the filing fees of the Rural Bank of Tinambac as it was respondent Baroy who received them and besides, on February 4, 1992, he was on sick leave. On her part Baroy claims that the bank paid voluntarily the filing fees. The records, however, shows that respondent Baroy sent a letter to the manager of the bank dated January 28, 1992 to the effect that if the bank would not pay she would submit all Rural Bank cases for dismissal (Annex 6, comment by respondent Baroy). Respondent Baroy should have checked whether the Rural Bank of Tinambac was really exempt from the payment of filing fees pursuant to Republic Act 720, as amended, instead of threatening the bank to have its cases be submitted to the court in order to have them dismissed. Here the payment of the filing fees was made on February 4, 1992, but the Four Hundred (P400.00) Pesos was only turned over to the Municipal Treasurer on March 12, 1992. Here, there is an undue delay again in complying with her obligation as accountable officer. In view of the foregoing findings that the evidence presented by the complainants sufficiently show that respondent Judge Lucio P. Palaypayon, Jr. had solemnized marriages, particularly that of Sammy Bocaya and Gina Besmonte, without a marriage license, and that it having been shown that he did not comply with his duty in closely 271

University of the Cordilleras College of Law First Year C S.Y. 2013 - 2014 supervising his clerk of court in the preparation of the monthly report of cases being submitted to the Supreme Court, particularly for the months of July and September, 1992 where it has been proven that the reports for said two (2) months were falsified with respect to the number of documents notarized, it is respectfully recommended that he be imposed a fine of TEN THOUSAND (P10,000.00) PESOS with a warning that the same or similar offenses will be more severely dealt with. The fact that Judge Palaypayon did not sign the marriage contracts or certificates of those marriages he solemnized without a marriage license, there were no dates placed in the marriage contracts to show when they were solemnized, the contracting parties were not furnished their marriage contracts and the Local Civil Registrar was not being sent any copy of the marriage contract, will not absolve him from liability. By solemnizing alone a marriage without a marriage license he as the solemnizing officer is the one responsible for the irregularity in not complying (with) the formal requ(i)sites of marriage and under Article 4(3) of the Family Code of the Philippines, he shall be civilly, criminally and administratively liable. Judge Palaypayon is likewise liable for his negligence or failure to comply with his duty of closely supervising his clerk of court in the performance of the latter's duties and functions, particularly the preparation of the monthly report of cases (Bendesula vs. Laya, 58 SCRA 16). His explanation that he only signed the monthly report of cases only when his clerk of court already signed the same, cannot be accepted. It is his duty to closely supervise her, to check and verify the records if the monthly reports prepared by his clerk of court do not contain false statements. It was held that "A judge cannot take refuge behind the inefficiency or incompetence of court personnel (Nidua vs. Lazaro, 174 SCRA 158). In view also of the foregoing finding that respondent Nelia Esmeralda-Baroy, the clerk of court of the Municipal Trial Court of Tinambac, Camarines Sur, has been found to have falsified the monthly report of cases for the months of July and September, 1992 with respect to the number of documents notarized, for having failed to account (for) the notarial fees she received for said two (2) months period; for having failed to account (for) the solemnization fees of those marriages allegedly not solemnized, but the solemnization fees were not returned; for unauthorized issuance of temporary receipts, some of which were issued unnumbered; for receiving the cash bond of Dacara on October 29, 1991 in the amount of One Thousand (P1,000.00) Pesos for which she issued only a temporary receipt (Exh. Y) and for depositing it with the Land Bank of the Philippines only on March 26, 1993, or after one year and five months in her possession and after this case was already filed; for withdrawing said cash bond of One Thousand (P1,000.00) Pesos on April 29, 1993 without any court order or authority and redepositing it only on July 23, 1993; for receiving a cash bond of Three Thousand (P3,000.00) Pesos from Alfredo Seprones in Crim. Case No. 5180, MTC, Tinambac, Camarines Sur, for which she issued only an unnumbered temporary receipt (Exhs. X and X-1) and for not depositing it with a bank or with the Municipal Treasurer until it was ordered released; and for requiring the Rural Bank of Tinambac, Camarines Sur to pay filing fees on February 4, 1992 for collection cases filed against farmers in the amount of Four Hundred (P400.00) Pesos, but turning over said amount to the Municipal Treasurer only on March 12, 1992, it is respectfully recommended that said respondent clerk of court Nelia Esmeralda-Baroy be dismissed from the service. It is provided that "Withdrawal of court deposits shall be by the clerk of court who shall issue official receipt to the provincial, city or municipal treasurer for the amount withdrawn. Court deposits cannot be withdrawn except by order of the court, . . . ." (Revised Manual of Instructions for Treasurers, Sec. 183, 184 and 626; p. 127, Manual for Clerks of Court). A circular also provides that the Clerks of Court shall immediately issue an official receipt upon receipt of deposits from party litigants and thereafter deposit intact the collection with the municipal, city or provincial treasurer and their deposits, can only be withdrawn upon proper receipt and order of the Court (DOJ Circular No. 52, 26 April 1968; p. 136, Manual for Clerks of Court). Supreme Court Memorandum Circular No. 5, 25 November 1982, also provides that "all collections of funds of fiduciary character including rental deposits, shall be deposited immediately by 272

University of the Cordilleras College of Law First Year C S.Y. 2013 - 2014 the clerk of court concerned upon receipt thereof with City, Municipal or Provincial Treasurer where his court is located" and that "no withdrawal of any of such deposits shall be made except upon lawful order of the court exercising jurisdiction over the subject matter. Respondent Baroy had either failed to comply with the foregoing circulars, or deliberately disregarded, or even intentionally violated them. By her conduct, she demonstrated her callous unconcern for the obligations and responsibility of her duties and functions as a clerk of court and accountable officer. The gross neglect of her duties shown by her constitute(s) a serious misconduct which warrant(s) her removal from office. In the case of Belen P. Ferriola vs. Norma Hiam, Clerk of Court, MTCC, Branch I, Batangas City; A.M. No. P-90-414; August 9, 1993, it was held that "The clerk of court is not authorized to keep funds in his/her custody; monies received by him/her shall be deposited immediately upon receipt thereof with the City, Municipal or Provincial Treasurer. Supreme Court Circular Nos. 5 dated November 25, 1982 and 5-A dated December 3, 1982. Respondent Hiam's failure to remit the cash bail bonds and fine she collected constitutes serious misconduct and her misappropriation of said funds constitutes dishonesty. "Respondent Norma Hiam was found guilty of dishonesty and serious misconduct prejudicial to the best interest of the service and (the Court) ordered her immediate dismissal (from) the service. xxx xxx xxx We here emphasize once again our adjuration that the conduct and behavior of everyone connected with an office charged with the dispensation of justice, from the presiding judge to the lowliest clerk, should be circumscribed with the heavy burden of responsibility. His conduct, at all times, must not only be characterized by propriety and decorum but, above all else, must be beyond suspicion. Every employee should be an example of integrity, uprightness and honesty. Integrity in a judicial office is more than a virtue, it is a necessity. It applies, without qualification as to rank or position, from the judge to the least of its personnel, they being standard-bearers of the exacting norms of ethics and morality imposed upon a Court of justice. On the charge regarding illegal marriages the Family Code pertinently provides that the formal requisites of marriage are, inter alia, a valid marriage license except in the cases provided for therein. Complementarily, it declares that the absence of any of the essential or formal requisites shall generally render the marriage void ab initio and that, while an irregularity in the formal requisites shall not affect the validity of the marriage, the party or parties responsible for the irregularity shall be civilly, criminally and administratively liable. The civil aspect is addressed to the contracting parties and those affected by the illegal marriages, and what we are providing for herein pertains to the administrative liability of respondents, all without prejudice to their criminal responsibility. The Revised Penal Code provides that "(p)riests or ministers of any religious denomination or sect, or civil authorities who shall perform or authorize any illegal marriage ceremony shall be punished in accordance with the provisions of the Marriage Law." This is of course, within the province of the prosecutorial agencies of the Government. The recommendation with respect to the administrative sanction to be imposed on respondent judge should, therefore, be modified. For one, with respect to the charge of illegal solemnization of marriages, it does appear that he had not taken to heart, but actually trifled with, the law's concern for the institution of marriage and the legal effects flowing from civil status. This, and his undeniable participation in the other offenses charged as hereinbefore narrated in detail, approximate such serious degree of misconduct and of gross negligence in the performance of judicial duties as to ineludibly require a higher penalty. WHEREFORE, the Court hereby imposes a FINE of P20,000.00 on respondent Judge Lucio P. Palaypayon. Jr., with a stern warning that any repetition of the same or similar offenses in the future will definitely be severely dealt with. Respondent Nelia Esmeralda-Baroy is hereby DISMISSED from the service, with forfeiture of all retirement benefits and with prejudice to employment in any branch, agency or instrumentality of the Government, including government-owned or controlled corporations. Let copies of this decision be spread on their records and furnished to the Office of the Ombudsman for appropriate action. 273

University of the Cordilleras College of Law First Year C S.Y. 2013 - 2014 SO ORDERED. Case Digest JUVY N. COSCA vs. HON. LUCIO P. PALAYPAYON, JR. A.M. No. MTJ-92-721 September 30, 1994 237 SCRA 249 PER CURIAM, J.: FACTS: The following are the complainants: Juvy N. Cosca (Stenographer 1), Edmundo B. Peralta (Interpreter 1), Ramon C. Sambo (Clerk II) and Apollo Villamora (Process Server). Respondents are Judge Lucio Palaypayon Jr., the presiding judge, and Nelia B. Esmeralda-Baroy, clerk of court II. All work in MTC-Tinambac, Camarines Sur. Complainants alleged that Palaypayon solemnized marriages even without the requisite of a marriage license. Hence, the following couples were able to get married just by paying the marriage fees to respondent Baroy: Alano P. Abellano & Nelly Edralin; Francisco Selpo & Julieta Carrido; Eddie Terrobias & Maria Gacer; Renato Gamay & Maricris Belga; Arsenio Sabater & Margarita Nacario; Sammy Bocaya & Gina Bismonte. As a consequence, the marriage contracts of the following couples did not reflect any marriage license number. In addition, Palaypayon did not sign the marriage contracts and did not indicate the date of solemnization reasoning out that he allegedly had to wait for the marriage license to be submitted by the parties which happens usually several days after the marriage ceremony. Palaypayon contends that marriage between Abellano & Edralin falls under Article 34 of the Civil Code thus exempted from the marriage license requirement. According to him, he gave strict instructions to complainant Sambo to furnish the couple copy of the marriage contract and to file the same with the civil registrar but the latter failed to do so. In order to solve the problem, the spouses subsequently formalized the marriage by securing a marriage license and executing their marriage contract, a copy of which was then filed with the civil registrar. The other five marriages were not illegally solemnized because Palaypayon did not sign their marriage contracts and the date and place of marriage are not included. It was alleged that copies of these marriage contracts are in the custody of complainant Sambo. The alleged marriage of Selpo & Carrido, Terrobias & Gacer, Gamay & Belga, Sabater & Nacario were not celebrated by him since he refused to solemnize them in the absence of a marriage license and that the marriage of Bocaya & Bismonte was celebrated even without the requisite license due to the insistence of the parties to avoid embarrassment with the guests which he again did not sign the marriage contract. An illegal solemnization of marriage was charged against the respondents. ISSUE: Whether or not the marriage solemnized by Judge Palaypayon were valid. HELD: Bocaya & Besmontes marriage was solemnized without a marriage license along with the other couples. The testimonies of Bocay and Pompeo Ariola including the photographs taken showed that it was really Judge Palaypayon who solemnized their marriage. Bocaya declared that they were advised by judge to return after 10 days after the solemnization and bring with them their marriage license. They already started living together as husband and wife even without the formal requisite. With respect to the photographs, judge explained that it was a simulated solemnization of marriage and not a real one. However, considering that there were pictures from the start of the wedding ceremony up to the signing of the marriage certificates in front of him. The court held that it is hard to believe that it was simulated. On the other hand, Judge Palaypayon admitted that he solemnized marriage between Abellano & Edralin and claimed it was under Article 34 of the Civil Code so the marriage license was dispensed with considering that the contracting parties executed a joint affidavit that they have been living together as husband and wife for almost 6 years already. However, it was shown in the marriage contract that Abellano was only 18 yrs 2months and 7 days old. If he and Edralin had been living together for 6 years already before they got married as what is stated in the joint affidavit, Abellano must have been less than 13 years old when they started living together which is hard to believe. Palaypayon should have been aware, as it is his duty to ascertain the qualification of the contracting parties who might have executed a false joint affidavit in order to avoid the marriage license requirement.

274

University of the Cordilleras College of Law First Year C S.Y. 2013 - 2014 Article 4 of the Family Code pertinently provides that in the absence of any of the essential or formal requisites shall render the marriage void ab initio whereas an irregularity in the formal requisite shall not affect the validity of the marriage but the party or parties responsible for the irregularity shall be civilly, criminally, and administratively liable. Republic vs Court of Appeals 236 SCRA 257 G.R. No. 103047 September 2, 1994 Full Case REPUBLIC OF THE PHILIPPINES, petitioner, vs. COURT OF APPEALS AND ANGELINA M. CASTRO, respondents. PUNO, J.: The case at bench originated from a petition filed by private respondent Angelina M. Castro in the Regional Trial Court of Quezon City seeking a judicial declaration of nullity of her marriage to Edwin F. Cardenas. 1 As ground therefor, Castro claims that no marriage license was ever issued to them prior to the solemnization of their marriage. Despite notice, defendant Edwin F. Cardenas failed to file his answer. Consequently, he was declared in default. Trial proceeded in his absence. The controlling facts are undisputed: On June 24, 1970, Angelina M. Castro and Edwin F. Cardenas were married in a civil ceremony performed by Judge Pablo M. Malvar, City Court Judge of Pasay City. The marriage was celebrated without the knowledge of Castro's parents. Defendant Cardenas personally attended to the processing of the documents required for the celebration of the marriage, including the procurement of the marriage, license. In fact, the marriage contract itself states that marriage license no. 3196182 was issued in the name of the contracting parties on June 24, 1970 in Pasig, Metro Manila. The couple did not immediately live together as husband and wife since the marriage was unknown to Castro's parents. Thus, it was only in March 1971, when Castro discovered she was pregnant, that the couple decided to live together. However, their cohabitation lasted only for four (4) months. Thereafter, the couple parted ways. On October 19, 1971, Castro gave birth. The baby was adopted by Castro's brother, with the consent of Cardenas. The baby is now in the United States. Desiring to follow her daughter, Castro wanted to put in order her marital status before leaving for the States. She thus consulted a lawyer, Atty. Frumencio E. Pulgar, regarding the possible annulment of her marriage. Through her lawyer's efforts, they discovered that there was no marriage license issued to Cardenas prior to the celebration of their marriage. As proof, Angelina Castro offered in evidence a certification from the Civil Register of Pasig, Metro Manila. It reads: February 20, 1987 TO WHOM IT MAY CONCERN: This is to certify that the names EDWIN F. CARDENAS and ANGELINA M. CASTRO who were allegedly married in the Pasay City Court on June 21, 1970 under an alleged (s)upportive marriage license no. 3196182 allegedly issued in the municipality on June 20, 1970 cannot be located as said license no. 3196182 does not appear from our records. Issued upon request of Mr. Ed Atanacio.

275

(Sgd) Senior Civil Registry Officer

University of the Cordilleras College of Law First Year C S.Y. 2013 - 2014 CENONA D.

QUINTOS

Castro testified that she did not go to the civil registrar of Pasig on or before June 24, 1970 in order to apply for a license. Neither did she sign any application therefor. She affixed her signature only on the marriage contract on June 24, 1970 in Pasay City. The trial court denied the petition. 2 It held that the above certification was inadequate to establish the alleged non-issuance of a marriage license prior to the celebration of the marriage between the parties. It ruled that the "inability of the certifying official to locate the marriage license is not conclusive to show that there was no marriage license issued." Unsatisfied with the decision, Castro appealed to respondent appellate court. She insisted that the certification from the local civil registrar sufficiently established the absence of a marriage license. As stated earlier, respondent appellate court reversed the Decision of the trial court. 3 It declared the marriage between the contracting parties null and void and directed the Civil Registrar of Pasig to cancel the subject marriage contract. Hence this petition for review on certiorari. Petitioner Republic of the Philippines urges that respondent appellate court erred when it ruled that the certification issued by the civil registrar that marriage license no. 3196182 was not in their record adequately proved that no such license was ever issued. Petitioner also faults the respondent court for relying on the self-serving and uncorroborated testimony of private respondent Castro that she had no part in the procurement of the subject marriage license. Petitioner thus insists that the certification and the uncorroborated testimony of private respondent are insufficient to overthrow the legal presumption regarding the validity of a marriage. Petitioner also points that in declaring the marriage between the parties as null and void, respondent appellate court disregarded the presumption that the solemnizing officer, Judge Pablo M. Malvar, regularly performed his duties when he attested in the marriage contract that marriage license no. 3196182 was duly presented to him before the solemnization of the subject marriage. The issues, being interrelated, shall be discussed jointly. The core issue presented by the case at bench is whether or not the documentary and testimonial evidence presented by private respondent are sufficient to establish that no marriage license was issued by the Civil Registrar of Pasig prior to the celebration of the marriage of private respondent to Edwin F. Cardenas. We affirm the impugned Decision. At the time the subject marriage was solemnized on June 24, 1970, the law governing marital relations was the New Civil Code. The law 4 provides that no marriage shall be solemnized without a marriage license first issued by a local civil registrar. Being one of the essential requisites of a valid marriage, absence of a license would render the marriage void ab initio. 5 Petitioner posits that the certification of the local civil registrar of due search and inability to find a record or entry to the effect that marriage license no. 3196182 was issued to the parties is not adequate to prove its non-issuance. We hold otherwise. The presentation of such certification in court is sanctioned by Section 29, Rule 132 of the Rules of Court, viz.: Sec. 29. Proof of lack of record. A written statement signed by an officer having custody of an official record or by his deputy, that after diligent search, no record or entry of a specified tenor is found to exist in the records of his office, accompanied by a certificate as above provided, is admissible as evidence that the records of his office contain no such record or entry. 276

University of the Cordilleras College of Law First Year C S.Y. 2013 - 2014 The above Rule authorized the custodian of documents to certify that despite diligent search, a particular document does not exist in his office or that a particular entry of a specified tenor was not to be found in a register. As custodians of public documents, civil registrars are public officers charged with the duty, inter alia, of maintaining a register book where they are required to enter all applications for marriage licenses, including the names of the applicants, the date the marriage license was issued and such other relevant data. 6 The certification of "due search and inability to find" issued by the civil registrar of Pasig enjoys probative value, he being the officer charged under the law to keep a record of all data relative to the issuance of a marriage license. Unaccompanied by any circumstance of suspicion and pursuant to Section 29, Rule 132 of the Rules of Court, a certificate of "due search and inability to find" sufficiently proved that his office did not issue marriage license no. 3196182 to the contracting parties. The fact that private respondent Castro offered only her testimony in support of her petition is, in itself, not a ground to deny her petition. The failure to offer any other witness to corroborate her testimony is mainly due to the peculiar circumstances of the case. It will be remembered that the subject marriage was a civil ceremony performed by a judge of a city court. The subject marriage is one of those commonly known as a "secret marriage" a legally non-existent phrase but ordinarily used to refer to a civil marriage celebrated without the knowledge of the relatives and/or friends of either or both of the contracting parties. The records show that the marriage between Castro and Cardenas was initially unknown to the parents of the former. Surely, the fact that only private respondent Castro testified during the trial cannot be held against her. Her husband, Edwin F. Cardenas, was duly served with notice of the proceedings and a copy of the petition. Despite receipt thereof, he chose to ignore the same. For failure to answer, he was properly declared in default. Private respondent cannot be faulted for her husband's lack of interest to participate in the proceedings. There was absolutely no evidence on record to show that there was collusion between private respondent and her husband Cardenas. It is noteworthy to mention that the finding of the appellate court that the marriage between the contracting parties is null and void for lack of a marriage license does not discount the fact that indeed, a spurious marriage license, purporting to be issued by the civil registrar of Pasig, may have been presented by Cardenas to the solemnizing officer. In fine, we hold that, under the circumstances of the case, the documentary and testimonial evidence presented by private respondent Castro sufficiently established the absence of the subject marriage license. IN VIEW WHEREOF, the petition is DENIED there being no showing of any reversible error committed by respondent appellate court. SO ORDERED.

277

University of the Cordilleras College of Law First Year C S.Y. 2013 - 2014 Case Digest REPUBLIC vs. COURT OF APPEALS G.R. No. 103047 September 2, 1994 236 SCRA 257 PUNO, J. FACTS: Angelina M. Castro and Edwin F. Cardenas were married in a civil ceremony without the knowledge of the formers parents. All the documents required for the celebration of the marriage which includes procurement of marriage license, was attended by Cardenas. It was stated in the marriage contract that marriage license no. 3196182 was issued. The cohabitation of Castro and Cardenas lasted only for four (4) months after which they parted ways. Castro sought the advice of a lawyer for a possible annulment of her marriage with Cardenas before leaving for the States to follow her daughter who was adopted by her brother with the consent of Cardenas. The Civil Registrar of Pasig issued a certification stating that Castro and Cardenas were allegedly married in the Pasay Court on June 21, 1970 under an alleged marriage license no. 3196182 which was allegedly issued on June 20, 1970 but such cannot be located since it does not appear in their records. It was then that she found out that there was no marriage license issued prior to the celebration of her marriage with Cardenas. Castro filed a petition seeking a judicial declaration of nullity of her marriage with Edwin Cardenas. The Regional Trial Court denied her petition. It ruled that inability of the certifying official to locate the marriage license is not conclusive to show that there was no marriage license issued. Castro appealed to respondent appellate court contending that the certification from the local civil registrar sufficiently established the absence of a marriage license. The respondent appellate court reversed the ruling of the trial court declaring that the marriage between the contracting parties is null and void and directed the Civil Registrar of Pasig to cancel the marriage contract. However, the Republic of the Philippines, the petitioner herein, brought a petition for review on certiorari which alleged that the certification and the uncorroborated testimony of Castro are not sufficient to overthrow the legal presumption regarding the validity of a marriage. ISSUE: Whether or not the documentary and testimonial evidence presented by private respondent are sufficient to establish that no marriage license was issued prior to the celebration of marriage. RULING: Yes. The Court ruled that the certification of "due search and inability to find" issued by the civil registrar of Pasig enjoys probative value, he being the officer charged under the law to keep a record of all data relative to the issuance of a marriage license. Unaccompanied by any circumstance of suspicion and pursuant to Section 29, Rule 132 of the Rules of Court, a certificate of "due search and inability to find" sufficiently proved that his office did not issue marriage license no. 3196182 to the contracting parties. The fact that private respondent Castro offered only her testimony in support of her petition is, in itself, not aground to deny her petition. The failure to offer any other witness to corroborate her testimony is mainly due to the peculiar circumstances of the case. The finding of the appellate court that the marriage between the contracting parties is null and void for lack of a marriage license does not discount the fact that indeed, a spurious marriage license, purporting to be issued by the civil registrar of Pasig, may have been presented by Cardenas to the solemnizing officer. It was held that under the circumstances of the case, the documentary and testimonial evidence presented by private respondent Castro sufficiently established the absence of the subject marriage license. Therefore, the petition is DENIED there being no showing of any reversible error committed by respondent appellate court.

278

University of the Cordilleras College of Law First Year C S.Y. 2013 - 2014 Quita vs Court of Appeals 300 SCRA 406 G.R. No. 124862 December 22, 1998 Full Case FE D. QUITA, petitioner, vs. COURT OF APPEALS and BLANDINA DANDAN, * respondents. BELLOSILLO, J.: FE D. QUITA and Arturo T. Padlan, both Filipinos, were married in the Philippines on 18 May 1941. They were not however blessed with children. Somewhere along the way their relationship soured. Eventually Fe sued Arturo for divorce in San Francisco, California, U.S.A. She submitted in the divorce proceedings a private writing dated 19 July 1950 evidencing their agreement to live separately from each other and a settlement of their conjugal properties. On 23 July 1954 she obtained a final judgment of divorce. Three (3) weeks thereafter she married a certain Felix Tupaz in the same locality but their relationship also ended in a divorce. Still in the U.S.A., she married for the third time, to a certain Wernimont. On 16 April 1972 Arturo died. He left no will. On 31 August 1972 Lino Javier Inciong filed a petition with the Regional Trial Court of Quezon City for issuance of letters of administration concerning the estate of Arturo in favor of the Philippine Trust Company. Respondent Blandina Dandan (also referred to as Blandina Padlan), claiming to be the surviving spouse of Arturo Padlan, and Claro, Alexis, Ricardo, Emmanuel, Zenaida and Yolanda, all surnamed Padlan, named in the children of Arturo Padlan opposed the petition and prayed for the appointment instead of Atty. Leonardo Casaba, which was resolved in favor of the latter. Upon motion of the oppositors themselves, Atty. Cabasal was later replaced by Higino Castillon. On 30 April 1973 the oppositors (Blandina and Padlan children) submitted certified photocopies of the 19 July 1950 private writing and the final judgment of divorce between petitioner and Arturo. Later Ruperto T. Padlan, claiming to be the sole surviving brother of the deceased Arturo, intervened. On 7 October 1987 petitioner moved for the immediate declaration of heirs of the decedent and the distribution of his estate. At the scheduled hearing on 23 October 1987, private respondent as well as the six (6) Padlan children and Ruperto failed to appear despite due notice. On the same day, the trial court required the submission of the records of birth of the Padlan children within ten (10) days from receipt thereof, after which, with or without the documents, the issue on the declaration of heirs would be considered submitted for resolution. The prescribed period lapsed without the required documents being submitted. The trial court invoking Tenchavez v. Escao which held that "a foreign divorce between Filipino citizens sought and decreed after the effectivity of the present Civil Code (Rep. Act 386) was not entitled to recognition as valid in this jurisdiction," disregarded the divorce between petitioner and Arturo. Consecuently, it expressed the view that their marriage subsisted until the death of Arturo in 1972. Neither did it consider valid their extrajudicial settlement of conjugal properties due to lack of judicial approval. On the other hand, it opined that there was no showing that marriage existed between private respondent and Arturo, much less was it shown that the alleged Padlan children had been acknowledged by the deceased as his children with her. As regards Ruperto, it found that he was a brother of Arturo. On 27 November 1987 only petitioner and Ruperto were declared the intestate heirs of Arturo. Accordingly, equal adjudication of the net hereditary estate was ordered in favor of the two intestate heirs. On motion for reconsideration, Blandina and the Padlan children were allowed to present proofs that the recognition of the children by the deceased as his legitimate children, except Alexis who was recognized as his illegitimate child, had been made in their respective records of birth. Thus on 15 February 1988 partial reconsideration was granted declaring the Padlan children, with the exception of Alexis, entitled to one-half of the estate to the exclusion of Ruperto Padlan, and petitioner to the other half. Private respondent was not declared an heir. Although it was stated in the aforementioned records of birth that she and Arturo were married on 22 April 1947, their marriage was clearly void since it was celebrated during the existence of his previous marriage to petitioner.

279

University of the Cordilleras College of Law First Year C S.Y. 2013 - 2014 In their appeal to the Court of Appeals, Blandina and her children assigned as one of the errors allegedly committed by the trial court the circumstance that the case was decided without a hearing, in violation of Sec. 1, Rule 90, of the Rules of Court, which provides that if there is a controversy before the court as to who are the lawful heirs of the deceased person or as to the distributive shares to which each person is entitled under the law, the controversy shall be heard and decided as in ordinary cases. Respondent appellate court found this ground alone sufficient to sustain the appeal; hence, on 11 September 1995 it declared null and void the 27 November 1987 decision and 15 February 1988 order of the trial court, and directed the remand of the case to the trial court for further proceedings. On 18 April 1996 it denied reconsideration. Should this case be remanded to the lower court for further proceedings? Petitioner insists that there is no need because, first, no legal or factual issue obtains for resolution either as to the heirship of the Padlan children or as to the decedent; and, second, the issue as to who between petitioner and private respondent is the proper hier of the decedent is one of law which can be resolved in the present petition based on establish facts and admissions of the parties. We cannot sustain petitioner. The provision relied upon by respondent court is clear: If there is a controversybefore the court as to who are the lawful heirs of the deceased person or as to the distributive shares to which each person is entitled under the law, the controversy shall be heard and decided as in ordinary cases. We agree with petitioner that no dispute exists either as to the right of the six (6) Padlan children to inherit from the decedent because there are proofs that they have been duly acknowledged by him and petitioner herself even recognizes them as heirs of Arturo Padlan; nor as to their respective hereditary shares. But controversy remains as to who is the legitimate surviving spouse of Arturo. The trial court, after the parties other than petitioner failed to appear during the scheduled hearing on 23 October 1987 of the motion for immediate declaration of heirs and distribution of estate, simply issued an order requiring the submission of the records of birth of the Padlan children within ten (10) days from receipt thereof, after which, with or without the documents, the issue on declaration of heirs would be deemed submitted for resolution. We note that in her comment to petitioner's motion private respondent raised, among others, the issue as to whether petitioner was still entitled to inherit from the decedent considering that she had secured a divorce in the U.S.A. and in fact had twice remarried. She also invoked the above quoted procedural rule. To this, petitioner replied that Arturo was a Filipino and as such remained legally married to her in spite of the divorce they obtained. Reading between the lines, the implication is that petitioner was no longer a Filipino citizen at the time of her divorce from Arturo. This should have prompted the trial court to conduct a hearing to establish her citizenship. The purpose of a hearing is to ascertain the truth of the matters in issue with the aid of documentary and testimonial evidence as well as the arguments of the parties either supporting or opposing the evidence. Instead, the lower court perfunctorily settled her claim in her favor by merely applying the ruling in Tenchavez v. Escao. Then in private respondent's motion to set aside and/or reconsider the lower court's decision she stressed that the citizenship of petitioner was relevant in the light of the ruling in Van Dorn v. Romillo Jr. that aliens may obtain divorces abroad, which may be recognized in the Philippines, provided they are valid according to their national law. She prayed therefore that the case be set for hearing. Petitioner opposed the motion but failed to squarely address the issue on her citizenship. The trial court did not grant private respondent's prayer for a hearing but proceeded to resolve her motion with the finding that both petitioner and Arturo were "Filipino citizens and were married in the Philippines." It maintained that their divorce obtained in 1954 in San Francisco, California, U.S.A., was not valid in Philippine jurisdiction. We deduce that the finding on their citizenship pertained solely to the time of their marriage as the trial court was not supplied with a basis to determine petitioner's citizenship at the time of their divorce. The doubt persisted as to whether she was still a Filipino citizen when their divorce was decreed. The trial court must have overlooked the materiality of this aspect. Once proved that she was no longer a Filipino citizen at the time of their divorce, Van Dorn would become applicable and petitioner could very well lose her right to inherit from Arturo. Respondent again raised in her appeal the issue on petitioner's citizenship; it did not merit enlightenment however from petitioner. In the present proceeding, petitioner's citizenship is brought anew to the fore by private respondent. She even furnishes the Court with the transcript of stenographic notes taken on 5 May 280

University of the Cordilleras College of Law First Year C S.Y. 2013 - 2014 1995 during the hearing for the reconstitution of the original of a certain transfer certificate title as well as the issuance of new owner's duplicate copy thereof before another trial court. When asked whether she was an American citizen petitioner answered that she was since 1954. Significantly, the decree of divorce of petitioner and Arturo was obtained in the same year. Petitioner however did not bother to file a reply memorandum to erase the uncertainty about her citizenship at the time of their divorce, a factual issue requiring hearings to be conducted by the trial court. Consequently, respondent appellate court did not err in ordering the case returned to the trial court for further proceedings. We emphasize however that the question to be determined by the trial court should be limited only to the right of petitioner to inherit from Arturo as his surviving spouse. Private respondent's claim to heirship was already resolved by the trial court. She and Arturo were married on 22 April 1947 while the prior marriage of petitioner and Arturo was subsisting thereby resulting in a bigamous marriage considered void from the beginning under Arts. 80 and 83 of the Civil Code. Consequently, she is not a surviving spouse that can inherit from him as this status presupposes a legitimate relationship. As regards the motion of private respondent for petitioner and a her counsel to be declared in contempt of court and that the present petition be dismissed for forum shopping, the same lacks merit. For forum shopping to exist the actions must involve the same transactions and same essential facts and circumstances. There must also be identical causes of action, subject matter and issue. The present petition deals with declaration of heirship while the subsequent petitions filed before the three (3) trial courts concern the issuance of new owner's duplicate copies of titles of certain properties belonging to the estate of Arturo. Obviously, there is no reason to declare the existence of forum shopping. WHEREFORE, the petition is DENIED. The decision of respondent Court of Appeals ordering the remand of the case to the court of origin for further proceedings and declaring null and void its decision holding petitioner Fe D. Quita and Ruperto T. Padlan as intestate heirs is AFFIRMED. The order of the appellate court modifying its previous decision by granting one-half (1/2) of the net hereditary estate to the Padlan children, namely, Claro, Ricardo, Emmanuel, Zenaida and Yolanda, with the exception of Alexis, all surnamed Padlan, instead of Arturo's brother Ruperto Padlan, is likewise AFFIRMED. The Court however emphasizes that the reception of evidence by the trial court should he limited to the hereditary rights of petitioner as the surviving spouse of Arturo Padlan. The motion to declare petitioner and her counsel in contempt of court and to dismiss the present petition for forum shopping is DENIED. SO ORDERED.

281

University of the Cordilleras College of Law First Year C S.Y. 2013 - 2014 Case Digest FE D. QUITA vs. COURT OF APPEALS and BLANDINA DANDAN G.R. No. 124862. December 22, 1998 300 SCRA 406 BELLOSILLO, J . Facts: Fe D. Quita, the petitioner, and Arturo T. Padlan, both Filipinos, were married in the Philippines on May 18, 1941. They got divorce in San Francisco on July 23, 1954. Both of them remarried another person. Arturo remarried Bladina Dandan, the respondent herewith. They were blessed with six children. On April 16, 1972, when Arturo died, the trial court was set to declared as to who will be the intestate heirs. The trial court invoking Tenchavez vs Escano case held that the divorce acquired by the petitioner is not recognized in our country. Private respondent stressed that the citizenship of petitioner was relevant in the light of the ruling in Van Dorn v. Rommillo Jr. that aliens who obtain divorce abroad are recognized in the Philippnes provided they are valid according to their national law. The petitioner herself answered that she was an American citizen since 1954. Through the hearing she also stated that Arturo was a Filipino at the time she obtained the divorce. Implying the she was no longer a Filipino citizen. The Trial court disregarded the respondents statement. The net hereditary estate was ordered in favor the Fe D. Quita and Ruperto, the brother of Arturo. Blandina and thePadlan children moved for reconsideration. On February 15, 1988 partial reconsideration was granted declaring the Padlan children, with the exception of Alexis, entitled to one-half of the estate to the exclusion of Ruperto Padlan, and the other half to Fe Quita.Private respondent was not declared an heir for her marriage to Arturo was declared void since it was celebrated during the existence of his previous marriage to petitioner.Blandina and her children appeal to the Court of Appeals that the case was decided without a hearing in violation of the Rules of Court. Issue: Whether or not Blandinas marriage to Arturo void ab initio. Whether or not Fe D. Quita be declared the primary beneficiary as surviving spouse of Arturo. Held: No. The marriage of Blandina and Arturo is not void. The citizenship of Fe D.Quita at the time of their divorce is relevant to this case. The divorce is valid here since she was already an alien at the time she obtained divorce, and such is valid in their countrys national law. Thus, Fe D. Quita is no longer recognized as a wife of Arturo. She cannot be the primary beneficiary or will be recognized as surviving spouse of Arturo.

282

University of the Cordilleras College of Law First Year C S.Y. 2013 - 2014 Garcia vs Recio 366 SCRA 437 G.R. No. 138322 October 2, 2001 Full Case GRACE J. GARCIA, a.k.a. GRACE J. GARCIA-RECIO, petitioner, vs. REDERICK A. RECIO, respondents. PANGANIBAN, J.: A divorce obtained abroad by an alien may be recognized in our jurisdiction, provided such decree is valid according to the national law of the foreigner. However, the divorce decree and the governing personal law of the alien spouse who obtained the divorce must be proven. Our courts do not take judicial notice of foreign laws and judgment; hence, like any other facts, both the divorce decree and the national law of the alien must be alleged and proven according to our law on evidence. The Case Before us is a Petition for Review under Rule 45 of the Rules of Court, seeking to nullify the January 7, 1999 Decision and the March 24, 1999 Order of the Regional Trial Court of Cabanatuan City, Branch 28, in Civil Case No. 3026-AF. The assailed Decision disposed as follows: "WHEREFORE, this Court declares the marriage between Grace J. Garcia and Rederick A. Recio solemnized on January 12, 1994 at Cabanatuan City as dissolved and both parties can now remarry under existing and applicable laws to any and/or both parties." The assailed Order denied reconsideration of the above-quoted Decision. The Facts Rederick A. Recio, a Filipino, was married to Editha Samson, an Australian citizen, in Malabon, Rizal, on March 1, 1987. They lived together as husband and wife in Australia. On May 18, 1989, a decree of divorce, purportedly dissolving the marriage, was issued by an Australian family court. On June 26, 1992, respondent became an Australian citizen, as shown by a "Certificate of Australian Citizenship" issued by the Australian government. Petitioner a Filipina and respondent were married on January 12, 1994 in Our Lady of Perpetual Help Church in Cabanatuan City. In their application for a marriage license, respondent was declared as "single" and "Filipino." Starting October 22, 1995, petitioner and respondent lived separately without prior judicial dissolution of their marriage. While the two were still in Australia, their conjugal assets were divided on May 16, 1996, in accordance with their Statutory Declarations secured in Australia. On March 3, 1998, petitioner filed a Complaint for Declaration of Nullity of Marriage in the court a quo, on the ground of bigamy respondent allegedly had a prior subsisting marriage at the time he married her on January 12, 1994. She claimed that she learned of respondent's marriage to Editha Samson only in November, 1997. In his Answer, respondent averred that, as far back as 1993, he had revealed to petitioner his prior marriage andits subsequent dissolution. He contended that his first marriage to an Australian citizen had been validly dissolved by a divorce decree obtained in Australian in 1989; thus, he was legally capacitated to marry petitioner in 1994.1wphi1.nt On July 7, 1998 or about five years after the couple's wedding and while the suit for the declaration of nullity was pending respondent was able to secure a divorce decree from a family court in Sydney, Australia because the "marriage ha[d] irretrievably broken down." Respondent prayed in his Answer that the Complained be dismissed on the ground that it stated no cause of action. The Office of the Solicitor General agreed with respondent. The court marked and admitted the 283

University of the Cordilleras College of Law First Year C S.Y. 2013 - 2014 documentary evidence of both parties. After they submitted their respective memoranda, the case was submitted for resolution. Thereafter, the trial court rendered the assailed Decision and Order. Ruling of the Trial Court The trial court declared the marriage dissolved on the ground that the divorce issued in Australia was valid and recognized in the Philippines. It deemed the marriage ended, but not on the basis of any defect in an essential element of the marriage; that is, respondent's alleged lack of legal capacity to remarry. Rather, it based its Decision on the divorce decree obtained by respondent. The Australian divorce had ended the marriage; thus, there was no more martial union to nullify or annual. Hence, this Petition. Issues Petitioner submits the following issues for our consideration: "I The trial court gravely erred in finding that the divorce decree obtained in Australia by the respondent ipso facto terminated his first marriage to Editha Samson thereby capacitating him to contract a second marriage with the petitioner. "2 The failure of the respondent, who is now a naturalized Australian, to present a certificate of legal capacity to marry constitutes absence of a substantial requisite voiding the petitioner' marriage to the respondent. "3 The trial court seriously erred in the application of Art. 26 of the Family Code in this case. "4 The trial court patently and grievously erred in disregarding Arts. 11, 13, 21, 35, 40, 52 and 53 of the Family Code as the applicable provisions in this case. "5 The trial court gravely erred in pronouncing that the divorce gravely erred in pronouncing that the divorce decree obtained by the respondent in Australia ipso facto capacitated the parties to remarry, without first securing a recognition of the judgment granting the divorce decree before our courts." The Petition raises five issues, but for purposes of this Decision, we shall concentrate on two pivotal ones: (1) whether the divorce between respondent and Editha Samson was proven, and (2) whether respondent was proven to be legally capacitated to marry petitioner. Because of our ruling on these two, there is no more necessity to take up the rest. The Court's Ruling The Petition is partly meritorious. First Issue: Proving the Divorce Between Respondent and Editha Samson 284

University of the Cordilleras College of Law First Year C S.Y. 2013 - 2014 Petitioner assails the trial court's recognition of the divorce between respondent and Editha Samson. Citing Adong v. Cheong Seng Gee, petitioner argues that the divorce decree, like any other foreign judgment, may be given recognition in this jurisdiction only upon proof of the existence of (1) the foreign law allowing absolute divorce and (2) the alleged divorce decree itself. She adds that respondent miserably failed to establish these elements. Petitioner adds that, based on the first paragraph of Article 26 of the Family Code, marriages solemnized abroad are governed by the law of the place where they were celebrated (the lex loci celebrationist). In effect, the Code requires the presentation of the foreign law to show the conformity of the marriage in question to the legal requirements of the place where the marriage was performed. At the outset, we lay the following basic legal principles as the take-off points for our discussion. Philippine law does not provide for absolute divorce; hence, our courts cannot grant it. A marriage between two Filipinos cannot be dissolved even by a divorce obtained abroad, because of Articles 15 and 17 of the Civil Code. In mixed marriages involving a Filipino and a foreigner, Article 26 of the Family Code allows the former to contract a subsequent marriage in case the divorce is "validly obtained abroad by the alien spouse capacitating him or her to remarry." A divorce obtained abroad by a couple, who are both aliens, may be recognized in the Philippines, provided it is consistent with their respective national laws. A comparison between marriage and divorce, as far as pleading and proof are concerned, can be made. Van Dorn v. Romillo Jr. decrees that "aliens may obtain divorces abroad, which may be recognized in the Philippines, provided they are valid according to their national law." Therefore, before a foreign divorce decree can be recognized by our courts, the party pleading it must prove the divorce as a fact and demonstrate its conformity to the foreign law allowing it. Presentation solely of the divorce decree is insufficient. Divorce as a Question of Fact Petitioner insists that before a divorce decree can be admitted in evidence, it must first comply with the registration requirements under Articles 11, 13 and 52 of the Family Code. These articles read as follows: "ART. 11. Where a marriage license is required, each of the contracting parties shall file separately a sworn application for such license with the proper local civil registrar which shall specify the following: xxx xxx xxx

"(5) If previously married, how, when and where the previous marriage was dissolved or annulled; xxx xxx xxx

"ART. 13. In case either of the contracting parties has been previously married, the applicant shall be required to furnish, instead of the birth of baptismal certificate required in the last preceding article, the death certificate of the deceased spouse or the judicial decree of annulment or declaration of nullity of his or her previous marriage. x x x. "ART. 52. The judgment of annulment or of absolute nullity of the marriage, the partition and distribution of the properties of the spouses, and the delivery of the children's presumptive legitimes shall be recorded in the appropriate civil registry and registries of property; otherwise, the same shall not affect their persons." Respondent, on the other hand, argues that the Australian divorce decree is a public document a written official act of an Australian family court. Therefore, it requires no further proof of its authenticity and due execution. Respondent is getting ahead of himself. Before a foreign judgment is given presumptive evidentiary value, the document must first be presented and admitted in evidence. A divorce obtained abroad is

285

University of the Cordilleras College of Law First Year C S.Y. 2013 - 2014 proven by the divorce decree itself. Indeed the best evidence of a judgment is the judgment itself. The decree purports to be a written act or record of an act of an officially body or tribunal of a foreign country. Under Sections 24 and 25 of Rule 132, on the other hand, a writing or document may be proven as a public or official record of a foreign country by either (1) an official publication or (2) a copy thereof attested by the officer having legal custody of the document. If the record is not kept in the Philippines, such copy must be (a) accompanied by a certificate issued by the proper diplomatic or consular officer in the Philippine foreign service stationed in the foreign country in which the record is kept and (b) authenticated by the seal of his office. The divorce decree between respondent and Editha Samson appears to be an authentic one issued by an Australian family court. However, appearance is not sufficient; compliance with the aforemetioned rules on evidence must be demonstrated. Fortunately for respondent's cause, when the divorce decree of May 18, 1989 was submitted in evidence, counsel for petitioner objected, not to its admissibility, but only to the fact that it had not been registered in the Local Civil Registry of Cabanatuan City. The trial court ruled that it was admissible, subject to petitioner's qualification.Hence, it was admitted in evidence and accorded weight by the judge. Indeed, petitioner's failure to object properly rendered the divorce decree admissible as a written act of the Family Court of Sydney, Australia. Compliance with the quoted articles (11, 13 and 52) of the Family Code is not necessary; respondent was no longer bound by Philippine personal laws after he acquired Australian citizenship in 1992. Naturalization is the legal act of adopting an alien and clothing him with the political and civil rights belonging to a citizen. Naturalized citizens, freed from the protective cloak of their former states, don the attires of their adoptive countries. By becoming an Australian, respondent severed his allegiance to the Philippines and the vinculum juris that had tied him to Philippine personal laws. Burden of Proving Australian Law Respondent contends that the burden to prove Australian divorce law falls upon petitioner, because she is the party challenging the validity of a foreign judgment. He contends that petitioner was satisfied with the original of the divorce decree and was cognizant of the marital laws of Australia, because she had lived and worked in that country for quite a long time. Besides, the Australian divorce law is allegedly known by Philippine courts: thus, judges may take judicial notice of foreign laws in the exercise of sound discretion. We are not persuaded. The burden of proof lies with "the party who alleges the existence of a fact or thing necessary in the prosecution or defense of an action." In civil cases, plaintiffs have the burden of proving the material allegations of the complaint when those are denied by the answer; and defendants have the burden of proving the material allegations in their answer when they introduce new matters. Since the divorce was a defense raised by respondent, the burden of proving the pertinent Australian law validating it falls squarely upon him. It is well-settled in our jurisdiction that our courts cannot take judicial notice of foreign laws. Like any other facts, they must be alleged and proved. Australian marital laws are not among those matters that judges are supposed to know by reason of their judicial function. The power of judicial notice must be exercised with caution, and every reasonable doubt upon the subject should be resolved in the negative. Second Issue: Respondent's Legal Capacity to Remarry Petitioner contends that, in view of the insufficient proof of the divorce, respondent was legally incapacitated to marry her in 1994. Hence, she concludes that their marriage was void ab initio. Respondent replies that the Australian divorce decree, which was validly admitted in evidence, adequately established his legal capacity to marry under Australian law. 286

University of the Cordilleras College of Law First Year C S.Y. 2013 - 2014 Respondent's contention is untenable. In its strict legal sense, divorce means the legal dissolution of a lawful union for a cause arising after marriage. But divorces are of different types. The two basic ones are (1) absolute divorce or a vinculo matrimonii and (2) limited divorce or a mensa et thoro. The first kind terminates the marriage, while the second suspends it and leaves the bond in full force. There is no showing in the case at bar which type of divorce was procured by respondent. Respondent presented a decree nisi or an interlocutory decree a conditional or provisional judgment of divorce. It is in effect the same as a separation from bed and board, although an absolute divorce may follow after the lapse of the prescribed period during which no reconciliation is effected. Even after the divorce becomes absolute, the court may under some foreign statutes and practices, still restrict remarriage. Under some other jurisdictions, remarriage may be limited by statute; thus, the guilty party in a divorce which was granted on the ground of adultery may be prohibited from remarrying again. The court may allow a remarriage only after proof of good behavior. On its face, the herein Australian divorce decree contains a restriction that reads: "1. A party to a marriage who marries again before this decree becomes absolute (unless the other party has died) commits the offence of bigamy." This quotation bolsters our contention that the divorce obtained by respondent may have been restricted. It did not absolutely establish his legal capacity to remarry according to his national law. Hence, we find no basis for the ruling of the trial court, which erroneously assumed that the Australian divorce ipso facto restored respondent's capacity to remarry despite the paucity of evidence on this matter. We also reject the claim of respondent that the divorce decree raises a disputable presumption or presumptive evidence as to his civil status based on Section 48, Rule 39 of the Rules of Court, for the simple reason that no proof has been presented on the legal effects of the divorce decree obtained under Australian laws. Significance of the Certificate of Legal Capacity Petitioner argues that the certificate of legal capacity required by Article 21 of the Family Code was not submitted together with the application for a marriage license. According to her, its absence is proof that respondent did not have legal capacity to remarry. We clarify. To repeat, the legal capacity to contract marriage is determined by the national law of the party concerned. The certificate mentioned in Article 21 of the Family Code would have been sufficient to establish the legal capacity of respondent, had he duly presented it in court. A duly authenticated and admitted certificate is prima facie evidence of legal capacity to marry on the part of the alien applicant for a marriage license. As it is, however, there is absolutely no evidence that proves respondent's legal capacity to marry petitioner. A review of the records before this Court shows that only the following exhibits were presented before the lower court: (1) for petitioner: (a) Exhibit "A" Complaint; (b) Exhibit "B" Certificate of Marriage Between Rederick A. Recto (Filipino-Australian) and Grace J. Garcia (Filipino) on January 12, 1994 in Cabanatuan City, Nueva Ecija;(c) Exhibit "C" Certificate of Marriage Between Rederick A. Recio (Filipino) and Editha D. Samson (Australian) on March 1, 1987 in Malabon, Metro Manila; (d) Exhibit "D" Office of the City Registrar of Cabanatuan City Certification that no information of annulment between Rederick A. Recto and Editha D. Samson was in its records; and (e) Exhibit "E" Certificate of Australian Citizenship of Rederick A. Recto; (2) for respondent: (Exhibit "1" Amended Answer; (b) Exhibit "S" Family Law Act 1975 Decree Nisi of Dissolution of Marriage in the Family Court of Australia; (c) Exhibit "3" Certificate of Australian Citizenship of Rederick A. Recto; (d) Exhibit "4" Decree Nisi of Dissolution of Marriage in the Family Court of Australia Certificate; and Exhibit "5" Statutory Declaration of the Legal Separation Between Rederick A. Recto and Grace J. Garcia Recio since October 22, 1995. Based on the above records, we cannot conclude that respondent, who was then a naturalized Australian citizen, was legally capacitated to marry petitioner on January 12, 1994. We agree with petitioner's contention that the court a quo erred in finding that the divorce decree ipso facto clothed respondent with the legal capacity to remarry without requiring him to adduce sufficient evidence to show the Australian 287

University of the Cordilleras College of Law First Year C S.Y. 2013 - 2014 personal law governing his status; or at the very least, to prove his legal capacity to contract the second marriage. Neither can we grant petitioner's prayer to declare her marriage to respondent null and void on the ground of bigamy. After all, it may turn out that under Australian law, he was really capacitated to marry petitioner as a direct result of the divorce decree. Hence, we believe that the most judicious course is to remand this case to the trial court to receive evidence, if any, which show petitioner's legal capacity to marry petitioner. Failing in that, then the court a quo may declare a nullity of the parties' marriage on the ground of bigamy, there being already in evidence two existing marriage certificates, which were both obtained in the Philippines, one in Malabon, Metro Manila dated March 1, 1987 and the other, in Cabanatuan City dated January 12, 1994. WHEREFORE, in the interest of orderly procedure and substantial justice, we REMAND the case to the court a quo for the purpose of receiving evidence which conclusively show respondent's legal capacity to marry petitioner; and failing in that, of declaring the parties' marriage void on the ground of bigamy, as above discussed. No costs. SO ORDERED. Melo, Puno, Vitug, and Sandoval-Gutierrez, JJ., concur.

288

University of the Cordilleras College of Law First Year C S.Y. 2013 - 2014 Case Digest GRACE J. GARCIA, a.k.a. GRACE J. GARCIA-RECIO vs. REDERICK A. RECIO G.R. No. 138322 October 2, 2001 366 SCRA 437 PANGANIBAN, J. Facts: Rederick Recio, a Filipino, was married to Editha Samson an Australian citizen, on March 1, 1987. On May 18, 1989 a decree of divorce dissolving the marriage was issued by the Australian Family Court. On June 26, 1992, respondent became an Australian citizen. Subsequently, respondent entered into marriage with petitioner a Filipina on January 12, 1994. Starting October 22, 1995, petitioner and respondent lived separately without prior judicial dissolution of their marriage. On March 3, 1998, petitioner filed a complaint for Declaration of Nullity of Marriage on the ground of bigamy. Respondent contended that his prior marriage had been validly dissolved by a decree of divorce obtained in Australia thus he is legally capacitated to marry petitioner. The trial court rendered the decision declaring the marriage between petitioner and respondent dissolved and both parties can now remarry. ISSUE: Whether or not the divorce obtained by respondent in Australia ipso facto capacitated him to remarry. HELD: The SC remanded the case to the trial court to receive evidence. Based on the records, the court cannot conclude that respondent who was then a naturalized Australian citizen was legally capacitated to marry petitioner. Neither can the court grant petitioners prayer to declare her marriage null and void on the ground of bigamy. After all it may turn out that under Australian law, he was really capacitated to marry petitioner as result of the divorce decree.

289

University of the Cordilleras College of Law First Year C S.Y. 2013 - 2014 Moreno vs Moreno 246 SCRA 120 A.M. No. MTJ-94-963 July 14, 1995 Full Case MARILOU NAMA MORENO, complainant, vs. JUDGE JOSE C. BERNABE, Metropolitan Trial Court, Branch 72, Pasig, Metro Manila, respondent. KAPUNAN, J.: The responsibility of a Judge is indeed heavy. As the incarnation of law and justice, it is his sworn duty to lead by example, to be the example. But how can he inspire the people to live by the law if he himself fails to do so? Marilou Nama Moreno filed this complaint against Judge Jose C. Bernabe of the Metropolitan Trial Court, Branch 72, Pasig, Metro Manila for grave misconduct and gross ignorance of the law. Complainant alleges that on October 4, 1993, she and Marcelo Moreno were married before respondent Judge Bernabe. She avers that Respondent Judge assured her that the marriage contract will be released ten (10) days after October 4, 1993. Complainant then visited the office of the Respondent Judge on October 15, 1993 only to find out that she could not get the marriage contract because the Office of the Local Civil Registrar failed to issue a marriage license. She claims that Respondent Judge connived with the relatives of Marcelo Moreno to deceive her. In his comment, Respondent denied that he conspired with the relatives of Marcelo Moreno to solemnize the marriage for the purpose of deceiving the complainant. Respondent contends: 1. That the Local Civil Registrar of Pasig has actually prepared the marriage license but it was not released due to the subsequent objection of the father of Marcelo Moreno; 2. That he did not violate the law nor did he have the slightest intention to violate the law when he, in good faith, solemnized the marriage, as he was moved only by a desire to help a begging and pleading complainant who wanted some kind of assurance or security due to her pregnant condition; 3. That in order to pacify complainant, Marcelo Moreno requested him to perform the marriage ceremony, with the express assurance that "the marriage license was definitely forthcoming since the necessary documents were complete;" 4. That the contracting parties were not known to him; and 5. That both parties, particularly the complainant, were fully apprised of the effects of a marriage performed without the required marriage license. In a Resolution dated August 10, 1994, we referred this matter for investigation, report and recommendation to Executive Judge Martin Villarama, Jr., of the Regional Trial Court of Pasig, Metro Manila, Branch 156. In his Memorandum of October 11, 1994, Judge Villarama, Jr. recommended the dismissal of the complaint against Respondent for failure of complainant to appear on any of the scheduled hearings and on the basis of a "Sinumpaang Salaysay" executed on behalf of complainant who has left for Singapore by her elder sister Sherlita N. Bendanillo expressly withdrawing her complaint against Respondent. Judge Villarama, however, also recommended that the Respondent be issued a stern warning "in view of the fact on record that he indeed solemnized a marriage without the requisite marriage license. . . ." On November 7, 1994, we referred the aforementioned Memorandum to the Office of the Court Administrator for evaluation, report and recommendation. 290

University of the Cordilleras College of Law First Year C S.Y. 2013 - 2014 In its Memorandum dated January 17, 1995, the Office of the Court Administrator stated: Careful study of the records reveal that indeed respondent Judge displayed his ignorance of the law when he solemnized the marriage without a marriage license. As a judge, he is presumed to be aware of the existence of Article 3(2) of the Family Code of the Philippines (E.O. 209, as amended by E.O. 227), which provides that one of the formal requisites of a marriage is a valid marriage license. Absence of said requisite will make the marriage void from the beginning (Article 35 , the Family Code of the Philippines). Judges are enjoined to show more than just a cursory acquaintance of the law and other established rules. It recommended that Respondent be held liable for misconduct for solemnizing a marriage without a marriage license and that the appropriate administrative sanctions be imposed against him. We concur with the findings and recommendation of the Office of the Court Administrator. Respondent, by his own admission that he solemnized the marriage between complainant and Marcelo Moreno without the required marriage license, has dismally failed to live up to his commitment to be the "embodiment of competence, integrity and independence" and to his promise to be "faithful to the law." Respondent cannot hide behind his claim of good faith and Christian motives which, at most, would serve only to mitigate his liability but not exonerate him completely. Good intentions could never justify violation of the law. Must we always repeat our reminder in Uy v. Dizon Capulong and several other cases that . . . the judge is the visible representation of law and justice from whom the people draw their will and awareness to obey the law. For the judge to return that regard, the latter must be the first to abide by the law and weave an example for the others to follow. The judge should be studiously careful to avoid even the slightest infraction of the law. To fulfill this mission, the judge should keep abreast of the law, the rulings and doctrines of this Court. If the judge is already aware of them, the latter should not deliberately refrain from applying them; otherwise such omission can never be excused. And have we not frequently stressed that: . . .judges should endeavor to maintain at all times the confidence and high respect accorded to those who wield the gavel of justice. Circular No. 13, dated July 1, 1987, enjoins judges "to conduct themselves strictly in accordance with the mandate of existing laws and the Code of Judicial Conduct that they be exemplars in their communities and the living personification of justice and the Rule of Law. . . . A case in point, a definite precedent and a clear basis in determining the liability of Respondent in the instant case is Cosca, et al. v. Palaypayon, Jr., et a1. where Judge Palaypayon, Jr. was duly fined and sternly warned for, among others, solemnizing marriages without licenses. We declared: . . . the conduct and behavior of everyone connected with an office charged with the dispensation of justice, from the presiding judge to the lowliest clerk, should be circumscribed with the heavy burden of responsibility. His conduct, at all times, must not only be characterized by propriety and decorum but, above all else, must be beyond suspicion. Every employee should be an example of integrity, uprightness and honesty. Integrity in a judicial office is more than a virtue, it is a necessity. It applies, without qualification as to rank or position, from the judge to the least of its personnel, they being standard-bearers of the exacting norms of ethics and morality imposed upon a Court of justice. On the charge regarding illegal marriages the Family Code pertinently provides that the formal requisites of marriage are, inter alia, a valid marriage license except in the cases provided for therein. Complementarily, it declares that the absence of any of the essential or formal requisites shall generally render the marriage void ab initio and that, while an 291

University of the Cordilleras College of Law First Year C S.Y. 2013 - 2014 irregularity in the formal requisites shall not affect the validity of the marriage, the party or parties responsible for the irregularity shall be civilly, criminally and administratively liable. The civil aspect is addressed to the contracting parties and those affected by the illegal marriages, and what we are providing for herein pertains to the administrative liability of respondents, all without prejudice to their criminal responsibility. The Revised Penal Code provides that "[p]riests or ministers of any religious denomination or sect, or civil authorities who shall perform or authorize any illegal marriage ceremony shall be punished in accordance with the provisions of the Marriage Law." This is of course, within the province of the prosecutorial agencies of the Government. Finally, on the alleged withdrawal of the complaint against Respondent, we reiterate our ruling in Imbing v. Tiongson: The fact that complainant has lost interest in prosecuting the administrative case against herein respondent judge will not necessarily warrant a dismissal thereof. Once charges have been filed, the Supreme Court may not be divested of its jurisdiction to investigate and ascertain the truth of the matter alleged in the complaint. The Court has an interest in the conduct of members of the Judiciary and in improving the delivery of justice to the people, and its efforts in that direction may not be derailed by the complainant's desistance from further prosecuting the case he or she initiated. To condition administrative actions upon the will of every complainant, who may, for one reason or another, condone a detestable act, is to strip this Court of its supervisory power to discipline erring members of the Judiciary. Definitely, personal interests are not material or controlling. What is involved here is a matter of public interest considering that respondent is no ordinary citizen but an officer of the court whose personal behavior not only upon the bench and in the performance of judicial duties, but also in his everyday life, should be beyond reproach. WHEREFORE, PREMISES CONSIDERED, Respondent is hereby ordered to pay a fine of P10,000.00 and is STERNLY WARNED that a repetition of the same or similar acts will be dealt with more severely. SO ORDERED. Padilla, Davide, Jr., Bellosillo and Quiason, JJ., concur.

292

University of the Cordilleras College of Law First Year C S.Y. 2013 - 2014 Case Digest MARILOU NAMA MORENO vs. JUDGE JOSE C. BERNABE A.M. No. MTJ-94-963 July 14, 1995 246 SCRA 120 KAPUNAN, J Facts: Marilou Nama Moreno and Marcelo Moreno were married before the respondent Judge Jose Bernabe on October 4, 1993 but did not process the papers for the marriage contract because the Office of the Local Civil Registrar failed to issue a marriage license. The complainant at that time was pregnant and begged to the judge to have her and her husband to be married by him. The complainant then filed a complaint allegedly for deceiving her that the marriage is valid. Respondent denied that he conspired with the relatives of Marcelo Moreno to solemnize the marriage for the purpose of deceiving the complainant and that he did not violate the law nor did he have the slightest intention to violate the law when he, in good faith, solemnized the marriage, as he was moved only by a desire to help a begging and pleading complainant who wanted some kind of assurance or security due to her pregnant condition. Issue: Whether or not that a Judge who held a wedding without issuing a marriage contract should be held liable even if the complaint had expressly withdrawn by the complainant. Held: Even with the withdrawal of the complainant against the respondent the Supreme Court insisted that it should still be dealt with accordingly as the accused was a member of the judiciary and a conduct of a higher level were expected. The judge displayed ignorance of the law which is unacceptable for his position and is therefore fined with 10, 000.00 pesos and is sternly warned that a repetition of a similar act should be punished severely.

293

University of the Cordilleras College of Law First Year C S.Y. 2013 - 2014 People vs Borromeo 133 SCRA 106 G.R. No. 117154 March 25, 1999 Full Case THE PEOPLE OF THE PHILIPPINES, plaintiff-appellee, vs. ERNESTO A. BORROMEO, accused-appellant. PURISIMA, J.: Appeal interposed by accused Ernesto Borromeo from the decision of the Regional Trial Court, Branch 54, Manila, finding him guilty of Illegal Recruitment in Criminal Case No. 93-129374 and of Estafa in Criminal Case Nos. 93-129376 to 93-129384. On November 9, 1993, Assistant Prosecutor Roy A. Cabatuando filed several Informations against the sold accused, alleging: In Criminal Case No. 93-129374 for Illegal Recruitment. That in or about and during the period comprised from February 16, 1993 up to July 28, 1993, inclusive, in the City of Manila, Philippines, the said accused conspiring and confederating with others whose true name, identities and present whereabouts are still unknown and helping one another, representing themselves to have the capacity to contract, enlist and transport Filipino workers for employment abroad, did and there willfully and unlawfully for a fee, recruit and promise employment to JOCELYNE DEVEZA Y GARCIA, DONNA PATRICIA V. ANTONIO, WESLEY PAJARILLGA, EDWIN ORTIZ, LEONARDO BROZO, ROBERTO PEREZ Y BROZO, EDUARDO NANA, LAMBERTO PINGA Y CONCEPCION, BENJAMIN G. FULGENCIO and NESTOR DIZON without first having secured the required license or authority from the Department of Labor and Employment. CONTRARY TO LAW. In Criminal Case No. 93-129375 for Estafa. That on or about March 22, 1993, in the City of Manila, Philippines, the said accused conspiring and confederating with others whose true names, identities and present whereabouts are still unknown and helping one another did then and there wilfully ( sic), unlawfully and feloniously defraud DONNA PATRICIA V. ANTONIO in the following manner, to wit: the said accused by means of false manifestations and fraudulent representations which he, she, they made to said DONNA PATRICIA V. ANTONIO to the effect that they had the power and capacity to recruit and employ DONNA PATRICIA V. ANTONIO as factory worker for Taiwan and could facilitate the processing of the pertinent papers if given the necessary amount to meet the requirements thereof, and by means of other similar deceits, induce and succeeded in inducing said DONNA PATRICIA V. ANTONIO to give and deliver, as in fact she gave and delivered to said accused the amount of P15,000.00 on the strength of said manifestations and representations, said accused well knowing that the same are false and fraudulent and were made solely, to obtain, as in fact they did obtain the amount of P15,000.00 which amount once in their possession, with intent to defraud, wilfully, unlawfully and feloniously misappropriated, misapplied and converted to their own personal use and benefit, to the damage and prejudice of said DONNA PATRICIA V. ANTONIO in the aforesaid amount of P15,000.00, Philippine Currency. CONTRARY TO LAW. In Criminal Case No. 93-129376 for Estafa.

294

University of the Cordilleras College of Law First Year C S.Y. 2013 - 2014 That on or about and during the period comprised from March 1, 1993 to July 28, 1993, inclusive, in the City of Manila, Philippines, the said accused conspiring and confederating with others whose true names, identities and present whereabouts are still unknown and helping one another did then and there wilfully, unlawfully and feloniously defraud EDWIN M. ORTIZ in the following manner, to wit: the said accused by means of false manifestations and fraudulent representations which he, she, they made to the said EDWIN M. ORTIZ to the effect that they had the power and capacity to recruit and employ EDWIN M. ORTIZ as factory worker for Taiwan and could facilitate the processing of the pertinent papers if given the necessary amount to meet the requirements thereof, and by means of other similar deceits, induce and succeeded in inducing said EDWlN M. ORTIZ to give and deliver, as in fact he gave and delivered to said accused the amount of P22,600.00 on the strength of said manifestations and representations, said accused well knowing that the same are false and fraudulent and were made solely, to obtain, as in fact they did obtain the amount of P22,600.00, which amount once in their possession, with intent to defraud, wilfully, unlawfully and feloniously misappropriated, misapplied and converted (sic) to their own personal use and benefit, to the damage and prejudice of said EDWIN M. ORTIZ in the aforesaid amount of P22,600.00, Philippine Currency. CONTRARY TO LAW. In Criminal Case No. 93-129377 for Estafa. That on or about and during the period comprised from February 22, 1993 to March 15, 1993 inclusive, in the City of Manila, Philippines, the said accused conspiring and confederating with others whose true names, identities and present whereabouts are still unknown and helping one another and then and there wilfully, unlawfully and feloniously defraud LEONARD S. BROZO in the following manner, to wit: the said accused by means of false manifestations and fraudulent representations which he, she, they made to LEONARD S. BROZO to the effect that they had the power and capacity to recruit and employ LEONARD S. BROZO as factory worker for Taiwan and could facilitate the processing of the pertinent papers if given the necessary amount to meet the requirements thereof and by means of other similar deceits, induce and succeeded in inducing said LEONARD S. BROZO to give and deliver, as in fact she gave and delivered to said accused the amount of P15,000.00 on the strength of said manifestations and representations, said accused well knowing that the same are false and fraudulent and were made solely, to obtain, as in fact they did obtain the amount of P15,000.00 which amount once in their possession, with intent to defraud, wilfully, unlawfully and feloniously misappropriated, misapplied and converted to their own personal use and benefit, to the damage and prejudice of said LEONARD S. BROZO in the aforesaid amount of P15,000.00, Philippine Currency. CONTRARY TO LAW. In Criminal Case No. 93-129378 for Estafa. That on or about and during the period comprised from February 22, 1993 and March 15, 1993, in the City of Manila, Philippines, the said accused conspiring and confederating with others whose true names, identities and present whereabouts are still unknown and helping one another and then and there wilfully, unlawfully and feloniously defraud ROBERTO BROZO PEREZ in the following manner, to wit: the said accused by means of false manifestations and fraudulent representations which he, she, they made to the said ROBERTO BROZO PEREZ to the effect that they had the power and capacity to recruit and employ ROBERTO BROZO PEREZ as factory worker for Taiwan and could facilitate the processing of the pertinent papers if given the necessary amount to meet the requirements thereof and by means of other similar deceits, induce and succeeded in inducing said ROBERTO BROZO PEREZ to give and deliver, as in fact she gave and delivered to said accused the amount of P15,000.00 on the strength of said manifestations and representations, said accused well knowing that the same are false and fraudulent and were made solely, to obtain, as in fact they did obtain the amount of P15,000.00 which amount once in their possession, with intent to defraud, wilfully, unlawfully and 295

University of the Cordilleras College of Law First Year C S.Y. 2013 - 2014 feloniously misappropriated, misapplied and converted to their own personal use and benefit, to the damage and prejudice of said ROBERTO BROZO PEREZ in the aforesaid amount of P15,000.00, Philippine Currency. CONTRARY TO LAW. In Criminal Case No. 93-129379 for Estafa. That on or about and during the period comprised from March 1, 1993, to July 28, 1993, inclusive, Philippines, the said accused conspiring and confederating with others whose true names, identities and present whereabouts are still unknown and helping one another and then and there wilfully, unlawfully and feloniously defraud LAMBERTO PINGA Y CONCEPCION in the following manner, to wit: the said accused by means of false manifestations and fraudulent representations which he, she, they made to the said LAMBERTO PINGA Y CONCEPCION to the effect that they had the power and capacity to recruit and employ LAMBERTO PINGA Y CONCEPCION as factory worker for Taiwan and could facilitate the processing of the pertinent papers if given the necessary amount to meet the requirements thereof and by means of other similar deceits, induce and succeeded, in inducing said LAMBERTO PINGA CONCEPCION to give and deliver, as in fact he gave and delivered to said accused the amount of P22,600.00 on the strength of said manifestation and representations, said accused knowing that the same are false and fraudulent and were made solely, to obtain, as in fact they did obtain the amount of P22,600.00 which amount once in their possession, with intent to defraud, wilfully, unlawfully and feloniously misappropriated, misapplied and converted to their own personal use and benefit, to the damage and prejudice of the said LAMBERTO PINGA Y CONCEPCION the aforesaid amount of P22,600.00, Philippine Currency. CONTRARY TO LAW. In Criminal Case No. 93-129380 for Estafa. That or about and during the period comprised from February 22, 1993 up to March 15, 1993, inclusive, in the City of Manila, Philippines, the said accused conspiring and confederating with others whose true names, identities and present whereabouts are still unknown and helping one another and then and there wilfully, unlawfully and feloniously defraud NESTOR N. DIZON in the following manner, to wit: the said accused by means of false manifestations and fraudulent representations which he, she, they made to the said NESTOR N. DIZON to the effect that they had the power and capacity to recruit and employ NESTOR N. DIZON as factory worker for Taiwan and could facilitate the processing of the pertinent papers if given the necessary amount to meet the requirements thereof and by means of other similar deceits, induce and succeeded in inducing said NESTOR N. DIZON to give and deliver, as in fact he gave and delivered to said accused the amount of P15,000.00 on the strength of said manifestations and representations, said accused well knowing that the same are false and fraudulent and were made solely, to obtain, as in fact they did obtain the amount of P15,000.00 which amount once in their possession, with intent to defraud, wilfully, unlawfully and feloniously misappropriated, misapplied and converted to their own personal use and benefit, to the damage and prejudice of said NESTOR N. DIZON in the aforesaid amount of P15,000.00, Philippine Currency. CONTRARY TO LAW. In Criminal Case No. 93-129381 for Estafa. That on or about and during the period comprised from February 16, 1993 up to March 25, 1993, inclusive, in the City of Manila, Philippines, the said accused conspiring and confederating with others whose true names, identities and present whereabouts are still unknown and helping one another and then and there wilfully, unlawfully and feloniously defraud WESLEY PAJARILLAGA in the following manner, to wit: the said accused by means of false manifestations and fraudulent representations which he, she, they made to the said WESLEY PAJARILLAGA to the effect that they had the power and capacity to 296

University of the Cordilleras College of Law First Year C S.Y. 2013 - 2014 recruit and employ WESLEY PAJARILLAGA as factory worker for Taiwan and could facilitate the processing of the pertinent papers if given the necessary amount to meet the requirements thereof and by means of other similar deceits, induce and succeeded in inducing said WESLEY PAJARILLAGA to give and deliver, as in fact he gave and delivered to said accused the amount of P15,000.00 on the strength of said manifestations and representations, said accused well knowing that the same are false and fraudulent and were made solely, to obtain, as in fact they did obtain the amount of P15, 000.00 which amount once in their possession, with intent to defraud, wilfully, unlawfully and feloniously misappropriated, misapplied and converted to their own personal use and benefit, to the damage and prejudice of said WESLEY PAJARILLAGA in the aforesaid amount of P15,000.00, Philippine Currency. CONTRARY TO LAW. In Criminal Case No. 93-129382 for Estafa. That on or about and during the period comprised from February 16, 1993 up to March 25, 1993, inclusive, in the City of Manila, Philippines, the said accused conspiring and confederating with others whose true names, identities and present whereabouts are still unknown and helping one another and then and there wilfully, unlawfully and feloniously defraud BENJAMIN G. FULGENCIO in the following manner, to wit: the said accused by means of false manifestations and fraudulent representations which he, she, they made to the said BENJAMIN G. FULGENCIO to the effect that they had the power and capacity to recruit and employ BENJAMIN G. FULGENCIO as factory worker for Taiwan and could facilitate the processing of the pertinent papers if given the necessary amount to meet the requirements thereof and by means of other similar deceits, induce and succeeded in inducing said BENJAMIN G. FULGENCIO to give and deliver, as in fact he gave and delivered to said accused the amount of P15,000.00 on the strength of said manifestations and representations, said accused well knowing that the same are false and fraudulent and were made solely, to obtain, as in fact they did obtain the amount of P15, 000.00 which amount once in their possession, with intent to defraud, wilfully, unlawfully and feloniously misappropriated, misapplied and converted to their own personal use and benefit, to the damage and prejudice of said BENJAMIN G. FULGENCIO in the aforesaid amount of P15,000.00, Philippine Currency. CONTRARY TO THE LAW In Criminal Case No. 93-129383 for Estafa. That on or about and during the period comprised from February 22, 1993 up to March 25, 1993, inclusive, in the City of Manila, Philippines, the said accused conspiring and confederating with others whose true names, identities and present whereabouts are still unknown and helping one another and then and there wilfully, unlawfully and feloniously defraud EDUARDO NANA in the following manner, to wit: the said accused by means of false manifestations and fraudulent representations which he, she, they made to the said EDUARDO NANA to the effect that they had the power and capacity to recruit and employ EDUARDO NANA as factory worker for Taiwan and could facilitate the processing of the pertinent papers if given the necessary amount to meet the requirements thereof and by means of other similar deceits, induce and succeeded in inducing said EDUARDO NANA to give and deliver, as in fact he gave and delivered to said accused the amount of P15,000.00 on the strength of said manifestations and representations, said accused well knowing that the same are false and fraudulent and were made solely, to obtain, as in fact they did obtain the amount of P15, 000.00 which amount once in their possession, with intent to defraud, wilfully, unlawfully and feloniously misappropriated, misapplied and converted to their own personal use and benefit, to the damage and prejudice of said EDUARDO NANA in the aforesaid amount of P15,000.00, Philippine Currency. CONTRARY TO THE LAW In Criminal Case No. 93-129384 for Estafa. 297

University of the Cordilleras College of Law First Year C S.Y. 2013 - 2014 That on or about March 19, 1993, in the City of Manila Philippines, the said accused conspiring and confederating with others whose true names, identities and present whereabouts are still unknown and helping one another and then and there wilfully, unlawfully and feloniously defraud JOCELYNE DEVEZA Y GARCIA in the following manner, to wit: the said accused by means of false manifestations and fraudulent representations which he, she, they made to the said JOCELYNE DEVEZ Y GARCIA to the effect that they had the power and capacity to recruit and employ BENJAMIN G. FULGENCIO as factory worker for Taiwan and could facilitate the processing of the pertinent papers if given the necessary amount to meet the requirements thereof and by means of other similar deceits, induce and succeeded in inducing said JOCELYNE DEVEZA Y GARCIA to give and deliver, as in fact he gave and delivered to said accused the amount of P15,000.00 on the strength of said manifestations and representations, said accused well knowing that the same are false and fraudulent and were made solely, to obtain, as in fact they did obtain the amount of P15, 000.00 which amount once in their possession, with intent to defraud, wilfully, unlawfully and feloniously misappropriated, misapplied and converted to their own personal use and benefit, to the damage and prejudice of said JOCELYNE DEVEZA Y GARCIA in the aforesaid amount of P15,000.00, Philippine Currency. CONTRARY TO THE LAW On January 17, 1994, with the accused entering negative pleas to all the above accusations, upon arraignment with the assistance of counsel, trial ensued with the prosecution presenting Nestor Dizon, Edwin Ortiz, Lamberto Pinga, Benjamin Fulgencio, Jocelyn Deveza, Leonardo Brozo, Wesley Pajarillaga, and Roberto Perez as its witnesses. For the defense, the accused took the witness stand as the lone witness on his behalf. Testified on by the above-named witnesses and as summarized by the Solicitor General in the Appellee's Brief, the version of the People runs as follows: I Complainant Nestor Dizon, . . ., is a "kumpadre" of William Ramos, brother-in-law of appellant Ernesto Borromeo (p. 2, TSN, February 7, 1994). In February 1993, William (or Willy) Ramos . . . introduced Dizon to appellant and his wife Elizabeth Ramos at Willy's house (pp. 2&4, 15&16, TSN, Feb. 7, 1994). Willy Ramos informed Dizon that appellant and his wife were in need of people to work as factory worker in Taiwan (p. 2, TSN, Ibid). Dizon then verified from Elizabeth Borromeo whether they really needed workers for Taiwan and she confirmed such information (p. 2, TSN, ibid). Appellant also told Dizon that he can go to Taiwan but he has to wait for a while (p. 3, TSN, Feb. 7, 1994). Dizon immediately told appellant and his wife that he wanted to apply for the job as worker in a textile factory in Taiwan. The Borromeo couple told Dizon that as part of his job application, he should give the amount of P40,000.00 (p. 3, TSN, ibid.). They told Dizon that he can go to Taiwan sometime in April (1993) if he can give the money. Consequently, Dizon accepted the condition requiring payment of the total amount of P15,000.00 (pp. 3-4, TSN, Feb. 2, 1994). On February 22, 1993, at the house of Willy Ramos, at 1727 M. Natividad St. Dizon gave to Elizabeth Borromeo, in the presence of appellant, P3,000.00 for his medical expenses (pp. 3-5, TSN, Feb. 2, 1994; Exh. "A") and P12,000.00 (Exh. "B") as downpayment for the P40,000.00, inclusive of the medical expenses (pp. 4-5, TSN, ibid.). Afterwards, Elizabeth issued two (2) receipts for the said amounts (Exhs. "A" and "B" ). Appellant and his wife Elizabeth assured that he will be leaving sometime in April 1993. Dizon asked the spouses why they failed to send him abroad and they explained that they were still arranging the processing of his papers including passport, application papers, biodata, birth certificate, marriage contract and other documents (pp. 5-6, TSN, Feb. 7, 1994). . . . xxx xxx xxx II 298

University of the Cordilleras College of Law First Year C S.Y. 2013 - 2014 . . . Edwin Ortiz is a resident of 1718 M. Natividad St., Sta. Cruz, Manila (p. 15, TSN, Feb. 7, 1994). In February 1993, he was introduced to appellant by William (Willy) Ramos at the latter's house at 1727 M. Natividad St., Sta. Cruz, Manila. Before such meeting, Willy Ramos had previously informed Ortiz that appellant and the latter's wife Elizabeth, sister of William Ramos, are recruiting workers for Taiwan (pp. 15-16, TSN, Feb. 7, 1994). Thereafter, Ortiz talked to the spouses and they confirmed that they can send workers to Taiwan (p.18, TSN, Feb. 7, 1994). Ortiz then told them that he was interested in applying for a job in Taiwan (pp. 17-18, TSN, Feb. 7, 1994). There and then, Ortiz applied with the Borromeo spouses . . . Eventually, the Borromeo spouses told Ortiz and his fellow applicants that they each had to undergo medical examination and pay P15,000.00 and the P3,000.00 thereof will answer for the medical examination fees while the P12,000.00 will be for the processing of their papers (p. 19, TSN, Feb. 7, 1994). Consequently, Ortiz paid to appellant the P3,000.00 for the medical examination for which are receipt was issued (Exh. "C") On March 3, 1993, he paid the amount of P8,000.00 (or P4,000.00 each) for himself and fellow job-applicant Lamberto Pinga to appellant who handed the money to his wife who then issued a receipt therefor (pp. 20-22, TSN, Feb. 7, 1994; Exhs. "E", E-1"). Later, Ortiz paid another P10,000.00 (P5,000 each) for himself and his co-applicant Lamberto Pinga who handed the money to his wife and the latter likewise issued in the name of Ortiz a receipt for the P10,000.00 (Exhs. "D", "D-l"; pp. 20-23, TSN, Feb. 7, 1994). Subsequently, appellant promised to Ortiz and Lamberto Pinga that after the processing of their papers, they can go to Taiwan within the month of May 1993 . . . xxx xxx xxx III Eduardo Nana, one of the complainants . . . In the first week of February 1993, he learned from his friend Willy Ramos . . . that his sister Elizabeth Ramos Borromeo and her husband Ernesto Borromeo (appellant) needed workers to be sent abroad (pp. 19-21, TSN, Feb. 9, 1994). Nana told Willy Ramos that he was interested to apply (p. 20, TSN, Feb. 9, 1994). . . . xxx xxx xxx . . . Appellant and his spouse confirmed that they can help Nana go abroad but he has to pay P3,000.00 and P12,000.00 for processing of documents (p. 21 TSN, Feb. 9, 1994). Nana agreed to pay the price and applied for job as a factory worker which according to the couple, fetched a salary of around $800.00 a moth (pp. 22-23, TSN, Ibid). On February 22, 1993, Nana paid P3,000.00 to Elizabeth Ramos-Borromeo who then counted the money (pp. 23-24, TSN, Feb. 9, 1994). Afterwards, she issued a receipt for the amount containing her signatures (pp. 24-25, TSN, Feb. 9, 1994; Exh. "H"). Such payment was made at the house of willy Ramos . . . Appellant assured Nana that if he completes payment of the additional P12,000.00, Nana could go to Taiwan (p. 25, TSN, ibid.) Accordingly, Nana made a second payment in the amount of 12,000.00 to the Borromeo couple on March 15, 1993 (Exh. "I"; pp. 25-26, TSN, Feb. 9, 1994). Subsequently, Elizabeth Ramos-Borromeo issued a receipt for said amount where she signed both her and appellant's name (pp. 23-30, TSN, Feb. 9, 1994). However, despite such payment, Nana was unable to go abroad as appellant and his wife had promised him (p. 26, TSN, Feb. 9, 1994). . . . xxx xxx xxx IV On March 1, 1993, Lamberto Pinga met appellant . . . when he applied with him for a job in Taiwan as factory worker (pp. 2-4, TSN, Feb. 14, 1994). Appellant told Pinga that he had to pay the processing fee of the papers in the amount of P15,000.00 before could go to Taiwan (pp. 4-7, TSN, Feb. 14, 1994). Pinga made a partial payment of P3,000.00 on 299

University of the Cordilleras College of Law First Year C S.Y. 2013 - 2014 the same day for which a receipt in yellow pad dated March 1, 1993 was issued (pp. 4-5, TSN, Feb. 14, 1994, Exh. "G"). . . . xxx xxx xxx . . . He made another partial payment on March 17, 1993 jointly with Ortiz in the amount of P10,000.00 for himself and Edwin Ortiz (or P5,000.00 for each of them ) for which receipt was issued (pp. 9-10, TSN, Feb. 14, 1994; Exh. "D", "D-1). At the time that he made the payments, appellant's wife and brother-in-law were also present (pp. 9-12, TSN, Feb. 14, 1994). Later, Pinga made another payment in the amount of P1,600.00 to appellant's brother-in-law William Ramos but no receipt therefor was issued (pp. 11-12, TSN, Feb. 14, 1994). Notwithstanding his payments, nothing happened to Pinga's application. . . . V Benjamin Fulgencio, one of the complainants in the subject Criminal Case is a resident of 1918 M. Natividad St., Sta. Cruz, Manila . . . In 1993, he (sic) learned from willy Ramos by phone that appellant, Ernesto Borromeo, Willy 's brother-in-law whom he had previously met in 1989, and his wife Elizabeth Ramos-Borromeo were recruiting workers for Taiwan (pp. 2-4, TSN, ibid.). . . . They eventually met at the house of Willy Ramos and Fulgencio verified from the Borromeo spouses whether they could really send workers in Taiwan.. . . . They told him that as part of his application, he had to pay a total of P15,000.00, P3,000,00 and P12,000.00 of which will answer for the medical expenses and processing of papers, respectively (pp. .4-5, TSN, Feb. 21, 1994). The couple told him that the total amount to be paid is P40,000.00 and that the P15,000,00 is only partial payment while the P25,000.00 balance thereof will be paid through salary deductions when he is already in Taiwan (pp. 4-5, TSN, ibid.). Fulgencio acceded to the aforesaid condition and paid to appellant the P3,000.00 for which a receipt was issued to him but which receipt has been misplaced (p. 5, TSN, Feb. 21, 1994). Appellant Fulgencio was given the receipt for the P3,000.00 he paid to appellant to whom he gave the money but it was signed by Mrs. Borromeo (pp. 4-5, TSN, Feb. 21, 1994). Fulgencio likewise paid the P12,00.00 for which he was issued a receipt (pp. 5-6, TSN, Feb. 21, 1994; Exh. "J", "J-1"). After such payment, appellant told Fulgencio that his salary will be $800.00 monthly (pp. 6-7, TSN, Feb. 21, 1994). Thereafter, the couple told Fulgencio that he will be leaving far abroad sometime in April 1993 (p. 7, TSN, ibid.). However, when April 1993 came, Fulgencio was unable to leave for abroad and nothing resulted from his application (p. 7, TSN, Feb. 21, 1994). . . . VI Jocelyn Devesa is a resident of 1714 M. Natividad St., Sta. Cruz, Manila . . . In 1993, she (sic) learned from her friend and neighbor William (Willy) Ramos that his brother-in-law Ernesto Borromeo (appellant) and his wife Beth Borromeo, Willy's sister, were recruiting workers for Taiwan (p. 2, TSN, Feb. 23, 1994), Jocelyn, asked Willy whether they are licensed recruiters and he answered that they were (p. 6, TSN, Feb. 23, 1994). He told her they are connected with the EER employment agency located at EDSA, Quezon City (p. 6, TSN, ibid.). . . . . . . They talked about the expenses they will incur and the date of their departure in connection with the job in Taiwan (p. 3, TSN, Feb. 23, 1994). . . . She told the applicants, including Jocelyn, that the total amount is P45,000.00 and that the P15,000.00 should be made as partial payment while the P30,000.00 will be paid through salary deduction when they are already in Taiwan (p. 3, TSN, Feb. 23, 1994). On March 19, 1993, Jocelyn Devesa paid the required P15,000.00 to appellant. He handed the amount to his wife Mrs. Beth Borromeo who then issued a receipt where the signature of Beth Borromeo appears over the printed name Ernesto Borromeo (Exh: "K", "K-1"; p.4, TSN, Feb. 23, 1994). The receipt was issued under her (Jocelyn) nickname "Joan" (Exh. "K" pp. 3-4, TSN, Feb. 23, 1994). . . . 300

University of the Cordilleras College of Law First Year C S.Y. 2013 - 2014 VII Leonardo Brozo, one of the complainants, similarly learned from his acquaintance William (Willy) Ramos that his brother-in-law Ernesto Borromeo and the latter's wife Beth Borromeo were recruiting persons to work abroad. . . . He (sic) then applied with them for a job as a laborer (pp. 7,13, TSN, Feb., 28, 1994). They told him to submit an NBI clearance and many others and to pay P15,000.00 out of the total P30,000,00, as applicant fee (p. 9, TSN, Feb. 28, 1994). On March 15, 1993, Brozo paid P12,000.00 for processing of his papers to appellant, who after counting the same turned it over to his wife Elizabeth Borromeo who then issued a receipt therefor ( Exh. "L-l"; pp. 12-13, TSN, Feb. 28, 1994). He also paid on March 26, 1993 the amount of P3,000.00 to appellant who after counting the money turned it over to Elizabeth Borromeo who then issued a receipt therefor (Exh. "L"; pp. 11-12, TSN, Feb. 28, 1994). The couple told Brozo that they will be the one to look for a job abroad for Brozo who will receive a salary of P30,000.00 (pp. 13-14, TSN, Feb. 28, 1994). . . . After Brozo had paid the P15,000.00, appellant kept on promising Brozo that he can go abroad but appellant failed in his promise and Brozo was unable to go abroad. (p. 14, TSN, Feb. 28, 1994). xxx xxx xxx VIII . . . On February 16, 1993, (Wesley) Pajarillaga paid P3,000.00 for medical expenses (p. 9, TSN, March 2 1994). He subsequently paid P7,000.00 on March 18, 1993 and P5,000.00 on March 25, 1993 to appellant . . . (pp. 9-10, TSN, March 2, 1994). Receipt for the P15,000.00 payment was issued but the same was lost by Pajarillaga (pp. 8-10, TSN, March 2, 1994). . . . IX In the first week of February 1993, Roberto Perez was introduced by Willy Ramos to Ernesto Borromeo, . . . Perez asked appellant if he was recruiting workers for Taiwan and the latter told Perez, he can help him go there (pp. 2-3, TSN, March 9, 1994). Perez told him that he was interested to go abroad. if he pays P3,000.00 and P12,000.00 (pp. 3-4, TSN, March 9, 1994). Perez applied for a job as a factory worker with a salary of $800.00 and paid the required payments (pp. 4-5, TSN, March 9, 1994). He paid the P3,000.00 and P12,000.00 to appellant and his wife for which a receipt dated March 15, 1993 was issued (Exhs. "M", "M-1"; pp. 4 and 6, TSN, March 9, 1994). On the aforesaid receipt, Elizabeth Borromeo printed her and appellant's respective names but it was only her signature which appeared thereon (p. 6, TSN, March 9, 1994). Since Perez was unable to leave for Taiwan despite his payments and despite appellant's promises, he checked from the POEA whether appellant and his wife were duly licensed recruiters (p. 7, TSN, March 9, 1994). They found out that they were not listed as licensed recruiters (p.7, TSN, ibid.). . . . For the defense, appellant placed reliance on mere denial, testifying thus: . . . that Elizabeth Ramos Borromeo is his common-law wife and William Ramos is his brother-in-law. He denied being at the house of his brother-in-law at 1727 M. Natividad St., Sta. Cruz, Manila when the complainants were introduced to his common-law wife, neither was he present when the complainant allegedly gave money to Elizabeth Ramos Borromeo. He does not even know who prepared the receipts evidencing the amounts given to Elizabeth Ramos Borromeo. In other words, he was not a privy to all the transactions that allegedly transpired between the complainants and his common-law wife in the house of his brother-in-law. He further testified that if at all the name ER Borromeo appears in any or all the receipts prepared by Elizabeth Ramos Borromeo, it is not his name because his complete name is Ernnesto Abeso Borromeo. His initial is E.A. Borromeo and not E. R. Borromeo and the latter initial is what appears in all the receipts presented by the complainants during the direct and cross-examinations. He came to know the complainants only when they went to their house at 27 Maamo St., Sikatuna Village, Diliman, Quezon City accompanied by his brother-in-law, William Ramos. All the time he stayed at home taking care of their two children and his fighting cocks.

301

University of the Cordilleras College of Law First Year C S.Y. 2013 - 2014 . . . Queried on how he was arrested, he answered that he was only invited by the NBI because of the problem of his wife. Several warrants of arrest were issued against his wife but when she could not be located because she was already in hiding, he was detained. Appellant declared that he cannot refund the money and the money was not given to me. "(TSN, pp. 2-20, May 4, 1994). After trial, the lower court found the evidence for the prosecution enough to convict and handed down the judgment appealed from, convicting and sentencing, the accused as follows: WHEREFORE, judgment is hereby rendered convicting the accused of the crime of illegal recruitment in Crim. Cases No. 94-129374 and of estafa in Crim Case No. 93129376 to 93-129384 and sentences him to life imprisonment and to pay a fine of P100,000.00 in the illegal recruitment case as provided for by Art. 38 (b) and Art. 39 of the Labor Code as amended by P.D. 2018; and in other cases sentences him to an indeterminate term of imprisonment of Two (2) years and Four (4) months of prision correctional as the minimum and Eight (8) years of Prision mayor as the maximum except in cases 93-129376 and 93-129379 where the accused is sentenced in each case to an indeterminate term of imprisonment from Ten (10) years and One (1) day of prision mayor as minimum, and Seventeen (17) years, Four (4) months and One (1) day of reclusion temporal as maximum. The accused is also adjudged liable to the complainants in Crim. Cases No. 93-129376 to 93-129384 for estafa; to pay as civil liability the sum of P15,000.00 for each complainant except in Crim, Cases No, 93129376 and 93-129379 where the accused is adjudged to pay P22,600.00 in each of the two (2) cases to the respective complainants thereof. Accused to pay the costs. The accused is acquitted in Case No. 93-129375 for lack of evidence. SO ORDERED. To buttress his plea for acquittal, appellant theorized in his assignment of errors, that: I THE TRIAL COURT GRAVELY ERRED IN GIVING FULL WEIGHT AND CREDENCE TO THE TESTIMONIES OF THE PROSECUTION WITNESSES AND IN DISREGARDING THE THEORY OF THE DEFENSE. II THE TRIAL COURT GRAVELY ERRED IN FINDING THE ACCUSEDAPPELLANT GUILTY BEYOND REASONABLE DOUBT OF THE CRIME CHARGED DESPITE INSUFFICIENCY OF EVIDENCE. On March 20, 1997, appellant presented a motion to withdraw the appeal. Acting thereupon, the Court directed his counsel to confer with appellant and ascertain the voluntariness of subject motion, and whether appellant was aware of its legal consequences. After confering with the accused on October 29, 1997, counsel prayed that the Motion to Withdraw the Appeal be disregarded. The pivotal issue at bar being factual and evidentiary, the credibility of the witnesses assumes extreme importance. Well-entrenched to the point of being elementary is the doctrine that on the issue of credibility of witnesses, findings arrived at by the trial court are accorded great weight and respect on appeal because of the singular opportunity of the lower court to observe the demeanor of the witnesses on the witness stand. . . . Factual findings of fact of the trial court pertaining to the credibility of witnesses command great weight and respect since it had the opportunity to observe their demeanor while they testified in court. . . . (People v. Tanedo, 266 SCRA 34) Well-entrenched is the rule that findings of trial courts on credibility of witnesses deserve a high degree of respect. (People v. Atad, 266 SCRA 262). Findings of the trial court which are factual in nature deserves to be respected and affirmed by the appellate courts. . . . (People v. Magallano, 266 SCRA 305). 302

University of the Cordilleras College of Law First Year C S.Y. 2013 - 2014 . . . Unless the trial judge plainly overlooked certain facts of substance and value which, if considered, might affect the result of the case, his assessment on credibility must be respected. (People v. Ramirez, 266 SCRA 335). . . . The age old-rule is that the task of assigning values to the testimonies of witnesses in the stand and weighing their credibility is best left to the trial court which forms its firsthand impressions as a witness testifies before it. . . . (People v. Sarabia, 266 SCRA 471). Factual findings and conclusions of the trial court are entitled to great weight and respect and should not be disturbed on appeal. . . . (People v. Cahindo, 266 SCRA 554). . . . Appellate courts accord the highest respect to the assessment and testimonies eyewitnesses by the trial court because of its unequaled opportunity to observe on the stand their demeanor and manner of testifying and to detect whether they are telling the truth or not. . . . (People v. Navales, 266 SCRA 569). . . . It is doctrinally settled that the evaluation of the testimony of the witnesses by the trial court is received on appeal with the highest respect, because it had the direct opportunity to observe the witnesses on the stand and detect if they were telling the truth. (People v. Dinglasan, 267 SCRA 26) . . . Absent any showing of a fact or any circumstance which the trial court failed to appreciate and which would have changed the result if it were considered, the factual findings laid down by the trial court remain binding upon us. (People v. Valles, 267 SCRA 103) After a thorough review and examination of the evidence on record, the Court finds no basis for disbelieving what the trial court of origin found and arrived at. Appellant berates the trustworthiness of the complaining witnesses who narrated how appellant conspired with his wife, Elizabeth, and brother-in-law, Willy Ramos, in the commission of the crimes charged. This deserves scant consideration; however, because the modus operandi of the spouses, Ernesto Borromeo and Willy Ramos, brother-in-law of Ernesto, has been established beyond any iota of doubt, to wit: Willy Ramos recruited applicants as factory workers in Taiwan, and introduced his recruits to the spouses, Ernesto Borromeo and Elizabeth Borromeo, who assured all and sundry of their ability and preparedness to send workers abroad, upon payment of P15,000.00 for medical expenses and processing fees. Cajoled by such promise and misrepresentation, the poor applicants paid the amounts demanded of them, only to discover later that the malefactors were not licensed and authorized to recruit workers for overseas employment. This fact was indubitably attested to by the Certification issued by the Philippine Overseas Employment Agency (Exhibit "F"). Ernesto Borromeo and Willy Ramos did not return the money of the complainants, who were unable to work abroad as promised by the appellant Ernesto Borromeo, his wife, Elizabeth Borromeo and brother-in-law, Willy Ramos. As regards the theory of appellant that there was no documentary evidence evidencing that he had received, misappropriated, misapplied and converted the money of complainants for his personal use and benefit, the straight forward narrations by the complainants negate such submission of appellants. Complainants testified as follows: Q. Now, did you actually pay the P3,000.00 for medical examination? A. Yes, sir. Q. To whom did, you pay? A. To Mr. and Mrs. Borromeo, sir. Q. To whom did you actually handed (sic) the money? A. I gave it to Mr. Borromeo, sir, and then he gave it to his wife. (p. 1920, TSN, February 7, 1994) 303

University of the Cordilleras College of Law First Year C S.Y. 2013 - 2014 Q. When you testified, you said that the amount of P3,000.00 was given to the accused himself and according to you the accused counted the money and then it was given to the wife? A. Yes, sir." (Cross-examination of Edwin Ortiz, p. 4, TSN, February 9, 1994). Q. That was the first time you paid the amount of P3,000.00? A. Yes, sir. Q. To whom did you give that money? A. To both the spouses, sir. (p., 23-24, TSN, Direct Examination of Edwin Nana, February 9, 1994). Q. Did you actually give this P3,000.00 for medical expenses and the P12,000.00 for processing of the papers? A. Yes, sir. Q. To whom did you give that amount of P3,000? A. To Mr. Borromeo. (p. 5, TSN, February 21, 1994, Direct Examination of B. Fulgencio) In the case of Lim v. Court of Appeals, 271 SCRA 12, "convert" and "misappropriate" were held to mean: an act of using or disposing of another's property as if it were one's own or devoting it to a purpose or use different from that agreed upon. The following elements of estafa have been established: 1) the accused defrauded another by a] abuse of confidence or b] by means of deceit and 2) the offended party or third party suffered damage or prejudice capable of pecuniary estimation. (Tan v. Court of Appeals, 283 SCRA 30, citing People v. Bautista, 241 SCRA 216, People v. Reyes, 282 SCRA 105) In the cases under scrutiny, estafa was consummated when the appellant together with wife Elizabeth and brother-in-law Willy Ramos falsely pretended to be capable of sending workers abroad, and as convinced by the appellant and his co-conspirators, the said applicants delivered their placement fee to appellant and his wife. With respect to the charge of illegal recruitment, no less than the Philippine Overseas Employment Administration confirmed that the spouses Borromeo, and Willy Ramos did not have any authority or license to recruit overseas workers. Neither was there a recruitment agency such as EER Employment Agency. In People v. Recio, 282 SCRA 274, it was held that: "Illegal recruitment is committed when two requisites concur, to wit: 1) that the offender has no valid license or authority required by law to enable one to lawfully engage in recruitment and placement of workers; and 2) that the offender undertakes either any activity within the meaning of "recruitment and placement" under Article 13(b) or any prohibited practices enumerated under Article 34 of the Labor Code as amended." And in the case of People v. Senoron, 267 SCRA 278, it was held that: "It is the lack of necessary license or authority that renders the recruitment activity unlawful or criminal" So, also, in People v. Tan Tiong Meng, 271 SCRA 125, this Court held that: . . . Accused- appellant's acts of accepting placement fees from job applicants and representing to them that he could get them jobs in Taiwan constitute recruitment and placement. . . . . There is illegal recruitment when one purports to have the ability to send a worker abroad though without authority or license to do so. (People v. Villas, 277 SCRA 391) Accused-appellant is likewise guilty of illegal recruitment in a large scale. His offense of illegal recruitment victimized three (3) or more persons, individually or as a group. 304

University of the Cordilleras College of Law First Year C S.Y. 2013 - 2014 In the case of People v. Manozca, G.R. No. 109779, March 13, 1997, the Court said: Prosecution also established that the accused by using the fictitious name of Nesty Santiago and Manolito Santiago and falsely pretending to possess the power and capacity to obtain and provide work for complainants in Singapore, obtained from the complainants various sums of money knowing fully well that he did not have such power or capacity in violation of Article 315, paragraph 2 (a) of the Revised Penal Code. In Criminal Case No. 90-13964 the amount defrauded from Arnulfo Caampued by accused is P14,500.00 while in Criminal Case No. 90-13963 the amount defrauded from Norlito Hular is P12,636.00. Anent the defense of denial invoked by the accused, the trial court erred not in rejecting the same. In the case ofPeople v. Magallano, 266 SCRA 305, the Court rationalized: Courts have generally viewed with disfavor the defense of denial on account of its aridity and the facility with which an accused could concoct the same to suit his defense. All things studiedly viewed in proper perspective, the mind of the court can rest easy on a finding of guilt of appellant. As regards the imposable penalty for the crimes of appellant; Article 315 of the Revised Penal Code, provides: Swindling (estafa). Any person who shall defraud another by any of the means mentioned herein below shall be punished by: 1st. The penalty of prision correccional in its maximum period to prision mayor in its minimum period, if the amount of the fraud is over 12,000 pesos but does not exceed 22,000 pesos; and if such amount exceeds the latter sum, the penalty provided in this paragraph shall be imposed in its maximum period, adding one year for each additional 10,000 pesos; but the total penalty which may be imposed shall not exceed twenty years. . .. Under Section 1 of Act 4103, the maximum term of the penalty is "that which, in view of the attending circumstances, could be properly imposed" under the Revised Penal Code, and the minimum should be "within the range of the penalty next lower to that prescribed" for the offense." The penalty next lower should be based on the penalty prescribed by the Revised Penal Code for the offense, without first considering any modifying circumstance attendant to the commission of the crime. The minimum of the indeterminate penalty is left to the sound discretion of the court, to fix from within the range of the penalty next lower without reference to the periods into which it might be subdivided. The modifying circumstances are considered only in the imposition of the maximum term of the indeterminate sentence. In Criminal Case Nos. 93-129377, 93-129378, 93-129380-84, the amount involved in each case is P15,000.00. Since the penalty prescribed by law therefor is prision correccional maximum to prision mayor minimum, the penalty next lower thereto is prision correccional minimum to prision correccional medium. Thus, the minimum of the indeterminate sentence to be meted here should be two (2) years, eleven (11) months and ten (10) days. Explicit is the said Indeterminate Sentence Law, that the maximum of the penalty shall be "that which, in view of the attending circumstances, could be properly imposed" under the Revised Penal Code. Accordingly, the maximum of subject indeterminate sentence is six (6) years, eight (8) months and twenty (20) days of prision mayor. In Criminal Case Nos. 93-129376 and 93-129379, the amount involved in each case is P22,600.00. Since the penalty prescribed by law therefor is prision correccional maximum to prision mayor minimum, to be imposed in itsmaximum period, the penalty next lower in degree should be four (4) years and two (2) months of prision correccional, and the maximum penalty therefor should be seven (7) years and four (4) months of prision mayor, in each case. WHEREFORE, the judgment of conviction appealed from is AFFIRMED, and accused-appellant is hereby sentenced, as follows: 305

University of the Cordilleras College of Law First Year C S.Y. 2013 - 2014 (1) In Criminal case No. 93-129374 for illegal recruitment, he is sentenced to suffer life imprisonment, and to pay a fine of P100,000.00; (2) In Criminal Case Nos. 93-129376 and 93-129379 for estafa, he is sentenced in each case, to an indeterminate penalty of four (4) years, two (2) months, as minimum, to seven (7) years and four (4) months, as maximum; (3) In Criminal Case Nos. 93-129377, 93-129378, 93-129380-84 for estafa, appellant, is sentenced in each case, to an indeterminate penalty of two (2) years, eleven (11) months and ten (10) days, as minimum, to six (6) years, eight (8) months and twenty (20) days, as maximum, and to pay the complainants P15,000.00 each; and (4) In Criminal Case No. 93-129375, appellant is ACQUITTED of the crime of estafa charged, for lack of evidence to prove his guilt beyond reasonable doubt. Costs against accused- appellant. SO ORDERED.

306

University of the Cordilleras College of Law First Year C S.Y. 2013 - 2014 Case Digest THE PEOPLE OF THE PHILIPPINES vs. ERNESTO A. BORROMEO G.R. No. 117154 March 25, 1999 133 SCRA 106 PURISIMA, J. Facts: At high noon on July 3, 1981, the four year old niece of Susana & Elias Borromeo told Matilde Taborada (mother of Susana) that Susana was screaming because Elias was killing her. Taborada told her to inform her son, Geronimo Taborada. Geronimo, in turn, told his father and together, they went to Susanas hut. There they found Susanas lifeless body next to her crying infant and Elias mumbling incoherently still with the weapon in his hands. The accused-appellant, Elias, said that because they were legally and validly married, he should only be liable for homicide and not parricide. He thinks such because there was no marriage contract issued on their wedding day and after that. However, in his testimony, he admitted that the victim was his wife and that they were married in a chapel by a priest. Issue: Does the non-execution of a marriage contract render a marriage void? Held: In the view of the law, a couple living together with the image of being married, are presumed married unless proven otherwise. This is attributed to the common order of society. Furthermore, the validity of a marriage resides on the fulfillment or presence of the requisites of the marriage which are : legal capacity and consent. The absence of the record of such marriage does not invalidate the same as long as the celebration and all requisites are present. Person living together in apparent matrimony are presumed, in the absence of any counter presumption or evidence special to the case, to be in fact married. The reason is that such is the common order of society, and if the parties were not what they thus hold themselves out as being, they would be living in constant violation of decency and law. (Son Cui vs. Guepangco, 22 Phil. 216). And, the mere fact that no record of the marriage exists in the registry of marriage does not invalidate said marriage, as long as in the celebration thereof, all requisites for its validity are present. The forwarding of a copy of the marriage certificate to the registry is not one of said requisites. (Pugeda vs. Trias, 4 SCRA 849). The appealed decision is AFFIRMED and the indemnity increased from 12,000 to 30,000

307

University of the Cordilleras College of Law First Year C S.Y. 2013 - 2014 Seguisabal vs Cabrera 106 SCRA 67 A.M. No. L-2209-CTJ August 27, 1981 Full Case ABDON SEGUISABAL, complainant, vs. HON. JOSE R. CABRERA, City Judge of Toledo City, respondent. MELENCIO-HERRERA, J.: In his verified Complaint filed on 18 June 1979, Abdon Seguisabal has charged City Judge Jose R. Cabrera of Toledo City with gross misconduct in office and gross ignorance of the law for having solemnized, on 14 April 1978, the marriage of Jaime Sayson and Marlyn Jagonoy without the requisite marriage license pursuant to Article 53 of the Civil Code, and for having failed to transmit a copy of the marriage contract, signed by him and the parties, to the Office of the Local Civil Registrar of Toledo City within fifteen (15) days from the date of solemnization as mandatorily required by Article 68 of the same Code. That respondent actually solemnized said marriage without the requisite license, is shown by the marriage contract issued to the contracting parties (Annexes "C", "C-1"). The f failure to transmit a copy of the marriage contract to the Local Civil Registrar is substantiated by the Certifications, both issued on 5 June 1979, by the Local Civil Registrar of Toledo City (Annex "A" and "B"), Required to comment, respondent Judge explained: On April 14, 1978 at around 12:00 o'clock noon, Jaime Sayson and Marlyn Jagonoy accompanied by the mother of Jaime Sayson the father of Marlyn Jagonoy and several others appeared before my Office bringing with them a Marriage Contract to be solemnized in marriage. I asked them for their Marriage License but they told me that the Local Civil Registrar of Toledo City cannot issue the same because the one in-charge was not in his Office, it being already 12:00 o'clock noon. The bride-to-be was three months pregnant. Presuming that the papers were in order inasmuch as the parents were present, I solemnized the marriage but told the parties to come back in the afternoon together with the Marriage License. The parties did not come in the afternoon and the papers left in my office were lost in the mass of paper works attendant in the Office of the City Judge, in fact, I have virtually forgotten about it myself. Sometime in May 1979, about a year after, a crying girl bringing with her a child appeared before me in my office and I Identified her as Marlyn Jagonoy. She informed me that her husband, Jaime Sayson, who was a draftee in the Philippine Army died in an encounter with the Muslim rebels in Maguindanao and subsequently died. The army authorities will give her the benefits if and when she can prove that she was actually married to the said Jaime Sayson. In sympathy and fairness to Marlyn Jagonoy whose marriage I actually solemnized, I searched for the papers and found them. I told the father of Marlyn to go to the Local Civil Registrar's Office in order that Marriage License be issued to her which he did but came back and told me that the Local Civil Registrar will not receive the papers and will not issue the Marriage License for the reason that the parties have not attended the Family Planning Seminar required by law before Marriage License may be issued. Believing that Family Planning was no longer necessary inasmuch as Jaime Sayson was already dead, I issued to them the Marriage Contract in order that they can enjoy the benefits accruing Jaime Sayson who died a hero's death in the service of the flag of the Republic. In issuing the Marriage Contract, I had done it in good faith and in sympathy and in fairness to the widow, Marlyn Jagonoy, whom I believe is entitled to the benefits she could enjoy it simply because of the technicality of the law. The issuance of the Marriage Contract made everybody happy, The parents of the boy and the parents of the girl were satisfied and are not even a party to this Complaint (Rollo, pp. 9-10).

308

University of the Cordilleras College of Law First Year C S.Y. 2013 - 2014 Respondent Judge further averred that the complainant herein was obviously ill-motivated and resorted to this administrative action out of spite because he had, on 24 November 1978, dismissed Criminal Case No. A-1712 for Alarm and Scandal filed by complainant against a certain Marcelo Rizal, and that complainant is the accused in Criminal Case No. A-1907 for Qualified Theft pending before respondent's Court. Considering the admissions made by respondent, and as observed in the Memorandum Report dated 22 October 1980 submitted by Deputy Court Administrator, Leo D. Medialdea, concurred in by Court Administrator, Justice Lorenzo Relova, there was no more need for a formal investigation to determine the administrative liability of respondent Judge. Respondent must be held guilty of the charge filed for in solemnizing the marriage of Jaime Sayson and Marlyn Jagonoy on 14 April 1978 without requiring the essential pre-requisite of a marriage license, respondent had undoubtedly transgressed Article 53(4) of the Civil Code in the absence of any showing that the subject marriage falls under marriages of an exceptional character wherein a license is not mandatorily required. Respondent was likewise remiss in his duty under Article 68 of the Civil Code to transmit to the Local Civil Registrar of Toledo City within fifteen (15) days from the date of solemnization of the marriage in question, a copy of the marriage contract duly signed by him as the solemnizing officer and by the contracting parties. The defense of good faith interposed by respondent is unavailing. As a judicial officer, he is expected to know the law on the solemnization of marriages. His feeling of sympathy and fairness to the widow, Marlyn Jagonoy" cannot serve as a license for him to deliberately transgress or dispense with legal requisites. In view, however, of respondent's twenty-seven (27) years and seven (7) months of service in the Judiciary, and considering that he has applied for retirement under Republic Act No. 5095 due to schemic heart ailment, we have mitigated the corresponding administrative sanction. WHEREFORE, finding respondent Judge to be guilty of gross neglect of duty, he shall pay a fine equivalent to three (3) months salary, the same to be deducted from his gratuity upon his retirement from the service. A copy of this Decision should be attached to his personal record. SO ORDERED, Teehankee, (Chairman), Makasiar, Fernandez and Guerrero, JJ., concur.

309

University of the Cordilleras College of Law First Year C S.Y. 2013 - 2014 Case Digest ABDON SEGUISABAL vs. HON. JOSE R. CABRERA A.M. No. L-2209-CTJ August 27, 1981 106 SCRA 67 MELENCIO-HERRERA, J. Facts: Abdon Seguisabal has charged City Judge Jose R. Cabrera of Toledo City with gross misconduct in office and gross ignorance of the law for having solemnized, on 14 April 1978, the marriage of Jaime Sayson and Marlyn Jagonoy without the requisite marriage and for having failed to transmit a copy of the marriage contract, signed by him and the parties, to the Office of the Local Civil Registrar of Toledo City within fifteen (15) days from the date of solemnization as mandatorily required. That respondent actually solemnized said marriage without the requisite license, is shown by the marriage contract issued to the contracting parties. The failure to transmit a copy of the marriage contract to the Local Civil Registrar is substantiated by the Certifications, both issued on 5 June 1979, by the Local Civil Registrar of Toledo City Issue: Whether or not Hon. Jose R. Cabrera is liable in solemnizing a marriage without requiring the essential pre-requisite of a marriage license. Ruling: Respondent must be held guilty of the charge filed for in solemnizing the marriage of Jaime Sayson and Marlyn Jagonoy on 14 April 1978 without requiring the essential pre-requisite of a marriage license, respondent had undoubtedly transgressed Article 53(4) of the Civil Code in the absence of any showing that the subject marriage falls under marriages of an exceptional character wherein a license is not mandatorily required. Respondent was likewise remiss in his duty under Article 68 of the Civil Code to transmit to the Local Civil Registrar of Toledo City within fifteen (15) days from the date of solemnization of the marriage in question, a copy of the marriage contract duly signed by him as the solemnizing officer and by the contracting parties. The defense of good faith interposed by respondent is unavailing. As a judicial officer, he is expected to know the law on the solemnization of marriages. His feeling of sympathy and fairness to the widow, Marlyn Jagonoy" cannot serve as a license for him to deliberately transgress or dispense with legal requisites. WHEREFORE, finding respondent Judge to be guilty of gross neglect of duty, he shall pay a fine equivalent to three (3) months salary, the same to be deducted from his gratuity upon his retirement from the service.

310

University of the Cordilleras College of Law First Year C S.Y. 2013 - 2014 Nial vs Bayadog 328 SCRA 122 G.R. No. 133778 March 14, 2000 Full Case ENGRACE NIAL for Herself and as Guardian ad Litem of the minors BABYLINE NIAL, INGRID NIAL, ARCHIE NIAL & PEPITO NIAL, JR., petitioners, vs. NORMA BAYADOG, respondent. YNARES-SANTIAGO, J.: May the heirs of a deceased person file a petition for the declaration of nullity of his marriage after his death? Pepito Nial was married to Teodulfa Bellones on September 26, 1974. Out of their marriage were born herein petitioners. Teodulfa was shot by Pepito resulting in her death on April 24, 1985. One year and 8 months thereafter or on December 11, 1986, Pepito and respondent Norma Badayog got married without any marriage license. In lieu thereof, Pepito and Norma executed an affidavit dated December 11, 1986 stating that they had lived together as husband and wife for at least five years and were thus exempt from securing a marriage license. On February 19, 1997, Pepito died in a car accident. After their father's death, petitioners filed a petition for declaration of nullity of the marriage of Pepito to Norma alleging that the said marriage was void for lack of a marriage license. The case was filed under the assumption that the validity or invalidity of the second marriage would affect petitioner's successional rights. Norma filed a motion to dismiss on the ground that petitioners have no cause of action since they are not among the persons who could file an action for "annulment of marriage" under Article 47 of the Family Code. Judge Ferdinand J. Marcos of the Regional Trial Court of Toledo City, Cebu, Branch 59, dismissed the petition after finding that the Family Code is "rather silent, obscure, insufficient" to resolve the following issues: (1) Whether or not plaintiffs have a cause of action against defendant in asking for the declaration of the nullity of marriage of their deceased father, Pepito G. Nial, with her specially so when at the time of the filing of this instant suit, their father Pepito G. Nial is already dead; (2) Whether or not the second marriage of plaintiffs' deceased father with defendant is null and void ab initio; (3) Whether or not plaintiffs are estopped from assailing the validity of the second marriage after it was dissolved due to their father's death. Thus, the lower court ruled that petitioners should have filed the action to declare null and void their father's marriage to respondent before his death, applying by analogy Article 47 of the Family Code which enumerates the time and the persons who could initiate an action for annulment of marriage. Hence, this petition for review with this Court grounded on a pure question of law. This petition was originally dismissed for non-compliance with Section 11, Rule 13 of the 1997 Rules of Civil Procedure, and because "the verification failed to state the basis of petitioner's averment that the allegations in the petition are "true and correct"." It was thus treated as an unsigned pleading which produces no legal effect under Section 3, Rule 7, of the 1997 Rules. However, upon motion of petitioners, this Court reconsidered the dismissal and reinstated the petition for review. The two marriages involved herein having been solemnized prior to the effectivity of the Family Code (FC), the applicable law to determine their validity is the Civil Code which was the law in effect at the time of their celebration. A valid marriage license is a requisite of marriage under Article 53 of the Civil Code, the absence of which renders the marriage void ab initio pursuant to Article 80(3) in relation to Article 58. The requirement and issuance of marriage license is the State's demonstration of its involvement and participation in every marriage, in the maintenance of which the general public is interested. This interest proceeds from the constitutional mandate that the State recognizes the sanctity of family life and of affording protection to the family as a basic "autonomous social 311

University of the Cordilleras College of Law First Year C S.Y. 2013 - 2014 institution." Specifically, the Constitution considers marriage as an "inviolable social institution," and is the foundation of family life which shall be protected by the State. This is why the Family Code considers marriage as "a special contract of permanent union" and case law considers it "not just an adventure but a lifetime commitment." However, there are several instances recognized by the Civil Code wherein a marriage license is dispensed with, one of which is that provided in Article 76, referring to the marriage of a man and a woman who have lived together and exclusively with each other as husband and wife for a continuous and unbroken period of at least five years before the marriage. The rationale why no license is required in such case is to avoid exposing the parties to humiliation, shame and embarrassment concomitant with the scandalous cohabitation of persons outside a valid marriage due to the publication of every applicant's name for a marriage license. The publicity attending the marriage license may discourage such persons from legitimizing their status. To preserve peace in the family, avoid the peeping and suspicious eye of public exposure and contain the source of gossip arising from the publication of their names, the law deemed it wise to preserve their privacy and exempt them from that requirement. There is no dispute that the marriage of petitioners' father to respondent Norma was celebrated without any marriage license. In lieu thereof, they executed an affidavit stating that "they have attained the age of majority, and, being unmarried, have lived together as husband and wife for at least five years, and that we now desire to marry each other." The only issue that needs to be resolved pertains to what nature of cohabitation is contemplated under Article 76 of the Civil Code to warrant the counting of the five year period in order to exempt the future spouses from securing a marriage license. Should it be a cohabitation wherein both parties are capacitated to marry each other during the entire five-year continuous period or should it be a cohabitation wherein both parties have lived together and exclusively with each other as husband and wife during the entire five-year continuous period regardless of whether there is a legal impediment to their being lawfully married, which impediment may have either disappeared or intervened sometime during the cohabitation period? Working on the assumption that Pepito and Norma have lived together as husband and wife for five years without the benefit of marriage, that five-year period should be computed on the basis of a cohabitation as "husband and wife" where the only missing factor is the special contract of marriage to validate the union. In other words, the five-year common-law cohabitation period, which is counted back from the date of celebration of marriage, should be a period of legal union had it not been for the absence of the marriage. This 5-year period should be the years immediately before the day of the marriage and it should be a period of cohabitation characterized by exclusivity meaning no third party was involved at anytime within the 5 years and continuity that is unbroken. Otherwise, if that continuous 5-year cohabitation is computed without any distinction as to whether the parties were capacitated to marry each other during the entire five years, then the law would be sanctioning immorality and encouraging parties to have common law relationships and placing them on the same footing with those who lived faithfully with their spouse. Marriage being a special relationship must be respected as such and its requirements must be strictly observed. The presumption that a man and a woman deporting themselves as husband and wife is based on the approximation of the requirements of the law. The parties should not be afforded any excuse to not comply with every single requirement and later use the same missing element as a pre-conceived escape ground to nullify their marriage. There should be no exemption from securing a marriage license unless the circumstances clearly fall within the ambit of the exception. It should be noted that a license is required in order to notify the public that two persons are about to be united in matrimony and that anyone who is aware or has knowledge of any impediment to the union of the two shall make it known to the local civil registrar. The Civil Code provides: Art. 63: . . . This notice shall request all persons having knowledge of any impediment to the marriage to advice the local civil registrar thereof. . . . Art. 64: Upon being advised of any alleged impediment to the marriage, the local civil registrar shall forthwith make an investigation, examining persons under oath. . . . This is reiterated in the Family Code thus: Art. 17 provides in part: . . . This notice shall request all persons having knowledge of any impediment to the marriage to advise the local civil registrar thereof. . . .

312

University of the Cordilleras College of Law First Year C S.Y. 2013 - 2014 Art. 18 reads in part: . . . In case of any impediment known to the local civil registrar or brought to his attention, he shall note down the particulars thereof and his findings thereon in the application for a marriage license. . . . This is the same reason why our civil laws, past or present, absolutely prohibited the concurrence of multiple marriages by the same person during the same period. Thus, any marriage subsequently contracted during the lifetime of the first spouse shall be illegal and void, subject only to the exception in cases of absence or where the prior marriage was dissolved or annulled. The Revised Penal Code complements the civil law in that the contracting of two or more marriages and the having of extramarital affairs are considered felonies, i.e., bigamy and concubinage and adultery. The law sanctions monogamy. In this case, at the time of Pepito and respondent's marriage, it cannot be said that they have lived with each other as husband and wife for at least five years prior to their wedding day. From the time Pepito's first marriage was dissolved to the time of his marriage with respondent, only about twenty months had elapsed. Even assuming that Pepito and his first wife had separated in fact, and thereafter both Pepito and respondent had started living with each other that has already lasted for five years, the fact remains that their five-year period cohabitation was not the cohabitation contemplated by law. It should be in the nature of a perfect union that is valid under the law but rendered imperfect only by the absence of the marriage contract. Pepito had a subsisting marriage at the time when he started cohabiting with respondent. It is immaterial that when they lived with each other, Pepito had already been separated in fact from his lawful spouse. The subsistence of the marriage even where there was actual severance of the filial companionship between the spouses cannot make any cohabitation by either spouse with any third party as being one as "husband and wife". Having determined that the second marriage involved in this case is not covered by the exception to the requirement of a marriage license, it is void ab initio because of the absence of such element. The next issue to be resolved is: do petitioners have the personality to file a petition to declare their father's marriage void after his death? Contrary to respondent judge's ruling, Article 47 of the Family Code cannot be applied even by analogy to petitions for declaration of nullity of marriage. The second ground for annulment of marriage relied upon by the trial court, which allows "the sane spouse" to file an annulment suit "at anytime before the death of either party" is inapplicable. Article 47 pertains to the grounds, periods and persons who can file an annulment suit, not a suit for declaration of nullity of marriage. The Code is silent as to who can file a petition to declare the nullity of a marriage. Voidable and void marriages are not identical. A marriage that is annulable is valid until otherwise declared by the court; whereas a marriage that is void ab initio is considered as having never to have taken place and cannot be the source of rights. The first can be generally ratified or confirmed by free cohabitation or prescription while the other can never be ratified. A voidable marriage cannot be assailed collaterally except in a direct proceeding while a void marriage can be attacked collaterally. Consequently, void marriages can be questioned even after the death of either party but voidable marriages can be assailed only during the lifetime of the parties and not after death of either, in which case the parties and their offspring will be left as if the marriage had been perfectly valid. That is why the action or defense for nullity is imprescriptible, unlike voidable marriages where the action prescribes. Only the parties to a voidable marriage can assail it but any proper interested party may attack a void marriage. Void marriages have no legal effects except those declared by law concerning the properties of the alleged spouses, regarding co-ownership or ownership through actual joint contribution, and its effect on the children born to such void marriages as provided in Article 50 in relation to Article 43 and 44 as well as Article 51, 53 and 54 of the Family Code. On the contrary, the property regime governing voidable marriages is generally conjugal partnership and the children conceived before its annulment are legitimate. Contrary to the trial court's ruling, the death of petitioner's father extinguished the alleged marital bond between him and respondent. The conclusion is erroneous and proceeds from a wrong premise that there was a marriage bond that was dissolved between the two. It should be noted that their marriage was void hence it is deemed as if it never existed at all and the death of either extinguished nothing. Jurisprudence under the Civil Code states that no judicial decree is necessary in order to establish the nullity of a marriage. "A void marriage does not require a judicial decree to restore the parties to their original rights or to make the marriage void but though no sentence of avoidance be absolutely necessary, yet as well for the sake of good order of society as for the peace of mind of all concerned, it is expedient 313

University of the Cordilleras College of Law First Year C S.Y. 2013 - 2014 that the nullity of the marriage should be ascertained and declared by the decree of a court of competent jurisdiction." "Under ordinary circumstances, the effect of a void marriage, so far as concerns the conferring of legal rights upon the parties, is as though no marriage had ever taken place. And therefore, being good for no legal purpose, its invalidity can be maintained in any proceeding in which the fact of marriage may be material, either direct or collateral, in any civil court between any parties at any time, whether before or after the death of either or both the husband and the wife, and upon mere proof of the facts rendering such marriage void, it will be disregarded or treated as non-existent by the courts." It is not like a voidable marriage which cannot be collaterally attacked except in direct proceeding instituted during the lifetime of the parties so that on the death of either, the marriage cannot be impeached, and is made good ab initio. But Article 40 of the Family Code expressly provides that there must be a judicial declaration of the nullity of a previous marriage, though void, before a party can enter into a second marriage and such absolute nullity can be based only on a final judgment to that effect. For the same reason, the law makes either the action or defense for the declaration of absolute nullity of marriage imprescriptible. Corollarily, if the death of either party would extinguish the cause of action or the ground for defense, then the same cannot be considered imprescriptible. However, other than for purposes of remarriage, no judicial action is necessary to declare a marriage an absolute nullity.1wphi1 For other purposes, such as but not limited to determination of heirship, legitimacy or illegitimacy of a child, settlement of estate, dissolution of property regime, or a criminal case for that matter, the court may pass upon the validity of marriage even in a suit not directly instituted to question the same so long as it is essential to the determination of the case. This is without prejudice to any issue that may arise in the case. When such need arises, a final judgment of declaration of nullity is necessary even if the purpose is other than to remarry. The clause "on the basis of a final judgment declaring such previous marriage void" in Article 40 of the Family Code connotes that such final judgment need not be obtained only for purpose of remarriage. WHEREFORE, the petition is GRANTED. The assailed Order of the Regional Trial Court, Toledo City, Cebu, Branch 59, dismissing Civil Case No. T-639, is REVERSED and SET ASIDE. The said case is ordered REINSTATED.1wphi1.nt SO ORDERED.

314

University of the Cordilleras College of Law First Year C S.Y. 2013 - 2014 Case Digest ENGRACE NIAL vs. NORMA BAYADOG G.R. No. 133778 March 14, 2000 328 SCRA 122 YNARES-SANTIAGO, J. Facts: Pepito Ninal was married with Teodulfa Bellones on September 26, 1974. They had 3 children Babyline Nial, Ingrid Nial, Archie Nial and Pepito Nial Jr.the petitioners. Due to the shot inflicted by Pepito to Teodulfa, the latter died on April 24, 1985. 1 year and 8 months later on December 11, 1986, Pepito and Norma Badayog got married without any marriage licence. They instituted an affidavit stating that they had lived together for at least 5 years exempting from securing the marriage license. Pepito died in a car accident on February 19, 1997. After his death, petitioners filed a petition for declaration of nullity of the marriage of Pepito and Norma alleging that said marriage was void for lack of marriage license. Issues: Whether or not the second marriage of Pepito was void. Whether or not the heirs of the deceased may file for the declaration of the nullity of Pepitos marriage and even after his death. Ruling:Yes, the second marriage of Pepito was void for absence of marriage licence. Yes, the heirs of the deceased may file for the declaration of the nullity of Pepit os marriage. The marriage will be disregarded or treated as non-existent by the courts upon mere proof of facts even after the latters death. Even though Pepito and Norma instituted an affidavit and claimed that they cohabit for at least 5 years, the marriage would not be valid because from the time of Pepitos first marriage was dissolved to the time of his marriage with Norma, only about 20 months had elapsed. Pepito and his first wife may had separated in fact, and thereafter both Pepito and Norma had started living with each other that has already lasted for five years, the fact remains that their five-year period cohabitation was not the cohabitation contemplated by law. Hence, his marriage to Norma is still void. Void marriages are deemed to have not taken place and cannot be the source of rights. It can be questioned even after the death of one of the parties and any proper interested party may attack a void marriage.

315

University of the Cordilleras College of Law First Year C S.Y. 2013 - 2014 Borja-Manzano vs Sanchez 354 SCRA 1 A.M. No. MTJ-00-1329 March 8, 2001 Full Case HERMINIA BORJA-MANZANO, petitioner, vs. JUDGE ROQUE R. SANCHEZ, MTC, Infanta, Pangasinan, respondent. DAVIDE, JR., C.J.: The solemnization of a marriage between two contracting parties who were both bound by a prior existing marriage is the bone of contention of the instant complaint against respondent Judge Roque R. Sanchez, Municipal Trial Court, Infanta, Pangasinan. For this act, complainant Herminia Borja-Manzano charges respondent Judge with gross ignorance of the law in a sworn Complaint-Affidavit filed with the Office of the Court Administrator on 12 May 1999. Complainant avers that she was the lawful wife of the late David Manzano, having been married to him on 21 May 1966 in San Gabriel Archangel Parish, Araneta Avenue, Caloocan City. Four children were born out of that marriage. On 22 March 1993, however, her husband contracted another marriage with one Luzviminda Payao before respondent Judge. When respondent Judge solemnized said marriage, he knew or ought to know that the same was void and bigamous, as the marriage contract clearly stated that both contracting parties were "separated." Respondent Judge, on the other hand, claims in his Comment that when he officiated the marriage between Manzano and Payao he did not know that Manzano was legally married. What he knew was that the two had been living together as husband and wife for seven years already without the benefit of marriage, as manifested in their joint affidavit. According to him, had he known that the late Manzano was married, he would have advised the latter not to marry again; otherwise, he (Manzano) could be charged with bigamy. He then prayed that the complaint be dismissed for lack of merit and for being designed merely to harass him. After an evaluation of the Complaint and the Comment, the Court Administrator recommended that respondent Judge be found guilty of gross ignorance of the law and be ordered to pay a fine of P2,000, with a warning that a repetition of the same or similar act would be dealt with more severely. On 25 October 2000, this Court required the parties to manifest whether they were willing to submit the case for resolution on the basis of the pleadings thus filed. Complainant answered in the affirmative. For his part, respondent Judge filed a Manifestation reiterating his plea for the dismissal of the complaint and setting aside his earlier Comment. He therein invites the attention of the Court to two separate affidavits of the late Manzano and of Payao, which were allegedly unearthed by a member of his staff upon his instruction. In those affidavits, both David Manzano and Luzviminda Payao expressly stated that they were married to Herminia Borja and Domingo Relos, respectively; and that since their respective marriages had been marked by constant quarrels, they had both left their families and had never cohabited or communicated with their spouses anymore. Respondent Judge alleges that on the basis of those affidavits, he agreed to solemnize the marriage in question in accordance with Article 34 of the Family Code. We find merit in the complaint. Article 34 of the Family Code provides: No license shall be necessary for the marriage of a man and a woman who have lived together as husband and wife for at least five years and without any legal impediment to marry each other. The contracting parties shall state the foregoing facts in an affidavit before any person authorized by law to administer oaths. The solemnizing officer shall also state under oath that he ascertained the qualifications of the contracting parties and found no legal impediment to the marriage. For this provision on legal ratification of marital cohabitation to apply, the following requisites must concur: 316

University of the Cordilleras College of Law First Year C S.Y. 2013 - 2014 1. The man and woman must have been living together as husband and wife for at least five years before the marriage; 2. The parties must have no legal impediment to marry each other; 3. The fact of absence of legal impediment between the parties must be present at the time of marriage; 4. The parties must execute an affidavit stating that they have lived together for at least five years [and are without legal impediment to marry each other]; and 5. The solemnizing officer must execute a sworn statement that he had ascertained the qualifications of the parties and that he had found no legal impediment to their marriage. Not all of these requirements are present in the case at bar. It is significant to note that in their separate affidavits executed on 22 March 1993 and sworn to before respondent Judge himself, David Manzano and Luzviminda Payao expressly stated the fact of their prior existing marriage. Also, in their marriage contract, it was indicated that both were "separated." Respondent Judge knew or ought to know that a subsisting previous marriage is a diriment impediment, which would make the subsequent marriage null and void. In fact, in his Comment, he stated that had he known that the late Manzano was married he would have discouraged him from contracting another marriage. And respondent Judge cannot deny knowledge of Manzanos and Payaos subsisting previous marriage, as the same was clearly stated in their separate affidavits which were subscribed and sworn to before him. The fact that Manzano and Payao had been living apart from their respective spouses for a long time already is immaterial. Article 63(1) of the Family Code allows spouses who have obtained a decree of legal separation to live separately from each other, but in such a case the marriage bonds are not severed. Elsewise stated, legal separation does not dissolve the marriage tie, much less authorize the parties to remarry. This holds true all the more when the separation is merely de facto, as in the case at bar. Neither can respondent Judge take refuge on the Joint Affidavit of David Manzano and Luzviminda Payao stating that they had been cohabiting as husband and wife for seven years. Just like separation, free and voluntary cohabitation with another person for at least five years does not severe the tie of a subsisting previous marriage. Marital cohabitation for a long period of time between two individuals who are legally capacitated to marry each other is merely a ground for exemption from marriage license. It could not serve as a justification for respondent Judge to solemnize a subsequent marriage vitiated by the impediment of a prior existing marriage. Clearly, respondent Judge demonstrated gross ignorance of the law when he solemnized a void and bigamous marriage. The maxim "ignorance of the law excuses no one" has special application to judges, who, under Rule 1.01 of the Code of Judicial Conduct, should be the embodiment of competence, integrity, and independence. It is highly imperative that judges be conversant with the law and basic legal principles. And when the law transgressed is simple and elementary, the failure to know it constitutes gross ignorance of the law. ACCORDINGLY, the recommendation of the Court Administrator is hereby ADOPTED, with the MODIFICATION that the amount of fine to be imposed upon respondent Judge Roque Sanchez is increased to P20,000. SO ORDERED.

317

University of the Cordilleras College of Law First Year C S.Y. 2013 - 2014 Case Digest HERMINIA BORJA-MANZANO vs. JUDGE ROQUE R. SANCHEZ, MTC A.M. No. MTJ-00-1329 March 8, 2001 354 SCRA 1 DAVIDE, JR., C.J. FACTS: Complainant, HerminiaBorja-Manzano charges respondent Judge Roque R. Sanchez through sworn Complaint-Affidavit filed with the Office of the Court Administrator on May 12, 1999, with gross ignorance of the law by solemnizing a marriage between her husband and another woman, who were both bound to prior existing marriage. HerminiaBorja-Manzano affirms that she is the lawful wife of David Manzano, having been married to him on 21 May 1966 in San Gabriel Archangel Parish, Araneta Avenue, Caloocan City.Four children were born out of that marriage.However, her husband contracted another marriage to LuzvimindaPayao on March 22, 1993 before Judge Roque R. Sanchez, who then knew or must know that such marriage is void or bigamous since both contracting parties were separated as stated in their marriage contract. ISSUE: Whether or not the respondent Judge is guilty. RULING: Yes. The Respondent Judge demonstrated gross ignorance of the law when he solemnized a void and bigamous marriage. Under Article 34 of Family Code, the requisites enumerated in the provision on legal ratification of marital cohabitation to apply are not all present. It is clearly stated in the affidavits executed by both David Manzano and Luzviminda Payao the fact of their prior existing marriage and both were separated. Marital cohabitation for a long period of time is a mere exemption from marriage license. Just like separation, free and voluntary cohabitation with another person for at least five years does not severe the tie of a subsisting previous marriage.The solemnizing officer knew and ought to know that a subsisting previous marriage is a diriment impediment, which would make the subsequent marriage null and void. Clearly, respondent Judge demonstrated gross ignorance of the law when he solemnized a void and bigamous marriage. The Court Administrator recommended that respondent be found guilty of gross ignorance of the law and be ordered to pay a fine of P2,000, with a warning that a repetition of the same or similar act would be dealt with more severely. The recommendation of the Court Administrator is adopted with modification that the fine of P2,000 is increased to P20,000.

318

University of the Cordilleras College of Law First Year C S.Y. 2013 - 2014 Yap vs Yap 145 SCRA 299 G.R. No. L-40003 October 28, 1986 Full Case SHIRLEY YAP, in her own behalf and in her capacity as Administratrix of the estate of MANING YAP, JAIME YAP, and TALINA BIANONG VDA. DE YAP, petitioners, vs. COURT OF APPEALS, NANCY J. YAP, MANING YAP, JR., JULIA YAP, JASMIN YAP, and SAMUEL YAP,respondents. GUTIERREZ, JR., J.: This is a petition to review the decision of the Court of Appeals which set aside the earlier decision of the then Court of First Instance of Lanao del Sur in Special Proceeding No. 1334 (R-61), declaring the petitioners as the legal heirs of the late Maning Yap entitled to inherit his estate and dismissing the opposition filed by the private respondents. The dispositive portion of the decision on appeal reads: WHEREFORE, the decision appealed from is hereby set aside and, after a complete and correct inventory is returned by the administratrix, the entire estate of the deceased Maning Yap shall be divided into two equal parts, one-half (1/2) corresponding to the petitioner Talina Bianong and her children Shirley Yap and Jaime Yap and the other half corresponding to the oppositors Nancy J. Yap and her children Maning Yap, Jr., Julia Yap, Jasmin Yap and Samuel Yap, without pronouncement as to costs. Maning Yap, during his lifetime married twice: first, to Talina Bianong in 1939 and second, to Nancy Yap on December 11, 1948. Maning Yap and Talina Bianong were married at Bara-as Plantation, Malabang, Lanao del Sur, in accordance with the Muslim rites and practices prescribed by the Islam religion professed by both of them. Immediately, after the marriage, the couple lived in the house of the parents of Maning Yap at the poblacion of Malabang, Lanao del Sur. Out of the marriage, four children were born; two of them died in infancy during the Japanese occupation, while the two others are petitioners Shirley Yap and Jaime Yap. While the first marriage was still subsisting, Maning Yap married Nancy J. Yap on December 11, 1948 in a civil ceremony performed by District Judge Juan Sarenas of the Court of First Instance of Cotabato. Nancy Yap entered into the marriage in the belief that Maning Yap was not a married man. They had four children, namely respondents Maning Yap, Jr., Julia Yap, Jasmin Yap and Samuel Yap. On February 21, 1964, Maning Yap died in Piagapo, Lanao del Sur, in the crash of an airplane of the Philippine Air Lines. At the time of his death he, therefore, had two families living separately about 80 kilometers apart. On March 3, 1964, Talina Bianong Vda. de Yap filed Special Proceeding No. 1334 (Intestate Estate of Maning Yap) before the Court of First Instance of Lanao del Sur, seeking the issuance of letters of administration for the estate of Maning Yap. Among other things, the petition alleged that Maning Yap left personal and real properties all located at Malabang, Lanao del Sur, with an approximate value of P100,000.00. The petition was opposed by Nancy J. Yap and her minor children on the ground that she is the legitimate widow of Maning Yap and that Maning Yap, Jr., Julia Yap, Jasmin Yap and Samuel Yap, all minors, are their legitimate children. Talina Bianong was initially appointed special administratrix of the intestate estate of Maning Yap. However, after a formal hearing and on recommendation of Talina, the lower court appointed Shirley Yap as regular administratrix of the intestate estate of Maning Yap. Various claims filed by the creditors against the intestate estate of Maning Yap were duly approved by the court and paid by the administratrix. Since there still existed a residue of the intestate estate consisting of real and personal properties and collectible debts after payments to creditors, the court set the case for hearing to arrive at a declaration of heirship for the purpose of liquidating the conjugal partnership of the late Maning Yap and his surviving spouse and to determine the heirs entitled to inherit his intestate estate. 319

University of the Cordilleras College of Law First Year C S.Y. 2013 - 2014 After trial, the lower court rendered decision declaring Talina Bianong and her children as the legal heirs of Maning Yap. The dispositive portion of the decision reads: IN VIEW OF ALL THE FOREGOING, judgment is hereby rendered: (a) Declaring Talina Bianong, Shirley Yap and Jaime Yap, the legal heirs of the late Maning Yap and entitled to inherit or succeed to his intestate with Talina Bianong, as his surviving spouse, and Shirleyt Yap and Jaime Yap, as his surviving legitimate children; (b) Adjudicating to Talina Bianong one-third (1/3) of the whole intestate estate of the late Maning Yap, as her share, pursuant to Art. 996 of the New Civil Code; to Shirley Yap, the other one-third (1/3) as her share and to Jaime Yap the remaining one-third (1/3), also as his share, pursuant to Art. 980 in conjunction with Art. 996 of the new Civil Code. The opposition and claim of the opposition is hereby dismissed without costs. Upon appeal by Nancy Yap and her children, the appellate court reversed and set aside the decision. As stated earlier, the Court of Appeals ruled that the estate of Maning Yap should be equally divided into two equal parts: one-half (1/2) to Talina Bianong and her children and the other half (1/2) to Nancy Yap and her children. The appellate court applied the ruling in Lao and Lao v. Dee Tim (45 Phil. 739). The facts in the cited case are similar to the case at bar in that Yap Siong in his lifetime contracted two marriages; first to Dee Tim on September 14, 1893 in China with whom he had three children and second to Maria Lao on June 24, 1903 with whom he had one child. Moreover, Maria Lao entered into the marriage believing that Yap Siong was not then a married man. Yap Siong died on September 1922 leaving properties which were claimed by the two families. In resolving the issue on how the properties of Yap Siong should be divided, this Court applied the Leyes de Partidas (Law 1; Title 13, Partida 4), to wit: xxx xxx xxx ... [W]here two wome innocently and in good faith are legally united in holy matrimony to the same man, their children and each family will be entitled to one-half of the estate of the husband upon distribution of his estate. That provision of the Leyes de Partidas is a very humane and wise law. It justly protects those who innocently have entered into the solemn relation of marriage and their descendants. The good faith of all the parties will be presumed until the contrary is positive proved. (Articles 69, Civil Code; Las Leyes de Matrimonio, section 96; Gaines v. Hennen, 65 U.S., 553). A woman who is deceied by a man who respresents himself as single and who marries him, she and her children born while the deception lasted, under the Spanish law, are entitled to all the rights of a legitimate wife and children. The common law allowing none of the incidents of a true marriage to follow another marriage entered into during the continuance of a first, was early found to work a great injustice upon the innocent parties to the second marriage, and specially upon the offspring of such second marriage. ... The petitioners now contend that Maning Yap died in 1964 when the New Civil Code had already super ed the old Spanish Civil Code. They state that pursuant to Article 2263 of the New Civil Code, the distribution of the estate of Maning Yap should be in accordance with, the new codal provisions and not the Leyes Partidas, which is an old law no longer applicable, We agree. Article 2263, a transitional provision in the New Civil Code which took effect on August 30, 1950 states: Rights to the inheritance of a person who died, with or without a will, before the effectivity of this Code, shall be governed by the Civil Code of 1889, by other previous laws, and by the Rules of Court. The inheritance of those who, with or without a will die after the beginning of the effectivity of this Code, shall be adjudicated and distributed in accordance with this new body of laws and by the Rules of Court; but the testamentary 320

University of the Cordilleras College of Law First Year C S.Y. 2013 - 2014 provisions shall be carried out insofar as they may be permitted by this Code. Therefore, legitimes, betterments, legacies and bequests shall be respected; however, their amount shall be reduced if in no other manner can every compulsory heir be given his full share according to this Code. (Rule 12a) The Report of the Code Commission explains the rule, to wit: The decisive fact which gives origin to the right of the heirs, devisees and legatees is the death of the decedent. This is the basis of the foregoing rule. No heir, devisee or legatee has any vested right until the moment of such death (Civil Code, Padilla, Volume VII, 1975, p. 712). We have accordingly ruled that the rights to the inheritance of a person who died before the effectivity of the New Civil Code shall be governed by the Civil Code of 1889, by other previous laws and by the Rules of Court (See Vidaurrazaga v. Court of Appeals, 91 Phil. 492; Canales v. Arrogante, 91 Phil. 9; and Morales, et al. v. Yaez, 98 Phil. 677), while the rights to the inheritance of a person who died after the effectivity of the New Civil Code shall be governed by the New Civil Code (Del Prado v. Santos, 18 SCRA 68). There is no dispute that the marriage of Talina Bianong to Maning Yap was valid and that the second marriage contracted by the latter with Nancy Yap was illegal and void pursuant to Act 3613 of the Philippine Legislature, the Marriage Law which was in force when the two marriages were celebrated to wit: SEC. 29. Illegal Marriages. Any marriage subsequently contracted by any person during the lifetime of the first spouse of such person with any person other than such first spouse shall be illegal and void from its performance, unless; (a) The first marriage was annulled or dissolved; (b) The first spouse had been absent for seven consecutive years at the time of the second marriage without the spouse present having news of the absentee being alive, or the absentee being generally considered as dead and believed to be so by the spouse present at the time of contracting such subsequent marriage, the marriage so contracted being valid in either case until declared null and void by a competent court. Bearing this in mind, how must the estate of Maning Yap be distributed? The records show that the real and personal properties under administration in the intestate estate proceedings of Maning Yap were acquired by Talina Bianong and the deceased Maning Yap during their marriage. Hence, these properties, in the absence of any evidence to the contrary are considered conjugal properties of Talina Bianong and Maning Yap (Article 142, New Civil Code). Considering that there was no liquidation of the conjugal partnership of gains during the lifetime of Maning Yap, such liquidation must be carried out in the intestate proceedings of Maning Yap, the deceased spouse as expressly provided in Section 2, Rule 73, Revised Rules of Court (Lapuz v. Eufemio, 43 SCRA 177). Article 142 of the New Civil Code provides: By means of the conjugal partnership of gains the husband and wife place in a common fund the fruits of their separate property and the income from their work or industry, and divide equally, upon the dissolution of the marriage or of the partnership, the net gains or benefits obtained indiscriminately by either spouse during the marriage. and Article 185 thereof states: The net remainder of the conjugal partnership of gains shall be divided equally between the husband and the wife or their respective heirs, unless a different basis of division was agreed upon in the marriage settlements.

321

University of the Cordilleras College of Law First Year C S.Y. 2013 - 2014 Pursuant to these provisions, the net remainder of the conjugal partnership of gains after money claims filed by creditors against the intestate estate of Maning Yap approved by the lower court have been paid by the administratrix should be equally divided between Maning Yap and Talina Bianong as their shares. The one-half share of Maning Yap would then comprise his intestate estate to be distributed among his heirs. (See also Vda. de Delizo v. Delizo, 69 SCRA 216) Under the law of succession in the New Civil Code, Maning Yap's legal heirs are Talina Bianong, her children Shirley Yap and Jaime Yap and the children of Nancy Yap by Maning Yap namely: Maning Yap, Jr., Julia Yap, Jasmin Yap and Samuel Yap. Talina Bianong, the first wife had not lost or relinquished her status as putative heir of her husband. She is entitled to share in Maning Yap's estate upon his death (Gomez v. Lipana, 33 SCRA 615). On the other hand, Nancy Yap, the second wife cannot inherit from Maning Yap because their marriage was void ab initio. (Art. 83, New Civil Code; People v. Mendoza, 95 Phil. 845) However, Nancy Yap's children by Maning Yap have the status of natural children by legal fiction and are considered compulsory heirs of the late Maning Yap. (Articles 89 and 887, New Civil Code). Considering the foregoing, the estate of Maning Yap which is one-half (1/2) pro indiviso of the net remainder of the conjugal partnership of gains of the first marriage (Articles 142 and 185 New Civil Code), the other half being the share of Talina Bianong, should be distributed as follows: a. To the legitimate children, Shirley Yap and Jaime Yap-one-half (1/2) of the resulting net estate to be divided equally between them pursuant to Article 888 of the New Civil Code; b. To the legitimate widow Talina Bianong one-fourth (1/4) of the net estate taken from the free portion or disposable half of the estate pursuant to Article 999 in relation. to Article 897 of the New Civil Code; and c. To the natural children by legal fiction --Maning Yap, Jr., Julia Yap, Jasmin Yap and Samuel Yap-the remaining one-fourth (1/4) of the net estate to be shared equally between them pursuant to the first and third paragraphs of Article 895 in relation to Article 983 of the New Civil Code. WHEREFORE, the instant PETITION is GRANTED. The questioned decision of the Court of Appeals is hereby REVERSED and SET ASIDE. The widow, Talina Bianong shall receive one half (1/2) of the whole intestate estate as her share in the net remainder of the conjugal partnership of gains. The other half, which is the net estate of the late Maning Yap, is distributed and adjudicated as stated above. SO ORDERED.

322

University of the Cordilleras College of Law First Year C S.Y. 2013 - 2014 Case Digest SHIRLEY YAP vs. COURT OF APPEALS, NANCY J. YAP, MANING YAP, JR., JULIA YAP, JASMIN YAP, and SAMUEL YAP G.R. No. L-40003 October 28, 1986 145 SCRA 299 GUTIERREZ, JR., J. FACTS: Maning Yap, married twice: first, to Talina Bianong in 1939 and second, to Nancy Yap on December 11, 1948. Maning Yap and Talina Bianong were married at Bara-as Plantation, Malabang, Lanao del Sur, in accordance with the Muslim rites and practices prescribed by the Islam religion professed by both of them. Out of the marriage, four children were born; two of them died in infancy during the Japanese occupation, while the two others are petitioners Shirley Yap and Jaime Yap. While the first marriage was still subsisting, Maning Yap married Nancy J. Yap on December 11, 1948 in a civil ceremony performed by District Judge Juan Sarenas of the Court of First Instance of Cotabato. Nancy Yap entered into the marriage in the belief that Maning Yap was not a married. They had four children namely respondents Maning Yap, Jr., Julia Yap, Jasmin Yap and Samuel Yap. On February 21, 1964, Maning Yap died. At the time of his death he, therefore, had two families living separately. Talina Bianong Vda. De Yap filed a Special Proceesing No. 1334, seeking the issuance of letters administration for the estate of Maning yap. The respondent Nancy J. Yap claim that she is the legitimate widow and her minor children are their legitimate children. ISSUE: Whether or not Talina Bianong Vda. de Yap and her children are the legal heirs of Maning Yap. RULING: The Court of Appeals ruled that the estate of Maning Yap should be equally divided into two equal parts: one-half (1/2) to Talina Bianong and her children and the other half (1/2) to Nancy Yap and her children. Under the law of succession in the New Civil Code, Maning Yap's legal heirs are Talina Bianong, her children Shirley Yap and Jaime Yap and the children of Nancy Yap by Maning Yap namely: Maning Yap, Jr., Julia Yap, Jasmin Yap and Samuel Yap. Talina Bianong, the first wife had not lost or relinquished her status as putative heir of her husband. She is entitled to share in Maning Yap's estate upon his death (Gomez v. Lipana, 33 SCRA 615). On the other hand, Nancy Yap, the second wife cannot inherit from Maning Yap because their marriage was void ab initio. (Art. 83, New Civil Code; People v. Mendoza, 95 Phil. 845) However, Nancy Yap's children by Maning Yap have the status of natural children by legal fiction and are considered compulsory heirs of the late Maning Yap. (Articles 89 and 887, New Civil Code).

323

University of the Cordilleras College of Law First Year C S.Y. 2013 - 2014 Santos vs Court of Appeals 240 SCRA 20y G.R. No. 112019 January 4, 1995 Full Case LEOUEL SANTOS, petitioner, vs. THE HONORABLE COURT OF APPEALS AND JULIA ROSARIO BEDIA-SANTOS, respondents. VITUG, J.: Concededly a highly, if not indeed the most likely, controversial provision introduced by the Family Code is Article 36 (as amended by E.O. No. 227 dated 17 July 1987), which declares: Art. 36. A marriage contracted by any party who, at the time of the celebration, was psychologically incapacitated to comply with the essential marital obligations of marriage, shall likewise be void even if such incapacity becomes manifest only after its solemnization. The present petition for review on certiorari, at the instance of Leouel Santos ("Leouel"), brings into fore the above provision which is now invoked by him. Undaunted by the decisions of the court a quo and the Court of Appeal, Leouel persists in beseeching its application in his attempt to have his marriage with herein private respondent, Julia Rosario Bedia-Santos ("Julia"), declared a nullity. It was in Iloilo City where Leouel, who then held the rank of First Lieutenant in the Philippine Army, first met Julia. The meeting later proved to be an eventful day for Leouel and Julia. On 20 September 1986, the two exchanged vows before Municipal Trial Court Judge Cornelio G. Lazaro of Iloilo City, followed, shortly thereafter, by a church wedding. Leouel and Julia lived with the latter's parents at the J. Bedia Compound, La Paz, Iloilo City. On 18 July 1987, Julia gave birth to a baby boy, and he was christened Leouel Santos, Jr. The ecstasy, however, did not last long. It was bound to happen, Leouel averred, because of the frequent interference by Julia's parents into the young spouses family affairs. Occasionally, the couple would also start a "quarrel" over a number of other things, like when and where the couple should start living independently from Julia's parents or whenever Julia would express resentment on Leouel's spending a few days with his own parents. On 18 May 1988, Julia finally left for the United Sates of America to work as a nurse despite Leouel's pleas to so dissuade her. Seven months after her departure, or on 01 January 1989, Julia called up Leouel for the first time by long distance telephone. She promised to return home upon the expiration of her contract in July 1989. She never did. When Leouel got a chance to visit the United States, where he underwent a training program under the auspices of the Armed Forces of the Philippines from 01 April up to 25 August 1990, he desperately tried to locate, or to somehow get in touch with, Julia but all his efforts were of no avail. Having failed to get Julia to somehow come home, Leouel filed with the regional trial Court of Negros Oriental, Branch 30, a complaint for "Voiding of marriage Under Article 36 of the Family Code" (docketed, Civil Case No. 9814). Summons was served by publication in a newspaper of general circulation in Negros Oriental. On 31 May 1991, respondent Julia, in her answer (through counsel), opposed the complaint and denied its allegations, claiming, in main, that it was the petitioner who had, in fact, been irresponsible and incompetent. A possible collusion between the parties to obtain a decree of nullity of their marriage was ruled out by the Office of the Provincial Prosecutor (in its report to the court). On 25 October 1991, after pre-trial conferences had repeatedly been set, albeit unsuccessfully, by the court, Julia ultimately filed a manifestation, stating that she would neither appear nor submit evidence. On 06 November 1991, the court a quo finally dismissed the complaint for lack of merit. 324

University of the Cordilleras College of Law First Year C S.Y. 2013 - 2014 Leouel appealed to the Court of Appeal. The latter affirmed the decision of the trial court. The petition should be denied not only because of its non-compliance with Circular 28-91, which requires a certification of non-shopping, but also for its lack of merit. Leouel argues that the failure of Julia to return home, or at the very least to communicate with him, for more than five years are circumstances that clearly show her being psychologically incapacitated to enter into married life. In his own words, Leouel asserts: . . . (T)here is no leave, there is no affection for (him) because respondent Julia Rosario Bedia-Santos failed all these years to communicate with the petitioner. A wife who does not care to inform her husband about her whereabouts for a period of five years, more or less, is psychologically incapacitated. The family Code did not define the term "psychological incapacity." The deliberations during the sessions of the Family Code Revision Committee, which has drafted the Code, can, however, provide an insight on the import of the provision. Art. 35. The following marriages shall be void from the beginning: xxx xxx xxx Art. 36. . . . (7) Those marriages contracted by any party who, at the time of the celebration, was wanting in the sufficient use of reason or judgment to understand the essential nature of marriage or was psychologically or mentally incapacitated to discharge the essential marital obligations, even if such lack of incapacity is made manifest after the celebration. On subparagraph (7), which as lifted from the Canon Law, Justice (Jose B.L.) Reyes suggested that they say "wanting in sufficient use," but Justice (Eduardo) Caguioa preferred to say "wanting in the sufficient use." On the other hand, Justice Reyes proposed that they say "wanting in sufficient reason." Justice Caguioa, however, pointed out that the idea is that one is not lacking in judgment but that he is lacking in the exercise of judgment. He added that lack of judgment would make the marriage voidable. Judge (Alicia Sempio-) Diy remarked that lack of judgment is more serious than insufficient use of judgment and yet the latter would make the marriage null and void and the former only voidable. Justice Caguioa suggested that subparagraph (7) be modified to read: "That contracted by any party who, at the time of the celebration, was psychologically incapacitated to discharge the essential marital obligations, even if such lack of incapacity is made manifest after the celebration." Justice Caguioa explained that the phrase "was wanting in sufficient use of reason of judgment to understand the essential nature of marriage" refers to defects in the mental faculties vitiating consent, which is not the idea in subparagraph (7), but lack of appreciation of one's marital obligations. Judge Diy raised the question: Since "insanity" is also a psychological or mental incapacity, why is "insanity" only a ground for annulment and not for declaration or nullity? In reply, Justice Caguioa explained that in insanity, there is the appearance of consent, which is the reason why it is a ground for voidable marriages, while subparagraph (7) does not refer to consent but to the very essence of marital obligations. Prof. (Araceli) Baviera suggested that, in subparagraph (7), the word "mentally" be deleted, with which Justice Caguioa concurred. Judge Diy, however, prefers to retain the word "mentally."

325

University of the Cordilleras College of Law First Year C S.Y. 2013 - 2014 Justice Caguioa remarked that subparagraph (7) refers to psychological impotence. Justice (Ricardo) Puno stated that sometimes a person may be psychologically impotent with one but not with another. Justice (Leonor Ines-) Luciano said that it is called selective impotency. Dean (Fortunato) Gupit stated that the confusion lies in the fact that in inserting the Canon Law annulment in the Family Code, the Committee used a language which describes a ground for voidable marriages under the Civil Code. Justice Caguioa added that in Canon Law, there are voidable marriages under the Canon Law, there are no voidable marriages Dean Gupit said that this is precisely the reason why they should make a distinction. Justice Puno remarked that in Canon Law, the defects in marriage cannot be cured. Justice Reyes pointed out that the problem is: Why is "insanity" a ground for void ab initio marriages? In reply, Justice Caguioa explained that insanity is curable and there are lucid intervals, while psychological incapacity is not. On another point, Justice Puno suggested that the phrase "even if such lack or incapacity is made manifest" be modified to read "even if such lack or incapacity becomes manifest." Justice Reyes remarked that in insanity, at the time of the marriage, it is not apparent. Justice Caguioa stated that there are two interpretations of the phrase "psychological or mentally incapacitated" in the first one, there is vitiation of consent because one does not know all the consequences of the marriages, and if he had known these completely, he might not have consented to the marriage. xxx xxx xxx Prof. Bautista stated that he is in favor of making psychological incapacity a ground for voidable marriages since otherwise it will encourage one who really understood the consequences of marriage to claim that he did not and to make excuses for invalidating the marriage by acting as if he did not understand the obligations of marriage. Dean Gupit added that it is a loose way of providing for divorce. xxx xxx xxx Justice Caguioa explained that his point is that in the case of incapacity by reason of defects in the mental faculties, which is less than insanity, there is a defect in consent and, therefore, it is clear that it should be a ground for voidable marriage because there is the appearance of consent and it is capable of convalidation for the simple reason that there are lucid intervals and there are cases when the insanity is curable. He emphasized that psychological incapacity does not refer to mental faculties and has nothing to do with consent; it refers to obligations attendant to marriage. xxx xxx xxx On psychological incapacity, Prof. (Flerida Ruth P.) Romero inquired if they do not consider it as going to the very essence of consent. She asked if they are really removing it from consent. In reply, Justice Caguioa explained that, ultimately, consent in general is effected but he stressed that his point is that it is not principally a vitiation of consent since there is a valid consent. He objected to the lumping together of the validity of the marriage celebration and the obligations attendant to marriage, which are completely different from each other, because they require a different capacity, which is eighteen years of age, for marriage but in contract, it is different. Justice Puno, however, felt that psychological incapacity is still a kind of vice of consent and that it should not be classified as a voidable marriage which is incapable of convalidation; it should be convalidated but there should be no prescription. In other words, as long as the defect has not been cured, there is always a right to annul the marriage and if the defect has been 326

University of the Cordilleras College of Law First Year C S.Y. 2013 - 2014 really cured, it should be a defense in the action for annulment so that when the action for annulment is instituted, the issue can be raised that actually, although one might have been psychologically incapacitated, at the time the action is brought, it is no longer true that he has no concept of the consequence of marriage. Prof. (Esteban) Bautista raised the question: Will not cohabitation be a defense? In response, Justice Puno stated that even the bearing of children and cohabitation should not be a sign that psychological incapacity has been cured. Prof. Romero opined that psychological incapacity is still insanity of a lesser degree. Justice Luciano suggested that they invite a psychiatrist, who is the expert on this matter. Justice Caguioa, however, reiterated that psychological incapacity is not a defect in the mind but in the understanding of the consequences of marriage, and therefore, a psychiatrist will not be a help. Prof. Bautista stated that, in the same manner that there is a lucid interval in insanity, there are also momentary periods when there is an understanding of the consequences of marriage. Justice Reyes and Dean Gupit remarked that the ground of psychological incapacity will not apply if the marriage was contracted at the time when there is understanding of the consequences of marriage. xxx xxx xxx Judge Diy proposed that they include physical incapacity to copulate among the grounds for void marriages. Justice Reyes commented that in some instances the impotence that in some instances the impotence is only temporary and only with respect to a particular person. Judge Diy stated that they can specify that it is incurable. Justice Caguioa remarked that the term "incurable" has a different meaning in law and in medicine. Judge Diy stated that "psychological incapacity" can also be cured. Justice Caguioa, however, pointed out that "psychological incapacity" is incurable. Justice Puno observed that under the present draft provision, it is enough to show that at the time of the celebration of the marriage, one was psychologically incapacitated so that later on if already he can comply with the essential marital obligations, the marriage is still void ab initio. Justice Caguioa explained that since in divorce, the psychological incapacity may occur after the marriage, in void marriages, it has to be at the time of the celebration of marriage. He, however, stressed that the idea in the provision is that at the time of the celebration of the marriage, one is psychologically incapacitated to comply with the essential marital obligations, which incapacity continues and later becomes manifest. Justice Puno and Judge Diy, however, pointed out that it is possible that after the marriage, one's psychological incapacity become manifest but later on he is cured. Justice Reyes and Justice Caguioa opined that the remedy in this case is to allow him to remarry. xxx xxx xxx Justice Puno formulated the next Article as follows: Art. 37. A marriage contracted by any party who, at the time of celebration, was psychologically incapacitated, to comply with essential obligations of marriage shall likewise be void from beginning even if such incapacity becomes manifest after solemnization. the the the its

Justice Caguioa suggested that "even if" be substituted with "although." On the other hand, Prof. Bautista proposed that the clause "although such incapacity becomes manifest after its solemnization" be deleted since it may encourage one to create the manifestation of psychological incapacity. Justice Caguioa pointed out that, as in other provisions, they cannot argue on the basis of abuse. 327

University of the Cordilleras College of Law First Year C S.Y. 2013 - 2014 Judge Diy suggested that they also include mental and physical incapacities, which are lesser in degree than psychological incapacity. Justice Caguioa explained that mental and physical incapacities are vices of consent while psychological incapacity is not a species of vice or consent. Dean Gupit read what Bishop Cruz said on the matter in the minutes of their February 9, 1984 meeting: "On the third ground, Bishop Cruz indicated that the phrase "psychological or mental impotence" is an invention of some churchmen who are moralists but not canonists, that is why it is considered a weak phrase. He said that the Code of Canon Law would rather express it as "psychological or mental incapacity to discharge . . ." Justice Caguioa remarked that they deleted the word "mental" precisely to distinguish it from vice of consent. He explained that "psychological incapacity" refers to lack of understanding of the essential obligations of marriage. Justice Puno reminded the members that, at the last meeting, they have decided not to go into the classification of "psychological incapacity" because there was a lot of debate on it and that this is precisely the reason why they classified it as a special case. At this point, Justice Puno, remarked that, since there having been annulments of marriages arising from psychological incapacity, Civil Law should not reconcile with Canon Law because it is a new ground even under Canon Law. Prof. Romero raised the question: With this common provision in Civil Law and in Canon Law, are they going to have a provision in the Family Code to the effect that marriages annulled or declared void by the church on the ground of psychological incapacity is automatically annulled in Civil Law? The other members replied negatively. Justice Puno and Prof. Romero inquired if Article 37 should be retroactive or prospective in application. Justice Diy opined that she was for its retroactivity because it is their answer to the problem of church annulments of marriages, which are still valid under the Civil Law. On the other hand, Justice Reyes and Justice Puno were concerned about the avalanche of cases. Dean Gupit suggested that they put the issue to a vote, which the Committee approved. The members voted as follows: (1) Justice Reyes, Justice Puno and Prof. Romero were for prospectivity. (2) Justice Caguioa, Judge Diy, Dean Gupit, Prof. Bautista and Director Eufemio were for retroactivity. (3) Prof. Baviera abstained. Justice Caguioa suggested that they put in the prescriptive period of ten years within which the action for declaration of nullity of the marriage should be filed in court. The Committee approved the suggestion. It could well be that, in sum, the Family Code Revision Committee in ultimately deciding to adopt the provision with less specificity than expected, has in fact, so designed the law as to allow some resiliency in its application. Mme. Justice Alicia V. Sempio-Diy, a member of the Code Committee, has been quoted by Mr. Justice Josue N. Bellosillo in Salita vs. Hon. Magtolis (G.R. No. 106429, 13 June 1994); thus:

328

University of the Cordilleras College of Law First Year C S.Y. 2013 - 2014 The Committee did not give any examples of psychological incapacity for fear that the giving of examples would limit the applicability of the provision under the principle of ejusdem generis. Rather, the Committee would like the judge to interpret the provision on a case-to-case basis, guided by experience, the findings of experts and researchers in psychological disciplines, and by decisions of church tribunals which, although not binding on the civil courts, may be given persuasive effect since the provision was taken from Canon Law. A part of the provision is similar to Canon 1095 of the New Code of Canon Law, which reads: Canon 1095. They are incapable of contracting marriage: 1. who lack sufficient use of reason; 2. who suffer from a grave defect of discretion of judgment concerning essentila matrimonial rights and duties, to be given and accepted mutually; 3. who for causes of psychological nature are unable to assume the essential obligations of marriage. (Emphasis supplied.) Accordingly, although neither decisive nor even perhaps all that persuasive for having no juridical or secular effect, the jurisprudence under Canon Law prevailing at the time of the code's enactment, nevertheless, cannot be dismissed as impertinent for its value as an aid, at least, to the interpretation or construction of the codal provision. One author, Ladislas Orsy, S.J., in his treaties, giving an account on how the third paragraph of Canon 1095 has been framed, states: The history of the drafting of this canon does not leave any doubt that the legislator intended, indeed, to broaden the rule. A strict and narrow norm was proposed first: Those who cannot assume the essential obligations of marriage because of a grave psycho-sexual anomaly (ob gravem anomaliam psychosexualem) are unable to contract marriage (cf. SCH/1975, canon 297, a new canon, novus); then a broader one followed: . . . because of a grave psychological anomaly (ob gravem anomaliam psychicam) . . . (cf. SCH/1980, canon 1049); then the same wording was retained in the text submitted to the pope ( cf. SCH/1982, canon 1095, 3); finally, a new version was promulgated: because of causes of a psychological nature (ob causas naturae psychiae). So the progress was from psycho-sexual to psychological anomaly, then the term anomaly was altogether eliminated. it would be, however, incorrect to draw the conclusion that the cause of the incapacity need not be some kind of psychological disorder; after all, normal and healthy person should be able to assume the ordinary obligations of marriage. Fr. Orsy concedes that the term "psychological incapacity" defies any precise definition since psychological causes can be of an infinite variety. In a book, entitled "Canons and Commentaries on Marriage," written by Ignatius Gramunt, Javier Hervada and LeRoy Wauck, the following explanation appears:

329

University of the Cordilleras College of Law First Year C S.Y. 2013 - 2014 This incapacity consists of the following: (a) a true inability to commit oneself to the essentials of marriage. Some psychosexual disorders and other disorders of personality can be the psychic cause of this defect, which is here described in legal terms. This particular type of incapacity consists of a real inability to render what is due by the contract. This could be compared to the incapacity of a farmer to enter a binding contract to deliver the crops which he cannot possibly reap; (b) this inability to commit oneself must refer to the essential obligations of marriage: the conjugal act, the community of life and love, the rendering of mutual help, the procreation and education of offspring; (c) the inability must be tantamount to a psychological abnormality. The mere difficulty of assuming these obligations, which could be overcome by normal effort, obviously does not constitute incapacity. The canon contemplates a true psychological disorder which incapacitates a person from giving what is due (cf. John Paul II, Address to R. Rota, Feb. 5, 1987). However, if the marriage is to be declared invalid under this incapacity, it must be proved not only that the person is afflicted by a psychological defect, but that the defect did in fact deprive the person, at the moment of giving consent, of the ability to assume the essential duties of marriage and consequently of the possibility of being bound by these duties. Justice Sempio-Diy cites with approval the work of Dr. Gerardo Veloso, a former Presiding Judge of the Metropolitan Marriage Tribunal of the Catholic Archdiocese of Manila (Branch 1), who opines that psychological incapacity must be characterized by (a) gravity, (b) juridical antecedence, and (c) incurability. The incapacity must be grave or serious such that the party would be incapable of carrying out the ordinary duties required in marriage; it must be rooted in the history of the party antedating the marriage, although the overt manifestations may emerge only after the marriage; and it must be incurable or, even if it were otherwise, the cure would be beyond the means of the party involved. It should be obvious, looking at all the foregoing disquisitions, including, and most importantly, the deliberations of the Family Code Revision Committee itself, that the use of the phrase "psychological incapacity" under Article 36 of the Code has not been meant to comprehend all such possible cases of psychoses as, likewise mentioned by some ecclesiastical authorities, extremely low intelligence, immaturity, and like circumstances (cited in Fr. Artemio Baluma's "Void and Voidable Marriages in the Family Code and their Parallels in Canon Law," quoting from the Diagnostic Statistical Manual of Mental Disorder by the American Psychiatric Association; Edward Hudson's "Handbook II for Marriage Nullity Cases"). Article 36 of the Family Code cannot be taken and construed independently of, but must stand in conjunction with, existing precepts in our law on marriage. Thus correlated, "psychological incapacity" should refer to no less than a mental (not physical) incapacity that causes a party to be truly incognitive of the basic marital covenants that concomitantly must be assumed and discharged by the parties to the marriage which, as so expressed by Article 68 of the Family Code, include their mutual obligations to live together, observe love, respect and fidelity and render help and support. There is hardly any doubt that the intendment of the law has been to confine the meaning of "psychological incapacity" to the most serious cases of personality disorders clearly demonstrative of an utter intensitivity or inability to give meaning and significance to the marriage. This pschologic condition must exist at the time the marriage is celebrated. The law does not evidently envision, upon the other hand, an inability of the spouse to have sexual relations with the other. This conclusion is implicit under Article 54 of the Family Code which considers children conceived prior to the judicial declaration of nullity of the void marriage to be "legitimate." The other forms of psychoses, if existing at the inception of marriage, like the state of a party being of unsound mind or concealment of drug addiction, habitual alcoholism, homosexuality or lesbianism, merely renders the marriage contract voidable pursuant to Article 46, Family Code. If drug addiction, habitual alcholism, lesbianism or homosexuality should occur only during the marriage, they become mere grounds for legal separation under Article 55 of the Family Code. These provisions of the Code, however, do not necessarily preclude the possibility of these various circumstances being themselves, depending on the degree and severity of the disorder, indicia of psychological incapacity. Until further statutory and jurisprudential parameters are established, every circumstance that may have some bearing on the degree, extent, and other conditions of that incapacity must, in every case, be carefully examined and evaluated so that no precipitate and indiscriminate nullity is peremptorily decreed. The well-considered opinions of psychiatrists, psychologists, and persons with expertise in psychological disciplines might be helpful or even desirable.

330

University of the Cordilleras College of Law First Year C S.Y. 2013 - 2014 Marriage is not an adventure but a lifetime commitment. We should continue to be reminded that innate in our society, then enshrined in our Civil Code, and even now still indelible in Article 1 of the Family Code, is that Art. 1. Marriage is a special contract of permanent union between a man a woman entered into in accordance with law for the establishment of conjugal and family life. It is the foundation of the family and an inviolable social institution whose nature, consequences, and incidents are governed by law and not subject to stipulation, except that marriage settlements may fix the property relations during the marriage within the limits provided by this Code. (Emphasis supplied.) Our Constitution is no less emphatic: Sec. 1. The State recognizes the Filipino family as the foundation of the nation. Accordingly, it shall strengthen its solidarity and actively promote its total development. Sec. 2. Marriage, as an inviolable social institution, is the foundation of the family and shall be protected by the State. (Article XV, 1987 Constitution). The above provisions express so well and so distinctly the basic nucleus of our laws on marriage and the family, and they are doubt the tenets we still hold on to. The factual settings in the case at bench, in no measure at all, can come close to the standards required to decree a nullity of marriage. Undeniably and understandably, Leouel stands aggrieved, even desperate, in his present situation. Regrettably, neither law nor society itself can always provide all the specific answers to every individual problem. WHEREFORE, the petition is DENIED. SO ORDERED. Separate Opinions

PADILLA, J., dissenting: It is difficult to dissent from a well-written and studied opinion as Mr. Justice Vitug's ponencia. But, after an extended reflection on the facts of this case, I cannot see my way clear into holding, as the majority do, that there is no ground for the declaration of nullity of the marriage between petitioner and private respondent. To my mind, it is clear that private respondent has been shown to be psychologically incapacitated to comply with at least one essential marital obligation, i.e. that of living and cohabiting with her husband, herein petitioner. On the other hand, it has not been shown that petitioner does not deserve to live and cohabit with his wife, herein private respondent. There appears to be no disagreement that the term "psychological incapacity" defies precision in definition. But, as used in Article 36 of the Family Code as a ground for the declaration of nullity of a marriage, the intent of the framers of the Code is evidently to expand and liberalize the grounds for nullifying a marriage, as well pointed out by Madam Justice Flerida Ruth P. Romero in her separate opinion in this case. While it is true that the board term "psychological incapacity" can open the doors to abuse by couples who may wish to have an easy way out of their marriage, there are, however, enough safeguards against this contingency, among which, is the intervention by the State, through the public prosecutor, to guard against collusion between the parties and/or fabrication of evidence.

331

University of the Cordilleras College of Law First Year C S.Y. 2013 - 2014 In their case at bench, it has been abundantly established that private respondent Julia Rosario BediaSantos exhibits specific behavior which, to my mind, shows that she is psychologically incapacitated to fulfill her essential marital obligations, to writ: a. It took her seven (7) months after she left for the United States to call up her husband. b. Julia promised to return home after her job contract expired in July 1989, but she never did and neither is there any showing that she informed her husband (herein petitioner) of her whereabouts in the U.S.A. c. When petitioner went to the United States on a mission for the Philippine Army, he exerted efforts to "touch base" with Julia; there were no similar efforts on the part of Julia; there were no similar efforts on the part of Julia to do the same. d. When petitioner filed this suit, more than five (5) years had elapsed, without Julia indicating her plans to rejoin the petitioner or her whereabouts. e. When petitioner filed this case in the trial court, Julia, in her answer, claimed that it is the former who has been irresponsible and incompetent. f. During the trial, Julia waived her right to appear and submit evidence. A spouse's obligation to live and cohabit with his/her partner in marriage is a basic ground rule in marriage, unless there are overpowering compelling reasons such as, for instance, an incurable contagious disease on the part of a spouse or cruelty of one partner, bordering on insanity. There may also be instances when, for economic and practical reasons, husband and wife have to live separately, but the marital bond between the spouses always remains. Mutual love and respect for each other would, in such cases, compel the absent spouse to at least have regular contracts with the other to inform the latter of his/her condition and whereabouts. In the present case, it is apparent that private respondent Julia Rosario Bedia-Santos has no intention of cohabiting with petitioner, her husband, or maintaining contact with him. In fact, her acts eloquently show that she does not want her husband to know of her whereabouts and neither has she any intention of living and cohabiting with him. To me there appears to be, on the part of private respondent, an unmistakeable indication of psychological incapacity to comply with her essential marital obligations, although these indications were made manifest after the celebration of the marriage. It would be a great injustice, I believe, to petitioner for this Court to give a much too restrictive interpretation of the law and compel the petitioner to continue to be married to a wife who for purposes of fulfilling her marital duties has, for all practical purposes, ceased to exist. Besides, there are public policy considerations involved in the ruling the Court makes today. Is it not, in effect directly or indirectly, facilitating the transformation of petitioner into a "habitual tryster" or one forced to maintain illicit relations with another woman or women with emerging problems of illegitimate children, simply because he is denied by private respondent, his wife, the companionship and conjugal love which he has sought from her and to which he is legally entitled? I do not go as far as to suggest that Art. 36 of the Family Code is a sanction for absolute divorce but I submit that we should not constrict it to non-recognition of its evident purpose and thus deny to one like petitioner, an opportunity to turn a new leaf in his life by declaring his marriage a nullity by reason of his wife's psychological incapacity to perform an essential marital obligation. I therefore vote to GRANT the petition and to DECLARE the marriage between petitioner Leouel Santos and private respondent Julia Rosario Bedia-Santos VOID on the basis of Article 36 of the Family Code. ROMERO, J., concurring:

332

University of the Cordilleras College of Law First Year C S.Y. 2013 - 2014 I agree under the circumstances of the case, petitioner is not entitled to have his marriage declared a nullity on the ground of psychological incapacity of private respondent. However, as a member of both the Family Law Revision Committee of the Integrated Bar of the Philippines and the Civil Code Revision Committee of the UP Law Center, I wish to add some observations. The letter dated April 15, 1985 of then Judge Alicia V. Sempio-Diy written in behalf of the Family Law and Civil Code Revision Committee to then Assemblywoman Mercedes Cojuangco-Teodoro traced the background of the inclusion of the present Article 36 in the Family Code. During its early meetings, the Family Law Committee had thought of including a chapter on absolute divorce in the draft of a new Family Code (Book I of the Civil Code) that it had been tasked by the IBP and the UP Law Center to prepare. In fact, some members of the Committee were in favor of a no-fault divorce between the spouses after a number of years of separation, legal or de-facto. Justice J.B.L. Reyes was then requested to prepare a proposal for an action for dissolution of marriage and the effects thereof based on two grounds: (a) five continuous years of separation between the spouses, with or without a judicial decree of legal separation, and (b) whenever a married person would have obtained a decree of absolute divorce in another country. Actually, such a proposal is one for absolute divorce but called by another name. Later, even the Civil Code Revision Committee took time to discuss the proposal of Justice Reyes on this matter. Subsequently, however, when the Civil Code Revision Committee and Family Law Committee started holding joint meetings on the preparation of the draft of the New Family Code, they agreed and formulated the definition of marriage as "a special contract of permanent partnership between a man and a woman entered into in accordance with law for the establishment of conjugal and family life. It is an inviolable social institution whose nature, consequences, and incidents are governed by law and not subject to stipulation, except that marriage settlements may fix the property relations during the marriage within the limits provided by law." With the above definition, and considering the Christian traditional concept of marriage of the Filipino people as a permanent, inviolable, indissoluble social institution upon which the family and society are founded, and also realizing the strong opposition that any provision on absolute divorce would encounter from the Catholic Church and the Catholic sector of our citizenry to whom the great majority of our people belong, the two Committees in their joint meetings did not pursue the idea of absolute divorce and instead opted for an action for judicial declaration of invalidity of marriage based on grounds available in the Canon Law. It was thought that such an action would not only be an acceptable alternative to divorce but would also solve the nagging problem of church annulments of marriages on grounds not recognized by the civil law of the State. Justice Reyes was thus requested to again prepare a draft of provisions on such action for celebration of invalidity of marriage. Still later, to avoid the overlapping of provisions on void marriages as found in the present Civil Code and those proposed by Justice Reyes on judicial declaration of invalidity of marriage on grounds similar to the Canon Law, the two Committees now working as a Joint Committee in the preparation of a New Family Code decided to consolidate the present provisions on void marriages with the proposals of Justice Reyes. The result was the inclusion of an additional kind of void marriage in the enumeration of void marriages in the present Civil Code, to wit: "(7) Those marriages contracted by any party who, at the time of the celebration, was wanting in the sufficient use of reason or judgment to understand the essential nature of marriage or was psychologically or mentally incapacitated to discharge the essential marital obligations, even if such lack of incapacity is made manifest after the celebration." as well as the following implementing provisions:

333

University of the Cordilleras College of Law First Year C S.Y. 2013 - 2014 "Art. 32. The absolute nullity of a marriage may be invoked or pleaded only on the basis of a final judgment declaring the marriage void, without prejudice to the provision of Article 34." "Art. 33. The action or defense for the declaration of the absolute nullity of a marriage shall not prescribe." xxx xxx xxx It is believed that many hopelessly broken marriages in our country today may already dissolved or annulled on the grounds proposed by the Joint Committee on declaration of nullity as well as annulment of marriages, thus rendering an absolute divorce law unnecessary. In fact, during a conference with Father Gerald Healy of the Ateneo University as well as another meeting with Archbishop Oscar Cruz of the Archdiocese of Pampanga, the Joint Committee was informed that since Vatican II, the Catholic Church has been declaring marriages null and void on the ground of "lack of due discretion" for causes that, in other jurisdictions, would be clear grounds for divorce, like teen-age or premature marriages; marriage to a man who, because of some personality disorder or disturbance, cannot support a family; the foolish or ridiculous choice of a spouse by an otherwise perfectly normal person; marriage to a woman who refuses to cohabit with her husband or who refuses to have children. Bishop Cruz also informed the Committee that they have found out in tribunal work that a lot of machismo among husbands are manifestations of their sociopathic personality anomaly, like inflicting physical violence upon their wives, constitutional indolence or laziness, drug dependence or addiction, and psychological anomaly. . . . (Emphasis supplied) Clearly, by incorporating what is now Article 36 into the Family Code, the Revision Committee referred to above intended to add another ground to those already listed in the Civil Code as grounds for nullifying a marriage, thus expanding or liberalizing the same. Inherent in the inclusion of the provision on psychological incapacity was the understanding that every petition for declaration of nullity based on it should be treated on a case-to-case basis; hence, the absence of a definition and an enumeration of what constitutes psychological incapacity. Moreover, the Committee feared that the giving of examples would limit the applicability of the provision under the principle ofejusdem generis. But the law requires that the same be existing at the time of marriage although it be manifested later. Admittedly, the provision on psychological incapacity, just like any other provision of law, is open to abuse. To prevent this, "the court shall take order the prosecuting attorney or fiscal assigned to it to appear on behalf of the State to take steps to prevent collusion between the parties and to take care that evidence is not fabricated or suppressed." Moreover, the judge, in interpreting the provision on a case-tocase basis, must be guided by "experience, the findings of experts and researchers in psychological disciplines, and by decisions of church tribunals which, although not binding on the civil courts, may be given persuasive effect since the provisions was taken from Canon Law." The constitutional and statutory provisions on the family will remain the lodestar which our society will hope to achieve ultimately. Therefore, the inclusion of Article 36 is not to be taken as an abandonment of the ideal which we all cherish. If at all, it is a recognition of the reality that some marriages, by reason of the incapacity of one of the contracting parties, fall short of this ideal; thus, the parties are constrained to find a way of putting an end to their union through some legally-accepted means. Any criticism directed at the way that judges have interpreted the provision since its enactment as to render it easier for unhappily-married couples to separate is addressed, not to the wisdom of the lawmakers but to the manner by which some members of the Bench have implemented the provision. These are not interchangeable, each being separate and distinct from the other.

Separate Opinions PADILLA, J., dissenting:

334

University of the Cordilleras College of Law First Year C S.Y. 2013 - 2014 It is difficult to dissent from a well-written and studied opinion as Mr. Justice Vitug's ponencia. But, after an extended reflection on the facts of this case, I cannot see my way clear into holding, as the majority do, that there is no ground for the declaration of nullity of the marriage between petitioner and private respondent. To my mind, it is clear that private respondent has been shown to be psychologically incapacitated to comply with at least one essential marital obligation, i.e. that of living and cohabiting with her husband, herein petitioner. On the other hand, it has not been shown that petitioner does not deserve to live and cohabit with his wife, herein private respondent. There appears to be no disagreement that the term "psychological incapacity" defies precision in definition. But, as used in Article 36 of the Family Code as a ground for the declaration of nullity of a marriage, the intent of the framers of the Code is evidently to expand and liberalize the grounds for nullifying a marriage, as well pointed out by Madam Justice Flerida Ruth P. Romero in her separate opinion in this case. While it is true that the board term "psychological incapacity" can open the doors to abuse by couples who may wish to have an easy way out of their marriage, there are, however, enough safeguards against this contingency, among which, is the intervention by the State, through the public prosecutor, to guard against collusion between the parties and/or fabrication of evidence. In their case at bench, it has been abundantly established that private respondent Julia Rosario BediaSantos exhibits specific behavior which, to my mind, shows that she is psychologically incapacitated to fulfill her essential marital obligations, to writ: a. It took her seven (7) months after she left for the United States to call up her husband. b. Julia promised to return home after her job contract expired in July 1989, but she never did and neither is there any showing that she informed her husband (herein petitioner) of her whereabouts in the U.S.A. c. When petitioner went to the United States on a mission for the Philippine Army, he exerted efforts to "touch base" with Julia; there were no similar efforts on the part of Julia; there were no similar efforts on the part of Julia to do the same. d. When petitioner filed this suit, more than five (5) years had elapsed, without Julia indicating her plans to rejoin the petitioner or her whereabouts. e. When petitioner filed this case in the trial court, Julia, in her answer, claimed that it is the former who has been irresponsible and incompetent. f. During the trial, Julia waived her right to appear and submit evidence. A spouse's obligation to live and cohabit with his/her partner in marriage is a basic ground rule in marriage, unless there are overpowering compelling reasons such as, for instance, an incurable contagious disease on the part of a spouse or cruelty of one partner, bordering on insanity. There may also be instances when, for economic and practical reasons, husband and wife have to live separately, but the marital bond between the spouses always remains. Mutual love and respect for each other would, in such cases, compel the absent spouse to at least have regular contracts with the other to inform the latter of his/her condition and whereabouts. In the present case, it is apparent that private respondent Julia Rosario Bedia-Santos has no intention of cohabiting with petitioner, her husband, or maintaining contact with him. In fact, her acts eloquently show that she does not want her husband to know of her whereabouts and neither has she any intention of living and cohabiting with him. To me there appears to be, on the part of private respondent, an unmistakeable indication of psychological incapacity to comply with her essential marital obligations, although these indications were made manifest after the celebration of the marriage.

335

University of the Cordilleras College of Law First Year C S.Y. 2013 - 2014 It would be a great injustice, I believe, to petitioner for this Court to give a much too restrictive interpretation of the law and compel the petitioner to continue to be married to a wife who for purposes of fulfilling her marital duties has, for all practical purposes, ceased to exist. Besides, there are public policy considerations involved in the ruling the Court makes today. Is it not, in effect directly or indirectly, facilitating the transformation of petitioner into a "habitual tryster" or one forced to maintain illicit relations with another woman or women with emerging problems of illegitimate children, simply because he is denied by private respondent, his wife, the companionship and conjugal love which he has sought from her and to which he is legally entitled? I do not go as far as to suggest that Art. 36 of the Family Code is a sanction for absolute divorce but I submit that we should not constrict it to non-recognition of its evident purpose and thus deny to one like petitioner, an opportunity to turn a new leaf in his life by declaring his marriage a nullity by reason of his wife's psychological incapacity to perform an essential marital obligation. I therefore vote to GRANT the petition and to DECLARE the marriage between petitioner Leouel Santos and private respondent Julia Rosario Bedia-Santos VOID on the basis of Article 36 of the Family Code. ROMERO, J., concurring: I agree under the circumstances of the case, petitioner is not entitled to have his marriage declared a nullity on the ground of psychological incapacity of private respondent. However, as a member of both the Family Law Revision Committee of the Integrated Bar of the Philippines and the Civil Code Revision Committee of the UP Law Center, I wish to add some observations. The letter dated April 15, 1985 of then Judge Alicia V. Sempio-Diy written in behalf of the Family Law and Civil Code Revision Committee to then Assemblywoman Mercedes Cojuangco-Teodoro traced the background of the inclusion of the present Article 36 in the Family Code. During its early meetings, the Family Law Committee had thought of including a chapter on absolute divorce in the draft of a new Family Code (Book I of the Civil Code) that it had been tasked by the IBP and the UP Law Center to prepare. In fact, some members of the Committee were in favor of a no-fault divorce between the spouses after a number of years of separation, legal or de-facto. Justice J.B.L. Reyes was then requested to prepare a proposal for an action for dissolution of marriage and the effects thereof based on two grounds: (a) five continuous years of separation between the spouses, with or without a judicial decree of legal separation, and (b) whenever a married person would have obtained a decree of absolute divorce in another country. Actually, such a proposal is one for absolute divorce but called by another name. Later, even the Civil Code Revision Committee took time to discuss the proposal of Justice Reyes on this matter. Subsequently, however, when the Civil Code Revision Committee and Family Law Committee started holding joint meetings on the preparation of the draft of the New Family Code, they agreed and formulated the definition of marriage as "a special contract of permanent partnership between a man and a woman entered into in accordance with law for the establishment of conjugal and family life. It is an inviolable social institution whose nature, consequences, and incidents are governed by law and not subject to stipulation, except that marriage settlements may fix the property relations during the marriage within the limits provided by law." With the above definition, and considering the Christian traditional concept of marriage of the Filipino people as a permanent, inviolable, indissoluble social institution upon which the family and society are founded, and also realizing the strong opposition that any provision on absolute divorce would encounter from the Catholic Church and the Catholic sector of our citizenry to whom the great majority of our people belong, the two Committees in their joint meetings did not pursue the idea of absolute divorce and instead opted for an action for judicial declaration of invalidity of marriage based on grounds available in the Canon Law. It was thought that such an action would not only be an acceptable alternative to divorce but would also solve the nagging problem of 336

University of the Cordilleras College of Law First Year C S.Y. 2013 - 2014 church annulments of marriages on grounds not recognized by the civil law of the State. Justice Reyes was thus requested to again prepare a draft of provisions on such action for celebration of invalidity of marriage. Still later, to avoid the overlapping of provisions on void marriages as found in the present Civil Code and those proposed by Justice Reyes on judicial declaration of invalidity of marriage on grounds similar to the Canon Law, the two Committees now working as a Joint Committee in the preparation of a New Family Code decided to consolidate the present provisions on void marriages with the proposals of Justice Reyes. The result was the inclusion of an additional kind of void marriage in the enumeration of void marriages in the present Civil Code, to wit: "(7) Those marriages contracted by any party who, at the time of the celebration, was wanting in the sufficient use of reason or judgment to understand the essential nature of marriage or was psychologically or mentally incapacitated to discharge the essential marital obligations, even if such lack of incapacity is made manifest after the celebration." as well as the following implementing provisions: "Art. 32. The absolute nullity of a marriage may be invoked or pleaded only on the basis of a final judgment declaring the marriage void, without prejudice to the provision of Article 34." "Art. 33. The action or defense for the declaration of the absolute nullity of a marriage shall not prescribe." xxx xxx xxx It is believed that many hopelessly broken marriages in our country today may already dissolved or annulled on the grounds proposed by the Joint Committee on declaration of nullity as well as annulment of marriages, thus rendering an absolute divorce law unnecessary. In fact, during a conference with Father Gerald Healy of the Ateneo University as well as another meeting with Archbishop Oscar Cruz of the Archdiocese of Pampanga, the Joint Committee was informed that since Vatican II, the Catholic Church has been declaring marriages null and void on the ground of "lack of due discretion" for causes that, in other jurisdictions, would be clear grounds for divorce, like teen-age or premature marriages; marriage to a man who, because of some personality disorder or disturbance, cannot support a family; the foolish or ridiculous choice of a spouse by an otherwise perfectly normal person; marriage to a woman who refuses to cohabit with her husband or who refuses to have children. Bishop Cruz also informed the Committee that they have found out in tribunal work that a lot of machismo among husbands are manifestations of their sociopathic personality anomaly, like inflicting physical violence upon their wives, constitutional indolence or laziness, drug dependence or addiction, and psychological anomaly. . . . (Emphasis supplied) Clearly, by incorporating what is now Article 36 into the Family Code, the Revision Committee referred to above intended to add another ground to those already listed in the Civil Code as grounds for nullifying a marriage, thus expanding or liberalizing the same. Inherent in the inclusion of the provision on psychological incapacity was the understanding that every petition for declaration of nullity based on it should be treated on a case-to-case basis; hence, the absence of a definition and an enumeration of what constitutes psychological incapacity. Moreover, the Committee feared that the giving of examples would limit the applicability of the provision under the principle ofejusdem generis. But the law requires that the same be existing at the time of marriage although it be manifested later. Admittedly, the provision on psychological incapacity, just like any other provision of law, is open to abuse. To prevent this, "the court shall take order the prosecuting attorney or fiscal assigned to it to appear on behalf of the State to take steps to prevent collusion between the parties and to take care that evidence is not fabricated or suppressed." Moreover, the judge, in interpreting the provision on a case-tocase basis, must be guided by "experience, the findings of experts and researchers in psychological disciplines, and by decisions of church tribunals which, although not binding on the civil courts, may be given persuasive effect since the provisions was taken from Canon Law."

337

University of the Cordilleras College of Law First Year C S.Y. 2013 - 2014 The constitutional and statutory provisions on the family will remain the lodestar which our society will hope to achieve ultimately. Therefore, the inclusion of Article 36 is not to be taken as an abandonment of the ideal which we all cherish. If at all, it is a recognition of the reality that some marriages, by reason of the incapacity of one of the contracting parties, fall short of this ideal; thus, the parties are constrained to find a way of putting an end to their union through some legally-accepted means. Any criticism directed at the way that judges have interpreted the provision since its enactment as to render it easier for unhappily-married couples to separate is addressed, not to the wisdom of the lawmakers but to the manner by which some members of the Bench have implemented the provision. These are not interchangeable, each being separate and distinct from the other.

338

University of the Cordilleras College of Law First Year C S.Y. 2013 - 2014 Case Digest LEOUEL SANTOS vs COURT OF APPEALS AND JULIA ROSARIO BEDIA-SANTOS G.R. No. 112019 January 4, 1995 240 SCRA 20 VITUG, J.: FACTS: Leouel, a First Lieutenant in the Philippine Army, met Julia in Iloilo. The two got married in 1986 before a municipal trial court followed shortly thereafter, by a church wedding. The couple lived with Julias parents at the J. Bedia Compound. Julia gave birth to a baby boy in 1987 and was named as Leouel Santos Jr. Occasionally, the couple will quarrel over a number of things aside from the interference of Julias parents into their family affairs. Julia left in 1988 to work in US as a nurse despite Leouels pleas to dissuade her. Seven months after her departure, she called her husband and promised to return home upon the expiration of her contract in July 1989 but she never did. Leouel got a chance to visit US where he underwent a training program under AFP, he desperately tried to locate or somehow get in touch with Julia but all his efforts were of no avail. Leouel filed a complaint to have their marriage declared void under Article 36 of the Family Code. He argued that failure of Julia to return home or to communicate with him for more than 5 years are circumstances that show her being psychologically incapacitated to enter into married life. ISSUE: Whether their marriage can be considered void under Article 36 of the Family Code. HELD: The intendment of the law has been to confine the meaning of psychological incapacity to the most serious cases of personal disorders clearly demonstrative of an utter insensitivity or inability to give meaning and significance to the marriage. This condition must exist at the time the marriage is celebrated. Undeniably and understandably, Leouel stands aggrieved, even desperate, in his present situation. Regrettably, neither law nor society itself can always provide all the specific answers to every individual problem. Wherefore, his petition was denied.

339

University of the Cordilleras College of Law First Year C S.Y. 2013 - 2014 Chi Ming Tsoi vs Court of Appeals 266 SCRA 324 G.R. No. 119190 January 16, 1997 Full Case CHI MING TSOI, petitioner, vs. COURT OF APPEALS and GINA LAO-TSOI, respondents. TORRES, JR., J.: Man has not invented a reliable compass by which to steer a marriage in its journey over troubled waters. Laws are seemingly inadequate. Over time, much reliance has been placed in the works of the unseen hand of Him who created all things. Who is to blame when a marriage fails? This case was originally commenced by a distraught wife against her uncaring husband in the Regional Trial Court of Quezon City (Branch 89) which decreed the annulment of the marriage on the ground of psychological incapacity. Petitioner appealed the decision of the trial court to respondent Court of Appeals (CA-G.R. CV No. 42758) which affirmed the Trial Court's decision November 29, 1994 and correspondingly denied the motion for reconsideration in a resolution dated February 14, 1995. The statement of the case and of the facts made by the trial court and reproduced by the Court of Appeals its decision are as follows: From the evidence adduced, the following acts were preponderantly established: Sometime on May 22, 1988, the plaintiff married the defendant at the Manila Cathedral, . . . Intramuros Manila, as evidenced by their Marriage Contract. (Exh. "A") After the celebration of their marriage and wedding reception at the South Villa, Makati, they went and proceeded to the house of defendant's mother. There, they slept together on the same bed in the same room for the first night of their married life. It is the version of the plaintiff, that contrary to her expectations, that as newlyweds they were supposed to enjoy making love, or having sexual intercourse, with each other, the defendant just went to bed, slept on one side thereof, then turned his back and went to sleep . There was no sexual intercourse between them during the first night. The same thing happened on the second, third and fourth nights. In an effort to have their honeymoon in a private place where they can enjoy together during their first week as husband and wife, they went to Baguio City. But, they did so together with her mother, an uncle, his mother and his nephew. They were all invited by the defendant to join them. [T]hey stayed in Baguio City for four (4) days. But, during this period, there was no sexual intercourse between them, since the defendant avoided her by taking a long walk during siesta time or by just sleeping on a rocking chair located at the living room. They slept together in the same room and on the same bed since May 22, 1988 until March 15, 1989. But during this period, there was no attempt of sexual intercourse between them. [S]he claims, that she did not: even see her husband's private parts nor did he see hers. Because of this, they submitted themselves for medical examinations to Dr. Eufemio Macalalag, a urologist at the Chinese General Hospital, on January 20, 1989. The results of their physical examinations were that she is healthy, normal and still a virgin, while that of her husband's examination was kept confidential up to this time. While no medicine was prescribed for her, the doctor prescribed medications for her husband which was also kept

340

University of the Cordilleras College of Law First Year C S.Y. 2013 - 2014 confidential. No treatment was given to her. For her husband, he was asked by the doctor to return but he never did. The plaintiff claims, that the defendant is impotent, a closet homosexual as he did not show his penis. She said, that she had observed the defendant using an eyebrow pencil and sometimes the cleansing cream of his mother. And that, according to her, the defendant married her, a Filipino citizen, to acquire or maintain his residency status here in the country and to publicly maintain the appearance of a normal man. The plaintiff is not willing to reconcile with her husband. On the other hand, it is the claim of the defendant that if their marriage shall be annulled by reason of psychological incapacity, the fault lies with his wife. But, he said that he does not want his marriage with his wife annulled for several reasons, viz: (1) that he loves her very much; (2) that he has no defect on his part and he is physically and psychologically capable; and, (3) since the relationship is still very young and if there is any differences between the two of them, it can still be reconciled and that, according to him, if either one of them has some incapabilities, there is no certainty that this will not be cured. He further claims, that if there is any defect, it can be cured by the intervention of medical technology or science. The defendant admitted that since their marriage on May 22, 1988, until their separation on March 15, 1989, there was no sexual contact between them. But, the reason for this, according to the defendant, was that everytime he wants to have sexual intercourse with his wife, she always avoided him and whenever he caresses her private parts, she always removed his hands. The defendant claims, that he forced his wife to have sex with him only once but he did not continue because she was shaking and she did not like it. So he stopped. There are two (2) reasons, according to the defendant , why the plaintiff filed this case against him, and these are: (1) that she is afraid that she will be forced to return the pieces of jewelry of his mother, and, (2) that her husband, the defendant, will consummate their marriage. The defendant insisted that their marriage will remain valid because they are still very young and there is still a chance to overcome their differences. The defendant submitted himself to a physical examination. His penis was examined by Dr. Sergio Alteza, Jr., for the purpose of finding out whether he is impotent . As a result thereof, Dr. Alteza submitted his Doctor's Medical Report. (Exh. "2"). It is stated there, that there is no evidence of impotency (Exh. "2-B"), and he is capable of erection. (Exh. "2-C") The doctor said, that he asked the defendant to masturbate to find out whether or not he has an erection and he found out that from the original size of two (2) inches, or five (5) centimeters, the penis of the defendant lengthened by one (1) inch and one centimeter. Dr. Alteza said, that the defendant had only a soft erection which is why his penis is not in its full length. But, still is capable of further erection, in that with his soft erection, the defendant is capable of having sexual intercourse with a woman. In open Court, the Trial Prosecutor manifested that there is no collusion between the parties and that the evidence is not fabricated." After trial, the court rendered judgment, the dispositive portion of which reads: ACCORDINGLY, judgment is hereby rendered declaring as VOID the marriage entered into by the plaintiff with the defendant on May 22, 1988 at the Manila Cathedral, Basilica of the Immaculate Conception, Intramuros, Manila, before the Rt. Rev. Msgr. Melencio de Vera. Without costs. Let a copy of this decision be furnished the Local Civil Registrar of Quezon City. Let another copy be furnished the Local Civil Registrar of Manila. SO ORDERED. 341

University of the Cordilleras College of Law First Year C S.Y. 2013 - 2014 On appeal, the Court of Appeals affirmed the trial court's decision. Hence, the instant petition. Petitioner alleges that the respondent Court of Appeals erred: I in affirming the conclusions of the lower court that there was no sexual intercourse between the parties without making any findings of fact. II in holding that the refusal of private respondent to have sexual communion with petitioner is a psychological incapacity inasmuch as proof thereof is totally absent. III in holding that the alleged refusal of both the petitioner and the private respondent to have sex with each other constitutes psychological incapacity of both. IV in affirming the annulment of the marriage between the parties decreed by the lower court without fully satisfying itself that there was no collusion between them. We find the petition to be bereft of merit. Petitioner contends that being the plaintiff in Civil Case No. Q-89-3141, private respondent has the burden of proving the allegations in her complaint; that since there was no independent evidence to prove the alleged non-coitus between the parties, there remains no other basis for the court's conclusion except the admission of petitioner; that public policy should aid acts intended to validate marriage and should retard acts intended to invalidate them; that the conclusion drawn by the trial court on the admissions and confessions of the parties in their pleadings and in the course of the trial is misplaced since it could have been a product of collusion; and that in actions for annulment of marriage, the material facts alleged in the complaint shall always be proved. Section 1, Rule 19 of the Rules of Court reads: Section 1. Judgment on the pleadings. Where an answer fails to tender an issue, or otherwise admits the material allegations of the adverse party's pleading, the court may, on motion of that party, direct judgment on such pleading. But in actions for annulment of marriage or for legal separation the material facts alleged in the complaint shall always be proved. The foregoing provision pertains to a judgment on the pleadings. What said provision seeks to prevent is annulment of marriage without trial. The assailed decision was not based on such a judgment on the pleadings. When private respondent testified under oath before the trial court and was cross-examined by oath before the trial court and was cross-examined by the adverse party, she thereby presented evidence in form of a testimony. After such evidence was presented, it be came incumbent upon petitioner to present his side. He admitted that since their marriage on May 22, 1988, until their separation on March 15, 1989, there was no sexual intercourse between them. To prevent collusion between the parties is the reason why, as stated by the petitioner, the Civil Code provides that no judgment annulling a marriage shall be promulgated upon a stipulation of facts or by confession of judgment (Arts. 88 and 101[par. 2]) and the Rules of Court prohibit such annulment without trial (Sec. 1, Rule 19). The case has reached this Court because petitioner does not want their marriage to be annulled. This only shows that there is no collusion between the parties. When petitioner admitted that he and his wife (private respondent) have never had sexual contact with each other, he must have been only telling the 342

University of the Cordilleras College of Law First Year C S.Y. 2013 - 2014 truth. We are reproducing the relevant portion of the challenged resolution denying petitioner's Motion for Reconsideration, penned with magisterial lucidity by Associate Justice Minerva Gonzaga-Reyes, viz: The judgment of the trial court which was affirmed by this Court is not based on a stipulation of facts. The issue of whether or not the appellant is psychologically incapacitated to discharge a basic marital obligation was resolved upon a review of both the documentary and testimonial evidence on record. Appellant admitted that he did not have sexual relations with his wife after almost ten months of cohabitation, and it appears that he is not suffering from any physical disability. Such abnormal reluctance or unwillingness to consummate his marriage is strongly indicative of a serious personality disorder which to the mind of this Court clearly demonstrates an 'utter insensitivity or inability to give meaning and significance to the marriage' within the meaning of Article 36 of the Family Code (See Santos vs. Court of Appeals, G.R. No. 112019, January 4, 1995). Petitioner further contends that respondent court erred in holding that the alleged refusal of both the petitioner and the private respondent to have sex with each other constitutes psychological incapacity of both. He points out as error the failure of the trial court to make "a categorical finding about the alleged psychological incapacity and an in-depth analysis of the reasons for such refusal which may not be necessarily due to physchological disorders" because there might have been other reasons, i.e., physical disorders, such as aches, pains or other discomforts, why private respondent would not want to have sexual intercourse from May 22, 1988 to March 15, 1989, in a short span of 10 months. First, it must be stated that neither the trial court nor the respondent court made a finding on who between petitioner and private respondent refuses to have sexual contact with the other. The fact remains, however, that there has never been coitus between them. At any rate, since the action to declare the marriage void may be filed by either party, i.e., even the psychologically incapacitated, the question of who refuses to have sex with the other becomes immaterial. Petitioner claims that there is no independent evidence on record to show that any of the parties is suffering from phychological incapacity. Petitioner also claims that he wanted to have sex with private respondent; that the reason for private respondent's refusal may not be psychological but physical disorder as stated above. We do not agree. Assuming it to be so, petitioner could have discussed with private respondent or asked her what is ailing her, and why she balks and avoids him everytime he wanted to have sexual intercourse with her. He never did. At least, there is nothing in the record to show that he had tried to find out or discover what the problem with his wife could be. What he presented in evidence is his doctor's Medical Report that there is no evidence of his impotency and he is capable of erection. Since it is petitioner's claim that the reason is not psychological but perhaps physical disorder on the part of private respondent, it became incumbent upon him to prove such a claim. If a spouse, although physically capable but simply refuses to perform his or her essential marriage obligations, and the refusal is senseless and constant, Catholic marriage tribunals attribute the causes to psychological incapacity than to stubborn refusal. Senseless and protracted refusal is equivalent to psychological incapacity. Thus, the prolonged refusal of a spouse to have sexual intercourse with his or her spouse is considered a sign of psychological incapacity. Evidently, one of the essential marital obligations under the Family Code is "To procreate children based on the universal principle that procreation of children through sexual cooperation is the basic end of marriage." Constant non- fulfillment of this obligation will finally destroy the integrity or wholeness of the marriage. In the case at bar, the senseless and protracted refusal of one of the parties to fulfill the above marital obligation is equivalent to psychological incapacity. As aptly stated by the respondent court, An examination of the evidence convinces Us that the husband's plea that the wife did not want carnal intercourse with him does not inspire belief. Since he was not physically impotent, but he refrained from sexual intercourse during the entire time (from May 22, 1988 to March 15, 1989) that he occupied the same bed with his wife, purely out of symphaty for her feelings, he deserves to be doubted for not having asserted his right seven though she balked (Tompkins vs. Tompkins, 111 Atl. 599, cited in I Paras, Civil Code, at p. 330). Besides, if it were true that it is the wife was 343

University of the Cordilleras College of Law First Year C S.Y. 2013 - 2014 suffering from incapacity, the fact that defendant did not go to court and seek the declaration of nullity weakens his claim. This case was instituted by the wife whose normal expectations of her marriage were frustrated by her husband's inadequacy. Considering the innate modesty of the Filipino woman, it is hard to believe that she would expose her private life to public scrutiny and fabricate testimony against her husband if it were not necessary to put her life in order and put to rest her marital status. We are not impressed by defendant's claim that what the evidence proved is the unwillingness or lack of intention to perform the sexual act, which is not phychological incapacity, and which can be achieved "through proper motivation." After almost ten months of cohabitation, the admission that the husband is reluctant or unwilling to perform the sexual act with his wife whom he professes to love very dearly, and who has not posed any insurmountable resistance to his alleged approaches, is indicative of a hopeless situation, and of a serious personality disorder that constitutes psychological incapacity to discharge the basic marital covenants within the contemplation of the Family Code. While the law provides that the husband and the wife are obliged to live together, observe mutual love, respect and fidelity (Art. 68, Family Code), the sanction therefor is actually the "spontaneous, mutual affection between husband and wife and not any legal mandate or court order" (Cuaderno vs. Cuaderno 120 Phil. 1298). Love is useless unless it is shared with another. Indeed, no man is an island, the cruelest act of a partner in marriage is to say "I could not have cared less." This is so because an ungiven self is an unfulfilled self. The egoist has nothing but himself. In the natural order, it is sexual intimacy which brings spouses wholeness and oneness. Sexual intimacy is a gift and a participation in the mystery of creation. It is a function which enlivens the hope of procreation and ensures the continuation of family relations. It appears that there is absence of empathy between petitioner and private respondent. That is a shared feeling which between husband and wife must be experienced not only by having spontaneous sexual intimacy but a deep sense of spiritual communion. Marital union is a two-way process. An expressive interest in each other's feelings at a time it is needed by the other can go a long way in deepening the marital relationship. Marriage is definitely not for children but for two consenting adults who view the relationship with love amor gignit amorem, respect, sacrifice and a continuing commitment to compromise, conscious of its value as a sublime social institution. This Court, finding the gravity of the failed relationship in which the parties found themselves trapped in its mire of unfulfilled vows and unconsummated marital obligations, can do no less but sustain the studied judgment of respondent appellate court. IN VIEW OF THE FOREGOING PREMISES , the assailed decision of the Court of Appeals dated November 29, 1994 is hereby AFFIRMED in all respects and the petition is hereby DENIED for lack of merit. SO ORDERED.

344

University of the Cordilleras College of Law First Year C S.Y. 2013 - 2014 Case Digest CHI MING TSOI vs. COURT OF APPEALS and GINA LAO-TSOI G.R. No. 119190 January 16, 1997 266 SCRA 324 TORRES, JR., J.: FACTS: Private respondent Gina Lao and petitioner Chi Ming Tsoi were married at the Manila Cathedral on May 22, 1988. Contrary to Ginas expectations that the newlyweds were to enjoy making love or having sexual intercourse with each other, the defendant just went to bed, slept on one side thereof, then turned his back and went to sleep. No sexual intercourse occurred during their first night, second, third and fourth night. From May 22, 1988 until March 15, 1989, they slept together in the same room and on the same bed but during this period, there was no attempt of sexual intercourse between them. A case was then filed to declare the annulment of the marriage on the ground of psychological incapacity. Gina alleged that Chi Ming was impotent, a closet homosexual as he did not show him his penis (clinically found to be only 3 inches and 1 cm. when erect). Defendant admitted that no sexual contact was ever made and according to him every time he wanted to have sexual intercourse with his wife, she always avoided him and whenever he caressed her private parts she always removed his hands. ISSUE: Is the refusal of private respondent to have sexual communion with petitioner a psychological incapacity? HELD: If a spouse, although physically capable but simply refuses to perform his or her essential marriage obligations, and the refusal is senseless and constant, Catholic marriage tribunals attribute the causes to psychological incapacity than to stubborn refusal. Senseless and protracted refusal is equivalent to psychological incapacity. Thus, the prolonged refusal of a spouse to have sexual intercourse with his or her spouse is considered a sign of psychological incapacity. Evidently, one of the essential marital obligations under the Family Code is To procreate children based on the universal principle that procreation of children through sexual cooperation is the basic end of marriage. Constant non-fulfillment of this obligation will finally destroy the integrity or wholeness of the marriage. In the case at bar, the senseless and protracted refusal of one of the parties to fulfill the above marital obligation is equivalent to psychological incapacity. While the law provides that the husband and the wife are obliged to live together, observe mutual love, respect and fidelity. (Art. 68, Family Code), the sanction therefore is actually the spontaneous, mutual affection between husband and wife and not any legal mandate or court order. Love is useless unless it is shared with another. Indeed, no man is an island, the cruelest act of a partner in marriage is to say I could not have cared less. This is so because an un given self is an unfulfilled self. The egoist has nothing but himself. In the natural order, it is sexual intimacy which brings spouses wholeness and oneness. Sexual intimacy is a gift and a participation in the mystery of creation. It is a function which enlivens the hope of procreation and ensures the continuation of family relations.

345

University of the Cordilleras College of Law First Year C S.Y. 2013 - 2014 Republic vs Molina 268 SCRA 198 G.R. No. 108763 February 13, 1997 Full Case REPUBLIC OF THE PHILIPPINES, vs. COURT OF APPEALS and RORIDEL OLAVIANO MOLINA, respondents. PANGANIBAN, J.: The Family Code of the Philippines provides an entirely new ground (in addition to those enumerated in the Civil Code) to assail the validity of a marriage, namely, "psychological incapacity." Since the Code's effectivity, our courts have been swamped with various petitions to declare marriages void based on this ground. Although this Court had interpreted the meaning of psychological incapacity in the recent case of Santos vs. Court of Appeals, still many judges and lawyers find difficulty in applying said novel provision in specific cases. In the present case and in the context of the herein assailed Decision of the Court of Appeals, the Solicitor General has labelled exaggerated to be sure but nonetheless expressive of his frustration Article 36 as the "most liberal divorce procedure in the world." Hence, this Court in addition to resolving the present case, finds the need to lay down specific guidelines in the interpretation and application of Article 36 of the Family Code. Before us is a petition for review on certiorari under Rule 45 challenging the January 25, 1993 Decision of the Court of Appeals in CA-G.R. CV No. 34858 affirming in toto the May 14, 1991 decision of the Regional Trial Court of La Trinidad, Benguet, which declared the marriage of respondent Roridel Olaviano Molina to Reynaldo Molina void ab initio, on the ground of "psychological incapacity" under Article 36 of the Family Code. The Facts This case was commenced on August 16, 1990 with the filing by respondent Roridel O. Molina of a verified petition for declaration of nullity of her marriage to Reynaldo Molina. Essentially, the petition alleged that Roridel and Reynaldo were married on April 14, 1985 at the San Agustin Church in Manila; that a son, Andre O. Molina was born; that after a year of marriage, Reynaldo showed signs of "immaturity and irresponsibility" as a husband and a father since he preferred to spend more time with his peers and friends on whom he squandered his money; that he depended on his parents for aid and assistance, and was never honest with his wife in regard to their finances, resulting in frequent quarrels between them; that sometime in February 1986, Reynaldo was relieved of his job in Manila, and since then Roridel had been the sole breadwinner of the family; that in October 1986 the couple had a very intense quarrel, as a result of which their relationship was estranged; that in March 1987, Roridel resigned from her job in Manila and went to live with her parents in Baguio City; that a few weeks later, Reynaldo left Roridel and their child, and had since then abandoned them; that Reynaldo had thus shown that he was psychologically incapable of complying with essential marital obligations and was a highly immature and habitually quarrel some individual who thought of himself as a king to be served; and that it would be to the couple's best interest to have their marriage declared null and void in order to free them from what appeared to be an incompatible marriage from the start. In his Answer filed on August 28, 1989, Reynaldo admitted that he and Roridel could no longer live together as husband and wife, but contended that their misunderstandings and frequent quarrels were due to (1) Roridel's strange behavior of insisting on maintaining her group of friends even after their marriage; (2) Roridel's refusal to perform some of her marital duties such as cooking meals; and (3) Roridel's failure to run the household and handle their finances. During the pre-trial on October 17, 1990, the following were stipulated: 1. That the parties herein were legally married on April 14, 1985 at the Church of St. Augustine, Manila; 2. That out of their marriage, a child named Albert Andre Olaviano Molina was born on July 29, 1986; 346

University of the Cordilleras College of Law First Year C S.Y. 2013 - 2014 3. That the parties are separated-in-fact for more than three years; 4. That petitioner is not asking support for her and her child; 5. That the respondent is not asking for damages; 6. That the common child of the parties is in the custody of the petitioner wife. Evidence for herein respondent wife consisted of her own testimony and that of her friends Rosemarie Ventura and Maria Leonora Padilla as well as of Ruth G. Lalas, a social worker, and of Dr. Teresita Hidalgo-Sison, a psychiatrist of the Baguio General Hospital and Medical Center. She also submitted documents marked as Exhibits "A" to "E-1." Reynaldo did not present any evidence as he appeared only during the pre-trial conference. On May 14, 1991, the trial court rendered judgment declaring the marriage void. The appeal of petitioner was denied by the Court of Appeals which affirmed in toto the RTC's decision. Hence, the present recourse. The Issue In his petition, the Solicitor General insists that "the Court of Appeals made an erroneous and incorrect interpretation of the phrase 'psychological incapacity' (as provided under Art. 36 of the Family Code) and made an incorrect application thereof to the facts of the case," adding that the appealed Decision tended "to establish in effect the most liberal divorce procedure in the world which is anathema to our culture." In denying the Solicitor General's appeal, the respondent Court relied heavily on the trial court's findings "that the marriage between the parties broke up because of their opposing and conflicting personalities." Then, it added it sown opinion that "the Civil Code Revision Committee (hereinafter referred to as Committee) intended to liberalize the application of our civil laws on personal and family rights. . . ." It concluded that: As ground for annulment of marriage, We view psychologically incapacity as a broad range of mental and behavioral conduct on the part of one spouse indicative of how he or she regards the marital union, his or her personal relationship with the other spouse, as well as his or her conduct in the long haul for the attainment of the principal objectives of marriage. If said conduct, observed and considered as a whole, tends to cause the union to self-destruct because it defeats the very objectives of marriage, then there is enough reason to leave the spouses to their individual fates. In the case at bar, We find that the trial judge committed no indiscretion in analyzing and deciding the instant case, as it did, hence, We find no cogent reason to disturb the findings and conclusions thus made. Respondent, in her Memorandum, adopts these discussions of the Court of Appeals. The petitioner, on the other hand, argues that "opposing and conflicting personalities" is not equivalent to psychological incapacity, explaining that such ground "is not simply the neglect by the parties to the marriage of their responsibilities and duties, but a defect in their psychological nature which renders them incapable of performing such marital responsibilities and duties." The Court's Ruling The petition is meritorious. In Leouel Santos vs. Court of Appeals this Court, speaking thru Mr. Justice Jose C. Vitug, ruled that "psychological incapacity should refer to no less than a mental (nor physical) incapacity . . . and that (t)here is hardly any doubt that the intendment of the law has been to confine the meaning of 'psychological incapacity' to the most serious cases of personality disorders clearly demonstrative of an utter insensitivity or inability to give meaning and significance to the marriage. This psychologic condition must exist at the time the marriage is celebrated." Citing Dr. Gerardo Veloso, a former 347

University of the Cordilleras College of Law First Year C S.Y. 2013 - 2014 presiding judge of the Metropolitan Marriage Tribunal of the Catholic Archdiocese of Manila, Justice Vitug wrote that "the psychological incapacity must be characterized by (a) gravity, (b) juridical antecedence, and (c) incurability." On the other hand, in the present case, there is no clear showing to us that the psychological defect spoken of is an incapacity. It appears to us to be more of a "difficulty," if not outright "refusal" or "neglect" in the performance of some marital obligations. Mere showing of "irreconciliable differences" and "conflicting personalities" in no wise constitutes psychological incapacity. It is not enough to prove that the parties failed to meet their responsibilities and duties as married persons; it is essential that they must be shown to be incapable of doing so, due to some psychological (nor physical) illness. The evidence adduced by respondent merely showed that she and her husband could nor get along with each other. There had been no showing of the gravity of the problem; neither its juridical antecedence nor its incurability. The expert testimony of Dr. Sison showed no incurable psychiatric disorder but only incompatibility, not psychological incapacity. Dr. Sison testified: COURT Q It is therefore the recommendation of the psychiatrist based on your findings that it is better for the Court to annul (sic) the marriage? A Yes, Your Honor. Q There is no hope for the marriage? A There is no hope, the man is also living with another woman. Q Is it also the stand of the psychiatrist that the parties are psychologically unfit for each other but they are psychologically fit with other parties? A Yes, Your Honor. Q Neither are they psychologically unfit for their professions? A Yes, Your Honor. The Court has no more questions. In the case of Reynaldo, there is no showing that his alleged personality traits were constitutive of psychological incapacity existing at the time of marriage celebration. While some effort was made to prove that there was a failure to fulfill pre-nuptial impressions of "thoughtfulness and gentleness" on Reynaldo's part of being "conservative, homely and intelligent" on the part of Roridel, such failure of expectation is nor indicative of antecedent psychological incapacity. If at all, it merely shows love's temporary blindness to the faults and blemishes of the beloved. During its deliberations, the Court decided to go beyond merely ruling on the facts of this case vis-avis existing law and jurisprudence. In view of the novelty of Art. 36 of the Family Code and the difficulty experienced by many trial courts interpreting and applying it, the Court decided to invite two amici curiae, namely, the Most Reverend Oscar V. Cruz, Vicar Judicial (Presiding Judge) of the National Appellate Matrimonial Tribunal of the Catholic Church in the Philippines, and Justice Ricardo C. Puno, a member of the Family Code Revision Committee. The Court takes this occasion to thank these friends of the Court for their informative and interesting discussions during the oral argument on December 3, 1996, which they followed up with written memoranda. From their submissions and the Court's own deliberations, the following guidelines in the interpretation and application of Art. 36 of the Family Code are hereby handed down for the guidance of the bench and the bar:

348

University of the Cordilleras College of Law First Year C S.Y. 2013 - 2014 (1) The burden of proof to show the nullity of the marriage belongs to the plaintiff. Any doubt should be resolved in favor of the existence and continuation of the marriage and against its dissolution and nullity. This is rooted in the fact that both our Constitution and our laws cherish the validity of marriage and unity of the family. Thus, our Constitution devotes an entire Article on the Family, recognizing it "as the foundation of the nation." It decrees marriage as legally "inviolable," thereby protecting it from dissolution at the whim of the parties. Both the family and marriage are to be "protected" by the state. The Family Code echoes this constitutional edict on marriage and the family and emphasizes thepermanence, inviolability and solidarity (2) The root cause of the psychological incapacity must be (a) medically or clinically identified, (b) alleged in the complaint, (c) sufficiently proven by experts and (d) clearly explained in the decision. Article 36 of the Family Code requires that the incapacity must be psychological not physical. although its manifestations and/or symptoms may be physical. The evidence must convince the court that the parties, or one of them, was mentally or physically ill to such an extent that the person could not have known the obligations he was assuming, or knowing them, could not have given valid assumption thereof. Although no example of such incapacity need be given here so as not to limit the application of the provision under the principle of ejusdem generis, nevertheless such root cause must be identified as a psychological illness and its incapacitating nature explained. Expert evidence may be given qualified psychiatrist and clinical psychologists. (3) The incapacity must be proven to be existing at "the time of the celebration" of the marriage. The evidence must show that the illness was existing when the parties exchanged their "I do's." The manifestation of the illness need not be perceivable at such time, but the illness itself must have attached at such moment, or prior thereto. (4) Such incapacity must also be shown to be medically or clinically permanent or incurable. Such incurability may be absolute or even relative only in regard to the other spouse, not necessarily absolutely against everyone of the same sex. Furthermore, such incapacity must be relevant to the assumption of marriage obligations, not necessarily to those not related to marriage, like the exercise of a profession or employment in a job. Hence, a pediatrician may be effective in diagnosing illnesses of children and prescribing medicine to cure them but may not be psychologically capacitated to procreate, bear and raise his/her own children as an essential obligation of marriage. (5) Such illness must be grave enough to bring about the disability of the party to assume the essential obligations of marriage. Thus, "mild characteriological peculiarities, mood changes, occasional emotional outbursts" cannot be accepted as root causes. The illness must be shown as downright incapacity or inability, nor a refusal, neglect or difficulty, much less ill will. In other words, there is a natal or supervening disabling factor in the person, an adverse integral element in the personality structure that effectively incapacitates the person from really accepting and thereby complying with the obligations essential to marriage. (6) The essential marital obligations must be those embraced by Articles 68 up to 71 of the Family Code as regards the husband and wife as well as Articles 220, 221 and 225 of the same Code in regard to parents and their children. Such non-complied marital obligation(s) must also be stated in the petition, proven by evidence and included in the text of the decision. (7) Interpretations given by the National Appellate Matrimonial Tribunal of the Catholic Church in the Philippines, while not controlling or decisive, should be given great respect by our courts. It is clear that Article 36 was taken by the Family Code Revision Committee from Canon 1095 of the New Code of Canon Law, which became effective in 1983 and which provides: The following are incapable of contracting marriage: Those who are unable to assume the essential obligations of marriage due to causes of psychological nature. Since the purpose of including such provision in our Family Code is to harmonize our civil laws with the religious faith of our people, it stands to reason that to achieve such harmonization, great persuasive weight should be given to decision of such appellate tribunal. Ideally subject to our law on evidence what is decreed as canonically invalid should also be decreed civilly void.

349

University of the Cordilleras College of Law First Year C S.Y. 2013 - 2014 This is one instance where, in view of the evident source and purpose of the Family Code provision, contemporaneous religious interpretation is to be given persuasive effect. Here, the State and the Church while remaining independent, separate and apart from each other shall walk together in synodal cadence towards the same goal of protecting and cherishing marriage and the family as the inviolable base of the nation. (8) The trial court must order the prosecuting attorney or fiscal and the Solicitor General to appear as counsel for the state. No decision shall he handed down unless the Solicitor General issues a certification, which will be quoted in the decision, briefly staring therein his reasons for his agreement or opposition, as the case may be, to the petition. The Solicitor General, along with the prosecuting attorney, shall submit to the court such certification within fifteen (15) days from the date the case is deemed submitted for resolution of the court. The Solicitor General shall discharge the equivalent function of the defensor vinculi contemplated under Canon 1095. In the instant case and applying Leouel Santos, we have already ruled to grant the petition. Such ruling becomes even more cogent with the use of the foregoing guidelines. WHEREFORE, the petition is GRANTED. The assailed Decision is REVERSED and SET ASIDE. The marriage of Roridel Olaviano to Reynaldo Molina subsists and remains valid. SO ORDERED. Narvasa, C.J., Davide, Jr., Bellosillo, Melo, Puno Francisco, Hermosisima, Jr., and Torres, Jr., JJ., concur. Regalado, Kapunan and Mendoza, JJ., concurs in the result. Separate Opinions

PADILLA, J., concuring opinion: I concur in the result of the decision penned by Mr. Justice Panganiban but only because of the peculiar facts of the case. As to whether or not the psychological incapacity exists in a given case calling for annulment of a marriage, depends crucially, more than in any field of the law, on the facts of the case. In Leouel Santos v. Court of Appeals and Julia Rosario-Bedia Santos, G.R. No. 112019, 4 January 1995, 240 SCRA 20-36, I maintained, and I still maintain, that there was psychological incapacity on the part of the wife to discharge the duties of a wife in a valid marriage. The facts of the present case, after an indepth study, do not support a similar conclusion. Obviously, each case must be judged, not on the basis of a priori assumptions, predilections or generalizations but according to its own facts. In the field of psychological incapacity as a ground for annulment of marriage, it is trite to say that no case is on "all fours" with another case. The trial judge must take pains in examining the actual millieu and the appellate court must, as much as possible, avoid substituting its own judgment for that of the trial court. ROMERO, J., separate opinion: The majority opinion, overturning that of the Court of Appeals which affirmed the Regional Trial Court ruling. upheld petitioner Solicitor General's position that "opposing and conflicting personalities" is not equivalent to psychological incapacity, for the latter "is not simply the neglect by the parties to the marriage of their responsibilities and duties, but a defect in their Psychological nature which renders them incapable of performing such marital responsibilities and duties. In the present case, the alleged personality traits of Reynaldo, the husband, did not constitute so much "psychological incapacity" as a "difficulty," if not outright "refusal" or "neglect" in the performance of some marital obligations. "It is not enough to prove that the parties failed to meet their responsibilities and duties as married persons; it is essential that they must be shown to be incapableof doing so, due to some psychological (not physical) illness."

350

University of the Cordilleras College of Law First Year C S.Y. 2013 - 2014 I would add that neither should the incapacity be the result of mental illness. For if it were due to insanity or defects in the mental faculties short of insanity, there is a resultant defect of vice of consent, thus rendering the marriage annulable under Art. 45 of the Family Code. That the intent of the members of the U.P. Law Center's Civil Code Revision Committee was to excludemental inability to understand the essential nature of marriage and focus strictly on psychological incapacity is demonstrated in the way the provision in question underwent revisions. At the Committee meeting of July 26, 1986, the draft provision read: (7) Those marriages contracted by any party who, at the time of the celebration, was wanting in the sufficient use of reason or judgment to understand the essential nature of marriage or was psychologically or mentally incapacitated to discharge the essential marital obligations, even if such lack of incapacity is made manifest after the celebration. The twists and turns which the ensuing discussion took finally produced the following revised provision even before the session was over: (7) That contracted by any party who, at the time of the celebration, was psychologically incapacitated to discharge the essential marital obligations, even if such lack or incapacity becomes manifest after the celebration. Noticeably, the immediately preceding formulation above has dropped any reference to "wanting in the sufficient use of reason or judgment to understand the essential nature or marriage" and to "mentally incapacitated." It was explained that these phrases refer to "defects in the mental faculties vitiating consent, which is not the idea . . . but lack of appreciation of one's marital obligation." There being a defect in consent, "it is clear that it should be a ground for voidable marriage because there is the appearance of consent and it is capable of convalidation for the simple reason that there are lucid intervals and there are sanity is curable. . . . Psychological incapacity does not refer to mental faculties and has nothing to do with consent; it refers to obligations attendant to marriage." My own position as a member of the Committee then was that psychological incapacity is, in a sense, insanity of a lesser degree. As to the proposal of Justice Caguioa to use the term "psychological or mental impotence," Archbishop Oscar Cruz opined in he earlier February 9, 1984 session that this term "is an invention of some churchmen who are moralists but not canonists, that is why it is considered a weak phrase." He said that the Code of Canon Law would rather express it as "psychological or mental incapacity to discharge. . . ." Justice Ricardo C. Puno opined that sometimes a person may be psychologically impotent with one but not with another. One of the guidelines enumerated in the majority opinion for the interpretation and application of Art. 36 is: "Such incapacity must also be shown to be medically or clinically permanent or incurable. Such incurability may be absolute or even relative only in regard to the other spouse, not necessarily absolutely against everyone of the same sex." The Committee, through Prof. Araceli T. Barrera, considered the inclusion of the phrase" and is incurable" but Prof. Esteban B. Bautista commented that this would give rise to the question of how they will determine curability and Justice Caguioa agreed that it would be more problematic. Yet the possibility that one may be cured after the psychological incapacity becomes manifest after the marriage was not ruled out by Justice Puno and Justice Alice Sempio-Diy. Justice Caguioa suggested that the remedy was to allow the afflicted spouse to remarry. For clarity, the Committee classified the bases for determining void marriages, viz: 1. lack of one or more of the essential requisites of marriage as contract; 2. reasons of public policy;

351

University of the Cordilleras College of Law First Year C S.Y. 2013 - 2014 3. special cases and special situations. The ground of psychological incapacity was subsumed under "special cases and special situations," hence its special treatment in Art. 36 in the Family Code as finally enacted. Nowhere in the Civil Code provisions on Marriage is there a ground for avoiding or annulling marriages that even comes close to being psychological in nature. Where consent is vitiated due to circumstances existing at the time of the marriage, such marriage which stands valid until annulled is capable of ratification or convalidation. On the other hand, for reasons of public policy or lack of essential requisites, some marriages are void from the beginning. With the revision of Book I of the Civil Code, particularly the provisions on Marriage, the drafters, now open to fresh winds of change in keeping with the more permissive mores and practices of the time, took a leaf from the relatively liberal provisions of Canon Law. Canon 1095 which states, inter alia, that the following persons are incapable of contracting marriage: "3. (those) who, because of causes of a psychological nature, are unable to assume the essential obligations of marriage" provided the model for what is now Art. 36 of the Family Code: "A marriage contracted by any party who, at the time of the celebration, was psychologically incapacitated to comply with the essential marital obligations of marriage, shall likewise be void even if such incapacity becomes manifest only after its solemnization. It bears stressing that unlike in Civil Law, Canon Law recognizes only two types of marriages with respect to their validity: valid and void. Civil Law, however, recognizes an intermediate state, the voidable or annullable marriages. When the Ecclesiastical Tribunal "annuls" a marriage, it actually declares the marriage null and void, i.e., it never really existed in the first place, for a valid sacramental marriage can never be dissolved. Hence, a properly performed and consummated marriage between two living Roman Catholics can only be nullified by the formal annulment process which entails a full tribunal procedure with a Court selection and a formal hearing. Such so-called church "annulments" are not recognized by Civil Law as severing the marriage ties as to capacitate the parties to enter lawfully into another marriage. The grounds for nullifying civil marriage, not being congruent with those laid down by Canon Law, the former being more strict, quite a number of married couples have found themselves in limbo freed from the marriage bonds in the eyes of the Catholic Church but yet unable to contract a valid civil marriage under state laws. Heedless of civil law sanctions, some persons contract new marriages or enter into live-in relationships. It was precisely to provide a satisfactory solution to such anomalous situations that the Civil Law Revision Committee decided to engraft the Canon Law concept of psychological incapacity into the Family Code and classified the same as a ground for declaring marriages void ab initio or totally in existent from the beginning. A brief historical note on the Old Canon Law (1917). This Old Code, while it did not provide directly for psychological incapacity, in effect recognized the same indirectly from a combination of three old canons: "Canon #1081 required persons to 'be capable according to law' in order to give valid consent; Canon #1082 required that persons 'be at least not ignorant' of the major elements required in marriage; and Canon #1087 (the force and fear category) required that internal and external freedom be present in order for consent to be valid. This line of interpretation produced two distinct but related grounds for annulment, called 'lack of due discretion' and 'lack of due competence.' Lack of due discretion means that the person did not have the ability to give valid consent at the time of the weddingand therefore the union is invalid. Lack of due competence means that the person was incapable of carrying out the obligations of the promise he or she made during the wedding ceremony. "Favorable annulment decisions by the Roman Rota in the 1950s and 1960s involving sexual disorders such as homosexuality and nymphomania laid the foundation for a broader approach to the kind of proof necessary for psychological grounds for annulment. The Rota had reasoned for the first time in several cases that the capacity to give valid consent at the time of marriage was probably not present in persons who had displayed such problems shortly after the marriage. The nature of this change was nothing short 352

University of the Cordilleras College of Law First Year C S.Y. 2013 - 2014 of revolutionary. Once the Rota itself had demonstrated a cautious willingness to use this kind of hindsight, the way was paved for what came after 1970. Diocesan Tribunals began to accept proof of serious psychological problems that manifested themselves shortly after the ceremony as proof of an inability to give valid consent at the time of the ceremony. Furthermore, and equally significant, the professional opinion of a psychological expert became increasingly important in such cases. Data about the person's entire life, both before and after the ceremony, were presented to these experts and they were asked to give professional opinions about a party's mental at the time of the wedding. These opinions were rarely challenged and tended to be accepted as decisive evidence of lack of valid consent. The Church took pains to point out that its new openness in this area did not amount to the addition of new grounds for annulment, but rather was an accommodation by the Church to the advances made in psychology during the past decades. There was now the expertise to provide the all-important connecting link between a marriage breakdown and premarital causes. During the 1970s, the Church broadened its whole idea of marriage from that of a legal contract to that of a covenant. The result of this was that it could no longer be assumed in annulment cases that a person who could intellectually understand the concept of marriage could necessarily give valid consent to marry. The ability to both grasp and assume the real obligations of a mature, lifelong commitment are now considered a necessary prerequisite to valid matrimonial consent. Rotal decisions continued applying the concept of incipient psychological incapacity, "not only to sexual anomalies but to all kinds of personality disorders that incapacitate a spouse or both spouses from assuming or carrying out the essential obligations of marriage. For marriage . . . is not merely cohabitation or the right of the spouses to each others' body for heterosexual acts, but is, in its totality, the right to the community of the whole of life, i.e., the right to a developing. lifelong relationship. Rotal decisions since 1973 have refined the meaning of psychological or psychic capacity for marriage as presupposing the development of an adult personality; as meaning the capacity of the spouses to give themselves to each other and to accept the other as a distinct person; that the spouses must be 'other oriented' since the obligations of marriage are rooted in a self-giving love; and that the spouses must have the capacity for interpersonal relationship because marriage is more than just a physical reality but involves a true intertwining of personalities. The fulfillment of the obligations of marriage depends. according to Church decisions, on the strength of this interpersonal relationship. A serious incapacity for interpersonal sharing and support is held to impair the relationship and consequently, the ability to fulfill the essential marital obligations. The marital capacity of one spouse is not considered in isolation but in reference to the fundamental relationship to the other spouse. Fr. Green, in an article in Catholic Mind, lists six elements necessary to the mature marital relationship: The courts consider the following elements crucial to the marital commitment: (1) a permanent and faithful commitment to the marriage partner; (2) openness to children and partner; (3) stability; (4) emotional maturity; (5) financial responsibility; (6) an ability to cope with the ordinary stresses and strains of marriage, etc. Fr. Green goes on to speak about some of the psychological conditions that might lead to the failure of a marriage: At stake is a type of constitutional impairment precluding conjugal communion even with the best intentions of the parties. Among the psychic factors possibly giving rise to his or her inability to fulfill marital obligations are the following: (1) antisocial personality with its fundamental lack of loyalty to persons or sense of moral values; (2) hyperesthesia, where the individual has no real freedom of sexual choice; (3) the inadequate personality where personal responses consistently fallshort of reasonable expectations. xxx xxx xxx The psychological grounds are the best approach for anyone who doubts whether he or she has a case for an annulment on any other terms. A situation that does not fit into any of the more traditional categories often fits very easily into the psychological category.

353

University of the Cordilleras College of Law First Year C S.Y. 2013 - 2014 As new as the psychological grounds are, experts are already detecting a shift in their use. Whereas originally the emphasis was on the parties' inability to exercise proper judgment at the time of the marriage (lack of due discretion), recent cases seem to be concentrating on the parties' to assume or carry out their responsibilities an obligations as promised (lack of due competence). An advantage to using the ground of lack of due competence is that the at the time the marriage was entered intocivil divorce and breakup of the family almost is of someone's failure out marital responsibilities as promised at the time the marriage was entered into. In the instant case, "opposing and conflicting personalities" of the spouses were not considered equivalent to psychological incapacity. As well in Santos v. Court of Appeals cited in the ponencia, the Court held that the failure of the wife to return home from the U.S. or to communicate with her husband for more then five years is not proof of her psychological incapacity as to render the marriage a nullity. Therefore, Art. 36 is inapplicable and the marriages remain valid and subsisting. However in the recent case of Chi Ming Tsoi v. Court of Appeals, this Court upheld both the Regional Trial Court and the Court of Appeals in declaring the presence of psychological incapacity on the part of the husband. Said petitioner husband, after ten (10) months' sleeping with his wife never had coitus with her, a fact he did not deny but he alleged that it was due to the physical disorder of his wife which, however, he failed to prove. Goaded by the indifference and stubborn refusal of her husband to fulfill a basic marital obligation described as "to procreate children based on the universal principle that procreation of children through sexual cooperation is the basic end of marriage," the wife brought the action in the lower court to declare the marriage null. The Court, quoting Dr. Gerardo Veloso, a former Presiding Judge of the Metropolitan Marriage Tribunal of the Catholic Archdiocese of Manila (Branch I) on Psychological incapacity concluded: If a spouse, although physically capable but simply refuses to perform his or her essential marriage obligations, and the refusal is senseless and constant, Catholic marriage tribunals attribute the causes to psychological incapacity than to stubborn refusal.Senseless and protracted refusal is equivalent to psychological incapacity. Thus, the prolonged refusal of a spouse to have sexual intercourse with his or her spouse is considered a sign of psychological incapacity. We declared: This Court, finding the gravity of the failed relationship in which the parties found themselves trapped in its mire of unfulfilled vows and unconsummated marital obligations, can do no less but sustain the studied judgment of respondent appellate court. 1 concur with the majority opinion that the herein marriage remains valid and subsisting absent psychological incapacity (under Art. 36 of the Family Code) on the part of either or both of the spouses.

VITUG, J., concurring: I fully concur with my esteemed 'colleague Mr. Justice Artemio V. Panganiban in his ponencia, and I find to be most helpful the guidelines that he prepared for the bench and the bar in the proper appreciation of Article 36 of Executive Order No. 209 ("The Family Code of the Philippines"). The term "psychological incapacity" was neither defined nor exemplified by the Family Code. Thus Art. 36. A marriage contracted by any party who, at the time of the celebration, was psychologically incapacitated to comply with the essential marital obligations of marriage, shall likewise be void even if such incapacity becomes manifest only after its solemnization. The Revision Committee, constituted under the auspices of the U.P. Law Center, which drafted the Code explained:

354

University of the Cordilleras College of Law First Year C S.Y. 2013 - 2014 (T)he Committee would like the judge to interpret the provision on a case-to-case basis, guided by experience, the findings of experts and researchers in psychological disciplines, and by decisions of church tribunals which, although not binding on the civil courts, may be given persuasive effect since the provision was taken from Canon Law. Article 36 of the Family Code was concededly taken from Canon 1095 of the New Code of Canon Law Canon 1095. (The following persons) are incapable of contracting marriage; (those) 1. who lack sufficient use of reason; 2. who suffer from a grave defect of discretion of judgment concerning essential matrimonial rights and duties, to be given and accepted mutually; 3. who for causes of psychological nature are unable to assume the essential obligations of marriage that should give that much value to Canon Law jurisprudence as an aid to the interpretation and construction of the statutory enactment. The principles in the proper application of the law teach us that the several provisions of a Code must be read like a congruent whole. Thus, in determining the import of "psychological incapacity" under Article 36, one must also read it along with, albeit to be taken as distinct from, the other grounds enumerated in the Code, like Articles 35, 37, 38 and 41 that would likewise, but for distinct reasons, render the marriage merely voidable, or Article 55 that could justify a petition for legal separation. Care must be observed so that these various circumstances are not applied so indiscriminately as if the law were indifferent on the matter. I would wish to reiterate the Court's' statement in Santos vs. Court of Appeals; viz: (T)he use of the phrase "psychological incapacity" under Article 36 of the Code has not been meant to comprehend all such possible cases of psychoses as, likewise mentioned by some ecclesiastical authorities, extremely low intelligence, immaturity, and like circumstances. . . Article 36 of the Family Code cannot be taken and construed independently of, but must stand in conjunction with, existing precepts in our law on marriage. Thus correlated, "psychological incapacity" should refer to no less than a mental (not physical) incapacity that causes a party to be truly incognitive of the basic marital covenants that concomitantly must be assumed and discharged by the parties to the marriage which, as so expressed by Article 68 of the Family Code, include their mutual obligations to live together, observe love, respect and fidelity and render help and support. There is hardly any doubt that the intendment of the law has been to confine the meaning of "psychological incapacity" to the most serious cases of personality disorders clearly demonstrative of an utter insensitivity or inability of the spouse to have sexual relations with the other. This conclusion is implicit under Article 54 of the Family Code which considers children conceived prior to the judicial declaration of nullity of the void marriage to be "legitimate." The other forms of psychoses, if existing at the inception of marriage, like the state of a party being of unsound mind or concealment of drug addiction, habitual alcoholism, homosexuality or lesbianism, merely renders the marriage contract voidable pursuant to Article 46, Family Code. If drug addiction, habitual alcoholism, lesbianism or homosexuality should occur only during the marriage, they become mere grounds for legal separation under Article 55 of the Family Code. These provisions of the Code, however, do not necessarily preclude the possibility of these various circumstances being themselves, depending on the degree and severity of the disorder, indicia of psychological incapacity. In fine, the term "psychological incapacity," to be a ground for then nullity of marriage under Article 36 of the Family Code, must be able to pass the following tests; viz: 355

University of the Cordilleras College of Law First Year C S.Y. 2013 - 2014 First, the incapacity must be psychological or mental, not physical, in nature; Second, the psychological incapacity must relate to the inability, not mere refusal, to understand, assume end discharge the basic marital obligations of living together, observing love, respect and fidelity and rendering mutual help and support; Third, the psychologic condition must exist at the time the marriage is contracted although its overt manifestations and the marriage may occur only thereafter; and Fourth, the mental disorder must be grave or serious and incurable. It may well be that the Family Code Revision Committee has envisioned Article 36, as not a few observers would suspect, as another form of absolute divorce or, as still others would also put it, to be a alternative to divorce; however, the fact still remains that the language of the law has failed to carry out, even if true, any such intendment. It might have indeed turned out for the better, if it were otherwise, there could be good reasons to doubt the constitutionality of the measure. The fundamental law itself, no less, has laid down in terse language its unequivocal command on how the State should regard marriage and the family, thus Section 2, Article XV: Sec. 2. Marriage, as an inviolable social institution, is the foundation of the family and shall be protected by the State. Section 12, Article II: Sec. 12. The State recognizes the sanctity of family life and shall protect and strengthen the family as a basic autonomous social institution . . . . Section 1, Article XV: Sec. 1. The State recognizes the Filipino family as the foundation of the nation. Accordingly, it shall strengthen its solidarity and actively promote its total development. (The 1987 Constitution) The case of Marcelino vs. Cruz, 121 SCRA 51, might here be significant not so much for the specific issue there resolved but for the tone it has set. The Court there has held that constitutional provisions are to be considered mandatory unless by necessary implication, a different intention is manifest such that to have them enforced strictly would cause more harm than by disregarding them. It is quite clear to me that the constitutional mandate on marriage and the family has not been meant to be simply directory in character, nor for mere expediency or convenience, but one that demands a meaningful, not half-hearted, respect.

Separate Opinions PADILLA, J., concuring opinion: I concur in the result of the decision penned by Mr. Justice Panganiban but only because of the peculiar facts of the case. As to whether or not the psychological incapacity exists in a given case calling for annulment of a marriage, depends crucially, more than in any field of the law, on the facts of the case. In Leouel Santos v. Court of Appeals and Julia Rosario-Bedia Santos, G.R. No. 112019, 4 January 1995, 240 SCRA 20-36, I maintained, and I still maintain, that there was psychological incapacity on the part of the wife to discharge the duties of a wife in a valid marriage. The facts of the present case, after an indepth study, do not support a similar conclusion. Obviously, each case must be judged, not on the basis of a priori assumptions, predilections or generalizations but according to its own facts. In the field of psychological incapacity as a ground for annulment of marriage, it is trite to say that no case is on "all fours" with another case. The trial judge must take pains in examining the actual millieu and the appellate court must, as much as possible, avoid substituting its own judgment for that of the trial court. 356

University of the Cordilleras College of Law First Year C S.Y. 2013 - 2014 ROMERO, J., separate opinion: The majority opinion, overturning that of the Court of Appeals which affirmed the Regional Trial Court ruling. upheld petitioner Solicitor General's position that "opposing and conflicting personalities" is not equivalent to psychological incapacity, for the latter "is not simply the neglect by the parties to the marriage of their responsibilities and duties, but a defect in their Psychological nature which renders them incapable of performing such marital responsibilities and duties. In the present case, the alleged personality traits of Reynaldo, the husband, did not constitute so much "psychological incapacity" as a "difficulty," if not outright "refusal" or "neglect" in the performance of some marital obligations. "It is not enough to prove that the parties failed to meet their responsibilities and duties as married persons; it is essential that they must be shown to be incapableof doing so, due to some psychological (not physical) illness." I would add that neither should the incapacity be the result of mental illness. For if it were due to insanity or defects in the mental faculties short of insanity, there is a resultant defect of vice of consent, thus rendering the marriage annulable under Art. 45 of the Family Code. That the intent of the members of the U.P. Law Center's Civil Code Revision Committee was to excludemental inability to understand the essential nature of marriage and focus strictly on psychological incapacity is demonstrated in the way the provision in question underwent revisions. At the Committee meeting of July 26, 1986, the draft provision read: (7) Those marriages contracted by any party who, at the time of the celebration, was wanting in the sufficient use of reason or judgment to understand the essential nature of marriage or was psychologically or mentally incapacitated to discharge the essential marital obligations, even if such lack of incapacity is made manifest after the celebration. The twists and turns which the ensuing discussion took finally produced the following revised provision even before the session was over: (7) That contracted by any party who, at the time of the celebration, was psychologically incapacitated to discharge the essential marital obligations, even if such lack or incapacity becomes manifest after the celebration. Noticeably, the immediately preceding formulation above has dropped any reference to "wanting in the sufficient use of reason or judgment to understand the essential nature or marriage" and to "mentally incapacitated." It was explained that these phrases refer to "defects in the mental faculties vitiating consent, which is not the idea . . . but lack of appreciation of one's marital obligation." There being a defect in consent, "it is clear that it should be a ground for voidable marriage because there is the appearance of consent and it is capable of convalidation for the simple reason that there are lucid intervals and there are sanity is curable. . . . Psychological incapacity does not refer to mental faculties and has nothing to do with consent; it refers to obligations attendant to marriage." My own position as a member of the Committee then was that psychological incapacity is, in a sense, insanity of a lesser degree. As to the proposal of Justice Caguioa to use the term "psychological or mental impotence," Archbishop Oscar Cruz opined in he earlier February 9, 1984 session that this term "is an invention of some churchmen who are moralists but not canonists, that is why it is considered a weak phrase." He said that the Code of Canon Law would rather express it as "psychological or mental incapacity to discharge. . . ." Justice Ricardo C. Puno opined that sometimes a person may be psychologically impotent with one but not with another. One of the guidelines enumerated in the majority opinion for the interpretation and application of Art. 36 is: "Such incapacity must also be shown to be medically or clinically permanent or incurable. Such incurability may be absolute or even relative only in regard to the other spouse, not necessarily absolutely against everyone of the same sex." 357

University of the Cordilleras College of Law First Year C S.Y. 2013 - 2014 The Committee, through Prof. Araceli T. Barrera, considered the inclusion of the phrase" and is incurable" but Prof. Esteban B. Bautista commented that this would give rise to the question of how they will determine curability and Justice Caguioa agreed that it would be more problematic. Yet the possibility that one may be cured after the psychological incapacity becomes manifest after the marriage was not ruled out by Justice Puno and Justice Alice Sempio-Diy. Justice Caguioa suggested that the remedy was to allow the afflicted spouse to remarry. For clarity, the Committee classified the bases for determining void marriages, viz: 1. lack of one or more of the essential requisites of marriage as contract; 2. reasons of public policy; 3. special cases and special situations. The ground of psychological incapacity was subsumed under "special cases and special situations," hence its special treatment in Art. 36 in the Family Code as finally enacted. Nowhere in the Civil Code provisions on Marriage is there a ground for avoiding or annulling marriages that even comes close to being psychological in nature. Where consent is vitiated due to circumstances existing at the time of the marriage, such marriage which stands valid until annulled is capable of ratification or convalidation. On the other hand, for reasons of public policy or lack of essential requisites, some marriages are void from the beginning. With the revision of Book I of the Civil Code, particularly the provisions on Marriage, the drafters, now open to fresh winds of change in keeping with the more permissive mores and practices of the time, took a leaf from the relatively liberal provisions of Canon Law. Canon 1095 which states, inter alia, that the following persons are incapable of contracting marriage: "3. (those) who, because of causes of a psychological nature, are unable to assume the essential obligations of marriage" provided the model for what is now Art. 36 of the Family Code: "A marriage contracted by any party who, at the time of the celebration, was psychologically incapacitated to comply with the essential marital obligations of marriage, shall likewise be void even if such incapacity becomes manifest only after its solemnization. It bears stressing that unlike in Civil Law, Canon Law recognizes only two types of marriages with respect to their validity: valid and void. Civil Law, however, recognizes an intermediate state, the voidable or annullable marriages. When the Ecclesiastical Tribunal "annuls" a marriage, it actually declares the marriage null and void, i.e., it never really existed in the first place, for a valid sacramental marriage can never be dissolved. Hence, a properly performed and consummated marriage between two living Roman Catholics can only be nullified by the formal annulment process which entails a full tribunal procedure with a Court selection and a formal hearing. Such so-called church "annulments" are not recognized by Civil Law as severing the marriage ties as to capacitate the parties to enter lawfully into another marriage. The grounds for nullifying civil marriage, not being congruent with those laid down by Canon Law, the former being more strict, quite a number of married couples have found themselves in limbo freed from the marriage bonds in the eyes of the Catholic Church but yet unable to contract a valid civil marriage under state laws. Heedless of civil law sanctions, some persons contract new marriages or enter into live-in relationships. It was precisely to provide a satisfactory solution to such anomalous situations that the Civil Law Revision Committee decided to engraft the Canon Law concept of psychological incapacity into the Family Code and classified the same as a ground for declaring marriages void ab initio or totally in existent from the beginning. A brief historical note on the Old Canon Law (1917). This Old Code, while it did not provide directly for psychological incapacity, in effect recognized the same indirectly from a combination of three old canons: 358

University of the Cordilleras College of Law First Year C S.Y. 2013 - 2014 "Canon #1081 required persons to 'be capable according to law' in order to give valid consent; Canon #1082 required that persons 'be at least not ignorant' of the major elements required in marriage; and Canon #1087 (the force and fear category) required that internal and external freedom be present in order for consent to be valid. This line of interpretation produced two distinct but related grounds for annulment, called 'lack of due discretion' and 'lack of due competence.' Lack of due discretion means that the person did not have the ability to give valid consent at the time of the weddingand therefore the union is invalid. Lack of due competence means that the person was incapable of carrying out the obligations of the promise he or she made during the wedding ceremony. "Favorable annulment decisions by the Roman Rota in the 1950s and 1960s involving sexual disorders such as homosexuality and nymphomania laid the foundation for a broader approach to the kind of proof necessary for psychological grounds for annulment. The Rota had reasoned for the first time in several cases that the capacity to give valid consent at the time of marriage was probably not present in persons who had displayed such problems shortly after the marriage. The nature of this change was nothing short of revolutionary. Once the Rota itself had demonstrated a cautious willingness to use this kind of hindsight, the way was paved for what came after 1970. Diocesan Tribunals began to accept proof of serious psychological problems that manifested themselves shortly after the ceremony as proof of an inability to give valid consent at the time of the ceremony. Furthermore, and equally significant, the professional opinion of a psychological expert became increasingly important in such cases. Data about the person's entire life, both before and after the ceremony, were presented to these experts and they were asked to give professional opinions about a party's mental at the time of the wedding. These opinions were rarely challenged and tended to be accepted as decisive evidence of lack of valid consent. The Church took pains to point out that its new openness in this area did not amount to the addition of new grounds for annulment, but rather was an accommodation by the Church to the advances made in psychology during the past decades. There was now the expertise to provide the all-important connecting link between a marriage breakdown and premarital causes. During the 1970s, the Church broadened its whole idea of marriage from that of a legal contract to that of a covenant. The result of this was that it could no longer be assumed in annulment cases that a person who could intellectually understand the concept of marriage could necessarily give valid consent to marry. The ability to both grasp and assume the real obligations of a mature, lifelong commitment are now considered a necessary prerequisite to valid matrimonial consent. Rotal decisions continued applying the concept of incipient psychological incapacity, "not only to sexual anomalies but to all kinds of personality disorders that incapacitate a spouse or both spouses from assuming or carrying out the essential obligations of marriage. For marriage . . . is not merely cohabitation or the right of the spouses to each others' body for heterosexual acts, but is, in its totality, the right to the community of the whole of life, i.e., the right to a developing. lifelong relationship. Rotal decisions since 1973 have refined the meaning of psychological or psychic capacity for marriage as presupposing the development of an adult personality; as meaning the capacity of the spouses to give themselves to each other and to accept the other as a distinct person; that the spouses must be 'other oriented' since the obligations of marriage are rooted in a self-giving love; and that the spouses must have the capacity for interpersonal relationship because marriage is more than just a physical reality but involves a true intertwining of personalities. The fulfillment of the obligations of marriage depends. according to Church decisions, on the strength of this interpersonal relationship. A serious incapacity for interpersonal sharing and support is held to impair the relationship and consequently, the ability to fulfill the essential marital obligations. The marital capacity of one spouse is not considered in isolation but in reference to the fundamental relationship to the other spouse. Fr. Green, in an article in Catholic Mind, lists six elements necessary to the mature marital relationship: The courts consider the following elements crucial to the marital commitment: (1) a permanent and faithful commitment to the marriage partner; (2) openness to children and partner; (3) stability; (4) emotional maturity; (5) financial responsibility; (6) an ability to cope with the ordinary stresses and strains of marriage, etc. Fr. Green goes on to speak about some of the psychological conditions that might lead to the failure of a marriage: 359

University of the Cordilleras College of Law First Year C S.Y. 2013 - 2014 At stake is a type of constitutional impairment precluding conjugal communion even with the best intentions of the parties. Among the psychic factors possibly giving rise to his or her inability to fulfill marital obligations are the following: (1) antisocial personality with its fundamental lack of loyalty to persons or sense of moral values; (2) hyperesthesia, where the individual has no real freedom of sexual choice; (3) the inadequate personality where personal responses consistently fallshort of reasonable expectations. xxx xxx xxx The psychological grounds are the best approach for anyone who doubts whether he or she has a case for an annulment on any other terms. A situation that does not fit into any of the more traditional categories often fits very easily into the psychological category. As new as the psychological grounds are, experts are already detecting a shift in their use. Whereas originally the emphasis was on the parties' inability to exercise proper judgment at the time of the marriage (lack of due discretion), recent cases seem to be concentrating on the parties' to assume or carry out their responsibilities an obligations as promised (lack of due competence). An advantage to using the ground of lack of due competence is that the at the time the marriage was entered intocivil divorce and breakup of the family almost is of someone's failure out marital responsibilities as promised at the time the marriage was entered into. In the instant case, "opposing and conflicting personalities" of the spouses were not considered equivalent to psychological incapacity. As well in Santos v. Court of Appeals cited in the ponencia, the Court held that the failure of the wife to return home from the U.S. or to communicate with her husband for more then five years is not proof of her psychological incapacity as to render the marriage a nullity. Therefore, Art. 36 is inapplicable and the marriages remain valid and subsisting. However in the recent case of Chi Ming Tsoi v. Court of Appeals, this Court upheld both the Regional Trial Court and the Court of Appeals in declaring the presence of psychological incapacity on the part of the husband. Said petitioner husband, after ten (10) months' sleeping with his wife never had coitus with her, a fact he did not deny but he alleged that it was due to the physical disorder of his wife which, however, he failed to prove. Goaded by the indifference and stubborn refusal of her husband to fulfill a basic marital obligation described as "to procreate children based on the universal principle that procreation of children through sexual cooperation is the basic end of marriage," the wife brought the action in the lower court to declare the marriage null. The Court, quoting Dr. Gerardo Veloso, a former Presiding Judge of the Metropolitan Marriage Tribunal of the Catholic Archdiocese of Manila (Branch I) on Psychological incapacity concluded: If a spouse, although physically capable but simply refuses to perform his or her essential marriage obligations, and the refusal is senseless and constant, Catholic marriage tribunals attribute the causes to psychological incapacity than to stubborn refusal.Senseless and protracted refusal is equivalent to psychological incapacity. Thus, the prolonged refusal of a spouse to have sexual intercourse with his or her spouse is considered a sign of psychological incapacity. We declared: This Court, finding the gravity of the failed relationship in which the parties found themselves trapped in its mire of unfulfilled vows and unconsummated marital obligations, can do no less but sustain the studied judgment of respondent appellate court. 1 concur with the majority opinion that the herein marriage remains valid and subsisting absent psychological incapacity (under Art. 36 of the Family Code) on the part of either or both of the spouses.

VITUG, J., concurring:

360

University of the Cordilleras College of Law First Year C S.Y. 2013 - 2014 I fully concur with my esteemed 'colleague Mr. Justice Artemio V. Panganiban in his ponencia, and I find to be most helpful the guidelines that he prepared for the bench and the bar in the proper appreciation of Article 36 of Executive Order No. 209 ("The Family Code of the Philippines"). The term "psychological incapacity" was neither defined nor exemplified by the Family Code. Thus Art. 36. A marriage contracted by any party who, at the time of the celebration, was psychologically incapacitated to comply with the essential marital obligations of marriage, shall likewise be void even if such incapacity becomes manifest only after its solemnization. The Revision Committee, constituted under the auspices of the U.P. Law Center, which drafted the Code explained: (T)he Committee would like the judge to interpret the provision on a case-to-case basis, guided by experience, the findings of experts and researchers in psychological disciplines, and by decisions of church tribunals which, although not binding on the civil courts, may be given persuasive effect since the provision was taken from Canon Law. Article 36 of the Family Code was concededly taken from Canon 1095 of the New Code of Canon Law Canon 1095. (The following persons) are incapable of contracting marriage; (those) 1. who lack sufficient use of reason; 2. who suffer from a grave defect of discretion of judgment concerning essential matrimonial rights and duties, to be given and accepted mutually; 3. who for causes of psychological nature are unable to assume the essential obligations of marriage that should give that much value to Canon Law jurisprudence as an aid to the interpretation and construction of the statutory enactment. The principles in the proper application of the law teach us that the several provisions of a Code must be read like a congruent whole. Thus, in determining the import of "psychological incapacity" under Article 36, one must also read it along with, albeit to be taken as distinct from, the other grounds enumerated in the Code, like Articles 35, 37, 38 and 41 that would likewise, but for distinct reasons, render the marriage merely voidable, or Article 55 that could justify a petition for legal separation. Care must be observed so that these various circumstances are not applied so indiscriminately as if the law were indifferent on the matter. I would wish to reiterate the Court's' statement in Santos vs. Court of Appeals; viz: (T)he use of the phrase "psychological incapacity" under Article 36 of the Code has not been meant to comprehend all such possible cases of psychoses as, likewise mentioned by some ecclesiastical authorities, extremely low intelligence, immaturity, and like circumstances. . . Article 36 of the Family Code cannot be taken and construed independently of, but must stand in conjunction with, existing precepts in our law on marriage. Thus correlated, "psychological incapacity" should refer to no less than a mental (not physical) incapacity that causes a party to be truly incognitive of the basic marital covenants that concomitantly must be assumed and discharged by the parties to the marriage which, as so expressed by Article 68 of the Family Code, include their mutual obligations to live together, observe love, respect and fidelity and render help and support. There is hardly any doubt that the intendment of the law has been to confine the meaning of "psychological incapacity" to the most serious cases of personality disorders clearly demonstrative of an utter insensitivity or inability of the spouse to have sexual relations with the other. This conclusion is implicit under Article 54 of the Family Code which considers children conceived prior to the judicial declaration of nullity of the void marriage to be "legitimate." 361

University of the Cordilleras College of Law First Year C S.Y. 2013 - 2014 The other forms of psychoses, if existing at the inception of marriage, like the state of a party being of unsound mind or concealment of drug addiction, habitual alcoholism, homosexuality or lesbianism, merely renders the marriage contract voidable pursuant to Article 46, Family Code. If drug addiction, habitual alcoholism, lesbianism or homosexuality should occur only during the marriage, they become mere grounds for legal separation under Article 55 of the Family Code. These provisions of the Code, however, do not necessarily preclude the possibility of these various circumstances being themselves, depending on the degree and severity of the disorder, indicia of psychological incapacity. In fine, the term "psychological incapacity," to be a ground for then nullity of marriage under Article 36 of the Family Code, must be able to pass the following tests; viz: First, the incapacity must be psychological or mental, not physical, in nature; Second, the psychological incapacity must relate to the inability, not mere refusal, to understand, assume end discharge the basic marital obligations of living together, observing love, respect and fidelity and rendering mutual help and support; Third, the psychologic condition must exist at the time the marriage is contracted although its overt manifestations and the marriage may occur only thereafter; and Fourth, the mental disorder must be grave or serious and incurable. It may well be that the Family Code Revision Committee has envisioned Article 36, as not a few observers would suspect, as another form of absolute divorce or, as still others would also put it, to be a alternative to divorce; however, the fact still remains that the language of the law has failed to carry out, even if true, any such intendment. It might have indeed turned out for the better, if it were otherwise, there could be good reasons to doubt the constitutionality of the measure. The fundamental law itself, no less, has laid down in terse language its unequivocal command on how the State should regard marriage and the family, thus Section 2, Article XV: Sec. 2. Marriage, as an inviolable social institution, is the foundation of the family and shall be protected by the State. Section 12, Article II: Sec. 12. The State recognizes the sanctity of family life and shall protect and strengthen the family as a basic autonomous social institution . . . . Section 1, Article XV: Sec. 1. The State recognizes the Filipino family as the foundation of the nation. Accordingly, it shall strengthen its solidarity and actively promote its total development. (The 1987 Constitution) The case of Marcelino vs. Cruz, 121 SCRA 51, might here be significant not so much for the specific issue there resolved but for the tone it has set. The Court there has held that constitutional provisions are to be considered mandatory unless by necessary implication, a different intention is manifest such that to have them enforced strictly would cause more harm than by disregarding them. It is quite clear to me that the constitutional mandate on marriage and the family has not been meant to be simply directory in character, nor for mere expediency or convenience, but one that demands a meaningful, not half-hearted, respect.

362

University of the Cordilleras College of Law First Year C S.Y. 2013 - 2014 Case Digest REPUBLIC vs. COURT OF APPEALS and RORIDEL OLAVIANO MOLINA G.R. No. 108763 February 13, 1997 268 SCRA 198 PANGANIBAN, J.: FACTS: The case at bar challenges the decision of CA affirming the marriage of the respondent Roridel Molina to Reynaldo Molina void in the ground of psychological incapacity. The couple got married in 1985, after a year, Reynaldo manifested signs of immaturity and irresponsibility both as husband and a father preferring to spend more time with friends whom he squandered his money, depends on his parents for aid and assistance and was never honest with his wife in regard to their finances. In 1986, the couple had an intense quarrel and as a result their relationship was estranged. Roridel quit her work and went to live with her parents in Baguio City in 1987 and a few weeks later, Reynaldo left her and their child. Since then he abandoned them. ISSUE: Whether or not the marriage is void on the ground of psychological incapacity. HELD: The marriage between Roridel and Reynaldo subsists and remains valid. What constitutes psychological incapacity is not mere showing of irreconcilable differences and confliction personalities. It is indispensable that the parties must exhibit inclinations which would not meet the essential marital responsibilities and duties due to some psychological illness. Reynaldos action at the time of the marriage did not manifest such characteristics that would comprise grounds for psychological incapacity. The evidence shown by Roridel merely showed that she and her husband cannot get along with each other and had not shown gravity of the problem neither its juridical antecedence nor its incurability. In addition, the expert testimony by Dr Sison showed no incurable psychiatric disorder but only incompatibility which is not considered as psychological incapacity. The following are the guidelines as to the grounds of psychological incapacity laid set forth in this case: burden of proof to show nullity belongs to the plaintiff root causes of the incapacity must be medically and clinically inclined such incapacity should be in existence at the time of the marriage such incapacity must be grave so as to disable the person in complying with the essentials of marital obligations of marriage such incapacity must be embraced in Art. 68-71 as well as Art 220, 221 and 225 of the Family Code decision of the National Matrimonial Appellate Court or the Catholic Church must be respected court shall order the prosecuting attorney and the fiscal assigned to it to act on behalf of the state.

363

University of the Cordilleras College of Law First Year C S.Y. 2013 - 2014 Hernandez vs Court of Appeals 320 SCRA 76 G.R. No. 126010. December 8, 1999 Full Case LUCITA ESTRELLA HERNANDEZ, petitioner vs COURT OF APPEALS and MARIO C. HERNANDEZ, respondents. MENDOZA, J.: This is a petition for review on certiorari of the decision of the Court of Appeals, dated January 30, 1996, affirming the decision of the Regional Trial Court, Branch 18, Tagaytay City, dated April 10, 1993, which dismissed the petition for annulment of marriage filed by petitioner. Petitioner Lucita Estrella Hernandez and private respondent Mario C. Hernandez were married at the Silang Catholic Parish Church in Silang, Cavite on January 1, 1981 (Exh. A). Three children were born to them, namely, Maie, who was born on May 3, 1982 (Exh. B), Lyra, born on May 22, 1985 (Exh. C), and Marian, born on June 15, 1989 (Exh. D). On July 10, 1992, petitioner filed before the Regional Trial Court, Branch 18, Tagaytay City, a petition seeking the annulment of her marriage to private respondent on the ground of psychological incapacity of the latter. She alleged that from the time of their marriage up to the time of the filing of the suit, private respondent failed to perform his obligation to support the family and contribute to the management of the household, devoting most of his time engaging in drinking sprees with his friends. She further claimed that private respondent, after they were married, cohabited with another woman with whom he had an illegitimate child, while having affairs with different women, and that, because of his promiscuity, private respondent endangered her health by infecting her with a sexually transmissible disease (STD). She averred that private respondent was irresponsible, immature and unprepared for the duties of a married life. Petitioner prayed that for having abandoned the family, private respondent be ordered to give support to their three children in the total amount ofP9,000.00 every month; that she be awarded the custody of their children; and that she be adjudged as the sole owner of a parcel of land located at Don Gregorio Subdivision I in Bo. Bucal, Dasmarias, Cavite, purchased during the marriage, as well as the jeep which private respondent took with him when he left the conjugal home on June 12, 1992. On October 8, 1992, because of private respondents failure to file his answer, the trial court issued an order directing the assistant provincial prosecutor to conduct an investigation to determine if there was collusion between the parties. Only petitioner appeared at the investigation on November 5, 1992. Nevertheless, the prosecutor found no evidence of collusion and recommended that the case be set for trial. Based on the evidence presented by the petitioner, the facts are as follows: Petitioner and private respondent met in 1977 at the Philippine Christian University in Dasmarias, Cavite. Petitioner, who is five years older than private respondent, was then in her first year of teaching zoology and botany. Private respondent, a college freshman, was her student for two consecutive semesters. They became sweethearts in February 1979 when she was no longer private respondents teacher. On January 1, 1981, they were married. Private respondent continued his studies for two more years. His parents paid for his tuition fees, while petitioner provided his allowances and other financial needs. The family income came from petitioners salary as a faculty member of the Philippine Christian University. Petitioner augmented her earnings by selling Tupperware products, as well as engaging in the buy-and-sell of coffee, rice and polvoron. From 1983 up to 1986, as private respondent could not find a stable job, it was agreed that he would help petitioner in her businesses by delivering orders to customers. However, because her husband was a spendthrift and had other women, petitioners business suffered. Private respondent often had smoking 364

University of the Cordilleras College of Law First Year C S.Y. 2013 - 2014 and drinking sprees with his friends and betted on fighting cocks. In 1982, after the birth of their first child, petitioner discovered two love letters written by a certain Realita Villena to private respondent. She knew Villena as a married student whose husband was working in Saudi Arabia. When petitioner confronted private respondent, he admitted having an extra-marital affair with Villena. Petitioner then pleaded with Villena to end her relationship with private respondent. For his part, private respondent said he would end the affairs, but he did not keep his promise. Instead, he left the conjugal home and abandoned petitioner and their child. When private respondent came back, however, petitioner accepted him, despite private respondents infidelity in the hope of saving their marriage. Upon the recommendation of a family friend, private respondent was able to get a job at Reynolds Philippines, Inc. in San Agustin, Dasmarias, Cavite in 1986. However, private respondent was employed only until March 31, 1991, because he availed himself of the early retirement plan offered by the company. He received P53,000.00 in retirement pay, but instead of spending the amount for the needs of the family, private respondent spent the money on himself and consumed the entire amount within four months of his retirement. While private respondent worked at Reynolds Philippines, Inc., his smoking, drinking, gambling and womanizing became worse. Petitioner discovered that private respondent carried on relationships with different women. He had relations with a certain Edna who worked at Yazaki; Angie, who was an operator of a billiard hall; Tess, a Japayuki; Myrna Macatangay, a secretary at the Road Master Drivers School in Bayan, Dasmarias, Cavite, with whom he cohabited for quite a while; and, Ruth Oliva, by whom he had a daughter named Margie P. Oliva, born on September 15, 1989 (Exh. E). When petitioner confronted private respondent about his relationship with Tess, he beat her up, as a result of which she was confined at the De la Salle University Medical Center in Dasmarias, Cavite on July 4-5, 1990 because of cerebral concussion (Exh. F). According to petitioner, private respondent engaged in extreme promiscuous conduct during the latter part of 1986. As a result, private respondent contracted gonorrhea and infected petitioner. They both received treatment at the Zapote Medical Specialists Center in Zapote, Bacoor, Cavite from October 22, 1986 until March 13, 1987 (Exhs. G & H). Petitioner averred that on one occasion of a heated argument, private respondent hit their eldest child who was then barely a year old. Private respondent is not close to any of their children as he was never affectionate and hardly spent time with them. On July 17, 1979, petitioner entered into a contract to sell (Exh. J) with F & C Realty Corporation whereby she agreed to buy from the latter a parcel of land at the Don Gregorio Heights Subdivision I in Bo. Bucal, Dasmarias, Cavite and placed a partial payment of P31,330.00. On May 26, 1987, after full payment of the amount of P51,067.10, inclusive of interests from monthly installments, a deed of absolute sale (Exh. K)was executed in her favor and TCT No. T-221529 (Exh. M) was duly issued. According to petitioner, on August 1, 1992, she sent a handwritten letter to private respondent expressing her frustration over the fact that her efforts to save their marriage proved futile. In her letter, petitioner also stated that she was allowing him to sell their owner-type jeepney and to divide the proceeds of the sale between the two of them. Petitioner also told private respondent of her intention to file a petition for the annulment of their marriage. It does not appear that private respondent ever replied to petitioners letter. By this time, he had already abandoned petitioner and their children. In October 1992, petitioner learned that private respondent left for the Middle East. Since then, private respondents whereabouts had been unknown. Ester Alfaro, petitioners childhood friend and co-teacher at the Philippine Christian University, testified during the hearing on the petition for annulment. She said that sometime in June 1979, petitioner introduced private respondent to her (Alfaro) as the formers sweetheart. Alfaro said she was not impressed with private respondent who was her student in accounting. She observed private respondent to be fun-loving, spending most of his time with campus friends. In November 1980, when petitioner 365

University of the Cordilleras College of Law First Year C S.Y. 2013 - 2014 asked Alfaro to be one of the secondary sponsors at her forthcoming wedding, Alfaro wanted to dissuade petitioner from going through with the wedding because she thought private respondent was not ready for married life as he was then unemployed. True enough, although the couple appeared happy during the early part of their marriage, it was not long thereafter that private respondent started drinking with his friends and going home late at night. Alfaro corroborated petitioners claim that private respondent was a habitual drunkard who carried on relationships with different women and continued hanging out with his friends. She also confirmed that petitioner was once hospitalized because she was beaten up by private respondent. After the first year of petitioners marriage, Alfaro tried to talk to private respondent, but the latter accused her of meddling with their marital life. Alfaro said that private respondent was not close to his children and that he had abandoned petitioner. On April 10, 1993, the trial court rendered a decision dismissing the petition for annulment of marriage filed by petitioner. The pertinent portion of the decision reads: The Court can underscore the fact that the circumstances mentioned by the petitioner in support of her claim that respondent was psychologically incapacitated to marry her are among the grounds cited by the law as valid reasons for the grant of legal separation (Article 55 of the Family Code) - not as grounds for a declaration of nullity of marriages or annulment thereof. Thus, Article 55 of the same code reads as follows: Art. 55. A petition for legal separation may be filed on any of the following grounds: (1) Repeated physical violence or grossly abusive conduct directed against the petitioner, a common child, or a child of the petitioner; .... (5) Drug addiction or habitual alcoholism of the respondent; .... (8) Sexual infidelity or perversion; .... (10) Abandonment of petitioner by respondent without justifiable cause for more than one year. .... If indeed Article 36 of the Family Code of the Philippines, which mentions psychological incapacity as a ground for the declaration of the nullity of a marriage, has intended to include the above-stated circumstances as constitutive of such incapacity, then the same would not have been enumerated as grounds for legal separation. In the same manner, this Court is not disposed to grant relief in favor of the petitioner under Article 46, paragraph (3) of the Family Code of the Philippines, as there is no dispute that the gonorrhea transmitted to the petitioner by respondent occurred sometime in 1986, or five (5) years after p etitioners marriage with respondent was celebrated in 1981. The provisions of Article 46, paragraph (3) of the same law should be taken in conjunction with Article 45, paragraph (3) of the same code, and a careful reading of the two (2) provisions of the law would require the existence of this ground (fraud) at the time of the celebration of the marriage. Hence, the annulment of petitioners marriage with the respondent on this ground, as alleged and proved in the instant case, cannot be legally accepted by the Court. Petitioner appealed to the Court of Appeals which, on January 30, 1996, rendered its decision affirming the decision of the trial court. Citing the ruling in Santos v. Court of Appeals, the Court of Appeals held: It is clear in the above law and jurisprudence that the psychological incapacity of a spouse, as a ground for declaration of nullity of marriage, must exist at the time of the celebration of marriage. More so, 366

University of the Cordilleras College of Law First Year C S.Y. 2013 - 2014 chronic sexual infidelity, abandonment, gambling and use of prohibited drugs are not grounds per se, of psychological incapacity of a spouse. We agree with the Solicitor General that petitioner-appellant failed to prove that her respondent-husband was psychologically incapacitated at the time of the celebration of the marriage. Certainly, petitionerappellants declaration that at the time of their marriage her respondent-husbands character was on the borderline between a responsible person and the happy-go-lucky, could not constitute the psychological incapacity in contemplation of Article 36 of the Family Code. In fact, petitioner-appellant herself ascribed said attitude to her respondent-husbands youth and very good looks, who was admittedly several years younger than petitioner-appellant who, herself, happened to be the college professor of her respondent-husband. Petitioner-appellant even described her respondent-husband not as a problem student but a normal one (p. 24, tsn, Dec. 8, 1992). The acts and attitudes complained of by petitioner-appellant happened after the marriage and there is no proof that the same have already existed at the time of the celebration of the marriage to constitute the psychological incapacity under Article 36 of the Family Code. Hence, this petition. Petitioner contends that the respondent Court of Appeals erred I. IN FINDING THAT THE PSYCHOLOGICAL INCAPACITY OF THE PRIVATE RESPONDENT TO COMPLY WITH HIS ESSENTIAL MARITAL OBLIGATIONS DID NOT EXIST FROM THE TIME OF THE CELEBRATION OF THE MARRIAGE. II. IN RULING THAT PRIVATE RESPONDENT WAS NOT PSYCHOLOGICALLY INCAPACITATED TO COMPLY WITH HIS ESSENTIAL MARITAL OBLIGATIONS. III. IN AFFIRMING THE DECISION OF THE TRIAL COURT DENYING THE AWARD OF PERMANENT CUSTODY OF THE CHILDREN TO PETITIONER. IV. IN AFFIRMING THE DECISION OF THE TRIAL COURT DENYING THE PRAYER FOR ISSUANCE OF AN ORDER REQUIRING PRIVATE RESPONDENT TO GIVE SUPPORT TO THE THREE CHILDREN IN THE AMOUNT OF P3,000.00 PER CHILD. V. IN NOT DECLARING THE REAL PROPERTY ACQUIRED BY PETITIONER AS HER EXCLUSIVE PROPERTY. The issue in this case is whether or not the marriage of petitioner and private respondent should be annulled on the ground of private respondents psychological incapacity. Petitioner alleges that the Court of Appeals erred in holding that petitioner failed to show that private respondents psychological incapacity existed at the time of the celebration of the marriage. She argues that the fact that the acts of incapacity of private respondent became manifest only after the celebration of their marriage should not be a bar to the annulment of their marriage. Art. 36 of the Family Code states: A marriage contracted by any party who, at the time of the celebration, was psychologically incapacitated to comply with the essential marital obligations of marriage, shall likewise be void even if such incapacity becomes manifest only after its solemnization. In Santos v. Court of Appeals, we held: Psychological incapacity should refer to no less than a mental (not physical) incapacity that causes a party to be truly in cognitive of the basic marital covenants that concomitantly must be assumed and discharged by the parties to the marriage which, as so expressed by Article 68 of the Family Code, include their mutual obligations to live together, observe love, respect and fidelity and render help and support. There is hardly any doubt that the intendment of the law has been to confine the meaning of psychological incapacity to the most serious cases of personality disorders clearly demonstrative of an 367

University of the Cordilleras College of Law First Year C S.Y. 2013 - 2014 utter insensitivity or inability to give meaning and significance to the marriage. This psychological condition must exist at the time the marriage is celebrated. The law does not evidently envision, upon the other hand, an inability of the spouse to have sexual relations with the other. This conclusion is implicit under Article 54 of the Family Code which considers children conceived prior to the judicial declaration of nullity of the void marriage to be legitimate. The other forms of psychoses, if existing at the inception of marriage, like the state of a party being of unsound mind or concealment of drug addiction, habitual alcoholism, homosexuality or lesbianism, merely renders the marriage contract voidable pursuant to Article 46, Family Code. If drug addiction, habitual alcoholism, lesbianism or homosexuality should occur only during the marriage, they become mere grounds for legal separation under Article 55 of the Family Code. These provisions of the Code, however, do not necessarily preclude the possibility of these various circumstances being themselves, depending on the degree and severity of the disorder, indicia of psychological incapacity. Until further statutory and jurisprudential parameters are established, every circumstance that may have some bearing on the degree, extent, and other conditions of that incapacity must, in every case, be carefully examined and evaluated so that no precipitate and indiscriminate nullity is peremptorily decreed. The well-considered opinions of psychiatrists, psychologists, and persons with expertise in psychological disciplines might be helpful or even desirable. In the instant case, other than her self-serving declarations, petitioner failed to establish the fact that at the time they were married, private respondent was suffering from a psychological defect which in fact deprived him of the ability to assume the essential duties of marriage and its concomitant responsibilities. As the Court of Appeals pointed out, no evidence was presented to show that private respondent was not cognizant of the basic marital obligations. It was not sufficiently proved that private respondent was really incapable of fulfilling his duties due to some incapacity of a psychological nature, and not merely physical. Petitioner says that at the outset of their marriage, private respondent showed lack of drive to work for his family. Private respondents parents and petitioner supported him through college. After his schooling, although he eventually found a job, he availed himself of the early retirement plan offered by his employer and spent the entire amount he received on himself. For a greater part of their marital life, private respondent was out of job and did not have the initiative to look for another. He indulged in vices and engaged in philandering, and later abandoned his family. Petitioner concludes that private respondents condition is incurable, causing the disintegration of their un ion and defeating the very objectives of marriage. However, private respondents alleged habitual alcoholism, sexual infidelity or perversion, and abandonment do not by themselves constitute grounds for finding that he is suffering from a psychological incapacity within the contemplation of the Family Code. It must be shown that these acts are manifestations of a disordered personality which make private respondent completely unable to discharge the essential obligations of the marital state, and not merel y due to private respondents youth and self-conscious feeling of being handsome, as the appellate court held. As pointed out in Republic of the Philippines v. Court of Appeals: The root cause of the psychological incapacity must be: (a) medically or clinically identified, (b) alleged in the complaint, (c) sufficiently proven by experts and (d) clearly explained in the decision. Article 36 of the Family Code requires that the incapacity must be psychological not physical, although its manifestations and/or symptoms may be physical. The evidence must convince the court that the parties, or one of them, was mentally or physically ill to such an extent that the person could not have known the obligations he was assuming, or knowing them, could not have given valid assumption thereof. Although no example of such incapacity need be given here so as not to limit the application of the provision under the principle of ejusdem generis (citing Salita v. Magtolis, supra) nevertheless such root cause must be identified as a psychological illness and its incapacitating nature fully explained. Expert evidence may be given by qualified psychiatrists and clinical psychologists. Moreover, expert testimony should have been presented to establish the precise cause of private respondents psychological incapacity, if any, in order to show that it existed at the inception of the 368

University of the Cordilleras College of Law First Year C S.Y. 2013 - 2014 marriage. The burden of proof to show the nullity of the marriage rests upon petitioner. The Court is mindful of the policy of the 1987 Constitution to protect and strengthen the family as the basic autonomous social institution and marriage as the foundation of the family. Thus, any doubt should be resolved in favor of the validity of the marriage. We, therefore, find no reason to reverse the ruling of respondent Court of Appeals whose conclusions, affirming the trial courts finding with regard to the non-existence of private respondents psychological incapacity at the time of the marriage, are entitled to great weight and even finality. Only where it is shown that such findings are whimsical, capricious, and arbitrary can these be overturned. The conclusion we have reached makes it unnecessary for us to pass upon petitioners contentions on the issue of permanent custody of children, the amount for their respective support, and the declaration of exclusive ownership of petitioner over the real property. These matters may more appropriately be litigated in a separate proceeding for legal separation, dissolution of property regime, and/or custody of children which petitioner may bring. WHEREFORE, the decision of the Court of Appeals is AFFIRMED. SO ORDERED. Bellosillo (Chairman), Quisumbing, Buena, and De Leon, Jr., JJ., concur.

369

University of the Cordilleras College of Law First Year C S.Y. 2013 - 2014 Case Digest LUCITA ESTRELLA HERNANDEZ vs. COURT OF APPEALS and MARIO C. HERNANDEZ G.R. No. 126010. December 8, 1999 320 SCRA 76 MENDOZA, J.: Facts: Petitioner Lucita Estrella Hernandez and private respondent Mario C. Hernandez were married at the Silang Catholic Parish Church in Silang, Cavite on January 1, 1981.Three children were born to them, namely, Maie, who was born on May 3, 1982, Lyra, born on May 22, 1985, and Marian, born on June 15, 1989. On July 10, 1992, petitioner filed before the Regional Trial Court, Branch 18, Tagaytay City, a petition seeking the annulment of her marriage to private respondent on the ground of psychological incapacity of the latter. She alleged that from the time of their marriage up to the time of the filing of the suit, private respondent failed to perform his obligation to support the family and contribute to the management of the household, devoting most of his time engaging in drinking sprees with his friends. She further claimed that private respondent, after they were married, cohabited with another woman with whom he had an illegitimate child, while having affairs with different women, and that, because of his promiscuity, private respondent endangered her health by infecting her with a sexually transmissible disease (STD). She averred that private respondent was irresponsible, immature and unprepared for the duties of a married life. Petitioner prayed that for having abandoned the family, private respondent be ordered to give support to their three children in the total amount ofP9,000.00 every month; that she be awarded the custody of their children; and that she be adjudged as the sole owner of a parcel of land located at Don Gregorio Subdivision I in Bo. Bucal, Dasmarias, Cavite, purchased during the marriage, as well as the jeep which private respondent took with him when he left the conjugal home on June 12, 1992. Issue: Whether or not the marriage of petitioner and private respondent should be annulled on the ground of private respondents psychological incapacity. Ruling: In Santos vs. Court of Appeals Psychological incapacity should refer to no less than a mental (not physical) incapacity that causes a party to be truly in cognitive of the basic marital covenants that concomitantly must be assumed and discharged by the parties to the marriage which, as so expressed by Article 68 of the Family Code, include their mutual obligations to live together, observe love, respect and fidelity and render help and support. There is hardly any doubt that the intendment of the law has been to confine the meaning of psychological incapacity to the most serious cases of personality disorders clearly demonstrative of an utter insensitivity or inability to give meaning and significance to the marriage. This psychological condition must exist at the time the marriage is celebrated. The law does not evidently envision, upon the other hand, an inability of the spouse to have sexual relations with the other. This conclusion is implicit under Article 54 of the Family Code which considers children conceived prior to the judicial declaration of nullity of the void marriage to be legitimate. In the instant case, other than her self-serving declarations, petitioner failed to establish the fact that at the time they were married, private respondent was suffering from a psychological defect which in fact deprived him of the ability to assume the essential duties of marriage and its concomitant responsibilities. As the Court of Appeals pointed out, no evidence was presented to show that private respondent was not cognizant of the basic marital obligations. It was not sufficiently proved that private respondent was really incapable of fulfilling his duties due to some incapacity of a psychological nature, and not merely physical. Petitioner says that at the outset of their marriage, private respondent showed lack of drive to work for his family. Private respondents parents and petitioner supported him through college. After his schooling, although he eventually found a job, he availed himself of the early retirement plan offered by his employer and spent the entire amount he received on himself. For a greater part of their marital life, private respondent was out of job and did not have the initiative to look for another. He indulged in vices and engaged in philandering, and later abandoned his family. Petitioner concludes that private respondents condition is incurable, causing the disintegration of their union and defeating the very objectives of marriage.

370

University of the Cordilleras College of Law First Year C S.Y. 2013 - 2014 Marcos vs Marcos 343 SCRA 755 G.R. No. 136490. October 19, 2000 Full Case BRENDA B. MARCOS, petitioner, vs. WILSON G. MARCOS, respondent. PANGANIBAN, J.: Psychological incapacity, as a ground for declaring the nullity of a marriage, may be established by the totality of evidence presented. There is no requirement, however, that the respondent should be examined by a physician or a psychologist as a conditio sine qua non for such declaration. The Case Before us is a Petition for Review on Certiorari under Rule 45 of the Rules of Court, assailing the July 24, 1998 Decision of the Court of Appeals (CA) in CA-GR CV No. 55588, which disposed as follows: "WHEREFORE, the contested decision is set aside and the marriage between the parties is hereby declared valid." Also challenged by petitioner is the December 3, 1998 CA Resolution denying her Motion for Reconsideration. Earlier, the Regional Trial Court (RTC) had ruled thus: "WHEREFORE, the marriage between petitioner Brenda B. Marcos and respondent Wilson G. Marcos, solemnized on September 6, 1982 in Pasig City is declared null and void ab initio pursuant to Art. 36 of the Family Code. The conjugal properties, if any, is dissolved [sic] in accordance with Articles 126 and 129 of the same Code in relation to Articles 50, 51 and 52 relative to the delivery of the legitime of [the] parties' children. In the best interest and welfare of the minor children, their custody is granted to petitioner subject to the visitation rights of respondent. "Upon finality of this Decision, furnish copy each to the Office of the Civil Registrar of Pasig City where the marriage was solemnized, the National Census and Statistics Office, Manila and the Register of Deeds of Mandaluyong City for their appropriate action consistent with this Decision. "SO ORDERED." The Facts The facts as found by the Court of Appeals are as follows: "It was established during the trial that the parties were married twice: (1) on September 6, 1982 which was solemnized by Judge Eriberto H. Espiritu at the Municipal Court of Pasig (Exh. A); and (2) on May 8, 1983 which was solemnized by Rev. Eduardo L. Eleazar, Command Chaplain, at the Presidential Security Command Chapel in Malacaang Park, Manila (Exh. A-1). Out of their marriage, five (5) children were born (Exhs. B, C, D, E and F). "Appellant Wilson G. Marcos joined the Armed Forces of the Philippines in 1973. Later on, he was transferred to the Presidential Security Command in Malacaang during the Marcos Regime. Appellee Brenda B. Marcos, on the other hand, joined the Women's Auxilliary Corps under the Philippine Air Force in 1978. After the Edsa Revolution, both of them sought a discharge from the military service. "They first met sometime in 1980 when both of them were assigned at the Malacaang Palace, she as an escort of Imee Marcos and he as a Presidential Guard of President Ferdinand Marcos. Through telephone conversations, they became acquainted and eventually became sweethearts. 371

University of the Cordilleras College of Law First Year C S.Y. 2013 - 2014 "After their marriage on September 6, 1982, they resided at No. 1702 Daisy Street, Hulo Bliss, Mandaluyong, a housing unit which she acquired from the Bliss Development Corporation when she was still single. "After the downfall of President Marcos, he left the military service in 1987 and then engaged in different business ventures that did not however prosper. As a wife, she always urged him to look for work so that their children would see him, instead of her, as the head of the family and a good provider. Due to his failure to engage in any gainful employment, they would often quarrel and as a consequence, he would hit and beat her. He would even force her to have sex with him despite her weariness. He would also inflict physical harm on their children for a slight mistake and was so severe in the way he chastised them. Thus, for several times during their cohabitation, he would leave their house. In 1992, they were already living separately. "All the while, she was engrossed in the business of selling "magic uling" and chickens. While she was still in the military, she would first make deliveries early in the morning before going to Malacaang. When she was discharged from the military service, she concentrated on her business. Then, she became a supplier in the Armed Forces of the Philippines until she was able to put up a trading and construction company, NS Ness Trading and Construction Development Corporation. "The 'straw that broke the camel's back' took place on October 16, 1994, when they had a bitter quarrel. As they were already living separately, she did not want him to stay in their house anymore. On that day, when she saw him in their house, she was so angry that she lambasted him. He then turned violent, inflicting physical harm on her and even on her mother who came to her aid. The following day, October 17, 1994, she and their children left the house and sought refuge in her sister's house. "On October 19, 1994, she submitted herself [to] medical examination at the Mandaluyong Medical Center where her injuries were diagnosed as contusions (Exh. G, Records, 153). "Sometime in August 1995, she together with her two sisters and driver, went to him at the Bliss unit in Mandaluyong to look for their missing child, Niko. Upon seeing them, he got mad. After knowing the reason for their unexpected presence, he ran after them with a samurai and even [beat] her driver. "At the time of the filing of this case, she and their children were renting a house in Camella, Paraaque, while the appellant was residing at the Bliss unit in Mandaluyong. "In the case study conducted by Social Worker Sonia C. Millan, the children described their father as cruel and physically abusive to them (Exh. UU, Records, pp. 85-100). "The appellee submitted herself to psychologist Natividad A. Dayan, Ph.D., for psychological evaluation (Exh. YY, Records, pp. 207-216), while the appellant on the other hand, did not. "The court a quo found the appellant to be psychologically incapacitated to perform his marital obligations mainly because of his failure to find work to support his family and his violent attitude towards appellee and their children, x x x." Ruling of the Court of Appeals Reversing the RTC, the CA held that psychological incapacity had not been established by the totality of the evidence presented. It ratiocinated in this wise: "Essential in a petition for annulment is the allegation of the root cause of the spouse's psychological incapacity which should also be medically or clinically identified, sufficiently proven by experts and clearly explained in the decision. The incapacity must be proven to be existing at the time of the celebration of the marriage and shown to be medically or clinically permanent or incurable. It must also be grave enough to bring about the disability of the parties to assume the essential obligations of marriage as set forth in Articles 68 to 71 and Articles 220 to 225 of the Family Code and such non-complied marital obligations must similarly be alleged in the petition, established by evidence and explained in the decision. 372

University of the Cordilleras College of Law First Year C S.Y. 2013 - 2014 "In the case before us, the appellant was not subjected to any psychological or psychiatric evaluation. The psychological findings about the appellant by psychiatrist Natividad Dayan were based only on the interviews conducted with the appellee. Expert evidence by qualified psychiatrists and clinical psychologists is essential if only to prove that the parties were or any one of them was mentally or psychically ill to be truly incognitive of the marital obligations he or she was assuming, or as would make him or her x x x unable to assume them. In fact, he offered testimonial evidence to show that he [was] not psychologically incapacitated. The root cause of his supposed incapacity was not alleged in the petition, nor medically or clinically identified as a psychological illness or sufficiently proven by an expert. Similarly, there is no evidence at all that would show that the appellant was suffering from an incapacity which [was] psychological or mental - not physical to the extent that he could not have known the obligations he was assuming: that the incapacity [was] grave, ha[d] preceded the marriage and [was] incurable." Hence, this Petition. Issues In her Memorandum,petitioner presents for this Court's consideration the following issues: "I. Whether or not the Honorable Court of Appeals could set aside the findings by the Regional Trial Court of psychological incapacity of a respondent in a Petition for declaration of nullity of marriage simply because the respondent did not subject himself to psychological evaluation. II. Whether or not the totality of evidence presented and the demeanor of all the witnesses should be the basis of the determination of the merits of the Petition." The Court's Ruling We agree with petitioner that the personal medical or psychological examination of respondent is not a requirement for a declaration of psychological incapacity. Nevertheless, the totality of the evidence she presented does not show such incapacity. Preliminary Issue: Need for Personal Medical Examination Petitioner contends that the testimonies and the results of various tests that were submitted to determine respondent's psychological incapacity to perform the obligations of marriage should not have been brushed aside by the Court of Appeals, simply because respondent had not taken those tests himself. Petitioner adds that the CA should have realized that under the circumstances, she had no choice but to rely on other sources of information in order to determine the psychological capacity of respondent, who had refused to submit himself to such tests. In Republic v. CA and Molina, the guidelines governing the application and the interpretation of psychological incapacity referred to in Article 36 of the Family Code were laid down by this Court as follows: "1) The burden of proof to show the nullity of the marriage belongs to the plaintiff. Any doubt should be resolved in favor of the existence and continuation of the marriage and against its dissolution and nullity. This is rooted in the fact that both our Constitution and our laws cherish the validity of marriage and unity of the family. Thus, our Constitution devotes an entire Article on the Family, recognizing it 'as the foundation of the nation.' It decrees marriage as legally 'inviolable,' thereby protecting it from dissolution at the whim of the parties. Both the family and marriage are to be 'protected' by the state. xxxxxxxxx 2) The root cause of the psychological incapacity must be: (a) medically or clinically identified, (b) alleged in the complaint, (c) sufficiently proven by experts and (d) clearly explained in the decision.Article 36 of the Family Code requires that the incapacity must be psychological - not physical, although its manifestations and/or symptoms may be physical. The evidence must convince the court that 373

University of the Cordilleras College of Law First Year C S.Y. 2013 - 2014 the parties, or one of them, was mentally or psychically ill to such an extent that the person could not have known the obligations he was assuming, or knowing them, could not have given valid assumption thereof. Although no example of such incapacity need be given here so as not to limit the application of the provision under the principle of ejusdem generis, nevertheless such root cause must be identified as a psychological illness and its incapacitating nature fully explained. Expert evidence may be given by qualified psychiatrists and clinical psychologists. 3) The incapacity must be proven to be existing at 'the time of the celebration' of the marriage. The evidence must show that the illness was existing when the parties exchanged their 'I do's.' The manifestation of the illness need not be perceivable at such time, but the illness itself must have attached at such moment, or prior thereto. 4) Such incapacity must also be shown to be medically or clinically permanent or incurable. Such incurability may be absolute or even relative only in regard to the other spouse, not necessarily absolutely against everyone of the same sex. Furthermore, such incapacity must be relevant to the assumption of marriage obligations, not necessarily to those not related to marriage, like the exercise of a profession or employment in a job. Hence, a pediatrician may be effective in diagnosing illnesses of children and prescribing medicine to cure them but not be psychologically capacitated to procreate, bear and raise his/her own children as an essential obligation of marriage. 5) Such illness must be grave enough to bring about the disability of the party to assume the essential obligations of marriage. Thus, 'mild characteriological peculiarities, mood changes, occasional emotional outbursts cannot be accepted as root causes. The illness must be shown as downright incapacity or inability, not a refusal, neglect or difficulty, much less ill will. In other words, there is a natal or supervening disabling factor in the person, an adverse integral element in the personality structure that effectively incapacitates the person from really accepting and thereby complying with the obligations essential to marriage. 6) The essential marital obligations must be those embraced by Articles 68 up to 71 of the Family Code as regards the husband and wife as well as Articles 220, 221 and 225 of the same Code in regard to parents and their children. Such non-complied marital obligation(s) must also be stated in the petition, proven by evidence and included in the text of the decision. 7) Interpretations given by the National Appellate Matrimonial Tribunal of the Catholic Church in the Philippines, while not controlling or decisive, should be given great respect by our courts. xxxxxxxxx (8) The trial court must order the prosecuting attorney or fiscal and the Solicitor General to appear as counsel for the state. No decision shall be handed down unless the Solicitor General issues a certification, which will be quoted in the decision, briefly stating therein his reasons for his agreement or opposition, as the case may be, to the petition. The Solicitor General, along with the prosecuting attorney, shall submit to the court such certification within fifteen (15) days from the date the case is deemed submitted for resolution of the court. The Solicitor General shall discharge the equivalent function of the defensor vinculi contemplated under Canon 1095." The guidelines incorporate the three basic requirements earlier mandated by the Court in Santos v. Court of Appeals: "psychological incapacity must be characterized by (a) gravity (b) juridical antecedence, and (c) incurability." The foregoing guidelines do not require that a physician examine the person to be declared psychologically incapacitated. In fact, the root cause may be "medically or clinically identified." What is important is the presence of evidence that can adequately establish the party's psychological condition. For indeed, if the totality of evidence presented is enough to sustain a finding of psychological incapacity, then actual medical examination of the person concerned need not be resorted to. Main Issue: Totality of Evidence Presented

374

University of the Cordilleras College of Law First Year C S.Y. 2013 - 2014 The main question, then, is whether the totality of the evidence presented in the present case -- including the testimonies of petitioner, the common children, petitioner's sister and the social worker -- was enough to sustain a finding that respondent was psychologically incapacitated. We rule in the negative. Although this Court is sufficiently convinced that respondent failed to provide material support to the family and may have resorted to physical abuse and abandonment, the totality of his acts does not lead to a conclusion of psychological incapacity on his part. There is absolutely no showing that his "defects" were already present at the inception of the marriage or that they are incurable. Verily, the behavior of respondent can be attributed to the fact that he had lost his job and was not gainfully employed for a period of more than six years. It was during this period that he became intermittently drunk, failed to give material and moral support, and even left the family home. Thus, his alleged psychological illness was traced only to said period and not to the inception of the marriage. Equally important, there is no evidence showing that his condition is incurable, especially now that he is gainfully employed as a taxi driver. Article 36 of the Family Code, we stress, is not to be confused with a divorce law that cuts the marital bond at the time the causes therefor manifest themselves. It refers to a serious psychological illness afflicting a party even before the celebration of the marriage. It is a malady so grave and so permanent as to deprive one of awareness of the duties and responsibilities of the matrimonial bond one is about to assume. These marital obligations are those provided under Articles 68 to 71, 220, 221 and 225 of the Family Code. Neither is Article 36 to be equated with legal separation, in which the grounds need not be rooted in psychological incapacity but on physical violence, moral pressure, moral corruption, civil interdiction, drug addiction, habitual alcoholism, sexual infidelity, abandonment and the like. At best, the evidence presented by petitioner refers only to grounds for legal separation, not for declaring a marriage void. Because Article 36 has been abused as a convenient divorce law, this Court laid down the procedural requirements for its invocation in Molina. Petitioner, however, has not faithfully observed them. In sum, this Court cannot declare the dissolution of the marriage for failure of petitioner to show that the alleged psychological incapacity is characterized by gravity, juridical antecedence and incurability; and for her failure to observe the guidelines outlined in Molina. WHEREFORE, the Petition is DENIED and assailed Decision AFFIRMED, except that portion requiring personal medical examination as a conditio sine qua non to a finding of psychological incapacity. No costs. SO ORDERED. Melo, (Chairman), Vitug, Purisima, and Gonzaga-Reyes, JJ., concur.

375

University of the Cordilleras College of Law First Year C S.Y. 2013 - 2014 Case Digest BRENDA B. MARCOS vs. WILSON G. MARCOS G.R. No. 136490. October 19, 2000 343 SCRA 755 PANGANIBAN, J.: Facts: Petitioner Brenda Marcos and Respondent Wilson Marcos were married twice and had five children. After the downfall of President Marcos, the respondent left the military service in1987. Consequently, due to the respondents failure to engage in any gainful employment, they would often quarrel and the respondent would hit and beat the petitioner. As a result, in 1992they were already living separately. Thus, petitioner filed for annulment of marriage assailing A r t . 3 6 o f t h e F a m i l y C o d e . T h e c o u r t a q u o f o u n d t h e r e s p o n d e n t t o b e p s y c h o l o g i c a l l y incapacitated to perform his marital obligations. However, the Court of Appeals reversed the decision of the RTC because psychological incapacity had not been established by the totality of the evidence presented. Issues: Whether personal medical or psychological examination of the respondent by a physician is a requirement for a declaration of psychological incapacity. Whether the totality of evidence presented in this case show psychological incapacity. Held: Psychological incapacity as a ground for declaring the nullity of a marriage, may be established by the totality of evidence presented. There is no requirement, however that the respondent be examined by a physician or a psychologist as a condition sine qua non for such declaration. Although this Court is sufficiently convinced that respondent failed to provide material support to the family and may have resorted to physical abuse and abandonment, the totality of his acts does not lead to a conclusion of psychological incapacity on his part. There is absolutely no showing that his defects were already present at the inception of the marriage or that they are incurable. Verily, the behavior of respondent can be attributed to the fact that he had lost his job and was not gainfully employed for a period of more than six years. It was during this period that he became intermittently drunk, failed to give material and moral support, and even left the family home. Thus, his alleged psychological illness was traced only to said period and not to the inception of the marriage. Equally important, there is no evidence showing that his condition is incurable, especially now that he is gainfully employed as a taxi driver. In sum, this Court cannot declare the dissolution of the marriage for failure of the petitioner to show that the alleged psychological incapacity is characterized by gravity, juridical antecedence and incurability and for her failure to observe the guidelines as outline in Republic v. CA and Molina.

376

University of the Cordilleras College of Law First Year C S.Y. 2013 - 2014 Zamora vs Court of Appeals 515 SCRA 19 G.R. No. 141917 February 7, 2007 Full Case BERNARDINO S. ZAMORA, Petitioner, vs. COURT OF APPEALS and NORMA MERCADO ZAMORA, Respondents. AZCUNA, J.: This is an appeal by certiorari under Rule 45 of the Rules of Court to annul and set aside the Decision and Resolution of the Court of Appeals (CA) dated August 5, 1999 and January 24, 2000 in CA-G.R. CV No. 53525, entitled "Bernardino S. Zamora v. Norma Mercado Zamora," which affirmed the dismissal of a complaint for declaration of nullity of marriage. The facts1 are: Petitioner and private respondent were married on June 4, 1970 in Cebu City. After their marriage, they lived together at No. 50-A Gorordo Avenue, Cebu City. The union did not produce any child. In 1972, private respondent left for the United States to work as a nurse. She returned to the Philippines for a few months, then left again in 1974. Thereafter, she made periodic visits to Cebu City until 1989, when she was already a U.S. citizen. Petitioner filed a complaint for declaration of nullity of marriage anchored on the alleged "psychological incapacity" of private respondent, as provided for under Article 36 of the Family Code. To support his position, he alleged that his wife was "horrified" by the mere thought of having children as evidenced by the fact that she had not borne petitioner a child. Furthermore, he also alleged that private respondent abandoned him by living in the United States and had in fact become an American citizen; and that throughout their marriage they lived together for not more than three years. On the other hand, private respondent denied that she refused to have a child. She portrayed herself as one who loves children as she is a nurse by profession and that she would from time to time borrow her husbands niece and nephews to care for them. She also faulted her husband for the breakup of their marriage, alleging that he had been unfaithful to her. He allegedly had two affairs with different women, and he begot at least three children with them. On June 22, 1995, the trial court rendered its decision thus: ... Plaintiff consented to defendants trip to the United States in 1974. She [defendant] wanted to earn money there because she wanted to help her husband build a big house at the Beverly Hills, Cebu City. Defendants testimony was corroborated by Paulina Martinez, a former househelp of the Zamoras.She always wanted to live in the Philippines before her husband committed infidelity. One reason why defendant seldom saw her husband while she was in the Philippines was because of the infidelity committed by her husband. No less than plaintiff himself admitted that he has a child with a certain [x x x]. The court is also convinced that he has two children with a certain [y y y]. The infidelity on the part of the plaintiff was one of the contributing factors which led to the estranged relationship between him and defendant. [N]othing in the evidence of plaintiff show[s] that the defendant suffered from any psychological incapacity or that she failed to comply with her essential marital obligations. There is no evidence of psychological incapacity on the part of defendant so that she could not carry out the ordinary duties required in married life. Neither has it been shown that there was an incurable defect on the part of defendant.

377

University of the Cordilleras College of Law First Year C S.Y. 2013 - 2014 ... WHEREFORE, in view of the foregoing, judgment is hereby rendered DISMISSING the complaint. Without special pronouncement as to cost. SO ORDERED.2 Petitioner appealed to the CA which rendered a Decision on August 5, 1999 affirming the ruling of the trial court. The pertinent portions of the CA decision read: Without delving further into both parties allegations, we must deny this appeal. In the case of Leouel Santos v. Court of Appeals,3the High Court ruled that, "psychological incapacity should refer to no less than a mental (not physical) incapacity x x x and that there is hardly any doubt that the intendment of the law has been to confine the meaning of psychological incapacity to the most serious cases of personality or inability to give meaning and significance to the marriage." Also, in Republic v. Court of Appeals and Molina,4 it was held that "mere showing of irreconcilable differences and conflicting personalities in no wise constitutes psychological incapacity. It is not enough to prove that the parties failed to meet their responsibilities and duties as married persons; it is essential that they must be shown to be incapable of doing so, due to some psychological (not physical) illness." This appeal does not fall in the category of psychological incapacity as defined in the aforementioned cases. The mere refusal of the appellee to bear a child is not equivalent to psychological incapacity, since even if such allegation is true, it is not shown or proven that this is due to psychological illness. As correctly stated by the appellee in her brief, the appellant even failed to present any psychologist or other medical expert to prove the psychological incapacity of defendant-appellee. This WE feel is a fatal omission on the part of the appellant, considering the doctrine laid down in the Santos and Molina cases (supra). WHEREFORE, in view of the foregoing, the decision of the Regional Trial Court, Branch 13 of Cebu City is hereby AFFIRMED. Appeal DISMISSED. SO ORDERED.5 Petitioner filed a motion for reconsideration but the same was denied by the CA in its Resolution dated January 24, 2000. Hence, this petition raising the following issues: 1) Whether or not the Court of Appeals misapplied facts of weight and substance affecting the result of the present case; 2) Whether or not Article 68 of the Family Code is applicable to this case; 3) Whether or not the presentation of psychologists and/or psychiatrists is still desirable, if evidence in this case already shows the psychological incapacity of private respondent; 4) Whether or not the presentation of psychologists and/or psychiatrists is still desirable, considering that the private respondent is a resident of the United States and living far away from the Philippines for more than twenty (20) years:

378

University of the Cordilleras College of Law First Year C S.Y. 2013 - 2014 5) Whether or not private respondents refusal to live with petitioner unde r one roof for more than twenty (20) years, her refusal to bear children with petitioner, and her living a solitary life in the United States for almost three (3) decades are enough indications of psychological incapacity to comply with essential marital obligations under Article 36 of the Family Code.6 Briefly, the issue is whether there can be a declaration of nullity of the marriage between petitioner and private respondent on the ground of psychological incapacity. Petitioner argues as follows: First, there is nothing in Santos v. CA,7 upon which private respondent relies, that requires as a conditio sine qua non the presentation of expert opinion of psychologists and psychiatrists in every petition filed under Article 36 of the Family Code. This Court merely said in that case that "[t]he well-considered opinions of psychiatrists, psychologists, and persons with expertise in psychological disciplines might be helpful or even desirable." However, no expert opinion is helpful or even desirable to determine whether private respondent has been living abroad and away from her husband for many years; whether she has a child; and whether she has made her residence abroad permanent by acquiring U.S. citizenship; and Second, Article 36 of the Family Code provides that a marriage contracted by any party who, at the time of the celebration, was psychologically incapacitated to comply with the essential marital obligations of marriage, shall likewise be void even if such incapacity becomes manifest only after its solemnization. Among the essential marital obligations embraced by Articles 68 to 71 of the same Code is to procreate children through sexual cooperation which is the basic end of marriage. To live together under one roof for togetherness spells the unity in marriage. The marriage had been existing for twenty four years when private respondent filed a legal separation case against petitioner. Throughout this period, private respondent deliberately and obstinately refused to comply with the essential marital obligation to live and cohabit with her husband. This Court rules as follows: It is true, as petitioner noted, that the case of Santos v. CA8 did not specifically mention that the presentation of expert opinion is a vital and mandatory requirement in filing a petition for the declaration of nullity of marriage grounded on psychological incapacity referred to under Article 36 of the Family Code. Even in the subsequent case of Republic v. Court of Appeals 9 (also known as the Molina case10 ), wherein the Court laid down the guidelines11 in the interpretation and application of the aforementioned article, examination of the person by a physician in order for the former to be declared psychologically incapacitated was likewise not considered a requirement.12 What is important, however, as stated in Marcos v. Marcos,13 is the presence of evidence that can adequately establish the partys psychological condition. If the totality of evidence presented is enough to sustain a finding of psychological incapacity, then actual medical examination of the person concerned need not be resorted to. Likewise, Section 2(d) of A.M. No. 02-11-10-SC or the Rule on Declaration of Absolute Nullity of Void Marriages and Annulment of Voidable Marriages, which took effect on March 15, 2003, states:1awphi1.net (d) What to allege. A petition under Article 36 of the Family Code shall specifically allege the complete facts showing that either or both parties were psychologically incapacitated from complying with the essential marital obligations of marriage at the time of the celebration of marriage even if such incapacity becomes manifest only after its celebration. The complete facts should allege the physical manifestations, if any, as are indicative of psychological incapacity at the time of the celebration of the marriage but expert opinion need not be alleged.14 The rule is that the facts alleged in the petition and the evidence presented, considered in totality, should be sufficient to convince the court of the psychological incapacity of the party concerned. Petitioner, however, failed to substantiate his allegation that private respondent is psychologically incapacitated. His 379

University of the Cordilleras College of Law First Year C S.Y. 2013 - 2014 allegations relating to her refusal to cohabit with him and to bear a child was strongly disputed, as the records undeniably bear out. Furthermore, the acts and behavior of private respondent that petitioner cited occurred during the marriage, and there is no proof that the former exhibited a similar predilection even before or at the inception of the marriage. Thus, based on the foregoing, the Court finds no reason to disturb the findings and conclusions reached by the trial court and the CA. WHEREFORE, the petition is DENIED. The Decision and Resolution of the Court of Appeals dated August 5, 1999 and January 24, 2000, respectively, in CA-G.R. CV No. 53525 are AFFIRMED. No costs. SO ORDERED.

380

University of the Cordilleras College of Law First Year C S.Y. 2013 - 2014 Case Digest BERNARDINO S. ZAMORA vs. COURT OF APPEALS and NORMA MERCADO ZAMORA G.R. No. 141917 February 7, 2007 515 SCRA 19 AZCUNA, J.: Facts: Petitioner and private respondent were married on June 4, 1970 in Cebu City. After their marriage, they lived together at No. 50-A Gorordo Avenue, Cebu City. The union did not produce any child. In 1972, private respondent left for the United States to work as a nurse. She returned to the Philippines for a few months, then left again in 1974. Thereafter, she made periodic visits to Cebu City until 1989, when she was already a U.S. citizen. Petitioner filed a complaint for declaration of nullity of marriage anchored on the alleged "psychological incapacity" of private respondent, as provided for under Article 36 of the Family Code. To support his position, he alleged that his wife was "horrified" by the mere thought of having children as evidenced by the fact that she had not borne petitioner a child. Furthermore, he also alleged that private respondent abandoned him by living in the United States and had in fact become an American citizen; and that throughout their marriage they lived together for not more than three years. On the other hand, private respondent denied that she refused to have a child. She portrayed herself as one who loves children as she is a nurse by profession and that she would from time to time borrow her husbands niece and nephews to care for them. She also faulted her husband for the breakup of their marriage, alleging that he had been unfaithful to her. He allegedly had two affairs with different women, and he begot at least three children with them. Issue: Whether there can be a declaration of nullity of the marriage between petitioner and private respondent on the ground of psychological incapacity. Ruling: No. In Marcos v. Marcos, stated that the presence of evidence that can adequately establish the partys psychological condition. If the totality of evidence presented is enough to sustain a finding of psychological incapacity, then actual medical examination of the person concerned need not be resorted to. But, the evidence presented, considered in totality, is not sufficient to convince the court of the psychological incapacity of the party concerned. Petitioner, however, failed to substantiate his allegation that private respondent is psychologically incapacitated. His allegations relating to her refusal to cohabit with him and to bear a child was strongly disputed, as the records undeniably bear out. Furthermore, the acts and behavior of private respondent that petitioner cited occurred during the marriage, and there is no proof that the former exhibited a similar predilection even before or at the inception of the marriage.

381

University of the Cordilleras College of Law First Year C S.Y. 2013 - 2014 Sermonia vs Court of Appeals 233 SCRA 15 G.R. No. 109454 June 14, 1994 Full Case JOSE C. SERMONIA, petitioner, vs. HON. COURT OF APPEALS, Eleventh Division, HON. DEOGRACIAS FELIZARDO, Presiding Judge, Regional Trial Court of Pasig, Br. 151, and JOSEPH SINSAY, respondents. BELLOSILLO, J.: Bigamy is an illegal marriage committed by contracting a second or subsequent marriage before the first marriage has been legally dissolved, or before the absent spouse has been declared presumptively dead by means of a judgment rendered in the proper proceedings. Bigamy carries with it the imposable penalty of prision mayor. Being punishable by an afflictive penalty, this crime prescribes in fifteen (15) years. The fifteen-year prescriptive period commences to run from the day on which the crime is discovered by the offended party, the authorities, or their agents . . . That petitioner contracted a bigamous marriage seems impliedly admitted. At least, it is not expressly denied. Thus the only issue for resolution is whether his prosecution for bigamy is already time-barred, which hinges on whether its discovery is deemed to have taken place from the time the offended party actually knew of the second marriage or from the time the document evidencing the subsequent marriage was registered with the Civil Registry consistent with the rule on constructive notice. The antecedents: In an information filed on 26 May 1992, petitioner Jose C. Sermonia was charged with bigamy before the Regional Trial Court of Pasig, Br. 151, for contracting marriage with Ma. Lourdes Unson on 15 February 1975 while his prior marriage to Virginia C. Nievera remained valid and subsisting. Petitioner moved to quash the information on the ground that his criminal liability for bigamy has been extinguished by prescription. In the order of 1 October 1992, respondent judge denied the motion to quash. On 27 October 1992, he likewise denied the motion to reconsider his order of denial. Petitioner challenged the above orders before the Court of Appeals through a petition for certiorari and prohibition. In the assailed decision of 21 January 1993, his petition was dismissed for lack of merit. In this recourse, petitioner contends that his criminal liability for bigamy has been obliterated by prescription. He avers that since the second marriage contract was duly registered with the Office of the Civil Registrar in 1975, such fact of registration makes it a matter of public record and thus constitutes notice to the whole world. The offended party therefore is considered to have had constructive notice of the subsequent marriage as of 1975; hence, prescription commenced to run on the day the marriage contract was registered. For this reason, the corresponding information for bigamy should have been filed on or before 1990 and not only in 1992. Petitioner likewise takes issue with the "alleged concealment of the bigamous marriage" as declared by the appellate court, insisting that the second marriage was publicly held at Our Lady of Nativity Church in Marikina on 15 February 1975, and adding for good measure that from the moment of registration the marriage contract was open to inspection by any interested person. On the other hand, the prosecution maintains that the prescriptive period does not begin from the commission of the crime but from the time of discovery by complainant which was in July 1991. While we concede the point that the rule on constructive notice in civil cases may be applied in criminal actions if the factual and legal circumstances so warrant, we agree with the view expounded by the Court

382

University of the Cordilleras College of Law First Year C S.Y. 2013 - 2014 of Appeals that it cannot apply in the crime of bigamy notwithstanding the possibility of its being more favorable to the accused. The appellate court succinctly explains Argued by the petitioner is that the principle of constructive notice should be applied in the case at bar, principally citing in support of his stand, the cases of People v. Reyes (175 SCRA 597); andPeople v. Dinsay (40 SCRA 50). This Court is of the view that the principle of constructive notice should not be applied in regard to the crime of bigamy as judicial notice may be taken of the fact that a bigamous marriage is generally entered into by the offender in secrecy from the spouse of the previous subsisting marriage. Also, a bigamous marriage is generally entered into in a place where the offender is not known to be still a married person, in order to conceal his legal impediment to contract another marriage. In the case of real property, the registration of any transaction involving any right or interest therein is made in the Register of Deeds of the place where the said property is located. Verification in the office of the Register of Deeds concerned of the transactions involving the said property can easily be made by any interested party. In the case of a bigamous marriage, verification by the offended person or the authorities of the same would indeed be quite difficult as such a marriage may be entered into in a place where the offender is not known to be still a married person. Be it noted that in the criminal cases cited by the petitioner wherein constructive notice was applied, involved therein were land or property disputes and certainly, marriage is not property. The non-application to the crime of bigamy of the principle of constructive notice is not contrary to the well entrenched policy that penal laws should be construed liberally in favor of the accused. To compute the prescriptive period for the offense of bigamy from registration thereof would amount to almost absolving the offenders thereof for liability therefor. While the celebration of the bigamous marriage may be said to be open and made of public record by its registration, the offender however is not truthful as he conceals from the officiating authority and those concerned the existence of his previous subsisting marriage. He does not reveal to them that he is still a married person. He likewise conceals from his legitimate spouse his bigamous marriage. And for these, he contracts the bigamous marriage in a place where he is not known to be still a married person. And such a place may be anywhere, under which circumstance, the discovery of the bigamous marriage is rendered quite difficult and would take time. It is therefore reasonable that the prescriptive period for the crime of bigamy should be counted only from the day on which the said crime was discovered by the offended party, the authorities or their agency (sic). Considering such concealment of the bigamous marriage by the offender, if the prescriptive period for the offense of bigamy were to be counted from the date of registration thereof, the prosecution of the violators of the said offense would almost be impossible. The interpretation urged by the petitioner would encourage fearless violations of a social institution cherished and protected by law. To this we may also add that the rule on constructive notice will make de rigueur the routinary inspection or verification of the marriages listed in the National Census Office and in various local civil registries all over the country to make certain that no second or even third marriage has been contracted without the knowledge of the legitimate spouse. This is too formidable a task to even contemplate. More importantly, while Sec. 52 of P.D. 1529 (Property Registration Decree) provides for constructive notice to all persons of every conveyance, mortgage, lease, lien, attachment, order, judgment, instrument or entry affecting registered land filed or entered in the office of the Register of Deeds for the province or city where the land to which it relates lies from the time of such registering, filing or entering, there is no counterpart provision either in Act No. 3753 (Act to Establish a Civil Register) or in Arts. 407 to 413 of the Civil Code, which leads us to the

383

University of the Cordilleras College of Law First Year C S.Y. 2013 - 2014 conclusion that there is no legal basis for applying the constructive notice rule to the documents registered in the Civil Register. Finally, petitioner would want us to believe that there was no concealment at all because his marriage contract with Ms. Unson was recorded in the Civil Registry which is open to all and sundry for inspection. We cannot go along with his argument because why did he indicate in the marriage contract that he was "single" thus obviously hiding his true status as a married man? Or for that matter, why did he not simply tell his first wife about the subsequent marriage in Marikina so that everything would be out in the open. The answer is obvious: He knew that no priest or minister would knowingly perform or authorize a bigamous marriage as this would subject him to punishment under the Marriage Law. Obviously, petitioner had no intention of revealing his duplicity to his first spouse and gambled instead on the probability that she or any third party would ever go to the local civil registrar to inquire. In the meantime, through the simple expedience of having the second marriage recorded in the local civil registry, he has set into motion the running of the fifteen-year prescriptive period against the unwary and the unsuspecting victim of his philandering. Were we to put our imprimatur to the theory advanced by petitioner, in all likelihood we would be playing right into the hands of philanderers. For we would be equating the contract of marriage with ordinary deeds of conveyance and other similar documents without due regard for the stability of marriage as an inviolable social institution, the preservation of which is a primary concern of our society. WHEREFORE, finding no reversible error in the questioned decision of the Court of Appeals, the same is AFFIRMED. SO ORDERED.

384

University of the Cordilleras College of Law First Year C S.Y. 2013 - 2014 Case Digest JOSE C. SERMONIA vs. COURT OF APPEALS G.R. No. 109454 June 14, 1994 233 SCRA 15 BELLOSILLO, J.: FACTS: On 26 May 1992, Jose C. Sermonia was charged with bigamy before the RTC of Pasig, for contracting marriage with Ma. Lourdes Unson on 15 February 1975 while his prior marriage to Virginia C. Nievera remained valid and subsisting. Petitioner moved to quash the information on the ground that his criminal liability for bigamy has been extinguished by prescription. In the order of 1 October 1992, respondent judge denied the motion to quash. On 27 October 1992, he likewise denied the motion to reconsider his order of denial. Petitioner challenged the above orders before the Court of Appeals through a petition for certiorari and prohibition. In the assailed decision of 21 January 1993, his petition was dismissed for lack of merit. Petitioner contends that his criminal liability for bigamy has been obliterated by prescription. He avers that since the second marriage contract was duly registered with the Office of the Civil Registrar in 1975, such fact of registration makes it a matter of public record and thus constitutes notice to the whole world. The offended party therefore is considered to have had constructive notice of the subsequent marriage as of 1975; hence, prescription commenced to run on the day the marriage contract was registered. For this reason, the corresponding information for bigamy should have been filed on or before 1990 and not only in 1992. On the other hand, the prosecution maintains that the prescriptive period does not begin from the commission of the crime but from the time of discovery by complainant which was in July 1991. ISSUE: Whether or not the prosecution of Jose C. Sermonia for bigamy has already prescribed. HELD: No. The non-application to the crime of bigamy of the principle of constructive notice is not contrary to the well entrenched policy that penal laws should be construed liberally in favor of the accused. To compute the prescriptive period for the offense of bigamy from registration thereof would amount to almost absolving the offenders thereof for liability therefor. While the celebration of the bigamous marriage may be said to be open and made of public record by its registration, the offender however is not truthful as he conceals from the officiating authority and those concerned the existence of his previous subsisting marriage. He does not reveal to them that he is still a married person. He likewise conceals from his legitimate spouse his bigamous marriage. And for these, he contracts the bigamous marriage in a place where he is not known to be still a married person. And such a place may be anywhere, under which circumstance, the discovery of the bigamous marriage is rendered quite difficult and would take time. It is therefore reasonable that the prescriptive period for the crime of bigamy should be counted only from the day on which the said crime was discovered by the offended party, the authorities or their agency.

385

University of the Cordilleras College of Law First Year C S.Y. 2013 - 2014 Bienvenido vs Court of Appeals 237 SCRA 676 G.R. No. 111717 October 24, 1994 Full Case NENITA BIENVENIDO, petitioner, vs. HON. COURT OF APPEALS, LUISITA CAMACHO and LUIS FAUSTINO C. CAMACHO, respondents. MENDOZA, J.: This is a petition for review of the decision of the Court of Appeals in CA-G.R. CV No. 24893, the dispositive portion of which reads: WHEREFORE, and upon all the foregoing, the decision of the court below dated August 29, 1989 is REVERSED. The deed of sale executed by the late Aurelio Camacho in favor of defendant Nenita T. Bienvenido and Transfer Certificate of Title No. 326681 of the Register of Deeds of Quezon City issued in her name are ANNULLED and in lieu thereof, a new transfer certificate of title in the name of the spouses Aurelio P. Camacho and Luisita C. Camacho shall ISSUE, herein declaring said spouses the owners of the property described in par. 8. of the complaint and DISMISSING the other prayers in the complaint as well as the defendant's counterclaim as baseless or without sufficient evidence in support thereof. With costs against the appellee. Petitioner filed a motion for reconsideration but her motion was denied in a resolution of the Court of Appeals promulgated on August 19, 1993. The background of this case is as follows: Aurelio P. Camacho married Consejo Velasco in Manila on October 3, 1942. On February 6, 1962, without his marriage to Consejo Velasco being dissolved, Aurelio P. Camacho contracted another marriage with respondent Luisita C. Camacho (Luisita) with whom he had been living since 1953 and by whom he begot a child, respondent Aurelio Luis Faustino C. Camacho (Chito) born on May 22, 1961. The marriage was solemnized in Tokyo, Japan where Aurelio and Luisita had been living since 1958. There were instances during Luisita and Aurelio's marriage when, because of their quarrels, one or the other left the dwelling place for long periods of time. In her case Luisita stayed on those occasions at various times in Davao City, Hongkong or Japan. In 1967 Aurelio met petitioner Nenita T. Bienvenido, who had been estranged from her husband, Luis Rivera. Aurelio courted her and apparently won her heart because from June 1968 until Aurelio's death on May 28, 1988, he lived with her, the last time in a duplex apartment on 84 Scout Delgado Street, Quezon City. Petitioner's daughter, Nanette, stayed with them as did Aurelio's son, Chito, who lived with them for about a year in 1976. On April 30, 1982, Aurelio bought the house and the lot on Delgado Street in which they were staying from the owners, Paz Lorenzo Infante and Suzette Infante-Moozca. In the deed of sale and Transfer Certificate of Title No. 288350 of the Registry of Deeds of Quezon City, issued in his name, Aurelio was described as single. On November 26, 1984, Aurelio executed a deed of sale of the property in favor of petitioner Nenita in consideration of the sum of P250,000.00, by virtue of which Transfer Certificate of Title No. 326681 was issued in petitioner's name on January 11, 1985. Between 1985 and 1987 Nenita and Luisita came to know each other. How they did is the subject of conflicting versions. Luisita claims that Nenita called her (Luisita's) residence several times, looking for Aurelio because the latter had allegedly left their dwelling place. Petitioner, according to Luisita, introduced herself as Mrs. Nenita Camacho.

386

University of the Cordilleras College of Law First Year C S.Y. 2013 - 2014 On the other hand petitioner claims it was the other way around that it was respondent Luisita who had called up their residence many times, also looking for Aurelio to urge him to file an application for American citizenship. On May 28, 1988, Aurelio died. Petitioner, using her Loyola Life Plan and Aurelio's account in the PCI Bank, took care of the funeral arrangements. Respondent Luisita was then in the United States with respondent Chito, having gone there, according to her, at the instance of Aurelio in order to look for a house in San Francisco so that Aurelio could follow and rejoin them. Upon learning of the death of Aurelio she and her son Chito came home on May 30, 1988. She had the remains of Aurelio transferred from the Loyola Memorial Chapels, first to the St. Ignatius Church and later to the Arlington Memorial Chapels. Luisita paid for the funeral services. Respondent Luisita was granted dealt benefits by the Armed Forces of the Philippines as the surviving spouse of Aurelio. Soon she also claimed ownership of the house and lot on Scout Delgado Street in which Nenita had been living. The two met at a barangay conciliation meeting but efforts to settle their dispute failed. On September 7, 1988, Luisita and her son Chito brought this case in the Regional Trial Court of Quezon City, seeking the annullment of the sale of the property to petitioner and the payment to them of damages. Luisita alleged that the deed of sale was a forgery and that in any event it was executed in fraud of her as the legitimate wife of Aurelio. In answer petitioner claimed that she and the late Aurelio had purchased the property in question using their joint funds which they had accumulated after living together for fourteen years, that the sale of the property by the late Aurelio to her was with respondent Luisita's consent; and that she was a purchaser in good faith. On August 29, 1989, the trial court rendered a decision upholding the sale of the property to petitioner and dismissing the complaint of Luisita. It found the deed of sale in favor of petitioner to be genuine and respondents Luisita and Chito to be in estoppel in not claiming the property until 1988 despite knowledge of the sale by the late Aurelio who had represented himself to be single. Respondents moved for a reconsideration but the trial court denied their motion. On appeal the respondents prevailed. On June 4, 1993, the Court of Appeals reversed the decision of the trial court and declared respondents to be the owners of the house and lot in dispute. Although Luisita had admitted that as early as 1985 she knew that Nenita had been staying in the premises, the appellate court held that respondents' action was not barred by laches because Luisita allegedly did not know that Nenita had obtained title to the property. On the merit, the Court of Appeals ruled that in the absence of proof to the contrary, Aurelio's first wife must be presumed to have been absent for seven years without Aurelio having news of her being alive when Aurelio contracted a second marriage. On this premise, it held (1) that the property in dispute belonged to the conjugal partnership of Aurelio and Luisita and (2) that the sale of the property to Nenita was void for the same reason that donations between persons who are guilty of concubinage or adultery are declared void under Art. 739 of the Civil Code. Hence this petition for review of the decision of the Court of Appeals. Petitioner claims that I THE COURT ERRED IN PRESUMING THE VALIDITY OF THE MARRIAGE BETWEEN AURELIO AND LUISITA [RESPONDENT HEREIN]; II THE COURT ERRED IN APPLYING ARTICLE 739 OF THE NCC AND DECLARING INVALID THE DEED OF SALE BETWEEN AURELIO AND NENITA [PETITIONER HEREIN]; III THE COURT ERRED IN RULING THAT THE SUBJECT PROPERTY FORMS PART OF THE CONJUGAL PROPERTIES OF AURELIO AND LUISITA. IV THE COURT ERRED IN NOT FINDING THAT PETITIONER IS NOT (sic) A PURCHASER IN GOOD FAITH AND LAWFUL OWNER OF SUBJECT PROPERTY. We find the petition to be meritorious. 387

University of the Cordilleras College of Law First Year C S.Y. 2013 - 2014 The resolution of this case hinges on the validity of Aurelio's marriage to respondent Luisita. If that marriage was valid then the property was property of their conjugal partnership and Luisita is the proper party to question the validity of the sale to Nenita. Otherwise, if the marriage is not valid, Luisita can not bring this suit. On the question of validity of Luisita's marriage to Aurelio, the Court of Appeals ruled: There is no dispute on the fact of appellant Luisita's marriage in 1962 to Aurelio. What is in question is the validity of that marriage considering Aurelio's purported previous marriage to Consejo Velasco. The appellee had attacked the validity of appellant's marriage in the trial below, on account of the previous marriage of Aurelio to Consejo Velasco, presenting evidence to that effect (Exhs. 43 and 44) to bolster her claim. Appellee likewise proved that Consejo Velasco although then a resident of Australia, is still alive. The burden of proof on the legality of appellant's marriage with Aurelio must rest on the appellee as the party who stands to benefit from a declaration of its invalidity. But appellee failed to prove that such second marriage (appellant's) was not valid because it was contracted at a time and on the assumption that the first spouse had been absent for seven years without the spouse present having news of the absentee being alive. This Court finds that the presumption of the validity of the marriage between Aurelio and Luisita has not been successfully assailed by appellee. The Court of Appeals thus presumed the validity of Aurelio's second marriage from the failure of petitioner to prove that at the time of such marriage Aurelio's first wife, Consejo, had not been absent for at least seven years and that Aurelio did not have news that his first wife was still alive. Petitioner had shown that on February 6, 1962, when Aurelio married Luisita, Aurelio's previous marriage to Consejo Velasco was still subsisting and, therefore, his second marriage was bigamous. It was the burden of herein respondents to prove that, at the time of his second marriage to respondent Luisita, Aurelio's first wife, Consejo Velasco, had been absent for at least seven years and that Aurelio had no news that she was alive. To assume these facts because petitioner has not disproved them would be to stand the principle on its head. Thus, Art. 83 of the Civil Code provides: Art. 83. Any marriage subsequently contracted by any person during the lifetime of the first spouse of such person with any person other than such first spouse shall be illegal and void from its performance, unless: (1) the first marriage was annulled or dissolved; or (2) the first spouse had been absent for seven consecutive years at the time of the second marriage without the spouse present having news of the absentee being alive, or if the absentee, though he has been absent for less than seven years, is generally considered as dead and believed to be so by the spouse present at the time of contracting such subsequent marriage, or if the absentee is presumed dead according to articles 390 and 391. The marriage so contracted shall be valid in any of the three cases until declared null and void by a competent court. As this Court has already explained, the general rule is that stated in the first sentence of this provision: "Any marriage subsequently contracted by any person during the lifetime of the first spouse of such person with any person other than such first spouse shall be illegal and void from its performance." The exceptions are those stated in paragraphs 1 and 2. The burden is on the party invoking any of the exceptions. Paragraph 2 mentions three cases when the subsequent marriage will not be considered void: (1) when the absent spouse has not been heard from for seven consecutive years and the present spouse has no news that he/she is alive; (2) when, although he/she has been absent for less than seven years, the absent spouse is generally considered to be dead and believed to be by the spouse present; and (3) when he/she is 388

University of the Cordilleras College of Law First Year C S.Y. 2013 - 2014 presumed to be dead after four years from the occurrence of any of the events enumerated in art. 391 of the Civil Code. In the case at bar, the burden of proof was on respondents to show that Luisita and Aurelio's marriage falls under any of these exceptions in order to be considered valid. They failed to discharge this burden. Instead the contrary appears. It has been held that the first exception refers to the subsequent marriage of the abandoned spouse and not the remarriage of the deserting spouse, after the period of seven years had lapsed. This exception cannot be invoked in this case in order to sustain the validity of Aurelio's marriage to Luisita because apparently it was Aurelio who had left his first wife. At the time of his second marriage to Luisita, he and Luisita had already been living together as husband and wife for five years. In fact the couple begot a child, in 1961, even before their marriage in 1962. What applies in this case, therefore, is the general rule, i.e., since Aurelio had a valid, subsisting marriage to Consejo Velaso, his subsequent marriage to respondent Luisita was void for being bigamous. Consequently, there is no basis for holding that the property in question was property of the conjugal partnership of Luisita and the late Aurelio because there was no such partnership in the first place. The Court of Appeals held that the sale of the property to Nenita is void on the principle embodied in Art. 739(1) of the Civil Code which declares donations made between persons who are guilty of adultery or concubinage at the time of the donation to be void. In the first place, an action for declaration of the nullity of such donations can only be brought by the innocent spouse, perhaps in this case by the first wife, but certainly not by Luisita whose marriage to Aurelio is itself void. The last paragraph of Art. 739 clearly provides: In the case referred to in No. 1, the action for declaration of nullify may be brought by the spouse of the donor or donee; and the guilt of the donor and donee may be proved by preponderance of evidence in the same action. In the second place, until otherwise shown in an appropriate action, the sale to petitioner must be presumed. Petitioner's ownership is evidenced by a deed of absolute sale executed with all the solemnity of a public document and by Transfer Certificate of Title No. 326681 issued in due course in her name. Petitioner is in possession of the property. It was error for the Court of Appeals to annul petitioner's title at the instance of one whose marriage to the seller is void. Indeed, the property in question was acquired by Aurelio during a long period of cohabitation with petitioner which lasted for twenty years (1968-1988). While petitioner knew respondent Chito to be Aurelio's son way back in 1976, there is nothing to show that she knew Aurelio to be married to Luisita. To the contrary, Aurelio represented himself to be single. As far as petitioner was concerned, Chito could have been Aurelio's child by a woman not his wife. There was, therefore, no basis for the Court of Appeals' ruling that Nenita was not a buyer in good faith of the property because she ought to have known that Aurelio was married to Luisita. WHEREFORE, the decision appealed from is REVERSED and another one is entered, DISMISSING the complaint against petitioner and DECLARING the deed of sale executed in her favor and Transfer Certificate of Title No. 326681 of the Register of Deeds of Quezon City issued in her name to be VALID. SO ORDERED. Narvasa, C.J., Regalado and Puno, JJ., concur.

389

University of the Cordilleras College of Law First Year C S.Y. 2013 - 2014 Case Digest NENITA BIENVENIDO vs. COURT OF APPEALS, LUISITA CAMACHO and LUIS FAUSTINO C. CAMACHO G.R. No. 111717 October 24, 1994 237 SCRA 676 MENDOZA, J.: Facts: Aurelio P. Camacho married Consejo Velasco in Manila on October 3,1942. On February 6, 1962, without his marriage to Consejo Velasco being dissolved, Aurelio P. Camacho contracted another marriage with respondent Luisita C. Camacho with whom he had been living since 1953 and by whom he begot a child, respondent Aurelio Luis Faustino C. Camacho (Chito) born on May 22, 1961. The marriage was solemnized in Tokyo, Japan where Aurelio and Luisita had been living since 1958. There were instances during Luisita and Aurelios marriage when, because of their quarrels, one or the other left the dwelling place for long periods of time. In her case Luisita stayed on those occasions at various times in Davao City, Hongkong or Japan. In 1967 Aurelio met petitioner Nenita T. Bienvenido, who had been estranged from her husband, Luis Rivera. He lived with her from June 1968 until Aurelios death o n May 28, 1988, he lived with her, the last time in a duplex apartment in Quezon City. Petitioners daughter, Nanette, stayed with them as did Aurelios son, Chito, who lived with them for about a year in 1976. On April 30, 1982, Aurelio bought the house and the lot on Delgado Street in which they were staying from the owners, Paz Lorenzo Infante and Suzette Infante-Moozca. In the deed of sale and Transfer Certificate of Title No. 288350 of the Registry of Deeds of Quezon City, issued in his name, Aurelio was described as single. On November 26, 1984, Aurelio executed a deed of sale of the property in favor of petitioner Nenita in consideration of the sum of P250,000.00, by virtue of which Transfer Certificate of Title No.326681 was issued in petitioners name on January 11, 1985. On September 7, 1988, Luisita and her son Chito brought this case in the Regional Trial Court of Quezon City, seeking the annulment of the sale of the property to petitioner and the payment to them of damages. Luisita alleged that the deed of sale was a forgery and that in any event it was executed in fraud of her as the legitimate wife of Aurelio. In answer petitioner Nenita claimed that she and the late Aurelio had purchased the property in question using their joint funds which they had accumulated after living together for fourteen years, that the sale of the property by the late Aurelio to her was with respondent Luisitas consent; and that she was a purchaser in good faith. Issue: Whether the marriage of Aurelio and Luisita is valid. Whether the deed of sale between Aurelio and Nenita is valid. Ruling: On August 29, 1989, the trial court rendered a decision upholding the sale of the property to petitioner and dismissing the complaint of Luisita. It found the deed of sale in favor of petitioner to be genuine and respondents Luisita and Chito to be in estoppel in not claiming the property until 1988 despite knowledge of the sale by the late Aurelio who had represented himself to be single. Respondents moved for a reconsideration but the trial court denied their motion. On appeal the respondents prevailed. On June 4, 1993, the Court of Appeals reversed the decision of the trial court and declared respondents to be the owners of the house and lot in dispute. Although Luisita had admitted that as early as 1985 she knew that Nenita had been staying in the premises, the appellate court held that respondents action was not barred by laches because Luisita allegedly did not know that Nenita had obtained title to the property. On the merit, the Court of Appeals ruled that in the absence of proof to the contrary, Aurelios first wife must be presumed to have been absent for seven years without Aurelio having news of her being alive when Aurelio contracted a second marriage. On this premise, it held (1)that the property in dispute belonged to the conjugal partnership of Aurelio and Luisita and (2) that the sale of the property to Nenita was void for the same reason

390

University of the Cordilleras College of Law First Year C S.Y. 2013 - 2014 that donations between persons who are guilty of concubinage or adultery are declared void under Art. 739 of the Civil Code. The decision appealed from is REVERSED and another one is entered, DISMISSING the complaint against petitioner and DECLARING the deed of sale executed in her favor and Transfer Certificate of Title No.326681 of the Register of Deeds of Quezon City issued in her name to be VALID. In the case at bar, the burden of proof was on respondents to show that Luisita and Aurelios marriage falls under any of these exceptions in order to be considered valid. They failed to discharge this burden. Instead the contrary appears. It has been held that the first exception refers to the subsequent marriage of the abandoned spouse and not the remarriage of the deserting spouse, after the period of seven years had lapsed. This exception cannot be invoked in this case in order to sustain the validity of Aurelios marriage to Luisita because apparently it was Aurelio who had left his first wife. At the time of his second marriage to Luisita, he and Luisita had already been living together as husband and wife for five years. In fact the couple begot a child, in 1961, even before their marriage in 1962. Consequently, there is no basis for holding that the property in question was property of the conjugal partnership of Luisita and the late Aurelio because there was no such partnership in the first place. The sale to petitioner must be presumed. Petitioners ownership is evidenced by a deed of absolute sale 7 executed with all the solemnity of a public document and by Transfer Certificate of Title No. 326681 issued in due course in her name. Indeed, the property in question was acquired by Aurelio during a long period of cohabitation with petitioner which lasted for twenty years (1968-1988). While petitioner knew respondent Chito to be Aurelios son way back in 1976, there is nothing to show that she knew Aurelio to be married to Luisita. To the contrary, Aurelio represented himself to be single. As far as petitioner was concerned, Chito could have been Aurelios child by a woman not his wi fe. There was, therefore, no basis for the Court of Appeals ruling that Nenita was not a buyer in good faith of the property because she ought to have known that Aurelio was married to Luisita.

391

University of the Cordilleras College of Law First Year C S.Y. 2013 - 2014 Terre vs Terre 211 SCRA 6 A.M. No. 2349 July 3, 1992 Full Case DOROTHY B. TERRE, complainant, vs. ATTY. JORDAN TERRE, respondent. PER CURIAM:J, In a sworn complaint filed with this Court on 24 December 1981, complainant Dorothy B. Terre charged respondent Jordan Terre, a member of the Philippine Bar with "grossly immoral conduct," consisting of contracting a second marriage and living with another woman other than complainant, while his prior marriage with complainant remained subsisting. The Court resolved to require respondent to answer the complaint. 1 Respondent successfully evaded five (5) attempts to serve a copy of the Court's Resolution and of the complaint by moving from one place to another, such that he could not be found nor reached in his alleged place of employment or residence. 2 On 24 April 1985, that is after three (3) years and a half, with still no answer from the respondent, the Court noted respondent's success in evading service of the complaint and the Court's Resolution and thereupon resolved to "suspend respondent Atty. Jordan Terre from the practice of law until after he appears and/or files his answer to the complaint against him" in the instant case. 3 On 28 September 1985, respondent finally filed an Answer with a Motion to Set Aside and/or Lift Suspension Order. In his Answer, Atty. Terre averred that he had contracted marriage with complainant Dorothy Terre on 14 June 1977 upon her representation that she was single; that he subsequently learned that Dorothy was married to a certain Merlito A. Bercenilla sometime in 1968; that when he confronted Dorothy about her prior marriage, Dorothy drove him out of their conjugal residence; that Dorothy had mockingly told him of her private meetings with Merlito A. Bercenilla and that the child she was then carrying (i.e., Jason Terre) was the son of Bercenilla; that believing in good faith that his marriage to complainant was null and void ab initio, he contracted marriage with Helina Malicdem at Dasol, Pangasinan. 4 In her Reply, complainant Dorothy denied that Jason Terre was the child of Merlito A. Bercenilla and insisted that Jason was the child of respondent Jordan Terre, as evidenced by Jason's Birth Certificate and physical resemblance to respondent. Dorothy further explained that while she had given birth to Jason Terre at the PAFGH registered as a dependent of Merlito Bercenilla, she had done so out of extreme necessity and to avoid risk of death or injury to the fetus which happened to be in a difficult breech position. According to Dorothy, she had then already been abandoned by respondent Jordan Terre, leaving her penniless and without means to pay for the medical and hospital bills arising by reason of her pregnancy. The Court denied respondent's Motion to Set Aside or Lift the Suspension Order and instead referred; by a Resolution dated 6 January 1986, the complaint to the Office of the Solicitor General for investigation, report and recommendation. 5 Then Solicitor Pio C. Guerrero was appointed investigator by the Office of the Solicitor General. He set the case for hearing on 7 July 1986 with notice to both parties. On 7 July 1986, complainant Dorothy appeared and presented her evidence ex parte, since respondent did not so appear. 6 The Investigating Solicitor scheduled and held another hearing on 19 August 1986, where he put clarificatory questions to the complainant; respondent once again did not appear despite notice to do so. Complainant finally offered her evidence and rested her case. The Solicitor set still another hearing for 2 October 1986, notifying respondent to present his evidence with a warning that should he fail once more to appear, the case would be deemed submitted for resolution. Respondent did not appear on 2 October 1986. The Investigating Solicitor accordingly considered respondent to have waived his right to present evidence 392

University of the Cordilleras College of Law First Year C S.Y. 2013 - 2014 and declared the case submitted for resolution. The parties were given time to submit their respective memoranda. Complainant Dorothy did so on 8 December 1986. Respondent Terre did not file his memorandum. On 26 February 1990, the Office of the Solicitor General submitted its "Report and Recommendation" to this Court. The Report summarized the testimony of the complainant in the following manner: Complainant Dorothy Terre took the witness stand and testified substantially as follows: she and respondent met for the first time in 1979 as fourth year high school classmates in Cadiz City High School (tsn, July 7, 1986, p. 9); she was then married to Merlito Bercenilla, while respondent was single (id.); respondent was aware of her marital status (ibid, p. 14); it was then that respondent started courting her but nothing happened of the courtship (ibid, p. 10); they [complainant and respondent] moved to Manila were they respectively pursued their education, respondent as a law student at the Lyceum University (tsn, July 7, 1986, p. 12, 15-16); respondent continued courting her, this time with more persistence (ibid, p. 11); she decided nothing would come of it since she was married but he [respondent] explained to her that their marriage was void ab initiosince she and her first husband were first cousins (ibid, p. 12); convinced by his explanation and having secured favorable advice from her mother and ex-in-laws, she agreed to marry him [respondent] (ibid, 12-13, 16); in their marriage license, despite her [complainant's] objection, he [respondent] wrote "single" as her status explaining that since her marriage was void ab initio, there was no need to go to court to declare it as such (ibid, 14-15); they were married before Judge Priscilla Mijares of the City Court of Manila on June 14, 1977 (Exhibit A; tsn, July 7, 1986, pp. 16-17); Jason Terre was born of their union on June 25, 1981 (Exhibit B, tsn, July 7, 1986, p. 18); all through their married state up to the time he [respondent] disappeared in 1981, complainant supported respondent, in addition to the allowance the latter was getting from his parents (ibid, pp. 19-20); she was unaware of the reason for his disappearance until she found out later that respondent married a certain Vilma [sic] Malicdem (Exhibit C, tsn, July 7, 1986, pp. 21-22); she then filed a case for abandonment of minor with the City Fiscal of Pasay City (ibid, p. 23) which was subsequently filed before Branch II of the City Court of Pasay City as Criminal Case No. 816159 (Exhibit D; tsn, July 7, 1986, p. 24); she likewise filed a case for bigamy against respondent and Helina Malicdem with the office of the Provincial Fiscal of Pangasinan, where a prima faciecase was found to exist (Exhibit E; tsn, July 7, pp. 25-26); additionally, complainant filed an administrative case against respondent with the Commission on Audit where he was employed, which case however was considered closed for being moot and academic when respondent was considered automatically separated from the service for having gone on absence without official leave (Exhibit F; tsn, July 7, 1986, pp. 28-29). 7 There is no dispute over the fact that complainant Dorothy Terre and respondent Jordan Terre contracted marriage on 14 July 1977 before Judge Priscilla Mijares. There is further no dispute over the fact that on 3 May 1981, respondent Jordan Terre married Helina Malicdem in Dasol, Pangasinan. When the second marriage was entered into, respondent's prior marriage with complainant was subsisting, no judicial action having been initiated or any judicial declaration obtained as to the nullity of such prior marriage of respondent with complainant. Respondent Jordan Terre sought to defend himself by claiming that he had believed in good faith that his prior marriage with complainant Dorothy Terre was null and void ab initio and that no action for a judicial declaration of nullity was necessary. The Court considers this claim on the part of respondent Jordan Terre as a spurious defense. In the first place, respondent has not rebutted complainant's evidence as to the basic facts which underscores the bad faith of respondent Terre. In the second place, that pretended defense is the same argument by which he had inveigled complainant into believing that her prior marriage to Merlito A. Bercenilla being incestuous and void ab initio (Dorothy and Merlito being allegedly first cousins to each other), she was free to contract a second marriage with the respondent. Respondent Jordan Terre, being a lawyer, knew or should have known that such an argument ran counter to the prevailing case law of this Court which holds that for purposes of determining whether a person is legally free to contract a second marriage, a judicial declaration that the first marriage was null and void ab initio is essential. 8 Even if we were to 393

University of the Cordilleras College of Law First Year C S.Y. 2013 - 2014 assume, arguendo merely, that Jordan Terre held that mistaken belief in good faith, the same result will follow. For if we are to hold Jordan Terre to his own argument, his first marriage to complainant Dorothy Terre must be deemed valid, with the result that his second marriage to Helina Malicdem must be regarded as bigamous and criminal in character. That the moral character of respondent Jordan Terre was deeply flawed is shown by other circumstances. As noted, he convinced the complainant that her prior marriage to Bercenilla was null and void ab initio, that she was still legally single and free to marry him. When complainant and respondent had contracted their marriage, respondent went through law school while being supported by complainant, with some assistance from respondent's parents. After respondent had finished his law course and gotten complainant pregnant, respondent abandoned the complainant without support and without the wherewithal for delivering his own child safely in a hospital. Thus, we agree with the Solicitor General that respondent Jordan Terre, by his actions, "eloquently displayed, not only his unfitness to remain as a member of the Bar, but likewise his inadequacy to uphold the purpose and responsibility of his gender" because marriage is a basic social institution. 9 In Pomperada v. Jochico, 10 the Court, in rejecting a petition to be allowed to take the oath as a member of the Bar and to sign the Roll of Attorneys, said through Mme. Justice Melencio-Herrera: It is evident that respondent fails to meet the standard of moral fitness for membership in the legal profession. Whether the marriage was a joke as respondent claims, or a trick played on her as claimed by complainant, it does not speak well of respondent's moral values. Respondent had made a mockery of marriage, a basic social institution which public policy cherishes and protects (Article 216, Civil Code). 11 In Bolivar v. Simbol, 12 the Court found the respondent there guilty of "grossly immoral conduct" because he made a dupe of complainant, living on her bounty and allowing her to spend for his schooling and other personal necessities while dangling before her the mirage of a marriage, marrying another girl as soon as he had finished his studies, keeping his marriage a secret while continuing to demand money from complainant. . . . ." The Court held such acts "indicative of a character not worthy of a member of the Bar." 13 We believe and so hold that the conduct of respondent Jordan Terre in inveigling complainant Dorothy Terre to contract a second marriage with him; in abandoning complainant Dorothy Terre after she had cared for him and supported him through law school, leaving her without means for the safe delivery of his own child; in contracting a second marriage with Helina Malicdem while his first marriage with complainant Dorothy Terre was subsisting, constituted "grossly immoral conduct" under Section 27 of Rule 138 of the Rules of Court, affording more than sufficient basis for disbarment of respondent Jordan Terre. He was unworthy of admission to the Bar in the first place. The Court will correct this error forthwith. WHEREFORE, the Court Resolved to DISBAR respondent Jordan Terre and to STRIKE OUT his name from the Roll of Attorneys. A copy of this decision shall be spread on the personal record of respondent Jordan Terre in the Bar Confidant's Office. A copy of this resolution shall also be furnished to the Integrated Bar of the Philippines and shall be circularized to all the courts of the land. SO ORDERED.

394

University of the Cordilleras College of Law First Year C S.Y. 2013 - 2014 Case Digest DOROTHY B. TERRE vs. ATTY. JORDAN TERRE A.M. No. 2349 July 3, 1992 211 SCRA 6 PER CURIAM: FACTS: Dorothy Terre was then married to a certain Merlito Bercenillo, her first cousin. Atty. Jordan Terre successfully convinced Dorothy that her marriage was void ab initio for the reason of public policy and that they are free to contract marriage. They got married in 1977 where he wrote single under Dorothys status. After getting Dorothy pregnant, Atty. Terre abandoned them and subsequently contracted another marriage to Helina Malicdem in 1986. Atty. Terre was charged with abandonment of minor and bigamy. ISSUE: Whether or not Atty. Terres marriage with Dorothy is null and void. HELD: Dorothys first marriage is indeed void ab initio considering that Merlito is her first cousin thereby against public policy. However, she did not file any declaration for the nullity of their marriage before she contracted her marriage with Atty. Terre thus, her second marriage is void. Article 40 states that the absolute nullity of a former marriage may be invoked for the purposes of remarriage on the basis solely of a final judgment declaring such previous marriage void.

395

University of the Cordilleras College of Law First Year C S.Y. 2013 - 2014 Buccat vs Buccat 72 Phil 21 G.R. No. 47101 April 25, 1941 Full Case GODOFREDO BUCCAT, plaintiff-appellant, vs. LUIDA MANGONON DE BUCCAT, defendant-respondent. HORRILLENO, J.: This issue has been raised to this superiority by the Court of First Instance of Baguio, as only raises a question purely of law. On March 20, 1939 the plaintiff inico the present case, in which the defendant failed to appear, despite having been duly summoned. Therefore, allowed the plaintiff to present evidence, the lower court decision, the matter in favor of the defendant. Hence this appeal. The applicant requests the annulment of their marriage had with the defendant Luisa Buccat Mangonon of the November 26, 1938, in Baguio City, the grounds that, in consenting to the marriage, he did so because the defendant had assured him that she was virgin. In the lower court's decision reveals the following facts: The plaintiff met the defendant in March 1938. After several interviews, both were committed on19 September of that year. On 26 November the same year, the plaintiff married the defendant in the Catholic Cathedral Baguio City. Delouse of living maritally for the space of eighty-nine days, the defendant gave birth to a child of nine months, the February 23, 1939. Following this event, the plaintiff abandoned the defendant and did not return to do with her marital life. We do not see any reason to vacate the judgment appealed. Indeed, it is improbable the plaintiff's allegation that the appellant and had not even suspected the gravid state of the defendant, being this, as has been proven, highly advanced in pregnant condition. Therefore, not necessary to estimate the fraud that speaks the appellant. He argued for this in the sense that countries not uncommon to find people of the abdomen developed, it seems childish to merit our consideration, especially as the applicant was a freshman in law. Marriage is a most sacred institution: it is the foundation on which society rests. You can stop this are necessary to clear and reliable. In this case no such evidence. Finding the judgment appealed in accordance with law, must be confirmed, as confirmed by the present, in all its parts, with costs against the appellant. So ordered.

396

University of the Cordilleras College of Law First Year C S.Y. 2013 - 2014 Case Digest GODOFREDO BUCCAT vs. LUIDA MANGONON DE BUCCAT G.R. No. 47101 April 25, 1941 72 PHIL 21 HORRILLENO, J.: Facts: Godofredo Buccat and Luida Mangonon de Buccat met in March 1938, became engaged in September, and got married in Nov 26.On Feb 23, 1939 (89 days after getting married) Luida, who was 9 months pregnant, gave birth to a son. Godofredo left Luida and on March 23, 1939, he filed for an annulment of their marriage on the grounds that when he agreed to married Luida, she assured him that she was a virgin. The Lower court decided in favor of Luida. Issue: Whether or not Luidas concealment of her pregnancy constituted a ground for the annulment of marriage Held: No. Clear and authentic proof is needed in order to nullify a marriage, a sacred institution in which the State is interested. In this case, the court did not find any proof that there was concealment of pregnancy constituting a ground for annulment; it was unlikely that Godofredo, a first-year law student, did not suspect anything about Luidas condition considering that she was in an advanced stage of pregnancy when they got married. SC affirmed the lower courts decision.

397

University of the Cordilleras College of Law First Year C S.Y. 2013 - 2014

Aquino vs Delizo 109 Phil 21


G.R. No. L-15853 Full Case FERNANDO AQUINO, petitioner, vs. CONCHITA DELIZO, respondent. GUTIERREZ DAVID, J.: This is a petition for certiorari to review a decision of the Court of Appeals affirming that of the Court of First Instance of Rizal which dismissed petitioner's complaint for annulment of his marriage with respondent Conchita Delizo. The dismissed complaint, which was filed on September 6, 1955, was based on the ground of fraud, it being alleged, among other things, that defendant Conchita Delizo, herein respondent, at the date of her marriage to plaintiff, herein petitioner Fernando Aquino, on December 27, 1954, concealed from the latter that fact that she was pregnant by another man, and sometime in April, 1955, or about four months after their marriage, gave birth to a child. In her answer, defendant claimed that the child was conceived out of lawful wedlock between her and the plaintiff. At the trial, the attorneys for both parties appeared and the court a quo ordered Assistant Provincial Fiscal Jose Goco to represent the State in the proceedings to prevent collusion. Only the plaintiff however, testified and the only documentary evidence presented was the marriage contract between the parties. Defendant neither appeared nor presented any evidence despite the reservation made by her counsel that he would present evidence on a later date. On June 16, 1956, the trial court noting that no birth certificate was presented to show that the child was born within 180 days after the marriage between the parties, and holding that concealment of pregnancy as alleged by the plaintiff does not constitute such fraud sa would annul a marriage dismissed the complaint. Through a verified "petition to reopen for reception of additional evidence", plaintiff tried to present the certificates of birth and delivery of the child born of the defendant on April 26, 1955, which documents, according to him, he had failed to secure earlier and produce before the trial court thru excusable negligence. The petition, however, was denied. On appeal to the Court of Appeals, that court held that there has been excusable neglect in plaintiff's inability to present the proof of the child's birth, through her birth certificate, and for that reason the court a quo erred in denying the motion for reception of additional evidence. On the theory, however, that it was not impossible for plaintiff and defendant to have had sexual intercourse during their engagement so that the child could be their own, and finding unbelievable plaintiff's claim that he did not notice or even suspect that defendant was pregnant when he married her, the appellate court, nevertheless, affirmed the dismissal of the complaint. On March 17, 1959, plaintiff filed a motion praying that the decision be reconsidered, or, if such reconsideration be denied, that the case be remanded to the lower court for new trial. In support of the motion, plaintiff attached as annexes thereof the following documents: 1. Affidavit of Cesar Aquino (Annex A) (defendant's brother-in-law and plaintiff's brother, with whom defendant was living at the time plaintiff met, courted and married her, and with whom defendant has begotten two more children, aside from her first born, in common-law relationship) admitting that he is the father of defendant's first born, Catherine Bess Aquino, and that he and defendant hid her pregnancy from plaintiff at the time of plaintiff's marriage to defendant; 2. Affidavit of defendant, Conchita Delizo (Annex "B") admitting her pregnancy by Cesar Aquino, her brother-in-law and plaintiff's own brother, at the time of her marriage to plaintiff and her having hidden this fact from plaintiff before and up to the time of their marriage; July 27, 1960

398

University of the Cordilleras College of Law First Year C S.Y. 2013 - 2014 3. Affidavit of Albert Powell (Annex "C") stating that he knew Cesar Aquino and defendant lived together as husband and wife before December 27, 1954, the date of plaintiff's marriage to defendant; 4. Birth Certificate of defendant's first born, Catherine Bess Aquino showing her date of birth to be April 26, 1955; 5. Birth Certificate (Annex "D") of Carolle Ann Aquino, the second child of defendant with Cesar Aquino, her brother-in-law; 6. Birth Certificate (Annex "E") of Chris Charibel Aquino, the third child of Cesar Aquino and defendant; and 7. Pictures of defendant showing her natural plumpness as early as 1952 to as late as November, 1954, the November, 1954 photo itself does not show defendant's pregnancy which must have been almost four months old at the time the picture was taken. Acting upon the motion, the Court of Appeals ordered the defendant Conchita Delizo and Assistant Provincial Fiscal of Rizal, who was representing the Government, to answer the motion for reconsideration, and deferred action on the prayer for new trial until after the case is disposed of. As both the defendant and the fiscal failed to file an answer and stating that it "does not believe the veracity of the contents of the motion and its annexes", the Court of Appeals, on August 6, 1959, denied the motion. From that order, the plaintiff brought the case to this Court thru the present petition for certiorari. After going over the record of the case, we find that the dismissal of plaintiff's complaint cannot be sustained. Under the new Civil Code, concealment by the wife of the fact that at the time of the marriage, she was pregnant by a man other than her husband constitutes fraud and is ground for annulment of marriage. (Art. 85, par. (4) in relation to Art. 86, par. (3). In the case of Buccat vs. Buccat (72 Phil., 19) cited in the decision sought to be reviewed, which was also an action for the annulment of marriage on the ground of fraud, plaintiff's claim that he did not even suspect the pregnancy of the defendant was held to be unbelievable, it having been proven that the latter was already in an advanced stage of pregnancy (7th month) at the time of their marriage. That pronouncement, however, cannot apply to the case at bar. Here the defendant wife was alleged to be only more than four months pregnant at the time of her marriage to plaintiff. At that stage, we are not prepared to say that her pregnancy was readily apparent, especially since she was "naturally plump" or fat as alleged by plaintiff. According to medical authorities, even on the 5th month of pregnancy, the enlargement of a woman's abdomen is still below the umbilicus, that is to say, the enlargement is limited to the lower part of the abdomen so that it is hardly noticeable and may, if noticed, be attributed only to fat formation on the lower part of the abdomen. It is only on the 6th month of pregnancy that the enlargement of the woman's abdomen reaches a height above the umbilicus, making the roundness of the abdomen more general and apparent. (See Lull, Clinical Obstetrics, p. 122) If, as claimed by plaintiff, defendant is "naturally plump", he could hardly be expected to know, merely by looking, whether or not she was pregnant at the time of their marriage more so because she must have attempted to conceal the true state of affairs. Even physicians and surgeons, with the aid of the woman herself who shows and gives her subjective and objective symptoms, can only claim positive diagnosis of pregnancy in 33% at five months. and 50% at six months. (XI Cyclopedia of Medicine, Surgery, etc. Pregnancy, p. 10). The appellate court also said that it was not impossible for plaintiff and defendant to have had sexual intercourse before they got married and therefore the child could be their own. This statement, however, is purely conjectural and finds no support or justification in the record. Upon the other hand, the evidence sought to be introduced at the new trial, taken together with what has already been adduced would, in our opinion, be sufficient to sustain the fraud alleged by plaintiff. The Court of Appeals should, therefore, not have denied the motion praying for new trial simply because defendant failed to file her answer thereto. Such failure of the defendant cannot be taken as evidence of collusion, especially since a provincial fiscal has been ordered of represent the Government precisely to prevent such collusion. As to the veracity of the contents of the motion and its annexes, the same can best be determined only after hearing evidence. In the circumstance, we think that justice would be better served if a new trial were ordered. 399

University of the Cordilleras College of Law First Year C S.Y. 2013 - 2014 Wherefore, the decision complained of is set aside and the case remanded to the court a quo for new trial. Without costs. Paras, C.J., Bengzon, Montemayor, Labrador, Concepcion, and Reyes, J.B.L., JJ., concur. Barrera, J., concurs in the result.

400

University of the Cordilleras College of Law First Year C S.Y. 2013 - 2014 Case Digest FERNANDO AQUINO vs. CONCHITA DELIZO G.R. No. L-15853 July 27, 1960 109 PHIL 21 GUTIERREZ DAVID, J.: Facts: Defendant Conchita Delizo, herein respondent, at the date of her marriage to plaintiff,

herein petitioner Fernando Aquino, on December 27, 1954, concealed from the latter that fact that she was pregnant by another man, and sometime in April, 1955, or about four months after their marriage, gave birth to a child. In her answer, defendant claimed that the child was conceived out of lawful wedlock between her and the plaintiff.
The Petitioner claimed that he did not notice that her wife was four months pregnant at the time of their marriage because the Respondent was naturally plump or fat. During the trial, Provincial Fiscal Jose Goco represent the state in the proceedings to prevent collusion. Only Aquino testified and the only documentary evidence presented was the marriage contract between the parties. Delizo did not appear nor presented any evidence. CFI-Rizal dismissed petitioners complaint for annulment of marriage, which was affirmed by CA thus a petition for certiorari to review the decisions. Issue: Whether or not concealment of pregnancy as alleged by Aquino does not constitute such fraud as would annul a marriage. Held: The concealment by the wife of the fact that at the time of the marriage, she was pregnant by a man other than her husband constitutes fraud and is a ground for annulment of marriage. Delizo was allegedly to be only more than four months pregnant at the time of her marriage. At this stage, it is hard to say that her pregnancy was readily apparent especially since she was naturally plump or fat. It is only on the 6thmonth of pregnancy that the enlargeme nt of the womans abdomen reaches a height above the umbilicus, making the roundness of the abdomen more general and apparent. In the following circumstances, the court remanded the case for new trial and decision complained is set aside.

401

University of the Cordilleras College of Law First Year C S.Y. 2013 - 2014 Anaya vs Palaroan 36 SCRA 97 G.R. No. L-27930 November 26, 1970 Full Case AURORA A. ANAYA, plaintiff-appellant, vs. FERNANDO O. PALAROAN, defendant-appellee. REYES, J.B.L., J.: Appeal from an order of dismissal, issued motu proprio by the Juvenile & Domestic Relations Court, Manila, of a complaint for annulment of marriage, docketed therein as Civil Case No. E-00431, entitled "Aurora A. Anaya, plaintiff vs. Fernando O. Palaroan, defendant." The complaint in said Civil Case No. E-00431 alleged, inter alia, that plaintiff Aurora and defendant Fernando were married on 4 December 1953; that defendant Fernando filed an action for annulment of the marriage on 7 January 1954 on the ground that his consent was obtained through force and intimidation, which action was docketed in the Court of First Instance of Manila as Civil Case No. 21589; that judgment was rendered therein on 23 September 1959 dismissing the complaint of Fernando, upholding the validity of the marriage and granting Aurora's counterclaim; that (per paragraph IV) while the amount of the counterclaim was being negotiated "to settle the judgment," Fernando had divulged to Aurora that several months prior to their marriage he had pre-marital relationship with a close relative of his; and that "the non-divulgement to her of the aforementioned pre-marital secret on the part of defendant that definitely wrecked their marriage, which apparently doomed to fail even before it had hardly commenced ... frank disclosure of which, certitude precisely precluded her, the Plaintiff herein from going thru the marriage that was solemnized between them constituted 'FRAUD', in obtaining her consent, within the contemplation of No. 4 of Article 85 of the Civil Code" (sic) (Record on Appeal, page 3). She prayed for the annulment of the marriage and for moral damages. Defendant Fernando, in his answer, denied the allegation in paragraph IV of the complaint and denied having had pre-marital relationship with a close relative; he averred that under no circumstance would he live with Aurora, as he had escaped from her and from her relatives the day following their marriage on 4 December 1953; that he denied having committed any fraud against her. He set up the defenses of lack of cause of action and estoppel, for her having prayed in Civil Case No. 21589 for the validity of the marriage and her having enjoyed the support that had been granted her. He counterclaimed for damages for the malicious filing of the suit. Defendant Fernando did not pray for the dismissal of the complaint but for its dismissal "with respect to the alleged moral damages." Plaintiff Aurora filed a reply with answer to the counterclaim, wherein she alleged: (1) that prior to their marriage on 4 December 1953, he paid court to her, and pretended to shower her with love and affection not because he really felt so but because she merely happened to be the first girl available to marry so he could evade marrying the close relative of his whose immediate members of her family were threatening him to force him to marry her (the close relative); (2) that since he contracted the marriage for the reason intimated by him, and not because he loved her, he secretly intended from the very beginning not to perform the marital duties and obligations appurtenant thereto, and furthermore, he covertly made up his mind not to live with her; (3) that the foregoing clandestine intentions intimated by him were prematurely concretized for him, when in order to placate and appease the immediate members of the family of the first girl (referent being the close relative) and to convince them of his intention not to live with plaintiff, carried on a courtship with a third girl with whom, after gaining the latter's love cohabited and had several children during the whole range of nine years that Civil Case No. 21589, had been litigated between them (parties); (Record on Appeal, pages 10-11)

402

University of the Cordilleras College of Law First Year C S.Y. 2013 - 2014 Failing in its attempt to have the parties reconciled, the court set the case for trial on 26 August 1966 but it was postponed. Thereafter, while reviewing the expendiente, the court realized that Aurora's allegation of the fraud was legally insufficient to invalidate her marriage, and, on the authority of Brown vs. Yambao, 102 Phil. 168, holding: It is true that the wife has not interposed prescription as a defense. Nevertheless, the courts can take cognizance thereof, because actions seeking a decree of legal separation, or annulment of marriage, involve public interest, and it is the policy of our law that no such decree be issued if any legal obstacles thereto appear upon the record. the court a quo required plaintiff to show cause why her complaint should not be dismissed. Plaintiff Aurora submitted a memorandum in compliance therewith, but the court found it inadequate and thereby issued an order, dated 7 October 1966, for the dismissal of the complaint; it also denied reconsideration. The main issue is whether or not the non-disclosure to a wife by her husband of his pre-marital relationship with another woman is a ground for annulment of marriage. We must agree with the lower court that it is not. For fraud as a vice of consent in marriage, which may be a cause for its annulment, comes under Article 85, No. 4, of the Civil Code, which provides: ART. 85. A marriage may be annulled for any of the following causes, existing at the time of the marriage: xxx xxx xxx (4) That the consent of either party was obtained by fraud, unless such party afterwards, with full knowledge of the facts constituting the fraud, freely cohabited with the other as her husband or his wife, as the case may be; This fraud, as vice of consent, is limited exclusively by law to those kinds or species of fraud enumerated in Article 86, as follows: ART. 86. Any of the following circumstances shall constitute fraud referred to in number 4 of the preceding article: (1) Misrepresentation as to the identity of one of the contracting parties; (2) Non-disclosure of the previous conviction of the other party of a crime involving moral turpitude, and the penalty imposed was imprisonment for two years or more; (3) Concealment by the wife of the fact that at the time of the marriage, she was pregnant by a man other than her husband. No other misrepresentation or deceit as to character, rank, fortune or chastity shall constitute such fraud as will give grounds for action for the annulment of marriage. The intention of Congress to confine the circumstances that can constitute fraud as ground for annulment of marriage to the foregoing three cases may be deduced from the fact that, of all the causes of nullity enumerated in Article 85, fraud is the only one given special treatment in a subsequent article within the chapter on void and voidable marriages. If its intention were otherwise, Congress would have stopped at Article 85, for, anyway, fraud in general is already mentioned therein as a cause for annulment. But Article 86 was also enacted, expressly and specifically dealing with "fraud referred to in number 4 of the preceding article," and proceeds by enumerating the specific frauds (misrepresentation as to identity, nondisclosure of a previous conviction, and concealment of pregnancy), making it clear that Congress intended to exclude all other frauds or deceits. To stress further such intention, the enumeration of the specific frauds was followed by the interdiction: "No other misrepresentation or deceit as to character, rank, fortune or chastity shall constitute such fraud as will give grounds for action for the annulment of marriage." 403

University of the Cordilleras College of Law First Year C S.Y. 2013 - 2014 Non-disclosure of a husband's pre-marital relationship with another woman is not one of the enumerated circumstances that would constitute a ground for annulment; and it is further excluded by the last paragraph of the article, providing that "no other misrepresentation or deceit as to ... chastity" shall give ground for an action to annul a marriage. While a woman may detest such non-disclosure of premarital lewdness or feel having been thereby cheated into giving her consent to the marriage, nevertheless the law does not assuage her grief after her consent was solemnly given, for upon marriage she entered into an institution in which society, and not herself alone, is interested. The lawmaker's intent being plain, the Court's duty is to give effect to the same, whether it agrees with the rule or not. But plaintiff-appellant Anaya emphasizes that not only has she alleged "non-divulgement" (the word chosen by her) of the pre-marital relationship of her husband with another woman as her cause of action, but that she has, likewise, alleged in her reply that defendant Fernando paid court to her without any intention of complying with his marital duties and obligations and covertly made up his mind not to live with her. Plaintiff-appellant contends that the lower court erred in ignoring these allegations in her reply. This second set of averments which were made in the reply (pretended love and absence of intention to perform duties of consortium) is an entirely new and additional "cause of action." According to the plaintiff herself, the second set of allegations is "apart, distinct and separate from that earlier averred in the Complaint ..." (Record on Appeal, page 76). Said allegations were, therefore, improperly alleged in the reply, because if in a reply a party-plaintiff is not permitted to amend or change the cause of action as set forth in his complaint (Calo vs. Roldan, 76 Phil. 445), there is more reason not to allow such party to allege a new and additional cause of action in the reply. Otherwise, the series of pleadings of the parties could become interminable. On the merits of this second fraud charge, it is enough to point out that any secret intention on the husband's part not to perform his marital duties must have been discovered by the wife soon after the marriage: hence her action for annulment based on that fraud should have been brought within four years after the marriage. Since appellant's wedding was celebrated in December of 1953, and this ground was only pleaded in 1966, it must be declared already barred. FOR THE FOREGOING REASONS, the appealed order is hereby affirmed. No costs. Concepcion, C.J., Makalintal, Zaldivar, Castro, Fernando, Teehankee, Barredo and Villamor, JJ., concur. Dizon and Makasiar, JJ., are on leave.

404

University of the Cordilleras College of Law First Year C S.Y. 2013 - 2014 Case Digest AURORA A. ANAYA vs. FERNANDO O. PALAROAN G.R. No. L-27930 November 26, 1970 36 SCRA 97 REYES, J.B.L., J. FACTS: Aurora Anaya and Fernando Palaroan were married in 1953. Palaroan filed for an action for annulment of the marriage in 1954 on the ground that his consent was obtained through force and intimidation. The complaint was dismissed and upheld the validity of the marriage and granting Auroras counterclaim. While the amount of counterclaim was being negotiated, Fernando divulged to her that several months prior to their marriage, he had premarital relationship with a close relative of his. According to her, the non-divulgement to her of such premarital secret constituted fraud in obtaining her consent. She prayed for the annulment of her marriage with Fernando on such ground. ISSUE: Whether or not the concealment to a wife by her husband of his premarital relationship with another woman is a ground for annulment of marriage. RULING: The concealment of a husbands premarital relationship with another woman was not one of those enumerated that would constitute fraud as ground for annulment and it is further excluded by the last paragraph providing that no other misinterpretation or deceit as tochastity shall give ground for an action to annul marriage. Hence, the case at bar does not constitute fraud and therefore would not warrant an annulment of marriage.

405

University of the Cordilleras College of Law First Year C S.Y. 2013 - 2014 Republic vs Nolasco 220 SCRA 20 G.R. No. 94053 March 17, 1993 Full Case REPUBLIC OF THE PHILIPPINES, petitioner, vs. GREGORIO NOLASCO, respondent. FELICIANO, J.: On 5 August 1988, respondent Gregorio Nolasco filed before the Regional Trial Court of Antique, Branch 10, a petition for the declaration of presumptive death of his wife Janet Monica Parker, invoking Article 41 of the Family Code. The petition prayed that respondent's wife be declared presumptively dead or, in the alternative, that the marriage be declared null and void. The Republic of the Philippines opposed the petition through the Provincial Prosecutor of Antique who had been deputized to assist the Solicitor-General in the instant case. The Republic argued, first, that Nolasco did not possess a "well-founded belief that the absent spouse was already dead," and second, Nolasco's attempt to have his marriage annulled in the same proceeding was a "cunning attempt" to circumvent the law on marriage. During trial, respondent Nolasco testified that he was a seaman and that he had first met Janet Monica Parker, a British subject, in a bar in England during one of his ship's port calls. From that chance meeting onwards, Janet Monica Parker lived with respondent Nolasco on his ship for six (6) months until they returned to respondent's hometown of San Jose, Antique on 19 November 1980 after his seaman's contract expired. On 15 January 1982, respondent married Janet Monica Parker in San Jose, Antique, in Catholic rites officiated by Fr. Henry van Tilborg in the Cathedral of San Jose. Respondent Nolasco further testified that after the marriage celebration, he obtained another employment contract as a seaman and left his wife with his parents in San Jose, Antique. Sometime in January 1983, while working overseas, respondent received a letter from his mother informing him that Janet Monica had given birth to his son. The same letter informed him that Janet Monica had left Antique. Respondent claimed he then immediately asked permission to leave his ship to return home. He arrived in Antique in November 1983. Respondent further testified that his efforts to look for her himself whenever his ship docked in England proved fruitless. He also stated that all the letters he had sent to his missing spouse at No. 38 Ravena Road, Allerton, Liverpool, England, the address of the bar where he and Janet Monica first met, were all returned to him. He also claimed that he inquired from among friends but they too had no news of Janet Monica. On cross-examination, respondent stated that he had lived with and later married Janet Monica Parker despite his lack of knowledge as to her family background. He insisted that his wife continued to refuse to give him such information even after they were married. He also testified that he did not report the matter of Janet Monica's disappearance to the Philippine government authorities. Respondent Nolasco presented his mother, Alicia Nolasco, as his witness. She testified that her daughterin-law Janet Monica had expressed a desire to return to England even before she had given birth to Gerry Nolasco on 7 December 1982. When asked why her daughter-in-law might have wished to leave Antique, respondent's mother replied that Janet Monica never got used to the rural way of life in San Jose, Antique. Alicia Nolasco also said that she had tried to dissuade Janet Monica from leaving as she had given birth to her son just fifteen days before, but when she (Alicia) failed to do so, she gave Janet Monica P22,000.00 for her expenses before she left on 22 December 1982 for England. She further claimed that she had no information as to the missing person's present whereabouts. The trial court granted Nolasco's petition in a Judgment dated 12 October 1988 the dispositive portion of which reads:

406

University of the Cordilleras College of Law First Year C S.Y. 2013 - 2014 Wherefore, under Article 41, paragraph 2 of the Family Code of the Philippines (Executive Order No. 209, July 6, 1987, as amended by Executive Order No. 227, July 17, 1987) this Court hereby declares as presumptively dead Janet Monica Parker Nolasco, without prejudice to her reappearance. The Republic appealed to the Court of Appeals contending that the trial court erred in declaring Janet Monica Parker presumptively dead because respondent Nolasco had failed to show that there existed a well founded belief for such declaration. The Court of Appeals affirmed the trial court's decision, holding that respondent had sufficiently established a basis to form a belief that his absent spouse had already died. The Republic, through the Solicitor-General, is now before this Court on a Petition for Review where the following allegations are made: 1. The Court of Appeals erred in affirming the trial court's finding that there existed a well-founded belief on the part of Nolasco that Janet Monica Parker was already dead; and 2. The Court of Appeals erred in affirming the trial Court's declaration that the petition was a proper case of the declaration of presumptive death under Article 41, Family Code. The issue before this Court, as formulated by petitioner is "[w]hether or not Nolasco has a well-founded belief that his wife is already dead." The present case was filed before the trial court pursuant to Article 41 of the Family Code which provides that: Art. 41. A marriage contracted by any person during the subsistence of a previous marriage shall be null and void, unless before the celebration of the subsequent marriage, the prior spouse had been absent for four consecutive years and the spouse present had a well-founded belief that the absent spouse was already dead. In case of disappearance where there is danger of death under the circumstances set forth in the provision of Article 391 of the Civil Code, an absence of only two years shall be sufficient. For the purpose of contracting the subsequent marriage under the preceding paragraph, the spouse present must institute a summary proceeding as provided in this Code for the declaration of presumptive death of the absentee, without prejudice to the effect of reappearance of the absent spouse. (Emphasis supplied). When Article 41 is compared with the old provision of the Civil Code, which it superseded, the following crucial differences emerge. Under Article 41, the time required for the presumption to arise has been shortened to four (4) years; however, there is need for a judicial declaration of presumptive death to enable the spouse present to remarry. Also, Article 41 of the Family Code imposes a stricter standard than the Civil Code: Article 83 of the Civil Code merely requires either that there be no news that such absentee is still alive; or the absentee is generally considered to be dead and believed to be so by the spouse present, or is presumed dead under Article 390 and 391 of the Civil Code. The Family Code, upon the other hand, prescribes as "well founded belief" that the absentee is already dead before a petition for declaration of presumptive death can be granted. As pointed out by the Solicitor-General, there are four (4) requisites for the declaration of presumptive death under Article 41 of the Family Code: 1. That the absent spouse has been missing for four consecutive years, or two consecutive years if the disappearance occurred where there is danger of death under the circumstances laid down in Article 391, Civil Code; 2. That the present spouse wishes to remarry; 3. That the present spouse has a well-founded belief that the absentee is dead; and 407

University of the Cordilleras College of Law First Year C S.Y. 2013 - 2014 4. That the present spouse files a summary proceeding for the declaration of presumptive death of the absentee. Respondent naturally asserts that he had complied with all these requirements. Petitioner's argument, upon the other hand, boils down to this: that respondent failed to prove that he had complied with the third requirement, i.e., the existence of a "well-founded belief" that the absent spouse is already dead. The Court believes that respondent Nolasco failed to conduct a search for his missing wife with such diligence as to give rise to a "well-founded belief" that she is dead. United States v. Biasbas, is instructive as to degree of diligence required in searching for a missing spouse. In that case, defendant Macario Biasbas was charged with the crime of bigamy. He set-up the defense of a good faith belief that his first wife had already died. The Court held that defendant had not exercised due diligence to ascertain the whereabouts of his first wife, noting that: While the defendant testified that he had made inquiries concerning the whereabouts of his wife, he fails to state of whom he made such inquiries. He did not even write to the parents of his first wife, who lived in the Province of Pampanga, for the purpose of securing information concerning her whereabouts. He admits that he had a suspicion only that his first wife was dead. He admits that the only basis of his suspicion was the fact that she had been absent. . . . In the case at bar, the Court considers that the investigation allegedly conducted by respondent in his attempt to ascertain Janet Monica Parker's whereabouts is too sketchy to form the basis of a reasonable or well-founded belief that she was already dead. When he arrived in San Jose, Antique after learning of Janet Monica's departure, instead of seeking the help of local authorities or of the British Embassy, he secured another seaman's contract and went to London, a vast city of many millions of inhabitants, to look for her there. Q After arriving here in San Jose, Antique, did you exert efforts to inquire the whereabouts of your wife? A Yes, Sir. Court: How did you do that? A I secured another contract with the ship and we had a trip to London and I went to London to look for her I could not find her (sic). (Emphasis supplied) Respondent's testimony, however, showed that he confused London for Liverpool and this casts doubt on his supposed efforts to locate his wife in England. The Court of Appeal's justification of the mistake, to wit: . . . Well, while the cognoscente (sic) would readily know the geographical difference between London and Liverpool, for a humble seaman like Gregorio the two places could mean one place in England, the port where his ship docked and where he found Janet. Our own provincial folks, every time they leave home to visit relatives in Pasay City, Kalookan City, or Paraaque, would announce to friends and relatives, "We're going to Manila." This apparent error in naming of places of destination does not appear to be fatal. is not well taken. There is no analogy between Manila and its neighboring cities, on one hand, and London and Liverpool, on the other, which, as pointed out by the Solicitor-General, are around three hundred fifty (350) kilometers apart. We do not consider that walking into a major city like Liverpool or

408

University of the Cordilleras College of Law First Year C S.Y. 2013 - 2014 London with a simple hope of somehow bumping into one particular person there which is in effect what Nolasco says he did can be regarded as a reasonably diligent search. The Court also views respondent's claim that Janet Monica declined to give any information as to her personal background even after she had married respondent too convenient an excuse to justify his failure to locate her. The same can be said of the loss of the alleged letters respondent had sent to his wife which respondent claims were all returned to him. Respondent said he had lost these returned letters, under unspecified circumstances. Neither can this Court give much credence to respondent's bare assertion that he had inquired from their friends of her whereabouts, considering that respondent did not identify those friends in his testimony. The Court of Appeals ruled that since the prosecutor failed to rebut this evidence during trial, it is good evidence. But this kind of evidence cannot, by its nature, be rebutted. In any case, admissibility is not synonymous with credibility. As noted before, there are serious doubts to respondent's credibility. Moreover, even if admitted as evidence, said testimony merely tended to show that the missing spouse had chosen not to communicate with their common acquaintances, and not that she was dead. Respondent testified that immediately after receiving his mother's letter sometime in January 1983, he cut short his employment contract to return to San Jose, Antique. However, he did not explain the delay of nine (9) months from January 1983, when he allegedly asked leave from his captain, to November 1983 when be finally reached San Jose. Respondent, moreover, claimed he married Janet Monica Parker without inquiring about her parents and their place of residence. Also, respondent failed to explain why he did not even try to get the help of the police or other authorities in London and Liverpool in his effort to find his wife. The circumstances of Janet Monica's departure and respondent's subsequent behavior make it very difficult to regard the claimed belief that Janet Monica was dead a well-founded one. In Goitia v. Campos-Rueda, the Court stressed that: . . . Marriage is an institution, the maintenance of which in its purity the public is deeply interested. It is a relationship for life and the parties cannot terminate it at any shorter period by virtue of any contract they make. . . . . (Emphasis supplied) By the same token, the spouses should not be allowed, by the simple expedient of agreeing that one of them leave the conjugal abode and never to return again, to circumvent the policy of the laws on marriage. The Court notes that respondent even tried to have his marriage annulled before the trial court in the same proceeding. In In Re Szatraw, the Court warned against such collusion between the parties when they find it impossible to dissolve the marital bonds through existing legal means. While the Court understands the need of respondent's young son, Gerry Nolasco, for maternal care, still the requirements of the law must prevail. Since respondent failed to satisfy the clear requirements of the law, his petition for a judicial declaration of presumptive death must be denied. The law does not view marriage like an ordinary contract. Article 1 of the Family Code emphasizes that. . . . Marriage is a special contract of permanent union between a man and a woman entered into in accordance with law for the establishment of conjugal and family life. It is the foundation of the familyand an inviolable social institution whose nature, consequences, and incidents are governed by law and not subject to stipulation, except that marriage settlements may fix the property relations during the marriage within the limits provided by this Code. (Emphasis supplied) In Arroyo, Jr. v. Court of Appeals, the Court stressed strongly the need to protect. . . . the basic social institutions of marriage and the family in the preservation of which the State bas the strongest interest; the public policy here involved is of the most fundamental kind. In Article II, Section 12 of the Constitution there is set forth the following basic state policy: The State recognizes the sanctity of family life and shall protect and strengthen the family as a basic autonomous social institution. . . . 409

University of the Cordilleras College of Law First Year C S.Y. 2013 - 2014 The same sentiment bas been expressed in the Family Code of the Philippines in Article 149: The family, being the foundation of the nation, is a basic social institution which public policy cherishes and protects. Consequently, family relations are governed by law and no custom, practice or agreement destructive of the family shall be recognized or given effect. In fine, respondent failed to establish that he had the well-founded belief required by law that his absent wife was already dead that would sustain the issuance of a court order declaring Janet Monica Parker presumptively dead. WHEREFORE, the Decision of the Court of Appeals dated 23 February 1990, affirming the trial court's decision declaring Janet Monica Parker presumptively dead is hereby REVERSED and both Decisions are hereby NULLIFIED and SET ASIDE. Costs against respondent. Bidin, Davide, Jr., Romero and Melo, JJ., concur. Gutierrez, Jr. J., is on leave.

410

University of the Cordilleras College of Law First Year C S.Y. 2013 - 2014 Case Digest REPUBLIC OF THE PHILIPPINES vs. GREGORIO NOLASCO G.R. No. 94053 March 17, 1993 220 SCRA 20 FELICIANO, J. FACTS: Gregorio Nolasco is a seaman. He met Janet Parker, a British, in a bar in England. After that, Janet started living with Nolasco in his ship for six months. It lasted until the contract of Nolasco expired then he brought her to his hometown in Antique. They got married in January 1982. Due to another contract, Nolasco left the province. In 1983, Nolasco received a letter from his mother informing him that his son had been born but 15 days after, Janet left. Nolasco went home and cut short his contract to find Janets whereabouts. He did so by securing another seamans contract going to London. He wrote several letters to the bar where they first met but it was all returned. Gregorio petitioned in 1988 for a declaration of presumptive death of Janet. ISSUE: Whether or not Nolasco had a well-founded belief that his wife, Janet, is already dead. RULING: The Supreme Court ruled that Nolascos efforts to locate Janet were not persistent to show that he has a well-founded belief that his wife was already dead because instead of seeking assistance of local authorities and the British Embassy, he even secured another contract. More so, while he was in London, he did not even try to solicit help of the authorities to find his wife.

411

University of the Cordilleras College of Law First Year C S.Y. 2013 - 2014 Tuason vs Court of Appeals 256 SCRA 15 G.R. No. 116607. April 10, 1996 Full Case EMILIO TUASON, petitioner, vs COURT OF APPEALS and MARIA VICTORIA L. TUASON, respondents. PUNO, J.: This petition for review on certiorari seeks to annul and set aside the decision dated July 29, 1994 of the Court of Appeals in CA-G.R. CV No. 37925 denying petitioners appeal from an order of the Regional Trial Court, Branch 149, Makati in Civil Case No. 3769. This case arose from the following facts: In 1989, private respondent Maria Victoria Lopez Tuason filed with the Regional Trial Court, Branch 149, Makati a petition for annulment or declaration of nullity of her marriage to petitioner Emilio R. Tuason. In her complaint, private respondent alleged that she and petitioner were married on June 3, 1972 and from this union, begot two children; that at the time of the marriage, petitioner was already psychologically incapacitated to comply with his essential marital obligations which became manifest afterward and resulted in violent fights between husband and wife; that in one of their fights, petitioner inflicted physical injuries on private respondent which impelled her to file a criminal case for physical injuries against him; that petitioner used prohibited drugs, was apprehended by the authorities and sentenced to a one-year suspended penalty and has not been rehabilitated; that petitioner was a womanizer, and in 1984, he left the conjugal home and cohabited with three women in succession, one of whom he presented to the public as his wife; that after he left the conjugal dwelling, petitioner gave minimal support to the family and even refused to pay for the tuition fees of their children compelling private respondent to accept donations and dole-outs from her family and friends; that petitioner likewise became a spendthrift and abused his administration of the conjugal partnership by alienating some of their assets and incurring large obligations with banks, credit card companies and other financial institutions, without private respondents consent; that attempts at reconciliation were made but they all failed because of petitioners refusal to reform. In addition to her prayer for annulment of marriage, priv ate respondent prayed for powers of administration to save the conjugal properties from further dissipation. Petitioner answered denying the imputations against him. As affirmative defense, he claimed that he and private respondent were a normal married couple during the first ten years of their marriage and actually begot two children during this period; that it was only in 1982 that they began to have serious personal differences when his wife did not accord the respect and dignity due him as a husband but treated him like apersona non grata; that due to the extreme animosities between them, he temporarily left the conjugal home for a cooling-off period in 1984; that it is private respondent who had been taking prohibited drugs and had a serious affair with another man; that petitioners work as owner and operator of a radio and television station exposed him to malicious gossip linking him to various women in media and the entertainment world; and that since 1984, he experienced financial reverses in his business and was compelled, with the knowledge of his wife, to dispose of some of the conjugal shares in exclusive golf and country clubs. Petitioner petitioned the court to allow him to return to the conjugal home and continue his administration of the conjugal partnership. After the issues were joined, trial commenced on March 30, 1990. Private respondent presented four witnesses, namely, herself; Dr. Samuel Wiley, a Canon Law expert and marriage counselor of both private respondent and petitioner; Ms. Adelita Prieto, a close friend of the spouses, and Any. Jose F. Racela IV, private respondents counsel. Private respondent likewise submitted documentary evidence consisting of newspaper articles of her husbands relationship with other women, his apprehension by the authorities for illegal possession of drugs; and copies of a prior church annulment decree. The parties marriage was clerically annulled by the Tribunal Metropolitanum Matrimoniale which was affirmed by the National Appellate Matrimonial Tribunal in 1986. During presentation of private respondents evidence, petitioner, on April 18, 1990, filed his Opposition to private respondents petition for appointment as administratrix of the conjugal partnership of gains. After private respondent rested her case, the trial court scheduled the reception of petitioners evidence on May 11, 1990. 412

University of the Cordilleras College of Law First Year C S.Y. 2013 - 2014 On May 8, 1990, two days before the scheduled hearing, a counsel for petitioner moved for a postponement on the ground that the principal counsel was out of the country and due to return on the first week of June. The court granted the motion and reset the hearing to June 8, 1990. On June 8, 1990, petitioner failed to appear. On oral motion of private respondent, the court declared petitioner to have waived his right to present evidence and deemed the case submitted for decision on the basis of the evidence presented. On June 29, 1990, the trial court rendered judgment declaring the nullity of private respondents marriage to petitioner and awarding custody of the children to private respondent. The court ruled: WHEREFORE, in view of the foregoing, the marriage contracted by Ma. Victoria L. Tuason and Emilio R. Tuason on June 3, 1972 is declared null and void oh initio on the ground of psychological incapacity on the part of the defendant under Sec. 36 of the Family Code. Let herein judgment of annulment be recorded in the registry of Mandaluyong, Metro Manila where the marriage was contracted and in the registry of Makati, Metro Manila where the marriage is annulled. The custody of the two (2) legitimate children of the plaintiff and the defendant is hereby awarded to the plaintiff. The foregoing judgment is without prejudice to the application of the other effects of annulment as provided for under Arts. 50 and 51 of the Family Code of the Philippines. Counsel for petitioner received a copy of this decision on August 24, 1990. No appeal was taken from the decision. On September 24, 1990, private respondent filed a Motion for Dissolution of Conjugal Partnership of Gains and Adjudication to Plaintiff of the Conjugal Properties. Petitioner opposed the motion on October 17, 1990 Also on the same day, October 17, 1990, petitioner, through new counsel, filed with the trial court a petition for relief from judgment of the June 29, 1990 decision. The trial court denied the petition on August 8, 1991. Petitioner appealed before the Court of Appeals the order of the trial court denying his petition for relief from judgment. On July 29, 1994, the Court of Appeals dismissed the appeal and affirmed the order of the trial court. Hence this petition. The threshold issue is whether a petition for relief from judgment is warranted under the circumstances of the case. We rule in the negative. A petition for relief from judgment is governed by Rule 38, Section 2 of the Revised Rules of Court which provides: Section 2. Petition to Court of First Instance for relief from judgment or other proceedings thereof. When a judgment or order is entered, or any other proceeding is taken, against a party in a court of first instance through fraud, accident, mistake, or excusable negligence, he may file a petition in such court and in the same cause praying that the judgment, order or proceeding be set aside. Under the rules, a final and executory judgment or order of the Regional Trial Court may be set aside on the ground of fraud, accident, mistake or excusable negligence. In addition, the petitioner must assert facts showing that he has a good, substantial and meritorious defense or cause of action. If the petition is granted, the court shall proceed to hear and determine the case as if a timely motion for new trial had been granted therein. In the case at bar, the decision annulling petitioners marriage to private respondent had already become final and executory when petitioner failed to appeal during the reglementary period. Petitioner however claims that the decision of the trial court was null and void for violation of his right to due process. He contends he was denied due process when, after failing to appear on two scheduled hearings, the trial court deemed him to have waived his right to present evidence and rendered judgment on the basis of the evidence for private respondent. Petitioner justifies his absence at the hearings on the ground that he was then confined for medical and/or rehabilitation reasons. In his affidavit of merit before the 413

University of the Cordilleras College of Law First Year C S.Y. 2013 - 2014 trial court, he attached a certification by Lt. Col. Plaridel F. Vidal, Director of the Narcotics Command, Drug Rehabilitation Center which states that on March 27, 1990 petitioner was admitted for treatment of drug dependency at the Drug Rehabilitation Center at Camp Bagong Diwa, Bicutan, Taguig, Metro Manila of the Philippine Constabulary-Integrated National Police. The records, however, show that the former counsel of petitioner did not inform the trial court of this confinement. And when the court rendered its decision, the same counsel was out of the country for which reason the decision became final and executory as no appeal was taken therefrom. The failure of petitioners counsel to notify him on time of the adverse judgment to enable him to appeal therefrom is negligence which is not excusable. Notice sent to counsel of record is binding upon the client and the neglect or failure of counsel to inform him of an adverse judgment resulting in the loss of his right to appeal is not a ground for setting aside a judgment valid and regular on its face. Similarly inexcusable was the failure of his former counsel to inform the trial court of petitioners confinement and medical treatment as the reason for his non-appearance at the scheduled hearings. Petitioner has not given any reason why his former counsel, intentionally or unintentionally, did not inform the court of this fact. This led the trial court to order the case deemed submitted for decision on the basis of the evidence presented by the private respondent alone. To compound the negligence of petitioners counsel, the order of the trial court was never assailed via a motion for reconsideration. Clearly, petitioner cannot now claim that he was deprived of due process. He may have lost his right to present evidence but he was not denied his day in court. As the records show, petitioner, through counsel, actively participated in the proceedings below. He filed his answer to the petition, crossexamined private respondents witnesses and even submitted his opposition to private respondents motion for dissolution of the conjugal partnership of gains. A petition for relief from judgment is an equitable remedy; it is allowed only in exceptional cases where there is no other available or adequate remedy. When a party has another remedy available to him, which may be either a motion for new trial or appeal from an adverse decision of the trial court, and he was not prevented by fraud, accident, mistake or excusable negligence from filing such motion or taking such appeal, he cannot avail himself of this petition. Indeed, relief will not be granted to a party who seeks avoidance from the effects of the judgment when the loss of the remedy at law was due to his own negligence; otherwise the petition for relief can be used to revive the right to appeal which had been lost thru inexcusable negligence. Petitioner also insists that he has a valid and meritorious defense. He cites the Family Code which provides that in actions for annulment of marriage or legal separation, the prosecuting officer should intervene for the state because the law looks with disfavor upon the haphazard declaration of annulment of marriages by default. He contends that when he failed to appear at the scheduled hearings, the trial court should have ordered the prosecuting officer to intervene for the state and inquire as to the reason for his non-appearance. Articles 48 and 60 of the Family Code read as follows: Art. 48. In all cases of annulment or declaration of absolute nullity of marriage, the Court shall order the prosecution attorney or fiscal assigned to it to appear on behalf of the State to take steps to prevent collusion between the parties and to take care that evidence is not fabricated or suppressed. In the cases referred to in the preceding paragraph, no judgment shall be based upon a stipulation of facts or confession of judgment. xxx xxx xxx

Art. 60. No decree of legal separation shall be based upon a stipulation of facts or a confession of judgment. In any case, the Court shall order the prosecuting attorney or fiscal assigned to it to take steps to prevent collusion between the parties and to take care that the evidence is not fabricated or suppressed. A grant of annulment of marriage or legal separation by default is fraught with the danger of collusion. Hence, in all cases for annulment, declaration of nullity of marriage and legal separation, the prosecuting attorney or fiscal is ordered to appear on behalf of the state for the purpose of preventing any collusion between the parties and to take care that their evidence is not fabricated or suppressed. If the defendant spouse fails to answer the complaint, the court cannot declare him or her in default but instead, 414

University of the Cordilleras College of Law First Year C S.Y. 2013 - 2014 should order the prosecuting attorney to determine if collusion exists between the parties. The prosecuting attorney or fiscal may oppose the application for legal separation or annulment through the presentation of his own evidence, if in his opinion, the proof adduced is dubious and fabricated. Our Constitution is committed to the policy of strengthening the family as a basic social institution. Our family law is based on the policy that marriage is not a mere contract, but a social institution in which the state is vitally interested. The state can find no stronger anchor than on good, solid and happy families. The break up of families weakens our social and moral fabric and, hence, their preservation is not the concern alone of the family members. The facts in the case at bar do not call for the strict application of Articles 48 and 60 of the Family Code. For one, petitioner was not declared in default by the trial court for failure to answer. Petitioner filed his answer to the complaint and contested the cause of action alleged by private respondent. He actively participated in the proceedings below by filing several pleadings and cross-examining the witnesses of private respondent. It is crystal clear that every stage of the litigation was characterized by a no-holds barred contest and not by collusion. The role of the prosecuting attorney or fiscal in annulment of marriage and legal separation proceedings is to determine whether collusion exists between the parties and to take care that the evidence is not suppressed or fabricated. Petitioners vehement opposition to the annulment proceedings negates the conclusion that collusion existed between the parties. There is no allegation by the petitioner that evidence was suppressed or fabricated by any of the parties. Under these circumstances, we are convinced that the non-intervention of a prosecuting attorney to assure lack of collusion between the contending parties is not fatal to the validity of the proceedings in the trial court. Petitioner also refutes the testimonies of private respondents witnesses, particularly Dr. Sa muel Wiley and Ms. Adelita Prieto, as biased, incredible and hearsay. Petitioner alleges that if he were able to present his evidence, he could have testified that he was not psychologically incapacitated at the time of the marriage as indicated by the fact that during their first ten years, he and private respondent lived together with their children as one normal and happy family, that he continued supporting his family even after he left the conjugal dwelling and that his work as owner and operator of a radio and television corporation places him in the public eye and makes him a good subject for malicious gossip linking him with various women. These facts, according to petitioner, should disprove the ground for annulment of his marriage to petitioner. Suffice it to state that the finding of the trial court as to the existence or non-existence of petitioners psychological incapacity at the time of the marriage is final and binding on us. Petitioner has not sufficiently shown that the trial courts factual findings and evaluation of the testimonies of private respondents witnesses vis-a-vis petitioners defenses are clearly and manifestly erroneous. IN VIEW WHEREOF, the petition is denied and the decision dated July 29, 1994 of the Court of Appeals in CA-G.R. CV No. 37925 is affirmed. SO ORDERED. Regalado (Chairman), Romero, and Mendoza, JJ., concur. Torres, Jr., J., on leave.

415

University of the Cordilleras College of Law First Year C S.Y. 2013 - 2014 Case Digest EMILIO TUASON vs. COURT OF APPEALS and MARIA VICTORIA L. TUASON G.R. No. 116607 April 10, 1996 256 SCRA 15 PUNO, J. FACTS: In 1989, private respondent Maria Victoria Lopez Tuason filed with the Regional Trial Court, Makati a petition for annulment or declaration of nullity of her marriage to petition Emilio R. Tuason. In her complaint, private respondent alleged that she and petitioner were married on June 3, 1972 and from this union, begot two children; that at the time of the marriage, petitioner was already psychologically incapacitated to comply with his essential marital obligations which became manifest afterward and resulted in violent fights between husband and wife; that petitioner used prohibited drugs, was apprehended by the authorities and sentenced to a one-year suspended penalty and has not been rehabilitated; that petitioner was a womanizer; that after he left the conjugal dwelling, petitioner gave minimal support to the family and even refused to pay for the tuition fees of their children compelling private respondent to accept donations and doleouts from her family and friends; that attempts at reconciliation were made but they all failed because of petitioners refusal to reform. In addition to her prayer for annulment, private respondent prayed for powers of administration to save the conjugal properties from further dissipation. After the issues were joined, the trial commenced on March 30, 1990. Private respondent presented her evidences. On the scheduled reception of the petitioners evidences on May 11, 1990, petitioners counsel filed for postponed on the ground that the principal counsel was out of the country. The court granted the motion and reset the hearing on June 8, 1990. On the said date, petitioner failed to appear. On oral motion private respondent, the court declared petitioner to have waived his right to present evidence and deemed the case submitted for decision on the basis of the evidence presented. On June 29, 1990, the trial court rendered judgment declaring the nullity of private respondents marriage to petitioner and awarding custody of the children to private respondent. The petitioner appealed but was again denied by the CA. At the Supreme Court, petitioner alleges that the proper process for annulment of marriage or legal separation was not properly followed by the trial court. ISSUE: Whether or not the trial court erred in the process of rendering the marriage null and void. RULING: A grant of annulment of marriage or legal separation by default is fraught with the danger of collusion. Hence, in all cases for annulment, declaration of nullity of marriage and legal separation, the prosecuting attorney or fiscal is ordered to appear on behalf of the state for the purpose of preventing any collusion between the parties and to take care that their evidence is not fabricated or suppressed. If the defendant spouse fails to answer the complaint, the court cannot declare him/her in default but instead, should order the prosecuting attorney to determine if collusion exists between the parties. The prosecuting attorney or fiscal may oppose the application for legal separation or annulment through the presentation of his evidence, if in his opinion, the proof adduced is dubious and fabricated. Our Constitution is committed to the policy of strengthening the family as a basic social institution. Our family law is based on the policy that marriage is not a mere contract, but a social institution in which the state is vitally interested. The facts at bar do not call for the strict implementation of Article 48 and 60 of the Family Code. For one, petitioner was not declared in default by the trial court for failure to answer. Petitioner actively participated in the proceedings below by filing several pleadings and cross-examining the witnesses of private respondent. It is crystal clear that every stage of the litigation was characterized by no-holds barred contest and not by collusion. The role of the prosecuting attorney or fiscal in annulment of marriage and legal separation proceedings is to determine whether collusion exists between the parties and to take care that the evidence is not suppressed or fabricated. Petitioners vehement opposition to the annulment proceedings negates the conclusion that collusion existed between the parties. There is no allegation by the petitioner that evidence was suppressed or fabricated by any of the parties. Under these circumstances, we are convinced that the non-intervention of a prosecuting attorney to assure lack of collusion between the contending parties is not fatal to the validity of the proceedings of the court.

416

University of the Cordilleras College of Law First Year C S.Y. 2013 - 2014

Atienza vs Brillante Jr. 243 SCRA 32


A.M. No. MTJ-92-706 March 29, 1995 Full Case LUPO ALMODIEL ATIENZA, complainant, vs. JUDGE FRANCISCO F. BRILLANTES, JR., Metropolitan Trial Court, Branch 28, Manila, respondent. QUIASON, J.: This is a complaint by Lupo A. Atienza for Gross Immorality and Appearance of Impropriety against Judge Francisco Brillantes, Jr., Presiding Judge of the Metropolitan Trial Court, Branch 20, Manila. Complainant alleges that he has two children with Yolanda De Castro, who are living together at No. 34 Galaxy Street, Bel-Air Subdivision, Makati, Metro Manila. He stays in said house, which he purchased in 1987, whenever he is in Manila. In December 1991, upon opening the door to his bedroom, he saw respondent sleeping on his (complainant's) bed. Upon inquiry, he was told by the houseboy that respondent had been cohabiting with De Castro. Complainant did not bother to wake up respondent and instead left the house after giving instructions to his houseboy to take care of his children. Thereafter, respondent prevented him from visiting his children and even alienated the affection of his children for him. Complainant claims that respondent is married to one Zenaida Ongkiko with whom he has five children, as appearing in his 1986 and 1991 sworn statements of assets and liabilities. Furthermore, he alleges that respondent caused his arrest on January 13, 1992, after he had a heated argument with De Castro inside the latter's office. For his part, respondent alleges that complainant was not married to De Castro and that the filing of the administrative action was related to complainant's claim on the Bel-Air residence, which was disputed by De Castro. Respondent denies that he caused complainant's arrest and claims that he was even a witness to the withdrawal of the complaint for Grave Slander filed by De Castro against complainant. According to him, it was the sister of De Castro who called the police to arrest complainant. Respondent also denies having been married to Ongkiko, although he admits having five children with her. He alleges that while he and Ongkiko went through a marriage ceremony before a Nueva Ecija town mayor on April 25, 1965, the same was not a valid marriage for lack of a marriage license. Upon the request of the parents of Ongkiko, respondent went through another marriage ceremony with her in Manila on June 5, 1965. Again, neither party applied for a marriage license. Ongkiko abandoned respondent 17 years ago, leaving their children to his care and custody as a single parent. Respondent claims that when he married De Castro in civil rites in Los Angeles, California on December 4, 1991, he believed, in all good faith and for all legal intents and purposes, that he was single because his first marriage was solemnized without a license. Under the Family Code, there must be a judicial declaration of the nullity of a previous marriage before a party thereto can enter into a second marriage. Article 40 of said Code provides: The absolute nullity of a previous marriage may be invoked for the purposes of remarriage on the basis solely of a final judgment declaring such previous marriage void. Respondent argues that the provision of Article 40 of the Family Code does not apply to him considering that his first marriage took place in 1965 and was governed by the Civil Code of the Philippines; while the second marriage took place in 1991 and governed by the Family Code.

417

University of the Cordilleras College of Law First Year C S.Y. 2013 - 2014 Article 40 is applicable to remarriages entered into after the effectivity of the Family Code on August 3, 1988 regardless of the date of the first marriage. Besides, under Article 256 of the Family Code, said Article is given "retroactive effect insofar as it does not prejudice or impair vested or acquired rights in accordance with the Civil Code or other laws." This is particularly true with Article 40, which is a rule of procedure. Respondent has not shown any vested right that was impaired by the application of Article 40 to his case. The fact that procedural statutes may somehow affect the litigants' rights may not preclude their retroactive application to pending actions. The retroactive application of procedural laws is not violative of any right of a person who may feel that he is adversely affected (Gregorio v. Court of Appeals, 26 SCRA 229 [1968]). The reason is that as a general rule no vested right may attach to, nor arise from, procedural laws (Billones v. Court of Industrial Relations, 14 SCRA 674 [1965]). Respondent is the last person allowed to invoke good faith. He made a mockery of the institution of marriage and employed deceit to be able to cohabit with a woman, who beget him five children. Respondent passed the Bar examinations in 1962 and was admitted to the practice of law in 1963. At the time he went through the two marriage ceremonies with Ongkiko, he was already a lawyer. Yet, he never secured any marriage license. Any law student would know that a marriage license is necessary before one can get married. Respondent was given an opportunity to correct the flaw in his first marriage when he and Ongkiko were married for the second time. His failure to secure a marriage license on these two occasions betrays his sinister motives and bad faith. It is evident that respondent failed to meet the standard of moral fitness for membership in the legal profession. While the deceit employed by respondent existed prior to his appointment as a Metropolitan Trial Judge, his immoral and illegal act of cohabiting with De Castro began and continued when he was already in the judiciary. The Code of Judicial Ethics mandates that the conduct of a judge must be free of a whiff of impropriety, not only with respect to his performance of his judicial duties but also as to his behavior as a private individual. There is no duality of morality. A public figure is also judged by his private life. A judge, in order to promote public confidence in the integrity and impartiality of the judiciary, must behave with propriety at all times, in the performance of his judicial duties and in his everyday life. These are judicial guideposts too self-evident to be overlooked. No position exacts a greater demand on moral righteousness and uprightness of an individual than a seat in the judiciary (Imbing v. Tiongzon, 229 SCRA 690 [1994]). WHEREFORE, respondent is DISMISSED from the service with forfeiture of all leave and retirement benefits and with prejudice to reappointment in any branch, instrumentality, or agency of the government, including government-owned and controlled corporations. This decision is immediately executory. SO ORDERED. Narvasa, C.J., Feliciano, Padilla, Bidin, Regalado, Davide, Jr., Romero, Bellosillo, Melo, Puno, Vitug, Kapunan, Mendoza and Francisco, JJ., concur.

418

University of the Cordilleras College of Law First Year C S.Y. 2013 - 2014 Case Digest LUPO ALMODIEL ATIENZA vs. JUDGE FRANCISCO F. BRILLANTES, JR. A.M. No. MTJ-92-706 March 29, 1995 243 SCRA 32 QUIASON, J. FACTS: Lupo Atienza lived together with Yolanda de Castro with whom he has two children. He purchased a house in Bel-Air, Makati where his family stayed. He stays there too whenever hes in Manila. In Dec., 1991, he was surprised to see Manila Metropolitan Trial Court Judge Francisco Brillantes sleeping on his bed. Their boy informed him that Brillantes had been cohabiting with de Castro. Later on, Brillantes prevented him from visiting his children. He claims that Brillantes is married to Zenaida Ongkiko with whom he has five children. Atienza filed a complaint for Gross Immorality & Appearance of Impropriety against Brillantes. Brillantes claims that his marriage to Ongkiko is not valid because of lack of marriage license. According to him, Ongkiko abandoned him 19 years ago leaving their children with him. He claims that he believed that he was single when he married de Castro because his first marriage was void. ISSUE: Whether or not Brillantes can contract a second marriage without a judicial declaration of nullity. RULING: No. He is dismissed from service, FC Art. 40: judicial declaration of nullity of previous marriage is needed before one can enter into a second marriage. Rule has retroactive effect thus applicable to Brillantes even if he got married under the Civil Code. Bad faith and sinister motives of Brillantes proven by his marriage to Ongkiko. They underwent two ceremonies however he never got a license. Then, he immorally and illegally cohabited with de Castro. Not fit for the judiciary.

419

University of the Cordilleras College of Law First Year C S.Y. 2013 - 2014 Domingo vs Court of Appeals 226 SCRA 572 G.R. No. 104818 September 17, 1993 Full Case ROBERTO DOMINGO, petitioner, vs. COURT OF APPEALS and DELIA SOLEDAD AVERA represented by her Attorney-in-Fact MOISES R. AVERA,respondents. ROMERO, J.: The instant petition seeks the reversal of respondent court's ruling finding no grave abuse of discretion in the lower court's order denying petitioner's motion to dismiss the petition for declaration of nullity of marriage and separation of property. On May 29, 1991, private respondent Delia Soledad A. Domingo filed a petition before the Regional Trial Court of Pasig entitled "Declaration of Nullity of Marriage and Separation of Property" against petitioner Roberto Domingo. The petition which was docketed as Special Proceedings No. 1989-J alleged among others that: they were married on November 29, 1976 at the YMCA Youth Center Bldg., as evidenced by a Marriage Contract Registry No. 1277K-76 with Marriage License No. 4999036 issued at Carmona, Cavite; unknown to her, he had a previous marriage with one Emerlina dela Paz on April 25, 1969 which marriage is valid and still existing; she came to know of the prior marriage only sometime in 1983 when Emerlina dela Paz sued them for bigamy; from January 23 1979 up to the present, she has been working in Saudi Arabia and she used to come to the Philippines only when she would avail of the one-month annual vacation leave granted by her foreign employer since 1983 up to the present, he has been unemployed and completely dependent upon her for support and subsistence; out of her personal earnings, she purchased real and personal properties with a total amount of approximately P350,000.00, which are under the possession and administration of Roberto; sometime in June 1989, while on her onemonth vacation, she discovered that he was cohabiting with another woman; she further discovered that he had been disposing of some of her properties without her knowledge or consent; she confronted him about this and thereafter appointed her brother Moises R. Avera as her attorney-in-fact to take care of her properties; he failed and refused to turn over the possession and administration of said properties to her brother/attorney-in-fact; and he is not authorized to administer and possess the same on account of the nullity of their marriage. The petition prayed that a temporary restraining order or a writ of preliminary injunction be issued enjoining Roberto from exercising any act of administration and ownership over said properties; their marriage be declared null and void and of no force and effect; and Delia Soledad be declared the sole and exclusive owner of all properties acquired at the time of their void marriage and such properties be placed under the proper management and administration of the attorney-in-fact. Petitioner filed a Motion to Dismiss on the ground that the petition stated no cause of action. The marriage being void ab initio, the petition for the declaration of its nullity is, therefore, superfluous and unnecessary. It added that private respondent has no property which is in his possession. On August 20, 1991, Judge Maria Alicia M. Austria issued an Order denying the motion to dismiss for lack of merit. She explained: Movant argues that a second marriage contracted after a first marriage by a man with another woman is illegal and void (citing the case of Yap v. Court of Appeals, 145 SCRA 229) and no judicial decree is necessary to establish the invalidity of a void marriage (citing the cases of People v. Aragon, 100 Phil. 1033; People v. Mendoza, 95 Phil. 845). Indeed, under the Yap case there is no dispute that the second marriage contracted by respondent with herein petitioner after a first marriage with another woman is illegal and void. However, as to whether or not the second marriage should first be judicially declared a nullity is not an issue in said case. In the case of Vda. de Consuegra v. GSIS, the Supreme Court ruled in explicit terms, thus: And with respect to the right of the second wife, this Court observed that although the second marriage can be presumed to be void ab initio as it was celebrated while the first marriage was still subsisting, still there is need for judicial declaration of its nullity. (37 SCRA 316, 326) 420

University of the Cordilleras College of Law First Year C S.Y. 2013 - 2014 The above ruling which is of later vintage deviated from the previous rulings of the Supreme Court in the aforecited cases of Aragon and Mendoza. Finally, the contention of respondent movant that petitioner has no property in his possession is an issue that may be determined only after trial on the merits. A motion for reconsideration was filed stressing the erroneous application of Vda. de Consuegra v. GSIS and the absence of justiciable controversy as to the nullity of the marriage. On September 11, 1991, Judge Austria denied the motion for reconsideration and gave petitioner fifteen (15) days from receipt within which to file his answer. Instead of filing the required answer, petitioner filed a special civil action of certiorari and mandamus on the ground that the lower court acted with grave abuse of discretion amounting to lack of jurisdiction in denying the motion to dismiss. On February 7, 1992, the Court of Appeals dismissed the petition. It explained that the case of Yap v. CA cited by petitioner and that of Consuegra v. GSIS relied upon by the lower court do not have relevance in the case at bar, there being no identity of facts because these cases dealt with the successional rights of the second wife while the instant case prays for separation of property corollary with the declaration of nullity of marriage. It observed that the separation and subsequent distribution of the properties acquired during the union can be had only upon proper determination of the status of the marital relationship between said parties, whether or not the validity of the first marriage is denied by petitioner. Furthermore, in order to avoid duplication and multiplicity of suits, the declaration of nullity of marriage may be invoked in this proceeding together with the partition and distribution of the properties involved. Citing Articles 48, 50 and 52 of the Family Code, it held that private respondent's prayer for declaration of absolute nullity of their marriage may be raised together with other incidents of their marriage such as the separation of their properties. Lastly, it noted that since the Court has jurisdiction, the alleged error in refusing to grant the motion to dismiss is merely one of law for which the remedy ordinarily would have been to file an answer, proceed with the trial and in case of an adverse decision, reiterate the issue on appeal. The motion for reconsideration was subsequently denied for lack of merit. Hence, this petition. The two basic issues confronting the Court in the instant case are the following. First, whether or not a petition for judicial declaration of a void marriage is necessary. If in the affirmative, whether the same should be filed only for purposes of remarriage. Second, whether or not SP No. 1989-J is the proper remedy of private respondent to recover certain real and personal properties allegedly belonging to her exclusively. Petitioner, invoking the ruling in People v. Aragon and People v. Mendoza, contends that SP. No. 1989-J for Declaration of Nullity of Marriage and Separation of Property filed by private respondent must be dismissed for being unnecessary and superfluous. Furthermore, under his own interpretation of Article 40 of the Family Code, he submits that a petition for declaration of absolute nullity of marriage is required only for purposes of remarriage. Since the petition in SP No. 1989-J contains no allegation of private respondent's intention to remarry, said petition should therefore, be dismissed. On the other hand, private respondent insists on the necessity of a judicial declaration of the nullity of their marriage, not for purposes of remarriage, but in order to provide a basis for the separation and distribution of the properties acquired during coverture. There is no question that the marriage of petitioner and private respondent celebrated while the former's previous marriage with one Emerlina de la Paz was still subsisting, is bigamous. As such, it is from the beginning. Petitioner himself does not dispute the absolute nullity of their marriage. The cases of People v. Aragon and People v. Mendoza relied upon by petitioner are cases where the Court had earlier ruled that no judicial decree is necessary to establish the invalidity of a void, bigamous 421

University of the Cordilleras College of Law First Year C S.Y. 2013 - 2014 marriage. It is noteworthy to observe that Justice Alex Reyes, however, dissented on these occasions stating that: Though the logician may say that where the former marriage was void there would be nothing to dissolve, still it is not for the spouses to judge whether that marriage was void or not. That judgment is reserved to the courts. . . . This dissenting opinion was adopted as the majority position in subsequent cases involving the same issue. Thus, in Gomez v. Lipana, the Court abandoned its earlier ruling in the Aragon and Mendoza cases. In reversing the lower court's order forfeiting the husband's share of the disputed property acquired during the second marriage, the Court stated that "if the nullity, or annulment of the marriage is the basis for the application of Article 1417, there is need for a judicial declaration thereof, which of course contemplates an action for that purpose." Citing Gomez v. Lipana, the Court subsequently held in Vda. de Consuegra v. Government Service Insurance System, that "although the second marriage can be presumed to be void ab initio as it was celebrated while the first marriage was still subsisting, still there is need for judicial declaration of such nullity." In Tolentino v. Paras, however, the Court turned around and applied the Aragon and Mendoza ruling once again. In granting the prayer of the first wife asking for a declaration as the lawful surviving spouse and the correction of the death certificate of her deceased husband, it explained that "(t)he second marriage that he contracted with private respondent during the lifetime of his first spouse is null and void from the beginning and of no force and effect. No judicial decree is necessary to establish the invalidity of a void marriage." However, in the more recent case of Wiegel v. Sempio-Diy the Court reverted to the Consuegra case and held that there was "no need of introducing evidence about the existing prior marriage of her first husband at the time they married each other, for then such a marriage though void still needs according to this Court a judicial declaration of such fact and for all legal intents and purposes she would still be regarded as a married woman at the time she contracted her marriage with respondent Karl Heinz Wiegel." Came the Family Code which settled once and for all the conflicting jurisprudence on the matter. A declaration of the absolute nullity of a marriage is now explicitly required either as a cause of action or a ground for defense. Where the absolute nullity of a previous marriage is sought to be invoked for purposes of contracting a second marriage, the sole basis acceptable in law for said projected marriage be free from legal infirmity is a final judgment declaring the previous marriage void. The Family Law Revision Committee and the Civil Code Revision Committee which drafted what is now the Family Code of the Philippines took the position that parties to a marriage should not be allowed to assume that their marriage is void even if such be the fact but must first secure a judicial declaration of the nullity of their marriage before they can be allowed to marry again. This is borne out by the following minutes of the 152nd Joint Meeting of the Civil Code and Family Law Committees where the present Article 40, then Art. 39, was discussed. B. Article 39. The absolute nullity of a marriage may be invoked only on the basis of a final judgment declaring the marriage void, except as provided in Article 41. Justice Caguioa remarked that the above provision should include not only void but also voidable marriages. He then suggested that the above provision be modified as follows: The validity of a marriage may be invoked only . . . Justice Reyes (J.B.L. Reyes), however, proposed that they say: The validity only . . . or invalidity of a marriage may be invoked

422

University of the Cordilleras College of Law First Year C S.Y. 2013 - 2014 On the other hand, Justice Puno suggested that they say: The invalidity of a marriage may be invoked only . . . Justice Caguioa explained that his idea is that one cannot determine for himself whether or not his marriage is valid and that a court action is needed. Justice Puno accordingly proposed that the provision be modified to read: The invalidity of a marriage may be invoked only on the basis of a final judgment annulling the marriage or declaring the marriage void, except as provided in Article 41. Justice Caguioa remarked that in annulment, there is no question. Justice Puno, however, pointed out that, even if it is a judgment of annulment, they still have to produce the judgment. Justice Caguioa suggested that they say: The invalidity of a marriage may be invoked only on the basis of a final judgment declaring the marriage invalid, except as provided in Article 41. Justice Puno raised the question: When a marriage is declared invalid, does it include the annulment of a marriage and the declaration that the marriage is void? Justice Caguioa replied in the affirmative. Dean Gupit added that in some judgments, even if the marriage is annulled, it is declared void. Justice Puno suggested that this matter be made clear in the provision. Prof. Baviera remarked that the original idea in the provision is to require first a judicial declaration of a void marriage and not annullable marriages, with which the other members concurred. Judge Diy added that annullable marriages are presumed valid until a direct action is filed to annul it, which the other members affirmed. Justice Puno remarked that if this is so, then the phrase "absolute nullity" can stand since it might result in confusion if they change the phrase to "invalidity" if what they are referring to in the provision is the declaration that the marriage is void. Prof. Bautista commented that they will be doing away with collateral defense as well as collateral attack. Justice Caguioa explained that the idea in the provision is that there should be a final judgment declaring the marriage void and a party should not declare for himself whether or not the marriage is void, while the other members affirmed. Justice Caguioa added that they are, therefore, trying to avoid a collateral attack on that point. Prof. Bautista stated that there are actions which are brought on the assumption that the marriage is valid. He then asked: Are they depriving one of the right to raise the defense that he has no liability because the basis of the liability is void? Prof. Bautista added that they cannot say that there will be no judgment on the validity or invalidity of the marriage because it will be taken up in the same proceeding. It will not be a unilateral declaration that, it is a void marriage. Justice Caguioa saw the point of Prof. Bautista and suggested that they limit the provision to remarriage. He then proposed that Article 39 be reworded as follows: The absolute nullity of a marriage for purposes of remarriage may be invoked only on the basis of final judgment . . . Justice Puno suggested that the above be modified as follows: The absolute nullity of a previous marriage may be invoked for purposes of establishing the validity of a subsequent marriage only on the basis of a final judgment declaring such previous marriage void, except as provided in Article 41.

423

University of the Cordilleras College of Law First Year C S.Y. 2013 - 2014 Justice Puno later modified the above as follows: For the purpose of establishing the validity of a subsequent marriage, the absolute nullity of a previous marriage may only be invoked on the basis of a final judgment declaring such nullity, except as provided in Article 41. Justice Caguioa commented that the above provision is too broad and will not solve the objection of Prof. Bautista. He proposed that they say: For the purpose of entering into a subsequent marriage, the absolute nullity of a previous marriage may only be invoked on the basis of a final judgment declaring such nullity, except as provided in Article 41. Justice Caguioa explained that the idea in the above provision is that if one enters into a subsequent marriage without obtaining a final judgment declaring the nullity of a previous marriage, said subsequent marriage is void ab initio. After further deliberation, Justice Puno suggested that they go back to the original wording of the provision as follows: The absolute nullity of a previous marriage may be invoked for purposes of remarriage only on the basis of a final judgment declaring such previous marriage void, except as provided in Article 41. In fact, the requirement for a declaration of absolute nullity of a marriage is also for the protection of the spouse who, believing that his or her marriage is illegal and void, marries again. With the judicial declaration of the nullity of his or her first marriage, the person who marries again cannot be charged with bigamy. Just over a year ago, the Court made the pronouncement that there is a necessity for a declaration of absolute nullity of a prior subsisting marriage before contracting another in the recent case of Terre v. Terre. The Court, in turning down the defense of respondent Terre who was charged with grossly immoral conduct consisting of contracting a second marriage and living with another woman other than complainant while his prior marriage with the latter remained subsisting, said that "for purposes of determining whether a person is legally free to contract a second marriage, a judicial declaration that the first marriage was null and void ab initio is essential." As regards the necessity for a judicial declaration of absolute nullity of marriage, petitioner submits that the same can be maintained only if it is for the purpose of remarriage. Failure to allege this purpose, according to petitioner's theory, will warrant dismissal of the same. Article 40 of the Family Code provides: Art. 40. The absolute nullity of a previous marriage may be invoked for purposes of remarriage on the basis solely of a final judgment declaring such previous marriage void. (n) Crucial to the proper interpretation of Article 40 is the position in the provision of the word "solely." As it is placed, the same shows that it is meant to qualify "final judgment declaring such previous marriage void." Realizing the need for careful craftsmanship in conveying the precise intent of the Committee members, the provision in question, as it finally emerged, did not state "The absolute nullity of a previous marriage may be invoked solely for purposes of remarriage . . .," in which case "solely" would clearly qualify the phrase "for purposes of remarriage." Had the phraseology been such, the interpretation of petitioner would have been correct and, that is, that the absolute nullity of a previous marriage may be invoked solely for purposes of remarriage, thus rendering irrelevant the clause "on the basis solely of a final judgment declaring such previous marriage void." That Article 40 as finally formulated included the significant clause denotes that such final judgment declaring the previous marriage void need not be obtained only for purposes of remarriage. Undoubtedly, 424

University of the Cordilleras College of Law First Year C S.Y. 2013 - 2014 one can conceive of other instances where a party might well invoke the absolute nullity of a previous marriage for purposes other than remarriage, such as in case of an action for liquidation, partition, distribution and separation of property between the erstwhile spouses, as well as an action for the custody and support of their common children and the delivery of the latters' presumptive legitimes. In such cases, evidence needs must be adduced, testimonial or documentary, to prove the existence of grounds rendering such a previous marriage an absolute nullity. These need not be limited solely to an earlier final judgment of a court declaring such previous marriage void. Hence, in the instance where a party who has previously contracted a marriage which remains subsisting desires to enter into another marriage which is legally unassailable, he is required by law to prove that the previous one was an absolute nullity. But this he may do on the basis solely of a final judgment declaring such previous marriage void. This leads us to the question: Why the distinction? In other words, for purposes of remarriage, why should the only legally acceptable basis for declaring a previous marriage an absolute nullity be a final judgment declaring such previous marriage void? Whereas, for purposes other than remarriage, other evidence is acceptable? Marriage, a sacrosanct institution, declared by the Constitution as an "inviolable social institution, is the foundation of the family;" as such, it "shall be protected by the State." In more explicit terms, the Family Code characterizes it as "a special contract of permanent union between a man and a woman entered into in accordance with law for the establishment of conjugal, and family life." So crucial are marriage and the family to the stability and peace of the nation that their "nature, consequences, and incidents are governed by law and not subject to stipulation . . ." As a matter of policy, therefore, the nullification of a marriage for the purpose of contracting another cannot be accomplished merely on the basis of the perception of both parties or of one that their union is so defective with respect to the essential requisites of a contract of marriage as to render it void ipso jure and with no legal effect and nothing more. Were this so, this inviolable social institution would be reduced to a mockery and would rest on very shaky foundations indeed. And the grounds for nullifying marriage would be as diverse and far-ranging as human ingenuity and fancy could conceive. For such a social significant institution, an official state pronouncement through the courts, and nothing less, will satisfy the exacting norms of society. Not only would such an open and public declaration by the courts definitively confirm the nullity of the contract of marriage, but the same would be easily verifiable through records accessible to everyone. That the law seeks to ensure that a prior marriage is no impediment to a second sought to be contracted by one of the parties may be gleaned from new information required in the Family Code to be included in the application for a marriage license, viz, "If previously married, how, when and where the previous marriage was dissolved and annulled." Reverting to the case before us, petitioner's interpretation of Art. 40 of the Family Code is, undoubtedly, quite restrictive. Thus, his position that private respondent's failure to state in the petition that the same is filed to enable her to remarry will result in the dismissal of SP No. 1989-J is untenable. His misconstruction of Art. 40 resulting from the misplaced emphasis on the term "solely" was in fact anticipated by the members of the Committee. Dean Gupit commented the word "only" may be misconstrued to refer to "for purposes of remarriage." Judge Diy stated that "only" refers to "final judgment." Justice Puno suggested that they say "on the basis only of a final judgment." Prof. Baviera suggested that they use the legal term "solely" instead of "only," which the Committee approved. (Emphasis supplied) Pursuing his previous argument that the declaration for absolute nullity of marriage is unnecessary, petitioner suggests that private respondent should have filed an ordinary civil action for the recovery of the properties alleged to have been acquired during their union. In such an eventuality, the lower court would not be acting as a mere special court but would be clothed with jurisdiction to rule on the issues of possession and ownership. In addition, he pointed out that there is actually nothing to separate or partition as the petition admits that all the properties were acquired with private respondent's money. The Court of Appeals disregarded this argument and concluded that "the prayer for declaration of absolute nullity of marriage may be raised together with the other incident of their marriage such as the separation of their properties."

425

University of the Cordilleras College of Law First Year C S.Y. 2013 - 2014 When a marriage is declared void ab initio, the law states that the final judgment therein shall provide for "the liquidation, partition and distribution of the properties of the spouses, the custody and support of the common children, and the delivery of their presumptive legitimes, unless such matters had been adjudicated in previous judicial proceedings." Other specific effects flowing therefrom, in proper cases, are the following: Art. 43. xxx xxx xxx (2) The absolute community of property or the conjugal partnership, as the case may be, shall be dissolved and liquidated, but if either spouse contracted said marriage in bad faith, his or her share of the net profits of the community property or conjugal partnership property shall be forfeited in favor of the common children or, if there are none, the children of the guilty spouse by a previous marriage or, in default of children, the innocent spouse; (3) Donations by reason of marriage shall remain valid, except that if the donee contracted the marriage in bad faith, such donations made to said donee are revoked by operation of law; (4) The innocent spouse may revoke the designation of the other spouse who acted in bad faith as a beneficiary in any insurance policy, even if such designation be stipulated as irrevocable; and (5) The spouse who contracted the subsequent marriage in bad faith shall be disqualified to inherit from the innocent spouse by testate and intestate succession. (n) Art. 44. If both spouses of the subsequent marriage acted in bad faith, said marriage shall be void ab initio and all donations by reason of marriage and testamentary disposition made by one in favor of the other are revoked by operation of law. (n) Based on the foregoing provisions, private respondent's ultimate prayer for separation of property will simply be one of the necessary consequences of the judicial declaration of absolute nullity of their marriage. Thus, petitioner's suggestion that in order for their properties to be separated, an ordinary civil action has to be instituted for that purpose is baseless. The Family Code has clearly provided the effects of the declaration of nullity of marriage, one of which is the separation of property according to the regime of property relations governing them. It stands to reason that the lower court before whom the issue of nullity of a first marriage is brought is likewise clothed with jurisdiction to decide the incidental questions regarding the couple's properties. Accordingly, the respondent court committed no reversible error in finding that the lower court committed no grave abuse of discretion in denying petitioner's motion to dismiss SP No. 1989-J. WHEREFORE, the instant petition is hereby DENIED. The decision of respondent Court dated February 7, 1992 and the Resolution dated March 20, 1992 are AFFIRMED. SO ORDERED. Bidin and Melo, JJ., concur. Feliciano, J., is on leave.

Separate Opinions VITUG, J., concurring: I concur with the opinion so well expressed by Mme. Justice Flerida Ruth P. Romero. I should like, however, to put in a modest observation.

426

University of the Cordilleras College of Law First Year C S.Y. 2013 - 2014 Void marriages are inexistent from the very beginning and, I believe, no judicial decree is required to establish their nullity, except in the following instances: (a) For purposes of remarriage pursuant to the provision of Article 40 of the Family Code; viz.: The absolute nullity of a previous marriage may be invoked for purposes of remarriage on the basis solely of a final judgment declaring such previous marriage void. (n) (b) A marriage celebrated prior to the effectivity of the Family Code in case a party thereto was psychologically incapacitated to comply with the essential marital obligations of marriage (Article 36, Family Code), where an action or defense for the declaration of nullity prescribes ten (10) years after the Family Code took effect (Article 39, Family Code); otherwise, the marriage is deemed unaffected by the Family Code. A void marriage, even without its being judicially declared a nullity, albeit the preferability for, and justiciability (fully discussed in the majority opinion) of, such a declaration, will not give it the status or the consequences of a valid marriage, saving only specific instances where certain effects of a valid marriage can still flow from the void marriage. Examples of these cases are children of void marriages under Article 36 (due to psychological incapacity) and Article 53, in relation to Article 52 (due to failure of partition, delivery of presumptive legitimes of children and recording thereof following the annulment or declaration of nullity a prior marriage), conceived or born before the judicial declaration of nullity of such void marriages, who the law deems as legitimate (Article 54, Family Code). In most, if not in all, other cases, a void marriage is to be considered extant per se. Neither the conjugal, partnership of gain under the old regime nor the absolute community of property under the new Code (absent a marriage settlement), will apply; instead, their property relations shall be governed by the coownership rules under either Article 147 or Article 148 of the Family Code. I must hasten to add as a personal view, however, that the exceptional effects on children of a void marriage because of the psychological incapacity of a party thereto should have been extended to cover even the personal and property relations of the spouses. Unlike the other cases of void marriages where the grounds therefor may be established by hard facts and with little uncertainty, the term "psychological incapacity" is so relative and unsettling that until a judicial declaration of nullity is made its interim effects can long and literally hang on the balance not only insofar as the spouses themselves are concerned but also as regards third persons with whom the spouses deal.

# Separate Opinions VITUG, J., concurring: I concur with the opinion so well expressed by Mme. Justice Flerida Ruth P. Romero. I should like, however, to put in a modest observation. Void marriages are inexistent from the very beginning and, I believe, no judicial decree is required to establish their nullity, except in the following instances: (a) For purposes of remarriage pursuant to the provision of Article 40 of the Family Code; viz.: The absolute nullity of a previous marriage may be invoked for purposes of remarriage on the basis solely of a final judgment declaring such previous marriage void. (n) (b) A marriage celebrated prior to the effectivity of the Family Code in case a party thereto was psychologically incapacitated to comply with the essential marital obligations of marriage (Article 36, Family Code), where an action or defense for the declaration of nullity prescribes ten (10) years after the Family Code took effect (Article 39, Family Code); otherwise, the marriage is deemed unaffected by the Family Code. A void marriage, even without its being judicially declared a nullity, albeit the preferability for, and justiciability (fully discussed in the majority opinion) of, such a declaration, will not give it the status or 427

University of the Cordilleras College of Law First Year C S.Y. 2013 - 2014 the consequences of a valid marriage, saving only specific instances where certain effects of a valid marriage can still flow from the void marriage. Examples of these cases are children of void marriages under Article 36 (due to psychological incapacity) and Article 53, in relation to Article 52 (due to failure of partition, delivery of presumptive legitimes of children and recording thereof following the annulment or declaration of nullity a prior marriage), conceived or born before the judicial declaration of nullity of such void marriages, who the law deems as legitimate (Article 54, Family Code). In most, if not in all, other cases, a void marriage is to be considered extant per se. Neither the conjugal, partnership of gain under the old regime nor the absolute community of property under the new Code (absent a marriage settlement), will apply; instead, their property relations shall be governed by the coownership rules under either Article 147 or Article 148 of the Family Code. I must hasten to add as a personal view, however, that the exceptional effects on children of a void marriage because of the psychological incapacity of a party thereto should have been extended to cover even the personal and property relations of the spouses. Unlike the other cases of void marriages where the grounds therefor may be established by hard facts and with little uncertainty, the term "psychological incapacity" is so relative and unsettling that until a judicial declaration of nullity is made its interim effects can long and literally hang on the balance not only insofar as the spouses themselves are concerned but also as regards third persons with whom the spouses deal.

428

University of the Cordilleras College of Law First Year C S.Y. 2013 - 2014 Case Digest ROBERTO DOMINGO vs. COURT OF APPEALS G.R. No. 104818 September 17, 1993 226 SCRA 572 ROMERO, J. FACTS: Delia Domingo, private respondent, filed a petition before RTC of Pasig for the declaration of nullity of marriage and separation of property against Roberto Domingo, petitioner. She alleged that they were married at Carmona, Cavite with evidences of marriage certificate and marriage license, unknown to her, petitioner had a previous marriage with Emerlina dela Paz which is still valid and existing. She came to know the prior marriage when Emerlina sued them for bigamy. She prays that their marriage be declared null and void and, as a consequence, to declare that she is the exclusive owner of all properties she acquired during the marriage and to recover them from him. Roberto moved to dismiss the petition on the ground that the marriage being void ab initio, the petition of declaration of nullity is unnecessary. It added that private respondent has no property which in his possession. ISSUE: Whether or not respondent may claim for the declaration of nullity of marriage and separation of property against petitioner on the ground of bigamy. RULING: There is no question that the marriage of petitioner and private respondent celebrated while the former's previous marriage with one Emerlina de la Paz was still subsisting is bigamous. As such, it is from the beginning. Petitioner himself does not dispute the absolute nullity of their marriage. The Court had ruled that no judicial decree is necessary to establish the invalidity of a void, bigamous marriage. The Family Code has clearly provided the effects of the declaration of nullity of marriage, one of which is the separation of property according to the regime of property relations governing them. It stands to reason that the lower court before whom the issue of nullity of a first marriage is brought is likewise clothed with jurisdiction to decide the incidental questions regarding the couple's properties.

429

University of the Cordilleras College of Law First Year C S.Y. 2013 - 2014 Beltran vs People 334 SCRA 106 G.R. No. 137567. June 20, 2000 Full Case MEYNARDO L. BELTRAN, petitioner, vs PEOPLE OF THE PHILIPPINES, and HON. JUDGE FLORENTINO TUAZON, JR., being the Judge of the RTC, Branch 139, Makati City, respondents. BUENA, J.: This petition for review, filed under Rule 45 of the 1997 Rules of Civil Procedure, seeks to review and set aside the Order dated January 28, 1999 issued by Judge Florentino A. Tuazon, Jr. of the Regional Trial Court of Makati City, Branch 139 in Special Civil Case No. 98-3056, entitled "Meynardo Beltran vs. People of the Philippines and Hon. Judge Alden Cervantes of the Metropolitan Trial Court of Makati city, Branch 61." The said Order denied petitioners prayer for the issuance of a writ of preliminary injunction to enjoin Judge Cervantes from proceeding with the trial of Criminal Case No. 236176, a concubinage case against petitioner on the ground that the pending petition for declaration of nullity of marriage filed by petitioner against his wife constitutes a prejudicial question. The antecedent facts of the case are undisputed: Petitioner Meynardo Beltran and wife Charmaine E. Felix were married on June 16, 1973 at the Immaculate Concepcion Parish Church in Cubao, Quezon City. On February 7, 1997, after twenty-four years of marriage and four children, petitioner filed a petition for nullity of marriage on the ground of psychological incapacity under Article 36 of the Family Code before Branch 87 of the Regional Trial Court of Quezon City. The case was docketed as Civil Case No. Q-9730192. In her Answer to the said petition, petitioner's wife Charmaine Felix alleged that it was petitioner who abandoned the conjugal home and lived with a certain woman named Milagros Salting. Charmaine subsequently filed a criminal complaint for concubinage under Article 334 of the Revised Penal Code against petitioner and his paramour before the City Prosecutor's Office of Makati who, in a Resolution dated September 16, 1997, found probable cause and ordered the filing of an Information against them. The case, docketed as Criminal Case No. 236176, was filed before the Metropolitan Trial Court of Makati City, Branch 61. On March 20, 1998, petitioner, in order to forestall the issuance of a warrant for his arrest, filed a Motion to Defer Proceedings Including the Issuance of the Warrant of Arrest in the criminal case. Petitioner argued that the pendency of the civil case for declaration of nullity of his marriage posed a prejudicial question to the determination of the criminal case. Judge Alden Vasquez Cervantes denied the foregoing motion in the Order dated August 31, 1998. Petitioner's motion for reconsideration of the said Order of denial was likewise denied in an Order dated December 9, 1998. In view of the denial of his motion to defer the proceedings in the concubinage case, petitioner went to the Regional Trial Court of Makati City, Branch 139 on certiorari, questioning the Orders dated August 31, 1998 and December 9, 1998 issued by Judge Cervantes and praying for the issuance of a writ of preliminary injunction. In an Order dated January 28, 1999, the Regional Trial Court of Makati denied the petition forcertiorari. Said Court subsequently issued another Order dated February 23, 1999, denying his motion for reconsideration of the dismissal of his petition. Undaunted, petitioner filed the instant petition for review. Petitioner contends that the pendency of the petition for declaration of nullity of his marriage based on psychological incapacity under Article 36 of the Family Code is a prejudicial question that should merit the suspension of the criminal case for concubinage filed against him by his wife. Petitioner also contends that there is a possibility that two conflicting decisions might result from the civil case for annulment of marriage and the criminal case for concubinage. In the civil case, the trial court 430

University of the Cordilleras College of Law First Year C S.Y. 2013 - 2014 might declare the marriage as valid by dismissing petitioner's complaint but in the criminal case, the trial court might acquit petitioner because the evidence shows that his marriage is void on ground of psychological incapacity. Petitioner submits that the possible conflict of the courts' ruling regarding petitioner's marriage can be avoided, if the criminal case will be suspended, until the court rules on the validity of marriage; that if petitioner's marriage is declared void by reason of psychological incapacity then by reason of the arguments submitted in the subject petition, his marriage has never existed; and that, accordingly, petitioner could not be convicted in the criminal case because he was never before a married man. Petitioner's contentions are untenable. The rationale behind the principle of prejudicial question is to avoid two conflicting decisions. It has two essential elements: (a) the civil action involves an issue similar or intimately related to the issue raised in the criminal action; and (b) the resolution of such issue determines whether or not the criminal action may proceed. The pendency of the case for declaration of nullity of petitioner's marriage is not a prejudicial question to the concubinage case. For a civil case to be considered prejudicial to a criminal action as to cause the suspension of the latter pending the final determination of the civil case, it must appear not only that the said civil case involves the same facts upon which the criminal prosecution would be based, but also that in the resolution of the issue or issues raised in the aforesaid civil action, the guilt or innocence of the accused would necessarily be determined. Article 40 of the Family Code provides: "The absolute nullity of a previous marriage may be invoked for purposes of remarriage on the basis solely of a final judgment declaring such previous marriage void." In Domingo vs. Court of Appeals, this Court ruled that the import of said provision is that for purposes of remarriage, the only legally acceptable basis for declaring a previous marriage an absolute nullity is a final judgment declaring such previous marriage void, whereas, for purposes of other than remarriage, other evidence is acceptable. The pertinent portions of said Decision read: "xxx Undoubtedly, one can conceive of other instances where a party might well invoke the absolute nullity of a previous marriage for purposes other than remarriage, such as in case of an action for liquidation, partition, distribution and separation of property between the erstwhile spouses, as well as an action for the custody and support of their common children and the delivery of the latters' presumptive legitimes. In such cases, evidence needs must be adduced, testimonial or documentary, to prove the existence of grounds rendering such a previous marriage an absolute nullity. These needs not be limited solely to an earlier final judgment of a court declaring such previous marriage void." So that in a case for concubinage, the accused, like the herein petitioner need not present a final judgment declaring his marriage void for he can adduce evidence in the criminal case of the nullity of his marriage other than proof of a final judgment declaring his marriage void. With regard to petitioner's argument that he could be acquitted of the charge of concubinage should his marriage be declared null and void, suffice it to state that even a subsequent pronouncement that his marriage is void from the beginning is not a defense. Analogous to this case is that of Landicho vs. Reloval cited in Donato vs. Luna where this Court held that: "xxx Assuming that the first marriage was null and void on the ground alleged by petitioner, that fact would not be material to the outcome of the criminal case. Parties to the marriage should not be permitted to judge for themselves its nullity, for the same must be submitted to the judgment of the competent courts and only when the nullity of the marriage is so declared can it be held as void, and so long as there is no such declaration the presumption is that the marriage exists. Therefore, he who contracts a 431

University of the Cordilleras College of Law First Year C S.Y. 2013 - 2014 second marriage before the judicial declaration of nullity of the first marriage assumes the risk of being prosecuted for bigamy." Thus, in the case at bar it must also be held that parties to the marriage should not be permitted to judge for themselves its nullity, for the same must be submitted to judgment of the competent courts and only when the nullity of the marriage is so declared can it be held as void, and so long as there is no such declaration the presumption is that the marriage exists for all intents and purposes. Therefore, he who cohabits with a woman not his wife before the judicial declaration of nullity of the marriage assumes the risk of being prosecuted for concubinage. The lower court therefore, has not erred in affirming the Orders of the judge of the Metropolitan Trial Court ruling that pendency of a civil action for nullity of marriage does not pose a prejudicial question in a criminal case for concubinage. WHEREFORE, for lack of merit, the instant petition is DISMISSED. SO ORDERED. Bellosillo, (Chairman), Mendoza, Quisumbing, and De Leon, Jr., JJ., concur.

432

University of the Cordilleras College of Law First Year C S.Y. 2013 - 2014 Case Digest MEYNARDO L. BELTRAN vs. PEOPLE OF THE PHILIPPINES G.R. No. 137567 June 20, 2000 334 SCRA 106 BUENA, J. FACTS: In 1973, Beltran and Charmaine Felix married each other. Theyve had 4 children since then but after 24 years of marriage Beltran filed an action for the declaration of the nullity of their marriage due to Felixs PI. Felix countered that Beltran left the conjugal home to cohabit with a certain Milagros and that she filed a case of concubinage against Beltran. In 1997, the lower court found probable cause against Beltran and Milagros. In order to forestall the issuance of a warrant of arrest against him, Beltran raised the issue that the civil case he filed is a prejudicial question to the criminal case filed by Milagros. He said that the courts hearing the cases may issue conflicting rulings if the criminal case will not be suspended until the civil case gets resolved. The lower court denied Beltrans petition and so did Judge Tuazon of the RTC upon appeal. Beltran then elevated the case to the SC. ISSUE: Whether or not the absolute nullity of a previous marriage be invoked as a prejudicial question in the case at bar. RULING: The rationale behind the principle of prejudicial question is to avoid two conflicting decisions. It has two essential elements: (a) the civil action involves an issue similar or intimately related to the issue raised in the criminal action; and (b) the resolution of such issue determines whether or not the criminal action may proceed. The pendency of the case for declaration of nullity of Beltrans marriage is not a prejudicial question to the concubinage case. For a civil case to be considered prejudicial to a criminal action as to cause the suspension of the latter pending the final determination of the civil case, it must appear not only that the said civil case involves the same facts upon which the criminal prosecution would be based, but also that in the resolution of the issue or issues raised in the aforesaid civil action, the guilt or innocence of the accused would necessarily be determined. The SC ruled that the import of said provision is that for purposes of remarriage, the only legally acceptable basis for declaring a previous marriage an absolute nullity is a final judgment declaring such previous marriage void, whereas, for purposes of other than remarriage, other evidence is acceptable. In a case for concubinage, the accused (Beltran) need not present a final judgment declaring his marriage void for he can adduce evidence in the criminal case of the nullity of his marriage other than proof of a final judgment declaring his marriage void. With regard to Beltrans argument that he could be acquitted of the charge of concubinage should his marriage be declared null and void, suffice it to state that even a subsequent pronouncement that his marriage is void from the beginning is not a defense.

433

University of the Cordilleras College of Law First Year C S.Y. 2013 - 2014 Mercado vs Tan 337 SCRA 122 G.R. No. 137110. August 1, 2000 Full Case VINCENT PAUL G. MERCADO a.k.a. VINCENT G. MERCADO, petitioner, vs CONSUELO TAN, respondent. PANGANIBAN, J.: A judicial declaration of nullity of a previous marriage is necessary before a subsequent one can be legally contracted. One who enters into a subsequent marriage without first obtaining such judicial declaration is guilty of bigamy. This principle applies even if the earlier union is characterized by statute as void.

The Case Before us is a Petition for Review on Certiorari assailing the July 14, 1998 Decision of the Court of Appeals (CA) in CA-GR CR No. 19830 and its January 4, 1999 Resolution denying reconsideration. The assailed Decision affirmed the ruling of the Regional Trial Court (RTC) of Bacolod City in Criminal Case No. 13848, which convicted herein petitioner of bigamy as follows: WHEREFORE, finding the guilt of accused Dr. Vincent Paul G. Mercado a.k.a. Dr. Vincent G. Mercado of the crime of Bigamy punishable under Article 349 of the Revised Penal Code to have been proven beyond reasonable doubt, [the court hereby renders] judgment imposing upon him a prison term of three (3) years, four (4) months and fifteen (15) days of prision correccional, as minimum of his indeterminate sentence, to eight (8) years and twenty-one (21) days of prision mayor, as maximum, plus accessory penalties provided by law. Costs against accused.

The Facts The facts are quoted by Court of Appeals (CA) from the trial courts judgment, as follows: From the evidence adduced by the parties, there is no dispute that accused Dr. Vincent Mercado and complainant Ma. Consuelo Tan got married on June 27, 1991 before MTCC-Bacolod City Br. 7 Judge Gorgonio J. Ibaez [by reason of] which a Marriage Contract was duly executed and signed by the parties. As entered in said document, the status of accused was single. There is no dispute either that at the time of the celebration of the wedding with complainant, accused was actually a married man, having been in lawful wedlock with Ma. Thelma Oliva in a marriage ceremony solemnized on April 10, 1976 by Judge Leonardo B. Caares, CFI-Br. XIV, Cebu City per Marriage Certificate issued in connection therewith, which matrimony was further blessed by Rev. Father Arthur Baur on October 10, 1976 in religious rites at the Sacred Heart Church, Cebu City. In the same manner, the civil marriage between accused and complainant was confirmed in a church ceremony on June 29, 1991 officiated by Msgr. Victorino A. Rivas, Judicial Vicar, Diocese of Bacolod City. Both marriages were consummated when out of the first consortium, Ma. Thelma Oliva bore accused two children, while a child, Vincent Paul, Jr. was sired by accused with complainant Ma. Consuelo Tan. On October 5, 1992, a letter-complaint for bigamy was filed by complainant through counsel with the City Prosecutor of Bacolod City, which eventually resulted [in] the institution of the present case before this Court against said accused, Dr. Vincent G. Mercado, on March 1, 1993 in an Information dated January 22, 1993. On November 13, 1992, or more than a month after the bigamy case was lodged in the Prosecutors Office, accused filed an action for Declaration of Nullity of Marriage against Ma. Thelma V. Oliva in RTC-Br. 22, Cebu City, and in a Decision dated May 6, 1993 the marriage between Vincent G. Mercado and Ma. Thelma V. Oliva was declared null and void. 434

University of the Cordilleras College of Law First Year C S.Y. 2013 - 2014 Accused is charged [with] bigamy under Article 349 of the Revised Penal Code for having contracted a second marriage with herein complainant Ma. Consuelo Tan on June 27, 1991 when at that time he was previously united in lawful marriage with Ma. Thelma V. Oliva on April 10, 1976 at Cebu City, without said first marriage having been legally dissolved. As shown by the evidence and admitted by accused, all the essential elements of the crime are present, namely: (a) that the offender has been previously legally married; (2) that the first marriage has not been legally dissolved or in case the spouse is absent, the absent spouse could not yet be presumed dead according to the Civil Code; (3) that he contract[ed] a second or subsequent marriage; and (4) that the second or subsequent marriage ha[d] all the essential requisites for validity. x x x While acknowledging the existence of the two marriage[s], accused posited the defense that his previous marriage ha[d] been judicially declared null and void and that the private complainant had knowledge of the first marriage of accused. It is an admitted fact that when the second marriage was entered into with Ma. Consuelo Tan on June 27, 1991, accuseds prior marriage with Ma. Thelma V. Oliva was subsisting, no judicial action having yet been initiated or any judicial declaration obtained as to the nullity of such prior marriage with Ma. Thelma V. Oliva. Since no declaration of the nullity of his first marriage ha[d] yet been made at the time of his second marriage, it is clear that accused was a married man when he contracted such second marriage with complainant on June 27, 1991. He was still at the time validly married to his first wife.

Ruling of the Court of Appeals Agreeing with the lower court, the Court of Appeals stated: Under Article 40 of the Family Code, the absolute nullity of a previous marriage may be invoked for purposes of remarriage on the basis solely of a final judgment declaring such previous marriage void. But here, the final judgment declaring null and void accuseds previous ma rriage came not before the celebration of the second marriage, but after, when the case for bigamy against accused was already tried in court. And what constitutes the crime of bigamy is the act of any person who shall contract a second subsequent marriage before the former marriage has been legally dissolved. Hence, this Petition.

The Issues In his Memorandum, petitioner raises the following issues: A Whether or not the element of previous legal marriage is present in order to convict petitioner. B Whether or not a liberal interpretation in favor of petitioner of Article 349 of the Revised Penal Code punishing bigamy, in relation to Articles 36 and 40 of the Family Code, negates the guilt of petitioner. C Whether or not petitioner is entitled to an acquittal on the basis of reasonable doubt.

The Courts Ruling The Petition is not meritorious.

435

University of the Cordilleras College of Law First Year C S.Y. 2013 - 2014 Main Issue:Effect of Nullity of Previous Marriage Petitioner was convicted of bigamy under Article 349 of the Revised Penal Code, which provides: The penalty of prision mayor shall be imposed upon any person who shall contract a second or subsequent marriage before the former marriage has been legally dissolved, or before the absent spouse has been declared presumptively dead by means of a judgment rendered in the proper proceedings. The elements of this crime are as follows: 1. That the offender has been legally married;

2. That the marriage has not been legally dissolved or, in case his or her spouse is absent, the absent spouse could not yet be presumed dead according to the Civil Code; 3. That he contracts a second or subsequent marriage; 4. That the second or subsequent marriage has all the essential requisites for validity. When the Information was filed on January 22, 1993, all the elements of bigamy were present. It is undisputed that petitioner married Thelma G. Oliva on April 10, 1976 in Cebu City. While that marriage was still subsisting, he contracted a second marriage, this time with Respondent Ma. Consuelo Tan who subsequently filed the Complaint for bigamy. Petitioner contends, however, that he obtained a judicial declaration of nullity of his first marriage under Article 36 of the Family Code, thereby rendering it void ab initio. Unlike voidable marriages which are considered valid until set aside by a competent court, he argues that a void marriage is deemed never to have taken place at all. Thus, he concludes that there is no first marriage to speak of. Petitioner also quotes the commentaries of former Justice Luis Reyes that it is now settled that if the first marriage is void from the beginning, it is a defense in a bigamy charge. But if the first marriage is voidable, it is not a defense. Respondent, on the other hand, admits that the first marriage was declared null and void under Article 36 of the Family Code, but she points out that that declaration came only after the Information had been filed. Hence, by then, the crime had already been consummated. She argues that a judicial declaration of nullity of a void previous marriage must be obtained before a person can marry for a subsequent time. We agree with the respondent. To be sure, jurisprudence regarding the need for a judicial declaration of nullity of the previous marriage has been characterized as conflicting. In People v. Mendoza, a bigamy case involving an accused who married three times, the Court ruled that there was no need for such declaration. In that case, the accused contracted a second marriage during the subsistence of the first. When the first wife died, he married for the third time. The second wife then charged him with bigamy. Acquitting him, the Court held that the second marriage was void ab initio because it had been contracted while the first marriage was still in effect. Since the second marriage was obviously void and illegal, the Court ruled that there was no need for a judicial declaration of its nullity. Hence, the accused did not commit bigamy when he married for the third time. This ruling was affirmed by the Court in People v. Aragon, which involved substantially the same facts. But in subsequent cases, the Court impressed the need for a judicial declaration of nullity. In Vda de Consuegra v. GSIS, Jose Consuegra married for the second time while the first marriage was still subsisting. Upon his death, the Court awarded one half of the proceeds of his retirement benefits to the first wife and the other half to the second wife and her children, notwithstanding the manifest nullity of the second marriage. It held: And with respect to the right of the second wife, this Court observes that although the second marriage can be presumed to be void ab initio as it was celebrated while the first marriage was still subsisting, still there is need for judicial declaration of such nullity. In Tolentino v. Paras, however, the Court again held that judicial declaration of nullity of a void marriage was not necessary. In that case, a man married twice. In his Death Certificate, his second wife was named as his surviving spouse. The first wife then filed a Petition to correct the said entry in the Death Certificate. The Court ruled in favor of the first wife, holding that the second marriage that he 436

University of the Cordilleras College of Law First Year C S.Y. 2013 - 2014 contracted with private respondent during the lifetime of the first spouse is null and void from the beginning and of no force and effect. No judicial decree is necessary to establish the invalidity of a void marriage. In Wiegel v. Sempio-Diy, the Court stressed the need for such declaration. In that case, Karl Heinz Wiegel filed an action for the declaration of nullity of his marriage to Lilia Olivia Wiegel on the ground that the latter had a prior existing marriage. After pretrial, Lilia asked that she be allowed to present evidence to prove, among others, that her first husband had previously been married to another woman. In holding that there was no need for such evidence, the Court ruled: x x x There is likewise no need of introducing evidence about the existing prior marriage of her first husband at the time they married each other, for then such a marriage though void still needs, according to this Court, a judicial declaration of such fact and for all legal intents and purposes she would still be regarded as a married woman at the time she contracted her marriage with respondent Karl Heinz Wiegel; x x x. Subsequently, in Yap v. CA, the Court reverted to the ruling in People v. Mendoza, holding that there was no need for such declaration of nullity. In Domingo v. CA, the issue raised was whether a judicial declaration of nullity was still necessary for the recovery and the separation of properties of erstwhile spouses. Ruling in the affirmative, the Court declared: The Family Code has settled once and for all the conflicting jurisprudence on the matter. A declaration of the absolute nullity of a marriage is now explicitly required either as a cause of action or a ground for defense; in fact, the requirement for a declaration of absolute nullity of a marriage is also for the protection of the spouse who, believing that his or her marriage is illegal and void, marries again. With the judicial declaration of the nullity of his or her first marriage, the person who marries again cannot be charged with bigamy. Unlike Mendoza and Aragon, Domingo as well as the other cases herein cited was not a criminal prosecution for bigamy. Nonetheless,Domingo underscored the need for a judicial declaration of nullity of a void marriage on the basis of a new provision of the Family Code, which came into effect several years after the promulgation of Mendoza and Aragon. In Mendoza and Aragon, the Court relied on Section 29 of Act No. 3613 (Marriage Law), which provided: Illegal marriages. Any marriage subsequently contracted by any person during the lifetime of the first spouse shall be illegal and void from its performance, unless: (a) The first marriage was annulled or dissolved; (b) The first spouse had been absent for seven consecutive years at the time of the second marriage without the spouse present having news of the absentee being alive, or the absentee being generally considered as dead and believed to be so by the spouse present at the time of contracting such subsequent marriage, the marriage as contracted being valid in either case until declared null and void by a competent court." The Court held in those two cases that the said provision plainly makes a subsequent marriage contracted by any person during the lifetime of his first spouse illegal and void from its performance, and no judicial decree is necessary to establish its invalidity, as distinguished from mere annulable marriages. The provision appeared in substantially the same form under Article 83 of the 1950 Civil Code and Article 41 of the Family Code. However, Article 40 of the Family Code, a new provision, expressly requires a judicial declaration of nullity of the previous marriage, as follows: ART. 40. The absolute nullity of a previous marriage may be invoked for purposes of remarriage on the basis solely of a final judgment declaring such marriage void. In view of this provision, Domingo stressed that a final judgment declaring such marriage void was necessary. Verily, the Family Code andDomingo affirm the earlier ruling in Wiegel. Thus, a Civil Law authority and member of the Civil Code Revision Commitee has observed: [Article 40] is also in line with the recent decisions of the Supreme Court that the marriage of a person may be null and void but there is need of a judicial declaration of such fact before that person can marry again; otherwise, the second marriage will also be void (Wiegel v. Sempio-Diy, Aug. 19/86, 143 SCRA 499, Vda. De Consuegra v. GSIS, 37 SCRA 315). This provision changes the old rule that where a 437

University of the Cordilleras College of Law First Year C S.Y. 2013 - 2014 marriage is illegal and void from its performance, no judicial decree is necessary to establish its validity (People v. Mendoza, 95 Phil. 843; People v. Aragon, 100 Phil. 1033). In this light, the statutory mooring of the ruling in Mendoza and Aragon that there is no need for a judicial declaration of nullity of a void marriage -- has been cast aside by Article 40 of the Family Code. Such declaration is now necessary before one can contract a second marriage. Absent that declaration, we hold that one may be charged with and convicted of bigamy. The present ruling is consistent with our pronouncement in Terre v. Terre, which involved an administrative Complaint against a lawyer for marrying twice. In rejecting the lawyers argument that he was free to enter into a second marriage because the first one was void ab initio, the Court ruled: for purposes of determining whether a person is legally free to contract a second marriage, a judicial declaration that the first marriage was null and void ab initio is essential. The Court further noted that the said rule was cast into statutory form by Article 40 of the Family Code. Significantly, it observed that the second marriage, contracted without a judicial declaration that the first marriage was void, was bigamous and criminal in character. Moreover, Justice Reyes, an authority in Criminal Law whose earlier work was cited by petitioner, changed his view on the subject in view of Article 40 of the Family Code and wrote in 1993 that a person must first obtain a judicial declaration of the nullity of a void marriage before contracting a subsequent marriage: It is now settled that the fact that the first marriage is void from the beginning is not a defense in a bigamy charge. As with a voidable marriage, there must be a judicial declaration of the nullity of a marriage before contracting the second marriage. Article 40 of the Family Code states that x x x. The Code Commission believes that the parties to a marriage should not be allowed to assume that their marriage is void, even if such is the fact, but must first secure a judicial declaration of nullity of their marriage before they should be allowed to marry again. x x x. In the instant case, petitioner contracted a second marriage although there was yet no judicial declaration of nullity of his first marriage. In fact, he instituted the Petition to have the first marriage declared void only after complainant had filed a letter-complaint charging him with bigamy. By contracting a second marriage while the first was still subsisting, he committed the acts punishable under Article 349 of the Revised Penal Code. That he subsequently obtained a judicial declaration of the nullity of the first marriage was immaterial. To repeat, the crime had already been consummated by then. Moreover, his view effectively encourages delay in the prosecution of bigamy cases; an accused could simply file a petition to declare his previous marriage void and invoke the pendency of that action as a prejudicial question in the criminal case. We cannot allow that. Under the circumstances of the present case, he is guilty of the charge against him.

Damages In her Memorandum, respondent prays that the Court set aside the ruling of the Court of Appeals insofar as it denied her claim of damages and attorneys fees. Her prayer has no merit. She did not appeal the ruling of the CA against her; hence, she cannot obtain affirmative relief from this Court. In any event, we find no reason to reverse or set aside the pertinent ruling of the CA on this point, which we quote hereunder: We are convinced from the totality of the evidence presented in this case that Consuelo Tan is not the innocent victim that she claims to be; she was well aware of the existence of the previous marriage when she contracted matrimony with Dr. Mercado. The testimonies of the defense witnesses prove this, and we find no reason to doubt said testimonies. xxx xxx xxx

Indeed, the claim of Consuelo Tan that she was not aware of his previous marriage does not inspire belief, especially as she had seen that Dr. Mercado had two (2) children with him. We are convinced that 438

University of the Cordilleras College of Law First Year C S.Y. 2013 - 2014 she took the plunge anyway, relying on the fact that the first wife would no longer return to Dr. Mercado, she being by then already living with another man. Consuelo Tan can therefore not claim damages in this case where she was fully conscious of the consequences of her act. She should have known that she would suffer humiliation in the event the truth [would] come out, as it did in this case, ironically because of her personal instigation. If there are indeed damages caused to her reputation, they are of her own willful making. WHEREFORE, the Petition is DENIED and the assailed Decision AFFIRMED. Costs against petitioner. SO ORDERED. Melo, (Chairman), Purisima, and Gonzaga-Reyes, JJ., concur. Vitug, J., see concurring and dissenting opinion.

CONCURRING AND DISSENTING OPINION VITUG, J.: At the pith of the controversy is the defense of the absolute nullity of a previous marriage in an indictment for bigamy. The majority opinion, penned by my esteemed brother, Mr. Justice Artemio V. Panganiban, enunciates that it is only a judicially decreed prior void marriage which can constitute a defense against the criminal charge. The civil law rule stated in Article 40 of the Family Code is a given but I have strong reservations on its application beyond what appears to be its expressed context. The subject of the instant petition is a criminal prosecution, not a civil case, and the ponencia affirms the conviction of petitioner Vincent Paul G. Mercado for bigamy. Article 40 of the Family code reads: ART. 40. The absolute nullity of a previous marriage may be invoked for purposes of remarriage on the basis solely of a final judgment declaring such previous marriage void. The phrase for purposes of remarriage is not at all insignificant. Void marriages, like void contracts, are inexistent from the very beginning. It is only by way of exception that the Family code requires a judicial declaration of nullity of the previous marriage before a subsequent marriage is contracted; without such declaration, the validity and the full legal consequence of the subsequent marriage would itself be in similar jeopardy under Article 53, in relation to Article 52, of the Family Code. Parenthetically, I would daresay that the necessity of a judicial declaration of nullity of a void marriage for the purpose of remarriage should be held to refer merely to cases where it can be said that a marriage, at least ostensibly, had taken place. No such judicial declaration of nullity, in my view, should still be deemed essential when the marriage, for instance, is between persons of the same sex or when either or both parties had not at all given consent to the marriage. Indeed, it is likely that Article 40 of the Family Code has been meant and intended to refer only to marriages declared void under the provisions of Articles 35, 36, 37, 38 and 53 thereof. In fine, the Family Code, I respectfully submit, did not have the effect of overturning the rule in criminal law and related jurisprudence. The Revised Penal Code expresses: Art. 349. Bigamy.---The penalty of prision mayor shall be imposed upon any person who shall contract a second or subsequent marriagebefore the former marriage has been legally dissolved, or before the absent spouse has been declared presumptively dead by means of a judgment rendered in the proper proceedings. Surely, the foregoing provision contemplated an existing, not void, prior marriage. Covered by article 349 would thus be, for instance, a voidable marriage, it obviously being valid and subsisting until set aside by a competent court. As early as People vs. Aragon, this Court has underscored:

439

University of the Cordilleras College of Law First Year C S.Y. 2013 - 2014 xxx Our Revised Penal Code is of recent enactment and had the rule enunciated in Spain and in America requiring judicial declaration of nullity of ab initio void marriages been within the contemplation of the legislature, an express provision to that effect would or should have been inserted in the law. In its absence, we are bound by said rule of strict interpretation. Unlike a voidable marriage which legally exists until judicially annulled (and therefore not a defense in bigamy if the second marriage were contracted prior to the decree of annulment), the complete nullity, however, of a previously contracted marriage, being a total nullity andinexistent, should be capable of being independently raised by way of a defense in a criminal case for bigamy. I see no incongruence between this rule in criminal law and that of the Family Code, and each may be applied within the respective spheres of governance. Accordingly, I vote to grant the petition.

440

University of the Cordilleras College of Law First Year C S.Y. 2013 - 2014 Case Digest VINCENT PAUL G. MERCADO vs. CONSUELO TAN G.R. No. 137110 August 1, 2000 337 SCRA 28 PANGANIBAN, J. FACTS: Dr. Vicent Mercado was previously married with Thelma Oliva in 1976 before he contracted marriage with Consuelo Tan in 1991 which the latter claims she did not know. Tan filed bigamy against Mercado and after a month the latter filed an action for declaration of nullity of marriage against Oliva. The decision in 1993 declared marriage between Mercado and Oliva null and void. ISSUE: Whether Mercado committed bigamy in spite of filing the declaration of nullity of the former marriage. RULING: A judicial declaration of nullity of a previous marriage is necessary before a subsequent one can be legally contracted. One who enters into a subsequent marriage without first obtaining such judicial declaration is guilty of bigamy. This principle applies even if the earlier union is characterized by statute as void. In the case at bar, Mercado only filed the declaration of nullity of his marriage with Olivia right after Tan filed bigamy case. Hence, by then, the crime had already been consummated. He contracted second marriage without the judicial declaration of the nullity. The fact that the first marriage is void from the beginning is not a defense in a bigamy charge.

441

University of the Cordilleras College of Law First Year C S.Y. 2013 - 2014 Carating-Siayngco vs Siayngco 441 SCRA 422 G.R. NO. 158896 October 27, 2004 Full Case JUANITA CARATING-SIAYNGCO, petitioner, vs. MANUEL SIAYNGCO, respondent. CHICO-NAZARIO, J.: This is a petition for review on certiorari of the decision of the Court of Appeals promulgated on 01 July 2003, reversing the decision of the Regional Trial Court (RTC), Branch 102, Quezon City, dated 31 January 2001, which dismissed the petition for declaration of nullity of marriage filed by respondent herein Judge Manuel Siayngco ("respondent Manuel"). Petitioner Juanita Carating-Siayngco ("Petitioner Juanita") and respondent Manuel were married at civil rites on 27 June 1973 and before the Catholic Church on 11 August 1973. After discovering that they could not have a child of their own, the couple decided to adopt a baby boy in 1977, who they named Jeremy. On 25 September 1997, or after twenty-four (24) years of married life together, respondent Manuel filed for the declaration of its nullity on the ground of psychological incapacity of petitioner Juanita. He alleged that all throughout their marriage, his wife exhibited an over domineering and selfish attitude towards him which was exacerbated by her extremely volatile and bellicose nature; that she incessantly complained about almost everything and anyone connected with him like his elderly parents, the staff in his office and anything not of her liking like the physical arrangement, tables, chairs, wastebaskets in his office and with other trivial matters; that she showed no respect or regard at all for the prestige and high position of his office as judge of the Municipal Trial Court; that she would yell and scream at him and throw objects around the house within the hearing of their neighbors; that she cared even less about his professional advancement as she did not even give him moral support and encouragement; that her psychological incapacity arose before marriage, rooted in her deep-seated resentment and vindictiveness for what she perceived as lack of love and appreciation from her own parents since childhood and that such incapacity is permanent and incurable and, even if treatment could be attempted, it will involve time and expense beyond the emotional and physical capacity of the parties; and that he endured and suffered through his turbulent and loveless marriage to her for twenty-two (22) years. In her Answer, petitioner Juanita alleged that respondent Manuel is still living with her at their conjugal home in Malolos, Bulacan; that he invented malicious stories against her so that he could be free to marry his paramour; that she is a loving wife and mother; that it was respondent Manuel who was remiss in his marital and family obligations; that she supported respondent Manuel in all his endeavors despite his philandering; that she was raised in a real happy family and had a happy childhood contrary to what was stated in the complaint. In the pre-trial order, the parties only stipulated on the following: 1. That they were married on 27 June 1973; 2. That they have one son who is already 20 years old. Trial on the merits ensued thereafter. Respondent Manuel first took the witness stand and elaborated on the allegations in his petition. He testified that his parents never approved of his marriage as they still harbored hope that he would return to the seminary. The early years of their marriage were difficult years as they had a hard time being accepted as husband and wife by his parents and it was at this period that his wife started exhibiting signs of being irritable and temperamental to him and his parents. She was also obsessive about cleanliness which became the common source of their quarrels. He, however, characterized their union as happy during that period of time in 1979 when they moved to Malolos as they were engrossed in furnishing their new house. In 1981, when he became busy with law school and with various community organizations, it was then that he felt that he and his wife started to drift apart. He then narrated incidents during their marriage that were greatly embarrassing and/or distressing to him, e.g., when his wife quarreled with an elderly neighbor; when she would visit him in his office and remark 442

University of the Cordilleras College of Law First Year C S.Y. 2013 - 2014 that the curtains were already dirty or when she kicked a trash can across the room or when she threw a ballpen from his table; when she caused his office drawer to be forcibly opened while he was away; when she confronted a female tenant of theirs and accused the tenant of having an affair with him; and other incidents reported to him which would show her jealous nature. Money matters continued to be a source of bitter quarrels. Respondent Manuel could not forget that he was not able to celebrate his appointment as judge in 1995 as his wife did not approve it, ostensibly for lack of money, but she was very generous when it came to celebrations of their parish priest. Respondent Manuel then denied that he was a womanizer or that he had a mistress. Lastly, respondent Manuel testified as to their conjugal properties and obligations. Next, LUCENA TAN, respondent Manuels Clerk of Court, testified that petitioner Juanita seldom went to respondent Manuels office. But when she was there, she would call witness to complain about the curtains and the cleanliness of the office. One time, witness remembered petitioner Juanita rummaging through respondent Manuels drawer looking for his address book while the latter was in Subic attending a conference. When petitioner Juanita could not open a locked drawer she called witness, telling the latter that she was looking for the telephone number of respondents hotel room in Subic. A process server was requested by petitioner Juanita to call for a locksmith in the town proper. When the locksmith arrived, petitioner Juanita ordered him to open the locked drawer. On another occasion, particularly in August of 1998, witness testified that she heard petitioner Juanita remark to respondent Manuel "sino bang batang bibinyagan na yan? Baka anak mo yan sa labas?" As his third witness, respondent Manuel presented DR. VALENTINA GARCIA whose professional qualifications as a psychiatrist were admitted by petitioner Juanita. From her psychiatric evaluation, Dr. Garcia concluded: To sum up, Manuel de Jesus Siayngco and Juanita Victoria Carating-Siayngco contributed to the marital collapse. There is a partner relational problem which affected their capacity to sustain the marital bond with love, support and understanding. The partner relational problem (coded V61/10 in the Fourth Edition of the Diagnostic and Statistical Manual of Mental Disorders or DSM IV) is secondary to the psychopathology of both spouses. Manuel and Juanita had engaged themselves in a defective communication pattern which is characteristically negative and deformed. This affected their competence to maintain the love and respect that they should give to each other. Marriage requires a sustained level of adaptation from both partners who are expected to use healthy strategies to solve their disputes and differences. Whereas Juanita would be derogatory, critical, argumentative, depressive and obsessive-compulsive, Manuel makes use of avoidance and suppression. In his effort to satisfy the self and to boost his masculine ego to cover up for his felt or imagined inadequacies, he became callused to the detrimental effects of his unfaithfulness and his failure to prioritize the marriage. Both spouses, who display narcissistic psychological repertoire (along with their other maladaptive traits), failed to adequately empathize (or to be responsive and sensitive) to each others needs and feelings. The matrimonial plot is not conducive to a healthy and a progressive marriage. Manuel and Juanita have shown their psychologically [sic] incapacity to satisfactorily comply with the fundamental duties of marriage. The clashing of their patterns of maladaptive traits, which warrant the diagnosis of personality disorder not otherwise specified (PDNOS, with code 301.9 as per DSM IV criteria) will bring about more emotional mishaps and psychopathology. These rigid sets of traits which were in existence before the marriage will tend to be pervasive and impervious to recovery. In her defense, petitioner Juanita denied respondent Manuels allegations. She insisted that they were a normal couple who had their own share of fights; that they were happily married until respondent Manuel started having extra-marital affairs which he had admitted to her. Petitioner Juanita professed that she would wish to preserve her marriage and that she truly loved her husband. She stated further that she has continuously supported respondent Manuel, waiting up for him while he was in law school to serve him food and drinks. Even when he already filed the present case, she would still attend to his needs. She remembered that after the pre-trial, while they were in the hallway, respondent Manuel implored her to give him a chance to have a new family.

443

University of the Cordilleras College of Law First Year C S.Y. 2013 - 2014 DR. EDUARDO MAABA, whose expertise as a psychiatrist was admitted by respondent Manuel, testified that he conducted a psychiatric evaluation on petitioner Juanita, the results of which were embodied in his report. Said report stated in part: Based on the clinical interviews and the results of the psychological tests, respondent Juanita Victoria Carating-Siayngco, was found to be a mature, conservative, religious and highly intelligent woman who possess [sic] more than enough psychological potentials for a mutually satisfying long term heterosexual relationship. Superego is strong and she is respectful of traditional institutions of society like the institution of marriage. She was also found to be a loving, nurturing and self-sacrificing woman who is capable of enduring severe environmental stress in her social milieu. Finally, she is reality-oriented and therefore capable of rendering fair and sound decision. In summary, the psychiatric evaluation found the respondent to be psychologically capacitated to comply with the basic and essential obligations of marriage. CRISPINA SEVILLA, a friend of the spouses Siayngco since 1992 described the Siayngcos as the ideal couple, sweet to each other. The couple would religiously attend prayer meetings in the community. Both were likewise leaders in their community. Witness then stated that she would often go to the house of the couple and, as late as March 2000, she still saw respondent Manuel there. On 31 January 2001, the trial court denied respondent Manuels petition for declaration of nullity of his marriage to petitioner Juanita holding in part that: The asserted psychological incapacity of the defendant is not preponderantly supported in evidence. The couple [was] happily married and after four years of marital bliss [was] blest with a son. Their life together continued years thereafter in peace and prosperity. The psychiatric finding that defendant has been critical, depressed and obsessive doubtless arose later in the parties relationship sometime in the early 90s when the defendant -wife started receiving letters that the plaintiff is playing footsy. xxx xxx xxx

The present state of our laws on marriage does not favor knee-jerk responses to slight stabs of the Pavlovian hammer on marital relations. A wife, as in the instant case, may have succumbed, due to her jealousy, to the constant delivery of irritating curtain lectures to her husband. But, as our laws now stand, the dissolution of the marriage is not the remedy in such cases. In contrast to some countries, our laws do not look at a marital partner as a mere refrigerator in the Kitchen even if he or she sometimes may sound like a firetruck. A motion for reconsideration was filed but was denied in an order dated 04 May 2001. On 01 July 2003, the Court of Appeals reversed the RTC decision, relying mainly on the psychiatric evaluation of Dr. Garcia finding both Manuel and Juanita psychologically incapacitated and on the case of Chi Ming Tsoi v. Court of Appeals. Thus: The report clearly explained the root cause of the alleged psychological incapacity of plaintiff Manuel and defendant Juanita. It appears that there is empathy between plaintiff and defendant. That is a shared feeling which between husband and wife must be experienced not only by having spontaneous sexual intimacy but a deep sense of spiritual communion. Marital union is a two-way process. An expressive interest in each others feelings at a time it is needed by the other can go a long way in deepening the marital relationship. Marriage is definitely not for children but for two consenting adults who view the relationship with love "amore gignit amorem", sacrifice and a continuing commitment to compromise conscious of its value as a sublime social institution (Chi Ming Tsoi vs. Court of Appeals, 266 SCRA 324). This court, finding the gravity of the failed relationship in which the parties found themselves trapped in its mire of unfulfilled vows and unconsummated marital obligations, can do no less, but reverse and set aside the decision of the lower court. Plaintiff Manuel is entitled to have his 444

University of the Cordilleras College of Law First Year C S.Y. 2013 - 2014 marriage declared a nullity on the ground of psychological incapacity, not only of defendant but also of himself. Petitioner contends that the Court of Appeals erred I. IN ITS FINDINGS INCAPACITATED THAT PETITIONER JUANITA IS PSYCHOLOGICALLY

II. IN ITS FINDINGS OF FACT THAT PETITIONER AND RESPONDENT SEPARATED ON MARCH 1997, THE TRUTH IS THAT THEY ARE STILL LIVING TOGETHER AS HUSBAND AND WIFE AT THE TIME OF THE FILING OF THE PETITION UP TO THE PRESENT III. WHEN IT DID NOT FOLLOW THE GUIDELINES LAID DOWN BY THE SUPREME COURT IN THE CASE OF REPUBLIC V. MOLINA IV. IN DECLARING THE MARRIAGE OF HEREIN PETITIONER AND RESPONDENT NULL AND VOID ON GROUND OF PSYCHOLOGICAL INCAPACITY UNDER ARTICLE 36 OF THE FAMILY CODE The Courts Ruling Our pronouncement in Republic v. Dagdag is apropos. There, we held that whether or not psychological incapacity exists in a given case calling for the declaration of the nullity of the marriage depends crucially on the facts of the case. Each case must be closely scrutinized and judged according to its own facts as there can be no case that is on "all fours" with another. This, the Court of Appeals did not heed. The Court of Appeals perfunctorily applied our ruling in Chi Ming Tsoi despite a clear divergence in its factual milieu with the case at bar. In Chi Ming Tsoi, the couple involved therein, despite sharing the same bed from the time of their wedding night on 22 May 1988 until their separation on 15 March 1989, never had coitus. The perplexed wife filed the petition for the declaration of the nullity of her marriage on the ground of psychological incapacity of her husband. We sustained the wife for the reason that an essential marital obligation under the Family Code is procreation such that "the senseless and protracted refusal of one of the parties to fulfill the above marital obligation is equivalent to psychological incapacity." On the other hand, sexual intimacy for procreation is a non-issue herein. Rather, we have here a case of a husband who is constantly embarrassed by his wifes outbursts and overbearing ways, who finds his wifes obsession with cleanliness and the tight reign on his wallet "irritants" and who is wounded by her lack of support and respect for his person and his position as a Judge. In our book, however, these inadequacies of petitioner Juanita which led respondent Manuel to file a case against her do not amount to psychological incapacity to comply with the essential marital obligations. It was in Santos v. Court of Appeals where we declared that "psychological incapacity" under Article 36 of the Family Code is not meant to comprehend all possible cases of psychoses. It should refer, rather, to no less than a mental (not physical) incapacity that causes a party to be truly incognitive of the basic marital covenants that concomitantly must be assumed and discharged by the parties to the marriage. Psychological incapacity must be characterized by (a) gravity, (b) juridical antecedence, and (c) incurability. In Republic v. Court of Appeals we expounded: (1) The burden of proof to show the nullity of marriage belongs to the plaintiff. Any doubt should be resolved in favor of the existence and continuation of the marriage and against its dissolution and nullity. This is rooted in the fact that both our Constitution and our laws cherish the validity of marriage and unity of the family. Thus, our Constitution devotes an entire Article on the Family, recognizing it "as the foundation of the nation." It decrees marriage as legally "inviolable," thereby protecting it from dissolution at the whim of the parties. Both the family and marriage are to be "protected" by the state. The Family Code echoes this constitutional edict on marriage and the family and emphasizes their permanence, inviolability and solidarity. (2) The root cause of the psychological incapacity must be: a) medically or clinically identified, b) alleged in the complaint, c) sufficiently proven by experts and d) clearly explained in the 445

University of the Cordilleras College of Law First Year C S.Y. 2013 - 2014 decision. Article 36 of the Family Code requires that the incapacity must be psychological not physical, although its manifestations and/or symptoms may be physical. The evidence must convince the court that the parties, or one of them, was mentally or physically ill to such an extent that the person could not have known the obligations he was assuming, or knowing them, could not have given valid assumption thereof. Although no example of such incapacity need be given here so as not to limit the application of the provision under the principle of ejusdem generis, nevertheless such root cause must be identified as a psychological illness and its incapacitating nature fully explained. Expert evidence may be given by qualified psychiatrists and clinical psychologists. (3) The incapacity must be proven to be existing at the "time of the celebration" of the marriage. The evidence must show that the illness was existing when the parties exchanged their "I dos." The manifestation of the illness need not be perceivable at such time, but the illness itself must have attached at such moment, or prior thereto. (4) Such incapacity must also be shown to be medically or clinically permanent or incurable. Such incurability may be absolute or even relative only in regard to the other spouse, not necessarily absolutely against everyone of the same sex. Furthermore, such incapacity must be relevant to the assumption of marriage obligations, not necessarily to those not related to marriage like the exercise of a profession or employment in a job. Hence, a pediatrician may be effective in diagnosing illnesses of children and prescribing medicine to cure them but may not be psychologically capacitated to procreate, bear and raise his/her own children as an essential obligation of marriage. (5) Such illness must be grave enough to bring about the disability of the party to assume the essential obligations of marriage. Thus, "mild characteriological peculiarities, mood changes, occasional emotional outbursts" cannot be accepted as root causes. The illness must be shown as downright incapacity or inability, not a refusal, neglect or difficulty, much less ill will. In other words, there is a natal or supervening disabling factor in the person, an adverse integral element in the personality structure that effectively incapacitates the person from really accepting and thereby complying with the obligations essential to marriage. (6) The essential marital obligations must be those embraced by Articles 68 up to 71 of the Family Code as regards the husband and wife as well as Articles 220, 221 and 225 of the same Code in regard to parents and their children. Such non-complied marital obligation(s) must also be stated in the petition, proven by evidence and included in the text of the decision. (7) Interpretations given by the National Appellate Matrimonial Tribunal of the Catholic Church in the Philippines, while not controlling or decisive, should be given great respect by our courts. With the foregoing pronouncements as compass, we now resolve the issue of whether or not the totality of evidence presented is enough to sustain a finding of psychological incapacity against petitioner Juanita and/or respondent Manuel. A. RE: PSYCHOLOGICAL INCAPACITY OF RESPONDENT MANUEL We reiterate that the state has a high stake in the preservation of marriage rooted in its recognition of the sanctity of married life and its mission to protect and strengthen the family as a basic autonomous social institution. With this cardinal state policy in mind, we held in Republic v. Court of Appeals that the burden of proof to show the nullity of marriage belongs to the plaintiff (respondent Manuel herein). Any doubt should be resolved in favor of the existence and continuation of the marriage and against its dissolution and nullity. In herein case, the Court of Appeals committed reversible error in holding that respondent Manuel is psychologically incapacitated. The psychological report of Dr. Garcia, which is respondent Manuels own evidence, contains candid admissions of petitioner Juanita, the person in the best position to gauge whether or not her husband fulfilled the essential marital obligations of marriage: She talked about her spouse, "My husband is kind, a good provider, cool, intelligent but a liar, masamang magalit at gastador. In spite of what he has done to me, I take care of him whenever he is sick. He is having extra marital affairs because he wants to have a child. I believe that our 446

University of the Cordilleras College of Law First Year C S.Y. 2013 - 2014 biggest problem is not having a child. It is his obsession to have a child with his girl now. He started his relationship with this girl in 1994. I even saw them together in the car. I think that it was the girl who encouraged him to file the petition." She feels that the problems in the relationship is [sic] "paulit-ulit," but, that she still is willing to pursue it. x x x. Overall, she feels that he is a good spouse and that he is not really psychologically incapacitated. He apparently told her, "You and Jeremy should give me a chance to have a new family." She answered and said, "Ikaw tinuruan mo akong to fight for my right. Ipaglalaban ko ang marriage natin." What emerges from the psychological report of Dr. Garcia as well as from the testimonies of the parties and their witnesses is that the only essential marital obligation which respondent Manuel was not able to fulfill, if any, is the obligation of fidelity. Sexual infidelity, per se, however, does not constitute psychological incapacity within the contemplation of the Family Code. It must be shown that respondent Manuels unfaithfulness is a manifestation of a disordered personality whi ch makes him completely unable to discharge the essential obligations of the marital state and not merely due to his ardent wish to have a child of his own flesh and blood. In herein case, respondent Manuel has admitted that: "I had [extra-marital] affairs because I wanted to have a child at that particular point." B. RE: PSYCHOLOGICAL INCAPACITY OF PETITIONER JUANITA As aforementioned, the presumption is always in favor of the validity of marriage. Semper praesumitur pro matrimonio. In the case at bar, respondent Manuel failed to prove that his wifes lack of respect for him, her jealousies and obsession with cleanliness, her outbursts and her controlling nature (especially with respect to his salary), and her inability to endear herself to his parents are grave psychological maladies that paralyze her from complying with the essential obligations of marriage. Neither is there any showing that these "defects" were already present at the inception of the marriage or that they are incurable. In fact, Dr. Maaba, whose expertise as a psychiatrist was admitted by respondent Manuel, reported that petitioner was psychologically capacitated to comply with the basic and essential obligations of marriage. The psychological report of respondent Manuels witness, Dr. Garcia, on the other hand, does not help his case any. Nothing in there supports the doctors conclusion that petitioner Juanita is psychologically incapacitated. On the contrary, the report clearly shows that the root cause of petitioner Juanitas behavior is traceable not from the inception of their marriage as required by law but from her experiences during the marriage, e.g., her in-laws disapproval of her as they wanted their son to enter the priesthood, her husbands philandering, admitted no less by him, and her inability to conceive. Dr. Garcias report paints a story of a husband and wife who grew professionally during the marriage, who pursued their individual dreams to the hilt, becoming busier and busier, ultimately sacrificing intimacy and togetherness as a couple. This was confirmed by respondent Manuel himself during his direct examination. Thus, from the totality of the evidence adduced by both parties, we have been allowed a window into the Siayngcoss life and have perceived therefrom a simple case of a married couple drifting apart, becoming strangers to each other, with the husband consequently falling out of love and wanting a way out. An unsatisfactory marriage, however, is not a null and void marriage. Mere showing of "irreconcilable differences" and "conflicting personalities" in no wise constitutes psychological incapacity. As we stated in Marcos v. Marcos: Article 36 of the Family Code, we stress, is not to be confused with a divorce law that cuts the marital bond at the time the causes therefore manifests themselves. It refers to a serious psychological illness afflicting a party even before the celebration of the marriage. It is a malady so grave and so permanent as to deprive one of awareness of the duties and responsibilities of the matrimonial bond one is about to assume. We are not downplaying the frustration and misery respondent Manuel might be experiencing in being shackled, so to speak, to a marriage that is no longer working. Regrettably, there are situations like this one, where neither law nor society can provide the specific answers to every individual problem.

447

University of the Cordilleras College of Law First Year C S.Y. 2013 - 2014 WHEREFORE, the petition for review is hereby GRANTED. The Decision dated 01 July 2003 of the Court of Appeals is hereby REVERSED and SET ASIDE. The Decision dated 31 January 2001 of the Regional Trial Court of Quezon City, Branch 102 is reinstated and given full force and effect. No costs. SO ORDERED. Puno, Austria-Martinez, Callejo, Sr., and Tinga, JJ., concur.

448

University of the Cordilleras College of Law First Year C S.Y. 2013 - 2014 Case Digest JUANITA CARATING-SIAYNGCO vs. MANUEL SIAYNGCO G.R. NO. 158896 October 27, 2004 441 SCRA 422 CHICO-NAZARIO, J. FACTS: Petitioner Juanita Carating-Siayngco ("Petitioner Juanita") and respondent Manuel were married at civil rites on 27 June 1973 and before the Catholic Church on 11 August 1973. After discovering that they could not have a child of their own, the couple decided to adopt a baby boy in 1977, who they named Jeremy. On 25 September 1997, or after twenty-four (24) years of married life together, respondent Manuel filed for the declaration of its nullity on the ground of psychological incapacity of petitioner Juanita. He alleged that all throughout their marriage, his wife exhibited an over domineering and selfish attitude towards him which was exacerbated by her extremely volatile and bellicose nature; that she incessantly complained about almost everything and anyone connected with him like his elderly parents, the staff in his office and anything not of her liking like the physical arrangement, tables, chairs, wastebaskets in his office and with other trivial matters; that she showed no respect or regard at all for the prestige and high position of his office as judge of the Municipal Trial Court; that she would yell and scream at him and throw objects around the house within the hearing of their neighbors; that she cared even less about his professional advancement as she did not even give him moral support and encouragement; that her psychological incapacity arose before marriage, rooted in her deep-seated resentment and vindictiveness for what she perceived as lack of love and appreciation from her own parents since childhood and that such incapacity is permanent and incurable and, even if treatment could be attempted, it will involve time and expense beyond the emotional and physical capacity of the parties; and that he endured and suffered through his turbulent and loveless marriage to her for twenty-two (22) years. In her Answer, petitioner Juanita alleged that respondent Manuel is still living with her at their conjugal home in Malolos, Bulacan; that he invented malicious stories against her so that he could be free to marry his paramour; that she is a loving wife and mother; that it was respondent Manuel who was remiss in his marital and family obligations; that she supported respondent Manuel in all his endeavors despite his philandering; that she was raised in a real happy family and had a happy childhood contrary to what was stated in the complaint. ISSUE: Whether or not the totality of evidence presented is enough to sustain a finding of psychological incapacity against petitioner Juanita and/or respondent Manuel. RULING: Sexual infidelity, per se, however, does not constitute psychological incapacity within the contemplation of the Family Code. It must be shown that respondent Manuels unfaithfulness is a manifestation of a disordered personality which makes him completely unable to discharge the essential obligations of the marital state and not merely due to his ardent wish to have a child of his own flesh and blood. An unsatisfactory marriage, however, is not a null and void marriage. Mere showing of "irreconcilable differences" and "conflicting personalities" in no wise constitutes psychological incapacity. As we stated in Marcos v. Marcos: Article 36 of the Family Code, we stress, is not to be confused with a divorce law that cuts the marital bond at the time the causes therefore manifests themselves. It refers to a serious psychological illness afflicting a party even before the celebration of the marriage. It is a malady so grave and so permanent as to deprive one of awareness of the duties and responsibilities of the matrimonial bond one is about to assume. We are not downplaying the frustration and misery respondent Manuel might be experiencing in being shackled, so to speak, to a marriage that is no longer working. Regrettably, there are situations like this one, where neither law nor society can provide the specific answers to every individual problem.

449

University of the Cordilleras College of Law First Year C S.Y. 2013 - 2014 Sin vs Sin 355 SCRA 28 G.R. No. 137590. March 26, 2001 Full Case FLORENCE MALCAMPO-SIN, petitioner, vs PHILIPP T. SIN, respondent. PARDO, J.: The Family Code emphasizes the permanent nature of marriage, hailing it as the foundation of the family. It is this inviolability which is central to our traditional and religious concepts of morality and provides the very bedrock on which our society finds stability. Marriage is immutable and when both spouses give their consent to enter it, their consent becomes irrevocable, unchanged even by their independent wills. However, this inviolability depends on whether the marriage exists and is valid. If it is void ab initio, the permanence of the union becomes irrelevant, and the Court can step in to declare it so. Article 36 of the Family Code is the justification. Where it applies and is duly proven, a judicial declaration can free the parties from the rights, obligations, burdens and consequences stemming from their marriage. A declaration of nullity of marriage under Article 36 of the Family Code requires the application of procedural and substantive guidelines. While compliance with these requirements mostly devolves upon petitioner, the State is likewise mandated to actively intervene in the procedure. Should there be noncompliance by the State with its statutory duty, there is a need to remand the case to the lower court for proper trial.

The Case What is before the Court is an appeal from a decision of the Court of Appeals which affirmed the decision of the Regional Trial Court, Branch 158, Pasig City dismissing petitioner Florence MalcampoSins (hereafter Florence) petition for declaration of nullity of marriage due to psychological incapacity for insufficiency of evidence.

The Facts On January 4, 1987, after a two-year courtship and engagement, Florence and respondent Philipp T. Sin (hereafter Philipp), a Portugese citizen, were married at St. Jude Catholic Parish in San Miguel, Manila. On September 20, 1994, Florence filed with the Regional Trial Court, Branch 158, Pasig City, a complaint for declaration of nullity of marriage against Philipp. Trial ensued and the parties presented their respective documentary and testimonial evidence. On June 16, 1995, the trial court dismissed Florences petition. On December 19, 1995, Florence filed with the trial court a notice of appeal to the Court of Appeals. After due proceedings, on April 30, 1998, the Court of Appeals promulgated its decision, the dispositive portion of which reads: IN THE LIGHT OF ALL THE FOREGOING, the Appeal is DISMISSED. The Decision appealed from is AFFIRMED. Cost against the Appellant. On June 23, 1998, petitioner filed with the Court of Appeals a motion for reconsideration of the aforequoted decision. On January 19, 1999, the Court of Appeals denied petitioners motion for reconsideration. 450

University of the Cordilleras College of Law First Year C S.Y. 2013 - 2014 Hence, this appeal.

The Courts Ruling We note that throughout the trial in the lower court, the State did not participate in the proceedings. While Fiscal Jose Danilo C. Jabson filed with the trial court a manifestation dated November 16, 1994, stating that he found no collusion between the parties, he did not actively participate therein. Other than entering his appearance at certain hearings of the case, nothing more was heard from him. Neither did the presiding Judge take any step to encourage the fiscal to contribute to the proceedings. The Family Code mandates: Article 48. In all cases of annulment or declaration of absolute nullity of marriage, the Court shall order the prosecuting attorney or fiscal assigned to it to appear on behalf of the State to take steps to prevent collusion between the parties and to take care that evidence is not fabricated or suppressed (underscoring ours). In the cases referred to in the preceeding paragraph, no judgment shall be based upon a stipulation of facts or confession of judgment. It can be argued that since the lower court dismissed the petition, the evil sought to be prevented (i.e., dissolution of the marriage) did not come about, hence, the lack of participation of the State was cured. Not so. The task of protecting marriage as an inviolable social institution requires vigilant and zealous participation and not mere pro-forma compliance. The protection of marriage as a sacred institution requires not just the defense of a true and genuine union but the exposure of an invalid one as well. This is made clear by the following pronouncement: (8) The trial court must order the prosecuting attorney or fiscal and the Solicitor General to appear as counsel for the state. No decision shall be handed down unless the Solicitor General issues a certification, which will be quoted in the decision, briefly stating therein his reasons for his agreement or opposition as the case may be, to the petition. The Solicitor-General shall discharge the equivalent function of the defensor vinculi contemplated under Canon 1095 (underscoring ours). The records are bereft of any evidence that the State participated in the prosecution of the case not just at the trial level but on appeal with the Court of Appeals as well. Other than the manifestation filed with the trial court on November 16, 1994, the State did not file any pleading, motion or position paper, at any stage of the proceedings. In Republic of the Philippines v. Erlinda Matias Dagdag, while we upheld the validity of the marriage, we nevertheless characterized the decision of the trial court as prematurely rendered since the investigating prosecutor was not given an opportunity to present controverting evidence before the judgment was rendered. This stresses the importance of the participation of the State. Having so ruled, we decline to rule on the factual disputes of the case, this being within the province of the trial court upon proper re-trial.

Obiter Dictum For purposes of re-trial, we guide the parties thus: In Republic vs. Court of Appeals, the guidelines in the interpretation and application of Article 36 of the Family Code are as follows (omitting guideline (8) in the enumeration as it was already earlier quoted): (1) The burden of proof to show the nullity of the marriage belongs to the plaintiff. Any doubt should be resolved in favor of the existence and continuation of the marriage and against its dissolution and nullity. This is rooted in the fact that both our Constitution and our laws cherish the validity of marriage and unity of the family. Thus, our Constitution devotes an entire Article on the Family, recognizing it as the foundation of the nation. It decrees marriage as legally inviolable, thereby protecting it from dissolution at the whim of the parties. Both the family and marriage are to be 451

University of the Cordilleras College of Law First Year C S.Y. 2013 - 2014 protected by the state. The Family Code echoes this constitutional edict on marriage and the family and emphasizes their permanence, inviolability and solidarity. (2) The root cause of the psychological incapacity must be: a) medically or clinically identified, b) alleged in the complaint, c) sufficiently proven by experts and d) clearly explained in the decision. Article 36 of the Family Code requires that the incapacity must be psychological-not physical, although its manifestations and/or symptoms may be physical. The evidence must convince the court that the parties, or one of them, was mentally or psychically (sic) ill to such an extent that the person could not have known the obligations he was assuming, or knowing them, could not have given valid assumption thereof. Although no example of such incapacity need be given here so as not to limit the application of the provision under the principle of ejusdem generis, nevertheless such root cause must be identified as a psychological illness and its incapacitating nature fully explained. Expert evidence may be given by qualified psychiatrists and clinical psychologists. (3) The incapacity must be proven to be existing at the time of the celebration of the marriage. The evidence must show that the illness was existing when the parties exchanged their I dos. The manifestation of the illness need not be perceivable at such time, but the illness itself must have attached at such moment, or prior thereto. (4) Such incapacity must also be shown to be medically or clinically permanent or incurable. Such incurability may be absolute or even relative only in regard to the other spouse, not necessarily absolutely against everyone of the same sex. Furthermore, such incapacity must be relevant to the assumption of marriage obligations, not necessarily to those not related to marriage, like the exercise of a profession or employment in a job. Hence, a pediatrician may be effective in diagnosing illnesses of children and prescribing medicine to cure them but may not be psychologically capacitated to procreate, bear and raise his/her own children as an essential obligation of marriage. (5) Such illness must be grave enough to bring about the disability of the party to assume the essential obligations of marriage. Thus, mild characteriological peculiarities, mood changes, occasional emotional outbursts cannot be accepted as root causes. The illness must be shown as downright incapacity or inability, not refusal, neglect or difficulty, much less ill will. In other words, there is a natal or supervening disabling factor in the person, an adverse integral element in the personality structure that effectively incapacitates the person from really accepting and thereby complying with the obligations essential to marriage. (6) The essential marital obligations must be those embraced by Articles 68 up to 71 of the Family Code as regards the husband and wife as well as Articles 220, 221 and 225 of the same Code in regard to parents and their children. Such non-complied marital obligation(s) must also be stated in the petition, proven by evidence and included in the text of the decision. (7) Interpretations given by the National Appellate Matrimonial Tribunal of the Catholic Church in the Philippines, while not controlling or decisive, should be given great respect by our courts.

The Fallo WHEREFORE, the Court REVERSES and SETS ASIDE the appealed decision of the Court of Appeals in CA-G. R. CV No. 51304, promulgated on April 30, 1998 and the decision of the Regional Trial Court, Branch 158, Pasig City in Civil Case No. 3190, dated June 16, 1995. Let the case be REMANDED to the trial court for proper trial. No costs. SO ORDERED. Davide, Jr., C.J. (Chairman), Puno, Kapunan, and Ynares-Santiago, JJ., concur.

452

University of the Cordilleras College of Law First Year C S.Y. 2013 - 2014 Case Digest FLORENCE MALCAMPO-SIN vs. PHILIPP T. SIN G.R. No. 137590 March 26, 2001 355 SCRA 28 PARDO, J. FACTS: Florence, the petitioner, was married with Philipp, a Portuguese citizen in January 1987. Florence filed in September 1994, a complaint for the declaration of nullity of their marriage. Trial ensued and the parties presented their respective documentary and testimonial evidence. In June 1995, trial court dismissed Florences petition and throughout its trial, the State did not participate in the proceedings. While Fiscal Jabson filed with the trial court a manifestation dated November 1994 stating that he found no collusion between the parties, he did not actively participated therein. Other than having appearance at certain hearings, nothing more was heard of him. ISSUE: Whether the declaration of nullity may be declared even with the absence of the participation of the State in the proceedings. RULING: Article 48 of the Family Code states that in all cases of annulment or declaration of absolute nullity of marriage, the Court shall order the prosecuting attorney or fiscal assigned to it to appear on behalf of the state to take steps to prevent collusion between the parties and to take care that evidence is not fabricated or suppressed. The trial court should have ordered the prosecuting attorney or fiscal and the Solicitor-General to appear as counsel for the state. No decision shall be handed down unless the Solicitor General issues a certification briefly stating his reasons for his agreement or opposition as the case may be, to the petition. The records are bereft of evidence that the State participated in the prosecution of the case thus, the case is remanded for proper trial.

453

University of the Cordilleras College of Law First Year C S.Y. 2013 - 2014 Choa vs Choa 392 SCRA 198 G.R. No. 143376. November 26, 2002 Full Case LENI O. CHOA, petitioner, vs ALFONSO C. CHOA, respondent. PANGANIBAN, J.: Though interlocutory in character, an order denying a demurrer to evidence may be the subject of a certiorari proceeding, provided the petitioner can show that it was issued with grave abuse of discretion; and that appeal in due course is not plain, adequate or speedy under the circumstances. Indeed, when the plaintiffs evidence is utterly and patently insufficient to prove the complaint, it would be capricious for a trial judge to deny the demurrer and to require the defendant to present evidence to controvert a nonexisting case. Verily, the denial constitutes an unwelcome imposition on the courts docket and an assault on the defendants resources and peace of mind. In short, such denial needlessly delays and, thus, effectively denies justice.

The Case Before us is a Petition for Review on Certiorari under Rule 45 of the Rules of Court, assailing the March 16, 2000 Decision and the May 22, 2000 Resolution of the Court of Appeals (CA) in CA-GR SP No. 53100. The decretal portion of the Decision reads as follows: WHEREFORE, the instant Petition is hereby DISMISSED for lack of merit. The assailed Resolution denied petitioners Motion for Reconsideration.

The Facts Petitioner and respondent were married on March 15, 1981. Out of this union, two children were born, Cheryl Lynne and Albryan. On October 27, 1993, respondent filed before the Regional Trial Court (RTC) of Negros Occidental, Branch 51, a Complaint for the annulment of his marriage to petitioner. The Complaint was docketed as Civil Case No. 93-8098. Afterwards he filed an Amended Complaint dated November 8, 1993 for the declaration of nullity of his marriage to petitioner based on her alleged psychological incapacity. The case went to trial with respondent presenting his evidence in chief. After his last witness testified, he submitted his Formal Offer of Exhibits dated February 20, 1998. Instead of offering any objection to it, petitioner filed a Motion to Dismiss (Demurrer to Evidence) dated May 11, 1998. The lower court then allowed a number of pleadings to be filed thereafter. Finally, the RTC issued its December 2, 1998 Order denying petitioners Demurrer to Evidence. It held that [respondent] established a quantum of evidence that the [petitioner] must controvert. After her Motion for Reconsideration was denied in the March 22, 1999 Order, petitioner elevated the case to the CA by way of a Petition for Certiorari, docketed as CA-GR No. 53100.

Ruling of the Court of Appeals The CA held that the denial of the demurrer was merely interlocutory; hence, certiorari under Rule 65 of the Rules of Court was not available. The proper remedy was for the defense to present evidence; and if an unfavorable decision was handed down later, to take an appeal therefrom. In any event, no grave abuse of discretion was committed by respondent judge in issuing the assailed Orders. 454

University of the Cordilleras College of Law First Year C S.Y. 2013 - 2014 The CA also ruled that the propriety of granting or denying a demurrer to evidence rests on the sound exercise of the [trial] courts discretion. Further, the [p]etitioner failed to show that the issues in the court below [had] been resolved arbitrarily or without basis. Hence, this Petition.

The Issues In her Memorandum, petitioner submits the following issues for our consideration: 1) Upon the denial of petitioners demurrer to evidence under Rule 33 of the 1997 Rules of Civil Procedure, is she under obligation, as a matter of inflexible rule, as what the Court of Appeals required of her, to present her evidence, and when an unfavorable [verdict] is handed down, appeal therefrom in the manner authorized by law, despite the palpably and patently weak and grossly insufficient or so inadequate evidence of the private respondent as plaintiff in the annulment of marriage case, grounded on psychological incapacity under Art. 36 of The Family Code? Or under such circumstances, can the extraordinary remedy of certiorari be directly and immediately resorted to by the petitioner; and 2) In upholding the lower courts denial of petitioners demurrer to evidence, did the Court of Appeals wantonly violate, ignore or disregard in a whimsical manner the doctrinal pronouncements of this Court in Molina (G.R. No. 108763, February 13, 1997, 268 SCRA 198) and Santos (G.R. No. 112019, January 14, 1995, 58 SCRA 17)? Simply stated, the issues are: (1) is certiorari available to correct an order denying a demurrer to evidence? and (2) in its denial, did the RTC commit grave abuse of discretion by violating or ignoring the applicable law and jurisprudence?

The Courts Ruling The Petition is meritorious.

First Issue: Resort to Certiorari Petitioner argues that the RTC denied her Demurrer to Evidence despite the patent weakness and gross insufficiency of respondents evidence. Thus, she was entitled to the immediate recourse of the extraordinary remedy of certiorari. Echoing the CA, respondent counters that appeal in due course, not certiorari, is the proper remedy. We clarify. In general, interlocutory orders are neither appealable nor subject to certiorari proceedings. However, this rule is not absolute. In Tadeo v. People, this Court declared that appeal -- not certiorari -- in due time was indeed the proper remedy, provided there was no grave abuse of discretion or excess of jurisdiction or oppressive exercise of judicial authority. In fact, Rules 41 and 65 of the Rules of Court expressly recognize this exception and allow certiorari when the lower court acts with grave abuse of discretion in the issuance of an interlocutory order. Rule 41 provides: No appeal may be taken from: xxx xxx (c) An interlocutory order; xxx xxx xxx 455 xxx

University of the Cordilleras College of Law First Year C S.Y. 2013 - 2014 In all the above instances where the judgment or final order is not appealable, the aggrieved party may file an appropriate special civil action under Rule 65. In turn, Section 1 of Rule 65 reads as follows: SEC. 1. Petition for certiorari -- When any tribunal, board or officer exercising judicial or quasi-judicial functions has acted without or in excess of its or his jurisdiction, or with grave abuse of discretion amounting to lack or excess of jurisdiction, and there is no appeal, nor any plain, speedy, and adequate remedy in the ordinary course of law, a person aggrieved thereby may file a verified petition in the proper court, alleging the facts with certainty and praying that judgment be rendered annulling or modifying the proceedings of such tribunal, board or officer, and granting such incidental reliefs as law and justice may require. Thus, a denial of a demurrer that is tainted with grave abuse of discretion amounting to lack or excess of jurisdiction may be assailed through a petition for certiorari. In Cruz v. People, this exception was stressed by the Court in this wise: Admittedly, the general rule that the extraordinary writ of certiorari is not available to challenge interlocutory orders of the trial court may be subject to exceptions. When the assailed interlocutory orders are patently erroneous or issued with grave abuse of discretion, the remedy of certiorari lies.

Second Issue: Denial of Demurrer to Evidence Having established that a writ of certiorari may be issued in exceptional circumstances, this Court is now tasked to determine whether the present case falls under the exception; that is, whether the RTC indeed committed a patent error or grave abuse of discretion in denying petitioners Demurrer to Evidence. A demurrer to evidence is defined as an objection or exception by one of the parties in an action at law, to the effect that the evidence which his adversary produced is insufficient in point of law (whether true or not) to make out his case or sustain the issue. The demurrer challenges the sufficiency of the plaintiffs evidence to sustain a verdict. In passing upon the sufficiency of the evidence raised in a demurrer, the court is merely required to ascertain whether there is competent or sufficient proof to sustain the indictment or to support a verdict of guilt. We have thoroughly reviewed the records of the present case, and we are convinced that the evidence against respondent (herein petitioner) is grossly insufficient to support any finding of psychological incapacity that would warrant a declaration of nullity of the parties marriage. First. Respondent claims that the filing by petitioner of a series of charges against him are proof of the latters psychological incapacity to comply with the essential obligations of marriage. These charges included Complaints for perjury, false testimony, concubinage and deportation. According to him, the filing and the prosecution of these cases clearly showed that his wife (herein petitioner) wanted not only to put him behind bars, but also to banish him from the country. He contends that this is very abnormal for a wife who, instead of protecting the name and integrity of her husband as the father of her children, had acted to the contrary. We do not agree. The documents presented by respondent during the trial do not in any way show the alleged psychological incapacity of his wife. It is the height of absurdity and inequity to condemn her as psychologically incapacitated to fulfill her marital obligations, simply because she filed cases against him. The evidence presented, even if taken as true, merely establishes the prosecution of the cases against him. To rule that the filings are sufficient to establish her psychological incapacity is not only totally erroneous, but also grave abuse of discretion bordering on absurdity. Second. Neither is the testimony of respondent, taken by itself or in conjunction with his documentary offerings, sufficient to prove petitioners alleged psychological incapacity. He testified in these words: Q Will you please tell us or explain to the Court what do you mean by psychologically incapacitated to comply with the essential obligations of marriage. What do you mean by that? 456

University of the Cordilleras College of Law First Year C S.Y. 2013 - 2014 Because before our marriage she was already on the family way, so at that time she even want it aborted by taking pills. She was even immature, carefree, and she lacked the intention of procreative sexuality. x x xxx x x

x x ATTY. CHUA:

And you consider her that she was carefree, she is psychologically incapacitated? Will you please elaborate on this what you mean by carefree approximating psychologically incapacitated? ATTY. MIRANO: I think we better ask the witness what he means by carefree. ATTY. CHUA: Okay. COURT: Witness may answer. WITNESS: She does not help in the household chores, she does not take care of the child, she wants me to hire an attendant in order to take care of the child. Even when the children were sick she does not bother to let the children see a doctor. x x ATTY. CHUA: Now. From the time of courtship up to the time of your marriage to the defendant, did you notice any characteristic or traits which you consider as psychological incapacity? WITNESS: Sometimes when I cannot visit at her house she gets mad at me, and she wont talk to me when I call her up by telephone. So, all she wanted for me to visit her everytime and even at the time when I am busy with some other things. So, I think that is all. Even if taken as true, the testimony of respondent basically complains about three aspects of petitioners personality; namely, her alleged (1) lack of attention to their children, (2) immaturity and (3) lack of an intention of procreative sexuality. None of these three, singly or collectively, constitutes psychological incapacity. Far from it. In Santos v. CA, this Court clearly explained that psychological incapacity must be characterized by (a) gravity, (b) juridical antecedence and (c) incurability. Said the Court: It should be obvious, looking at all the foregoing disquisitions, including, and most importantly, the deliberations of the Family Code Revision Committee itself, that the use of the phrase psychological incapacity under Article 36 of the Code has not been meant to comprehend all such possible cases of psychoses as, likewise mentioned by some ecclesiastical authorities, extremely low intelligence, immaturity, and like circumstances (cited in Fr. Artemio Baluma's Void and Voidable Marriages in the Family Code and their Parallels in Canon Law, quoting from the Diagnostic Statistical Manual of Mental Disorder by the American Psychiatric Association; Edward Hudson's Handbook II for Marriage Nullity Cases). Article 36 of the Family Code cannot be taken and construed independently of but must stand in conjunction with, existing precepts in our law on marriage. Thus correlated, psychological incapacity should refer to no less than a mental (not physical) incapacity that causes a party to be truly incognitive of the basic marital covenants that concomitantly must be assumed and discharged by the parties to the marriage which, as so expressed by Article 68 of the Family Code, include their mutual obligations to live together, observe love, respect and fidelity and render help and support. There is hardly any doubt that the intendment of the law has been to confine the meaning of psychological incapacity to the most serious cases of personality disorders clearly demonstrative of an utter insensitivity or inability to give meaning 457 x x xxx STENOGRAPHER (reads back the question of Atty. Chua): x x

University of the Cordilleras College of Law First Year C S.Y. 2013 - 2014 and significance to the marriage. This psychologic condition must exist at the time the marriage is celebrated. Furthermore, in Republic v. Molina, we ruled that the psychological incapacity must be more than just a difficulty, a refusal or a neglect in the performance of some marital obligations. We stressed that a mere showing of irreconcilable differences and conflicting personalities in no wise constitutes psychological incapacity. In the case at bar, the evidence adduced by respondent merely shows that he and his wife could not get along with each other. There was absolutely no showing of the gravity or juridical antecedence or incurability of the problems besetting their marital union. Sorely lacking in respondents evidence is proof that the psychological incapacity was grave enough to bring about the disability of a party to assume the essential obligations of marriage. In Molina, we affirmed that mild characterological peculiarities, mood changes and occasional emotional outbursts cannot be accepted as root causes of psychological incapacity. The illness must be shown as downright incapacity or inability, not a refusal, neglect or difficulty, much less ill will. In other words, there should be a natal or supervening disabling factor in the person, an adverse integral element in the personality structure that effectively incapacitates the person from really accepting and thereby complying with the obligations essential to marriage. Respondents pious peroration that petitioner lacked the intention of procreative sexuality is easily belied by the fact that two children were born during their union. Moreover, there is absolutely no showing that the alleged defect was already existing at the time of the celebration of the marriage. Third. Most telling is the insufficiency, if not incompetency, of the supposed expert testimony presented by respondent. His witness, Dr. Antonio M. Gauzon, utterly failed to identify and prove the root cause of the alleged psychological incapacity. Specifically, his testimony did not show that the incapacity, if true, was medically or clinically permanent or incurable. Neither did he testify that it was grave enough to bring about the disability of the party to assume the essential obligations of marriage. The pertinent portions of his testimony are quoted thus: ATTY. CHUA: And then finally and ultimately you reached the conclusion that both parties, meaning the husband and the wife in the present case have a personality which is normal. That is your conclusion? WITNESS: They are normal, but they cannot mix together. Q. So as a general proposition, both of them are of normal personality, only that they are not compatible with each other? A. Yes. Q. And by normal personality, you mean that neither of them suffer from any personality disorder, bordering on abnormality? A. Yes. Q. But Doctor, is not a fact or a fact of life, that no couple could be or are perfectly match? A. Precisely, if there is a problem, marital problem, there should be somebody who knows how to handle marriage, that should try to intervene. Q. You mean expert advise or services should be needed by the couple? A. Yes. Q. Now, if the couple are mature enough and each of them practises what we call maximum tolerance and give and take, will that serve the purpose? A. That would served the purpose of getting well. Q. Yes? A. Yes. Q. Meaning to say that the incompatibility could be harmonized? 458

University of the Cordilleras College of Law First Year C S.Y. 2013 - 2014 A. Yes, because they are supposedly normal, but both of them are personally disordered. It cannot be harmonized. So this case, if only they have tried professional help to take care of their marital problem, it could have been solved. Q. Or the situation could have been remedied? A. Yes. But I would like to say that it must be somebody who is an expert. Not just any from Tom, Dick and Harry could handle this. That means from the very beginning they have personalities which they were incompatible. So if anybody would handle that, they will not mix, they will be always quarreling with each other. They should not have got married. x x x x xxx Q. Yes. So in this present case, your expert opinion was sought by the plaintiff, and you found out that both are normal? A. With different personalities. So that they were incompatible. Q. Normal, simply incompatible. A. Yes, with personalities different from each other, which I mentioned there in my last page. That they are like oil and water, immiscible. Like oil and water, they will not mix. Q. You also mentioned that the plaintiff. Meaning to say the husband told you about the frequent quarrels had with the wife. Did he ever tell you that was a serious or major quarrel? A. Actually there was no major quarrel. It was all petty quarrels. x x x x xxx Q. So the problem of this couple is fundamentally a conflicting personalities? A. Yes. x x x x xxx Q. Now, you mentioned that you maybe able to make them reconcile? A. Yes. Q. You mean that given the time and opportunity, things could be worked out? A. Yes. Q. You mean reconciliation at this stage with expert services, and the advise of those who possess the necessary [expertise] could be worked out? A. Yes, as I said it can be done by therapy. Family therapy. x x x x xxx Q. Doctor, you draw your conclusion that there is psychological inc[a]pacity existing in this case? A. Yes. Q. Because of the A. The incompatibility. Q. Incompatibility. A. Yes. His testimony established merely that the spouses had an incompatibility, a defect that could possibly be treated or alleviated through psychotherapy. We need not expound further on the patent insufficiency of the expert testimony to establish the psychological incapacity of petitioner. x x x x x x x x

459

University of the Cordilleras College of Law First Year C S.Y. 2013 - 2014 Furthermore, the assessment of petitioner by Dr. Gauzon was based merely on descriptions communicated to him by respondent. The doctor never conducted any psychological examination of her. Neither did he ever claim to have done so. In fact, his Professional Opinionbegan with the statement [I]f what Alfonso Choa said about his wife Leni is true, x x x. The expert witness testified thus: ATTY. CHUA Q Doctor, in this professional opinion of yours, you gathered most of your material data from the plaintiff who is the husband?

WITNESS A Yes. By the way, I requested the husband Alfonso, if it was possible for me to interview Leni, and he said, he doesnt know. He doesnt know. Now, Doctor if we were to request you to conduct the same personal interview and written psychological examination on the part of the wife, [w]ould you be willing to do that? Sure for a fee. I maybe able to make them reconcile.

ATTY. CHUA Q

WITNESS A

Obviously, Dr. Gauzon had no personal knowledge of the facts he testified to, as these had merely been relayed to him by respondent. The former was working on pure suppositions and secondhand information fed to him by one side. Consequently, his testimony can be dismissed as unscientific and unreliable. Dr. Gauzon tried to save his credibility by asserting that he was able to assess petitioners character, not only through the descriptions given by respondent, but also through the formers at least fifteen hours of study of the voluminous transcript of records of this case. Even if it took the good doctor a whole day or a whole week to examine the records of this case, we still find his assessment of petitioners psychological state sorely insufficient and methodologically flawed. As to respondents argument -- that because Dr. Gauzons testimony had never been objected to, the objection raised thereafter was deemed waived -- the Supreme Court has already ruled on the matter. It held that although the question of admissibility of evidence could not be raised for the first time on appeal, hearsay or unreliable evidence should be disregarded whether objected to or not, because it has no probative value. We are, of course, mindful of the ruling that a medical examination is not a conditio sine qua non to a finding of psychological incapacity, so long as the totality of evidence presented is enough to establish the incapacity adequately. Here, however, the totality of evidence presented by respondent was completely insufficient to sustain a finding of psychological incapacity -- more so without any medical, psychiatric or psychological examination. The trial court should have carefully studied and assessed the evidence presented by respondent and taken into account the prevailing jurisprudence on the matter. It could then have easily concluded, as we conclude now, that it was useless to proceed further with the tedious process of hearing contravening proof. His evidence was obviously, grossly and clearly insufficient to support a declaration of nullity of marriage based on psychological incapacity. Withal, it was grave abuse of discretion for the RTC to deny the Demurrer and to violate or ignore this Courts rulings in point. Indeed, continuing the process of litigation would have been a total waste of time and money for the parties and an unwelcome imposition on the trial courts docket. We have already ruled that grave abuse of discretion may arise when a lower court or tribunal violates or contravenes the Constitution, the law or existing jurisprudence. Any decision, order or resolution of a lower court tantamount to overruling a judicial pronouncement of the highest Court is unmistakably a very grave abuse of discretion. There is no reason to believe that an appeal would prove to be a plain, speedy or adequate remedy in the case at bar. An appeal would not promptly relieve petitioner from the injurious effects of the patently mistaken Orders maintaining the baseless action of respondent. It would only compel her to go needlessly through a protracted trial, which would further clog the court dockets with another futile case. WHEREFORE, the Petition is hereby GRANTED and the assailed CA Decision REVERSED and SET ASIDE. Respondents Demurrer to Evidence is GRANTED, and the case 460

University of the Cordilleras College of Law First Year C S.Y. 2013 - 2014 for declaration of nullity of marriage based on the alleged psychological incapacity of petitioner isDISMISSED. No pronouncement as to costs. SO ORDERED. Sandoval-Gutierrez, Corona, and Carpio-Morales, JJ., concur. Puno, (Chairman), J., abroad on official leave.

461

University of the Cordilleras College of Law First Year C S.Y. 2013 - 2014 Case Digest LENI O. CHOA vs. ALFONSO C. CHOA G.R. No. 143376 November 26, 2002 392 SCRA 198 PANGANIBAN, J. FACTS: Leni Choa, petitioner, and Alfonso Choa, respondent, were married on March 15, 1981. Out of this union, two children were born. On October 27, 1993, respondent filed a complaint for the annulment of his marriage to petitioner. Also filed an amended complaint for the declaration of nullity of his marriage based on her alleged psychological incapacity. The case went on trial with the respondent presenting his evidence. However, petitioner filed a motion to dismiss the evidence. RTC denied petitioners demurrer to evidence on the ground that petitioner must controvert the established quantum evidence of respondent. Petitioner elevated the case to CA after the motion of reconsideration was denied. CA held that denial of the demurrer was merely interlocutory and petitioner in her defense must present evidence. ISSUE: Whether or not petitioners obligated to present her evidence despite the inadequate evidence of respondent in the annulment of marriage case grounded on psychological incapacity. RULING: The petition is meritorious. However, the evidence against petitioner is grossly insufficient to support any finding of psychological incapacity that would warrant a declaration of nullity of the parties marriage. Respondent claims that the filing by petitioner of a series of charges against him are proof of the latters psychological incapacity to comply with the essential obligations of marriage. These charges included Complaints for perjury, false testimony, concubinage and deportation. The documents presented by respondent during the trial do not in any way show the alleged psychological incapacity of his wife. It is the height of absurdity and inequity to condemn her as psychologically incapacitated to fulfill her marital obligations, simply because she filed cases against him. The evidence presented merely establishes the prosecution of the cases against him. To rule that the filings are sufficient to establish her psychological incapacity is not only totally erroneous, but also grave abuse of discretion bordering on absurdity. Court clearly explained that "psychological incapacity must be characterized by (a) gravity, (b) juridical antecedence and (c) incurability. The evidence adduced by respondent merely shows that he and his wife could not get along with each other. There was absolutely no showing of the gravity or juridical antecedence or incurability of the problems besetting their marital union.

462

University of the Cordilleras College of Law First Year C S.Y. 2013 - 2014

Republic vs Iyoy 470 SCRA 508


G.R. No. 152577 September 21, 2005 Full Case REPUBLIC OF THE PHILIPPINES, Petitioners, vs. CRASUS L. IYOY, Respondent. CHICO-NAZARIO, J.: In this Petition for Review on Certiorari under Rule 45 of the Rules of Court, petitioner Republic of the Philippines, represented by the Office of the Solicitor General, prays for the reversal of the Decision of the Court of Appeals in CA-G.R. CV No. 62539, dated 30 July 2001, affirming the Judgment of the Regional Trial Court (RTC) of Cebu City, Branch 22, in Civil Case No. CEB-20077, dated 30 October 1998, declaring the marriage between respondent Crasus L. Iyoy and Fely Ada Rosal-Iyoy null and void on the basis of Article 36 of the Family Code of the Philippines. The proceedings before the RTC commenced with the filing of a Complaint for declaration of nullity of marriage by respondent Crasus on 25 March 1997. According to the said Complaint, respondent Crasus married Fely on 16 December 1961 at Bradford Memorial Church, Jones Avenue, Cebu City. As a result of their union, they had five children Crasus, Jr., Daphne, Debbie, Calvert, and Carlos who are now all of legal ages. After the celebration of their marriage, respondent Crasus discovered that Fely was "hottempered, a nagger and extravagant." In 1984, Fely left the Philippines for the United States of America (U.S.A.), leaving all of their five children, the youngest then being only six years old, to the care of respondent Crasus. Barely a year after Fely left for the U.S.A., respondent Crasus received a letter from her requesting that he sign the enclosed divorce papers; he disregarded the said request. Sometime in 1985, respondent Crasus learned, through the letters sent by Fely to their children, that Fely got married to an American, with whom she eventually had a child. In 1987, Fely came back to the Philippines with her American family, staying at Cebu Plaza Hotel in Cebu City. Respondent Crasus did not bother to talk to Fely because he was afraid he might not be able to bear the sorrow and the pain she had caused him. Fely returned to the Philippines several times more: in 1990, for the wedding of their eldest child, Crasus, Jr.; in 1992, for the brain operation of their fourth child, Calvert; and in 1995, for unknown reasons. Fely continued to live with her American family in New Jersey, U.S.A. She had been openly using the surname of her American husband in the Philippines and in the U.S.A. For the wedding of Crasus, Jr., Fely herself had invitations made in which she was named as "Mrs. Fely Ada Micklus." At the time the Complaint was filed, it had been 13 years since Fely left and abandoned respondent Crasus, and there was no more possibility of reconciliation between them. Respondent Crasus finally alleged in his Complaint that Felys acts brought danger and dishonor to the family, and clearly demonstrated her psychological incapacity to perform the essential obligations of marriage. Such incapacity, being incurable and continuing, constitutes a ground for declaration of nullity of marriage under Article 36, in relation to Articles 68, 70, and 72, of the Family Code of the Philippines. Fely filed her Answer and Counterclaim with the RTC on 05 June 1997. She asserted therein that she was already an American citizen since 1988 and was now married to Stephen Micklus. While she admitted being previously married to respondent Crasus and having five children with him, Fely refuted the other allegations made by respondent Crasus in his Complaint. She explained that she was no more hottempered than any normal person, and she may had been indignant at respondent Crasus on certain occasions but it was because of the latters drunkenness, womanizing, and lack of sincere effort to find employment and to contribute to the maintenance of their household. She could not have been extravagant since the family hardly had enough money for basic needs. Indeed, Fely left for abroad for financial reasons as respondent Crasus had no job and what she was then earning as the sole breadwinner in the Philippines was insufficient to support their family. Although she left all of her children with respondent Crasus, she continued to provide financial support to them, as well as, to respondent Crasus. Subsequently, Fely was able to bring her children to the U.S.A., except for one, Calvert, who had to stay behind for medical reasons. While she did file for divorce from respondent Crasus, she denied having herself sent a letter to respondent Crasus requesting him to sign the enclosed divorce papers. After securing a divorce from respondent Crasus, Fely married her American husband and acquired American citizenship. She argued that her marriage to her American husband was legal because now being an American citizen, her status shall be governed by the law of her present nationality. Fely also pointed out that respondent Crasus himself was presently living with another woman who bore him a child. She also 463

University of the Cordilleras College of Law First Year C S.Y. 2013 - 2014 accused respondent Crasus of misusing the amount of P90,000.00 which she advanced to him to finance the brain operation of their son, Calvert. On the basis of the foregoing, Fely also prayed that the RTC declare her marriage to respondent Crasus null and void; and that respondent Crasus be ordered to pay to Fely the P90,000.00 she advanced to him, with interest, plus, moral and exemplary damages, attorneys fees, and litigation expenses. After respondent Crasus and Fely had filed their respective Pre-Trial Briefs, the RTC afforded both parties the opportunity to present their evidence. Petitioner Republic participated in the trial through the Provincial Prosecutor of Cebu. Respondent Crasus submitted the following pieces of evidence in support of his Complaint: (1) his own testimony on 08 September 1997, in which he essentially reiterated the allegations in his Complaint; (2) the Certification, dated 13 April 1989, by the Health Department of Cebu City, on the recording of the Marriage Contract between respondent Crasus and Fely in the Register of Deeds, such marriage celebration taking place on 16 December 1961; and (3) the invitation to the wedding of Crasus, Jr., their eldest son, wherein Fely openly used her American husbands surname, Micklus. Felys counsel filed a Notice, and, later on, a Motion, to take the deposition of witnesses, namely, Fely and her children, Crasus, Jr. and Daphne, upon written interrogatories, before the consular officers of the Philippines in New York and California, U.S.A, where the said witnesses reside. Despite the Orders and Commissions issued by the RTC to the Philippine Consuls of New York and California, U.S.A., to take the depositions of the witnesses upon written interrogatories, not a single deposition was ever submitted to the RTC. Taking into account that it had been over a year since respondent Crasus had presented his evidence and that Fely failed to exert effort to have the case progress, the RTC issued an Order, dated 05 October 1998, considering Fely to have waived her right to present her evidence. The case was thus deemed submitted for decision. Not long after, on 30 October 1998, the RTC promulgated its Judgment declaring the marriage of respondent Crasus and Fely null and void ab initio, on the basis of the following findings The ground bearing defendants psychological incapacity deserves a reasonable consideration. As observed, plaintiffs testimony is decidedly credible. The Court finds that defendant had indeed exhibited unmistakable signs of psychological incapacity to comply with her marital duties such as striving for family unity, observing fidelity, mutual love, respect, help and support. From the evidence presented, plaintiff adequately established that the defendant practically abandoned him. She obtained a divorce decree in the United States of America and married another man and has establish [ sic] another family of her own. Plaintiff is in an anomalous situation, wherein he is married to a wife who is already married to another man in another country. Defendants intolerable traits may not have been apparent or manifest before the marriage, the FAMILY CODE nonetheless allows the annulment of the marriage provided that these were eventually manifested after the wedding. It appears to be the case in this instance. Certainly defendants posture being an irresponsible wife erringly reveals her very low regard for that sacred and inviolable institution of marriage which is the foundation of human society throughout the civilized world. It is quite evident that the defendant is bereft of the mind, will and heart to comply with her marital obligations, such incapacity was already there at the time of the marriage in question is shown by defendants own attitude towards her marriage to plaintiff. In sum, the ground invoked by plaintiff which is defendants psychological incapacity to comply with the essential marital obligations which already existed at the time of the marriage in question has been satisfactorily proven. The evidence in herein case establishes the irresponsibility of defendant Fely Ada Rosal Iyoy, firmly. Going over plaintiffs testimony which is decidedly credible, the Court finds that the defendant had indeed exhibited unmistakable signs of such psychological incapacity to comply with her marital obligations. These are her excessive disposition to material things over and above the marital stability. That such incapacity was already there at the time of the marriage in question is shown by defendants own attitude towards her marriage to plaintiff. And for these reasons there is a legal ground to declare the marriage of plaintiff Crasus L. Iyoy and defendant Fely Ada Rosal Iyoy null and void ab initio.

464

University of the Cordilleras College of Law First Year C S.Y. 2013 - 2014 Petitioner Republic, believing that the afore-quoted Judgment of the RTC was contrary to law and evidence, filed an appeal with the Court of Appeals. The appellate court, though, in its Decision, dated 30 July 2001, affirmed the appealed Judgment of the RTC, finding no reversible error therein. It even offered additional ratiocination for declaring the marriage between respondent Crasus and Fely null and void, to wit Defendant secured a divorce from plaintiff-appellee abroad, has remarried, and is now permanently residing in the United States. Plaintiff-appellee categorically stated this as one of his reasons for seeking the declaration of nullity of their marriage Article 26 of the Family Code provides: "Art. 26. All marriages solemnized outside the Philippines in accordance with the laws in force in the country where they were solemnized, and valid there as such, shall also be valid in this country, except those prohibited under Articles 35(1), (4), (5) and (6), 36, 37 and 38. "WHERE A MARRIAGE BETWEEN A FILIPINO CITIZEN AND A FOREIGNER IS VALIDLY CELEBRATED AND A DIVORCE IS THEREAFTER VALIDLY OBTAINED ABROAD BY THE ALIEN SPOUSE CAPACITATING HIM OR HER TO REMARRY, THE FILIPINO SPOUSE SHALL LIKEWISE HAVE CAPACITY TO REMARRY UNDER PHILIPPINE LAW." The rationale behind the second paragraph of the above-quoted provision is to avoid the absurd and unjust situation of a Filipino citizen still being married to his or her alien spouse, although the latter is no longer married to the Filipino spouse because he or she has obtained a divorce abroad. In the case at bench, the defendant has undoubtedly acquired her American husbands citizenship and thus has become an alien as well. This Court cannot see why the benefits of Art. 26 aforequoted can not be extended to a Filipino citizen whose spouse eventually embraces another citizenship and thus becomes herself an alien. It would be the height of unfairness if, under these circumstances, plaintiff would still be considered as married to defendant, given her total incapacity to honor her marital covenants to the former. To condemn plaintiff to remain shackled in a marriage that in truth and in fact does not exist and to remain married to a spouse who is incapacitated to discharge essential marital covenants, is verily to condemn him to a perpetual disadvantage which this Court finds abhorrent and will not countenance. Justice dictates that plaintiff be given relief by affirming the trial courts declaration of the nullity of the marriage of the parties. After the Court of Appeals, in a Resolution, dated 08 March 2002, denied its Motion for Reconsideration, petitioner Republic filed the instant Petition before this Court, based on the following arguments/grounds I. Abandonment by and sexual infidelity of respondents wife do not per se constitute psychological incapacity. II. The Court of Appeals has decided questions of substance not in accord with law and jurisprudence considering that the Court of Appeals committed serious errors of law in ruling that Article 26, paragraph 2 of the Family Code is inapplicable to the case at bar. In his Comment to the Petition, respondent Crasus maintained that Felys psychological incapacity was clearly established after a full-blown trial, and that paragraph 2 of Article 26 of the Family Code of the Philippines was indeed applicable to the marriage of respondent Crasus and Fely, because the latter had already become an American citizen. He further questioned the personality of petitioner Republic, represented by the Office of the Solicitor General, to institute the instant Petition, because Article 48 of the Family Code of the Philippines authorizes the prosecuting attorney or fiscal assigned to the trial court, not the Solicitor General, to intervene on behalf of the State, in proceedings for annulment and declaration of nullity of marriages. After having reviewed the records of this case and the applicable laws and jurisprudence, this Court finds the instant Petition to be meritorious. 465

University of the Cordilleras College of Law First Year C S.Y. 2013 - 2014 I The totality of evidence presented during trial is insufficient to support the finding of psychological incapacity of Fely. Article 36, concededly one of the more controversial provisions of the Family Code of the Philippines, reads ART. 36. A marriage contracted by any party who, at the time of the celebration, was psychologically incapacitated to comply with the essential marital obligations of marriage, shall likewise be void even if such incapacity becomes manifest only after its solemnization. Issues most commonly arise as to what constitutes psychological incapacity. In a series of cases, this Court laid down guidelines for determining its existence. In Santos v. Court of Appeals, the term psychological incapacity was defined, thus ". . . [P]sychological incapacity" should refer to no less than a mental (not physical) incapacity that causes a party to be truly cognitive of the basic marital covenants that concomitantly must be assumed and discharged by the parties to the marriage which, as so expressed by Article 68 of the Family Code, include their mutual obligations to live together, observe love, respect and fidelity and render help and support. There is hardly any doubt that the intendment of the law has been to confine the meaning of "psychological incapacity" to the most serious cases of personality disorders clearly demonstrative of an utter insensitivity or inability to give meaning and significance to the marriage. This psychological condition must exist at the time the marriage is celebrated The psychological incapacity must be characterized by (a) Gravity It must be grave or serious such that the party would be incapable of carrying out the ordinary duties required in a marriage; (b) Juridical Antecedence It must be rooted in the history of the party antedating the marriage, although the overt manifestations may emerge only after the marriage; and (c) Incurability It must be incurable or, even if it were otherwise, the cure would be beyond the means of the party involved. More definitive guidelines in the interpretation and application of Article 36 of the Family Code of the Philippines were handed down by this Court in Republic v. Court of Appeals and Molina, which, although quite lengthy, by its significance, deserves to be reproduced below (1) The burden of proof to show the nullity of the marriage belongs to the plaintiff. Any doubt should be resolved in favor of the existence and continuation of the marriage and against its dissolution and nullity. This is rooted in the fact that both our Constitution and our laws cherish the validity of marriage and unity of the family. Thus, our Constitution devotes an entire Article on the Family, recognizing it "as the foundation of the nation." It decrees marriage as legally "inviolable," thereby protecting it from dissolution at the whim of the parties. Both the family and marriage are to be "protected" by the state. The Family Code echoes this constitutional edict on marriage and the family and emphasizes their permanence, inviolability and solidarity. (2) The root cause of the psychological incapacity must be (a) medically or clinically identified, (b) alleged in the complaint, (c) sufficiently proven by experts and (d) clearly explained in the decision. Article 36 of the Family Code requires that the incapacity must be psychological - not physical, although its manifestations and/or symptoms may be physical. The evidence must convince the court that the parties, or one of them, was mentally or psychically ill to such an extent that the person could not have known the obligations he was assuming, or knowing them, could not have given valid assumption thereof. Although no example of such incapacity need be given here so as not to limit the application of the provision under the principle of ejusdem generis, nevertheless such root cause must be identified as a

466

University of the Cordilleras College of Law First Year C S.Y. 2013 - 2014 psychological illness and its incapacitating nature fully explained. Expert evidence may be given by qualified psychiatrists and clinical psychologists. (3) The incapacity must be proven to be existing at "the time of the celebration" of the marriage. The evidence must show that the illness was existing when the parties exchanged their "I do's." The manifestation of the illness need not be perceivable at such time, but the illness itself must have attached at such moment, or prior thereto. (4) Such incapacity must also be shown to be medically or clinically permanent or incurable. Such incurability may be absolute or even relative only in regard to the other spouse, not necessarily absolutely against everyone of the same sex. Furthermore, such incapacity must be relevant to the assumption of marriage obligations, not necessarily to those not related to marriage, like the exercise of a profession or employment in a job (5) Such illness must be grave enough to bring about the disability of the party to assume the essential obligations of marriage. Thus, "mild characteriological peculiarities, mood changes, occasional emotional outbursts" cannot be accepted as root causes. The illness must be shown as downright incapacity or inability, not a refusal, neglect or difficulty, much less ill will. In other words, there is a natal or supervening disabling factor in the person, an adverse integral element in the personality structure that effectively incapacitates the person from really accepting and thereby complying with the obligations essential to marriage. (6) The essential marital obligations must be those embraced by Articles 68 up to 71 of the Family Code as regards the husband and wife as well as Articles 220, 221 and 225 of the same Code in regard to parents and their children. Such non-complied marital obligation(s) must also be stated in the petition, proven by evidence and included in the text of the decision. (7) Interpretations given by the National Appellate Matrimonial Tribunal of the Catholic Church in the Philippines, while not controlling or decisive, should be given great respect by our courts (8) The trial court must order the prosecuting attorney or fiscal and the Solicitor General to appear as counsel for the state. No decision shall be handed down unless the Solicitor General issues a certification, which will be quoted in the decision, briefly stating therein his reasons for his agreement or opposition, as the case may be, to the petition. The Solicitor General, along with the prosecuting attorney, shall submit to the court such certification within fifteen (15) days from the date the case is deemed submitted for resolution of the court. The Solicitor General shall discharge the equivalent function of the defensor vinculi contemplated under Canon 1095. A later case, Marcos v. Marcos, further clarified that there is no requirement that the defendant/respondent spouse should be personally examined by a physician or psychologist as a condition sine qua non for the declaration of nullity of marriage based on psychological incapacity. Such psychological incapacity, however, must be established by the totality of the evidence presented during the trial. Using the guidelines established by the afore-mentioned jurisprudence, this Court finds that the totality of evidence presented by respondent Crasus failed miserably to establish the alleged psychological incapacity of his wife Fely; therefore, there is no basis for declaring their marriage null and void under Article 36 of the Family Code of the Philippines. The only substantial evidence presented by respondent Crasus before the RTC was his testimony, which can be easily put into question for being self-serving, in the absence of any other corroborating evidence. He submitted only two other pieces of evidence: (1) the Certification on the recording with the Register of Deeds of the Marriage Contract between respondent Crasus and Fely, such marriage being celebrated on 16 December 1961; and (2) the invitation to the wedding of Crasus, Jr., their eldest son, in which Fely used her American husbands surname. Even considering the admissions made by Fely herself in her Answer to respondent Crasuss Complaint filed with the RTC, the evidence is not enough to convince this Court that Fely had such a grave mental illness that prevented her from assuming the essential obligations of marriage. It is worthy to emphasize that Article 36 of the Family Code of the Philippines contemplates downright incapacity or inability to take cognizance of and to assume the basic marital obligations; not a mere 467

University of the Cordilleras College of Law First Year C S.Y. 2013 - 2014 refusal, neglect or difficulty, much less, ill will, on the part of the errant spouse. Irreconcilable differences, conflicting personalities, emotional immaturity and irresponsibility, physical abuse, habitual alcoholism, sexual infidelity or perversion, and abandonment, by themselves, also do not warrant a finding of psychological incapacity under the said Article. As has already been stressed by this Court in previous cases, Article 36 "is not to be confused with a divorce law that cuts the marital bond at the time the causes therefore manifest themselves. It refers to a serious psychological illness afflicting a party even before the celebration of marriage. It is a malady so grave and so permanent as to deprive one of awareness of the duties and responsibilities of the matrimonial bond one is about to assume." The evidence may have proven that Fely committed acts that hurt and embarrassed respondent Crasus and the rest of the family. Her hot-temper, nagging, and extravagance; her abandonment of respondent Crasus; her marriage to an American; and even her flaunting of her American family and her American surname, may indeed be manifestations of her alleged incapacity to comply with her marital obligations; nonetheless, the root cause for such was not identified. If the root cause of the incapacity was not identified, then it cannot be satisfactorily established as a psychological or mental defect that is serious or grave; neither could it be proven to be in existence at the time of celebration of the marriage; nor that it is incurable. While the personal examination of Fely by a psychiatrist or psychologist is no longer mandatory for the declaration of nullity of their marriage under Article 36 of the Family Code of the Philippines, by virtue of this Courts ruling in Marcos v. Marcos, respondent Crasus must still have complied with the requirement laid down in Republic v. Court of Appeals and Molina that the root cause of the incapacity be identified as a psychological illness and that its incapacitating nature be fully explained. In any case, any doubt shall be resolved in favor of the validity of the marriage. No less than the Constitution of 1987 sets the policy to protect and strengthen the family as the basic social institution and marriage as the foundation of the family. II Article 26, paragraph 2 of the Family Code of the Philippines is not applicable to the case at bar. According to Article 26, paragraph 2 of the Family Code of the Philippines Where a marriage between a Filipino citizen and a foreigner is validly celebrated and a divorce is thereafter validly obtained abroad by the alien spouse capacitating him or her to remarry, the Filipino spouse shall likewise have capacity to remarry under Philippine law. As it is worded, Article 26, paragraph 2, refers to a special situation wherein one of the couple getting married is a Filipino citizen and the other a foreigner at the time the marriage was celebrated. By its plain and literal interpretation, the said provision cannot be applied to the case of respondent Crasus and his wife Fely because at the time Fely obtained her divorce, she was still a Filipino citizen . Although the exact date was not established, Fely herself admitted in her Answer filed before the RTC that she obtained a divorce from respondent Crasus sometime after she left for the United States in 1984, after which she married her American husband in 1985. In the same Answer, she alleged that she had been an American citizen since 1988. At the time she filed for divorce, Fely was still a Filipino citizen, and pursuant to the nationality principle embodied in Article 15 of the Civil Code of the Philippines, she was still bound by Philippine laws on family rights and duties, status, condition, and legal capacity, even when she was already living abroad. Philippine laws, then and even until now, do not allow and recognize divorce between Filipino spouses. Thus, Fely could not have validly obtained a divorce from respondent Crasus. III The Solicitor General is authorized to intervene, on behalf of the Republic, in proceedings for annulment and declaration of nullity of marriages. Invoking Article 48 of the Family Code of the Philippines, respondent Crasus argued that only the prosecuting attorney or fiscal assigned to the RTC may intervene on behalf of the State in proceedings for 468

University of the Cordilleras College of Law First Year C S.Y. 2013 - 2014 annulment or declaration of nullity of marriages; hence, the Office of the Solicitor General had no personality to file the instant Petition on behalf of the State. Article 48 provides ART. 48. In all cases of annulment or declaration of absolute nullity of marriage, the Court shall order the prosecuting attorney or fiscal assigned to it to appear on behalf of the State to take steps to prevent collusion between the parties and to take care that the evidence is not fabricated or suppressed. That Article 48 does not expressly mention the Solicitor General does not bar him or his Office from intervening in proceedings for annulment or declaration of nullity of marriages. Executive Order No. 292, otherwise known as the Administrative Code of 1987, appoints the Solicitor General as the principal law officer and legal defender of the Government. His Office is tasked to represent the Government of the Philippines, its agencies and instrumentalities and its officials and agents in any litigation, proceeding, investigation or matter requiring the services of lawyers. The Office of the Solicitor General shall constitute the law office of the Government and, as such, shall discharge duties requiring the services of lawyers. The intent of Article 48 of the Family Code of the Philippines is to ensure that the interest of the State is represented and protected in proceedings for annulment and declaration of nullity of marriages by preventing collusion between the parties, or the fabrication or suppression of evidence; and, bearing in mind that the Solicitor General is the principal law officer and legal defender of the land, then his intervention in such proceedings could only serve and contribute to the realization of such intent, rather than thwart it. Furthermore, the general rule is that only the Solicitor General is authorized to bring or defend actions on behalf of the People or the Republic of the Philippines once the case is brought before this Court or the Court of Appeals. While it is the prosecuting attorney or fiscal who actively participates, on behalf of the State, in a proceeding for annulment or declaration of nullity of marriage before the RTC, the Office of the Solicitor General takes over when the case is elevated to the Court of Appeals or this Court. Since it shall be eventually responsible for taking the case to the appellate courts when circumstances demand, then it is only reasonable and practical that even while the proceeding is still being held before the RTC, the Office of the Solicitor General can already exercise supervision and control over the conduct of the prosecuting attorney or fiscal therein to better guarantee the protection of the interests of the State. In fact, this Court had already recognized and affirmed the role of the Solicitor General in several cases for annulment and declaration of nullity of marriages that were appealed before it, summarized as follows in the case of Ancheta v. Ancheta In the case of Republic v. Court of Appeals [268 SCRA 198 (1997)], this Court laid down the guidelines in the interpretation and application of Art. 48 of the Family Code, one of which concerns the role of the prosecuting attorney or fiscal and the Solicitor General to appear as counsel for the State: (8) The trial court must order the prosecuting attorney or fiscal and the Solicitor General to appear as counsel for the state. No decision shall be handed down unless the Solicitor General issues a certification, which will be quoted in the decision, briefly stating therein his reasons for his agreement or opposition, as the case may be, to the petition. The Solicitor General, along with the prosecuting attorney, shall submit to the court such certification within fifteen (15) days from the date the case is deemed submitted for resolution of the court. The Solicitor General shall discharge the equivalent function of the defensor vinculi contemplated under Canon 1095. [Id., at 213] This Court in the case of Malcampo-Sin v. Sin [355 SCRA 285 (2001)] reiterated its pronouncement in Republic v. Court of Appeals [Supra.] regarding the role of the prosecuting attorney or fiscal and the Solicitor General to appear as counsel for the State Finally, the issuance of this Court of the Rule on Declaration of Absolute Nullity of Void Marriages and Annulment of Voidable Marriages, which became effective on 15 March 2003, should dispel any other doubts of respondent Crasus as to the authority of the Solicitor General to file the instant Petition on behalf of the State. The Rule recognizes the authority of the Solicitor General to intervene and take part in the proceedings for annulment and declaration of nullity of marriages before the RTC and on appeal to higher courts. The pertinent provisions of the said Rule are reproduced below Sec. 5. Contents and form of petition. 469

University of the Cordilleras College of Law First Year C S.Y. 2013 - 2014 (4) It shall be filed in six copies. The petitioner shall serve a copy of the petition on the Office of the Solicitor General and the Office of the City or Provincial Prosecutor, within five days from the date of its filing and submit to the court proof of such service within the same period. Sec. 18. Memoranda. The court may require the parties and the public prosecutor, in consultation with the Office of the Solicitor General, to file their respective memoranda in support of their claims within fifteen days from the date the trial is terminated. It may require the Office of the Solicitor General to file its own memorandum if the case is of significant interest to the State. No other pleadings or papers may be submitted without leave of court. After the lapse of the period herein provided, the case will be considered submitted for decision, with or without the memoranda. Sec. 19. Decision. (2) The parties, including the Solicitor General and the public prosecutor, shall be served with copies of the decision personally or by registered mail. If the respondent summoned by publication failed to appear in the action, the dispositive part of the decision shall be published once in a newspaper of general circulation. (3) The decision becomes final upon the expiration of fifteen days from notice to the parties. Entry of judgment shall be made if no motion for reconsideration or new trial, or appeal is filed by any of the parties, the public prosecutor, or the Solicitor General. Sec. 20. Appeal. (2) Notice of Appeal. An aggrieved party or the Solicitor General may appeal from the decision by filing a Notice of Appeal within fifteen days from notice of denial of the motion for reconsideration or new trial. The appellant shall serve a copy of the notice of appeal on the adverse parties. Given the foregoing, this Court arrives at a conclusion contrary to those of the RTC and the Court of Appeals, and sustains the validity and existence of the marriage between respondent Crasus and Fely. At most, Felys abandonment, sexual infidelity, and bigamy, give respondent Crasus grounds to file for legal separation under Article 55 of the Family Code of the Philippines, but not for declaration of nullity of marriage under Article 36 of the same Code. While this Court commiserates with respondent Crasus for being continuously shackled to what is now a hopeless and loveless marriage, this is one of those situations where neither law nor society can provide the specific answer to every individual problem. WHEREFORE, the Petition is GRANTED and the assailed Decision of the Court of Appeals in CA-G.R. CV No. 62539, dated 30 July 2001, affirming the Judgment of the RTC of Cebu City, Branch 22, in Civil Case No. CEB-20077, dated 30 October 1998, is REVERSED and SET ASIDE. The marriage of respondent Crasus L. Iyoy and Fely Ada Rosal-Iyoy remains valid and subsisting. SO ORDERED.

470

University of the Cordilleras College of Law First Year C S.Y. 2013 - 2014 Case Digest REPUBLIC vs. CRASUS IYOY G.R. No. 152577. September 21, 2005 470 SCRA 461 CHICO-NAZARIO, J.: Facts: On December 16, 1961 Crasus Iyoy and Ada Rosal-Iyoy got married. They had five children Crasus, Jr., Daphne, Debbie, Calvert, and Carlos. After the celebration of their marriage, Crasus discovered that Fely was "hot-tempered, a nagger and extravagant." In 1984, Fely went to the US and sent letters to Crasus asking him to sign divorce papers which he disregarded. In 1985, Crasus learned, through the letters sent by Fely to their children, that Fely got married to an American, with whom she eventually had a child. Fely went back to the Philippines on several occasions for the wedding of their eldest child, Crasus, Jr. in 1992 and for the brain operation of their fourth child, Calvert. Fely continued to live with her American family in New Jersey. She had been openly using the surname of her American husband. For the wedding of Crasus, Jr., Fely herself had invitations made in which she was named as "Mrs. Fely Ada Micklus." In March 25, 1997, Crasus filed a complaint for declaration of nullity alleging that Felys acts brought danger and dishonor to the family and were manifestations of her psychological incapacity. Crasus submitted his testimony, the certification of the recording of their marriage contract, and the invitation where Fely used her new husbands last name as evidences. Fely denied the claims and asserted that Crasus was a drunkard, womanizer, had no job, and that since 1988 she was already an American citizen and not covered by our laws. Issue: Whether or not abandonment and sexual infidelity per se constitute psychological incapacity Ruling: The Supreme Court arrives at a conclusion contrary to those of the RTC and the Court of Appeals, and sustains the validity and existence of the marriage between respondent Crasus and Fely. At most, Felys abandonment, sexual infidelity, and bigamy, give respondent Crasus grounds to file for legal separation under Article 55 of the Family Code of the Philippines, but not for declaration of nullity of marriage under Article 36 of the same Code. The root cause of psychological incapacity was not proven. There was also improper application of Art. 26 of the Family Code. As it is worded, Article 26, paragraph 2, refers to a special situation wherein one of the couple getting married is a Filipino citizen and the other a foreigner at the time the marriage was celebrated. By its plain and literal interpretation, the said provision cannot be applied to the case of respondent Crasus and his wife Fely because at the time Fely obtained her divorce, she was still a Filipino citizen. Furthermore, Article 36 contemplates downright incapacity or inability to take cognizance of and to assume the basic marital obligations; not a mere refusal, neglect or difficulty, much less, ill will, on the part of the errant spouse. Irreconcilable differences, conflicting personalities, emotional immaturity and irresponsibility, physical abuse, habitual alcoholism, sexual infidelity or perversion, and abandonment, by themselves, also do not warrant a finding of psychological incapacity under the said Article. Article 36 is not to be confused with a divorce law that cuts the marital bond at the time the causes therefore manifest themselves. It refers to a serious psychological illness afflicting a party even before the celebration of marriage. It is a malady so grave and so permanent as to deprive one of awareness of the duties and responsibilities of the matrimonial bond one is about to assume.

471

University of the Cordilleras College of Law First Year C S.Y. 2013 - 2014 Pesca vs Pesca 356 SCRA 588 G.R. No. 136921 April 17, 2001 Full Case LORNA GUILLEN PESCA, petitioner vs. ZOSIMO A PESCA, respondent. VITUG, J.: Submitted for review is the decision of the Court of Appeals, promulgated on 27 May 1998, in C.A. G.R. CV. No. 52374, reversing the decision of the Regional Trial Court ("RTC") of Caloocan City, Branch 130, which has declared the marriage between petitioner and respondent to be null and void ab initio on the ground of psychological incapacity on the part of respondent. Petitioner Lorna G. Pesca and respondent Zosimo A. Pesca first met sometime in 1975 while on board an inter-island vessel bound for Bacolod City. After a whirlwind courtship, they got married on 03 March 1975. Initially, the young couple did not live together as petitioner was still a student in college and respondent, a seaman, had to leave the country on board an ocean-going vessel barely a month after the marriage. Six months later, the young couple established their residence in Quezon City until they were able to build their own house in Caloocan City where they finally resided. It was blissful marriage for the couple during the two months of the year that they could stay together - when respondent was on vacation. The union begot four children, 19-year old Ruhem, 17-year old Rez, 11-year old Ryan, and 9year old Richie. It started in 1988, petitioner said, when she noticed that respondent surprisingly showed signs of "psychological incapacity" to perform his marital covenant. His "true color" of being an emotionally immature and irresponsible husband became apparent. He was cruel and violent. He was a habitual drinker, staying with friends daily from 4:00 o'clock in the afternoon until 1:00 o'clock in the morning. When cautioned to stop or, to at least, minimize his drinking, respondent would beat, slap and kick her. At one time, he chased petitioner with a loaded shotgun and threatened to kill her in the presence of the children. The children themselves were not spared from physical violence. Finally, on 19 November 1992, petitioner and her children left the conjugal abode to live in the house of her sister in Quezon City as they could no longer bear his violent ways. Two months later, petitioner decided to forgive respondent, and she returned home to give him a chance to change. But, to her dismay, things did not so turn out as expected. Indeed, matters became worse. On the morning of 22 March 1994, about eight o'clock, respondent assaulted petitioner for about half an hour in the presence of the children. She was battered black and blue. She submitted herself to medical examination at the Quezon City General Hospital, which diagnosed her injuries as contusions and abrasions. Petitioner filed a complaint with the barangay authorities, and a case was filed against respondent for slight physical injuries. He was convicted by the Metropolitan Trial Court of Caloocan City and sentenced to eleven days of imprisonment. This time, petitioner and her children left the conjugal home for good and stayed with her sister. Eventually, they decided to rent an apartment. Petitioner sued respondent before the Regional Trial Court for the declaration of nullity of their marriage invoking psychological incapacity. Petitioner likewise sought the custody of her minor children and prayed for support pendente lite . Summons, together with a copy of the complaint, was served on respondent on 25 April 1994 by personal service by the sheriff. As respondent failed to file an answer or to enter his appearance within the reglementary period, the trial court ordered the city prosecutor to look into a possible collusion between the parties. Prosecutor Rosa C. Reyes, on 03 August 1994, submitted her report to the effect that she found no evidence to establish that there was collusion between the parties. On 11 January 1995, respondent belatedly filed, without leave of court, an answer, and the same, although filed late, was admitted by the court. In his answer, respondent admitted the fact of his marriage with petitioner and the birth of their children. He also confirmed the veracity of Annex "A" of the complaint 472

University of the Cordilleras College of Law First Year C S.Y. 2013 - 2014 which listed the conjugal property. Respondent vehemently denied, however, the allegation that he was psychologically incapacitated. On 15 November 1995, following hearings conducted by it, the trial court rendered its decision declaring the marriage between petitioner and respondent to be null and void ab initio on the basis of psychological incapacity on the part of respondent and ordered the liquidation of the conjugal partnership. Respondent appealed the above decision to the Court of Appeals, contending that the trial court erred, particularly, in holding that there was legal basis to declare the marriage null and void and in denying his motion to reopen the case. The Court of Appeals reversed the decision of the trial court and declared the marriage between petitioner and respondent valid and subsisting. The appellate court said: "Definitely the appellee has not established the following: That the appellant showed signs of mental incapacity as would cause him to be truly incognitive of the basic marital covenant, as so provided for in Article 68 of the Family Code; that the incapacity is grave, has preceded the marriage and is incurable; that his incapacity to meet his marital responsibility is because of a psychological, not physical illness; that the root cause of the incapacity has been identified medically or clinically, and has been proven by an expert; and that the incapacity is permanent and incurable in nature. "The burden of proof to show the nullity of marriage lies in the plaintiff and any doubt should be resolved in favor of the existence and continuation of the marriage and against its dissolution and nullity." Petitioner, in her plea to this Court, would have the decision of the Court of Appeals reversed on the thesis that the doctrine enunciated in Santos vs. Court of Appeals, promulgated on 14 January 1995, as well as the guidelines set out in Republic vs. Court of Appeals and Molina, promulgated on 13 February 1997, should have no retroactive application and, on the assumption that the Molina ruling could be applied retroactively, the guidelines therein outlined should be taken to be merely advisory and not mandatory in nature. In any case, petitioner argues, the application of the Santos and Molinadicta should warrant only a remand of the case to the trial court for further proceedings and not its dismissal. Be that as it may, respondent submits, the appellate court did not err in its assailed decision for there is absolutely no evidence that has been shown to prove psychological incapacity on his part as the term has been so defined in Santos. Indeed, there is no merit in the petition. The term "psychological incapacity," as a ground for the declaration of nullity of a marriage under Article 36 of the Family Code, has been explained by the Court, in Santos and reiterated in Molina. The Court, in Santos, concluded: "It should be obvious, looking at all the foregoing disquisitions, including, and most importantly, the deliberations of the Family Code Revision Committee itself, that the use of the phrase 'psychological incapacity' under Article 36 of the Code has not been meant to comprehend all such possible cases of psychoses as, likewise mentioned by some ecclesiastical authorities, extremely low intelligence, immaturity, and like circumstances (cited in Fr. Artemio Balumad's 'Void and Voidable Marriages in the Family Code and their Parallels in Canon Law,' quoting form the Diagnostic Statistical Manuel of Mental Disorder by the American Psychiatric Association; Edward Hudson's 'Handbook II for Marriage Nullity Cases'). Article 36 of the Family. Code cannot be taken and construed independently of, but must stand in conjunction with, existing precepts in our law on marriage. Thus correlated, 'psychological incapacity' should refer to no less than a mental (not physical) incapacity that causes a party to be truly incognitive of the basic marital covenants that concomitantly must be assumed and discharged by the parties to the marriage which, as so expressed by Article 68 of the Family Code, include their mutual obligations to live together, observe love, respect and fidelity and render help and support. There is hardly any doubt that the intendment of the law has been to confine the meaning of 'psychological incapacity' to the most serious cases of personality disorders clearly demonstrative

473

University of the Cordilleras College of Law First Year C S.Y. 2013 - 2014 of an utter insensitivity or inability to give meaning and significance to the marriage. This psychologic condition must exist at the time the marriage is celebrated." The- "doctrine of stare decisis," ordained in Article 8 of the Civil Code, expresses that judicial decisions applying or interpreting the law shall form part of the legal system of the Philippines. The rule follows the settled legal maxim - "legis interpretado legis vim obtinet" - that the interpretation placed upon the written law by a competent court has the force of law. The interpretation or construction placed by the courts establishes the contemporaneous legislative intent of the law. The latter as so interpreted and construed would thus constitute a part of that law as of the date the statute is enacted. It is only when a prior ruling of this Court finds itself later overruled, and a different view is adopted, that the new doctrine may have to be applied prospectively in favor of parties who have relied on the old doctrine and have acted in good faith in accordance therewith under the familiar rule of "lex prospicit, non respicit." The phrase "psychological incapacity ," borrowed from Canon law, is an entirely novel provision in our statute books, and, until the relatively recent enactment of the Family Code, the concept has escaped jurisprudential attention. It is in Santos when, for the first time, the Court has given life to the term. Molina, that followed, has additionally provided procedural guidelines to assist the courts and the parties in trying cases for annulment of marriages grounded on psychological incapacity. Molina has strengthened, not overturned, Santos. At all events, petitioner has utterly failed, both in her allegations in the complaint and in her evidence, to make out a case of psychological incapacity on the part of respondent, let alone at the time of solemnization of the contract, so as to warrant a declaration of nullity of the marriage. Emotional immaturity and irresponsibility, invoked by her, cannot be equated with psychological incapacity. The Court reiterates its reminder that marriage is an inviolable social institution and the foundation of the family that the State cherishes and protects. While the Court commisserates with petitioner in her unhappy marital relationship with respondent, totally terminating that relationship, however, may not necessarily be the fitting denouement to it. In these cases, the law has not quite given up, neither should we. WHEREFORE, the herein petition is DENIED. No costs. SO ORDERED.

474

University of the Cordilleras College of Law First Year C S.Y. 2013 - 2014 Case Digest LORNA PESCA vs. ZOSIMO PESCA G.R. No. 136921. April 17, 2001 356 SCRA 588 VITUG, J.: Facts: Lorna G. Pesca and Zosimo A. Pesca got married on March 3, 1975. Initially, the young couple did not live together as petitioner was still a student in college and respondent, a seaman, had to leave the country on board an ocean-going vessel barely a month after the marriage. Six months later, the young couple established their residence in Quezon City until they were able to build their own house in Caloocan City where they finally resided. The union begot four children, 19year old Ruhem, 17-year old Rez, 11-year old Ryan, and 9-year old Richie. In 1988, the petitioner said she noticed that respondent surprisingly showed signs of psychological incapacity. His true colors of emotionally immature and irresponsible husband became apparent. The respondent allegedly would slap, beat and kick her and one time with a loaded shotgun threatened to kill her in the presence of their children. After some other beating, a case was filed against the respondent for slight physical injuries and was sentenced to eleven days of imprisonment. On November 15, 1995, the trial court rendered its decision declaring the marriage between petitioner and respondent to be null and void ab initio on the basis of psychological incapacity on the part of the respondent and ordered a liquidation of conjugal partnership. The respondent appealed the above decision to the Court of Appeals, contending that the trial court erred, and particularly in holding that there was legal basis to declare the marriage null and void. The Court of Appeals reversed the decision of the trial court and declared the marriage between the petitioner and respondent valid and subsisting. Issue: Whether or not the marriage is null and void for the ground of psychological incapacity Ruling: The Supreme Court affirmed the decision of the appellate court thereby dismissing the petition. At all events, petitioner has utterly failed, both in her allegations in the complaint and in her evidence, to make out a case of psychological incapacity on the part of respondent, let alone at the time of solemnization of the contract, so as to warrant a declaration of nullity of the marriage. Emotional immaturity and irresponsibility, invoked by her, cannot be equated with psychological incapacity. The Court reiterates its reminder that marriage is an inviolable social institution and the foundation of the family that the State cherishes and protects. While the Court commiserates with petitioner in her unhappy marital relationship with respondent, totally terminating that relationship, however, may not necessarily be the fitting denouement to it. In these cases, the law has not quite given up, neither should we.

475

University of the Cordilleras College of Law First Year C S.Y. 2013 - 2014 Ferraris vs Ferraris 495 SCRA 396 G.R. No. 162368 July 17, 2006 Full Case MA. ARMIDA PEREZ-FERRARIS, petitioner, vs. BRIX FERRARIS, respondent. YNARES-SANTIAGO, J.: This resolves the motion for reconsideration filed by petitioner Ma. Armida Perez-Ferraris of the Resolution dated June 9, 2004 denying the petition for review on certiorari of the Decision and Resolution of the Court of Appeals dated April 30, 2003 and February 24, 2004, respectively, for failure of the petitioner to sufficiently show that the Court of Appeals committed any reversible error. On February 20, 2001, the Regional Trial Court of Pasig City, Branch 151 rendered a Decision denying the petition for declaration of nullity of petitioner's marriage with Brix Ferraris. The trial court noted that suffering from epilepsy does not amount to psychological incapacity under Article 36 of the Civil Code and the evidence on record were insufficient to prove infidelity. Petitioner's motion for reconsideration was denied in an Order dated April 20, 2001 where the trial court reiterated that there was no evidence that respondent is mentally or physically ill to such an extent that he could not have known the obligations he was assuming, or knowing them, could not have given valid assumption thereof. Petitioner appealed to the Court of Appeals which affirmed in toto the judgment of the trial court. It held that the evidence on record did not convincingly establish that respondent was suffering from psychological incapacity or that his "defects" were incurable and already present at the inception of the marriage. The Court of Appeals also found that Dr. Dayan's testimony failed to establish the substance of respondent's psychological incapacity; that she failed to explain how she arrived at the conclusion that the respondent has a mixed personality disorder; that she failed to clearly demonstrate that there was a natal or supervening disabling factor or an adverse integral element in respondent's character that effectively incapacitated him from accepting and complying with the essential marital obligations. Petitioner's motion for reconsideration was denied for lack of merit; thus, she filed a petition for review on certiorari with this Court. As already stated, the petition for review was denied for failure of petitioner to show that the appellate tribunal committed any reversible error. Petitioner filed the instant motion for reconsideration. The Court required respondent Brix Ferraris to file comment but failed to comply; thus, he is deemed to have waived the opportunity to file comment. Further, the Court directed the Office of the Solicitor General (OSG) to comment on petitioner's motion for reconsideration which it complied on March 2, 2006. After considering the arguments of both the petitioner and the OSG, the Court resolves to deny petitioner's motion for reconsideration. The issue of whether or not psychological incapacity exists in a given case calling for annulment of marriage depends crucially, more than in any field of the law, on the facts of the case. Such factual issue, however, is beyond the province of this Court to review. It is not the function of the Court to analyze or weigh all over again the evidence or premises supportive of such factual determination. It is a wellestablished principle that factual findings of the trial court, when affirmed by the Court of Appeals, are binding on this Court, save for the most compelling and cogent reasons, like when the findings of the appellate court go beyond the issues of the case, run contrary to the admissions of the parties to the case, or fail to notice certain relevant facts which, if properly considered, will justify a different conclusion; or when there is a misappreciation of facts, which are unavailing in the instant case. The term "psychological incapacity" to be a ground for the nullity of marriage under Article 36 of the Family Code, refers to a serious psychological illness afflicting a party even before the celebration of the marriage. It is a malady so grave and so permanent as to deprive one of awareness of the duties and responsibilities of the matrimonial bond one is about to assume. As all people may have certain quirks and idiosyncrasies, or isolated characteristics associated with certain personality disorders, there is hardly any doubt that the intendment of the law has been to confine the meaning of "psychological incapacity" to 476

University of the Cordilleras College of Law First Year C S.Y. 2013 - 2014 the most serious cases of personality disorders clearly demonstrative of an utter insensitivity or inability to give meaning and significance to the marriage. It is for this reason that the Court relies heavily on psychological experts for its understanding of the human personality. However, the root cause must be identified as a psychological illness and its incapacitating nature must be fully explained, which petitioner failed to convincingly demonstrate. As aptly held by the Court of Appeals: Simply put, the chief and basic consideration in the resolution of marital annulment cases is the presence of evidence that can adequately establish respondent's psychological condition. Here, appellant contends that there is such evidence. We do not agree. Indeed, the evidence on record did not convincingly establish that respondent was suffering from psychological incapacity. There is absolutely no showing that his "defects" were already present at the inception of the marriage, or that those are incurable. Quite apart from being plainly self-serving, petitioner's evidence showed that respondent's alleged failure to perform his so-called marital obligations was not at all a manifestation of some deepseated, grave, permanent and incurable psychological malady. To be sure, the couple's relationship before the marriage and even during their brief union (for well about a year or so) was not all bad. During that relatively short period of time, petitioner was happy and contented with her life in the company of respondent. In fact, by petitioner's own reckoning, respondent was a responsible and loving husband. x x x. Their problems began when petitioner started doubting respondent's fidelity. It was only when they started fighting about the calls from women that respondent began to withdraw into his shell and corner, and failed to perform his so-called marital obligations. Respondent could not understand petitioner's lack of trust in him and her constant naggings. He thought her suspicions irrational. Respondent could not relate to her anger, temper and jealousy. x x x. xxxx At any rate, Dr. Dayan did not explain how she arrived at her diagnosis that respondent has a mixed personality disorder called "schizoid," and why he is the "dependent and avoidant type." In fact, Dr. Dayan's statement that one suffering from such mixed personality disorder is dependent on others for decision x x x lacks specificity; it seems to belong to the realm of theoretical speculation. Also, Dr. Dayan's information that respondent had extramarital affairs was supplied by the petitioner herself. Notably, when asked as to the root cause of respondent's alleged psychological incapacity, Dr. Dayan's answer was vague, evasive and inconclusive. She replied that such disorder "can be part of his family upbringing" x x x. She stated that there was a history of respondent's parents having difficulties in their relationship. But this input on the supposed problematic history of respondent's parents also came from petitioner. Nor did Dr. Dayan clearly demonstrate that there was really "a natal or supervening disabling factor" on the part of respondent, or an "adverse integral element" in respondent's character that effectively incapacitated him from accepting, and, thereby complying with, the essential marital obligations. Of course, petitioner likewise failed to prove that respondent's supposed psychological or mental malady existed even before the marriage. All these omissions must be held up against petitioner, for the reason that upon her devolved the onus of establishing nullity of the marriage. Indeed, any doubt should be resolved in favor of the validity of the marriage and the indissolubility of the marital vinculum. We find respondent's alleged mixed personality disorder, the "leaving-the-house" attitude whenever they quarreled, the violent tendencies during epileptic attacks, the sexual infidelity, the abandonment and lack of support, and his preference to spend more time with his band mates than his family, are not rooted on some debilitating psychological condition but a mere refusal or unwillingness to assume the essential obligations of marriage. In Republic v. Court of Appeals,wheretherein respondent preferred to spend more time with his friends than his family on whom he squandered his money, depended on his parents for aid and assistance, and was dishonest to his wife regarding his finances, the Court held that the psychological defects spoken of were more of a "difficulty," if not outright "refusal" or "neglect" in the performance of some marital obligations and that a mere showing of irreconcilable differences and conflicting personalities in no wise constitute psychological incapacity; it is not enough to prove that the parties failed to meet their

477

University of the Cordilleras College of Law First Year C S.Y. 2013 - 2014 responsibilities and duties as married persons; it is essential that they must be shown to be incapable of doing so, due to some psychological, not physical, illness. Also, we held in Hernandez v. Court of Appeals that habitual alcoholism, sexual infidelity or perversion, and abandonment do not by themselves constitute grounds for declaring a marriage void based on psychological incapacity. While petitioner's marriage with the respondent failed and appears to be without hope of reconciliation, the remedy however is not always to have it declared void ab initio on the ground of psychological incapacity. An unsatisfactory marriage, however, is not a null and void marriage. No less than the Constitution recognizes the sanctity of marriage and the unity of the family; it decrees marriage as legally "inviolable" and protects it from dissolution at the whim of the parties. Both the family and marriage are to be "protected" by the state. Thus, in determining the import of "psychological incapacity" under Article 36, it must be read in conjunction with, although to be taken as distinct from Articles 35, 37, 38, and 41 that would likewise, but for different reasons, render the marriage void ab initio, or Article 45 that would make the marriage merely voidable, or Article 55 that could justify a petition for legal separation. Care must be observed so that these various circumstances are not applied so indiscriminately as if the law were indifferent on the matter. Article 36 should not to be confused with a divorce law that cuts the marital bond at the time the causes therefor manifest themselves. Neither it is to be equated with legal separation, in which the grounds need not be rooted in psychological incapacity but on physical violence, moral pressure, moral corruption, civil interdiction, drug addiction, habitual alcoholism, sexual infidelity, abandonment and the like. WHEREFORE, in view of the foregoing, the motion for reconsideration of the Resolution dated June 9, 2004 denying the petition for review on certiorari for failure of the petitioner to sufficiently show that the Court of Appeals committed any reversible error, is DENIED WITH FINALITY. SO ORDERED.

Case Digest
MA. ARMIDA-FERRARIS vs. BRIX FERRARIS G.R. No. 162368, July 17, 2006 495 SCRA 396 YNARES-SANTIAGO, J.: Facts: The Regiona Trial Court denied the petition for declaration of nullity of marriage between petitioner Ma. Armida-Ferraris with Brix Ferraris. It found that his "violence" during episodes of epilepsy did not constitute psychological incapacity. The Court of Appeals affirmed the decision and cited that the evidence on record did not convincingly establish that respondent was suffering from psychological incapacity or that his "defects" were incurable and already present at the inception of the marriage. The Court of Appeals also found that Dr. Dayan's testimony failed to establish the substance of respondent's psychological incapacity; that she failed to explain how she arrived at the conclusion that the respondent has a mixed personality disorder; that she failed to clearly demonstrate that there was a natal or supervening disabling factor or an adverse integral element in respondent's character that effectively incapacitated him from accepting and complying with the essential marital obligations. The petitioner then filed a motion to the Supreme Court. Issue: Whether or not epilepsy constitute psychological incapacity Ruling: The Supreme Court denied the petition with finality. The Supreme Court found respondent's alleged mixed personality disorder, the "leaving-the-house" attitude whenever they quarreled, the violent tendencies during epileptic attacks, the sexual infidelity, the abandonment and lack of support, and his preference to spend more time with his band mates than his family, are not rooted on some debilitating psychological condition but a mere refusal or unwillingness to 478

University of the Cordilleras College of Law First Year C S.Y. 2013 - 2014 assume the essential obligations of marriage. In determining the import of "psychologicalincapacity" under Article 36, it must be read in conjunction with, although to be taken as distinct from Articles 35, 37, 38, and 41 that would likewise, but for different reasons, render the marriage void ab initio, or Article 45 that would make the marriage merely voidable, or Article 55 that could justify a petition for legal separation. Care must be observed so that these various circumstances are not applied so indiscriminately as if the law were indifferent on the matter. Article 36 should not to be confused with a divorce law that cuts the marital bond at the time the causes therefor manifest themselves. Neither it is to be equated with legal separation, in which the grounds need not be rooted in psychological incapacity but on physical violence, moral pressure, moral corruption, civil interdiction, drug addiction, habitual alcoholism, sexual infidelity, abandonment and the like.

479

University of the Cordilleras College of Law First Year C S.Y. 2013 - 2014 Dedel vs Court of Appeals 421 SCRA 461 G.R. No. 151867 January 29, 2004 Full Case DAVID B. DEDEL, Petitioner, vs. COURT OF APPEALS and SHARON L. CORPUZ-DEDEL a.k.a. JANE IBRAHIM, Respondents. REPUBLIC OF THE PHILIPPINES, Oppositor-Respondent. YNARES-SANTIAGO, J.: Petitioner David B. Dedel met respondent Sharon L. Corpuz Dedel while he was working in the advertising business of his father. The acquaintance led to courtship and romantic relations, culminating in the exchange of marital vows before the City Court of Pasay on September 28, 1966. The civil marriage was ratified in a church wedding on May 20, 1967. The union produced four children, namely: Beverly Jane, born on September 18, 1968; Stephanie Janice born on September 9, 1969; Kenneth David born on April 24, 1971; and Ingrid born on October 20, 1976. The conjugal partnership, nonetheless, acquired neither property nor debt. Petitioner avers that during the marriage, Sharon turned out to be an irresponsible and immature wife and mother. She had extra-marital affairs with several men: a dentist in the Armed Forces of the Philippines; a Lieutenant in the Presidential Security Command and later a Jordanian national. Sharon was once confirmed in the Manila Medical City for treatment by Dr. Lourdes Lapuz, a clinical psychiatrist. Petitioner alleged that despite the treatment, Sharon did not stop her illicit relationship with the Jordanian national named Mustafa Ibrahim, whom she married and with whom she had two children. However, when Mustafa Ibrahim left the country, Sharon returned to petitioner bringing along her two children by Ibrahim. Petitioner accepted her back and even considered the two illegitimate children as his own. Thereafter, on December 9, 1995, Sharon abandoned petitioner to join Ibrahim in Jordan with their two children. Since then, Sharon would only return to the country on special occasions. Finally, giving up all hope of a reconciliation with Sharon, petitioner filed on April 1, 1997 a petition seeking the declaration of nullity of his marriage on the ground of psychological incapacity, as defined in Article 36 of the Family Code, before the Regional Trial Court of Makati City, Branch 149. Summons was effected by publication in the Pilipino Star Ngayon, a newspaper of general circulation in the country considering that Sharon did not reside and could not be found in the Philippines. Petitioner presented Dr. Natividad A. Dayan, who testified that she conducted a psychological evaluation of petitioner and found him to be conscientious, hardworking, diligent, a perfectionist who wants all tasks and projects completed up to the final detail and who exerts his best in whatever he does. On the other hand, Dr. Dayan declared that Sharon was suffering from Anti-Social Personality Disorder exhibited by her blatant display of infidelity; that she committed several indiscretions and had no capacity for remorse, even bringing with her the two children of Mustafa Ibrahim to live with petitioner. Such immaturity and irresponsibility in handling the marriage like her repeated acts of infidelity and abandonment of her family are indications of Anti-Social Personality Disorder amounting to psychological incapacity to perform the essential obligations of marriage. After trial, judgment was rendered, the dispositive portion of which reads: WHEREFORE, in the light of the foregoing, the civil and church marriages between DAVID B. DEDEL and SHARON L. CORPUZ celebrated on September 28, 1966 and May 20, 1967 are hereby declared null and void on the ground of psychological incapacity on the part of the respondent to perform the essential obligations of marriage under Article 36 of the Family Code.

480

University of the Cordilleras College of Law First Year C S.Y. 2013 - 2014 Accordingly, the conjugal partnership of gains existing between the parties is dissolved and in lieu thereof a regime of complete separation of property between the said spouses is established in accordance with the pertinent provisions of the Family Code, without prejudice to rights previously acquired by creditors. Let a copy of this Decision be duly recorded in the proper civil and property registries in accordance with Article 52 of the Family Code. SO ORDERED. Respondent Republic of the Philippines, through the Solicitor General, appealed alleging that I THE LOWER COURT ERRED IN GRANTING THE PETITION DESPITE THE ABSENCE OF A VALID GROUND FOR DECLARATION OF NULLITY OF MARRIAGE. II THE LOWER COURT ERRED IN DECLARING THAT THE CHURCH MARRIAGE BETWEEN PETITIONER IS NULL AND VOID. III THE LOWER COURT ERRED IN RENDERING A DECISION WITHOUT A CERTIFICATION HAVING BEEN ISSUED BY THE SOLICITOR GENERAL AS REQUIRED IN THE MOLINA CASE. The Court of Appeals recalled and set aside the judgment of the trial court and ordered dismissal of the petition for declaration of nullity of marriage. Petitioners motion for reconsideration was denied in a Resolution dated January 8, 2002. Hence, the instant petition. Petitioner contends that the appellate court gravely abused its discretion and manifestly erred in its conclusion that the: (1) respondent was not suffering from psychological incapacity to perform her marital obligations; (2) psychological incapacity of respondent is not attended by gravity, juridical antecedence and permanence or incurability; and (3) totality of evidence submitted by the petitioner falls short to prove psychological incapacity suffered by respondent. The main question for resolution is whether or not the totality of the evidence presented is enough to sustain a finding that respondent is psychologically incapacitated. More specifically, does the aberrant sexual behavior of respondent adverted to by petitioner fall within the term "psychological incapacity?" In Santos v. Court of Appeals, it was ruled: x x x "psychological incapacity" should refer to no less than a mental (not physical) incapacity that causes a party to be truly incognitive of the basic marital covenants that concomitantly must be assumed and discharged by the parties to the marriage which, as so expressed in Article 68 of the Family Code, include their mutual obligations to live together, observe love, respect and fidelity and render help and support. There is hardly any doubt that the intendment of the law has been to confine the meaning of "psychological incapacity" to the most serious cases of personality disorders clearly demonstrative of an utter insensitivity of inability to give meaning and significance to the marriage. This psychological condition must exist at the time the marriage is celebrated. The law does not evidently envision, upon the other hand, an inability of the spouse to have sexual relations with the other. This conclusion is implicit under Article 54 of the Family Code which considers children conceived prior to the judicial declaration of nullity of the void marriage to be "legitimate." The other forms of psychoses, if existing at the inception of marriage, like the state of a party being of unsound mind or concealment of drug addiction, habitual alcoholism, homosexuality or lesbianism, merely renders the marriage contract voidable pursuant to Article 46, Family Code. If drug addiction, habitual alcoholism, lesbianism or homosexuality should occur only during the marriage, they become mere grounds for legal separation under Article 55 of the Family Code. These provisions, however, do not 481

University of the Cordilleras College of Law First Year C S.Y. 2013 - 2014 necessarily preclude the possibility of these various circumstances being themselves, depending on the degree and severity of the disorder, indicia of psychological incapacity. Until further statutory and jurisprudential parameters are established, every circumstance that may have some bearing on the degree, extent and other conditions of that incapacity must, in every case, be carefully examined and evaluated so that no precipitate and indiscriminate nullity is peremptorily decreed. The well-considered opinion of psychiatrists, psychologists and persons with expertise in psychological disciplines might be helpful or even desirable. The difficulty in resolving the problem lies in the fact that a personality disorder is a very complex and elusive phenomenon which defies easy analysis and definition. In this case, respondents sexual infidelity can hardly qualify as being mentally or psychically ill to such an extent that she could not have known the obligations she was assuming, or knowing them, could not have given a valid assumption thereof. It appears that respondents promiscuity did not exist prior to or at the inception of the marriage. What is, in fact, disclosed by the records is a blissful marital union at its celebration, later affirmed in church rites, and which produced four children. Respondents sexual infidelity or perversion and abandonment do not by themselves constitute psychological incapacity within the contemplation of the Family Code. Neither could her emotional immaturity and irresponsibility be equated with psychological incapacity. It must be shown that these acts are manifestations of a disordered personality which make respondent completely unable to discharge the essential obligations of the marital state, not merely due to her youth, immaturity or sexual promiscuity. At best, the circumstances relied upon by petitioner are grounds for legal separation under Article 55 of the Family Code. However, we pointed out in Marcos v. Marcos that Article 36 is not to be equated with legal separation in which the grounds need not be rooted in psychological incapacity but on physical violence, moral pressure, civil interdiction, drug addiction, habitual alcoholism, sexual infidelity, abandonment and the like. In short, the evidence presented by petitioner refers only to grounds for legal separation, not for declaring a marriage void. We likewise agree with the Court of Appeals that the trial court has no jurisdiction to dissolve the church marriage of petitioner and respondent. The authority to do so is exclusively lodged with the Ecclesiastical Court of the Roman Catholic Church. All told, we find no cogent reason to disturb the ruling of the appellate court.1wphi1 We cannot deny the grief, frustration and even desperation of petitioner in his present situation. Regrettably, there are circumstances, like in this case, where neither law nor society can provide the specific answers to every individual problem. While we sympathize with petitioners marital predicament, our first and foremost duty is to apply the law no matter how harsh it may be. WHEREFORE, in view of the foregoing, the petition is DENIED. The decision of the Court of Appeals in CA-G.R. CV No. 60406, which ordered the dismissal of Civil Case No. 97-467 before the Regional Trial Court of Makati, Branch 149, is AFFIRMED. No costs. SO ORDERED.

482

University of the Cordilleras College of Law First Year C S.Y. 2013 - 2014 Case Digest DAVID DEDEL vs. COURT OF APPEALS G.R. No. 151867. January 29, 2004 421 SCRA 461 YNARES-SANTIAGO, J.: Facts: David B. Dedel married Sharon L. Corpuz Dedel before the City Court of Pasay on September 28, 1966. The civil marriage was ratified in a church wedding on May 20, 1967. Petitioner avers that during the marriage, Sharon turned out to be an irresponsible and immature wife and mother. She had extra-marital affairs with several men: a dentist in the Armed Forces of the Philippines; a Lieutenant in the Presidential Security Command and later a Jordanian national. Sharon was once confirmed in the Manila Medical City for treatment by Dr. Lourdes Lapuz, a clinical psychiatrist. Petitioner alleged that despite the treatment, Sharon did not stop her illicit relationship with the Jordanian national named Mustafa Ibrahim, whom she married and with whom she had two children. However, when Mustafa Ibrahim left the country, Sharon returned to petitioner bringing along her two children by Ibrahim. Petitioner accepted her back and even considered the two illegitimate children as his own. On December 9, 1995, Sharon abandoned petitioner to join Ibrahim in Jordan with their two children. The petitioner then filed a motion for the declaration of nullity of the marriage based on psychological incapacity. The petitioner presented Dr. Dayan Natividad who alleged that Sharon suffered from Anti-Social disorder exhibited by her blatant display of infidelity and abandonment. These characteristics render her unable to perform essential marital obligations. The Regional Trial Court declared the marriage as null and void. The Office of the Solicitor General appealed and the Court of Appeals reversed the earlier ruling. The petitioner then filed a motion to the Supreme Court alleging that there was error in the judgment of the Court of Appeals. Issue: Whether or not aberrant sexual behavior of respondent adverted to by petitioner fall within the term psychological incapacity Ruling: The Supreme Court dismissed the petition and affirmed the ruling the ruling of the Court of Appeals. In this case, respondents sexual infidelity can hardly qualify as being mentally or psychically ill to such an extent that she could not have known the obligations she was assuming, or knowing them, could not have given a valid assumption thereof. It appears that respondents promiscuity did not exist prior to or at the inception of the marriage. What is, in fact, disclosed by the records is a blissful marital union at its celebration, later affirmed in church rites, and which produced four children. Respondents sexual infidelity or perversion and abandonment do not by themselves constitute psychological incapacity within the contemplation of the Family Code. Neither could her emotional immaturity and irresponsibility be equated with psychological incapacity.

483

University of the Cordilleras College of Law First Year C S.Y. 2013 - 2014 Tongol vs Tongol 537 SCRA 135 G.R. No. 157610 October 19, 2007 Full Case ORLANDO G. TONGOL, Petitioner, vs. FILIPINAS M. TONGOL, Respondent. AUSTRIA-MARTINEZ, J.: Assailed in the present Petition for Review on Certiorari under Rule 45 of the Rules of Court is the Decision of the Court of Appeals (CA) dated September 25, 2002 in CA-G.R. CV No. 66245, and its Resolution of March 19, 2003, denying petitioner's motion for reconsideration. The CA Decision affirmed, in toto, the Decision of the Regional Trial Court (RTC) of Makati City, Branch 149, which dismissed the petition for declaration of nullity of marriage filed by herein petitioner Orlando Tongol. The facts of the case are as follows: Orlando G. Tongol (Orlando) and Filipinas M. Tongol (Filipinas) were married on August 27, 1967. Out of their union, they begot four children, namely: Crisanto, born in 1968; Olivia, born in 1969; Frederick, born in 1971, and; Ma. Cecilia, born in 1972. On May 13, 1994, Orlando and Filipinas filed a petition for dissolution of their conjugal partnership of gains, which was granted in a Judgment issued by the RTC of Makati City, Branch 143 on April 24, 1995. On August 19, 1996, Orlando filed before the RTC of Makati City a verified petition for the declaration of nullity of his marriage with Filipinas on the ground that the latter is psychologically incapacitated to comply with her essential marital obligations. In his Petition, Orlando contended that he and Filipinas got married over the objection of the latter's family; their marriage was not a happy one because of her parents' continued interference and attempt to break up their union; greatly influenced by her parents, Filipinas, even at the early stages of their marriage, already treated Orlando with contempt and without the love and respect due him as her husband; when Orlando started a junk shop business, Filipinas ridiculed him instead of giving him encouragement; later on, his business became successful and he was able to embark upon another business venture; he put up a pharmaceutical company which also became profitable; Filipinas then became interested and began to interfere in the operation of the business; however, because of her bad attitude, the employees were aloof; she also resented the fact that her husband got along well with the employees; as a result, she quarreled with her husband causing the latter embarrassment; she even suspected that the income of the business was being given to her husband's relatives; their continued fighting persisted and affected their children; efforts at reconciliation proved futile because their differences had become irreconcilable and their marriage impossible; in 1990, Orlando decided to live separately from Filipinas; in 1994, the spouses filed a petition for dissolution of their property relationship; and the petition was granted in 1995. In her Answer with Counter-Petition, Filipinas admitted that efforts at reconciliation have been fruitless and that their marriage is a failure. However, she claims that their marriage failed because it is Orlando who is psychologically incapacitated to fulfill his obligations as a married man. Evidence for Orlando consisted of his own testimony, that of his sister, Angelina Tongol, and of Annaliza Guevara, an employee in the pharmaceutical company owned by the spouses Tongol. Orlando also presented Dr. Cecilia Villegas, a psychiatrist who conducted a psychological examination of both parties. Orlando submitted documents evidencing their marriage, the birth of their four children, the RTC decision granting the petition for dissolution of their conjugal partnership of gains, and the written evaluation of Dr. Villegas regarding the spouses' psychological examination. On the other hand, record shows that evidence for Filipinas only consisted of her own testimony. On June 30, 1999, the RTC of Makati City, Branch 149, rendered a Decision dismissing the petition.

484

University of the Cordilleras College of Law First Year C S.Y. 2013 - 2014 On appeal, the CA affirmed, in toto, the Decision of the RTC. Hence, herein petition raising the following issues: 1. "WHETHER OR NOT THE EVIDENCE SUPPORTS THE FINDINGS OF THE TRIAL COURT AND THE HONORABLE COURT OF APPEALS THAT DRA. CECILIA VILLEGAS FAILED TO STATE WHETHER OR NOT RESPONDENT'S INADEQUATE PERSONALITY DISORDER WAS GRAVE, PERMANENT AND INCURABLE" (par. 12, p. 3, Annex "A", hereof). 2. "WITH ALL DUE RESPECT, THE COURT OF APPEALS ERRED IN DISMISSING THE APPEAL" (p. 7, ibid.). 3. "WITH ALL DUE RESPECT, THE COURT OF APPEALS ERRED IN DENYING THE MOTION FOR RECONSIDERATION" (Annex "B", hereof). The basic issue to be resolved in the instant case is whether or not the totality of the evidence presented in the present case is enough to sustain a finding that herein respondent is psychologically incapacitated to comply with her essential marital obligations. In Santos v. Court of Appeals, the term psychological incapacity was defined as: [N]o less than a mental (not physical) incapacity that causes a party to be truly incognitive of the basic marital covenants that concomitantly must be assumed and discharged by the parties to the marriage which, as so expressed by Article 68 of the Family Code, include their mutual obligations to live together, observe love, respect and fidelity and render help and support. There is hardly any doubt that the intendment of the law has been to confine the meaning of "psychological incapacity" to the most serious cases of personality disorders clearly demonstrative of an utter insensitivity or inability to give meaning and significance to the marriage. This psychologic condition must exist at the time the marriage is celebrated. x x x Psychological incapacity must be characterized by: (a) Gravity It must be grave or serious such that the party would be incapable of carrying out the ordinary duties required in a marriage; (b) Juridical Antecedence It must be rooted in the history of the party antedating the marriage, although the overt manifestations may emerge only after the marriage; and (c) Incurability It must be incurable or, even if it were otherwise, the cure would be beyond the means of the party involved. While the CA has already extensively quoted the ruling in Republic of the Philippines v. Court of Appeals and Molina, wherein the guidelines in the interpretation and application of Article 36 of the Family Code was laid down, this Court finds it significant to reproduce the same quoted portion, to wit: (1) The burden of proof to show the nullity of the marriage belongs to the plaintiff. Any doubt should be resolved in favor of the existence and continuation of the marriage and against its dissolution and nullity. This is rooted in the fact that both our Constitution and our laws cherish the validity of marriage and unity of the family. Thus, our Constitution devotes an entire Article on the Family, recognizing it "as the foundation of the nation." It decrees marriage as legally "inviolable," thereby protecting it from dissolution at the whim of the parties. Both the family and marriage are to be "protected" by the state. The Family Code echoes this constitutional edict on marriage and the family and emphasizes their permanence, inviolability and solidarity. (2) The root cause of the psychological incapacity must be (a) medically or clinically identified, (b) alleged in the complaint, (c) sufficiently proven by experts and (d) clearly explained in the decision. Article 36 of the Family Code requires that the incapacity must be psychological - not 485

University of the Cordilleras College of Law First Year C S.Y. 2013 - 2014 physical, although its manifestations and/or symptoms may be physical. The evidence must convince the court that the parties, or one of them, was mentally or psychically ill to such an extent that the person could not have known the obligations he was assuming, or knowing them, could not have given valid assumption thereof. Although no example of such incapacity need be given here so as not to limit the application of the provision under the principle of ejusdem generis, nevertheless such root cause must be identified as a psychological illness and its incapacitating nature fully explained. Expert evidence may be given by qualified psychiatrists and clinical psychologists. (3) The incapacity must be proven to be existing at "the time of the celebration" of the marriage. The evidence must show that the illness was existing when the parties exchanged their "I do's." The manifestation of the illness need not be perceivable at such time, but the illness itself must have attached at such moment, or prior thereto. (4) Such incapacity must also be shown to be medically or clinically permanent or incurable. Such incurability may be absolute or even relative only in regard to the other spouse, not necessarily absolutely against everyone of the same sex. Furthermore, such incapacity must be relevant to the assumption of marriage obligations, not necessarily to those not related to marriage, like the exercise of a profession or employment in a job. x x x (5) Such illness must be grave enough to bring about the disability of the party to assume the essential obligations of marriage. Thus, "mild characteriological peculiarities, mood changes, occasional emotional outbursts" cannot be accepted as root causes. The illness must be shown as downright incapacity or inability, not a refusal, neglect or difficulty, much less ill will. In other words, there is a natal or supervening disabling factor in the person, an adverse integral element in the personality structure that effectively incapacitates the person from really accepting and thereby complying with the obligations essential to marriage. (6) The essential marital obligations must be those embraced by Articles 68 up to 71 of the Family Code as regards the husband and wife as well as Articles 220, 221 and 225 of the same Code in regard to parents and their children. Such non-complied marital obligation(s) must also be stated in the petition, proven by evidence and included in the text of the decision. (7) Interpretations given by the National Appellate Matrimonial Tribunal of the Catholic Church in the Philippines, while not controlling or decisive, should be given great respect by our courts. x xx (8) The trial court must order the prosecuting attorney or fiscal and the Solicitor General to appear as counsel for the state. No decision shall be handed down unless the Solicitor General issues a certification, which will be quoted in the decision, briefly stating therein his reasons for his agreement or opposition, as the case may be, to the petition. The Solicitor General, along with the prosecuting attorney, shall submit to the court such certification within fifteen (15) days from the date the case is deemed submitted for resolution of the court. The Solicitor General shall discharge the equivalent function of the defensor vinculi contemplated under Canon 1095. Under the Rule on Declaration of Absolute Nullity of Void Marriages and Annulment of Voidable Marriages, which took effect on March 15, 2003, the foregoing guidelines have been modified. Section 2(d) of the said Rule provides: SEC. 2. Petition for declaration of absolute nullity of void marriages.xxxx (d) What to allege.- A petition under Article 36 of the Family Code shall specifically allege the complete facts showing that either or both parties were psychologically incapacitated from complying with the essential marital obligations of marriage at the time of the celebration of marriage even if such incapacity becomes manifest only after its celebration. The complete facts should allege the physical manifestations, if any, as are indicative of psychological incapacity at the time of the celebration of the marriage but expert opinion need not be alleged. 486

University of the Cordilleras College of Law First Year C S.Y. 2013 - 2014 The new Rule dispensed with the certification from the Solicitor General, stating therein his reasons for his agreement or opposition to the petition. Attachment of expert opinions to the petition is also dispensed with. In the instant case, the RTC and the CA gave credence to the conclusion of the examining psychiatrist, Dr. Villegas, that respondent is suffering from Inadequate Personality Disorder. However, both courts ruled that the behavior exhibited by respondent does not amount to psychological incapacity as contemplated under Article 36 of the Family Code. This Court finds no cogent reason to depart from the assessment of the RTC and the CA for the following reasons: First, petitioner relies heavily on the findings of Dr. Villegas who made the following written evaluation regarding respondent's psychological makeup: xxxx On the other hand, Mrs. Filipinas Mendoza-Tongol belonged to a matriarchal family where the mother assumed a more active and dominant role. She was left to the care of her aunt and developed a basic feeling a (sic) rejection. The only college graduate among 7 children her operating intellectual ability is low-average. Sudden change overwhelmed her. When seized by an impulse, she is likely to give way, even minor pressures upset her and when this happens, emotional control could not be relied upon. In marriage when her husband shows good relationship with their employees, especially with females, she became (sic) suspicious, jealous, and threatened, and this is related to her basic feelings of rejection in early life. She coped (sic) up with her uncomfortable feelings by exhibiting temper tantrums, irritability and dominance, a replica of her mother's attitude, but to the distaste of her husband. At present she is depressed, though hostile, and now living in the expectation of further rejection. Additionally, she is threatened by a neurological illness (tremor of the hands) for which she is consulting a neurologist. Based on the above findings, it is the opinion of the undersigned that Mr. Orlando Tongol is suffering from some depressive features, which seems to be a recent development as a result of marital problems. On the other hand, Mrs. Tongol is suffering from an Inadequate Personality Disorder, with hysterical coloring, which renders her psychologically incapacitated to perform the duties and responsibilities of marriage. She is unable to cope with the sudden work and environmental shifts, that overwhelmed her, due to insufficient psychological inner resources. In her testimony, Dr. Villegas explained respondent's personality disorder in this wise: ATTY. VILLAREAL xxxx Q- What exactly do you mean [by] inadequate personality disorder? A- Inadequate personality disorder means, there are not times that in all aspects of her life, she could not function in the way that she feels or she is confident. She has always been very much in doubt of her own capabilities, Sir. Q- What about hysterical coloring? A- Hysterical coloring means, there is always an exaggeration of her psychological reactions to any stresses, Sir. Q- Exaggeration in what aspect?

487

University of the Cordilleras College of Law First Year C S.Y. 2013 - 2014 A- Exaggeration in any emotional reactions or situations like if she would be seeing the husband talking to some employees then, she is suddenly irritable and would present some tantrums. In short, she cannot control her emotion at the moment of stresses circulations, Sir. When asked how such personality disorder affects respondent's capacity to assume the essential obligations of marriage, Dr. Villegas expounded as follows: ATTY. RENDOR xxxx Q- How about Mrs. Tongol, what are your findings? A- Mrs. Tongol is a college graduate and she finished commerce. Basically, she has a feeling of rejection from the start of her development and this was carried on into her adult life. When the husband started having some good relationship with his employees, then she started to get jealous and she would embarrass him in front of their employees and insulted him and would go into tantrums and this was very much resented by Mr. Tongol, Sir. ATTY. RENDOR Q- In your expert opinion, Doctor, can you tell us the reason why Mrs. Tongol acted in such a way? A- Because of her basic rejection at that time, Sir. She was afraid that Mr. Tongol was already rejecting her as a wife and being attracted to other people, but it is the way of how Mrs. Tongol reacted to her own feelings of rejection, Sir. xxxx Q- What made you say that because of inadequate personality disorder, Mrs. Tongol rendered her psychological (sic) incapacitated to perform the duties and responsibilities of the marriage. What is your basis in saying that? A- She belongs to a very matriarchal family. The mother was very dominant. She always gets what she wanted in the house. In short, she was the authority in the house and during her growing up stage, she was given up to the aunt, for the aunt to take care of her. She only came back to the family when she was already a sort of an early teenager. With this, there has always been a feeling of rejection during her personality development. Besides, she feels that she is one of those not favor (sic) by the mother during her growing up stage, Sir. Q- Based on your examination of the spouses, what do you recommend as far as the marriage is concerned, considering that this is a petition for the annulment of marriage? A- I could recommend that they have their marriage annulled because it will only be sufferings from (sic) both of them because on the part of Mrs. Tongol, it is one that is more or less permanent and Mr. Tongol is also suffering from some depression, Sir. The Court can only gather from the foregoing explanations of Dr. Villegas that as a child, Filipinas had always felt rejected, especially by her mother; that she never got rid of those feelings of rejection even when she became an adult and got married; that her fits of jealousy and temper tantrums, every time she sees her husband having a good interaction with their employees, are ways of coping up with her feelings of rejection. However, Dr. Villegas failed to link respondent's personality disorder to her conclusion that respondent is psychologically incapacitated to perform her obligations as wife and mother. The Court cannot see how respondent's personality disorder which, according to Dr. Villegas, is inextricably linked to her feelings of rejection, would render her unaware of the essential marital obligations, or to borrow the terms used in Santos, "to be truly incognitive of the basic marital covenants that concomitantly must be assumed and discharged by the parties to the marriage." What has been established in the instant case is that, by reason of her feelings of inadequacy and rejection, respondent not only encounters a lot of difficulty but even refuses to assume some of her obligations towards her husband, such as respect, help

488

University of the Cordilleras College of Law First Year C S.Y. 2013 - 2014 and support for him. However, this Court has ruled that psychological incapacity must be more than just a "difficulty," a "refusal" or a "neglect" in the performance of some marital obligations. As held in Santos: There is hardly any doubt that the intendment of the law has been to confine the meaning of "psychological incapacity" to the most serious cases of personality disorders clearly demonstrative of an utter insensitivity or inability to give meaning and significance to the marriage. This psychologic condition must exist at the time the marriage is celebrated. Second, Dr. Villegas also failed to fully and satisfactorily explain if the personality disorder of respondent is grave enough to bring about her disability to assume the essential obligations of marriage. Petitioner contends that respondent's exaggerated reactions to normal situations, her unreasonable feelings of rejection brought about by her dysfunctional upbringing, are all indications of the gravity of her psychological condition. Even granting that respondent's psychological disorder is serious, the fact remains that there is no evidence to prove that such condition is of such nature as to render respondent incapable of carrying out the ordinary duties required in marriage. Third, there is no evidence that such incapacity is incurable. Neither in her written evaluation nor in her testimony did Dr. Villegas categorically and conclusively characterize respondent's inadequate personality disorder as permanent or incurable. Dr. Villegas was not sure of the permanence or incurability of respondent's illness as shown by her following statement: I could recommend that they have their marriage annulled because it will only be sufferings from (sic) both of them because on the part of Mrs. Tongol, it is one that is more or less permanent and Mr. Tongol is also suffering from some depression, Sir. (Emphasis supplied) Fourth, the psychological incapacity considered under Article 36 of the Family Code is not meant to comprehend all possible cases of psychoses. The fourth guideline in Molina requires that the psychological incapacity as understood under Article 36 of the Family Code must be relevant to the assumption of marriage obligations, not necessarily to those not related to marriage, like the exercise of a profession or employment in a job. In the present case, the testimonies of both petitioner and respondent as well as the other witnesses regarding the spouses' differences and misunderstanding basically revolve around and are limited to their disagreement regarding the management of their business. In fact, respondent herself, in her Memorandum submitted to the trial court, claimed that their quarrels arose solely from their disagreement on how to run their business. This is confirmed by the testimony of petitioner's sister who lived with the spouses for a considerable period of time. However, a mere showing of irreconcilable differences and conflicting personalities in no wise constitutes psychological incapacity. In addition, it is true that the marital obligations of a husband and wife enumerated under the Family Code include the mutual responsibility of the spouses to manage the household and provide support for the family, which means that compliance with this obligation necessarily entails the management of the income and expenses of the household. While disagreements on money matters would, no doubt, affect the other aspects of one's marriage as to make the wedlock unsatisfactory, this is not a sufficient ground to declare a marriage null and void. In the present case, respondent's disagreement with her husband's handling of the family's business and finances and her propensity to start a fight with petitioner spouse regarding these matters can hardly be considered as a manifestation of the kind of psychological incapacity contemplated under Article 36 of the Family Code. In fact, the Court takes judicial notice of the fact that disagreements regarding money matters is a common, and even normal, occurrence between husbands and wives. Fifth, marital obligation includes not only a spouse's obligation to the other spouse but also one's obligation toward their children. In the present case, no evidence was presented to show that respondent had been remiss in performing her obligations toward their children as enumerated in Article 220 of the Family Code. It is settled that Article 36 of the Family Code is not to be confused with a divorce law that cuts the marital bond at the time the causes therefor manifest themselves. It refers to a serious psychological illness afflicting a party even before the celebration of marriage. It is a malady so grave and so permanent as to deprive one of awareness of the duties and responsibilities of the matrimonial bond one is about to assume. In the instant case, the Court finds no error in the findings of the RTC, as affirmed by the CA, that the aversive behavior of petitioner and respondent towards each other is a mere indication of

489

University of the Cordilleras College of Law First Year C S.Y. 2013 - 2014 incompatibility brought about by their different family backgrounds as well as their attitudes, which developed after their marriage. In sum, it is not disputed that respondent is suffering from a psychological disorder.1wphi1 However, the totality of the evidence presented in the present case does not show that her personality disorder is of the kind contemplated by Article 36 of the Family Code as well as jurisprudence as to render her psychologically incapacitated or incapable of complying with the essential obligations of marriage. It remains settled that the State has a high stake in the preservation of marriage rooted in its recognition of the sanctity of married life and its mission to protect and strengthen the family as a basic autonomous social institution. Hence, any doubt should be resolved in favor of the existence and continuation of the marriage and against its dissolution and nullity. WHEREFORE, the petition is DENIED. The September 25, 2002 Decision and March 19, 2003 Resolution of the Court of Appeals in CA-G.R. CV No. 66245 are AFFIRMED. SO ORDERED.

490

University of the Cordilleras College of Law First Year C S.Y. 2013 - 2014 Case Digest ORLANDO TONGOL vs. FILIPINAS TONGOL G.R. No. 157610 October 19, 2007 537 SCRA 135 AUSTRIA-MARTINEZ, J.: Facts: Orlando G. Tongol and Filipinas M. Tongol were married on August 27, 1967. They had four children: Crisanto, Olivia, Frederick, and Ma. Cecilia. On August 19, 1996, Orlando filed a petition for the declaration of nullity of his marriage with Filipinas on the ground that the latter is psychologically incapacitated to comply with her essential marital obligations. Orlando contended that their marriage was not a happy one because of her parents' continued interference and attempt to break up their union. Filipinas ridiculed Orlando when he started a junk shop business and put up a pharmaceutical company which also became profitable. Filipinas became interested and began to interfere in the operation of the business; however, their continued fighting persisted and affected their children. In 1990, Orlando decided to live separately from Filipinas. Filipinas admitted that efforts at reconciliation have been fruitless and that their marriage is a failure because their differences had become irreconcilable. However, she claims that it is Orlando who is psychologically incapacitated to fulfill his obligations as a married man. On June 30, 1999, the RTC of Makati City, dismissed the petition. Issue: Whether or not the totality of the evidence presented in the present case is enough to sustain a finding that herein respondent is psychologically incapacitated to comply with her essential marital obligations Ruling: The Court finds no error in the findings of the RTC, as affirmed by the CA, that the aversive behavior of petitioner and respondent towards each other is a mere indication of incompatibility brought about by their different family backgrounds as well as their attitudes, which developed after their marriage. In her testimony, Dr. Villegas explained that Mrs. Tongol is suffering from an Inadequate Personality Disorder, with hysterical coloring, which renders her psychologically incapacitated to perform the duties and responsibilities of marriage. She is unable to cope with the sudden work and environmental shifts that overwhelmed her, due to insufficient psychological inner resources. In sum, it is not disputed that respondent is suffering from a psychological disorder. However, the totality of the evidence presented in the present case does not show that her personality disorder is of the kind contemplated by Article 36 of the Family Code as well as jurisprudence as to render her psychologically incapacitated or incapable of complying with the essential obligations of marriage. It remains settled that the State has a high stake in the preservation of marriage rooted in its recognition of the sanctity of married life and its mission to protect and strengthen the family as a basic autonomous social institution. Hence, any doubt should be resolved in favor of the existence and continuation of the marriage and against its dissolution and nullity.

491

University of the Cordilleras College of Law First Year C S.Y. 2013 - 2014

Antonio vs Reyes 484 SCRA 353


G.R. No. 155800. March 10, 2006 Full Case LEONILO ANTONIO Petitioner, vs. MARIE IVONNE F. REYES, Respondent. TINGA, J.: Statistics never lie, but lovers often do, quipped a sage. This sad truth has unsettled many a love transformed into matrimony. Any sort of deception between spouses, no matter the gravity, is always disquieting. Deceit to the depth and breadth unveiled in the following pages, dark and irrational as in the modern noir tale, dims any trace of certitude on the guilty spouses capability to fulfill the marital obligations even more. The Petition for Review on Certiorari assails the Decision and Resolution of the Court of Appeals dated 29 November 2001 and 24 October 2002. The Court of Appeals had reversed the judgment of the Regional Trial Court (RTC) of Makati declaring the marriage of Leonilo N. Antonio (petitioner) and Marie Ivonne F. Reyes (respondent), null and void. After careful consideration, we reverse and affirm instead the trial court. Antecedent Facts Petitioner and respondent met in August 1989 when petitioner was 26 years old and respondent was 36 years of age. Barely a year after their first meeting, they got married before a minister of the Gospel at the Manila City Hall, and through a subsequent church wedding at the Sta. Rosa de Lima Parish, Bagong Ilog, Pasig, Metro Manila on 6 December 1990. Out of their union, a child was born on 19 April 1991, who sadly died five (5) months later. On 8 March 1993, petitioner filed a petition to have his marriage to respondent declared null and void. He anchored his petition for nullity on Article 36 of the Family Code alleging that respondent was psychologically incapacitated to comply with the essential obligations of marriage. He asserted that respondents incapacity existed at the time their marriage was celebrated and still subsists up to the present. As manifestations of respondents alleged psychological incapacity, petitioner claimed that respondent persistently lied about herself, the people around her, her occupation, income, educational attainment and other events or things, to wit: (1) She concealed the fact that she previously gave birth to an illegitimate son, and instead introduced the boy to petitioner as the adopted child of her family. She only confessed the truth about the boys parentage when petitioner learned about it from other sources after their marriage. (2) She fabricated a story that her brother-in-law, Edwin David, attempted to rape and kill her when in fact, no such incident occurred. (3) She misrepresented herself as a psychiatrist to her obstetrician, Dr. Consuelo Gardiner, and told some of her friends that she graduated with a degree in psychology, when she was neither. (4) She claimed to be a singer or a free-lance voice talent affiliated with Blackgold Recording Company (Blackgold); yet, not a single member of her family ever witnessed her alleged singing activities with the group. In the same vein, she postulated that a luncheon show was held at the Philippine Village Hotel in her honor and even presented an invitation to that effect but petitioner discovered per certification by the Director of Sales of said hotel that no such occasion had taken place. (5) She invented friends named Babes Santos and Via Marquez, and under those names, sent lengthy letters to petitioner claiming to be from Blackgold and touting her as the "number one moneymaker" in the commercial industry worth P2 million. Petitioner later found out that respondent herself was the one 492

University of the Cordilleras College of Law First Year C S.Y. 2013 - 2014 who wrote and sent the letters to him when she admitted the truth in one of their quarrels. He likewise realized that Babes Santos and Via Marquez were only figments of her imagination when he discovered they were not known in or connected with Blackgold. (6) She represented herself as a person of greater means, thus, she altered her payslip to make it appear that she earned a higher income. She bought a sala set from a public market but told petitioner that she acquired it from a famous furniture dealer. She spent lavishly on unnecessary items and ended up borrowing money from other people on false pretexts. (7) She exhibited insecurities and jealousies over him to the extent of calling up his officemates to monitor his whereabouts. When he could no longer take her unusual behavior, he separated from her in August 1991. He tried to attempt a reconciliation but since her behavior did not change, he finally left her for good in November 1991. In support of his petition, petitioner presented Dr. Dante Herrera Abcede (Dr. Abcede), a psychiatrist, and Dr. Arnulfo V. Lopez (Dr. Lopez), a clinical psychologist, who stated, based on the tests they conducted, that petitioner was essentially a normal, introspective, shy and conservative type of person. On the other hand, they observed that respondents persistent and constant lying to petitioner was abnormal or pathological. It undermined the basic relationship that should be based on love, trust and respect. They further asserted that respondents extreme jealousy was also pathological. It reached the point of paranoia since there was no actual basis for her to suspect that petitioner was having an affair with another woman. They concluded based on the foregoing that respondent was psychologically incapacitated to perform her essential marital obligations. In opposing the petition, respondent claimed that she performed her marital obligations by attending to all the needs of her husband. She asserted that there was no truth to the allegation that she fabricated stories, told lies and invented personalities. She presented her version, thus: (1) She concealed her child by another man from petitioner because she was afraid of losing her husband. (2) She told petitioner about Davids attempt to rape and kill her because she surmised such intent from Davids act of touching her back and ogling her from head to foot. (3) She was actually a BS Banking and Finance graduate and had been teaching psychology at the Pasig Catholic School for two (2) years. (4) She was a free-lance voice talent of Aris de las Alas, an executive producer of Channel 9 and she had done three (3) commercials with McCann Erickson for the advertisement of Coca-cola, Johnson & Johnson, and Traders Royal Bank. She told petitioner she was a Blackgold recording artist although she was not under contract with the company, yet she reported to the Blackgold office after office hours. She claimed that a luncheon show was indeed held in her honor at the Philippine Village Hotel on 8 December 1979. (5) She vowed that the letters sent to petitioner were not written by her and the writers thereof were not fictitious. Bea Marquez Recto of the Recto political clan was a resident of the United States while Babes Santos was employed with Saniwares. (6) She admitted that she called up an officemate of her husband but averred that she merely asked the latter in a diplomatic matter if she was the one asking for chocolates from petitioner, and not to monitor her husbands whereabouts. (7) She belied the allegation that she spent lavishly as she supported almost ten people from her monthly budget of P7,000.00. In fine, respondent argued that apart from her non-disclosure of a child prior to their marriage, the other lies attributed to her by petitioner were mostly hearsay and unconvincing. Her stance was that the totality of the evidence presented is not sufficient for a finding of psychological incapacity on her part. 493

University of the Cordilleras College of Law First Year C S.Y. 2013 - 2014 In addition, respondent presented Dr. Antonio Efren Reyes (Dr. Reyes), a psychiatrist, to refute the allegations anent her psychological condition. Dr. Reyes testified that the series of tests conducted by his assistant, together with the screening procedures and the Comprehensive Psycho-Pathological Rating Scale (CPRS) he himself conducted, led him to conclude that respondent was not psychologically incapacitated to perform the essential marital obligations. He postulated that regressive behavior, gross neuroticism, psychotic tendencies, and poor control of impulses, which are signs that might point to the presence of disabling trends, were not elicited from respondent. In rebuttal, Dr. Lopez asseverated that there were flaws in the evaluation conducted by Dr. Reyes as (i) he was not the one who administered and interpreted respondents psychological evaluation, and (ii) he made use of only one instrument called CPRS which was not reliable because a good liar can fake the results of such test. After trial, the lower court gave credence to petitioners evidence and held that respondents propensity to lying about almost anythingher occupation, state of health, singing abilities and her income, among othershad been duly established. According to the trial court, respondents fantastic ability to invent and fabricate stories and personalities enabled her to live in a world of make-believe. This made her psychologically incapacitated as it rendered her incapable of giving meaning and significance to her marriage. The trial court thus declared the marriage between petitioner and respondent null and void. Shortly before the trial court rendered its decision, the Metropolitan Tribunal of the Archdiocese of Manila annulled the Catholic marriage of the parties, on the ground of lack of due discretion on the part of the parties. During the pendency of the appeal before the Court of Appeals, the Metropolitan Tribunals ruling was affirmed with modification by both the National Appellate Matrimonial Tribunal, which held instead that only respondent was impaired by a lack of due discretion. Subsequently, the decision of the National Appellate Matrimonial Tribunal was upheld by the Roman Rota of the Vatican. Petitioner duly alerted the Court of Appeals of these rulings by the Catholic tribunals. Still, the appellate court reversed the RTCs judgment. While conceding that respondent may not have been completely honest with petitioner, the Court of Appeals nevertheless held that the totality of the evidence presented was insufficient to establish respondents psychological incapacity. It declared that the requirements in the case of Republic v. Court of Appeals governing the application and interpretation of psychological incapacity had not been satisfied. Taking exception to the appellate courts pronouncement, petitioner elevated the case to this Court. He contends herein that the evidence conclusively establish respondents psychological incapacity. In considering the merit of this petition, the Court is heavily influenced by the credence accorded by the RTC to the factual allegations of petitioner. It is a settled principle of civil procedure that the conclusions of the trial court regarding the credibility of witnesses are entitled to great respect from the appellate courts because the trial court had an opportunity to observe the demeanor of witnesses while giving testimony which may indicate their candor or lack thereof. The Court is likewise guided by the fact that the Court of Appeals did not dispute the veracity of the evidence presented by petitioner. Instead, the appellate court concluded that such evidence was not sufficient to establish the psychological incapacity of respondent. Thus, the Court is impelled to accept the factual version of petitioner as the operative facts. Still, the crucial question remains as to whether the state of facts as presented by petitioner sufficiently meets the standards set for the declaration of nullity of a marriage under Article 36 of the Family Code. These standards were definitively laid down in the Courts 1997 ruling in Republic v. Court of Appeals (also known as the Molina case), and indeed the Court of Appeals cited the Molina guidelines in reversing the RTC in the case at bar. Since Molina was decided in 1997, the Supreme Court has yet to squarely affirm the declaration of nullity of marriage under Article 36 of the Family Code. In fact, even before Molina was handed down, there was only one case, Chi Ming Tsoi v. Court of Appeals, wherein the Court definitively concluded that a spouse was psychologically incapacitated under Article 36. This state of jurisprudential affairs may have led to the misperception that the remedy afforded by Article 36 of the Family Code is hollow, insofar as the Supreme Court is concerned. Yet what Molina and the succeeding cases did ordain was a set of guidelines which, while undoubtedly onerous on the petitioner seeking the declaration of nullity, still leave room for a decree of nullity under the proper circumstances.

494

University of the Cordilleras College of Law First Year C S.Y. 2013 - 2014 Molina did not foreclose the grant of a decree of nullity under Article 36, even as it raised the bar for its allowance. Legal Guides to Understanding Article 36 Article 36 of the Family Code states that "[a] marriage contracted by any party who, at the time of the celebration, was psychologically incapacitated to comply with the essential marital obligations of marriage, shall likewise be void even if such incapacity becomes manifest only after its solemnization." The concept of psychological incapacity as a ground for nullity of marriage is novel in our body of laws, although mental incapacity has long been recognized as a ground for the dissolution of a marriage. The Spanish Civil Code of 1889 prohibited from contracting marriage persons "who are not in the full enjoyment of their reason at the time of contracting marriage." Marriages with such persons were ordained as void, in the same class as marriages with underage parties and persons already married, among others. A partys mental capacity was not a ground for divorce under the Divorce Law of 1917, but a marriage where "either party was of unsound mind" at the time of its celebration was cited as an "annullable marriage" under the Marriage Law of 1929. Divorce on the ground of a spouses incurable insanity was permitted under the divorce law enacted during the Japanese occupation. Upon the enactment of the Civil Code in 1950, a marriage contracted by a party of "unsound mind" was classified under Article 85 of the Civil Code as a voidable marriage. The mental capacity, or lack thereof, of the marrying spouse was not among the grounds for declaring a marriage void ab initio. Similarly, among the marriages classified as voidable under Article 45 (2) of the Family Code is one contracted by a party of unsound mind. Such cause for the annulment of marriage is recognized as a vice of consent, just like insanity impinges on consent freely given which is one of the essential requisites of a contract. The initial common consensus on psychological incapacity under Article 36 of the Family Code was that it did not constitute a specie of vice of consent. Justices Sempio-Diy and Caguioa, both members of the Family Code revision committee that drafted the Code, have opined that psychological incapacity is not a vice of consent, and conceded that the spouse may have given free and voluntary consent to a marriage but was nonetheless incapable of fulfilling such rights and obligations. Dr. Tolentino likewise stated in the 1990 edition of his commentaries on the Family Code that this "psychological incapacity to comply with the essential marital obligations does not affect the consent to the marriage." There were initial criticisms of this original understanding of Article 36 as phrased by the Family Code committee. Tolentino opined that "psychologically incapacity to comply would not be juridically different from physical incapacity of consummating the marriage, which makes the marriage only voidable under Article 45 (5) of the Civil Code x x x [and thus] should have been a cause for annulment of the marriage only." At the same time, Tolentino noted "[it] would be different if it were psychological incapacity to understand the essential marital obligations, because then this would amount to lack of consent to the marriage." These concerns though were answered, beginning with Santos v. Court of Appeals, wherein the Court, through Justice Vitug, acknowledged that "psychological incapacity should refer to no less than a mental (not physical) incapacity that causes a party to be truly incognitive of the basic marital covenants that concomitantly must be assumed and discharged by the parties to the marriage." The notion that psychological incapacity pertains to the inability to understand the obligations of marriage, as opposed to a mere inability to comply with them, was further affirmed in the Molina case. Therein, the Court, through then Justice (now Chief Justice) Panganiban observed that "[t]he evidence [to establish psychological incapacity] must convince the court that the parties, or one of them, was mentally or psychically ill to such extent that the person could not have known the obligations he was assuming, or knowing them, could not have given valid assumption thereto." Jurisprudence since then has recognized that psychological incapacity "is a malady so grave and permanent as to deprive one of awareness of the duties and responsibilities of the matrimonial bond one is about to assume." It might seem that this present understanding of psychological incapacity deviates from the literal wording of Article 36, with its central phase reading "psychologically incapacitated to comply with the essential marital obligations of marriage." At the same time, it has been consistently recognized by this Court that the intent of the Family Code committee was to design the law as to allow some resiliency in its application, by avoiding specific examples that would limit the applicability of the provision under the principle of ejusdem generis. Rather, the preference of the revision committee was for 495

University of the Cordilleras College of Law First Year C S.Y. 2013 - 2014 "the judge to interpret the provision on a case-to-case basis, guided by experience, in the findings of experts and researchers in psychological disciplines, and by decisions of church tribunals which, although not binding on the civil courts, may be given persuasive effect since the provision was taken from Canon Law." We likewise observed in Republic v. Dagdag: Whether or not psychological incapacity exists in a given case calling for annulment of a marriage, depends crucially, more than in any field of the law, on the facts of the case. Each case must be judged, not on the basis of a priori assumptions, predilections or generalizations but according to its own facts. In regard to psychological incapacity as a ground for annulment of marriage, it is trite to say that no case is on "all fours" with another case. The trial judge must take pains in examining the factual milieu and the appellate court must, as much as possible, avoid substituting its own judgment for that of the trial court. The Court thus acknowledges that the definition of psychological incapacity, as intended by the revision committee, was not cast in intractable specifics. Judicial understanding of psychological incapacity may be informed by evolving standards, taking into account the particulars of each case, current trends in psychological and even canonical thought, and experience. It is under the auspices of the deliberate ambiguity of the framers that the Court has developed the Molina rules, which have been consistently applied since 1997. Molina has proven indubitably useful in providing a unitary framework that guides courts in adjudicating petitions for declaration of nullity under Article 36. At the same time, the Molina guidelines are not set in stone, the clear legislative intent mandating a case-to-case perception of each situation, and Molina itself arising from this evolutionary understanding of Article 36. There is no cause to disavow Molina at present, and indeed the disposition of this case shall rely primarily on that precedent. There is need though to emphasize other perspectives as well which should govern the disposition of petitions for declaration of nullity under Article 36. Of particular notice has been the citation of the Court, first in Santos then in Molina, of the considered opinion of canon law experts in the interpretation of psychological incapacity. This is but unavoidable, considering that the Family Code committee had bluntly acknowledged that the concept of psychological incapacity was derived from canon law, and as one member admitted, enacted as a solution to the problem of marriages already annulled by the Catholic Church but still existent under civil law. It would be disingenuous to disregard the influence of Catholic Church doctrine in the formulation and subsequent understanding of Article 36, and the Court has expressly acknowledged that interpretations given by the National Appellate Matrimonial Tribunal of the local Church, while not controlling or decisive, should be given great respect by our courts. Still, it must be emphasized that the Catholic Church is hardly the sole source of influence in the interpretation of Article 36. Even though the concept may have been derived from canon law, its incorporation into the Family Code and subsequent judicial interpretation occurred in wholly secular progression. Indeed, while Church thought on psychological incapacity is merely persuasive on the trial courts, judicial decisions of this Court interpreting psychological incapacity are binding on lower courts. Now is also opportune time to comment on another common legal guide utilized in the adjudication of petitions for declaration of nullity under Article 36. All too frequently, this Court and lower courts, in denying petitions of the kind, have favorably cited Sections 1 and 2, Article XV of the Constitution, which respectively state that "[t]he State recognizes the Filipino family as the foundation of the nation. Accordingly, it shall strengthen its solidarity and actively promote its total developmen[t]," and that "[m]arriage, as an inviolable social institution, is the foundation of the family and shall be protected by the State." These provisions highlight the importance of the family and the constitutional protection accorded to the institution of marriage. But the Constitution itself does not establish the parameters of state protection to marriage as a social institution and the foundation of the family. It remains the province of the legislature to define all legal aspects of marriage and prescribe the strategy and the modalities to protect it, based on whatever sociopolitical influences it deems proper, and subject of course to the qualification that such legislative enactment itself adheres to the Constitution and the Bill of Rights. This being the case, it also falls on the legislature to put into operation the constitutional provisions that protect marriage and the family. This has been accomplished at present through the enactment of the Family Code, which defines marriage and the family, spells out the corresponding legal effects, imposes the limitations that affect married and family life, as well as prescribes the grounds for declaration of nullity and those for legal separation. 496

University of the Cordilleras College of Law First Year C S.Y. 2013 - 2014 While it may appear that the judicial denial of a petition for declaration of nullity is reflective of the constitutional mandate to protect marriage, such action in fact merely enforces a statutory definition of marriage, not a constitutionally ordained decree of what marriage is. Indeed, if circumstances warrant, Sections 1 and 2 of Article XV need not be the only constitutional considerations to be taken into account in resolving a petition for declaration of nullity. Indeed, Article 36 of the Family Code, in classifying marriages contracted by a psychologically incapacitated person as a nullity, should be deemed as an implement of this constitutional protection of marriage. Given the avowed State interest in promoting marriage as the foundation of the family, which in turn serves as the foundation of the nation, there is a corresponding interest for the State to defend against marriages ill-equipped to promote family life. Void ab initio marriages under Article 36 do not further the initiatives of the State concerning marriage and family, as they promote wedlock among persons who, for reasons independent of their will, are not capacitated to understand or comply with the essential obligations of marriage. These are the legal premises that inform us as we decide the present petition. Molina Guidelines As Applied in This Case As stated earlier, Molina established the guidelines presently recognized in the judicial disposition of petitions for nullity under Article 36. The Court has consistently applied Molina since its promulgation in 1997, and the guidelines therein operate as the general rules. They warrant citation in full: 1) The burden of proof to show the nullity of the marriage belongs to the plaintiff. Any doubt should be resolved in favor of the existence and continuation of the marriage and against its dissolution and nullity. This is rooted in the fact that both our Constitution and our laws cherish the validity of marriage and unity of the family. Thus, our Constitution devotes an entire Article on the Family, recognizing it "as the foundation of the nation." It decrees marriage as legally "inviolable," thereby protecting it from dissolution at the whim of the parties. Both the family and marriage are to be "protected" by the state. The Family Code echoes this constitutional edict on marriage and the family and emphasizes their permanence, inviolability and solidarity. 2) The root cause of the psychological incapacity must be: (a) medically or clinically identified, (b) alleged in the complaint, (c) sufficiently proven by experts and (d) clearly explained in the decision. Article 36 of the Family Code requires that the incapacity must be psychologicalnot physical, although its manifestations and/or symptoms may be physical. The evidence must convince the court that the parties, or one of them, was mentally or psychically ill to such an extent that the person could not have known the obligations he was assuming, or knowing them, could not have given valid assumption thereof. Although no example of such incapacity need be given here so as not to limit the application of the provision under the principle of ejusdem generis, nevertheless such root cause must be identified as a psychological illness and its incapacitating nature fully explained. Expert evidence may be given by qualified psychiatrists and clinical psychologists. 3) The incapacity must be proven to be existing at "the time of the celebration" of the marriage. The evidence must show that the illness was existing when the parties exchanged their "I dos." The manifestation of the illness need not be perceivable at such time, but the illness itself must have attached at such moment, or prior thereto. 4) Such incapacity must also be shown to be medically or clinically permanent or incurable. Such incurability may be absolute or even relative only in regard to the other spouse, not necessarily absolutely against everyone of the same sex. Furthermore, such incapacity must be relevant to the assumption of marriage obligations, not necessarily to those not related to marriage, like the exercise of a profession or employment in a job. Hence, a pediatrician may be effective in diagnosing illnesses of children and prescribing medicine to cure them but not be psychologically capacitated to procreate, bear and raise his/her own children as an essential obligation of marriage.

497

University of the Cordilleras College of Law First Year C S.Y. 2013 - 2014 5) Such illness must be grave enough to bring about the disability of the party to assume the essential obligations of marriage. Thus, "mild characteriological peculiarities, mood changes, occasional emotional outbursts" cannot be accepted as root causes. The illness must be shown as downright incapacity or inability, not a refusal, neglect or difficulty, much less ill will. In other words, there is a natal or supervening disabling factor in the person, an adverse integral element in the personality structure that effectively incapacitates the person from really accepting and thereby complying with the obligations essential to marriage. 6) The essential marital obligations must be those embraced by Articles 68 up to 71 of the Family Code as regards the husband and wife as well as Articles 220, 221 and 225 of the same Code in regard to parents and their children. Such non-complied marital obligation(s) must also be stated in the petition, proven by evidence and included in the text of the decision. 7) Interpretations given by the National Appellate Matrimonial Tribunal of the Catholic Church in the Philippines, while not controlling or decisive, should be given great respect by our courts. It is clear that Article 36 was taken by the Family Code Revision Committee from Canon 1095 of the New Code of Canon Law, which became effective in 1983 and which provides: "The following are incapable of contracting marriage: Those who are unable to assume the essential obligations of marriage due to causes of psychological nature." Since the purpose of including such provision in our Family Code is to harmonize our civil laws with the religious faith of our people, it stands to reason that to achieve such harmonization, great persuasive weight should be given to decisions of such appellate tribunal. Ideallysubject to our law on evidence what is decreed as canonically invalid should also be decreed civilly void. Molina had provided for an additional requirement that the Solicitor General issue a certification stating his reasons for his agreement or opposition to the petition. This requirement however was dispensed with following the implementation of A.M. No. 02-11-10-SC, or the Rule on Declaration of Absolute Nullity of Void Marriages and Annulment of Voidable Marriages. Still, Article 48 of the Family Code mandates that the appearance of the prosecuting attorney or fiscal assigned be on behalf of the State to take steps to prevent collusion between the parties and to take care that evidence is not fabricated or suppressed. Obviously, collusion is not an issue in this case, considering the consistent vigorous opposition of respondent to the petition for declaration of nullity. In any event, the fiscals participation in the hearings before the trial court is extant from the records of this case. As earlier noted, the factual findings of the RTC are now deemed binding on this Court, owing to the great weight accorded to the opinion of the primary trier of facts, and the refusal of the Court of Appeals to dispute the veracity of these facts. As such, it must be considered that respondent had consistently lied about many material aspects as to her character and personality. The question remains whether her pattern of fabrication sufficiently establishes her psychological incapacity, consistent with Article 36 and generally, the Molina guidelines. We find that the present case sufficiently satisfies the guidelines in Molina. First. Petitioner had sufficiently overcome his burden in proving the psychological incapacity of his spouse. Apart from his own testimony, he presented witnesses who corroborated his allegations on his wifes behavior, and certifications from Blackgold Records and the Philippine Village Hotel Pavillon which disputed respondents claims pertinent to her alleged singing career. He also presented two (2) expert witnesses from the field of psychology who testified that the aberrant behavior of respondent was tantamount to psychological incapacity. In any event, both courts below considered petitioners evidence as credible enough. Even the appellate court acknowledged that respondent was not totally honest with petitioner. As in all civil matters, the petitioner in an action for declaration of nullity under Article 36 must be able to establish the cause of action with a preponderance of evidence. However, since the action cannot be considered as a non-public matter between private parties, but is impressed with State interest, the Family Code likewise requires the participation of the State, through the prosecuting attorney, fiscal, or Solicitor General, to take steps to prevent collusion between the parties and to take care that evidence is not fabricated or suppressed. Thus, even if the petitioner is able establish the psychological incapacity of

498

University of the Cordilleras College of Law First Year C S.Y. 2013 - 2014 respondent with preponderant evidence, any finding of collusion among the parties would necessarily negate such proofs. Second. The root cause of respondents psychological incapacity has been medically or clinically identified, alleged in the complaint, sufficiently proven by experts, and clearly explained in the trial courts decision. The initiatory complaint alleged that respondent, from the start, had exhibited unusual and abnormal behavior "of peren[n]ially telling lies, fabricating ridiculous stories, and inventing personalities and situations," of writing letters to petitioner using fictitious names, and of lying about her actual occupation, income, educational attainment, and family background, among others. These allegations, initially characterized in generalities, were further linked to medical or clinical causes by expert witnesses from the field of psychology. Petitioner presented two (2) such witnesses in particular. Dr. Abcede, a psychiatrist who had headed the department of psychiatry of at least two (2) major hospitals, testified as follows: WITNESS: Given that as a fact, which is only based on the affidavit provided to me, I can say that there are a couple of things that [are] terribly wrong with the standards. There are a couple of things that seems ( sic) to be repeated over and over again in the affidavit. One of which is the persistent, constant and repeated lying of the "respondent"; which, I think, based on assessment of normal behavior of an individual, is abnormal or pathological. x x x ATTY. RAZ: (Back to the witness) Q- Would you say then, Mr. witness, that because of these actuations of the respondent she is then incapable of performing the basic obligations of her marriage? A- Well, persistent lying violates the respect that one owes towards another. The lack of concern, the lack of love towards the person, and it is also something that endangers human relationship. You see, relationship is based on communication between individuals and what we generally communicate are our thoughts and feelings. But then when one talks and expresse[s] their feelings, [you] are expected to tell the truth. And therefore, if you constantly lie, what do you think is going to happen as far as this relationship is concerned. Therefore, it undermines that basic relationship that should be based on love, trust and respect. Q- Would you say then, Mr. witness, that due to the behavior of the respondent in constantly lying and fabricating stories, she is then incapable of performing the basic obligations of the marriage? xxx ATTY. RAZ: (Back to the witness) Q- Mr. witness, based on the testimony of Mr. Levy Mendoza, who is the third witness for the petitioner, testified that the respondent has been calling up the petitioners officemates and ask him ( sic) on the activities of the petitioner and ask him on the behavior of the petitioner. And this is specifically stated on page six (6) of the transcript of stenographic notes, what can you say about this, Mr. witness? A- If an individual is jealous enough to the point that he is paranoid, which means that there is no actual basis on her suspect (sic) that her husband is having an affair with a woman, if carried on to the extreme, then that is pathological. That is not abnormal. We all feel jealous, in the same way as we also lie every now and then; but everything that is carried out in extreme is abnormal or pathological. If there is no basis in reality to the fact that the husband is having an affair with another woman and if she persistently believes that the husband is having an affair with different women, then that is pathological and we call that paranoid jealousy. Q- Now, if a person is in paranoid jealousy, would she be considered psychologically incapacitated to perform the basic obligations of the marriage? A- Yes, Maam. 499

University of the Cordilleras College of Law First Year C S.Y. 2013 - 2014 The other witness, Dr. Lopez, was presented to establish not only the psychological incapacity of respondent, but also the psychological capacity of petitioner. He concluded that respondent "is [a] pathological liar, that [she continues] to lie [and] she loves to fabricate about herself." These two witnesses based their conclusions of psychological incapacity on the case record, particularly the trial transcripts of respondents testimony, as well as the supporting affidavits of petitioner. While these witnesses did not personally examine respondent, the Court had already held in Marcos v. Marcos that personal examination of the subject by the physician is not required for the spouse to be declared psychologically incapacitated.We deem the methodology utilized by petitioners witnesses as sufficient basis for their medical conclusions. Admittedly, Drs. Abcede and Lopezs common conclusion of respondents psychological incapacity hinged heavily on their own acceptance of petitioners version as the true set of facts. However, since the trial court itself accepted the veracity of petitioners factual premises, there is no cause to dispute the conclusion of psychological incapacity drawn therefrom by petitioners expert witnesses. Also, with the totality of the evidence presented as basis, the trial court explicated its finding of psychological incapacity in its decision in this wise: To the mind of the Court, all of the above are indications that respondent is psychologically incapacitated to perform the essential obligations of marriage. It has been shown clearly from her actuations that respondent has that propensity for telling lies about almost anything, be it her occupation, her state of health, her singing abilities, her income, etc. She has this fantastic ability to invent and fabricate stories and personalities. She practically lived in a world of make believe making her therefore not in a position to give meaning and significance to her marriage to petitioner. In persistently and constantly lying to petitioner, respondent undermined the basic tenets of relationship between spouses that is based on love, trust and respect. As concluded by the psychiatrist presented by petitioner, such repeated lying is abnormal and pathological and amounts to psychological incapacity. Third. Respondents psychological incapacity was established to have clearly existed at the time of and even before the celebration of marriage. She fabricated friends and made up letters from fictitious characters well before she married petitioner. Likewise, she kept petitioner in the dark about her natural childs real parentage as she only confessed when the latter had found out the truth after their marriage. Fourth. The gravity of respondents psychological incapacity is sufficient to prove her disability to assume the essential obligations of marriage. It is immediately discernible that the parties had shared only a little over a year of cohabitation before the exasperated petitioner left his wife. Whatever such circumstance speaks of the degree of tolerance of petitioner, it likewise supports the belief that respondents psychological incapacity, as borne by the record, was so grave in extent that any p rolonged marital life was dubitable. It should be noted that the lies attributed to respondent were not adopted as false pretenses in order to induce petitioner into marriage. More disturbingly, they indicate a failure on the part of respondent to distinguish truth from fiction, or at least abide by the truth. Petitioners witnesses and the trial court were emphatic on respondents inveterate proclivity to telling lies and the pathologic nature of her mistruths, which according to them, were revelatory of respondents inability to understand and perform the essential obligations of marriage. Indeed, a person unable to distinguish between fantasy and reality would similarly be unable to comprehend the legal nature of the marital bond, much less its psychic meaning, and the corresponding obligations attached to marriage, including parenting. One unable to adhere to reality cannot be expected to adhere as well to any legal or emotional commitments. The Court of Appeals somehow concluded that since respondent allegedly tried her best to effect a reconciliation, she had amply exhibited her ability to perform her marital obligations. We are not convinced. Given the nature of her psychological condition, her willingness to remain in the marriage hardly banishes nay extenuates her lack of capacity to fulfill the essential marital obligations. Respondents ability to even comprehend what the essential marital obligations are is impaired at best. Considering that the evidence convincingly disputes respondents ability to adhere to the truth, her avowals as to her commitment to the marriage cannot be accorded much credence. At this point, it is worth considering Article 45(3) of the Family Code which states that a marriage may be annulled if the consent of either party was obtained by fraud, and Article 46 which enumerates the circumstances constituting fraud under the previous article, clarifies that "no other misrepresentation or 500

University of the Cordilleras College of Law First Year C S.Y. 2013 - 2014 deceit as to character, health, rank, fortune or chastity shall constitute such fraud as will give grounds for action for the annulment of marriage." It would be improper to draw linkages between misrepresentations made by respondent and the misrepresentations under Articles 45 (3) and 46. The fraud under Article 45(3) vitiates the consent of the spouse who is lied to, and does not allude to vitiated consent of the lying spouse. In this case, the misrepresentations of respondent point to her own inadequacy to cope with her marital obligations, kindred to psychological incapacity under Article 36. Fifth. Respondent is evidently unable to comply with the essential marital obligations as embraced by Articles 68 to 71 of the Family Code. Article 68, in particular, enjoins the spouses to live together, observe mutual love, respect and fidelity, and render mutual help and support. As noted by the trial court, it is difficult to see how an inveterate pathological liar would be able to commit to the basic tenets of relationship between spouses based on love, trust and respect. Sixth. The Court of Appeals clearly erred when it failed to take into consideration the fact that the marriage of the parties was annulled by the Catholic Church. The appellate court apparently deemed this detail totally inconsequential as no reference was made to it anywhere in the assailed decision despite petitioners efforts to bring the matter to its attention. Such deliberate ignorance is in contravention of Molina, which held that interpretations given by the National Appellate Matrimonial Tribunal of the Catholic Church in the Philippines, while not controlling or decisive, should be given great respect by our courts. As noted earlier, the Metropolitan Tribunal of the Archdiocese of Manila decreed the invalidity of the marriage in question in a Conclusion dated 30 March 1995, citing the "lack of due discretion" on the part of respondent. Such decree of nullity was affirmed by both the National Appellate Matrimonial Tribunal, and the Roman Rota of the Vatican. In fact, respondents psychological incapacity was considered so grave that a restrictive clause was appended to the sentence of nullity prohibiting respondent from contracting another marriage without the Tribunals consent. In its Decision dated 4 June 1995, the National Appellate Matrimonial Tribunal pronounced: The JURISRPRUDENCE in the Case maintains that matrimonial consent is considered ontologically defective and wherefore judicially ineffective when elicited by a Part Contractant in possession and employ of a discretionary judgment faculty with a perceptive vigor markedly inadequate for the practical understanding of the conjugal Covenant or serious impaired from the correct appreciation of the integral significance and implications of the marriage vows. The FACTS in the Case sufficiently prove with the certitude required by law that based on the depositions of the Partes in Causa and premised on the testimonies of the Common and Expert Witnesse[s], the Respondent made the marriage option in tenure of adverse personality constracts that were markedly antithetical to the substantive content and implications of the Marriage Covenant, and that seriously undermined the integrality of her matrimonial consent in terms of its deliberative component. In other words, afflicted with a discretionary faculty impaired in its practico-concrete judgment formation on account of an adverse action and reaction pattern, the Respondent was impaired from eliciting a judicially binding matrimonial consent. There is no sufficient evidence in the Case however to prove as well the fact of grave lack of due discretion on the part of the Petitioner. Evidently, the conclusion of psychological incapacity was arrived at not only by the trial court, but also by canonical bodies. Yet, we must clarify the proper import of the Church rulings annulling the marriage in this case. They hold sway since they are drawn from a similar recognition, as the trial court, of the veracity of petitioners allegations. Had the trial court instead appreciated respondents version as correct, and the appellate court affirmed such conclusion, the rulings of the Catholic Church on this matter would have diminished persuasive value. After all, it is the factual findings of the judicial trier of facts, and not that of the canonical courts, that are accorded significant recognition by this Court. Seventh. The final point of contention is the requirement in Molina that such psychological incapacity be shown to be medically or clinically permanent or incurable. It was on this score that the Court of Appeals reversed the judgment of the trial court, the appellate court noting that it did not appear certain that respondents condition was incurable and that Dr. Abcede did not testify to such effect. Petitioner points out that one month after he and his wife initially separated, he returned to her, desiring to make their marriage work. However, respondents aberrant behavior remained unchanged, as she 501

University of the Cordilleras College of Law First Year C S.Y. 2013 - 2014 continued to lie, fabricate stories, and maintained her excessive jealousy. From this fact, he draws the conclusion that respondents condition is incurable. From the totality of the evidence, can it be definitively concluded that respondents condition is incurable? It would seem, at least, that respondents psychosis is quite grave, and a cure thereof a remarkable feat. Certainly, it would have been easier had petitioners expert witnesses characterized respondents condition as incurable. Instead, they remained silent on whether the psychological incapacity was curable or incurable. But on careful examination, there was good reason for the experts taciturnity on this point. The petitioners expert witnesses testified in 1994 and 1995, and the trial court rendered its decision on 10 August 1995. These events transpired well before Molina was promulgated in 1997 and made explicit the requirement that the psychological incapacity must be shown to be medically or clinically permanent or incurable. Such requirement was not expressly stated in Article 36 or any other provision of the Family Code. On the other hand, the Court in Santos, which was decided in January 1995, began its discussion by first citing the deliberations of the Family Code committee, then the opinion of canonical scholars, before arriving at its formulation of the doctrinal definition of psychological incapacity.Santos did refer to Justice Caguioas opinion expressed during the deliberations that "psychological incapacity is incurable," and the view of a former presiding judge of the Metropolitan Marriage Tribunal of the Archdiocese of Manila that psychological incapacity must be characterized "by (a) gravity, (b) juridical antecedence, and (c) incurability." However, in formulating the doctrinal rule on psychological incapacity, the Court in Santos omitted any reference to incurability as a characteristic of psychological incapacity. This disquisition is material as Santos was decided months before the trial court came out with its own ruling that remained silent on whether respondents psychological incapacity was incurable. Certainly, Santos did not clearly mandate that the incurability of the psychological incapacity be established in an action for declaration of nullity. At least, there was no jurisprudential clarity at the time of the trial of this case and the subsequent promulgation of the trial courts decision that required a medical finding of incurability. Such requisite arose only with Molina in 1997, at a time when this case was on appellate review, or after the reception of evidence. We are aware that in Pesca v. Pesca, the Court countered an argument that Molina and Santos should not apply retroactively with the observation that the interpretation or construction placed by the courts of a law constitutes a part of that law as of the date the statute in enacted. Yet we approach this present case from utterly practical considerations. The requirement that psychological incapacity must be shown to be medically or clinically permanent or incurable is one that necessarily cannot be divined without expert opinion. Clearly in this case, there was no categorical averment from the expert witnesses that respondents psychological incapacity was curable or incurable simply because there was no legal necessity yet to elicit such a declaration and the appropriate question was not accordingly propounded to him. If we apply Pesca without deep reflection, there would be undue prejudice to those cases tried before Molina or Santos, especially those presently on appellate review, where presumably the respective petitioners and their expert witnesses would not have seen the need to adduce a diagnosis of incurability. It may hold in those cases, as in this case, that the psychological incapacity of a spouse is actually incurable, even if not pronounced as such at the trial court level. We stated earlier that Molina is not set in stone, and that the interpretation of Article 36 relies heavily on a case-to-case perception. It would be insensate to reason to mandate in this case an expert medical or clinical diagnosis of incurability, since the parties would have had no impelling cause to present evidence to that effect at the time this case was tried by the RTC more than ten (10) years ago. From the totality of the evidence, we are sufficiently convinced that the incurability of respondents psychological incapacity has been established by the petitioner. Any lingering doubts are further dispelled by the fact that the Catholic Church tribunals, which indubitably consider incurability as an integral requisite of psychological incapacity, were sufficiently convinced that respondent was so incapacitated to contract marriage to the degree that annulment was warranted.

502

University of the Cordilleras College of Law First Year C S.Y. 2013 - 2014 All told, we conclude that petitioner has established his cause of action for declaration of nullity under Article 36 of the Family Code. The RTC correctly ruled, and the Court of Appeals erred in reversing the trial court. There is little relish in deciding this present petition, pronouncing as it does the marital bond as having been inexistent in the first place. It is possible that respondent, despite her psychological state, remains in love with petitioner, as exhibited by her persistent challenge to the petition for nullity. In fact, the appellate court placed undue emphasis on respondents avowed commitment to remain in the marriage. Yet the Court decides these cases on legal reasons and not vapid sentimentality. Marriage, in legal contemplation, is more than the legitimatization of a desire of people in love to live together. WHEREFORE, the petition is GRANTED. The decision of the RTC dated 10 August 1995, declaring the marriage between petitioner and respondent NULL and VOID under Article 36 of the Family Code, is REINSTATED. No costs. SO ORDERED.

503

University of the Cordilleras College of Law First Year C S.Y. 2013 - 2014 Case Digest ANTONIOvs. REYES G.R. No. 155800. March 10, 2006 484 SCRA 353 TINGA, J.: Facts: Leonilo N. Antonio and Marie Ivonne F. Reyes met in August 1989 when petitioner was 26 years old and respondent was 36 years of age. Barely a year after their first meeting, they got married on December 6, 1990. Out of their union, a child was born on April 19, 1991, who sadly died five (5) months later. On March 8, 1993, petitioner filed a petition to have his marriage to respondent declared null and void on the ground of psychological incapacity. As manifestations of respondents alleged psychological incapacity, petitioner claimed that respondent persistently lied about herself, the people around her, her occupation, income, educational attainment and other events or things, to wit: (1) She concealed the fact that she previously gave birth to an illegitimate son, and instead introduced the boy to petitioner as the adopted child of her family. She only confessed the truth about the boys parentage when petitioner learned about it from other sources after their marriage. (2) She fabricated a story that her brother-in-law, Edwin David, attempted to rape and kill her when in fact, no such incident occurred. (3) She misrepresented herself as a psychiatrist to her obstetrician, Dr. Consuelo Gardiner, and told some of her friends that she graduated with a degree in psychology, when she was neither. (4) She claimed to be a singer or a free-lance voice talent affiliated with Blackgold Recording Company (Blackgold); yet, not a single member of her family ever witnessed her alleged singing activities with the group. In the same vein, she postulated that a luncheon show was held at the Philippine Village Hotel in her honor and even presented an invitation to that effect but petitioner discovered per certification by the Director of Sales of said hotel that no such occasion had taken place. (5) She invented friends named Babes Santos and Via Marquez, and under those names, sent lengthy letters to petitioner claiming to be from Blackgold and touting her as the "number one moneymaker" in the commercial industry worth P2 million. Petitioner later found out that respondent herself was the one who wrote and sent the letters to him when she admitted the truth in one of their quarrels. He likewise realized that Babes Santos and Via Marquez were only figments of her imagination when he discovered they were not known in or connected with Blackgold. (6) She represented herself as a person of greater means, thus, she altered her payslip to make it appear that she earned a higher income. She bought a sala set from a public market but told petitioner that she acquired it from a famous furniture dealer. She spent lavishly on unnecessary items and ended up borrowing money from other people on false pretexts. (7) She exhibited insecurities and jealousies over him to the extent of calling up his officemates to monitor his whereabouts. When he could no longer take her unusual behavior, he separated from her in August 1991. He tried to attempt a reconciliation but since her behavior did not change, he finally left her for good in November 1991 The Trial Court declared the marriage as null and void since it found that respondent's propensity to lie rendered her incapable of giving meaning and significance to her marriage. The Church also annuled the Catholic marriage of the parties. The Court of Appeals nevertheless held that the totality of the evidence presented was insufficient to establish respondents psychological incapacity and thus reversed the decision of the trial court. The petitioner then filed a motion to the Supreme Court. Issue: Whether or not pathological lying constitute psychological incapacity Ruling: The Supreme Court granted the petition and reinstated the decision of the Trial Court. The Molina guidelines did not foreclose the grant of a decree of nullity under Article 36, even as it 504

University of the Cordilleras College of Law First Year C S.Y. 2013 - 2014 raised the bar for its allowance. The guidelines are in fact used to interpret incapacity in the present case. From the totality of the evidence, we are sufficiently convinced that the incurability of respondents psychological incapacity has been established by the petitioner. Any lingering doubts are further dispelled by the fact that the Catholic Church tribunals, which indubitably consider incurability as an integral requisite of psychological incapacity, were sufficiently convinced that respondent was so incapacitated to contract marriage to the degree that annulment was warranted. It should be noted that the lies attributed to respondent were not adopted as false pretenses in order to induce petitioner into marriage. More disturbingly, they indicate a failure on the part of respondent to distinguish truth from fiction, or at least abide by the truth. Petitioners witnesses and the trial court were emphatic on respondents inveterate proclivity to telling lies and the pathologic nature of her mistruths, which according to them, were revelatory of respondents inability to understand and perform the essential obligations of marriage. Indeed, a person unable to distinguish between fantasy and reality would similarly be unable to comprehend the legal nature of the marital bond, much less its psychic meaning, and the corresponding obligations attached to marriage, including parenting. One unable to adhere to reality cannot be expected to adhere as well to any legal or emotional commitments.

505

University of the Cordilleras College of Law First Year C S.Y. 2013 - 2014 Villanueva vs Court of Appeals 505 SCRA 564 G.R. No. 132955 October 27, 2006 Full Case ORLANDO VILLANUEVA, petitioner, vs. HON. COURT OF APPEALS and LILIA CANALITA-VILLANUEVA, respondents. YNARES-SANTIAGO, J.: This petition for review under Rule 45 of the Rules of Court assails the January 26, 1998 Decision of the Court of Appeals in CA-G.R. CV No. 51832, affirming with modification the Decision dated January 12, 1996 of the Regional Trial Court of Valenzuela, Metro Manila, Branch 172 in Civil Case No. 3997-V-92 (a) dismissing petitioner's petition for the annulment of his marriage to private respondent and (b) ordering him to pay moral and exemplary damages, attorneys fees and costs. Also assailed is the March 5, 1998 Resolution denying petitioners motion for reconsideration. The antecedent facts are as follows: Petitioner Orlando Villanueva and private respondent Lilia Canalita-Villanueva got married on April 13, 1988 in Puerto Princesa, Palawan. On November 17, 1992, Orlando filed with the trial court a petition for annulment of his marriage alleging that threats of violence and duress forced him into marrying Lilia, who was already pregnant; that he did not get her pregnant prior to the marriage; that he never cohabited with her after the marriage; and that he later learned that private respondent's child died during delivery on August 29, 1988. In her answer with compulsory counterclaim, Lilia prayed for the dismissal of the petition, arguing that petitioner freely and voluntarily married her; that petitioner stayed with her in Palawan for almost a month after their marriage; that petitioner wrote letters to her after he returned to Manila, during which private respondent visited him personally; and that petitioner knew about the progress of her pregnancy, which ended in their son being born prematurely. Private respondent also prayed for the payment of moral and exemplary damages, attorneys fees and costs. On January 12, 1996, the trial court rendered judgment the dispositive portion of which states: WHEREFORE, judgment is hereby rendered as follows: 1) Dismissing the above-entitled case; and 2) Ordering the plaintiff to pay the defendant moral damages in the amount of P100,000.00, exemplary damages in the amount of P50,000.00, and attorney's fees in the amount of P20,000.00, plus the costs of suit. SO ORDERED. The Court of Appeals affirmed the trial courts dismissal of the petition and the award of attorneys fees and costs, but reduced the award of moral and exemplary damages to P50,000.00 and P25,000.00, respectively. The Court of Appeals denied petitioners motion for reconsideration, hence, the instant petition for review based on the following assigned errors: I. THE RESPONDENT COURT OF APPEALS COMMITTED A GRAVE ABUSE OF DISCRETION IN NOT GRANTING THE ANNULMENT OF MARRIAGE THE CONSENT OF THE PETITIONER HAVING BEEN OBTAINED BY FRAUD, INTIMIDATION AND UNDUE AND IMPROPER PRESSURE AND INFLUENCE PLUS THE FACT THAT THERE WAS NO COHABITATION WHATSOEVER BETWEEN PETITIONER AND PRIVATE RESPONDENT.

506

University of the Cordilleras College of Law First Year C S.Y. 2013 - 2014 II. THE RESPONDENT COURT OF APPEALS COMMITTED GROSS ERROR IN AWARDING MORAL AND EXEMPLARY DAMAGES AS WELL AS ATTORNEY'S FEES, SAID AWARDS NOT BEING THOSE ALLOWED BY LAW. The issues for resolution are (a) whether the subject marriage may be annulled on the ground of vitiated consent; and (b) whether petitioner should be liable for moral and exemplary damages as well as attorneys fees and costs. The petition is partly granted. Factual findings of the Court of Appeals, especially if they coincide with those of the trial court, as in the instant case, are generally binding on this Court. We affirm the findings of the Court of Appeals that petitioner freely and voluntarily married private respondent and that no threats or intimidation, duress or violence compelled him to do so, thus To begin with, We are at once disturbed by the circumstance that despite the alleged coerced consent which supposedly characterized his marriage with Lilia on April 13, 1988, it was only on November 17, 1992 or after a span of not less than four (4) years and eight (8) months when Orlando took serious step to have the same marriage annulled. Unexplained, the prolonged inaction evidently finds basis in Lilias allegation that this annulment suit was filed by Orlando solely in the hope that a favorable judgment thereon would bolster his defense, if not altogether bring about his acquittal in the criminal case for bigamy which was then already pending against him. Unfortunately, however, let alone the fact that the criminal case was admittedly decided ahead with a judgment of conviction against Orlando x x x even the very outcome of the present case disappointed his expectation. At this late, with his appeal in the bigamy case still pending with this Court x x x Orlando must be hoping against hope that with a decree of annulment ensuing from this Court, he may yet secure an acquittal in the same bigamy charge. Viewed in this perspective, the instant appeal is, therefore, understandable. But even in terms of merit, the recourse must have to fall. Appellant anchored his prayer for the annulment of his marriage on the ground that he did not freely consent to be married to the appellee. He cited several incidents that created on his mind a reasonable and well-grounded fear of an imminent and grave danger to his life and safety, to wit: the harassing phone calls from the appellee and strangers as well as the unwanted visits by three men at the premises of the University of the East after his classes thereat, and the threatening presence of a certain Ka Celso, a supposed member of the New Peoples Army whom appellant claimed to have been hired by appellee and who accompanied him in going to her home province of Palawan to marry her. The Court is not convinced that appellants apprehension of danger to his person is so overwhelming as to deprive him of the will to enter voluntarily to a contract of marriage. It is not disputed that at the time he was allegedly being harassed, appellant worked as a security guard in a bank. Given his employment at that time, it is reasonable to assume that appellant knew the rudiments of self-defense, or, at the very least, the proper way to keep himself out of harms way. For sure, it is even doubtful if threats were indeed made to bear upon appellant, what with the fact that he never sought the assistance of the security personnel of his school nor the police regarding the activities of those who were threatening him. And neither did he inform the judge about his predicament prior to solemnizing their marriage. Appellant also invoked fraud to annul his marriage, as he was made to believe by appellee that the latter was pregnant with his child when they were married. Appellants excuse that he could not have impregnated the appellee because he did not have an erection during their tryst is flimsy at best, and an outright lie at worst. The complaint is bereft of any reference to his inability to copulate with the appellee. His counsel also conceded before the lower court that his client had a sexual relationship with the appellee x x x. He also narrated x x x that sometime in January 1988, he and the appellee went to a hotel where "the sexual act was consummated, with the defendant on top" x x x. 507

University of the Cordilleras College of Law First Year C S.Y. 2013 - 2014 Instead of providing proofs that he was tricked into marrying his wife, appellant resorted to undermining the credibility of the latter by citing her testimony that her child was born, and died, on August 29, 1989, a year off from August 29, 1988, the date of fetal death as appearing in the registry of deaths of the Office of the Civil Registrar of Puerto Princesa City x x x. To Our mind, appellant cannot make capital of the lapse because it is inconsequential, as there is no controversy regarding the date of death of appellees fetus. Nevertheless, during the continuation of the cross-examination of the appellee, she declared that her child was prematurely born on August 29, 1988, matching the date in the certification of the Civil Registrar x x x. The Court is not prepared to disbelieve the appellee and throw overboard her entire testimony simply on account of her confusion as to the exact date of the death of the fetus, especially when she herself had presented documentary evidence that put August 29, 1988 as the date her fetus died. Appellants propensity to rely on his perceived weakness of the appellees evidence continues in his argument that if indeed there is truth to her claim that she was impregnated sometime in December 1987, then she could not have a premature delivery on August 29, 1988, as she had testified during the trial, because the 35-week period of pregnancy is complete by that time. Whether the appellees impression that she had delivered prematurely is correct or not will not affect the fact that she had delivered a fetus on August 29, 1988. In the light of appellants admission that he had a sexual intercourse with his wife in January 1988, and his failure to attribute the latters pregnancy to any other man, appellant cannot complain that he was deceived by the appellee into marrying her. Appellant also puts in issue the lower courts appreciation of the le tters allegedly written by him to the appellee. During his cross-examination, when confronted with thirteen (13) letters, appellant identified the seven (7) letters that he sent to the appellee, but denied the remaining six (6) x x x. The letters admitted by the appellant contained expressions of love and concern for his wife, and hardly the rantings of a man under duress. During the re-direct examination, however, appellant suddenly changed mind and denied authorship of those seven (7) letters, claiming that he was forced to admit them because he was threatened with harm by the appellee. If he was laboring under duress when he made the admission, where did he find the temerity to deny his involvement with the remaining six (6) letters? The recantation can only be motivated by a hindsight realization by the appellant of the evidentiary weight of those letters against his case. As to the second assignment of error, appellant cannot claim that his marriage should be annulled due to the absence of cohabitation between him and his wife. Lack of cohabitation is, per se, not a ground to annul a marriage. Otherwise, the validity of a marriage will depend upon the will of the spouses who can terminate the marital union by refusing to cohabitate. The failure to cohabit becomes relevant only if it arises as a result of the perpetration of any of the grounds for annulling the marriage, such as lack of parental consent, insanity, fraud, intimidation, or undue influence x x x. Since the appellant failed to justify his failure to cohabit with the appellee on any of those grounds, the validity of his marriage must be upheld. We also agree that private respondent is entitled to attorneys fees. Article 2208 (11) of the Civil Code provides that attorneys may be awarded where the court deems it just and equitable under the circumstances, as in the instant case. We, however, delete the award of moral and exemplary damages for lack of factual and legal basis. There is nothing in the records or in the appealed decision that would support an award of moral damages. In justifying the award, the Court of Appeals merely said thus: It is not difficult to imagine the suffering of the appellee from the baseless portrayal of her by the appellant as the perpetrator of fraudulent schemes to trap an unwilling mate. x xx

508

University of the Cordilleras College of Law First Year C S.Y. 2013 - 2014 However, the aforesaid finding is only a supposition as it has no reference to any testimony of private respondent detailing her alleged physical suffering, mental anguish, fright, serious anxiety, besmirched reputation, wounded feelings, moral shock, social humiliation, and similar injury as would entitle her to moral damages. In Mahinay v. Velasquez, Jr., we held that: In order that moral damages may be awarded, there must be pleading and proof of moral suffering, mental anguish, fright and the like. While respondent alleged in his complaint that he suffered mental anguish, serious anxiety, wounded feelings and moral shock, he failed to prove them during the trial. Indeed, respondent should have taken the witness stand and should have testified on the mental anguish, serious anxiety, wounded feelings and other emotional and mental suffering he purportedly suffered to sustain his claim for moral damages. Mere allegations do not suffice; they must be substantiated by clear and convincing proof. No other person could have proven such damages except the respondent himself as they were extremely personal to him. As private respondent is not entitled to moral damages, a fortiori, she is not entitled to exemplary damages. This is clear in Article 2234 of the Civil Code, which provides: ART. 2234. While the amount of the exemplary damages need not be proved, the plaintiff must show that he is entitled to moral, temperate or compensatory damages before the court may consider the question of whether or not exemplary damages should be awarded. In case liquidated damages have been agreed upon, although no proof of loss is necessary in order that such liquidated damages may be recovered, nevertheless, before the court may consider the question of granting exemplary in addition to the liquidated damages, the plaintiff must show that he would be entitled to moral, temperate or compensatory damages were it not for the stipulation for liquidated damages. Hence, exemplary damages is allowed only in addition to moral damages such that no exemplary damages can be awarded unless the claimant first establishes his clear right to moral damages. In the instant case, private respondent failed to satisfactorily establish her claim for moral damages, thus she is not likewise entitled to exemplary damages. WHEREFORE, the petition is PARTLY GRANTED. The January 26, 1998 Decision of the Court of Appeals in CA-G.R. CV No. 51832 affirming with modification the January 12, 1996 Decision of the Regional Trial Court of Valenzuela, Metro Manila, Branch 172 in Civil Case No. 3997-V-92 dismissing petitioners petition for the annulment of his marriage with private respondent, is AFFIRMED. However, the award of moral and exemplary damages is DELETED for lack of basis. SO ORDERED. Panganiban, C.J. (Chairperson), Austria-Martinez, Callejo, Sr., and Chico-Nazario, JJ., concur.

509

University of the Cordilleras College of Law First Year C S.Y. 2013 - 2014 Case Digest VILLANUEVA vs. COURT OF APPEALS G.R. No. 132955 October 27, 2006 505 SCRA 564 YNARES-SANTIAGO, J.: Facts: Petitioner Orlando Villanueva and private respondent Lilia Canalita-Villanueva got married on April 13, 1988 in Puerto Princesa, Palawan. On November 17, 1992, Orlando filed with the trial court a petition for annulment of his marriage alleging that threats of violence and duress forced him into marrying Lilia, who was already pregnant; that he did not get her pregnant prior to the marriage; that he never cohabited with her after the marriage; and that he later learned that private respondent's child died during delivery on August 29, 1988. In her counterclaim, Lilia prayed for the dismissal of the petition, arguing that petitioner freely and voluntarily married her; that petitioner stayed with her in Palawan for almost a month after their marriage; that petitioner wrote letters to her after he returned to Manila, during which private respondent visited him personally; and that petitioner knew about the progress of her pregnancy, which ended in their son being born prematurely. Issue: Whether or not the marriage may be annulled on the ground of vitiated consent Ruling: We affirm the findings of the Court of Appeals that petitioner freely and voluntarily married private respondent and that no threats or intimidation, duress or violence compelled him to do so. The Court is not convinced that appellants apprehension of danger to his person is so overwhelming as to deprive him of the will to enter voluntarily to a contract of marriage. It is not disputed that at the time he was allegedly being harassed, appellant worked as a security guard in a bank. Given his employment at that time, it is reasonable to assume that appellant knew the rudiments of self-defense, or, at the very least, the proper way to keep himself out of harms way. For sure, it is even doubtful if threats were indeed made to bear upon appellant, what with the fact that he never sought the assistance of the security personnel of his school nor the police regarding the activities of those who were threatening him. And neither did he inform the judge about his predicament prior to solemnizing their marriage. Appellant also invoked fraud to annul his marriage, as he was made to believe by appellee that the latter was pregnant with his child when they were married. Appellants excuse that he could not have impregnated the appellee because he did not have an erection during their tryst is flimsy at best, and an outright lie at worst. The complaint is bereft of any reference to his inability to copulate with the appellee. The petitioners petition for the annulment of his marriage with private respondent is dismissed.

510

University of the Cordilleras College of Law First Year C S.Y. 2013 - 2014 Ngo-Te vs Yu-Te G.R. No. 161 SCRA 793 G.R. No. 161793 February 13, 2009 Full Case EDWARD KENNETH NGO TE, Petitioner, vs. ROWENA ONG GUTIERREZ YU-TE, Respondent, REPUBLIC OF THE PHILIPPINES, Oppositor. NACHURA, J.: Far from novel is the issue involved in this petition. Psychological incapacity, since its incorporation in our laws, has become a clichd subject of discussion in our jurisprudence. The Court treats this case, however, with much ado, it having realized that current jurisprudential doctrine has unnecessarily imposed a perspective by which psychological incapacity should be viewed, totally inconsistent with the way the concept was formulatedfree in form and devoid of any definition. For the resolution of the Court is a petition for review on certiorari under Rule 45 of the Rules of Court assailing the August 5, 2003 Decision of the Court of Appeals (CA) in CA-G.R. CV No. 71867. The petition further assails the January 19, 2004 Resolution denying the motion for the reconsideration of the challenged decision. The relevant facts and proceedings follow. Petitioner Edward Kenneth Ngo Te first got a glimpse of respondent Rowena Ong Gutierrez Yu-Te in a gathering organized by the Filipino-Chinese association in their college. Edward was then initially attracted to Rowenas close friend; but, as the latter already had a boyfriend, the young man decided to court Rowena. That was in January 1996, when petitioner was a sophomore student and respondent, a freshman. Sharing similar angst towards their families, the two understood one another and developed a certain degree of closeness towards each other. In March 1996, or around three months after their first meeting, Rowena asked Edward that they elope. At first, he refused, bickering that he was young and jobless. Her persistence, however, made him relent. Thus, they left Manila and sailed to Cebu that month; he, providing their travel money and she, purchasing the boat ticket. However, Edwards P80,000.00 lasted for only a month. Their pension house accommodation and daily sustenance fast depleted it. And they could not find a job. In April 1996, they decided to go back to Manila. Rowena proceeded to her uncles house and Edward to his parents home. As his family wa s abroad, and Rowena kept on telephoning him, threatening him that she would commit suicide, Edward agreed to stay with Rowena at her uncles place. On April 23, 1996, Rowenas uncle brought the two to a court to get married. He was then 25 years old, and she, 20. The two then continued to stay at her uncles place where Edward was treated like a prisonerhe was not allowed to go out unaccompanied. Her uncle also showed Edward his guns and warned the latter not to leave Rowena. At one point, Edward was able to call home and talk to his brother who suggested that they should stay at their parents home and live with them. Edward relayed this to Rowena who, however, suggested that he should get his inheritance so that they could live on their own. Edward talked to his father about this, but the patriarch got mad, told Edward that he would be disinherited, and insisted that Edward must go home. After a month, Edward escaped from the house of Rowenas uncle, and stayed with his parents. His family then hid him from Rowena and her family whenever they telephoned to ask for him. In June 1996, Edward was able to talk to Rowena. Unmoved by his persistence that they should live with his parents, she said that it was better for them to live separate lives. They then parted ways.

511

University of the Cordilleras College of Law First Year C S.Y. 2013 - 2014 After almost four years, or on January 18, 2000, Edward filed a petition before the Regional Trial Court (RTC) of Quezon City, Branch 106, for the annulment of his marriage to Rowena on the basis of the latters psychological incapacity. This was docketed as Civil Case No. Q-00-39720. As Rowena did not file an answer, the trial court, on July 11, 2000, ordered the Office of the City Prosecutor (OCP) of Quezon City to investigate whether there was collusion between the parties. In the meantime, on July 27, 2000, the Office of the Solicitor General (OSG) entered its appearance and deputized the OCP to appear on its behalf and assist it in the scheduled hearings. On August 23, 2000, the OCP submitted an investigation report stating that it could not determine if there was collusion between the parties; thus, it recommended trial on the merits. The clinical psychologist who examined petitioner found both parties psychologically incapacitated, and made the following findings and conclusions: BACKGROUND DATA & BRIEF MARITAL HISTORY: EDWARD KENNETH NGO TE is a [29-year-old] Filipino male adult born and baptized Born Again Christian at Manila. He finished two years in college at AMA Computer College last 1994 and is currently unemployed. He is married to and separated from ROWENA GUTIERREZ YU-TE. He presented himself at my office for a psychological evaluation in relation to his petition for Nullification of Marriage against the latter by the grounds of psychological incapacity. He is now residing at 181 P. Tuazon Street, Quezon City. Petitioner got himself three siblings who are now in business and one deceased sister. Both his parents are also in the business world by whom he [considers] as generous, hospitable, and patient. This said virtues are said to be handed to each of the family member. He generally considers himself to be quiet and simple. He clearly remembers himself to be afraid of meeting people. After 1994, he tried his luck in being a Sales Executive of Mansfield International Incorporated. And because of job incompetence, as well as being quiet and loner, he did not stay long in the job until 1996. His interest lie[s] on becoming a full servant of God by being a priest or a pastor. He [is] said to isolate himself from his friends even during his childhood days as he only loves to read the Bible and hear its message. Respondent is said to come from a fine family despite having a lazy father and a disobedient wife. She is said to have not finish[ed] her collegiate degree and shared intimate sexual moments with her boyfriend prior to that with petitioner. In January of 1996, respondent showed her kindness to petitioner and this became the foundation of their intimate relationship. After a month of dating, petitioner mentioned to respondent that he is having problems with his family. Respondent surprisingly retorted that she also hates her family and that she actually wanted to get out of their lives. From that [time on], respondent had insisted to petitioner that they should elope and live together. Petitioner hesitated because he is not prepared as they are both young and inexperienced, but she insisted that they would somehow manage because petitioner is rich. In the last week of March 1996, respondent seriously brought the idea of eloping and she already bought tickets for the boat going to Cebu. Petitioner reluctantly agreed to the idea and so they eloped to Cebu. The parties are supposed to stay at the house of a friend of respondent, but they were not able to locate her, so petitioner was compelled to rent an apartment. The parties tried to look for a job but could not find any so it was suggested by respondent that they should go back and seek help from petitioners parents. When the parties arrived at the house of petitioner, all of his whole family was all out of the country so respondent decided to go back to her home for the meantime while petitioner stayed behind at their home. After a few days of separation, respondent called petitioner by phone and said she wanted to talk to him. Petitioner responded immediately and when he arrived at their house, respondent confronted petitioner as to why he appeared to be cold, respondent acted irrationally and even threatened to commit suicide. Petitioner got scared so he went home again. Respondent would call by phone every now and then and became angry as petitioner does not know what to do. Respondent went to the extent of threatening to file a case against petitioner and scandalize his family in the newspaper. Petitioner asked her how he would be able to make amends and at this point in time[,] respondent brought the idea of marriage. Petitioner[,] out of frustration in life[,] agreed to her to pacify her. And so on April 23, 1996, respondents uncle brought the parties to Valenzuela[,] and on that very same day[,] petitioner was made to sign the Marriage Contract before the Judge. Petitioner actually never applied for any Marriage License.

512

University of the Cordilleras College of Law First Year C S.Y. 2013 - 2014 Respondent decided that they should stay first at their house until after arrival of the parents of petitioner. But when the parents of petitioner arrived, respondent refused to allow petitioner to go home. Petitioner was threatened in so many ways with her uncle showing to him many guns. Respondent even threatened that if he should persist in going home, they will commission their military friends to harm his family. Respondent even made petitioner sign a declaration that if he should perish, the authorities should look for him at his parents[ ]and relatives[ ]houses. Sometime in June of 1996, petitioner was able to escape and he went home. He told his parents about his predicament and they forgave him and supported him by giving him military escort. Petitioner, however, did not inform them that he signed a marriage contract with respondent. When they knew about it[,] petitioner was referred for counseling. Petitioner[,] after the counseling[,] tried to contact respondent. Petitioner offered her to live instead to[sic] the home of petitioners parents while they are still studying. Respondent refused the idea and claimed that she would only live with him if they will have a separate home of their own and be away from his parents. She also intimated to petitioner that he should already get his share of whatever he would inherit from his parents so they can start a new life. Respondent demanded these not knowing [that] the petitioner already settled his differences with his own family. When respondent refused to live with petitioner where he chose for them to stay, petitioner decided to tell her to stop harassing the home of his parents. He told her already that he was disinherited and since he also does not have a job, he would not be able to support her. After knowing that petitioner does not have any money anymore, respondent stopped tormenting petitioner and informed petitioner that they should live separate lives. The said relationship between Edward and Rowena is said to be undoubtedly in the wreck and weaklyfounded. The break-up was caused by both parties[] unreadiness to commitment and their young age. He was still in the state of finding his fate and fighting boredom, while she was still egocentrically involved with herself. TESTS ADMINISTERED: Revised Beta Examination Bender Visual Motor Gestalt Test Draw A Person Test Rorschach Psychodiagnostic Test Sachs Sentence Completion Test MMPI TEST RESULTS & EVALUATION: Both petitioner and respondent are dubbed to be emotionally immature and recklessly impulsive upon swearing to their marital vows as each of them was motivated by different notions on marriage. Edward Kenneth Ngo Te, the petitioner in this case[,] is said to be still unsure and unready so as to commit himself to marriage. He is still founded to be on the search of what he wants in life. He is absconded as an introvert as he is not really sociable and displays a lack of interest in social interactions and mingling with other individuals. He is seen too akin to this kind of lifestyle that he finds it boring and uninteresting to commit himself to a relationship especially to that of respondent, as aggravated by her dangerously aggressive moves. As he is more of the reserved and timid type of person, as he prefer to be religiously attached and spend a solemn time alone. ROWENA GUTIERREZ YU-TE, the respondent, is said to be of the aggressive-rebellious type of woman. She is seen to be somewhat exploitative in her [plight] for a life of wealth and glamour. She is seen to take move on marriage as she thought that her marriage with petitioner will bring her good fortune because he is part of a rich family. In order to have her dreams realized, she used force and threats knowing that [her] husband is somehow weak-willed. Upon the realization that there is really no chance for wealth, she gladly finds her way out of the relationship. REMARKS: 513

University of the Cordilleras College of Law First Year C S.Y. 2013 - 2014 Before going to marriage, one should really get to know himself and marry himself before submitting to marital vows. Marriage should not be taken out of intuition as it is profoundly a serious institution solemnized by religious and law. In the case presented by petitioner and respondent[,] (sic) it is evidently clear that both parties have impulsively taken marriage for granted as they are still unaware of their own selves. He is extremely introvert to the point of weakening their relationship by his weak behavioral disposition. She, on the other hand[,] is extremely exploitative and aggressive so as to be unlawful, insincere and undoubtedly uncaring in her strides toward convenience. It is apparent that she is suffering the grave, severe, and incurable presence of Narcissistic and Antisocial Personality Disorder that started since childhood and only manifested during marriage. Both parties display psychological incapacities that made marriage a big mistake for them to take. The trial court, on July 30, 2001, rendered its Decision declaring the marriage of the parties null and void on the ground that both parties were psychologically incapacitated to comply with the essential marital obligations. The Republic, represented by the OSG, timely filed its notice of appeal. On review, the appellate court, in the assailed August 5, 2003 Decision in CA-G.R. CV No. 71867, reversed and set aside the trial courts ruling. It ruled that petitioner failed to prove the psychological incapacity of respondent. The clinical psychologist did not personally examine respondent, and relied only on the information provided by petitioner. Further, the psychological incapacity was not shown to be attended by gravity, juridical antecedence and incurability. In sum, the evidence adduced fell short of the requirements stated in Republic v. Court of Appeals and Molina needed for the declaration of nullity of the marriage under Article 36 of the Family Code. The CA faulted the lower court for rendering the decision without the required certification of the OSG briefly stating therein the OSGs reasons for its agreement with or opposition to, as the case may be, the petition. The CA later denied petitioners motion for reconsideration in the likewise assailed January 19, 2004 Resolution. Dissatisfied, petitioner filed before this Court the instant petition for review on certiorari. On June 15, 2005, the Court gave due course to the petition and required the parties to submit their respective memoranda. In his memorandum, petitioner argues that the CA erred in substituting its own judgment for that of the trial court. He posits that the RTC declared the marriage void, not only bec ause of respondents psychological incapacity, but rather due to both parties psychological incapacity. Petitioner also points out that there is no requirement for the psychologist to personally examine respondent. Further, he avers that the OSG is bound by the actions of the OCP because the latter represented it during the trial; and it had been furnished copies of all the pleadings, the trial court orders and notices. For its part, the OSG contends in its memorandum, that the annulment petition filed before the RTC contains no statement of the essential marital obligations that the parties failed to comply with. The root cause of the psychological incapacity was likewise not alleged in the petition; neither was it medically or clinically identified. The purported incapacity of both parties was not shown to be medically or clinically permanent or incurable. And the clinical psychologist did not personally examine the respondent. Thus, the OSG concludes that the requirements in Molina were not satisfied. The Court now resolves the singular issue of whether, based on Article 36 of the Family Code, the marriage between the parties is null and void. I. We begin by examining the provision, tracing its origin and charting the development of jurisprudence interpreting it. Article 36 of the Family Code provides: Article 36. A marriage contracted by any party who, at the time of the celebration, was psychologically incapacitated to comply with the essential marital obligations of marriage, shall likewise be void even if such incapacity becomes manifest only after its solemnization. As borne out by the deliberations of the Civil Code Revision Committee that drafted the Family Code, Article 36 was based on grounds available in the Canon Law. Thus, Justice Flerida Ruth P. Romero elucidated in her separate opinion in Santos v. Court of Appeals: 514

University of the Cordilleras College of Law First Year C S.Y. 2013 - 2014 However, as a member of both the Family Law Revision Committee of the Integrated Bar of the Philippines and the Civil Code Revision Commission of the UP Law Center, I wish to add some observations. The letter dated April 15, 1985 of then Judge Alicia V. Sempio-Diy written in behalf of the Family Law and Civil Code Revision Committee to then Assemblywoman Mercedes Cojuangco-Teodoro traced the background of the inclusion of the present Article 36 in the Family Code. "During its early meetings, the Family Law Committee had thought of including a chapter on absolute divorce in the draft of a new Family Code (Book I of the Civil Code) that it had been tasked by the IBP and the UP Law Center to prepare. In fact, some members of the Committee were in favor of a no-fault divorce between the spouses after a number of years of separation, legal or de facto. Justice J.B.L. Reyes was then requested to prepare a proposal for an action for dissolution of marriage and the effects thereof based on two grounds: (a) five continuous years of separation between the spouses, with or without a judicial decree of legal separation, and (b) whenever a married person would have obtained a decree of absolute divorce in another country. Actually, such a proposal is one for absolute divorce but called by another name. Later, even the Civil Code Revision Committee took time to discuss the proposal of Justice Reyes on this matter. Subsequently, however, when the Civil Code Revision Committee and Family Law Committee started holding joint meetings on the preparation of the draft of the New Family Code, they agreed and formulated the definition of marriage as a special contract of permanent partnership between a man and a woman entered into in accordance with law for the establishment of conjugal and family life. It is an inviolable social institution whose nature, consequences, and incidents are governed by law and not subject to stipulation, except that marriage settlements may fix the property relations during the marriage within the limits provided by law. With the above definition, and considering the Christian traditional concept of marriage of the Filipino people as a permanent, inviolable, indissoluble social institution upon which the family and society are founded, and also realizing the strong opposition that any provision on absolute divorce would encounter from the Catholic Church and the Catholic sector of our citizenry to whom the great majority of our people belong, the two Committees in their joint meetings did not pursue the idea of absolute divorce and, instead, opted for an action for judicial declaration of invalidity of marriage based on grounds available in the Canon Law. It was thought that such an action would not only be an acceptable alternative to divorce but would also solve the nagging problem of church annulments of marriages on grounds not recognized by the civil law of the State. Justice Reyes was, thus, requested to again prepare a draft of provisions on such action for celebration of invalidity of marriage. Still later, to avoid the overlapping of provisions on void marriages as found in the present Civil Code and those proposed by Justice Reyes on judicial declaration of invalidity of marriage on grounds similar to the Canon Law, the two Committees now working as a Joint Committee in the preparation of a New Family Code decided to consolidate the present provisions on void marriages with the proposals of Justice Reyes. The result was the inclusion of an additional kind of void marriage in the enumeration of void marriages in the present Civil Code, to wit: (7) those marriages contracted by any party who, at the time of the celebration, was wanting in the sufficient use of reason or judgment to understand the essential nature of marriage or was psychologically or mentally incapacitated to discharge the essential marital obligations, even if such lack or incapacity is made manifest after the celebration. as well as the following implementing provisions: Art. 32. The absolute nullity of a marriage may be invoked or pleaded only on the basis of a final judgment declaring the marriage void, without prejudice to the provision of Article 34. Art. 33. The action or defense for the declaration of the absolute nullity of a marriage shall not prescribe. xxxxxxxxx It is believed that many hopelessly broken marriages in our country today may already be dissolved or annulled on the grounds proposed by the Joint Committee on declaration of nullity as well as annulment of marriages, thus rendering an absolute divorce law unnecessary. In fact, during a conference with Father Gerald Healy of the Ateneo University, as well as another meeting with Archbishop Oscar Cruz of the 515

University of the Cordilleras College of Law First Year C S.Y. 2013 - 2014 Archdiocese of Pampanga, the Joint Committee was informed that since Vatican II, the Catholic Church has been declaring marriages null and void on the ground of "lack of due discretion" for causes that, in other jurisdictions, would be clear grounds for divorce, like teen-age or premature marriages; marriage to a man who, because of some personality disorder or disturbance, cannot support a family; the foolish or ridiculous choice of a spouse by an otherwise perfectly normal person; marriage to a woman who refuses to cohabit with her husband or who refuses to have children. Bishop Cruz also informed the Committee that they have found out in tribunal work that a lot of machismo among husbands are manifestations of their sociopathic personality anomaly, like inflicting physical violence upon their wives, constitutional indolence or laziness, drug dependence or addiction, and psychosexual anomaly. In her separate opinion in Molina, she expounded: At the Committee meeting of July 26, 1986, the draft provision read: "(7) Those marriages contracted by any party who, at the time of the celebration, was wanting in the sufficient use of reason or judgment to understand the essential nature of marriage or was psychologically or mentally incapacitated to discharge the essential marital obligations, even if such lack of incapacity is made manifest after the celebration." The twists and turns which the ensuing discussion took finally produced the following revised provision even before the session was over: "(7) That contracted by any party who, at the time of the celebration, was psychologically incapacitated to discharge the essential marital obligations, even if such lack or incapacity becomes manifest after the celebration." Noticeably, the immediately preceding formulation above has dropped any reference to "wanting in the sufficient use of reason or judgment to understand the essential nature of marriage" and to "mentally incapacitated." It was explained that these phrases refer to "defects in the mental faculties vitiating consent, which is not the idea . . . but lack of appreciation of one's marital obligation." There being a defect in consent, "it is clear that it should be a ground for voidable marriage because there is the appearance of consent and it is capable of convalidation for the simple reason that there are lucid intervals and there are cases when the insanity is curable . . . Psychological incapacity does not refer to mental faculties and has nothing to do with consent; it refers to obligations attendant to marriage." My own position as a member of the Committee then was that psychological incapacity is, in a sense, insanity of a lesser degree. As to the proposal of Justice Caguioa to use the term "psychological or mental impotence," Archbishop Oscar Cruz opined in the earlier February 9, 1984 session that this term "is an invention of some churchmen who are moralists but not canonists, that is why it is considered a weak phrase." He said that the Code of Canon Law would rather express it as "psychological or mental incapacity to discharge . . ." Justice Ricardo C. Puno opined that sometimes a person may be psychologically impotent with one but not with another. One of the guidelines enumerated in the majority opinion for the interpretation and application of Art. 36 is: "Such incapacity must also be shown to be medically or clinically permanent or incurable. Such incurability may be absolute or even relative only in regard to the other spouse, not necessarily absolutely against everyone of the same sex." The Committee, through Prof. Araceli T. Barrera, considered the inclusion of the phrase "and is incurable" but Prof. Esteban B. Bautista commented that this would give rise to the question of how they will determine curability and Justice Caguioa agreed that it would be more problematic. Yet, the possibility that one may be cured after the psychological incapacity becomes manifest after the marriage was not ruled out by Justice Puno and Justice Alice Sempio-Diy. Justice Caguioa suggested that the remedy was to allow the afflicted spouse to remarry. For clarity, the Committee classified the bases for determining void marriages, viz.: 1. lack of one or more of the essential requisites of marriage as contract; 516

University of the Cordilleras College of Law First Year C S.Y. 2013 - 2014 2. reasons of public policy; 3. special cases and special situations. The ground of psychological incapacity was subsumed under "special cases and special situations," hence, its special treatment in Art. 36 in the Family Code as finally enacted. Nowhere in the Civil Code provisions on Marriage is there a ground for avoiding or annulling marriages that even comes close to being psychological in nature. Where consent is vitiated due to circumstances existing at the time of the marriage, such marriage which stands valid until annulled is capable of ratification or convalidation. On the other hand, for reasons of public policy or lack of essential requisites, some marriages are void from the beginning. With the revision of Book I of the Civil Code, particularly the provisions on Marriage, the drafters, now open to fresh winds of change in keeping with the more permissive mores and practices of the time, took a leaf from the relatively liberal provisions of Canon Law. Canon 1095 which states, inter alia, that the following persons are incapable of contracting marriage: "3. (those) who, because of causes of a psychological nature, are unable to assume the essential obligations of marriage" provided the model for what is now Art. 36 of the Family Code: "A marriage contracted by any party who, at the time of the celebration, was psychologically incapacitated to comply with the essential marital obligations of marriage, shall likewise be void even if such incapacity becomes manifest only after its solemnization." It bears stressing that unlike in Civil Law, Canon Law recognizes only two types of marriages with respect to their validity: valid and void. Civil Law, however, recognizes an intermediate state, the voidable or annullable marriages. When the Ecclesiastical Tribunal "annuls" a marriage, it actually declares the marriage null and void, i.e., it never really existed in the first place, for a valid sacramental marriage can never be dissolved. Hence, a properly performed and consummated marriage between two living Roman Catholics can only be nullified by the formal annulment process which entails a full tribunal procedure with a Court selection and a formal hearing. Such so-called church "annulments" are not recognized by Civil Law as severing the marriage ties as to capacitate the parties to enter lawfully into another marriage. The grounds for nullifying civil marriage, not being congruent with those laid down by Canon Law, the former being more strict, quite a number of married couples have found themselves in limbofreed from the marriage bonds in the eyes of the Catholic Church but yet unable to contract a valid civil marriage under state laws. Heedless of civil law sanctions, some persons contract new marriages or enter into live-in relationships. It was precisely to provide a satisfactory solution to such anomalous situations that the Civil Law Revision Committee decided to engraft the Canon Law concept of psychological incapacity into the Family Codeand classified the same as a ground for declaring marriages void ab initio or totally inexistent from the beginning. A brief historical note on the Old Canon Law (1917). This Old Code, while it did not provide directly for psychological incapacity, in effect, recognized the same indirectly from a combination of three old canons: "Canon #1081 required persons to be capable according to law in order to give valid consent; Canon #1082 required that persons be at least not ignorant of the major elements required in marriage; and Canon #1087 (the force and fear category) required that internal and external freedom be present in order for consent to be valid. This line of interpretation produced two distinct but related grounds for annulment called lack of due discretion and lack of due competence. Lack of due discretion means that the person did not have the ability to give valid consent at the time of the wedding and, therefore, the union is invalid. Lack of due competence means that the person was incapable of carrying out the obligations of the promise he or she made during the wedding ceremony." Favorable annulment decisions by the Roman Rota in the 1950s and 1960s involving sexual disorders such as homosexuality and nymphomania laid the foundation for a broader approach to the kind of proof necessary for psychological grounds for annulment. The Rota had reasoned for the first time in several 517

University of the Cordilleras College of Law First Year C S.Y. 2013 - 2014 cases that the capacity to give valid consent at the time of marriage was probably not present in persons who had displayed such problems shortly after the marriage. The nature of this change was nothing short of revolutionary. Once the Rota itself had demonstrated a cautious willingness to use this kind of hindsight, the way was paved for what came after 1970. Diocesan Tribunals began to accept proof of serious psychological problems that manifested themselves shortly after the ceremony as proof of an inability to give valid consent at the time of the ceremony. Interestingly, the Committee did not give any examples of psychological incapacity for fear that by so doing, it might limit the applicability of the provision under the principle of ejusdem generis. The Committee desired that the courts should interpret the provision on a case-to-case basis; guided by experience, the findings of experts and researchers in psychological disciplines, and by decisions of church tribunals which, although not binding on the civil courts, may be given persuasive effect since the provision itself was taken from the Canon Law. The law is then so designed as to allow some resiliency in its application. Yet, as held in Santos, the phrase "psychological incapacity" is not meant to comprehend all possible cases of psychoses. It refers to no less than a mental (not physical) incapacity that causes a party to be truly noncognitive of the basic marital covenants that concomitantly must be assumed and discharged by the parties to the marriage which, as expressed by Article 68 of the Family Code, include their mutual obligations to live together, observe love, respect and fidelity; and render help and support. The intendment of the law has been to confine it to the most serious of cases of personality disorders clearly demonstrative of an utter insensitivity or inability to give meaning and significance to the marriage. This interpretation is, in fact, consistent with that in Canon Law, thus: 3.5.3.1. The Meaning of Incapacity to Assume. A sharp conceptual distinction must be made between the second and third paragraphs of C.1095, namely between the grave lack of discretionary judgment and the incapacity to assume the essential obligation. Mario Pompedda, a rotal judge, explains the difference by an ordinary, if somewhat banal, example. Jose wishes to sell a house to Carmela, and on the assumption that they are capable according to positive law to enter such contract, there remains the object of the contract, viz, the house. The house is located in a different locality, and prior to the conclusion of the contract, the house was gutted down by fire unbeknown to both of them. This is the hypothesis contemplated by the third paragraph of the canon. The third paragraph does not deal with the psychological process of giving consent because it has been established a priori that both have such a capacity to give consent, and they both know well the object of their consent [the house and its particulars]. Rather, C.1095.3 deals with the object of the consent/contract which does not exist. The contract is invalid because it lacks its formal object. The consent as a psychological act is both valid and sufficient. The psychological act, however, is directed towards an object which is not available. Urbano Navarrete summarizes this distinction: the third paragraph deals not with the positing of consent but with positing the object of consent. The person may be capable of positing a free act of consent, but he is not capable of fulfilling the responsibilities he assumes as a result of the consent he elicits. Since the address of Pius XII to the auditors of the Roman Rota in 1941 regarding psychic incapacity with respect to marriage arising from pathological conditions, there has been an increasing trend to understand as ground of nullity different from others, the incapacity to assume the essential obligations of marriage, especially the incapacity which arises from sexual anomalies. Nymphomania is a sample which ecclesiastical jurisprudence has studied under this rubric. The problem as treated can be summarized, thus: do sexual anomalies always and in every case imply a grave psychopathological condition which affects the higher faculties of intellect, discernment, and freedom; or are there sexual anomalies that are purely so that is to say, they arise from certain physiological dysfunction of the hormonal system, and they affect the sexual condition, leaving intact the higher faculties however, so that these persons are still capable of free human acts. The evidence from the empirical sciences is abundant that there are certain anomalies of a sexual nature which may impel a person towards sexual activities which are not normal, either with respect to its frequency [nymphomania, satyriasis] or to the nature of the activity itself [sadism, masochism, homosexuality]. However, these anomalies notwithstanding, it is altogether possible that the higher faculties remain intact such that a person so afflicted continues to have an adequate understanding of what marriage is and of the gravity of its responsibilities. In fact, he can choose marriage freely. The question though is whether such a person can assume those responsibilities which he cannot fulfill, although he may be able to understand them. In this latter hypothesis, the incapacity to assume the essential obligations of marriage issues from the incapacity to posit the object of consent, rather than the incapacity to posit consent itself. 518

University of the Cordilleras College of Law First Year C S.Y. 2013 - 2014 Ecclesiastical jurisprudence has been hesitant, if not actually confused, in this regard. The initial steps taken by church courts were not too clear whether this incapacity is incapacity to posit consent or incapacity to posit the object of consent. A case c. Pinna, for example, arrives at the conclusion that the intellect, under such an irresistible impulse, is prevented from properly deliberating and its judgment lacks freedom. This line of reasoning supposes that the intellect, at the moment of consent, is under the influence of this irresistible compulsion, with the inevitable conclusion that such a decision, made as it was under these circumstances, lacks the necessary freedom. It would be incontrovertible that a decision made under duress, such as this irresistible impulse, would not be a free act. But this is precisely the question: is it, as a matter of fact, true that the intellect is always and continuously under such an irresistible compulsion? It would seem entirely possible, and certainly more reasonable, to think that there are certain cases in which one who is sexually hyperaesthetic can understand perfectly and evaluate quite maturely what marriage is and what it implies; his consent would be juridically ineffective for this one reason that he cannot posit the object of consent, the exclusive jus in corpus to be exercised in a normal way and with usually regularity. It would seem more correct to say that the consent may indeed be free, but is juridically ineffective because the party is consenting to an object that he cannot deliver. The house he is selling was gutted down by fire. 3.5.3.2. Incapacity as an Autonomous Ground. Sabattani seems to have seen his way more clearly through this tangled mess, proposing as he did a clear conceptual distinction between the inability to give consent on the one hand, and the inability to fulfill the object of consent, on the other. It is his opinion that nymphomaniacs usually understand the meaning of marriage, and they are usually able to evaluate its implications. They would have no difficulty with positing a free and intelligent consent. However, such persons, capable as they are of eliciting an intelligent and free consent, experience difficulty in another sphere: delivering the object of the consent. Anne, another rotal judge, had likewise treated the difference between the act of consenting and the act of positing the object of consent from the point of view of a person afflicted with nymphomania. According to him, such an affliction usually leaves the process of knowing and understanding and evaluating intact. What it affects is the object of consent: the delivering of the goods. 3.5.3.3 Incapacity as Incapacity to Posit the Object of Consent. From the selected rotal jurisprudence cited, supra, it is possible to see a certain progress towards a consensus doctrine that the incapacity to assume the essential obligations of marriage (that is to say, the formal object of consent) can coexist in the same person with the ability to make a free decision, an intelligent judgment, and a mature evaluation and weighing of things. The decision coram Sabattani concerning a nymphomaniac affirmed that such a spouse can have difficulty not only with regard to the moment of consent but also, and especially, with regard to the matrimonium in facto esse. The decision concludes that a person in such a condition is incapable of assuming the conjugal obligation of fidelity, although she may have no difficulty in understanding what the obligations of marriage are, nor in the weighing and evaluating of those same obligations. Prior to the promulgation of the Code of Canon Law in 1983, it was not unusual to refer to this ground as moral impotence or psychic impotence, or similar expressions to express a specific incapacity rooted in some anomalies and disorders in the personality. These anomalies leave intact the faculties of the will and the intellect. It is qualified as moral or psychic, obviously to distinguish it from the impotence that constitutes the impediment dealt with by C.1084. Nonetheless, the anomalies render the subject incapable of binding himself in a valid matrimonial pact, to the extent that the anomaly renders that person incapable of fulfilling the essential obligations. According to the principle affirmed by the long tradition of moral theology: nemo ad impossibile tenetur. xxxx 3.5.3.5 Indications of Incapacity. There is incapacity when either or both of the contractants are not capable of initiating or maintaining this consortium. One immediately thinks of those cases where one of the parties is so self-centered [e.g., a narcissistic personality] that he does not even know how to begin a union with the other, let alone how to maintain and sustain such a relationship. A second incapacity could be due to the fact that the spouses are incapable of beginning or maintaining a heterosexual consortium, which goes to the very substance of matrimony. Another incapacity could arise when a spouse is unable to concretize the good of himself or of the other party. The canon speaks, not of the bonum partium, but of the bonum conjugum. A spouse who is capable only of realizing or contributing to the good of the other party qua persona rather than qua conjunx would be deemed incapable of contracting marriage. Such would be the case of a person who may be quite capable of procuring the economic good and the 519

University of the Cordilleras College of Law First Year C S.Y. 2013 - 2014 financial security of the other, but not capable of realizing the bonum conjugale of the other. These are general strokes and this is not the place for detained and individual description. A rotal decision c. Pinto resolved a petition where the concrete circumstances of the case concerns a person diagnosed to be suffering from serious sociopathy. He concluded that while the respondent may have understood, on the level of the intellect, the essential obligations of marriage, he was not capable of assuming them because of his "constitutional immorality." Stankiewicz clarifies that the maturity and capacity of the person as regards the fulfillment of responsibilities is determined not only at the moment of decision but also and especially during the moment of execution of decision. And when this is applied to constitution of the marital consent, it means that the actual fulfillment of the essential obligations of marriage is a pertinent consideration that must be factored into the question of whether a person was in a position to assume the obligations of marriage in the first place. When one speaks of the inability of the party to assume and fulfill the obligations, one is not looking at matrimonium in fieri, but also and especially at matrimonium in facto esse. In [the] decision of 19 Dec. 1985, Stankiewicz collocated the incapacity of the respondent to assume the essential obligations of marriage in the psychic constitution of the person, precisely on the basis of his irresponsibility as regards money and his apathy as regards the rights of others that he had violated. Interpersonal relationships are invariably disturbed in the presence of this personality disorder. A lack of empathy (inability to recognize and experience how others feel) is common. A sense of entitlement, unreasonable expectation, especially favorable treatment, is usually present. Likewise common is interpersonal exploitativeness, in which others are taken advantage of in order to achieve ones ends. Authors have made listings of obligations considered as essential matrimonial obligations. One of them is the right to the communio vitae. This and their corresponding obligations are basically centered around the good of the spouses and of the children. Serious psychic anomalies, which do not have to be necessarily incurable, may give rise to the incapacity to assume any, or several, or even all of these rights. There are some cases in which interpersonal relationship is impossible. Some characteristic features of inability for interpersonal relationships in marriage include affective immaturity, narcissism, and antisocial traits. Marriage and Homosexuality. Until 1967, it was not very clear under what rubric homosexuality was understood to be invalidating of marriage that is to say, is homosexuality invalidating because of the inability to evaluate the responsibilities of marriage, or because of the inability to fulfill its obligations. Progressively, however, rotal jurisprudence began to understand it as incapacity to assume the obligations of marriage so that by 1978, Parisella was able to consider, with charity, homosexuality as an autonomous ground of nullity. This is to say that a person so afflicted is said to be unable to assume the essential obligations of marriage. In this same rotal decision, the object of matrimonial consent is understood to refer not only to the jus in corpus but also the consortium totius vitae. The third paragraph of C.1095 [incapacity to assume the essential obligations of marriage] certainly seems to be the more adequate juridical structure to account for the complex phenomenon that homosexuality is. The homosexual is not necessarily impotent because, except in very few exceptional cases, such a person is usually capable of full sexual relations with the spouse. Neither is it a mental infirmity, and a person so afflicted does not necessarily suffer from a grave lack of due discretion because this sexual anomaly does not by itself affect the critical, volitive, and intellectual faculties. Rather, the homosexual person is unable to assume the responsibilities of marriage because he is unable to fulfill this object of the matrimonial contract. In other words, the invalidity lies, not so much in the defect of consent, as in the defect of the object of consent. 3.5.3.6 Causes of Incapacity. A last point that needs to be addressed is the source of incapacity specified by the canon: causes of a psychological nature. Pompedda proffers the opinion that the clause is a reference to the personality of the contractant. In other words, there must be a reference to the psychic part of the person. It is only when there is something in the psyche or in the psychic constitution of the person which impedes his capacity that one can then affirm that the person is incapable according to the hypothesis contemplated by C.1095.3. A person is judged incapable in this juridical sense only to the extent that he is found to have something rooted in his psychic constitution which impedes the assumption of these obligations. A bad habit deeply engrained in ones consciousness would not seem to qualify to be a source of this invalidating incapacity. The difference being that there seems to be some freedom, however remote, in the development of the habit, while one accepts as given ones psychic constitution. It would seem then that the law insists that the source of the incapacity must be one which is not the fruit of some degree of freedom.

520

University of the Cordilleras College of Law First Year C S.Y. 2013 - 2014 Conscious of the laws intention that it is the courts, on a case-to-case basis, that should determine whether a party to a marriage is psychologically incapacitated, the Court, in sustaining the lower courts judgment of annulment in Tuason v. Court of Appeals, ruled that the findings of the trial court are final and binding on the appellate courts. Again, upholding the trial courts findings and declaring that its decision was not a judgment on the pleadings, the Court, in Tsoi v. Court of Appeals, explained that when private respondent testified under oath before the lower court and was cross-examined by the adverse party, she thereby presented evidence in the form of testimony. Importantly, the Court, aware of parallel decisions of Catholic marriage tribunals, ruled that the senseless and protracted refusal of one of the parties to fulfill the marital obligation of procreating children is equivalent to psychological incapacity. The resiliency with which the concept should be applied and the case-to-case basis by which the provision should be interpreted, as so intended by its framers, had, somehow, been rendered ineffectual by the imposition of a set of strict standards in Molina, thus: From their submissions and the Court's own deliberations, the following guidelines in the interpretation and application of Art. 36 of the Family Code are hereby handed down for the guidance of the bench and the bar: (1) The burden of proof to show the nullity of the marriage belongs to the plaintiff. Any doubt should be resolved in favor of the existence and continuation of the marriage and against its dissolution and nullity. This is rooted in the fact that both our Constitution and our laws cherish the validity of marriage and unity of the family. Thus, our Constitution devotes an entire Article on the Family, recognizing it "as the foundation of the nation." It decrees marriage as legally "inviolable," thereby protecting it from dissolution at the whim of the parties. Both the family and marriage are to be "protected" by the state. The Family Code echoes this constitutional edict on marriage and the family and emphasizes their permanence, inviolability and solidarity. (2) The root cause of the psychological incapacity must be (a) medically or clinically identified, (b) alleged in the complaint, (c) sufficiently proven by experts and (d) clearly explained in the decision. Article 36 of the Family Code requires that the incapacity must be psychological not physical, although its manifestations and/or symptoms may be physical. The evidence must convince the court that the parties, or one of them, was mentally or psychically ill to such an extent that the person could not have known the obligations he was assuming, or knowing them, could not have given valid assumption thereof. Although no example of such incapacity need be given here so as not to limit the application of the provision under the principle of ejusdem generis, nevertheless such root cause must be identified as a psychological illness and its incapacitating nature fully explained. Expert evidence may be given by qualified psychiatrists and clinical psychologists. (3) The incapacity must be proven to be existing at "the time of the celebration" of the marriage. The evidence must show that the illness was existing when the parties exchanged their "I do's." The manifestation of the illness need not be perceivable at such time, but the illness itself must have attached at such moment, or prior thereto. (4) Such incapacity must also be shown to be medically or clinically permanent or incurable. Such incurability may be absolute or even relative only in regard to the other spouse, not necessarily absolutely against everyone of the same sex. Furthermore, such incapacity must be relevant to the assumption of marriage obligations, not necessarily to those not related to marriage, like the exercise of a profession or employment in a job. Hence, a pediatrician may be effective in diagnosing illnesses of children and prescribing medicine to cure them but may not be psychologically capacitated to procreate, bear and raise his/her own children as an essential obligation of marriage. (5) Such illness must be grave enough to bring about the disability of the party to assume the essential obligations of marriage. Thus, "mild characterological peculiarities, mood changes, occasional emotional outbursts" cannot be accepted as root causes. The illness must be shown as downright incapacity or inability, not a refusal, neglect or difficulty, much less ill will. In other 521

University of the Cordilleras College of Law First Year C S.Y. 2013 - 2014 words, there is a natal or supervening disabling factor in the person, an adverse integral element in the personality structure that effectively incapacitates the person from really accepting and thereby complying with the obligations essential to marriage. (6) The essential marital obligations must be those embraced by Articles 68 up to 71 of the Family Code as regards the husband and wife as well as Articles 220, 221 and 225 of the same Code in regard to parents and their children. Such non-complied marital obligation(s) must also be stated in the petition, proven by evidence and included in the text of the decision. (7) Interpretations given by the National Appellate Matrimonial Tribunal of the Catholic Church in the Philippines, while not controlling or decisive, should be given great respect by our courts. It is clear that Article 36 was taken by the Family Code Revision Committee from Canon 1095 of the New Code of Canon Law, which became effective in 1983 and which provides: "The following are incapable of contracting marriage: Those who are unable to assume the essential obligations of marriage due to causes of psychological nature." Since the purpose of including such provision in our Family Code is to harmonize our civil laws with the religious faith of our people, it stands to reason that to achieve such harmonization, great persuasive weight should be given to decisions of such appellate tribunal. Ideally subject to our law on evidencewhat is decreed as canonically invalid should also be decreed civilly void. This is one instance where, in view of the evident source and purpose of the Family Code provision, contemporaneous religious interpretation is to be given persuasive effect. Here, the State and the Churchwhile remaining independent, separate and apart from each othershall walk together in synodal cadence towards the same goal of protecting and cherishing marriage and the family as the inviolable base of the nation. (8) The trial court must order the prosecuting attorney or fiscal and the Solicitor General to appear as counsel for the state. No decision shall be handed down unless the Solicitor General issues a certification, which will be quoted in the decision, briefly stating therein his reasons for his agreement or opposition, as the case may be, to the petition. The Solicitor General, along with the prosecuting attorney, shall submit to the court such certification within fifteen (15) days from the date the case is deemed submitted for resolution of the court. The Solicitor General shall discharge the equivalent function of the defensor vinculi contemplated under Canon 1095. Noteworthy is that in Molina, while the majority of the Courts membership concurred in the ponencia of then Associate Justice (later Chief Justice) Artemio V. Panganiban, three justices concurred "in the result" and another threeincluding, as aforesaid, Justice Romerotook pains to compose their individual separate opinions. Then Justice Teodoro R. Padilla even emphasized that "each case must be judged, not on the basis of a priori assumptions, predelictions or generalizations, but according to its own facts. In the field of psychological incapacity as a ground for annulment of marriage, it is trite to say that no case is on all fours with another case. The trial judge must take pains in examining the factual milieu and the appellate court must, as much as possible, avoid substituting its own judgment for that of the trial court." Predictably, however, in resolving subsequent cases, the Court has applied the aforesaid standards, without too much regard for the laws clear intention that each case is to be treated differently, as "courts should interpret the provision on a case-to-case basis; guided by experience, the findings of experts and researchers in psychological disciplines, and by decisions of church tribunals." In hindsight, it may have been inappropriate for the Court to impose a rigid set of rules, as the one in Molina, in resolving all cases of psychological incapacity. Understandably, the Court was then alarmed by the deluge of petitions for the dissolution of marital bonds, and was sensitive to the OSGs exaggeration of Article 36 as the "most liberal divorce procedure in the world." The unintended consequences of Molina, however, has taken its toll on people who have to live with deviant behavior, moral insanity and sociopathic personality anomaly, which, like termites, consume little by little the very foundation of their families, our basic social institutions. Far from what was intended by the Court, Molina has become a strait-jacket, forcing all sizes to fit into and be bound by it. Wittingly or unwittingly, the Court, in conveniently applying Molina, has allowed diagnosed sociopaths, schizophrenics, nymphomaniacs, narcissists and the like, to continuously debase and pervert the sanctity of marriage.

522

University of the Cordilleras College of Law First Year C S.Y. 2013 - 2014 Ironically, the Roman Rota has annulled marriages on account of the personality disorders of the said individuals. The Court need not worry about the possible abuse of the remedy provided by Article 36, for there are ample safeguards against this contingency, among which is the intervention by the State, through the public prosecutor, to guard against collusion between the parties and/or fabrication of evidence. The Court should rather be alarmed by the rising number of cases involving marital abuse, child abuse, domestic violence and incestuous rape. In dissolving marital bonds on account of either partys psychological incapacity, the Court is n ot demolishing the foundation of families, but it is actually protecting the sanctity of marriage, because it refuses to allow a person afflicted with a psychological disorder, who cannot comply with or assume the essential marital obligations, from remaining in that sacred bond. It may be stressed that the infliction of physical violence, constitutional indolence or laziness, drug dependence or addiction, and psychosexual anomaly are manifestations of a sociopathic personality anomaly. Let it be noted that in Article 36, there is no marriage to speak of in the first place, as the same is void from the very beginning. To indulge in imagery, the declaration of nullity under Article 36 will simply provide a decent burial to a stillborn marriage. The prospect of a possible remarriage by the freed spouses should not pose too much of a concern for the Court. First and foremost, because it is none of its business. And second, because the judicial declaration of psychological incapacity operates as a warning or a lesson learned. On one hand, the normal spouse would have become vigilant, and never again marry a person with a personality disorder. On the other hand, a would-be spouse of the psychologically incapacitated runs the risk of the latters disorder recurring in their marriage. Lest it be misunderstood, we are not suggesting the abandonment of Molina in this case. We simply declare that, as aptly stated by Justice Dante O. Tinga in Antonio v. Reyes, there is need to emphasize other perspectives as well which should govern the disposition of petitions for declaration of nullity under Article 36. At the risk of being redundant, we reiterate once more the principle that each case must be judged, not on the basis of a priori assumptions, predilections or generalizations but according to its own facts. And, to repeat for emphasis, courts should interpret the provision on a case-to-case basis; guided by experience, the findings of experts and researchers in psychological disciplines, and by decisions of church tribunals. II. We now examine the instant case. The parties whirlwind relationship lasted more or less six (6) months. They met in January 1996, eloped in March, exchanged marital vows in May, and parted ways in June. The psychologist who provided expert testimony found both parties psychologically incapacitated. Petitioners behavioral pattern falls under the classification of dependent personality disorder, and respondents, that of the narcissistic and antisocial personality disorder. By the very nature of Article 36, courts, despite having the primary task and burden of decision-making, must not discount but, instead, must consider as decisive evidence the expert opinion on the psychological and mental temperaments of the parties. Justice Romero explained this in Molina, as follows: Furthermore, and equally significant, the professional opinion of a psychological expert became increasingly important in such cases. Data about the person's entire life, both before and after the ceremony, were presented to these experts and they were asked to give professional opinions about a party's mental capacity at the time of the wedding. These opinions were rarely challenged and tended to be accepted as decisive evidence of lack of valid consent. The Church took pains to point out that its new openness in this area did not amount to the addition of new grounds for annulment, but rather was an accommodation by the Church to the advances made in psychology during the past decades. There was now the expertise to provide the all-important connecting link between a marriage breakdown and premarital causes. 523

University of the Cordilleras College of Law First Year C S.Y. 2013 - 2014 During the 1970s, the Church broadened its whole idea of marriage from that of a legal contract to that of a covenant. The result of this was that it could no longer be assumed in annulment cases that a person who could intellectually understand the concept of marriage could necessarily give valid consent to marry. The ability to both grasp and assume the real obligations of a mature, lifelong commitment are now considered a necessary prerequisite to valid matrimonial consent. Rotal decisions continued applying the concept of incipient psychological incapacity, "not only to sexual anomalies but to all kinds of personality disorders that incapacitate a spouse or both spouses from assuming or carrying out the essential obligations of marriage. For marriage . . . is not merely cohabitation or the right of the spouses to each other's body for heterosexual acts, but is, in its totality the right to the community of the whole of life; i.e., the right to a developing lifelong relationship. Rotal decisions since 1973 have refined the meaning of psychological or psychic capacity for marriage as presupposing the development of an adult personality; as meaning the capacity of the spouses to give themselves to each other and to accept the other as a distinct person; that the spouses must be other oriented since the obligations of marriage are rooted in a self-giving love; and that the spouses must have the capacity for interpersonal relationship because marriage is more than just a physical reality but involves a true intertwining of personalities. The fulfillment of the obligations of marriage depends, according to Church decisions, on the strength of this interpersonal relationship. A serious incapacity for interpersonal sharing and support is held to impair the relationship and consequently, the ability to fulfill the essential marital obligations. The marital capacity of one spouse is not considered in isolation but in reference to the fundamental relationship to the other spouse. Fr. Green, in an article in Catholic Mind, lists six elements necessary to the mature marital relationship: "The courts consider the following elements crucial to the marital commitment: (1) a permanent and faithful commitment to the marriage partner; (2) openness to children and partner; (3) stability; (4) emotional maturity; (5) financial responsibility; (6) an ability to cope with the ordinary stresses and strains of marriage, etc." Fr. Green goes on to speak about some of the psychological conditions that might lead to the failure of a marriage: "At stake is a type of constitutional impairment precluding conjugal communion even with the best intentions of the parties. Among the psychic factors possibly giving rise to his or her inability to fulfill marital obligations are the following: (1) antisocial personality with its fundamental lack of loyalty to persons or sense of moral values; (2) hyperesthesia, where the individual has no real freedom of sexual choice; (3) the inadequate personality where personal responses consistently fall short of reasonable expectations. xxxx The psychological grounds are the best approach for anyone who doubts whether he or she has a case for an annulment on any other terms. A situation that does not fit into any of the more traditional categories often fits very easily into the psychological category. As new as the psychological grounds are, experts are already detecting a shift in their use. Whereas originally the emphasis was on the parties' inability to exercise proper judgment at the time of the marriage (lack of due discretion), recent cases seem to be concentrating on the parties' incapacity to assume or carry out their responsibilities and obligations as promised (lack of due competence). An advantage to using the ground of lack of due competence is that at the time the marriage was entered into civil divorce and breakup of the family almost always is proof of someone's failure to carry out marital responsibilities as promised at the time the marriage was entered into."1avvphi1 Hernandez v. Court of Appeals emphasizes the importance of presenting expert testimony to establish the precise cause of a partys psychological incapacity, and to show that it existed at the inception of the marriage. And as Marcos v. Marcos asserts, there is no requirement that the person to be declared psychologically incapacitated be personally examined by a physician, if the totality of evidence presented is enough to sustain a finding of psychological incapacity. Verily, the evidence must show a link, medical or the like, between the acts that manifest psychological incapacity and the psychological disorder itself.

524

University of the Cordilleras College of Law First Year C S.Y. 2013 - 2014 This is not to mention, but we mention nevertheless for emphasis, that the presentation of expert proof presupposes a thorough and in-depth assessment of the parties by the psychologist or expert, for a conclusive diagnosis of a grave, severe and incurable presence of psychological incapacity. Parenthetically, the Court, at this point, finds it fitting to suggest the inclusion in the Rule on Declaration of Absolute Nullity of Void Marriages and Annulment of Voidable Marriages, an option for the trial judge to refer the case to a court-appointed psychologist/expert for an independent assessment and evaluation of the psychological state of the parties. This will assist the courts, who are no experts in the field of psychology, to arrive at an intelligent and judicious determination of the case. The rule, however, does not dispense with the parties prerogative to present their own expert witnesses. Going back, in the case at bench, the psychological assessment, which we consider as adequate, produced the findings that both parties are afflicted with personality disordersto repeat, dependent personality disorder for petitioner, and narcissistic and antisocial personality disorder for respondent. We note that The Encyclopedia of Mental Health discusses personality disorders as follows A group of disorders involving behaviors or traits that are characteristic of a persons recent and longterm functioning. Patterns of perceiving and thinking are not usually limited to isolated episodes but are deeply ingrained, inflexible, maladaptive and severe enough to cause the individual mental stress or anxieties or to interfere with interpersonal relationships and normal functioning. Personality disorders are often recognizable by adolescence or earlier, continue through adulthood and become less obvious in middle or old age. An individual may have more than one personality disorder at a time. The common factor among individuals who have personality disorders, despite a variety of character traits, is the way in which the disorder leads to pervasive problems in social and occupational adjustment. Some individuals with personality disorders are perceived by others as overdramatic, paranoid, obnoxious or even criminal, without an awareness of their behaviors. Such qualities may lead to trouble getting along with other people, as well as difficulties in other areas of life and often a tendency to blame others for their problems. Other individuals with personality disorders are not unpleasant or difficult to work with but tend to be lonely, isolated or dependent. Such traits can lead to interpersonal difficulties, reduced self-esteem and dissatisfaction with life. Causes of Personality Disorders Different mental health viewpoints propose a variety of causes of personality disorders. These include Freudian, genetic factors, neurobiologic theories and brain wave activity. Freudian Sigmund Freud believed that fixation at certain stages of development led to certain personality types. Thus, some disorders as described in the Diagnostic and Statistical Manual of Mental Disorders (3d ed., rev.) are derived from his oral, anal and phallic character types. Demanding and dependent behavior (dependent and passive-aggressive) was thought to derive from fixation at the oral stage. Characteristics of obsessionality, rigidity and emotional aloofness were thought to derive from fixation at the anal stage; fixation at the phallic stage was thought to lead to shallowness and an inability to engage in intimate relationships.lawphil.net However, later researchers have found little evidence that early childhood events or fixation at certain stages of development lead to specific personality patterns. Genetic Factors Researchers have found that there may be a genetic factor involved in the etiology of antisocial and borderline personality disorders; there is less evidence of inheritance of other personality disorders. Some family, adoption and twin studies suggest that schizotypal personality may be related to genetic factors. Neurobiologic Theories In individuals who have borderline personality, researchers have found that low cerebrospinal fluid 5-hydroxyindoleacetic acid (5-HIAA) negatively correlated with measures of aggression and a past history of suicide attempts. Schizotypal personality has been associated with low platelet monoamine oxidase (MAO) activity and impaired smooth pursuit eye movement. Brain Wave Activity Abnormalities in electroencephalograph (EEG) have been reported in antisocial personality for many years; slow wave is the most widely reported abnormality. A study of borderline patients reported that 38 percent had at least marginal EEG abnormalities, compared with 19 percent in a control group.

525

University of the Cordilleras College of Law First Year C S.Y. 2013 - 2014 Types of Disorders According to the American Psychiatric Associations Diagnostic and Statistical Manual of Mental Disorders (3d ed., rev., 1987), or DSM-III-R, personality disorders are categorized into three major clusters: Cluster A: Paranoid, schizoid and schizotypal personality disorders. Individuals who have these disorders often appear to have odd or eccentric habits and traits. Cluster B: Antisocial, borderline, histrionic and narcissistic personality disorders. Individuals who have these disorders often appear overly emotional, erratic and dramatic. Cluster C: Avoidant, dependent, obsessive-compulsive and passive-aggressive personality disorders. Individuals who have these disorders often appear anxious or fearful. The DSM-III-R also lists another category, "personality disorder not otherwise specified," that can be used for other specific personality disorders or for mixed conditions that do not qualify as any of the specific personality disorders. Individuals with diagnosable personality disorders usually have long-term concerns, and thus therapy may be long-term. Dependent personality disorder is characterized in the following manner A personality disorder characterized by a pattern of dependent and submissive behavior. Such individuals usually lack self-esteem and frequently belittle their capabilities; they fear criticism and are easily hurt by others comments. At times they actually bring about dominance by others through a quest for overprotection. Dependent personality disorder usually begins in early adulthood. Individuals who have this disorder may be unable to make everyday decisions without advice or reassurance from others, may allow others to make most of their important decisions (such as where to live), tend to agree with people even when they believe they are wrong, have difficulty starting projects or doing things on their own, volunteer to do things that are demeaning in order to get approval from other people, feel uncomfortable or helpless when alone and are often preoccupied with fears of being abandoned. and antisocial personality disorder described, as follows Characteristics include a consistent pattern of behavior that is intolerant of the conventional behavioral limitations imposed by a society, an inability to sustain a job over a period of years, disregard for the rights of others (either through exploitiveness or criminal behavior), frequent physical fights and, quite commonly, child or spouse abuse without remorse and a tendency to blame others. There is often a faade of charm and even sophistication that masks disregard, lack of remorse for mistreatment of others and the need to control others. Although characteristics of this disorder describe criminals, they also may befit some individuals who are prominent in business or politics whose habits of self-centeredness and disregard for the rights of others may be hidden prior to a public scandal. During the 19th century, this type of personality disorder was referred to as moral insanity. The term described immoral, guiltless behavior that was not accompanied by impairments in reasoning.lawphil.net According to the classification system used in the Diagnostic and Statistical Manual of Mental Disorders (3d ed., rev. 1987), anti-social personality disorder is one of the four "dramatic" personality disorders, the others being borderline, histrionic and narcissistic. The seriousness of the diagnosis and the gravity of the disorders considered, the Court, in this case, finds as decisive the psychological evaluation made by the expert witness; and, thus, rules that the marriage of the parties is null and void on ground of both parties psychological incapacity. We further consider that the trial court, which had a first-hand view of the witnesses deportment, arrived at the same conclusion. Indeed, petitioner, who is afflicted with dependent personality disorder, cannot assume the essential marital obligations of living together, observing love, respect and fidelity and rendering help and support, 526

University of the Cordilleras College of Law First Year C S.Y. 2013 - 2014 for he is unable to make everyday decisions without advice from others, allows others to make most of his important decisions (such as where to live), tends to agree with people even when he believes they are wrong, has difficulty doing things on his own, volunteers to do things that are demeaning in order to get approval from other people, feels uncomfortable or helpless when alone and is often preoccupied with fears of being abandoned. As clearly shown in this case, petitioner followed everything dictated to him by the persons around him. He is insecure, weak and gullible, has no sense of his identity as a person, has no cohesive self to speak of, and has no goals and clear direction in life. Although on a different plane, the same may also be said of the respondent. Her being afflicted with antisocial personality disorder makes her unable to assume the essential marital obligations. This finding takes into account her disregard for the rights of others, her abuse, mistreatment and control of others without remorse, her tendency to blame others, and her intolerance of the conventional behavioral limitations imposed by society. Moreover, as shown in this case, respondent is impulsive and domineering; she had no qualms in manipulating petitioner with her threats of blackmail and of committing suicide. Both parties being afflicted with grave, severe and incurable psychological incapacity, the precipitous marriage which they contracted on April 23, 1996 is thus, declared null and void. WHEREFORE, premises considered, the petition for review on certiorari is GRANTED. The August 5, 2003 Decision and the January 19, 2004 Resolution of the Court of Appeals in CA-G.R. CV No. 71867 are REVERSED and SET ASIDE, and the Decision, dated July 30, 2001, REINSTATED. SO ORDERED.

527

University of the Cordilleras College of Law First Year C S.Y. 2013 - 2014 Case Digest NGO TE vs. YU-TE G.R. No. 161793 February 13, 2009 NACHURA, J.: Facts: The parties whirlwind relationship lasted more or less six (6) months. They met in January 1996, eloped in March, exchanged marital vows in May, and parted ways in June. After almost four years, on January 18, 2000, Edward filed a petition before the Regional Trial Court in Quezon City for the annulment of his marriage to Rowena on the basis of the latters psychological incapacity. The psychologist who provided expert testimony found both parties psychologically incapacitated. Petitioners behavioral pattern falls under the classification of dependent personality disorder, and the respondents, that of the narcissistic and antisocial personality disorder. The trial court, on July 30, 2001, rendered its decision declaring the marriage of the parties null and void on the ground that both parties were psychologically incapacitated to comply with the essential marital obligations. On review, the appellate court reversed and set aside the trials court ruling. It ruled that petitioner failed to prove the psychological incapacity of respondent, for the clinical psychologist did not personally examine respondent, and relied only on the information provided by petitioner. Further, the psychological incapacity was not shown to be attended by gravity, juridical antecedence and incurability. In sum, the evidence adduced fell short of the requirements stated in the Molina case needed for the declaration of nullity of the marriage under Art. 36 of the Family Code. Dissatisfied, petitioner filed before the SC the instant petition for review on certiorari. He posited that the trial court declared the marriage void, not only because of respondents psychological incapacity, but rather due to both parties psychological incapacity. He also pointed out that there is no requirement for the psychologist to personally examine respondent. Issue: Whether or not based on Article 36 of the Family Code, the marriage between the parties is null and void Ruling: The fulfillment of the obligations of marriage depends on the strength of this interpersonal relationship. A serious incapacity for interpersonal sharing and support is held to impair the relationship and consequently, the ability to fulfill the essential marital obligations. The marital capacity of one spouse is not considered in isolation but in reference to the fundamental relationship to the other spouse. The petitioner, who is afflicted with dependent personality disorder, cannot assume the essential marital obligations of living together, observing love, respect and fidelity and rendering help and support, for he is unable to make everyday decisions without advice from others, allows others to make most of his important decisions (such as where to live), tends to agree with people even when he believes they are wrong, has difficulty doing things on his own, volunteers to do things that are demeaning in order to get approval from other people, feels uncomfortable or helpless when alone and is often preoccupied with fears of being abandoned. As clearly shown in this case, petitioner followed everything dictated to him by the persons around him. He is insecure, weak and gullible, has no sense of his identity as a person, has no cohesive self to speak of, and has no goals and clear direction in life. Although on a different plane, the same may also be said of the respondent. Her being afflicted with antisocial personality disorder makes her unable to assume the essential marital obligations. This finding takes into account her disregard for the rights of others, her abuse, mistreatment and control of others without remorse, her tendency to blame others, and her intolerance of the conventional behavioral limitations imposed by society. Moreover, as shown in this case, respondent is impulsive and domineering; she had no qualms in manipulating petitioner with her threats of blackmail and of committing suicide. Both parties being afflicted with grave, severe and incurable psychological incapacity, the 528

University of the Cordilleras College of Law First Year C S.Y. 2013 - 2014 precipitous marriage which they contracted on April 23, 1996 is thus, declared null and void.

529

University of the Cordilleras College of Law First Year C S.Y. 2013 - 2014

VII. Legal Seperation

530

University of the Cordilleras College of Law First Year C S.Y. 2013 - 2014 Goitia vs Campos Rueda35 Phil 252 G.R. No. 11263 November 2, 1916 Full Case ELOISA GOITIA DE LA CAMARA, plaintiff-appellant, vs. JOSE CAMPOS RUEDA, defendant-appellee. TRENT, J.: This is an action by the wife against her husband for support outside of the conjugal domicile. From a judgment sustaining the defendant's demurrer upon the ground that the facts alleged in the complaint do not state a cause of action, followed by an order dismissing the case after the plaintiff declined to amend, the latter appealed. It was urged in the first instance, and the court so held, that the defendant cannot be compelled to support the plaintiff, except in his own house, unless it be by virtue of a judicial decree granting her a divorce or separation from the defendant. The parties were legally married in the city of Manila on January 7, 1915, and immediately thereafter established their residence at 115 Calle San Marcelino, where they lived together for about a month, when the plaintiff returned to the home of her parents. The pertinent allegations of the complaint are as follows: That the defendant, one month after he had contracted marriage with the plaintiff, demanded of her that she perform unchaste and lascivious acts on his genital organs; that the plaintiff spurned the obscene demands of the defendant and refused to perform any act other than legal and valid cohabitation; that the defendant, since that date had continually on other successive dates, made similar lewd and indecorous demands on his wife, the plaintiff, who always spurned them, which just refusals of the plaintiff exasperated the defendant and induce him to maltreat her by word and deed and inflict injuries upon her lips, her face and different parts of her body; and that, as the plaintiff was unable by any means to induce the defendant to desist from his repugnant desires and cease from maltreating her, she was obliged to leave the conjugal abode and take refuge in the home of her parents. Marriage in this jurisdiction is a contract entered into in the manner and with the solemnities established by General Orders No. 68, in so far as its civil effects are concerned requiring the consent of the parties. (Garcia vs. Montague, 12 Phil. Rep., 480, citing article 1261 of Civil Code.) Upon the termination of the marriage ceremony, a conjugal partnership is formed between the parties. (Sy Joc Lieng vs. Encarnacion, 16 Phil. Rep., 137.) To this extent a marriage partakes of the nature of an ordinary contract. But it is something more than a mere contract. It is a new relation, the rights, duties, and obligations of which rest not upon the agreement of the parties but upon the general law which defines and prescribes those rights, duties, and obligations .Marriage is an institution, in the maintenance of which in its purity the public is deeply interested. It is a relation for life and the parties cannot terminate it at any shorter period by virtue of any contract they may make .The reciprocal rights arising from this relation, so long as it continues, are such as the law determines from time to time, and none other. When the legal existence of the parties is merged into one by marriage, the new relation is regulated and controlled by the state or government upon principles of public policy for the benefit of society as well as the parties. And when the object of a marriage is defeated by rendering its continuance intolerable to one of the parties and productive of no possible good to the community, relief in some way should be obtainable. With these principles to guide us, we will inquire into the status of the law touching and governing the question under consideration. Articles 42 to 107 of the Civil Code are not in force in the Philippine Islands (Benedicto vs. De la Rama, 3 Phil .Rep., 34). Articles 44 to 78 of the Law of Civil Marriage of 1870, in force in the Peninsula, were extended to the Philippine Islands by royal decree on April 13, 1883 (Ebreo vs. Sichon, 4 Phil. Rep., 705). Articles 44, 45, and 48 of this law read: ART. 44. The spouses are obliged to be faithful to each other and to mutually assist each other. ART. 45. The husband must live with and protect his wife. (The second paragraph deals with the management of the wife's property.) ART. 48. The wife must obey her husband, live with him, and follow him when he charges his domicile or residence.

531

University of the Cordilleras College of Law First Year C S.Y. 2013 - 2014 Notwithstanding the provisions of the foregoing paragraph, the court may for just cause relieve her from this duty when the husband removes his residence to a foreign country. And articles 143 and 149 of the Civil Code are as follows: ART. 143. The following are obliged to support each other reciprocally to the whole extent specified in the preceding article. 1. The consorts. xxx xxx xxx ART. (149) 49. The person obliged to give support may, at his option, satisfy it, either by paying the pension that may be fixed or by receiving and maintaining in his own home the person having the right to the same. Article 152 of the Civil Code gives the instances when the obligation to give support shall cease. The failure of the wife to live with her husband is not one of them. The above quoted provisions of the Law of Civil Marriage and the Civil Code fix the duties and obligations of the spouses. The spouses must be faithful to, assist, and support each other. The husband must live with and protect his wife. The wife must obey and live with her husband and follow him when he changes his domicile or residence, except when he removes to a foreign country. But the husband who is obliged to support his wife may, at his option, do so by paying her a fixed pension or by receiving and maintaining her in his own home. May the husband, on account of his conduct toward his wife, lose this option and be compelled to pay the pension? Is the rule established by article 149 of the Civil Code absolute? The supreme court of Spain in its decision of December 5, 1903, held:. That in accordance with the ruling of the supreme court of Spain in its decisions dated May 11, 1897, November 25, 1899, and July 5, 1901, the option which article 149 grants the person, obliged to furnish subsistence, between paying the pension fixed or receiving and keeping in his own house the party who is entitled to the same, is not so absolute as to prevent cases being considered wherein, either because this right would be opposed to the exercise of a preferential right or because of the existence of some justifiable cause morally opposed to the removal of the party enjoying the maintenance, the right of selection must be understood as being thereby restricted. Whereas the only question discussed in the case which gave rise to this appeal was whether there was any reason to prevent the exercise of the option granted by article 149 of the Civil Code to the person obliged to furnish subsistence, to receive and maintain in his own house the one who is entitled to receive it; and inasmuch as nothing has been alleged or discussed with regard to the parental authority of Pedro Alcantara Calvo, which he ha not exercised, and it having been set forth that the natural father simply claims his child for the purpose of thus better attending to her maintenance, no action having been taken by him toward providing the support until, owing to such negligence, the mother was obliged to demand it; it is seen that these circumstances, together with the fact of the marriage of Pedro Alcantara, and that it would be difficult for the mother to maintain relations with her daughter, all constitute an impediment of such a nature as to prevent the exercise of the option in the present case, without prejudice to such decision as may be deemed proper with regard to the other questions previously cited in respect to which no opinion should be expressed at this time. The above was quoted with approval in United States and De Jesus vs. Alvir (9 Phil. Rep., 576), wherein the court held that the rule laid down in article 149 of the Civil Code "is not absolute." but it is insisted that there existed a preexisting or preferential right in each of these cases which was opposed to the removal of the one entitled to support. It is true that in the first the person claiming the option was the natural father of the child and had married a woman other than the child's mother, and in the second the right to support had already been established by a final judgment in a criminal case. Notwithstanding these facts the two cases clearly established the proposition that the option given by article 149 of the Civil Code may not be exercised in any and all cases. Counsel for the defendant cite, in support of their contention, the decision of the supreme court of Spain, dated November 3, 1905. In this case Don Berno Comas, as a result of certain business reverses and in order no to prejudice his wife, conferred upon her powers to administer and dispose of her property. When she left him he gave her all the muniments of title, mortgage credits, notes, P10,000 in accounts receivable, and the key to the safe in which he kept a large amount of jewels, thus depriving himself of all his possessions and being reduced in consequence to want. Subsequently he instituted this 532

University of the Cordilleras College of Law First Year C S.Y. 2013 - 2014 civil action against his wife, who was then living in opulence, for support and the revocation of the powers heretofore granted in reference to the administration and disposal of her property. In her answer the wife claimed that the plaintiff (her husband) was not legally in a situation to claim support and that the powers voluntarily conferred and accepted by her were bilateral and could not be canceled by the plaintiff. From a judgment in favor of the plaintiff the defendant wife appealed to the Audencia Territorial wherein, after due trial, judgment was rendered in her favor dismissing the action upon the merits. The plaintiff appealed to the supreme court and that high tribunal, in affirming the judgment of the Audencia Territorial, said: Considering that article 143, No. 1, of the Civil Code, providing that the spouses are mutually obliged to provide each other with support, cannot but be subordinate to the other provisions of said Code which regulates the family organization and the duties of spouses not legally separated, among which duties are those of their living together and mutually helping each other, as provided in article 56 of the aforementioned code; and taking this for granted, the obligation of the spouse who has property to furnish support to the one who has no property and is in need of it for subsistence, is to be understood as limited to the case where, in accordance with law, their separation has been decreed, either temporarily or finally and this case, with respect to the husband, cannot occur until a judgment of divorce is rendered, since, until then, if he is culpable, he is not deprived of the management of his wife's property and of the product of the other property belonging to the conjugal partnership; and Considering that, should the doctrine maintained in the appeal prevail, it would allow married persons to disregard the marriage bond and separate from each other of their own free will, thus establishing, contrary to the legal provision contained in said article 56 of the Civil Code, a legal status entirely incompatible with the nature and effects of marriage in disregard of the duties inherent therein and disturbing the unity of the family, in opposition to what the law, in conformity with good morals, has established; and. Considering that, as the spouses D. Ramon Benso and Doa Adela Galindo are not legally separated, it is their duty to live together and afford each other help and support; and for this reason, it cannot be held that the former has need of support from his wife so that he may live apart from her without the conjugal abode where it is his place to be, nor of her conferring power upon him to dispose even of the fruits of her property in order therewith to pay the matrimonial expenses and, consequently, those of his own support without need of going to his wife; wherefore the judgment appealed from, denying the petition of D. Ramon Benso for support, has not violated the articles of the Civil Code and the doctrine invoked in the assignments of error 1 and 5 of the appeal. From a careful reading of the case just cited and quoted from it appears quite clearly that the spouses separated voluntarily in accordance with an agreement previously made. At least there are strong indications to this effect, for the court says, "should the doctrine maintained in the appeal prevail, it would allow married persons to disregard the marriage bond and separate from each other of their own free will." If this be the true basis upon which the supreme court of Spain rested its decision, then the doctrine therein enunciated would not be controlling in cases where one of the spouses was compelled to leave the conjugal abode by the other or where the husband voluntarily abandons such abode and the wife seeks to force him to furnish support. That this is true appears from the decision of the same high tribunal, dated October 16, 1903. In this case the wife brought an action for support against her husband who had willfully and voluntarily abandoned the conjugal abode without any cause whatever. The supreme court, reversing the judgment absolving the defendant upon the ground that no action for divorce, etc., had been instituted, said: In the case at bar, it has been proven that it was Don Teodoro Exposito who left the conjugal abode, although he claims, without however proving his contention, that the person responsible for this situation was his wife, as she turned him out of the house. From this state of affairs it results that it is the wife who is party abandoned, the husband not having prosecuted any action to keep her in his company and he therefore finds himself, as long as he consents to the situation, under the ineluctable obligation to support his wife in fulfillment of the natural duty sanctioned in article 56 of the Code in relation with paragraph 1 of article 143. In not so holding, the trial court, on the mistaken ground that for the fulfillment of this duty the situation or relation of the spouses should be regulated in the manner it indicates, has made the errors of law assigned in the first three grounds alleged, because the nature of the duty of affording mutual support is compatible

533

University of the Cordilleras College of Law First Year C S.Y. 2013 - 2014 and enforcible in all situations, so long as the needy spouse does not create any illicit situation of the court above described.lawphil.net If we are in error as to the doctrine enunciated by the supreme court of Spain in its decision of November 3, 1905, and if the court did hold, as contended by counsel for the defendant in the case under consideration, that neither spouse can be compelled to support the other outside of the conjugal abode, unless it be by virtue of a final judgment granting the injured one a divorce or separation from the other, still such doctrine or holding would not necessarily control in this jurisdiction for the reason that the substantive law is not in every particular the same here as it is in Spain. As we have already stated, articles 42 to 107 of the Civil Code in force in the Peninsula are not in force in the Philippine Islands. The law governing the duties and obligations of husband and wife in this country are articles 44 to 78 of the Law of Civil Marriage of 1870 .In Spain the complaining spouse has, under article 105 of the Civil Code, various causes for divorce, such as adultery on the part of the wife in every case and on the part of the husband when public scandal or disgrace of the wife results therefrom; personal violence actually inflicted or grave insults: violence exercised by the husband toward the wife in order to force her to change her religion; the proposal of the husband to prostitute his wife; the attempts of the husband or wife to corrupt their sons or to prostitute their daughters; the connivance in their corruption or prostitution; and the condemnation of a spouse to perpetual chains or hard labor, while in this jurisdiction the only ground for a divorce is adultery. (Benedicto vs. De la Rama, 3 Phil .Rep., 34, 45.) This positive and absolute doctrine was announced by this court in the case just cited after an exhaustive examination of the entire subject. Although the case was appealed to the Supreme Court of the United States and the judgment rendered by this court was there reversed, the reversal did not affect in any way or weaken the doctrine in reference to adultery being the only ground for a divorce. And since the decision was promulgated by this court in that case in December, 1903, no change or modification of the rule has been announced. It is, therefore, the well settled and accepted doctrine in this jurisdiction. But it is argued that to grant support in an independent suit is equivalent to granting divorce or separation, as it necessitates a determination of the question whether the wife has a good and sufficient cause for living separate from her husband; and, consequently, if a court lacks power to decree a divorce, as in the instant case, power to grant a separate maintenance must also be lacking. The weakness of this argument lies in the assumption that the power to grant support in a separate action is dependent upon a power to grant a divorce. That the one is not dependent upon the other is apparent from the very nature of the marital obligations of the spouses. The mere act of marriage creates an obligation on the part of the husband to support his wife. This obligation is founded not so much on the express or implied terms of the contract of marriage as on the natural and legal duty of the husband; an obligation, the enforcement of which is of such vital concern to the state itself that the laws will not permit him to terminate it by his own wrongful acts in driving his wife to seek protection in the parental home. A judgment for separate maintenance is not due and payable either as damages or as a penalty; nor is it a debt in the strict legal sense of the term, but rather a judgment calling for the performance of a duty made specific by the mandate of the sovereign. This is done from necessity and with a view to preserve the public peace and the purity of the wife; as where the husband makes so base demands upon his wife and indulges in the habit of assaulting her. The pro tanto separation resulting from a decree for separate support is not an impeachment of that public policy by which marriage is regarded as so sacred and inviolable in its nature; it is merely a stronger policy overruling a weaker one; and except in so far only as such separation is tolerated as a means of preserving the public peace and morals may be considered, it does not in any respect whatever impair the marriage contract or for any purpose place the wife in the situation of a feme sole. The foregoing are the grounds upon which our short opinion and order for judgment, heretofore filed in this case, rest. Torres, Johnson and Carson, JJ., concur. Separate Opinions MORELAND, J., concurring: I based my vote in this case upon the ground that a husband cannot, by his own wrongful acts, relieve himself from the duty to support his wife imposed by law; and where a husband, by wrongful, illegal, and unbearable conduct, drives his wife from the domicile fixed by him, he cannot take advantage of her departure to abrogate the law applicable to the marital relation and repudiate his duties thereunder. In law and for all purposes within its purview, the wife still remains an inmate of the conjugal domicile; for I regard it as a principle of law universally recognized that where a person by his wrongful and illegal acts creates a condition which under ordinary circumstances would produce the loss of rights or status pertaining to another, the law will, whenever necessary to protect fully the rights or status of the person affected by such acts, regard the condition by such acts created as not existing and will recur to and act 534

University of the Cordilleras College of Law First Year C S.Y. 2013 - 2014 upon the original situation of the parties to determine their relative rights or the status of the person adversely affected. I do not believe, therefore, that the case is properly conceived by defendant, when the consideration thereof proceeds solely on the theory that the wife is outside the domicile fixed by the husband. Under the facts alleged in the complainant the wife is legally still within the conjugal domicile.

535

University of the Cordilleras College of Law First Year C S.Y. 2013 - 2014 Case Digest ELOISA GOITIA Y DELA CAMARA vs. JOSE CAMPOS RUEDA 35 PHIL 252 G.R. No. 11263. Decided On: November 2, 1916 Ponente: TRENT, J.: Facts: This is an action by the wife against the husband for support outside of the conjugal domicile. Eloitia Goitia and Jose Campos Rueda were legally married on January 7, 1915. After a month of living together, the wife returned to the home of her parents due to the following reasons: that the husband demand wife to perform unchaste and lascivious acts on his genital organs; that whenever wife rejected husbands indecorous demands, husband would maltreat wife by words and inflict injuries on wifes lips, face and different parts of her body; and that because she was unable to desist husbands repugnant desires and maltreatment, she was obliged to leave the conjugal home. The wife also seeks for support from his husband even if she lives separately. The husband on the other hand, seeks the relief of the courts in compelling his wife to return back to their conjugal home. Issue: Whether or not the husband can be compelled to support the wife outside the conjugal domicile Ruling: Marriage is something more than a mere contract. It is a new relation, the rights, duties and obligations of which rest not upon the agreement of the parties but upon the general law which defines and prescribes those rights, duties and obligations. When the legal existence is merged into one by marriage, the new relation is regulated and controlled by the government upon principles of public policy for the benefit of the society as well as the parties. Marriage is an institution and its maintenance is in its purity which the public is deeply interested. In the case at bar, when the continuance of the marriage becomes intolerable to one or both parties and gives no possible good to the community, relief from the court should be attainable. The Supreme Court made the observation that implied approval by the court of a wifes separate residence from her husband does not necessarily violate the sacredness and inviolability of the marriage. Since separation de-facto is allowed in this case, it is only due to the fact that public peace and wifes purity must be preserved. Lastly, the husband cannot, by his own wrongful acts, relieve himself from the duty to support his wife imposed by law; and where a husband, by wrongful, illegal and unbearable conduct, drives his wife from the domicile fixed by him, he cannot take the advantage of her departure to abrogate his duty to still support his wife. In law, the wife is legally still within the conjugal domicile, even if living separately, thus she is entitled to support and maintenance by the husband.

536

University of the Cordilleras College of Law First Year C S.Y. 2013 - 2014 Arroyo vs Vasquez De Arroyo 42 Phil 54 G.R. No. L-17014 August 11, 1921 MARIANO B. ARROYO, plaintiff-appellant, vs. DOLORES C. VASQUEZ DE ARROYO, defendant-appellee. STREET, J.: Mariano B. Arroyo and Dolores C. Vasquez de Arroyo were united in the bonds of wedlock by marriage in the year 1910, and since that date, with a few short intervals of separation, they have lived together as man and wife in the city of Iloilo until July 4, 1920, when the wife went away from their common home with the intention of living thenceforth separate from her husband. After efforts had been made by the husband without avail to induce her to resume marital relations, this action was initiated by him to compel her to return to the matrimonial home and live with him as a dutiful wife. The defendant answered, admitting the fact of marriage, and that she had left her husband's home without his consent; but she averred by way of defense and cross-complaint that she had been compelled to leave by cruel treatment on the part of her husband. Accordingly she in turn prayed for affirmative relief, to consist of (1) a decree of separation; (2) a liquidation of the conjugal partnership; (3) and an allowance for counsel fees and permanent separate maintenance. Upon hearing the cause the lower court gave judgment in favor of the defendant, authorizing her to live apart from her husband, granting her alimony at the rate of P400 per month, and directing that the plaintiff should pay to the defendant's attorney the sum of P1,000 for his services to defendant in the trial of the case. The plaintiff thereupon removed the case with the usual formalities by appeal to this court. The trial judge, upon consideration of the evidence before him, reached the conclusion that the husband was more to blame than his wife and that his continued ill-treatment of her furnished sufficient justification for her abandonment of the conjugal home and the permanent breaking off of marital relations with him. We have carefully examined and weighed every line of the proof, and are of the opinion that the conclusion stated is wholly untenable. The evidence shows that the wife is afflicted with a disposition of jealousy towards her husband in an aggravated degree; and to his cause are chiefly traceable without a doubt the many miseries that have attended their married life. In view of the decision which we are to pronounce nothing will be said in this opinion which will make the resumption of married relations more difficult to them or serve as a reminder to either of the mistakes of the past; and we prefer to record the fact that so far as the proof in this record shows neither of the spouses has at any time been guilty of conjugal infidelity, or has given just cause to the other to suspect illicit relations with any person. The tales of cruelty on the part of the husband towards the wife, which are the basis of the cross-action, are in our opinion no more than highly colored versions of personal wrangles in which the spouses have allowed themselves from time to time to become involved and would have little significance apart from the morbid condition exhibited by the wife. The judgment must therefore be recorded that the abandonment by her of the marital home was without sufficient justification in fact. In examining the legal questions involved, it will be found convenient to dispose first of the defendant's cross-complaint. To begin with, the obligation which the law imposes on the husband to maintain the wife is a duty universally recognized in civil society and is clearly expressed in articles 142 and 143 of the Civil code. The enforcement of this obligation by the wife against the husband is not conditioned upon the procurance of a divorce by her, nor even upon the existence of a cause for divorce. Accordingly it had been determined that where the wife is forced to leave the matrimonial abode and to live apart from her husband, she can, in this jurisdiction, compel him to make provision for her separate maintenance (Goitia vs. Campos Rueda, 35 Phil., 252); and he may be required to pay the expenses, including attorney's fees, necessarily incurred in enforcing such obligation, (Mercado vs. Ostrand and Ruiz, 37 Phil., 179.) Nevertheless, the interests of both parties as well as of society at large require that the courts should move with caution in enforcing the duty to provide for the separate maintenance of the wife, for this step involves a recognition of the de facto separation of the spouses a state which is abnormal and fraught with grave danger to all concerned. From this consideration it follows that provision should not be made for separate maintenance in favor of the wife unless it appears that the continued cohabitation of the pair has become impossible and separation necessary from the fault of the husband. In Davidson vs Davidson, the Supreme Court of Michigan, speaking through the eminent jurist, Judge Thomas M. Cooley, held that an action for the support of the wife separate from the husband will only be sustained when the reasons for it are imperative (47 Mich., 151). That imperative necessity is the only ground on which such a proceeding can be maintained also appears from the decision in Schindel vs. Schindel (12 Md., 294). In the State of South Carolina, where judicial divorces have never been procurable on any ground, the Supreme court fully recognizes the right of the wife to have provision for separate maintenance, where it is impossible for her to continue safely to cohabit with her husband; but 537

University of the Cordilleras College of Law First Year C S.Y. 2013 - 2014 the same court has more than once rejected the petition of the wife for separate maintenance where it appeared that the husband's alleged cruelty or ill-treatment was provoked by the wife's own improper conduct. (Rhame vs. Rhame, 1 McCord's Chan. [S. Car.], 197; 16 Am. Dec., 597; Boyd vs. Boyd, Har. Eq. [S. Car.], 144.) Upon one occasion Sir William Scott, pronouncing the judgment of the English Ecclesiastical Court in a case where cruelty on the part of the husband was relied upon to secure a divorce for the wife, made use of the following eloquent words, which are perhaps even more applicable in a proceeding for separate maintenance in a jurisdiction where, as here, a divorce cannot be obtained except on the single ground of adultery and this, too, after the conviction of the guilty spouse in a criminal prosecution for that crime. Said he: That the duty of cohabitation is released by the cruelty of one of the parties is admitted, but the question occurs, What is cruelty? . . . What merely wounds the mental feelings is in few cases to be admitted where they are not accompanied with bodily injury, either actual or menaced. Mere austerity of temper, petulance of manners, rudeness of language, a want of civil attention and accommodation, even occasional sallies of passion, if they do not threaten bodily harm, do not amount to legal cruelty: they are high moral offenses in the marriage-state undoubtedly, not innocent surely in any state of life, but still they are not that cruelty against which the law can relieve. Under such misconduct of either of the parties, for it may exist on the one side as well as on the other, the suffering party must bear in some degree the consequences of an injudicious connection; must subdue by decent resistance or by prudent conciliation; and if this cannot be done, both must suffer in silence. . . . The humanity of the court has been loudly and repeatedly invoked. Humanity is the second virtue of courts, but undoubtedly the first is justice. If it were a question of humanity simply, and of humanity which confined its views merely to the happiness of the present parties, it would be a question easily decided upon first impressions. Every body must feel a wish to sever those who wish to live separate from each other, who cannot live together with any degree of harmony, and consequently with any degree of happiness; but my situation does not allow me to indulge the feelings, much less the first feelings of an individual. The law has said that married persons shall not be legally separated upon the mere disinclination of one or both to cohabit together. . . . To vindicate the policy of the law is no necessary part of the office of a judge; but if it were, it would not be difficult to show that the law in this respect has acted with its usual wisdom and humanity with that true wisdom, and that real humanity, that regards the general interests of mankind. For though in particular cases the repugnance of the law to dissolve the obligations of matrimonial cohabitation may operate with great severity upon individual, yet it must be carefully remembered that the general happiness of the married life is secured by its indissolubility. When people understand that they must live together, except for a very few reasons known to the law, they learn to soften by mutual accommodation that yoke which they know cannot shake off; they become good husbands and good wives form the necessity of remaining husbands and wives; for necessity is a powerful master in teaching the duties which it imposes. . . . In this case, as in many others, the happiness of some individuals must be sacrificed to the greater and more general good. (Evans vs. Evans, 1 Hag. Con., 35; 161 Eng. Reprint, 466, 467.) In the light of the considerations stated, it is obvious that the cross-complaint is not well founded and none of the relief sought therein can be granted. The same considerations that require the dismissal of the cross-complaint conclusively prove that the plaintiff, Mariano B. Arroyo, has done nothing to forfeit his right to the marital society of his wife and that she is under an obligation, both moral and legal, to return to the common home and cohabit with him. The only question which here arises is as to the character and extent of the relief which may be properly conceded to him by judicial decree. The action is one by which the plaintiff seeks the restitution of conjugal rights; and it is supposed in the petitory part of the complaint that he is entitled to a permanent mandatory injunction requiring the defendant to return to the conjugal home and live with him as a wife according to the precepts of law and morality. Of course if such a decree were entered, in unqualified terms, the defendant would be liable to attachment for contempt, in case she should refuse to obey it; and, so far as the present writer is aware, the question is raised for the first time in this jurisdiction whether it is competent for the court to make such an order. 538

University of the Cordilleras College of Law First Year C S.Y. 2013 - 2014 Upon examination of the authorities we are convinced that it is not within the province of the courts of this country to attempt to compel one of the spouses to cohabit with, and render conjugal rights to, the other. Of course where the property rights of one of the pair are invaled, an action for restitution of such rights can be maintained. But we are disinclined to sanction the doctrine that an order, enforcible by process of contempt, may be entered to compel the restitution of the purely personal rights of consortium. At best such an order can be effective for no other purpose than to compel the spouses to live under the same roof; and the experience of these countries where the court of justice have assumed to compel the cohabitation of married people shows that the policy of the practice is extremely questionable. Thus in England, formerly the Ecclesiastical Court entertained suits for the restitution of conjugal rights at the instance of either husband or wife; and if the facts were found to warrant it that court would make a mandatory decree, enforcible by process of contempt in case of disobedience, requiring the delinquent party to live with the other and render conjugal rights. Yet this practice was sometimes criticized even by the judges who felt bound to enforce such orders, and in Weldon vs. Weldon (9 P. D., 52), decided in 1883, Sir James Hannen, President in the Probate, Divorce and Admiralty Division of the High Court of Justice, expressed his regret that the English law on the subject was not the same as that which prevailed in Scotland, where a decree of adherence, equivalent to the decree for the restitution of conjugal rights in England, could be obtained by the injured spouse, but could not be enforced by imprisonment. Accordingly, in obedience to the growing sentiment against the practice, the Matrimonial Causes Act (1884) abolished the remedy of imprisonment; though a decree for the restitution of conjugal rights can still be procured, and in case of disobedience may serve in appropriate cases as the basis of an order for the periodical payment of a stipend in the character of alimony. In the voluminous jurisprudence of the United States, only one court, so far as we can discover, has ever attempted to make a peremptory order requiring one of the spouses to live with the other; and that was in a case where a wife was ordered to follow and live with her husband, who had changed his domicile to the City of New Orleans. The decision referred to (Gahn vs. Darby, 36 La. Ann., 70) was based on a provision of the Civil Code of Louisiana similar to article 56 of the Spanish Civil Code. It was decided many years ago, and the doctrine evidently has not been fruitful even in the State of Louisiana. In other states of the American Union the idea of enforcing cohabitation by process of contempt is rejected. (21 Cyc., 1148.) In a decision of January 2, 1909, the supreme court of Spain appears to have affirmed an order of the Audencia Territorial de Valladolid requiring a wife to return to the marital domicile, and in the alternative, upon her failure to do so, to make a particular disposition of certain money and effects then in her possession and to deliver to her husband, as administrator of the ganancial property, all income, rents, and interest which might accrue to her from the property which she had brought to the marriage. (113 Jur. Civ., pp. 1, 11.) but it does not appear that this order for the return of the wife to the marital domicile was sanctioned by any other penalty than the consequences that would be visited upon her in respect to the use and control of her property; and it does not appear that her disobedience to that order would necessarily have been followed by imprisonment for contempt. We are therefore unable to hold that Mariano B. Arroyo in this case is entitled to the unconditional and absolute order for the return of the wife to the marital domicile, which is sought in the petitory part of the complaint; though he is, without doubt, entitled to a judicial declaration that his wife has presented herself without sufficient cause and that it is her duty to return. Therefore, reversing the judgment appealed from, in respect both to the original complaint and the crossbill, it is declared that Dolores Vasquez de Arroyo has absented herself from the marital home without sufficient cause; and she is admonished that it is her duty to return. The plaintiff is absolved from the cross-complaint, without special pronouncement as to costs of either instance. So ordered. Mapa, C.J., Johnson, Araullo, Avancea and Villamor, JJ., concur.

539

University of the Cordilleras College of Law First Year C S.Y. 2013 - 2014 Case Digest MARIANO ARROYO vs. DOLORES VAZQUEZ DE ARROYO 42 PHIL 54 G.R No. 17014. August 11, 1921 STREET, J.: Facts: Mariano Arroyo and Dolores Vazquez de Arroyo were united in the bonds of wedlock by marriage in 1910 and since that date, with a few short intervals of separation, they lived together as husband and wife until July 4, 1920, when the wife went away from their common home with the intention to live separately from her husband. After efforts had been made by the husband without avail to induce her to resume marital relations, an action was initiated by him to compel her to return to the matrimonial home and live with him as a dutiful wife. The wife answered that she had left her husbands home without his consent because of the cruel treatment on the part of her husband. Upon hearing the cause, the lower court gave judgment in favor of the wife, authorizing her to live apart from her husband. They concluded that the husband was more to blame than the wife and that his continued ill-treatment was a sufficient justification for her abandonment of the conjugal home. Issue: Whether or not the lower court erred in their judgment favoring the wife Whether or not it is competent for the court to make such an order for the return of the wife to the marital domicile Ruling: To begin with, the obligation which the law imposes on the husband to maintain the wife is a duty universally recognized in civil society and is clearly expressed in Art. 142 and 143 of the Civil Code. Accordingly, it has been determined that where the wife is forced to leave the matrimonial abode, she can, compel him to make provision for her separate maintenance. Nevertheless, the interests of both parties and the society at large require that the courts should move with caution in enforcing the duty to provide for the separate maintenance of the wife. From this consideration, it follows that provisions should be made for separate maintenance in favor of the wife unless it appears that the continued cohabitation of the pair has become impossible and separation necessary from the fault of the husband. We are therefore hold that Mariano Arroyo in this case is entitled to the unconditional and absolute return of the wife to the marital domicile. He is entitled to a judicial declaration that his wife has absented herself without sufficient cause and that it is her duty to return. Therefore, reversing the judgment of the lower court, it is declared that Dolores Vazquez de Arroyo has absented herself from the marital home without sufficient cause; and that she is admonished that it is her duty to return.

540

University of the Cordilleras College of Law First Year C S.Y. 2013 - 2014 People vs Schneckenburger, Et Al. 73 SCRA 413 G.R. No. L-48183 November 10, 1941 Full Case THE PEOPLE OF THE PHILIPPINES, plaintiff-appellee, vs. RODOLFO A. SCHNECKENBURGER, ET AL., defendants-appellants. MORAN, J.: On March 16, 1926, the accused Rodolfo A. Schneckenburger married the compliant Elena Ramirez Cartagena and after seven years of martial life, they agreed, for reason of alleged incompatibility of character, to live separately each other and on May 25, 1935 they executed a document which in part recites as follows: Que ambos comparecientes convienen en vivir separados el uno del otro por el resto de su vida y se comprometen, y obligan reciprocamente a no molastarse ni intervenir ni mezclarse bajo ningun concepto en la vida publica o privada de los mismos, entre si, quendado cada uno de los otorgantes en completa libertad de accion en calquier acto y todos concepto. On June 15, 1935, the accused Schneckenburger, without leaving the Philippines, secured a decree of divorce from the civil court of Juarez, Bravos District, State of Chihuahua, Mexico. On May 11, 1936, he contracted another marriage with his co-accused, Julia Medel, in the justice of the peace court of Malabon, Rizal, and since then they lived together as husband and wife in the city of Manila. Because of the nullity of the divorce decreed by the Mexico Court, complaint herein instituted two actions against the accused, one for bigamy in the Court of First Instance of Rizal and the other concubinage in the court of First Instance of Manila. The first culminated in the conviction of the accused for which he was sentenced to penalty of two months and one day of arresto mayor. On the trial for the offense of concubinage accused interposed the plea of double jeopardy, and the case was dismissed; but, upon appeal by the fiscal, this Court held the dismissal before the trial to be premature this was under the former procedure and without deciding the question of double jeopardy, remanded the case to the trial court for trial on the merits. Accused was convicted of concubinage through reckless imprudence and sentenced to a penalty of two months and one day of arresto mayor. Hence this appeal. As to appellant's plea of double jeopardy, it need only be observed that the office of bigamy for which he was convicted and that of concubinage for which he stood trial in the court below are two distinct offenses in law and in fact as well as in the mode of their prosecution. The celebration of the second marriage, with the first still existing, characterizes the crime of bigamy; on the other hand, in the present case, mere cohabitation by the husband with a woman who is not his wife characterizes the crime of concubinage. The first in an offense against civil status which may be prosecuted at the instance of the state; the second, an offense against chastity and may be prosecuted only at the instance of the offended party. And no rule is more settled in law than that, on the matter of double jeopardy, the test is not whether the defendant has already been tried for the same act, but whether he has been put in jeopardy for the same offense. (Diaz v. U. S., 223 U. S., 422; People v. Cabrera, 43 Phil., 82) Upon the other hand, we believe and so hold that the accused should be acquitted of the crime of concubinage. The document executed by and between the accused and the complaint in which they agreed to be "en completa libertad de accion en cualquier acto y en todos conceptos," while illegal for the purpose for which it was executed, constitutes nevertheless a valid consent to the act of concubinage within the meaning of section 344 of the Revised Penal Code. There can be no doubt that by such agreement, each party clearly intended to forego to illicit acts of the other. We said before (People vs. Guinucod, 58 Phil., 621) that the consent which bars the offended party from instituting a criminal prosecution in cases of adultery, concubinage, seduction, abduction, rape and acts of lasciviousness is that which has been given expressly or impliedly after the crime has been committed. We are now convinced that this is a narrow view in way warranted by the language, as well as the manifest policy, of the law. The second paragraph of article 344 of the Revised Penal Code provides: The offended party cannot institute criminal prosecution without including both the guilty parties, if they are both alive, nor, in any case, if he shall have consented or pardoned the offenders. (Emphasis ours.) As the term "pardon" unquestionably refers to the offense after its commission, "consent" must have been intended agreeably with its ordinary usage, to refer to the offense prior to its commission. No logical difference can indeed be perceived between prior and subsequent consent, for in both instances as the offended party has chosen to compromise with his/her dishonor, he/she becomes unworthy to come to court and invoke its aid in the vindication of the wrong. For instance, a husband who believers his wife 541

University of the Cordilleras College of Law First Year C S.Y. 2013 - 2014 another man for adultery, is as unworthy, if not more, as where, upon acquiring knowledge of the adultery after its commission, he says or does nothing. We, therefore, hold that the prior consent is as effective as subsequent consent to bar the offended party from prosecuting the offense. In this arriving at this conclusion we do not wish to be misconstrued as legalizing an agreement to do an illicit act, in violation of law. Our view must be taken only to mean that an agreement of the tenor entered into between the parties herein, operates, within the plain language and manifest policy of the law, to bar the offended party from prosecuting the offense. If there is anything morally condemnatory in a situation of his character, the remedy lies not with us but with the legislative department of the government. What the law is, not what it should be, defines the limits of our authority. Judgment is reversed and the accused is hereby acquitted, without costs. Avancea, C.J., Abad Santos, Diaz and Horilleno, JJ., concur.

542

University of the Cordilleras College of Law First Year C S.Y. 2013 - 2014 Case Digest PEOPLE OF THE PHILIPPINES vs. RODOLFO SCHNCKENBERGER 73 SCRA 413 G.R. No. 48183. November 10, 1941 MORAN, J.: Facts: On Mach 16, 1926, the accused, Rodolfo Schneckenberger married the complainant Elena Cartegena and after 7 years of marital life, they agreed, for reason of alleged incompatibility of character, to live separately from each other. On June 15, 1935, Rodolfo, without leaving the Philippines, secured a divorce from the civil court of Juarez, Mexico. On May 11, 1936, he contracted another marriage with his co-accused, Julia Medel. Complainant herein instituted two actions, one for bigamy in the Court of First Instance of Rizal and the other for concubinage in the Court of First Instance of Manila. The first culminated in the conviction of the accused. On the trial of concubinage, Rodolfo interposed the plea of double jeopardy, and the case was dismissed. Upon appeal by the fiscal, Rodolfo was convicted of concubinage through reckless imprudence and sentenced to a penalty of two months and one day of arresto mayor. Hence this appeal. Issue: Whether or not the court erred in convicting accused in the offense of concubinage Ruling: As to appellants plea for double jeopardy, it need only be observed that the offense of bigamy for which he was convicted and that of concubinage for which he stood trial are two distinct offenses in law and in fact as well as the mode of their prosecution. The celebration of the second marriage, with the first still existing, characterizes bigamy; in the present case, mere cohabitation by the husband with a woman who is not his wife characterizes concubinage. Upon the other hand, we believe and so hold that the accused should be acquitted of the crime of concubinage. The document executed by and between the accused clearly shows that each party intended to forego the illicit acts of the other. As the term pardon unquestionably refers to the offense after its commission, consent must have been intended, agreeably with its ordinary usage, to refer to the offense prior its commission. No logical difference can indeed be perceived between prior and subsequent consent, for in both instances as the offended party has chosen to compromise with his/her dishonor, he/she becomes unworthy to come to court and invoke its aid in the vindication of the wrong. In arriving at this conclusion, we do not wish to be misconstrued as legalizing an agreement to do an illicit act, in violation of law. Our view must be taken only to mean that an agreement of the tenor entered into between the parties herein, operates, within the plain language and manifest policy of the law, to bar the offended party from prosecuting the offense. The accused is acquitted in the crime of concubinage.

543

University of the Cordilleras College of Law First Year C S.Y. 2013 - 2014 Laperal vs Republic 6 SCRA 357 G.R. No. L-18008 October 30, 1962 Full Case ELISEA LAPERAL, petitioner, vs. REPUBLIC OF THE PHILIPPINES, oppositor. BARRERA, J.: On May 10, 1960, Elisea Laperal filed in the Court of First Instance of Baguio (Sp Proc. No. 433) a petition which reads: 1. That petitioner has been a bona fide resident of the City of Baguio for the last three years prior to the date of the filing of this petition; 2. That petitioner's maiden name is ELISEA LAPERAL; that on March 24, 1939, she married Mr. Enrique R. Santamaria; that in a partial decision entered on this Honorable Court on January 18, 1958, in Civil Case No. 356 of this Court, entitled 'Enrique R. Santamaria vs. Elisea L. Santamaria' Mr. Enrique Santamaria was given a decree of legal separation from her; that the said partial decision is now final; 3. That during her marriage to Enrique R. Santamaria, she naturally used, instead of her maiden name, that of Elisea L. Santamaria; that aside from her legal separation from Enrique R. Santamaria, she has also ceased to live with him for many years now; 4. That in view of the fact that she has been legally separated from Mr. Enrique R. Santamaria and has likewise ceased to live with him for many years, it is desirable that she be allowed to change her name and/or be permitted to resume using her maiden name, to wit: ELISEA LAPERAL. WHEREFORE, petitioner respectfully prayed that after the necessary proceedings are had, she be allowed to resume using her maiden name of Elisea Laperal. The petition was opposed by the City Attorney of Baguio on the ground that the same violates the provisions of Article 370 (should be 372) of the Civil Code, and that it is not sanctioned by the Rules of Court. In its decision of October 31, 1960, the court denied the petition for the reason that Article 372 of the Civil Code requires the wife, even after she is decreed legally separated from her husband, to continue using the name and surname she employed before the legal separation. Upon petitioner's motion, however, the court, treating the petition as one for change of name, reconsidered its decision and granted the petition on the ground that to allow petitioner, who is a businesswoman decreed legally separated from her husband, to continue using her married name would give rise to confusion in her finances and the eventual liquidation of the conjugal assets. Hence, this appeal by the State. The contention of the Republic finds support in the provisions of Article 372 of the New Civil Code which reads: ART. 372. When legal separation has been granted, the wife shall continue using her name and surname employed before the legal separation. (Emphasis supplied) Note that the language of the statute is mandatory that the wife, even after the legal separation has been decreed, shall continue using her name and surname employed before the legal separation. This is so because her married status is unaffected by the separation, there being no severance of the vinculum. It seems to be the policy of the law that the wife should continue to use the name indicative of her unchanged status for the benefit of all concerned. The appellee contends, however, that the petition is substantially for change of her name from Elisea L. Santamaria, the one she has been using, since her marriage, to Elisea Laperal, her maiden name, giving as reason or cause therefor her being legally separated from the husband Enrique R. Santamaria, and the fact that they have ceased to live together for many years. There seems to be no dispute that in the institution of these proceedings, the procedure prescribed in Rule 103 of the Rules of Court for change of name has been observed. But from the petition quoted in full at the beginning of these opinion, the only reason relied upon for the change of name is the fact that petitioner is legally separated from her husband and has, in fact, ceased to live with him for many years. It is doubtful, to say the least, whether Rule 103 which refers to change of name in general, may prevail over the specific provisions of Article 372 of the New Civil Code with regards to married women legally separated from their husbands. Even, however, applying Rule 103 to this case, the fact of legal separation 544

University of the Cordilleras College of Law First Year C S.Y. 2013 - 2014 alone which is the only basis for the petition at bar is, in our opinion, not a sufficient ground to justify a change of the name of herein petitioner, for to hold otherwise would be to provide an easy circumvention of the mandatory provisions of Article 372. It is true that in the second decision which reconsidered the first it is stated that as the petitioner owns extensive business interests, the continued used of her husband surname may cause undue confusion in her finances and the eventual liquidation of the conjugal assets. This finding is however without basis. In the first place, these were not the causes upon which the petition was based; hence, obviously no evidence to this effect had been adduced. Secondly, with the issuance of the decree of legal separation in 1958, the conjugal partnership between petitioner and her husband had automatically been dissolved and liquidated. (Art. 106[2], Civil Cod). Consequently, there could be no more occasion for an eventual liquidation of the conjugal assets. WHEREFORE, the order of the lower court of December 1, 1960, granting the petition, is hereby set aside and the petition dismissed. Without costs. So ordered. Bengzon, C.J., Padilla, Bautista Angelo, Labrador, Concepcion, Reyes, J.B.L., Paredes, Dizon, Regala and Makalintal, JJ., concur.

545

University of the Cordilleras College of Law First Year C S.Y. 2013 - 2014 Case Digest Elisea Laperal vs Republic of the Philippines GR No. L-18008 October 30, 1962 J. Barrerra Facts: Elisea Laperal married Enrique Santamaria on March 24, 1939. However, they filed a petition for legal separation and it was granted by the court on January 18, 1958. Thereafter, they lived separately. Prior to the legal separation, petitioner used the name Elisea L. Santamaria. However, now that the petition for legal separation has been granted, the petitioner wants to revert to using her maiden name, Elisea Laperal. This was opposed by the City Attorney of Baguio on the ground that it violates Art. 372 of the Civil Code. She was claiming that continuing to use her married name would give rise to confusion in her finances and the eventual liquidation of the conjugal assets. Issue: WoN petitioner be allowed to use her maiden name Elisea Laperal Ruling: The fact of legal separation alone which is the only basis for the petition at bar is, in our opinion, not a sufficient ground to justify a change of the name of herein petitioner. It is true that in the second decision which reconsidered the first it is stated that as the petitioner owns extensive business interests, the continued used of her husband surname may cause undue confusion in her finances and the eventual liquidation of the conjugal assets. This finding is however without basis. In the first place, these were not the causes upon which the petition was based; hence, obviously no evidence to this effect had been adduced.

546

University of the Cordilleras College of Law First Year C S.Y. 2013 - 2014 Tenchavez vs Escano 15 SCRA 355 G.R. No. L-19671 November 29, 1965 Full Case PASTOR B. TENCHAVEZ, plaintiff-appellant, vs. VICENTA F. ESCAO, ET AL., defendants-appellees. REYES, J.B.L., J.: Direct appeal, on factual and legal questions, from the judgment of the Court of First Instance of Cebu, in its Civil Case No. R-4177, denying the claim of the plaintiff-appellant, Pastor B. Tenchavez, for legal separation and one million pesos in damages against his wife and parents-in-law, the defendantsappellees, Vicente, Mamerto and Mena,1 all surnamed "Escao," respectively.2 The facts, supported by the evidence of record, are the following: Missing her late afternoon classes on 24 February 1948 in the University of San Carlos, Cebu City, where she was then enrolled as a second year student of commerce, Vicenta Escao, 27 years of age (scion of a well-to-do and socially prominent Filipino family of Spanish ancestry and a "sheltered colegiala"), exchanged marriage vows with Pastor Tenchavez, 32 years of age, an engineer, ex-army officer and of undistinguished stock, without the knowledge of her parents, before a Catholic chaplain, Lt. Moises Lavares, in the house of one Juan Alburo in the said city. The marriage was the culmination of a previous love affair and was duly registered with the local civil register. Vicenta's letters to Pastor, and his to her, before the marriage, indicate that the couple were deeply in love. Together with a friend, Pacita Noel, their matchmaker and go-between, they had planned out their marital future whereby Pacita would be the governess of their first-born; they started saving money in a piggy bank. A few weeks before their secret marriage, their engagement was broken; Vicenta returned the engagement ring and accepted another suitor, Joseling Lao. Her love for Pastor beckoned; she pleaded for his return, and they reconciled. This time they planned to get married and then elope. To facilitate the elopement, Vicenta had brought some of her clothes to the room of Pacita Noel in St. Mary's Hall, which was their usual trysting place. Although planned for the midnight following their marriage, the elopement did not, however, materialize because when Vicente went back to her classes after the marriage, her mother, who got wind of the intended nuptials, was already waiting for her at the college. Vicenta was taken home where she admitted that she had already married Pastor. Mamerto and Mena Escao were surprised, because Pastor never asked for the hand of Vicente, and were disgusted because of the great scandal that the clandestine marriage would provoke (t.s.n., vol. III, pp. 1105-06). The following morning, the Escao spouses sought priestly advice. Father Reynes suggested a recelebration to validate what he believed to be an invalid marriage, from the standpoint of the Church, due to the lack of authority from the Archbishop or the parish priest for the officiating chaplain to celebrate the marriage. The recelebration did not take place, because on 26 February 1948 Mamerto Escao was handed by a maid, whose name he claims he does not remember, a letter purportedly coming from San Carlos college students and disclosing an amorous relationship between Pastor Tenchavez and Pacita Noel; Vicenta translated the letter to her father, and thereafter would not agree to a new marriage. Vicenta and Pastor met that day in the house of Mrs. Pilar Mendezona. Thereafter, Vicenta continued living with her parents while Pastor returned to his job in Manila. Her letter of 22 March 1948 (Exh. "M"), while still solicitous of her husband's welfare, was not as endearing as her previous letters when their love was aflame. Vicenta was bred in Catholic ways but is of a changeable disposition, and Pastor knew it. She fondly accepted her being called a "jellyfish." She was not prevented by her parents from communicating with Pastor (Exh. "1-Escao"), but her letters became less frequent as the days passed. As of June, 1948 the newlyweds were already estranged (Exh. "2-Escao"). Vicenta had gone to Jimenez, Misamis Occidental, to escape from the scandal that her marriage stirred in Cebu society. There, a lawyer filed for her a petition, drafted by then Senator Emmanuel Pelaez, to annul her marriage. She did not sign the petition (Exh. "B-5"). The case was dismissed without prejudice because of her non-appearance at the hearing (Exh. "B-4"). 547

University of the Cordilleras College of Law First Year C S.Y. 2013 - 2014 On 24 June 1950, without informing her husband, she applied for a passport, indicating in her application that she was single, that her purpose was to study, and she was domiciled in Cebu City, and that she intended to return after two years. The application was approved, and she left for the United States. On 22 August 1950, she filed a verified complaint for divorce against the herein plaintiff in the Second Judicial District Court of the State of Nevada in and for the County of Washoe, on the ground of "extreme cruelty, entirely mental in character." On 21 October 1950, a decree of divorce, "final and absolute", was issued in open court by the said tribunal. In 1951 Mamerto and Mena Escao filed a petition with the Archbishop of Cebu to annul their daughter's marriage to Pastor (Exh. "D"). On 10 September 1954, Vicenta sought papal dispensation of her marriage (Exh. "D"-2). On 13 September 1954, Vicenta married an American, Russell Leo Moran, in Nevada. She now lives with him in California, and, by him, has begotten children. She acquired American citizenship on 8 August 1958. But on 30 July 1955, Tenchavez had initiated the proceedings at bar by a complaint in the Court of First Instance of Cebu, and amended on 31 May 1956, against Vicenta F. Escao, her parents, Mamerto and Mena Escao, whom he charged with having dissuaded and discouraged Vicenta from joining her husband, and alienating her affections, and against the Roman Catholic Church, for having, through its Diocesan Tribunal, decreed the annulment of the marriage, and asked for legal separation and one million pesos in damages. Vicenta claimed a valid divorce from plaintiff and an equally valid marriage to her present husband, Russell Leo Moran; while her parents denied that they had in any way influenced their daughter's acts, and counterclaimed for moral damages. The appealed judgment did not decree a legal separation, but freed the plaintiff from supporting his wife and to acquire property to the exclusion of his wife. It allowed the counterclaim of Mamerto Escao and Mena Escao for moral and exemplary damages and attorney's fees against the plaintiff-appellant, to the extent of P45,000.00, and plaintiff resorted directly to this Court. The appellant ascribes, as errors of the trial court, the following: 1. In not declaring legal separation; in not holding defendant Vicenta F. Escao liable for damages and in dismissing the complaint;. 2. In not holding the defendant parents Mamerto Escano and the heirs of Doa Mena Escao liable for damages;. 3 In holding the plaintiff liable for and requiring him to pay the damages to the defendant parents on their counterclaims; and. 4. In dismissing the complaint and in denying the relief sought by the plaintiff. That on 24 February 1948 the plaintiff-appellant, Pastor Tenchavez, and the defendant-appellee, Vicenta Escao, were validly married to each other, from the standpoint of our civil law, is clearly established by the record before us. Both parties were then above the age of majority, and otherwise qualified; and both consented to the marriage, which was performed by a Catholic priest (army chaplain Lavares) in the presence of competent witnesses. It is nowhere shown that said priest was not duly authorized under civil law to solemnize marriages. The chaplain's alleged lack of ecclesiastical authorization from the parish priest and the Ordinary, as required by Canon law, is irrelevant in our civil law, not only because of the separation of Church and State but also because Act 3613 of the Philippine Legislature (which was the marriage law in force at the time) expressly provided that SEC. 1. Essential requisites. Essential requisites for marriage are the legal capacity of the contracting parties and consent. (Emphasis supplied) The actual authority of the solemnizing officer was thus only a formal requirement, and, therefore, not essential to give the marriage civil effects,3 and this is emphasized by section 27 of said marriage act, which provided the following:

548

University of the Cordilleras College of Law First Year C S.Y. 2013 - 2014 SEC. 27. Failure to comply with formal requirements. No marriage shall be declared invalid because of the absence of one or several of the formal requirements of this Act if, when it was performed, the spouses or one of them believed in good faith that the person who solemnized the marriage was actually empowered to do so, and that the marriage was perfectly legal. The good faith of all the parties to the marriage (and hence the validity of their marriage) will be presumed until the contrary is positively proved (Lao vs. Dee Tim, 45 Phil. 739, 745; Francisco vs. Jason, 60 Phil. 442, 448). It is well to note here that in the case at bar, doubts as to the authority of the solemnizing priest arose only after the marriage, when Vicenta's parents consulted Father Reynes and the archbishop of Cebu. Moreover, the very act of Vicenta in abandoning her original action for annulment and subsequently suing for divorce implies an admission that her marriage to plaintiff was valid and binding. Defendant Vicenta Escao argues that when she contracted the marriage she was under the undue influence of Pacita Noel, whom she charges to have been in conspiracy with appellant Tenchavez. Even granting, for argument's sake, the truth of that contention, and assuming that Vicenta's consent was vitiated by fraud and undue influence, such vices did not render her marriage ab initio void, but merely voidable, and the marriage remained valid until annulled by a competent civil court. This was never done, and admittedly, Vicenta's suit for annulment in the Court of First Instance of Misamis was dismissed for non-prosecution. It is equally clear from the record that the valid marriage between Pastor Tenchavez and Vicenta Escao remained subsisting and undissolved under Philippine law, notwithstanding the decree of absolute divorce that the wife sought and obtained on 21 October 1950 from the Second Judicial District Court of Washoe County, State of Nevada, on grounds of "extreme cruelty, entirely mental in character." At the time the divorce decree was issued, Vicenta Escao, like her husband, was still a Filipino citizen.4 She was then subject to Philippine law, and Article 15 of the Civil Code of the Philippines (Rep. Act No. 386), already in force at the time, expressly provided: Laws relating to family rights and duties or to the status, condition and legal capacity of persons are binding upon the citizens of the Philippines, even though living abroad. The Civil Code of the Philippines, now in force, does not admit absolute divorce, quo ad vinculo matrimonii; and in fact does not even use that term, to further emphasize its restrictive policy on the matter, in contrast to the preceding legislation that admitted absolute divorce on grounds of adultery of the wife or concubinage of the husband (Act 2710). Instead of divorce, the present Civil Code only provides for legal separation (Title IV, Book 1, Arts. 97 to 108), and, even in that case, it expressly prescribes that "the marriage bonds shall not be severed" (Art. 106, subpar. 1). For the Philippine courts to recognize and give recognition or effect to a foreign decree of absolute divorce betiveen Filipino citizens could be a patent violation of the declared public policy of the state, specially in view of the third paragraph of Article 17 of the Civil Code that prescribes the following: Prohibitive laws concerning persons, their acts or property, and those which have for their object public order, policy and good customs, shall not be rendered ineffective by laws or judgments promulgated, or by determinations or conventions agreed upon in a foreign country. Even more, the grant of effectivity in this jurisdiction to such foreign divorce decrees would, in effect, give rise to an irritating and scandalous discrimination in favor of wealthy citizens, to the detriment of those members of our polity whose means do not permit them to sojourn abroad and obtain absolute divorces outside the Philippines. From this point of view, it is irrelevant that appellant Pastor Tenchavez should have appeared in the Nevada divorce court. Primarily because the policy of our law cannot be nullified by acts of private parties (Civil Code,Art. 17, jam quot.); and additionally, because the mere appearance of a non-resident consort cannot confer jurisdiction where the court originally had none (Area vs. Javier, 95 Phil. 579). From the preceding facts and considerations, there flows as a necessary consequence that in this jurisdiction Vicenta Escao's divorce and second marriage are not entitled to recognition as valid; for her previous union to plaintiff Tenchavez must be declared to be existent and undissolved. It follows, 549

University of the Cordilleras College of Law First Year C S.Y. 2013 - 2014 likewise, that her refusal to perform her wifely duties, and her denial of consortium and her desertion of her husband constitute in law a wrong caused through her fault, for which the husband is entitled to the corresponding indemnity (Civil Code, Art. 2176). Neither an unsubstantiated charge of deceit nor an anonymous letter charging immorality against the husband constitute, contrary to her claim, adequate excuse. Wherefore, her marriage and cohabitation with Russell Leo Moran is technically "intercourse with a person not her husband" from the standpoint of Philippine Law, and entitles plaintiff-appellant Tenchavez to a decree of "legal separation under our law, on the basis of adultery" (Revised Penal Code, Art. 333). The foregoing conclusions as to the untoward effect of a marriage after an invalid divorce are in accord with the previous doctrines and rulings of this court on the subject, particularly those that were rendered under our laws prior to the approval of the absolute divorce act (Act 2710 of the Philippine Legislature). As a matter of legal history, our statutes did not recognize divorces a vinculo before 1917, when Act 2710 became effective; and the present Civil Code of the Philippines, in disregarding absolute divorces, in effect merely reverted to the policies on the subject prevailing before Act 2710. The rulings, therefore, under the Civil Code of 1889, prior to the Act above-mentioned, are now, fully applicable. Of these, the decision in Ramirez vs. Gmur, 42 Phil. 855, is of particular interest. Said this Court in that case: As the divorce granted by the French Court must be ignored, it results that the marriage of Dr. Mory and Leona Castro, celebrated in London in 1905, could not legalize their relations; and the circumstance that they afterwards passed for husband and wife in Switzerland until her death is wholly without legal significance. The claims of the very children to participate in the estate of Samuel Bishop must therefore be rejected. The right to inherit is limited to legitimate, legitimated and acknowledged natural children. The children of adulterous relations are wholly excluded. The word "descendants" as used in Article 941 of the Civil Code cannot be interpreted to include illegitimates born of adulterous relations. (Emphasis supplied) Except for the fact that the successional rights of the children, begotten from Vicenta's marriage to Leo Moran after the invalid divorce, are not involved in the case at bar, the Gmur case is authority for the proposition that such union is adulterous in this jurisdiction, and, therefore, justifies an action for legal separation on the part of the innocent consort of the first marriage, that stands undissolved in Philippine law. In not so declaring, the trial court committed error. True it is that our ruling gives rise to anomalous situations where the status of a person (whether divorced or not) would depend on the territory where the question arises. Anomalies of this kind are not new in the Philippines, and the answer to them was given in Barretto vs. Gonzales, 58 Phil. 667: The hardship of the existing divorce laws in the Philippine Islands are well known to the members of the Legislature. It is the duty of the Courts to enforce the laws of divorce as written by Legislature if they are constitutional. Courts have no right to say that such laws are too strict or too liberal. (p. 72) The appellant's first assignment of error is, therefore, sustained. However, the plaintiff-appellant's charge that his wife's parents, Dr. Mamerto Escao and his wife, the late Doa Mena Escao, alienated the affections of their daughter and influenced her conduct toward her husband are not supported by credible evidence. The testimony of Pastor Tenchavez about the Escao's animosity toward him strikes us to be merely conjecture and exaggeration, and are belied by Pastor's own letters written before this suit was begun (Exh. "2-Escao" and "Vicenta," Rec. on App., pp. 270-274). In these letters he expressly apologized to the defendants for "misjudging them" and for the "great unhappiness" caused by his "impulsive blunders" and "sinful pride," "effrontery and audacity" [sic]. Plaintiff was admitted to the Escao house to visit and court Vicenta, and the record shows nothing to prove that he would not have been accepted to marry Vicente had he openly asked for her hand, as good manners and breeding demanded. Even after learning of the clandestine marriage, and despite their shock at such unexpected event, the parents of Vicenta proposed and arranged that the marriage be recelebrated in strict conformity with the canons of their religion upon advice that the previous one was canonically defective. If no recelebration of the marriage ceremony was had it was not due to defendants Mamerto Escao and his wife, but to the refusal of Vicenta to proceed with it. That the spouses Escao did not seek 550

University of the Cordilleras College of Law First Year C S.Y. 2013 - 2014 to compel or induce their daughter to assent to the recelebration but respected her decision, or that they abided by her resolve, does not constitute in law an alienation of affections. Neither does the fact that Vicenta's parents sent her money while she was in the United States; for it was natural that they should not wish their daughter to live in penury even if they did not concur in her decision to divorce Tenchavez (27 Am. Jur. 130-132). There is no evidence that the parents of Vicenta, out of improper motives, aided and abetted her original suit for annulment, or her subsequent divorce; she appears to have acted independently, and being of age, she was entitled to judge what was best for her and ask that her decisions be respected. Her parents, in so doing, certainly cannot be charged with alienation of affections in the absence of malice or unworthy motives, which have not been shown, good faith being always presumed until the contrary is proved. SEC. 529. Liability of Parents, Guardians or Kin. The law distinguishes between the right of a parent to interest himself in the marital affairs of his child and the absence of rights in a stranger to intermeddle in such affairs. However, such distinction between the liability of parents and that of strangers is only in regard to what will justify interference. A parent isliable for alienation of affections resulting from his own malicious conduct, as where he wrongfully entices his son or daughter to leave his or her spouse, but he is not liable unless he acts maliciously, without justification and from unworthy motives. He is not liable where he acts and advises his child in good faith with respect to his child's marital relations in the interest of his child as he sees it, the marriage of his child not terminating his right and liberty to interest himself in, and be extremely solicitous for, his child's welfare and happiness, even where his conduct and advice suggest or result in the separation of the spouses or the obtaining of a divorce or annulment, or where he acts under mistake or misinformation, or where his advice or interference are indiscreet or unfortunate, although it has been held that the parent is liable for consequences resulting from recklessness. He may in good faith take his child into his home and afford him or her protection and support, so long as he has not maliciously enticed his child away, or does not maliciously entice or cause him or her to stay away, from his or her spouse. This rule has more frequently been applied in the case of advice given to a married daughter, but it is equally applicable in the case of advice given to a son. Plaintiff Tenchavez, in falsely charging Vicenta's aged parents with racial or social discrimination and with having exerted efforts and pressured her to seek annulment and divorce, unquestionably caused them unrest and anxiety, entitling them to recover damages. While this suit may not have been impelled by actual malice, the charges were certainly reckless in the face of the proven facts and circumstances. Court actions are not established for parties to give vent to their prejudices or spleen. In the assessment of the moral damages recoverable by appellant Pastor Tenchavez from defendant Vicente Escao, it is proper to take into account, against his patently unreasonable claim for a million pesos in damages, that (a) the marriage was celebrated in secret, and its failure was not characterized by publicity or undue humiliation on appellant's part; (b) that the parties never lived together; and (c) that there is evidence that appellant had originally agreed to the annulment of the marriage, although such a promise was legally invalid, being against public policy (cf. Art. 88, Civ. Code). While appellant is unable to remarry under our law, this fact is a consequence of the indissoluble character of the union that appellant entered into voluntarily and with open eyes rather than of her divorce and her second marriage. All told, we are of the opinion that appellant should recover P25,000 only by way of moral damages and attorney's fees. With regard to the P45,000 damages awarded to the defendants, Dr. Mamerto Escao and Mena Escao, by the court below, we opine that the same are excessive. While the filing of this unfounded suit must have wounded said defendants' feelings and caused them anxiety, the same could in no way have seriously injured their reputation, or otherwise prejudiced them, lawsuits having become a common occurrence in present society. What is important, and has been correctly established in the decision of the court below, is that said defendants were not guilty of any improper conduct in the whole deplorable affair. This Court, therefore, reduces the damages awarded to P5,000 only. Summing up, the Court rules:

551

University of the Cordilleras College of Law First Year C S.Y. 2013 - 2014 (1) That a foreign divorce between Filipino citizens, sought and decreed after the effectivity of the present Civil Code (Rep. Act 386), is not entitled to recognition as valid in this jurisdiction; and neither is the marriage contracted with another party by the divorced consort, subsequently to the foreign decree of divorce, entitled to validity in the country; (2) That the remarriage of divorced wife and her co-habitation with a person other than the lawful husband entitle the latter to a decree of legal separation conformably to Philippine law; (3) That the desertion and securing of an invalid divorce decree by one consort entitles the other to recover damages; (4) That an action for alienation of affections against the parents of one consort does not lie in the absence of proof of malice or unworthy motives on their part. WHEREFORE, the decision under appeal is hereby modified as follows; (1) Adjudging plaintiff-appellant Pastor Tenchavez entitled to a decree of legal separation from defendant Vicenta F. Escao; (2) Sentencing defendant-appellee Vicenta Escao to pay plaintiff-appellant Tenchavez the amount of P25,000 for damages and attorneys' fees; (3) Sentencing appellant Pastor Tenchavez to pay the appellee, Mamerto Escao and the estate of his wife, the deceased Mena Escao, P5,000 by way of damages and attorneys' fees. Neither party to recover costs. Bengzon, C.J., Bautista Angelo, Concepcion, Dizon, Regala, Makalintal, Bengzon, J.P. and Zaldivar, JJ., concur.

552

University of the Cordilleras College of Law First Year C S.Y. 2013 - 2014 Case Digest Tenchavez vs Escano G.R. No. L-19671 November 29, 1965 J. Reyes Facts: In February 1948, Tenchavez and Escao secretly married each other and of course without the knowledge of Escaos parents who were of prominent social status. The marriage was celebrated by a military chaplain. When Escaos parents learned of this, they insisted a church wedding to be held but Escao withdrew from having a re-celebration because she heard that Tenchavez was having an affair with another woman. Eventually, their relationship went sour; 2 years later, Escao went to the US where she acquired a decree of absolute divorce and she subsequently became an American citizen and also married an American. In 1955, Tenchavez initiated a case for legal separation and further alleged that Escaos parents dissuaded their daughter to go abroad and causing her to be estranged from him hence hes asking for damages in the amount of P1,000,000.00. The lower court did not grant the legal separation being sought for and at the same time awarded a P45,000.00 worth of counter-claim by the Escaos. Issue: Whether or not damages should be awarded to either party in the case at bar

Held: Yes. On the part of Tenchavez: His marriage with Escao was a secret one and the failure of said marriage did not result to public humiliation; that they never lived together and he even consented to annulling the marriage earlier (because Escao filed for annulment before she left for the US but the same was dismissed due to her non-appearance in court); that he failed to prove that Escaos parents dissuaded their daughter to leave Tenchavez and as such his P1,000,000.00 claim cannot be awarded. HOWEVER, by reason of the fact that Escao left without the knowledge of Tenchavez and being able to acquire a divorce decree; and Tenchavez being unable to remarry, the SC awarded P25,000.00 only by way of moral damages and attorneys fees to be paid by Escao and not her parents. On the part of Escaos parents: It is true that the P1,000,000.00 for damages suit by Tenchavez against the Escaos is unfounded and the same must have wounded their feelings and caused them anxiety, the same could in no way have seriously injured their reputation, or otherwise prejudiced them, lawsuits having become a common occurrence in present society. What is important, and has been correctly established in the decision of the lower court, is that they were not guilty of any improper conduct in the whole deplorable affair. The SC reduced the damages awarded from P45,000.00 to P5,000.00 only.

553

University of the Cordilleras College of Law First Year C S.Y. 2013 - 2014 Macadangdang vs Court of Appeals 108 SCRA 314 G.R. No. L-38287 October 23, 1981 Full Case ANTONIO MACADANGDANG, petitioner, vs. THE COURT OF APPEALS; HONORABLE ALEJANDRO E. SEBASTIAN, in his capacity as Presiding Judge, Court of First Instance of Davao, 16th Judicial District, Sala 1, Tagum, Davao del Norte; FILOMENA GAVIANA, MACADANGDANG; and ROLANDO RAMA, respondents. MAKASIAR, J.: This petition for certiorari, prohibition and injunction with prayer for temporary restraining order presents for review the Court of Appeal's resolution dated December 21, 1973, which dismissed the petition in CA-G.R. No. Sp-02656-R, petitioner's motion for reconsideration of the said resolution having been denied on January 29, 1974. From the records, it appears that respondent Filomena Gaviana Macadangdang (hereinafter referred to as private respondent) and petitioner Antonio Macadangdang contracted marriage in 1946 after having lived together for two years. From a humble buy-and-sell business and sari-sari store operation in Davao City, the spouses moved to Mawab Davao del Norte where, through hard work and good fortune, their small business grew and expanded into merchandising, trucking, transportation, rice and corn mill business, abaca stripping, real estate and others. They were blessed with six children, three of whom were already of majority age and the other three were still minors as of the time this case was initiated in the lower court. With their established businesses and accumulated wealth, their once simple life became complicated and their relationship started to suffer setbacks. While the economic or material aspect of their marriage was stabilized the physical and spiritual aspects became shaky. Both accused each other of indulging in extramarital relations. Married life for them became so intolerable that they separated in 1965 when private respondent left for Cebu for good. When she returned to Davao in 1971, she learned of the illicit affairs of her estranged husband. Then and there, she decided to take the initial action. On April 28, 1971, private respondent (plaintiff therein) instituted a complaint for legal separation in the Court of First Instance of Davao, Branch VI I I at Tagum, Davao, which complaint was docketed as Civil Case No. 109 and entitled "Filomena Gaviana Macadangdang vs. Antonio Macadangdang" [P. 156, rec]. Petitioner (then defendant) filed his answer with counterclaim dated May 31, 1971 [p. 158, rec]. On February 9, 1972, private respondent filed a petition for appointment of administrator, to administer the estate of the conjugal partnership pending the termination of the case [p. 100, rec.]. Petitioner opposed the aforesaid petition in a pleading dated February 21, 1972 [P. 102, rec] On January 4, 1973, the petition for appointment of administrator not having been acted upon, the trial court handed down its decision, the dispositive portion of which states thus: Wherefore, judgment is hereby rendered ordering the legal separation of plaintiff and the defendant, or what under the old law was separation from bed and board a mensa et thoro with all the legal effects attendant thereto, particularly the dissolution and liquidation of the conjugal community of property. Since there is no complete list of the community property which has to be divided, pending the dissolution of the conjugal property, the defendant is ordered to pay to plaintiff P10,000.00 for her support, for any way he had been disposing some of the properties or mortgaging them without sharing the plaintiff any part of the fruits or proceeds thereof until the court can appoint an administrator, as prayed for by plaintiff in a separate petition, who will take over the administration and management of all the conjugal partnership properties, and act as guardian of the minor children; to protect said properties from dissipation, and who will submit a complete inventory of said properties so that the Court can make a just division, such division to be embodied in a supplemental decision. ... [pp. 104-115, rec.]. On August 7, 1973, private respondent filed a motion praying that she be allowed to withdraw P10,000.00 from the lease rental of a portion of their conjugal property deposited by Francisco Dizon [p. 116, rec.]. Respondent Judge acted on the aforesaid motion by issuing the order of August 13, 1973 which directed the clerk of court "to deliver, under receipt, to plaintiff Filomena Gaviana Macadangdang and/or to her counsel, Atty. Marcial Fernandez, the amount of P10,000.00" [p. 118, rec]. On August 25, 1973, private respondent filed another motion for the appointment of an administrator, reiterating her previous petition and urging favorable action thereon "to impede unlawful sequestration of

554

University of the Cordilleras College of Law First Year C S.Y. 2013 - 2014 some conjugal assets and clandestine transfers" by petitioner [p. 120, rec.]. Petitioner again filed his opposition dated September 6, 1973 [p. 122, rec.]. On September 20, 1973, respondent Judge issued an order directing plaintiff's counsel "to submit three (3) names for appointment as administrator, including in the list, if possible, a banking institution authorized to handle cases of administration of properties, furnishing a copy of said list to defendant, who shag be given three (3) days from receipt thereof to present his observations and objections to said recommended persons or entity, after which the Court will select the administrator as may seem best suited for the purpose" [pp. 126-127, rec] Petitioner then filed a motion for reconsideration dated October 3, 1973 of the order of September 20, 1973 with prayer that he be allowed to continue administering the conjugal properties in accordance with law [p. 128, rec.]. This motion for reconsideration was denied in the order of October 13, 1973 [p. 133, rec]. On October 13, 1973, herein private respondent filed a motion for appointment of administrator and submission of complete fist of conjugal assets by defendant, submitting therein three nominees for administrator [p. 135, rec]. On October 23, 1973, petitioner filed his second motion for reconsideration praying therein that the orders of September 20, 1973 and October 13, 1973 be reconsidered by not proceeding with the appointment of an administrator of the conjugal properties of the parties [p. 137, rec]. Respondent Judge denied the aforesaid second motion for reconsideration in his order of November 19, 1973, reiterating therein his ruling that the decree of legal separation had become final [p. 141, rec]. Petitioner brought the case to the Court of Appeals in a petition for certiorari and prohibition with writ of preliminary injunction and/or temporary restraining order filed on December 18, 1973. Said petition sought to review, set aside and declare null and void the orders of September 20, 1973, October 13, 1973 and November 19, 1973 of respondent Judge; to prohibit respondent Judge from carrying out and executing the aforecited orders; and to prohibit him from treating, regarding and construing his decision of January 4, 1973 as being "final and executory" as well as from enforcing the same in any manner whatsoever [pp. 1, 4, & 5, CA rec.]. The Court of Appeals, in its resolution of December 21, 1973, ruled that the questioned January 4, 1973 decision of the lower court had become final and, consequently, the appointment of an administrator was valid and that the petition was not sufficient in substance, since the applicable law and jurisprudence afford the petitioner no valid cause to impugn the three questioned orders. The appellate court accordingly dismissed the petition [pp. 70-80, rec]. Hence, this appeal from the resolution of December 21, 1973. On February 6, 1980, counsel for petitioner, through a notice of death and motion to dismiss, informed this Court that petitioner Antonio Macadangdang died on November 30, 1979 and as a consequence thereof, this case and Civil Case No. 109 of the Court of First Instance of Davao have become moot and academic [p. 516, rec.]. Private respondent, when required to comment on the aforesaid motion, moved for a resolution of this case although she believes that petitioner's death has posed new intervening circumstances that would affect the entire purpose in filing the same. In effect, private respondent agrees with petitioner's counsel that her husband's death has rendered the instant petition moot and academic [pp. 522, 524, rec.]. Petitioner had averred that the Court of Appeals gravely erred in holding that respondent Judge's incomplete decision of January 4, 1973 had become final and executory and that the same Court committed an error in holding that the appointment of an administrator in the case below was proper. Private respondent, upon the other hand, has always maintained that 1. the decision of January 4, 1973 had become final and executory when the petitioner failed to appeal therefrom within the reglementary period of 30 days from receipt thereof, despite the non-issuance of a supplemental decision regarding the division of the conjugal properties; and 2. the appointment of an administrator pending the actual division of said properties is proper being a must and an exercise of the sound discretion of the Honorable Presiding Judge in the Court of First Instance of Davao, Branch VIII in Tagum [pp. 193-194, rec]. Did petitioner's death on November 30, 1979 render the case moot and academic? Legal problems do not cease simply because one of the parties dies; the same problems may come up again in another case of similar magnitude. Considering also the far-reaching significance and implications of a pronouncement

555

University of the Cordilleras College of Law First Year C S.Y. 2013 - 2014 on the very important issues involved, this Court feels bound to meet said issues frontally and come out with a decisive resolution of the same. Thus, the questions for resolution have been narrowed down to the following: 1. Whether the decision of the trial court dated January 4, 1973 in Civil Case No. 109 finding herein petitioner guilty of concubinage and decreeing legal separation between him and his wife Filomena Gaviana Macadangdang (private respondent herein) had already become final and executory long before the herein petition was filed; 2. Should the children of both spouses predecease the surviving spouse, whether the intestate heirs of the deceased could inherit from the innocent surviving spouse, particularly where the latter's share in the conjugal assets is concerned, in view of Article 106, No. 4 of the New Civil Code; and 3. The effect of the pendency of Special Proceedings No. 134 in the Court of First Instance of Davao for the settlement of the estate of the deceased petitioner herein, on the decision in Civil Case No. 109 as well as on the instant petition. In support of his contention that the Court of Appeals committed grave error in holding that respondent Judge's incomplete decision of January 4, 1973 had become final and executory, petitioner had consistently asserted the following reasons: 1. Private respondent's complaint for legal separation and division of properties was a single complaint. Thus, she explicitly prayed: xxx xxx xxx 3. That upon trial of this action judgment be rendered ordering the legal separation of the plaintiff and the defendant and the division of all the assets of the conjugal partnership, ... [p. 157, rec) In this single action, private respondent asked the trial court to decide if petitioner and she should be legally separated, and if they should, what properties would form part of the conjugal regime and which properties would be assigned to each spouse. 2. Of the aforesaid issues, the lower court resolved only the issue of legal separation and reserved for supplemental decision the division of the conjugal properties. Petitioner had further argued that Inasmuch as the Decision failed to dispose of all the issues before the Court, which necessitated the announcement of a forthcoming supplemental decision, petitioner respectfully submits that the Decision was an incomplete judgment. In Santos v. de Guzman, 1 SCRA 1048, is found this very succinct explanation of what an incomplete judgment is: ... There was but one case before the lower court. Its first decision (of June 12, 1956) was, as already stated, incomplete the same not having resolved the issues involved in the litigation. For this reason the trial had to be reopened and a supplemental decision had to be rendered ... (at p. 1053; emphasis supplied). WE do not find merit in petitioner's submission that the questioned decision had not become final and executory since the law explicitly and clearly provides for the dissolution and liquidation of the conjugal partnership of gains of the absolute community of property as among the effects of the final decree of legal separation. Article 106 of the Civil Code thus reads: Art. 106. The decree of legal separation shall have the following effects: 1) The spouses shall be entitled to live separately from each other, but the marriage bonds shall not be severed; 2) The conjugal partnership of gains or the absolute conjugal community of property shall be dissolved and liquidated but the offending spouse shall have no right to any share of the profits earned by the partnership or community, without prejudice to the provisions of article 176; xxx xxx xxx [emphasis supplied]. The aforequoted provision mandates the dissolution and liquidation of the property regime of the spouses upon finality of the decree of legal separation. Such dissolution and liquidation are necessary consequences of the final decree. This legal effect of the decree of legal separation ipso facto or

556

University of the Cordilleras College of Law First Year C S.Y. 2013 - 2014 automatically follows, as an inevitable incident of, the judgment decreeing legal separationfor the purpose of determining the share of each spouse in the conjugal assets. Even American courts have made definite pronouncements on the aforestated legal effect of a divorce (legal separation) decree. Generally speaking, the purpose of a decree in divorce insofar as the disposition of property is concerned is to fix and make certain the property rights and interests of the parties (Mich-Westgate vs. Westgate, 288 N.W. 860, 291 Mich. 18, 300 [1] p. 354, C.J.S. Vol. 27B); and it has been held that the provisions of the decree should definitely and finally determine the property rights and interests of the parties (Wash.Shaffer vs. Shaffer, 262 P. 2d. 763, 43 Wash. 2d 629; 300 [11 p. 354 C.J.S. Vol. 27B); and that any attempted reservation of such questions for future determination is improper and error (Mich.Karwowski vs. Karwowski, 20 N.W. 2d 851, 313 Mich. 167, 300 11] p. 354, C.J.S., Vol. 27B; emphasis supplied). Some statutes providing for the division or disposition of the property of the parties to a divorce have been held mandatory and hence to require the court to decree some division of their property rights (U.S.Pearce vs. CIR, 62 S. Ct. 154, 315 U.S. 543, 86 L. ed. 1016, construing Texas statute; 291 [1] p. 263 C.J.S. Vol. 27B). Likewise, it has been held that the settlement of some pro-property rights between the parties is an incident of every decree of divorce where there is any property involved (Utah-Smith vs. Smith, 291 P. 298, 77 Utah 60, 291 [1] p. 264, C.J.S., Vol. 27B). It has been held that notwithstanding the division of property between the parties, the subject matter of a divorce action remains the marital status of the parties, the settlement of the property rights being merely incidental(Wash.-State ex rel. Atkins vs. Superior Court of King Country, 97 P. 2d. 139, 1 Wash. 2d 677; 291 [1] p. 264 C.J.S., Vol. 27B; emphasis supplied). Under other authorities, by the very nature of the litigation, all property rights growing out of marital relations are settled and included in divorce proceedings (Ind.-Novak vs. Novak, 133 N.E. 2d 578, 126 Ind. App. 428) and a decree of divorce is an adjudication of all property rights connected with the marriage and precludes the parties as to all matters which might have been legitimately proved in support of charges or defenses in the action (U.S.Spreckles vs. Wakefield, C.C.A. 286 F. 465) and bars any action thereafter brought by either party to determine the question of property rights (Fla.Cooper vs. Cooper, 69 So. 2d 881; Finston vs. Finston, 37 So. 2d 423,160 Fla. 935; p. 751, C.J.S. Vol. 27A). An absolute divorce ordinarily terminates all property rights and interests, not actually vested, of divorced persons in property of each other, which are dependent on the marriage (U.S.Cockrill vs. Woodson, D.C. Mo., 70 F. 752), at least where no proceedings have been taken to vacate or modify the decree by appeal until the statutory time therefor has expired (Kan.Roberts vs. Fagan 92 P. 559, 76 Kan. 536). Accordingly, unless the court granting the decree is without jurisdiction, inchoate rights of the wife in the husband's property are usually cut off (KyBowling vs. Little, 206 S.W. 1, 182 Ky 86) especially where by the terms of the decree all property obtained by either spouse from or through the other during the marriage is restored to such spouse (Tex. Houston, etc., R. Co. vs. Helm, Civ. App. 93 S.W. 697; pp. 752753, C.J.S. Vol. 27A). Enunciating with directness and finality, one U.S. court held: "The part of a divorce suit regarding property is a part of the very divorce action itself" (Tex.Ex parte Scott 123 S.W. 2d. 306, 313, 133 Tex. 1, answers to certified questions conformed to, Civ. App. 126, S.W. 2d 525; 291 [1] p. 264, C.J.S. Vol. 27B). Petitioner erred in invoking the case of Vda. de Zaldarriaga vs. Zaldarriaga which in turn cited the doctrine of Fuentebella vs. Carrascoso, which We have already declared abrogated in the case of Miranda vs. Court of Appeals (L-33007, 71 SCRA 295, [June 18, 1976]). In this case, this Court explicitly stated: For the guidance of the bench and bar, the court declares as abandoned the doctrine of Fuentebella vs. Carrascoso and adopts the opposite rule that judgments for recovery with accounting are final and appealable (without need of awaiting the accounting) and would become final and executory if not appealed within the reglementary period. In resolving the question of whether or not the judgment directing an accounting in an action for recovery of properties is final and appealable, this Court further explained: The judgment "directing an accounting is appealable, regardless of whether the accounting is the principal relief sought or a mere incident or consequence of the judgment which grants recovery and delivery of absconded properties as the principal relief and expressly provides that"a judgment or order directing an accounting in an

557

University of the Cordilleras College of Law First Year C S.Y. 2013 - 2014 action, shall not be stayed after its rendition and before an appeal is taken or during the pendency of an appeal. xxx xxx xxx If a judgment which directs solely an accounting is appealable notwithstanding that it "does not finally dispose of the action and the accounting has yet to be rendered to complete the relief sought," much more so is a judgment which orders accounting as a mere incident appealable, because the judgment which orders the delivery of properties does finally dispose of the action on its merits, xxx xxx xxx Imperative and controlling considerations of public policy and of sound practice in the courts to achieve the desideratum of just, speedy and inexpensive determination of every action militate against such a novel and unprecedented situation where a judgment on the merits for recovery of properties would be left dangling and would be considered as "interlocutory" and subject to revision and alteration at will for as long as the accounting ordered as a mere incident and logical consequence has not been rendered and acted upon by the trial court. xxx xxx xxx The Court, however, deems it proper for the guidance of the bench and bar to now declare as is clearly indicated from the compelling reasons and considerations hereinabove stated: that the court considers the better rule to be that stated in H.E. Heacock Co. vs. American Trading Co. (53 Phil. 481 [19291, to wit, that where the primary purpose of a case is to ascertain and determine whobetween plaintiff and defendant is the true owner and entitled to the exclusive use of the disputed property, "the judgment ... rendered by the lower court [is] a judgment on the merits as to those questions, and (that) the order of the court for an accounting was based upon and is incidental to the judgment on the merits. That is to say, that the judgment ... (is) a final judgment ... ; that in this kind of a case an accounting is a mere incident to the judgment; that an appeal lies from the rendition of the judgment as rendered ... xxx xxx xxx that accordingly, the contrary ruling in Fuentebella vs. Carrascoso which expressly reversed the Heacock case and a line of similar decisions (Africa vs. Africa, 42 Phil. 934; Villanueva vs. Capistrano; Prophylactic Brush Co., et al. vs. Court of Appeals, G.R. No. 46254, Nov. 23, 1938 [Unpublished) and ruled that such a decision for recovery of property with accounting 'is not final but merely interlocutory and therefore not appealable and subsequent cases Adhering to the same Zaldarriaga vs. Enriquez, 1 SCRA 1188) must be now in turn abandoned and set aside. xxx xxx xxx The Court's considered opinion is that imperative considerations of public policy and of sound practice in the courts and adherence to the constitutional mandate of simplified, just, speedy and inexpensive determination of every action can for considering such judgments for recovery of property with accounting as final judgments which are duly appealable (and would therefore become final and executory if not appealed within the reglementary period) with the accounting as a mereincident of the judgment to be rendered during the course of the appeal as provided in Rule 39, section 4 or to be implemented at the execution stage upon final affirmance on appeal of the judgment (as in Court of Industrial Relations unfair labor practice cases ordering reinstatement of the worker with accounting, computation and payment of his backwages less earnings elsewhere during his layoff) and that the only reason given in Fuentebella for the contrary ruling, viz, "the general harm that would follow from throwing the door open to multiplicity of appeals in a single case is of lesser import and consequence". Considering the aforestated well-established jurisprudence on the matter, the clear mandate of Article 106 of the Civil Code and the aforequoted ruling in the Miranda case, the decision of the trial court dated January 4, 1973 decreeing the legal separation between then spouses Antonio Macadangdang and Filomena Gaviana Macadangdang had long become final and executory and the division of the conjugal property in a "supplemental decision" is a mere incident of the decree of legal separation. Since We have ruled on the finality of the judgment decreeing the spouses' legal separation as of January 4, 1973, the remaining issue for Our resolution is the final disposition of their conjugal partnership of 558

University of the Cordilleras College of Law First Year C S.Y. 2013 - 2014 gains which partnership, by reason of the final decree, had been automatically dissolved. The law (Articles 106, 107 and 176 of the Civil Code) clearly spells out the effects of a final decree of legal separation on the conjugal property. The death on November 30, 1979 of herein petitioner who was declared the guilty spouse by the trial court, before the liquidation of the conjugal property is effected, poses a new problem which can be resolved simply by the application of the rules on intestate succession with respect to the properties of the deceased petitioner. Thus, the rules on dissolution and liquidation of the conjugal partnership of gains under the aforecited provisions of the Civil Code would be applied effective January 4, 1973 when the decree of legal separation became final. Upon the liquidation and distribution conformably with the law governing the effects of the final decree of legal separation, the law on intestate succession should take over in the disposition of whatever remaining properties have been allocated to petitioner. This procedure involves details which properly pertain to the lower court. The properties that may be allocated to the deceased petitioner by virtue of the liquidation of the conjugal assets, shall be distributed in accordance with the laws of intestate succession in Special Proceedings No. 134. WHEREFORE, THIS PETITION IS HEREBY DISMISSED, WITH COSTS AGAINST PETITIONER'S ESTATE. SO ORDERED. Fernandez, Guerrero and Melencio-Herrera, JJ., concur. Separate Opinions TEEHANKEE, J., concurring: I concur in the result. Assuming the finality of the January 4, 1973 decision of legal separation, and considering that "In effect, private respondent agrees with petitioner's counsel that her husband's death has rendered the instant petition moot and academic" (main opinion, at page 5), it might be the more efficacious and compassionate procedure to consider moot the legal separation proceedings and the division and liquidation of the conjugal partnership and properties that would have followed in a "supplemental decision", had not death intervened. There would seem to be no need of continuing these legal separation proceedings, following through herein on the division of the conjugal properties as of January 4, 1973 and simultaneously proceeding with the intestate proceedings for the settlement of the deceased's estate, Sp. Proc. No. 134, when such division and settlement could more expeditiously be accomplished in the intestate proceedings. After all, it would appear that the forfeiture of profits in the conjugal partnership of gains imposed by Article 106, paragraph 2 of the Civil Code would not apply in the light of the provision of Article 176 that such forfeiture "shall not apply ... if the conjugal partnership property came mostly or entirely from the work or industry, or from the wages and salaries, or from the fruits of the separate property of the guilty spouse." Prescinding therefrom, there is no point in the respondent wife's insisting upon such forfeiture, since with petitioner's death, the provision of paragraph 4 of Article 106 that "The offending spouse shall be disqualified from inheriting from the innocent spouse by intestate succession" has likewise become moot and of no application. What is more, respondent wife and the children of their marriage are after all the intestate and forced heirs of the deceased petitioner and the inheritors of his estate. Even the costs awarded against petitioner's estate would thus come in effect from their own pockets. Separate Opinions TEEHANKEE, J., concurring: I concur in the result. Assuming the finality of the January 4, 1973 decision of legal separation, and considering that "In effect, private respondent agrees with petitioner's counsel that her husband's death has rendered the instant petition moot and academic" (main opinion, at page 5), it might be the more efficacious and compassionate procedure to consider moot the legal separation proceedings and the division and liquidation of the conjugal partnership and properties that would have followed in a "supplemental decision", had not death intervened. There would seem to be no need of continuing these legal separation proceedings, following through herein on the division of the conjugal properties as of January 4, 1973 and simultaneously proceeding with the intestate proceedings for the settlement of the deceased's estate, Sp. Proc. No. 134, when such division and settlement could more expeditiously be accomplished in the intestate proceedings. After all, it would appear that the forfeiture of profits in the conjugal partnership of gains imposed by Article 106, paragraph 2 of the Civil Code would not apply in the light of the provision of Article 176 that 559

University of the Cordilleras College of Law First Year C S.Y. 2013 - 2014 such forfeiture "shall not apply ... if the conjugal partnership property came mostly or entirely from the work or industry, or from the wages and salaries, or from the fruits of the separate property of the guilty spouse." Prescinding therefrom, there is no point in the respondent wife's insisting upon such forfeiture, since with petitioner's death, the provision of paragraph 4 of Article 106 that "The offending spouse shall be disqualified from inheriting from the innocent spouse by intestate succession" has likewise become moot and of no application. What is more, respondent wife and the children of their marriage are after all the intestate and forced heirs of the deceased petitioner and the inheritors of his estate. Even the costs awarded against petitioner's estate would thus come in effect from their own pockets.

560

University of the Cordilleras College of Law First Year C S.Y. 2013 - 2014 Case Digest Macadangdang vs Court of Appeals G.R. No. L-38287 October 23, 1981 J. Makasiar Facts: Respondent Filomena Gaviana Macadangdang and petitioner Antonio Macadangdang were married in 1946 after having lived together for two years and had 6 children. They started a buy and sell business and sari-sari store in Davao City. Through hard work and good fortune, their business grew and expanded into merchandising, trucking, transportation, rice and corn mill business, abaca stripping, real estate etc. Their relationship became complicated and both indulged in extramarital relations. Married life became intolerable so they separated in 1965 when private respondent left for Cebu for good. When she returned in Davao in 1971, she learned of the illicit affairs of her estranged husband. She then decided to take the initial action. In April 1971, she instituted a complaint for legal separation. Issue: Whether or not the death of a spouse after a final decree of legal separation has effect on the legal separation. Ruling: The death of a spouse after a final decree of legal separation has no effect on the legal separation. When the decree itself is issued, the finality of the separation is complete after the lapse of the period to appeal the decision to a higher court even if the effects, such as the liquidation of the property, have not yet been commenced nor terminated. The law clearly spells out the effect of a final decree of legal separation on the conjugal property. Therefore, upon the liquidation and distribution confortably with the effects of such final decree, the law on intestate succession should take over the disposition of whatever remaining properties have been allocated to the deceased spouse. Such dissolution and liquidation are necessary consequences of the final decree. Article 106 of the Civil Code, now Article 63 of the Family Code provides the effects of the decree of legal separation. These legal effects ipso facto or automatically follows, as an inevitable incident of the judgment decreeing legal separation, for the purpose of determining the share of each spouse in the conjugal assets.

561

University of the Cordilleras College of Law First Year C S.Y. 2013 - 2014 Lapuz-Sy vs Eufemio 43 SCRA 177 G.R. No. L-30977 January 31, 1972 Full Case CARMEN LAPUZ SY, represented by her substitute MACARIO LAPUZ, petitioner-appellant, vs. EUFEMIO S. EUFEMIO alias EUFEMIO SY UY, respondent-appellee. REYES J.B.L., J.:p Petition, filed after the effectivity of Republic Act 5440, for review by certiorari of an order, dated 29 July 1969, of the Juvenile and Domestic Relations Court of Manila, in its Civil Case No. 20387, dismissing said case for legal separation on the ground that the death of the therein plaintiff, Carmen O. Lapuz Sy, which occurred during the pendency of the case, abated the cause of action as well as the action itself. The dismissal order was issued over the objection of Macario Lapuz, the heir of the deceased plaintiff (and petitioner herein) who sought to substitute the deceased and to have the case prosecuted to final judgment. On 18 August 1953, Carmen O. Lapuz Sy filed a petition for legal separation against Eufemio S. Eufemio, alleging, in the main, that they were married civilly on 21 September 1934 and canonically on 30 September 1934; that they had lived together as husband and wife continuously until 1943 when her husband abandoned her; that they had no child; that they acquired properties during their marriage; and that she discovered her husband cohabiting with a Chinese woman named Go Hiok at 1319 Sisa Street, Manila, on or about March 1949. She prayed for the issuance of a decree of legal separation, which, among others, would order that the defendant Eufemio S. Eufemio should be deprived of his share of the conjugal partnership profits. In his second amended answer to the petition, herein respondent Eufemio S. Eufemio alleged affirmative and special defenses, and, along with several other claims involving money and other properties, counterclaimed for the declaration of nullity ab initio of his marriage with Carmen O. Lapuz Sy, on the ground of his prior and subsisting marriage, celebrated according to Chinese law and customs, with one Go Hiok, alias Ngo Hiok. Issues having been joined, trial proceeded and the parties adduced their respective evidence. But before the trial could be completed (the respondent was already scheduled to present surrebuttal evidence on 9 and 18 June 1969), petitioner Carmen O. Lapuz Sy died in a vehicular accident on 31 May 1969. Counsel for petitioner duly notified the court of her death. On 9 June 1969, respondent Eufemio moved to dismiss the "petition for legal separation" 1 on two (2) grounds, namely: that the petition for legal separation was filed beyond the one-year period provided for in Article 102 of the Civil Code; and that the death of Carmen abated the action for legal separation. On 26 June 1969, counsel for deceased petitioner moved to substitute the deceased Carmen by her father, Macario Lapuz. Counsel for Eufemio opposed the motion. On 29 July 1969, the court issued the order under review, dismissing the case. 2 In the body of the order, the court stated that the motion to dismiss and the motion for substitution had to be resolved on the question of whether or not the plaintiff's cause of action has survived, which the court resolved in the negative. Petitioner's moved to reconsider but the motion was denied on 15 September 1969. After first securing an extension of time to file a petition for review of the order of dismissal issued by the juvenile and domestic relations court, the petitioner filed the present petition on 14 October 1969. The same was given due course and answer thereto was filed by respondent, who prayed for the affirmance of the said order. 3 Although the defendant below, the herein respondent Eufemio S. Eufemio, filed counterclaims, he did not pursue them after the court below dismissed the case. He acquiesced in the dismissal of said counterclaims by praying for the affirmance of the order that dismissed not only the petition for legal separation but also his counterclaim to declare the Eufemio-Lapuz marriage to be null and void ab initio. But petitioner Carmen O. Lapuz Sy (through her self-assumed substitute for the lower court did not act on the motion for substitution) stated the principal issue to be as follows: When an action for legal separation is converted by the counterclaim into one for a declaration of nullity of a marriage, does the death of a party abate the proceedings? The issue as framed by petitioner injects into it a supposed conversion of a legal separation suit to one for declaration of nullity of a marriage, which is without basis, for even petitioner asserted that "the respondent has acquiesced to the dismissal of his counterclaim" (Petitioner's Brief, page 22). Not only this. The petition for legal separation and the counterclaim to declare the nullity of the self same marriage 562

University of the Cordilleras College of Law First Year C S.Y. 2013 - 2014 can stand independent and separate adjudication. They are not inseparable nor was the action for legal separation converted into one for a declaration of nullity by the counterclaim, for legal separation presupposes a valid marriage, while the petition for nullity has a voidable marriage as a pre-condition. The first real issue in this case is: Does the death of the plaintiff before final decree, in an action for legal separation, abate the action? If it does, will abatement also apply if the action involves property rights? . An action for legal separation which involves nothing more than the bed-and-board separation of the spouses (there being no absolute divorce in this jurisdiction) is purely personal. The Civil Code of the Philippines recognizes this in its Article 100, by allowing only the innocent spouse (and no one else) to claim legal separation; and in its Article 108, by providing that the spouses can, by their reconciliation, stop or abate the proceedings and even rescind a decree of legal separation already rendered. Being personal in character, it follows that the death of one party to the action causes the death of the action itself actio personalis moritur cum persona. ... When one of the spouses is dead, there is no need for divorce, because the marriage is dissolved. The heirs cannot even continue the suit, if the death of the spouse takes place during the course of the suit (Article 244, Section 3). The action is absolutely dead (Cass., July 27, 1871, D. 71. 1. 81; Cass. req., May 8, 1933, D. H. 1933, 332.") 4 . Marriage is a personal relation or status, created under the sanction of law, and an action for divorce is a proceeding brought for the purpose of effecting a dissolution of that relation. The action is one of a personal nature. In the absence of a statute to the contrary, the death of one of the parties to such action abates the action, for the reason that death has settled the question of separation beyond all controversy and deprived the court of jurisdiction, both over the persons of the parties to the action and of the subject-matter of the action itself. For this reason the courts are almost unanimous in holding that the death of either party to a divorce proceeding, before final decree, abates the action. 1 Corpus Juris, 208; Wren v. Moss, 2 Gilman, 72; Danforth v. Danforth, 111 Ill. 236; Matter of Grandall, 196 N.Y. 127, 89 N.E. 578; 134 Am St. Rep. 830; 17 Ann. Cas. 874; Wilcon v. Wilson, 73 Mich, 620, 41 N.W. 817; Strickland v. Strickland, 80 Ark. 452, 97 S. W. 659; McCurley v. McCurley, 60 Md. 185, 45 Am. Rep. 717; Begbie v. Begbie, 128 Cal. 155, 60 Pac. 667, 49 L.R.A. 141. 5 The same rule is true of causes of action and suits for separation and maintenance (Johnson vs. Bates, Ark. 101 SW 412; 1 Corpus Juris 208). A review of the resulting changes in property relations between spouses shows that they are solely the effect of the decree of legal separation; hence, they can not survive the death of the plaintiff if it occurs prior to the decree. On the point, Article 106 of the Civil Code provides: . Art. 106. The decree of legal separation shall have the following effects: (1) The spouses shall be entitled to live separately from each other, but the marriage bonds shall not be severed; . (2) The conjugal partnership of gains or the absolute conjugal community of property shall be dissolved and liquidated, but the offending spouse shall have no right to any share of the profits earned by the partnership or community, without prejudice to the provisions of article 176; (3) The custody of the minor children shall be awarded to the innocent spouse, unless otherwise directed by the court in the interest of said minors, for whom said court may appoint a guardian; (4) The offending spouse shall be disqualified from inheriting from the innocent spouse by intestate succession. Moreover, provisions in favor of the offending spouse made in the will of the innocent one shall be revoked by operation of law. From this article it is apparent that the right to the dissolution of the conjugal partnership of gains (or of the absolute community of property), the loss of right by the offending spouse to any share of the profits earned by the partnership or community, or his disqualification to inherit by intestacy from the innocent spouse as well as the revocation of testamentary provisions in favor of the offending spouse made by the innocent one, are all rights and disabilities that, by the very terms of the Civil Code article, are vested exclusively in the persons of the spouses; and by their nature and intent, such claims and disabilities are difficult to conceive as assignable or transmissible. Hence, a claim to said rights is not a claim that "is not thereby extinguished" after a party dies, under Section 17, Rule 3, of the Rules of Court, to warrant continuation of the action through a substitute of the deceased party.

563

University of the Cordilleras College of Law First Year C S.Y. 2013 - 2014 Sec. 17. Death of party. After a party dies and the claim is not thereby extinguished, the court shall order, upon proper notice, the legal representative of the deceased to appear and to be substituted for the deceased, within a period of thirty (30) days, or within such time as may be granted... The same result flows from a consideration of the enumeration of the actions that survive for or against administrators in Section 1, Rule 87, of the Revised Rules of Court: SECTION 1. Actions which may and which may not be brought against executor or administrator. No action upon a claim for the recovery of money or debt or interest thereon shall be commenced against the executor or administrator; but actions to recover real or personal property, or an interest therein, from the estate, or to enforce a lien thereon, and actions to recover damages for an injury to person or property, real or personal, may be commenced against him. Neither actions for legal separation or for annulment of marriage can be deemed fairly included in the enumeration.. A further reason why an action for legal separation is abated by the death of the plaintiff, even if property rights are involved, is that these rights are mere effects of decree of separation, their source being the decree itself; without the decree such rights do not come into existence, so that before the finality of a decree, these claims are merely rights in expectation. If death supervenes during the pendency of the action, no decree can be forthcoming, death producing a more radical and definitive separation; and the expected consequential rights and claims would necessarily remain unborn. As to the petition of respondent-appellee Eufemio for a declaration of nullity ab initio of his marriage to Carmen Lapuz, it is apparent that such action became moot and academic upon the death of the latter, and there could be no further interest in continuing the same after her demise, that automatically dissolved the questioned union. Any property rights acquired by either party as a result of Article 144 of the Civil Code of the Philippines 6 could be resolved and determined in a proper action for partition by either the appellee or by the heirs of the appellant. In fact, even if the bigamous marriage had not been void ab initio but only voidable under Article 83, paragraph 2, of the Civil Code, because the second marriage had been contracted with the first wife having been an absentee for seven consecutive years, or when she had been generally believed dead, still the action for annulment became extinguished as soon as one of the three persons involved had died, as provided in Article 87, paragraph 2, of the Code, requiring that the action for annulment should be brought during the lifetime of any one of the parties involved. And furthermore, the liquidation of any conjugal partnership that might have resulted from such voidable marriage must be carried out "in the testate or intestate proceedings of the deceased spouse", as expressly provided in Section 2 of the Revised Rule 73, and not in the annulment proceeding. ACCORDINGLY, the appealed judgment of the Manila Court of Juvenile and Domestic Relations is hereby affirmed. No special pronouncement as to costs. Concepcion, C.J., Makalintal, Zaldivar, Castro, Fernando, Teehankee, Barredo, Villamor and Makasiar, JJ., concur.

564

University of the Cordilleras College of Law First Year C S.Y. 2013 - 2014 Case Digest Lapuz-Sy vs Eufemio G.R. No. L-30977 January 31, 1972 J. Reyes Facts: Carmen Lapuz-Sy filed a petition for legal separation against Eufemio Eufemio on August 1953. They were married civilly on September 21, 1934 and canonically after nine days. They had lived together as husband and wife continuously without any children until 1943 when her husband abandoned her. They acquired properties during their marriage. Petitioner then discovered that her husband cohabited with a Chinese woman named Go Hiok on or about 1949. She prayed for the issuance of a decree of legal separation, which among others, would order that the defendant Eufemio should be deprived of his share of the conjugal partnership profits. Eufemio counterclaimed for the declaration of nullity of his marriage with Lapuz-Sy on the ground of his prior and subsisting marriage with Go Hiok. Trial proceeded and the parties adduced their respective evidence. However, before the trial could be completed, respondent already scheduled to present surrebuttal evidence, petitioner died in a vehicular accident on May 1969. Her counsel duly notified the court of her death. Eufemio moved to dismiss the petition for legal separation on June 1969 on the grounds that the said petition was filed beyond the oneyear period provided in Article 102 of the Civil Code and that the death of Carmen abated the action for legal separation. Petitioners counsel moved to substitute the deceased Carmen by her father, Macario Lapuz. Issue: Whether the death of the plaintiff, before final decree in an action for legal separation, abate the action and will it also apply if the action involved property rights. Ruling: An action for legal separation is abated by the death of the plaintiff, even if property rights are involved. These rights are mere effects of decree of separation, their source being the decree itself; without the decree such rights do not come into existence, so that before the finality of a decree, these claims are merely rights in expectation. If death supervenes during the pendency of the action, no decree can be forthcoming, death producing a more radical and definitive separation; and the expected consequential rights and claims would necessarily remain unborn. The petition of Eufemio for declaration of nullity is moot and academic and there could be no further interest in continuing the same after her demise, that automatically dissolved the questioned union. Any property rights acquired by either party as a result of Article 144 of the Civil Code of the Philippines 6 could be resolved and determined in a proper action for partition by either the appellee or by the heirs of the appellant.

565

University of the Cordilleras College of Law First Year C S.Y. 2013 - 2014 Pcete vs Carriago 231 SCRA 321 G.R. No. L-53880 March 17, 1994 Full Case ENRICO L. PACETE, CLARITA DE LA CONCEPCION, EMELDA C. PACETE, EVELINA C. PACETE and EDUARDO C. PACETE, petitioners, vs. HON. GLICERIO V. CARRIAGA, JR. and CONCEPCION (CONCHITA) ALANIS PACETE, respondents. VITUG, J.: The issue in this petition for certiorari is whether or not the Court of First Instance (now Regional Trial Court) of Cotabato, Branch I, in Cotabato City, gravely abused its discretion in denying petitioners' motion for extension of time to file their answer in Civil Case No. 2518, in declaring petitioners in default and in rendering its decision of 17 March 1980 which, among other things, decreed the legal separation of petitioner Enrico L. Pacete and private respondent Concepcion Alanis and held to be null and void ab initio the marriage of Enrico L. Pacete to Clarita de la Concepcion. On 29 October 1979, Concepcion Alanis filed with the court below a complaint for the declaration of nullity of the marriage between her erstwhile husband Enrico L. Pacete and one Clarita de la Concepcion, as well as for legal separation (between Alanis and Pacete), accounting and separation of property. In her complaint, she averred that she was married to Pacete on 30 April 1938 before the Justice of the Peace of Cotabato, Cotabato; that they had a child named Consuelo who was born on 11 March 1943; that Pacete subsequently contracted (in 1948) a second marriage with Clarita de la Concepcion in Kidapawan, North Cotabato; that she learned of such marriage only on 01 August 1979; that during her marriage to Pacete, the latter acquired vast property consisting of large tracts of land, fishponds and several motor vehicles; that he fraudulently placed the several pieces of property either in his name and Clarita or in the names of his children with Clarita and other "dummies;" that Pacete ignored overtures for an amicable settlement; and that reconciliation between her and Pacete was impossible since he evidently preferred to continue living with Clarita. The defendants were each served with summons on 15 November 1979. They filed a motion for an extension of twenty (20) days from 30 November 1979 within which to file an answer. The court granted the motion. On 18 December 1979, appearing through a new counsel, the defendants filed a second motion for an extension of another thirty (30) days from 20 December 1979. On 07 January 1980, the lower court granted the motion but only for twenty (20) days to be counted from 20 December 1979 or until 09 January 1980. The Order of the court was mailed to defendants' counsel on 11 January 1980. Likely still unaware of the court order, the defendants, on 05 February 1980, again filed another motion (dated 18 January 1980) for an extension of "fifteen (15) days counted from the expiration of the 30-day period previously sought" within which to file an answer. The following day, or on 06 February 1980, the court denied this last motion on the ground that it was "filed after the original period given . . . as first extension had expired." 1 The plaintiff thereupon filed a motion to declare the defendants in default, which the court forthwith granted. The plaintiff was then directed to present her evidence. 2 The court received plaintiff's evidence during the hearings held on 15, 20, 21 and 22 February 1980. On 17 March 1980, the court 3 promulgated the herein questioned decision, disposing of the case, thus WHEREFORE, order is hereby issued ordering: 1. The issuance of a Decree of Legal Separation of the marriage between, the plaintiff, Concepcion (Conchita) Alanis Pacete and the herein defendants, Enrico L. Pacete, in accordance with the Philippine laws and with consequences, as provided for by our laws; 2. That the following properties are hereby declared as the conjugal properties of the partnership of the plaintiff, Concepcion (Conchita) Alanis Pacete and the defendant, Enrico L. Pacete, half and half, to wit: 1. The parcel of land covered by TCT No. V-815 which is a parcel of land situated in the barrio of Langcong, Municipality of Matanog (previously of Parang), province of Maguindanao (previously of Cotabato province) with an area of 45,265 square meters registered in the name of Enrico Pacete, Filipino, of legal age, married to Conchita Alanis as shown in Exhibits "B" and "B-1" for the plaintiff. 2. A parcel of land covered by Transfer Certificate of Title No. T-20442, with an area of 538 square meters and covered by Tax Declaration No. 2650 (74) in the name of Enrico Pacete, situated in the Poblacion of Kidapawan, North Cotabato, together with all its 566

University of the Cordilleras College of Law First Year C S.Y. 2013 - 2014 improvements, which parcel of land, as shown by Exhibits "K-1" was acquired by way of absolute deed of sale executed by Amrosio Mondog on January 14, 1965. 3. A parcel of land covered by Transfer Certificate of Title No. T-20424 and covered by Tax Declaration No. 803 (74), with an area of 5.1670 hectares, more or less, as shown by Exhibit "R", the same was registered in the name of Enrico Pacete and the same was acquired by Enrico Pacete last February 17, 1967 from Ambag Ampoy, as shown by Exhibit "R-1", situated at Musan, Kidapawan, North Cotabato. 4. A parcel of land situated at Lanao, Kidapawan, North Cotabato, with an area of 5.0567 hectares, covered by Tax Declaration No. 4332 (74), as shown by Exhibit "S", and registered in the name of Enrico Pacete. 5. A parcel of land covered by Transfer Certificate of Title No. T-9750, situated at Lika, Mlang, North Cotabato, with an area of 4.9841 hectares and the same is covered by Tax Declaration No. 803 (74) and registered in the name of Enrico Pacete and which land was acquired by Enrico Pacete from Salvador Pacete on September 24, 1962, as shown by Exhibit "Q-1". 6. A parcel of land covered by Transfer Certificate of Title No. T-9944, with an area of 9.9566 and also covered by Tax Declaration No. 8608 (74) and registered in the name of the defendant Enrico L. Pacete which Enrico L. Pacete acquired from Sancho Balingcos last October 22, 1962, as shown by Exhibit "L-1" and which parcel of land is situated at (Kialab), Kiab, Matalam, North Cotabato. 7. A parcel of land covered by Transfer Certificate of Title No. T-9227, situated at Kiab, Matalam, North Cotabato, with an area of 12.04339 hectares, more or less, and also covered by Tax Declaration No. 8607 (74) both in the name of the defendant Enrico L. Pacete which he acquired last October 15, 1962 from Minda Bernardino, as shown by Exhibit "M-1". 8. A parcel of land covered by Transfer Certificate of Title No. T-9228, situated at Kiab, Matalam, North Cotabato, with an area of 10.8908 hectares, registered in the name of Enrico Pacete and also covered by Tax Declaration No. 5781 (74) in the name of Enrico Pacete and which parcel of land he acquired last September 25, 1962 from Conchita dela Torre, as shown by Exhibit "P-1". 9. A parcel of land covered by Transfer Certificate of Title No. T-10301, situated at Linao, Matalam, North Cotabato, with an area of 7.2547 hectares, registered in the name of Enrico Pacete and also covered by Tax Declaration No. 8716 (74) also in the name of Enrico Pacete which Enrico Pacete acquired from Agustin Bijo last July 16, 1963, as shown by Exhibit "N-1". 10. A parcel of land covered by Transfer Certificate of Title No. 12728 in the name of the defendant, Enrico L. Pacete, with an area of 10.9006 hectares, situated at Linao, Matalam, North Cotabato and is also covered by Tax Declaration No. 5745 (74) in the name of Enrico Pacete, as shown on Exhibit "O" and which Enrico Pacete acquired last December 31, 1963 from Eliseo Pugni, as shown on Exhibit "0-1". 3. Ordering the Cancellation of Original Certificate of Title No. P-34243 covering Lot No. 1066, issued in the name of Evelina Pacete, situated at Kiab, Matalam, North Cotabato, and ordering the registration of the same in the joint name of Concepcion (Conchita) Alanis Pacete and Enrico L. Pacete as their conjugal property, with address on the part of Concepcion (Conchita) Alanis Pacete at Parang, Maguindanao and on the part of Enrico L. Pacete at Kidapawan, North Cotabato. 4. Ordering likewise the cancellation of Original Certificate of Title No. V-20101, covering Lot No. 77, in the name of Eduardo C. Pacete, situated at New Lawaan, Mlang, North Cotabato, and the issuance of a new Transfer Certificate of Title in the joint name of (half and half) Concepcion (Conchita) Alanis Pacete and Enrico L. Pacete. 5. Ordering likewise the cancellation of Original Certificate of Title No. P-29890, covering Lot 1068, situated at Kiab, Matalam, North Cotabato, with an area of 12.1031 hectares, in the name of Emelda C. Pacete and the issuance of a new Transfer Certificate of Title in the joint name (half and half) of Concepcion (Conchita) Alanis Pacete and Enrico L. Pacete; and declaring that the fishpond situated at Barrio Tumanan, Bislig, Surigao Del Sur, with an area of 48 hectares and covered by Fishpond Lease Agreement of Emelda C. Pacete, dated July 29, 1977 be cancelled and in lieu thereof, the joint name 567

University of the Cordilleras College of Law First Year C S.Y. 2013 - 2014 of Concepcion (Conchita) Alanis Pacete and her husband, Enrico L. Pacete, be registered as their joint property, including the 50 hectares fishpond situated in the same place, Barrio Timanan, Bislig, Surigao del Sur. 6. Ordering the following motor vehicles to be the joint properties of the conjugal partnership of Concepcion (Conchita) Alanis Pacete and Enrico L. Pacete, viz: a. Motor vehicle with Plate No. T-RG-783; Make, Dodge; Motor No. T137-20561; Chassis No. 83920393, and Type, Mcarrier; b. Motor vehicle with Plate No. T-RG-784; Make, Dodge; Motor No. T214-229547; Chassis No. 10D-1302-C; and Type, Mcarrier; c. Motor vehicle with Plate No. J-PR-818; Make, Ford; Motor No. GRW-116188; Chassis No. HOCC-GPW-1161-88-C; Type, Jeep; d. Motor vehicle with Plate No. TH-5J-583; Make, Ford: Motor No. F70MU5-11111; Chassis No. HOCC-GPW-1161188-G; Type, Stake; e. Motor vehicle with Plate No. TH-5J-584; Make, Hino; Motor No. ED300-45758; Chassis No. KB222-22044; Type, Stake; and f. Motor vehicle with Plate No. TH-5J-585; Make, Ford: Motor No. LTC-780-Dv; Chassis No. 10F-13582-K; Type, Stake. 7. Ordering the defendant Enrico L. Pacete to pay the plaintiff the sum of P46,950.00 which is the share of the plaintiff in the unaccounted income of the ricemill and corn sheller for three years from 1971 to 1973. 8. Ordering the defendant, Enrico L. Pacete, to reimburse the plaintiff the monetary equipment of 30% of whether the plaintiff has recovered as attorney's fees; 9. Declaring the subsequent marriage between defendant Enrico L. Pacete and Clarita de la Concepcion to be void ab initio; and 10. Ordering the defendants to pay the costs of this suit. 4 Hence, the instant special civil action of certiorari. Under ordinary circumstances, the petition would have outrightly been dismissed, for, as also pointed out by private respondents, the proper remedy of petitioners should have instead been either to appeal from the judgment by default or to file a petition for relief from judgment. 5 This rule, however, is not inflexible; a petition forcertiorari is allowed when the default order is improperly declared, or even when it is properly declared, where grave abuse of discretion attended such declaration. 6 In these exceptional instances, the special civil action ofcertiorari to declare the nullity of a judgment by default is available. 7 In the case at bench, the default order unquestionably is not legally sanctioned. The Civil Code provides: Art. 101. No decree of legal separation shall be promulgated upon a stipulation of facts or by confession of judgment. In case of non-appearance of the defendant, the court shall order the prosecuting attorney to inquire whether or not a collusion between the parties exists. If there is no collusion, the prosecuting attorney shall intervene for the State in order to take care that the evidence for the plaintiff is not fabricated. The provision has been taken from Article 30 of the California Civil Code, 8 and it is, in substance, reproduced in Article 60 of the Family Code. 9 Article 101 reflects the public policy on marriages, and it should easily explain the mandatory tenor of the law. InBrown v. Yambao, 10 the Court has observed: The policy of Article 101 of the new Civil Code, calling for the intervention of the state attorneys in case of uncontested proceedings for legal separation (and of annulment of marriages, under Article 88), is to emphasize that marriage is more than a mere contract; that it is a social institution in which the state is vitally interested, so that its continuation or interruption can not be made to depend upon the parties themselves (Civil Code, Article 52; Adong vs. Cheong Gee, 43 Phil. 43; Ramirez v. Gmur, 42 Phil. 855; Goitia v. Campos, 35 Phil. 252). It is consonant with this policy that the inquiry by the Fiscal should be allowed to focus upon any relevant matter that may indicate whether the proceedings for separation or annulment are fully justified or not.

568

University of the Cordilleras College of Law First Year C S.Y. 2013 - 2014 Article 103 of the Civil Code, now Article 58 of the Family Code, further mandates that an action for legal separation must "in no case be tried before six months shall have elapsed since the filing of the petition," obviously in order to provide the parties a "cooling-off" period. In this interim, the court should take steps toward getting the parties to reconcile. The significance of the above substantive provisions of the law is further underscored by the inclusion of the following provision in Rule 18 of the Rules of Court: Sec. 6. No defaults in actions for annulments of marriage or for legal separation. If the defendant in an action for annulment of marriage or for legal separation fails to answer, the court shall order the prosecuting attorney to investigate whether or not a collusion between the parties exists, and if there is no collusion, to intervene for the State in order to see to it that the evidence submitted is not fabricated. The special prescriptions on actions that can put the integrity of marriage to possible jeopardy are impelled by no less than the State's interest in the marriage relation and its avowed intention not to leave the matter within the exclusive domain and the vagaries of the parties to alone dictate. It is clear that the petitioner did, in fact, specifically pray for legal separation. 11 That other remedies, whether principal or incidental, have likewise been sought in the same action cannot dispense, nor excuse compliance, with any of the statutory requirements aforequoted. WHEREFORE, the petition for certiorari is hereby GRANTED and the proceedings below, including the Decision of 17 March 1980 appealed from, are NULLIFIED and SET ASIDE. No costs. SO ORDERED. Feliciano, Bidin, Romero and Melo, JJ., concur.

569

University of the Cordilleras College of Law First Year C S.Y. 2013 - 2014 Case Digest Pacete vs Carriaga G.R. No. L-53880 March 17, 1994 J. Vitug Facts: Concepcion Alanis filed a complaint on October 1979, for the Declaration of Nullity of Marriage between her erstwhile husband Enrico Pacete and one Clarita de la Concepcion, as well as for legal separation between her and Pacete, accounting and separation of property. She averred in her complaint that she was married to Pacete on April 1938 and they had a child named Consuelo; that Pacete subsequently contracted a second marriage with Clarita de la Concepcion and that she learned of such marriage only on August 1979. Reconciliation between her and Pacete was impossible since he evidently preferred to continue living with Clarita. The defendants were each served with summons. They filed an extension within which to file an answer, which the court partly granted. Due to unwanted misunderstanding, particularly in communication, the defendants failed to file an answer on the date set by the court. Thereafter, the plaintiff filed a motion to declare the defendants in default, which the court forthwith granted. The court received plaintiffs evidence during the hearings held on February 15, 20, 21, and 22, 1980. After trial, the court rendered a decision in favor of the plaintiff on March 17, 1980. Issue: Whether or not the RTC gravely abused its discretion in denying petitioners motion for extension of time to file their answer, in declaring petitioners in default and in rendering its decision on March 17, 1980 which decreed the legal separation of Pacete and Alanis and held to be null and void the marriage of Pacete to Clarita. Ruling: The Civil Code provides that no decree of legal separation shall be promulgated upon a stipulation of facts or by confession of judgment. In case of non-appearance of the defendant, the court shall order the prosecuting attorney to inquire whether or not collusion between parties exists. If there is no collusion, the prosecuting attorney shall intervene for the State in order to take care that the evidence for the plaintiff is not fabricated. The above stated provision calling for the intervention of the state attorneys in case of uncontested proceedings for legal separation (and of annulment of marriages, under Article 88) is to emphasize that marriage is more than a mere contract. Article 103 of the Civil Code, now Article 58 of the Family Code, further mandates that an action for legal separation must in no case be tried before six months shall have elapsed since the filing of the petition, obviously in order to provide the parties a cooling-off period. In this interim, the court should take steps toward getting the parties to reconcile. The significance of the above substantive provisions of the law is further or underscored by the inclusion of a provision in Rule 18 of the Rules of Court which provides that no defaults in actions for annulments of marriage or for legal separation. Therefore, if the defendant in an action for annulment of marriage or for legal separation fails to answer, the court shall order the prosecuting attorney to investigate whether or not a collusion between the parties exists, and if there is no collusion, to intervene for the State in order to see to it that the evidence submitted is not fabricated.

570

University of the Cordilleras College of Law First Year C S.Y. 2013 - 2014 Alcantara vs Alcantara 531 SCRA 446 G.R. No. 167746 August 28, 2007 Full Case RESTITUTO M. ALCANTARA -versusROSITA A. ALCANTARA and HON. COURT OF APPEALS, Respondents. CHICO-NAZARIO, J.: Before this Court is a Petition for Review on Certiorari filed by petitioner Restituto Alcantara assailing the Decision of the Court of Appeals dated 30 September 2004 in CA-G.R. CV No. 66724 denying petitioners appeal and affirming the decision of the Regional Trial Court (RTC) of Makati City, Branch 143, in Civil Case No. 97-1325 dated 14 February 2000, dismissing his petition for annulment of marriage. The antecedent facts are: A petition for annulment of marriage was filed by petitioner against respondent Rosita A. Alcantara alleging that on 8 December 1982 he and respondent, without securing the required marriage license, went to the ManilaCity Hall for the purpose of looking for a person who could arrange a marriage for them.They met a person who, for a fee, arranged their wedding before a certain Rev. Aquilino Navarro, a Minister of the Gospel of the CDCC BR Chapel.They got married on the same day, 8 December 1982.Petitioner and respondent went through another marriage ceremony at the San Jose de Manuguit Church in Tondo, Manila, on 26 March 1983.The marriage was likewise celebrated without the parties securing a marriage license.The alleged marriage license, procured in Carmona, Cavite, appearing on the marriage contract, is a sham, as neither party was a resident of Carmona, and they never went to Carmona to apply for a license with the local civil registrar of the said place.On 14 October 1985, respondent gave birth to their child Rose Ann Alcantara.In 1988, they parted ways and lived separate lives.Petitioner prayed that after due hearing, judgment be issued declaring their marriage void and ordering the Civil Registrar to cancel the corresponding marriage contract and its entry on file. Answering petitioners petition for annulment of marriage, respondent asserts the validity of their marriage and maintains that there was a marriage license issued as evidenced by a certification from the Office of the Civil Registry of Carmona, Cavite. Contrary to petitioners representation, respondent gave birth to their first child named Rose Ann Alcantara on 14 October 1985 and to another daughter named Rachel Ann Alcantara on 27 October 1992. Petitioner has a mistress with whom he has three children.Petitioner only filed the annulment of their marriage to evade prosecution for concubinage.Respondent, in fact, has filed a case for concubinage against petitioner before the MetropolitanTrialCourtofMandaluyongCity, Branch 60.Respondent prays that the petition for annulment of marriage be denied for lack of merit. On 14 February 2000, the RTC of Makati City, Branch 143, rendered its Decision disposing as follows: The foregoing considered, judgment is rendered as follows: 1. The Petition is dismissed for lack of merit; 2. Petitioner is ordered to pay respondent the sum of twenty thousand pesos (P20,000.00) per month as support for their two (2) children on the first five (5) days of each month; and 3. To pay the costs. As earlier stated, the Court of Appeals rendered its Decision dismissing the petitioners appeal.His Motion for Reconsideration was likewise denied in a resolution of the Court of Appeals dated 6 April 2005. The Court of Appeals held that the marriage license of the parties is presumed to be regularly issued and petitioner had not presented any evidence to overcome the presumption.Moreover, the parties marriage contract being a public document is a prima facie proof of the questioned marriage under Section 44, Rule 130 of the Rules of Court. In his Petition before this Court, petitioner raises the following issues for resolution: a. The Honorable Court of Appeals committed a reversible error when it ruled that the Petition for Annulment has no legal and factual basis despite the evidence on record that there was no marriage license at the precise moment of the solemnization of the marriage. b. The Honorable Court of Appeals committed a reversible error when it gave weight to the Marriage License No. 7054133 despite the fact that the same was not identified and offered as evidence during the trial, and was not the Marriage license number appearing on the face of the marriage contract.

571

University of the Cordilleras College of Law First Year C S.Y. 2013 - 2014 c. The Honorable Court of Appeals committed a reversible error when it failed to apply the ruling laid down by this Honorable Court in the case of Sy vs. Court of Appeals.(G.R. No. 127263, 12 April 2000 [330 SCRA 550]). d. The Honorable Court of Appeals committed a reversible error when it failed to relax the observance of procedural rules to protect and promote the substantial rights of the party litigants. We deny the petition. Petitioner submits that at the precise time that his marriage with the respondent was celebrated, there was no marriage license because he and respondent just went to the ManilaCity Hall and dealt with a fixer who arranged everything for them.The wedding took place at the stairs in ManilaCity Hall and not in CDCC BR Chapel where Rev. Aquilino Navarro who solemnized the marriage belongs. He and respondent did not go to Carmona, Cavite, to apply for a marriage license.Assuming a marriage license from Carmona, Cavite, was issued to them, neither he nor the respondent was a resident of the place. The certification of the Municipal Civil Registrar of Carmona, Cavite, cannot be given weight because the certification states that Marriage License number 7054133 was issued in favor of Mr. Restituto Alcantara and Miss Rosita Almario but their marriage contract bears the number 7054033 for their marriage license number. The marriage involved herein having been solemnized on 8 December 1982, or prior to the effectivity of the Family Code, the applicable law to determine its validity is the Civil Code which was the law in effect at the time of its celebration. A valid marriage license is a requisite of marriage under Article 53 of the Civil Code, the absence of which renders the marriage void ab initio pursuant to Article 80(3) in relation to Article 58 of the same Code. Article 53 of the Civil Code which was the law applicable at the time of the marriage of the parties states: Art. 53.No marriage shall be solemnized unless all these requisites are complied with: (1) Legal capacity of the contracting parties; (2)Their consent, freely given; (3) Authority of the person performing the marriage; and (4) A marriage license, except in a marriage of exceptional character. The requirement and issuance of a marriage license is the States demonstration of its involvement and participation in every marriage, in the maintenance of which the general public is interested. Petitioner cannot insist on the absence of a marriage license to impugn the validity of his marriage. The cases where the court considered the absence of a marriage license as a ground for considering the marriage void are clear-cut. In Republic of the Philippines v. Court of Appeals, the Local Civil Registrar issued a certification of due search and inability to find a record or entry to the effect that Marriage License No. 3196182 was issued to the parties. The Court held that the certification of due search and inability to find a record or entry as to the purported marriage license, issued by the Civil Registrar of Pasig, enjoys probative value, he being the officer charged under the law to keep a record of all data relative to the issuance of a marriage license. Based on said certification, the Court held that there is absence of a marriage license that would render the marriage void ab initio. In Cario v. Cario, the Court considered the marriage of therein petitioner Susan Nicdao and the deceased Santiago S. Carino as void ab initio.The records reveal that the marriage contract of petitioner and the deceased bears no marriage license number and, as certified by the Local Civil Registrar of San Juan, Metro Manila, their office has no record of such marriage license. The court held that the certification issued by the local civil registrar is adequate to prove the non-issuance of the marriage license. Their marriage having been solemnized without the necessary marriage license and not being one of the marriages exempt from the marriage license requirement, the marriage of the petitioner and the deceased is undoubtedly void ab initio. In Sy v. Court of Appeals, the marriage license was issued on 17 September 1974, almost one year after the ceremony took place on 15 November 1973.The Court held that the ineluctable conclusion is that the marriage was indeed contracted without a marriage license. In all these cases, there was clearly an absence of a marriage license which rendered the marriage void. Clearly, from these cases, it can be deduced that to be considered void on the ground of absence of a marriage license, the law requires that the absence of such marriage license must be apparent on the marriage contract, or at the very least, supported by a certification from the local civil registrar that no 572

University of the Cordilleras College of Law First Year C S.Y. 2013 - 2014 such marriage license was issued to the parties.In this case, the marriage contract between the petitioner and respondent reflects a marriage license number.A certification to this effect was also issued by the local civil registrar of Carmona, Cavite. The certification moreover is precise in that it specifically identified the parties to whom the marriage license was issued, namely Restituto Alcantara and Rosita Almario, further validating the fact that a license was in fact issued to the parties herein. The certification of Municipal Civil Registrar Macrino L. Diaz of Carmona, Cavite, reads: This is to certify that as per the registry Records of Marriage filed in this office, Marriage License No. 7054133 was issued in favor of Mr. Restituto Alcantara and Miss Rosita Almario on December 8, 1982. This Certification is being issued upon the request of Mrs. Rosita A. Alcantara for whatever legal purpose or intents it may serve. This certification enjoys the presumption that official duty has been regularly performed and the issuance of the marriage license was done in the regular conduct of official business. The presumption of regularity of official acts may be rebutted by affirmative evidence of irregularity or failure to perform a duty. However, the presumption prevails until it is overcome by no less than clear and convincing evidence to the contrary. Thus, unless the presumption is rebutted, it becomes conclusive. Every reasonable intendment will be made in support of the presumption and, in case of doubt as to an officers act being lawful or unlawful, construction should be in favor of its lawfulness. Significantly, apart from these, petitioner, by counsel, admitted that a marriage license was, indeed, issued in Carmona, Cavite. Petitioner, in a faint attempt to demolish the probative value of the marriage license, claims that neither he nor respondent is a resident of Carmona, Cavite. Even then, we still hold that there is no sufficient basis to annul petitioner and respondents marriage. Issuance of a marriage license in a city or municipality, not the residence of either of the contracting parties, and issuance of a marriage license despite the absence of publication or prior to the completion of the 10-day period for publication are considered mere irregularities that do not affect the validity of the marriage. An irregularity in any of the formal requisites of marriage does not affect its validity but the party or parties responsible for the irregularity are civilly, criminally and administratively liable. Again, petitioner harps on the discrepancy between the marriage license number in the certification of the Municipal Civil Registrar, which states that the marriage license issued to the parties is No. 7054133, while the marriage contract states that the marriage license number of the parties is number 7054033.Once more, this argument fails to sway us.It is not impossible to assume that the same is a mere a typographical error, as a closer scrutiny of the marriage contract reveals the overlapping of the numbers 0 and 1, such that the marriage license may read either as 7054133 or 7054033. It therefore does not detract from our conclusion regarding the existence and issuance of said marriage license to the parties. Under the principle that he who comes to court must come with clean hands, petitioner cannot pretend that he was not responsible or a party to the marriage celebration which he now insists took place without the requisite marriage license. Petitioner admitted that the civil marriage took place because he initiated it. Petitioner is an educated person. He is a mechanical engineer by profession. He knowingly and voluntarily went to the Manila City Hall and likewise, knowingly and voluntarily, went through a marriage ceremony. He cannot benefit from his action and be allowed to extricate himself from the marriage bond at his mere say-so when the situation is no longer palatable to his taste or suited to his lifestyle. We cannot countenance such effrontery. His attempt to make a mockery of the institution of marriage betrays his bad faith. Petitioner and respondent went through a marriage ceremony twice in a span of less than one year utilizing the same marriage license. There is no claim that he went through the second wedding ceremony in church under duress or with a gun to his head. Everything was executed without nary a whimper on the part of the petitioner. In fact, for the second wedding of petitioner and respondent, they presented to the San Jose de Manuguit Church the marriage contract executed during the previous wedding ceremony before the Manila City Hall. This is confirmed in petitioners testimony as follows WITNESS As I remember your honor, they asked us to get the necessary document prior to the wedding. COURT What particular document did the church asked you to produce? I am referring to the San Jose de Manuguit church. WITNESS 573

University of the Cordilleras College of Law First Year C S.Y. 2013 - 2014 I dont remember your honor. COURT Were you asked by the church to present a Marriage License? WITNESS I think they asked us for documents and I said we have already a Marriage Contract and I dont know if it is good enough for the marriage and they accepted it your honor. COURT In other words, you represented to the San Jose de Manuguit church that you have with you already a Marriage Contract? WITNESS Yes your honor. COURT That is why the San Jose de Manuguit church copied the same marriage License in the Marriage Contract issued which Marriage License is Number 7054033. WITNESS Yes your honor. The logical conclusion is that petitioner was amenable and a willing participant to all that took place at that time. Obviously, the church ceremony was confirmatory of their civil marriage, thereby cleansing whatever irregularity or defect attended the civil wedding. Likewise, the issue raised by petitioner that they appeared before a fixer who arranged everything for them and who facilitated the ceremony before a certain Rev. Aquilino Navarro, a Minister of the Gospel of the CDCC Br Chapel will not strengthen his posture. The authority of the officer or clergyman shown to have performed a marriage ceremony will be presumed in the absence of any showing to the contrary. Moreover, the solemnizing officer is not duty-bound to investigate whether or not a marriage license has been duly and regularly issued by the local civil registrar. All the solemnizing officer needs to know is that the license has been issued by the competent official, and it may be presumed from the issuance of the license that said official has fulfilled the duty to ascertain whether the contracting parties had fulfilled the requirements of law. Semper praesumitur pro matrimonio. The presumption is always in favor of the validity of the marriage. Every intendment of the law or fact leans toward the validity of the marriage bonds. The Courts look upon this presumption with great favor. It is not to be lightly repelled; on the contrary, the presumption is of great weight. Wherefore, premises considered, the instant Petition is Denied for lack of merit. The decision of the Court of Appeals dated 30 September 2004 affirming the decision of the Regional Trial Court, Branch 143 of Makati City, dated 14 February 2000, are AFFIRMED. Costs against petitioner. SO ORDERED.

574

University of the Cordilleras College of Law First Year C S.Y. 2013 - 2014 Case Digest Alcantara vs Alcantara G.R. No. 167746 August 28, 2007 J. Chico-Nazario Facts: Restituto filed a petition for annulment of marriage against Rosita alleging that on 8 Dec 1982 he and Rosita, without securing the required marriage license, went to the Manila City Hall for the purpose of looking for a fixer who could arrange a marriage for them before a certain Rev. Navarro. They got married on the same day. Restituto and Rosita went through another marriage ceremony in Tondo, Manila, on 26 March 1983. The marriage was again celebrated without the parties securing a marriage license. The alleged marriage license, procured in Carmona, Cavite, appearing on the marriage contract, is a sham, as neither party was a resident of Carmona, and they never went to Carmona to apply for a license with the local civil registrar of the said place. In 1988, they parted ways and lived separate lives. Petitioner prayed that after due hearing, judgment be issued declaring their marriage void and ordering the Civil Registrar to cancel the corresponding marriage contract and its entry on file. Rosita however asserts the validity of their marriage and maintains that there was a marriage license issued as evidenced by a certification from the Office of the Civil Registry of Carmona, Cavite. Restituto has a mistress with whom he has three children. Restituto only filed the annulment of their marriage to evade prosecution for concubinage. Rosita, in fact, has filed a case for concubinage against Restituto. Issue: Whether or not their marriage is valid. Ruling: The requirement and issuance of a marriage license is the States demonstration of its involvement and participation in every marriage, in the maintenance of which the general public is interested. Petitioner cannot insist on the absence of a marriage license to impugn the validity of his marriage. The cases where the court considered the absence of a marriage license as a ground for considering the marriage void are clear-cut. In this case, the marriage contract between the petitioner and respondent reflects a marriage license number. A certification to this effect was also issued by the local civil registrar of Carmona, Cavite. The certification moreover is precise in that it specifically identified the parties to whom the marriage license was issued, namely Restituto Alcantara and Rosita Almario, further validating the fact that a license was in fact issued to the parties herein.Petitioner, in a faint attempt to demolish the probative value of the marriage license, claims that neither he nor respondent is a resident of Carmona, Cavite. Even then, we still hold that there is no sufficient basis to annul petitioner and respondents marriage. Issuance of a marriage license in a city or municipality, not the residence of either of the contracting parties, and issuance of a marriage license despite the absence of publication or prior to the completion of the 10-day period for publication are considered mere irregularities that do not affect the validity of the marriage. An irregularity in any of the formal requisites of marriage does not affect its validity but the party or parties responsible for the irregularity are civilly, criminally and administratively liable. Semper praesumitur pro matrimonio. The presumption is always in favor of the validity of the marriage. Every intendment of the law or fact leans toward the validity of the marriage bonds. The Courts look upon this presumption with great favor. It is not to be lightly repelled; on the contrary, the presumption is of great weight.

575

University of the Cordilleras College of Law First Year C S.Y. 2013 - 2014

VIII. Rights and Obligations between Husband and Wife

576

University of the Cordilleras College of Law First Year C S.Y. 2013 - 2014 Pelayo vs Iauron 12 Phil 453 G.R. No. L-4089 January 12, 1909 Full Case ARTURO PELAYO, plaintiff-appellant, vs. MARCELO LAURON, ET AL., defendants-appellees. TORRES, J.: On the 23rd of November, 1906, Arturo Pelayo, a physician residing in Cebu, filed a complaint against Marcelo Lauron and Juana Abella setting forth that on or about the 13th of October of said year, at night, the plaintiff was called to the house of the defendants, situated in San Nicolas, and that upon arrival he was requested by them to render medical assistance to their daughter-in-law who was about to give birth to a child; that therefore, and after consultation with the attending physician, Dr. Escao, it was found necessary, on account of the difficult birth, to remove the fetus by means of forceps which operation was performed by the plaintiff, who also had to remove the afterbirth, in which services he was occupied until the following morning, and that afterwards, on the same day, he visited the patient several times; that the just and equitable value of the services rendered by him was P500, which the defendants refuse to pay without alleging any good reason therefor; that for said reason he prayed that the judgment be entered in his favor as against the defendants, or any of them, for the sum of P500 and costs, together with any other relief that might be deemed proper. In answer to the complaint counsel for the defendants denied all of the allegation therein contained and alleged as a special defense, that their daughter-in-law had died in consequence of the said childbirth, and that when she was alive she lived with her husband independently and in a separate house without any relation whatever with them, and that, if on the day when she gave birth she was in the house of the defendants, her stay their was accidental and due to fortuitous circumstances; therefore, he prayed that the defendants be absolved of the complaint with costs against the plaintiff. The plaintiff demurred to the above answer, and the court below sustained the demurrer, directing the defendants, on the 23rd of January, 1907, to amend their answer. In compliance with this order the defendants presented, on the same date, their amended answer, denying each and every one of the allegations contained in the complaint, and requesting that the same be dismissed with costs. As a result of the evidence adduced by both parties, judgment was entered by the court below on the 5th of April, 1907, whereby the defendants were absolved from the former complaint, on account of the lack of sufficient evidence to establish a right of action against the defendants, with costs against the plaintiff, who excepted to the said judgment and in addition moved for a new trial on the ground that the judgment was contrary to law; the motion was overruled and the plaintiff excepted and in due course presented the corresponding bill of exceptions. The motion of the defendants requesting that the declaration contained in the judgment that the defendants had demanded therefrom, for the reason that, according to the evidence, no such request had been made, was also denied, and to the decision the defendants excepted. Assuming that it is a real fact of knowledge by the defendants that the plaintiff, by virtue of having been sent for by the former, attended a physician and rendered professional services to a daughter-in-law of the said defendants during a difficult and laborious childbirth, in order to decide the claim of the said physician regarding the recovery of his fees, it becomes necessary to decide who is bound to pay the bill, whether the father and mother-in-law of the patient, or the husband of the latter. According to article 1089 of the Civil Code, obligations are created by law, by contracts, by quasicontracts, and by illicit acts and omissions or by those in which any kind of fault or negligence occurs. Obligations arising from law are not presumed. Those expressly determined in the code or in special laws, etc., are the only demandable ones. Obligations arising from contracts have legal force between the contracting parties and must be fulfilled in accordance with their stipulations. (Arts. 1090 and 1091.) The rendering of medical assistance in case of illness is comprised among the mutual obligations to which the spouses are bound by way of mutual support. (Arts. 142 and 143.) If every obligation consists in giving, doing or not doing something (art. 1088), and spouses are mutually bound to support each other, there can be no question but that, when either of them by reason of illness should be in need of medical assistance, the other is under the unavoidable obligation to furnish the necessary services of a physician in order that health may be restored, and he or she may be freed from the sickness by which life is jeopardized; the party bound to furnish such support is therefore liable for all expenses, including the fees of the medical expert for his professional services. This liability originates from the above-cited mutual obligation which the law has expressly established between the married couple. 577

University of the Cordilleras College of Law First Year C S.Y. 2013 - 2014 In the face of the above legal precepts it is unquestionable that the person bound to pay the fees due to the plaintiff for the professional services that he rendered to the daughter-in-law of the defendants during her childbirth, is the husband of the patient and not her father and mother- in-law, the defendants herein. The fact that it was not the husband who called the plaintiff and requested his assistance for his wife is no bar to the fulfilment of the said obligation, as the defendants, in view of the imminent danger, to which the life of the patient was at that moment exposed, considered that medical assistance was urgently needed, and the obligation of the husband to furnish his wife in the indispensable services of a physician at such critical moments is specially established by the law, as has been seen, and compliance therewith is unavoidable; therefore, the plaintiff, who believes that he is entitled to recover his fees, must direct his action against the husband who is under obligation to furnish medical assistance to his lawful wife in such an emergency. From the foregoing it may readily be understood that it was improper to have brought an action against the defendants simply because they were the parties who called the plaintiff and requested him to assist the patient during her difficult confinement, and also, possibly, because they were her father and motherin-law and the sickness occurred in their house. The defendants were not, nor are they now, under any obligation by virtue of any legal provision, to pay the fees claimed, nor in consequence of any contract entered into between them and the plaintiff from which such obligation might have arisen. In applying the provisions of the Civil Code in an action for support, the supreme court of Spain, while recognizing the validity and efficiency of a contract to furnish support wherein a person bound himself to support another who was not his relative, established the rule that the law does impose the obligation to pay for the support of a stranger, but as the liability arose out of a contract, the stipulations of the agreement must be held. (Decision of May 11, 1897.) Within the meaning of the law, the father and mother-in-law are strangers with respect to the obligation that devolves upon the husband to provide support, among which is the furnishing of medical assistance to his wife at the time of her confinement; and, on the other hand, it does not appear that a contract existed between the defendants and the plaintiff physician, for which reason it is obvious that the former cannot be compelled to pay fees which they are under no liability to pay because it does not appear that they consented to bind themselves. The foregoing suffices to demonstrate that the first and second errors assigned to the judgment below are unfounded, because, if the plaintiff has no right of action against the defendants, it is needless to declare whether or not the use of forceps is a surgical operation. Therefore, in view of the consideration hereinbefore set forth, it is our opinion that the judgment appealed from should be affirmed with the costs against the appellant. So ordered.

578

University of the Cordilleras College of Law First Year C S.Y. 2013 - 2014 Nancy Go and Alex Go vs Court of Appeals 272 SCRA 752 G.R. No. 114791 May 29, 1997 Full Case NANCY GO AND ALEX GO, petitioners, vs. THE HONORABLE COURT OF APPEALS, HERMOGENES ONG and JANE C. ONG, respondents. ROMERO, J.: No less than the Constitution commands us to protect marriage as an inviolable social institution and the foundation of the family. 1 In our society, the importance of a wedding ceremony cannot be underestimated as it is the matrix of the family and, therefore, an occasion worth reliving in the succeeding years. It is in this light that we narrate the following undisputed facts: Private respondents spouses Hermogenes and Jane Ong were married on June 7, 1981, in Dumaguete City. The video coverage of the wedding was provided by petitioners at a contract price of P1,650.00. Three times thereafter, the newlyweds tried to claim the video tape of their wedding, which they planned to show to their relatives in the United States where they were to spend their honeymoon, and thrice they failed because the tape was apparently not yet processed. The parties then agreed that the tape would be ready upon private respondents' return. When private respondents came home from their honeymoon, however, they found out that the tape had been erased by petitioners and therefore, could no longer be delivered. Furious at the loss of the tape which was supposed to be the only record of their wedding, private respondents filed on September 23, 1981 a complaint for specific performance and damages against petitioners before the Regional Trial Court, 7th Judicial District, Branch 33, Dumaguete City. After a protracted trial, the court a quorendered a decision, to wit: WHEREFORE, judgment is hereby granted: 1. Ordering the rescission of the agreement entered into between plaintiff Hermogenes Ong and defendant Nancy Go; 2. Declaring defendants Alex Go and Nancy Go jointly and severally liable to plaintiffs Hermogenes Ong and Jane C. Ong for the following sums: a) P450.00 , the down payment made at contract time; b) P75,000.00, as moral damages; c) P20,000.00, as exemplary damages; d) P5,000.00, as attorney's fees; and e) P2,000.00, as litigation expenses; Defendants are also ordered to pay the costs. SO ORDERED. Dissatisfied with the decision, petitioners elevated the case to the Court of Appeals which, on September 14, 1993, dismissed the appeal and affirmed the trial court's decision. Hence, this petition. Petitioners contend that the Court of Appeals erred in not appreciating the evidence they presented to prove that they acted only as agents of a certain Pablo Lim and, as such, should not have been held liable. In addition, they aver that there is no evidence to show that the erasure of the tape was done in bad faith so as to justify the award of damages. 2 The petition is not meritorious. Petitioners claim that for the video coverage, the cameraman was employed by Pablo Lim who also owned the video equipment used. They further assert that they merely get a commission for all customers solicited for their principal. 3 This contention is primarily premised on Article 1883 of the Civil Code which states thus: Art. 1883. If an agent acts in his own name, the principal has no right of action against the persons with whom the agent has contracted; neither have such persons against the principal.

579

University of the Cordilleras College of Law First Year C S.Y. 2013 - 2014 In such case the agent is the one directly bound in favor of the person with whom he has contracted, as if the transaction were his own, except when the contract involves things belonging to the principal. xxx xxx xxx Petitioners' argument that since the video equipment used belonged to Lim and thus the contract was actually entered into between private respondents and Lim is not deserving of any serious consideration. In the instant case, the contract entered into is one of service, that is, for the video coverage of the wedding. Consequently, it can hardly be said that the object of the contract was the video equipment used. The use by petitioners of the video equipment of another person is of no consequence. It must also be noted that in the course of the protracted trial below, petitioners did not even present Lim to corroborate their contention that they were mere agents of the latter. It would not be unwarranted to assume that their failure to present such a vital witness would have had an adverse result on the case. 4 As regards the award of damages, petitioners would impress upon this Court their lack of malice or fraudulent intent in the erasure of the tape. They insist that since private respondents did not claim the tape after the lapse of thirty days, as agreed upon in their contract, the erasure was done in consonance with consistent business practice to minimize losses. 5 We are not persuaded. As correctly observed by the Court of Appeals, it is contrary to human nature for any newlywed couple to neglect to claim the video coverage of their wedding; the fact that private respondents filed a case against petitioners belies such assertion. Clearly, petitioners are guilty of actionable delay for having failed to process the video tape. Considering that private respondents were about to leave for the United States, they took care to inform petitioners that they would just claim the tape upon their return two months later. Thus, the erasure of the tape after the lapse of thirty days was unjustified. In this regard, Article 1170 of the Civil Code provides that "those who in the performance of their obligations are guilty of fraud, negligence or delay, and those who is any manner contravene the tenor thereof, are liable for damages." In the instant case, petitioners and private respondents entered into a contract whereby, for a fee, the former undertook to cover the latter's wedding and deliver to them a video copy of said event. For whatever reason, petitioners failed to provide private respondents with their tape. Clearly, petitioners are guilty of contravening their obligation to said private respondents and are thus liable for damages. The grant of actual or compensatory damages in the amount of P450.00 is justified, as reimbursement of the downpayment paid by private respondents to petitioners. 6 Generally, moral damages cannot be recovered in an action for breach of contract because this case is not among those enumerated in Article 2219 of the Civil Code. However, it is also accepted in this jurisdiction that liability for a quasi-delict may still exist despite the presence of contractual relations, that is, the act which violates the contract may also constitute a quasi-delict. 7 Consequently, moral damages are recoverable for the breach of contract which was palpably wanton, reckless, malicious or in bad faith, oppressive or abusive. 8 Petitioners' act or omission in recklessly erasing the video coverage of private respondents' wedding was precisely the cause of the suffering private respondents had to undergo. As the appellate court aptly observed: Considering the sentimental value of the tapes and the fact that the event therein recorded a wedding which in our culture is a significant milestone to be cherished and remembered could no longer be reenacted and was lost forever, the trial court was correct in awarding the appellees moral damages albeit in the amount of P75,000.00, which was a great reduction from plaintiffs' demand in the complaint in compensation for the mental anguish, tortured feelings, sleepless nights and humiliation that the appellees suffered and which under the circumstances could be awarded as allowed under Articles 2217 and 2218 of the Civil Code. 9 Considering the attendant wanton negligence committed by petitioners in the case at bar, the award of exemplary damages by the trial court is justified 10 to serve as a warning to all entities engaged in the same business to observe due diligence in the conduct of their affairs. The award of attorney' s fees and litigation expenses are likewise proper, consistent with Article 2208 11 of the Civil Code.

580

University of the Cordilleras College of Law First Year C S.Y. 2013 - 2014 Finally, petitioner Alex Go questions the finding of the trial and appellate courts holding him jointly and severally liable with his wife Nancy regarding the pecuniary liabilities imposed. He argues that when his wife entered into the contract with private respondent, she was acting alone for her sole interest. 12 We find merit in this contention. Under Article 117 of the Civil Code (now Article 73 of the Family Code), the wife may exercise any profession, occupation or engage in business without the consent of the husband. In the instant case, we are convinced that it was only petitioner Nancy Go who entered into the contract with private respondent. Consequently, we rule that she is solely liable to private respondents for the damages awarded below, pursuant to the principle that contracts produce effect only as between the parties who execute them. 13 WHEREFORE, the assailed decision dated September 14, 1993 is hereby AFFIRMED with the MODIFICATION that petitioner Alex Go is absolved from any liability to private respondents and that petitioner Nancy Go is solely liable to said private respondents for the judgment award. Costs against petitioners. SO ORDERED.

581

University of the Cordilleras College of Law First Year C S.Y. 2013 - 2014 Narag vs Narag 291 SCRA 451 A.C. No. 3405 June 29, 1998 Full Case JULIETA B. NARAG, complainant, vs. ATTY. DOMINADOR M. NARAG, respondent. PER CURIAM: Good moral character is a continuing qualification required of every member of the bar. Thus, when a lawyer fails to meet the exacting standard of moral integrity, the Supreme Court may withdraw his or her privilege to practice law. On November 13, 1989, Mrs. Julieta B. Narag filed an administrative complaint 1 for disbarment against her husband, Atty. Dominador M. Narag, whom she accused of having violated Canons 1 and 6, Rule 1.01 of the Code of Ethics for Lawyers. 2 The complainant narrated: The St. Louis College of Tuguegarao engaged the services of Atty. Dominador M. Narag in the early seventies as a full-time college instructor in the College of Arts and Sciences and as a professor in the Graduate School. In 1984, Ms. Gina Espita, 17 years old and a first year college student, enrolled in subjects handled by Atty. Narag. Exerting his influence as her teacher, and as a prominent member of the legal profession and then member of the Sangguniang Bayan of Tuguegarao, Atty. Narag courted Ms. Espita, gradually lessening her resistance until the student acceded to his wishes. They then maintained an illicit relationship known in various circles in the community, but which they managed to from me. It therefore came as a terrible embar[r]assment to me, with unspeakable grief and pain when my husband abandoned us, his family, to live with Ms. Espita, in utterly scandalous circumstances. It appears that Atty. Narag used his power and influence as a member of the Sangguniang Panlalawigan of Cagayan to cause the employment of Ms. Espita at the Department of Trade and Industry Central Office at Makati, Metro Manila. Out of gratitude perhaps, for this gesture, Ms. Espita agreed to live with Atty. Narag, her sense of right[e]ousness and morals completely corrupted by a member of the Bar. It is now a common knowledge in the community that Atty. Dominador M. Narag has abandoned us, his family, to live with a 22-year-old woman, who was his former student in the tertiary level[.] 3 This Court, in a Resolution dated December 18, 1989, referred the case to the Integrated Bar of the Philippines (IBP) for investigation, report and recommendation. 4 On June 26, 1990, the office of then Chief Justice Marcelo B. Fernan received from complainant another letter seeking the dismissal of the administrative complaint. She alleged therein that (1) she fabricated the allegations in her complaint to humiliate and spite her husband; (2) all the love letters between the respondent and Gina Espita were forgeries; and (3) she was suffering from "emotional confusion arising from extreme jealousy." The truth, she stated, was that her husband had remained a faithful and responsible family man. She further asserted that he had neither entered into an amorous relationship with one Gina Espita nor abandoned his family. 5 Supporting her letter were an Affidavit of Desistance 6 and a Motion to Dismiss, 7 attached as Annexes A and B, which she filed before the IBP commission on bar discipline. 8 In a Decision dared October 8, 1991, the IBP Board of Governors 9dismissed the complaint of Mrs. Narag for failure to prosecute. 10 The case took an unexpected turn when, on November 25, 1991, this Court 11 received another letter 12 from the complainant, with her seven children 13 as co-signatories, again appealing for the disbarment of her husband. She explained that she had earlier dropped the case against him because of his continuous threats against her. 14 In his Comment on the complainant's letter of November 11, 1991, filed in compliance with this Court's Resolution issued on July 6, 1992, 15 respondent prayed that the decision of the Board of Governors be affirmed. Denying that he had threatened, harassed or intimidated his wife, he alleged that she had voluntarily executed her Affidavit of Desistance 16 and Motion to Dismiss, 17 even appearing before the investigating officer, Commissioner Racela, to testify under oath "that she prepared the Motion to Dismiss and Affidavit of Desistance on her own free will and affirmed the contents thereof."

582

University of the Cordilleras College of Law First Year C S.Y. 2013 - 2014 In addition, he professed his love for his wife and his children and denied abandoning his family to live with his paramour. However, he described his wife as a person emotionally disturbed, viz: What is pitiable here is the fact that Complainant is an incurably jealous and possessive woman, and every time the streak of jealousy rears its head, she fires off letters or complaints against her husband in every conceivable forum, all without basis, and purely on impulse, just to satisfy the consuming demands of her "loving" jealousy. Then, as is her nature, a few hours afterwards, when her jealousy cools off, she repents and feels sorry for her acts against the Respondent. Thus, when she wrote the Letter of November 11, 1991, she was then in the grips of one of her bouts of jealousy. 18 On August 24, 1992, this Court issued another Resolution referring the Comment of respondent to the IBP. 19 In the hearing before IBP Commissioner Plaridel C. Jose, respondent alleged the following: 20 2. Your Respondent comes from very poor parents who have left him not even a square meter of land, but gave him the best legacy in life: a purposeful and meaningful education. Complainant comes from what she claims to be very rich parents who value material possession more than education and the higher and nobler aspirations in life. Complainant abhors the poor. 3. Your Respondent has a loving upbringing, nurtured in the gentle ways of love, forgiveness, humility, and concern for the poor. Complainant was reared and raised in an entirely different environment. Her value system is the very opposite. 4. Your Respondent loves his family very dearly, and has done all he could in thirty-eight (38) years of marriage to protect and preserve his family. He gave his family sustenance, a comfortable home, love, education, companionship, and most of all, a good and respected name. He was always gentle and compassionate to his wife and children. Even in the most trying times, he remained calm and never inflicted violence on them. His children are all now full-fledged professionals, mature, and gainfully employed. . . . xxx xxx xxx Your Respondent subscribes to the sanctity of marriage as a social institution. On the other hand, consumed by insane and unbearable jealousy, Complainant has been systematically and unceasingly destroying the very foundations of their marriage and their family. Their marriage has become a torture chamber in which Your Respondent has been incessantly BEATEN, BATTERED, BRUTALIZED, TORTURED, ABUSED, and HUMILIATED, physically, mentally, and emotionally, by the Complainant, in public and at home. Their marriage has become a nightmare. For thirty-eight years, your Respondent suffered in silence and bore the pain of his misfortune with dignity and with almost infinite patience, if only to preserve their family and their marriage. But this is not to be. The Complainant never mellowed and never became gentl[e], loving, and understanding. In fact, she became more fierce and predatory. Hence, at this point in time, the light at the tunnel for Your Respondent does not seem in sight. The darkness continues to shroud the marital and familial landscape. Your Respondent has to undergo a catharsis, a liberation from enslavement. Paraphrasing Dorfman in "Death and the Maiden", can the torturer and the tortured co-exist and live together? Hence, faced with an absolutely uncomprehending and uncompromising mind whose only obsession now is to destroy, destroy, and destroy, Your Respondent, with perpetual regret and with great sorrow, filed a Petition for Annulment of Marriage, Spl. Proc. No. 566, RTC, Branch III, Tuguegarao, Cagayan. . . . 5. Complainant is a violent husband-beater, vitriolic and unbending. But your Respondent never revealed these destructive qualities to other people. He preserved the good name and dignity of his wife. This is in compliance with the marital vow to love, honor or obey your spouse, for better or for worse, in sickness and in health . . . Even in this case, Your Respondent never revealed anything derogatory to his wife. It is only now that he is constrained to reveal all these things to defend himself. On the other hand, for no reason at all, except a jealous rage, Complainant tells everyone, everywhere, that her husband is worthless, good-for-nothing, evil and immoral. She goes to colleges and universities, professional organizations, religious societies, and all other 583

University of the Cordilleras College of Law First Year C S.Y. 2013 - 2014 sectors of the community to tell them how evil, bad and immoral her husband is. She tells them not to hire him as professor, as Counsel, or any other capacity because her husband is evil, bad, and immoral. Is this love? Since when did love become an instrument to destroy a man's dearest possession in life his good name, reputation and dignity? Because of Complainant's virulent disinformation campaign against her husband, employing every unethical and immoral means to attain his ends, Your Respondent has been irreparably and irreversibly disgraced, shamed, and humiliated. Your Respondent is not a scandalous man. It is he who has been mercilessly scandalized and crucified by the Complainant. 21 To prove the alleged propensity of his wife to file false charges, respondent presented as evidence the following list of the complaints she had filed against him and Gina Espita: 3.1 Complaint for Immorality/Neglect of Duty . . . 3.2 Complaint for Immorality/Neglect of Duty, DILG, Adm. Case No. P5-90. . . . 3.3 Complaint for Concubinage. Provincial Prosecutor's Office of Cagayan. I.S No. 89-114. . . . 3.4 Complaint for Anti-Graft and Corrupt Practices and concubinage. OMBUDSMAN Case No. 1-92-0083. . . . 3.5 Complaint for Civil Support. RTC, Tuguegarao, Civil Case No. 4061. DISMISSED. 3.6 Complaint for Concubinage. Provincial Prosecutor's Office of Cagayan. I.S. No. 92-109. DISMISSED. (. . .). Complainant filed Motion for Reconsideration. DENIED. (. . .). 3.7 Complaint for Disbarment (. . .) with S[upreme] C[ourt]. Withdrawn (. . .). DISMISSED by IBP Board of Governors (. . .). Re-instituted (. . .). 3.8 Complaint for Disbarment, again (. . .). Adm. Case No. 3405. Pending. 3.9 Complaint for Concubinage, again (. . .). Third MCTC, Tumauini, Isabela. Pending. . . . 22 In his desperate effort to exculpate himself, he averred: I. That all the alleged love letters and envelopes (. . .), picture (. . .) are inadmissible in evidence as enunciated by the Supreme Court in "Cecilia Zulueta vs. Court of Appeals, et.al.", G.R. No. 107383, February 20, 1996. (. . .). xxx xxx xxx II. That respondent is totally innocent of the charges: He never courted Gina Espita in the Saint Louis College of Tuguegarao. He never caused the employment of said woman in the DTI. He never had or is having any illicit relationship with her anywhere, at any time. He never lived with her as husband and wife anywhere at any time, be it in Centro Tumauini or any of its barangays, or in any other place. He never begot a child or children with her. Finally, respondent submits that all the other allegations of Mrs. Narag are false and fabricated, . . . xxx xxx xxx III. Respondent never abandoned his family[.] Mrs. Narag and her two sons forcibly drove respondent Narag out of the conjugal home. After that, Atty. Narag tried to return to the conjugal home many times with the help of mutual friends to save the marriage and the family from collapse. He tried several times to reconcile with Mrs. Narag. In fact, in one of the hearings of the disbarment case, he offered to return home and to reconcile with Mrs. Narag. But Mrs. Narag refused all these efforts of respondent Narag. . . . IV. Complainant Julieta B. Narag is an unbearably jealous, violent, vindictive, scandalous, virulent and merciless wife since the beginning of the marriage, who incessantly beat, battered, brutalized, tortured, abuse[d], scandalized, and humiliated respondent Atty. Narag, physically, mentally, emotionally, and psychologically, . . . V. Complainant Julieta Narag's claim in her counter-manifestation dated March 28, 1996, to the effect that the affidavit of Dominador B. Narag, Jr., dated February 27, 1996 was 584

University of the Cordilleras College of Law First Year C S.Y. 2013 - 2014 obtained through force and intimidation, is not true. Dominador, Jr., executed his affidavit freely, voluntarily, and absolutely without force or intimidation, as shown by the transcript of stenographic notes of the testimonies of Respondent Atty. Narag and Tuguegarao MTC Judge Dominador Garcia during the trial of Criminal Case No. 12439, People vs. Dominador M. Narag, et. al., before the Tuguegarao MTC on May 3, 1996. . . . xxx xxx xxx VI. Respondent Atty. Narag is now an old man a senior citizen of 63 years sickly, abandoned, disgraced, weakened and debilitated by progressively degenerative gout and arthritis, and hardly able to earn his own keep. His very physical, medical, psychological, and economic conditions render him unfit and unable to do the things attributed to him by the complainant. Please see the attached medical certificates, . . ., among many other similar certificates touching on the same ailments. Respondent is also suffering from hypertension.23 On July 18, 1997, the investigating officer submitted his report, 24 recommending the indefinite suspension of Atty. Narag from the practice of law. The material portions of said report read as follows: Culled from the voluminous documentary and testimonial evidence submitted by the contending parties, two (2) issues are relevant for the disposition of the case, namely: a) Whether there was indeed a commission of alleged abandonment of respondent's own family and [whether he was] living with his paramour, Gina Espita; b) Whether the denial under oath that his illegitimate children with Gina Espita (Aurelle Dominic and Kyle Dominador) as appearing on paragraph 1(g) of respondent's Comment vis-a-vis his handwritten love letters, the due execution and contents of which, although he objected to their admissibility for being allegedly forgeries, were never denied by him on the witness stand much less presented and offered proof to support otherwise. Except for the testimonies of respondent's witnesses whose testimonies tend to depict the complaining wife, Mrs. Narag, as an incurably jealous wife and possessive woman suffering everytime with streaks of jealousy, respondent did not present himself on the witness stand to testify and be cross-examined on his sworn comment; much less did he present his alleged paramour, Gina Espita, to disprove the adulterous relationship between him and their having begotten their illegitimate children, namely: Aurelle Dominic N. Espita and Kyle Dominador N. Espita. Worse, respondent's denial that he is the father of the two is a ground for disciplinary sanction (Morcayda v. Naz, 125 SCRA 467). Viewed from all the evidence presented, we find the respondent subject to disciplinary action as a member of the legal profession. 25 In its Resolution 26 issued on August 23, 1997, the IBP adopted and approved the investigating commissioner's recommendation for the indefinite suspension of the respondent. 27 Subsequently the complaint sought the disbarment of her husband in a Manifestation/Comment she filed on October 20, 1997. The IBP granted this stiffer penalty and, in its Resolution dated November 30, 1997, denied respondent's Motion for Reconsideration. After a careful scrutiny of the records of the proceedings and the evidence presented by the parties, we find that the conduct of respondent warrants the imposition of the penalty of disbarment. The Code of Professional Responsibility provides: Rule 1.01 A lawyer shall not engage in unlawful, dishonest, immoral or deceitful conduct. CANON 7 A lawyer shall at all times uphold the integrity and dignity of the legal profession, and support the activities of the Integrated Bar. Rule 7.03 A lawyer shall not engage in conduct that adversely reflects on his fitness to practice law, nor should he, whether in public or private life, behave in a scandalous manner to the discredit of the legal profession. Thus, good moral character is not only a condition precedent 28 to the practice of law, but a continuingqualification for all members of the bar. Hence, when a lawyer is found guilty of gross immoral conduct, he may be suspended or disbarred. 29 Immoral conduct has been defined as that conduct which is so willful, flagrant, or shameless as to show indifference to the opinion of good and respectable members of the community. 30 Furthermore, such 585

University of the Cordilleras College of Law First Year C S.Y. 2013 - 2014 conduct must not only be immoral, but grossly immoral. That is, it must be so corrupt as to constitute a criminal act or so unprincipled as to be reprehensible to a high degree 31 or committed under such scandalous or revolting circumstances as to shock the common sense of decency. 32 We explained in Barrientos vs. Daarol 33 that, "as officers of the court, lawyers must not only in fact be of good moral character but must also be seen to be of good moral character and leading lives in accordance with the highest moral standards of the community. More specifically, a member of the Bar and officer of the court is not only required to refrain from adulterous relationships or the keeping of mistresses but must also so behave himself as to avoid scandalizing the public by creating the belief that he is flouting those moral standards." Respondent Narag is accused of gross immorality for abandoning his family in order to live with Gina Espita. The burden of proof rests upon the complainant, and the Court will exercise its disciplinary power only if she establishes her case by clear, convincing and satisfactory evidence. 34 Presented by complainant as witnesses, aside from herself. 35 were: Charlie Espita, 36 Magdalena Bautista, 37Bienvenido Eugenio, 38 Alice Carag, 39 Dr. Jervis B. Narag, 40 Dominador Narag, Jr., 41 and Nieves F. Reyes. 42 Charlie Espita, brother of the alleged paramour Gina Espita, corroborated complainant's charge against respondent in these categorical statements he gave to the investigating officer: Q Mr. Witness, do you know Atty. Narag? A Yes, Your Honor, he is the live-in partner of my sister, Gina Espita. Q If Atty. Narag is here, can you point [to] him? A Yes, sir. (Witness pointed to the respondent, Atty. Dominador Narag) Q Why do you know Atty. Narag? ATTY. NARAG: Already answered. He said I am the live-in partner. CONTINUATION OF THE DIRECT A Because he is the live-in partner of my sister and that they are now living together as husband and wife and that they already have two children, Aurelle Dominic and Kyle Dominador. xxx xxx xxx During cross-examination conducted by the respondent himself, Charlie Espita repeated his account that his sister Gina was living with the respondent, with whom she had two children: Q Mr. Espita, you claim that Atty. Narag is now living with your sister as husband and wife. You claim that? A Yes, sir. Q Why do you say that? A Because at present you are living together as husband and wife and you have already two children and I know that is really an immoral act which you cannot just allow me to follow since my moral values don't allow me that my sister is living with a married man like you. Q How do you know that Atty. Narag is living with your sister? Did you see them in the house? A Yes, si[r]. xxx xxx xxx Q You said also that Atty. Narag and your sister have two children, Aurelle Dominic and Kyle Dominador, is it not? A Yes, sir. Q How do you know that they are the children of Atty. Narag? A Because you are staying together in that house and you have left your family. 44 In addition, Charlie Espita admitted (1) that it was he who handed to Mrs. Narag the love letters respondent had sent to his sister, and (2) that Atty. Narag tried to dissuade him from appearing at the disbarment proceedings. 45 Witness Bienvenido Eugenio strengthened the testimony of Charlie Espita in this wise: 586

University of the Cordilleras College of Law First Year C S.Y. 2013 - 2014 Q Mr. Witness, do you know the respondent in this case? A I know him very well, sir. Q Could you please tell us why do you know him? A Because he was always going to the house of my son-in-law by the name of Charlie Espita. xxx xxx xxx Q Mr. Eugenio, do you know the residence of Atty. Dominador M. Narag? A At that time, he [was] residing in the house of Reynaldo Angubong, sir. Q And this is located where? A Centro Tamauini, Isabela, sir. Q And you specifically, categorically state under oath that this is the residence of Atty. Narag? A Yes, sir. xxx xxx xxx Q And under oath this is where Atty. Narag and Gina Espita are allegedly living as husband and wife, is it not? A Yes, sir. 46 Witness Nieves Reyes, a neighbor and friend of the estranged couple, testified that she learned from the Narag children Randy, Bong and Rowena that their father left his family, that she and her husband prodded the complainant to accept the respondent back, that the Narag couple again separated when the respondent "went back to his woman," and that Atty. Narag had maltreated his wife. 47 On the strength of the testimony of her witnesses, the complainant was able to establish that respondent abandoned his family and lived with another woman. Absent any evidence showing that these witnesses had an ill motive to testify falsely against the respondent, their testimonies are deemed worthy of belief. Further, the complainant presented as evidence the love letters that respondent had sent to Gina. In these letters, respondent clearly manifested his love for Gina and her two children, whom he acknowledged as his own. In addition, complainant, also submitted as evidence the cards that she herself had received from him. Guided by the rule that handwriting may be proved through a comparison of one set of writings with those admitted or treated by the respondent as genuine, we affirm that the two sets of evidence were written by one and the same person. 48Besides, respondent did not present any evidence to prove that the love letters were not really written by him; he merely denied that he wrote them. While the burden of proof is upon the complainant, respondent has the duty not only to himself but also to the court to show that he is morally fit to remain a member of the bar. Mere denial does not suffice. Thus, when his moral character is assailed, such that his right to continue practicing his cherished profession is imperiled, he must meet the charges squarely and present evidence, to the satisfaction of the investigating body and this Court, that he is morally fit to have his name in the Roll of Attorneys. 49 This he failed to do. Respondent adamantly denies abandoning his family to live with Gina Espita. At the same time, he depicts his wife as a "violent husband-beater, vitriolic and unbending," and as an "insanely and pathologically jealous woman," whose only obsession was to "destroy, destroy and destroy" him as shown by her filing of a series of allegedly unfounded charges against him (and Gina Espita). To prove his allegation, he presented ninety-eight (98) pieces of documentary evidence 50 and ten (10) witnesses. 51 We note, however, that the testimonies of the witnesses of respondent did not establish the fact that he maintained that moral integrity required by the profession that would render him fit to continue practicing law. Neither did their testimonies destroy the fact, as proven by the complainant, that he had abandoned his family and lived with Gina Espita, with whom he had two children. Some of them testified on matters which they had no actual knowledge of, but merely relied on information from either respondent himself or other people, while others were presented to impeach the good character of his wife. Respondent may have provided well for his family they enjoyed a comfortable life and his children finished their education. He may have also established himself as a successful lawyer and a seasoned politician. But these accomplishments are not sufficient to show his moral fitness to continue being a member of the noble profession of law. We remind respondent that parents have not only rights but also duties e.g., to support, educate and instruct their children according to right precepts and good example; and to give them love, companionship and understanding, as well as moral and spiritual guidance. 52 As a husband, he is also 587

University of the Cordilleras College of Law First Year C S.Y. 2013 - 2014 obliged to live with his wife; to observe mutual love, respect and fidelity; and to render help and support. 53 Respondent himself admitted that his work required him to be often away from home. But the evidence shows that he was away not only because of his work; instead, he abandoned his family to live with her paramour, who bore him two children. It would appear, then, that he was hardly in a position to be a good husband or a good father. His children, who grew up mostly under the care of their mother, must have scarcely felt the warmth of their father's love. Respondent's son, Jervis B. Narag, showed his resentment towards his father's moral frailties in his testimony: Q My question is this, is there any sin so grievous that it cannot be forgiven, is there a fault that is so serious that it is incapable of forgiveness? A That depends upon the sin or fault, sir, but if the sin or fault is with the emotional part of myself, I suppose I cannot forgive a person although am a God-fearing person, but I h[av]e to give the person a lesson in order for him or her to at least realize his mistakes, sir. xxx xxx xxx COMR. JOSE: I think it sounds like this. Assuming for the sake of argument that your father is the worst, hardened criminal on earth, would you send him to jail and have him disbarred? That is the question. CONTINUATION. A With the reputation that he had removed from us, I suppose he has to be given a lesson. At this point in time, I might just forgive him if he will have to experience all the pains that we have also suffered for quite sometime. Q Dr. Narag, your father gave you life, his blood runs in your veins, his flesh is your flesh, his bones are your bones and you now disown him because he is the worst man on earth, is that what you are saying. A Sort of, sir. Q You are now telling that as far [as] you are concerned because your father has sinned, you have no more father, am I correct? A Long before, sir, I did not feel much from my father even when I was still a kid because my father is not always staying with us at home. So, how can you say that? Yes, he gave me life, why not? But for sure, sir, you did not give me love. 54 Another son, Dominador Narag, Jr., narrated before the investigating officer the trauma he went through: Q In connection with that affidavit, Mr. Witness, which contains the fact that your father is maintaining a paramour, could you please tell this Honorable Commission the effect on you? A This has a very strong effect on me and this includes my brothers and sisters, especially my married life, sir. And it also affected my children so much, that I and my wife ha[ve] parted ways. It hurts to say that I and my wife parted ways. This is one reason that affected us. Q Will you please tell us specifically why you and your wife parted ways? A Because my wife wa[s] ashamed of what happened to my family and that she could not face the people, our community, especially because my wife belongs to a well-known family in our community. Q How about the effect on your brothers and sisters? Please tell us what are those. A Well, sir, this has also affected the health of my elder sister because she knows so well that my mother suffered so much and she kept on thinking about my mother. xxx xxx xxx Q Why did your wife leave you? A The truth is because of the things that had happened in our family, Your Honor. Q In your wife's family? A In our family, sir. Q And what do you mean by that? 588

University of the Cordilleras College of Law First Year C S.Y. 2013 - 2014 A What meant by that is my father had an illicit relationship and that my father went to the extent of scolding my wife and calling my wife a "puta" in provincial government, which my mother-inlaw hated him so much for this, which really affected us. And then my wife knew for a fact that my father has an illicit relationship with Gina Espita, whom he bore two children by the name of Aurelle Dominic and Kyle Dominador, which I could prove and I stand firm to this, Your Honor. 55 Although respondent piously claims adherence to the sanctity of marriage, his acts prove otherwise. A husband is not merely a man who has contracted marriage. Rather, he is a partner who has solemnly sworn to love and respect his wife and remain faithful to her until death. We reiterate our ruling in Cordova vs. Cordova 56: "The moral delinquency that affects the fitness of a member of the bar to continue as such includes conduct that outrages the generally accepted moral standards of the community, conduct for instance, which makes a mockery of the inviolable social institution of marriage." In Toledo vs. Toledo, 57 the respondent was disbarred from the practice of law, when he abandoned his lawful wife and cohabited with another woman who had borne him a child. Likewise, in Obusan vs. Obusan, 58 the respondent was disbarred after the complainant proved that he had abandoned her and maintained an adulterous relationship with a married woman. This Court declared that respondent failed to maintain the highest degree of morality expected and required of a member of the bar. In the present case, the complainant was able to establish, by clear and convincing evidence, that respondent had breached the high and exacting moral standards set for members of the law profession. As held in Maligsa vs. Cabanting, 59 "a lawyer may be disbarred for any misconduct, whether in his professional or private capacity, which shows him to be wanting in moral character, in honesty, probity and good demeanor or unworthy to continue as an officer of the court." WHEREFORE, Dominador M. Narag is hereby DISBARRED and his name is ORDERED STRICKEN from the Roll of Attorneys. Let copies of this Decision be in the personal record of Respondent Narag; and furnished to all courts of the land, the Integrated Bar of the Philippines, and the Office of the Bar Confidant. SO ORDERED.

589

University of the Cordilleras College of Law First Year C S.Y. 2013 - 2014 Mangonon vs Court of Appeals 494 SCRA 1 G.R. No. 125041 June 30, 2006 Full Case MA. BELEN B. MANGONON, for and in behalf of her minor children REBECCA ANGELA DELGADO and REGINA ISABEL DELGADO. Petitioner, vs. HON. COURT OF APPEALS, HON. JUDGE JOSEFINA GUEVARA-SALONGA, Presiding Judge, RTC-Makati, Branch 149, FEDERICO C. DELGADO and FRANCISCO C. DELGADO, Respondents. CHICO-NAZARIO, J.: Before Us is a Petition for Review on Certiorari assailing the Decision_1 of the Court of Appeals dated 20 March 1996, affirming the Order, dated 12 September 1995_2 of the Regional Trial Court (RTC), Branch 149, Makati, granting support pendente lite to Rebecca Angela (Rica) and Regina Isabel (Rina), both surnamed Delgado. The generative facts leading to the filing of the present petition are as follows: On 17 March 1994, petitioner Ma. Belen B. Mangonon filed, in behalf of her then minor children Rica and Rina, a Petition for Declaration of Legitimacy and Support, with application for support pendente lite with the RTC Makati._3In said petition, it was alleged that on 16 February 1975, petitioner and respondent Federico Delgado were civilly married by then City Court Judge Eleuterio Agudo in Legaspi City, Albay. At that time, petitioner was only 21 years old while respondent Federico was only 19 years old. As the marriage was solemnized without the required consent per Article 85 of the New Civil Code,_4 it was annulled on 11 August 1975 by the Quezon City Juvenile and Domestic Relations Court._5 On 25 March 1976, or within seven months after the annulment of their marriage, petitioner gave birth to twins Rica and Rina. According to petitioner, she, with the assistance of her second husband Danny Mangonon, raised her twin daughters as private respondents had totally abandoned them. At the time of the institution of the petition, Rica and Rina were about to enter college in the United States of America (USA) where petitioner, together with her daughters and second husband, had moved to and finally settled in. Rica was admitted to the University of Massachusetts (Amherst) while Rina was accepted by the Long Island University and Western New England College. Despite their admissions to said universities, Rica and Rina were, however, financially incapable of pursuing collegiate education because of the following: i) The average annual cost for college education in the US is about US$22,000/year, broken down as follows: Tuition Fees US$13,000.00 Room & Board 5,000.00 Books 1,000.00 Yearly Transportation & Meal Allowance 3,000.00 Total US$ 22,000.00 or a total of US$44,000.00, more or less, for both Rica and Rina ii) Additionally, Rica and Rina need general maintenance support each in the amount of US$3,000.00 per year or a total of US$6,000 per year. iii) Unfortunately, petitioners monthly income from her 2 jobs is merely US$1,200 after taxes which she can hardly give general support to Rica and Rina, much less their required college educational support. iv) Neither can petitioners present husband be compelled to share in the general support and college education of Rica and Rina since he has his own son with petitioner and own daughter (also in college) to attend to. v) Worse, Rica and Rinas petitions for Federal Student Aid have been rejected by the U.S. Department of Education._6 Petitioner likewise averred that demands_7 were made upon Federico and the latters father, Francisco,8 for general support and for the payment of the required college education of Rica and Rina. The twin sisters even exerted efforts to work out a settlement concerning these matters with respondent Federico and respondent Francisco, the latter being generally known to be financially well-off._9 These demands, however, remained unheeded. Considering the impending deadline for admission to college and 590

University of the Cordilleras College of Law First Year C S.Y. 2013 - 2014 the opening of classes, petitioner and her then minor children had no choice but to file the petition before the trial court. Petitioner also alleged that Rica and Rina are her legitimate daughters by respondent Federico since the twin sisters were born within seven months from the date of the annulment of her marriage to respondent Federico. However, as respondent Federico failed to sign the birth certificates of Rica and Rina, it was imperative that their status as legitimate children of respondent Federico, and as granddaughters of respondent Francisco, be judicially declared pursuant to Article 173 of the Family Code._10 As legitimate children and grandchildren, Rica and Rina are entitled to general and educational support under Articles 174_11 and 195(b)_12 in relation to Articles 194(1 and 2)_13 and 199(c)_14 of the Family Code. Petitioner alleged that under these provisions, in case of default on the part of the parents, the obligation to provide support falls upon the grandparents of the children; thus, respondent Federico, or in his default, respondent Francisco should be ordered to provide general and educational support for Rica and Rina in the amount of US$50,000.00, more or less, per year. Petitioner also claimed that she was constrained to seek support pendente lite from private respondents who are millionaires with extensive assets both here and abroad - in view of the imminent opening of classes, the possibility of a protracted litigation, and Rica and Rinas lack of financial means to pursue their college education in the USA. In his Answer,_15 respondent Francisco stated that as the birth certificates of Rica and Rina do not bear the signature of respondent Federico, it is essential that their legitimacy be first established as "there is no basis to claim support until a final and executory judicial declaration has been made as to the civil status of the children."_16Whatever good deeds he may have done to Rica and Rina, according to respondent Francisco, was founded on pure acts of Christian charity. He, likewise, averred that the order of liability for support under Article 199 of the Family Code is not concurrent such that the obligation must be borne by those more closely related to the recipient. In this case, he maintained that responsibility should rest on the shoulders of petitioner and her second husband, the latter having voluntarily assumed the duties and responsibilities of a natural father. Even assuming that he is responsible for support, respondent Francisco contends that he could not be made to answer beyond what petitioner and the father could afford. On 24 May 1994, petitioner filed a Motion to Declare Defendant (respondent herein) Federico in Default._17 This was favorably acted upon by the trial court in the Order dated 16 June 1994._18 On 5 August 1994, respondent Federico filed a Motion to Lift Order of Default alleging that the summons and a copy of the petition were not served in his correct address._19 Attached thereto was his Answer_20 where he claimed that petitioner had no cause of action against him. According to him, he left for abroad and stayed there for a long time "[w]ithin the first one hundred twenty (120) days of the three hundred days immediately preceding March 25, 1976" and that he only came to know about the birth of Rica and Rina when the twins introduced themselves to him seventeen years later. In order not to antagonize the two, respondent Federico claimed he did not tell them that he could not be their father. Even assuming that Rica and Rina are, indeed, his daughters, he alleged that he could not give them the support they were demanding as he was only making P40,000.00 a month. Finding sufficient ground in the motion filed by respondent Federico, the trial court lifted its Order dated 16 June 1994 and admitted his Answer._21 In the meantime, on 25 April 1994, petitioner filed an Urgent Motion to Set Application for Support Pendente Lite for Hearing because Rica and Rina both badly needed immediate financial resources for their education._22 This Motion was opposed by respondent Francisco._23 After both parties submitted supplemental pleadings to bolster their respective positions, the trial court resolved the motion in an Order dated 12 September 1995 in this wise: WHEREFORE, in the light of the foregoing considerations, respondents are hereby directed to provide a monthly support (pendente lite) of P5,000.00 each or a total of P10,000.00 for the education of Rebecca Angela and Regina Isabel Delgado to be delivered within the first five days of each month without need of demand._24 Unsatisfied with the Order of the trial court, petitioner brought the case to the Court of Appeals via Petition for Certiorari. The Court of Appeals affirmed the holding of the trial court and disposed the petition in the following manner: WHEREFORE, the petition for certiorari is hereby DISMISSED and the Order of the lower court dated September 12, 1995 is hereby AFFIRMED._25 Petitioners Motion for Reconsideration was denied through the Resolution of the Court of Appeals dated 16 May 1996._26

591

University of the Cordilleras College of Law First Year C S.Y. 2013 - 2014 Petitioner is now before this Court claiming that the Decision of the Court of Appeals was tainted with the following errors: RESPONDENT COURT OF APPEALS ERRED IN CONCLUDING THAT RESPONDENT JUDGE DID NOT COMMIT GRAVE ABUSE OF DISCRETION IN FIXING THE AMOUNT OF MONTHLY SUPPORT PENDENTE LITE GRANTED TO PETITIONERS CHILDREN AT A MEASLEY P5,000.00 PER CHILD. I. RESPONDENT COURT IGNORED EVIDENCE ON RECORD OF THE FINANCIAL INCAPACITY OF RICA AND RINAS PARENTS IN DEFAULT OF WHOM THE OBLIGATION TO GIVE SUPPORT DEVOLVES ON THE GRANDFATHER. II. IT BEING ESTABLISHED THAT THE PERSON OBLIGED TO GIVE SUPPORT GRANDFATHER DON PACO IS UNDOUBTEDLY CAPABLE OF GIVING THE AMOUNT DEMANDED, RESPONDENT COURT ERRED IN NOT HOLDING THAT RESPONDENT JUDGE ACTED WITH GRAVE ABUSE OF DISCRETION IN FIXING AN AMOUNT OF SUPPORT PENDENTE LITE THAT IS OBVIOUSLY INADEQUATE TO SUPPORT THE EDUCATIONAL REQUIREMENTS OF THE RECIPIENTS._27 At the time of the filing of the present Petition, it is alleged that Rica had already entered Rutgers University in New Jersey with a budget of US$12,500.00 for academic year 1994-1995. She was able to obtain a tuition fee grant of US$1,190.00 and a Federal Stafford loan from the US government in the amount of US$2,615.00._28 In order to defray the remaining balance of Ricas education for said school year, petitioner claims that she had to secure a loan under the Federal Direct Student Loan Program. Meanwhile, Rina entered CW Post, Long Island University, where she was expected to spend US$20,000.00 for the school year 1994-1995. She was given a financial grant of US$6,000.00, federal work study assistance of US$2,000.00, and a Federal Stafford loan of US$2,625.00._29 Again, petitioner obtained a loan to cover the remainder of Rinas school budget for the year. Petitioner concedes that under the law, the obligation to furnish support to Rica and Rina should be first imposed upon their parents. She contends, however, that the records of this case demonstrate her as well as respondent Federicos inability to give the support needed for Rica and Rinas college education. Consequently, the obligation to provide support devolves upon respondent Francisco being the grandfather of Rica and Rina. Petitioner also maintains that as respondent Francisco has the financial resources to help defray the cost of Rica and Rinas schooling, the Court of Appeals then erred in sustaining the trial courts Order directing respondent Federico to pay Rica and Rina the amount of award P5,000.00 each as monthly support pendente lite. On the other hand, respondent Francisco argues that the trial court correctly declared that petitioner and respondent Federico should be the ones to provide the support needed by their twin daughters pursuant to Article 199 of the Family Code. He also maintains that aside from the financial package availed of by Rica and Rina in the form of state tuition aid grant, work study program and federal student loan program, petitioner herself was eligible for, and had availed herself of, the federal parent loan program based on her income and properties in the USA. He, likewise, insists that assuming he could be held liable for support, he has the option to fulfill the obligation either by paying the support or receiving and maintaining in the dwelling here in the Philippines the person claiming support._30 As an additional point to be considered by this Court, he posits the argument that because petitioner and her twin daughters are now US citizens, they cannot invoke the Family Code provisions on support as "[l]aws relating to family rights and duties, or to the status, condition and legal capacity of persons are binding upon citizens of the Philippines, even though living abroad."_31 Respondent Federico, for his part, continues to deny having sired Rica and Rina by reiterating the grounds he had previously raised before the trial court. Like his father, respondent Federico argues that assuming he is indeed the father of the twin sisters, he has the option under the law as to how he would provide support. Lastly, he assents with the declaration of the trial court and the Court of Appeals that the parents of a child should primarily bear the burden of providing support to their offspring. The petition is meritorious. As a preliminary matter, we deem it necessary to briefly discuss the essence of support pendente lite. The pertinent portion of the Rules of Court on the matter provides:

592

University of the Cordilleras College of Law First Year C S.Y. 2013 - 2014 Rule 61 SUPPORT PENDENTE LITE SECTION 1. Application.- At the commencement of the proper action or proceeding, or at any time prior to the judgment or final order, a verified application for support pendente lite may be filed by any party stating the grounds for the claim and the financial conditions of both parties, and accompanied by affidavits, depositions or other authentic documents in support thereof. xxxx SEC. 4. Order.- The court shall determine provisionally the pertinent facts, and shall render such orders as justice and equity may require, having due regard to the probable outcome of the case and such other circumstances as may aid in the proper resolution of the question involved. If the application is granted, the court shall fix the amount of money to be provisionally paid or such other forms of support as should be provided, taking into account the necessities of the applicant and the resources or means of the adverse party, and the terms of payment or mode for providing the support. If the application is denied, the principal case shall be tried and decided as early as possible. Under this provision, a court may temporarily grant support pendente lite prior to the rendition of judgment or final order. Because of its provisional nature, a court does not need to delve fully into the merits of the case before it can settle an application for this relief. All that a court is tasked to do is determine the kind and amount of evidence which may suffice to enable it to justly resolve the application. It is enough that the facts be established by affidavits or other documentary evidence appearing in the record._32lavvphi1.net After the hearings conducted on this matter as well as the evidence presented, we find that petitioner was able to establish, by prima facie proof, the filiation of her twin daughters to private respondents and the twins entitlement to support pendente lite. In the words of the trial court By and large, the status of the twins as children of Federico cannot be denied. They had maintained constant communication with their grandfather Francisco. As a matter of fact, respondent Francisco admitted having wrote several letters to Rica and Rina (Exhs. A, B, C, D, E, F, G, G-1 to G-30). In the said letters, particularly at the bottom thereof, respondent Francisco wrote the names of Rica and Rina Delgado. He therefore was very well aware that they bear the surname Delgado. Likewise, he referred to himself in his letters as either "Lolo Paco" or "Daddy Paco." In his letter of October 13, 1989 (Exh. G-21), he said "as the grandfather, am extending a financial help of US$1,000.00." On top of this, respondent Federico even gave the twins a treat to Hongkong during their visit to the Philippines. Indeed, respondents, by their actuations, have shown beyond doubt that the twins are the children of Federico.33 Having addressed the issue of the propriety of the trial courts grant of support pendente lite in favor of Rica and Rina, the next question is who should be made liable for said award. The pertinent provision of the Family Code on this subject states: ART. 199. Whenever two or more persons are obliged to give support, the liability shall devolve upon the following persons in the order herein provided: (1) The spouse; (2) The descendants in the nearest degree; (3) The ascendants in the nearest degree; and (4) The brothers and sisters. An eminent author on the subject explains that the obligation to give support rests principally on those more closely related to the recipient. However, the more remote relatives may be held to shoulder the responsibility should the claimant prove that those who are called upon to provide support do not have the means to do so.34 In this case, both the trial court and the Court of Appeals held respondent Federico liable to provide monthly support pendente lite in the total amount of P10,000.00 by taking into consideration his supposed income ofP30,000.00 to P40,000.00 per month. We are, however, unconvinced as to the veracity of this ground relied upon by the trial court and the Court of Appeals. It is a basic procedural edict that questions of fact cannot be the proper subject of a petition for review under Rule 45 of the 1997 Rules of Civil Procedure. The rule finds a more stringent application where the Court of Appeals upholds the findings of fact of the trial court; in such a situation, this Court, as the final arbiter, is generally bound to adopt the facts as determined by the appellate and the lower courts. This rule, however, is not ironclad as it admits of the following recognized exceptions: "(1) when the findings are grounded entirely on speculation, surmises or conjectures; (2) when the inference made is manifestly mistaken, absurd or impossible; (3) when there is grave abuse of discretion; (4) when the judgment is 593

University of the Cordilleras College of Law First Year C S.Y. 2013 - 2014 based on a misapprehension of facts; (5) when the findings of facts are conflicting; (6) when in making its findings the Court of Appeals went beyond the issues of the case, or its findings are contrary to the admissions of both the appellant and the appellee; (7) when the findings are contrary to that of the trial court; (8) when the findings are conclusions without citation of specific evidence on which they are based; (9) when the facts set forth in the petition as well as in the petitioners main and reply briefs are not disputed by the respondent; (10) when the findings of fact are premised on the supposed absence of evidence and contradicted by the evidence on record; and (11) when the Court of Appeals manifestly overlooked certain relevant facts not disputed by the parties, which, if properly considered, would justify a different conclusion."35 The case at bar falls within the seventh and eleventh exceptions. The trial court gave full credence to respondent Federicos allegation in his Answer36 and his testimony37 as to the amount of his income. We have, however, reviewed the records of this case and found them bereft of evidence to support his assertions regarding his employment and his earning. Notably, he was even required by petitioners counsel to present to the court his income tax return and yet the records of this case do not bear a copy of said document.38 This, to our mind, severely undermines the truthfulness of respondent Federicos assertion with respect to his financial status and capacity to provide support to Rica and Rina. In addition, respondent Francisco himself stated in the witness stand that as far as he knew, his son, respondent Federico did not own anything "Atty. Lopez: I have here another letter under the letter head of Mr. & Mrs. Dany Mangonon, dated October 19, 1991 addressed to Mr. Francisco Delgado signed by "sincerely, Danny Mangonon, can you remember." xxxx WITNESS: A: I do remember this letter because it really irritated me so much that I threw it away in a waste basket. It is a very demanding letter, that is what I do not like at all. ATTY. LOPEZ: Q: It is stated in this letter that "I am making this request to you and not to your son, Rico, for reasons we both are aware of." Do you know what reason that is? A: Yes. The reason is that my son do not have fix employment and do not have fix salary and income and they want to depend on the lolo. x x x xlavvphi1.net Q: Would you have any knowledge if Federico owns a house and lot? A: Not that I know. I do not think he has anything. Q: How about a car? A: Well, his car is owned by my company.39 Respondent Federico himself admitted in court that he had no property of his own, thus: Q: You also mentioned that you are staying at Mayflower Building and you further earlier testified that this building belongs to Citadel Corporation. Do you confirm that? A: Yes, sir. Q: What car are you driving, Mr. Witness? A: I am driving a lancer, sir. Q: What car, that registered in the name of the corporation? A: In the corporation, sir. Q: What corporation is that? A: Citadel Commercial, Inc., sir. Q: What properties, if any, are registered in your name, do you have any properties, Mr. Witness? A: None, sir."40 (Emphasis supplied.) Meanwhile, respondent Francisco asserts that petitioner possessed the capacity to give support to her twin daughters as she has gainful employment in the USA. He even went as far as to state that petitioners income abroad, when converted to Philippine peso, was much higher than that received by a trial court

594

University of the Cordilleras College of Law First Year C S.Y. 2013 - 2014 judge here in the Philippines. In addition, he claims that as she qualified for the federal parent loan program, she could very well support the college studies of her daughters. We are unconvinced. Respondent Franciscos assertion that petitioner had the means to support her daughters education is belied by the fact that petitioner was even forced by her financial status in the USA to secure the loan from the federal government. If petitioner were really making enough money abroad, she certainly would not have felt the need to apply for said loan. The fact that petitioner was compelled to take out a loan is enough indication that she did not have enough money to enable her to send her daughters to college by herself. Moreover, even Rica and Rina themselves were forced by the circumstances they found themselves in to secure loans under their names so as not to delay their entrance to college. There being prima facie evidence showing that petitioner and respondent Federico are the parents of Rica and Rina, petitioner and respondent Federico are primarily charged to support their childrens college education. In view however of their incapacities, the obligation to furnish said support should be borne by respondent Francisco. Under Article 199 of the Family Code, respondent Francisco, as the next immediate relative of Rica and Rina, is tasked to give support to his granddaughters in default of their parents. It bears stressing that respondent Francisco is the majority stockholder and Chairman of the Board of Directors of Citadel Commercial, Incorporated, which owns and manages twelve gasoline stations, substantial real estate, and is engaged in shipping, brokerage and freight forwarding. He is also the majority stockholder and Chairman of the Board of Directors of Citadel Shipping which does business with Hyundai of Korea. Apart from these, he also owns the Citadel Corporation which, in turn, owns real properties in different parts of the country. He is likewise the Chairman of the Board of Directors of Isla Communication Co. and he owns shares of stocks of Citadel Holdings. In addition, he owns real properties here and abroad.41 It having been established that respondent Francisco has the financial means to support his granddaughters education, he, in lieu of petitioner and respondent Federico, should be held liable for support pendente lite. Anent respondent Francisco and Federicos claim that they have the option under the law as to how they could perform their obligation to support Rica and Rina, respondent Francisco insists that Rica and Rina should move here to the Philippines to study in any of the local universities. After all, the quality of education here, according to him, is at par with that offered in the USA. The applicable provision of the Family Code on this subject provides: Art. 204. The person obliged to give support shall have the option to fulfill the obligation either by paying the allowance fixed, or by receiving and maintaining in the family dwelling the person who has a right to receive support. The latter alternative cannot be availed of in case there is a moral or legal obstacle thereto. Under the abovecited provision, the obligor is given the choice as to how he could dispense his obligation to give support. Thus, he may give the determined amount of support to the claimant or he may allow the latter to stay in the family dwelling. The second option cannot be availed of in case there are circumstances, legal or moral, which should be considered. In this case, this Court believes that respondent Francisco could not avail himself of the second option. From the records, we gleaned that prior to the commencement of this action, the relationship between respondent Francisco, on one hand, and petitioner and her twin daughters, on the other, was indeed quite pleasant. The correspondences exchanged among them expressed profound feelings of thoughtfulness and concern for one anothers well-being. The photographs presented by petitioner as part of her exhibits presented a seemingly typical family celebrating kinship. All of these, however, are now things of the past. With the filing of this case, and the allegations hurled at one another by the parties, the relationships among the parties had certainly been affected. Particularly difficult for Rica and Rina must be the fact that those who they had considered and claimed as family denied having any familial relationship with them. Given all these, we could not see Rica and Rina moving back here in the Philippines in the company of those who have disowned them. Finally, as to the amount of support pendente lite, we take our bearings from the provision of the law mandating the amount of support to be proportionate to the resources or means of the giver and to the necessities of the recipient.42 Guided by this principle, we hold respondent Francisco liable for half of the amount of school expenses incurred by Rica and Rina as support pendente lite. As established by petitioner, respondent Francisco has the financial resources to pay this amount given his various business endeavors. Considering, however, that the twin sisters may have already been done with their education by the time of the promulgation of this decision, we deem it proper to award support pendente lite in arrears43 to be computed from the time they entered college until they had finished their respective studies. 595

University of the Cordilleras College of Law First Year C S.Y. 2013 - 2014 The issue of the applicability of Article 15 of the Civil Code on petitioner and her twin daughters raised by respondent Francisco is best left for the resolution of the trial court. After all, in case it would be resolved that Rica and Rina are not entitled to support pendente lite, the court shall then order the return of the amounts already paid with legal interest from the dates of actual payment.44 WHEREFORE, premises considered, this Petition is PARTIALLY GRANTED. The Decision of the Court of Appeals dated 20 March 1996 and Resolution dated 16 May 1996 affirming the Order dated 12 September 1995 of the Regional Trial Court, Branch 149, Makati, fixing the amount of support pendente lite to P5,000.00 for Rebecca Angela and Regina Isabel, are hereby MODIFIED in that respondent Francisco Delgado is hereby held liable for support pendente lite in the amount to be determined by the trial court pursuant to this Decision. Let the records of this case be remanded to the trial court for the determination of the proper amount of support pendente lite for Rebecca Angela and Regina Isabel as well as the arrearages due them in accordance with this Decision within ten (10) days from receipt hereof. Concomitantly, the trial court is directed to proceed with the trial of the main case and the immediate resolution of the same with deliberate dispatch. The RTC Judge, Branch 149, Makati, is further directed to submit a report of his compliance with the directive regarding the support pendente lite within ten (10) days from compliance thereof. SO ORDERED.

596

University of the Cordilleras College of Law First Year C S.Y. 2013 - 2014

IX. Property Relations between Husband and Wife

597

University of the Cordilleras College of Law First Year C S.Y. 2013 - 2014 Jocson vs Court of Appeals 170 SCRA 333 G.R. No. L-55322 February 16, 1989 Full Case MOISES JOCSON, petitioner, vs. HON. COURT OF APPEALS, AGUSTINA JOCSON-VASQUEZ, ERNESTO VASQUEZ, respondents. Dolorfino and Dominguez Law Officers for petitioner. Gabriel G. Mascardo for private respondents. MEDIALDEA, J.: This is a petition for review on certiorari under Rule 45 of the Rules of Court of the decision of the Court of Appeals in CA- G.R. No. 63474, promulgated on April 30, 1980, entitled "MOISES JOCSON, plaintiff-appellee, versus AGUSTINA JOCSON-VASQUEZ and ERNESTO VASQUEZ, defendantappellants," upholding the validity of three (3) documents questioned by Moises Jocson, in total reversal of the decision of the then Court of First Instance of Cavite, Branch I, which declared them as null and void; and of its resolution, dated September 30, 1980, denying therein appellee's motion for reconsideration. Petitioner Moises Jocson and respondent Agustina Jocson-Vasquez are the only surviving offsprings of the spouses Emilio Jocson and Alejandra Poblete, while respondent Ernesto Vasquez is the husband of Agustina. Alejandra Poblete predeceased her husband without her intestate estate being settled. Subsequently, Emilio Jocson also died intestate on April 1, 1972. As adverted to above, the present controversy concerns the validity of three (3) documents executed by Emilio Jocson during his lifetime. These documents purportedly conveyed, by sale, to Agustina JocsonVasquez what apparently covers almost all of his properties, including his one-third (1/3) share in the estate of his wife. Petitioner Moises Jocson assails these documents and prays that they be declared null and void and the properties subject matter therein be partitioned between him and Agustina as the only heirs of their deceased parents. The documents, which were presented as evidence not by Moises Jocson, as the party assailing its validity, but rather by herein respondents, are the following: 1) "Kasulatan ng Bilihan ng Lupa," marked as Exhibit 3 (pp. 12-13, Records) for the defendant in the court a quo, dated July 27, 1968. By this document Emilio Jocson sold to Agustina Jocson-Vasquez six (6) parcels of land, all located at Naic, Cavite, for the sum of ten thousand P10,000.00 pesos. On the same document Emilio Jocson acknowledged receipt of the purchase price, thus: Na ngayon, alang-alang sa halagang SAMPUNG LIBONG PISO (P10,000) salaping Pilipino na aking tinanggap ng buong kasiyahan loob at ang pagkakatanggap ay aking hayagang inaamin sa pamamagitan ng kasulatang ito, sa aking anak na si Agustina Jocson, na may sapat na gulang, mamamayang Pilipino, asawa ni Ernesto Vasquez, at naninirahan sa Poblacion, Naic, Cavite, ay aking ipinagbile ng lubusan at kagyat at walang ano mang pasubali ang nabanggit na anim na pirasong lupa na nasa unang dahon ng dokumentong ito, sa nabanggit na Agustina Jocson, at sa kaniyang tagapagmana o makakahalili at gayon din nais kong banggitin na kahit na may kamurahan ang ginawa kong pagbibile ay dahilan sa ang nakabile ay aking anak na sa akin at mapaglingkod, madamayin at ma-alalahanin, na tulad din ng isa ko pang anak na lalaki. Ang kuartang tinanggap ko na P10,000.00, ay gagamitin ko sa aking katandaan at mga huling araw at sa aking mga ibang mahahalagang pangangailangan. [Emphasis supplied] Na nais ko ring banggitin na ang ginawa kong ito ay hindi labag sa ano mang batas o kautusan, sapagkat ang aking pinagbile ay akin at nasa aking pangalan. Ang mga lupang nasa pangalan ng aking nasirang asawa ay hindi ko ginagalaw ni pinakikialaman at iyon ay dapat na hatiin ng dalawa kong anak alinsunod sa umiiral na batas (p. 13, Records.) 2) "Kasulatan ng Ganap na Bilihan,"dated July 27,1968, marked as Exhibit 4 (p. 14, Records). On the face of this document, Emilio Jocson purportedly sold to Agustina Jocson-Vasquez, for the sum of FIVE THOUSAND (P5,000.00) PESOS, two rice mills and a camarin (camalig) located at Naic, Cavite. As in the first document, Moises Jocson acknowledged receipt of the purchase price:

598

University of the Cordilleras College of Law First Year C S.Y. 2013 - 2014 'Na alang-alang sa halagang LIMANG LIBONG PISO (P5,000.00) salaping Pilipino na aking tinanggap ng buong kasiyahan loob sa aking anak na Agustina Jocson .... Na ang halagang ibinayad sa akin ay may kamurahan ng kaunti ngunit dahil sa malaking pagtingin ko sa kaniya ... kaya at pinagbile ko sa kaniya ang mga nabanggit na pagaari kahit na hindi malaking halaga ... (p. 14, Records). 3) Lastly, the "Deed of Extrajudicial Partition and Adjudication with Sale, "dated March 9, 1969, marked as Exhibit 2 (p. 10-11, Records), whereby Emilio Jocson and Agustina Jocson-Vasquez, without the participation and intervention of Moises Jocson, extrajudicially partitioned the unsettled estate of Alejandra Poblete, dividing the same into three parts, one-third (1/3) each for the heirs of Alejandra Poblete, namely: Emilio Jocson, Agustina Jocson-Vasquez and Moises Jocson. By the same instrument, Emilio sold his one- third (1/3) share to Agustin for the sum of EIGHT THOUSAND (P8,000.00) PESOS. As in the preceding documents, Emilio Jocson acknowledged receipt of the purchase price: Now for and in consideration of the sum of only eight thousand (P8,000.00) pesos, which I, the herein Emilio Jocson had received from my daughter Agustina Jocson, do hereby sell, cede, convey and transfer, unto the said Agustina Jocson, her heirs and assigns, administrators and successors in interests, in the nature of absolute and irrevocable sale, all my rights, interest, shares and participation, which is equivalent to one third (1/3) share in the properties herein mentioned and described the one third being adjudicated unto Agustina Jocson and the other third (1/3) portion being the share of Moises Jocson. (p. 11, Records). These documents were executed before a notary public. Exhibits 3 and 4 were registered with the Office of the Register of Deeds of Cavite on July 29, 1968 and the transfer certificates of title covering the properties therein in the name of Emilio Jocson, married to Alejandra Poblete," were cancelled and new certificates of title were issued in the name of Agustina Jocson-Vasquez. Exhibit 2 was not registered with the Office of the Register of Deeds. Herein petitioner filed his original complaint (Record on Appeal, p. 27, Rollo) on June 20,1973 with the then Court of First Instance of Naic, Cavite (docketed as Civil Case No. TM- 531), and which was twice amended. In his Second Amended Complaint (pp. 47-58, Record on Appeal), herein petitioner assailed the above documents, as aforementioned, for being null and void. It is necessary to partly quote the allegation of petitioner in his complaint for the reason that the nature of his causes of action is at issue, thus: 8. [With regard the first document, that] the defendants, through fraud, deceit, undue pressure and influence and other illegal machinations, were able to induce, led, and procured their father ... to sign [the] contract of sale ..., for the simulated price of P10,000.00, which is a consideration that is shocking to the conscience of ordinary man and despite the fact that said defendants have no work or livelihood of their own ...; that the sale is null and void, also, because it is fictitious, simulated and fabricated contract x x x (pp. 52-53, Record on Appeal). [Emphasis supplied] xxx xxx xxx 12. [With regards the second and third document, that they] are null and void because the consent of the father, Emilio Jocson, was obtained with fraud, deceit, undue pressure, misrepresentation and unlawful machinations and trickeries committed by the defendant on him; and that the said contracts are simulated, fabricated and fictitious, having been made deliberately to exclude the plaintiff from participating and with the dishonest and selfish motive on the part of the defendants to defraud him of his legitimate share on said properties [subject matter thereof]; and that without any other business or employment or any other source of income, defendants who were just employed in the management and administration of the business of their parents, would not have the sufficient and ample means to purchase the said properties except by getting the earnings of the business or by simulated consideration ... (pp. 54-55, Record on Appeal). [Emphasis supplied] Petitioner explained that there could be no real sale between a father and daughter who are living under the same roof, especially so when the father has no need of money as the properties supposedly sold were all income-producing. Further, petitioner claimed that the properties mentioned in Exhibits 3 and 4 are the unliquidated conjugal properties of Emilio Jocson and Alejandra Poblete which the former, therefore, cannot validly sell (pp. 53, 57, Record on Appeal). As far as Exhibit 2 is concerned, petitioner questions

599

University of the Cordilleras College of Law First Year C S.Y. 2013 - 2014 not the extrajudicial partition but only the sale by his father to Agustina of the former's 1/3 share (p. 13, Rollo). The trial court sustained the foregoing contentions of petitioner (pp. 59-81, Record on Appeal). It declared that the considerations mentioned in the documents were merely simulated and fictitious because: 1) there was no showing that Agustina Jocson-Vasquez paid for the properties; 2) the prices were grossly inadequate which is tantamount to lack of consideration at all; and 3) the improbability of the sale between Emilio Jocson and Agustina Jocson-Vasquez, taking into consideration the circumstances obtaining between the parties; and that the real intention of the parties were donations designed to exclude Moises Jocson from participating in the estate of his parents. It further declared the properties mentioned in Exhibits 3 and 4 as conjugal properties of Emilio Jocson and Alejandra Poblete, because they were registered in the name of "Emilio Jocson, married to Alejandra Poblete" and ordered that the properties subject matter of all the documents be registered in the name of herein petitioners and private respondents. On appeal, the Court of Appeals in CA-G.R. No. 63474-R rendered a decision (pp. 29-42, Rollo) and reversed that of the trial court's and ruled that: 1. That insofar as Exhibits 3 and 4 are concerned the appellee's complaint for annulment, which is indisputably based on fraud, and undue influence, is now barred by prescription, pursuant to the settled rule that an action for annulment of a contract based on fraud must be filed within four (4) years, from the discovery of the fraud, ... which in legal contemplation is deemed to be the date of the registration of said document with the Register of Deeds ... and the records admittedly show that both Exhibits 3 and 4, were all registered on July 29, 1968, while on the other hand, the appellee's complaint was filed on June 20, 1973, clearly beyond the aforesaid four-year prescriptive period provided by law; 2. That the aforesaid contracts, Exhibits 2, 3, and 4, are decisively not simulated or fictitious contracts, since Emilio Jocson actually and really intended them to be effective and binding against him, as to divest him of the full dominion and ownership over the properties subject of said assailed contracts, as in fact all his titles over the same were all cancelled and new ones issued to appellant Agustina Jocson-Vasquez ...; 3. That in regard to Exhibit 2, the same is valid and subsisting, and the partition with sale therein made by and between Emilio Jocson and Agustina Jocson-Vasquez, affecting the 2/3 portion of the subject properties described therein have all been made in accordance with Article 996 of the New Civil Code on intestate succession, and the appellee's (herein petitioner) remaining 1/3 has not been prejudiced (pp. 41-42, Rollo). In this petition for review, Moises Jocson raised the following assignments of errors: 1. HAS THE RESPONDENT COURT OF APPEALS ERRED IN CONCLUDING THAT THE SUIT FOR THE ANNULMENT OF CONTRACTS FILED BY PETITIONERS WITH THE TRIAL COURT IS "BASED ON FRAUD" AND NOT ON ITS INEXISTENCE AND NULLITY BECAUSE OF IT'S BEING SIMULATED OR FICTITIOUS OR WHOSE CAUSE IS CONTRARY TO LAW, MORALS AND GOOD CUSTOMS? II. HAS THE RESPONDENT COURT OF APPEALS ERRED IN CONCLUDING THAT THE COMPLAINT FILED BY PETITIONER IN THE TRIAL COURT IS BARRED BY PRESCRIPTION? III. HAS THE RESPONDENT COURT OF APPEALS ERRED IN NOT DECLARING AS INEXISTENT AND NULL AND VOID THE CONTRACTS IN QUESTION AND IN REVERSING THE DECLARING DECISION OF THE TRIAL COURT? (p. 2, Rollo) I. The first and second assignments of errors are related and shall be jointly discussed. According to the Court of Appeals, herein petitioner's causes of action were based on fraud. Under Article 1330 of the Civil Code, a contract tainted by vitiated consent, as when consent was obtained through fraud, is voidable; and the action for annulment must be brought within four years from the time of the discovery of the fraud (Article 1391, par. 4, Civil Code), otherwise the contract may no longer be contested. Under present jurisprudence, discovery of fraud is deemed to have taken place at the time the convenant was registered with the Register of Deeds (Gerona vs. De Guzman, No. L-19060, May 29,1964, 11 SCRA 153). Since Exhibits 3 and 4 were registered on July 29, 1968 but Moises Jocson filed

600

University of the Cordilleras College of Law First Year C S.Y. 2013 - 2014 his complaint only on June 20, 1973, the Court of Appeals ruled that insofar as these documents were concerned, petitioner's "annulment suit" had prescribed. If fraud were the only ground relied upon by Moises Jocson in assailing the questioned documents, We would have sustained the above pronouncement. But it is not so. As pointed out by petitioner, he further assailed the deeds of conveyance on the ground that they were without consideration since the amounts appearing thereon as paid were in fact merely simulated. According to Article 1352 of the Civil Code, contracts without cause produce no effect whatsoever. A contract of sale with a simulated price is void (Article 1471; also Article 1409 [3]]), and an action for the declaration of its nullity does not prescribe (Article 1410, Civil Code; See also, Castillo v. Galvan, No. L27841, October 20, l978, 85 SCRA 526). Moises Jocsons saction, therefore, being for the judicial declaration of nullity of Exhibits 3 and 4 on the ground of simulated price, is imprescriptible. II. For petitioner, however, the above discussion may be purely academic. The burden of proof in showing that contracts lack consideration rests on he who alleged it. The degree of proof becomes more stringent where the documents themselves show that the vendor acknowledged receipt of the price, and more so where the documents were notarized, as in the case at bar. Upon consideration of the records of this case, We are of the opinion that petitioner has not sufficiently proven that the questioned documents are without consideration. Firstly, Moises Jocson's claim that Agustina Jocson-Vasquez had no other source of income other than what she derives from helping in the management of the family business (ricefields and ricemills), and which was insufficient to pay for the purchase price, was contradicted by his own witness, Isaac Bagnas, who testified that Agustina and her husband were engaged in the buy and sell of palay and rice (p. 10, t.s.n., January 14, 1975). Amazingly, petitioner himself and his wife testified that they did not know whether or not Agustina was involved in some other business (p. 40, t.s.n., July 30, 1974; p. 36, t.s.n., May 24, 1974). On the other hand, Agustina testified that she was engaged in the business of buying and selling palay and rice even before her marriage to Ernesto Vasquez sometime in 1948 and continued doing so thereafter (p. 4, t.s.n., March 15, 1976). Considering the foregoing and the presumption that a contract is with a consideration (Article 1354, Civil Code), it is clear that petitioner miserably failed to prove his allegation. Secondly, neither may the contract be declared void because of alleged inadequacy of price. To begin with, there was no showing that the prices were grossly inadequate. In fact, the total purchase price paid by Agustina Jocson-Vasquez is above the total assessed value of the properties alleged by petitioner. In his Second Amended Complaint, petitioner alleged that the total assessed value of the properties mentioned in Exhibit 3 was P8,920; Exhibit 4, P3,500; and Exhibit 2, P 24,840, while the purchase price paid was P10,000, P5,000, and P8,000, respectively, the latter for the 1/3 share of Emilio Jocson from the paraphernal properties of his wife, Alejandra Poblete. And any difference between the market value and the purchase price, which as admitted by Emilio Jocson was only slight, may not be so shocking considering that the sales were effected by a father to her daughter in which case filial love must be taken into consideration (Alsua-Betts vs. Court of Appeals, No. L-46430-31, April 30, 1979, 92 SCRA 332). Further, gross inadequacy of price alone does not affect a contract of sale, except that it may indicate a defect in the consent, or that the parties really intended a donation or some other act or contract (Article 1470, Civil Code) and there is nothing in the records at all to indicate any defect in Emilio Jocson's consent. Thirdly, any discussion as to the improbability of a sale between a father and his daughter is purely speculative which has no relevance to a contract where all the essential requisites of consent, object and cause are clearly present. There is another ground relied upon by petitioner in assailing Exhibits 3 and 4, that the properties subject matter therein are conjugal properties of Emilio Jocson and Alejandra Poblete. It is the position of petitioner that since the properties sold to Agustina Jocson-Vasquez under Exhibit 3 were registered in the name of "Emilio Jocson, married to Alejandra Poblete," the certificates of title he presented as evidence (Exhibits "E', to "J', pp. 4-9, Records) were enough proof to show that the properties covered therein were acquired during the marriage of their parents, and, therefore, under Article 160 of the Civil Code, presumed to be conjugal properties. Article 160 of the Civil Code provides that: All property of the marriage is presumed to belong to the conjugal partnership, unless it be proved that it pertains exclusively to the husband or to the wife. In Cobb-Perez vs. Hon. Gregorio Lantin, No. L-22320, May 22, 1968, 23 SCRA 637, 644, We held that: 601

University of the Cordilleras College of Law First Year C S.Y. 2013 - 2014 Anent their claim that the shares in question are conjugal assets, the spouses Perez adduced not a modicum of evidence, although they repeatedly invoked article 160 of the New Civil Code which provides that ... . As interpreted by this Court, the party who invokes this presumption must first prove that the property in controversy was acquired during the marriage. In other words, proof of acquisition during the coverture is a condition sine qua non for the operation of the presumption in favor of conjugal ownership. Thus in Camia de Reyes vs. Reyes de Ilano [62 Phil. 629, 639], it was held that "according to law and jurisprudence, it is sufficient to prove that the Property was acquired during the marriage in order that the same may be deemed conjugal property." In the recent case of Maramba vs. Lozano, et. al. [L-21533, June 29, 1967, 20 SCRA 474], this Court, thru Mr. Justice Makalintal, reiterated that "the presumption under Article 160 of the Civil Code refers to property acquired during the marriage," and then concluded that since "there is no showing as to when the property in question was acquired...the fact that the title is in the wife's name alone is determinative." Similarly, in the case at bar, since there is no evidence as to when the shares of stock were acquired, the fact that they are registered in the name of the husband alone is an indication that the shares belong exclusively to said spouse.' This pronouncement was reiterated in the case of Ponce de Leon vs. Rehabilitation Finance Corporation, No. L-24571, December 18, 1970, 36 SCRA 289, and later in Torela vs. Torela, No. 1,27843, October 11, 1979, 93 SCRA 391. It is thus clear that before Moises Jocson may validly invoke the presumption under Article 160 he must first present proof that the disputed properties were acquired during the marriage of Emilio Jocson and Alejandra Poblete. The certificates of title, however, upon which petitioner rests his claim is insufficient. The fact that the properties were registered in the name of "Emilio Jocson, married to Alejandra Poblete" is no proof that the properties were acquired during the spouses' coverture. Acquisition of title and registration thereof are two different acts. It is well settled that registration does not confer title but merely confirms one already existing (See Torela vs. Torela, supra). It may be that the properties under dispute were acquired by Emilio Jocson when he was still a bachelor but were registered only after his marriage to Alejandra Poblete, which explains why he was described in the certificates of title as married to the latter. Contrary to petitioner's position, the certificates of title show, on their face, that the properties were exclusively Emilio Jocson's, the registered owner. This is so because the words "married to' preceding "Alejandra Poblete' are merely descriptive of the civil status of Emilio Jocson Litam v. Rivera, 100 Phil. 354; Stuart v. Yatco, No. L-16467, April 27, 1962, 4 SCRA 1143; Magallon v. Montejo, G.R. No. L73733, December 16, 1986, 146 SCRA 282). In other words, the import from the certificates of title is that Emilio Jocson is the owner of the properties, the same having been registered in his name alone, and that he is married to Alejandra Poblete. We are not unmindful that in numerous cases We consistently held that registration of the property in the name of only one spouse does not negate the possibility of it being conjugal (See Bucoy vs. Paulino, No. L-25775, April 26, 1968, 23 SCRA 248). But this ruling is not inconsistent with the above pronouncement for in those cases there was proof that the properties, though registered in the name of only one spouse, were indeed conjugal properties, or that they have been acquired during the marriage of the spouses, and therefore, presumed conjugal, without the adverse party having presented proof to rebut the presumption (See Mendoza vs- Reyes, No. L-31618, August 17, 1983, 124 SCRA 154). In the instant case, had petitioner, Moises Jocson, presented sufficient proof to show that the disputed properties were acquired during his parents' coverture. We would have ruled that the properties, though registered in the name of Emilio Jocson alone, are conjugal properties in view of the presumption under Article 160. There being no such proof, the condition sine qua non for the application of the presumption does not exist. Necessarily, We rule that the properties under Exhibit 3 are the exclusive properties of Emilio Jocson. There being no showing also that the camarin and the two ricemills, which are the subject of Exhibit 4, were conjugal properties of the spouses Emilio Jocson and Alejandra Poblete, they should be considered, likewise, as the exclusive properties of Emilio Jocson, the burden of proof being on petitioner. ACCORDINGLY, the petition is DISMISSED and the decision of the Court of Appeals is AFFIRMED. SO ORDERED. Narvasa, Cruz, Gancayco and Grio-Aquino, JJ., concur.

602

University of the Cordilleras College of Law First Year C S.Y. 2013 - 2014

Case Digest Jocson vs Court of Appeals G.R. No. L-55322 Decided on: February 16, 1989 Ponente: J. Medialdea Facts: Spouses Emilio Jocson & Alejandra Poblete had 2 children: Moises Jocson & Agustina JocsonVasquez. Agustina is married to Ernesto Vasquez. Alejandra died intestate. On April 1, 1972, Emilio died intestate. On June 20, 1973, Moises filed a complaint, assailing the validity of 3 documents executed by Emilio during his lifetime. He prays that the following be declared null and void and that the properties involved be partitioned between him and his sister: 1. Deed of Sale executed July 27, 1968 wherein Emilio sold to Agustina 6 parcels of land in Naic, Cavite for P10,000.00. Deed included Emilios manifestation that the lands were sold at a low price because it was his loving, helpful & thoughtful daughter who bought the property. He says his son possesses such qualities too. He further claims that the sale did not violate any law & that he did not touch his wifes properties. He acknowledged receipt of payment. 2. Deed of Sale executed July 27, 1968, selling 2 rice mills and a camalig in Naic, Cavite to Agustina for P5,000.00. Emilio acknowledged receipt too. 3. Deed of Extrajudicial Partition & Adjudication w/Sale executed March 9, 1969 wherein Emilio & Agustina, excluding Moises, extra-judicially partitioned unsettled estate of Alejandra dividing such into 3. Emilio sold his share to Agustina. All documents were executed before a notary public. Nos. 1 & 2 were registered with the Register of Deeds. Old certificates were cancelled & new certificates issued in the name of Agustina. Moises allegations: 1. #1 is null and void because his fathers consent was obtained by fraud, deceit, undue pressure, influence & other illegal machinations. He also alleges that property was sold for a simulated price considering that his sister had no work or livelihood of her own. Also, he claims that the contract is fictitious, simulated and fabricated. 2. Same allegations regarding #2 & #3 with additional allegation that he was deliberately excluded and they intended to defraud him of his legitimate share. He also claims that defendants were employed in their parents business and they must have used business earnings or simulated consideration in order to purchase the properties. 3. No real sale between dad and daughter living under same roof. 4. Dad didnt need money since sold properties were all income-producitng. 5. #1 & #2 are unliquidated conjugal properties that Emilio cant validly sell. 6. He only questions sale of dads share to sister but not extra judicial partition in #3. RTC: decided in favor of petitioner. Documents were simulated & fictitious because: 1. No proof that Agustina paid for the properties; 2. Prices were grossly inadequate tantamount to lack of consideration at all; 603

University of the Cordilleras College of Law First Year C S.Y. 2013 - 2014 3. Improbability of sale considering circumstances. Designed to exclude Moises. Numbers 1 and 2 were declared as conjugal properties by virtue of registration papers which declared that Emilio Jocson, married to Alejandra Poblete. The RTC ordered the registration of property to two children. CA: reversed. Nos. 1 & 2 barred by prescription because annulment of contract based on fraud must be filed 4 years from discovery of such which begins on the date of the registration with the Register of Deeds. All documents actually and intended to be binding and effective against Emilio. Proof of such: issuance of new titles. Partition with sale in #3 is valid since it was done in accordance with the New Civil Code Art. 996 on intestate succession and Moises 1/3 has not been prejudiced. Issue: Whether or not properties in #1 & #2 were conjugal properties of Emilio & wife. Ruling: NO. Art. 160 of the Family Code provides that all property of marriage is presumed to belong to Conjugal Partnership unless proven otherwise. Condition sine qua non (main thing) would be for the party who invokes this to prove that properties were indeed acquired during the marriage (Cobb-Perez vs. Lantin). Thus, Moises has to present proof that the properties in question were indeed obtained during the marriage of their parents before he can invoke the presumption. However, titles used by RTC in declaring properties as Conjugal Property (see RTC decision in bold letters) are insufficient proof. Doesnt say when properties were obtained. Acquisition of title (actual owning of land) is different from registration. Possible that Emilio acquired properties when he was still a bachelor & only registered such after marriage.

604

University of the Cordilleras College of Law First Year C S.Y. 2013 - 2014 Mariano vs Court of Appeals 174 SCRA 59 G.R. No. L-51283 June 7, 1989 Full Case LOURDES MARIANO, petitioner, vs. COURT OF APPEALS, and DANIEL SANCHEZ, respondents. NARVASA, J.: The proceedings at bar concern (1) an attempt by a married man to prevent execution against conjugal property of a judgment rendered against his wife, for obligations incurred by the latter while engaged in a business that had admittedly redounded to the benefit of the family, and (2) the interference by a court with the proceedings on execution of a co-equal or coordinate court. Both acts being proscribed by law, correction is called for and will hereby be effected. The proceedings originated from a suit filed by Esther Sanchez against Lourdes Mariano in the Court of First Instance at Caloocan City, 1 for recovery of the value of ladies' ready made dresses allegedly purchased by and delivered to the latter. 2 A writ of preliminary attachment issued at Esther Sanchez' instance, upon a bond posted by Veritas Insurance Company in the amount of P 11,000.00, and resulted in the seizure of Lourdes Mariano's property worth P 15,000.00 or so. 3 Her motion for the discharge of the attachment having been denied, 4 Lourdes Mariano went up to the Court of Appeals on certiorari. That Court ordered 5 the Trial Court to receive evidence on whether or not the attachment had been improvidently or irregularly issued. 6 The Trial Court did so, came to the conclusion that the attachment had indeed been improperly issued, and consequently dissolved it. 7 Trial then ensued upon the issues arising from the complaint as well as Lourdes Mariano's answer with counterclaim-which included a claim for damages resulting from wrongful attachment. Thereafter judgment was rendered in favor of defendant Lourdes Mariano and against plaintiff Esther Sanchez containing the following dispositions, to wit: 8 1. On the complaint, defendant is ordered to pay unto the plaintiff for the value of the dishonored check (Exhs. G-1, H and I) in the total amount of P 1,512.00. 2. On the counterclaim, the plaintiff is ordered to pay unto defendant the following, as follows: a) P 7,500.00 for loss of income of the defendant for 75 days; b) P 16,000.00 for the value of attached goods; c) P 25,000.00 for moral and exemplary damages; d) P 5,000.00 as attorney's fees plus costs of suit. The Veritas Insurance Company which issued the attachment bond is ordered to pay unto the defendant the full insurance coverage of P 11,000.00 to answer for the total liability of the plaintiff thereof Esther Sanchez sought to perfect an appeal by filing a notice of appeal, an appeal bond and a record on appeal. 9 Pending approval of the record on appeal, Lourdes Mariano filed a motion for the immediate execution of the judgment which the Court granted. 10 In virtue of the writ of execution which afterwards issued in due course, the sheriff garnished the sum of P 11,000.00 from Veritas Insurance Company, and levied on real and personal property belonging to the conjugal partnership of Esther Sanchez and her husband, Daniel Sanchez. Esther Sanchez then filed a petition for certiorari with the Court of Appeals, praying for the annulment of the execution pending appeal authorized by the Trial Court; but her petition was adjudged to be without merit and was accordingly dismissed. 11 Daniel Sanchez, Esther's husband, now made his move. He filed a complaint for annulment of the execution in the Court of First Instance at Quezon City in his capacity as administrator of the conjugal partnership. 12 He alleged that the conjugal assets could not validly be made to answer for obligations exclusively contracted by his wife, and that, moreover, some of the personal property levied on, such as household appliances and utensils necessarily used in the conjugal dwelling, were exempt from execution. He also applied for a preliminary injunction pending adjudication of the case on the merits. 13 The Quezon City Court issued an order setting the matter of the injunction for hearing, and commanding the sheriff, in the meantime, to desist from proceeding with the auction sale of the property subject of Daniel Sanchez' claim. 14 Lourdes Mariano filed a motion to dismiss the action; this, the Court denied. 15 She then instituted a special civil action of certiorari in the Court of Appeals 16 where she 605

University of the Cordilleras College of Law First Year C S.Y. 2013 - 2014 initially enjoyed some measure of success: her petition was given due course, and the Quezon City Court was restrained by the Appellate Court's Seventh Division 17 from further proceeding with the case. 18 Eventually, however, the Eighth Division 19 came to the conclusion that there was no merit in her cause and dismissed her petition. 20 It ruled that the Quezon City Court had not interfered with the execution process of the Caloocan Court because Daniel Sanchez's action in the former court raised an issue-the validity of the sheriffs levy on the conjugal partnership assets of the Sanchez spouses different from those adjudicated in the Caloocan Court, and Sanchez was not a party to the case tried by the latter. From this verdict Lourdes Mariano has appealed to this Court, contending that the Appellate Court committed reversible error1) in ruling that the conjugal partnership of Daniel and Esther Sanchez could not be made liable for Esther's judgment obligation arising from the spouses' joint business with Lourdes Mariano; 2) in ruling that the Quezon City Court of First Instance had not interfered with the execution process of the Caloocan Court of First Instance; and 3) when its Eighth Division decided the petition of Lourdes Mariano although the case had been raffled to the Seventh Division and the latter had in fact given due course to the petition. 1. There is no dispute about the fact that Esther Sanchez was engaged in business not only without objection on the part of her husband, Daniel, but in truth with his consent and approval. 21 It is also established that, as expressly acknowledged by Esther herself and never denied by Daniel, the profits from the business had been used to meet, in part at least, expenses for the support of her family, i.e., the schooling of the children, food and other household expenses. 22 Under the circumstances, Lourdes Mariano action against Esther Sanchez was justified, the litigation being "incidental to the ... business in which she is engaged 23 and consequently, the conjugal partnership of Daniel and Esther Sanchez was liable for the debts and obligations contracted by Esther in her business since the income derived therefrom, having been used to defray some of the expenses for the maintenance of the family and the education of the children, had redounded to the benefit of the partnership. 24 It was therefore error for the Court of Appeals to have ruled otherwise. 2. It was also error for the Court of Appeals to have held that there was no interference by the Quezon City Court of First Instance with the execution process of the Caloocan Court. The rule, one of great importance in the administration of justice, is that a Court of First Instance has no power to restrain by means of injunction the execution of a judgment or decree of another judge of concurrent or coordinate jurisdictions. 25 But this is precisely what was done by the Quezon City Court of First Instance: it enjoined the execution of a judgment authorized and directed by a co-equal and coordinate court, the Caloocan City Court of First Instance. It did so on the claim of Daniel Sanchez that the property being levied on belonged to the conjugal partnership and could not be made liable for the wife's obligations. The question that arises is whether such a claim that property levied on in execution of a judgment is not property of the judgment debtor, Daniel Sanchez's wife, but of the conjugal partnership of the Sanchez Spouses is properly cognizable by a Court other than that which rendered judgment adversely to the wife. To be sure, Section 17, Rule 39 of the Rules of Court, authorizes a "third person," i.e., "any other person than the judgment debtor or his agent," to vindicate "his claim to the property by any proper action." The section reads as follows: 26 SEC. 17. Proceedings where property claimed by third person.-If property levied on be claimed by any other person than the judgment debtor or his agent, and such person make an affidavit of his title thereto or right to the possession thereof, stating the grounds of such right or title, and serve the same upon the officer making the levy, and a copy thereof upon the judgment creditor, the officer shall not be bound to keep the property, unless such judgment creditor or his agent, on demand of the officer, indemnify the officer against such claim by a bond in a sum not greater than the value of the property levied on. In case of disagreement as to such value, the same shall be determined by the court issuing the writ of execution. The officer is not liable for damages, for the taking or keeping of the property, to any third-party claimant, unless a claim is made by the latter and unless an action for damages 606

University of the Cordilleras College of Law First Year C S.Y. 2013 - 2014 is brought by him against the officer within one hundred twenty (120) days from the date of the filing of the bond. But nothing herein contained shall prevent such claimant or any third person from vindicating his claim to the property by any proper action. xxx xxx xxx The "proper action" referred to in the section "is and should be an entirely separate and distinct action from that in which execution has issued, if instituted by a stranger to the latter suit:" 27 and in "such separate action, the court may issue a writ of preliminary injunction against the sheriff enjoining him from proceeding with the execution sale."28 "Upon the other hand, if the claim of impropriety on the part of the sheriff in the execution proceedings is made by a party to the action, not a stranger thereto, any relief therefrom may be applied for with, and obtained from, only the executing court; and this is true even if a new party has been impleaded in the suit." 29 In the case at bar, the husband of the judgment debtor cannot be deemed a "stranger" to the case prosecuted and adjudged against his wife. A strikingly similar situation was presented in a case decided by this Court as early as 1976, Rejuso v. Estipona. 30 There, the sheriff tried to evict petitioner Rejuso and his family from their house and lot which had been sold in execution of a money judgment rendered by the Court of First Instance of Davao against Rejuso. What Rejuso did was to institute, together with his wife, Felisa, a separate suit in the same court against the sheriff and the judgment creditor, Estipona, for the purpose of annulling the levy, execution sale, and writ of possession issued in the first action in respect of their residential house and lot, on the theory that that property was conjugal in character and "hence, not subject to such proceedings considering that Felisa was not a party to the previous case." The action was however dismissed by the court on the ground that it had "no jurisdiction over the subject matter of the action or the nature of the action and of the relief sought." 31 The dismissal was had on motion of Estipona who argued that the court had no jurisdiction to "vacate or annul and/or enjoin the enforcement of the process issued by another branch in another case," and since Rejuso had already raised the same issues in the first case, without success, he should not be allowed to "get from another branch ... what he failed to get ... (from) Branch l." This Court affirmed that judgment of dismissal, 32 holding that Rejuso's action was barred by res adjudicata; and "(a)s regards Felisa Rejuso, who is a new party in Civil Case No. 5102" (the second action) it was ruled that... her remedy, if it has not yet been barred by the statute of limitations or become stale in some other way, is within Civil Case No. 4435 (the first suit). Indeed, it is superfluous to start a new action on a matter which can be more simply and conveniently litigated within a former proceeding of which it is more logically and legally an integral part. (Ipekdjian Merchandising Co., Inc, v. CTA, 8 SCRA 59 [1963]). Actually, the court in which the former proceeding was pending has exclusive jurisdiction thereof (De Leon vs. Salvador, 36 SCRA 567), the fact that the two cases are in the same Branch of the same Court of First Instance and presided over by the same Judge notwithstanding. After all, it is simpler and more convenient to observe such practice, which insures also consistency in the resolutions of related questions because they are to be determined in most if not all instances by the same judge. In any case, whether by intervention in the court issuing the writ, or by separate action, it is unavailing for either Esther Sanchez or her husband, Daniel, to seek preclusion of the enforcement of the writ of possession against their conjugal assets. For it being established, as aforestated, that Esther had engaged in business with her husband's consent, and the income derived therefrom had been expended, in part at least, for the support of her family, the liability of the conjugal assets to respond for the wife's obligations in the premises cannot be disputed. The petitioner's appeal must therefore be sustained. However, the petitioner's theory that the Eighth Division of the Appellate Court had improperly taken cognizance of the case which had been raffled to the Seventh Division, must be rejected. It is without foundation, and was evidently made without attempt to ascertain the relevant facts and applicable rules. The case had originally been assigned to Mr. Justice Isidro C. Borromeo for study and report while he was still a member of the Seventh Division. The case was brought by him to the Eighth Division when he was subsequently transferred thereto; and he had ultimately written the opinion for the division after due deliberation with his colleagues. All of this took place in accordance with the Rules of the Court of Appeals. WHEREFORE, the Decision of the Court of Appeals subject of the petition is REVERSED AND SET ASIDE, and the Regional Trial Court (formerly Court of First Instance) at Quezon City is ORDERED to dismiss Civil Case No. 20415 entitled "Daniel P. Sanchez v. Deputy Sheriff Mariano V. Cachero, et al.," with prejudice. Costs against private respondents. SO ORDERED. 607

University of the Cordilleras College of Law First Year C S.Y. 2013 - 2014 Case Digerst Mariano vs Court of Appeals G.R. No. L-51283 Decided on: June 7, 1989 Ponente: J. Narvasa Facts: The proceedings at bar concern (1) an attempt by a married man to prevent execution against conjugal property of a judgment rendered against his wife, for obligations incurred by the latter while engaged in a business that had admittedly redounded to the benefit of the family, and (2) the interference by a court with the proceedings on execution of a co-equal or coordinate court. Both acts being proscribed by law, correction is called for and will hereby be effected. The proceedings originated from a suit filed by Esther Sanchez against Lourdes Mariano in the Court of First Instance at Caloocan City, for recovery of the value of ladies ready-made dresses allegedly purchased by and delivered to the latter. Daniel Sanchez, Esthers husband, now made his move. He filed a complaint for annulment of the execution in the Court of First Instance at Quezon City in his capacity as administrator of the conjugal partnership. He alleged that the conjugal assets could not validly be made to answer for obligations exclusively contracted by his wife, and that, moreover, some of the personal property levied on, such as household appliances and utensils necessarily used in the conjugal dwelling, were exempt from execution. Issue: Whether or not the claim that property levied on in execution of a judgment is not property of the judgment debtor, Daniel Sanchezs wife, but of the conjugal partnership of the Sanchez Spouses Ruling: In the case at bar, the husband of the judgment debtor cannot be deemed a stranger to the case prosecuted and adjudged against his wife. In any case, whether by intervention in the court issuing the writ, or by separate action, it is unavailing for either Esther Sanchez or her husband, Daniel, to seek preclusion of the enforcement of the writ of possession against their conjugal assets. For it being established, as aforementioned, that Esther had engaged in business with her husbands consent, and the income derived there from had been expended, in part at least, for the support of her family, the liability of the conjugal assets to respond for the wifes obligations in the premises cannot be disputed.

608

University of the Cordilleras College of Law First Year C S.Y. 2013 - 2014 Stasa, Inc. vs Court of Appeals 182 SCRA 879 G.R. No. 79385 February 28, 1990 Full Case STASA INCORPORATED, petitioner, vs. HON. COURT OF APPEALS AND MARIA LOURDES R. LICUANAN, respondents. PARAS, J.: In this petition for review on certiorari, petitioner corporation seeks the reversal of the decision dated April 14,1987 of respondent Court of Appeals * which allowed private respondent's petition for certiorari and prohibition and reversed the Orders of September 16, 1985 and March 7, 1986 of the Regional Trial Court of Manila, Branch 42. Petitioner filed the within petition on August 18, 1987. On October 6, 1987, private respondent filed her Manifestation submitting the Entry of Judgment of respondent Court of Appeals which certifies that the assailed decision of April 14, 1987 became final and executory on August 8, 1987. The facts as found by the Court of Appeals on the basis of the records and pleadings appear to be: On February 19, 1975, a complaint for ejectment, docketed as Civil Case No. 0239933, was filed by plaintiff Delfin San Jose, against defendant Mariano Aquino, alleging nonpayment of rentals and conversion of the premises, located at No. 2684 Down, New Panaderos, Sta. Ana, Manila, from residential to commercial purposes (page 37, rollo). The City Court of Manila, Branch 13 in said case, rendered a decision, dated June 20, 1982, in favor of plaintiff and against defendant, ordering the latter and all persons claiming under him to vacate the subject leased premises and to pay plaintiff rentals for the occupation of the same. Subsequent to the filing of Civil Case No. 0239933 plaintiff Delfin San Jose assigned the leased premises to STASA, Inc. The said leased premises were later transferred to and acquired by herein respondent STASA, INC. from Delfin San Jose. On January 26, 1981, plaintiff STASA, INC. (a family corporation, represented by Delfin San Jose, as President and General Manager), filed another ejectment suit, docketed as Civil Case No. 061208, over the same apartment house located at No. 2684 Down, New Panaderos, Sta. Ana, Manila, against defendant Mariano Aquino, but this time, joining therein as party-defendants Ricardo Licuanan, Jr. and one 'Fortuna.' The City Court of Manila, Branch 2, rendered a judgment, dated January 6, 1983, in favor of plaintiff STASA, INC. and against defendants Ricardo Licuanan and 'Fortuna,' excluding defendant Mariano Aquino who had already vacated the premises, and all persons claiming right under said defendants, ordering them to vacate the said premises and to pay the arrears in rentals and reasonable compensation for the use of the property. Thereafter, on October 12, 1983, plaintiff STASA, INC. filed a complaint for a sum of money with the Regional Trial Court of Manila, Branch 42, docketed as Civil Case No. 83-20734 against defendant Mariano Aquino and defendant Maria Lourdes Licuanan. An amended answer, dated September 11, 1984 was filed by defendant Mariano Aquino and defendant Maria Lourdes Licuanan, contending that the complaint should be dismissed upon the ground of res judicata in view of the decisions of the RTC-Manila in the ejectment cases, Civil Cases Nos. 0239933 and 061208. On February 20, 1985, the parties in civil case for sum of money submitted a Joint Stipulation of Facts and Statement of Issues. Pertinent provisions thereof are as follows: l. That Delfin M. San Jose, Sr. was the previous owner and lessor of the premises known as 2634-Down New Panaderos St., Sta. Ana, Manila; 2. That title thereto was later transferred to and acquired by Stasa, Inc., plaintiff in the instant case; 3. That plaintiff Stasa, Inc. is a family corporation and Delfin San Jose, Sr. is the President, General Manager and Chairman of the Board of Directors, and his children are the Board Members; xxx xxx xxx

609

University of the Cordilleras College of Law First Year C S.Y. 2013 - 2014 15. That defendant Maria Lourdes Licuanan was never made defendant and neither was she impleaded in Civil Case Nos. 239933 and 061208-IV, but she was the actual occupant of the said premises; 16. That in the two aforementioned cases, defendant Maria Licuanan testified in open court as the wife and/or common-law-wife of defendant Mariano Aquino; 17. That on 12 October 1983, the present action was filed with this Honorable Court, again by plaintiff Stasa, Incorporated, based on the same lease contract over the same premises, 2684 Down New Panaderos St., Sta. Ana, Manila, for the collection of the same unpaid rentals, twice litigated before the defunct City Court of Manila and both won by herein plaintiff but which had remained unexecuted up to date hereof. On August 28, 1985, plaintiff STASA, INC. filed a Manifestation with the RTC-Manila praying that the execution or satisfaction of the decisions in Civil Cases Nos. 0239933 and 061208 against the defendant Maria Lourdes Licuanan, the wife of defendant Mariano Aquino, be allowed. On September 12, 1985, defendant Maria Lourdes Licuanan filed a motion to dismiss the complaint contending that the instant case is in fact an action to enforce collection of rentals per decision of the City Court of Manila, Branch 13, for ejectment, in Civil Case No. 0239933; that the instant action for execution should have been filed before the City Court of Manila and not before the RTC; that the decision in Civil Case No. 0239933 cannot be enforced in this complaint for collection of sum of money (Civil Case No. 83-20734) against defendant Licuanan, not being a party in Civil Case No. 0239933; that Civil Case No. 83-20734 is barred by prior judgment under Sec.1 (f) Rule 13 of the Rules of Court. On September 13, 1985, plaintiff STASA, INC. opposed the motion to dismiss, alleging that the questioned decisions of the defunct City Court of Manila in Civil Case No. 0239923 and 061208 are ejectment cases, separate and distinct from the present actions for collection of sum of money. On September 16, 1985, the RTC-Manila issued an order denying the motion to dismiss for lack of merit, it appearing that the same is not indubitable. On December 18, 1985, defendant Maria Lourdes Licuanan filed a motion for reconsideration of the order dated September 16, 1985 on the following grounds; lack of jurisdiction of the RTC over the nature of the case; lack of cause of action and res judicata. On January 21, 1986, plaintiff STASA, INC. filed its opposition to the motion for reconsideration, and, in the order of March 7, 1986, the RTC-Manila denied the motion for reconsideration, for lack of merit. (pp. 12-14, Rollo) Private respondent (the petitioner) brought this case before respondent appellate court on a petition for certiorari and prohibition as the aforecited Order of September 16, 1985 which denied her motion to dismiss the complaint for collection of sum of money for lack of merit and the Order of March 7, 1986 which denied the motion for reconsideration. Respondent Court of Appeals handed down the questioned decision granting the petition and reversing and setting aside the aforecited orders of the trial court. Petitioner thus resorted to this petition. Petitioner corporation anchors this petition on these averments: 1. The respondent Court of Appeals erred in reversing the orders of the trial court and in dismissing Civil Case No. 83-20734. Said civil action is separate from the two previous cases and private respondent was not impleaded as a party defendant in these cases because then defendant Mariano Aquino and herein private respondent misrepresented themselves before and during the occupancy of subject premises that they were legally married and for which reason, petitioner filed the ejectment cases against Aquino, the alleged head of the family, for arrears in rentals and for the illegal conversion of the residential apartment into a commercial one. It was only during the hearing of the two ejectment cases when private respondent revealed that they were merely common law husband and wife and, that when a decision was rendered in favor of petitioner and when a writ of execution was issued, the same could not be enforced against the alleged couple. Neither could the writ be enforced against their properties because they had been conveniently transferred in the name of private respondent. 2. Respondent appellate court also erred in concluding that Civil Case No. 83-20734 (for collection of sum of money) is barred by the prior judgment in Civil Cases Nos. 0239933 and 061208 for ejectment.

610

University of the Cordilleras College of Law First Year C S.Y. 2013 - 2014 Petitioner also alleges that respondent court absolved private respondent from the payment of rental arrears alleged in the complaint for ejectment despite the fact that she was the lessee-occupant of the subject apartment, the administrator of Trading housed in the said apartment and that she was the only one who testified in the cases for ejectment. Private respondent, on the other hand, contends that in the two ejectment cases, she was neither named a party defendant nor held liable for unpaid rentals; that not having been impleaded in the two ejectment cases she cannot be held liable in the third case for collection of rentals, and that under the principle of res judicata, issues and matters already raised, discussed and passed upon in the ejectment cases cannot again be raised and litigated in the third civil case for collection of money. The sole issue for the Court's resolution is: Whether or not the judgment in the previous two ejectment cases would bar the third case of the collection of sum of money. The answer is definitely and clearly in the negative. For the principle of res judicata to apply in any given case, four requisites must be fully satisfied as enunciated in Arguson v. Miclat, 133 SCRA 678, and in other cases and these are: (1) the presence of a final former judgment; (2) the former judgment was rendered by a court having jurisdiction over the subject matter and the parties; (3) the former judgment is a judgment on the merits; and (4) there is, between the first and the second action identity of parties, of subject matter, and of cause of action. These four elements must be complied with; otherwise, res judicata would not be sustained. The established facts clearly reveal that there has never been any identity of parties between the two ejectment cases and the case for collection of money. Private respondent herself has asserted that she was never impleaded in the ejectment cases nor was she held responsible for the unpaid rentals. It should be noted that in the "Stipulation of Facts and Statement of Issues" submitted by both parties in the civil case for collection of sum of money, private respondent declared that she "was never made defendant and neither was she impleaded in Civil Case Nos. 0239933 and 061208-IV, but she was the actual occupant of the said premises." (No. 15 of Stipulation of Facts) It cannot even be assumed that she was impliedly impleaded as defendants in the ejectment cases because she and Mariano Aquino revealed their misrepresentation as lawful husband and wife. There never was a conjugal partnership against which the writ of execution could be enforced. Note that in the case of G. Tractors, Inc. v. Court of Appeals, 57402, 135 SCRA 192, this Court ruled that there is no need to include the name of the wife where husband is sued in order to bind the conjugal partnership. Conversely speaking, therefore, there is a need to include the name of the common-law-wife, as in this case, in order to bind her property or make her answerable for any liability. It should also be noted that in the case of Martinez v. Court of Appeals, 139 SCRA 558, this Court declared that the fact alone that the parties in both actions may be the same persons will not spell identity of parties, which presupposes that they are adversary-parties in both first and second cases litigating in the same capacity.(Emphasis supplied). Evidently and indubitably, therefore, there is no identity of parties between the ejectment cases and the case for a sum of money filed against private respondent. At this juncture, this Court cannot pass judgment in this case without making an observation on the manner with which private respondent and her common-law husband pursued the previous cases. It appears that private respondent and her live-in partner indulged in an orchestrated and manipulative handling of the ejectment cases considering that initially, they posed as a legally married coupled but during the litigation, sensing perhaps that they would lose in the cases, suddenly revealed that they were not legally married but were merely living together as husband and wife. They obviously planned and made use of such a scheme in order to avoid the adjudged liability resulting from losing in the cases. Their manipulation of the two cases is even confirmed by the consequent transfer of the properties in the name of private respondent against whom the writ of execution could never be enforced since she was never a party defendant. Then suddenly conveniently she now takes refuge in the principle of "identity of parties", a posture so inconsistent with her attempt to hide her true and real status when it served her deceitful intention. WHEREFORE, the decision of the Court of Appeals is hereby REVERSED and SET ASIDE and the Orders dated September 16, 1985 and March 7, 1986 of Branch 42, Regional Trial Court of Manila are hereby REINSTATED. Let this case be remanded to the trial court for the implementation of the orders. SO ORDERED.

611

University of the Cordilleras College of Law First Year C S.Y. 2013 - 2014 Case Digest Stasa, Inc. vs Court of Appeals G.R. No. 79385 Decided on: February 28, 1990 Ponente: J. Paras Facts: Mariano Aquino and Maria Lourdes Licuanan were living in an apartment located in Sta. Ana, Manila. The owner of the building, Delfin San Jose, subsequently sold the property to Stasa, Inc., a family corporation represented by Delfin San Jose. The respondents did not pay the lease for some months. Due to this, Stasa, Inc filed a complaint to compel the respondents to pay their dues. During the trial, Aquino and Licuanan were presumed to be married. They lost their case in the lower court. They appealed the decision to the Court of Appeals, and it ruled in their favor. Subsequently, the case was brought to the Supreme Court. During the hearing, it was revealed by Licuanan that she and Aquino are merely common law husband and wife. Issue: WoN the Court of Appeals erred in their decision Ruling: It appears that private respondent and her live-in partner indulged in an orchestrated and manipulative handling of the ejectment cases considering that initially, they posed as a legally married coupled but during the litigation, sensing perhaps that they would lose in the cases, suddenly revealed that they were not legally married but were merely living together as husband and wife. They obviously planned and made use of such a scheme in order to avoid the adjudged liability resulting from losing in the cases. Their manipulation of the two cases is even confirmed by the consequent transfer of the properties in the name of private respondent against whom the writ of execution could never be enforced since she was never a party defendant. Then suddenly conveniently she now takes refuge in the principle of "identity of parties", a posture so inconsistent with her attempt to hide her true and real status when it served her deceitful intention. WHEREFORE, the decision of the Court of Appeals is hereby REVERSED and SET ASIDE and the Orders dated September 16, 1985 and March 7, 1986 of Branch 42, Regional Trial Court of Manila are hereby REINSTATED. Let this case be remanded to the trial court for the implementation of the orders.

612

University of the Cordilleras College of Law First Year C S.Y. 2013 - 2014 Villanueva vs IAC 192 SCRA 21 G.R. No. 74577 : December 4, 1990 Full Case CONSOLACION VILLANUEVA, Petitioner, vs. THE INTERMEDIATE APPELLATE COURT, JESUS BERNAS and REMEDIOS Q. BERNAS, Respondents. NARVASA, J.: The spouses Graciano Aranas and Nicolasa Bunsa were the owners in fee simple of a parcel of land identified as Lot 13, their ownership being evidenced by Original Certificate of Title No. 0-3239 issued by the Register of Deeds of Capiz on June 19, 1924. After they died, their surviving children, Modesto Aranas and Federico Aranas, adjudicated the land to themselves under a deed of extrajudicial partition executed on May 2, 1952. The southern portion, described as Lot 13-C, was thereby assigned to Modesto; the northern, to Federico. On March 21, 1953, Modesto Aranas obtained a Torrens title in his name from the Capiz Registry of Property, numbered T-1346. He died on April 20, 1973, at the age of 81 years. His wife, Victoria Comorro, predeceased him dying at age 70 on July 16, 1971. They had no children. Now, it appears that Modesto was survived by two (2) illegitimate children named Dorothea Aranas Ado and Teodoro C. Aranas. These two borrowed P18,000.00 from Jesus Bernas. As security therefor they mortgaged to Bernas their father's property, Lot 13-C. In the "Loan Agreement with Real Estate Mortgage" executed between them and Bernas on October 30, 1975, they described themselves as the absolute co-owners of Lot 13-C. A relative, Raymundo Aranas, signed the agreement as a witness. Dorothea and Teodoro failed to pay their loan. As a result, Bernas caused the extrajudicial foreclosure of the mortgage over Lot 13-C on June 29, 1977 and acquired the land at the auction sale as the highest bidder. After the foreclosure sale, Dorothea and Teodoro executed a deed of Extrajudicial Partition dated June 21, 1978, in which they adjudicated the same Lot 13-C unto themselves in equal shares proindiviso.: On October 25, 1978 Bernas consolidated his ownership over Lot 13-C, the mortgagors having failed to redeem the same within the reglementary period, and had the latter's title (No. T-1346 in the name of Modesto Aranas) cancelled and another issued in his name, TCT No. T-15121. About a month later, or on November 24, 1978, Consolacion Villanueva and Raymundo Aranas who, as aforestated, was an instrumental witness in the deed of mortgage executed by Dorothea and Teodoro Aranas on October 30, 1975 filed a complaint with the Regional Trial Court at Roxas City against Jesus Bernas and his spouse, Remedios Bernas. The case was docketed as Civil Case No. V-4188, and assigned to Branch 14. In their complaint, the plaintiffs prayed that the latter's title over Lot 13-C, TCT No. T-15121, be cancelled and they be declared co-owners of the land. They grounded their cause of action upon their alleged discovery on or about November 20, 1978 of two (2) wills, one executed on February 11, 1958 by Modesto Aranas, and the other, executed on October 29, 1957 by his wife, Victoria Comorro. Victoria Comorro's will allegedly bequeathed to Consolacion and Raymundo, and to Dorothea and Teodoro Aranas, in equal shares pro indiviso, all of said Victoria Comorro's "interests, rights and properties, real and personal . . . as her net share from (the) conjugal partnership property with her husband, Modesto Aranas . . ." Modesto Aranas' will, on the other hand, bequeathed to Dorothea and Teodoro Aranas (his illegitimate children) all his interests in his conjugal partnership with Victoria "as well as his own capital property brought by him to (his) marriage with his said wife." At the pre-trial, the parties stipulated on certain facts, including the following: 1) that the property in question was registered before the mortgage in the name of the late Modesto Aranas, married to Victoria Comorro, (covered by) TCT No. 1346, issued on March 21, 1953; 2) that the wills above described were probated only after the filing of the case (No. V-4188); 613

University of the Cordilleras College of Law First Year C S.Y. 2013 - 2014 3) that Consolacion Villanueva and Raymundo Aranas are not children of either Modesto Aranas or Victoria Comorro; 4) that the lot in question is not expressly mentioned in the will; and 5) that TCT No. 15121 exists, and was issued in favor of defendant spouses Jesus Bernas and Remedios Bernas.: Trial ensued after which judgment was rendered adversely to the plaintiffs, Consolacion Villanueva and Raymundo Aranas. The dispositive part of the judgment reads as follows: WHEREFORE, IN VIEW OF THE FOREGOING, judgment is hereby rendered in favor of the defendants and against the plaintiffs as follows: The plaintiffs' complaint is hereby dismissed and ordering the plaintiffs, jointly and severally, to pay the defendants the following: 1) THREE THOUSAND FIVE HUNDRED PESOS (P3,500.00) as attorney's fees; 2) FIVE HUNDRED PESOS (P500.00) as actual damages; 3) TEN THOUSAND PESOS (P10,000.00) as moral damages; 4) Declaring the defendants spouses Jesus Bernas and Remedios O. Bernas as legal owners of Lot No. 13-C and including all the improvements thereon; 5) Declaring the loan agreement with real estate mortgage (Exh. '2') entered into by Dorothea Aranas Ado married to Reynaldo F. Ado and Teodoro C. Aranas and Jesus Bernas married to Remedios O. Bernas, over the lot in question executed on October 30, 1975 before Notary Public Roland D. Abalajon and the corresponding Certificate of Title No. T-15121 registered in the name of Jesus Bernas (defendants spouses) as having been executed and issued in accordance with law, are declared legal and valid; 6) For failure to prove all other counter-claim and damages, the same are hereby dismissed. 7) To pay costs of this suit. SO ORDERED." The plaintiffs appealed to the Intermediate Appellate Court, where they succeeded only in having the award of actual and moral damages deleted, the judgment of the Regional Trial Court having been otherwise affirmed in toto. From this judgment of the Appellate Court, Consolacion Villanueva appealed to this Court. Her coplaintiff, Raymundo Aranas, did not. The only question is, what right was acquired by Consolacion Villanueva over Lot 13-C and the improvements thereon standing by virtue of Victoria Camorro's last will and testament giving to her all of said Victoria's "interests, rights and properties, real and personal . . . as her net share from (the) conjugal partnership property with her husband, Modesto Aranas . . ." She is admittedly, not named an heiress in Modesto Aranas' will.: Certain it is that the land itself, Lot 13-C, was not "conjugal partnership property" of Victoria Comorro and her husband, Modesto Aranas. It was the latter's exclusive, private property, which he had inherited from his parents Graciano Aranas and Nicolasa Bunsa, the original owners of the property registered solely in his name, under TCT T-1346. Whether Modesto succeeded to the property prior or subsequent to his marriage to Victoria Comorro the record being unfortunately none too clear on the point is inconsequential. The property should be regarded as his own exclusively, as a matter of law. This is what Article 148 of the Civil Code clearly decrees: that to be considered as "the exclusive property of each spouse" is inter alia, "that which is brought to the marriage as his or her own," or "that which each acquires, during the marriage, by lucrative title." Thus, even if it be assumed that Modesto's acquisition

614

University of the Cordilleras College of Law First Year C S.Y. 2013 - 2014 by succession of Lot 13-C took place during his marriage to Victoria Comorro, the lot would nonetheless be his "exclusive property" because acquired by him, "during the marriage, by lucrative title." Moreover, Victoria Comorro died on July 16, 1971, about two (2) years ahead of her husband, Modesto Aranas, exclusive owner of Lot 13-C, who passed away on April 20, 1973. Victoria never therefore inherited any part of Lot 13-C and hence, had nothing of Lot 13-C to bequeath by will or otherwise to Consolacion Villanueva or anybody else. It would seem, however, that there are improvements standing on Lot 13-C, and it is to these improvements that Consolacion Villanueva's claims are directed. The question then is, whether or not the improvements are conjugal property, so that Victoria Comorro may be said to have acquired a right over them by succession, as voluntary heir of Victoria Comorro. The Civil Code says that improvements, "whether for utility or adornment, made on the separate property of the spouses through advancements from the partnership or through the industry of either the husband or the wife, belong to the conjugal partnership," and buildings "constructed, at the expense of the partnership, during the marriage on land belonging to one of the spouses, also pertain to the partnership, but the value of the land shall be reimbursed to the spouse who owns the same." Proof, therefore, is needful of the time of the making or construction of the improvements and the source of the funds used therefor, in order to determine the character of the improvements as belonging to the conjugal partnership or to one spouse separately. No such proof was presented or proferred by Consolacion Villanueva or anyone else. What is certain is that the land on which the improvements stand was the exclusive property of Modesto Aranas and that where, as here, property is registered in the name of one spouse only and there is no showing of when precisely the property was acquired, the presumption is that it belongs exclusively to said spouse. It is not therefore possible to declare the improvements to be conjugal in character. Yet another consideration precludes relief to Consolacion Villanueva and that is, that when Lot 13-C was mortgaged to Jesus Bernas, the title was free of any lien, encumbrance or adverse claim presented by or for Consolacion Villanueva or anybody else, and that when Bernas subsequently consolidated his ownership over Lot 13-C and obtained title in his name, the Registry of Deeds contained no record of any lien, encumbrance or adverse claim affecting the property. Furthermore, Bernas' mode of acquisition of ownership over the property, i.e., by a mortgage sale, appears in all respects to be regular, untainted by any defect whatsoever. Bernas must therefore be deemed to have acquired indefeasible and clear title to Lot 13-C which cannot be defeated or negated by claims subsequently arising and of which he had no knowledge or means of knowing prior to their assertion and ventilation.: Finally, it bears stressing that the conclusion of the Intermediate Appellate Court that the evidence establishes that the property in question was the exclusive property of one spouse, not conjugal, is a factual one which, absent any satisfactory showing of palpable error or grave abuse of discretion on the part of the Appellate Court in reaching it, is not reviewable by this Court. WHEREFORE, the judgment of the Intermediate Appellate Court subject of this appeal, being in accord with the evidence and applicable law and jurisprudence, is AFFIRMED, with costs against the petitioner. SO ORDERED.

615

University of the Cordilleras College of Law First Year C S.Y. 2013 - 2014 Case Digest Villanueva vs Intermediate Appellate Court G.R. No. 74577 Decided on: December 4, 1990 Ponente: J. Narvasa Facts: Dorothea and Teodoro Aranas borrowed P18,000 from private respondent Jesus Bernas, mortgaging as collateral their father Modestos property, Lot 13-C. In the loan agreement between them on Oct. 30, 1975, the Aranas described themselves as the absolute co-owners. When Dorothea and Teodoro failed to pay the loan, Bernas caused the extrajudicial foreclosure of the mortgage in 1977 and acquired the land as the highest bidder. After the foreclosure sale, the Aranases executed a deed of extrajudicial partition in 1978, in which they adjudicated the same lot 13-C unto themselves in equal share pro-indiviso. Bernas then consolidated his ownership over the lot when the mortgagors failed to redeem it withn the reglementary period, and had the title in the name of Modesto cancelled and another TCT issued in his name. On November 24, 1978, herein petitioner Consolacion Villanueva and Raymundo Aranas (witness to the deed of mortgage in 1975) filed a complaint with the RTC of Roxas ity against respondents spouses Jesus and Remedios Bernas, for the cancellation of the TCT under the name of the Bernases, and they (Villanueva and Aranas) be declared co-owners of the land. Petitioner alleged that spouses Modesto and Victoria Aranas in 1987 and 1958 executed two separate wills: first bequeathing to Consolacion and Raymundo and to Dorothea and Teodoro, in equal shares pro diviso, all of said Victorias shares from the conjugal partnership property; and second Modestos interests in his conjugal partnership with Victoria as well as his separate properties bequeathed to Dorothea and Teodoro (his illegitimate children). The trial court dismissed the complaint, declaring herein respondents as the legal owners of the disputed property. Upon appeal, the Intermediate Appellate Court (IAC) affirmed the lower courts decision in toto. Issue: Whether or not Villanueva had a right over Lot 13-C and the improvements thereon made by Victoria rendered the lot as conjugal property. Ruling: Lot 13-C was not a conjugal partnership property of Victoria and Modesto. It was Modestos exclusive, private property, which he inherited from his parents. Moreover, since Victoria died ahead of Modesto, Victoria did not inherit said lot from him and therefore had nothing of Lot 13C to bequeath by will of otherwise to Consolacion. Article 158 of the Civil Code says that improvements, whether for utility or adornment made on the separate property of the spouses through advancements from the partnership or through the industry of either spouse belong to the conjugal partnership, and buildings constructed at the expense of the partnership during the marriage on land belonging to one of the spouses also pertain to the partnership, but the value of the land shall be reimbursed to the spouse who owns the same. Proof, therefore, is needful of the time of the making or construction of the improvements and the source of the funds used therefor in order to determine the character of the improvements as belonging to the conjugal partnership or to one spouse separately. No such proof was presented or proffered by Villanueva. What is certain is that the land on which the improvements stand was the exclusive property of Modesto and that where the property is registered in the name of one spouse only and there is no showing of when precisely the property was acquired, the presumption is that is belongs exclusively to said spouse [PNB vs. CA, 153 SCRA 435, 1987]. It is not therefore possible to declare the improvements to be conjugal in character. Furthermore, Bernas mode of acquisition of ownership over the property appears in all respect to be regular, untainted by any defect whatsoever. Bernas must therefore be deemed to have 616

University of the Cordilleras College of Law First Year C S.Y. 2013 - 2014 acquired indefeasible and clear title to Lot 13-C which cannot be defeated or negated by claims subsequently arising and of which he had no knowledge or means of knowing prior to their assertion and ventilation.

617

University of the Cordilleras College of Law First Year C S.Y. 2013 - 2014 Belcodero vs Court of Appeals 227 SCRA 303 G.R. No. 89667 October 20, 1993 Full Case JOSEPHINE B. BELCODERO, petitioner, vs. THE HONORABLE COURT OF APPEALS, et al., respondents. VITUG, J.: This case involves the question of ownership over a piece of land acquired by a husband while living with a paramour and after having deserted his lawful wife and children. The property had been bought by the husband on installment basis prior to the effectivity of the Civil Code of 1950 but the final deed, as well as the questioned conveyance by him to his common law spouse, has ensued during the latter Code's regime. Now, of course, we have to likewise take note of the new Family Code which took effect on 03 August 1988. Let us begin by paraphrasing the factual findings of the appellate court below. The husband, Alayo D. Bosing, married Juliana Oday on 27 July 1927, with whom he had three children, namely, Flora, Teresita, and Gaido. In 1946, he left the conjugal home, and he forthwith started to live instead with Josefa Rivera with whom he later begot one child, named Josephine Bosing, now Josephine Balcobero. On 23 August 1949, Alayo purchased a parcel of land on installment basis from the Magdalena Estate, Inc. In the deed, he indicated his civil status as, "married to Josefa R. Bosing," the common-law wife. In a letter, dated 06 October 1959, which he addressed to Magdalena Estate, Inc., he authorized the latter to transfer the lot in the name of his "wife Josefa R. Bosing." The final deed of sale was executed by Magdalena Estate, Inc., on 24 October 1959. A few days later, or on 09 November 1959, Transfer Certificate of Title No. 48790 was issued in the name of "Josefa R. Bosing, . . . married to Alayo Bosing, . . ." On 06 June 1958, Alayo married Josefa even while his prior marriage with Juliana was still subsisting. Alayo died on 11 march 1967. About three years later, or on 17 September 1970, Josefa and Josephine executed a document of extrajudicial partition and sale of the lot in question, which was there described as "conjugal property" of Josefa and deceased Alayo. In this deed, Josefa's supposed one-half (1/2) interest as surviving spouse of Alayo, as well as her one-fourth (1/4) interest as heir, was conveyed to Josephine for a P10,000.00 consideration, thereby completing for herself, along with her one-fourth (1/4) interest as the surviving child of Alayo, a full "ownership" of the property. The notice of extrajudicial partition was published on 04, 05 and 06 November 1970 in the Evening Post; the inheritance and estate taxes were paid; and a new Transfer Certificate of Title No. 198840 was issued on 06 June 1974 in the name of Josephine. On 30 October 1980, Juliana (deceased Alayo's real widow) and her three legitimate children filed with the court a quo an action for reconveyance of the property. On the basis of he above facts, the trial court ruled in favor of the plaintiffs, and it ordered that . . . Josephine Bosing executed a deed of reconveyance of the property in question to the legal heirs of the deceased Alayo D. Bosing, and that both defendants pay, jointly and severally, actual damages by way of attorney's fees and expenses in litigation, TEN THOUSAND (P10,000.00) PESOS as moral damages, pus TEN THOUSAND (P10,000.00) PESOS exemplary damages to prevent future frauds. The defendants went to the Court of Appeals which affirmed the trial court's order for reconveyance but reversed the decision on the award for damages, thus WHEREFORE, the judgment appealed from is hereby AFFIRMED insofar as defendant Josephine Bosing is ordered to execute a deed of reconveyance of the property granting the same to the legal heirs of the deceased Alayo D. Bosing, and REVERSED insofar as it awards actual, moral and exemplary damages. 1 Hence, the instant petition for review 2 submitting that 1. THE RESPONDENT COURT ERRED IN NOT HOLDING THAT THE ACTION FOR RECONVEYANCE HAD LONG PRESCRIBED. 2. THE RESPONDENT COURT ERRED IN FINDING THAT, THE ACTION FOR RECONVEYANCE IS BASED UPON AN IMPLIED OR CONSTRUCTIVE TRUST.

618

University of the Cordilleras College of Law First Year C S.Y. 2013 - 2014 3. THE RESPONDENT COURT ERRED IN NOT HOLDING THAT, THE PROPERTY IN QUESTION BELONGS EXCLUSIVELY TO THE PETITIONERS. 4. THE RESPONDENT COURT ERRED IN NOT GRANTING PETITIONER'S MOTION FOR NEW TRIAL BASED ON NEWLY DISCOVERED EVIDENCE, AND LIKEWISE ERRED IN HOLDING THAT EVEN IF A NEW TRIAL IS GRANTED THE SAME WOULD NOT SERVE A USEFUL PURPOSE. We rule for affirmance. The first three issues are interrelated, and the same will thus be jointly discussed. Whether the property in question was acquired by Alayo in 1949 when an agreement for its purchase on installment basis was entered into between him and Magdalena Estate, Inc., or in 1959 when a deed of sale was finally executed by Magdalena Estate, Inc., the legal results would be the same. The property remained as belonging to the conjugal partnership of Alayo and his legitimate wife Juliana. Under both the new Civil Code (Article 160) and the old Civil Code (Article 1407), "all property of the marriage is presumed to belong to the conjugal partnership, unless it be proved that it pertains exclusively to the husband or to the wife." This presumption has not been convincingly rebutted. It cannot be seriously contended that, simply because the property was titled in the name of Josefa at Alayo's request, she should thereby be deemed to be its owner. The property unquestionably was acquired by Alayo. Alayo's letter, dated 06 October 1959, to Magdalena Estate, Inc., merely authorized the latter to have title to the property transferred to her name. More importantly, she implicitly recognized Alayo's ownership when, three years after the death of Alayo, she and Josephine executed the deed of extrajudicial partition and sale in which she asserted a one-half (1/2) interest in the property in what may be described as her share in the "conjugal partnership" with Alayo, plus another one-fourth (1/4) interest as "surviving widow," the last one-fourth (1/4) going to Josephine as the issue of the deceased. Observe that the above adjudication would have exactly conformed with a partition in intestacy had they been the sole and legitimate heirs of the decedent. The appellate court below, given the above circumstances, certainly cannot be said to have been without valid basis in concluding that the property really belonged to the lawful conjugal partnership between Alayo and his true spouse Juliana. As regards the property relation between common-law spouses, Article 144 of the Civil Code merely codified the law established through judicial precedents under the old code (Margaret Maxey vs. Court of Appeals, G.R. No. L-45870, 11 May 1984). In both regimes, the co-ownership rule had more than once been repudiated when either or both spouses suffered from an impediment to marry (Jeroniza vs. Jose, 89 SCRA 306). The present provisions under Article 147 and Article 148 of the Family Code did not much deviate from the old rules; in any case, its provisions cannot apply to this case without interdicting prior vested rights (Article 256, Family Code). It was at the time that 'the adjudication of ownership was made following Alayo's demise (not when Alayo merely allowed the property to be titled in Josefa's name which clearly was not intended to be adversarial to Alayo's interest), that a constructive trust was deemed to have been created by operation of law under the provisions of Article 1456 of the Civil Code. Article 1456. If the property is acquired through mistake or fraud, the person obtaining it is, by force of law, considered a trustee of an implied trust for the benefit of the person from whom the property comes. The applicable prescriptive period for an action seeking a reconveyance of the property by the beneficiaries thereof is ten (10) years (Article 1144, Civil Code). Ordinarily, that period starts from the establishment of the implied trust being the day when the cause of action would be considered to have accrued (Article 1150, Civil Code). Unfortunately for Josefa and Josephine, however, the property involved in this case is a realty titled under the Torrens System. The prescriptive period is thus to be counted from the time the transaction affecting the property is registered with the corresponding issuance of a new certificate of title. 3 Between the time Transfer of Certificate of Title No. 198840 was issued on 06 June 1974, and the filing of the action for the reconveyance of the property with the court a quo on 30 October 1980, barely a period of six (6) years and four (4) months had elapsed. The case has accordingly been initiated seasonably. The four-year prescriptive period, mentioned in passing by the petitioners, would have had some value and relevance had the private respondents or their predecessor in interest been parties to the extrajudicial partition and sale. In that event, the latter's action could only then be predicated on a vitiation of consent 4 where the applicable statutory limitation would be four years. 5

619

University of the Cordilleras College of Law First Year C S.Y. 2013 - 2014 The last issue raises the supposed error in the rejection of a new trial on the basis of newly discovered evidence. We concur with the resolution of the appellate court below (on appellants' [petitioners herein] motion for reconsideration thereat), thus Appellants' prayer for a new trial based upon what they claim is newly discovered evidence deserves scant consideration. Appellant proposes to prove (1) that Josefa Bosing sold certain property for P8,000.00 in 1948 and was therefore in a financial position to make the payments to Magdalena Estate Inc. and (2) that appellee Juliana Bosing got married in 1961 to one Burayos Ballit, and thus, "forfeited" her right to the conjugal partnership. The first ground is not meritorious. It is not newly discovered evidence. As described in appellants' Motion the documents were "not discovered or considered as necessary evidence during the trial of the case below" by the former counsel; it is therefore more properly considered as forgotten evidence, which the appellant knew or should have known during the trial (Tesoro vs. Court of Appeals, 54 SCRA 296; Republic vs. Vda. de Castelvi, 58 SCRA 336). Moreover, assuming the sale is proven, it does not follow that the proceeds were used to pay the lot in question; the payments were made in installments, not in one lump sum. Neither is the second ground deserving of merit. Assuming that the marriage to Ballit in 1961 is duly proven, and that this provided a cause for legal separation and consequent disqualification of the guilty spouse to succeed to the husband's intestate estate under Article 1002 of the Civil Code, the fact remains that no action for legal separation was brought by the husband during his lifetime and within the period provided by law. It is too late to raise the issue at this time. Accordingly, assuming that the Motion for New Trial complies with the formal requisites for such motion (See Minister of Natural Resources vs. Heirs of Orval Hughes, et al., G.R. No. 62662, prom. November 12, 1987), a question We don't find necessary to decide, a new trial would not serve a useful purpose in altering the result of the questioned decision. WHEREFORE, the decision appealed from in the instant petition for review on certiorari is AFFIRMED. SO ORDERED.

620

University of the Cordilleras College of Law First Year C S.Y. 2013 - 2014 Case Digest Belcodero vs Court of Appeals G.R. No. 89667 Decided on: October 20, 1993 Ponente: J. Vitug Facts:This case involves the question of ownership over a piece of land acquired by a husband while living with a paramour and after having deserted his lawful wife and children. The property had been bought by the husband on installment basis prior to the effectivity of the Civil Code of 1950 but the final deed, as well as the questioned conveyance by him to his common law spouse, has ensued during the latter Codes regime. Now, of course, we have to likewise take note of the new Family Code which took effect on 03 August 1988. The property was acquired by Alayo then transferred title to the name of the second wife. Alayo Bosing died and left the property to his paramour turned second wife (first marriage still subsisting). First wife petitioned. Issue: Won the property in question belonged to the conjugal partnership of Alayo and Juliana Ruling: The property remained as belonging to the conjugal partnership of Alayo and his legitimate wife Juliana. Under both the new Civil Code (Article 160) and the old Civil Code (Article 1407), all property of the marriage is presumed to belong to the conjugal partnership, unless it be proved that it pertains exclusively to the husband or to the wife. This presumption has not been convincingly rebutted. It cannot be seriously contended that, simply because the property was titled in the name of Josefa at Alayos request, she should thereby be deemed to be its owner. The property unquestionably was acquired by Alayo it was just transferred to Josefa.

621

University of the Cordilleras College of Law First Year C S.Y. 2013 - 2014 Ayala Development Corp. vs Court of Appeals 286 SCRA 272 G.R. No. 118305. February 12, 1998 Full Case AYALA INVESTMENT & DEVELOPMENT CORP. and ABELARDO MAGSAJO, petitioners, vs. COURT OF APPEALS and SPOUSES ALFREDO & ENCARNACION CHING, respondents. MARTINEZ, J.: Under Article 161 of the Civil Code, what debts and obligations contracted by the husband alone are considered for the benefit of the conjugal partnership which are chargeable against the co njugal partnership? Is a surety agreement or an accommodation contract entered into by the husband in favor of his employer within the contemplation of the said provision? These are the issues which we will resolve in this petition for review. The petitioner assails the decision dated April 14, 1994 of the respondent Court of Appeals in Spouses Alfredo and Encarnacion Ching vs. Ayala Investment and Development Corporation,et. al., docketed as CA-G.R. CV No. 29632,[1] upholding the decision of the Regional Trial Court of Pasig, Branch 168, which ruled that the conjugal partnership of gains of respondents-spouses Alfredo and Encarnacion Ching is not liable for the payment of the debts secured by respondent-husband Alfredo Ching. A chronology of the essential antecedent facts is necessary for a clear understanding of the case at bar. Philippine Blooming Mills (hereinafter referred to as PBM) obtained a P50,300,000.00 loan from petitioner Ayala Investment and Development Corporation (hereinafter referred to as AIDC). As added security for the credit line extended to PBM, respondent Alfredo Ching, Executive Vice President of PBM, executed security agreements on December 10, 1980 and on March 20, 1981 making himself jointly and severally answerable with PBMs indebtedness to AIDC. PBM failed to pay the loan. Thus, on July 30, 1981, AIDC filed a case for sum of money against PBM and respondent-husband Alfredo Ching with the then Court of First Instance of Rizal (Pasig), Branch VIII, entitled Ayala Investment and Development Corporation vs. Philippine Blooming Mills and Alfredo Ching, docketed as Civil Case No. 42228. After trial, the court rendered judgment ordering PBM and respondent-husband Alfredo Ching to jointly and severally pay AIDC the principal amount of P50,300,000.00 with interests. Pending appeal of the judgment in Civil Case No. 42228, upon motion of AIDC, the lower court issued a writ of execution pending appeal. Upon AIDCs putting up of an P8,000,000.00 bond, a writ of execution dated May 12, 1982 was issued. Thereafter, petitioner Abelardo Magsajo, Sr., Deputy Sheriff of Rizal and appointed sheriff in Civil Case No. 42228, caused the issuance and service upon respondents-spouses of a notice of sheriff sale dated May 20, 1982 on three (3) of their conjugal properties. Petitioner Magsajo then scheduled the auction sale of the properties levied. On June 9, 1982, private respondents filed a case of injunction against petitioners with the then Court of First Instance of Rizal (Pasig), Branch XIII, to enjoin the auction sale alleging that petitioners cannot enforce the judgment against the conjugal partnership levied on the ground that, among others, the subject loan did not redound to the benefit of the said conjugal partnership.[2]Upon application of private respondents, the lower court issued a temporary restraining order to prevent petitioner Magsajo from proceeding with the enforcement of the writ of execution and with the sale of the said properties at public auction. AIDC filed a petition for certiorari before the Court of Appeals,[3] questioning the order of the lower court enjoining the sale. Respondent Court of Appeals issued a Temporary Restraining Order on June 25, 622

University of the Cordilleras College of Law First Year C S.Y. 2013 - 2014 1982, enjoining the lower court[4] from enforcing its Order of June 14, 1982, thus paving the way for the scheduled auction sale of respondents-spouses conjugal properties. On June 25, 1982, the auction sale took place. AIDC being the only bidder, was issued a Certificate of Sale by petitioner Magsajo, which was registered on July 2, 1982. Upon expiration of the redemption period, petitioner sheriff issued the final deed of sale on August 4, 1982 which was registered on August 9, 1983. In the meantime, the respondent court, on August 4, 1982, decided CA-G.R. SP No. 14404, in this manner: WHEREFORE, the petition for certiorari in this case is granted and the challenged order of the respondent Judge dated June 14, 1982 in Civil Case No. 46309 is hereby set aside and nullified. The same petition insofar as it seeks to enjoin the respondent Judge from proceeding with Civil Case No. 46309 is, however, denied. No pronouncement is here made as to costs. x x x x.[5] On September 3, 1983, AIDC filed a motion to dismiss the petition for injunction filed before Branch XIII of the CFI of Rizal (Pasig) on the ground that the same had become moot and academic with the consummation of the sale. Respondents filed their opposition to the motion arguing, among others, that where a third party who claims ownership of the property attached or levied upon, a different legal situation is presented; and that in this case, two (2) of the real properties are actually in the name of Encarnacion Ching, a non-party to Civil Case No. 42228. The lower court denied the motion to dismiss. Hence, trial on the merits proceeded. Private respondents presented several witnesses. On the other hand, petitioners did not present any evidence. On September 18, 1991, the trial court promulgated its decision declaring the sale on execution null and void. Petitioners appealed to the respondent court, which was docketed as CA-G.R. CV No. 29632. On April 14, 1994, the respondent court promulgated the assailed decision, affirming the decision of the regional trial court. It held that: The loan procured from respondent-appellant AIDC was for the advancement and benefit of Philippine Blooming Mills and not for the benefit of the conjugal partnership of petitionersappellees. xxx xxx xxx

As to the applicable law, whether it is Article 161 of the New Civil Code or Article 1211 of the Family Code-suffice it to say that the two provisions are substantially the same. Nevertheless, We agree with the trial court that the Family Code is the applicable law on the matter x x x x x x. Article 121 of the Family Code provides that The conjugal partnership shall be liable for: x x x (2) All debts and obligations contracted during the marriage by the designated Administrator-Spouse for the benefit of the conjugal partnership of gains x x x. The burden of proof that the debt was contracted for the benefit of the conjugal partnership of gains, lies with the creditor-party litigant claiming as such. In the case at bar, respondent-appellant AIDC failed to prove that the debt was contracted by appellee-husband, for the benefit of the conjugal partnership of gains. The dispositive portion of the decision reads: WHEREFORE, in view of all the foregoing, judgment is hereby rendered DISMISSING the appeal. The decision of the Regional Trial Court is AFFIRMED in toto.[6] Petitioner filed a Motion for Reconsideration which was denied by the respondent court in a Resolution dated November 28, 1994.[7] 623

University of the Cordilleras College of Law First Year C S.Y. 2013 - 2014 Hence, this petition for review. Petitioner contends that the respondent court erred in ruling that the conjugal partnership of private respondents is not liable for the obligation by the respondent-husband. Specifically, the errors allegedly committed by the respondent court are as follows: I. RESPONDENT COURT ERRED IN RULING THAT THE OBLIGATION INCURRED BY RESPONDENT HUSBAND DID NOT REDOUND TO THE BENEFIT OF THE CONJUGAL PARTNERSHIP OF THE PRIVATE RESPONDENT. II RESPONDENT COURT ERRED IN RULING THAT THE ACT OF RESPONDENT HUSBAND IN SECURING THE SUBJECT LOAN IS NOT PART OF HIS INDUSTRY, BUSINESS OR CAREER FROM WHICH HE SUPPORTS HIS FAMILY. Petitioners in their appeal point out that there is no need to prove that actual benefit redounded to the benefit of the partnership; all that is necessary, they say, is that the transaction was entered into for the benefit of the conjugal partnership. Thus, petitioners aver that: The wordings of Article 161 of the Civil Code is very clear: for the partnership to be held liable, the husband must have contracted the debt for the benefit of the partnership, thus: Art. 161. The conjugal partnership shall be liable for: 1) all debts and obligations contracted by the husband for the benefit of the conjugal partnership x x x.

There is a difference between the phrases: redounded to the benefit of or benefited from (on the one hand) and for the benefit of (on the other). The former require that actual benefit must have been realized; the latter requires only that the transaction should be one which normally would produce benefit to the partnership, regardless of whether or not actual benefit accrued.[8] We do not agree with petitioners that there is a difference between the terms redounded to the benefit of or benefited from on the one hand; and for the benefit of on the other. They mean one and the same thing. Article 161 (1) of the Civil Code and Article 121 (2) of the Family Code are similarly worded, i.e., both use the term for the benefit of. On the other hand, Article 122 of the Family Code provides that The payment of personal debts by the husband or the wife before or during the marriage shall not be charged to the conjugal partnership except insofar as they redounded to the benefit of the family. As can be seen, the terms are used interchangeably. Petitioners further contend that the ruling of the respondent court runs counter to the pronouncement of this Court in the case of Cobb-Perez vs. Lantin,[9] that the husband as head of the family and as administrator of the conjugal partnership is presumed to have contracted obligations for the benefit of the family or the conjugal partnership. Contrary to the contention of the petitioners, the case of Cobb-Perez is not applicable in the case at bar. This Court has, on several instances, interpreted the term for the benefit of the conjugal partnership. In the cases of Javier vs. Osmea,[10] Abella de Diaz vs. Erlanger & Galinger, Inc.,[11] Cobb-Perez vs. Lantin[12] and G-Tractors, Inc. vs. Court of Appeals,[13] cited by the petitioners, we held that: The debts contracted by the husband during the marriage relation, for and in the exercise of the industry or profession by which he contributes toward the support of his family, are not his personal and private debts, and the products or income from the wifes own property, which, like those of her husbands, are liable for the payment of the marriage expenses, cannot be excepted from the payment of such debts. (Javier) The husband, as the manager of the partnership (Article 1412, Civil Code), has a right to embark the partnership in an ordinary commercial enterprise for gain, and the fact that the wife may not approve of a venture does not make it a private and personal one of the husband. (Abella de Diaz) 624

University of the Cordilleras College of Law First Year C S.Y. 2013 - 2014 Debts contracted by the husband for and in the exercise of the industry or profession by which he contributes to the support of the family, cannot be deemed to be his exclusive and private debts. (Cobb-Perez) x x x if he incurs an indebtedness in the legitimate pursuit of his career or profession or suffers losses in a legitimate business, the conjugal partnership must equally bear the indebtedness and the losses, unless he deliberately acted to the prejudice of his family. (GTractors) However, in the cases of Ansaldo vs. Sheriff of Manila, Fidelity Insurance & Luzon Insurance Co., Liberty Insurance Corporation vs. Banuelos,[15] and Luzon Surety Inc. vs. De Garcia,[16]cited by the respondents, we ruled that:
[14]

The fruits of the paraphernal property which form part of the assets of the conjugal partnership, are subject to the payment of the debts and expenses of the spouses, but not to the payment of the personal obligations (guaranty agreements) of the husband, unless it be proved that such obligations were productive of some benefit to the family. (Ansaldo; parenthetical phrase ours.) When there is no showing that the execution of an indemnity agreement by the husband redounded to the benefit of his family, the undertaking is not a conjugal debt but an obligation personal to him. (Liberty Insurance) In the most categorical language, a conjugal partnership under Article 161 of the new Civil Code is liable only for such debts and obligations contracted by the husband for the benefit of the conjugal partnership. There must be the requisite showing then of some advantage which clearly accrued to the welfare of the spouses. Certainly, to make a conjugal partnership respond for a liability that should appertain to the husband alone is to defeat and frustrate the avowed objective of the new Civil Code to show the utmost concern for the solidarity and well-being of the family as a unit. The husband, therefore, is denied the power to assume unnecessary and unwarranted risks to the financial stability of the conjugal partnership. (Luzon Surety, Inc.) From the foregoing jurisprudential rulings of this Court, we can derive the following conclusions: (A) If the husband himself is the principal obligor in the contract, i.e., he directly received the money and services to be used in or for his own business or his own profession, that contract falls within the term x x x x obligations for the benefit of the conjugal partnership. Here, no actual benefit may be proved. It is enough that the benefit to the family is apparent at the time of the signing of the contract. From the very nature of the contract of loan or services, the family stands to benefit from the loan facility or services to be rendered to the business or profession of the husband. It is immaterial, if in the end, his business or profession fails or does not succeed. Simply stated, where the husband contracts obligations on behalf of the family business, the law presumes, and rightly so, that such obligation will redound to the benefit of the conjugal partnership. (B) On the other hand, if the money or services are given to another person or entity, and the husband acted only as a surety or guarantor, that contract cannot, by itself, alone be categorized as falling within the context of obligations for the benefit of the conjugal partnership. The contract of loan or services is clearly for the benefit of the principal debtor and not for the surety or his family. No presumption can be inferred that, when a husband enters into a contract of surety or accommodation agreement, it is for the benefit of the conjugal partnership. Proof must be presented to establish benefit redounding to the conjugal partnership. Thus, the distinction between the Cobb-Perez case, and we add, that of the three other companion cases, on the one hand, and that of Ansaldo, Liberty Insurance and Luzon Surety, is that in the former, the husband contracted the obligation for his own business; while in the latter, the husband merely acted as a surety for the loan contracted by another for the latters business.

625

University of the Cordilleras College of Law First Year C S.Y. 2013 - 2014 The evidence of petitioner indubitably show that co-respondent Alfredo Ching signed as surety for the P50M loan contracted on behalf of PBM. Petitioner should have adduced evidence to prove that Alfredo Chings acting as surety redounded to the benefit of the conjugal partnership. The reason for this is as lucidly explained by the respondent court: The loan procured from respondent-appellant AIDC was for the advancement and benefit of Philippine Blooming Mills and not for the benefit of the conjugal partnership of petitionersappellees. Philippine Blooming Mills has a personality distinct and separate from the family of petitioners-appellees - this despite the fact that the members of the said family happened to be stockholders of said corporate entity. xxx xxx xxx

x x x. The burden of proof that the debt was contracted for the benefit of the conjugal partnership of gains, lies with the creditor-party litigant claiming as such. In the case at bar, respondent-appellant AIDC failed to prove that the debt was contracted by appellee-husband, for the benefit of the conjugal partnership of gains. What is apparent from the facts of the case is that the judgment debt was contracted by or in the name of the Corporation Philippine Blooming Mills and appellee-husband only signed as surety thereof. The debt is clearly a corporate debt and respondent-appellants right of recourse against appellee-husband as surety is only to the extent of his corporate stockholdings. It does not extend to the conjugal partnership of gains of the family of petitioners-appellees. x x x x x x. [17] Petitioners contend that no actual benefit need accrue to the conjugal partnership. To support this contention, they cite Justice J.B.L. Reyes authoritative opinion in the Luzon Surety Company case: I concur in the result, but would like to make of record that, in my opinion, the words all debts and obligations contracted by the husband for the benefit of the conjugal partnership used in Article 161 of the Civil Code of the Philippines in describing the charges and obligations for which the conjugal partnership is liable do not require that actual profit or benefit must accrue to the conjugal partnership from the husbands transaction; but it suffices that the transaction should be one that normally would produce such benefit for the partnership. This is the ratio behind our ruling in Javier vs. Osmea, 34 Phil. 336, that obligations incurred by the husband in the practice of his profession are collectible from the conjugal partnership. The aforequoted concurring opinion agreed with the majority decision that the conjugal partnership should not be made liable for the surety agreement which was clearly for the benefit of a third party. Such opinion merely registered an exception to what may be construed as a sweeping statement that in all cases actual profit or benefit must accrue to the conjugal partnership. The opinion merely made it clear that no actual benefits to the family need be proved in some cases such as in the Javier case. There, the husband was the principal obligor himself. Thus, said transaction was found to be one that would normally produce x x x benefit for the partnership. In the later case of G-Tractors, Inc., the husband was also the principal obligor - not merely the surety. This latter case, therefore, did not create any precedent. It did not also supersede the Luzon Surety Company case, nor any of the previous accommodation contract cases, where this Court ruled that they were for the benefit of third parties. But it could be argued, as the petitioner suggests, that even in such kind of contract of accommodation, a benefit for the family may also result, when the guarantee is in favor of the husbands employer. In the case at bar, petitioner claims that the benefits the respondent family would reasonably anticipate were the following: (a) The employment of co-respondent Alfredo Ching would be prolonged and he would be entitled to his monthly salary of P20,000.00 for an extended length of time because of the loan he guaranteed;

626

University of the Cordilleras College of Law First Year C S.Y. 2013 - 2014 (b) The shares of stock of the members of his family would appreciate if the PBM could be rehabilitated through the loan obtained; (c) His prestige in the corporation would be enhanced and his career would be boosted should PBM survive because of the loan. However, these are not the benefits contemplated by Article 161 of the Civil Code. The benefits must be one directly resulting from the loan. It cannot merely be a by-product or a spin-off of the loan itself. In all our decisions involving accommodation contracts of the husband,[18] we underscored the requirement that: there must be the requisite showing x x x of some advantage which clearly accrued to the welfare of the spouses or benefits to his family or that such obligations are productive of some benefit to the family. Unfortunately, the petition did not present any proof to show: (a) Whether or not the corporate existence of PBM was prolonged and for how many months or years; and/or (b) Whether or not the PBM was saved by the loan and its shares of stock appreciated, if so, how much and how substantial was the holdings of the Ching family. Such benefits (prospects of longer employment and probable increase in the value of stocks) might have been already apparent or could be anticipated at the time the accommodation agreement was entered into. But would those benefits qualify the transaction as one of the obligations x x x for the benefit of the conjugal partnership? Are indirect and remote probable benefits, the ones referred to in Article 161 of the Civil Code? The Court of Appeals in denying the motion for reconsideration, disposed of these questions in the following manner: No matter how one looks at it, the debt/credit extended by respondents-appellants is purely a corporate debt granted to PBM, with petitioner-appellee-husband merely signing as surety. While such petitioner-appellee-husband, as such surety, is solidarily liable with the principal debtor AIDC, such liability under the Civil Code provisions is specifically restricted by Article 122 (par. 1) of the Family Code, so that debts for which the husband is liable may not be charged against conjugal partnership properties. Article 122 of the Family Code is explicit The payment of personal debts contracted by the husband or the wife before or during the marriage shall not be charged to the conjugal partnership except insofar as they redounded to the benefit of the family. Respondents-appellants insist that the corporate debt in question falls under the exception laid down in said Article 122 (par. one). We do not agree. The loan procured from respondentappellant AIDC was for the sole advancement and benefit of Philippine Blooming Mills and not for the benefit of the conjugal partnership of petitioners-appellees. x x x appellee-husband derives salaries, dividends benefits from Philippine Blooming Mills (the debtor corporation), only because said husband is an employee of said PBM. These salaries and benefits, are not the benefits contemplated by Articles 121 and 122 of the Family Code. The benefits contemplated by the exception in Article 122 (Family Code) is that benefit derived directly from the use of the loan. In the case at bar, the loan is a corporate loan extended to PBM and used by PBM itself, not by petitioner-appellee-husband or his family. The alleged benefit, if any, continuously harped by respondents-appellants, are not only incidental but also speculative.[19] We agree with the respondent court. Indeed, considering the odds involved in guaranteeing a large amount (P50,000,000.00) of loan, the probable prolongation of employment in PBM and increase in value of its stocks, would be too small to qualify the transaction as one for the benefit of the suretys family. Verily, no one could say, with a degree of certainty, that the said contract is even productive of some benefits to the conjugal partnership. We likewise agree with the respondent court (and this view is not contested by the petitioners) that the provisions of the Family Code is applicable in this case. These provisions highlight the underlying

627

University of the Cordilleras College of Law First Year C S.Y. 2013 - 2014 concern of the law for the conservation of the conjugal part nership; for the husbands duty to protect and safeguard, if not augment, not to dissipate it. This is the underlying reason why the Family Code clarifies that the obligations entered into by one of the spouses must be those that redounded to the benefit of the family and that the measure of the partnerships liability is to the extent that the family is benefited.[20] These are all in keeping with the spirit and intent of the other provisions of the Civil Code which prohibits any of the spouses to donate or convey gratuitously any part of the conjugal property.[21] Thus, when co-respondent Alfredo Ching entered into a surety agreement he, from then on, definitely put in peril the conjugal property (in this case, including the family home) and placed it in danger of being taken gratuitously as in cases of donation. In the second assignment of error, the petitioner advances the view that acting as surety is part of the business or profession of the respondent-husband. This theory is new as it is novel. The respondent court correctly observed that: Signing as a surety is certainly not an exercise of an industry or profession, hence the cited cases of Cobb-Perez vs. Lantin; Abella de Diaz vs. Erlanger & Galinger; G-Tractors, Inc. vs. CA do not apply in the instant case. Signing as a surety is not embarking in a business.[22] We are likewise of the view that no matter how often an executive acted or was persuaded to act, as a surety for his own employer, this should not be taken to mean that he had thereby embarked in the business of suretyship or guaranty. This is not to say, however, that we are unaware that executives are often asked to stand as surety for their companys loan obligations. This is especially true if the corporate officials have sufficient property of their own; otherwise, their spouses signatures are required in order to bind the conjugal partnerships. The fact that on several occasions the lending institutions did not require the signature of the wife and the husband signed alone does not mean that being a surety became part of his profession. Neither could he be presumed to have acted for the conjugal partnership. Article 121, paragraph 3, of the Family Code is emphatic that the payment of personal debts contracted by the husband or the wife before or during the marriage shall not be charged to the conjugal partnership except to the extent that they redounded to the benefit of the family. Here, the property in dispute also involves the family home. The loan is a corporate loan not a personal one. Signing as a surety is certainly not an exercise of an industry or profession nor an act of administration for the benefit of the family. On the basis of the facts, the rules, the law and equity, the assailed decision should be upheld as we now uphold it. This is, of course, without prejudice to petitioners right to enforce the obligation in its favor against the PBM receiver in accordance with the rehabilitation program and payment schedule approved or to be approved by the Securities & Exchange Commission. WHEREFORE, the petition for review should be, as it is hereby, DENIED for lack of merit. SO ORDERED.

628

University of the Cordilleras College of Law First Year C S.Y. 2013 - 2014 Case Digest Ayala Development Corporation vs Court of Appeals G.R. No. 118305 Decided on: February 12, 1998 Ponente: J. Martinez Facts: Philippine Blooming Mills (PBM) obtained P50,300,000.00 loan from petitioner Ayala Investment and Development Corporation (AIDC). Respondent Alfredo Ching, EVP of PBM, executed security agreements on December 1980 and March 1981 making him jointly and severally answerable with PBMs indebtedness to AIDC. PBM failed to pay the loan hence filing of complaint against PBM and Ching. The RTC rendered judgment ordering PBM and Ching to jointly and severally pay AIDC the principal amount with interests. Pending the appeal of the judgment, RTC issued writ of execution. Thereafter, Magsajo, appointed deputy sheriff, caused the issuance and service upon respondent spouses of the notice of sheriff sale on 3 of their conjugal properties on May 1982. Respondent spouses filed injunction against petitioners on the ground that subject loan did not redound to the benefit of the said conjugal partnership. CA issued a TRP enjoining lower court from enforcing its order paving way for the scheduled auction sale of respondent spouses conjugal properties. A certificate of sale was issued to AIDC, being the only bidder and was registered on July 1982. Issue: Whether or not the debts and obligations contracted by the husband alone is considered for the benefit of the conjugal partnership and is it chargeable. Ruling: The loan procured from AIDC was for the advancement and benefit of PBM and not for the benefit of the conjugal partnership of Ching. Furthermore, AIDC failed to prove that Ching contracted the debt for the benefit of the conjugal partnership of gains. PBM has a personality distinct and separate from the family of Ching despite the fact that they happened to be stockholders of said corporate entity. Clearly, the debt was a corporate debt and right of recourse to Ching as surety is only to the extent of his corporate stockholdings. Based from the foregoing jurisprudential rulings of the court, if the money or services are given to another person or entity, and the husband acted only as a surety or guarantor, that contract cannot, by itself, alone be categorized as falling within the context of obligations for the benefit of the conjugal partnership. The contract of loan or services is clearly for the benefit of the principal debtor and not for the surety or his family. Ching only signed as a surety for the loan contracted with AIDC in behalf of PBM. Signing as a surety is certainly not an exercise of an industry or profession, it is not embarking in a business. Hence, the conjugal partnership should not be made liable for the surety agreement which was clearly for the benefit of PBM. The court did not support the contention of the petitioner that a benefit for the family may have resulted when the guarantee was in favor of Chings employment (prolonged tenure, appreciation of shares of stocks, prestige enhanced) since the benefits contemplated in Art. 161 of the Civil Code must be one directly resulting from the loan. It must not be a mere by product or a spin-off of the loan itself.

629

University of the Cordilleras College of Law First Year C S.Y. 2013 - 2014 Toda vs Court of Appeals 183 SCRA 713 Full Case G.R. Nos. 78583-4 March 26, 1990 BENIGNO TODA, JR., petitioner, vs. COURT OF APPEALS and ROSE MARIE TUASON-TODA, respondents. G.R. Nos.78696-7 March 26,1990 ROSE MARIE TUASON-TODA, petitioner, vs. BENIGNO TODA, JR., respondent. REGALADO, J.: These consolidated cases seek a review of the decision of the Court of Appeals promulgated on January 29, 1987 1 in CA-G.R. CV Nos. 06675 and 07936, the dispositive portion of which reads: WHEREFORE, judgment is hereby rendered: 1. Ordering the payment of the cash dividends declared on July 1, 1981 amounting to P2,191.62 and those declared on July 25, 1981 amounting to P40,196.12 to Rose Marie Toda as her separate property. The cash dividends declared on April 25, 1981 amounting to P37,196.30 (sic) are hereby adjudicated to Benigno Toda, Jr. as his share in the conjugal partnership assets; the portion of the order dated November 2, 1981 with respect to the payment of the amount of P360,095.12 to Rose Marie T. Toda is set aside; 2. Ordering the payment of the amount of P4,1623,982.24 to Rose Marie Toda representing the balance of P15, 749,135.32 obligated to be paid as estate taxes by Benigno Toda, Jr.; 3. Setting aside the order of the lower court dated June 2, 1982 directing Benigno Toda, Jr. to pay interest and non-payment penalty of 18% and 5%, respectively; and 4. Setting aside the order of the lower court directing the annotation of lien on the property of Benigno Toda, Jr. SO ORDERED. Benigno Toda, Jr. (Benigno for brevity) and Rose Marie Tuason-Toda (Rose Marie for brevity) were married on June 9, 1951 and were blessed with two children. Individual differences and the alleged infidelity of Benigno, however, marred the conjugal union thereby prompting Rose Marie to file on December 18, 1979 in the former Court of First Instance of Rizal, 2 as Civil Case No. 35566, a petition for termination of conjugal partnership for alleged mismanagement and dissipation of conjugal funds against Benigno. After hearings were held, the parties in order to avoid further "disagreeable proceedings," filed on April 1, 1981 a joint petition forjudicial approval of dissolution of conjugal partnership under Article 191 of the Civil Code, docketed as Special Proceeding No. 9478, 3 which was consolidated with the aforesaid civil case. This petition which was signed by the parties on March 30, 1981, embodied a compromise agreement allocating to the spouses their respective shares in the conjugal partnership assets and dismissing with prejudice the said Civil Case No. 35566, CA-G.R. No. 11123-SP of the Court of Appeals and G.R. No. 56121 of this Court. The said petition and the compromise agreement therein were approved by the trial court in its order of June 9, 1981. 4 Thereafter, several orders were issued by the lower court pertaining to the interpretation and implementation of the compromise agreement, as follows: 1. Order, dated November 20, 1981, ordering Benigno, inter alia, to pay Rose Marie the cash dividends on the shares declared on April 25, 1981 amounting to P37,126.30; that declared on July 25, 1981 amounting to P40,196.12; that declared on July 1, 1981, given on September 25, 1981 amounting to P2,191.62; and the payment of P360,095.12 to Rose Marie which is the balance of P2 million paid on April 4, 1981; 5 2. Order, dated June 2, 1982, ordering Benigno to pay Rose Marie interest at 18% per annum on the amounts required to be paid in the order of November 20,1981, as well as 5% non-payment penalty should the said order of November 20,1981 be sustained on appeal; 6

630

University of the Cordilleras College of Law First Year C S.Y. 2013 - 2014 3. Order, dated December 9, 1982, denying Benigno's motion to inhibit Judge Rizalina Bonifacio Vera from hearing the case; 7 4. Order, dated March 1, 1983, ordering the annotation of a lien on certain properties of Benigno as security for any and all amounts that he may finally be ordered to pay to Rose Marie under the compromise agreement; 8 and 5. Order, dated March 14, 1983, ordering Benigno to pay Rose Marie the amount of P4,623,929.24, with interest and penalties thereon at the rates stipulated in the compromise agreement from date of at the rates stipulated in the compromise agreement from date of demand by Rose Marie. 9 The compromise agreement which, as earlier stated, was incorporated in the petition for dissolution of the conjugal partnership and was approved by the court below, contains the following stipulaitons: xxx xxx xxx 4. For the best interest of each of them, petitioners have agreed to dissolve their conjugal partnership and to partition the assets thereof, under the following terms and conditions this document, a pleading, being intended by them to embody and evidence their agreement; (a) Petitioners as the parties hereto agree upon the dissolution of their conjugal partnership during the marriage and further agree to obtain judicial approval of their said agreement as provided by Article 191 of the Civil Code. (b) The following shall be adjudicated to petitioner Rose Marie Tuason-Toda: (1) Forty Million Peson (P40,000,000.00) to be paid as follows: (a) Petitioner Benigno Toda, Jr. shall assume the payment of the estate taxes, interest and penalties thereon, pertaining to the estate of petitioner Rose Marie Tuason Toda's late brother Manuel Tuason, Jr. in the sum of P15,749,135.32 as of March 31, 1981 all interest and penalty charges after March 31, 1981 to be the responsibility of petitioner Benigno Toda, Jr. (b) P2,000,000.00 to be paid within 30 days after signing of this agreement. (c) The balance shall be paid within six (6) months after date of signing of this agreement. If not paid when due, the balance shall bear interest at 18% per annum until paid and there shall be a 5% non-payment penalty. The proceeds from any sale of or loss with respect to, Rubicon's shares in Philippine Air Lines, Inc., shares of Cibeles Insurance Corporation or Hermana Mayor shall be applied when received against the aforesaid balance, except to the extent such proceeds are used to satisfy any other obligation under this agreement. (2) All shares of stock in San Nguel Corporation registered solely in the name of petitioner Rose Marie Tuason Toda whether stock dividends or stocks acquired on pre-emptive rights including those acquired in the names of both petitioners Benigno Toda, Jr. and Rose Marie Tuason Toda (whetherjointly or alternately 'and/or'), free from all liens and encumbrances. (3) All shares of stock in San Miguel Corporation acquired whether as stock dividends of or on pre-emptive zighta pertaining to the shares of stock in said corporation of petitioner Rose Marie Tuason Toda's brother the late Manuel Tuason, Jr. (of course, the original shares of the latter pertain to petitioner Rose Marie Tuason Toda also), free from all liens and encumbrances except for the estate tax lien. Petitioner Rose Marie Tuason Toda hereby grants petitioner Benigno Toda, Jr. an irrevocable proxy, for three years through the 1983 stockholders' meeting whether annual or special to elect directors for all shares of stock she owns

631

University of the Cordilleras College of Law First Year C S.Y. 2013 - 2014 directly or indirectly including those from the late Manuel Tuason, Jr. in San Miguel Corporation. (4) The Banaba Forbes Park conjugal dwelling and its contents free from all liens and encumbrances except that petitioner Benigno Toda, Jr. shall remove therefrom his personal effects including furniture and appliances in his study room and T.V. room and, from the family rooin, all antiques, rugs, paintings of Old Fort Manila, books and mementos. Petitioner Benigno Toda, Jr. commits that no servant now living in the Tolentino street apartments shall be evicted. (5) The San Francisco apartment at Apartment 905, No. 1750 Taylor Street, San Francisco, California, U.SA., and its contents, free from all liens and encumbrances, except that petitioner Benigno Toda, Jr. shall remove therefrom his personal effects. (6) The artifacts already removed by petitioner Rose Marie Tuason Toda from the Madrid Apartment at No. 4 San Pedro de Valdivia. She shall return to it its silver ware, china ware, paintings and etchings. She may retain the three fans encased in glass and may remove her clothes, perfumes and toiletries, the Sansa painting ofa shell dedicated to her, the painting of the Madonna and tapestry hanging in her bedroom, 5 Persian rugs, 1 writing desk and chair and the 2 lamps thereon and 1 lamp on the night table, and the statuette given her by Hagedorn. (7) Jewelry. (8) Motor vehicles registered in her name. (9) Within forty-five (45) days from signing of this agreement, One Million Pesos (Pl,000,000.00) as attorneys' fees petitioner Rose Marie Tuason Toda agreeing to hold petitioner Benigno Toda, Jr. harmless from any claim fo attorneys' fees and expenses that may be filed against the conjugal partnership or herself for services rendered to her in the prosecution of her claims against said conjugal partnership or against petitioner Benigno Toda, Jr. or to secure her paraphernal estate. (10) Two shares with two lots in Valley Golf & Country Club. (11) One share in Club Puerta de Hierro in Madrid, Spain if there is one registered in petitioner Rose Marie Tuason Toda's name. (12) Share in Montemar Beach Club in Bagac, Bataan petitioner Rose Marie Tuason Toda agreeing to assume the balance of the acquisition cost thereof. (c) All other properties of the conjugal partnership of whatever and wherever located shall be adjudicated to petitioner Benigno Toda, Jr. even though acquired in the name of petitioner Rose Marie Tuason Toda or both of them she undertaking to execute the corresponding deeds of conveyances. (d) Petitioner Benigno Toda, Jr. shall assume the payment of all conjugal obligations, petitioner Rose Marie Tuason Toda representing and warranting that she has no pending obligation or incurred no obligation chargeable to the conjugal partnership except those listed in Annex 'A' hereof. If the Rosaria Apartment is subject to a mortgage loan and such loan is a conjugal debt, petitioner Benigno Toda, Jr. shall assume such loan and shall obtain the discharge of the mortgage. (e) After the signing of this document: (1) Each of them shall own, dispose of, possess, administer and enjoy his or her separate estate, present and future, without the consent of the other; (2) All earnings from any profession business or industry shall likewise belong to each of them respectively; (3) All expenses and obligations incurred by each of them shall be their respective and separate responsibilities. 632

University of the Cordilleras College of Law First Year C S.Y. 2013 - 2014 (f) With the signing of this document, Civil Case No. 35566 of this same Court, CA-G.R. No. 11123-SP and SC-G.R. No. L-56121 shall be deemed dismissed with prejudice as between the parties hereto. 10 The parties then prayed that judgment be rendered: (a) Approving the agreement for voluntary dissolution and partition of the conjugal partnership; (b) declaring the conjugal partnership of petitioners dissolved and adjudicating to each of them his or her share in the properties and assets of said conjugal partnership in accordance with the agreement embodied in paragraph 4 hereof; and (c) enjoining the parties to comply with the terms and conditions of the aforesaid agreement. 11 Ironically, the said agreement failed to fully subserve the intended amicable settlement of all the disputes of the spouses. Instead, as lamented by the counsel of one of them, the compromise agreement which was designed to terminate a litigation spawned two new petitions, with each party initiating one against the other. Thus, illustrative of the saying that a solution which creates another problem is no solution, the contradictory interpretations placed by the parties on some provisions of the agreement resulted in appeals to respondent court and, eventually, the present recourse to us. Benigno appealed from the aforestated orders of the trial court of November 20, 1981, June 2, 1982, December 9, 1982, March 1, 1983 and March 14, 1983 containing the directives hereinbefore respectively set out. The same were disposed of by the Court of Appeals as explained at the start of this decision. Rose Marie now submits that the Court of Appeals erred: 1. In holding that the compromise agreement of the parties herein became effective only after its judicial approval on June 9, 1981 and not upon its execution on March 30,1981; 2. In setting aside the order of the lower court dated June 2, 1981 directing Benigno to pay interest of eighteen percent and non-payment penalty of five percent; and 3. In setting aside the order of the lower court directing the annotation of Rose Marie's lien on Benigno's property. 12 On the other hand, Benigno contends in his present petition before us that: 1. The Court of Appeals erred on a question of law when it affirmed the lower court's award of P4,623,929.24 without trial and evidence-taking and overruled petitioner's claim of violation of his due process right; 2. The Court of Appeals erred on a question of law and due process when it upheld the lower court's denial of petitioner's motion for her inhibition/disqualification; 3. Since the document (the parties' compromise agreement) explicitly provided for assumption of liability rather than agency to pay and since there was no evidence-taking, the Court of Appeals finding of an agency to pay is reviewable as a question of law; and 4. The Court of Appeals on a question of law involving the parol evidence rule. 13 The award of cash dividends basically depends on the date of effectivity of the compromise agreement as this will determine whether the same is conjugal property or separate property of the spouses. We are in agreement with the holding of the Court of Appeals that the compromise agreement became effective only on June 9, 1981, the date when it was approved by the trial court, and not on March 30,1981 when it was signed by the parties. Under Article 190 of the Civil Code, 14 "(i)n the absence of an express declaration in the marriage settlements, the separation of property between spouses during the marriage shall not take place save in virtue of a judicial order." Hence, the separation of property is not effected by the mere execution of the contract or agreement of the parties, but by the decree of the court approving the same. It, therefore, becomes effective on y upon judicial approval, without which it is void. 15 Furthermore, Article 192 of said Code explicitly provides that the conjugal partnership is dissolved only upon the issuance of a decree of separation of property. Consequently, the conjugal partnership of Benigno and Rose Marie should be considered dissolved only on June 9, 1981 when the trial court approved their joint petition for voluntary dissolution of their conjugal partnership. Conformably thereto, the cash dividends declared on July 1, 1981 and July 25,1981 in the amount of P2,191.62 and P40,196.12, respectively, should pertain to Rose Marie; and that declared on April 2,5, 1981 in the amount of P37,126.30 ought to be paid to Benigno, pursuant to Paragraph 4 (c) of the compromise agreement which awards to Benigno the conjugal assets not otherwise specifically assigned to Rose Marie. 633

University of the Cordilleras College of Law First Year C S.Y. 2013 - 2014 With respect to the amount of P360,095.12 which Benigrio deducted from the P2 million supposed to be paid to Rose Marie, it is not clear from the records where said amount came from. The Court of Appeals, in holding that it is conjugal and therefore belongs to Benigno, presumed it to be in the nature of cash dividends declared prior to the approval of the compromise agreement by reason of the fact that the amount was deducted by Benigno from the P2 million which he paid on April 14,1981. While no sufficient proof was adduced to conclusively explain such deduction, there exists the legal presumption that all property of the marriage belongs to the conjugal partnership absent any proof that it is the exclusive property of either spouse. 16 Since Rose Marie failed to prove that the amount forms part of her paraphernal property, it is presumed to be conjugal property. Consequently, Benigno is entitled to the said amount of P360,095.12, hence he rightfully deducted the same from the amount due to Rose Marie. The issue regarding the annotation of the lien on Benigno's properties has been mooted by our resolution dated Aprjl 3, 1989 wherein, at his instance, we ordered the cancellation thereof upon his posting of the corresponding bond. In our resolution of February 26, 1990, we noted Benigno's comphance, approved the bond he filed, and ordered the cancellation of the hens annotated on the certificates of title of the propertiesinvolved. Likewise, the order denying the motion to inhibit Judge Rizalina Bonifacio Vera has become academic considering that she no longer presides over the court where the case was filed. Besides, as correctly explained by respondent court, the groundfor inhibition raised by Benigno is not valid it being merely on the basis of the judge having acquired knowledge of the facts surrounding the agreement of the parties, hence she would be a material witness to the issue of the true agreement which is contested by the parties. However, those facts came to the knowledge of the judge in the course of her efforts to effect a compromise between parties and are also known to the parties.This is not a ground for disqualification; on the contrary, said, acts of the judge were in accord with the rule encouraging compromises in litigations, especially between members of the same family. Anent the tax savings of P4,623,982.24 obtained by Benigno, we hold that this forms part of the P40 million allocated to Rose Marie under paragraph 4 (b) (1) of the compromise agreement.We give credit to the ratiocination thereon of the trial court as quoted with approval by respondent court: The records show that petitioner Benigno Toda, Jr. paid only Pl,125,152.48 in estate taxes, although the amount stated in the m Compromise Agreement was P15,749,135.32. The balance of P4,623,929.24 is now being claimed by both parties as aforestated. In the opinion of this court, the pertinent terms of the Agreement as quoted, are clear and do not require any interpretation. In brief, under, the Agreement, petitioner Rose Marie T. Toda is adjudicated the fixed sum of P40 million, to be paid as follows: (a) Payment by petitioner Benigno Toda, Jr. of the estate taxes, interests and penalties thereon, pertaining to the estate of the late Manuel Tuason, Jr. in the amount of Pl5,749,135.32 as of March 31, 1982; (b) P2 million within 30 days after signing of the Agreement; (c) the balance within six months after date of signing of the Agreement. This Court notes that the amount of taxes, interests and penalties is fixed at P15,749,135.32 and this figure was provided by Benigno Toda, Jr. There is no provision as contended by petitioner Benigno Toda, Jr. that the amount was only an assumed liability and that he could attempt to reduce it by suit or compromise. It is clear that if the amount of P4,623,929.24 is to be credited to Benigno Toda, Jr. then the P40 million which petitioner Rose Marie T. Toda is to receive would be short by that amount. This Court is also of the opinion that under the Agreement, petitioner Benigno Toda, Jr. was constituted as agent to pay to the government the liability of the estate of the late Manuel Tuason, Jr. in the fixed amount of P15,749,135.32 and if he was able to secure a reduction thereof, then he should deliver to his principal such reduction... 17 We do not believe that Benigno was denied due process when the trial court resolved the motion of Rose Marie for the payment of P4,623,982.24 without the benefit of a hearing. The records disclose that the hearing thereon was postponed twice at the instance of Benigno, which prompted the court to thereafter consider the motion submitted for resolution on the basis of the allegations therein and the answer filed by counsel for both parties. Benigno cannot now be heard to claim that he was deprived of his day in court. Furthermore, respondent court correctly held that the issue involved was more of a question of interpretation of a contract rather than a determination of facts. Benigno failed to make a plausible showing that the supposed evidence he had intended to present, if any, would not be merely collateral matters. Considering that the amount of P4,623,982.24 actually forms an integral part of the P40 million (minus the lawful and authorized deductions that may be made therefrom) which Benigno categorically undertook to pay to Rose Marie, the same must earn interest at the rate of 18% per annum and 5% non634

University of the Cordilleras College of Law First Year C S.Y. 2013 - 2014 payment penalty, the same being included in and within the contemplation of Paragraph 4 (b) (1) (c) of the compromise agreement. Said provision of the agrdement provides for the payment of the interest and penalty upon non-payment of the balance of the P40 million after the specific authorized deductions therefrom. Since the amount of P4,623,982.24 was not to be lawfully deducted by Benigno, as hereinbefore explained, it constitutes part of the contemplated contingent balance which might tum out to be due to Rose Marie and, therefore, subject to the imposition of said increments on Benigno's liability. WHEREFORE, the judgment appealed from is hereby AFFIRMED, with the modification that Benigno Toda, Jr. is hereby ordered to pay Rose Marie Tuason Toda interest at the rate of a 18% per annum and 5% non-payment penalty on the tax savings of P4,623,982.24 from date of formal demand until the same is fully paid. SO ORDERED.

635

University of the Cordilleras College of Law First Year C S.Y. 2013 - 2014 Case Digest Toda vs CA G.R. Nos. 78583-4 Decided on: March 26, 1990 Ponente: J. Regalado Facts: Benigno Toda, Jr. (Benigno for brevity) and Rose Marie Tuason- Toda (Rose Marie for brevity) were married on June 9, 1951 and were blessed with two children. Individual differences and the alleged infidelity of Benigno, however, mar of First Instance of Rizal, as Civil Case No. 35566, a petition for termination of conjugal partnership for alleged mismanagement and dissipation of conjugal funds against Benigno. After hearings were held, the parties in order to avoid further" disagreeable proceedings," filed on April 1, 1981 a joint petition for judicial approval of dissolution of conjugal partnership under Article191 of the Civil Code, docketed as Special Proceeding No. 9478, which was consolidated with the aforesaid civil case. This petition which was signed by the parties on March 30, 1981, embodied a compromise agreement allocating to the spouses their respective shares in the conjugal partnership assets and dismissing with prejudice the said Civil Case No. 35566, CA-G.R. No. 11123-SP of the Court of Appeals and G.R. No. 56121 of this Court. The said petition and the compromise agreement therein were approved by the trial court in its order of June 9, 1981. Thereafter, several orders were issued by the lower court pertaining to the interpretation and implementation of the compromise agreement Issue: When did the compromise agreement become effective? Ruling: We are in agreement with the holding of the Court of Appeals that the compromise agreement became effective only on June 9, 1981, the date it was approved by the trial court and not on March 30, 1981 when it was signed by the parties. Under Article 134 of the Family Code in the absence of express declaration in the marriage settlements, the separation of property between the spouses during the marriage shall not take place save in virtue of a judicial order hen ce, the separation of property is not effected by mere execution of the contract or agreement of the parties, but by the decree of the court approving the same. It, therefore, becomes effective only upon judicial approval, without which it is void. Furthermore, Article 137 of the said code explicitly provides that the conjugal partnership is dissolved only upon the issuance of a decree of separation of property.

636

University of the Cordilleras College of Law First Year C S.Y. 2013 - 2014

X.

Property Regime of Unions Without Marriage

637

University of the Cordilleras College of Law First Year C S.Y. 2013 - 2014 ACRE vs Yuttikki 534 SCRA 224 G.R. No. 153029 September 27, 2007 Full Case BEATRIZ, ALLAN, MARY ANN, JOCELYN, WELMA, ROWEL and SOFRONIO WENDEL II, all surnamed ACRE, vs EVANGELINE YUTTIKKI, SANDOVAL-GUTIERREZ, J.: For our resolution is the instant Petition for Review on Certiorari under Rule 45 of the 1997 Rules of Civil Procedure, as amended, assailing the Decision[1] dated March 11, 2002 rendered by the Court of Appeals in CA-G.R. CV No. 64656, entitled Beatriz Acre, et al., plaintiffs-appellants, v. Evangeline Yuttikki, defendant-appellant. Beatriz Acre, petitioner, and Sofronio Acre, Jr. were married on November 8, 1957. Their union produced six children, also petitioners. Sometime in 1972, Sofronio left the conjugal dwelling because of constant marital dispute. Later, petitioners found that he married Evangeline Yuttikki, respondent, on May 18, 1972 while his marriage to Beatriz was still subsisting. On November 16, 1996, Sofronio died. His union with respondent lasted for more than 24 years. During respondents marriage with Sofronio, they acquired the following properties: (a) a motor vehicle; (b) two parcels of land covered by Transfer Certificate of Title (TCT) No. 116740 in the name of Evangeline Y. Acre, married to Sofronio V. Acre, Jr.; (c) and TCT No. 100087 registered in the names of Evangeline Y. Acre, married to Sofronio V. Acre, Jr. and Nellie Y. Del Mar, married to Jose Del Mar. Petitioners filed with the Regional Trial Court, Branch 58, Cebu City, a complaint for reconveyance and recovery of properties and/or partition with damages. They alleged that Sofronio alone acquired the subject properties with his funds. In a Decision dated November 24, 1998, the trial court dismissed the complaint concluding that the two parcels of land are owned in common by respondent and Sofronio. Thereupon, petitioners filed a notice of appeal which was likewise dismissed by the Court of Appeals in its Resolution of March 11, 2002 for lack of merit, finding that In view of the failure of the plain tiffs-appellants to prove by preponderance of evidence their entitlement to the properties in question, the land covered by TCT No. 100087 is exclusively owned by defendant-appellee and with respect to the property covered by TCT No. 116740, the defendant-appellee co-owned such property with her sister Nellie Del Mar. Petitioners contend that the Court of Appeals erred in declaring respondent the owner of the contested properties. Undeniably, the marriage between respondent and Sofronio is bigamous considering that their union was celebrated while he was still married to Beatriz. As such, their property regime is covered by Article 148 of the Family Code providing that all properties acquired by the parties out of their actual joint contribution of money, property, or industry shall be governed by the rules on co-ownership.[2] Hence, if there is no contribution from either or both of the spouses, there can be no co-ownership.[3] Petitioners failed to present any evidence to establish that Sofronio made an actual contribution in acquiring the contested properties. Clearly, co-ownership does not exist here. The Court of Appeals held: 638

University of the Cordilleras College of Law First Year C S.Y. 2013 - 2014 In the instant case, the property covered by TCT No. 100087 contains a recital that the property is registered in the name of Evangeline Y. Acre, married to Sofronio V. Acre, Jr. On the other hand, RCT No. 116740 shows that the property described therein was registered in the names of Evangeline Y. Acre, married to Sofronio Acre, and Nellie Y. Del Mar, married to Jose Del Mar. Therefore, the certificates of title on its face show that the disputed properties were exclusively owned by defendant-appellee (with respect to TCT No. 100087) and co-owned by the defendant-appellee with her sister Nellie (with regard to TCT No. 116740). The rule is well-settled that the words married to preceding Sofronio Acre, Jr. are merely descriptive of the civil status of the defendant-appellee. WHEREFORE, we DENY the instant petition and AFFIRM the assailed Decision of the Court of Appeals in CA-G.R. CV No. 64656. Costs against petitioners. SO ORDERED.

639

University of the Cordilleras College of Law First Year C S.Y. 2013 - 2014 Enrico vs Heirs of Spouses Medinaceli 534 SCRA 418 G.R. No. 173614 September 28, 2007 Full Case LOLITA D. ENRICO, Petitioner, vs. HEIRS OF SPS. EULOGIO B. MEDINACELI AND REPRESENTED BY VILMA M. ARTICULO, Respondents. CHICO-NAZARIO, J.: The instant Petition for Certiorari filed under Rule 65 of the 1997 Rules of Civil Procedure assails the Order, dated 3 May 2006 of the Regional Trial Court (RTC) of Aparri, Cagayan, Branch 6, in Civil Case No. II-4057, granting reconsideration of its Order, dated 11 October 2005, and reinstating respondents Complaint for Declaration of Nullity of Marriage. On 17 March 2005, respondents, heirs of Spouses Eulogio B. Medinaceli (Eulogio) and Trinidad CatliMedinaceli (Trinidad) filed with the RTC, an action for declaration of nullity of marriage of Eulogio and petitioner Lolita D. Enrico. Substantially, the complaint alleged, inter alia, that Eulogio and Trinidad were married on 14 June 1962, in Lal-lo, Cagayan.3 They begot seven children, herein respondents, namely: Eduardo, Evelyn, Vilma, Mary Jane, Haizel, Michelle and Joseph Lloyd.4 On 1 May 2004, Trinidad died.5 On 26 August 2004, Eulogio married petitioner before the Municipal Mayor of Lal-lo, Cagayan.6 Six months later, or on 10 February 2005, Eulogio passed away.7 In impugning petitioners marriage to Eulogio, respondents averred that the same was entered into without the requisite marriage license. They argued that Article 348 of the Family Code, which exempts a man and a woman who have been living together for at least five years without any legal impediment from securing a marriage license, was not applicable to petitioner and Eulogio because they could not have lived together under the circumstances required by said provision. Respondents posited that the marriage of Eulogio to Trinidad was dissolved only upon the latters death, or on 1 May 2004, which was barely three months from the date of marriage of Eulogio to petitioner. Therefore, petitioner and Eulogio could not have lived together as husband and wife for at least five years. To further their cause, respondents raised the additional ground of lack of marriage ceremony due to Eulogios serious illness which made its performance impossible. In her Answer, petitioner maintained that she and Eulogio lived together as husband and wife under one roof for 21 years openly and publicly; hence, they were exempted from the requirement of a marriage license. From their union were born Elvin Enrico and Marco Enrico, all surnamed Medinaceli, on 28 October 1988 and 30 October 1991, respectively. She further contended that the marriage ceremony was performed in the Municipal Hall of Lal-lo, Cagayan, and solemnized by the Municipal Mayor. As an affirmative defense, she sought the dismissal of the action on the ground that it is only the contracting parties while living who can file an action for declaration of nullity of marriage. On 11 October 2005, the RTC issued an Order,9 granting the dismissal of the Complaint for lack of cause of action. It cited A.M. No. 02-11-10-SC,10 dated 7 March 2003, promulgated by the Supreme Court En Banc as basis. The RTC elucidated on its position in the following manner: The Complaint should be dismissed. 1) Administrative Matter No. 02-11-10-SC promulgated by the Supreme Court which took effect on March 15, 2003 provides in Section 2, par. (a)11 that a petition for Declaration of Absolute Nullity of a Void Marriage may be filed solely by the husband or the wife. The language of this rule is plain and simple which states that such a petition may be filed solely by the husband or the wife. The rule is clear and unequivocal that only the husband or the wife may file the petition for Declaration of Absolute Nullity of a Void Marriage. The reading of this Court is that the right to bring such petition is exclusive 640

TRINIDAD

CATLI-MEDINACELI,

University of the Cordilleras College of Law First Year C S.Y. 2013 - 2014 and this right solely belongs to them. Consequently, the heirs of the deceased spouse cannot substitute their late father in bringing the action to declare the marriage null and void.12 (Emphasis supplied.) The dispositive portion of the Order, thus, reads: WHEREFORE, [the] Motion to Dismiss raised as an affirmative defense in the answer is hereby GRANTED. Accordingly, the Complaint filed by the [respondents] is hereby DISMISSED with costs de officio. 13 Respondents filed a Motion for Reconsideration thereof. Following the filing by petitioner of her Comment to the said motion, the RTC rendered an Order14 dated 3 May 2006, reversing its Order of 11 October 2005. Hence, the RTC reinstated the complaint on the ratiocination that the assailed Order ignored the ruling in Nial v. Bayadog,15 which was on the authority for holding that the heirs of a deceased spouse have the standing to assail a void marriage even after the death of the latter. It held that Section 2(a) of A.M. No. 02-11-20-SC, which provides that a petition for declaration of absolute nullity of void marriage may be filed solely by the husband or the wife, applies only where both parties to a void marriage are still living.16 Where one or both parties are deceased, the RTC held that the heirs may file a petition to declare the marriage void. The RTC expounded on its stance, thus: The questioned Order disregarded the case of Nial vs. Bayadog, 328 SCRA 122 (March 14, 2000) in which the Supreme Court, First Division, held that the heirs of a deceased person may file a petition for the declaration of his marriage after his death. The Order subject of this motion for reconsideration held that the case of Nial vs. Bayadog is now superseded by the new Rule on Declaration of Absolute Nullity of Marriages (hereinafter referred to as the Rule) because the Supreme Court has rejected the case of Nial vs. Bayadog by approving the Rule on Nullity of Void Marriages. The Order further held that it is only the husband or the wife who is (sic) the only parties allowed to file an action for declaration of nullity of their marriage and such right is purely personal and is not transmissible upon the death of the parties. It is admitted that there seems to be a conflict between the case of Nial vs. Bayadog and Section 2(a) of the Rule. In view of this, the Court shall try to reconcile the case of Nial vs. Bayadog and the Rule. To reconcile, the Court will have to determine [the] basic rights of the parties. The rights of the legitimate heirs of a person who entered into a void marriage will be prejudiced particularly with respect to their successional rights. During the lifetime of the parent[,] the heirs have only an inchoate right over the property of the said parents. Hence, during the lifetime of the parent, it would be proper that it should solely be the parent who should be allowed to file a petition to declare his marriage void. However, upon the death of the parent his heirs have already a vested right over whatever property left by the parent. Such vested right should not be frustrated by any rules of procedure such as the Rule. Rules of Procedure cannot repeal rights granted by substantive law. The heirs, then, have a legal standing in Court. If the heirs are prohibited from questioning the void marriage entered by their parent, especially when the marriage is illegal and feloniously entered into, it will give premium to such union because the guilty parties will seldom, if ever at all, ask for the annulment of the marriage. Such void marriage will be given a semblance of validity if the heirs will not be allowed to file the petition after the death of the parent. For these reasons, this Court believes that Sec. 2(a) of the Rules on Declaration of Absolute Nullity of Marriage is applicable only when both parties to a (sic) void marriage are still living. Upon the death of anyone of the guilty party to the void marriage, his heirs may file a petition to declare the the (sic) marriage void, but the Rule is not applicable as it was not filed b the husband or the wife. It shall be the ordinary rule of civil procedure which shall be applicable.17 Perforce, the decretal portion of the RTC Order of 3 May 2006 states: In view of the foregoing, the Court grants the motion for reconsideration dated October 31, 2005 and reinstate this case.18 641

University of the Cordilleras College of Law First Year C S.Y. 2013 - 2014 Aggrieved, petitioner filed a Motion for Reconsideration of the foregoing Order; however, on 1 June 2006, the RTC denied the said motion on the ground that no new matter was raised therein.19 Hence, the instant Petition under Rule 65 of the 1997 Rules of Civil Procedure on the sole question of whether the case law as embodied in Nial, or the Rule on Declaration of Absolute Nullity of Void Marriages and Annulment of Voidable Marriages, as specified in A.M. No. 02-11-10-SC of the Supreme Court applies to the case at bar. At the outset, we note that petitioner took an abbreviated route to this Court, countenancing the hierarchy of courts. We have earlier emphasized that while the Supreme Court has the concurrent jurisdiction with the Court of Appeals and the RTCs (for writs enforceable within their respective regions), to issue writs of mandamus, prohibition or certiorari, the litigants are well advised against taking a direct recourse to this Court.20 Instead, they should initially seek the proper relief from the lower courts. As a court of last resort, this Court should not be burdened with the task of dealing with causes in the first instance. Where the issuance of an extraordinary writ is concurrently within the competence of the Court of Appeals or the RTC, litigants must observe the principle of hierarchy of courts.21 However, it cannot be gainsaid that this Court has the discretionary power to brush aside procedural lapses if compelling reasons, or the nature and importance of the issues raised, warrant the immediate exercise of its jurisdiction.22 Moreover, notwithstanding the dismissibility of the instant Petition for its failure to observe the doctrine on the hierarchy of courts, this Court will proceed to entertain the case grounded as it is on a pure question of law. Petitioner maintains that A.M. No. 02-11-10-SC governs the instant case. A contrario, respondents posit that it is Nial which is applicable, whereby the heirs of the deceased person were granted the right to file a petition for the declaration of nullity of his marriage after his death. We grant the Petition. In reinstating respondents Complaint for Declaration of Nullit y of Marriage, the RTC acted with grave abuse of discretion. While it is true that Nial in no uncertain terms allowed therein petitioners to file a petition for the declaration of nullity of their fathers marriage to therein respondent after the death of their father, we cannot, however, apply its ruling for the reason that the impugned marriage therein was solemnized prior to the effectivity of the Family Code. The Court in Nial recognized that the applicable law to determine the validity of the two marriages involved therein is the Civil Code, which was the law in effect at the time of their celebration.23 What we have before us belongs to a different milieu, i.e., the marriage sought to be declared void was entered into during the effectivity of the Family Code. As can be gleaned from the facts, petitioners marriage to Eulogio was celebrated in 2004. The Rule on Declaration of Absolute Nullity of Void Marriages and Annulment of Voidable Marriages as contained in A.M. No. 02-11-10-SC is explicit in its scope, to wit: Section 1. Scope. This Rule shall govern petitions for declaration of absolute nullity of void marriages and annulment of voidable marriages under the Family Code of the Philippines. The Rules of Court shall apply suppletorily. (Emphasis supplied.) The categorical language of A.M. No. 02-11-10-SC leaves no room for doubt. The coverage extends only to those marriages entered into during the effectivity of the Family Code which took effect on 3 August 1988. Moreover, A.M. No. 02-11-10-SC took effect on 15 March 2003, following its publication in a newspaper of general circulation. Thus, contrary to the opinion of the RTC, there is no need to reconcile the 642

University of the Cordilleras College of Law First Year C S.Y. 2013 - 2014 provisions of A.M. No. 02-11-10-SC with the ruling in Nial, because they vary in scope and application. As has been emphasized, A.M. No. 02-11-10-SC covers marriages under the Family Code of the Philippines, and is prospective in its application. The marriage of petitioner to Eulogio was celebrated on 26 August 2004, and it squarely falls within the ambit of A.M. No. 02-11-10-SC. Hence, in resolving the issue before us, we resort to Section 2(a) of A.M. No. 02-11-10-SC, which provides: Section 2. Petition for declaration of absolute nullity of void marriages. (a) Who may file. A petition for declaration of absolute nullity of void marriage may be filed solely by the husband or the wife. (n) (Emphasis supplied.) There is no ambiguity in the Rule. Absolute sententil expositore non indiget. When the language of the law is clear, no explanation of it is required. Section 2(a) of A.M. No. 02-11-10-SC, makes it the sole right of the husband or the wife to file a petition for declaration of absolute nullity of void marriage. The Rationale of the Rules on Annulment of Voidable Marriages and Declaration of Absolute Nullity of Void Marriages, Legal Separation and Provisional Orders explicates on Section 2(a) in the following manner, viz: 1. Only an aggrieved or injured spouse may file petitions for annulment of voidable marriages and declaration of absolute nullity of void marriages. Such petitions cannot be filed by the compulsory or intestate heirs of the spouses or by the State. [Section 2; Section 3, paragraph a] Only an aggrieved or injured spouse may file a petition for annulment of voidable marriages or declaration of absolute nullity of void marriages. Such petition cannot be filed by compulsory or intestate heirs of the spouses or by the State. The Committee is of the belief that they do not have a legal right to file the petition. Compulsory or intestate heirs have only inchoate rights prior to the death of their predecessor, and hence can only question the validity of the marriage of the spouses upon the death of a spouse in a proceeding for the settlement of the estate of the deceased spouse filed in the regular courts. On the other hand, the concern of the State is to preserve marriage and not to seek its dissolution. (Emphasis supplied.) Respondents clearly have no cause of action before the court a quo. Nonetheless, all is not lost for respondents. While A.M. No. 02-11-10-SC declares that a petition for declaration of absolute nullity of void marriage may be filed solely by the husband or the wife, it does not mean that the compulsory or intestate heirs are already without any recourse under the law. They can still protect their successional right, for, as stated in the Rationale of the Rules on Annulment of Voidable Marriages and Declaration of Absolute Nullity of Void Marriages, Legal Separation and Provisional Orders, compulsory or intestate heirs can still question the validity of the marriage of the spouses, not in a proceeding for declaration of nullity, but upon the death of a spouse in a proceeding for the settlement of the estate of the deceased spouse filed in the regular courts. WHEREFORE, the Petition is GRANTED. Civil Case No. II-4057 filed before the Regional Trial Court of Aparri, Cagayan, Branch 6, is ORDERED DISMISSED without prejudice to challenging the validity of the marriage of Lolita D. Enrico to Eulogio B. Medinaceli in a proceeding for the settlement of the estate of the latter. No costs. SO ORDERED.

643

University of the Cordilleras College of Law First Year C S.Y. 2013 - 2014 Case Digest: Enrico vs Heirs of Spouses Medinaceli 534 SCRA 418 G.R. No. 1736 14 Decided on: September 28, 2007 Ponente: CHICO-NAZARIO, J.: Facts: On 17 March 2005, respondents, heirs of Spouses Eulogio B. Medinaceli (Eulogio) and Trinidad Catli-Medinaceli (Trinidad) filed with the RTC, an action for declaration of nullity of marriage of Eulogio and petitioner Lolita D. Enrico. Substantially, the complaint alleged, inter alia, that Eulogio and Trinidad were married on 14 June 1962, in Lal-lo, Cagayan.[3] They begot seven children. On 1 May2004, Trinidad died.[5] On 26, in her Answer, petitioner maintained that she and Eulogio lived together as husband and wife under one roof for 21 years openly and publicly; hence, they were exempted from the requirement of a marriage license. From their union were born Elvin Enrico and Marco Enrico, all surnamed Medinaceli, on 28 October 1988and 30 October 1991, respectively. She further contended that the marriage ceremony was performed in the Municipal Hall of Lal-lo, Cagayan, and solemnized by the Municipal Mayor. As an affirmative defense, she sought the dismissal of the action on the ground that it is only the contracting parties while living who can file an action for declaration of nullity of marriage Issue: Whether or not A.M. No. 02-11-10-SC governs the instant case. Ruling: Respondents clearly have no cause of action before the court a quo. Nonetheless, all is not lost for respondents. While A.M. No. 02-11-10-SC declares that a petition for declaration of absolute nullity of void marriage may be filed solely by the husband or the wife, it does not mean that he compulsory or intestate heirs are already without any recourse under the law. They can still protect their successional right, for, as stated in the Rationale of the Rules on Annulment of Voidable Marriages and Declaration of Absolute Nullity of Void Marriages, Legal Separation and Provisional Orders, compulsory or intestate heirs can still question the validity of the marriage of the spouses, not in a proceeding for declaration of nullity, but upon the death of a spouse in a proceeding for the settlement of the estate of the deceased spouse filed in the regular courts. WHEREFORE, the Petition is GRANTED. Civil Case No. II-4057 filed before the Regional Trial Court of Aparri, Cagayan, Branch 6, is ORDERED DISMISSED without prejudice to challenging the validity of the marriage of Lolita D. Enrico to Eulogio B. Medinaceli in a proceeding for the settlement of the estate of the latter.

644

University of the Cordilleras College of Law First Year C S.Y. 2013 - 2014 Valdez vs RTC 260 SCRA 221 G.R. No. 122749. July 31, 1996 Full Case ANTONIO A. S. VALDES, petitioner, vs. REGIONAL TRIAL COURT, BRANCH 102, QUEZON CITY, and CONSUELO M. GOMEZ-VALDES, respondents. VITUG, J.: The petition for review bewails, purely on a question of law, an alleged error committed by the Regional Trial Court in Civil Case No. Q-92-12539. Petitioner avers that the court a quo has failed to apply the correct law that should govern the disposition of a family dwelling in a situation where a marriage is declared void ab initio because of psychological incapacity on the part of either or both of the parties to the contract. The pertinent facts giving rise to this incident are, by and large, not in dispute. Antonio Valdes and Consuelo Gomez were married on 05 January 1971. Begotten during the marriage were five children. In a petition, dated 22 June 1992, Valdes sought the declaration of nullity of the marriage pursuant to Article 36 of the Family Code (docketed Civil Case No. Q-92-12539, Regional Trial Court of Quezon City, Branch 102). After hearing the parties following the joinder of issues, the trial court,[1] in its decision of 29 July 1994, granted the petition; viz: "WHEREFORE, judgment is hereby rendered as follows: "(1) The marriage of petitioner Antonio Valdes and respondent Consuelo Gomez-Valdes is hereby declared null and void under Article 36 of the Family Code on the ground of their mutual psychological incapacity to comply with their essential marital obligations; "(2) The three older children, Carlos Enrique III, Antonio Quintin and Angela Rosario shall choose which parent they would want to stay with. "Stella Eloisa and Joaquin Pedro shall be placed in the custody of their mother, herein respondent Consuelo Gomez-Valdes. "The petitioner and respondent shall have visitation rights over the children who are in the custody of the other. "(3) The petitioner and respondent are directed to start proceedings on the liquidation of their common properties as defined by Article 147 of the Family Code, and to comply with the provisions of Articles 50, 51 and 52 of the same code, within thirty (30) days from notice of this decision. "Let a copy of this decision be furnished the Local Civil Registrar of Mandaluyong, Metro Manila, for proper recording in the registry of marriages."[2] (Italics ours) Consuelo Gomez sought a clarification of that portion of the decision directing compliance with Articles 50, 51 and 52 of the Family Code. She asserted that the Family Code contained no provisions on the procedure for the liquidation of common property in "unions without marriage." Parenthetically, during the hearing on the motion, the children filed a joint affidavit expressing their desire to remain with their father, Antonio Valdes, herein petitioner. In an Order, dated 05 May 1995, the trial court made the following clarification: "Consequently, considering that Article 147 of the Family Code explicitly provides that the property acquired by both parties during their union, in the absence of proof to the contrary, are presumed to have

645

University of the Cordilleras College of Law First Year C S.Y. 2013 - 2014 been obtained through the joint efforts of the parties and will be owned by them in equal shares, plaintiff and defendant will own their 'family home' and all their other properties for that matter in equal shares. "In the liquidation and partition of the properties owned in common by the plaintiff and defendant, the provisions on co-ownership found in the Civil Code shall apply."[3] (Italics supplied) In addressing specifically the issue regarding the disposition of the family dwelling, the trial court said: "Considering that this Court has already declared the marriage between petitioner and respondent as null and void ab initio, pursuant to Art. 147, the property regime of petitioner and respondent shall be governed by the rules on co-ownership. "The provisions of Articles 102 and 129 of the Family Code finds no application since Article 102 refers to the procedure for the liquidation of the conjugal partnership property and Article 129 refers to the procedure for the liquidation of the absolute community of property."[4] Petitioner moved for a reconsideration of the order. The motion was denied on 30 October 1995. In his recourse to this Court, petitioner submits that Articles 50, 51 and 52 of the Family Code should be held controlling; he argues that: "I "Article 147 of the Family Code does not apply to cases where the parties are psychological incapacitated. "II "Articles 50, 51 and 52 in relation to Articles 102 and 129 of the Family Code govern the disposition of the family dwelling in cases where a marriage is declared void ab initio, including a marriage declared void by reason of the psychological incapacity of the spouses. "III "Assuming arguendo that Article 147 applies to marriages declared void ab initio on the ground of the psychological incapacity of a spouse, the same may be read consistently with Article 129. "IV "It is necessary to determine the parent with whom majority of the children wish to stay."[5] The trial court correctly applied the law. In a void marriage, regardless of the cause thereof, the property relations of the parties during the period of cohabitation is governed by the provisions of Article 147 or Article 148, such as the case may be, of the Family Code. Article 147 is a remake of Article 144 of the Civil Code as interpreted and so applied in previous cases;[6] it provides: "ART. 147. When a man and a woman who are capacitated to marry each other, live exclusively with each other as husband and wife without the benefit of marriage or under a void marriage, their wages and salaries shall be owned by them in equal shares and the property acquired by both of them through their work or industry shall be governed by the rules on co-ownership. "In the absence of proof to the contrary, properties acquired while they lived together shall be presumed to have been obtained by their joint efforts, work or industry, and shall be owned by them in equal shares. For purposes of this Article, a party who did not participate in the acquisition by the other party of any property shall be deemed to have contributed jointly in the acquisition thereof if the former's efforts consisted in the care and maintenance of the family and of the household. "Neither party can encumber or dispose by acts inter vivos of his or her share in the property acquired during cohabitation and owned in common, without the consent of the other, until after the termination of their cohabitation.

646

University of the Cordilleras College of Law First Year C S.Y. 2013 - 2014 "When only one of the parties to a void marriage is in good faith, the share of the party in bad faith in the co-ownership shall be forfeited in favor of their common children. In case of default of or waiver by any or all of the common children or their descendants, each vacant share shall belong to the respective surviving descendants. In the absence of descendants, such share shall belong to the innocent party. In all cases, the forfeiture shall take place upon termination of the cohabitation." This peculiar kind of co-ownership applies when a man and a woman, suffering no legal impediment to marry each other, so exclusively live together as husband and wife under a void marriage or without the benefit of marriage. The term "capacitated" in the provision (in the first paragraph of the law) refers to the legal capacity of a party to contract marriage, i.e., any "male or female of the age of eighteen years or upwards not under any of the impediments mentioned in Articles 37 and 38"[7] of the Code. Under this property regime, property acquired by both spouses through their work and industry shall be governed by the rules on equal co-ownership. Any property acquired during the union is prima facie presumed to have been obtained through their joint efforts. A party who did not participate in the acquisition of the property shall still be considered as having contributed thereto jointly if said party's "efforts consisted in the care and maintenance of the family household."[8] Unlike the conjugal partnership of gains, the fruits of the couple's separate property are not included in the co-ownership. Article 147 of the Family Code, in substance and to the above extent, has clarified Article 144 of the Civil Code; in addition, the law now expressly provides that (a) Neither party can dispose or encumber by act inter vivos his or her share in co-ownership property, without the consent of the other, during the period of cohabitation; and (b) In the case of a void marriage, any party in bad faith shall forfeit his or her share in the coownership in favor of their common children; in default thereof or waiver by any or all of the common children, each vacant share shall belong to the respective surviving descendants, or still in default thereof, to the innocent party. The forfeiture shall take place upon the termination of the cohabitation[9] or declaration of nullity of the marriage.[10] When the common-law spouses suffer from a legal impediment to marry or when they do not live exclusively with each other (as husband and wife ),only the property acquired by both of them through their actual joint contribution of money, property or industry shall be owned in common and in proportion to their respective contributions. Such contributions and corresponding shares, however, are prima facie presumed to be equal. The share of any party who is married to another shall accrue to the absolute community or conjugal partnership, as the case may be, if so existing under a valid marriage. If the party who has acted in bad faith is not validly married to another, his or her share shall be forfeited in the manner already heretofore expressed.[11] In deciding to take further cognizance of the issue on the settlement of the parties' common property, the trial court acted neither imprudently nor precipitately; a court which has jurisdiction to declare the marriage a nullity must be deemed likewise clothed with authority to resolve incidental and consequential matters. Nor did it commit a reversible error in ruling that petitioner and private respondent own the "family home" and all their common property in equal shares, as well as in concluding that, in the liquidation and partition of the property owned in common by them, the provisions on co-ownership under the Civil Code, not Articles 50, 51 and 52, in relation to Articles 102 and 129,[12] of the Family Code, should aptly prevail. The rules set up to govern the liquidation of either the absolute community or the conjugal partnership of gains, the property regimes recognized for valid and voidable marriages (in the latter case until the contract is annulled ),are irrelevant to the liquidation of the co-ownership that exists between common-law spouses. The first paragraph of Article 50 of the Family Code, applying paragraphs (2 ),(3 ),(4) and (5) of Article 43,[13] relates only, by its explicit terms, to voidable marriages and, exceptionally, to void marriages under Article 40[14] of the Code, i.e., the declaration of nullity of a subsequent marriage contracted by a spouse of a prior void marriage before the latter is judicially declared void. The latter is a special rule that somehow recognizes the philosophy and an old doctrine that void 647

University of the Cordilleras College of Law First Year C S.Y. 2013 - 2014 marriages are inexistent from the very beginning and no judicial decree is necessary to establish their nullity. In now requiring for purposes of remarriage, the declaration of nullity by final judgment of the previously contracted void marriage, the present law aims to do away with any continuing uncertainty on the status of the second marriage. It is not then illogical for the provisions of Article 43, in relation to Articles 41[15] and 42,[16] of the Family Code, on the effects of the termination of a subsequent marriage contracted during the subsistence of a previous marriage to be made applicable pro hac vice. In all other cases, it is not to be assumed that the law has also meant to have coincident property relations, on the one hand, between spouses in valid and voidable marriages (before annulment) and, on the other, between common-law spouses or spouses of void marriages, leaving to ordain, in the latter case, the ordinary rules on co-ownership subject to the provision of Article 147 and Article 148 of the Family Code. It must be stressed, nevertheless, even as it may merely state the obvious, that the provisions of the Family Code on the "family home," i.e., the provisions found in Title V, Chapter 2, of the Family Code, remain in force and effect regardless of the property regime of the spouses. WHEREFORE, the questioned orders, dated 05 May 1995 and 30 October 1995, of the trial court are AFFIRMED. No costs. SO ORDERED.

648

University of the Cordilleras College of Law First Year C S.Y. 2013 - 2014 Case Digest: Valdez vs RTC 260 SCRA 221 G.R. No. 122749 Decided on: July 31, 1996 Ponente: VITUG, J.: FACTS: Antonio Valdez and Consuelo Gomez were married in 1971 and begotten 5 children. Valdez filed a petition in 1992 for a declaration of nullity of their marriage pursuant to Article 36 of the Family Code, which was granted hence, marriage is null and void on the ground of their mutual psychological incapacity. Stella and Joaquin are placed under the custody of their mother while the other 3 siblings are free to choose which they prefer. Gomez sought a clarification of that portion in the decision regarding the procedure for the liquidation of common property i n unions without marriage. During the hearing on the motion, the children filed a joint affidavit expressing desire to stay with their father. ISSUE: Whether or not the property regime should be based on co-ownership. HELD: The Supreme Court ruled that in a void marriage, regardless of the cause thereof, the property relations of the parties are governed by the rules on co-ownership. Any property acquired during the union is prima facie presumed to have been obtained through their joint efforts. A party who did not participate in the acquisition of the property shall be considered as having contributed thereto jointly if said partys efforts consisted in the care and maintenance of the family.

649

University of the Cordilleras College of Law First Year C S.Y. 2013 - 2014 Agapay vs Palang 276 SCRA 340 G.R. No. 116668. July 28, 1997 Full Case ERLINDA A. AGAPAY, petitioner, vs. CARLINA (CORNELIA) V. PALANG and HERMINIA P. DELA CRUZ, respondents. ROMERO, J.: Before us is a petition for review of the decision of the Court of Appeals in CA-G.R. CV No. 24199 entitled Erlinda Agapay v. Carlina (Cornelia) Palang and Herminia P. Dela Cruz dated June 22, 1994 involving the ownership of two parcels of land acquired during the cohabitation of petitioner and private respondents legitimate spouse. Miguel Palang contracted his first marriage on July 16, 1949 when he took private respondent Carlina (or Cornelia) Vallesterol as a wife at the Pozorrubio Roman Catholic Church in Pangasinan. A few months after the wedding, in October 1949, he left to work in Hawaii. Miguel and Carlinas only chi ld, Herminia Palang, was born on May 12, 1950. Miguel returned in 1954 for a year. His next visit to the Philippines was in 1964 and during the entire duration of his year-long sojourn he stayed in Zambales with his brother, not in Pangasinan with his wife and child. The trial court found evidence that as early as 1957, Miguel had attempted to divorce Carlina in Hawaii.[1] When he returned for good in 1972, he refused to live with private respondents, but stayed alone in a house in Pozorrubio, Pangasinan. On July 15, 1973, the then sixty-three-year-old Miguel contracted his second marriage with nineteenyear-old Erlinda Agapay, herein petitioner.[2] Two months earlier, on May 17, 1973, Miguel and Erlinda, as evidenced by the Deed of Sale, jointly purchased a parcel of agricultural land located at San Felipe, Binalonan, Pangasinan with an area of 10,080 square meters. Consequently, Transfer Certificate of Title No. 101736 covering said rice land was issued in their names. A house and lot in Binalonan, Pangasinan was likewise purchased on September 23, 1975, allegedly by Erlinda as the sole vendee. TCT No. 143120 covering said property was later issued in her name. On October 30, 1975, Miguel and Cornelia Palang executed a Deed of Donation as a form of compromise agreement to settle and end a case filed by the latter.[3] The parties therein agreed to donate their conjugal property consisting of six parcels of land to their only child, Herminia Palang.[4] Miguel and Erlindas cohabitation produced a son, Kristopher A. Palang, born on December 6, 1977. In 1979, Miguel and Erlinda were convicted of Concubinage upon Carlinas complaint.[5] Two years later, on February 15, 1981, Miguel died. On July 11, 1981, Carlina Palang and her daughter Herminia Palang de la Cruz, herein private respondents, instituted the case at bar, an action for recovery of ownership and possession with damages against petitioner before the Regional Trial Court in Urdaneta, Pangasinan (Civil Case No. U-4265). Private respondents sought to get back the riceland and the house and lot both located at Binalonan, Pangasinan allegedly purchased by Miguel during his cohabitation with petitioner. Petitioner, as defendant below, contended that while the riceland covered by TCT No. 101736 is registered in their names (Miguel and Erlinda), she had already given her half of the property to their son Kristopher Palang. She added that the house and lot covered by TCT No. 143120 is her sole property, having bought the same with her own money. Erlinda added that Carlina is precluded from claiming aforesaid properties since the latter had already donated their conjugal estate to Herminia.

650

University of the Cordilleras College of Law First Year C S.Y. 2013 - 2014 After trial on the merits, the lower court rendered its decision on June 30, 1989 dismissing the complaint after declaring that there was little evidence to prove that the subject properties pertained to the conjugal property of Carlina and Miguel Palang. The lower court went on to provide for the intestate shares of the parties, particularly of Kristopher Palang, Miguels illegitimate son. The dispositive portion of the decision reads: WHEREFORE, premises considered, judgment is hereby rendered1) Dismissing the complaint, with costs against plaintiffs; 2) Confirming the ownership of defendant Erlinda Agapay of the residential lot located at Poblacion, Binalonan, Pangasinan, as evidenced by TCT No. 143120, Lot 290-B including the old house standing therein; 3) Confirming the ownership of one-half (1/2) portion of that piece of agricultural land situated at Balisa, San Felipe, Binalonan, Pangasinan, consisting of 10,080 square meters and as evidenced by TCT No. 101736, Lot 1123-A to Erlinda Agapay; 4) Adjudicating to Kristopher Palang as his inheritance from his deceased father, Miguel Palang, the onehalf (1/2) of the agricultural land situated at Balisa, San Felipe, Binalonan, Pangasinan, under TCT No. 101736 in the name of Miguel Palang, provided that the former (Kristopher) executes, within 15 days after this decision becomes final and executory, a quit-claim forever renouncing any claims to annul/reduce the donation to Herminia Palang de la Cruz of all conjugal properties of her parents, Miguel Palang and Carlina Vallesterol Palang, dated October 30, 1975, otherwise, the estate of deceased Miguel Palang will have to be settled in another separate action; 5) No pronouncement as to damages and attorneys fees. SO ORDERED.[6] On appeal, respondent court reversed the trial courts decision. The Court of Appeals rendered its decision on July 22, 1994 with the following dispositive portion: WHEREFORE, PREMISES CONSIDERED, the appealed decision is hereby REVERSED and another one entered: 1. Declaring plaintiffs-appellants the owners of the properties in question; 2. Ordering defendant-appellee to vacate and deliver the properties in question to herein plaintiffsappellants; 3. Ordering the Register of Deeds of Pangasinan to cancel Transfer Certificate of Title Nos. 143120 and 101736 and to issue in lieu thereof another certificate of title in the name of plaintiffsappellants. No pronouncement as to costs.[7] Hence, this petition. Petitioner claims that the Court of Appeals erred in not sustaining the validity of two deeds of absolute sale covering the riceland and the house and lot, the first in favor of Miguel Palang and Erlinda Agapay and the second, in favor of Erlinda Agapay alone. Second, petitioner contends that respondent appellate court erred in not declaring Kristopher A. Palang as Miguel Palangs illegitimate son and thus entitled to inherit from Miguels estate. Third, respondent court erred, according to petitioner, in not finding that there is sufficient pleading and evidence that Kristoffer A. Palang or Christopher A. Palang should be

651

University of the Cordilleras College of Law First Year C S.Y. 2013 - 2014 considered as party-defendant in Civil Case No. U-4625 before the trial court and in CA-G.R. No. 24199.[8] After studying the merits of the instant case, as well as the pertinent provisions of law and jurisprudence, the Court denies the petition and affirms the questioned decision of the Court of Appeals. The first and principal issue is the ownership of the two pieces of property subject of this action. Petitioner assails the validity of the deeds of conveyance over the same parcels of land. There is no dispute that the transfers of ownership from the original owners of the riceland and the house and lot, Corazon Ilomin and the spouses Cespedes, respectively, were valid. The sale of the riceland on May 17, 1973, was made in favor of Miguel and Erlinda. The provision of law applicable here is Article 148 of the Family Code providing for cases of cohabitation when a man and a woman who are not capacitated to marry each other live exclusively with each other as husband and wife without the benefit of marriage or under a void marriage. While Miguel and Erlinda contracted marriage on July 15, 1973, said union was patently void because the earlier marriage of Miguel and Carlina was still susbsisting and unaffected by the latters de facto separation. Under Article 148, only the properties acquired by both of the parties through their actual joint contribution of money, property or industry shall be owned by them in common in proportion to their respective contributions. It must be stressed that actual contribution is required by this provision, in contrast to Article 147 which states that efforts in the care and maintenance of the family and household, are regarded as contributions to the acquisition of common property by one who has no salary or income or work or industry. If the actual contribution of the party is not proved, there will be no co-ownership and no presumption of equal shares.[9] In the case at bar, Erlinda tried to establish by her testimony that she is engaged in the business of buy and sell and had a sari-sari store[10] but failed to persuade us that she actually contributed money to buy the subject riceland. Worth noting is the fact that on the date of conveyance, May 17, 1973, petitioner was only around twenty years of age and Miguel Palang was already sixty-four and a pensioner of the U.S. Government. Considering her youthfulness, it is unrealistic to conclude that in 1973 she contributed P3,750.00 as her share in the purchase price of subject property,[11] there being no proof of the same. Petitioner now claims that the riceland was bought two months before Miguel and Erlinda actually cohabited. In the nature of an afterthought, said added assertion was intended to exclude their case from the operation of Article 148 of the Family Code. Proof of the precise date when they commenced their adulterous cohabitation not having been adduced, we cannot state definitively that the riceland was purchased even before they started living together. In any case, even assuming that the subject property was bought before cohabitation, the rules of co-ownership would still apply and proof of actual contribution would still be essential. Since petitioner failed to prove that she contributed money to the purchase price of the riceland in Binalonan, Pangasinan, we find no basis to justify her co-ownership with Miguel over the same. Consequently, the riceland should, as correctly held by the Court of Appeals, revert to the conjugal partnership property of the deceased Miguel and private respondent Carlina Palang. Furthermore, it is immaterial that Miguel and Carlina previously agreed to donate their conjugal property in favor of their daughter Herminia in 1975. The trial court erred in holding that the decision adopting their compromise agreement in effect partakes the nature of judicial confirmation of the separation of property between spouses and the termination of the conjugal partnership.[12] Separation of property between spouses during the marriage shall not take place except by judicial order or without judicial conferment when there is an express stipulation in the marriage settlements.[13] The judgment which resulted from the parties compromise was not specifically and expressly for separation of property and should not be so inferred.

652

University of the Cordilleras College of Law First Year C S.Y. 2013 - 2014 With respect to the house and lot, Erlinda allegedly bought the same for P20,000.00 on September 23, 1975 when she was only 22 years old. The testimony of the notary public who prepared the deed of conveyance for the property reveals the falsehood of this claim. Atty. Constantino Sagun testified that Miguel Palang provided the money for the purchase price and directed that Erlindas name alone be placed as the vendee.[14] The transaction was properly a donation made by Miguel to Erlinda, but one which was clearly void and inexistent by express provision of law because it was made between persons guilty of adultery or concubinage at the time of the donation, under Article 739 of the Civil Code. Moreover, Article 87 of the Family Code expressly provides that the prohibition against donations between spouses now applies to donations between persons living together as husband and wife without a valid marriage,[15] for otherwise, the condition of those who incurred guilt would turn out to be better than those in legal union.[16] The second issue concerning Kristopher Palangs status and claim a s an illegitimate son and heir to Miguels estate is here resolved in favor of respondent courts correct assessment that the trial court erred in making pronouncements regarding Kristophers heirship and filiation inasmuch as questions as to who are the heirs of the decedent, proof of filiation of illegitimate children and the determination of the estate of the latter and claims thereto should be ventilated in the proper probate court or in a special proceeding instituted for the purpose and cannot be adjudicated in the instant ordinary civil action which is for recovery of ownership and possession.[17] As regards the third issue, petitioner contends that Kristopher Palang should be considered as partydefendant in the case at bar following the trial courts decision which expressly found that Kristopher had not been impleaded as party defendant but theorized that he had submitted to the courts jurisdiction through his mother/guardian ad litem.[18] The trial court erred gravely. Kristopher, not having been impleaded, was, therefore, not a party to the case at bar. His mother, Erlinda, cannot be called his guardian ad litem for he was not involved in the case at bar. Petitioner adds that there is no need for Kristopher to file another action to prove that he is the illegitimate son of Miguel, in order to avoid multiplicity of suits.[19] Petitioners grave error has been discussed in the preceeding paragraph where the need for probate proceedings to resolve the settlement of Miguels estate and Kristopher s successional rights has been pointed out. WHEREFORE, the instant petition is hereby DENIED. The questioned decision of the Court of Appeals is AFFIRMED. Costs against petitioner. SO ORDERED.

653

University of the Cordilleras College of Law First Year C S.Y. 2013 - 2014 Case Digest: Agapay vs Palang 276 SCRA 340 G.R. No. 116668. Decided on: July 28, 1997 Facts: Miguel Palang contracted marriage with Carlina in Pangasinan on 1949. He left to work in Hawaii a few months after the wedding. Their only child Herminia was born in May 1950. The trial court found evident that as early as 1957, Miguel attempted to Divorce Carlina in Hawaii. When he returned for good in 1972, he refused to live with Carlina and stayed alone in a house in Pozzorubio Pangasinan. The 63 year old Miguel contracted a subsequent marriage with 19 year old Erlinda Agapay, herein petitioner. 2 months earlier, they jointly purchased a parcel of agricultural land located at Binalonan Pangasinan. A house and lot in the same place was likewise purchased. On the other hand, Miguel and Carlina executed a Deed of Donation as a form of compromise agreement and agreed to donate their conjugal property consisting of 6 parcels of land to their child Herminia. Miguel and Erlindas cohabitation produced a son named Kristopher. In 1979, they were convicted of concubinage upon Carlinas complaint. 2 years later, Miguel died. Carlina and her daughter instituted this case for recovery of ownership and possession with damages against petitioner. They sought to get back the land and the house and lot located at Binalonan allegedly purchase by Miguel during his cohabitation with petitioner. The lower court dismissed the complaint but CA reversed the decision. ISSUE: Whether the agricultural land and the house and lot should be awarded in favor of Erlinda Agapay. HELD: The sale of the riceland on May 17, 1973, was made in favor of Miguel and Erlinda. However, their marriage is void because of the subsisting marriage with Carlina. Only the properties acquired by both parties through their actual joint contribution shall be owned by them in proportion to their respective contributions. It is required that there be an actual contribution. If actual contribution is not proved, there will be no co-ownership and no presumption of equal shares. Erlinda established in her testimony that she was engaged in the business of buy and sell and had a sarisari store. However, she failed to persuade the court that she actually contributed money to but the subjected riceland. When the land was acquired, she was only around 20 years old compared to Miguel who was already 64 years old and a pensioner of the US Government. Considering his youthfulness, its unrealistic how she could have contributed the P3,750 as her share. Thus, the court finds no basis to justify the co-ownership with Miguel over the same. Hence, the Riceland should, as correctly held by CA, revert to the conjugal partnership property of the deceased and Carlina. It is immaterial that Miguel and Carlina previously agreed to donate their conjugal property in favor of Herminia. Separation of property between spouses during the marriage shall not take place except by judicial order or without judicial conferment when there is an express stipulation in the marriage settlements. The judgment resulted from the compromise was not specifically for separation of property and should not be so inferred. With respect to the house and lot, Atty Sagun, notary public who prepared the deed of conveyance for the property revealed the falsehood of Erlindas claim that she bought such property for P20,000 when she was 22 years old. The lawyer testified that Miguel provided the money for the purchase price and directed Erlindas name alone be placed as the vendee.

654

University of the Cordilleras College of Law First Year C S.Y. 2013 - 2014 The transaction made by Miguel to Erlinda was properly a donation and which was clearly void and inexistent by express provision of the law because it was made between persons guilty of adultery or concubinage at the time of the donation. Moreover, Article 87 of the Family Code, expressly provides that the prohibition against donation between spouses now applies to donations between persons living together as husband and wife without a valid marriage, for otherwise, the condition of those who incurred guilt would turn out to be better than those in legal union.

655

University of the Cordilleras College of Law First Year C S.Y. 2013 - 2014 Tumlos vs Fernandez 330 SCRA 718 G.R. No. 137650. April 12, 2000 Full Case GUILLERMA TUMLOS, petitioner, vs. SPOUSES MARIO FERNANDEZ and LOURDES FERNANDEZ, respondents. PANGANIBAN, J.: Under Article 148 of the Family Code, a man and a woman who are not legally capacitated to marry each other, but who nonetheless live together conjugally, may be deemed co-owners of a property acquired during the cohabitation only upon proof that each made an actual contribution to its acquisition. Hence, mere cohabitation without proof of contribution will not result in a co-ownership. The Case Before us is a Petition for Review under Rule 45 of the Rules of Court, assailing the November 19, 1998 Decision of the Court of Appeals[1] (CA), which reversed the October 7, 1997 Order of the Regional Trial Court (RTC).[2] The dispositive part of the CA Decision reads: Jur-is "WHEREFORE, the instant petition is GRANTED, and the questioned orders of the court a quo dated October 7, 1997 and November 11, 1997, are hereby REVERSED and SET ASIDE. The judgment of the court a quo dated June 5, 1997 is hereby REINSTATED. Costs against the private respondents."[3] The assailed Order of the RTC disposed as follows: Supr-ema "Wherefore, the decision of this Court rendered on June 5, 1997 affirming in toto the appealed judgment of the [MTC] is hereby reconsidered and a new one is entered reversing said decision of the [MTC] and dismissing the complaint in the above-entitled case."[4] Petitioner also assails the February 14, 1999 CA Resolution denying the Motion for Reconsideration. The Facts The Court of Appeals narrates the facts as follows: "[Herein respondents] were the plaintiffs in Civil Case No. 6756, an action for ejectment filed before Branch 82 of the MTC of Valenzuela, Metro Manila against [herein Petitioner] Guillerma Tumlos, Toto Tumlos, and Gina Tumlos. In their complaint dated July 5, 1996, the said spouses alleged that they are the absolute owners of an apartment building located at ARTE SUBDIVISION III, Lawang Bato, Valenzuela, Metro Manila; that through tolerance they had allowed the defendants-private respondents to occupy the apartment building for the last seven (7) years, since 1989, without the payment of any rent; that it was agreed upon that after a few months, defendant Guillerma Tumlos will pay P1,600.00 a month while the other defendants promised to pay P1,000.00 a month, both as rental, which agreement was not complied with by the said defendants; that they have demanded several times [that] the defendants x x x vacate the premises, as they are in need of the property for the construction of a new building; and that they have also demanded payment of P84,000.00 from Toto and Gina Tumlos representing rentals for seven (7) years and payment of P143,600.00 from Guillerma Tumlos as unpaid rentals for seven (7) years, but the said demands went unheeded. They then prayed that the defendants be ordered to vacate the property in question and to pay the stated unpaid rentals, as well as to jointly pay P30,000.00 in attorneys fees. "[Petitioner] Guillerma Tumlos was the only one who filed an answer to the complaint. She averred therein that the Fernandez spouses had no cause of action against her, since she is a co-owner of the subject premises as evidenced by a Contract to Sell wherein it was stated that she is a co-vendee of the

656

University of the Cordilleras College of Law First Year C S.Y. 2013 - 2014 property in question together with [Respondent] Mario Fernandez. She then asked for the dismissal of the complaint. "After an unfruitful preliminary conference on November 15, 1996, the MTC required the parties to submit their affidavits and other evidence on the factual issues defined in their pleadings within ten (10) days from receipt of such order, pursuant to section 9 of the Revised Rule on Summary Procedure. [Petitioner] Guillerma Tumlos submitted her affidavit/position paper on November 29, 1996, while the [respondents] filed their position paper on December 5, 1996, attaching thereto their marriage contract, letters of demand to the defendants, and the Contract to Sell over the disputed property. The MTC thereafter promulgated its judgment on January 22, 1997[.]Scs-daad xxxxxxxxx "Upon appeal to the [RTC], [petitioner and the two other] defendants alleged in their memorandum on appeal that [Respondent] Mario Fernandez and [Petitioner] Guillerma had an amorous relationship, and that they acquired the property in question as their love nest. It was further alleged that they lived together in the said apartment building with their two (2) children for around ten(10) years, and that Guillerma administered the property by collecting rentals from the lessees of the other apartments, until she discovered that [Respondent Mario] deceived her as to the annulment of his marriage. It was also during the early part of 1996 when [Respondent Mario] accused her of being unfaithful and demonstrated his baseless [jealousy]. "In the same memorandum, [petitioner and the two other] defendants further averred that it was only recently that Toto Tumlos was temporarily accommodated in one of the rooms of the subject premises while Gina Tumlos acted as a nanny for the children. In short, their presence there [was] only transient and they [were] not tenants of the Fernandez spouses. "On June 5, 1997, the [RTC] rendered a decision affirming in toto the judgment of the MTC. S-daad "The [petitioner and the two other defendants] seasonably filed a motion for reconsideration on July 3, 1997, alleging that the decision of affirmance by the RTC was constitutionally flawed for failing to point out distinctly and clearly the findings of facts and law on which it was based vis--vis the statements of issues they have raised in their memorandum on appeal. They also averred that the Contract to Sell presented by the plaintiffs which named the buyer as Mario P. Fernandez, of legal age, marri ed to Lourdes P. Fernandez, should not be given credence as it was falsified to appear that way. According to them, the Contract to Sell originally named Guillerma Fernandez as the spouse of [Respondent Mario]. As found by the [RTC] in its judgment, a new Contract to Sell was issued by the sellers naming the [respondents] as the buyers after the latter presented their marriage contract and requested a change in the name of the vendee-wife. Such facts necessitate the conclusion that Guillerma was really a co-owner thereof, and that the [respondents] manipulated the evidence in order to deprive her of her rights to enjoy and use the property as recognized by law. Sd-aamiso xxxxxxxxx "The [RTC], in determining the question of ownership in order to resolve the issue of possession, ruled therein that the Contract to Sell submitted by the Fernandez spouses appeared not to be authentic, as there was an alteration in the name of the wife of [Respondent] Mario Fernandez. Hence, the contract presented by the [respondents] cannot be given any weight. The court further ruled that Guillerma and [Respondent Mario] acquired the property during their cohabitation as husband and wife, although without the benefit of marriage. From such findings, the court concluded that [Petitioner] Guillerma Tumlos was a co-owner of the subject property and could not be ejected therefrom. "The [respondents] then filed a motion for reconsideration of the order of reversal, but the same was denied by the [RTC]."[5] As earlier stated, the CA reversed the RTC. Hence, this Petition filed by Guillerma Tumlos only.[6] 657

University of the Cordilleras College of Law First Year C S.Y. 2013 - 2014 Ruling of the Court of Appeals The CA rejected petitioners claim that she and Respondent Mario Fernandez were co -owners of the disputed property. The CA ruled: Scnc-m "From the inception of the instant case, the only defense presented by private respondent Guillerma is her right as a co-owner of the subject property[.] xxxxxxxxx This claim of co-ownership was not satisfactorily proven by Guillerma, as correctly held by the trial court. No other evidence was presented to validate such claim, except for the said affidavit/position paper. As previously stated, it was only on appeal that Guillerma alleged that she cohabited with the petitionerhusband without the benefit of marriage, and that she bore him two (2) children. Attached to her memorandum on appeal are the birth certificates of the said children. Such contentions and documents should not have been considered by the x x x (RTC), as they were not presented in her affidavit/position paper before the trial court (MTC). xxxxxxxxx "However, even if the said allegations and documents could be considered, the claim of co-ownership must still fail. As [herein Respondent] Mario Fernandez is validly married to [Respondent] Lourdes Fernandez (as per Marriage Contract dated April 27, 1968, p. 45, Original Record), Guillerma and Mario are not capacitated to marry each other. Thus, the property relations governing their supposed cohabitation is that found in Article 148 of Executive Order No. 209, as amended, otherwise known as the Family Code of the Philippines[.] xxxxxxxxx "It is clear that actual contribution is required by this provision, in contrast to Article 147 of the Family Code which states that efforts in the care and maintenance of the family and household are regarded as contributions to the acquisition of common property by one who has no salary or income or work or industry (Agapay v. Palang, 276 SCRA 340). The care given by one party [to] the home, children, and household, or spiritual or moral inspiration provided to the other, is not included in Article 148 (Handbook on the Family Code of the Philippines by Alicia V. Sempio-Diy, 1988 ed., p. 209). Hence, if actual contribution of the party is not proved, there will be no co-ownership and no presumption of equal shares (Agapay, supra at p. 348, citing Commentaries and Jurisprudence on the Civil Code of the Philippines Volume I by Arturo M. Tolentino, 1990 ed., p. 500). "In the instant case, no proof of actual contribution by Guillerma Tumlos in the purchase of the subject property was presented. Her only evidence was her being named in the Contract to Sell as the wife of [Respondent] Mario Fernandez. Since she failed to prove that she contributed money to the purchase price of the subject apartment building, We find no basis to justify her co-ownership with [Respondent Mario]. The said property is thus presumed to belong to the conjugal partnership property of Mario and Lourdes Fernandez, it being acquired during the subsistence of their marriage and there being no other proof to the contrary (please see Article 116 of the Family Code). "The court a quo (RTC) also found that [Respondent Mario] has two (2) children with Guillerma who are in her custody, and that to eject them from the apartment building would be to run counter with the obligation of the former to give support to his minor illegitimate children, which indispensably includes dwelling. As previously discussed, such finding has no leg to stand on, it being based on evidence presented for the first time on appeal. Nc-mmis xxxxxxxxx

658

University of the Cordilleras College of Law First Year C S.Y. 2013 - 2014 "Even assuming arguendo that the said evidence was validly presented, the RTC failed to consider that the need for support cannot be presumed. Article 203 of the Family Code expressly provides that the obligation to give support shall be demandable from the time the person who has a right to receive the same needs it for maintenance, but it shall not be paid except from the date of judicial or extrajudicial demand. x x x. Nc-m "In contrast to the clear pronouncement of the Supreme Court, the RTC instead presumed that Guillerma and her children needed support from [Respondent Mario]. Worse, it relied on evidence not properly presented before the trial court (MTC). "With regard to the other [defendants], Gina and Toto Tumlos, a close perusal of the records shows that they did not file any responsive pleading. Hence, judgment may be rendered against them as may be warranted by the facts alleged in the complaint and limited to what is prayed for therein, as provided for in Section 6 of the Revised Rules on Summary Procedure. There was no basis for the public respondent to dismiss the complaint against them."[7] (emphasis in the original) Ol-dmiso The Issues In her Memorandum, petitioner submits the following issues for the consideration of the Court: "I. The Court of Appeals gravely erred and abused its discretion in not outrightly dismissing the petition for review filed by respondents. "II. The Court of Appeals erred in finding that petitioner is not the co-owner of the property in litis. "III. Corollary thereto, the Court of Appeals erred in applying Art. 148 of the Family Code in the case at bar. Man-ikan "IV. The Court of Appeals erred in disregarding the substantive right of support vis--vis the remedy of ejectment resorted to by respondents."[8] In resolving this case, we shall answer two questions: (a) Is the petitioner a co-owner of the property? (b) Can the claim for support bar this ejectment suit? We shall also discuss these preliminary matters: (a) whether the CA was biased in favor of respondents and (b) whether the MTC had jurisdiction over the ejectment suit. Manik-s The Courts Ruling The Petition has no merit. Preliminary Matters Petitioner submits that the CA exhibited partiality in favor of herein respondents. This bias, she argues, is manifest in the following: Man-ikx 1. The CA considered the respondents Petition for Review[9] despite their failure to attach several pleadings as well as the explanation for the proof of service, despite the clear mandate of Section 11[10] of Rule 13 of the Revised Rules of Court and despite the ruling in Solar Team Entertainment, Inc. v. Ricafort.[11] 2. It allowed respondents to submit the pleadings that were not attached. 3. It considered respondents Reply dated May 20, 1998, which had allegedly been filed out of time. Ne xold 4. It declared that the case was submitted for decision without first determining whether to give due course to the Petition, pursuant to Section 6, Rule 42 of the Rules of Court.[12] 659

University of the Cordilleras College of Law First Year C S.Y. 2013 - 2014 The CA, for its part, succinctly dismissed these arguments in this wise: Mi-so "It is too late in the day now to question the alleged procedural error after we have rendered the decision. More importantly, when the private respondent filed their comment to the petition on April 26, 1998, they failed to question such alleged procedural error. Neither have they questioned all the resolutions issued by the Court after their filing of such comment. They should, therefore, be now considered in estoppel to question the same."[13] We agree with the appellate court. Petitioner never raised these matters before the CA. She cannot be allowed now to challenge its Decision on grounds of alleged technicalities being belatedly raised as an afterthought. In this light, she cannot invoke Solar[14] because she never raised this issue before the CA. Spp-edjo More important, we find it quite sanctimonious indeed on petitioners part to rely, on the one hand, on these procedural technicalities to overcome the appealed Decision and, on the other hand, assert that the RTC may consider the new evidence she presented for the first time on appeal. Such posturing only betrays the futility of petitioners assertion, if not its absence of merit. One other preliminary matter. Petitioner implies that the court of origin, the Municipal Trial Court (MTC), did not have jurisdiction over the "nature of the case," alleging that the real question involved is one of ownership. Since the issue of possession cannot be settled without passing upon that of ownership, she maintains that the MTC should have dismissed the case. Josp-ped This contention is erroneous. The issue of ownership may be passed upon by the MTC to settle the issue of possession.[15] Such disposition, however, is not final insofar as the issue of ownership is concerned,[16] which may be the subject of another proceeding brought specifically to settle that question. Having resolved these preliminary matters, we now move on to petitioners substantive contentions. Spped First Issue: Petitioner as Co-owner Petitioners central theory and main defense against respondents action for ejectment is her claim of co ownership over the property with Respondent Mario Fernandez. At the first instance before the MTC, she presented a Contract to Sell indicating that she was his spouse. The MTC found this document insufficient to support her claim. The RTC, however, after considering her allegation that she had been cohabiting with Mario Fernandez as shown by evidence presented before it,[17] ruled in her favor. Misspped On the other hand, the CA held that the pieces of evidence adduced before the RTC could no longer be considered because they had not been submitted before the MTC. Hence, the appellate court concluded that "[t]he claim of co-ownership was not satisfactorily proven x x x."[18] We agree with the petitioner that the RTC did not err in considering the evidence presented before it. Nonetheless, we reject her claim that she was a co-owner of the disputed property. Missc Evidence Presented on Appeal Before the RTC In ruling that the RTC erred in considering on appeal the evidence presented by petitioner, the CA relied on the doctrine that issues not raised during trial could not be considered for the first time during appeal.[19] We disagree. In the first place, there were no new matters or issues belatedly raised during the appeal before the RTC. The defense invoked by petitioner at the very start was that she was a co-owner. To support her claim, she presented a Contract to Sell dated November 14, 1986, which stated that Mario 660

University of the Cordilleras College of Law First Year C S.Y. 2013 - 2014 Fernandez was legally married to her. The allegation that she was cohabiting with him was a mere elaboration of her initial theory. In the second place, procedural rules are generally premised on considerations of fair play. Respondents never objected when the assailed evidence was presented before the RTC. Thus, they cannot claim unfair surprise or prejudice. Scmis Petitioner Not a Co-Owner Under Article 144 of the Civil Code Even considering the evidence presented before the MTC and the RTC, we cannot accept petitioners submission that she is a co-owner of the disputed property pursuant to Article 144 of the Civil Code.[20] As correctly held by the CA, the applicable law is not Article 144 of the Civil Code, but Article 148 of the Family Code which provides: "Art. 148. In cases of cohabitation not falling under the preceding Article,[21] only the properties acquired by both of the parties through their actual joint contribution of money, property, or industry shall be owned by them in common in proportion to their respective contributions. In the absence of proof to the contrary, their contributions and corresponding shares are presumed to be equal. The same rule and presumption shall apply to joint deposits of money and evidences of credit. "If one of the parties is validly married to another, his or her share in the co-ownership shall accrue to the absolute community or conjugal partnership existing in such valid marriage. If the party who acted in bad faith is not validly married to another, his or her share shall be forfeited in the manner provided in the last paragraph of the preceding Article. "The foregoing rules on forfeiture shall likewise apply even if both parties are in bad faith." Sc Article 144 of the Civil Code applies only to a relationship between a man and a woman who are not incapacitated to marry each other,[22] or to one in which the marriage of the parties is void[23] from the beginning.[24] It does not apply to a cohabitation that amounts to adultery or concubinage, for it would be absurd to create a co-ownership where there exists a prior conjugal partnership or absolute community between the man and his lawful wife.[25] Based on evidence presented by respondents, as well as those submitted by petitioner herself before the RTC, it is clear that Mario Fernandez was incapacitated to marry petitioner because he was legally married to Lourdes Fernandez. It is also clear that, as readily admitted by petitioner, she cohabited with Mario in a state of concubinage. Therefore, Article 144 of the Civil Code is inapplicable. As stated above, the relationship between petitioner and Respondent Mario Fernandez is governed by Article 148 of the Family Code. Justice Alicia V. Sempio-Diy points out[26] that "[t]he Family Code has filled the hiatus in Article 144 of the Civil Code by expressly regulating in its Article 148 the property relations of couples living in a state of adultery or concubinage." x-sc Hence, petitioners argument -- that the Family Code is inapplicable because the cohabitation and the acquisition of the property occurred before its effectivity -- deserves scant consideration. Suffice it to say that the law itself states that it can be applied retroactively if it does not prejudice vested or acquired rights.[27] In this case, petitioner failed to show any vested right over the property in question. Moreover, to resolve similar issues, we have applied Article 148 of the Family Code retroactively.[28] No Evidence of Actual Joint Contribution Another consideration militates against petitioners claim that she is a co -owner of the property. In Agapay,[29] the Court ruled: "Under Article 148, only the properties acquired by both of the parties through their actual joint contribution of money, property or industry shall be owned by them in common in proportion to their 661

University of the Cordilleras College of Law First Year C S.Y. 2013 - 2014 respective contributions. It must be stressed that the actual contribution is required by this provision, in contrast to Article 147 which states that efforts in the care and maintenance of the family and household, are regarded as contributions to the acquisition of common property by one who has no salary or income or work or industry. If the actual contribution of the party is not proved, there will be no co-ownership and no presumption of equal shares." (emphasis ours) xl-aw In this case, petitioner fails to present any evidence that she had made an actual contribution to purchase the subject property. Indeed, she anchors her claim of co-ownership merely on her cohabitation with Respondent Mario Fernandez. Likewise, her claim of having administered the property during the cohabitation is unsubstantiated. In any event, this fact by itself does not justify her claim, for nothing in Article 148 of the Family Code provides that the administration of the property amounts to a contribution in its acquisition. Clearly, there is no basis for petitioners claim of co-ownership. The property in question belongs to the conjugal partnership of respondents. Hence, the MTC and the CA were correct in ordering the ejectment of petitioner from the premises. Sc-lex Second Issue: Support versus Ejectment Petitioner contends that since Respondent Mario Fernandez failed to repudiate her claim regarding the filiation of his alleged sons, Mark Gil and Michael Fernandez, his silence on the matter amounts to an admission. Arguing that Mario is liable for support, she advances the theory that the childrens right to support, which necessarily includes shelter, prevails over the right of respondents to eject her. We disagree. It should be emphasized that this is an ejectment suit whereby respondents seek to exercise their possessory right over their property. It is summary in character and deals solely with the issue of possession of the property in dispute. Here, it has been shown that they have a better right to possess it than does the petitioner, whose right to possess is based merely on their tolerance. Scl-aw Moreover, Respondent Mario Fernandez alleged failure to repudiate petitioners claim of filiation is not relevant to the present case. Indeed, it would be highly improper for us to rule on such issue. Besides, it was not properly taken up below.[30] In any event, Article 298[31] of the Civil Code requires that there should be an extrajudicial demand.[32] None was made here. The CA was correct when it said: "Even assuming arguendo that the said evidence was validly presented, the RTC failed to consider that the need for support cannot be presumed. Article [298] of the [New Civil Code] expressly provides that the obligation to give support shall be demandable from the time the person who has a right to receive the same need it for maintenance, but it shall not be paid except from the date of judicial and extrajudicial demand."[33] WHEREFORE, the Petition is DENIED and the appealed Decision AFFIRMED. Costs against petitioner. Rtc-spped SO ORDERED.

662

University of the Cordilleras College of Law First Year C S.Y. 2013 - 2014 Case Digest: Tumlos vs Fernandez 330 SCRA 718 G.R. No. 137650. Decided on: April 12, 2000 Ponente: PANGANIBAN, J.: FACTS: Mario and Lourdes Fernandez were plaintiffs in an action for ejectment filed against Guillerma, Gina and Toto Tumlos. In the complaint, spouses Fernandez alleged that they are the absolute owners of an apartment building that through their tolerance they allowed the Tumlos to occupy the apartment for the last 7 years without payment of any rent. It was agreed that Guillerma will pay 1,600 a month while the other defendants promised to pay 1,000 a month which was not complied with. Demand was made several times for the defendants to vacate the premises as they are in need of the property for the construction of a new building. Defendants appealed to RTC that Mario and Guillerma had an amorous relationship and that they acquired the property in question as their love nest. It was likewise alleged that they lived together in the said apartment building with their 2 children for about 10 years and that Gullerma administered the property by collecting rentals from the lessees until she discovered that Mario deceived her as to the annulment of their marriage. ISSUE: WON Guillerma is a co-owner of the said apartment under Article 148. HELD: SC rejected the claim that Guillerma and Mario were co-owners of the subject property. The claim was not satisfactorily proven by Guillerma since there were no other evidence presented to validate it except for the said affidavit. Even if the allegations of having cohabited with Mario and that she bore him two children were true, the claim of co-ownership still cannot be accepted. Mario is validly married with Lourdes hence Guillerma and Mario are not capacitated to marry each other. The property relation governing their supposed cohabitation is under Article 148 of the Family Code. Actual contribution is required by the said provision in contrast to Art 147 which states that efforts in the care and maintenance of the family and household are regarded as contributions to the acquisitions of common property by one who has no salary, income, work or industry. Such is not included in Art 148. If actual contribution is not proven then there can be no co-ownership and no presumption of equal shares.

663

University of the Cordilleras College of Law First Year C S.Y. 2013 - 2014 Malilin Jr. vs Castillo 351 SCRA 127 G.R. No. 136803 June 16, 2000 Full Case EUSTAQUIO MALLILIN, JR., petitioner, vs. MA. ELVIRA CASTILLO, respondent. MENDOZA, J.: This is a petition for review of the amended decision1 of the Court of Appeals dated May 7, 1998 in CA G.R. CV No. 48443 granting respondent's motion for reconsideration of its decision dated November 7, 1996, and of the resolution dated December 21, 1998 denying petitioner's motion for reconsideration. The factual and procedural antecedents are as follows: On February 24, 1993, petitioner Eustaquio Mallilin, Jr. filed a complaint2 for "Partition and/or Payment of Co-Ownership Share, Accounting and Damages" against respondent Ma. Elvira Castillo. The complaint, docketed as Civil Case No. 93-656 at the Regional Trial Court in Makati City, alleged that petitioner and respondent, both married and with children, but separated from their respective spouses, cohabited after a brief courtship sometime in 1979 while their respective marriages still subsisted. During their union, they set up the Superfreight Customs Brokerage Corporation, with petitioner as president and chairman of the board of directors, and respondent as vice-president and treasurer. The business flourished and petitioner and respondent acquired real and personal properties which were registered solely in respondent's name. In 1992, due to irreconcilable differences, the couple separated. Petitioner demanded from respondent his share in the subject properties, but respondent refused alleging that said properties had been registered solely in her name. In her Amended Answer,3 respondent admitted that she engaged in the customs brokerage business with petitioner but alleged that the Superfreight Customs Brokerage Corporation was organized with other individuals and duly registered with the Securities and Exchange Commission in 1987. She denied that she and petitioner lived as husband and wife because the fact was that they were still legally married to their respective spouses. She claimed to be the exclusive owner of all real personal properties involved in petitioner's action for partition on the ground that they were acquired entirely out of her own money and registered solely in her name. On November 25, 1994, respondent filed a Motion for Summary Judgment,4 in accordance with Rule 34 of the Rules of Court.5 She contended that summary judgment was proper, because the issues raised in the pleadings were sham and not genuine, to wit: A. The main issue is Can plaintiff validly claim the partition and/or payment of co-ownership share, accounting and damages, considering that plaintiff and defendant are admittedly both married to their respective spouses under still valid and subsisting marriages, even assuming as claimed by plaintiff, that they lived together as husband and wife without benefit of marriage? In other words, can the parties be considered as co-owners of the properties, under the law, considering the present status of the parties as both married and incapable of marrying each other, even assuming that they lived together as husband and wife (?) B. As a collateral issue, can the plaintiff be considered as an unregistered co-owner of the real properties under the Transfer Certificates of Title duly registered solely in the name of defendant Ma. Elvira Castillo? This issue is also true as far as the motor vehicles in question are concerned which are also registered in the name of defendant.6

664

University of the Cordilleras College of Law First Year C S.Y. 2013 - 2014 On the first point, respondent contended that even if she and petitioner actually cohabited, petitioner could not validly claim a part of the subject real and personal properties because Art. 144 of the Civil Code, which provides that the rules on co-ownership shall govern the properties acquired by a man and a woman living together as husband and wife but not married, or under a marriage which is void ab initio, applies only if the parties are not in any way incapacitated to contract marriage.7 In the parties' case, their union suffered the legal impediment of a prior subsisting marriage. Thus, the question of fact being raised by petitioner, i.e., whether they lived together as husband and wife, was irrelevant as no co-ownership could exist between them. As to the second issue, respondent maintained that petitioner cannot be considered an unregistered coowner of the subject properties on the ground that, since titles to the land are solely in her name, to grant petitioner's prayer would be to allow a collateral attack on the validity of such titles. Petitioner opposed respondent's Motion for Summary Judgment.8 He contended that the case presented genuine factual issues and that Art. 144 of the Civil Code had been repealed by the Family Code which now allows, under Art. 148, a limited co-ownership even though a man and a woman living together are not capacitated to marry each other. Petitioner also asserted that an implied trust was constituted when he and respondent agreed to register the properties solely in the latter's name although the same were acquired out of the profits made from their brokerage business. Petitioner invoked the following provisions of the Civil Code: Art. 1452. If two or more persons agree to purchase property and by common consent the legal title is taken in the name of one of them for the benefit of all, a trust is created by force of law in favor of the others in proportion to the interest of each. Art. 1453. When the property is conveyed to a person in reliance upon his declared intention to hold it for, or transfer it to another grantor, there is an implied trust in favor of the person whose benefit is contemplated. On January 30, 1995, the trial court rendered its decision9 granting respondent's motion for summary judgment. It ruled that an examination of the pleadings shows that the issues involved were purely legal. The trial court also sustained respondent's contention that petitioner's action for partition amounted to a collateral attack on the validity of the certificates of title covering the subject properties. It held that even if the parties really had cohabited, the action for partition could not be allowed because an action for partition among co-owners ceases to be so and becomes one for title if the defendant, as in the present case, alleges exclusive ownership of the properties in question. For these reasons, the trial court dismissed Civil Case No. 93-656. On appeals, the Court of Appeals on November 7, 1996, ordered the case remanded to the court of origin for trial on the merits. It cited the decision in Roque v. Intermediate Appellate Court 10 to the effect that an action for partition is at once an action for declaration of co-ownership and for segregation and conveyance of a determinate portion of the properties involved. If the defendant asserts exclusive title over the property, the action for partition should not be dismissed. Rather, the court should resolve the case and if the plaintiff is unable to sustain his claimed status as a co-owner, the court should dismiss the action, not because the wrong remedy was availed of, but because no basis exists for requiring the defendant to submit to partition. Resolving the issue whether petitioner's action for partition was a collateral attack on the validity of the certificates of title, the Court of Appeals held that since petitioner sought to compel respondent to execute documents necessary to effect transfer of what he claimed was his share, petitioner was not actually attacking the validity of the titles but in fact, recognized their validity. Finally, the appellate court upheld petitioner's position that Art. 144 of the Civil Code had been repealed by Art. 148 of the Family Code. Respondent moved for reconsideration of the decision of Court of Appeals. On May 7, 1998, nearly two years after its first decision, the Court of Appeals granted respondent's motion and reconsidered its prior decision. In its decision now challenged in the present petition, it held 665

University of the Cordilleras College of Law First Year C S.Y. 2013 - 2014 Prefatorily, and to better clarify the controversy on whether this suit is a collateral attack on the titles in issue, it must be underscored that plaintiff-appellant alleged in his complaint that all the nine (9) titles are registered in the name of defendant-appellee, Ma. Elvira T. Castillo, except one which appears in the name of Eloisa Castillo (see par. 9, Complaint). However, a verification of the annexes of such initiatory pleading shows some discrepancies, to wit: 1. TCT No. 149046 (Annex A) = Elvira T. Castillo, single 2. TCT No. 168208 (Annex B) = do 3. TCT No. 37046 (Annex C) = do 4. TCT No. 37047 (Annex D) = do 5. TCT No. 37048 (Annex E) = do 6. TCT No. 30368 (Annex F) = Steelhaus Realty & Dev. Corp. 7. TCT No. 30369 (Annex G) = do 8. TCT No. 30371 (Annex F) = do 9. TCT No. (92323) 67881 (Annex I) = Eloisa Castillo In this action, plaintiff-appellant seeks to be declared as 1/2 co-owner of the real properties covered by the above listed titles and eventually for their partition [par. (a), Prayer; p. 4 Records]. Notably, in order to achieve such prayer for a joint co-ownership declaration, it is unavoidable that the individual titles involved be altered, changed, cancelled or modified to include therein the name of the appellee as a registered 1/2 co-owner. Yet, no cause of action or even a prayer is contained filed. Manifestly, absent any cause or prayer for the alteration, cancellation, modification or changing of the titles involved, the desired declaration of co-ownership and eventual partition will utterly be an indirect or collateral attack on the subject titled in this suit. It is here that We fell into error, such that, if not rectified will surely lead to a procedural lapse and a possible injustice. Well settled is the rules that a certificate of title cannot be altered, modified or cancelled except in a direct proceeding in accordance with law. In this jurisdiction, the remedy of the landowner whose property has been wrongfully or erroneously registered in another name is, after one year from the date of the decree, not to set aside the decree, but respecting it as incontrovertible and no longer open to review, to bring an action for reconveyance or, if the property had passed into the hands of an innocent purchaser for value, for damages. Verily, plaintiffappellant should have first pursued such remedy or any other relief directly attacking the subject titles before instituting the present partition suit. Apropos, the case at bench appears to have been prematurely filed. Lastly, to grant the partition prayed for by the appellant will in effect rule and decide against the properties registered in the names of Steelhouse Realty and Development Corporation and Eloisa Castillo, who are not parties in the case. To allow this to happen will surely result to injustice and denial of due process of law. . . . 11 Petitioner moved for reconsideration but his motion was denied by the Court of Appeals in its resolution dated December 21, 1998. Hence this petition. Petitioner contends that: (1) the Court of Appeals, in its first decision of November 7, 1996, was correct in applying the Roque ruling and in rejecting respondent's claim that she was the sole owner of the subject properties and that the partition suit was a collateral attack on the titles; (2) the Court of Appeals correctly 666

University of the Cordilleras College of Law First Year C S.Y. 2013 - 2014 rules in its first decision that Art. 148 of the Family Code governs the co-ownership between the parties, hence, the complaint for partition is proper; (3) with respect to the properties registered in the name of Steelhouse Realty, respondent admitted ownership thereof and, at the very least, these properties could simply be excluded and the partition limited to the remaining real and personal properties; and (4) the Court of Appeals erred in not holding that under the Civil Code, there is an implied trust in his favor. 12 The issue in this case is really whether summary judgment, in accordance with Rule 35 of the Rules of Court, is proper. We rule in the negative. First. Rule 35, 3 of the Rules of Court provides that summary judgment is proper only when, based on the pleadings, depositions, and admissions on file, and after summary hearing, it is shown that except as to the amount of damages, there is no veritable issue regarding any material fact in the action and the movant is entitled to judgment as a matter of law. 1 Conversely, where the pleadings tender a genuine issue, i.e., an issue of fact the resolution of which calls for the presentation of evidence, as distinguished from an issue which is sham, fictitious, contrived, set-up in bad faith, or patently unsubstantial, summary judgment is not proper. 14 In the present case, we are convinced that genuine issues exist. Petitioner anchors his claim of coownership on two factual grounds: first, that said properties were acquired by him and respondent during their union from 1979 to 1992 from profits derived from their brokerage business; and second, that said properties were registered solely in respondent's name only because they agreed to that arrangement, thereby giving rise to an implied trust in accordance with Art. 1452 and Art. 1453 of the Civil Code. These allegations are denied by respondent. She denies that she and petitioner lived together as husband and wife. She also claims that the properties in question were acquired solely by her with her own money and resources. With such conflicting positions, the only way to ascertain the truth is obviously through the presentation of evidence by the parties. The trial court ruled that it is immaterial whether the parties actually lived together as husband and wife because Art. 144 of the Civil Code cannot be made to apply to them as they were both incapacitated to marry each other. Hence, it was impossible for a co-ownership to exist between them. We disagree. Art. 144 of the Civil Code provides: When a man and a woman live together as husband and wife, but they are not married, or their marriage is void from the beginning, the property acquired by either or both of them through their work or industry or their wages and salaries shall be governed by the rules on co-ownership. This provision of the Civil Code, applies only to cases in which a man and a woman live together as husband and wife without the benefit of marriage provided they are not incapacitated or are without impediment to marry each other, 15 or in which the marriage is void ab initio, provided it is not bigamous. Art. 144, therefore, does not cover parties living in an adulterous relationship. However, Art. 148 of the Family Code now provides for a limited co-ownership in cases where the parties in union are incapacitated to marry each other. It states: In cases of cohabitation not falling under the preceding article, 16 only the properties acquired by both of the parties through their actual joint contribution of money, property or industry shall be owned by them in common in proportion to their respective contributions. In the absence of proof to the contrary, their contributions and corresponding shares are presumed to be equal. The same rule and presumption shall apply to joint deposits of money and evidences of credits. If one of the parties is validly married to another, his or her share in the co-ownership shall accrue to the absolute community or conjugal partnership existing in such valid marriage. If the party who acted in bad faith is not validly married to another, his or her share shall be forfeited in the manner provided in the last paragraph of the preceding article. 667

University of the Cordilleras College of Law First Year C S.Y. 2013 - 2014 The foregoing rules on forfeiture shall likewise apply even if both parties are in bad faith. It was error for the trial court to rule that, because the parties in this case were not capacitated to marry each other at the time that they were alleged to have been living together, they could not have owned properties in common. The Family Code, in addition to providing that a co-ownership exists between a man and a woman who live together as husband and wife without the benefit of marriage, likewise provides that, if the parties are incapacitated to marry each other, properties acquired by them through their joint contribution of money, property or industry shall be owned by them in common in proportion to their contributions which, in the absence of proof to the contrary, is presumed to be equal. There is thus co-ownership eventhough the couple are not capacitated to marry each other. In this case, there may be a co-ownership between the parties herein. Consequently, whether petitioner and respondent cohabited and whether the properties involved in the case are part of the alleged coownership are genuine and material. All but one of the properties involved were alleged to have been acquired after the Family Code took effect on August 3, 1988. With respect to the property acquired before the Family Code took effect if it is shown that it was really acquired under the regime of the Civil Code, then it should be excluded. Petitioner also alleged in paragraph 7 of his complaint that: Due to the effective management, hardwork and enterprise of plaintiff assisted by defendant, their customs brokerage business grew and out of the profits therefrom, the parties acquired real and personal properties which were, upon agreement of the parties, listed and registered in defendant's name with plaintiff as the unregistered co-owner of all said properties. 17 On the basis of this, he contends that an implied trust existed pursuant to Art. 1452 of the Civil Code which provides that "(I)f two or more persons agree to purchase property and by common consent the legal title is taken in the name of one of them for the benefit of all, a trust is created by force of law in favor of the others in proportion to the interest of each." We do not think this is correct. The legal relation of the parties is already specifically covered by Art. 148 of the Family Code under which all the properties acquired by the parties out of their actual joint contributions of money, property or industry shall constitute a co-ownership. Co-ownership is a form of trust and every co-owner is a trustee for the other. 18 The provisions of Art. 1452 and Art. 1453 of the Civil Code, then are no longer material since a trust relation already inheres in a co-ownership which is governed under Title III, Book II of the Civil Code. Second. The trial court likewise dismissed petitioner's action on the ground that the same amounted to a collateral attack on the certificates of title involved. As already noted, at first, the Court of Appeals ruled that petitioner's action does not challenge the validity of respondent's titles. However, on reconsideration, it reversed itself and affirmed the trial court. It noted that petitioner's complaint failed to include a prayer for the alteration, cancellation, modification, or changing of the titles involved. Absent such prayer, the appellate court ruled that a declaration of co-ownership and eventual partition would involve an indirect or collateral attack on the titles. We disagree. A torrens title, as a rule, is conclusive and indefeasible. Proceeding from this, P.D. No. 1529, 19 48 provides that a certificate of title shall not be subject to collateral attack and cannot be altered, modified, or canceled except in a direct proceeding. When is an action an attack on a title? It is when the object of the action or proceeding is to nullify the title, and thus challenge the judgment pursuant to which the title was decreed. The attack is direct when the object of an action or proceeding is to annul or set aside such judgment, or enjoin its enforcement. On the other hand, the attack is indirect or collateral when, in an action to obtain a different relief, an attack on the judgment is nevertheless made as an incident thereof. 20 In his complaint for partition, consistent with our ruling in Roque regarding the nature of an action for partition, petitioner seeks first, a declaration that he is a co-owner of the subject properties; and second, 668

University of the Cordilleras College of Law First Year C S.Y. 2013 - 2014 the conveyance of his lawful shares. He does not attack respondent's titles. Petitioner alleges no fraud, mistake, or any other irregularity that would justify a review of the registration decree in respondent's favor. His theory is that although the subject properties were registered solely in respondent's name, but since by agreement between them as well as under the Family Code, he is co-owner of these properties and as such is entitled to the conveyance of his shares. On the premise that he is a co-owner, he can validly seek the partition of the properties in co-ownership and the conveyance to him of his share. Thus, in Guevara v. Guevara, 21 in which a parcel of land bequeathed in a last will and testament was registered in the name of only one of the heirs, with the understanding that he would deliver to the others their shares after the debts of the original owner had been paid, this Court ruled that notwithstanding the registration of the land in the name of only one of the heirs, the other heirs can claim their shares in "such action, judicial or extrajudicial, as may be necessary to partition the estate of the testator." 22 Third. The Court of Appeals also reversed its first decision on the ground that to order partition will, in effect, rule and decide against Steelhouse Realty Development Corporation and Eloisa Castillo, both strangers to the present case, as to the properties registered in their names. This reasoning, however, ignores the fact that the majority of the properties involved in the present case are registered in respondent's name, over which petitioner claims rights as a co-owner. Besides, other than the real properties, petitioner also seeks partition of a substantial amount of personal properties consisting of motor vehicles and several pieces of jewelry. By dismissing petitioner's complaint for partition on grounds of due process and equity, the appellate court unwittingly denied petitioner his right to prove ownership over the claimed real and personal properties. The dismissal of petitioner's complaint is unjustified since both ends may be amply served by simply excluding from the action for partition the properties registered in the name of Steelhouse Realty and Eloisa Castillo. WHEREFORE, the amended decision of the Court of Appeals, dated May 7, 1998, is REVERSED and the case is REMANDED to the Regional Trial Court, Branch 59, Makati City for further proceedings on the merits. SO ORDERED. Bellosillo, Quisumbing and De Leon, Jr., JJ., concur. Buena, J., took no part.

669

University of the Cordilleras College of Law First Year C S.Y. 2013 - 2014 Case Digest: Malilin Jr. vs Castillo 351 SCRA 127 G.R. No. 136803 Decided on: June 16, 2000 Ponente: MENDOZA, J.: Facts: Petitioner Eustaquio Mallilin, Jr. filed a complaint for "Partition and/or Payment of Co-Ownership Share, Accounting and Damages" against respondent Ma. Elvira Castillo. The complaint alleged that petitioner and respondent, both married and with children, but separated from their respective spouses, cohabited after a brief courtship while their respective marriages still subsisted. During their union, they set up the Superfreight Customs Brokerage Corporation, with petitioner as president and chairman of the board of directors, and respondent as vice-president and treasurer. The business flourished and petitioner and respondent acquired real and personal properties which were registered solely in respondent's name. Due to irreconcilable differences, the couple separated. Petitioner demanded from respondent his share in the subject properties, but respondent refused alleging that said properties had been registered solely in her name. Respondent admitted that she engaged in the customs brokerage business with petitioner but alleged that the Superfreight Customs Brokerage Corporation was organized with other individuals and duly registered with the SEC. She denied that she and petitioner lived as husband and wife because the fact was that they were still legally married to their respective spouses. She claimed to be the exclusive owner of all real personal properties involved in petitioner's action for partition on the ground that they were acquired entirely out of her own money and registered solely in her name. Issue: Whether or not the parties are considered as co-owners of the properties. Ruling: A co-ownership exists between a man and a woman who live together as husband and wife without the benefit of marriage, likewise provides that, if the parties are incapacitated to marry each other, properties acquired by them through their joint contribution of money, property or industry shall be owned by them in common in proportion to their contributions which, in the absence of proof to the contrary, is presumed to be equal. There is thus co-ownership even though the couple are not capacitated to marry each other.

670

University of the Cordilleras College of Law First Year C S.Y. 2013 - 2014 CARIO vs CARIO [G.R. No. 132529. February 2, 2001] Full Case SUSAN NICDAO CARIO, petitioner, vs. SUSAN YEE CARIO, respondent. YNARES-SANTIAGO, J.: The issue for resolution in the case at bar hinges on the validity of the two marriages contracted by the deceased SPO4 Santiago S. Cario, whose death benefits is now the subject of the controversy between the two Susans whom he married. Before this Court is a petition for review on certiorari seeking to set aside the decision[1] of the Court of Appeals in CA-G.R. CV No. 51263, which affirmed in toto the decision[2] of the Regional Trial Court of Quezon City, Branch 87, in Civil Case No. Q-93-18632. During the lifetime of the late SPO4 Santiago S. Cario, he contracted two marriages, the first was on June 20, 1969, with petitioner Susan Nicdao Cario (hereafter referred to as Susan Nicdao), with whom he had two offsprings, namely, Sahlee and Sandee Cario; and the second was on November 10, 1992, with respondent Susan Yee Cario (hereafter referred to as Susan Yee), with whom he had no children in their almost ten year cohabitation starting way back in 1982. In 1988, SPO4 Santiago S. Cario became ill and bedridden due to diabetes complicated by pulmonary tuberculosis. He passed away on November 23, 1992, under the care of Susan Yee, who spent for his medical and burial expenses. Both petitioner and respondent filed claims for monetary benefits and financial assistance pertaining to the deceased from various government agencies. Petitioner Susan Nicdao was able to collect a total of P146,000.00 from MBAI, PCCUI, Commutation, NAPOLCOM, [and] Pag-ibig,[3] while respondent Susan Yee received a total of P21,000.00 from GSIS Life, Burial (GSIS) and burial (SSS).[4] On December 14, 1993, respondent Susan Yee filed the instant case for collection of sum of money against petitioner Susan Nicdao praying, inter alia, that petitioner be ordered to return to her at least onehalf of the one hundred forty-six thousand pesos (P146,000.00) collectively denominated as death benefits which she (petitioner) received from MBAI, PCCUI, Commutation, NAPOLCOM, [and] Pagibig. Despite service of summons, petitioner failed to file her answer, prompting the trial court to declare her in default. Respondent Susan Yee admitted that her marriage to the deceased took place during the subsistence of, and without first obtaining a judicial declaration of nullity of, the marriage between petitioner and the deceased. She, however, claimed that she had no knowledge of the previous marriage and that she became aware of it only at the funeral of the deceased, where she met petitioner who introduced herself as the wife of the deceased. To bolster her action for collection of sum of money, respondent contended that the marriage of petitioner and the deceased is void ab initio because the same was solemnized without the required marriage license. In support thereof, respondent presented: 1) the marriage certificate of the deceased and the petitioner which bears no marriage license number;[5] and 2) a certification dated March 9, 1994, from the Local Civil Registrar of San Juan, Metro Manila, which reads This is to certify that this Office has no record of marriage license of the spouses SANTIAGO CARINO (sic) and SUSAN NICDAO, who are married in this municipality on June 20, 1969. Hence, we cannot issue as requested a true copy or transcription of Marriage License number from the records of this archives. This certification is issued upon the request of Mrs. Susan Yee Cario for whatever legal purpose it may serve.[6]

671

University of the Cordilleras College of Law First Year C S.Y. 2013 - 2014 On August 28, 1995, the trial court ruled in favor of respondent, Susan Yee, holding as follows: WHEREFORE, the defendant is hereby ordered to pay the plaintiff the sum of P73,000.00, half of the amount which was paid to her in the form of death benefits arising from the death of SPO4 Santiago S. Cario, plus attorneys fees in the amount of P5,000.00, and costs of suit. IT IS SO ORDERED.[7] On appeal by petitioner to the Court of Appeals, the latter affirmed in toto the decision of the trial court. Hence, the instant petition, contending that: I. THE HONORABLE COURT OF APPEALS GRAVELY ERRED IN AFFIRMING THE FINDINGS OF THE LOWER COURT THAT VDA. DE CONSUEGRA VS. GSIS IS APPLICABLE TO THE CASE AT BAR. II. THE HONORABLE COURT OF APPEALS GRAVELY ERRED IN APPLYING EQUITY IN THE INSTANT CASE INSTEAD OF THE CLEAR AND UNEQUIVOCAL MANDATE OF THE FAMILY CODE. III. THE HONORABLE COURT OF APPEALS GRAVELY ERRED IN NOT FINDING THE CASE OF VDA. DE CONSUEGRA VS GSIS TO HAVE BEEN MODIFIED, AMENDED AND EVEN ABANDONED BY THE ENACTMENT OF THE FAMILY CODE.[8] Under Article 40 of the Family Code, the absolute nullity of a previous marriage may be invoked for purposes of remarriage on the basis solely of a final judgment declaring such previous marriage void. Meaning, where the absolute nullity of a previous marriage is sought to be invoked for purposes of contracting a second marriage, the sole basis acceptable in law, for said projected marriage to be free from legal infirmity, is a final judgment declaring the previous marriage void.[9] However, for purposes other than remarriage, no judicial action is necessary to declare a marriage an absolute nullity. For other purposes, such as but not limited to the determination of heirship, legitimacy or illegitimacy of a child, settlement of estate, dissolution of property regime, or a criminal case for that matter, the court may pass upon the validity of marriage even after the death of the parties thereto, and even in a suit not directly instituted to question the validity of said marriage, so long as it is essential to the determination of the case.[10] In such instances, evidence must be adduced, testimonial or documentary, to prove the existence of grounds rendering such a previous marriage an absolute nullity. These need not be limited solely to an earlier final judgment of a court declaring such previous marriage void.[11] It is clear therefore that the Court is clothed with sufficient authority to pass upon the validity of the two marriages in this case, as the same is essential to the determination of who is rightfully entitled to the subject death benefits of the deceased. Under the Civil Code, which was the law in force when the marriage of petitioner Susan Nicdao and the deceased was solemnized in 1969, a valid marriage license is a requisite of marriage,[12] and the absence thereof, subject to certain exceptions,[13] renders the marriage void ab initio.[14] In the case at bar, there is no question that the marriage of petitioner and the deceased does not fall within the marriages exempt from the license requirement. A marriage license, therefore, was indispensable to the validity of their marriage. This notwithstanding, the records reveal that the marriage contract of petitioner and the deceased bears no marriage license number and, as certified by the Local Civil Registrar of San Juan, Metro Manila, their office has no record of such marriage license. In Republic v. 672

University of the Cordilleras College of Law First Year C S.Y. 2013 - 2014 Court of Appeals,[15] the Court held that such a certification is adequate to prove the non-issuance of a marriage license. Absent any circumstance of suspicion, as in the present case, the certification issued by the local civil registrar enjoys probative value, he being the officer charged under the law to keep a record of all data relative to the issuance of a marriage license. Such being the case, the presumed validity of the marriage of petitioner and the deceased has been sufficiently overcome. It then became the burden of petitioner to prove that their marriage is valid and that they secured the required marriage license. Although she was declared in default before the trial court, petitioner could have squarely met the issue and explained the absence of a marriage license in her pleadings before the Court of Appeals and this Court. But petitioner conveniently avoided the issue and chose to refrain from pursuing an argument that will put her case in jeopardy. Hence, the presumed validity of their marriage cannot stand. It is beyond cavil, therefore, that the marriage between petitioner Susan Nicdao and the deceased, having been solemnized without the necessary marriage license, and not being one of the marriages exempt from the marriage license requirement, is undoubtedly void ab initio. It does not follow from the foregoing disquisition, however, that since the marriage of petitioner and the deceased is declared void ab initio, the death benefits under scrutiny would now be awarded to respondent Susan Yee. To reiterate, under Article 40 of the Family Code, for purposes of remarriage, there must first be a prior judicial declaration of the nullity of a previous marriage, though void, before a party can enter into a second marriage, otherwise, the second marriage would also be void. Accordingly, the declaration in the instant case of nullity of the previous marriage of the deceased and petitioner Susan Nicdao does not validate the second marriage of the deceased with respondent Susan Yee. The fact remains that their marriage was solemnized without first obtaining a judicial decree declaring the marriage of petitioner Susan Nicdao and the deceased void. Hence, the marriage of respondent Susan Yee and the deceased is, likewise, void ab initio. One of the effects of the declaration of nullity of marriage is the separation of the property of the spouses according to the applicable property regime.[16] Considering that the two marriages are void ab initio, the applicable property regime would not be absolute community or conjugal partnership of property, but rather, be governed by the provisions of Articles 147 and 148 of the Family Code on Property Regime of Unions Without Marriage. Under Article 148 of the Family Code, which refers to the property regime of bigamous marriages, adulterous relationships, relationships in a state of concubine, relationships where both man and woman are married to other persons, multiple alliances of the same married man,[17] ... [O]nly the properties acquired by both of the parties through their actual joint contribution of money, property, or industry shall be owned by them in common in proportion to their respective contributions ... In this property regime, the properties acquired by the parties through their actual joint contribution shall belong to the co-ownership. Wages and salaries earned by each party belong to him or her exclusively. Then too, contributions in the form of care of the home, children and household, or spiritual or moral inspiration, are excluded in this regime.[18] Considering that the marriage of respondent Susan Yee and the deceased is a bigamous marriage, having been solemnized during the subsistence of a previous marriage then presumed to be valid (between petitioner and the deceased), the application of Article 148 is therefore in order. The disputed P146,000.00 from MBAI [AFP Mutual Benefit Association, Inc.], NAPOLCOM, Commutation, Pag-ibig, and PCCUI, are clearly renumerations, incentives and benefits from governmental agencies earned by the deceased as a police officer. Unless respondent Susan Yee presents proof to the contrary, it could not be said that she contributed money, property or industry in the 673

University of the Cordilleras College of Law First Year C S.Y. 2013 - 2014 acquisition of these monetary benefits. Hence, they are not owned in common by respondent and the deceased, but belong to the deceased alone and respondent has no right whatsoever to claim the same. By intestate succession, the said death benefits of the deceased shall pass to his legal heirs. And, respondent, not being the legal wife of the deceased is not one of them. As to the property regime of petitioner Susan Nicdao and the deceased, Article 147 of the Family Code governs. This article applies to unions of parties who are legally capacitated and not barred by any impediment to contract marriage, but whose marriage is nonetheless void for other reasons, like the absence of a marriage license. Article 147 of the Family Code reads Art. 147. When a man and a woman who are capacitated to marry each other, live exclusively with each other as husband and wife without the benefit of marriage or under a void marriage, their wages and salaries shall be owned by them in equal shares and the property acquired by both of them through their work or industry shall be governed by the rules on co-ownership. In the absence of proof to the contrary, properties acquired while they lived together shall be presumed to have been obtained by their joint efforts, work or industry, and shall be owned by them in equal shares. For purposes of this Article, a party who did not participate in the acquisition by the other party of any property shall be deemed to have contributed jointly in the acquisition thereof if the formers efforts consisted in the care and maintenance of the family and of the household. xxx xxx xxx

When only one of the parties to a void marriage is in good faith, the share of the party in bad faith in the co-ownership shall be forfeited in favor of their common children. In case of default of or waiver by any or all of the common children or their descendants, each vacant share shall belong to the respective surviving descendants. In the absence of descendants, such share shall belong to the innocent party. In all cases, the forfeiture shall take place upon termination of the cohabitation. In contrast to Article 148, under the foregoing article, wages and salaries earned by either party during the cohabitation shall be owned by the parties in equal shares and will be divided equally between them, even if only one party earned the wages and the other did not contribute thereto.[19] Conformably, even if the disputed death benefits were earned by the deceased alone as a government employee, Article 147 creates a co-ownership in respect thereto, entitling the petitioner to share one-half thereof. As there is no allegation of bad faith in the present case, both parties of the first marriage are presumed to be in good faith. Thus, one-half of the subject death benefits under scrutiny shall go to the petitioner as her share in the property regime, and the other half pertaining to the deceased shall pass by, intestate succession, to his legal heirs, namely, his children with Susan Nicdao. In affirming the decision of the trial court, the Court of Appeals relied on the case of Vda. de Consuegra v. Government Service Insurance System,[20] where the Court awarded one-half of the retirement benefits of the deceased to the first wife and the other half, to the second wife, holding that: ... [S]ince the defendants first marriage has not been dissolved or declared void the conjugal partnership established by that marriage has not ceased. Nor has the first wife lost or relinquished her status as putative heir of her husband under the new Civil Code, entitled to share in his estate upon his death should she survive him. Consequently, whether as conjugal partner in a still subsisting marriage or as such putative heir she has an interest in the husbands share in the property here in dispute.... And with respect to the right of the second wife, this Court observed that although the second marriage can be presumed to be void ab initio as it was celebrated while the first marriage was still subsisting, still there is need for judicial declaration of such nullity. And inasmuch as the conjugal partnership formed by the second marriage was dissolved before judicial declaration of its nullity, [t]he only just and equitable solution in this case would be to recognize the right of the second wife to her share of one-half in the property acquired by her and her husband, and consider the other half as pertaining to the conjugal partnership of the first marriage.[21] 674

University of the Cordilleras College of Law First Year C S.Y. 2013 - 2014 It should be stressed, however, that the aforecited decision is premised on the rule which requires a prior and separate judicial declaration of nullity of marriage. This is the reason why in the said case, the Court determined the rights of the parties in accordance with their existing property regime. In Domingo v. Court of Appeals,[22] however, the Court, construing Article 40 of the Family Code, clarified that a prior and separate declaration of nullity of a marriage is an all-important condition precedent only for purposes of remarriage. That is, if a party who is previously married wishes to contract a second marriage, he or she has to obtain first a judicial decree declaring the first marriage void, before he or she could contract said second marriage, otherwise the second marriage would be void. The same rule applies even if the first marriage is patently void because the parties are not free to determine for themselves the validity or invalidity or their marriage. However, for purposes other than to remarry, like for filing a case for collection of sum of money anchored on a marriage claimed to be valid, no prior and separate judicial declaration of nullity is necessary. All that a party has to do is to present evidence, testimonial or documentary, that would prove that the marriage from which his or her rights flow is in fact valid. Thereupon, the court, if material to the determination of the issues before it, will rule on the status of the marriage involved and proceed to determine the rights of the parties in accordance with the applicable laws and jurisprudence. Thus, in Nial v. Bayadog,[23] the Court explained: [T]he court may pass upon the validity of marriage even in a suit not directly instituted to question the same so long as it is essential to the determination of the case. This is without prejudice to any issue that may arise in the case. When such need arises, a final judgment of declaration of nullity is necessary even if the purpose is other than to remarry. The clause on the basis of a final judgment declaring such previous marriage void in Article 40 of the Family Code connoted that such final judgment need not be obtained only for purpose of remarriage. WHEREFORE, the petition is GRANTED, and the decision of the Court of Appeals in CA-G.R. CV No. 51263 which affirmed the decision of the Regional Trial Court of Quezon City ordering petitioner to pay respondent the sum of P73,000.00 plus attorneys fees in the amount of P5,000.00, is REVERSED and SET ASIDE. The complaint in Civil Case No. Q-93-18632, is hereby DISMISSED. No pronouncement as to costs. SO ORDERED. Davide, Jr., C.J. (Chairman), Kapunan, and Pardo, JJ., concur. Puno J., on official leave.

675

University of the Cordilleras College of Law First Year C S.Y. 2013 - 2014 Case Digest CARIO vs CARIO G.R. No. 132529 Decided on: February 2, 2001 Ponente: YNARES-SANTIAGO, J.: FACTS: Santiago CArio married petitioner Susan Nicdao on June 20, 1969, with whom he had two children, Sahlee and Sandee. On November 10, 1982, SPO4 Cario also married respondent Susan Yee. In 1988, SPO4 Cario became bedridden due to diabetes and tuberculosis, and died on November 23, 1992, under the care of Susan Yee who spent for his medical and burial expenses. Both Susans filed claims for monetary benefits and financial assistance from various government agencies pertaining to the deceased. Nicdao was able to collect P146,000 from MBAI, PCCVI, commutation, NAPOLCOM and Pag-ibig, while Yee received a total of P21,000 from GSIS burial and SSS burial insurance. On December 14, 1993, Yee filed for collection of money against NIcdao, praying that Nicdao be ordered to return to her at least one-half of the P146,000 NIcdao had collected. For failing to file her answer, NIcdao was declared in default. Yee admitted that her marriage to the deceased took place during the subsistence of and without first obtaining a judicial declaration of nullity of the marriage between Nicdao and Cario. But she claimed good faith, having no knowledge of the previous marriage until at the funeral where she met Nicdao who introduced herself as the wife of the deceased. Yee su bmitted that Carios marriage to Nicdao was void because it was solemnized without the required marriage license. ISSUES: (1) Whether or not the subsequent marriage is null and void; (2) Whether or not, if yes to above, the wife of the deceased is entitled to collect the death benefits from government agencies despite the nullity of their marriage. HELD: Under Article 40 of the Family Code, the nullity of a previous marriage may be invoked for purposes of remarriage on the basis solely of a final judgment declaring such marriage void. Meaning, where the absolute nullity of a previous marriage is sought to be invoked for purposes of contracting a second marriage, the sole basis acceptable in law, for said projected marriage to be free from legal infirmity, is a final judgment declaring the previous marriage void. However, for purposes other than remarriage, no judicial action is necessary to declare a marriage an absolute nullity. For other purposes, such as but not limited to the determination of heirship, legitimacy or illegitimacy of a child, settlement of estate, dissolution of property regime, or a criminal case for that matter, the court may pass upon the validity of marriage even after the death of the parties thereto, and even in a suit not directly instituted to question the validity of said marriage, so long as it is essential to the determination of the case. Under the Civil Code which was the law in force when the marriage of petitioner and the deceased was solemnized in 1969, a valid marriage license is a requisite of marriage, and the absence thereof, subject to certain exceptions, renders the marriage void ab initio. It does not follow, however, that since the marriage of Nicdao and the deceased was void ab initio, the death benefits would now be awarded to Yee. To reiterate, under Article 40 of the Family Code, for purposes of remarriage, there must be a prior judicial declaration of the nullity of a previous marriage, though void, before a party can enter into a second marriage; otherwise, the second marriage would also be void. One of the effects of the declaration of nullity of marriage is the separation of the property of the spouses according to the applicable property regime. Considering that the two marriages are void ab initio, the applicable property regime would be not absolute community nor conjugal 676

University of the Cordilleras College of Law First Year C S.Y. 2013 - 2014 partnership of property, but governed by the provisions of Articles 147 and 148 of the Family Code, on Property Regime of Unions Without Marriage.

677

University of the Cordilleras College of Law First Year C S.Y. 2013 - 2014 Saguid vs Court of Appeals 403 SCRA 678 G.R. No. 150611. June 10, 2003 Full Case JACINTO SAGUID, petitioner, vs. HON. COURT OF APPEALS, THE REGIONAL TRIAL COURT, BRANCH 94, BOAC, MARINDUQUE and GINA S. REY, respondents. YNARES-SANTIAGO, J.: The regime of limited co-ownership of property governing the union of parties who are not legally capacitated to marry each other, but who nonetheless live together as husband and wife, applies to properties acquired during said cohabitation in proportion to their respective contributions. Co-ownership will only be up to the extent of the proven actual contribution of money, property or industry. Absent proof of the extent thereof, their contributions and corresponding shares shall be presumed to be equal.[1] Seventeen-year old Gina S. Rey was married,[2] but separated de facto from her husband, when she met petitioner Jacinto Saguid in Marinduque, sometime in July 1987.[3] After a brief courtship, the two decided to cohabit as husband and wife in a house built on a lot owned by Jacintos father.[4] T heir cohabitation was not blessed with any children. Jacinto made a living as the patron of their fishing vessel Saguid Brothers.[5] Gina, on the other hand, worked as a fish dealer, but decided to work as an entertainer in Japan from 1992 to 1994 when her relationship with Jacintos relatives turned sour. Her periodic absence, however, did not ebb away the conflict with petitioners relatives. In 1996, the couple decided to separate and end up their 9-year cohabitation.[6] On January 9, 1997, private respondent filed a complaint for Partition and Recovery of Personal Property with Receivership against the petitioner with the Regional Trial Court of Boac, Marinduque. She alleged that from her salary of $1,500.00 a month as entertainer in Japan, she was able to contribute P70,000.00 in the completion of their unfinished house. Also, from her own earnings as an entertainer and fish dealer, she was able to acquire and accumulate appliances, pieces of furniture and household effects, with a total value of P111,375.00. She prayed that she be declared the sole owner of these personal properties and that the amount of P70,000.00, representing her contribution to the construction of their house, be reimbursed to her. Private respondent testified that she deposited part of her earnings in her savings account with First Allied Development Bank.[7] Her Pass Book shows that as of May 23, 1995, she had a balance of P21,046.08.[8] She further stated that she had a total of P35,465.00[9] share in the joint account deposit which she and the petitioner maintained with the same bank.[10] Gina declared that said deposits were spent for the purchase of construction materials, appliances and other personal properties.[11] In his answer[12] to the complaint, petitioner claimed that the expenses for the construction of their house were defrayed solely from his income as a captain of their fishing vessel. He averred that private respondents meager income as fish dealer rendered her unable to contribute in the construction of said house. Besides, selling fish was a mere pastime to her; as such, she was contented with the small quantity of fish allotted to her from his fishing trips. Petitioner further contended that Gina did not work continuously in Japan from 1992 to 1994, but only for a 6-month duration each year. When their house was repaired and improved sometime in 1995-1996, private respondent did not share in the expenses because her earnings as entertainer were spent on the daily needs and business of her parents. From his income in the fishing business, he claimed to have saved a total of P130,000.00, P75,000.00 of which was placed in a joint account deposit with private respondent. This savings, according to petitioner was spent in purchasing the disputed personal properties. On May 21, 1997, the trial court declared the petitioner as in default for failure to file a pre-trial brief as required by Supreme Court Circular No. 1-89.[13]

678

University of the Cordilleras College of Law First Year C S.Y. 2013 - 2014 On May 26, 1997, petitioner filed a motion for reconsideration[14] of the May 21, 1997 order, which was denied on June 2, 1997, and private respondent was allowed to present evidence ex parte.[15] Petitioner filed another motion for reconsideration but the same was also denied on October 8, 1997. On July 15, 1998, a decision[16] was rendered in favor of private respondent, the dispositive portion of which reads: WHEREFORE, in view of all the foregoing, judgment is hereby rendered in favor of the plaintiff Gina S. Rey against defendant Jacinto Saguid: a) Ordering the partition of the house identified as plaintiffs Exhibit C and D and directing the defendant to return and/or reimburse to the plaintiff the amount of seventy thousand pesos (P70,000,00) which the latter actually contributed to its construction and completion; b) Declaring the plaintiff as the exclusive owner of the personal properties listed on Exhibit M;

c) Ordering the defendant, and/or anyone in possession of the aforesaid personal properties, to return and/or deliver the same to the plaintiff; and d) Ordering the defendant to pay the plaintiff moral damages in the sum of fifty thousand pesos (P50,000.00) plus the costs of suit. SO ORDERED.[17] On appeal, said decision was affirmed by the Court of Appeals; however, the award of P50,000.00 as moral damages was deleted for lack of basis.[18] The appellate court ruled that the propriety of the order which declared the petitioner as in default became moot and academic in view of the effectivity of the 1997 Rules of Civil Procedure. It explained that the new rules now require the filing of a pre-trial brief and the defendants non-compliance therewith entitles the plaintiff to present evidence ex parte. Both parties filed motions for reconsideration which were denied; hence, petitioner filed the instant petition based on the following assigned errors: A. THE HONORABLE COURT OF APPEALS COMMIT[TED] A REVERSIBLE ERROR IN APPLYING RETROACTIVELY THE 1997 RULES OF CIVIL PROCEDURE IN THE PRESENT CASE AND HOLDING THE FIRST ASSIGNED ERROR THEREIN MOOT AND ACADEMIC THUS, FAILED TO RULE ON THE PROPRIETY OF THE TRIAL COURTS REFUSAL TO SET ASIDE THE ORDER OF DEFAULT DUE TO MISTAKE AND/OR EXCUSABLE NEGLIGENCE COMMITTED BY PETITIONER. B. THE HONORABLE COURT OF APPEALS COMMIT[TED] A REVERSIBLE ERROR IN RELYING ON THE FACTUAL FINDINGS OF THE TRIAL COURT WHICH RECEIVED THE EVIDENCE OF HEREIN RESPONDENT ONLY EX PARTE.[19] The issues for resolution are: (1) whether or not the trial court erred in allowing private respondent to present evidence ex parte; and (2) whether or not the trial courts decision is supported by evidence. Under Section 6, Rule 18 of the 1997 Rules of Civil Procedure, the failure of the defendant to file a pretrial brief shall have the same effect as failure to appear at the pre-trial, i.e., the plaintiff may present his evidence ex parte and the court shall render judgment on the basis thereof.[20] The remedy of the defendant is to file a motion for reconsideration[21] showing that his failure to file a pre-trial brief was due to fraud, accident, mistake or excusable neglect.[22] The motion need not really stress the fact that the defendant has a valid and meritorious defense because his answer which contains his defenses is already on record.[23] 679

University of the Cordilleras College of Law First Year C S.Y. 2013 - 2014 In the case at bar, petitioner insists that his failure to file a pre-trial brief is justified because he was not represented by counsel. This justification is not, however, sufficient to set aside the order directing private respondent to present evidence ex parte, inasmuch as the petitioner chose at his own risk not to be represented by counsel. Even without the assistance of a lawyer, petitioner was able to file a motion for extension to file answer,[24] the required answer stating therein the special and affirmative defenses,[25] and several other motions.[26] If it were true that petitioner did not understand the import of the April 23, 1997 order directing him to file a pre-trial brief, he could have inquired from the court or filed a motion for extension of time to file the brief. Instead, he waited until May 26, 1997, or 14 days from his alleged receipt of the April 23, 1997 order before he filed a motion asking the court to excuse his failure to file a brief. Pre-trial rules are not to be belittled or dismissed because their non-observance may result in prejudice to a partys substantive rights. Like all rules, they should be followed except only for the most persuasive of reasons when they may be relaxed to relieve a litigant of an injustice not commensurate with the degree of his thoughtlessness in not complying with the procedure prescribed.[27] In the instant case, the fact that petitioner was not assisted by a lawyer is not a persuasive reason to relax the application of the rules. There is nothing in the Constitution which mandates that a party in a noncriminal proceeding be represented by counsel and that the absence of such representation amounts to a denial of due process. The assistance of lawyers, while desirable, is not indispensable. The legal profession is not engrafted in the due process clause such that without the participation of its members the safeguard is deemed ignored or violated.[28] However, the Court of Appeals erred in ruling that the effectivity of the 1997 Rules of Civil Procedure, specifically, Section 6, Rule 18 thereof, rendered moot and academic the issue of whether or not the plaintiff may be allowed to present evidence ex parte for failure of the defendant to file a pre-trial brief. While the rules may indeed be applied retroactively, the same is not called for in the case at bar. Even before the 1997 Rules of Civil Procedure took effect on July 1, 1997, the filing of a pre-trial brief was required under Circular No. 1-89 which became effective on February 1, 1989. Pursuant to the said circular, [f]ailure to file pre-trial briefs may be given the same effect as the failure to appear at the pretrial, that is, the party may be declared non-suited or considered as in default.[29] Coming now to the substantive issue, it is not disputed that Gina and Jacinto were not capacitated to marry each other because the former was validly married to another man at the time of her cohabitation with the latter. Their property regime therefore is governed by Article 148[30] of the Family Code, which applies to bigamous marriages, adulterous relationships, relationships in a state of concubinage, relationships where both man and woman are married to other persons, and multiple alliances of the same married man. Under this regime, only the properties acquired by both of the parties through their actual joint contribution of money, property, or industry shall be owned by them in common in proportion to their respective contributions ...[31] Proof of actual contribution is required.[32] In the case at bar, although the adulterous cohabitation of the parties commenced in 1987, which is before the date of the effectivity of the Family Code on August 3, 1998, Article 148 thereof applies because this provision was intended precisely to fill up the hiatus in Article 144 of the Civil Code.[33] Before Article 148 of the Family Code was enacted, there was no provision governing property relations of couples living in a state of adultery or concubinage. Hence, even if the cohabitation or the acquisition of the property occurred before the Family Code took effect, Article 148 governs.[34] In the cases of Agapay v. Palang,[35] and Tumlos v. Fernandez,[36] which involved the issue of coownership of properties acquired by the parties to a bigamous marriage and an adulterous relationship, respectively, we ruled that proof of actual contribution in the acquisition of the property is essential. The claim of co-ownership of the petitioners therein who were parties to the bigamous and adulterous union is without basis because they failed to substantiate their allegation that they contributed money in the purchase of the disputed properties. Also in Adriano v. Court of Appeals,[37] we ruled that the fact that the controverted property was titled in the name of the parties to an adulterous relationship is not sufficient proof of co-ownership absent evidence of actual contribution in the acquisition of the property.

680

University of the Cordilleras College of Law First Year C S.Y. 2013 - 2014 As in other civil cases, the burden of proof rests upon the party who, as determined by the pleadings or the nature of the case, asserts an affirmative issue. Contentions must be proved by competent evidence and reliance must be had on the strength of the partys own evidence and not upon the weakness of the opponents defense.[38] This applies with more vigor where, as in the instant case, the plaintiff was allowed to present evidence ex parte. The plaintiff is not automatically entitled to the relief prayed for. The law gives the defendant some measure of protection as the plaintiff must still prove the allegations in the complaint. Favorable relief can be granted only after the court is convinced that the facts proven by the plaintiff warrant such relief.[39] Indeed, the party alleging a fact has the burden of proving it and a mere allegation is not evidence.[40] In the case at bar, the controversy centers on the house and personal properties of the parties. Private respondent alleged in her complaint that she contributed P70,000.00 for the completion of their house. However, nowhere in her testimony did she specify the extent of her contribution. What appears in the record are receipts[41] in her name for the purchase of construction materials on November 17, 1995 and December 23, 1995, in the total amount of P11,413.00. On the other hand, both parties claim that the money used to purchase the disputed personal properties came partly from their joint account with First Allied Development Bank. While there is no question that both parties contributed in their joint account deposit, there is, however, no sufficient proof of the exact amount of their respective shares therein. Pursuant to Article 148 of the Family Code, in the absence of proof of extent of the parties respective contribution, their share shall be presumed to be equal. Here, the disputed personal properties were valued at P111,375.00, the existence and value of which were not questioned by the petitioner. Hence, their share therein is equivalent to one-half, i.e., P55,687.50 each. The Court of Appeals thus erred in affirming the decision of the trial court which granted the reliefs prayed for by private respondent. On the basis of the evidence established, the extent of private respondents co-ownership over the disputed house is only up to the amount of P11,413.00, her proven contribution in the construction thereof. Anent the personal properties, her participation therein should be limited only to the amount of P55,687.50. As regards the trial courts award of P50,000.00 as moral damages, the Court of Appeals correctly deleted the same for lack of basis. WHEREFORE, in view of all the foregoing, the Decision of the Court of Appeals in CA-G.R. CV No 64166 is AFFIRMED with MODIFICATION. Private respondent Gina S. Rey is declared co-owner of petitioner Jacinto Saguid in the controverted house to the extent of P11,413.00 and personal properties to the extent of P55,687.50. Petitioner is ordered to reimburse the amount of P67,100.50 to private respondent, failing which the house shall be sold at public auction to satisfy private respondents claim. SO ORDERED. Davide, Jr., C.J., (Chairman), Vitug, Carpio, and Azcuna, JJ., concur.

681

University of the Cordilleras College of Law First Year C S.Y. 2013 - 2014 Case Digest: Saguid vs. CA, G.R. No. 150611, June 10, 2003 G.R. No. 150611. Decided on: June 10, 2003 Ponente: YNARES-SANTIAGO, J.: Facts: 17yrs old Gina S. Rey was married, but separated de facto from her husband, when she met petitioner Jacinto Saguid. After a brief courtship, the two decided to cohabit as husband and wife (no children). Jacinto made a living as the patron of their fishing vessel "Saguid Brothers." Gina, on the other hand, worked as a fish dealer, but decided to work as an entertainer in Japan from 1992 to 1994 when her relationship with Jacintos relatives turned sour. In 1996, the couple decided to separate and end up their 9-year cohabitation. On January 9, 1997, private respondent filed a complaint for Partition and Recovery of Personal Property with Receivership against the petitioner with the RTC- Boac, Marinduque. She alleged that from her salary of $1,500.00 a month as entertainer in Japan, she was able to contribute P70,000.00 in the completion of their unfinished house. Also, from her own earnings as an entertainer and fish dealer, she was able to acquire and accumulate appliances, pieces of furniture and household effects, with a total value of P111,375.00. She prayed that she be declared the sole owner of these personal properties and that the amount of P70,000.00, representing her contribution to the construction of their house, be reimbursed to her. Petitioner claimed that the expenses for the construction of their house were defrayed solely from his income as a captain of their fishing vessel. He further contended that Gina did not work continuously in Japan from 1992 to 1994, but only for a 6-month duration each year. When their house was repaired and improved sometime in 1995-1996, private respondent did not share in the expenses because her earnings as entertainer were spent on the daily needs and business of her parents. From his income in the fishing business, he claimed to have saved a total of P130,000.00, P75,000.00 of which was placed in a joint account deposit with private respondent. This savings, according to petitioner was spent in purchasing the disputed personal properties. TC ruled in favor of Gina. CA affirmed. Issue: Whether or not the trial courts decision is supported by evidence Held: Yes. Decision affirmed. In the case at bar, their property regime is governed by Article148 of the Family Code, which applies to bigamous marriages, adulterous relationships, relationships in a state of concubinage, relationships where both man and woman are married to other persons, and multiple alliances of the same married man. Under this regime, "only the properties acquired by both of the parties through their actual joint contribution of money, property, or industry shall be owned by them in common in proportion to their respective contributions ..." Proof of actual contribution is required. As in other civil cases, the burden of proof rests upon the party who, as determined by the pleadings or the nature of the case, asserts an affirmative issue. Contentions must be proved by competent evidence and reliance must be had on the strength of the partys own evidence and not upon the weakness of the opponents defense. This applies with more vigor where, as in the instant case, the plaintiff was allowed to present evidence ex parte. The plaintiff is not automatically entitled to the relief prayed for. The law gives the defendant some measure of protection as the plaintiff must still prove the allegations in the complaint. Favorable relief can be granted only after the court is convinced that the facts proven by the plaintiff warrant such relief. Indeed, the party alleging a fact has the burden of proving it and a mere allegation is not evidence. Both parties claim that the money used to purchase the disputed personal properties came partly from their joint account with First Allied Development Bank. While there is no question that both parties contributed in their joint account deposit, there is, however, no sufficient proof of the exact amount of their respective shares therein.

682

University of the Cordilleras College of Law First Year C S.Y. 2013 - 2014 Pursuant to Article 148of the Family Code, in the absence of proof of extent of the parties respective contribution, their share shall be presumed to be equal. Here, the disputed personal properties were valued at P111,375.00, the existence and value of which were not questioned by the petitioner. Hence, their share therein is equivalent to one-half, i.e., P55,687.50 each. On the basis of the evidence established, the extent of private respondents co-ownership over the disputed house is only up to the amount of P11,413.00, her proven contribution in the construction thereof. Anent the personal properties, her participation therein should be limited only to the amount of P 55,687.50.

683

University of the Cordilleras College of Law First Year C S.Y. 2013 - 2014 Heirs of Marcelino vs Heirs of Fortunato Doronio 541 SCRA 479 G.R. No. 169454 December 27, 2007 Full Case THE HEIRS OF MARCELINO DORONIO, NAMELY: REGINA AND FLORA, BOTH SURNAMED DORONIO, Petitioners, vs. HEIRS OF FORTUNATO DORONIO, NAMELY: TRINIDAD ROSALINA DORONIO-BALMES, MODING DORONIO, FLORENTINA DORONIO, AND ANICETA ALCANTARA-MANALO, Respondents. REYES, R.T., J.: For Our review on certiorari is the Decision_1 of the Court of Appeals (CA) reversing that_2 of the Regional Trial Court (RTC), Branch 45, Anonas, Urdaneta City, Pangasinan, in an action for reconveyance and damages. The CA declared respondents as rightful owners of one-half of the subject property and directed petitioners to execute a registerable document conveying the same to respondents. The Facts Spouses Simeon Doronio and Cornelia Gante, now both deceased, were the registered owners of a parcel of land located at Barangay Cabalitaan, Asingan, Pangasinan covered by Original Certificate of Title (OCT) No. 352._3 The courts below described it as follows: Un terreno (Lote 1018), situada en el municipio de Asingan, Linda por el NE; con propriedad de Gabriel Bernardino; con el SE con propriedad de ZacariasNajorda y Alejandro Najorda; por el SO con propriedad de Geminiano Mendoza y por el NO con el caminoparaVillasis; midiendouna extension superficial mil cientocincuenta y dos metros cuadrados._4 The spouses had children but the records fail to disclose their number. It is clear, however, that MarcelinoDoronio and FortunatoDoronio, now both deceased, were among them and that the parties in this case are their heirs. Petitioners are the heirs of MarcelinoDoronio, while respondents are the heirs of FortunatoDoronio. On April 24, 1919, a private deed of donation propter nuptias_5 was executed by spouses Simeon Doronio and Cornelia Gante in favor of MarcelinoDoronio and the latters wife, Veronica Pico. One of the properties subject of said deed of donation is the one that it described as follows: Fourth A piece of residential land located in the barrio of Cabalitian but we did not measure it, the area is bounded on the north by Gabriel Bernardino; on the east by FortunatoDoronio; on the south by Geminiano Mendoza and on the west by a road to Villasis. Constructed on said land is a house of light materials also a part of the dowry. Value 200.00._6 It appears that the property described in the deed of donation is the one covered by OCT No. 352. However, there is a significant discrepancy with respect to the identity of the owner of adjacent property at the eastern side. Based on OCT No. 352, the adjacent owners are ZacariasNajorda and Alejandro Najorda, whereas based on the deed of donation, the owner of the adjacent property is FortunatoDoronio. Furthermore, said deed of donation remained a private document as it was never notarized._7 Both parties have been occupying the subject land for several decades_8 although they have different theories regarding its present ownership. According to petitioners, they are now the owners of the entire property in view of the private deed of donation propter nuptias in favor of their predecessors, MarcelinoDoronio and Veronica Pico. Respondents, on the other hand, claim that only half of the property was actually incorporated in the said deed of donation because it stated that FortunatoDoronio, instead of ZacariasNajorda and Alejandro Najorda, is the owner of the adjacent property at the eastern side. Respondents posit that the donors respected and segregated the possession of FortunatoDoronio of the eastern half of the land. They are the ones who have been possessing said land occupied by their predecessor, FortunatoDoronio. Eager to obtain the entire property, the heirs of MarcelinoDoronio and Veronica Pico filed, on January 11, 1993, before the RTC in Urdaneta, Pangasinan a petition "For the Registration of a Private Deed of 684

University of the Cordilleras College of Law First Year C S.Y. 2013 - 2014 Donation"_9 docketed as Petition Case No. U-920. No respondents were named in the said petition_10 although notices of hearing were posted on the bulletin boards of Barangay Cabalitaan, Municipalities of Asingan and Lingayen._11 During the hearings, no one interposed an objection to the petition._12 After the RTC ordered a general default,_13 the petition was eventually granted on September 22, 1993. This led to the registration of the deed of donation, cancellation of OCT No. 352 and issuance of a new Transfer Certificate of Title (TCT) No. 44481 in the names of Marcelino Doronio and Veronica Pico._14 Thus, the entire property was titled in the names of petitioners predecessors. On April 28, 1994, the heirs of Fortunato Doronio filed a pleading before the RTC in the form of a petition in the same Petition Case No. U-920. The petition was for the reconsideration of the decision of the RTC that ordered the registration of the subject deed of donation. It was prayed in the petition that an order be issued declaring null and void the registration of the private deed of donation and that TCT No. 44481 be cancelled. However, the petition was dismissed on May 13, 1994 on the ground that the decision in Petition Case No. U-920 had already become final as it was not appealed. Determined to remain in their possessed property, respondent heirs of FortunatoDoronio (as plaintiffs) filed an action for reconveyance and damages with prayer for preliminary injunction_15 against petitioner heirs of MarcelinoDoronio (as defendants) before the RTC, Branch 45, Anonas, Urdaneta City, Pangasinan. Respondents contended, among others, that the subject land is different from what was donated as the descriptions of the property under OCT No. 352 and under the private deed of donation were different. They posited that spouses Simeon Doronio and Cornelia Gante intended to donate only one-half of the property. During the pre-trial conference, the parties stipulated, among others, that the property was originally covered by OCT No. 352 which was cancelled by TCT No. 44481. They also agreed that the issues are: (1) whether or not there was a variation in the description of the property subject of the private deed of donation and OCT No. 352; (2) whether or not respondents had acquired one-half of the property covered by OCT No. 352 by acquisitive prescription; (3) whether or not the transfer of the whole property covered by OCT No. 352 on the basis of the registration of the private deed of donation notwithstanding the discrepancy in the description is valid; (4) whether or not respondents are entitled to damages; and (5) whether or not TCT No. 44481 is valid._16 RTC Decision After due proceedings, the RTC ruled in favor of petitioner heirs of MarcelinoDoronio (defendants). It concluded that the parties admitted the identity of the land which they all occupy;_ 17 that a title once registered under the torrens system cannot be defeated by adverse, open and notorious possession or by prescription;_18 that the deed of donation in consideration of the marriage of the parents of petitioners is valid, hence, it led to the eventual issuance of TCT No. 44481 in the names of said parents;_ 19 and that respondent heirs of FortunatoDoronio (plaintiffs) are not entitled to damages as they are not the rightful owners of the portion of the property they are claiming._20 The RTC disposed of the case, thus: WHEREFORE, premises considered, the Court hereby renders judgment DISMISSING the herein Complaint filed by plaintiffs against defendants._21 Disagreeing with the judgment of the RTC, respondents appealed to the CA. They argued that the trial court erred in not finding that respondents predecessor-in-interest acquired one-half of the property covered by OCT No. 352 by tradition and/or intestate succession; that the deed of donation dated April 26, 1919 was null and void; that assuming that the deed of donation was valid, only one-half of the property was actually donated to MarcelinoDoronio and Veronica Pico; and that respondents acquired ownership of the other half portion of the property by acquisitive prescription._22 CA Disposition In a Decision dated January 26, 2005, the CA reversed the RTC decision with the following disposition:

685

University of the Cordilleras College of Law First Year C S.Y. 2013 - 2014 WHEREFORE, the assailed Decision dated June 28, 2002 is REVERSED and SET ASIDE. Declaring the appellants as rightful owners of one-half of the property now covered by TCT No. 44481, the appellees are hereby directed to execute a registerable document conveying the same to appellants. SO ORDERED._23 The appellate court determined that "(t)he intention to donate half of the disputed property to appellees predecessors can be gleaned from the disparity of technical descriptions appearing in the title (OCT No. 352) of spouses Simeon Doronio and Cornelia Gante and in the deed of donation propter nuptias executed on April 24, 1919 in favor of appellees predecessors."_24 The CA based its conclusion on the disparity of the following technical descriptions of the property under OCT No. 352 and the deed of donation, to wit: The court below described the property covered by OCT No. 352 as follows: "Un terreno (Lote 1018), situada en el municipio de Asingan, Linda por el NE; con propriedad de Gabriel Bernardino; con el SE con propriedad de ZacariasNajorda y Alejandro Najorda; por el SO con propriedad de Geminiano Mendoza y por el NO con el caminoparaVillasis; midiendouna extension superficial mil cientocincuenta y dos metros cuadrados." On the other hand, the property donated to appellees predecessors was described in the deed of donation as: "Fourth A piece of residential land located in the barrio of Cabalitian but we did not measure it, the area is bounded on the north by Gabriel Bernardino; on the east by FortunatoDoronio; on the south by Geminiano Mendoza and on the west by a road to Villasis. Constructed on said land is a house of light materials also a part of the dowry. Value 200.00."_25 (Emphasis ours) Taking note "that the boundaries of the lot donated to MarcelinoDoronio and Veronica Pico differ from the boundaries of the land owned by spouses Simeon Doronio and Cornelia Gante," the CA concluded that spouses Simeon Doronio and Cornelia Gante donated only half of the property covered by OCT No. 352._26 Regarding the allegation of petitioners that OCT No. 352 is inadmissible in evidence, the CA pointed out that, "while the OCT is written in the Spanish language, this document already forms part of the records of this case for failure of appellees to interpose a timely objection when it was offered as evidence in the proceedings a quo. It is a well-settled rule that any objection to the admissibility of such evidence not raised will be considered waived and said evidence will have to form part of the records of the case as competent and admitted evidence."_27 The CA likewise ruled that the donation of the entire property in favor of petitioners predecessors is invalid on the ground that it impairs the legitime of respondents predecessor, FortunatoDoronio. On this aspect, the CA reasoned out: Moreover, We find the donation of the entire property in favor of appellees predecessors invalid as it impairs the legitime of appellants predecessor. Article 961 of the Civil Cod e is explicit. "In default of testamentary heirs, the law vests the inheritance, x xx, in the legitimate x xx relatives of the deceased, x xx." As Spouses Simeon Doronio and Cornelia Gante died intestate, their property shall pass to their lawful heirs, namely: Fortunato and MarcelinoDoronio. Donating the entire property to MarcelinoDoronio and Veronica Pico and excluding another heir, Fortunato, tantamounts to divesting the latter of his rightful share in his parents inheritance. Besides, a persons prero gative to make donations is subject to certain limitations, one of which is that he cannot give by donation more than what he can give by will (Article 752, Civil Code). If he does, so much of what is donated as exceeds what he can give by will is deemed inofficious and the donation is reducible to the extent of such excess._28 Petitioners were not pleased with the decision of the CA. Hence, this petition under Rule 45. Issues Petitioners now contend that the CA erred in: 1. DECLARING ADMISSIBILITY OF THE ORIGINAL CERTIFICATE OF TITLE NO. 352 DESPITE OF LACK OF TRANSLATION THEREOF. 686

University of the Cordilleras College of Law First Year C S.Y. 2013 - 2014 2. (RULING THAT) ONLY HALF OF THE DISPUTED PROPERTY WAS DONATED TO THE PREDECESSORS-IN-INTEREST OF THE HEREIN APPELLANTS. 3. (ITS) DECLARATION THAT THE DONATION PROPTER NUPTIAS IS INNOFICIOUS, IS PREMATURE, AND THUS IT IS ILLEGAL AND UNPROCEDURAL._29 Our Ruling OCT No. 352 in Spanish Although Not Translated into English or Filipino Is Admissible For Lack of Timely Objection Petitioners fault the CA for admitting OCT No. 352 in evidence on the ground that it is written in Spanish language. They posit that "(d)ocumentary evidence in an unofficial language shall not be admitted as evidence, unless accompanied with a translation into English or Filipino."_30 The argument is untenable. The requirement that documents written in an unofficial language must be accompanied with a translation in English or Filipino as a prerequisite for its admission in evidence must be insisted upon by the parties at the trial to enable the court, where a translation has been impugned as incorrect, to decide the issue._31 Where such document, not so accompanied with a translation in English or Filipino, is offered in evidence and not objected to, either by the parties or the court, it must be presumed that the language in which the document is written is understood by all, and the document is admissible in evidence._32 Moreover, Section 36, Rule 132 of the Revised Rules of Evidence provides: SECTION 36. Objection. Objection to evidence offered orally must be made immediately after the offer is made. Objection to a question propounded in the course of the oral examination of a witness shall be made as soon as the grounds therefor shall become reasonably apparent. An offer of evidence in writing shall be objected to within three (3) days after notice of the offer unless a different period is allowed by the court. In any case, the grounds for the objections must be specified. (Emphasis ours) Since petitioners did not object to the offer of said documentary evidence on time, it is now too late in the day for them to question its admissibility. The rule is that evidence not objected may be deemed admitted and may be validly considered by the court in arriving at its judgment._33 This is true even if by its nature, the evidence is inadmissible and would have surely been rejected if it had been challenged at the proper time._34 As a matter of fact, instead of objecting, petitioners admitted the contents of Exhibit "A," that is, OCT No. 352 in their comment_35 on respondents formal offer of documentary evidence. In the said comment, petitioners alleged, among others, that "Exhibits A, B, C, D, E, F and G, are admitted but not for the purpose they are offered because these exhibits being public and official documents are the best evidence of that they contain and not for what a party would like it to prove."_36 Said evidence was admitted by the RTC._37Once admitted without objection, even though not admissible under an objection, We are not inclined now to reject it._38 Consequently, the evidence that was not objected to became property of the case, and all parties to the case are considered amenable to any favorable or unfavorable effects resulting from the said evidence._39 Issues on Impairment of Legitime Should Be Threshed Out in a Special Proceeding, Not in Civil Action for Reconveyance and Damages On the other hand, petitioners are correct in alleging that the issue regarding the impairment of legitime of FortunatoDoronio must be resolved in an action for the settlement of estates of spouses Simeon Doronio and Cornelia Gante. It may not be passed upon in an action for reconveyance and damages. A probate court, in the exercise of its limited jurisdiction, is the best forum to ventilate and adjudge the issue of impairment of legitime as well as other related matters involving the settlement of estate._ 40 687

University of the Cordilleras College of Law First Year C S.Y. 2013 - 2014 An action for reconveyance with damages is a civil action, whereas matters relating to settlement of the estate of a deceased person such as advancement of property made by the decedent, partake of the nature of a special proceeding. Special proceedings require the application of specific rules as provided for in the Rules of Court._41 As explained by the Court in Natcher v. Court of Appeals:_42 Section 3, Rule 1 of the 1997 Rules of Civil Procedure defines civil action and special proceedings, in this wise: a) A civil action is one by which a party sues another for the enforcement or protection of a right, or the prevention or redress of a wrong. A civil action may either be ordinary or special. Both are governed by the rules for ordinary civil actions, subject to specific rules prescribed for a special civil action. x xxx c) A special proceeding is a remedy by which a party seeks to establish a status, a right or a particular fact. As could be gleaned from the foregoing, there lies a marked distinction between an action and a special proceeding. An action is a formal demand of ones right in a court of justice in the manner prescribed by the court or by the law. It is the method of applying legal remedies according to definite established rules. The term "special proceeding" may be defined as an application or proceeding to establish the status or right of a party, or a particular fact. Usually, in special proceedings, no formal pleadings are required unless the statute expressly so provides. In special proceedings, the remedy is granted generally upon an application or motion. Citing American Jurisprudence, a noted authority in Remedial Law expounds further: It may accordingly be stated generally that actions include those proceedings which are instituted and prosecuted according to the ordinary rules and provisions relating to actions at law or suits in equity, and that special proceedings include those proceedings which are not ordinary in this sense, but is instituted and prosecuted according to some special mode as in the case of proceedings commenced without summons and prosecuted without regular pleadings, which are characteristics of ordinary actions x xx. A special proceeding must therefore be in the nature of a distinct and independent proceeding for particular relief, such as may be instituted independently of a pending action, by petition or motion upon notice. Applying these principles, an action for reconveyance and annulment of title with damages is a civil action, whereas matters relating to settlement of the estate of a deceased person such as advancement of property made by the decedent, partake of the nature of a special proceeding, which concomitantly requires the application of specific rules as provided for in the Rules of Court. Clearly, matters which involve settlement and distribution of the estate of the decedent fall within the exclusive province of the probate court in the exercise of its limited jurisdiction. Thus, under Section 2, Rule 90 of the Rules of Court, questions as to advancement made or alleged to have been made by the deceased to any heir may be heard and determined by the court having jurisdiction of the estate proceedings, and the final order of the court thereon shall be binding on the person raising the questions and on the heir. While it may be true that the Rules used the word "may," it is nevertheless clear that the same provision contemplates a probate court when it speaks of the "court having jurisdiction of the estate proceedings." Corollarily, the Regional Trial Court in the instant case, acting in its general jurisdiction, is devoid of authority to render an adjudication and resolve the issue of advancement of the real property in favor of herein petitioner Natcher, inasmuch as Civil Case No. 71075 for reconveyance and annulment of title with damages is not, to our mind, the proper vehicle to thresh out said question. Moreover, under the present circumstances, the RTC of Manila, Branch 55, was not properly constituted as a probate court so as to validly pass upon the question of advancement made by the decedent Graciano Del Rosario to his wife, herein petitioner Natcher.

688

University of the Cordilleras College of Law First Year C S.Y. 2013 - 2014 We likewise find merit in petitioners contention that before any conclusion about the legal share due to a compulsory heir may be reached, it is necessary that certain steps be taken first._43 The net estate of the decedent must be ascertained, by deducting all payable obligations and charges from the value of the property owned by the deceased at the time of his death; then, all donations subject to collation would be added to it. With the partible estate thus determined, the legitime of the compulsory heir or heirs can be established; and only then can it be ascertained whether or not a donation had prejudiced the legitimes._44 Declaration of Validity of Donation Can Be Challenged by an Interested Party Not Impleaded in Petition for Quieting of Title or Declaratory Relief or Where There is No Res Judicata. Moreover, This Court Can Consider a Factual Matter or Unassigned Error in the Interest of Substantial Justice. Nevertheless, petitioners cannot preclude the determination of validity of the deed of donation on the ground that (1) it has been impliedly admitted by respondents; (2) it has already been determined with finality by the RTC in Petition Case No. U-920; or (3) the only issue in an action for reconveyance is who has a better right over the land._45 The validity of the private deed of donation propter nuptias in favor of petitioners predecessors was one of the issues in this case before the lower courts. The pre-trial order_46 of the RTC stated that one of the issues before it is "(w)hether or not the transfer of the whole property covered by OCT No. 352 on the basis of the private deed of donation notwithstanding the discrepancy in the description is valid." Before the CA, one of the errors assigned by respondents is that "THE TRIAL COURT ERRED IN NOT FINDING THAT THE PRIVATE DEED OF DONATION DATED APRIL 26, 1919 WAS NULL AND VOID."_47 The issue of the validity of donation is likewise brought to Us by petitioners as they stated in their Memorandum_48 that one of the issues to be resolved is regarding the alleged fact that "THE HONORABLE COURT OF APPEALS ERRED IN FINDING THE DONATION INVALID." We are thus poised to inspect the deed of donation and to determine its validity. We cannot agree with petitioners contention that respondents may no longer question the validity of the deed of donation on the ground that they already impliedly admitted it. Under the provisions of the Civil Code, a void contract is inexistent from the beginning. The right to set up the defense of its illegality cannot be waived._49 The right to set up the nullity of a void or non-existent contract is not limited to the parties as in the case of annullable or voidable contracts; it is extended to third persons who are directly affected by the contract._50 Consequently, although respondents are not parties in the deed of donation, they can set up its nullity because they are directly affected by the same._51 The subject of the deed being the land they are occupying, its enforcement will definitely affect them. Petitioners cannot also use the finality of the RTC decision in Petition Case No. U-920_52 as a shield against the verification of the validity of the deed of donation. According to petitioners, the said final decision is one for quieting of title._53 In other words, it is a case for declaratory relief under Rule 64 (now Rule 63) of the Rules of Court, which provides: SECTION 1. Who may file petition. Any person interested under a deed, will, contract or other written instrument, or whose rights are affected by a statute, executive order or regulation, or ordinance, may, before breach or violation thereof, bring an action to determine any question of construction or validity arising under the instrument or statute and for a declaration of his rights or duties thereunder. An action for the reformation of an instrument, to quiet title to real property or remove clouds therefrom, or to consolidate ownership under Article 1607 of the Civil Code, may be brought under this rule.

689

University of the Cordilleras College of Law First Year C S.Y. 2013 - 2014 SECTION 2. Parties. All persons shall be made parties who have or claim any interest which would be affected by the declaration; and no declaration shall, except as otherwise provided in these rules, prejudice the rights of persons not parties to the action. (Emphasis ours) However, respondents were not made parties in the said Petition Case No. U-920.1wphi1 Worse, instead of issuing summons to interested parties, the RTC merely allowed the posting of notices on the bulletin boards of Barangay Cabalitaan, Municipalities of Asingan and Lingayen, Pangasinan. As pointed out by the CA, citing the ruling of the RTC: x xx In the said case or Petition No. U-920, notices were posted on the bulletin boards of barangay Cabalitaan, Municipalities of Asingan and Lingayen, Pangasinan, so that there was a notice to the whole world and during the initial hearing and/or hearings, no one interposed objection thereto._54 Suits to quiet title are not technically suits in rem, nor are they, strictly speaking, in personam, but being against the person in respect of the res, these proceedings are characterized as quasi in rem._ 55 The judgment in such proceedings is conclusive only between the parties._56 Thus, respondents are not bound by the decision in Petition Case No. U-920 as they were not made parties in the said case. The rules on quieting of title_57 expressly provide that any declaration in a suit to quiet title shall not prejudice persons who are not parties to the action. That respondents filed a subsequent pleading_58 in the same Petition Case No. U-920 after the decision there had become final did not change the fact that said decision became final without their being impleaded in the case. Said subsequent pleading was dismissed on the ground of finality of the decision._59 Thus, the RTC totally failed to give respondents their day in court. As a result, they cannot be bound by its orders. Generally accepted is the principle that no man shall be affected by any proceeding to which he is a stranger, and strangers to a case are not bound by judgment rendered by the court._60 Moreover, for the principle of res judicata to apply, the following must be present: (1) a decision on the merits; (2) by a court of competent jurisdiction; (3) the decision is final; and (4) the two actions involve identical parties, subject matter and causes of action._61 The fourth element is not present in this case. The parties are not identical because respondents were not impleaded in Petition Case No. U-920. While the subject matter may be the same property covered by OCT No. 352, the causes of action are different. Petition Case No. U-920 is an action for declaratory relief while the case below is for recovery of property. We are not persuaded by petitioners posture that the only issue in this action for reconveyance is who has a better right over the land; and that the validity of the deed of donation is beside the point._62 It is precisely the validity and enforceability of the deed of donation that is the determining factor in resolving the issue of who has a better right over the property. Moreover, notwithstanding procedural lapses as to the appropriateness of the remedies prayed for in the petition filed before Us, this Court can brush aside the technicalities in the interest of justice. In some instances, this Court even suspended its own rules and excepted a case from their operation whenever the higher interests of justice so demanded._63 Moreover, although respondents did not directly raise the issue of validity of the deed of donation at the commencement of the case before the trial court, it was stipulated_64 by the parties during the pre-trial conference. In any event, this Court has authority to inquire into any question necessary in arriving at a just decision of a case before it._65 Though not specifically questioned by the parties, additional issues may also be included, if deemed important for substantial justice to be rendered._66 Furthermore, this Court has held that although a factual issue is not squarely raised below, still in the interest of substantial justice, this Court is not prevented from considering a pivotal factual matter. The Supreme Court is clothed with ample authority to review palpable errors not assigned as such if it finds that their consideration is necessary in arriving at a just decision._67 A rudimentary doctrine on appealed cases is that this Court is clothed with ample authority to review matters, even if they are not assigned as errors on appeal, if it finds that their consideration is necessary at arriving at a just decision of the case._68 Also, an unassigned error closely related to an error properly

690

University of the Cordilleras College of Law First Year C S.Y. 2013 - 2014 assigned or upon which the determination of the question raised by the error properly assigned is dependent, will be considered by the appellate court notwithstanding the failure to assign it as an error._69 Donation Propter Nuptias of Real Property Made in a Private Instrument Before the New Civil Code Took Effect on August 30, 1950 is Void We now focus on the crux of the petition, which is the validity of the deed of donation.1avvphi1 It is settled that only laws existing at the time of the execution of a contract are applicable to it and not the later statutes, unless the latter are specifically intended to have retroactive effect._70 Accordingly, the Old Civil Code applies in this case as the donation propter nuptias was executed in 1919, while the New Civil Code took effect only on August 30, 1950. Under the Old Civil Code, donations propter nuptias must be made in a public instrument in which the property donated must be specifically described._71 Article 1328 of the Old Civil Code provides that gifts propter nuptias are governed by the rules established in Title 2 of Book 3 of the same Code. Article 633 of that title provides that the gift of real property, in order to be valid, must appear in a public document._72 It is settled that a donation of real estate propter nuptias is void unless made by public instrument._73 In the instant case, the donation propter nuptias did not become valid. Neither did it create any right because it was not made in a public instrument._74 Hence, it conveyed no title to the land in question to petitioners predecessors. Logically, then, the cancellation of OCT No. 352 and the issuance of a new TCT No. 44481 in favor of petitioners predecessors have no legal basis. The title to the subject property should, therefore, be restored to its original owners under OCT No. 352. Direct reconveyance to any of the parties is not possible as it has not yet been determined in a proper proceeding who among the heirs of spouses Simeon Doronio and Cornelia Gante is entitled to it. It is still unproven whether or not the parties are the only ones entitled to the properties of spouses Simeon Doronio and Cornelia Gante. As earlier intimated, there are still things to be done before the legal share of all the heirs can be properly adjudicated._75 Titled Property Cannot Be Acquired By Another By Adverse Possession or Extinctive Prescription Likewise, the claim of respondents that they became owners of the property by acquisitive prescription has no merit. Truth to tell, respondents cannot successfully invoke the argument of extinctive prescription. They cannot be deemed the owners by acquisitive prescription of the portion of the property they have been possessing. The reason is that the property was covered by OCT No. 352. A title once registered under the torrens system cannot be defeated even by adverse, open and notorious possession; neither can it be defeated by prescription._76 It is notice to the whole world and as such all persons are bound by it and no one can plead ignorance of the registration._77 The torrens system is intended to guarantee the integrity and conclusiveness of the certificate of registration, but it cannot be used for the perpetration of fraud against the real owner of the registered land._78 The system merely confirms ownership and does not create it. Certainly, it cannot be used to divest the lawful owner of his title for the purpose of transferring it to another who has not acquired it by any of the modes allowed or recognized by law. It cannot be used to protect a usurper from the true owner, nor can it be used as a shield for the commission of fraud; neither does it permit one to enrich himself at the expense of another._79 Where such an illegal transfer is made, as in the case at bar, the law presumes that no registration has been made and so retains title in the real owner of the land._80 Although We confirm here the invalidity of the deed of donation and of its resulting TCT No. 44481, the controversy between the parties is yet to be fully settled. The issues as to who truly are the present owners of the property and what is the extent of their ownership remain unresolved. The same may be properly

691

University of the Cordilleras College of Law First Year C S.Y. 2013 - 2014 threshed out in the settlement of the estates of the registered owners of the property, namely: spouses Simeon Doronio and Cornelia Gante. WHEREFORE, the appealed Decision is REVERSED AND SET ASIDE. A new one is entered: (1) Declaring the private deed of donation propter nuptias in favor of petitioners predecessors NULL AND VOID; and (2) Ordering the Register of Deeds of Pangasinan to: (a) CANCEL Transfer Certificate of Title No. 44481 in the names of MarcelinoDoronio and Veronica Pico; and (b) RESTORE Original Certificate of Title No. 352 in the names of its original owners, spouses Simeon Doronio and Cornelia Gante. SO ORDERED.

692

University of the Cordilleras College of Law First Year C S.Y. 2013 - 2014 Case Digest Heirs of Marcelino vs Heirs of Fortunato Doronio 541 SCRA 479 G.R. No. 169454 Decided on: December 27, 2007 Ponente: REYES, R.T., J.: FACTS: Spouses Simeon Doronio and Cornelia Gante deceased, were the registered owners of a parcel of land located. Marcelino Doronio and Fortunato Doronio, deceased, were the children of the spouses and the parties in this case are their heirs. Petitioners are the heirs of Marcelino Doronio, while respondents are the heirs of Fortunato Doronio. Eager to obtain the entire property, the heirs of Marcelino Doronio and Veronica Pico filed before the RTC in Urdaneta, Pangasinana petition "For the Registration of a Private Deed of Donation" docketed as Petition Case No. U-920. No respondents were named in the said petition although notices of hearing were posted on the bulletin boards of Barangay Cabalitaan, Municipalities of Asingan and Lingayen. During the hearings, no one interposed an objection to the petition. After the RTC ordered a general default, the petition was eventually granted on September 22, 1993. This led to the registration of the deed of donation, cancellation of OCT No. 352 and issuance of anew Transfer Certificate of Title (TCT) No. 44481 in the names of Marcelino Doronio and Veronica Pico. Thus, the entire property was titled in the names of petitioners predecessors. On April 28, 1994,the heirs of Fortunato Doronio filed a pleading before the RTC in the form of a petition in the same Petition Case No. U-920. The petition was for the reconsideration of the decision of the RTC that ordered the registration of the subject deed of donation. It was prayed in the petition that an order be issued declaring null and void the registration of the private deed of donation and that TCT No. 44481 be cancelled. However, the petition was dismissed on the ground that the decision in Petition Case No. U-920 had already become final as it was not appealed. Issue: Can respondents be bound by the decision in Petition Case No. U-920 even if they were not made parties in the said case? Held: Petitioners cannot use the finality of the RTC decision in Petition Case No. U-920 as a shield against the verification of the validity of the deed of donation. According to petitioners, the said final decision is one for quieting of title. In other words, it is a case for declaratory relief under Rule 64 (now Rule 63) of the Rules of Court. Suits to quiet title are not technically suits in rem, nor are they, strictly speaking, in personam, but being against the person in respect of the res, these proceedings are characterized as quasi in rem. The judgment in such proceedings is conclusive only between the parties. Thus, respondents are not bound by the decision in Petition Case No. U-920 as they were not made parties in the said case. The rules on quieting of title expressly provide that any declaration in a suit to quiet title shall not prejudice persons who are not parties to the action. That respondents filed a subsequent pleading in the same Petition Case No. U-920 after the decision there had become final did not change the fact that said decision became final without their being impleaded in the case. Said subsequent pleading was dismissed on the ground of finality of the decision.

693

University of the Cordilleras College of Law First Year C S.Y. 2013 - 2014

XI. The Family Home

694

University of the Cordilleras College of Law First Year C S.Y. 2013 - 2014 Sandejas vs Ignacio Jr. 541 SCRA 61 G.R. No. 155033 December 19, 2007 Full Case ALICE A.I. SANDEJAS, ROSITA A.I. CUSI, PATRICIA A.I. SANDEJAS and BENJAMIN A.I. ESPIRITU, vs SPS. ARTURO IGNACIO, JR. and EVELYN IGNACIO, AUSTRIA-MARTINEZ, J.: Before the Court is a Petition for Review on Certiorari under Rule 45 of the Rules of Court assailing the Decision of the Court of Appeals (CA) in CA-G.R. CV No. 62404 promulgated on August 27, 2002, which affirmed with modification the Decision of the Regional Trial Court (RTC) of Pasig City, Branch 158, in Civil Case No. 65146 dated December 18, 1998. The facts of the case, as summarized by the RTC, are as follows: It appears from the plaintiffs' [petitioners] evidence that Arturo [respondent] is the elder brother of Alice [petitioner] and Rosita [petitioner], Benjamin [petitioner] and Patricia [petitioner] are Arturo's nephew and niece. Arturo and his wife Evelyn [respondent] are residents of the United States. In October 1993, Arturo leased from Dr. Borja a condominium unit identified as Unit 28-C Gilmore Townhomes located at Granada St.,Quezon City. The lease was for the benefit of Benjamin who is the occupant of the unit. The rentals were paid by Ignacio. The term of the lease is for one (1) year and will expire on October 15, 1994. It appears that Arturo was intending to renew the lease contract. As he had to leave for theU.S., Arturo drew up a check, UCPB Check No. GRH-560239 and wrote on it the name of the payee, Dr. Manuel Borja, but left blank the date and amount. He signed the check. The check was intended as payment for the renewal of the lease. The date and the amount were left blank because Arturo does not know when it will be renewed and the new rate of the lease. The check was left with Arturo's sister-in-law, who was instructed to deliver or give it to Benjamin. The check later came to the possession of Alice who felt that Arturo cheated their sister in the amount of three million pesos (P3,000,000.00). She believed that Arturo and Rosita had a joint and/or money market placement in the amount of P3 million with the UCPB branch at Ortigas Ave., San Juan and that Ignacio preterminated the placement and ran away with it, which rightfully belonged to Rosita. Alice then inquired from UCPB Greenhills branch if Arturo still has an account with them. On getting a confirmation, she together with Rosita drew up a scheme to recover the P3 million from Arturo. Alice filled up the date of the check with March 17, 1995 and the amount with three million only. Alice got her driver, Kudera, to stand as the payee of the check, Dr. Borja. Alice and Rosita came to SBC Greenhills Branch together with a man (Kudera) who[m] they introduced as Dr. Borja to the then Assistant Cashier Luis. After introducing the said man as Dr. Borja, Rosita, Alice and the man who was later identified as Kudera opened a Joint Savings Account No. 271-410554-7. As initial deposit for the Joint Savings Account, Alice, Rosita and Kudera deposited the check. No ID card was required of Mr. Kudera because it is an internal policy of the bank that when a valued client opens an account, an identification card is no longer required (TSN, April 21, 1997, pp. 15-16). SBC also allowed the check to be deposited without the endorsement of the impostor Kudera. SBC officials stamped on the dorsal portion of the check endorsement/lack of endorsement guaranteed and sent the check for clearing to the Philippine Clearing House Corporation. 695

University of the Cordilleras College of Law First Year C S.Y. 2013 - 2014 On 21 March 1995, after the check had already been cleared by the drawer bank UCPB, Rosita withdrew P1 million from Joint Savings Account and deposited said amount to the current account of Alice with SBC Greenhills Branch. On the same date, Alice caused the transfer of P2 million from the Joint Savings Account to two (2) Investment Savings Account[s] in the names of Alice, Rosita and/or Patricia. ... On April 4, 1995, a day after Evelyn and Atty. Sanz inquired about the identity of the persons and the circumstances surrounding the deposit and withdrawal of the check, the three million pesos in the two investment savings account[s] and in the current account just opened with SBC were withdrawn by Alice and Rosita. On June 18, 1995, Arturo Ignacio, Jr. and Evelyn Ignacio (respondents) filed a verified complaint for recovery of a sum of money and damages against Security Bank and Trust Company (SBTC) and its officers, namely: Rene Colin D. Gray, Manager; and Sonia Ortiz-Luis, Cashier. The complaint also impleaded herein petitioner Benjamin A.I. Espiritu (Benjamin), a John Doe, representing himself as Manuel N. Borja; and a Jane Doe. On November 7, 1995, the complaint was amended by additionally impleading herein petitioners Alice A.I. Sandejas (Alice), Rosita A.I. Cusi (Rosita) and Patricia A.I. Sandejas (Patricia) as defendants who filed their respective answers and counterclaims. After trial, the RTC following dispositive portion: rendered judgment dated December 18, 1998 with the

WHEREFORE, in view of the foregoing, judgment is rendered in favor of plaintiffs as against defendants Security Bank and Trust Co., Rene Colin Gray, Sonia Ortiz Luis, Alice A.I. Sandejas and Rosita A.I. Cusi, ordering them to pay jointly and severally the plaintiffs the following amounts: (1) P3,000,000.00 plus legal interest on 1995 until the entire amount is fully paid; (2) P500,000.00 as moral damages; (3) P200,000.00 as exemplary damages; (4) P300,000.00 as attorney's fees; plus (5) the cost of suit. it from March 17,

In turn, plaintiffs are directed to pay Benjamin A.I. Espiritu the amount of P100,000.00 as moral damages, P50,000.00 as exemplary damages and another P50,000.00 as attorney's fees. The counterclaims of Patricia A.I. Sandejas are dismissed. SO ORDERED. Both parties appealed the RTC Decision to the CA. On August 14, 1999, during the pendency of the appeal with the CA, herein respondent Arturo Ignacio, Jr. (Arturo) died. On August 27, 2002, the CA promulgated the presently assailed Decision, disposing as follows: WHEREFORE, in view of the foregoing, the assailed decision of the trial court is hereby AFFIRMED with the MODIFICATION that the judgment shall read as follows:

696

University of the Cordilleras College of Law First Year C S.Y. 2013 - 2014 The defendants-appellants Security Bank and Trust Company, Rene Colin D. Gray, Sonia Ortiz-Luis, Alice A.I. Sandejas, and Rosita A.I.Cusi, are hereby ordered to jointly and severally pay the plaintiffs the following amounts: 1. P3,000,000.00 plus legal interest computed 17, 1995 until the entire amount is fully paid; 2. P200,000.00 as moral damages; 3. P100,000.00 as exemplary damages; 4. P50,000.00 as attorney's fees; plus 5. the costs of suit. from March

The award of moral damages, exemplary damages, and attorney's fees in favor of Benjamin Espiritu is DELETED. SO ORDERED. Petitioners and SBTC, together with Gray and Ortiz-Luis, filed their respective petitions for review before this Court. However, the petition filed by SBTC, Gray and Ortiz-Luis, docketed as G.R. No. 155038, was denied in a Resolution issued by this Court on November 20, 2002, for their failure to properly verify the petition, submit a valid certification of non-forum shopping, and attach to the petition the duplicate original or certified true copy of the assailed CA Decision. Said Resolution became final and executory on April 9, 2003. On the other hand, the instant petition was given due course. Petitioners enumerated the following grounds in support of their petition: I. THE COURT OF APPEALS HAD DECIDED A QUESTION OF SUBSTANCE NOT HERETOFORE DECIDED BY THIS COURT AND/OR HAD DECIDED IT IN A WAY PROBABLY NOT IN ACCORD WITH EQUITY, THE LAW AND THE APPLICABLE DECISIONS OF THIS COURT, SUCH AS: (a) IN NOT HOLDING THAT AS BETWEEN SIBLINGS, THE AGGRIEVED SIBLING HAS THE RIGHT TO TAKE MEASURES OR STEPS TO PROTECT HIS OWN INTEREST OR PROPERTY RIGHTS FROM AN ACT OF THE GUILTY SIBLING; (b) IN NOT HOLDING THAT THE ACT OF ROSITA AND ALICE IN FILLING OUT THE BLANK PORTIONS OF THE CHECK TO RECOVER WHAT ARTURO, JR. TOOK FROM AND DUE ROSITA, DID NOT GIVE RISE TO AN ACTIONABLE TORT; (c) IN NOT HOLDING THAT THE CRIMINAL ACT OF ARTURO, JR. IN SUBMITTING AN AFFIDAVIT OF LOSS OF THE CERTIFICATE OF TIME DEPOSIT FOR P3,000,000 THAT RIGHTFULLY BELONGED TO ROSITA JUST TO BE ABLE TO PRE-TERMINATE THE TIME DEPOSIT AND GET ITS FACE VALUE, WHEN HE KNEW IT WAS NOT LOST BUT IN FACT INTACT AND IN THE POSSESSION OF ROSITA, IS A DISHONEST AND REPREHENSIBLE ACT THAT JUSTIFIED ROSITA AND ALICE IN TAKING MEANS TO REGAIN THE MONEY AND TO DENY ARTURO, JR. ANY RIGHT TO RECOVER THE SAID AMOUNT AS WELL AS TO AN AWARD OF DAMAGES;

697

University of the Cordilleras College of Law First Year C S.Y. 2013 - 2014 (d) IN NOT HOLDING THAT THE CRIMINAL ACT OF ARTURO, JR. IN SUBMITTING AN AFFIDAVIT OF LOSS OF THE OWNER'S COPY OF THE TITLE IN MORAYTA AND IN TESTIFYING IN COURT AS TO SUCH, WHEN THAT IS NOT THE TRUTH AS HE KNEW THAT THE ORIGINAL OWNER'S COPY OF THE TITLE WAS WITH ROSITA, IS ANOTHER DISHONEST AND REPREHENSIBLE ACT THAT SHOULD NOT HAVE ENTITLED HIM TO ANY AWARD OF DAMAGES; AND (e) IN NOT APPLYING THE RULE ON PARI DELICTO UNDER ART. 1412 OF THE CIVIL CODE. II. THE COURT OF APPEALS HAD DEPARTED FROM THE USUAL COURSE OF JUDICIAL PROCEEDINGS WHEN IT FAILED TO RESOLVE IN THE APPEAL THE COUNTERCLAIM OF ROSITA AGAINST ARTURO, JR. FOR THE RECOVERY OF THE AMOUNTS LEGALLY HERS THAT SHOULD JUSTIFY ALICE'S BEING ABSOLVED FROM ANY LIABILITY FOR USING THE CHECK IN RECOVERING THE AMOUNT RIGHTFULLY BELONGING TO ROSITA; III. THE COURT OF APPEALS HAD DEPARTED FROM THE USUAL COURSE OF JUDICIAL PROCEEDINGS WHEN IT REVERSED THE TRIAL COURT'S FINDING THAT RESPONDENT WAS GUILTY OF BAD FAITH AND MALICE THAT ENTITLED PETITIONER BENJAMIN A.I. ESPIRITU TO THE AWARD OF DAMAGES NOTWITHSTANDING THAT THERE WAS AMPLE EVIDENCE SHOWN THAT SUCH BAD FAITH AND MALICE WAS MADE AS A LEVERAGE TO COMPEL ARTURO'S SIBLINGS TO RETURN TO HIM THE P3,000,000 WHICH WAS NOT HIS; and, IV. THE COURT OF APPEALS HAD DECIDED THE CASE NOT IN ACCORD WITH LAW WHEN IT DELETED THE AWARD OF DAMAGES TO PETITIONER ESPIRITU AND IN NOT HAVING RULED THAT HE WAS ENTITLED TO A HIGHER AWARD OF DAMAGES CONSIDERING THE CIRCUMSTANCES OF THE CASE AS WELL AS IN NOT HAVING RULED THAT PATRICIA WAS ENTITLED TO AN AWARD OF DAMAGES. Petitioners argue that the CA overlooked and ignored vital pieces of evidence showing that the encashment of the subject check was not fraudulent and, on the contrary, was justified under the circumstances; and that such encashment did not amount to an actionable tort and that it merely called for the application of the civil law rule on pari delicto. In support of these arguments, petitioners contend that the principal adversaries in the present case are full blooded siblings; that the law recognizes the solidarity of family which is why it is made a condition that earnest efforts towards a compromise be exerted before one family member can institute a suit against the other; that even if Arturo previously defrauded Rosita and deprived her of her lawful share in the sale of her property, petitioners Rosita and Alice did not precipitately file suit against him and instead took extra-legal measures to protect Rosita's property rights and at the same time preserve the solidarity of their family and save it from public embarrassment. Petitioners also aver that Rosita's and Alice's act of encashing the subject check is not fraudulent because they did not have any unlawful intent and that they merely took from Arturo what rightfully belonged to Rosita. Petitioners contend that even granting that the act of Rosita and Alice amounted to an actionable tort, they could not be adjudged liable to return the amount to respondents or to pay damages in their favor, because the civil law rule on pari delicto dictates that, when both parties are at fault, neither of them could expect positive relief from courts of justice and, instead, are left in the state where they were at the time of the filing of the case.

698

University of the Cordilleras College of Law First Year C S.Y. 2013 - 2014 Petitioners also contend that the CA erred in failing to award damages to Patricia even if the appellate court sustained the trial court's finding that she was not a party to the fraudulent acts committed by Rosita and Alice. Petitioners argue that even if Patricia did not bother to know the details of the cases against her and left everything to her mother, she did not even know the nature of the case against her, or her superiors in the bank where she worked did not know whether she was the plaintiff or defendant, these were not reasons to deny her award of damages. The fact remains that she had been maliciously dragged into the case and that the suit had adversely affected her work and caused her mental worries and anguish, besmirched reputation, embarrassment and humiliation. As to Benjamin, petitioners aver that the CA also erred in deleting the award of damages and attorney's fees in his favor. Petitioners assert that the trial court found that Benjamin suffered mental anguish, wounded feelings and moral shock as a result of the filing of the present case. Citing the credentials and social standing of Benjamin, petitioners claim that the award of damages and attorney's fees in his favor should be increased. Lastly, petitioners contend that the award of damages and attorney's fees to respondents should be deleted for their failure to establish malice or bad faith on the part of petitioners Alice and Rosita in recovering the P3,000,000.00 which Arturo took from Rosita; and that it is Rosita who is entitled to damages and attorney's fees for Arturo's failure and refusal to give her share in the sale of her property in Morayta. In their Memorandum, respondents simply contend that the issues raised by petitioners are factual in nature and that the settled rule is that questions of fact are not subject to review by the Supreme Court in a petition for review on certiorari under Rule 45 of the Rules of Court. While there are exceptions to this rule, respondents assert that petitioners failed to show that the instant case falls under any of these exceptions. The Courts Ruling The Court finds the petition bereft of merit. There is no compelling reason for the Court to disturb the findings of facts of the lower courts. The trial court's findings are as follows: (1) Rosita failed to establish that there is an agreement between her and Arturo that the latter will give her one-third of the proceeds of the sale of the Morayta property; (2) petitioners were not able to establish by clear and sufficient evidence that the P3,000,000.00 which they took from Arturo when they encashed the subject check was part of the proceeds of the sale of the Morayta property; (3) Rosita's counterclaim is permissive and she failed to pay the full docket and filing fees for her counterclaim. Petitioners challenge the findings of the RTC and insist that they should not be held liable for encashing the subject check because Arturo defrauded Rosita and that he committed deceitful acts which deprived her of her rightful share in the sale of her building in Morayta; that the amount of P3,000,000.00 represented by the check which they encashed formed part of the proceeds of the said sale; that Alice and Rosita were merely moved by their desire to recover from Arturo, Rosita's supposed share in the sale of her property. However, the Court agrees with respondents that only questions of law are entertained in petitions for review on certiorari under Rule 45 of the Rules of Court. The trial courts findings of fact, which the Court of Appeals affirmed, are generally binding and conclusive upon this court. There are recognized exceptions to this rule, among which are: (1) the conclusion is grounded on speculations, surmises or conjectures; (2) the inference is manifestly mistaken, absurd or impossible; (3) there is grave abuse of discretion; (4) the judgment is based on a misapprehension of facts; (5) the findings of facts are conflicting; (6) there is no citation of specific evidence on which the factual findings are based; (7) the finding of absence of facts is contradicted by the presence of evidence on record; (8) the findings of the 699

University of the Cordilleras College of Law First Year C S.Y. 2013 - 2014 CA are contrary to the findings of the trial court; (9) the CA manifestly overlooked certain relevant and undisputed facts that, if properly considered, would justify a different conclusion; (10) the findings of the CA are beyond the issues of the case; and (11) such findings are contrary to the admissions of both parties. In the instant case, petitioners failed to demonstrate that their petition falls under any one of the above exceptions. Petitioners' assignments of errors boil down to the basic issue of whether or not Alice and Rosita are justified in encashing the subject check given the factual circumstances established in the present case. Petitioners' posture is not sanctioned by law. If they truly believe that Arturo took advantage of and violated the rights of Rosita, petitioners should have sought redress from the courts and should not have simply taken the law into their own hands. Our laws are replete with specific remedies designed to provide relief for the violation of one's rights. In the instant case, Rosita could have immediately filed an action for the nullification of the sale of the building she owns in light of petitioners' claim that the document bearing her conformity to the sale of the said building was taken by Arturo from her without her knowledge and consent. Or, in the alternative, as the CA correctly held, she could have brought a suit for the collection of a sum of money to recover her share in the sale of her property in Morayta. In a civilized society such as ours, the rule of law should always prevail. To allow otherwise would be productive of nothing but mischief, chaos and anarchy. As a lawyer, who has sworn to uphold the rule of law, Rosita should know better. She must go to court for relief. It is true that Article 151 of the Family Code requires that earnest efforts towards a compromise be made before family members can institute suits against each other. However, nothing in the law sanctions or allows the commission of or resort to any extra-legal or illegal measure or remedy in order for family members to avoid the filing of suits against another family member for the enforcement or protection of their respective rights. Petitioners invoke the rule of pari delicto to support their contention that respondents do not deserve any relief from the courts. The principle of pari delicto provides that when two parties are equally at fault, the law leaves them as they are and denies recovery by either one of them. Indeed, one who seeks equity and justice must come to court with clean hands. However, in the present case, petitioners were not able to establish that respondents are also at fault. Thus, the principle of pari delicto cannot apply. In any case, the application of the pari delicto principle is not absolute, as there are exceptions to its application. One of these exceptions is where the application of the pari delicto rule would violate wellestablished public policy. The prevention of lawlessness and the maintenance of peace and order are established public policies. In the instant case, to deny respondents relief on the ground of pari delicto would put a premium on the illegal act of petitioners in taking from respondents what the former claim to be rightfully theirs. Petitioners also question the trial court's ruling that their counterclaim is permissive. This Court has laid down the following tests to determine whether a counterclaim is compulsory or not, to wit: (1) Are the issues of fact or law raised by the claim and the counterclaim largely the same? (2) Would res judicata bar a subsequent suit on defendants claims, absent the compulsory countercla im rule? (3) Will substantially the same evidence support or refute plaintiffs claim as well as the defendants counterclaim? and (4) Is there any logical relation between the claim and the counterclaim, such that the conduct of separate trials of the respective claims of the parties would entail a substantial duplication of effort and time by the parties and the court? Tested against the above-mentioned criteria, this Court agrees with the view of the RTC that Rosita's counterclaim for the recovery of her alleged share in the sale of the Morayta property is permissive in nature. The evidence needed to prove respondents' claim to recover the amount 700

University of the Cordilleras College of Law First Year C S.Y. 2013 - 2014 of P3,000,000.00 from petitioners is different from that required to establish Rosita's demands for the recovery of her alleged share in the sale of the subject Morayta property. The recovery of respondents' claim is not contingent or dependent upon the establishment of Rosita's counterclaim such that conducting separate trials will not result in the substantial duplication of the time and effort of the court and the parties. In Sun Insurance Office, Ltd., (SIOL) v. Asuncio, In this Court laid down the rules on the payment of filing fees, to wit: 1. It is not simply the filing of the complaint or appropriate initiatory pleading, but the payment of the prescribed docket fee, that vests a trial court with jurisdiction over the subject-matter or nature of the action. Where the filing of the initiatory pleading is not accompanied by payment of the docket fee, the court may allow payment of the fee within a reasonable time but in no case beyond the applicable prescriptive or reglementary period. 2. The same rule applies to permissive counterclaims, third-party claims and similar pleadings, which shall not be considered filed until and unless the filing fee prescribed therefor is paid. The court may allow payment of said fee within a reasonable time but also in no case beyond its applicable prescriptive or reglementary period. 3. Where the trial court acquires jurisdiction over a claim by the filing of the appropriate pleading and payment of the prescribed filing fee but, subsequently, the judgment awards a claim not specified in the pleading, or if specified the same has been left for determination by the court, the additional filing fee therefor shall constitute a lien on the judgment. It shall be the responsibility of the Clerk of Court or his duly authorized deputy to enforce said lien and assess and collect the additional fee. In order for the trial court to acquire jurisdiction over her permissive counterclaim, Rosita is bound to pay the prescribed docket fees. Since it is not disputed that Rosita never paid the docket and filing fees, the RTC did not acquire jurisdiction over her permissive counterclaim. Nonetheless, the trial court ruled on the merits of Rosita's permissive counterclaim by dismissing the same on the ground that she failed to establish that there is a sharing agreement between her and Arturo with respect to the proceeds of the sale of the subject Morayta property and that the amount of P3,000,000.00 represented by the check which Rosita and Aliceen cashed formed part of the proceeds of the said sale. It is settled that any decision rendered without jurisdiction is a total nullity and may be struck down at any time, even on appeal before this Court. In the present case, considering that the trial court did not acquire jurisdiction over the permissive counterclaim of Rosita, any proceeding taken up by the trial court and any ruling or judgment rendered in relation to such counterclaim is considered null and void. In effect, Rosita may file a separate action against Arturo for recovery of a sum of money. However, Rosita's claims for damages and attorney's fees are compulsory as they necessarily arise as a result of the filing by respondents of their complaint. Being compulsory in nature, payment of docket fees is not required. Nonetheless, since petitioners are found to be liable to return to respondents the amount of P3,000,000.00 as well as to pay moral and exemplary damages and attorney's fees, it necessarily follows that Rosita's counterclaim for damages and attorney's fees should be dismissed as correctly done by the RTC and affirmed by the CA. As to Patricia's entitlement to damages, this Court has held that while no proof of pecuniary loss is necessary in order that moral damages may be awarded, the amount of indemnity being left to the discretion of the court, it is nevertheless essential that the claimant should satisfactorily show the 701

University of the Cordilleras College of Law First Year C S.Y. 2013 - 2014 existence of the factual basis of damages and its causal connection to defendants acts. This is so because moral damages, though incapable of pecuniary estimation, are in the category of an award designed to compensate the claimant for actual injury suffered and not to impose a penalty on the wrongdoer. Moreover, additional facts must be pleaded and proven to warrant the grant of moral damages under the Civil Code, these being, social humiliation, wounded feelings, grave anxiety, etc. that resulted from the act being complained of. In the present case, both the RTC and the CA were not convinced that Patricia is entitled to damages. Quoting the RTC, the CA held thus: With respect to Patricia, she did not even bother to know the details of the case against her, she left everything to the hands of her mother Alice. Her attitude towards the case appears weird, she being a banker who seems so concerned of her reputation. Aside from the parties to this case, her immediate superiors in the BPI knew that she is involved in a case. They did not however know whether she is the plaintiff or the defendant in the case. Further, they did not know the nature of the case that she is involved in. It appears that Patricia has not suffered any of the injuries enumerated in Article 2217 of the Civil Code, thus, she is not entitled to moral damages and attorney's fees. This Court finds no cogent reason to depart from the above-quoted findings as Patricia failed to satisfactorily show the existence of the factual basis for granting her moral damages and the causal connection of such fact to the act of respondents in filing a complaint against her. In addition, and with respect to Benjamin, the Court agrees with the CA that in the absence of a wrongful act or omission, or of fraud or bad faith, moral damages cannot be awarded. The adverse result of an action does not per se make the action wrongful, or the party liable for it. One may err, but error alone is not a ground for granting such damages. In the absence of malice and bad faith, the mental anguish suffered by a person for having been made a party in a civil case is not the kind of anxiety which would warrant the award of moral damages. A resort to judicial processes is not, per se, evidence of ill will upon which a claim for damages may be based. In China Banking Corporation v. Court of Appeals, this Court held: Settled in our jurisprudence is the rule that moral damages cannot be recovered from a person who has filed a complaint against another in good faith, or without malice or bad faith (Philippine National Bank v. Court of Appeals, 159 SCRA 433 [1988]; R & B Surety and Insurance v. Intermediate Appellate Court, 129 SCRA 736 [1984]). If damage results from the filing of the complaint, it is damnum absque injuria (IlocosNorte Electrical Company v. Court of Appeals, 179 SCRA 5 [1989]). In the present case, the Court agrees with the RTC and the CA that petitioners failed to establish that respondents were moved by bad faith or malice in impleading Patricia and Benjamin. Hence, Patricia and Benjamin are not entitled to damages. The Court sustains the award of moral and exemplary damages as well as attorney's fees in favor of respondents. As to moral damages, Article 20 of the Civil Code provides that every person who, contrary to law, willfully or negligently causes damage to another, shall indemnify the latter for the same. In addition, Article 2219 (10) of the Civil Code provides that moral damages may be recovered in acts or actions referred to in Articles 21, 26, 27, 28, 29, 30, 32, 34 and 35 of the same Code. More particularly, Article 21 of the said Code provides that any person who willfully causes loss or injury to another in a manner 702

University of the Cordilleras College of Law First Year C S.Y. 2013 - 2014 that is contrary to morals, good customs, or public policy shall compensate the latter for the damage. In the present case, the act of Alice and Rosita in fraudulently encashing the subject check to the prejudice of respondents is certainly a violation of law as well as of the public policy that no one should put the law into his own hands. As to SBTC and its officers, their negligence is so gross as to amount to a willfull injury to respondents. The banking system has become an indispensable institution in the modern world and plays a vital role in the economic life of every civilized society. Whether as mere passive entities for the safe-keeping and saving of money or as active instruments of business and commerce, banks have attained a ubiquitous presence among the people, who have come to regard them with respect and even gratitude and most of all, confidence. For this reason, banks should guard against injury attributable to negligence or bad faith on its part. There is no hard-and-fast rule in the determination of what would be a fair amount of moral damages since each case must be governed by its own peculiar facts. The yardstick should be that it is not palpably and scandalously excessive. Moreover, the social standing of the aggrieved party is essential to the determination of the proper amount of the award. Otherwise, the goal of enabling him to obtain means, diversions, or amusements to restore him to the status quo ante would not be achieved. In the present case, the Court finds no cogent reason to modify the amount of moral damages granted by the CA. Likewise, the Court finds no compelling reason to disturb the modifications made by the CA on the award of exemplary damages and attorney's fees. Under Article 2229 of the Civil Code, exemplary or corrective damages are imposed by way of example or correction for the public good, in addition to moral, temperate, liquidated, or compensatory damages. In the instant case, the award of exemplary damages in favor of respondents is in order for the purpose of deterring those who intend to enforce their rights by taking measures or remedies which are not in accord with law and public policy. On the part of respondent bank, the public relies on a bank's sworn profession of diligence and meticulousness in giving irreproachable service. Hence, the level of meticulousness must be maintained at all times by the banking sector. In the present case the award of exemplary damages is justified by the brazen acts of petitioners Rosita and Alice in violating the law coupled with the gross negligence committed by respondent bank and its officers in allowing the subject check to be deposited which later paved the way for its encashment. As to attorney's fees, Article 2208 of the same Code provides, among others, that attorney's fees may be recovered when exemplary damages are awarded or when the defendant's act or omission has compelled the plaintiff to litigate with third persons or to incur expenses to protect his interest. WHEREFORE, the instant petition is DENIED. The Decision of the Court of Appeals dated August 27, 2002 in CA-G.R. CV No. 62404 is AFFIRMED. Costs against the petitioners. SO ORDERED.

703

University of the Cordilleras College of Law First Year C S.Y. 2013 - 2014 Case Digest Sandejas vs Ignacio Jr. 541 SCRA 61 G.R. No. 155033 Decided on: December 19, 2007 Ponente: AUSTRIA-MARTINEZ, J.: FACTS: Arturo drew up a check, UCPB Check No. GRH-560239 and wrote on it the name of the payee, Dr. Manuel Borja, but left blank the date and amount. He signed the check. The check was left with Arturo's sister-in-law, who was instructed to deliver or give it to Benjamin. The check later came to the possession of Alice who felt that Arturo cheated their sister Rosita in the amount of three million pesos (P3,000,000.00). She believed that Arturo and Rosita had a joint and/or money market placement in the amount of P3 million with the UCPB branch at Ortigas Ave., San Juan and that Ignacio pre-terminated the placement and ran away with it, which rightfully belonged to Rosita. She together with Rosita drew up a scheme to recover the P3 million from Arturo. Alice got her driver, Kudera, to stand as the payee of the check, Dr. Borja. Alice and Rosita came to SBC Greenhills Branch together with a man (Kudera) who[m] they introduced as Dr. Borja to the then Assistant Cashier Luis. They opened a Joint Savings Account. As initial deposit for the Joint Savings Account, Alice, Rosita and Kudera deposited the check. Thereafter, they successfully withdraw the amount. Arturo Ignacio, Jr. and Evelyn Ignacio (respondents) filed a verified complaint for recovery of a sum of money and damages. Judgment is rendered in favor of plaintiffs as against defendants Security Bank and Trust Co., Rene Colin Gray, Sonia Ortiz Luis, Alice A.I. Sandejas and Rosita A.I. Cusi. The counterclaims of Patricia A.I. Sandejas are dismissed. Both parties appealed the RTC Decision to the CA. The defendants-appellants, Security Bank and Trust Company, Rene Colin D. Gray, Sonia Ortiz-Luis, Alice A.I. Sandejas,and Rosita A.I. Cusi, are ordered to jointly and severally pay the plaintiffs. Petitioners and SBTC,together with Gray and Ortiz-Luis, filed their respective petitions for review before this Court. ISSUE: 1.Whether or not Alice and Rosita are justified in encashing the subject check given the factual circumstances established in the present case. 2. Whether or not the petitioners can hold respondent liable for moral damages as effect of his complaint. DECISION OF THE COURT: Petitioners' posture is not sanctioned by law. If they truly believe that Arturo took advantage of and violated the rights of Rosita, petitioners should have sought redress from the courts and should not have simply taken the law into their own hands. Our laws are replete with specific remedies designed to provide relief for the violation of one's rights. It is true that Article151 of the Family Code requires that earnest efforts towards a compromise be made before family members can institute suits against each other. However, nothing in the law sanctions or allows the commission of or resort to any extra-legal or illegal measure or remedy in order for family members to avoid the filing of suits against another family member for the enforcement or protection of their respective rights. As to Patricia's entitlement to damages, this Court has held that while no proof of pecuniary loss is necessary in order that moral damages may be awarded, the amount of indemnity being left to the discretion of the court, it is nevertheless essential that the claimant should satisfactorily show the existence of the factual basis of damages and its causal connection to defendants acts. In the present case, both the RTC and the CA were not convinced that Patricia is entitled to damages. In addition, and with respect to Benjamin, the Court agrees with the CA that in the absence of a wrongful act or omission, or of fraud or bad faith, moral damages cannot be awarded. WHEREFORE, the instant petition is DENIED. The Decision of the Court of Appeals dated August 27, 2002 in CA-G.R. CV No. 62404 is AFFIRMED. Costs against the petitioners

704

University of the Cordilleras College of Law First Year C S.Y. 2013 - 2014 Modequili vs Breva 185 SCRA 766 G.R. No. 86355 May 31, 1990 Full Case JOSE MODEQUILLO, petitioner, vs. HON. AUGUSTO V. BREVA FRANCISCO SALINAS, FLORIPER ABELLAN-SALINAS, JUANITO CULAN-CULAN and DEPUTY SHERIFF FERNANDO PLATA respondents. GANCAYCO, J.: The issue in this petition is whether or not a final judgment of the Court of Appeals in an action for damages may be satisfied by way of execution of a family home constituted under the Family Code. The facts are undisputed. On January 29, 1988, a judgment was rendered by the Court of Appeals in CA-G.R. CV No. 09218 entitled "Francisco Salinas, et al. vs. Jose Modequillo, et al.," the dispositive part of which read as follows: WHEREFORE, the decision under appeal should be, as it is hereby, reversed and set aside. Judgment is hereby rendered finding the defendants-appellees Jose Modequillo and Benito Malubay jointly and severally liable to plaintiffs-appellants as herein below set forth. Accordingly, defendants-appellees are ordered to pay jointly and severally to: 1. Plaintiffs-appellants, the Salinas spouses: a. the amount of P30,000.00 by way of compensation for the death of their son Audie Salinas; b. P10,000.00 for the loss of earnings by reason of the death of said Audie Salinas; c. the sum of P5,000.00 as burial expenses of Audie Salinas; and d. the sum of P5,000.00 by way of moral damages. 2. Plaintiffs-appellants Culan-Culan: a. the sum of P5,000.00 for hospitalization expenses of Renato Culan- Culan; and b. P5,000.00 for moral damages. 3. Both plaintiff-appellants Salinas and Culan-Culan, P7,000.00 for attorney's fees and litigation expenses. All counterclaims and other claims are hereby dismissed. The said judgment having become final and executory, a writ of execution was issued by the Regional Trial Court of Davao City to satisfy the said judgment on the goods and chattels of the defendants Jose Modequillo and Benito Malubay at Malalag, Davao del Sur. On July 7, 1988, the sheriff levied on a parcel of residential land located at Poblacion Malalag, Davao del Sur containing an area of 600 square meters with a market value of P34,550.00 and assessed value of P7,570.00 per Tax Declaration No. 87008-01359, registered in the name of Jose Modequillo in the office of the Provincial Assessor of Davao del Sur; and a parcel of agricultural land located at Dalagbong Bulacan, Malalag, Davao del Sur containing an area of 3 hectares with a market value of P24,130.00 and 705

University of the Cordilleras College of Law First Year C S.Y. 2013 - 2014 assessed value of P9,650.00 per Tax Declaration No. 87-08-01848 registered in the name of Jose Modequillo in the office of the Provincial Assessor of Davao del Sur. A motion to quash and/or to set aside levy of execution was filed by defendant Jose Modequillo alleging therein that the residential land located at Poblacion Malalag is where the family home is built since 1969 prior to the commencement of this case and as such is exempt from execution, forced sale or attachment under Articles 152 and 153 of the Family Code except for liabilities mentioned in Article 155 thereof, and that the judgment debt sought to be enforced against the family home of defendant is not one of those enumerated under Article 155 of the Family Code. As to the agricultural land although it is declared in the name of defendant it is alleged to be still part of the public land and the transfer in his favor by the original possessor and applicant who was a member of a cultural minority was not approved by the proper government agency. An opposition thereto was filed by the plaintiffs. In an order dated August 26, 1988, the trial court denied the motion. A motion for reconsideration thereof was filed by defendant and this was denied for lack of merit on September 2, 1988. Hence, the herein petition for review on certiorari wherein it is alleged that the trial court erred and acted in excess of its jurisdiction in denying petitioner's motion to quash and/or to set aside levy on the properties and in denying petitioner' motion for reconsideration of the order dated August 26, 1988. Petitioner contends that only a question of law is involved in this petition. He asserts that the residential house and lot was first occupied as his family residence in 1969 and was duly constituted as a family home under the Family Code which took effect on August 4, 1988. Thus, petitioner argues that the said residential house and lot is exempt from payment of the obligation enumerated in Article 155 of the Family Code; and that the decision in this case pertaining to damages arising from a vehicular accident took place on March 16, 1976 and which became final in 1988 is not one of those instances enumerated under Article 155 of the Family Code when the family home may be levied upon and sold on execution. It is further alleged that the trial court erred in holding that the said house and lot became a family home only on August 4, 1988 when the Family Code became effective, and that the Family Code cannot be interpreted in such a way that all family residences are deemed to have been constituted as family homes at the time of their occupancy prior to the effectivity of the said Code and that they are exempt from execution for the payment of obligations incurred before the effectivity of said Code; and that it also erred when it declared that Article 162 of the Family Code does not state that the provisions of Chapter 2, Title V have a retroactive effect. Articles 152 and 153 of the Family Code provide as follows: Art. 152. The family home, constituted jointly by the husband and the wife or by an unmarried head of a family, is the dwelling house where they and their family reside, and the land on which it is situated. Art. 153. The family home is deemed constituted on a house and lot from the time it is occupied as a family residence. From the time of its constitution and so long as any of its beneficiaries actually resides therein, the family home continues to be such and is exempt from execution, forced sale or attachment except as hereinafter provided and to the extent of the value allowed by law. Under the Family Code, a family home is deemed constituted on a house and lot from the time it is occupied as a family residence. There is no need to constitute the same judicially or extrajudicially as required in the Civil Code. If the family actually resides in the premises, it is, therefore, a family home as contemplated by law. Thus, the creditors should take the necessary precautions to protect their interest before extending credit to the spouses or head of the family who owns the home. Article 155 of the Family Code also provides as follows: Art. 155. The family home shall be exempt from execution, forced sale or attachment except: 706

University of the Cordilleras College of Law First Year C S.Y. 2013 - 2014 (1) For non-payment of taxes; (2) For debts incurred prior to the constitution of the family home; (3) For debts secured by mortgages on the premises before or after such constitution; and (4) For debts due to laborers, mechanics, architects, builders, material men and others who have rendered service or furnished material for the construction of the building. The exemption provided as aforestated is effective from the time of the constitution of the family home as such, and lasts so long as any of its beneficiaries actually resides therein. In the present case, the residential house and lot of petitioner was not constituted as a family home whether judicially or extrajudicially under the Civil Code. It became a family home by operation of law only under Article 153 of the Family Code. It is deemed constituted as a family home upon the effectivity of the Family Code on August 3, 1988 not August 4, one year after its publication in the Manila Chronicle on August 4, 1987 (1988 being a leap year). The contention of petitioner that it should be considered a family home from the time it was occupied by petitioner and his family in 1969 is not well- taken. Under Article 162 of the Family Code, it is provided that "the provisions of this Chapter shall also govern existing family residences insofar as said provisions are applicable." It does not mean that Articles 152 and 153 of said Code have a retroactive effect such that all existing family residences are deemed to have been constituted as family homes at the time of their occupation prior to the effectivity of the Family Code and are exempt from execution for the payment of obligations incurred before the effectivity of the Family Code. Article 162 simply means that all existing family residences at the time of the effectivity of the Family Code, are considered family homes and are prospectively entitled to the benefits accorded to a family home under the Family Code. Article 162 does not state that the provisions of Chapter 2, Title V have a retroactive effect. Is the family home of petitioner exempt from execution of the money judgment aforecited No. The debt or liability which was the basis of the judgment arose or was incurred at the time of the vehicular accident on March 16, 1976 and the money judgment arising therefrom was rendered by the appellate court on January 29, 1988. Both preceded the effectivity of the Family Code on August 3, 1988. This case does not fall under the exemptions from execution provided in the Family Code. As to the agricultural land subject of the execution, the trial court correctly ruled that the levy to be made by the sheriff shall be on whatever rights the petitioner may have on the land. WHEREFORE, the petition is DISMISSED for lack of merit. No pronouncement as to costs. SO ORDERED.

707

University of the Cordilleras College of Law First Year C S.Y. 2013 - 2014 Case Digest Modequili vs Breva 185 SCRA 766 G.R. No. 86355 Decided on: May 31, 1990 Ponente: GANCAYCO, J.:

FACTS: The sheriff levied on a parcel of residential land located at Poblacion Malalag, Davao del Sur on July 1988, registered in the name of Jose Mondequillo and a parcel of agricultural land located at Dalagbong Bulacan, Malalag, Davao de Sur also registered in the latters name. A motion to quash was filed by the petitioner alleging that the residential land is where the family home is built since 1969 prior the commencement of this case and as such is exempt from execution, forced sale or attachment under Article 152 and 153 except for liabilities mentioned in Article 155 thereof, and that the judgment sought to be enforced against the family home is not one of those enumerated. With regard to the agricultural land, it is alleged that it is still part of the public land and the transfer in his favor by the original possessor and applicant who was a member of a cultural minority. The residential house in the present case became a family home by operation of law under Article 153. ISSUE: WON the subject property is deemed to be a family home. HELD: The petitioners contention that it should be considered a family home from the time it was occupied by petitioner and his family in 1969 is not well-taken. Under Article 162 of the Family Code, it provides that the provisions of this Chapter shall govern existing family residences insofar as said provisions are applicable. It does not mean that Article 152 and 153 shall have a retroactive effect such that all existing family residences are deemed to have been constituted as family homes at the time of their occupation prior to the effectivity of the Family Code and are exempt from the execution for payment of obligations incurred before the effectivity of the Code. The said article simply means that all existing family residences at the time of the effectivity of the Family Code, are considered family homes and are prospectively entitled to the benefits accorded to a family home under the FC. The debt and liability which was the basis of the judgment was incurred prior the effectivity of the Family Code. This does not fall under the exemptions from execution provided in the FC. As to the agricultural land, trial court correctly ruled that the levy to be made shall be on whatever rights the petitioner may have on the land. Petition was dismissed.

708

University of the Cordilleras College of Law First Year C S.Y. 2013 - 2014 Manacop vs Court of Appeals 215 SCRA 773 G.R. No. 97898. August 11, 1997 Full Case FLORANTE F. MANACOP, petitioner, vs. COURT OF APPEALS and E & L MERCANTILE, INC., respondents. PANGANIBAN, J.: May a writ of execution of a final and executory judgment issued before the effectivity of the Family Code be executed on a house and lot constituted as a family home under the provision of said Code? Statement of the Case This is the principal question posed by petitioner in assailing the Decision of Respondent Court of Appeals in CA-G.R. SP No. 18906 promulgated on February 21, 1990 and its Resolution promulgated on March 21, 1991, affirming the orders issued by the trial court commanding the issuance of various writs of execution to enforce the latters decision in Civil Case No. 53271. The Facts Petitioner Florante F. Manacop and his wife Eulaceli purchased on March 10, 1972 a 446-square-meter residential lot with a bungalow, in consideration of P75,000.00. The property, located in Commonwealth Village, Commonwealth Avenue, Quezon City, is covered by Transfer Certificate of Title No. 174180. On March 17, 1986, Private Respondent E & L Mercantile, Inc. filed a complaint against petitioner and F.F. Manacop Construction Co., Inc. before the Regional Trial Court of Pasig, Metro Manila to collect an indebtedness of P3,359,218.45. Instead of filing an answer, petitioner and his company entered into a compromise agreement with private respondent, the salient portion of which provides: c. That defendants will undertake to pay the amount of P2,000,000.00 as and when their means permit, but expeditiously as possible as their collectibles will be collected. (sic) On April 20, 1986, the trial court rendered judgment approving the aforementioned compromise agreement. It enjoined the parties to comply with the agreement in good faith. On July 15, 1986, private respondent filed a motion for execution which the lower court granted on September 23, 1986. However, execution of the judgment was delayed. Eventually, the sheriff levied on several vehicles and other personal properties of petitioner. In partial satisfaction of the judgment debt, these chattels were sold at public auction for which certificates of sale were correspondingly issued by the sheriff. On August 1, 1989, petitioner and his company filed a motion to quash the alias writs of execution and to stop the sheriff from continuing to enforce them on the ground that the judgment was not yet executory. They alleged that the compromise agreement had not yet matured as there was no showing that they had the means to pay the indebtedness or that their receivables had in fact been collected. They buttressed their motion with supplements and other pleadings. On August 11, 1989, private respondent opposed the motion on the following grounds: (a) it was too late to question the September 23, 1986 Order considering that more than two years had elapsed; (b) the second alias writ of execution had been partially implemented; and (c) petitioner and his company were in bad faith in refusing to pay their indebtedness notwithstanding that from February 1984 to January 5, 1989, they had collected the total amount of P41,664,895.56. On September 21, 1989, private respondent filed an opposition to petitioner and his companys addendum to the motion to quash the writ of 709

University of the Cordilleras College of Law First Year C S.Y. 2013 - 2014 execution. It alleged that the property covered by TCT No. 174180 could not be considered a family home on the grounds that petitioner was already living abroad and that the property, having been acquired in 1972, should have been judicially constituted as a family home to exempt it from execution. On September 26, 1989, the lower court denied the motion to quash the writ of execution and the prayers in the subsequent pleadings filed by petitioner and his company. Finding that petitioner and his company had not paid their indebtedness even though they collected receivables amounting to P57,224,319.75, the lower court held that the case had become final and executory. It also ruled that petitioners residence was not exempt from execution as it was not duly constituted as a family home, pursuant to the Civil Code. Hence, petitioner and his company filed with the Court of Appeals a petition for certiorari assailing the lower courts Orders of September 23, 1986 and September 26, 1989. On February 21, 1990, Respondent Court of Appeals rendered its now questioned Decision dismissing the petition for certiorari. The appellate court quoted with approval the findings of the lower court that: (a) the judgment based on the compromise agreement had become final and executory, stressing that petitioner and his company had collected the total amount of P57,224,319.75 but still failed to pay their indebtedness and (b) there was no showing that petitioners residence had been duly constituted as a family home to exempt it from execution. On the second finding, the Court of Appeals added that: x xx. We agree with the respondent judge that there is no showing in evidence that petitioner Maacops residence under TCT 174180 has been duly constituted as a family home in accordance with law. For one thing, it is the clear implication of Article 153 that the family home continues to be so deemed constituted so long as any of its beneficiaries enumerated in Article 154 actually resides therein. Conversely, it ceases to continue as such family home if none of its beneficiaries actually occupies it. There is no showing in evidence that any of its beneficiaries is actually residing therein. On the other hand, the unrefuted assertion of private respondent is that petitioner FloranteMaacop had already left the country and is now, together with all the members of his family, living in West Covina, Los Angeles, California, U.S.A. Petitioner and his company filed a motion for reconsideration of this Decision on the ground that the property covered by TCT No. 174180 was exempt from execution. On March 21, 1991, the Court of Appeals rendered the challenged Resolution denying the motion. It anchored its ruling on Modequillov. Breva, which held that all existing family residences at the time of the effectivity of the Family Code are considered family homes and are prospectively entitled to the benefits accorded to a family home under the Family Code. Applying the foregoing pronouncements to this case, the Court of Appeals explained: The record of the present case shows that petitioners incurred the debt of P3,468,000.00 from private respondent corporation on February 18, 1982 (Annex `A, Petition). The judgment based upon the compromise agreement was rendered by the court on April 18, 1986 (Annex `C, Ibid). Paraphrasing the aforecitedModequillo case, both the debt and the judgment preceded the effectivity of the Family Code on August 3, 1988. Verily, the case at bar does not fall under the exemptions from execution provided under Article 155 of the Family Code. Undeterred, petitioner filed the instant petition for review on certiorari arguing that the Court of Appeals misapplied Modequillo. He contends that there was no need for him to constitute his house and lot as a family home for it to be treated as such since he was and still is a resident of the same property from the time it was levied upon and up to this moment. The Issue

710

University of the Cordilleras College of Law First Year C S.Y. 2013 - 2014 As stated in the opening sentence of this Decision, the issue in this case boils down to whether a final and executory decision promulgated and a writ of execution issued before the effectivity of the Family Code can be executed on a family home constituted under the provisions of the said Code. The Courts Ruling We answer the question in the affirmative. The Court of Appeals committed no reversible error. On the contrary, its Decision and Resolution are supported by law and applicable jurisprudence. No Novel Issue At the outset, the Court notes that the issue submitted for resolution in the instant case is not entirely new. In Manacopv. Court of Appeals, petitioner himself as a party therein raised a similar question of whether this very same property was exempt from preliminary attachment for the same excuse that it was his family home. In said case, F.F. Cruz & Co., Inc. filed a complaint for a sum of money. As an incident in the proceedings before it, the trial court issued a writ of attachment on the said house and lot. In upholding the trial court (and the Court of Appeals) in that case, we ruled that petitioner incurred the indebtedness in 1987 or prior to the effectivity of the Family Code on August 3, 1988. Hence, petitioners family home was not exempt from attachment by sheer force of exclusion embodied in paragraph 2, Article 155 of the Family Code cited in Modequillo, where the Court categorically ruled: Under the Family Code, a family home is deemed constituted on a house and lot from the time it is occupied as a family residence. There is no need to constitute the same judicially or extrajudicially as required in the Civil Code. If the family actually resides in the premises, it is, therefore, a family home as contemplated by law. Thus, the creditors should take the necessary precautions to protect their interest before extending credit to the spouses or head of the family who owns the home. Article 155 of the Family Code also provides as follows: Art. 155. The family home shall be exempt from execution, forced sale or attachment except: (1) For nonpayment of taxes; (2) For debts incurred prior to the constitution of the family home; (3) For debts secured by mortgages on the premises before or after such constitution; and (4) For debts due to laborers, mechanics, architects, builders, materialmen and others who have rendered service or furnished material for the construction of the building. The exemption provided as aforestated is effective from the time of the constitution of the family home as such, and lasts so long as any of its beneficiaries actually resides therein. In the present case, the residential house and lot of petitioner was not constituted as a family home whether judicially or extrajudicially under the Civil Code. It became a family home by operation of law only under Article 153 of the Family Code. It is deemed constituted as a family home upon the effectivity of the Family Code on August 3, 1988 not August 4, one year after its publication in the Manila Chronicle on August 4, 1987 (1988 being a leap year). The contention of petitioner that it should be considered a family home from the time it was occupied by petitioner and his family in 1960 is not well-taken. Under Article 162 of the Family Code, it is provided that `the provisions of this Chapter shall also govern existing family residences insofar as said provisions are applicable. It does not mean that Articles 152 and 153 of said Code have a retroactive effect such that all existing family residences are deemed to have been constituted as family homes at the time of their occupation prior to the effectivity of the Family Code and are exempt from execution for the payment of 711

University of the Cordilleras College of Law First Year C S.Y. 2013 - 2014 obligations incurred before the effectivity of the Family Code. Article 162 simply means that all existing family residences at the time of the effectivity of the Family Code, are considered family homes and are prospectively entitled to the benefits accorded to a family home under the Family Code. Article 162 does not state that the provisions of Chapter 2, Title V have a retroactive effect. Is the family home of petitioner exempt from execution of the money judgment aforecited? No. The debt or liability which was the basis of the judgment arose or was incurred at the time of the vehicular accident on March 16, 1976 and the money judgment arising therefrom was rendered by the appellate court on January 29, 1988. Both preceded the effectivity of the Family Code on August 3, 1988. This case does not fall under the exemptions from execution provided in the Family Code. Article 153 of the Family Code Has No Retroactive Effect Petitioner contends that the trial court erred in holding that his residence was not exempt from execution in view of his failure to show that the property involved has been duly constituted as a family home in accordance with law. He asserts that the Family Code and Modequillo require simply the occupancy of the property by the petitioner, without need for its judicial or extrajudicial constitution as a family home. Petitioner is only partly correct. True, under the Family Code which took effect on August 3, 1988. The subject property became his family home under the simplified process embodied in Article 153 of said Code. However, Modequillo explicitly ruled that said provision of the Family Code does not have retroactive effect. In other words, prior to August 3, 1988, the procedure mandated by the Civil Code had to be followed for a family home to be constituted as such. There being absolutely no proof that the subject property was judicially or extrajudicially constituted as a family home, it follows that the laws protective mantle cannot be availed of by petitioner. Since the debt involved herein was incurred and the assailed orders of the trial court issued prior to August 3, 1988, the petitioner cannot be shielded by the benevolent provisions of the Family Code. List of Beneficiary-Occupants Restricted to Those Enumerated in the Code In view of the foregoing discussion, there is no reason to address the other arguments of petitioner other than to correct his misconception of the law. Petitioner contends that he should be deemed residing in the family home because his stay in the United States is merely temporary. He asserts that the person staying in the house is his overseer and that whenever his wife visited this country, she stayed in the family home. This contention lacks merit. The law explicitly provides that occupancy of the family home either by the owner thereof or by any of its beneficiaries must be actual. That which is actual is something real, or actually existing, as opposed to something merely possible, or to something which is presumptive or constructive. Actual occupancy, however, need not be by the owner of the house specifically. Rather, the property may be occupied by the beneficiaries enumerated by Article 154 of the Family Code. Art. 154. The beneficiaries of a family home are: (1) The husband and wife, or an unmarried person who is the head of the family; and (2) Their parents, ascendants, descendants, brothers and sisters, whether the relationship be legitimate or illegitimate, who are living in the family home and who depend upon the head of the family for lead support. This enumeration may include the in-laws where the family home is constituted jointly by the husband and wife. But the law definitely excludes maids and overseers. They are not the beneficiaries contemplated by the Code. Consequently, occupancy of a family home by an overseer like Carmencita V. Abat in this case is insufficient compliance with the law. 712

University of the Cordilleras College of Law First Year C S.Y. 2013 - 2014 WHEREFORE, the petition is hereby DENIED for utter lack of merit. This Decision is immediately executory. Double costs against petitioner. SO ORDERED

Case Digest Manacop vs CA GR No. 104875 Decided on: November 13, 1992 Ponente: PANGANIBAN, J.: FACTS: Florante Manacop and his wife Euaceli purchased on March 1972, a residential lot with a bungalow located in Quezon City. The petitioner failed to pay the sub-contract cost pursuant to a deed of assignment signed between petitioners corporation and private respondent herein (FF Cruz & Co). The latter filed a complaint for the recovery for the sum of money with a prayer for preliminary attachment against the former. Consequently, the corresponding writ for the provisional remedy was issued which triggered the attachment of a parcel of land in Quezon City owned by the Manacop Construction President, the petitioner. The latter insists that the attached property is a family home having been occupied by him and his family since 1972 and is therefore exempt from attachment. ISSUE: WON the subject property is indeed exempted from attachment. HELD: The residential house and lot of petitioner became a family home by operation of law under Article 153 of the Family Code. Such provision does not mean that said article has a retroactive effect such that all existing family residences, petitioners included, are deemed to have been constituted as family homes at the time of their occupation prior to the effectivity of the Family Code and henceforth, are exempt from execution for the payment of obligations incurred before the effectivity of the Family Code on August 3, 1988. Since petitioner incurred debt in 1987, it preceded the effectivity of the Code and his property is therefore not exempt form attachment. The petition was dismissed by SC.

713

University of the Cordilleras College of Law First Year C S.Y. 2013 - 2014

Guerrero vs RTC, Ilocos Norte 229 SCRA 274


G.R. No. 109068 January 10, 1994 Full Case GAUDENCIO GUERRERO, petitioner, vs. REGIONAL TRIAL COURT OF ILOCOS NORTE, BR. XVI, JUDGE LUIS B. BELLO, JR., PRESIDING, and PEDRO G. HERNANDO, respondents. BELLOSILLO, J.: Filed by petitioner as an accion publicana against private respondent, this case assumed another dimension when it was dismissed by respondent Judge on the ground that the parties being brother-in-law the complaint should have alleged that earnest efforts were first exerted towards a compromise. Admittedly, the complaint does not allege that the parties exerted earnest towards a compromise and that the same failed. However, private respondent Pedro G. Hernando apparently overlooked this alleged defect since he did not file any motion to dismiss nor attack the complaint on this ground in his answer. It was only on 7 December 1992, at the pre-trial conference, that the relationship of petitioner Gaudencio Guerrero and respondent Hernando was noted by respondent Judge Luis B. Bello, Jr., they being married to half-sisters hence are brothers-in-law, and on the basis thereof respondent Judge gave petitioner five (5) days "to file his motion and amended complaint" to allege that the parties were very close relatives, their respective wives being sisters, and that the complaint to be maintained should allege that earnest efforts towards a compromise were exerted but failed. Apparently, respondent Judge considered this deficiency a jurisdictional defect. On 11 December 1992, Guerrero moved to reconsider the 7 December 1992 Order claiming that since brothers by affinity are not members of the same family, he was not required to exert efforts towards a compromise. Guerrero likewise argued that Hernando was precluded from raising this issue since he did not file a motion to dismiss nor assert the same as an affirmative defense in his answer. On 22 December 1992, respondent Judge denied the motion for reconsideration holding that "[f]ailure to allege that earnest efforts towards a compromise is jurisdictional such that for failure to allege same the court would be deprived of its jurisdiction to take cognizance of the case." He warned that unless the complaint was amended within five (5) days the case would be dismissed. On 29 January 1993, the 5-day period having expired without Guerrero amending his complaint, respondent Judge dismissed the case, declaring the dismissal however to be without prejudice. Guerrero appeals by way of this petition for review the dismissal by the court a quo. He raises these legal issues: (a) whether brothers by affinity are considered members of the same family contemplated in Art. 217, par. (4), and Art. 222 of the New Civil Code, as well as under Sec. 1, par. (j), Rule 16, of the Rules of Court requiring earnest efforts towards a compromise before a suit between them may be instituted and maintained; and, (b) whether the absence of an allegation in the complaint that earnest efforts towards a compromise were exerted, which efforts failed, is a ground for dismissal for lack of jurisdiction. The Constitution protects the sanctity of the family and endeavors to strengthen it as a basic autonomous social institution. This is also embodied in Art. 149, and given flesh in Art. 151, of the Family Code, which provides: Art. 151. No suit between members of the same family shall prosper unless it should appear from the verified complaint or petition that earnest efforts toward a compromise have been made, but

714

University of the Cordilleras College of Law First Year C S.Y. 2013 - 2014 that the same had failed. If it is shown that no such efforts were in fact made, the case must be dismissed. This rule shall not apply to cases which may not be the subject of compromise under the Civil Code. Considering that Art. 151 herein-quoted starts with the negative word "No", the requirement is mandatory that the complaint or petition, which must be verified, should allege that earnest efforts towards a compromise have been made but that the same failed, so that "[i]f it is shown that no such efforts were in fact made, the case must be dismissed." Further, Art. 151 is contemplated by Sec. 1, par. (j), Rule 16, of the Rules of Court which provides as a ground for motion to dismiss "(t)hat the suit is between members of the same family and no earnest efforts towards a compromise have been made." The Code Commission, which drafted the precursor provision in the Civil Code, explains the reason for the requirement that earnest efforts at compromise be first exerted before a complaint is given due course This rule is introduced because it is difficult to imagine a sadder and more tragic spectacle than a litigation between members of the same family. It is necessary that every effort should be made toward a compromise before a litigation is allowed to breed hate and passion in the family. It is known that a lawsuit between close relatives generates deeper bitterness than between strangers . . . A litigation in a family is to be lamented far more than a lawsuit between strangers . . . But the instant case presents no occasion for the application of the above-quoted provisions. As early as two decades ago, we already ruled in Gayon v. Gayon that the enumeration of "brothers and sisters" as members of the same family does not comprehend "sisters-in-law". In that case, then Chief Justice Concepcion emphasized that "sisters-in-law" (hence, also "brothers-in-law") are not listed under Art. 217 of the New Civil Code as members of the same family. Since Art. 150 of the Family Code repeats essentially the same enumeration of "members of the family", we find no reason to alter existing jurisprudence on the matter. Consequently, the court a quo erred in ruling that petitioner Guerrero, being a brother-in-law of private respondent Hernando, was required to exert earnest efforts towards a compromise before filing the present suit. In his Comment, Hernando argues that ". . . although both wives of the parties were not impleaded, it remains a truism that being spouses of the contending parties, and the litigation involves ownership of real property, the spouses' interest and participation in the land in question cannot be denied, making the suit still a suit between half-sisters . . ." Finding this argument preposterous, Guerrero counters in his Reply that his "wife has no actual interest and participation in the land subject of the . . . suit, which the petitioner bought, according to his complaint, before he married his wife." This factual controversy however may be best left to the court a quo to resolve when it resumes hearing the case. As regards the second issue, we need only reiterate our ruling in O'Laco v. Co Cho Chit, citing Mendoza v. Court of Appeals, that the attempt to compromise as well as the inability to succeed is a condition precedent to the filing of a suit between members of the same family, the absence of such allegation in the complaint being assailable at any stage of the proceeding, even on appeal, for lack of cause of action. It is not therefore correct, as petitioner contends, that private respondent may be deemed to have waived the aforesaid defect in failing to move or dismiss or raise the same in the Answer. On the other hand, we cannot sustain the proposition of private respondent that the case was, after all, also dismissed pursuant to

715

University of the Cordilleras College of Law First Year C S.Y. 2013 - 2014 Sec. 3, Rule 17, of the Rules of Court for failure of petitioner to comply with the court's order to amend his complaint. A review of the assailed orders does not show any directive which Guerrero supposedly defied. The Order of 7 December 1992 merely gave Guerrero five (5) days to file his motion and amended complaint with a reminder that the complaint failed to allege that earnest efforts were exerted towards a compromise. The Order of 22 December 1992, which denied Guerrero's motion for reconsideration, simply stated that "Plaintiff if it (sic) so desire must amend the complaint otherwise, the court will have to dismiss the case (emphasis supplied) . . ." The Order of 29 January 1993 dismissing the case without prejudice only made reference to an earlier order "admonishing" counsel for Guerrero to amend the complaint, and an "admonition" is not synonymous with "order". Moreover, since the assailed orders do not find support in our jurisprudence but, on the other hand, are based on an erroneous interpretation and application of the law, petitioner could not be bound to comply with them. WHEREFORE, the petition is GRANTED and the appealed Orders of 7 December 1992, 22 December 1992 and 29 January 1993 are SET ASIDE. The Regional Trial Court of Laoag City, Branch 16, or whichever branch of the court the case may now be assigned, is directed to continue with Civil Case No. 10084-16 with deliberate dispatch. SO ORDERED.

716

University of the Cordilleras College of Law First Year C S.Y. 2013 - 2014

XII. Paternity and Filiation

717

University of the Cordilleras College of Law First Year C S.Y. 2013 - 2014

Jao vs. Ca G.R. No. 128314. May 29, 2002 Full Case RODOLFO V. JAO, petitioner, vs. COURT OF APPEALS and PERICO V. JAO, respondents. DECISION YNARES-SANTIAGO, J.: Rodolfo and PericoJao were the only sons of the spouses Ignacio JaoTayag and Andrea V. Jao, who died intestate in 1988 and 1989, respectively. The decedents left real estate, cash, shares of stock and other personal properties. On April 17, 1991, Perico instituted a petition for issuance of letters of administration before the Regional Trial Court of Quezon City, Branch 99, over the estate of his parents, docketed as Special Proceedings No. Q-91-8507. Pending the appointment of a regular administrator, Perico moved that he be appointed as special administrator. He alleged that his brother, Rodolfo, was gradually dissipating the assets of the estate. More particularly, Rodolfo was receiving rentals from real properties without rendering any accounting, and forcibly opening vaults belonging to their deceased parents and disposing of the cash and valuables therein. Rodolfo moved for the dismissal of the petition on the ground of improper venue. He argued that the deceased spouses did not reside in Quezon City either during their lifetime or at the time of their deaths. The decedents actual residence was in Angeles City, Pampanga, where his late mother used to run and operate a bakery. As the health of his parents deteriorated due to old age, they stayed in Rodolfos residence at 61 Scout Gandia Street, Quezon City, solely for the purpose of obtaining medical treatment and hospitalization. Rodolfo submitted documentary evidence previously executed by the decedents, consisting of income tax returns, voters affidavits, statements of assets and liabilities, real estate tax payments, motor vehicle registration and passports, all indicating that their permanent residence was in Angeles City, Pampanga. In his opposition, Perico countered that their deceased parents actually resided in Rodolfos house in Quezon City at the time of their deaths. As a matter of fact, it was conclusively declared in their death certificates that their last residence before they died was at 61 Scout Gandia Street, Quezon City. Rodolfo himself even supplied the entry appearing on the death certificate of their mother, Andrea, and affixed his own signature on the said document. Rodolfo filed a rejoinder, stating that he gave the information regarding the decedents residence on the death certificates in good faith and through honest mistake. He gave his residence only as reference, considering that their parents were treated in their late years at the Medical City General Hospital in Mandaluyong, Metro Manila. Their stay in his house was merely transitory, in the same way that they were taken at different times for the same purpose to Pericos residence at Legaspi Towers in Roxas Boulevard. The death certificates could not, therefore, be deemed conclusive evidence of the decedents residence in light of the other documents showing otherwise. The court required the parties to submit their respective nominees for the position. Both failed to comply, whereupon the trial court ordered that the petition be archived. Subsequently, Perico moved that the intestate proceedings be revived. After the parties submitted the names of their respective nominees, the trial court designated Justice Carlos L. Sundiam as special administrator of the estate of Ignacio JaoTayag and Andrea Jao. On April 6, 1994, the motion to dismiss filed by petitioner Rodolfo was denied, to wit:

718

University of the Cordilleras College of Law First Year C S.Y. 2013 - 2014

A mere perusal of the death certificates of the spouses issued separately in 1988 and 1989, respectively, confirm the fact that Quezon City was the last place of residence of the decedents. Surprisingly, the entries appearing on the death certificate of Andrea V. Jao were supplied by movant, Rodolfo V. Jao, whose signature appears in said document. Movant, therefore, cannot disown his own representation by taking an inconsistent position other than his own admission. xxx xxxxxx. WHEREFORE, in view of the foregoing consideration, this court DENIES for lack of merit movants motion to dismiss. SO ORDERED. Rodolfo filed a petition for certiorari with the Court of Appeals, which was docketed as CA-G.R. SP No. 35908. On December 11, 1996, the Court of Appeals rendered the assailed decision, the dispositive portion of which reads: WHEREFORE, no error, much less any grave abuse of discretion of the court a quo having been shown, the petition for certiorari is hereby DISMISSED. The questioned order of the respondent Judge is affirmed in toto. SO ORDERED. Rodolfos motion for reconsideration was denied by the Court of Appeals in the assailed resolution dated February 17, 1997. Hence, this petition for review, anchored on the following grounds: I RESPONDENT COURT HAD DECIDED A QUESTION OF SUBSTANCE IN A WAY NOT IN ACCORD WITH THE LAW AND IS DIRECTLY CONTRADICTORY TO THE APPLICABLE DECISION ALREADY RENDERED BY THIS HONORABLE COURT. II RESPONDENT COURT ERRED IN DISREGARDING THE RULING OF THIS HONORABLE COURT IN THE CASE OF EUSEBIO VS. EUSEBIO, 100 PHILS. 593, WHICH CLEARLY INTERPRETED WHAT IS MEANT BY RESIDENCE IN SEC. 1 OF RULE 73 OF THE RULES OF COURT. III RESPONDENT COURT ERRED IN HOLDING THAT PHYSICAL PRESENCE IN A PLACE AT THE TIME OF DEATH IS DETERMINATIVE OF DECEDENTS RESIDENCE RATHER THAN THE INTENTION OF THE DECEDENTS TO ESTABLISH THEIR PERMANENT RESIDENCE IN ANOTHER PLACE. IV RESPONDENT COURT ERRED IN APPLYING BY ANALOGY THE RESIDENCE CONTEMPLATED IN SEC. 2 OF RULE 4 FOR THE PURPOSE OF SERVING SUMMONS TO A DEFENDANT IN A PERSONAL ACTION TO THE RESIDENCE CONTEMPLATED IN SEC. 1 OF RULE 73 FOR THE PURPOSE OF DETERMINING VENUE IN THE SETTLEMENT OF THE ESTATE OF A DECEASED. V

719

University of the Cordilleras College of Law First Year C S.Y. 2013 - 2014

RESPONDENT COURT ERRED IN GIVING MORE WEIGHT TO THE ENTRY OF PETITIONER AND PRIVATE RESPONDENT IN THE RESPECTIVE DEATH CERTIFICATES OF THE DECEDENTS RATHER THAN THE OVERWHELMING EVIDENCE SHOWING THE CLEAR INTENTION OF THE DECEDENTS TO ESTABLISH THEIR PERMANENT RESIDENCE IN ANGELES CITY. VI RESPONDENT COURT ERRED IN APPLYING THE PRINCIPLE OF ESTOPPEL AS AGAINST PETITIONER WHICH CAN NOT BE MORE PERSUASIVE THAN THE CLEAR INTENTION OF THE DECEDENTS THEMSELVES TO ESTABLISH PERMANENT RESIDENCE IN ANGELES CITY. VII RESPONDENT COURT ERRED IN DISMISSING THE PETITION FOR CERTIORARI DESPITE THE CLEAR ABUSE OF DISCRETION ON THE PART OF THE TRIAL COURT IN INSISTING TO TAKE COGNIZANCE OF SP. PROCEEDING NO. Q-91-8507.[13] The main issue before us is: where should the settlement proceedings be had --- in Pampanga, where the decedents had their permanent residence, or in Quezon City, where they actually stayed before their demise? Rule 73, Section 1 of the Rules of Court states: Where estate of deceased persons be settled. If the decedent is an inhabitant of the Philippines at the time of his death, whether a citizen or an alien, his will shall be proved, or letters of administration granted, and his estate settled, in the Court of First Instance in the province in which he resides at the time of his death, and if he is an inhabitant of a foreign country, the Court of First Instance of any province in which he had estate. The court first taking cognizance of the settlement of the estate of a decedent shall exercise jurisdiction to the exclusion of all other courts. The jurisdiction assumed by a court, so far as it depends on the place of residence of the decedent, or of the location of his estate, shall not be contested in a suit or proceeding, except in an appeal from that court, in the original case, or when the want of jurisdiction appears on the record. (underscoring ours) Clearly, the estate of an inhabitant of the Philippines shall be settled or letters of administration granted in the proper court located in the province where the decedent resides at the time of his death. Petitioner Rodolfo invokes our ruling in the case of Eusebio v. Eusebio, et al., where we held that the situs of settlement proceedings shall be the place where the decedent had his permanent residence or domicile at the time of death. In determining residence at the time of death, the following factors must be considered, namely, the decedent had: (a) capacity to choose and freedom of choice; (b) physical presence at the place chosen; and (c) intention to stay therein permanently. While it appears that the decedents in this case chose to be physically present in Quezon City for medical convenience, petitioner avers that they never adopted Quezon City as their permanent residence. The contention lacks merit. The facts in Eusebio were different from those in the case at bar. The decedent therein, Andres Eusebio, passed away while in the process of transferring his personal belongings to a house in Quezon City. He was then suffering from a heart ailment and was advised by his doctor/son to purchase a Quezon City residence, which was nearer to his doctor. While he was able to acquire a house in Quezon City, Eusebio died even before he could move therein. In said case, we ruled that Eusebio retained his domicile --- and hence, residence --- in San Fernando, Pampanga. It
720

University of the Cordilleras College of Law First Year C S.Y. 2013 - 2014

cannot be said that Eusebio changed his residence because, strictly speaking, his physical presence in Quezon City was just temporary. In the case at bar, there is substantial proof that the decedents have transferred to petitioners Quezon City residence. Petitioner failed to sufficiently refute respondents assertion that their elderly parents stayed in his house for some three to four years before they died in the late 1980s. Furthermore, the decedents respective death certificates state that they were both residents of Quezon City at the time of their demise. Significantly, it was petitioner himself who filled up his late mothers death certificate. To our mind, this unqualifiedly shows that at that time, at least, petitioner recognized his deceased mothers residence to be Quezon City. Moreover, petitioner failed to contest the entry in Ignacios death certificate, accomplished a year earlier by respondent. The recitals in the death certificates, which are admissible in evidence, were thus properly considered and presumed to be correct by the court a quo. We agree with the appellate courts observation that since the death certificates were accomplished even before petitioner and respondent quarrelled over their inheritance, they may be relied upon to reflect the true situation at the time of their parents death. The death certificates thus prevailed as proofs of the decedents residence at the time of death, over the numerous documentary evidence presented by petitioner. To be sure, the documents presented by petitioner pertained not to residence at the time of death, as required by the Rules of Court, but to permanent residence or domicile. In Garcia-Fule v. Court of Appeals, we held: xxx xxxxxx the term resides connotes ex vi termini actual residence as distinguished from legal residence or domicile. This term resides, like the terms residing and residence, is elastic and should be interpreted in the light of the object or purpose of the statute or rule in which it is employed. In the application of venue statutes and rules Section 1, Rule 73 of the Revised Rules of Court is of such nature residence rather than domicile is the significant factor. Even where the statute uses the word domicile still it is construed as meaning residence and not domicile in the technical sense. Some cases make a distinction between the terms residence and domicile but as generally used in statutes fixing venue, the terms are synonymous, and convey the same meaning as the term inhabitant. In other words, resides should be viewed or understood in its popular sense, meaning, the personal, actual or physical habitation of a person, actual residence or place of abode. It signifies physical presence in a place and actual stay thereat. In this popular sense, the term means merely residence, that is, personal residence, not legal residence or domicile. Residence simply requires bodily presence as an inhabitant in a given place, while domicile requires bodily presence in that place and also an intention to make it ones domicile. No particular length of time of residence is required though; however, the residence must be more than temporary. Both the settlement court and the Court of Appeals found that the decedents have been living with petitioner at the time of their deaths and for some time prior thereto. We find this conclusion to be substantiated by the evidence on record. A close perusal of the challenged decision shows that, contrary to petitioners assertion, the court below considered not only the decedents physical presence in Quezon City, but also other factors indicating that the decedents stay therein was more than temporary. In the absence of any substantial showing that the lower courts factual findings stemmed from an erroneous apprehension of the evidence presented, the same must be held to be conclusive and binding upon this Court. Petitioner strains to differentiate between the venue provisions found in Rule 4, Section 2, on ordinary civil actions, and Rule 73, Section 1, which applies specifically to settlement proceedings. He argues that while venue in the former understandably refers to actual physical residence for the purpose of serving summons, it is the permanent residence of the decedent which is significant in Rule 73, Section 1. Petitioner insists that venue for the settlement of
721

University of the Cordilleras College of Law First Year C S.Y. 2013 - 2014

estates can only refer to permanent residence or domicile because it is the place where the records of the properties are kept and where most of the decedents properties are located. Petitioners argument fails to persuade. It does not necessarily follow that the records of a persons properties are kept in the place where he permanently resides. Neither can it be presumed that a persons properties can be found mostly in the place where he establishes his domicile. It may be that he has his domicile in a place different from that where he keeps his records, or where he maintains extensive personal and business interests. No generalizations can thus be formulated on the matter, as the question of where to keep records or retain properties is entirely dependent upon an individuals choice and peculiarities. At any rate, petitioner is obviously splitting straws when he differentiates between venue in ordinary civil actions and venue in special proceedings. In Raymond v. Court of Appeals and Bejer v. Court of Appeals, we ruled that venue for ordinary civil actions and that for special proceedings have one and the same meaning. As thus defined, residence, in the context of venue provisions, means nothing more than a persons actual residence or place of abode, provided he resides therein with continuity and consistency. All told, the lower court and the Court of Appeals correctly held that venue for the settlement of the decedents intestate estate was properly laid in the Quezon City court. WHEREFORE, in view of the foregoing, the petition is DENIED, and the decision of the Court of Appeals in CA-G.R. SP No. 35908 is AFFIRMED. SO ORDERED

722

University of the Cordilleras College of Law First Year C S.Y. 2013 - 2014

Case Digest JAO VS COURT OF APPEALS G.R. NO. 128314. May 29, 2002. Date Decided: May 29, 2002 Ponente: YNARES-SANTIAGO, J.: Facts: Petitioner (Rodolfo Jao) and Private Respondent (PericoJao) were the sons of the deceased Spouses Ignacio and Andrea Jao who died intestate in 1988 and 1989.Private respondent filed a petition for the issuance of letters of administration in the RTC of Quezon City over the estate of his parents. Pending the appointment of a regular administrator, private respondent Perico moved that he be appointed as special administrator, alleging that petitioner Rodolfo was dissipating the assets of the estate. Petitioner moved for the dismissal of the petition on the ground of improper venue. He alleged that his parents did not reside in Quezon City during their lifetime but in Angeles City, Pampanga. He submitted documentary evidence showing that his deceased parents were residents of Angeles City, Pampanga. Private respondent Perico countered that his parents resides in Quezon City and in fact, actually resided in petitioners house as shown in the death certificate presented before the court. Petitioner argued that his parents stay in Quezon City was merely transitory and that the death certificates could not be deemed conclusive evidence of the decedents residence. The trial court ruled in favor of private respondent Perico. The CA affirmed in to the trial courts decision. Hence, this petition. Issue: Whether or not the settlement proceeding was properly laid in Quezon City. Held: Yes. The settlement proceeding was properly laid in Quezon City. As provided for under the Rules of Court, the estate of an inhabitant of the Philippines shall be settled or letters of administration granted in the proper court located in the province where the decedent resides at the time of his death. The Rules of Court refers to residence at the time of death, not to the permanent residence or domicile. In the case of Garcia-Fule vs CA, it was held that the term resides connotes ex vi termini actual residence as distinguished from legal residence or domicile. xxx resides should be viewed or understood in its popular sense, meaning the personal, actual or physical habitation of a person, actual residence or place of abode. It signifies physical presence in a place and actual stay thereat. In this popular sense, the term means merely residence, that is, personal residence, not legal residence or domicile. Residence simply requires bodily presence as an inhabitant in a given place, while domicile requires bodily presence and also an intention to make it ones domicile. No particular length of time is required; however, the residence must me more than temporary. In the case at bar, it was found that the decedents have been living in Quezon City at the time of their death and sometime prior thereto, and as was also shown in the death certificate presented by private respondent. Thus, the venue for the settlement of the decedents intestate was properly laid in the Quezon City.

723

University of the Cordilleras College of Law First Year C S.Y. 2013 - 2014

Uyguangco vs. Ca G.R. No. 76873 Full Case

October 26, 1989

DOROTEA, VIRGILIO, APOLINARIO, JR., SULPICIO & DOMINADOR, all surnamed UYGUANGCO, petitioners, vs. COURT OF APPEALS, Judge SENEN PENARANDA and GRACIANO BACJAO UYGUANGCO, respondents.

CRUZ, J.: The issue before the Court is not the status of the private respondent, who has been excluded from the family and inheritance of the petitioners. What we are asked to decide is whether he should be allowed to prove that he is an illegitimate child of his claimed father, who is already dead, in the absence of the documentary evidence required by the Civil Code. The trial court said he could and was sustained by the respondent Court of Appeals. The latter court held that the trial judge had not committed any grave abuse of discretion or acted without jurisdiction in allowing the private respondent to prove his filiation. Moreover, the proper remedy was an ordinary appeal and not a petition for prohibition. The petitioners ask for a reversal of these rulings on the ground that they are not in accordance with law and jurisprudence. ApolinarioUyguangco died intestate in 1975, leaving his wife, Dorotea, four legitimate children (her co-petitioners herein), and considerable properties which they divided among themselves. Claiming to be an illegitimate son of the deceased Apolinario, and having been left out in the extrajudicial settlement of his estate, GracianoBacjaoUyguangco filed a complaint for partition against all the petitioners. Graciano alleged that he was born in 1952 to ApolinarioUyguangco and AnastaciaBacjao and that at the age of 15 he moved to his father's hometown at Medina, Misamis Oriental, at the latter's urging and also of Dorotea and his half-brothers. Here he received support from his father while he was studying at the Medina High School, where he eventually graduated. He was also assigned by his father, without objection from the rest of the family, as storekeeper at the Uyguangco store in Mananom from 1967 to 1973. In the course of his presentation of evidence at the trial, the petitioners elicited an admission from Graciano that he had none of the documents mentioned in Article 278 to show that he was the illegitimate son of Apolinario Uyguangco. These are "the record of birth, a will, a statement before a court of record, or (in) any authentic writing." The petitioners thereupon moved for the dismissal of the case on the ground that the private respondent could no longer prove his alleged filiation under the applicable provisions of the Civil Code. Specifically, the petitioners argued that the only evidence allowed under Article 278 to prove the private respondent's claim was not available to him as he himself had admitted. Neither could he now resort to the provisions of Article 285 because he was already an adult when his alleged father died in 1975, and his claim did not come under the exceptions. The said article provides as follows: ART. 285. The action for the recognition of natural children may be brought only during the lifetime of the presumed parents, except in the following cases: (1) If the father or mother died during the minority of the child, in which case the latter may file the action before the expiration of four years from the attainment of his majority;
724

University of the Cordilleras College of Law First Year C S.Y. 2013 - 2014

(2) If after the death of the father or of the mother a document should appear of which nothing had been heard and in which either or both parents recognize the child. In this case, the action must be commenced within four years from the finding of the document. As earlier related, the motion to dismiss was denied, prompting the petitioners to seek relief in vain from the respondent court. In the case now before us, the petitioners reiterate and emphasize their position that allowing the trial to proceed would only be a waste of time and effort. They argue that the complaint for partition is actually an action for recognition as an illegitimate child, which, being already barred, is a clear attempt to circumvent the said provisions. The private respondent insists, on the other hand, that he has a right to show under Article 283 that he is "in continuous possession of the status of a child of his alleged father by the direct acts of the latter or of his family." We find that this case must be decided under a new if not entirely dissimilar set of rules because the parties have been overtaken by events, to use the popular phrase. The Civil Code provisions they invoke have been superseded, or at least modified, by the corresponding articles in the Family Code, which became effective on August 3,1988. Under the Family Code, it is provided that: Art. 175. Illegitimate children may establish their illegitimate filiation in the same way and on the same evidence as legitimate children. The following provision is therefore also available to the private respondent in proving his illegitimate filiation: Art. 172. The filiation of legitimate children is established by any of the following: (1) The record of birth appearing in the civil register or a final judgment; or

(2) An admission of legitimate filiation in a public document or a private handwritten instrument and signed by the parent concerned. In the absence of the foregoing evidence, the legitimate filiation shall be proved by: (1) (2) The open and continuous possession of the status of a legitimate child; or Any other means allowed by the Rules of Court and special laws.

While the private respondent has admitted that he has none of the documents mentioned in the first paragraph (which are practically the same documents mentioned in Article 278 of the Civil Code except for the "private handwritten instrument signed by the parent himself'''), he insists that he has nevertheless been "in open and continuous possession of the status of an illegitimate child," which is now also admissible as evidence of filiation. Thus, he claims that he lived with his father from 1967 until 1973, receiving support from him during that time; that he has been using the surname Uyguangco without objection from his father and the petitioners as shown in his high school diploma, a special power of attorney executed in his favor by Dorotea Uyguangco, and another one by Sulpicio Uyguangco; that he has shared in the profits of the copra business of the Uyguangcos, which is a strictly family business; that he was a director, together with the petitioners, of the Alu and Sons Development Corporation, a family corporation; and that in the addendum to the original extrajudicial settlement concluded by the petitioners he was given a share in his deceased father's estate.

725

University of the Cordilleras College of Law First Year C S.Y. 2013 - 2014

It must be added that the illegitimate child is now also allowed to establish his claimed filiation by "any other means allowed by the Rules of Court and special laws," like his baptismal certificate, a judicial admission, a family Bible in which his name has been entered, common reputation respecting his pedigree, admission by silence, the testimonies of witnesses, and other kinds of proof admissible under Rule 130 of the Rules of Court. The problem of the private respondent, however, is that, since he seeks to prove his filiation under the second paragraph of Article 172 of the Family Code, his action is now barred because of his alleged father's death in 1975. The second paragraph of this Article 175 reads as follows: The action must be brought within the same period specified in Article 173, except when the action is based on the second paragraph of Article 172, in which case the action may be brought during the lifetime of the alleged parent. (Italics supplied.) It is clear that the private respondent can no longer be allowed at this time to introduce evidence of his open and continuous possession of the status of an illegitimate child or prove his alleged filiation through any of the means allowed by the Rules of Court or special laws. The simple reason is that Apolinario Uyguangco is already dead and can no longer be heard on the claim of his alleged son's illegitimate filiation. In her Handbook on the Family Code of the Philippines, Justice Alicia Sempio-Diy explains the rationale of the rule, thus: "It is a truism that unlike legitimate children who are publicly recognized, illegitimate children are usually begotten and raised in secrecy and without the legitimate family being aware of their existence. Who then can be sure of their filiation but the parents themselves? But suppose the child claiming to be the illegitimate child of a certain person is not really the child of the latter? The putative parent should thus be given the opportunity to affirm or deny the child's filiation, and this, he or she cannot do if he or she is already dead." Finally, it must be observed that the provisions invoked by the parties are among those affected by the following articles in the Family Code: Art. 254. Titles III, IV, V, VI VII, VIII, IX, XI and XV of Book I of Republic Act No. 386, otherwise known as the Civil Code of the Philippines, as amended, and Articles 17,18,19, 27, 28, 29, 30, 31, 39, 40, 41 and 42 of Presidential Decree No. 603, otherwise known as the Child and Youth Welfare Code, as amended, and all laws, decrees, executive orders, proclamations, rules and regulations, or parts thereof, inconsistent herewith are hereby repealed. Art. 256. This Code shall have retroactive effect insofar as it does not prejudice or impair vested or acquired rights in accordance with the Civil Code or other laws. Graciano's complaint is based on his contention that he is the illegitimate child of Apolinario Uyguangco, whose estate is the subject of the partition sought. If this claim can no longer be proved in an action for recognition, with more reason should it be rejected in the said complaint, where the issue of Graciano's filiation is being raised only collaterally. The complaint is indeed a circumvention of Article 172, which allows proof of the illegitimate child's filiation under the second paragraph thereof only during the lifetime of the alleged parent. Considering that the private respondent has, as we see it, established at least prima facie proof of his alleged filiation, we find it regrettable that his action should be barred under the said article. But that is the law and we have no choice but to apply it. Even so, the Court expresses the hope that the parties will arrive at some kind of rapprochement, based on fraternal and moral ties if not the strict language of the law, that will allow the private respondent an equitable share in the disputed estate. Blood should tell. WHEREFORE, the petition is GRANTED, and Civil Case No. 9067 in the Regional Trial Court of Misamis Oriental, Branch 20, is hereby DISMISSED. It is so ordered.
726

University of the Cordilleras College of Law First Year C S.Y. 2013 - 2014

Case Digest Uyguangco vs. Ca Date Decided: October 26, 1989 Ponente Cruz Facts: Apolinario Uyguangco died intestate in 1975, leaving his wife, four legitimate children and properties which they divided among themselves. Graciano Uyguangco filed a complaint for partition against the petitioners, claiming that as the illegitimate son of the deceased and a Anastacia Bacjao, he must not be left out of the extrajudicial settlement of the estate. He also claims that he received support from his father while in high school and was also assigned by his father as storekeeper at the Uyguangco store. Petitioners moved to dismiss the case on the ground that Graciano could not prove his alleged filiation having none of the documents required in Art. 278 of the NCC (i.e. record of birth, a will, a statement before a court of record or in any authentic writing. Neither may he resort to Art. 285 of the NCC because he was already an adult when his alleged dad died. Graciano insists however, that he is in continuous possession of the status of a child of his alleged father by the direct acts of the latter or of his family as is under Art. 283of the NCC. Issue: WON Graciano may adequately prove filiation. Held: NO Ratio: The Civil Code provisions they invoke have been superseded or at least modified by the corresponding articles in the FC. Since illegitimate children may establish their illegitimate filiation in the same way and on the same evidence as legitimate children (Art 175), Graciano may establish his filiation by the means given in Art. 172. Thus while he has no record of birth appearing in the civil registrar or a final judgment or an admission of legitimate filiation in a public document or a private handwritten instrument and signed by the parent concerned, he insists that he has nevertheless been in an open and continuous possession of the status of an illegitimate child, which is admissible as evidence of filiation under Art.172.As proof to this open and continuous possessionhe claims that he lived with his father from 1967 until 1973, received support from him, used the name Uyguangco without objection, a special power of attorney executed in his favour by Apolinarios wife, and another one by Suplcio Uyguangco, shared in the profits of the copra family business of the Uyguangcos and was even given a share in his deceased fathers estate as found in the addendum to the original extrajudicial settlement concluded bythepetitioners. However, since his father has already died, his action is now barred as Art. 172specifically requires that when the action is based on other proofs of filiation such as open and continuous possession, the action must be brought during the lifetime of the alleged parent.

727

University of the Cordilleras College of Law First Year C S.Y. 2013 - 2014

Aruego vs. CA G.R. No. 112193. March 13, 1996 Full Case JOSE E. ARUEGO, JR., SIMEONA SAN JUAN ARUEGO, MA. IMMACULADA T. ALANON, ROBERTO A. TORRES, CRISTINA A. TORRES, JUSTO JOSE TORRES and AGUSTIN TORRES, petitioners, vs. THE HON. COURT OF APPEALS, THIRTEENTH DIVISION and ANTONIA ARUEGO, respondents. HERMOSISIMA, JR., J.: On March 7, 1983, a Complaint for Compulsory Recognition and Enforcement of Successional Rights was filed before Branch 30 of the Regional Trial Court of Manila by the minors, private respondent Antonia F. Aruego and her alleged sister Evelyn F. Aruego, represented by their mother and natural guardian, Luz M. Fabian. Named defendants therein were Jose E. Aruego, Jr. and the five (5) minor children of the deceased Gloria A. Torres, represented by their father and natural guardian, Justo P. Torres, Jr., now the petitioners herein. In essence, the complaint avers that the late Jose M. Aruego, Sr., a married man, had an amorous relationship with Luz M. Fabian sometime in 1959 until his death on March 30, 1982. Out of this relationship were born Antonia F. Aruego and Evelyn F. Aruego on October 5, 1962 and September 3, 1963, respectively. The complaint prayed for an Order praying that herein private respondent and Evelyn be declared the illegitimate children of the deceased Jose M. Aruego, Sr; that herein petitioners be compelled to recognize and acknowledge them as the compulsory heirs of the deceased Jose M. Aruego; that their share and participation in the estate of their deceased father be determined and ordered delivered to them. The main basis of the action for compulsory recognition is their alleged open and continuous possession of the status of illegitimate children as stated in paragraphs 6 and 7 of the Complaint, to wit: 6. The plaintiffs father, Jose M. Aruego, acknowledged and recognized the herein plaintiffs as his children verbally among plaintiffs and their mothers family friends, as well as by myriad different paternal ways, including but not limited to the following: (a) Regular support and educational expenses; (b) Allowance to use his surname; (c) Payment of maternal bills; (d) Payment of baptismal expenses and attendance therein; (e) Taking them to restaurants and department stores on occasions of family rejoicing; (f) Attendance to school problems of plaintiffs; (g) Calling and allowing plaintiffs to his office every now and then; (h) Introducing them as such children to family friends. 7. The plaintiffs are thus, in continuous possession of the status of (illegitimate) children of the deceased Jose M. Aruego who showered them, with the continuous and clear manifestations of paternal care and affection as above outlined. Petitioners denied all these allegations.
728

University of the Cordilleras College of Law First Year C S.Y. 2013 - 2014

After trial, the lower court rendered judgment, dated June 15, 1992, the dispositive portion of which reads: WHEREFORE, judgment is rendered 1. Declaring Antonia Aruego as illegitimate daughter of Jose Aruego and Luz Fabian; 2. Evelyn Fabian is not an illegitimate daughter of Jose Aruego with Luz Fabian; 3. Declaring that the estate of deceased Jose Aruego are the following: x xxxxxxxx 4. Antonia Aruego is entitled to a share equal to portion of share of the legitimate children of Jose Aruego; 5. Defendants are hereby ordered to recognize Antonia Aruego as the illegitimate daughter of Jose Aruego with Luz Fabian; 6. Defendants are hereby ordered to deliver to Antonia Aruego (her) share in the estate of Jose Aruego, Sr.; 7. Defendants to play (sic) plaintiffs (Antonia Aruego) counsel the sum of P10,000.00 as atty.s fee; 8. Cost against the defendants. Herein petitioners filed a Motion for Partial Reconsideration of the decision alleging loss of jurisdiction on the part of the trial court over the complaint by virtue of the passage of Executive Order No. 209 (as amended by Executive Order No. 227), otherwise known as the Family Code of the Philippines which took effect on August 3, 1988. This motion was denied by the lower court in the Order, dated January 14, 1993. Petitioners interposed an appeal but the lower court refused to give it due course on the ground that it was filed out of time. A Petition for Prohibition and Certiorari with prayer for a Writ of Preliminary Injunction was filed by herein petitioners before respondent Court of Appeals, the petition was dismissed for lack of merit in a decision promulgated on August 31, 1993. A Motion for Reconsideration when filed was denied by the respondent court in a minute resolution, dated October 13, 1993. Hence, this Petition for Review on Certiorari under Rule 45 alleging the following grounds: A RESPONDENT COURT HAD DECIDED A QUESTION OF SUBSTANCE IN A WAY NOT IN ACCORD WITH THE LAW AND IS DIRECTLY CONTRADICTORY TO THE APPLICABLE DECISION ALREADY ISSUED BY THIS HONORABLE COURT. B RESPONDENT COURT ERRED IN HOLDING THAT THE PETITION FILED BY PETITIONERS BEFORE IT DOES NOT INVOLVE A QUESTION OF JURISDICTION. C
729

University of the Cordilleras College of Law First Year C S.Y. 2013 - 2014

RESPONDENT COURT HAD CLEARLY ERRED IN RULING THAT THERE IS NO PERCEPTIBLE DIFFERENCE BETWEEN THE CIVIL CODE PROVISION AND THOSE OF THE FAMILY CODE ANENT THE TIME AN ACTION FOR COMPULSORY RECOGNITION MAY BE MADE AND THAT THERE IS NO DIFFERENCE UNDER THE CIVIL CODE FROM THAT OF THE FAMILY CODE CONCERNING THE REQUIREMENT THAT AN ACTION FOR COMPULSORY RECOGNITION ON THE GROUND OF CONTINUOUS POSSESSION OF THE STATUS OF AN ILLEGITIMATE CHILD SHOULD BE FILED DURING THE LIFETIME OF THE PUTATIVE PARENT, IN UTTER DISREGARD OF THE RULING OF THIS HONORABLE COURT IN THE UYGUANGCO CASE THAT THE CIVIL CODE PROVISION HAD BEEN SUPERSEDED, OR AT LEAST MODIFIED BY THE CORRESPONDING ARTICLES IN THE FAMILY CODE. D RESPONDENT COURT ERRED IN DISMISSING PETITIONERS PETITION FOR PROHIBITION AND IN HOLDING THAT PETITIONERS REMEDY IS THAT OF AN APPEAL WHICH ALLEGEDLY HAD ALREADY BEEN LOST. Private respondents action for compulsory recognition as an illegitimate child was brought under Book I, Title VIII of the Civil Code on PERSONS, specifically Article 285 thereof, which states the manner by which illegitimate children may prove their filiation, to wit: Art. 285. The action for the recognition of natural children may be brought only during the lifetime of the presumed parents, except in the following cases: (1) If the father or mother died during the minority of the child, in which case the latter may file the action before the expiration of four years from the attainment of his majority; x xx. Petitioners, on the other hand, submit that with the advent of the New Family Code on August 3, 1988, the trial court lost jurisdiction over the complaint of private respondent on the ground of prescription, considering that under Article 175, paragraph 2, in relation to Article 172 of the New Family Code, it is provided that an action for compulsory recognition of illegitimate filiation, if based on the open and continuous possession of the status of an illegitimate child, must be brought during the lifetime of the alleged parent without any exception, otherwise the action will be barred by prescription. The law cited reads: Article 172. The filiation of legitimate children is established by any of the following: (1) The record of birth appearing in the civil register or a final judgment; or

(2) An admission of legitimate filiation in a public document or a private handwritten instrument and signed by the parent concerned. In the absence of the foregoing evidence, the legitimate filiation shall be proved by: (1) (2) The open and continuous possession of the status of a legitimate child; or Any other means allowed by the Rules of Court and special laws.

Article 175. Illegitimate children may establish their illegitimate filiation in the same way and on the same evidence as legitimate children. The action must be brought within the same period specified in Article 173 [during the lifetime of the child], except when the action is based on the second paragraph of Article 172, in which case the action may be brought during the lifetime of the alleged parent.

730

University of the Cordilleras College of Law First Year C S.Y. 2013 - 2014

In the case at bench, petitioners point out that, since the complaint of private respondent and her alleged sister was filed on March 7, 1983, or almost one (1) year after the death of their presumed father on March 30, 1982, the action has clearly prescribed under the new rule as provided in the Family Code. Petitioners, further, maintain that even if the action was filed prior to the effectivity of the Family Code, this new law must be applied to the instant case pursuant to Article 256 of the Family Code which provides: This Code shall have retroactive effect insofar as it does not prejudice or impair vested or acquired rights in accordance with the Civil Code or other laws. The basic question that must be resolved in this case, therefore, appears to be: Should the provisions of the Family Code be applied in the instant case? As a corollary Will the application of the Family Code in this case prejudice or impair any vested right of the private respondent such that it should not be given retroactive effect in this particular case? The phrase vested or acquired rights under Article 256, is not defined by the Family Code. The Committee did not define what is meant by a vested or acquired right, thus leaving it to the courts to determine what it means as each particular issue is submitted to them. It is difficult to provide the answer for each and every question that may arise in the future. In Tayag vs. Court of Appeals, a case which involves a similar complaint denominated as Claim for Inheritance but treated by this court as one to compel recognition as an illegitimate child brought prior to the effectivity of the Family Code by the mother of the minor child, and based also on the open and continuous possession of the status of an illegitimate child, we had occasion to rule that: Under the circumstances obtaining in the case at bar, we hold that the right of action of the minor child has been vested by the filing of the complaint in court under the regime of the Civil Code and prior to the effectivity of the Family Code. We herein adopt our ruling in the recent case of Republic of the Philippines vs. Court of Appeals, et. al. where we held that the fact of filing of the petition already vested in the petitioner her right to file it and to have the same proceed to final adjudication in accordance with the law in force at the time, and such right can no longer be prejudiced or impaired by the enactment of a new law. xxx xxxxxx Accordingly, Article 175 of the Family Code finds no proper application to the instant case since it will ineluctably affect adversely a right of private respondent and, consequentially, of the minor child she represents, both of which have been vested with the filing of the complaint in court. The trial court is, therefore, correct in applying the provisions of Article 285 of the Civil Code and in holding that private respondents cause of action has not yet prescribed. Tayag applies four-square with the case at bench. The action brought by private respondent Antonia Aruego for compulsory recognition and enforcement of successional rights which was filed prior to the advent of the Family Code, must be governed by Article 285 of the Civil Code and not by Article 175, paragraph 2 of the Family Code. The present law cannot be given retroactive effect insofar as the instant case is concerned, as its application will prejudice the vested right of private respondent to have her case decided under Article 285 of the Civil Code. The right was vested to her by the fact that she filed her action under the regime of the Civil Code. Prescinding from this, the conclusion then ought to be that the action was not yet barred, notwithstanding the fact that it was brought when the putative father was already deceased, since private respondent was then still a minor when it was filed, an exception to the general rule provided under Article 285 of the Civil Code. Hence, the trial court, which acquired jurisdiction over the case by the filing of the complaint, never lost jurisdiction over the same despite the passage of E.O. No. 209, also known as the Family Code of the Philippines. Our ruling herein reinforces the principle that the jurisdiction of a court, whether in criminal or civil cases, once attached cannot be ousted by subsequent happenings or events, although of a
731

University of the Cordilleras College of Law First Year C S.Y. 2013 - 2014

character which would have prevented jurisdiction from attaching in the first instance, and it retains jurisdiction until it finally disposes of the case. WHEREFORE, the petition is DENIED and the decision of the Court of Appeals dated August 31, 1993 and its Resolution dated October 13, 1993 are hereby AFFIRMED. SO ORDERED.

732

University of the Cordilleras College of Law First Year C S.Y. 2013 - 2014

CASE DIGEST: ARUEGO VS. CA Date Decided: March 13, 1996 Ponente: HERMOSISIMA The present law cannot be given retroactive effect insofar as the instant case is concerned, as its application will prejudice the vested right of private respondent to have her case decided under Article 285 of the Civil Code. The courts response in the petitioner contention that the Family Code should be applied in the instant case. Ponente: Justice HERMOSISIMA, JR., 1996FACTS:Private respondent Antonia & Evelyn Aruego, as represented by her mother Fabian, filed a petition, in the RTC, compelling the Aruego children of Torres to recognize and acknowledge them as compulsory heirs of the deceased Jose. M. Aruego; on the grounds that they possess an open and continuous possession of the status of illegitimate children to wit: (a) The plaintiffs' father, Jose M. Aruego, acknowledged and recognized the herein plaintiffs as his children verbally among plaintiffs' and their mother's family friends, as well as by myriad different paternal ways. (b) The plaintiffs are thus, incontinuous possession of the status of (illegitimate) children of the deceased Jose M. Aruego who showered them, with the continuous and clear manifestations of paternal care and affection as above outlined. Petitioner denied all these allegations. The lower court rendered judgment in favour of the private respondent, declaring Antonia Aruegoas illegitimate daughter of Jose M. Aruego. Petitioner filed a motion for partial reconsideration alleging that the trial court lost its jurisdiction over the complaint by virtue of the passage of Family Code of the Philippines. The motion was denied the lower court. Petitioner filed a petition for Prohibition and Certiorari with prayer for a Writ of Preliminary Injunction before the Respondent Court of Appeals. The petition was dismissed for lack of merit. A motion for reconsideration filed by the petitioner was also dismissed by the respondent court.

733

University of the Cordilleras College of Law First Year C S.Y. 2013 - 2014

People vs. Malapo G.R. No. 123115. August 25, 1998 Full Case PEOPLE OF THE PHILIPPINES, plaintiff-appellee, vs. NIXON MALAPO, accusedappellant. MENDOZA, J.: This is an appeal from the decision rendered on June 23, 1995 by the Regional Trial Court of Iriga City, Branch 36, convicting accused-appellant Nixon Malapo of rape and sentencing him to suffer the penalty of reclusion perpetua and to pay the victim Amalia Trinidad the sum of P50,000.00 in moral damages. The information against accused-appellant alleged: The undersigned Assistant City Prosecutor of Iriga City, upon sworn complaint originally filed by the offended party hereto attached, hereby accuses one NIXON MALAPO of Salvacion, Iriga City of the crime of RAPE, committed as follows: That sometime on the month of September, 1991 at Salvacion, Iriga City, Philippines, and within the jurisdiction of this Honorable Court, the said accused, entered the house of one Nenita I. No, aunt of Complainant AMALIA TRINIDAD who was then and there alone, and by means of force and intimidation, did, then and there willfully, unlawfully and feloniously succeeded in having carnal knowledge of said Amalia Trinidad against her will and consent and as a result she has become pregnant and delivered a baby at the Iriga City Puericulture Center. CONTRARY TO LAW. Three witnesses testified against him: complainant Amalia Trinidad; Amalias guardian, Nenita No; and a guidance counselor and first cousin of Nenita No, Bernardita Marquinez. Nenita No identified accused-appellant as her long-time neighbor. She testified that Amalia Trinidad had been under her care and custody since 1978 when Amalia was just seven years old. She said Amalia was able to finish the sixth grade of her primary education. It appears that Amalia is a retardate who was a former ward of the Elsie Gaches Village institution. Mrs. Nenita No and her husband were given custody of Amalia on November 19, 1978 on the basis of the following psychological evaluation: Amalia is seemingly an example of a pseudoretardate. She might have been deprived of intellectual stimulations which explains her lag in cognitive development. She is still categorized within the normal classification of children. She must continue attending the centers special school to catch-up for whatever educational deficiency she may have. Mrs. No told the court that, sometime during the first week of September 1991, Amalia was left alone in their house at Salvacion, Iriga City, as she and her husband taught in school, while their four natural children attended classes. At around 10:30 in the morning of that day, when Mrs. No came home from her class, she found accused-appellant Malapo in the yard of her house. Accused-appellant was in haste. She stopped him and asked why he was in a hurry, to which accused-appellant replied he had gathered firewood. This puzzled Mrs. No as they had no firewood at the back of their house. Mrs. No said she found Amalia inside their house crying. Mrs. No tried to find out why Amalia was crying, but she would not say anything. On May 18, 1992, Amalia finally told Mrs. Nos cousin, Bernardita Marquinez, that she had been raped by accused-appellant.

734

University of the Cordilleras College of Law First Year C S.Y. 2013 - 2014

Taking the witness stand, Amalia Trinidad recounted how at around 9:30 in the morning in September 1991, while she was alone at home, accused-appellant Nixon Malapo entered their house. Amalia was then cooking. Upon seeing accused-appellant, she tried to run away, but Malapo caught her hand and brought her to the dining room. The accused-appellant then caused her to fall on the floor, covered her mouth, and forcibly removed her short pants and undergarment. Next, he removed his pants, lay on top of her, and forced his sexual organ into her private part, causing lacerations and bleeding in her vagina. Amalia said she tried to punch the accused-appellant and to remove his hand from her mouth, but he was too strong for her. After he had succeeded in having sexual intercourse with her, accused-appellant left after warning her that he would kill her if she reported the incident to Mrs. No or to anyone else. For this reason, Amalia said, when Mrs. No asked why she was crying, she did not tell her what had happened to her. She confirmed that it was only when she was about to give birth to her baby on May 18, 1992 that she told Bernardita Marquinez that she had been raped by accusedappellant. Amalia pointed to accused-appellant in court as the person who had raped her. She testified that, prior to the date of the alleged crime, she did not harbor any ill will or grudge against him, but, as a result of her abuse, she said she suffered from wounded feelings which made her cry very often. The last witness for the prosecution was Bernardita Marquinez, a resident of Iriga City and guidance counselor of the University of Saint Anthony. She was presented to corroborate the testimonies of Mrs. No and the victim regarding the events on May 18, 1992 and afterward. Accused-appellant Nixon Malapo testified on his behalf, basically claiming alibi as his defense. He presented as witnesses Felipe Edroso and Santos Ramos to corroborate his claim that he and Ramos worked together as duck watchers hired by Edroso in San Jose, Buhi, Camarines Sur, about fifteen kilometers away from Salvacion, Iriga City, from July 1991 until January 1992. Accused-appellant alleged that Amalia three times failed to identify him: When Amalia was brought before the barangay captains office to confront accused-appellant, Amalia failed to identify him despite Mrs. Nos effort to make her point to him. Amalia again failed to identif y him as her alleged assailant when they were taken to the police headquarters and, still later, before Prosecutor Jose Tagum of the Iriga City Prosecutors office. Accused-appellant submitted as documentary evidence a medical certificateshowing that the alleged victim gave birth to a full-term male baby on May 18, 1992. He argues that if Amalia had been raped in September of 1991, she could not have been delivered of her baby on May 18, 1992. On June 23, 1995, the trial court rendered its decision finding accused-appellant guilty. The dispositive portion of its decision reads: WHEREFORE, premises considered, the Court finds the accused, NIXON MALAPO, guilty beyond reasonable doubt of the crime of rape defined and penalized under Article 335 of the Revised Penal Code before its amendment by Rep. Act No. 7659, as charged in the information, and hereby sentences the said accused to suffer the penalty of reclusion perpetua; that said accused is further ordered to indemnify the private offended party, AMALIA TRINIDAD, of the sum of P50,000.00 as moral damages, and to pay the costs. SO ORDERED. Hence, this appeal. Accused-appellants sole contention is this: As, according to Exhibit 1-A, the baby was a full term baby, it is unlikely, nay unbelievable, that same baby was the fruit of the alleged rape perpetrated sometime in September 1991, because from September 15, 1991 (assuming that the rape took place on September 15, 1991, there being no evidence as to when in September 1991 the rape took place) to May 18, 1992 when the baby
735

University of the Cordilleras College of Law First Year C S.Y. 2013 - 2014

was born, is a period of only eight (8) months and three (3) days, contrary to the Certificate (Exh. 1 and 1-A) that the baby was full term when delivered. Consequently, that the appellant had raped the complaining witness in September 1991 and, as a result, she became pregnant and gave birth to her baby on May 18, 1992 is simply improbable; hence, obviously a lie. It could therefore be that the baby of the complaining witness was fathered by another man; hence, there is serious doubt that the appellant had raped same complaining witness in September 1991 causing her to become pregnant and to deliver a baby on May 18, 1992. It cannot be argued that the victim must have been already pregnant when she was allegedly raped because there is no evidence to this effect. The information and the prosecution evidence are to the effect that the baby was the fruit of the alleged rape. The contention has no merit. A textbook on pediatrics states that Infants delivered before the thirty-seventh week of gestation with a birth weight of less than 2,500 grams (American) or 2,275 grams (Filipino) are considered premature. An infant can therefore be considered a full-term baby if it weighs more than 2,275 grams even if it is born before the thirty-seventh week which is less than 9.3 months. Since according to the medical certificate (Exh. 1) Amalias baby weighed 2.4 kilograms or 2,400 grams, it was a full-term baby even if it was born before the normal gestation period. Article 166 of the Family Code provides: Legitimacy of a child may be impugned only on the following grounds: (1) That it was physically impossible for the husband to have sexual intercourse with his wife within the first 120 days of the 300 days which immediately preceded the birth of the child because of: (a) the physical incapacity of the husband to have sexual intercourse with his wife; (b) the fact that the husband and wife were living separately in such a way that sexual intercourse was not possible; or (c) serious illness of the husband, which absolutely prevented sexual intercourse; (2) That it is proved that for biological or other scientific reasons, the child could not have been that of the husband, except in the instance provided in the second paragraph of Article 164; . . . In the case at bar, it can be inferred that conception occurred at or about the time that accusedappellant is alleged to have committed the crime, i.e., within 120 days from the commission of the offense in September 1991. Pursuant to Art. 166 of the Family Code, accused-appellant can overcome the presumption that Amalias child was begotten as a result of her having been raped in September 1991 only if he can show either that it was physically impossible for him to have sexual intercourse because of impotence or serious illness which absolutely prevents him from having sexual intercourse or that Amalia had sexual intercourse with another man. However, accused-appellant has not shown either of these. The testimony of Amalia, as corroborated by Nenita No and Bernardita Marquinez, leaves no doubt in our mind that accused-appellant is the father of the child. Therefore, in accordance with Art. 345 of the Revised Penal Code, accused-appellant should be ordered to pay support.

736

University of the Cordilleras College of Law First Year C S.Y. 2013 - 2014

In any event, the impregnation of a woman is not an element of rape. Proof that the child was fathered by another man does not show that accused-appellant is not guilty, considering the positive testimony of Amalia that accused-appellant had abused her. As held in People v. Alib: ... Under Article 335 of the Revised Penal Code, rape is committed by having carnal knowledge of a woman under any of the following circumstances: (1) By using force or intimidation; (2) When the woman is deprived of reason or otherwise unconscious; and (3) When the woman is under twelve years of age, even though neither of the circumstances mentioned in the two next preceding paragraphs shall be present. It is therefore quite clear that the pregnancy of the victim is not required. For the conviction of an accused, it is sufficient that the prosecution establish beyond reasonable doubt that he had carnal knowledge of the offended party and that he had committed such act under any of the circumstances enumerated above. Carnal knowledge is defined as the act of a man having sexual bodily connections with a woman; Indeed, the findings of the trial court deserve the great respect usually accorded the findings of triers of facts who had the opportunity of observing the demeanor of the witnesses while testifying. Amalias inability to recall the exact date she was raped cannot affect her credibility, especially considering her condition of feeblemindedness. In People v. Quinones, which involved the rape of a 25-year old retardate who also could not recount when she was raped by the accused in that case, this Court held: [T]he date of the occurrence of the rape is not an essential element in the commission of the rape. That is why the Amended Information reads: [t]hat on or about the 5th of June 1989 . . . Suffice it to say that it was shown that rape under Art. 335, par. (2), of the Revised Penal Code was committed, and that the evidence presented established beyond a ray of doubt that accusedappellant was responsible therefor. It is noteworthy that in this appeal accused-appellant does not reiterate his original defense that in September 1991 he was not in Salvacion, Iriga because he was then tending a duck farm in San Jose, Buhi, Camarines, Sur, fifteen kilometers away. For alibi is inherently a weak defense which cannot prevail over the positive identification of the accused. Furthermore, his claim that he was elsewhere at the time of the crime is belied by his own witness, Santos Ramos, who admitted that he and accused-appellant took turns going home to their families in Salvacion. By testifying that he did not go home in September 1991, Santos, by implication, admitted that it was accused-appellants turn to go home in that month. Santos Ramos testified: Q But the fact is, you and Nixon Malapo did never leave the ducks you were tending to from the time that they were brought at Salay in July up to the time you left in January, 1992? A Q A Sometimes, one of us also leave. And when one of you leave, where do you go? We go home to our house, sir.

Q And will you tell the Court how many times you went home between July, 1991 to January, 1992? A Q About five (5) times, sir? When were these five (5) times?
737

University of the Cordilleras College of Law First Year C S.Y. 2013 - 2014

I went home in October and December.

Q You said five (5) times - you said you went home five (5) times, in October and December only? A Q A Q A Yes, sir. You did not go home in August? No, sir. You did not also go home in September? No, sir.

Answering questions from the trial judge, he said: Q During that period between July, 1991 to January, 1992, do you remember if Nixon Malapo also visited? A Q A Q A Yes, Your Honor. Do you remember how many times did he visit his family during that period? Yes, Your Honor, the same number of times that I went home. He went home on those days when you were on duty? Yes, Your Honor.

In addition to the foregoing, Felipe Edroso, the other defense witness, testified: PROS. CANUTO: Q In September 1991 he [accused-appellant] also used to leave San Jose, Salay, and visited his family in Salvacion, stayed there for about a day and then returned to his work in San Jose, Salay? A Yes, Sir.

Q And in October 1991, he also would leave San Jose Salay, to visit his family in Salvacion and stayed with his family for about a day and then returned [to] his work in San Jose, Salay? A Q A Q A Yes, Sir. Now, in 1991 do you know whether Nixon Malapo was married or not? Yes, Sir. Did they have children at that time? Yes, Sir.

Q And you will agree with me that this could be the reason of [sic] the fact that he already had his wife, had his family that he did not continuously stay in Buhi, for three (3) months, but that
738

University of the Cordilleras College of Law First Year C S.Y. 2013 - 2014

he would leave San Jose, Salay and visited his family once in a while in Salvacion during that time/period? A Very seldom, Sir.

In conclusion, we hold that the trial court correctly found accused-appellant guilty of rape. However, it failed to order accused-appellant to pay indemnity. After reciting that, in all criminal cases, unless the offended party reserves the right to institute a separate civil action, she has a right to recover civil indemnity, the trial court awarded the complainant in this case moral damages only. As we have explained in a number of cases, the indemnity provided in criminal law as civil liability is the equivalent of actual or compensatory damages in civil law. It is, therefore, separate and distinct from any award of moral damages. As currently fixed, the indemnity for rape is P50,000.00. However, as we have recently held in People v. Victor, if rape is committed or is qualified by any of the circumstances which under the law (R.A. No. 4111 and R.A. No. 7659) would justify the imposition of the death penalty, the indemnity shall be in an amount not less than P75,000.00. Since in this case the rape is not qualified, the indemnity should be P50,000.00. This is in addition to the amount of P50,000.00 awarded by the trial court as moral damages. It should be added that the latter amount is automatically granted in rape cases without need of any proof. It is assumed that the offended party has suffered moral injuries entitling her to the award of such damages. As we explained in the recent case of People v. Prades: The conventional requirement of allegata et probata in civil procedure and for essentially civil cases should be dispensed with in criminal prosecutions for rape with the civil aspect included therein, since no appropriate pleadings are filed wherein such allegations can be made. Corollarily, the fact that complainant has suffered the trauma of mental, physical and psychological sufferings which constitute the bases for moral damages are too obvious to still require the recital thereof at the trial by the victim, since the Court itself even assumes and acknowledges such agony on her part as a gauge of her credibility. What exists by necessary implication as being ineludibly present in the case need not go through the superfluity of still being proved through a testimonial charade. Mention was earlier made that since Amalias baby was begotten as a result of the rape, accused-appellant is liable for support. Under Art. 345 of the Revised Penal Code, in addition to the indemnification of the offended party, persons guilty of rape must in every case support the offspring. Although said article also provides for the acknowledgment of the child unless the offender is married, this Court has already ruled that: Article 176 of the Family Code confers parental authority over illegitimate children on the mother, and likewise provides for their entitlement to support in conformity with the Family Code. As such, there is no further need for the prohibition against acknowledgment of the offspring by an offender who is married which would vest parental authority in him. Therefore, under Article 345 of the Revised Penal Code, the offender in a rape case who is married can only be sentenced to indemnify the victim and support the offspring, if there be any. In the instant case then, the accused should also be ordered to support his illegitimate offspring, Tracy Jhuen Nieto, with Marie Elena Nieto, but in light of Article 201 of the Family Code, the amount and terms thereof are to be determined by the trial court only after due notice and hearing. Therefore, given the fact that Amalias child is conclusively the illegitimate child of the accused appellant,[27] the acknowledgment in this instance should be understood to refer only to the affiliation of the child.[28] WHEREFORE, the decision of the Regional Trial Court is AFFIRMED, with the MODIFICATION that the accused-appellant is ordered to pay complainant Amalia Trinidad the
739

University of the Cordilleras College of Law First Year C S.Y. 2013 - 2014

sum of P50,000.00 as indemnity, in addition to the amount of P50,000.00 granted by the trial court as moral damages, as well as to acknowledge the filiation of complainants offspring and to give support, the amount of which shall be determined by the trial court. Accordingly, the records of this case are hereby REMANDED to the Regional Trial Court for the fixing of the amount of support. SO ORDERED.

740

University of the Cordilleras College of Law First Year C S.Y. 2013 - 2014

Sayson vs. CA G.R. Nos. 89224-25 January 23, 1992 Full Case MAURICIO SAYSON, ROSARIO SAYSON-MALONDA, BASILISA SAYSON-LIRIO, REMEDIOS SAYSON-REYES and JUANA C. BAUTISTA, petitioners, vs. THE HONORABLE COURT OF APPEALS, DELIA SAYSON, assisted by her husband, CIRILO CEDO, JR., EDMUNDO SAYSON AND DORIBEL SAYSON, respondents.

CRUZ, J.: At issue in this case is the status of the private respondents and their capacity to inherit from their alleged parents and grandparents. The petitioners deny them that right, asserting if for themselves to the exclusion of all others. The relevant genealogical facts are as follows. Eleno and Rafaela Sayson begot five children, namely, Mauricio, Rosario, Basilisa, Remedios and Teodoro. Eleno died on November 10, 1952, and Rafaela on May 15, 1976. Teodoro, who had married Isabel Bautista, died on March 23, 1972. His wife died nine years later, on March 26, 1981. Their properties were left in the possession of Delia, Edmundo, and Doribel, all surnamed Sayson, who claim to be their children. On April 25, 1983, Mauricio, Rosario, Basilisa, and Remedios, together with Juana C. Bautista, Isabel's mother, filed a complaint for partition and accounting of the intestate estate of Teodoro and Isabel Sayson. It was docketed as Civil Case No. 1030 in Branch 13 of the Regional Trial Court of Albay. The action was resisted by Delia, Edmundo and Doribel Sayson, who alleged successional rights to the disputed estate as the decedents' lawful descendants. On July 11, 1983, Delia, Edmundo and Doribel filed their own complaint, this time for the accounting and partition of the intestate estate of Eleno and Rafaela Sayson, against the couple's four surviving children. This was docketed as Civil Case No. 1042 in the Regional Trial Court of Albay, Branch 12. The complainants asserted the defense they raised in Civil Case No. 1030, to wit, that Delia and Edmundo were the adopted children and Doribel was the legitimate daughter of Teodoro and Isabel. As such, they were entitled to inherit Teodoro's share in his parents' estate by right of representation. Both cases were decided in favor of the herein private respondents on the basis of practically the same evidence. Judge Rafael P. Santelices declared in his decision dated May 26, 1986, that Delia and Edmundo were the legally adopted children of Teodoro and Isabel Sayson by virtue of the decree of adoption dated March 9, 1967. Doribel was their legitimate daughter as evidenced by her birth certificate dated February 27, 1967. Consequently, the three children were entitled to inherit from Eleno and Rafaela by right of representation. In his decision dated September 30, 1986, Judge Jose S. Saez dismissed Civil Case No. 1030, holding that the defendants, being the legitimate heirs of Teodoro and Isabel as established by the aforementioned evidence, excluded the plaintiffs from sharing in their estate. Both cases were appealed to the Court of Appeals, where they were consolidated. In its own decision dated February 28, 1989, the respondent court disposed as follows:

741

University of the Cordilleras College of Law First Year C S.Y. 2013 - 2014

WHEREFORE, in Civil Case No. 1030 (CA-G.R. No. 11541), the appealed decision is hereby AFFIRMED. In Civil case No. 1042 (CA-G.R. No. 12364), the appealed decision is MODIFIED in that Delia and EdmundoSayson are disqualified from inheriting from the estate of the deceased spouses Eleno and Rafaela Sayson, but is affirmed in all other respects. SO ORDERED. That judgment is now before us in this petition for review by certiorari. Reversal of the respondent court is sought on the ground that it disregarded the evidence of the petitioners and misapplied the pertinent law and jurisprudence when it declared the private respondents as the exclusive heirs of Teodoro and Isabel Sayson. The contention of the petitioners is that Delia and Edmundo were not legally adopted because Doribel had already been born on February 27, 1967, when the decree of adoption was issued on March 9, 1967. The birth of Doribel disqualified her parents from adopting. The pertinent provision is Article 335 of the Civil Code, naming among those who cannot adopt "(1) Those who have legitimate, legitimated, acknowledged natural children, or natural children by legal fiction." Curiously enough, the petitioners also argue that Doribel herself is not the legitimate daughter of Teodoro and Isabel but was in fact born to one Edita Abila, who manifested in a petition for guardianship of the child that she was her natural mother. The inconsistency of this position is immediately apparent. The petitioners seek to annul the adoption of Delia and Edmundo on the ground that Teodoro and Isabel already had a legitimate daughter at the time but in the same breath try to demolish this argument by denying that Doribel was born to the couple. On top of this, there is the vital question of timeliness. It is too late now to challenge the decree of adoption, years after it became final and executory. That was way back in 1967. Assuming the petitioners were proper parties, what they should have done was seasonably appeal the decree of adoption, pointing to the birth of Doribel that disqualified Teodoro and Isabel from adopting Delia and Edmundo. They did not. In fact, they should have done this earlier, before the decree of adoption was issued. They did not, although Mauricio claimed he had personal knowledge of such birth. As the respondent court correctly observed: When Doribel was born on February 27, 1967, or about TEN (10) days before the issuance of the Order of Adoption, the petitioners could have notified the court about the fact of birth of DORIBEL and perhaps withdrew the petition or perhaps petitioners could have filed a petition for the revocation or rescission of the adoption (although the birth of a child is not one of those provided by law for the revocation or rescission of an adoption). The court is of the considered opinion that the adoption of the plaintiffs DELIA and EDMUNDO SAYSON is valid, outstanding and binding to the present, the same not having been revoked or rescinded. Not having any information of Doribel's birth to Teodoro and Isabel Sayson, the trial judge cannot be faulted for granting the petition for adoption on the finding inter alia that the adopting parents were not disqualified. A no less important argument against the petitioners is that their challenge to the validity of the adoption cannot be made collaterally, as in their action for partition, but in a direct proceeding frontally addressing the issue. The settled rule is that a finding that the requisite jurisdictional facts exists, whether erroneous or not, cannot be questioned in a collateral proceeding, for a presumption arises in such cases where
742

University of the Cordilleras College of Law First Year C S.Y. 2013 - 2014

the validity of the judgment is thus attacked that the necessary jurisdictional facts were proven [Freeman on Judgments, Vol. I, Sec. 350, pp. 719-720]. (Emphasis supplied.) In the case of Santos v. Aranzanso, this Court declared: Anent this point, the rulings are summed up in 2 American Jurisprudence, 2nd Series, Adoption, Sec. 75, p. 922, thus: An adoption order implies the finding of the necessary facts and the burden of proof is on the party attacking it; it cannot be considered void merely because the fact needed to show statutory compliance is obscure. While a judicial determination of some particular fact, such as the abandonment of his next of kin to the adoption, may be essential to the exercise of jurisdiction to enter the order of adoption, this does not make it essential to the jurisdictional validity of the decree that the fact be determined upon proper evidence, or necessarily in accordance with the truth; a mere error cannot affect the jurisdiction, and the determination must stand until reversed on appeal, and hence cannot be collaterally attacked. If this were not the rule, the status of adopted children would always be uncertain, since the evidence might not be the same at all investigations, and might be regarded with different effect by different tribunals, and the adoption might be held by one court to have been valid, while another court would hold it to have been of no avail. (Emphasis supplied.) On the question of Doribel's legitimacy, we hold that the findings of the trial courts as affirmed by the respondent court must be sustained. Doribel's birth certificate is a formidable piece of evidence. It is one of the prescribed means of recognition under Article 265 of the Civil Code and Article 172 of the Family Code. It is true, as the petitioners stress, that the birth certificate offers only prima facie evidence of filiation and may be refuted by contrary evidence. However, such evidence is lacking in the case at bar. Mauricio's testimony that he was present when Doribel was born to Edita Abila was understandbly suspect, coming as it did from an interested party. The affidavit of Abila denying her earlier statement in the petition for the guardianship of Doribel is of course hearsay, let alone the fact that it was never offered in evidence in the lower courts. Even without it, however, the birth certificate must be upheld in line with Legaspi v. Court of Appeals, where we ruled that "the evidentiary nature of public documents must be sustained in the absence of strong, complete and conclusive proof of its falsity or nullity." Another reason why the petitioners' challenge must fail is the impropriety of the present proceedings for that purpose. Doribel's legitimacy cannot be questioned in a complaint for partition and accounting but in a direct action seasonably filed by the proper party. The presumption of legitimacy in the Civil Code . . . does not have this purely evidential character. It serves a more fundamental purpose. It actually fixes a civil status for the child born in wedlock, and that civil status cannot be attacked collaterally. The legitimacy of the child can be impugned only in a direct action brought for that purpose, by the proper parties, and within the period limited by law. The legitimacy of the child cannot be contested by way of defense or as a collateral issue in another action for a different purpose. . . . (Emphasis supplied.) In consequence of the above observations, we hold that Doribel, as the legitimate daughter of Teodoro and Isabel Sayson, and Delia and Edmundo, as their adopted children, are the exclusive heirs to the intestate estate of the deceased couple, conformably to the following Article 979 of the Civil Code: Art. 979. Legitimate children and their descendants succeed the parents and other ascendants, without distinction as to sex or age, and even if they should come from different marriages.
743

University of the Cordilleras College of Law First Year C S.Y. 2013 - 2014

An adopted child succeeds to the property of the adopting parents in the same manner as a legitimate child. The philosophy underlying this article is that a person's love descends first to his children and grandchildren before it ascends to his parents and thereafter spreads among his collateral relatives. It is also supposed that one of his purposes in acquiring properties is to leave them eventually to his children as a token of his love for them and as a provision for their continued care even after he is gone from this earth. Coming now to the right of representation, we stress first the following pertinent provisions of the Civil Code: Art. 970. Representation is a right created by fiction of law, by virtue of which the representative is raised to the place and the degree of the person represented, and acquires the rights which the latter would have if he were living or if he could have inherited. Art. 971. The representative is called to the succession by the law and not by the person represented. The representative does not succeed the person represented but the one who the person represented would have succeeded. Art. 981. Should children of the deceased and descendants of other children who are dead, survive, the former shall inherit in their own right, and the latter by right of representation. There is no question that as the legitimate daughter of Teodoro and thus the granddaughter of Eleno and Rafaela, Doribel has a right to represent her deceased father in the distribution of the intestate estate of her grandparents. Under Article 981, quoted above, she is entitled to the share her father would have directly inherited had he survived, which shall be equal to the shares of her grandparents' other children. But a different conclusion must be reached in the case of Delia and Edmundo, to whom the grandparents were total strangers. While it is true that the adopted child shall be deemed to be a legitimate child and have the same right as the latter, these rights do not include the right of representation. The relationship created by the adoption is between only the adopting parents and the adopted child and does not extend to the blood relatives of either party. In sum, we agree with the lower courts that Delia and Edmundo as the adopted children and Doribel as the legitimate daughter of Teodoro Sayson and Isabel Bautista, are their exclusive heirs and are under no obligation to share the estate of their parents with the petitioners. The Court of Appeals was correct, however, in holding that only Doribel has the right of representation in the inheritance of her grandparents' intestate estate, the other private respondents being only the adoptive children of the deceased Teodoro. WHEREFORE, the petition is DENIED, and the challenged decision of the Court of Appeals is AFFIRMED in toto, with costs against the petitioners.

744

University of the Cordilleras College of Law First Year C S.Y. 2013 - 2014

CASE DIGEST Sayson Vs. CA Date Decided: January 23, 1992 Ponenete: Cruz FACTS: Eleno and Rafaela Sayson begot five children, namely, Mauricio, Rosario, Basilisa, Remedios and Teodoro. Eleno died on November 10, 1952, and Rafaela on May15,1976. Teodoro, who had married Isabel Bautista, died on March 23, 1972. His wife died nine years later. Their properties were left in the possession of Delia, Edmundo, and Doribel, all surnamed Sayson, who claim to be their children. Mauricio, Rosario, Basilisa, and Remedios, together with Juana C. Bautista, Isabel's mother, filed a complaint for partition and accounting of the intestate estate of Teodoro and Isabel Sayson. Delia, Edmundo and Doribel filed their own complaint, this time for the accounting and partition of the intestate estate of Eleno and Rafaela Sayson, against the couple's four surviving children. Both cases filed on the Lower Court were decided in favour Delia, et al. on the basis of practically the same evidence. The Lower Court declared that Delia and Edmundo were the legally adopted children of Teodoro and Isabel Sayson by virtue of the decree of adoption. Doribel was their legitimate daughter as evidenced by her birth certificate. Consequently, the three children were entitled to inherit from Eleno and Rafaela by right of representation. Both cases were appealed to the Court of Appeals, where they were consolidated. The appellate court affirmed that Delia, et al. are entitled to the intestate estate of spouses Teodoro and Isabel Sayson. However, Delia and Edmundo are disqualified from inheriting from the estate of the deceased spouses Eleno and Rafaela Sayson. ISSUE: W/N CA is correct in holding that Delia and Edmundo are disqualified to inherit from the estate of the deceased spouses Eleno and Rafaela Sayson. HELD: A different conclusion must be reached in the case of Delia and Edmundo, to whom the grandparents were total strangers. While it is true that the adopted child shall be deemed to be a legitimate child and have the same right as the latter, these rights do not include the right of representation. The relationship created by the adoption is between only the adopting parents and the adopted child and does not extend to the blood relatives of either party. In sum, we agree with the lower courts that Delia and Edmundo as the adopted children and Doribelas the legitimate daughter of Teodoro Sayson and Isabel Bautista, are their exclusive heirs and are under no obligation to share the estate of their parents with the petitioners. The Court of Appeals was correct, however, in holding that only Doribel has the right of representation in the inheritance of her grandparents' intestate estate, the other private respondents being only the adoptive children of the deceased Teodoro.

745

University of the Cordilleras College of Law First Year C S.Y. 2013 - 2014

Tayag vs. Ca G.R. No. 95229 Full Case

June 9, 1992

CORITO OCAMPO TAYAG, petitioner, vs. HON. COURT OF APPEALS and EMILIE DAYRIT CUYUGAN, respondent.

REGALADO, J.: The instant petition seeks to reverse and set aside the decision of respondent Court of Appeals in CA-G.R. SP No. 20222, entitled "Corito Ocampo Tayag vs. Hon. Norberto C. Ponce, Judge, Regional Trial Court of San Fernando, Pampanga and Emilde Dayrit Cuyugan," promulgated on May 10, 1990, and its resolution denying petitioner's motion for reconsideration. Said decision, now before us for review, dismissed petitioner's Petition for Certiorari and Prohibition with Preliminary Injunction on the ground that the denial of the motion to dismiss Civil Case No. 7938 of the court a quo is an interlocutory order and cannot be the subject of the said special civil action, ordinary appeal in due time being petitioner's remedy. In said Civil Case No, 7938, herein private respondent, in her capacity as mother and legal guardian of minor Chad D. Cuyugan, filed on April 9, 1987 a complaint denominated "Claim for Inheritance" against herein petitioner as the administratrix of the estate of the late Atty. Ricardo Ocampo. The operative allegations in said complaint are as follows: xxx xxx xxx

2. Plaintiff is the mother and legal guardian of her minor son, Chad Cuyugan, by the father of the defendant, the late Atty. Ricardo Ocampo; and the defendant is the known administratrix of the real and personal properties left by her deceased father, said Atty. Ocampo, who died intestate in Angeles City on September 28, 1983; 3. Plaintiff has been estranged from her husband, Jose Cuyugan, for several years now and during which time, plaintiff and Atty. Ricardo Ocampo had illicit amorous relationship with each other that, as a consequence thereof, they begot a child who was christened Chad Cuyugan in accordance with the ardent desire and behest of said Atty. Ocampo; 4. Chad, the son of plaintiff by the late Atty. Ricardo Ocampo, who was born in Angeles City on October 5, 1980 bad been sired, showered with exceptional affection, fervent love and care by his putative father for being his only son as can be gleaned from indubitable letters and documents of the late Atty. Ocampo to herein plaintiff, excerpts from some of which are hereunder reproduced; . . . Keep good keep faith keep Chad and yourself for me alone and for me all the time. As I have now I shall save my heart to you and to Chad. . . . Please take good care and pray to Sto. Nio for our sake and for the child sake. . . . Keep him. Take good care of him. . . . I'm proud that you are his mother. . . I'm proud of him and you. Let me bless him by my name and let me entitle him to all what I am and what I've got. . . . I have vowed to recognize him and be my heir. . . . How is CHAD and you . . .
746

University of the Cordilleras College of Law First Year C S.Y. 2013 - 2014

. . . Why should we not start now to own him, jointly against the whole world. After all we love each other and CHAD is the product of our love. 5. The minor, Chad D. Cuyugan, although illegitimate is nevertheless entitled to a share in the intestate estate left by his deceased father, Atty. Ricardo Ocampo as one of the surviving heirs; 6. The deceased Atty. Ricardo Ocampo, at the time of his death was the owner of real and personal property, located in Baguio City, Angeles City and in the Province of Pampanga with approximate value of several millions of pesos; 7. The estate of the late Atty. Ocampo has not as yet been inventoried by the defendant and the inheritance of the surviving heirs including that of said Chad has not likewise been ascertained; 8. The only known surviving heirs of the deceased Atty. Ricardo Ocampo are his children, namely: Corito O. Tayag, Rivina O. Tayag, Evita O. Florendo, Felina Ocampo, and said minor Chad, for and in whose behalf this instant complaint is filed; 9. Plaintiff has no means of livelihood and she only depends on the charity of friends and relatives for the sustenance of her son, Chad, such that it is urgent, necessary and imperative that said child be extended financial support from the estate of his putative father, Atty. Ricardo Ocampo; 10. Several demands, verbal and written, have been made for defendant to grant Chad's lawful inheritance, but despite said demands, defendant failed and refused and still fails and refused and still fails and refuses to satisfy the claim for inheritance against the estate of the late Atty. Ocampo; xxx xxx xxx

Plaintiff thereafter prays, among others, that judgment be rendered ordering defendant to render an inventory and accounting of the real and personal properties left by Atty. Ricardo Ocampo; to determine and deliver the share of the minor child Chad in the estate of the deceased; and to give him support pendentelite. Petitioner, as defendant therein, filed her answer with counterclaim on June 3, 1987, disputing the material allegations in the complaint. She maintained by way of affirmative defenses, inter alia, that the complaint states no cause of action; that the action is premature; that the suit as barred by prescription; that respondent Cuyugan has no legal and judicial personality to bring the suit; that the lower court was no jurisdiction over the nature of the action; and that there is improper joinder of causes of action. After the hearing of the motion to dismiss on the grounds asserted as affirmative defenses, the trial court issued the following order on October 20, 1987: xxx xxx xxx

The Court is of the considered opinion that there is a need of further proceedings to adduce evidence on the various claims of the parties so as to hear their respective sides WHEREFORE, resolution on the preliminary hearing which partakes of the nature of a motion to dismiss requiring additional evidence is in the meantime held in abeyance. The Motion to Dismiss is hereby denied and the case as set for pre-trial . . .

747

University of the Cordilleras College of Law First Year C S.Y. 2013 - 2014

With the denial of her motion for reconsideration of said order on November 19, 1987, petitioner filed on December 10, 1987 a petition for certiorari and prohibition before the Court of Appeals, docketed therein as CA-G.R. SP No. 13464, which was granted by the Sixth Division of respondent court on August 2, 1989 and enjoined respondent judge to resolve petitioner's motion praying for the dismissal of the complaint based on the affirmative defenses within ten (10) days from notice thereof. In compliance with said decision of respondent court, the trial court acted on and thereafter denied the motion to dismiss, which had been pleaded in the affirmative defenses in Civil Case No. 7938, in an order dated October 24, 1989, resolving the said motion in the following manner: xxx xxx xxx

The Court now resolves: No. 1. The complaint sufficiently shows that a cause of action exists in favor of the plaintiff. A cause of action being the "primary right to redress a wrong" (Marquez vs. Valera, 48 OG 5272), which apparently on the face of the complaint, plaintiff has a right to enforce through this case. Defendant's protestation that there is no sufficient cause of action is therefore untenable. No. 2. The present action. despite the claim of defendant is not premature. It is exactly filed in order to prove filiation, and then recognition. To go about the step by step procedure outlined by the defendant by filing one action after another is definitely violative of the prohibition against splitting a cause of action. No. 3. It is not the plaintiff that is now bringing the case before the Court. It is (her) spurious child that she represents as natural guardian that is instituting the action. No. 4. Prescription has not set in if we consider that a spurious child may file an action for recognition within four years from his attainment of majority (New Civil Code. Art, 285, No. 2). Whether the letters of the putative father, Atty. Ocampo, is evidence, that should be inquired into in a hearing on the merits. No. 5. Several causes of action may be joined in one complaint as was done in this case. The defendant's claim that there was a misjoinder is untenable. No. 6. The Court being a court of general jurisdiction, and of special jurisdiction, such as a probate court has capacity to entertain a complaint such as the one now before it. The nature of the case "CLAIM FOR INHERITANCE" does not control the body of the complaint. From all the foregoing, the Court finds that the complaint is sufficient' in form and substance and, therefore, the motion to dismiss could not be granted until after trial on the merits in which it should be shown that the allegations of the complaint are unfounded or a special defense to the action exists. WHEREFORE, the Motion to Dismiss is hereby DENIED. Petitioner's motion for reconsideration of said order was denied by the trial court on January 30, 1990. As a consequence, another petition for certiorari and prohibition with preliminary injunction was filed by petitioner on March 12, 1990 with respondent court, docketed as CAG.R. SP No. 20222, praying that the orders dated October 24, 1989 and January 30, 1990 of the trial court be annulled and set aside for having been issued with grave abuse of discretion amounting to lack or excess of jurisdiction.

748

University of the Cordilleras College of Law First Year C S.Y. 2013 - 2014

On May 10, 1990, as earlier stated, respondent court promulgated its decision dismissing the petition, and likewise denied petitioner's motion for reconsideration in a resolution dated September 5, 1990, hence the present petition for review on certiorari. In elevating the case before us, petitioner relies on these grounds: a. The Honorable Respondent Court of Appeals dismissed Petitioner's Petition for Certiorari and Prohibition in UTTER DISREGARD OF APPLICABLE DECISIONS OF THIS HONORABLE COURT providing clear exceptions to the general rule that interlocutory orders may not be elevated by way of the special civil action of certiorari; b. Respondent Court refused to resolve certain issues raised by Petitioner before the Regional Trial Court and before Respondent Court of Appeals involving QUESTIONS OF SUBSTANCE not theretofore determined by this Honorable Court, such as the interpretation and application of Art. 281 of the Civil Code requiring judicial approval when the recognition of an illegitimate minor child does not take place in a record of birth or in a will: of Art. 175, Par. 2, in relation to Art. 172, Par. 2 of the Family Code, providing for the prescriptive period with respect to the action to establish illegitimate filiation; and of Art. 285 of the Civil Code, providing for the prescriptive period with respect to the action for recognition of a natural child; and c. Respondent Court has sanctioned a DEPARTURE by the Regional Trial Court from the accepted and usual course of judicial proceedings. Petitioner contends that the action to claim for inheritance filed by herein private respondent in behalf of the minor child, Chad Cuyugan, is premature and the complaint states no cause of action, she submits that the recognition of the minor child, either voluntarily or by judicial action, by the alleged putative father must first be established before the former can invoke his right to succeed and participate in the estate of the latter. Petitioner asseverates that since there is no allegation of such recognition in the complaint denominated as "Claim for Inheritance," then there exists no basis for private respondent's aforesaid claim and, consequently, the complaint should be dismissed. The instant case is similar to the case of Paulino vs. Paulino, et al., wherein the petitioner, as plaintiff, brought an action against the private respondents, as defendants, to compel them to give her share of inheritance in the estate of the late Marcos Paulino, claiming and alleging, inter alia, that she is the illegitimate child of the deceased; that no proceedings for the settlement of the deceased's estate had been commenced in court; and that the defendants had refused and failed to deliver her share in the estate of the deceased. She accordingly prayed that the defendants therein be ordered to deliver her aforesaid share. The defendants moved for the dismissal of her complaint on the ground that it states no cause of action and that, even if it does, the same is barred by prescription. The only difference between the afore cited case and the case at bar is that at the time of the filing of the complaint therein, the petitioner in that case had already reached the age of majority, whereas the claimant in the present case is still a minor. In Paulino, we held that an illegitimate child, to be entitled to support and successional rights from the putative or presumed parent, must prove his filiation to the latter. We also said that it is necessary to allege in the complaint that the putative father had acknowledged and recognized the illegitimate child because such acknowledgment is essential to and is the basis of the right to inherit. There being no allegation of such acknowledgment, the action becomes one to compel recognition which cannot be brought after the death of the putative father. The ratio decidendi in Paulino, therefore, is not the absence of a cause of action for failure of the petitioner to allege the fact of acknowledgment in the complaint, but the prescription of the action. Applying the foregoing principles to the case at bar, although petitioner contends that the complaint filed by herein private respondent merely alleges that the minor Chad Cuyugan is an illegitimate child of the deceased and is actually a claim for inheritance, from the allegations
749

University of the Cordilleras College of Law First Year C S.Y. 2013 - 2014

therein the same may be considered as one to compel recognition. Further that the two causes of action, one to compel recognition and the other to claim inheritance, may be joined in one complaint is not new in our jurisprudence. As early as 1922, we had occasion to rule thereon in Briz vs. Briz, et al., 12 wherein we said: The question whether a person in the position of the present plaintiff can any event maintain a complex action to compel recognition as a natural child and at the same time to obtain ulterior relief in the character of heir, is one which, in the opinion of this court must be answered in the affirmative, provided always that the conditions justifying the joinder of the two distinct causes of action are present in the particular case. In, other words, there is no absolute necessity requiring that the action to compel acknowledgment should have been instituted and prosecuted to a successful conclusion prior to the action in which that same plaintiff seers additional relief in the character of heir. Certainly, there is nothing so peculiar to the action to compel acknowledgment as to require that a rule should be here applied different from that generally applicable in other cases. . . The conclusion above stated, though not heretofore explicitly formulated by this court, is undoubtedly to some extent supported by our prior decisions. Thus, we have held in numerous cases, and the doctrine must be considered well settled, that a natural child having a right to compel acknowledgment, but who has not been in fact legally acknowledged, may maintain partition proceedings for the division of the inheritance against his co-heirs . . .; and the same person may intervene in proceedings for the distribution of the estate of his deceased natural father, or mother . . . In neither of these situations has it been thought necessary for the plaintiff to show a prior decree compelling acknowledgment. The obvious reason is that in partition suits and distribution proceedings the other persons who might take by inheritance are before the court; and the declaration of heirship is appropriate to such proceedings. The next question to be resolved is whether the action to compel recognition has prescribed. Petitioner argues that assuming arguendo that the action is one to compel recognition, private respondent's cause of action has prescribed for the reason that since filiation is sought to be proved by means of a private handwritten instrument signed by the parent concerned, then under paragraph 2, Article 175 of the Family Code, the action to establish filiation of the illegitimate minor child must be brought during the lifetime of the alleged putative father. In the case at bar, considering that the complaint was filed after the death of the alleged parent, the action has prescribed and this is another ground for the dismissal of the complaint. Petitioner theorizes that Article 285 of the Civil Code is not applicable to the case at bar and, instead, paragraph 2, Article 175 of the Family Code should be given retroactive effect. The theory is premised on the supposition that the latter provision of law being merely procedural in nature, no vested rights are created, hence it can be made to apply retroactively. Article 285 of the Civil Code provides: Art. 285. The action for the recognition of natural children may be brought only during the lifetime of the presumed parents, except in the following cases: (1) If the father or mother died during the minority of the child, in which case the latter may file the action before the expiration of four years from the attainment of his majority; xxx xxx xxx

On the other hand, Article 175 of the Family Code reads: Art. 175. Illegitimate children may establish their illegitimate filiation in the same way and on the same evidence as legitimate children.
750

University of the Cordilleras College of Law First Year C S.Y. 2013 - 2014

The action must be brought within the same period specified in Article 173, except when the action is based on the second paragraph of Article 172, in which case the action may be brought during the lifetime of the alleged parent. Under the last-quoted provision of law, therefore, if the action is based on the record of birth of the child, a final judgment, or an admission by the parent of the child's filiation in a public document or in a private handwritten signed instrument, then the action may be brought during the lifetime of the child. However, if the action is based on the open and continuous possession by the child of the status of an illegitimate child, or on other evidence allowed by the Rules of Court and special laws, the view has been expressed that the action must be brought during the lifetime of the alleged parent. Petitioner submits that Article 175 of the Family Code applies in which case the complaint should have been filed during the lifetime of the putative father, failing which the same must be dismissed on the ground of prescription. Private respondent, however, insists that Article 285 of the Civil Code is controlling and, since the alleged parent died during the minority of the child, the action for filiation may be filed within four years from the attainment of majority of the minor child. Article 256 of the Family Code states that "[t]his Code shall have retroactive effect insofar as it does not prejudice or impair vested or acquired rights in accordance with the Civil Code or other laws." It becomes essential, therefore, to determine whether the right of the minor child to file an action for recognition is a vested right or not. Under the circumstances obtaining in the case at bar, we hold that the right of action of the minor child bas been vested by the filing of the complaint in court under the regime of the Civil Code and prior to the effectivity of the Family Code. We herein adopt our ruling in the recent case of Republic of the Philippines vs. Court of Appeals, et al. where we held that the fact of filing of the petition already vested in the petitioner her right to file it and to have the same proceed to final adjudication in accordance with the law in force at the time, and such right can no longer be prejudiced or impaired by the enactment of a new law. Even assuming ex gratia argumenti that the provision of the Family Code in question is procedural in nature, the rule that a statutory change in matters of procedure may affect pending actions and proceedings, unless the language of the act excludes them from its operation, is not so pervasive that it may be used to validate or invalidate proceedings taken before it goes into effective, since procedure must be governed by the law regulating it at the time the question of procedure arises especially where vested rights may be prejudiced. Accordingly, Article 175 of the Family Code finds no proper application to the instant case since it will ineluctably affect adversely a right of private respondent and, consequentially, of the mind child she represents, both of which have been vested with the filing of the complaint in court. The trial court is therefore, correct in applying the provisions of Article 285 of the Civil Code and in holding that private respondent's cause of action has not yet prescribed. Finally, we conform with the holding of the Court of Appeals that the questioned order of the court below denying the motion to dismiss is interlocutory and cannot be the subject of a petition for certiorari. The exceptions to this rule invoked by petitioner and allegedly obtaining in the case at bar, are obviously not present and may not be relied upon. WHEREFORE, the petition at bar is DENIED and the assailed decision and resolution of respondent Court of Appeals are hereby AFFIRMED in toto. SO ORDERED.

751

University of the Cordilleras College of Law First Year C S.Y. 2013 - 2014

CASE DIGEST Tayag vs. CA Date Decided: June 9, 1992 Ponente: Regalado Facts:Petition for Review on certiorari of the decision of the Court of Appeals. A deed of conveyance was executed by Galicia Sr. and Labuguin in favor of respondent Leyva for an undivided one-half piece of land in Nueva Ecija for PhP 50,000. The PhP 3,000 initial payment was complete, but the succeeding installments of PhP 10,000 each were received as follows: PhP 9,707 on first installment and PhP 6,926.41 as payment for the loan of Galicia Sr. to Philippine Veterans Bank. Levya then consigned his supposed last installment of PhP 20,000 to the court upon claims of two different parties, but only consigned PhP 18,250. Issue: Whether Galicia Sr. has a right to cancel the deed of conveyance due to partial fullfilment of obligations. Held: The fact that in spite of delay, the petitioners still accepted the payments waives their right to contest the non-fulfillment of the obligation. When the obligee accepts the performance in spite of knowledge of incompleteness or irregularity and without expressing any protest or objection, the obligation is deemed complied with. There is a valid consignation since it was established that two heirs of Galicia Sr. claimed the payment. Respondent Leyva substantially complied with the obligation, and it is deemed extinguished.

752

University of the Cordilleras College of Law First Year C S.Y. 2013 - 2014

Benitez-Badua vs. CA G.R. No. 105625 January 24, 1994 Full Case MARISSA BENITEZ-BADUA, petitioner, vs. COURT OF APPEALS, VICTORIA BENITEZ LIRIO AND FEODOR BENITEZ AGUILAR, respondents. Reynaldo M. Alcantara for petitioner. Augustus Cesar E. Azura for private respondents.

PUNO, J.: This is a petition for review of the Decision of the 12th Division of the Court of Appeals in CAG.R. No. CV No. 30862 dated May 29, 1992. The facts show that the spouses Vicente Benitez and Isabel Chipongian owned various properties especially in Laguna. Isabel died on April 25, 1982. Vicente followed her in the grave on November 13, 1989. He died intestate. The fight for administration of Vicente's estate ensued. On September 24, 1990, private respondents Victoria Benitez-Lirio and Feodor Benitez Aguilar (Vicente's sister and nephew, respectively) instituted Sp. Proc. No. 797 (90) before the RTC of San Pablo City, 4th Judicial Region, Br. 30. They prayed for the issuance of letters of administration of Vicente's estate in favor of private respondent Aguilar. They alleged, inter alia, viz.: xxx xxx xxx

4. The decedent is survived by no other heirs or relatives be they ascendants or descendants, whether legitimate, illegitimate or legally adopted; despite claims or representation to the contrary, petitioners can well and truly establish, given the chance to do so, that said decedent and his spouse Isabel Chipongian who pre-deceased him, and whose estate had earlier been settled extra-judicial, were without issue and/or without descendants whatsoever, and that one Marissa Benitez-Badua who was raised and cared by them since childhood is, in fact, not related to them by blood, nor legally adopted, and is therefore not a legal heir; . . . On November 2, 1990, petitioner opposed the petition. She alleged that she is the sole heir of the deceased Vicente Benitez and capable of administering his estate. The parties further exchanged reply and rejoinder to buttress their legal postures. The trial court then received evidence on the issue of petitioner's heirship to the estate of the deceased. Petitioner tried to prove that she is the only legitimate child of the spouses Vicente Benitez and Isabel Chipongian. She submitted documentary evidence, among others: (1) her Certificate of Live Birth (Exh. 3); (2) Baptismal Certificate (Exh. 4); (3) Income Tax Returns and Information Sheet for Membership with the GSIS of the late Vicente naming her as his daughter (Exhs. 10 to 21); and (4) School Records (Exhs. 5 & 6). She also testified that the said spouses reared an continuously treated her as their legitimate daughter. On the other hand, private respondents tried to prove, mostly thru testimonial evidence, that the said spouses failed to beget a child during their marriage; that the late Isabel, then thirty six (36) years of age, was even referred to Dr.Constantino Manahan, a noted obstetrician-gynecologist, for treatment. Their primary witness, Victoria Benitez-Lirio, elder sister of the late Vicente, then 77 years of age, categorically declared that petitioner was not the biological child of the said spouses who were unable to physically procreate.

753

University of the Cordilleras College of Law First Year C S.Y. 2013 - 2014

On December 17, 1990, the trial court decided in favor of the petitioner. It dismissed the private respondents petition for letters and administration and declared petitioner as the legitimate daughter and sole heir of the spouses Vicente O. Benitez and Isabel Chipongian. The trial court relied on Articles 166 and 170 of the Family Code. On appeal, however, the Decision of the trial court was reversed on May 29, 1992 by the 17th Division of the Court of Appeals. The dispositive portion of the Decision of the appellate court states: WHEREFORE, the decision appealed from herein is REVERSED and another one entered declaring that appellee Marissa Benitez is not the biological daughter or child by nature of the spouse Vicente O. Benitez and Isabel Chipongian and, therefore, not a legal heir of the deceased Vicente O. Benitez. Her opposition to the petition for the appointment of an administrator of the intestate of the deceased Vicente O. Benitez is, consequently, DENIED; said petition and the proceedings already conducted therein reinstated; and the lower court is directed to proceed with the hearing of Special proceeding No. SP-797 (90) in accordance with law and the Rules. Costs against appellee. SO ORDERED. In juxtaposition, the appellate court held that the trial court erred in applying Articles 166 and 170 of the Family Code. In this petition for review, petitioner contends: 1. The Honorable Court of Appeals committed error of law and misapprehension of facts when it failed to apply the provisions, more particularly, Arts. 164, 166, 170 and 171 of the Family Code in this case and in adopting and upholding private respondent's theory that the instant case does not involve an action to impugn the legitimacy of a child; 2. Assuming arguendo that private respondents can question or impugn directly or indirectly, the legitimacy of Marissa's birth, still the respondent appellate Court committed grave abuse of discretion when it gave more weight to the testimonial evidence of witnesses of private respondents whose credibility and demeanor have not convinced the trial court of the truth and sincerity thereof, than the documentary and testimonial evidence of the now petitioner Marissa Benitez-Badua; 3. The Honorable Court of Appeals has decided the case in a way not in accord with law or with applicable decisions of the supreme Court, more particularly, on prescription or laches. We find no merit to the petition. Petitioner's insistence on the applicability of Articles 164, 166, 170 and 171 of the Family Code to the case at bench cannot be sustained. These articles provide: Art. 164. Children conceived or born during the marriage of the parents are legitimate.

Children conceived as a result of artificial insemination of the wife with sperm of the husband or that of a donor or both are likewise legitimate children of the husband and his wife, provided, that both of them authorized or ratified such insemination in a written instrument executed and signed by them before the birth of the child. The instrument shall be recorded in the civil registry together with the birth certificate of the child. Art. 166. Legitimacy of child may be impugned only on the following grounds:

754

University of the Cordilleras College of Law First Year C S.Y. 2013 - 2014

1) That it was physically impossible for the husband to have sexual intercourse with his wife within the first 120 days of the 300 days which immediately preceded the birth of the child because of: a) the physical incapacity of the husband to have sexual intercourse with his wife;

b) the fact that the husband and wife were living separately in such a way that sexual intercourse was not possible; or c) serious illness of the husband, which absolutely prevented sexual intercourse.

2) That it is proved that for biological or other scientific reasons, the child could not have been that of the husband except in the instance provided in the second paragraph of Article 164; or 3) That in case of children conceived through artificial insemination, the written authorization or ratification of either parent was obtained through mistake, fraud, violence, intimidation, or undue influence. Art. 170. The action to impugn the legitimacy of the child shall be brought within one year from the knowledge of the birth or its recording in the civil register, if the husband or, in a proper case, any of his heirs, should reside in the city or municipality where the birth took place or was recorded. If the husband or, in his default, all of his heirs do not reside at the place of birth as defined in the first paragraph or where it was recorded, the period shall be two years if they should reside in the Philippines; and three years if abroad. If the birth of the child has been concealed from or was unknown to the husband or his heirs, the period shall be counted from the discovery or knowledge of the birth of the child or of the fact of registration of said birth, whichever is earlier. Art. 171. The heirs of the husband may impugn the filiation of the child within the period prescribed in the preceding Article only in the following case: 1) 2) 3) If the husband should die before the expiration of the period fixed for bringing his action; If he should die after the filing of the complaint, without having desisted therefrom; or If the child was born after the death of the husband.

A careful reading of the above articles will show that they do not contemplate a situation, like in the instant case, where a child is alleged not to be the child of nature or biological child of a certain couple. Rather, these articles govern a situation where a husband (or his heirs) denies as his own a child of his wife. Thus, under Article 166, it is the husband who can impugn the legitimacy of said child by proving: (1) it was physically impossible for him to have sexual intercourse, with his wife within the first 120 days of the 300 days which immediately preceded the birth of the child; (2) that for biological or other scientific reasons, the child could not have been his child; (3) that in case of children conceived through artificial insemination, the written authorization or ratification by either parent was obtained through mistake, fraud, violence, intimidation or undue influence. Articles 170 and 171 reinforce this reading as they speak of the prescriptive period within which the husband or any of his heirs should file the action impugning the legitimacy of said child. Doubtless then, the appellate court did not err when it refused to apply these articles to the case at bench. For the case at bench is not one where the heirs of the late Vicente are contending that petitioner is not his child by Isabel. Rather, their clear submission is that petitioner was not born to Vicente and Isabel. Our ruling in Cabatbat-Lim vs. Intermediate Appellate Court, 166 SCRA 451, 457 cited in the impugned decision is apropos, viz.:
755

University of the Cordilleras College of Law First Year C S.Y. 2013 - 2014

Petitioners' recourse to Article 263 of the New Civil Code [now Article 170 of the Family Code] is not well-taken. This legal provision refers to an action to impugn legitimacy. It is inapplicable to this case because this is not an action to impugn the legitimacy of a child, but an action of the private respondents to claim their inheritance as legal heirs of their childless deceased aunt. They do not claim that petitioner VioletaCabatbat Lim is an illegitimate child of the deceased, but that she is not the decedent's child at all. Being neither legally adopted child, nor an acknowledged natural child, nor a child by legal fiction of Esperanza Cabatbat, Violeta is not a legal heir of the deceased. We now come to the factual finding of the appellate court that petitioner was not the biological child or child of nature of the spouses Vicente Benitez and Isabel Chipongian. The appellate court exhaustively dissected the evidence of the parties as follows: . . . And on this issue, we are constrained to say that appellee's evidence is utterly insufficient to establish her biological and blood kinship with the aforesaid spouses, while the evidence on record is strong and convincing that she is not, but that said couple being childless and desirous as they were of having a child, the late Vicente O. Benitez took Marissa from somewhere while still a baby, and without he and his wife's legally adopting her treated, cared for, reared, considered, and loved her as their own true child, giving her the status as not so, such that she herself had believed that she was really their daughter and entitled to inherit from them as such. The strong and convincing evidence referred to us are the following: First, the evidence is very cogent and clear that Isabel Chipongian never became pregnant and, therefore, never delivered a child. Isabel's own only brother and sibling, Dr. Lino Chipongian, admitted that his sister had already been married for ten years and was already about 36 years old and still she has not begotten or still could not bear a child, so that he even had to refer her to the late Dr.Constantino Manahan, a well-known and eminent obstetrician-gynecologist and the OB of his mother and wife, who treated his sister for a number of years. There is likewise the testimony of the elder sister of the deceased Vicente O. Benitez, Victoria Benitez Lirio, who then, being a teacher, helped him (he being the only boy and the youngest of the children of their widowed mother) through law school, and whom Vicente and his wife highly respected and consulted on family matters, that her brother Vicente and his wife Isabel being childless, they wanted to adopt her youngest daughter and when she refused, they looked for a baby to adopt elsewhere, that Vicente found two baby boys but Isabel wanted a baby girl as she feared a boy might grow up unruly and uncontrollable, and that Vicente finally brought home a baby girl and told his elder sister Victoria he would register the baby as his and his wife's child. Victoria Benitez Lirio was already 77 years old and too weak to travel and come to court in San Pablo City, so that the taking of her testimony by the presiding judge of the lower court had to be held at her residence in Paraaque, MM. Considering, her advanced age and weak physical condition at the time she testified in this case, Victoria Benitez Lirio's testimony is highly trustworthy and credible, for as one who may be called by her Creator at any time, she would hardly be interested in material things anymore and can be expected not to lie, especially under her oath as a witness. There were also several disinterested neighbors of the couple Vicente O. Benitez and Isabel Chipongian in Nagcarlan, Laguna (Sergio Fule, Cecilia Coronado, and Benjamin C. Asendido) who testified in this case and declared that they used to see Isabel almost everyday especially as she had drugstore in the ground floor of her house, but they never saw her to have been pregnant, in 1954 (the year appellee Marissa Benitez was allegedly born, according to her birth certificate Exh. "3") or at any time at all, and that it is also true with the rest of their townmates. Ressureccion A. Tuico, Isabel Chipongian's personal beautician who used to set her hair once a week at her (Isabel's) residence, likewise declared that she did not see Isabel ever become pregnant, that she knows that Isabel never delivered a baby, and that when she saw the baby Marissa in her crib one day she went to Isabel's house to set the latter's hair, she was surprised and asked the latter where the baby came from, and "she told me that the child was brought by Atty. Benitez and told me not to tell about it" (p. 10, tsn, Nov. 29, 1990).

756

University of the Cordilleras College of Law First Year C S.Y. 2013 - 2014

The facts of a woman's becoming pregnant and growing big with child, as well as her delivering a baby, are matters that cannot be hidden from the public eye, and so is the fact that a woman never became pregnant and could not have, therefore, delivered a baby at all. Hence, if she is suddenly seen mothering and caring for a baby as if it were her own, especially at the rather late age of 36 (the age of Isabel Chipongian when appellee Marissa Benitez was allegedly born), we can be sure that she is not the true mother of that baby. Second, appellee's birth certificate Exh. "3" with the late Vicente O. Benitez appearing as the informant, is highly questionable and suspicious. For if Vicente's wife Isabel, who wads already 36 years old at the time of the child's supposed birth, was truly the mother of that child, as reported by Vicente in her birth certificate, should the child not have been born in a hospital under the experienced, skillful and caring hands of Isabel's obstetriciangynecologistDr.Constantino Manahan, since delivery of a child at that late age by Isabel would have been difficult and quite risky to her health and even life? How come, then, that as appearing in appellee's birth certificate, Marissa was supposedly born at the Benitez home in Avenida Rizal, Nagcarlan, Laguna, with no physician or even a midwife attending? At this juncture, it might be meet to mention that it has become a practice in recent times for people who want to avoid the expense and trouble of a judicial adoption to simply register the child as their supposed child in the civil registry. Perhaps Atty. Benitez, though a lawyer himself, thought that he could avoid the trouble if not the expense of adopting the child Marissa through court proceedings by merely putting himself and his wife as the parents of the child in her birth certificate. Or perhaps he had intended to legally adopt the child when she grew a little older but did not come around doing so either because he was too busy or for some other reason. But definitely, the mere registration of a child in his or her birth certificate as the child of the supposed parents is not a valid adoption, does not confer upon the child the status of an adopted child and the legal rights of such child, and even amounts of simulation of the child's birth or falsification of his or her birth certificate, which is a public document. Third, if appellee Marissa Benitez is truly the real, biological daughter of the late Vicente O. Benitez and his wife Isabel Chipongian, why did he and Isabel's only brother and sibling Dr. Nilo Chipongian, after Isabel's death on April 25, 1982, state in the extrajudicial settlement Exh. "E" that they executed her estate, "that we are the sole heirs of the deceased ISABEL CHIPONGIAN because she died without descendants or ascendants?" Dr. Chipongian, placed on a witness stand by appellants, testified that it was his brother-in-law Atty. Vicente O. Benitez who prepared said document and that he signed the same only because the latter told him to do so (p. 24, tsn, Nov. 22, 1990). But why would Atty. Benitez make such a statement in said document, unless appellee Marissa Benitez is not really his and his wife's daughter and descendant and, therefore, not his deceased wife's legal heir? As for Dr. Chipongian, he lamely explained that he signed said document without understanding completely the meaning of the words "descendant and ascendant" (p. 21, tsn, Nov. 22, 1990). This we cannot believe, Dr.Chipongian being a practicing pediatrician who has even gone to the United States (p. 52, tsn, Dec. 13, 1990). Obviously, Dr.Chipongian was just trying to protect the interests of appellee, the foster-daughter of his deceased sister and brother-in-law, as against those of the latter's collateral blood relatives. Fourth, it is likewise odd and strange, if appellee Marissa Benitez is really the daughter and only legal heir of the spouses Vicente O. Benitez and Isabel Chipongian, that the latter, before her death, would write a note to her husband and Marissa stating that: even without any legal papers, I wish that my husband and my child or only daughter will inherit what is legally my own property, in case I die without a will, and in the same handwritten note, she even implored her husband

757

University of the Cordilleras College of Law First Year C S.Y. 2013 - 2014

that any inheritance due him from my property when he die to make our own daughter his sole heir. This do [sic] not mean what he legally owns or his inherited property. I leave him to decide for himself regarding those. (Exhs. "F-1", "F-1-A" and "F-1-B") We say odd and strange, for if Marissa Benitez is really the daughter of the spouses Vicente O. Benitez and Isabel Chipongian, it would not have been necessary for Isabel to write and plead for the foregoing requests to her husband, since Marissa would be their legal heir by operation of law. Obviously, Isabel Chipongian had to implore and supplicate her husband to give appellee although without any legal papers her properties when she dies, and likewise for her husband to give Marissa the properties that he would inherit from her (Isabel), since she well knew that Marissa is not truly their daughter and could not be their legal heir unless her (Isabel's) husband makes her so. Finally, the deceased Vicente O. Benitez' elder sister Victoria Benitez Lirio even testified that her brother Vicente gave the date December 8 as Marissa's birthday in her birth certificate because that date is the birthday of their (Victoria and Vicente's) mother. It is indeed too much of a coincidence for the child Marissa and the mother of Vicente and Victoria to have the same birthday unless it is true, as Victoria testified, that Marissa was only registered by Vicente as his and his wife's child and that they gave her the birth date of Vicente's mother. We sustain these findings as they are not unsupported by the evidence on record. The weight of these findings was not negated by documentary evidence presented by the petitioner, the most notable of which is her Certificate of Live Birth (Exh. "3") purportedly showing that her parents were the late Vicente Benitez and Isabel Chipongian. This Certificate registered on December 28, 1954 appears to have been signed by the deceased Vicente Benitez. Under Article 410 of the New Civil Code, however, "the books making up the Civil Registry and all documents relating thereto shall be considered public documents and shall be prima facie evidence of the facts therein stated." As related above, the totality of contrary evidence, presented by the private respondents sufficiently rebutted the truth of the content of petitioner's Certificate of Live Birth. of said rebutting evidence, the most telling was the Deed of Extra-Judicial Settlement of the Estate of the Deceased Isabel Chipongian (Exh. "E") executed on July 20, 1982 by Vicente Benitez, and Dr. Nilo Chipongian, a brother of Isabel. In their notarized document, they stated that "(they) are the sole heirs of the deceased Isabel Chipongian because she died without descendants or ascendants". In executing this Deed, Vicente Benitez effectively repudiated the Certificate of Live Birth of petitioner where it appeared that he was petitioner's father. The repudiation was made twenty-eight years after he signed petitioner's Certificate of Live Birth. IN VIEW WHEREOF, the petition for review is dismissed for lack of merit. Costs against petitioner. SO ORDERED.

758

University of the Cordilleras College of Law First Year C S.Y. 2013 - 2014

CASE DIGEST Benitez-BaduaVs. CA Date Decided: January 24, 1994 Ponente: Puno FACTS: Spouses Vicente Benitez and Isabel Chipongian-Benitez owned various properties in Laguna. Isabel died in 1982 while Vicente died in 1989. Vicente died intestate. After Vicentes death, his sister and nephew filed a case for issuance of letters of administration of Vicente's estate. They claim that Isable and Vicente were childless and that one Marissa Benitez-Badua who was raised and cared by them since childhood is not their child and was not legally adopted by them. Marissa opposed the petition and alleged that she is the sole heir of deceased Vicente and capable of administering his estate. She presented Certificate of Live Birth, as one of her evidences to prove her claim. The trial court decided in favor of Marissa and dismissed the petition of Vicentes sister and nephew. The trial court relied on Articles 166 and 170 of the Family Code on impugning the legitimacy of the child. The appellate court reversed the trial court ad held that Articles 166 and 170 of the Family Code are not applicable. ISSUE: Are Articles 166 and 170 of the Family Code applicable in this case? HELD: No. Articles 164, 166, 170 and 171 of the Family Code do not contemplate a situation, like in the instant case, where a child is alleged not to be the child of nature or biological child of a certain couple. Rather, these articles govern a situation where a husband (or his heirs) denies as his own a child of his wife. Thus, under Article 166, it is the husband who can impugn the legitimacy of said child. As regards the Birth Certificate which lists Marissa as the child of Isabel and Vicente, the court said that it is highly questionable and suspicions. Under Article 410 of the New Civil Code, is states that, "the books making up the Civil Registry and all documents relating thereto shall be considered public documents and shall be prima facie evidence of the facts therein stated." The totality of contrary evidence presented by the sister and nephew of Vicente show that Isabel did not become pregnant as testified to by the witnesses. Also the Deed of Extra-Judicial Settlement of the Estate of the Deceased Isabel Chipongian executed by Vicente himself and by Isabels brother, stated that they are the sole heirs of the deceased Isabel Chipongian because she died without descendants or ascendants". In executing this Deed, Vicente effectively repudiated the Certificate of Live Birth of Marissa where it appeared that Vicente was Marissas father. Therefore, Marissas petition before the Supreme Court was dismissed.

759

University of the Cordilleras College of Law First Year C S.Y. 2013 - 2014

Fernandez vs CA G.R. No. 108366 Full Case

February 16, 1994

JOHN PAUL E. FERNANDEZ, ET AL., petitioners, vs. THE COURT OF APPEALS and CARLITO S. FERNANDEZ, respondents. Erlinda B. Espejo for petitioners. C.B. Carbon & Associates for private respondent. PUNO, J.: The legal dispute between the parties began when the petitioners filed Civil Case No. Q-45567 for support against the private respondent before the RTC of Quezon City. The complaint was dismissed on December 9, 1986 by Judge Antonio P. Solano, who found that "(t)here is nothing in the material allegations in the complaint that seeks to compel (private respondent) to recognize or acknowledge (petitioners) as his illegitimate children," and that there was no sufficient and competent evidence to prove the petitioners filiation. Petitioners plodded on. On February 19, 1987, they file the case at bench, another action for recognition and support against the private respondent before another branch of the RTC of Quezon City, Branch 87. The case was docketed as Civil Case No. Q-50111. The evidence shows that VIOLETA P. ESGUERRA, single, is the mother and guardian ad litem of the two petitioners, CLARO ANTONIO FERNANDEZ and JOHN PAUL FERNANDEZ, met sometime in 1983, at the Meralco Compound tennis courts. A Meralco employee and a tennis enthusiast, Carlito used to spend his week-ends regularly at said courts, where Violeta's father served as tennis instructor. Violeta pointed to Carlito as the father of her two sons. She claimed that they started their illicit sexual relationship six (6) months after their first meeting. The tryst resulted in the birth of petitioner Claro Antonio on March 1, 1984, and of petitioner John Paul on not know that Carlito was married until the birth of her two children. She averred they were married in civil rites in October, 1983. In March, 1985, however, she discovered that the marriage license which they used was spurious. To bolster their case, petitioners presented the following documentary evidence: their certificates of live birth, identifying respondent Carlito as their father; the baptismal certificate of petitioner Claro which also states that his father is respondent Carlito; photographs of Carlito taken during the baptism of petitioner Claro; and pictures of respondent Carlito and Claro taken at the home of VioletaEsguerra. Petitioners likewise presented as witnesses, Rosario Cantoria, Dr. Milagros Villanueva, Ruby Chua Cu, and Fr. Liberato Fernandez. The first three witnesses told the trial court that VioletaEsguerra had, at different times, introduced the private respondent to them as her "husband". Fr. Fernandez, on the other hand, testified that Carlito was the one who presented himself as the father of petitioner Claro during the latter's baptism. In defense, respondent Carlito denied Violeta's allegations that he sired the two petitioners. He averred he only served as one of the sponsors in the baptism of petitioner Claro. This claim was corroborated by the testimony of Rodante Pagtakhan, an officemate of respondent Carlito who also stood as a sponsor of petitioner Claro during his baptism. The Private respondent also presented as witness, Fidel Arcagua, a waiter of the Lighthouse Restaurant. He disputed Violeta's allegation that she and respondent Carlito frequented the said restaurant during their affair. Arcagua stated he never saw Violeta Esguerra and respondent Carlito together at the said

760

University of the Cordilleras College of Law First Year C S.Y. 2013 - 2014

restaurant. Private respondent also declared he only learned he was named in the birth certificates of both petitioners as their father after he was sued for support in Civil Case No. Q-45567. Based on the evidence adduced by the parties, the trial court ruled in favor of petitioners, viz.: In view of the above, the Court concludes and so holds that the plaintiffs minors (petitioners herein) are entitled to the relief's prayed for in the complaint. The defendant (herein private respondent) is hereby ordered to recognize Claro Antonio Carlito Fernandez, now aged 6, and John Paul Fernandez, now aged 41/2 as his sons. As the defendant has admitted that he has a supervisory job at the Meralco, he shall give the plaintiffs support in the amount of P2,000 each a month, payment to be delivered to Violeta Esguerra, the children's mother and natural guardian, with arrears reckoned as of the filing of the complaint on February 19, 1987. SO ORDERED. On appeal, the decision was set aside and petitioners complaint dismissed by the respondent Court of Appeals 8 in its impugned decision, dated October 20, 1992. It found that the "proof relied upon by the (trial) court (is) inadequate to prove the (private respondent's) paternity and filiation of (petitioners)." It further held that the doctrine of res judicata applied because of the dismissal of the petitioners complaint in Civil Case No. Q-45567. Petitioners' motion for reconsideration was denied on December 22, 1992. Petitioners now contend that the respondent appellate court erred in: (1) not giving full faith and credit to the testimony in of Violeta Esguerra; (2) not giving weight and value to the testimony of Father Liberato Fernandez; (3) not giving probative value to the numerous pictures of respondent Carlito Fernandez taken during the baptismal ceremony and inside the bedroom of Violeta Esguerra; (4) not giving probative value to the birth certificates of petitioners; (5) giving so much credence to the self-serving and incredible testimony of respondent Carlito Fernandez; and (6) holding that the principle of res judicata is applicable in the case at bar. We find no merit in the petition. The rule is well-settled that findings of facts of the Court of Appeals may be reviewed by this court only under exceptional circumstances. One such situation is when the findings of the appellate court clash with those of the trial court as in the case at bench. It behooves us therefore to exercise our extraordinary power, and settle the issue of whether the ruling of the appellate court that private respondent is not the father of the petitioners is substantiated by the evidence on record. We shall first examine the documentary evidence offered by the petitioners which the respondent court rejected as insufficient to prove their filiation. Firstly, we hold that petitioners cannot rely on the photographs showing the presence of the private respondent in the baptism of petitioner Claro (Exh. "B-8", Exh. "B-12", Exh. "H" and Exh. "I"). These photographs are far from proofs that private respondent is the father of petitioner Claro. As explained by the private respondent, he was in the baptism as one of the sponsors of petitioner Claro. His testimony was corroborated by RodantePagtakhan. Secondly, the pictures taken in the house of Violeta showing private respondent showering affection to Claro fall short of the evidence required to prove paternity (Exhibits "B", "B-1", "B2", "B-7", "B-14" and "B-15"). As we held in Tan vs. Trocio, 192 SCRA 764, viz: . . . The testimonies of complainant and witness MarilouPangandaman, another maid, to show unusual closeness between Respondent and Jewel, like playing with him and giving him paternity. The same must be said of . . . (the) pictures of Jewels and Respondent showing allegedly their physical likeness to each other. Said evidence is inconclusive to prove paternity and much less would prove violation of complaint's person and honor. (Emphasis supplied)
761

University of the Cordilleras College of Law First Year C S.Y. 2013 - 2014

Thirdly, the baptismal certificates (Exh. "D") of petitioner Claro naming private respondent as his father has scant evidentiary value. There is no showing that private respondent participated in its preparation. On this score, we held in Berciles vs. Systems, et al. 128 SCRA 53 (1984): As to the baptismal certificates, Exh. "7-A", the rule is that although the baptismal record of a natural child describes her as a child of the record the decedent had no intervening, the baptismal record cannot be held to be a voluntary recognition of parentage. . . . The reason for this rule that canonical records do not constitute the authentic document prescribed by Arts. 115 and 117 to prove the legitimate filiation of a child is that such canonical record is simply proof of the only act to which the priest may certify by reason of his personal knowledge, an act done by himself or in his presence, like the administration of the sacrament upon a day stated; it is no proof of the declarations in the record with respect to the parentage of the child baptized, or of prior and distinct facts which require separate and concrete evidence. In Macandang vs. Court of Appeals, 100 SCRA 73 (1980), we also ruled that while baptismal certificates may be considered public documents, they can only serve as evidence of the administration of the sacraments on the dates so specified. They are not necessarily competent evidence of the veracity of entries therein with respect to the child's paternity. Fourth, the certificates of live birth (Exh. "A"; Exh. "B") of the petitioners identifying private respondent as their father are not also competent evidence on the issue of their paternity. Again, the records do no show that private respondent had a hand in the preparation of said certificates. In rejecting these certificates, the ruling of the respondent court is in accord with our pronouncement in Roces vs. Local Civil Registrar, 102 Phil. 1050 (1958), viz: . . . Section 5 of Act No. 3793 and Article 280 of the Civil Code of the Philippines explicity prohibited, not only the naming of the father or the child born outside wedlock, when the birth certificates, or the recognition, is not filed or made by him, but, also, the statement of any information or circumstances by which he could be identified. Accordingly, the Local Civil Registrar had no authority to make or record the paternity of an illegitimate child upon the information of a third person and the certificate of birth of an illegitimate child, when signed only by the mother of the latter, is incompetent evidence of fathership of said child. (Emphasis supplied) We reiterated this rule in Berciles, op. cit., when we held that "a birth certificate no signed by the alleged father therein indicated is not competent evidence of paternity." We have also reviewed the relevant testimonies of the witnesses for the petitioners and we are satisfied that the respondent appellate court properly calibrated their weight. Petitioners capitalize on the testimony of Father Liberato Fernandez who solemnized the baptismal ceremony of petitioner Claro. He declared on the witness stand: Q Do you recall Father, whether on that occasion when you called for the father and the mother of the child, that both father and mother were present? A Q A Q A Yes. Would you able to recognized the father and the mother who were present at that time? Yes. Please point to the court? There (witness pointing to the defendant, Carlito Fernandez).

762

University of the Cordilleras College of Law First Year C S.Y. 2013 - 2014

Q him? A Q A

For instance, just give us more specifically what question do you remember having asked

Yes, like for example, do you renounce Satan and his works? What was the answer of Fernandez? Yes, I do.

Q I just want to be sure, Father, will you please look at the defendant again. I want to be sure if he is the person who appeared before you on that occasion? A I am sure.

(TSN, May 23, 1986, pp. 14-16) However, on cross examination, Father Fernandez admitted that he has to be shown a picture of the private respondent by Violeta Esguerra to recognize the private respondent, viz: Q When was the, approximately, when you were first shown this picture by Violeta Esguerra? A Q A Q A Q I cannot recall. At least the month and the year? It must be in 1986. What month in 1986. It is difficult. . . When was the first time you know you are going to testify here?

A Let us see, you came there two times and first one was you want to get a baptismal certificate and then the second time was I asked you for what is this? And you said it is for the court. Q On the second time that Ms. Violeta Esguerra went to your place, you were already informed that you will testify here before this Honorable Court? A Yes.

Q And you were informed by this Ms. Violeta Esguerra that this man wearing the blue Tshirt is the father? A Q A Yes, sir. So, it was Violeta Esguerra who. . . Yes.

(TSN, May 23, 1986, pp. 18 to 22) Indeed, there is no proof that Father Fernandez is a close friend of Violeta Esguerra and the private respondent which should render unquestionable his identification of the private respondent during petitioner Claro's baptism. In the absence of this proof, we are not prepared to
763

University of the Cordilleras College of Law First Year C S.Y. 2013 - 2014

concede that Father Fernandez who officiates numerous baptismal ceremonies day in and day out can remember the parents of the children he has baptized. We cannot also disturb the findings of the respondent court on the credibility of Violeta Esguerra. Her testimony is highly suspect as it is self-serving and by itself, is insufficient to prove the paternity of the petitioners. We shall not pass upon the correctness of the ruling of the respondent appellate court applying the doctrine of res judicata as additional reason in dismissing petitioners action for recognition and support. It is unnecessary considering our findings that petitioners evidence failed to substantiate their cause of action. IN VIEW WHEREOF, the petition is DISMISSED and the Decision of the respondent court in CA-G.R. CV No. 29182 is AFFIRMED. Costs against petitioners. SO ORDERED.

764

University of the Cordilleras College of Law First Year C S.Y. 2013 - 2014 De Santos vs. Hon. Angeles G.R. No. 105619 December 12, 1995 Full Case MARIA ROSARIO DE SANTOS, petitioner, vs. HON. ADORACION G. ANGELES, JUDGE, REGIONAL TRIAL COURT OF CALOOCAN CITY, BRANCH 121 and CONCHITA TALAG DE SANTOS, respondents.

ROMERO, J.: Can natural children by legal fiction be legitimized? There being no explicit provision of law in point, the Court is called upon to cast illumination in a gray area even as it fills up unintentional interstices in the fabric of Civil Law with overlays of philosophical, historical and sociological strands. For an understanding of how the issue arose, we now proceed to unravel the pertinent factual background. On February 7, 1941, Dr. Antonio de Santos married Sofia Bona, which union was blessed with a daughter, herein petitioner Maria Rosario de Santos. After some time, their relationship became strained to the breaking point. Thereafter, Antonio fell in love with a fellow doctor, Conchita Talag, private respondent herein. Antonio sought a formal dissolution of his first marriage by obtaining a divorce decree from a Nevada court in 1949. Obviously aware that said decree was a worthless scrap of paper in our jurisdiction which then, as now, did not recognize divorces, Antonio proceeded to Tokyo, Japan in 1951 to marry private respondent, with whom he had been cohabiting since his de facto separation from Sofia. This union produced eleven children. On March 30, 1967, Sofia died in Guatemala. Less than a month later, on April 23, 1967, Antonio and private respondent contracted a marriage in Tagaytay City celebrated under Philippine laws. On March 8, 1981, Antonio died intestate leaving properties with an estimated value of P15,000,000.00. On May 15, 1981, private respondent went to court 1 asking for the issuance of letters of administration in her favor in connection with the settlement of her late husband's estate. She alleged, among other things, that the decedent was survived by twelve legitimate heirs, namely, herself, their ten surviving children, and petitioner. There being no opposition, her petition was granted. After six years of protracted intestate proceedings, however, petitioner decided to intervene. Thus, in a motion she filed sometime in November 1987, she argued inter alia that private respondent's children were illegitimate. This was challenged by private respondent although the latter admitted during the hearing that all her children were born prior to Sofia's death in 1967. On November 14, 1991, after approval of private respondent's account of her administration, the court a quo passed upon petitioner's motion. The court, citing the case of Francisco H. Tongoy, et al. v. Court of Appeals, et al. (23 SCRA 99 [1983]), declared private respondent's ten children legitimated and thereupon instituted and declared them, along with petitioner and private respondent, as the heirs of Antonio de Santos. Petitioner sought a reconsideration of said order but this was denied in the court's order dated January 9, 1992. Hence, she filed the instant petition for certiorari on June 16, 1992, contending that since only natural children can be legitimized, the trial court mistakenly declared as legitimated her half brothers and sisters. This argument is tenable. Article 269 of the Civil Code expressly states: Art. 269. Only natural children can be legitimated. Children born outside wedlock of parents who, at the time of the conception of the former, were not disqualified by any impediment to marry each other, are natural. 765

University of the Cordilleras College of Law First Year C S.Y. 2013 - 2014 In other words, a child's parents should not have been disqualified to marry each other at the time of conception for him to qualify as a "natural child." In the case at bench, there is no question that all the children born to private respondent and deceased Antonio de Santos were conceived and born when the latter's valid marriage to petitioner's mother was still subsisting. That private respondent and the decedent were married abroad after the latter obtained in Nevada, U.S.A. a decree of divorce from his legitimate wife does not change this fact, for a divorce granted abroad was not recognized in this jurisdiction at the time. Evidently, the decedent was aware of this fact, which is why he had to have the marriage solemnized in Tokyo, outside of the Philippines. It may be added here that he was likewise aware of the nullity of the Tokyo marriage for after his legitimate, though estranged wife died, he hastily contracted another marriage with private respondent, this time here in Tagaytay. It must be noted that while Article 269, which falls under the general heading of "Paternity and Filiation," specifically deals with "Legitimated Children," Article 89, a provision subsumed under the general title on "Marriage," deals principally with void and voidable marriages and secondarily, on the effects of said marriages on their offspring. It creates another category of illegitimate children, those who are "conceived or born of marriages which are void from the beginning," but because there has been a semblance of marriage, they are classified as "acknowledged natural children" and, accordingly, enjoy the same status, rights and obligations as such kind of children. In the case at bench, the marriage under question is considered "void from the beginning" because bigamous, contracted when a prior valid marriage was still subsisting. It follows that the children begotten of such union cannot be considered natural children proper for at the time of their conception, their parents were disqualified from marrying each other due to the impediment of a prior subsisting marriage. What term should then be coined to distinguish them from natural children proper (those "born outside of wedlock of parents who, at the time of the conception of the former, were not disqualified by any impediment to marry each other")? A legal fiction had to be resorted to, that device contrived by law to simulate a fact or condition which, strictly and technically speaking, is not what it purports to be. In this case, the term "natural children by legal fiction" was invented, thus giving rise to another category of illegitimate children, clearly not to be confused with "natural children" as defined under Art. 269 but by fiction of law to be equated with acknowledged natural children and, consequently, enjoying the status, rights and obligations of the latter. Does this cluster of rights include the right to be legitimated? Under the Civil Code, there exists a hierarchy of children classified on the basis of rights granted by law, which must be preserved by strictly construing the substantive provisions of the law in force. Under the prevailing Civil Code (which may be considered "old" in light of the new provisions of the Family Code on "Persons"), much emphasis is laid on the classification of children vis-a-vis their parents, and the corresponding rights they are entitled to under the law. Thus, the title on "Paternity and Filiation" devotes two whole chapters to legitimate children alone, and one chapter on those deemed by law to be possessed of the rights of the former, such as legitimated children, because of their compliance with certain requisites laid down by law; two other chapters deal with illegitimate children composed of recognized natural children, and those other than natural, or spurious, whether recognized or not. The well-ordered delineation of such distinctions among these groups demonstrates a clear intent on the part of the framers of the Civil Code to compartmentalize and separate one from the other, for legitimacy/illegitimacy determines the substantive rights accruing to the different categories of children. It must be noted that before said Code was enacted, other classes of illegitimate children were recognized, such as, "manceres" or the offspring of prostitutes and the "sacrilegious" or children of those who had received Holy Orders. Subsequently, the Civil Code, in an effort to keep in step with modern times, limited illegitimate filiation to those which are incestuous, adulterous and illicit. At the core of the institution of legitimacy held sacrosanct by Spanish tradition and culture, lies the "inviolable social institution" known as marriage. This union, absent any formal or substantial defect or of any vice of consent, is virtually adamantine. On the whole, the status of a marriage determines in large part the filiation of its resultant issue. Thus, a child born within a valid marriage is legitimate, while one born outside of wedlock is illegitimate. If, however, the latter's parents were, at the time of the child's conception, not legally barred from marrying each other and subsequently do so, the child's filiation improves as he becomes legitimized and the "legitimated" child eventually enjoys all the privileges and rights associated with legitimacy. Without such marriage, the natural child's rights depend on whether he

766

University of the Cordilleras College of Law First Year C S.Y. 2013 - 2014 is acknowledged or recognized by his parents, but he does not rise to the level of a legitimate child in the manner that the legitimated child does. A child conceived or born of a marriage which is void ab initio or one which is declared a nullity is illegitimate since there is no marriage to speak of, but it is the law which accords him the rights of an acknowledged natural child. Finally, there are illegitimate children who are referred to as "spurious" or derisively denominated as "bastards" because of their doubtful origins. There is no marriage valid or otherwise which would give any semblance of legality to the child's existence. Nothing links child to parent aside from the information appearing in the birth certificate. When such child is recognized by one or both parents, he acquires certain rights nowhere approaching those of his legitimate counterparts. The Civil Code provides three rights which, in varying degrees, are enjoyed by children, depending on their filiation: use of surname, succession, and support. Legitimate children and legitimated children are entitled to all three. Thus, they "shall principally use the surname of the father," and shall be entitled to support from their legitimate ascendants and descendants, as well as to a legitime consisting of one-half of the hereditary estate of both parents, and to other successional rights, such as the right of representation. "These rights as effects of legitimacy cannot be renounced." Natural children recognized by both parents and natural children by legal fiction shall principally use the surname of the father. If a natural child is recognized by only one parent, the child shall follow the surname of such recognizing parent. Both types of children are entitled to receive support from the parent recognizing them. They also cannot be deprived of their legitime equivalent to one-half of that pertaining to each of the legitimate children or descendants of the recognizing parent, to be taken from the free disposable portion of the latter's estate. Recognized illegitimate children other than natural, or spurious issues, are, in their minority, under the parental authority of their mothers and, naturally, take the latter's surname. The only support which they are entitled to is from the recognizing parent, and their legitime, also to be taken from the free portion, consists of four-fifths of the legitime of an acknowledged natural child or two-fifths that of each legitimate child. It must also be observed that while the legitime of a legitimate child is fairly secured by law, the legitime of any recognized illegitimate child, taken as it is from the free portion of the hereditary estate which the child shares with the surviving spouse, may be reduced if it should exceed said portion. Unrecognized illegitimate children are not entitled to any of the rights above mentioned. These distinctions gain more relevance if we were to consider that while a legitimated child may enjoy the same successional rights granted to legitimate children, a natural child by legal fiction cannot rise beyond that to which an acknowledged natural child is entitled, insofar as his hereditary rights are concerned. It is thus incongruous to conclude, as private respondent maintains, that petitioner's half siblings can rise to her level by the fact of being legitimized, for two reasons: First, they failed to meet the most important requisite of legitimation, that is, that they be natural children within the meaning of Article 269; second, natural children by legal fiction cannot demand that they be legitimized simply because it is one of the rights enjoyed by acknowledged natural children. It may be argued that legitimation is a right vouchsafed to acknowledged natural children and, therefore, by the same token, to natural children by legal fiction. This conclusion is arrived at through a syllogism as simple as it is deceptive, which runs as follows: The respondent's children are natural children by legal fiction. Therefore, they have the same status, rights and obligations as acknowledged natural children. Acknowledged natural children have the right to be legitimated.

767

University of the Cordilleras College of Law First Year C S.Y. 2013 - 2014 Ergo, respondent's children have the right to be legitimated (as in fact they were "deemed legitimated" by the subsequent valid marriage of their parents in the Philippines in 1967). The above line of reasoning follows the Euclidian geometric proposition that things equal to the same thing are equal to each other. This may hold true in the realm of instructional, as opposed to descriptive science, where the former calls for the application of absolute, mathematical rules with precision but not to the latter, particularly those which deal with the social sciences where human relationships are central to a study whose main concern is not to leave out anything of significance. The former deals with inanimate things, those which a scientist has described as the "dead aspect of nature," excluding all factors regarded as superfluous to obtaining absolute results and nothing more. It does not concern itself so much with the whole truth as with those aspects or parts only through which the inexorable result can be obtained. To apply the strict rules of syllogism, where the basic premise is defective, to the arena of paternity and filiation, especially in the determination of the status and rights of the different kinds of illegitimate children vis-a-vis the legitimate ones, is bound to spawn mischief and results never intended by the framers of the provisions of the law under review. Pursued to its logical, undeviating conclusion, it may eventually be postulated that "adulterous children shall enjoy the status, rights and obligations of legitimate children," a doctrine which no moral philosophy under our social and cultural milieu can countenance. This conclusion not only presumes that children other than those who are "natural" can be legitimized in the first place, but also grants acknowledged natural children (and, consequently, natural children by legal fiction) a "right" to be legitimized when no such right exists. Legitimation is not a "right" which is demandable by a child. It is a privilege, available only to natural children proper, as defined under Art. 269. Although natural children by legal fiction have the same rights as acknowledged natural children, it is a quantum leap in the syllogism to conclude that, therefore, they likewise have the right to be legitimated, which is not necessarily so, especially, as in this case, when the legally existing marriage between the children's father and his estranged first wife effectively barred a "subsequent marriage" between their parents. The question that must be confronted next is: How are the offspring of the second union affected by the first wife's death and the ensuing celebration of a valid marriage between her widower and his ostensible second wife? Natural children by legal fiction cannot be legitimized in this fashion. Our archaic law on family relations, patterned as it is after Spanish Civil Law, frowns upon illegal relations such that the benefits of legitimation under Chapter 3 of Title VIII do not extend, nor were they intended to extend, to natural children by legal fiction. Article 269 itself clearly limits the privilege of legitimation to natural children as defined thereunder. There was, therefore, from the outset, an intent to exclude children conceived or born out of illicit relations from the purview of the law. Another point to be considered is that although natural children can be legitimized, and natural children by legal fiction enjoy the rights of acknowledged natural children, this does not necessarily lead to the conclusion that natural children by legal fiction can likewise be legitimized. As has been pointed out, much more is involved here than the mere privilege to be legitimized. The rights of other children, like the petitioner in the case at bench, may be adversely affected as her testamentary share may well be reduced in the event that her ten surviving half siblings should be placed on par with her, when each of them is rightfully entitled to only half of her share. The provisions of law invoked by private respondent are couched in simple and unmistakable language, not at all subject to interpretation, and they all point to the correctness of petitioner's claim. If it should be asserted that we now trench on a gray area of law that calls for interpretation, or a lacuna that cries for filling up, then we have to pierce the shroud unintentionally created by the letter of the law and expose its spirit as evincing intent, in this case one which decidedly favors legitimacy over illegitimacy. The hierarchy of children so painstakingly erected by law and the corresponding gradation of their rights may conceivably be shattered by elevating natural children by legal fiction who are incontestably illegitimate children to the level of natural children proper, whose filiation would otherwise be legitimate had their parents blessed their union with a valid marriage. Finally, attention must be drawn to the fact that this case has been decided under the provisions of the Civil Code, not the Family Code which now recognizes only two classes of children: legitimate and illegitimate. "Natural children by legal fiction" are nothing if not pure fiction. 768

University of the Cordilleras College of Law First Year C S.Y. 2013 - 2014 WHEREFORE, the instant petition is hereby GRANTED. The assailed orders of the court a quo dated November 14, 1991 and January 9, 1992, are NULLIFIED and SET ASIDE. Petitioner Maria Rosario de Santos is hereby declared the SOLE LEGITIMATE CHILD of the decedent Antonio de Santos and, as such, entitled to all the rights accorded to her by law. SO ORDERED.

769

University of the Cordilleras College of Law First Year C S.Y. 2013 - 2014 Marquino vs Intermediate Appellate Court 233 SCRA 130 G.R. No. 72078 June 27, 1994 Full Case EUTIQUIO MARQUINO and MARIA TERENAL-MARQUINO Survived by: LUZ. T. MARQUINO, ANA T. MARQUINO and EVA T. MARQUINO, petitioners, vs. THE HON. INTERMEDIATE APPELLATE COURT, FIRST CIVIL CASES DIVISION, BIBIANA ROMANO-PAGADORA, Survived By: PEDRO PAGADORA, EMY R. PAGADORA, JUNE R. PAGADORA, EDGAR R. PAGADORA, MAY R. PAGADORA, MAGO R. PAGADORA, ARDEN R. PAGADORA, and MARS R. PAGADORA, respondents. PUNO, J.: For resolution are the following issues: (1) the effect of the death of the natural child during the pendency of her action for recognition; and (2) the effect of the death of the putative parent also during the pendency of the case. The facts are as follows: Respondent Bibiana Romano-Pagadora filed Civil Case No. 5197, an action for Judicial Declaration of Filiation, Annulment of Partition, Support, and Damages against petitioner Eutiquio Marquino on January 10, 1971 before the then Court of First Instance of Negros Occidental. Also impleaded as defendants, were Maria Terenal-Marquino, wife of Eutiquio Marquino, and their legitimate children Luz, Ana, and Eva, all surnamed Terenal-Marquino. The records show that Bibiana was born on December 2, 1926 at Piapi, Dumaguete City, of Gregoria Romano and allegedly of Eutiquio Marquino. 1 At that time, Eutiquio was still single. Bibiana became personally known to the Marquino family when she was hired as domestic helper in their household at Luke Wright Street, Dumaguete City. She always received financial assistance from them. Thus, she claimed that she enjoyed continuous possession of the status of an acknowledged natural child by direct and unequivocal acts of her father and his family. The Marquinos, on the other hand, strongly denied her allegations. During the pendency of the case and before respondent Bibiana could finish presenting her evidence, she died on March 17, 1979. On March 23, 1979, her heirs were ordered substituted for her as partiesplaintiffs. On May 17, 1983, petitioners filed a Motion to Dismiss. They averred that the action for recognition is intransmissible to the heirs being a personal act. 2 The trial court dismissed the case. Respondents appealed to the respondent Intermediate Appellate Court (now Court of Appeals). On August 20, 1983, Eutiquio Marquino died while the case was pending appeal. On June 17, 1985, respondent court invoking the case of Banaga vs. Pascacio, (No. 4848-R, July 31, 1954, 50 O.G. No. 12, p. 5908) reversed the controverted order. It ruled: [A]fter the death of the natural child, the heirs of said deceased natural child, cannot bring the action to compel recognition, but may however, continue the action already filed to compel recognition. xxx xxx xxx

Summarizing, We hold that the death of the putative parent while the case against him for recognition of his alleged child is pending will not extinguish the action but the same can be continued with the heirs substituted for said deceased parents because: a) the law does not require that the case be brought and decided while the putative parent is alive;

b) that would be adding another requisite for the action which is not sanctioned by the law or jurisprudence;

770

University of the Cordilleras College of Law First Year C S.Y. 2013 - 2014 c) it would be unfair to the plaintiff child to have his action for recognition depend on the speed of the Court in disposing of the case and on a fortuitous event. This is because if the court takes, let us say, 10 years to decide the case, the chances that the defendant parent would survive the case is very much less, especially if he was already of advanced age at the time the action is brought; d) there are no compelling reasons not to allow substitution of the deceased parent with his heirs, for with the death of the defendant parent, the effects of recognition will practically be limited to successional rights. WHEREFORE, finding merit in this appeal, we hereby SET ASIDE the Order of the trial Court dated August 13, 1983 and remand the case to the Court of origin for continuation of the trial by the heirs of plaintiff against the heirs of defendant Eutiquio Marquino, without pronouncement as to costs. SO ORDERED. 3 The Motion for Reconsideration was denied on May 19, 1985. Hence, this petition for review on certiorari. Petitioners hold respondent court to be in error, in these respects: I IN RULING THAT AFTER THE DEATH OF THE NATURAL CHILD, THE HEIRS OF SAID DECEASED NATURAL CHILD, CANNOT BRING THE ACTION TO COMPEL RECOGNITION, BUT THEY MAY HOWEVER, CONTINUE THE ACTION ALREADY FILED TO COMPEL RECOGNITION. II IN RULING THAT THE DEATH OF THE PUTATIVE PARENT WHILE THE CASE AGAINST HIM FOR RECOGNITION OF HIS ALLEGED CHILD IS PENDING WILL NOT EXTINGUISH THE ACTION BUT THE SAME CAN BE CONTINUED WITH THE HEIRS SUBSTITUTED FOR SAID DECEASED PARENT. 4 The Court writes finis to this controversy after twenty-three (23) years of protracted litigation. The first issue to be resolved is whether or not the right of action to compel recognition is intransmissible in character. Article 285 of the Civil Code provides that an action for recognition of natural children may be brought only during the lifetime of the presumed parents, except in the following cases: (1) If the father or mother died during the minority of the child, in which case the latter may file the action before the expiration of four years from the attainment of his majority; (2) If after the death of the father or of the mother a document should appear of which nothing had been heard and in which either or both parents recognize the child. In this case, the action must be commenced within four years from discovery of the document. The rationale for the rule is to give the alleged parents opportunity to be heard. The reason for the exceptions is to protect the heirs. 5 In Conde vs. Abaya, 6 we held that the right of action for the acknowledgment of natural children to which Article 285 (Article 137, Old Civil code) refers, can never be transmitted. The reason is that the code makes no mention of it in any case, not even as an exception. 7 In the case at bench, it is evident that Bibiana was a natural child. She was born out of wedlock on December 2, 1926, of Gregoria Romano and allegedly of Eutiquio Marquino who at that time was single. Bibiana sued for compulsory recognition while Eutiquio was still alive. Sadly, she died on March 17, 1983 before she could present her proof of recognition. Her death tolled the action considering its personal nature and intransmissibility. As explained in the case of Conde vs. Abaya, 8 viz.: 771

University of the Cordilleras College of Law First Year C S.Y. 2013 - 2014 It is most illogical and contrary to every rule of correct interpretation that the right of action to secure acknowledgment by the natural child should be presumed to be transmitted, independently, as a rule to his heirs, while the right to claim legitimacy from his predecessor is not expressly, independently, or, as a general rule conceded to the heirs of the legitimate child, but only relatively and as an exception. Consequently, the pretension that the right of action on the part of the child to obtain the acknowledgment of his natural filiation is transmitted to his descendants, is altogether unfounded. No legal provision exists to sustain such pretension, nor can an argument of presumption be based on the lesser claim when there is no basis for the greater one, and when it is only given as an exception in well-defined cases. It is placing the heirs of the natural child on a better footing than the heirs of the legitimate one, when, as a child is not better than, nor even equal to, that of a legitimate child. This ruling was reiterated in the recent case of Heirs of Raymundo C. Banas vs. Heirs of Bibiano Banas 9 thus: Granting that, after the death of Bibiano Banas Raymundo could file an action for compulsory recognition against Bibiano's heirs, still plaintiffs-appellants cannot invoke Raymundo's right to file such action, because it is not transmissible to the natural child's heirs; the right is purely a personal one to the natural child. The second issue for resolution is whether or not after the death of the putative father the action for recognition of a natural child can be continued against the heirs of the former. We rule against its continuance. In an action for compulsory recognition, the party in the best position to oppose the same is the putative parent himself. 10 The need to hear the side of the putative parent is an overwhelming consideration because of the unsettling effects of such an action on the peace and harmonious relationship in the family of the putative parent. For this reason, Article 285 provides only two (2) exceptions when an action for recognition transcends the death of the putative parent. Neither of these exceptions obtains in the case at bench. Firstly, the death of Eutiquio did not occur during the minority of Bibiana. In fact, she was already forty-five (45) years old when the recognition case was filed on January 10, 1971. Secondly, no document was discovered, before unknown, in which Bibiana was expressly acknowledged as a natural child. Consequently, the respondent court erred in ruling that the action can still be continued against the heirs of Eutiquio. 11 Our public policy at that time supports the rule limiting actions for recognition during the lifetime of the presumed parents, to quote: Public policy, indeed public necessity, demands that before an illegitimate child be admitted into a legitimate family, every requisite of the law must be completely and fully complied with. No one should ever be permitted upon doubtful evidence to take from legitimate children the property which they and their parents have, by industry, fidelity, and frugality, acquired. To do so would in many instances where the legitimate children had "labored unsparingly in order that they might have the comforts of life and joys of home," be manifestly contrary to every plainest principles of justice. And again, if this can ever be done upon oral testimony alone, after the lips of the alleged father and mother have been closed by death, such testimony must be clear, strong, and convincing. 12 Our law providing for the intransmissibility of an action for recognition, however, has been superseded by the New Family Code which took effect on August 3, 1988. Under Article 173 of the Family Code, it is now provided: The action to claim legitimacy may be brought by the child during his or her lifetime and shall be transmitted to the heirs should the child die during minority or in a state of insanity. In these cases, the heirs shall have a period of five (5) years within which to institute the action. The action commenced by the child shall survive notwithstanding the death of either or both of the parties. (Emphasis supplied) Pursuant to this provision, the child can bring the action during his or her entire lifetime (not during the lifetime of the parents) and even after the death of the parents. In other words, the action does not prescribe as long as he lives. 13

772

University of the Cordilleras College of Law First Year C S.Y. 2013 - 2014 Be that as it may, Article 173 of the Family Code cannot be given retroactive effect so as to apply to the case at bench because it will prejudice the vested rights of petitioners transmitted to them at the time of the death of their father, Eutiquio Marquino. "Vested right" is a right in property which has become fixed and established and is no longer open to doubt or controversy. 14 It expresses the concept of present fixed interest, which in right reason and natural justice should be protected against arbitrary State action. 15 WHEREFORE, the decision of the Court of Appeals dated June 17, 1985 is REVERSED and SET ASIDE. The Complaint in Civil Case No. 5197 of the then Court of First Instance of Negros Occidental is DISMISSED. No costs. SO ORDERED.

773

University of the Cordilleras College of Law First Year C S.Y. 2013 - 2014 Case Digest: Marquino vs Intermediate Appellate Court 233 SCRA 130 G.R. No. 72078 June 27, 1994 PUNO, J.: FACTS: Respondent Bibiana Romano-Pagadora filed Civil Case No. 5197, an action for Judicial Declaration of Filiation, Annulment of Partition, Support, and Damages against petitioner Eutiquio Marquino on January 10, 1971 before the then Court of First Instance of Negros Occidental. Also impleaded as defendants, were Maria Terenal-Marquino, wife of Eutiquio Marquino, and their legitimate children Luz, Ana, and Eva, all surnamed Terenal-Marquino. The records show that Bibiana was born on December 2, 1926 at Piapi, Dumaguete City, of Gregoria Romano and allegedly of Eutiquio Marquino. 1 At that time, Eutiquio was still single. Bibiana became personally known to the Marquino family when she was hired as domestic helper in their household at Luke Wright Street, Dumaguete City. She always received financial assistance from them. Thus, she claimed that she enjoyed continuous possession of the status of an acknowledged natural child by direct and unequivocal acts of her father and his family. The Marquinos, on the other hand, strongly denied her allegations. Issue: During the pendency of the case and before respondent Bibiana could finish presenting her evidence, she died on March 17, 1979. On March 23, 1979, her heirs were ordered substituted for her as parties-plaintiffs. On May 17, 1983, petitioners filed a Motion to Dismiss. They averred that the action for recognition is intransmissible to the heirs being a personal act. 2 The trial court dismissed the case. Summarizing, We hold that the death of the putative parent while the case against him for recognition of his alleged child is pending will not extinguish the action but the same can be continued with the heirs substituted for said deceased parents because: a) the law does not require that the case be brought and decided while the putative parent is alive; b) that would be adding another requisite for the action which is not sanctioned by the law or jurisprudence; c) it would be unfair to the plaintiff child to have his action for recognition depend on the speed of the Court in disposing of the case and on a fortuitous event. This is because if the court takes, let us say, 10 years to decide the case, the chances that the defendant parent would survive the case is very much less, especially if he was already of advanced age at the time the action is brought; d) there are no compelling reasons not to allow substitution of the deceased parent with his heirs, for with the death of the defendant parent, the effects of recognition will practically be limited to successional rights. Ruling: Pursuant to this provision, the child can bring the action during his or her entire lifetime (not during the lifetime of the parents) and even after the death of the parents. In other words, the action does not prescribe as long as he lives. Be that as it may, Article 173 of the Family Code cannot be given retroactive effect so as to apply to the case at bench because it will prejudice the vested rights of petitioners transmitted to them at the time of the death of their father, Eutiquio Marquino. "Vested right" is a right in property which has become fixed and established and is no longer open to doubt or controversy. It expresses the concept of present fixed interest, which in right reason and natural justice should be protected against arbitrary State action.

774

University of the Cordilleras College of Law First Year C S.Y. 2013 - 2014 Rodriguez vs Court of Appeals 245 SCRA 150 G.R. No. 85723 June 19, 1995 Full Case BIENVENIDO RODRIGUEZ, petitioner, vs. COURT OF APPEALS and CLARITO AGBULOS, respondents. QUIASON, J.: This is a petition for review on certiorari under Rule 45 of the Revised Rules of Court of the Decision of the Court of Appeals dated November 2, 1988 in CA-G.R. SP No. 14276, which allowed, in an action for compulsory recognition, the testimony of the mother of a natural child on the identity of the putative father. I On October 15, 1986, an action for compulsory recognition and support was brought before the Regional Trial Court, Branch 9, Baguio-Benguet, by respondent Alarito (Clarito) Agbulos against Bienvenido Rodriguez, petitioner herein. At the trial, the plaintiff presented his mother, Felicitas Agbulos Haber, as first witness. In the course of her direct examination, she was asked by counsel to reveal the identity of the plaintiff's father but the defendant's counsel raised a timely objection which the court sustained. The plaintiff filed before this Court a petition for review on certiorari questioning the said order in UDK 8516 entitled Clarito Agbulos v. Hon. Romeo A. Brawner and Bienvenido Rodriguez." On March 18, 1988, this Court referred the petition to the Court of Appeals (CA-G.R. SP No. 14276), which promulgated the questioned Decision dated November 2, 1988. II In the instant petition for review on certiorari, petitioner alleged that the Court of Appeals erred: (1) in not dismissing the petition for certiorari on the ground that the order of the trial court disallowing the testimony of Felicitas Agbulos Haber was interlocutory and could not be reviewed separately from the judgment; and (2) in reversing the said order and allowing the admission of said testimony. As a rule, errors of judgment or of procedure, not relating to the court's jurisdiction nor involving grave abuse of discretion, are not reviewable by certiorari under Rule 65 of the Revised Rules of Court (Villalon v. Intermediate Appellate Court, 144 SCRA 443 [1986]). However, there are exceptions to said rule. For instance, certiorari is justified in order to prevent irreparable damages and injury to a party, where the trial judge capriciously and whimsically exercised his judgment, or where there may be danger of failure of justice. Certiorari may also be availed of where an appeal would be slow, inadequate and insufficient (Presco v. Court of Appeals, 192 SCRA 232 [1990]; Saludes v. Pajarillo, 78 Phil. 754 [1947]). We find that had the appellate court sanctioned the trial court's disallowance of the testimony of plaintiff's mother, private respondent would have been deprived of a speedy and adequate remedy considering the importance of said testimony and the erroneous resolution of the trial court. On the merits of his petition, petitioner contended that Felicitas Agbulos Haber should not be allowed to reveal the name of the father of private respondent because such revelation was prohibited by Article 280 of the Civil Code of the Philippines. Said Article provided: When the father or the mother makes the recognition separately, he or she shall not reveal he name of the person with whom he or she had the child; neither shall he or she state any circumstance whereby the other party may be identified. On the other hand, private respondent argued that his mother should be allowed to testify on the identity of his father, pursuant to paragraph 4, Article 283 of the Civil Code of the Philippines and Section 30, Rule 130 of the Revised Rules of Court. Article 283 of the Civil Code of the Philippines provided:

775

University of the Cordilleras College of Law First Year C S.Y. 2013 - 2014 In any of the following cases, the father is obliged to recognize the child as his natural child: (1) In cases of rape, abduction or seduction, when the period of the offense coincides more or less with that of the conception; (2) When the child is in continuous possession of status of a child of the alleged father by the direct acts of the latter or of his family; (3) When the child was conceived during the time when the mother cohabited with the supposed father; (4) When the child has in his favor any evidence or proof that the defendant is his father.

Section 30, Rule 130 of the Revised Rules of Court provides: Testimony generally confined to personal knowledge; hearsay excluded. A witness can testify only to those facts which he knows of his own knowledge, that is, which are derived from his own perception, except as otherwise provided in these rules. Private respondent cannot invoke our decision in Navarro v. Bacalla, 15 SCRA 114 (1965). While we ruled in Navarro that the testimony of the mother of the plaintiff in said case, could be used to establish his paternity, such testimony was admitted during the trial without objection and the defendant accepted the finding of the trial court that he was the father of the plaintiff. In the case at bench, petitioner timely objected to the calling of the mother of private respondent to the witness stand to name petitioner as the father of said respondent. Likewise, in Navarro we clearly stated: We are not ruling whether the mere testimony of the mother, without more, is sufficient to prove the paternity of the child. Neither are we ruling on the scope of Art. 280, New Civil Code which enjoins the mother in making a separate and voluntary recognition of a child from revealing the name of the father, specifically, as to whether the mother's testimony identifying the father is admissible in an action to compel recognition if and when a timely objection to such oral evidence is interposed (at p. 117). Navarro, therefore, is not the end but only the beginning of our quest, which felicitously was reached with our conclusion that the prohibition in Article 280 against the identification of the father or mother of a child applied only in voluntary and not in compulsory recognition. This conclusion becomes abundantly clear if we consider the relative position of the progenitor of Article 280, which was Article 132 of the Spanish Civil Code of 1889, with the other provisions on the acknowledgement of natural children of the same Code. Article 132 was found in Section I (Acknowledgment of Natural Children), Chapter IV (Illegitimate Children), Title V (Paternity and Filiation), Book First (Persons) of the Spanish Civil Code of 1889. The first article in said Section provided: Art. 129 A natural child may be acknowledged by the father and mother jointly or by either of them alone. The next article provided: Art. 130 In case the acknowledgment is made by only one of the parents, it shall be presumed that the child is a natural one if the parent acknowledging it was, at the time of the conception, legally competent to contract marriage. The article immediately preceding Article 132 provided: Art. 131 The acknowledgment of a natural child must be made in the record of birth, in a will, or in some other public document. Article 132 of the Spanish Civil Code provided: 776

University of the Cordilleras College of Law First Year C S.Y. 2013 - 2014 When the acknowledgment is made separately by the father or the mother, the name of the child's other parent shall not be revealed by the parent acknowledging it, nor shall any circumstance be mentioned by which such person might be recognized. No public officer shall authenticate any document drawn in violation of this provision and should he do so notwithstanding this prohibition shall be liable to a fine of from 125 to 500 pesetas, and the words containing such revelation shall be striken out. Article 280 of the Civil Code of the Philippines was found in Section 1 (Recognition of Natural Children), Chapter 4 (Illegitimate Children), Title VIII (Paternity and Filiation) of said Code. The whole section was repealed by the Family Code. The first article of this section was Article 276 which was a reproduction of Article 129 of the Spanish Civil Code. The second article was Article 277 which was a reproduction of Article 130 of the Spanish Civil Code. The third article was Article 278 which was a reproduction of Article 131 of the Spanish Civil Code. However, unlike in the Spanish Civil Code, wherein the progenitor of Article 280 followed immediately the progenitor of Article 278, a new provision was inserted to separate Article 280 from Article 278. The new provision, Article 279, provided: A minor who may contract marriage without parental consent cannot acknowledge a natural child, unless the parent or guardian approves the acknowledgment, or unless the recognition is made in the will. If the sequencing of the provisions in the Spanish Civil Code were maintained in the Civil Code of the Philippines, and Article 280 was numbered Article 279, it becomes clear that the prohibition against the identification by the parent acknowledging a child of the latter's other parent refers to the voluntary recognition provided for in Article 278. Senator Arturo M. Tolentino is of the view that the prohibition in Article 280 does not apply in an action for compulsory recognition. According to him: The prohibition to reveal the name or circumstance of the parent who does not intervene in the separate recognition is limited only to the very act of making such separate recognition. It does not extend to any other act or to cases allowed by law. Thus, when a recognition has been made by one parent, the name of the other parent may be revealed in an action by the child to compel such other parent to recognize him also (I Commentaries and Jurisprudence on the Civil Code of the Philippines 590 [1985]). Justice Eduardo Caguioa also opines that the said prohibition refers merely to the act of recognition. "It does prevent inquiry into the identity of the other party in case an action is brought in court to contest recognition on the ground that the child is not really natural because the other parent had no legal capacity to contract marriage" (I Comments and Cases on Civil Law 380 [1967] citing In re Estate of Enriquez, 29 Phil. 167 [1915]). We have not lost sight of our decision in Infante v. Fiqueras, 4 Phil. 738 (1905), where we rejected the testimony of the mother of a child that the defendant was the father of the plaintiff. The action for recognition in that case was brought under Article 135 of the Spanish Civil Code, which limited actions to compel recognition to cases when an indubitable writing existed wherein the father expressly acknowledged his paternity and when the child was in the uninterrupted possession of the status of a natural child of the defendant father justified by the conduct of the father himself or that of his family. The action filed by private respondent herein was brought under Article 283 of the Civil Code of the Philippines, which added new grounds for filing an action for recognition: namely, xxx xxx xxx

3) When the child was conceived during the time when the mother cohabited with the supposed father; 4) When the child has in his favor any evidence or proof that the defendant is his father.

777

University of the Cordilleras College of Law First Year C S.Y. 2013 - 2014 Likewise, the testimony of the mother of the plaintiff in Infante was not admissible under the procedural law then in force, which was the Law of Bases of May 11, 1888. Said law in pertinent part provided: No se admitira la investigation de la paternidad si no en los casos de delito o cuando existe escrito del padre en el que conste su voluntad indubitada de reconnocer per suyo al hijo, deliberadamente expresada con ese fin, o cuando medie posesion de estado. Se permitira la investigacion de la maternidad. Traditionally, there was a free inquiry into the paternity of children allowed by French royal decrees but the investigation of paternity was forbidden by the French Revolutionary Government in order to repress scandal and blackmail. This prohibition passed to the French Civil Code and from it to the Spanish Civil Code of 1889 (I Reyes and Puno, An Outline of Philippine Civil Code 266 [4th ed.]). Worth noting is the fact that no similar prohibition found in Article 280 of the Civil Code of the Philippines has been replicated in the present Family Code. This undoubtedly discloses the intention of the legislative authority to uphold the Code Commission's stand to liberalize the rule on the investigation of the paternity of illegitimate children. Articles 276, 277, 278, 279 and 280 of the Civil Code of the Philippines were repealed by the Family Code, which now allows the establishment of illegitimate filiation in the same way and on the same evidence as legitimate children (Art. 175). Under Article 172 of the Family Code, filiation of legitimate children is by any of the following: The filiation of legitimate children is established by any of the following: (1) The record of birth appearing in the civil register or a final judgment; or

(2) An admission of legitimate filiation in a public document or a private handwritten instrument and signed by the parent concerned. In the absence of the foregoing evidence, the legitimate filiation shall be proved by: (1) (2) The open and continuous possession of the status of a legitimate child; or Any other means allowed by the Rules of Court and special laws. (265a, 266a, 267a)

Of interest is that Article 172 of the Family Code adopts the rule in Article 283 of the Civil Code of the Philippines, that filiation may be proven by "any evidence or proof that the defendant is his father." WHEREFORE, the Decision of the Court of Appeals is AFFIRMED. The trial court is DIRECTED to PROCEED with dispatch in the disposition of the action for compulsory recognition. SO ORDERED.

778

University of the Cordilleras College of Law First Year C S.Y. 2013 - 2014 Case Digest: Rodriguez vs Court of Appeals 245 SCRA 150 G.R. No. 85723 June 19, 1995 QUIASON, J.: Facts: This is a petition for review on certiorari under Rule 45 of the Revised Rules of Court of the Decision of the Court of Appeals dated November 2, 1988 in CA-G.R. SP No. 14276, which allowed, in an action for compulsory recognition, the testimony of the mother of a natural child on the identity of the putative father. On October 15, 1986, an action for compulsory recognition and support was brought before the Regional Trial Court, Branch 9, Baguio-Benguet, by respondent Alarito (Clarito) Agbulos against Bienvenido Rodriguez, petitioner herein. At the trial, the plaintiff presented his mother, Felicitas Agbulos Haber, as first witness. In the course of her direct examination, she was asked by counsel to reveal the identity of the plaintiff's father but the defendant's counsel raised a timely objection which the court sustained. Issue: In the instant petition for review on certiorari, petitioner alleged that the Court of Appeals erred: (1) in not dismissing the petition for certiorari on the ground that the order of the trial court disallowing the testimony of Felicitas Agbulos Haber was interlocutory and could not be reviewed separately from the judgment; and (2) in reversing the said order and allowing the admission of said testimony. Ruling: Errors of judgment or of procedure, not relating to the court's jurisdiction nor involving grave abuse of discretion, are not reviewable by certiorari under Rule 65 of the Revised Rules of Court (Villalon v. Intermediate Appellate Court, 144 SCRA 443 [1986]). However, there are exceptions to said rule. For instance, certiorari is justified in order to prevent irreparable damages and injury to a party, where the trial judge capriciously and whimsically exercised his judgment, or where there may be danger of failure of justice. Certiorari may also be availed of where an appeal would be slow, inadequate and insufficient (Presco v. Court of Appeals, 192 SCRA 232 [1990]; Saludes v. Pajarillo, 78 Phil. 754 [1947]).

779

University of the Cordilleras College of Law First Year C S.Y. 2013 - 2014 De Santos vs Angeles 251 SCRA 206 G.R. No. 105619 December 12, 1995 Full Case MARIA ROSARIO DE SANTOS, petitioner, vs. HON. ADORACION G. ANGELES, JUDGE, REGIONAL TRIAL COURT OF CALOOCAN CITY, BRANCH 121 and CONCHITA TALAG DE SANTOS, respondents. ROMERO, J.: Can natural children by legal fiction be legitimized? There being no explicit provision of law in point, the Court is called upon to cast illumination in a gray area even as it fills up unintentional interstices in the fabric of Civil Law with overlays of philosophical, historical and sociological strands. For an understanding of how the issue arose, we now proceed to unravel the pertinent factual background. On February 7, 1941, Dr. Antonio de Santos married Sofia Bona, which union was blessed with a daughter, herein petitioner Maria Rosario de Santos. After some time, their relationship became strained to the breaking point. Thereafter, Antonio fell in love with a fellow doctor, Conchita Talag, private respondent herein. Antonio sought a formal dissolution of his first marriage by obtaining a divorce decree from a Nevada court in 1949. Obviously aware that said decree was a worthless scrap of paper in our jurisdiction which then, as now, did not recognize divorces, Antonio proceeded to Tokyo, Japan in 1951 to marry private respondent, with whom he had been cohabiting since his de facto separation from Sofia. This union produced eleven children. On March 30, 1967, Sofia died in Guatemala. Less than a month later, on April 23, 1967, Antonio and private respondent contracted a marriage in Tagaytay City celebrated under Philippine laws. On March 8, 1981, Antonio died intestate leaving properties with an estimated value of P15,000,000.00. On May 15, 1981, private respondent went to court 1 asking for the issuance of letters of administration in her favor in connection with the settlement of her late husband's estate. She alleged, among other things, that the decedent was survived by twelve legitimate heirs, namely, herself, their ten surviving children, and petitioner. There being no opposition, her petition was granted. After six years of protracted intestate proceedings, however, petitioner decided to intervene. Thus, in a motion she filed sometime in November 1987, she argued inter alia that private respondent's children were illegitimate. This was challenged by private respondent although the latter admitted during the hearing that all her children were born prior to Sofia's death in 1967. On November 14, 1991, after approval of private respondent's account of her administration, the court a quo passed upon petitioner's motion. The court, citing the case of Francisco H. Tongoy, et al. v. Court of Appeals, et al. (23 SCRA 99 [1983]), declared private respondent's ten children legitimated and thereupon instituted and declared them, along with petitioner and private respondent, as the heirs of Antonio de Santos. Petitioner sought a reconsideration of said order but this was denied in the court's order dated January 9, 1992. Hence, she filed the instant petition for certiorari on June 16, 1992, contending that since only natural children can be legitimized, the trial court mistakenly declared as legitimated her half brothers and sisters. This argument is tenable. Article 269 of the Civil Code expressly states: Art. 269. Only natural children can be legitimated. Children born outside wedlock of parents who, at the time of the conception of the former, were not disqualified by any impediment to marry each other, are natural.

780

University of the Cordilleras College of Law First Year C S.Y. 2013 - 2014 In other words, a child's parents should not have been disqualified to marry each other at the time of conception for him to qualify as a "natural child." In the case at bench, there is no question that all the children born to private respondent and deceased Antonio de Santos were conceived and born when the latter's valid marriage to petitioner's mother was still subsisting. That private respondent and the decedent were married abroad after the latter obtained in Nevada, U.S.A. a decree of divorce from his legitimate wife does not change this fact, for a divorce granted abroad was not recognized in this jurisdiction at the time. Evidently, the decedent was aware of this fact, which is why he had to have the marriage solemnized in Tokyo, outside of the Philippines. It may be added here that he was likewise aware of the nullity of the Tokyo marriage for after his legitimate, though estranged wife died, he hastily contracted another marriage with private respondent, this time here in Tagaytay. It must be noted that while Article 269, which falls under the general heading of "Paternity and Filiation," specifically deals with "Legitimated Children," Article 89, a provision subsumed under the general title on "Marriage," deals principally with void and voidable marriages and secondarily, on the effects of said marriages on their offspring. It creates another category of illegitimate children, those who are "conceived or born of marriages which are void from the beginning," but because there has been a semblance of marriage, they are classified as "acknowledged natural children" and, accordingly, enjoy the same status, rights and obligations as such kind of children. In the case at bench, the marriage under question is considered "void from the beginning" because bigamous, contracted when a prior valid marriage was still subsisting. It follows that the children begotten of such union cannot be considered natural children proper for at the time of their conception, their parents were disqualified from marrying each other due to the impediment of a prior subsisting marriage. What term should then be coined to distinguish them from natural children proper (those "born outside of wedlock of parents who, at the time of the conception of the former, were not disqualified by any impediment to marry each other")? A legal fiction had to be resorted to, that device contrived by law to simulate a fact or condition which, strictly and technically speaking, is not what it purports to be. In this case, the term "natural children by legal fiction" was invented, thus giving rise to another category of illegitimate children, clearly not to be confused with "natural children" as defined under Art. 269 but by fiction of law to be equated with acknowledged natural children and, consequently, enjoying the status, rights and obligations of the latter. Does this cluster of rights include the right to be legitimated? Under the Civil Code, there exists a hierarchy of children classified on the basis of rights granted by law, which must be preserved by strictly construing the substantive provisions of the law in force. Under the prevailing Civil Code (which may be considered "old" in light of the new provisions of the Family Code on "Persons"), much emphasis is laid on the classification of children vis-a-vis their parents, and the corresponding rights they are entitled to under the law. Thus, the title on "Paternity and Filiation" devotes two whole chapters to legitimate children alone, and one chapter on those deemed by law to be possessed of the rights of the former, such as legitimated children, because of their compliance with certain requisites laid down by law; two other chapters deal with illegitimate children composed of recognized natural children, and those other than natural, or spurious, whether recognized or not. The well-ordered delineation of such distinctions among these groups demonstrates a clear intent on the part of the framers of the Civil Code to compartmentalize and separate one from the other, for legitimacy/illegitimacy determines the substantive rights accruing to the different categories of children. It must be noted that before said Code was enacted, other classes of illegitimate children were recognized, such as, "manceres" or the offspring of prostitutes and the "sacrilegious" or children of those who had received Holy Orders. Subsequently, the Civil Code, in an effort to keep in step with modern times, limited illegitimate filiation to those which are incestuous, adulterous and illicit. At the core of the institution of legitimacy held sacrosanct by Spanish tradition and culture, lies the "inviolable social institution" known as marriage. This union, absent any formal or substantial defect or of any vice of consent, is virtually adamantine. On the whole, the status of a marriage determines in large part the filiation of its resultant issue. Thus, a child born within a valid marriage is legitimate, while one born outside of wedlock is illegitimate. If, however, the latter's parents were, at the time of the child's conception, not legally barred from marrying each other and subsequently do so, the child's filiation improves as he becomes legitimized and the "legitimated" child eventually enjoys all the privileges and rights associated with legitimacy. Without such marriage, the natural child's rights depend on whether he is acknowledged or recognized by his parents, but he does not rise to the level of a legitimate child in the 781

University of the Cordilleras College of Law First Year C S.Y. 2013 - 2014 manner that the legitimated child does. A child conceived or born of a marriage which is void ab initio or one which is declared a nullity is illegitimate since there is no marriage to speak of, but it is the law which accords him the rights of an acknowledged natural child. Finally, there are illegitimate children who are referred to as "spurious" or derisively denominated as "bastards" because of their doubtful origins. There is no marriage valid or otherwise which would give any semblance of legality to the child's existence. Nothing links child to parent aside from the information appearing in the birth certificate. When such child is recognized by one or both parents, he acquires certain rights nowhere approaching those of his legitimate counterparts. The Civil Code provides three rights which, in varying degrees, are enjoyed by children, depending on their filiation: use of surname, succession, and support. Legitimate children and legitimated children are entitled to all three. 2 Thus, they "shall principally use the surname of the father," 3 and shall be entitled to support from their legitimate ascendants and descendants, 4 as well as to a legitime consisting of one-half of the hereditary estate of both parents, 5 and to other successional rights, such as the right of representation. "These rights as effects of legitimacy cannot be renounced." 6 Natural children recognized by both parents and natural children by legal fiction shall principally use the surname of the father. 7 If a natural child is recognized by only one parent, the child shall follow the surname of such recognizing parent. 8 Both types of children are entitled to receive support from the parent recognizing them. 9 They also cannot be deprived of their legitime equivalent to one-half of that pertaining to each of the legitimate children or descendants of the recognizing parent, to be taken from the free disposable portion of the latter's estate. 10 Recognized illegitimate children other than natural, or spurious issues, are, in their minority, under the parental authority of their mothers and, naturally, take the latter's surname. 11 The only support which they are entitled to is from the recognizing parent, 12 and their legitime, also to be taken from the free portion, consists of four-fifths of the legitime of an acknowledged natural child or two-fifths that of each legitimate child. 13 It must also be observed that while the legitime of a legitimate child is fairly secured by law, 14 the legitime of any recognized illegitimate child, taken as it is from the free portion of the hereditary estate which the child shares with the surviving spouse, may be reduced if it should exceed said portion. 15 Unrecognized illegitimate children are not entitled to any of the rights above mentioned. 16 These distinctions gain more relevance if we were to consider that while a legitimated child may enjoy the same successional rights granted to legitimate children, a natural child by legal fiction cannot rise beyond that to which an acknowledged natural child is entitled, insofar as his hereditary rights are concerned. It is thus incongruous to conclude, as private respondent maintains, that petitioner's half siblings can rise to her level by the fact of being legitimized, for two reasons: First, they failed to meet the most important requisite of legitimation, that is, that they be natural children within the meaning of Article 269; second, natural children by legal fiction cannot demand that they be legitimized simply because it is one of the rights enjoyed by acknowledged natural children. It may be argued that legitimation is a right vouchsafed to acknowledged natural children and, therefore, by the same token, to natural children by legal fiction. This conclusion is arrived at through a syllogism as simple as it is deceptive, which runs as follows: The respondent's children are natural children by legal fiction. Therefore, they have the same status, rights and obligations as acknowledged natural children. Acknowledged natural children have the right to be legitimated. Ergo, respondent's children have the right to be legitimated (as in fact they were "deemed legitimated" by 782

University of the Cordilleras College of Law First Year C S.Y. 2013 - 2014 the subsequent valid marriage of their parents in the Philippines in 1967). The above line of reasoning follows the Euclidian geometric proposition that things equal to the same thing are equal to each other. This may hold true in the realm of instructional, as opposed to descriptive science, where the former calls for the application of absolute, mathematical rules with precision but not to the latter, particularly those which deal with the social sciences where human relationships are central to a study whose main concern is not to leave out anything of significance. The former deals with inanimate things, those which a scientist has described as the "dead aspect of nature," excluding all factors regarded as superfluous to obtaining absolute results and nothing more. It does not concern itself so much with the whole truth as with those aspects or parts only through which the inexorable result can be obtained. To apply the strict rules of syllogism, where the basic premise is defective, to the arena of paternity and filiation, especially in the determination of the status and rights of the different kinds of illegitimate children vis-a-vis the legitimate ones, is bound to spawn mischief and results never intended by the framers of the provisions of the law under review. Pursued to its logical, undeviating conclusion, it may eventually be postulated that "adulterous children shall enjoy the status, rights and obligations of legitimate children," a doctrine which no moral philosophy under our social and cultural milieu can countenance. This conclusion not only presumes that children other than those who are "natural" can be legitimized in the first place, but also grants acknowledged natural children (and, consequently, natural children by legal fiction) a "right" to be legitimized when no such right exists. Legitimation is not a "right" which is demandable by a child. It is a privilege, available only to natural children proper, as defined under Art. 269. Although natural children by legal fiction have the same rights as acknowledged natural children, it is a quantum leap in the syllogism to conclude that, therefore, they likewise have the right to be legitimated, which is not necessarily so, especially, as in this case, when the legally existing marriage between the children's father and his estranged first wife effectively barred a "subsequent marriage" between their parents. The question that must be confronted next is: How are the offspring of the second union affected by the first wife's death and the ensuing celebration of a valid marriage between her widower and his ostensible second wife? Natural children by legal fiction cannot be legitimized in this fashion. Our archaic law on family relations, patterned as it is after Spanish Civil Law, frowns upon illegal relations such that the benefits of legitimation under Chapter 3 of Title VIII do not extend, nor were they intended to extend, to natural children by legal fiction. Article 269 itself clearly limits the privilege of legitimation to natural children as defined thereunder. There was, therefore, from the outset, an intent to exclude children conceived or born out of illicit relations from the purview of the law. Another point to be considered is that although natural children can be legitimized, and natural children by legal fiction enjoy the rights of acknowledged natural children, this does not necessarily lead to the conclusion that natural children by legal fiction can likewise be legitimized. As has been pointed out, much more is involved here than the mere privilege to be legitimized. The rights of other children, like the petitioner in the case at bench, may be adversely affected as her testamentary share may well be reduced in the event that her ten surviving half siblings should be placed on par with her, when each of them is rightfully entitled to only half of her share. The provisions of law invoked by private respondent are couched in simple and unmistakable language, not at all subject to interpretation, and they all point to the correctness of petitioner's claim. If it should be asserted that we now trench on a gray area of law that calls for interpretation, or a lacuna that cries for filling up, then we have to pierce the shroud unintentionally created by the letter of the law and expose its spirit as evincing intent, in this case one which decidedly favors legitimacy over illegitimacy. The hierarchy of children so painstakingly erected by law and the corresponding gradation of their rights may conceivably be shattered by elevating natural children by legal fiction who are incontestably illegitimate children to the level of natural children proper, whose filiation would otherwise be legitimate had their parents blessed their union with a valid marriage. Finally, attention must be drawn to the fact that this case has been decided under the provisions of the Civil Code, not the Family Code which now recognizes only two classes of children: legitimate and illegitimate. "Natural children by legal fiction" are nothing if not pure fiction.

783

University of the Cordilleras College of Law First Year C S.Y. 2013 - 2014 WHEREFORE, the instant petition is hereby GRANTED. The assailed orders of the court a quo dated November 14, 1991 and January 9, 1992, are NULLIFIED and SET ASIDE. Petitioner Maria Rosario de Santos is hereby declared the SOLE LEGITIMATE CHILD of the decedent Antonio de Santos and, as such, entitled to all the rights accorded to her by law. SO ORDERED. Feliciano, Regalado, Davide, Jr., Melo, Puno, Vitug and Mendoza, JJ., concur. Separate Opinions HERMOSISIMA, JR., J., concurring: Do children born out of adulterous relationships have the right to be legitimated under the New Civil Code? This I believe is the resultant issue in this case. In declaring what the law is on this matter, we could not be so unmindful of the highest regard that our society places on the institution of marriage and the maintenance of which in its purity the public is deeply interested, for it is the foundation of the family and of society, Without it there could be neither civilization nor progress. 1 No less than the Constitution, of which we should be the staunchest vanguard as we are its ablest defender, marshals us to protect marriage as an inviolable social institution and the foundation of the family, 2 for it cannot be denied that the welfare of society is served and nurtured by a court that exercises its judicial prerogatives not in a vacuum of cold logic but in the context of the loftiest and most enduring social values which the citizens, albeit struggling and fumbling in their daily living, try to approximate in their own lives. The citizens, after all, are our constituents; 3 and so their best interests, embodied in the scale of values which they extol, are an integral part of the great flux that is the law. As we are concerned with its exposition, we must strive to continuously refurbish the image of the law vis-avis the welfare of society, to keep it bright, and to subject it to constant re-analysis so as to keep it in touch with what has always been right, what is just and fair under present circumstances, and what is most beneficial for the future generations. 4 It is in this light that we appreciate this case with the following antecedent facts: Dr. Antonio de Santos married Sofia Bona on February 7, 1941. Out of this union was born in 1942 petitioner Maria Rosario de Santos. However, Antonio and Sofia subsequently parted ways. While separated de facto from Sofia, Antonio, in 1949, secured a divorce decree against her in Nevada, U.S.A. He then married private respondent Dr. Conchita Talag in Tokyo, Japan, in 1951. Antonio and Conchita had eleven children who were all born between the years 1951 to 1967. On March 30, 1967, Sofia died in Guatemala. Thereafter, Antonio married private respondent, for the second time, in Tagaytay City. Antonio then died on March 8, 1981 at the Capitol Medical Center. Is special proceedings filed by private respondent on May 15, 1981, before the Regional Trial Court of Caloocan City, the court granted her petition for letters of administration since such petition was unopposed. In the course of the proceedings, however, petitioner intervened alleging, among others, that the ten surviving children of private respondent were illegitimate. After the approval of the Income and Expenses Statement of the decedent's estate pursuant to Sec. 1, Rule 90 of the Revised Rules of Court on May 6, 1991, the trial court issued an order on November 14, 1991 declaring that the ten children of the deceased and private respondent must be deemed legitimated and therefore entitled to inherit as legitimate heirs. Consequently, the sole issue raised in the instant petition for certiorari is whether or not said children can be legitimated. A logical cold deduction based on some pertinent laws would appear to answer this issue in the affirmative, in this wise: Article 80 of the New Civil Code considers as marriages void from the beginning, bigamous marriages not falling under Art. 83 (2). Article 89 of the same Code, in turn, bestows upon children conceived or born of marriages void from the beginning, referred to as natural children by legal fiction, the status, rights and obligations of acknowledged natural children. Among the rights of acknowledged natural 784

University of the Cordilleras College of Law First Year C S.Y. 2013 - 2014 children is the right of legitimation granted to them under Article 269 in relation to 271 of the same Code. Since private respondent's children were all born after her marriage to the deceased in Tokyo in 1951, which marriage is considered bigamous, hence, void from the beginning, because of its celebration while the marriage between the deceased and his first wife, Sofia Bona, still subsisted, said children are natural children by legal fiction who have the rights of acknowledged natural children, including the right to be legitimated, and they may now be considered legitimated since the deceased and private respondent validly married for the second time after Sofia's demise. It happens that the law may lose its character of being a law by an excess of caprice in its administration, but it could hardly cease to be law because of its rigid logical application according to its tenor. When its rigid logical application, however, amounts to absurdity, the law not only becomes incapable of just administration but may also become an instrument of legal injustice. Clearly for us to read the law in the aforegoing manner is to dangerously teeter on the fulcrum of legal folly for there is no scaling down its unacceptable implications. If children born out of an extramarital relationship, but whose parents contracted a bigamous marriage and still another marriage subsequent thereto upon the death of the first spouse of the adulterous parent, may eventually be legitimated, then children of adulterous spouses, by the expedient contrivance of a bigamous marriage, may later on be legitimated. The adulterous spouse may still prove himself virtuous and heroic by risking prosecution for bigamy if only to give his child out of wedlock the chance, that slim chance, to be legitimated, that is, if he prays enough that his first spouse dies ahead of him so he could eventually validly marry his paramour. As such, in the mildest terms, the law would seem to condone extramarital relationships by providing the seemingly confessant adulterer a way to be a conscientious parent to his illegitimate children without having to give up his illicit relations with their mother. At its worst, such a reading of the law amounts to a mockery of the institution of marriage, which is, under our Constitution and family laws, an inviolable social institution imbued with public interest and traditionally and constantly held to be a priority in our culture's scale of values, for nothing stops the public from concluding that marriage and a bigamous marriage at that (with its accompanying criminal consequences), is actually a backdoor to legitimating adulterous children. The letter of Article 89 of the New Civil Code must be transcended and the absurd and sheepishly grotesque consequences of its application in the instant case, rejected. It is not enough that the law exists to be administered justly; in addition and more importantly, the law needs to possess a just content. The law must by itself aim at and endeavor to conform to, some criteria of rightness which repose on values espoused by the very society it seeks to serve. As it is our duty to declare the law as it is, there is no escaping the task of revealing the justness of the law in accordance with society's avowed values. Consequently, it has been called a golden rule of statutory interpretation that unreasonableness of the result produced by one possible interpretation of a statute is reason for rejecting that interpretation in favor of another which would produce a reasonable result. 5 In resolving the issue at hand, I believe the emphasis should be on Article 269 which is, after all, the law squarely in point under the premises of this case. Taking the letter of Article 269 as it is, it clearly prescribes the limits of its applicability upon only natural children. Surely there is no canon against using both common sense and common weal in construing the law as saying what it obviously means: Chapter 3 Art. 269. Only natural children can be ligitimated. Children born outside wedlock of parents who, at the time of the conception of the former, were not disqualified by any impediment to marry each other, are natural. xxx xxx xxx

Art. 271. Only natural children who have been recognized by the parents before or after the celebration of the marriage, or have been declared natural children by final judgment, may be considered legitimated by subsequent marriage. (Emphasis ours) Legitimation is a right granted by law only to natural children who, because their parents could have legally married at the time they were conceived, cannot be substantially differentiated from legitimate children once their parents do marry after their birth. This is because said parents can marry any time, there being no legal impediment preventing them from validly contracting marriage. The situation obtaining respecting legitimate children and legitimated natural children is certainly distinct from that 785

University of the Cordilleras College of Law First Year C S.Y. 2013 - 2014 respecting adulterous children because the parents of adulterous children are admittedly incapacitated to marry each other at the time said children were conceived. It may easily be said, thus, that to interpret the law as allowing adulterous children to be put on equal footing with the legitimate children, would be putting a premium on adulterous relationships, which is frowned upon by the society itself. Even the law on succession under the New Civil Code distinguishes the respective hereditary rights of the different kinds of children and significantly assigns a diminishing share in accordance with the degree of illegitimacy of the child concerned. Thus, Article 895 provides that the legitime of each of the acknowledged natural children and each of the natural children by legal fiction shall consist of one-half of the legitime of each of the legitimate children or descendants and that of illegitimate children who is neither of the above, four-fifths of the legitime of an acknowledged natural child. It is, therefore, evident that the treatment accorded children under the New Civil Code is determined by the circumstances under which they have been conceived and born, particularly, the capacity to marry of their parents at the time that they were conceived. Private respondent's children were precisely born when their deceased father was still legally married to Sofia Bona. The marriage of the deceased and private respondent in Tokyo, Japan, in 1951 could not have given a semblance of legitimacy to their subsequent cohabitation and their issues since such marriage was contracted during the subsistence of the deceased's marriage with Sofia Bona. The relationship between the deceased and private respondent, therefore, was no less adulterous notwithstanding an attempt to legitimize the same through a bigamous marriage. There is no other way to put it but that the deceased and private respondent were having illicit relations; they were fully aware of the legal and moral consequences of their actions, and they seemingly, in bad faith, attempted to circumvent the law in their favor by contracting a bigamous marriage to the prejudice of the legitimate issue in the person of the petitioner. There is no quibbling that private respondent's children are adulterous children whose status, by the simple expedient of a bigamous marriage contracted by parties fully aware of their incapacity to marry, could never have been intended by the law to be equated to that of petitioner who is the legitimate child of the deceased in view of the public policy involved in preserving the sanctity of marriage and preventing the proliferation of illegitimate issues. As the earlier interpretation has been shown to lead to unreasonable results with socially virulent implications, and the same originates from two provisions, namely, Article 89 and Article 269 of the New Civil Code, we are wont to state that they are irreconcilable provisions. And the applicable statutory rule is that where there is an irreconcilable conflict between the different provisions of a statute, the provision last in order of position will prevail, since it is the latest expression of the legislative will. 6 More than that Article 269 is the latest expression of the legislative will, however, Article 269 on its face specifically states the law on legitimation, limits its applicability to natural children, and is resonantly silent on the right of adulterous children to be legitimated in the same way as children born to parents who, at the time of their conception, were legally capable to marry each other. All told, the law tenders to us in no unpretentious terms the basis to rule that private respondent's children, being adulterous children, have no right to be legitimated under the New Civil Code. Such a ruling is not only in accord with the explicit, unequivocal language of Article 269 but more importantly animates and upholds the public policy as regards the institution of marriage as the foundation of society. Needless to say, such ruling sits well with the need to obviate any legal injustice and social absurdity that may result if we were to rule otherwise. The final cause of law is the welfare of society. The rule that misses its aim cannot permanently justify its existence. "Ethical considerations can no more be excluded from the administration of justice which is the end and purpose of all civil laws than one can exclude the vital air from his room and live. 7 The final rendering of the meaning of a statute is an act of judgment. 8 This court has so judged this case at bench, and so we will perhaps be judged thereby. I, therefore, vote to grant the petition, set aside the assailed order of the Regional Trial Court, and remand thereto the case for further proceedings. VITUG, J., dissenting: I vote to resolve the controversy in favor of the child. I take it to be the legislative intent that the pertinent provisions of the Civil Code on children in the book on persons and family relations are meant to enhance the child's interest and welfare. This intent finds exemplification in Article 89 of the Civil Code by explicitly providing that natural children by legal fiction (among them those conceived or born of void 786

University of the Cordilleras College of Law First Year C S.Y. 2013 - 2014 marriages because the parents suffer from an impediment to marry) shall have the same status, rights and obligations as acknowledged natural children. If then under Article 269, in relation to Article 270, of the Civil Code, acknowledged natural children are given the right to be legitimated by the subsequent marriage of the parents, the law must, by virtue of Article 89 aforesaid, likewise extend unqualifiedly to natural children by legal fiction. No matter how well legal calisthenics are played, there is, I must point out, not a single provision of the Code that limits or circumscribes the scope and application of Article 89. The law, I respectfully submit, should be so construed as to attain congruity, rather than a division, among its several provisions. The rule is expressed in the maxim interpretare et concordare legibus est optimus interpretendi upon the theory that the legislature is presumed not to have enacted conflicting provisions of law but that, on the contrary, it must have meant to give them such parity and consequence as a uniform jurisprudential system. Most regrettably, I still perceive coolness, if not outright hostility, towards illegitimate children who have not been fortunate enough to be conceived or born under a better family circumstance. It is not enough that they are unjustly ostracized by a segment in society; they are also called names bastards, outcasts, adulterous, spurious that certainly they do not deserve. If at all, their situation needs sympathy, not hatred or condemnation. Any conflict of view, however, would soon be a thing of the past, for as so keenly observed by Mme. Justice Flerida Ruth P. Romero, the Family Code, which became effective on 03 August 1988, has deleted any reference to natural children by legal fiction. The Family Code presently categorizes children of void marriages into two kinds the legitimates which include those conceived or born of void marriages under Article 36 and Article 52 of the Family Code before the judicial declaration of nullity of such void marriages and the illegitimates or children conceived or born of all other void marriages (but evidently maintaining, for legitimation purposes, the distinction between those whose parents, at the time of conception, were not disqualified to marry and those whose parents were disqualified). Narvasa, C.J. and Bellosillo, J., concurs. KAPUNAN, J., dissenting: The principal issue in the case at bench may be capsulized as to whether or not the trial court committed grave abuse of discretion amounting to a lack or excess of jurisdiction in considering the private respondent's children legitimated under the facts established herein, and in declaring and instituting said children as heirs of the decedent. As the law unequivocally gives them such a right, I respectfully dissent from the majority. I begin by observing that, taking their cue from the lower court's inappropriate lifting of an editor's precis or statement from the syllabus of the case of Tongoy vs. Court of Appeals, 1 both parties in the case at bench have placed too much emphasis and reliance on the case of Tongoy, 2 the facts and circumstances of which are not exactly on all fours with those obtaining in the case at bench. The italicized portion of the syllabus of cases appearing in official or unofficial reports of Supreme Court Decisions or Resolutions 3 generally reflect the editor's summary of a discussion of an issue or a specific point in a case, and, taken out of context, could be misleading and inappropriate for citation. Judges should strive to read cases which might have a bearing on cases before them in their entirety, and quote or obtain their citations from the body of the decision, not the syllabus. The principal issue in Tongoy, 4 hinged "on the absence of an acknowledgment (by the father prior to his death of his illegitimate children) through any of the modes recognized by the Old Civil Code." 5 It is not, however clear from the Court's discussion of the facts of the case, whether the illegitimate children were sired during the subsistence of the first marriage or after the death of the first wife. On the sale issue of the father's acknowledgment, the Court therein took a liberal view, recognizing the fact that the children "were in continuous possession of the status of natural, or even legitimated, children" 6 and that they were "treated as legitimate children not only by their parents but also by the entire clan," 7 in declaring, on equitable grounds, that the children therein were legitimate heirs. For better appreciation of our ruling in Tongoy, let me quote the following: The remaining assignment of error dwells on the question of whether or not respondents Amado, Ricardo, Cresenciano and Norberto, all surnamed Tongoy, may be considered legitimated by virtue of the marriage of their parents, Francisco Tongoy and Antonina Pabello, subsequent to their births and shortly before 787

University of the Cordilleras College of Law First Year C S.Y. 2013 - 2014 Francisco died on September 15, 1926. Petitioners maintain that since the said respondents were never acknowledged by their father, they could not have been legitimated by the subsequent marriage of their parents, much less could they inherit from the estate of their father, the predecessor-in-interest of Luis D. Tongoy, who is admittedly the half brother of the said respondents. Both the trial court and the respondent appellate court have found overwhelming evidence to sustain the following conclusions: that Amado P. Tongoy, Ricardo P. Tongoy, Cresenciano P. Tongoy and Norberto P. Tongoy were born illegitimate to Antonina Pabello on August 19, 1910 (Exh. A), August 12, 1922 (Exh. B), December 1, 1915 (Exhs. C and C-1) and August 4, 1922 (Exh. D), respectively; that Francisco Tongoy was their father; that said Francisco Tongoy had before them and Antonina Pabello two legitimate children by his first wife, namely, Luis D. Tongoy and Patricio D. Tongoy; that Francisco Tongoy and Antonina Pabello were married sometime before his death on September 15, 1926 (Exh. H); that shortly thereafter, Luis D. Tongoy and Patricio D. Tongoy executed an Extra-Judicial Declaration of Heirs, leaving out their half-brothers Amado, Ricardo, Cresenciano, and Norberto, who were then still minors; that respondents Amado, Ricardo, Cresenciano and Norberto were known and accepted by the whole clan as children of Francisco; that they had lived in Hacienda Pulo with their parents, but when they went to school, they stayed in the old family home at Washington Street, Bacolod, together with their grandmother, Agatona Tongoy; that everybody in Bacolod knew them to be part of the Tongoy-Sonora clan; and that Luis D. Tongoy as administrator of Hacienda Pulo, also spent for the education of Ricardo Tongoy until he became a lawyer; and that even petitioners admit the fact that they were half-brothers of the late Luis D. Tongoy. The bone of contention, however, hinges on the absence of an acknowledgment through any of the modes recognized by the Old Civil Code (please see Articles 131 and 135 of the Old Civil Code), such that legitimation could not have taken place in view of the provisions of Art. 121 of the same Code which states that "children shall be considered legitimated by a subsequent marriage only when they have been acknowledged by the parents before or after the celebration thereof." Of course, the overwhelming evidence found by respondent Court of Appeals conclusively shows that respondents Amado, Ricardo, Cresenciano and Norberto have been in continuous possession of the status of natural, or even legitimated children. Still, it recognizes the fact that such continuous possession of status is not, per se, a sufficient acknowledgment but only a ground to compel recognition (Alabat vs. Alabat, 21 SCRA 1379; Pua vs. Chan, 21 SCRA 753; Larena vs. Rubio, 43 Phil. 1017). Be that as it may, WE cannot but agree with the liberal view taken by respondent Court of Appeals when it said: . . . It does not seem equally manifest, however, that defendants-appellants stand on a purely technical point in the light of overwhelming evidence that appellees were natural children of Francisco Tongoy and Antonina Pabello, and were treated as legitimate children not only by their parents but only by the entire clan. Indeed, it does not make much sense that appellees should be deprived of their hereditary rights as undoubted natural children of their father, when the only plausible reason that the latter could have had in mind when he married his second wife Antonina Pabello just over a month before his death was to give legitimate status to their children. It is not in keeping with the more liberal attitude taken by the New Civil Code towards illegitimate children and the more compassionate trend of the New Society to insist on a very literal application of the law in requiring the formalities of compulsory acknowledgment, when the only result is to unjustly deprive children who are otherwise entitled to hereditary rights. From the very nature of things, it is hardly to be expected of appellees, having been reared as legitimate children by their parents and treated as such by everybody, to bring an action to compel their parents to acknowledge them. In the hitherto cited case of Ramos vs. Ramos, supra, the Supreme Court showed the way out of patent injustice and inequity that might result in some cases simply because of the implacable insistence on the technical amenities for acknowledgment. Thus, it held Unacknowledged natural children have no rights whatsoever (Buenaventura vs. Urbano, 5 Phil. 1; Siguiong vs. Siguiong, 8 Phil. 5, 11; Infante vs. Figueras, 4 Phil. 738; Crisolo vs. Macadaeg, 94 Phil. 862). The fact that the plaintiffs, as natural children of Martin Ramos, received shares in his estate implied that they were acknowledged. Obviously, defendants Agustin Ramos and Granada Ramos and the late Jose Ramos and members of his family had treated them as his children. Presumably, that fact was well-known in the community. Under the circumstances, Agustin Ramos and Granada Ramos and the heirs of Jose Ramos, are estopped from attacking plaintiffs' status as acknowledged natural children (See Arts. 283 [4] and 2666 [3], New Civil Code). [Ramos vs. Ramos, supra].

788

University of the Cordilleras College of Law First Year C S.Y. 2013 - 2014 With the same logic, estoppel should also operate in this case in favor of appellees, considering, as already explained in detail, that they have always been treated as acknowledged and legitimated children of the second marriage of Francisco Tongoy, not only by their presumed parents who raised them as their children, but also by the entire Tongoy-Sonora clan, including Luis D. Tongoy himself who had furnished sustenance to the clan in his capacity as administrator of Hacienda Pulo and had in fact supported the law studies of appellee Ricardo P. Tongoy in Manila, the same way he did with Jesus T. Sonora in his medical studies. As already pointed out, even defendants-appellants have not questioned the fact that appellees are half-brothers of Luis D. Tongoy. As a matter of fact, they are really children of Francisco Tongoy and Antonina Pabello, and only the technicality that their acknowledgment as natural children has not been formalized in any of the modes prescribed by law appears to stand in the way of granting them their hereditary rights. But estoppel, as already indicated, precludes defendants-appellants from attacking appellees' status as acknowledged natural or legitimated children of Francisco Tongoy. In addition to estoppel, this is decidedly one instance when technicality should give way to conscience, equity and justice (cf. Vda. de Sta. Ana vs. Rivera, L-22070, October 29, 1966, 18 SCRA 588) [pp. 196-198, Vol. 1, rec.]. It is time that WE, too, take a liberal view in favor of natural children who, because they enjoy the blessings and privileges of an acknowledged natural child and even of a legitimated child, found if rather awkward, if not unnecessary, to institute an action for recognition against their natural parents, who, without their asking, have been showering them with the same love, care and material support as are accorded to legitimate children. The right to participate in their father's inheritance should necessarily follow. 8 However, acknowledgment is clearly not at issue here. Petitioner makes no pretense that private respondent's children are not entitled to hereditary rights. 9 She herself admits that the decedent acknowledged his paternity of the private respondent's children and that they are indeed her brothers and sisters. 10 What herein petitioner claims she opposes "is their being judicially declared legitimated (by the respondent court) so as to entitle them to enjoy the same rights as a legitimate heir," 11 to her prejudice. Citing Article 269 of the New Civil Code as "the law in point" in the case at bench, she contends that the trial court erred in declaring her half brothers and sisters legitimated because under the New Civil Code only natural children could be legitimated. I find this contention, to which the majority of this divided Court agrees, absolutely untenable. The New Civil Code appears to limit the right to legitimation only to those children conceived by parents not disqualified by any impediment to marry each other, bestowing upon them, prior to such legitimation, the status of natural children. Article 269 which provides the cornerstone for the majority's holding today states that: Art. 269. Only natural children can be legitimated. Children born outside wedlock of parents who, at the time of the conception of the former, were not disqualified by any impediment to marry each other, are natural. The rule is, however, not absolute because even children conceived or born out of marriages void from the very beginning under the Civil Code possess the status of natural children by legal fiction and enjoy the same rights as acknowledged natural children. Article 89 provides: Art. 89. Children conceived or born out of marriages which are void from the beginning shall have the same status, rights and obligations as acknowledged natural children, and are called natural children by legal fiction. Children conceived of voidable marriages before the decree of annulment shall be considered legitimate; and children conceived thereafter shall have the same status, rights and obligations as acknowledged natural children, and are also called natural children by legal fiction. Article 89, a creature of legislation (through the Code Commission) which has remained unmolested since 1950 I must stress, is not an accidental provision. The Civil Code Commission clearly intended Article 89, notwithstanding its location in the Code, as a piece of reform, an exception to the rule furnished by Article 269. More importantly, Article 89 (unlike Article 269 which came from the Spanish Civil Code of 1889) was a new provision deliberately introduced by the Code Commission as one of its revolutionary reforms thirty five years ago. And doubt about the intention of this piece of legislation should have been laid to rest by the following explanation from the Code Commission's Report:

789

University of the Cordilleras College of Law First Year C S.Y. 2013 - 2014 This proposed reform is based on the fact that such children have been brought into this world through no fault of their own, but through that of their parents. To visit punishment upon them is most unjust. Moreover, though the marriage is void, or voidable, at least there was a semblance of legality to the relationship between the parents. This circumstance should cast a mantle of protection over the children, who by legal fiction should be treated as acknowledged natural children. 12 Since the decedent's 1951 marriage in Tokyo with the private respondent was invalid, 13 being one of those marriages classified as void from the very beginning under the Civil Code, 14 the status of her children clearly falls under Article 89 which puts them on par, at least in terms of rights and obligations, with acknowledged natural children. Since the rights of acknowledged natural children include the right of legitimation under Article 270 of the Civil Code by the subsequent valid marriage of their parents, 15 it therefore plainly follows that by virtue of Article 89, in relation to Article 270, the private respondent's children were deemed legitimated by the subsequent valid marriage of their parents in the Philippines in 1967. This position is hardly an isolated one. Virtually all Civil and Family Code commentators are united in the belief that Article 89 furnishes an escape valve for children found under the circumstances existing in the case at bench. Dr. Arturo Tolentino, in his commentary on the Civil Code, for example, writes: Under Article 89, natural children by legal fiction "shall have the same status, rights and obligations as acknowledged natural children." Theoretically therefore, natural children by legal fiction can be legitimated. . . . . The following children by legal fiction . . . can be legitimated: . . . (2) those born of a bigamous marriage, for the parents can marry each other again upon the widowhood of the parent who married twice. . . . . 16 In the same token, Prof. Ernesto L. Pineda, a member of the Family Code Revision Committee acknowledges this exception the rule, stating that: By way of exception, some natural children by legal fiction (Art. 89, NCC) can be legitimated such as (a) those born of couples who married while below the allowable marrying age but who contracted a new marriage after reaching the proper age; (b) those born of bigamous marriages but where the parents married each other upon the widowhood of the disqualified parent; (c) those born of parents who got married before an unauthorized officer and the parents contracted a new marriage before an authorized one; (d) those born of parents who got married without a marriage license (where license was required) and the parents contracted a subsequent valid marriage; and (e) children conceived after (not before) the decree of annulment of a voidable marriage. 17 Justice Alicia V. Sempio-Diy, writing on the New Family Code 18 underscores the difference in treatment of the subject of legitimation between the Family Code and the Civil Code thus: Under the Civil Code, children of bigamous marriages, who are natural children by legal fiction, can be legitimated, since the parents can marry each other upon the death of the first husband or wife of the parent who married twice. Unfortunately for such children, they can no longer be legitimated under the Family Code, which has limited the kind of children to legitimate and illegitimate and abolished the category of natural children by legal fiction. 19 "Parenthetically," another commentator on the Family Code, Prof. Melencio Sta. Maria writes, "under the Civil Code provisions of legitimation which were repealed by the Family Code," there can be an instances where such children could be legitimated. 20 Elaborating on these provisions in his 1995 commentary; he states: This is so because according to the repealed Article 271 of the Civil Code only acknowledged natural children can be legitimated, and also according to the repealed Article 89 of the Civil Code, a child born inside a void marriage was considered a natural child by legal fiction with all the rights of an acknowledged natural child. Since a natural child by legal fiction has all the rights of an acknowledged natural child and the statutory right to be legitimated was one of the rights of an acknowledged natural child, the subject child therefore can be legitimated if the parents subsequently validly remarried. 21 Clearly, the weight of authority in this country recognizes that under the Civil Code, Article 89 unequivocally furnishes an exception to the rule that only acknowledged natural children or those who by law have been declared natural children by final judgment can be legitimated. This exception was, in fact, 790

University of the Cordilleras College of Law First Year C S.Y. 2013 - 2014 acknowledged by the Family Code Revision Committee in its Meeting of August 24, 1985, when it decided not to accord the same privilege extended by Article 89 to similarly situated illegitimate children (under the family Code's simplified classification) in the provisions of the new code. However, for children born under the Civil Code, the exception is a legal fact which could not be ignored. If under Article 269, in relation to Article 270 of the Civil Code, acknowledged natural children are given the right to be legitimated by the subsequent marriage of the parents, the law must, by virtue of Article 89, also extend unqualifiedly to natural children by legal fiction. This not only harmonizes Article 89 with the Civil Code articles on the rights of acknowledged natural children and the articles on legitimation but also leads to a result which enhances the welfare and interest of the child. As Justice Vitug in his 1993 Compendium of Civil Law and Jurisprudence writes: The provisions of Art. 269 and 271 of the Civil Code, in a literal sense appear to limit legitimation in favor of acknowledged natural children or those who by law have been declared natural children by final judgment. Considering, however, that natural children by legal fiction (such as those born of void marriages because the parents suffer from an impediment to marry) are expressly given the same status, rights and obligations as acknowledged natural children (Art. 89 Civil Code), and because all doubts should be resolved in favor of the child, it is submitted that the rules on legitimation should likewise extend to such children. 22 Indeed, it hardly makes sense that the children of private respondent should be deprived of their full hereditary rights as legitimated children when the facts and circumstances of the case at bench clearly show the decedent's intention to remove, once and for all, all manner of legal and moral obstacles to his second and apparently blissful union with the private respondent. For immediately after the death of his first wife in Guatemala in 1967, the decedent wasted no time in obtaining a Philippine marriage in Tagaytay with his second wife. With a fairly considerable estate, it was not entirely remote that the decedent had in mind not only the intention to legitimatize his union with the private respondent but also the intention to accord legitimate status to his children with his second wife. Given the nature of their relationship and the clear intendment of the Civil Code under Article 89 to place natural children by legal fiction on equal standing with acknowledged natural children, a patent injustice and inequity will result if we uphold herein petitioner's implacable position. Given the clear intendment of the legislature in enacting the new provision (Article 89) over thirty years ago when many of the members of this Court were still law students, the majority's holding in the case at bench amounts to a belated judicial veto of a valid piece of legislation. I vote to DENY the instant petition. Narvasa, C.J., Padilla, Bellosillo, and Francisco JJ., concur. PANGANIBAN, J., dissenting: With all due respect, I dissent from the well-written ponencia of Mme. Justice Flerida Ruth P. Romero. The pertinent portions of Arts. 89, 269, 270 and 271 of the New Civil Code which are the codal provisions in point, read as follows: Art. 89. Children conceived or born of marriages which are void from the beginning shall have the same status, rights and obligations as acknowledged natural children, and are called natural children by legal fiction. xxx xxx xxx

Art. 269. Only natural children can be legitimated. Children born outside wedlock of parents who, at the time of the conception of the former, were not disqualified by any impediment to marry each other, are natural. Art. 270. Legitimation shall take place by the subsequent marriage of the parents.

Art. 271. Only natural children who have been recognized by the parents before or after the celebration of the marriage, or have been declared natural children by final judgment, may be considered legitimated by subsequent marriage. . . . Art. 89 has been repealed by the Family Code (Executive Order No. 209) which took effect on August 3, 791

University of the Cordilleras College of Law First Year C S.Y. 2013 - 2014 1988 (Uyguangco vs. Court of Appeals, 178 SCRA 684 [1989]; Atienza vs. Brillantes, A.M. No. MTJ-92706, March 29, 1995). It was one of the provisions under Title III, Book I of the New Civil Code which have been omitted from the text of the present Family Code. But it was the law in force at the time the legitimation in the case at bench took place and should, consequently, govern the present controversy. Art. 89 was a new provision in the sense that, unlike Articles 269 to 271, aforequoted, which all came from the Spanish Civil Code of 1889, Art. 89 was one of the reforms instituted by the Code Commission that drafted the New Civil Code. The Code Commission justified this new article in this wise: This proposed reform is based on the fact that such children have been brought into the world through no fault of their own, but through that of their parents. To visit punishment upon them is most unjust. Moreover, though the marriage was void, or voidable, at least there was a semblance of legality of the relationship between the parents. This circumstance should cast the mantle of protection over the children, who by legal fiction should be treated as acknowledged natural children. (Report of the Code Commission, at p. 81.) In conferring upon natural children by legal fiction the same status, rights and obligations of acknowledged natural children, the clear intention of the law was to put them at par with the latter although in fact they are not. They are not in fact natural because they were conceived in the presence, not absence, of an impediment between the parents. They are natural only by figment of law. Thus, the name natural children by legal fiction. But this legal fiction precisely operates to exempt them from the requirement under Art. 269 that there be no impediment between the parents at the time of the conception as well as from the requirement of recognition by both parents under Art. 271. Plainly, this is the conclusion that can rationally be given to the express, unequivocal declaration in Art. 89 that natural children by legal fiction "shall have the same status, rights and obligations as acknowledged natural children" neither imposing any condition nor subjecting the grant of status to any qualification or exception of any kind. Had the intention been to deprive them of the right of legitimation, the law would have said so. Or it would have inserted a condition that they could be legitimated only if they can show compliance with Arts. 269 and 271 of the Code. The fact that these insertions were not made can only mean that the law intended to exempt this special class of natural children from the strict requirements normally imposed on ordinary natural children. Under the provisions of the New Civil Code, legitimation takes place when three requisites are met: (a) that the child be a natural child; (b) that he be recognized by both parents either before or after a valid marriage; and (c) that there be a subsequent valid marriage of the parents (cf. Paras, Civil Code of the Philippines Annotated, 1984 Ed., Vol. I, p. 651). A natural child by legal fiction possesses the first two requisites from inception by virtue of Art. 89, which places him on the same plane as an acknowledged natural child. In that sense, he has an advantage over a natural child as defined by Art. 269, for the latter would still need to be recognized by both parents in order to have the status and rights of an acknowledged natural child. Thus, for the purpose of legitimation, the natural child by legal fiction needs to fulfill only the third requisite: a valid subsequent marriage between his parents (cf. Paras, op. cit., p. 651; Tolentino, Civil Code of the Philippines, 1987 Reprinting, Vol. I, p. 570). Where the impediment is permanent or perpetual, such as incest or the fact that one or both of the parties have been found guilty of killing the spouse of one of them, no legitimation can ever take place as no valid marriage can ever be made between the parents (Tolentino, op. cit., p. 570). But the bigamous character of a marriage is terminable by, among other causes, the death of the first spouse, making a subsequent marriage valid. And that simply was what happened in the case at bench. Prior to the repeal of Art. 89 by the Family Code, it was suggested by some civil law scholars that a distinction should be made between natural children by legal fiction who were conceived during the existence of an impediment, on the one hand, and those who were conceived after the disappearance of such impediment, on the other. Their theory was that only the latter would qualify for legitimation. Such a stance would have been juridically sound were it not for the fact that Art. 89 does not classify natural children by legal fiction into the two suggested categories based on the presence or absence of impediment at the time of conception of the child. Ubi lex non distinguit nec nos distinguere debemus; where the law does not distinguish, we should not (Gesolgon vs. Lacson, 2 SCRA 553, 556 [1961]; Libudan vs. Gil, 45 SCRA 17, 33 [1972]; Guevarra vs. Inocentes, 16 SCRA 379, 385 [1966]; Robles vs. Chromite Mining Co., 104 Phil. 688, 690 [1958]). Besides, as already pointed out, the conferment on natural children by legal fiction of the same status, rights and obligations as those of acknowledged natural children under Art. 89, New Civil Code, evidently exempted the former from the requirements 792

University of the Cordilleras College of Law First Year C S.Y. 2013 - 2014 imposed upon ordinary natural children by Arts. 269 and 271 of said Code. And, finally, Art. 220 of said Code fortifies this conclusion, because said provision declares that in case of doubt, every intendment of law or fact leans toward, among other things, "the legitimacy of children". The doubt if there be such at all should therefore be resolved in favor of sustaining the right to legitimation of the eleven (11) offsprings of the decedent with private respondent Conchita Talag, regardless of the presence or absence of an impediment to marry on the part of their parents at the time of the conception of each of them. Indeed, it is hardly fair to stigmatize and create social and successional prejudice against children who had no fault in nor control over the marital impediments which bedeviled their parents. They are the victims, not the perpetrators, of these vagaries of life. Why then should they suffer their consequences? In the final analysis, there are really no illegitimate children; there are only illegitimate parents. And this dissent finds its philosophy in this: that children, unarguably born and reared innocent in this world, should benefit by every intendment of the law, particularly where as in this case their parents, who originally suffered from a marital impediment, would now want to overcome the improvident social and successional consequences of such condition. Therefore, it is most unfair that the these innocent children should be condemned to continue suffering the consequences of the impediment they did not cause, when the very impediment itself has disappeared. The mere fact that such legitimation would impact adversely upon the petitioner's successional rights as the lone legitimate child of the first marriage is no reason to deny the children of the second marriage of their own legal right to be deemed legitimated. Precisely, legitimation produces such an effect i.e., diminution of successional rights of the legitimate children. Art. 272 of the New Civil Code provides in fact that "(c)hildren who are legitimated by subsequent marriage shall enjoy the same rights as legitimate children." When the legislature decided to grant to children of void carriages the same status, rights, and obligations as those of acknowledged natural children, it is presumed to have carefully weighed precisely these consequences upon the rights of the other children in the family. The policy then was to cast a mantle of protection upon children of void marriages. That policy is evidently enforced by enabling them to get legitimated in the same manner as acknowledged natural children namely, by the subsequent valid marriage of their parents. If the Family Code, by repealing Art. 89 of the New Civil Code, is to be viewed as having reversed or denigrated that policy (although, by and large, it appears to have maintained the policy in many other areas of family law), such reversal or denigration should not, and cannot, in any case impair rights already acquired by and thus vested in the private respondents. One last point. Both petitioner and private respondent admit that the eleven (1l) children of the decedent with private respondent Conchita Talag were born after the celebration of the bigamous marriage on July 25, 1951. (See Petition, item 6, page 5; Rollo p. 6.) If any one of them was born prior thereto, such child, not being a natural child by legal fiction but spurious, cannot claim the special benefit granted under Art. 89 of the New Civil Code. Unlike his brothers and sisters who are natural children by legal fiction, he can only inherit by showing that he has been recognized by the decedent as the latter's illegitimate child either voluntarily or by final judgment in a proper paternity suit (Paterno vs. Paterno, 20 SCRA 585; Noble vs. Noble, 18 SCRA 1104; Clemena vs. Clemena, 24 SCRA 720; Divinagracia vs. Rovira, 72 SCRA 307; Tolentino, The Civil Code of the Philippines, 1987 Reprinting, Vol. I, pp. 616-617.) Separate Opinions HERMOSISIMA, JR., J., concurring: Do children born out of adulterous relationships have the right to be legitimated under the New Civil Code? This I believe is the resultant issue in this case. In declaring what the law is on this matter, we could not be so unmindful of the highest regard that our society places on the institution of marriage and the maintenance of which in its purity the public is deeply interested, for it is the foundation of the family and of society, Without it there could be neither civilization nor progress. 1 No less than the Constitution, of which we should be the staunchest vanguard as we are its ablest defender, marshals us to protect marriage as an inviolable social institution and the foundation of the family, 2 for it cannot be denied that the welfare of society is served and nurtured by a court that exercises its judicial prerogatives not in a vacuum of cold logic but in the context of the loftiest and most enduring social values which the citizens, albeit struggling and fumbling in their daily living, try to approximate in their own lives. The citizens, after all, are our constituents; 3 and so their best interests, embodied in the scale of values which they extol, are an integral part of the great flux that is the law. As we are concerned with its exposition, we must strive to continuously refurbish the image of the law vis-avis the welfare of society, to keep it bright, and to subject it to constant re-analysis so as to keep it in 793

University of the Cordilleras College of Law First Year C S.Y. 2013 - 2014 touch with what has always been right, what is just and fair under present circumstances, and what is most beneficial for the future generations. 4 It is in this light that we appreciate this case with the following antecedent facts: Dr. Antonio de Santos married Sofia Bona on February 7, 1941. Out of this union was born in 1942 petitioner Maria Rosario de Santos. However, Antonio and Sofia subsequently parted ways. While separated de facto from Sofia, Antonio, in 1949, secured a divorce decree against her in Nevada, U.S.A. He then married private respondent Dr. Conchita Talag in Tokyo, Japan, in 1951. Antonio and Conchita had eleven children who were all born between the years 1951 to 1967. On March 30, 1967, Sofia died in Guatemala. Thereafter, Antonio married private respondent, for the second time, in Tagaytay City. Antonio then died on March 8, 1981 at the Capitol Medical Center. Is special proceedings filed by private respondent on May 15, 1981, before the Regional Trial Court of Caloocan City, the court granted her petition for letters of administration since such petition was unopposed. In the course of the proceedings, however, petitioner intervened alleging, among others, that the ten surviving children of private respondent were illegitimate. After the approval of the Income and Expenses Statement of the decedent's estate pursuant to Sec. 1, Rule 90 of the Revised Rules of Court on May 6, 1991, the trial court issued an order on November 14, 1991 declaring that the ten children of the deceased and private respondent must be deemed legitimated and therefore entitled to inherit as legitimate heirs. Consequently, the sole issue raised in the instant petition for certiorari is whether or not said children can be legitimated. A logical cold deduction based on some pertinent laws would appear to answer this issue in the affirmative, in this wise: Article 80 of the New Civil Code considers as marriages void from the beginning, bigamous marriages not falling under Art. 83 (2). Article 89 of the same Code, in turn, bestows upon children conceived or born of marriages void from the beginning, referred to as natural children by legal fiction, the status, rights and obligations of acknowledged natural children. Among the rights of acknowledged natural children is the right of legitimation granted to them under Article 269 in relation to 271 of the same Code. Since private respondent's children were all born after her marriage to the deceased in Tokyo in 1951, which marriage is considered bigamous, hence, void from the beginning, because of its celebration while the marriage between the deceased and his first wife, Sofia Bona, still subsisted, said children are natural children by legal fiction who have the rights of acknowledged natural children, including the right to be legitimated, and they may now be considered legitimated since the deceased and private respondent validly married for the second time after Sofia's demise. It happens that the law may lose its character of being a law by an excess of caprice in its administration, but it could hardly cease to be law because of its rigid logical application according to its tenor. When its rigid logical application, however, amounts to absurdity, the law not only becomes incapable of just administration but may also become an instrument of legal injustice. Clearly for us to read the law in the aforegoing manner is to dangerously teeter on the fulcrum of legal folly for there is no scaling down its unacceptable implications. If children born out of an extramarital relationship, but whose parents contracted a bigamous marriage and still another marriage subsequent thereto upon the death of the first spouse of the adulterous parent, may eventually be legitimated, then children of adulterous spouses, by the expedient contrivance of a bigamous marriage, may later on be legitimated. The adulterous spouse may still prove himself virtuous and heroic by risking prosecution for bigamy if only to give his child out of wedlock the chance, that slim chance, to be legitimated, that is, if he prays enough that his first spouse dies ahead of him so he could eventually validly marry his paramour. As such, in the mildest terms, the law would seem to condone extramarital relationships by providing the seemingly confessant adulterer a way to be a conscientious parent to his illegitimate children without having to give up his illicit relations with their mother. At its worst, such a reading of the law amounts to a mockery of the institution of marriage, which is, under our Constitution and family laws, an inviolable social institution imbued with public interest and traditionally and constantly held to be a priority in our culture's scale of values, for nothing stops the public from concluding that marriage and a bigamous marriage at that (with its accompanying criminal 794

University of the Cordilleras College of Law First Year C S.Y. 2013 - 2014 consequences), is actually a backdoor to legitimating adulterous children. The letter of Article 89 of the New Civil Code must be transcended and the absurd and sheepishly grotesque consequences of its application in the instant case, rejected. It is not enough that the law exists to be administered justly; in addition and more importantly, the law needs to possess a just content. The law must by itself aim at and endeavor to conform to, some criteria of rightness which repose on values espoused by the very society it seeks to serve. As it is our duty to declare the law as it is, there is no escaping the task of revealing the justness of the law in accordance with society's avowed values. Consequently, it has been called a golden rule of statutory interpretation that unreasonableness of the result produced by one possible interpretation of a statute is reason for rejecting that interpretation in favor of another which would produce a reasonable result. 5 In resolving the issue at hand, I believe the emphasis should be on Article 269 which is, after all, the law squarely in point under the premises of this case. Taking the letter of Article 269 as it is, it clearly prescribes the limits of its applicability upon only natural children. Surely there is no canon against using both common sense and common weal in construing the law as saying what it obviously means: Chapter 3 Art. 269. Only natural children can be ligitimated. Children born outside wedlock of parents who, at the time of the conception of the former, were not disqualified by any impediment to marry each other, are natural. xxx xxx xxx

Art. 271. Only natural children who have been recognized by the parents before or after the celebration of the marriage, or have been declared natural children by final judgment, may be considered legitimated by subsequent marriage. (Emphasis ours) Legitimation is a right granted by law only to natural children who, because their parents could have legally married at the time they were conceived, cannot be substantially differentiated from legitimate children once their parents do marry after their birth. This is because said parents can marry any time, there being no legal impediment preventing them from validly contracting marriage. The situation obtaining respecting legitimate children and legitimated natural children is certainly distinct from that respecting adulterous children because the parents of adulterous children are admittedly incapacitated to marry each other at the time said children were conceived. It may easily be said, thus, that to interpret the law as allowing adulterous children to be put on equal footing with the legitimate children, would be putting a premium on adulterous relationships, which is frowned upon by the society itself. Even the law on succession under the New Civil Code distinguishes the respective hereditary rights of the different kinds of children and significantly assigns a diminishing share in accordance with the degree of illegitimacy of the child concerned. Thus, Article 895 provides that the legitime of each of the acknowledged natural children and each of the natural children by legal fiction shall consist of one-half of the legitime of each of the legitimate children or descendants and that of illegitimate children who is neither of the above, four-fifths of the legitime of an acknowledged natural child. It is, therefore, evident that the treatment accorded children under the New Civil Code is determined by the circumstances under which they have been conceived and born, particularly, the capacity to marry of their parents at the time that they were conceived. Private respondent's children were precisely born when their deceased father was still legally married to Sofia Bona. The marriage of the deceased and private respondent in Tokyo, Japan, in 1951 could not have given a semblance of legitimacy to their subsequent cohabitation and their issues since such marriage was contracted during the subsistence of the deceased's marriage with Sofia Bona. The relationship between the deceased and private respondent, therefore, was no less adulterous notwithstanding an attempt to legitimize the same through a bigamous marriage. There is no other way to put it but that the deceased and private respondent were having illicit relations; they were fully aware of the legal and moral consequences of their actions, and they seemingly, in bad faith, attempted to circumvent the law in their favor by contracting a bigamous marriage to the prejudice of the legitimate issue in the person of the petitioner. There is no quibbling that private respondent's children are adulterous children whose status, by the simple expedient of a bigamous marriage contracted by parties fully aware of their incapacity to marry, could never have been intended by the law to be equated to that of petitioner who is the legitimate child of the deceased in view of the public policy involved in preserving the sanctity of marriage and preventing the proliferation of illegitimate issues. As the earlier interpretation has been shown to lead to 795

University of the Cordilleras College of Law First Year C S.Y. 2013 - 2014 unreasonable results with socially virulent implications, and the same originates from two provisions, namely, Article 89 and Article 269 of the New Civil Code, we are wont to state that they are irreconcilable provisions. And the applicable statutory rule is that where there is an irreconcilable conflict between the different provisions of a statute, the provision last in order of position will prevail, since it is the latest expression of the legislative will. 6 More than that Article 269 is the latest expression of the legislative will, however, Article 269 on its face specifically states the law on legitimation, limits its applicability to natural children, and is resonantly silent on the right of adulterous children to be legitimated in the same way as children born to parents who, at the time of their conception, were legally capable to marry each other. All told, the law tenders to us in no unpretentious terms the basis to rule that private respondent's children, being adulterous children, have no right to be legitimated under the New Civil Code. Such a ruling is not only in accord with the explicit, unequivocal language of Article 269 but more importantly animates and upholds the public policy as regards the institution of marriage as the foundation of society. Needless to say, such ruling sits well with the need to obviate any legal injustice and social absurdity that may result if we were to rule otherwise. The final cause of law is the welfare of society. The rule that misses its aim cannot permanently justify its existence. "Ethical considerations can no more be excluded from the administration of justice which is the end and purpose of all civil laws than one can exclude the vital air from his room and live. 7 The final rendering of the meaning of a statute is an act of judgment. 8 This court has so judged this case at bench, and so we will perhaps be judged thereby. I, therefore, vote to grant the petition, set aside the assailed order of the Regional Trial Court, and remand thereto the case for further proceedings. VITUG, J., dissenting: I vote to resolve the controversy in favor of the child. I take it to be the legislative intent that the pertinent provisions of the Civil Code on children in the book on persons and family relations are meant to enhance the child's interest and welfare. This intent finds exemplification in Article 89 of the Civil Code by explicitly providing that natural children by legal fiction (among them those conceived or born of void marriages because the parents suffer from an impediment to marry) shall have the same status, rights and obligations as acknowledged natural children. If then under Article 269, in relation to Article 270, of the Civil Code, acknowledged natural children are given the right to be legitimated by the subsequent marriage of the parents, the law must, by virtue of Article 89 aforesaid, likewise extend unqualifiedly to natural children by legal fiction. No matter how well legal calisthenics are played, there is, I must point out, not a single provision of the Code that limits or circumscribes the scope and application of Article 89. The law, I respectfully submit, should be so construed as to attain congruity, rather than a division, among its several provisions. The rule is expressed in the maxim interpretare et concordare legibus est optimus interpretendi upon the theory that the legislature is presumed not to have enacted conflicting provisions of law but that, on the contrary, it must have meant to give them such parity and consequence as a uniform jurisprudential system. Most regrettably, I still perceive coolness, if not outright hostility, towards illegitimate children who have not been fortunate enough to be conceived or born under a better family circumstance. It is not enough that they are unjustly ostracized by a segment in society; they are also called names bastards, outcasts, adulterous, spurious that certainly they do not deserve. If at all, their situation needs sympathy, not hatred or condemnation. Any conflict of view, however, would soon be a thing of the past, for as so keenly observed by Mme. Justice Flerida Ruth P. Romero, the Family Code, which became effective on 03 August 1988, has deleted any reference to natural children by legal fiction. The Family Code presently categorizes children of void marriages into two kinds the legitimates which include those conceived or born of void marriages under Article 36 and Article 52 of the Family Code before the judicial declaration of nullity of such void marriages and the illegitimates or children conceived or born of all other void marriages (but evidently maintaining, for legitimation purposes, the distinction between those whose parents, at the time of conception, were not disqualified to marry and those whose parents were disqualified).

796

University of the Cordilleras College of Law First Year C S.Y. 2013 - 2014 Narvasa, C.J. and Bellosillo, J., concurs. KAPUNAN, J., dissenting: The principal issue in the case at bench may be capsulized as to whether or not the trial court committed grave abuse of discretion amounting to a lack or excess of jurisdiction in considering the private respondent's children legitimated under the facts established herein, and in declaring and instituting said children as heirs of the decedent. As the law unequivocally gives them such a right, I respectfully dissent from the majority. I begin by observing that, taking their cue from the lower court's inappropriate lifting of an editor's precis or statement from the syllabus of the case of Tongoy vs. Court of Appeals, 1 both parties in the case at bench have placed too much emphasis and reliance on the case of Tongoy, 2 the facts and circumstances of which are not exactly on all fours with those obtaining in the case at bench. The italicized portion of the syllabus of cases appearing in official or unofficial reports of Supreme Court Decisions or Resolutions 3 generally reflect the editor's summary of a discussion of an issue or a specific point in a case, and, taken out of context, could be misleading and inappropriate for citation. Judges should strive to read cases which might have a bearing on cases before them in their entirety, and quote or obtain their citations from the body of the decision, not the syllabus. The principal issue in Tongoy, 4 hinged "on the absence of an acknowledgment (by the father prior to his death of his illegitimate children) through any of the modes recognized by the Old Civil Code." 5 It is not, however clear from the Court's discussion of the facts of the case, whether the illegitimate children were sired during the subsistence of the first marriage or after the death of the first wife. On the sale issue of the father's acknowledgment, the Court therein took a liberal view, recognizing the fact that the children "were in continuous possession of the status of natural, or even legitimated, children" 6 and that they were "treated as legitimate children not only by their parents but also by the entire clan," 7 in declaring, on equitable grounds, that the children therein were legitimate heirs. For better appreciation of our ruling in Tongoy, let me quote the following: The remaining assignment of error dwells on the question of whether or not respondents Amado, Ricardo, Cresenciano and Norberto, all surnamed Tongoy, may be considered legitimated by virtue of the marriage of their parents, Francisco Tongoy and Antonina Pabello, subsequent to their births and shortly before Francisco died on September 15, 1926. Petitioners maintain that since the said respondents were never acknowledged by their father, they could not have been legitimated by the subsequent marriage of their parents, much less could they inherit from the estate of their father, the predecessor-in-interest of Luis D. Tongoy, who is admittedly the half brother of the said respondents. Both the trial court and the respondent appellate court have found overwhelming evidence to sustain the following conclusions: that Amado P. Tongoy, Ricardo P. Tongoy, Cresenciano P. Tongoy and Norberto P. Tongoy were born illegitimate to Antonina Pabello on August 19, 1910 (Exh. A), August 12, 1922 (Exh. B), December 1, 1915 (Exhs. C and C-1) and August 4, 1922 (Exh. D), respectively; that Francisco Tongoy was their father; that said Francisco Tongoy had before them and Antonina Pabello two legitimate children by his first wife, namely, Luis D. Tongoy and Patricio D. Tongoy; that Francisco Tongoy and Antonina Pabello were married sometime before his death on September 15, 1926 (Exh. H); that shortly thereafter, Luis D. Tongoy and Patricio D. Tongoy executed an Extra-Judicial Declaration of Heirs, leaving out their half-brothers Amado, Ricardo, Cresenciano, and Norberto, who were then still minors; that respondents Amado, Ricardo, Cresenciano and Norberto were known and accepted by the whole clan as children of Francisco; that they had lived in Hacienda Pulo with their parents, but when they went to school, they stayed in the old family home at Washington Street, Bacolod, together with their grandmother, Agatona Tongoy; that everybody in Bacolod knew them to be part of the Tongoy-Sonora clan; and that Luis D. Tongoy as administrator of Hacienda Pulo, also spent for the education of Ricardo Tongoy until he became a lawyer; and that even petitioners admit the fact that they were half-brothers of the late Luis D. Tongoy. The bone of contention, however, hinges on the absence of an acknowledgment through any of the modes recognized by the Old Civil Code (please see Articles 131 and 135 of the Old Civil Code), such that legitimation could not have taken place in view of the provisions of Art. 121 of the same Code which states that "children shall be considered legitimated by a subsequent marriage only when they have been acknowledged by the parents before or after the celebration thereof."

797

University of the Cordilleras College of Law First Year C S.Y. 2013 - 2014 Of course, the overwhelming evidence found by respondent Court of Appeals conclusively shows that respondents Amado, Ricardo, Cresenciano and Norberto have been in continuous possession of the status of natural, or even legitimated children. Still, it recognizes the fact that such continuous possession of status is not, per se, a sufficient acknowledgment but only a ground to compel recognition (Alabat vs. Alabat, 21 SCRA 1379; Pua vs. Chan, 21 SCRA 753; Larena vs. Rubio, 43 Phil. 1017). Be that as it may, WE cannot but agree with the liberal view taken by respondent Court of Appeals when it said: . . . It does not seem equally manifest, however, that defendants-appellants stand on a purely technical point in the light of overwhelming evidence that appellees were natural children of Francisco Tongoy and Antonina Pabello, and were treated as legitimate children not only by their parents but only by the entire clan. Indeed, it does not make much sense that appellees should be deprived of their hereditary rights as undoubted natural children of their father, when the only plausible reason that the latter could have had in mind when he married his second wife Antonina Pabello just over a month before his death was to give legitimate status to their children. It is not in keeping with the more liberal attitude taken by the New Civil Code towards illegitimate children and the more compassionate trend of the New Society to insist on a very literal application of the law in requiring the formalities of compulsory acknowledgment, when the only result is to unjustly deprive children who are otherwise entitled to hereditary rights. From the very nature of things, it is hardly to be expected of appellees, having been reared as legitimate children by their parents and treated as such by everybody, to bring an action to compel their parents to acknowledge them. In the hitherto cited case of Ramos vs. Ramos, supra, the Supreme Court showed the way out of patent injustice and inequity that might result in some cases simply because of the implacable insistence on the technical amenities for acknowledgment. Thus, it held Unacknowledged natural children have no rights whatsoever (Buenaventura vs. Urbano, 5 Phil. 1; Siguiong vs. Siguiong, 8 Phil. 5, 11; Infante vs. Figueras, 4 Phil. 738; Crisolo vs. Macadaeg, 94 Phil. 862). The fact that the plaintiffs, as natural children of Martin Ramos, received shares in his estate implied that they were acknowledged. Obviously, defendants Agustin Ramos and Granada Ramos and the late Jose Ramos and members of his family had treated them as his children. Presumably, that fact was well-known in the community. Under the circumstances, Agustin Ramos and Granada Ramos and the heirs of Jose Ramos, are estopped from attacking plaintiffs' status as acknowledged natural children (See Arts. 283 [4] and 2666 [3], New Civil Code). [Ramos vs. Ramos, supra]. With the same logic, estoppel should also operate in this case in favor of appellees, considering, as already explained in detail, that they have always been treated as acknowledged and legitimated children of the second marriage of Francisco Tongoy, not only by their presumed parents who raised them as their children, but also by the entire Tongoy-Sonora clan, including Luis D. Tongoy himself who had furnished sustenance to the clan in his capacity as administrator of Hacienda Pulo and had in fact supported the law studies of appellee Ricardo P. Tongoy in Manila, the same way he did with Jesus T. Sonora in his medical studies. As already pointed out, even defendants-appellants have not questioned the fact that appellees are half-brothers of Luis D. Tongoy. As a matter of fact, they are really children of Francisco Tongoy and Antonina Pabello, and only the technicality that their acknowledgment as natural children has not been formalized in any of the modes prescribed by law appears to stand in the way of granting them their hereditary rights. But estoppel, as already indicated, precludes defendants-appellants from attacking appellees' status as acknowledged natural or legitimated children of Francisco Tongoy. In addition to estoppel, this is decidedly one instance when technicality should give way to conscience, equity and justice (cf. Vda. de Sta. Ana vs. Rivera, L-22070, October 29, 1966, 18 SCRA 588) [pp. 196-198, Vol. 1, rec.]. It is time that WE, too, take a liberal view in favor of natural children who, because they enjoy the blessings and privileges of an acknowledged natural child and even of a legitimated child, found if rather awkward, if not unnecessary, to institute an action for recognition against their natural parents, who, without their asking, have been showering them with the same love, care and material support as are accorded to legitimate children. The right to participate in their father's inheritance should necessarily follow. 8 However, acknowledgment is clearly not at issue here. Petitioner makes no pretense that private respondent's children are not entitled to hereditary rights. 9 She herself admits that the decedent acknowledged his paternity of the private respondent's children and that they are indeed her brothers and sisters. 10 What herein petitioner claims she opposes "is their being judicially declared legitimated (by the respondent court) so as to entitle them to enjoy the same rights as a legitimate heir," 11 to her prejudice. 798

University of the Cordilleras College of Law First Year C S.Y. 2013 - 2014 Citing Article 269 of the New Civil Code as "the law in point" in the case at bench, she contends that the trial court erred in declaring her half brothers and sisters legitimated because under the New Civil Code only natural children could be legitimated. I find this contention, to which the majority of this divided Court agrees, absolutely untenable. The New Civil Code appears to limit the right to legitimation only to those children conceived by parents not disqualified by any impediment to marry each other, bestowing upon them, prior to such legitimation, the status of natural children. Article 269 which provides the cornerstone for the majority's holding today states that: Art. 269. Only natural children can be legitimated. Children born outside wedlock of parents who, at the time of the conception of the former, were not disqualified by any impediment to marry each other, are natural. The rule is, however, not absolute because even children conceived or born out of marriages void from the very beginning under the Civil Code possess the status of natural children by legal fiction and enjoy the same rights as acknowledged natural children. Article 89 provides: Art. 89. Children conceived or born out of marriages which are void from the beginning shall have the same status, rights and obligations as acknowledged natural children, and are called natural children by legal fiction. Children conceived of voidable marriages before the decree of annulment shall be considered legitimate; and children conceived thereafter shall have the same status, rights and obligations as acknowledged natural children, and are also called natural children by legal fiction. Article 89, a creature of legislation (through the Code Commission) which has remained unmolested since 1950 I must stress, is not an accidental provision. The Civil Code Commission clearly intended Article 89, notwithstanding its location in the Code, as a piece of reform, an exception to the rule furnished by Article 269. More importantly, Article 89 (unlike Article 269 which came from the Spanish Civil Code of 1889) was a new provision deliberately introduced by the Code Commission as one of its revolutionary reforms thirty five years ago. And doubt about the intention of this piece of legislation should have been laid to rest by the following explanation from the Code Commission's Report: This proposed reform is based on the fact that such children have been brought into this world through no fault of their own, but through that of their parents. To visit punishment upon them is most unjust. Moreover, though the marriage is void, or voidable, at least there was a semblance of legality to the relationship between the parents. This circumstance should cast a mantle of protection over the children, who by legal fiction should be treated as acknowledged natural children. 12 Since the decedent's 1951 marriage in Tokyo with the private respondent was invalid, 13 being one of those marriages classified as void from the very beginning under the Civil Code, 14 the status of her children clearly falls under Article 89 which puts them on par, at least in terms of rights and obligations, with acknowledged natural children. Since the rights of acknowledged natural children include the right of legitimation under Article 270 of the Civil Code by the subsequent valid marriage of their parents, 15 it therefore plainly follows that by virtue of Article 89, in relation to Article 270, the private respondent's children were deemed legitimated by the subsequent valid marriage of their parents in the Philippines in 1967. This position is hardly an isolated one. Virtually all Civil and Family Code commentators are united in the belief that Article 89 furnishes an escape valve for children found under the circumstances existing in the case at bench. Dr. Arturo Tolentino, in his commentary on the Civil Code, for example, writes: Under Article 89, natural children by legal fiction "shall have the same status, rights and obligations as acknowledged natural children." Theoretically therefore, natural children by legal fiction can be legitimated. . . . . The following children by legal fiction . . . can be legitimated: . . . (2) those born of a bigamous marriage, for the parents can marry each other again upon the widowhood of the parent who married twice. . . . . 16 In the same token, Prof. Ernesto L. Pineda, a member of the Family Code Revision Committee acknowledges this exception the rule, stating that: 799

University of the Cordilleras College of Law First Year C S.Y. 2013 - 2014 By way of exception, some natural children by legal fiction (Art. 89, NCC) can be legitimated such as (a) those born of couples who married while below the allowable marrying age but who contracted a new marriage after reaching the proper age; (b) those born of bigamous marriages but where the parents married each other upon the widowhood of the disqualified parent; (c) those born of parents who got married before an unauthorized officer and the parents contracted a new marriage before an authorized one; (d) those born of parents who got married without a marriage license (where license was required) and the parents contracted a subsequent valid marriage; and (e) children conceived after (not before) the decree of annulment of a voidable marriage. 17 Justice Alicia V. Sempio-Diy, writing on the New Family Code 18 underscores the difference in treatment of the subject of legitimation between the Family Code and the Civil Code thus: Under the Civil Code, children of bigamous marriages, who are natural children by legal fiction, can be legitimated, since the parents can marry each other upon the death of the first husband or wife of the parent who married twice. Unfortunately for such children, they can no longer be legitimated under the Family Code, which has limited the kind of children to legitimate and illegitimate and abolished the category of natural children by legal fiction. 19 "Parenthetically," another commentator on the Family Code, Prof. Melencio Sta. Maria writes, "under the Civil Code provisions of legitimation which were repealed by the Family Code," there can be an instances where such children could be legitimated. 20 Elaborating on these provisions in his 1995 commentary; he states: This is so because according to the repealed Article 271 of the Civil Code only acknowledged natural children can be legitimated, and also according to the repealed Article 89 of the Civil Code, a child born inside a void marriage was considered a natural child by legal fiction with all the rights of an acknowledged natural child. Since a natural child by legal fiction has all the rights of an acknowledged natural child and the statutory right to be legitimated was one of the rights of an acknowledged natural child, the subject child therefore can be legitimated if the parents subsequently validly remarried. 21 Clearly, the weight of authority in this country recognizes that under the Civil Code, Article 89 unequivocally furnishes an exception to the rule that only acknowledged natural children or those who by law have been declared natural children by final judgment can be legitimated. This exception was, in fact, acknowledged by the Family Code Revision Committee in its Meeting of August 24, 1985, when it decided not to accord the same privilege extended by Article 89 to similarly situated illegitimate children (under the family Code's simplified classification) in the provisions of the new code. However, for children born under the Civil Code, the exception is a legal fact which could not be ignored. If under Article 269, in relation to Article 270 of the Civil Code, acknowledged natural children are given the right to be legitimated by the subsequent marriage of the parents, the law must, by virtue of Article 89, also extend unqualifiedly to natural children by legal fiction. This not only harmonizes Article 89 with the Civil Code articles on the rights of acknowledged natural children and the articles on legitimation but also leads to a result which enhances the welfare and interest of the child. As Justice Vitug in his 1993 Compendium of Civil Law and Jurisprudence writes: The provisions of Art. 269 and 271 of the Civil Code, in a literal sense appear to limit legitimation in favor of acknowledged natural children or those who by law have been declared natural children by final judgment. Considering, however, that natural children by legal fiction (such as those born of void marriages because the parents suffer from an impediment to marry) are expressly given the same status, rights and obligations as acknowledged natural children (Art. 89 Civil Code), and because all doubts should be resolved in favor of the child, it is submitted that the rules on legitimation should likewise extend to such children. 22 Indeed, it hardly makes sense that the children of private respondent should be deprived of their full hereditary rights as legitimated children when the facts and circumstances of the case at bench clearly show the decedent's intention to remove, once and for all, all manner of legal and moral obstacles to his second and apparently blissful union with the private respondent. For immediately after the death of his first wife in Guatemala in 1967, the decedent wasted no time in obtaining a Philippine marriage in Tagaytay with his second wife. With a fairly considerable estate, it was not entirely remote that the decedent had in mind not only the intention to legitimatize his union with the private respondent but also the intention to accord legitimate status to his children with his second wife. Given the nature of their relationship and the clear intendment of the Civil Code under Article 89 to place natural children by legal 800

University of the Cordilleras College of Law First Year C S.Y. 2013 - 2014 fiction on equal standing with acknowledged natural children, a patent injustice and inequity will result if we uphold herein petitioner's implacable position. Given the clear intendment of the legislature in enacting the new provision (Article 89) over thirty years ago when many of the members of this Court were still law students, the majority's holding in the case at bench amounts to a belated judicial veto of a valid piece of legislation. I vote to DENY the instant petition. Narvasa, C.J., Padilla, Bellosillo and Francisco, JJ., concur. PANGANIBAN, J., dissenting: With all due respect, I dissent from the well-written ponencia of Mme. Justice Flerida Ruth P. Romero. The pertinent portions of Arts. 89, 269, 270 and 271 of the New Civil Code which are the codal provisions in point, read as follows: Art. 89. Children conceived or born of marriages which are void from the beginning shall have the same status, rights and obligations as acknowledged natural children, and are called natural children by legal fiction. xxx xxx xxx

Art. 269. Only natural children can be legitimated. Children born outside wedlock of parents who, at the time of the conception of the former, were not disqualified by any impediment to marry each other, are natural. Art. 270. Legitimation shall take place by the subsequent marriage of the parents.

Art. 271. Only natural children who have been recognized by the parents before or after the celebration of the marriage, or have been declared natural children by final judgment, may be considered legitimated by subsequent marriage. . . . Art. 89 has been repealed by the Family Code (Executive Order No. 209) which took effect on August 3, 1988 (Uyguangco vs. Court of Appeals, 178 SCRA 684 [1989]; Atienza vs. Brillantes, A.M. No. MTJ-92706, March 29, 1995). It was one of the provisions under Title III, Book I of the New Civil Code which have been omitted from the text of the present Family Code. But it was the law in force at the time the legitimation in the case at bench took place and should, consequently, govern the present controversy. Art. 89 was a new provision in the sense that, unlike Articles 269 to 271, aforequoted, which all came from the Spanish Civil Code of 1889, Art. 89 was one of the reforms instituted by the Code Commission that drafted the New Civil Code. The Code Commission justified this new article in this wise: This proposed reform is based on the fact that such children have been brought into the world through no fault of their own, but through that of their parents. To visit punishment upon them is most unjust. Moreover, though the marriage was void, or voidable, at least there was a semblance of legality of the relationship between the parents. This circumstance should cast the mantle of protection over the children, who by legal fiction should be treated as acknowledged natural children. (Report of the Code Commission, at p. 81.) In conferring upon natural children by legal fiction the same status, rights and obligations of acknowledged natural children, the clear intention of the law was to put them at par with the latter although in fact they are not. They are not in fact natural because they were conceived in the presence, not absence, of an impediment between the parents. They are natural only by figment of law. Thus, the name natural children by legal fiction. But this legal fiction precisely operates to exempt them from the requirement under Art. 269 that there be no impediment between the parents at the time of the conception as well as from the requirement of recognition by both parents under Art. 271. Plainly, this is the conclusion that can rationally be given to the express, unequivocal declaration in Art. 89 that natural children by legal fiction "shall have the same status, rights and obligations as acknowledged natural children" neither imposing any condition nor subjecting the grant of status to any qualification or exception of any kind. Had the intention been to 801

University of the Cordilleras College of Law First Year C S.Y. 2013 - 2014 deprive them of the right of legitimation, the law would have said so. Or it would have inserted a condition that they could be legitimated only if they can show compliance with Arts. 269 and 271 of the Code. The fact that these insertions were not made can only mean that the law intended to exempt this special class of natural children from the strict requirements normally imposed on ordinary natural children. Under the provisions of the New Civil Code, legitimation takes place when three requisites are met: (a) that the child be a natural child; (b) that he be recognized by both parents either before or after a valid marriage; and (c) that there be a subsequent valid marriage of the parents (cf. Paras, Civil Code of the Philippines Annotated, 1984 Ed., Vol. I, p. 651). A natural child by legal fiction possesses the first two requisites from inception by virtue of Art. 89, which places him on the same plane as an acknowledged natural child. In that sense, he has an advantage over a natural child as defined by Art. 269, for the latter would still need to be recognized by both parents in order to have the status and rights of an acknowledged natural child. Thus, for the purpose of legitimation, the natural child by legal fiction needs to fulfill only the third requisite: a valid subsequent marriage between his parents (cf. Paras, op. cit., p. 651; Tolentino, Civil Code of the Philippines, 1987 Reprinting, Vol. I, p. 570). Where the impediment is permanent or perpetual, such as incest or the fact that one or both of the parties have been found guilty of killing the spouse of one of them, no legitimation can ever take place as no valid marriage can ever be made between the parents (Tolentino, op. cit., p. 570). But the bigamous character of a marriage is terminable by, among other causes, the death of the first spouse, making a subsequent marriage valid. And that simply was what happened in the case at bench. Prior to the repeal of Art. 89 by the Family Code, it was suggested by some civil law scholars that a distinction should be made between natural children by legal fiction who were conceived during the existence of an impediment, on the one hand, and those who were conceived after the disappearance of such impediment, on the other. Their theory was that only the latter would qualify for legitimation. Such a stance would have been juridically sound were it not for the fact that Art. 89 does not classify natural children by legal fiction into the two suggested categories based on the presence or absence of impediment at the time of conception of the child. Ubi lex non distinguit nec nos distinguere debemus; where the law does not distinguish, we should not (Gesolgon vs. Lacson, 2 SCRA 553, 556 [1961]; Libudan vs. Gil, 45 SCRA 17, 33 [1972]; Guevarra vs. Inocentes, 16 SCRA 379, 385 [1966]; Robles vs. Chromite Mining Co., 104 Phil. 688, 690 [1958]). Besides, as already pointed out, the conferment on natural children by legal fiction of the same status, rights and obligations as those of acknowledged natural children under Art. 89, New Civil Code, evidently exempted the former from the requirements imposed upon ordinary natural children by Arts. 269 and 271 of said Code. And, finally, Art. 220 of said Code fortifies this conclusion, because said provision declares that in case of doubt, every intendment of law or fact leans toward, among other things, "the legitimacy of children". The doubt if there be such at all should therefore be resolved in favor of sustaining the right to legitimation of the eleven (11) offsprings of the decedent with private respondent Conchita Talag, regardless of the presence or absence of an impediment to marry on the part of their parents at the time of the conception of each of them. Indeed, it is hardly fair to stigmatize and create social and successional prejudice against children who had no fault in nor control over the marital impediments which bedeviled their parents. They are the victims, not the perpetrators, of these vagaries of life. Why then should they suffer their consequences? In the final analysis, there are really no illegitimate children; there are only illegitimate parents. And this dissent finds its philosophy in this: that children, unarguably born and reared innocent in this world, should benefit by every intendment of the law, particularly where as in this case their parents, who originally suffered from a marital impediment, would now want to overcome the improvident social and successional consequences of such condition. Therefore, it is most unfair that the these innocent children should be condemned to continue suffering the consequences of the impediment they did not cause, when the very impediment itself has disappeared. The mere fact that such legitimation would impact adversely upon the petitioner's successional rights as the lone legitimate child of the first marriage is no reason to deny the children of the second marriage of their own legal right to be deemed legitimated. Precisely, legitimation produces such an effect i.e., diminution of successional rights of the legitimate children. Art. 272 of the New Civil Code provides in fact that "(c)hildren who are legitimated by subsequent marriage shall enjoy the same rights as legitimate children." When the legislature decided to grant to children of void carriages the same status, rights, and obligations as those of acknowledged natural children, it is presumed to have carefully weighed precisely these consequences upon the rights of the other children in the family. The policy then was to cast a mantle of protection upon children of void marriages. That policy is evidently enforced by enabling them to get legitimated in the same manner as acknowledged natural children namely, by the subsequent 802

University of the Cordilleras College of Law First Year C S.Y. 2013 - 2014 valid marriage of their parents. If the Family Code, by repealing Art. 89 of the New Civil Code, is to be viewed as having reversed or denigrated that policy (although, by and large, it appears to have maintained the policy in many other areas of family law), such reversal or denigration should not, and cannot, in any case impair rights already acquired by and thus vested in the private respondents. One last point. Both petitioner and private respondent admit that the eleven (1l) children of the decedent with private respondent Conchita Talag were born after the celebration of the bigamous marriage on July 25, 1951. (See Petition, item 6, page 5; Rollo p. 6.) If any one of them was born prior thereto, such child, not being a natural child by legal fiction but spurious, cannot claim the special benefit granted under Art. 89 of the New Civil Code. Unlike his brothers and sisters who are natural children by legal fiction, he can only inherit by showing that he has been recognized by the decedent as the latter's illegitimate child either voluntarily or by final judgment in a proper paternity suit (Paterno vs. Paterno, 20 SCRA 585; Noble vs. Noble, 18 SCRA 1104; Clemena vs. Clemena, 24 SCRA 720; Divinagracia vs. Rovira, 72 SCRA 307; Tolentino, The Civil Code of the Philippines, 1987 Reprinting, Vol. I, pp. 616-617.)

803

University of the Cordilleras College of Law First Year C S.Y. 2013 - 2014 Case Digest: De Santos vs Angeles 251 SCRA 206 G.R. No. 105619 December 12, 1995 ROMERO, J.: Facts: There being no explicit provision of law in point, the Court is called upon to cast illumination in a gray area even as it fills up unintentional interstices in the fabric of Civil Law with overlays of philosophical, historical and sociological strands. For an understanding of how the issue arose, we now proceed to unravel the pertinent factual background. On February 7, 1941, Dr. Antonio de Santos married Sofia Bona, which union was blessed with a daughter, herein petitioner Maria Rosario de Santos. After some time, their relationship became strained to the breaking point. Thereafter, Antonio fell in love with a fellow doctor, Conchita Talag, private respondent herein. Antonio sought a formal dissolution of his first marriage by obtaining a divorce decree from a Nevada court in 1949. Obviously aware that said decree was a worthless scrap of paper in our jurisdiction which then, as now, did not recognize divorces, Antonio proceeded to Tokyo, Japan in 1951 to marry private respondent, with whom he had been cohabiting since his de facto separation from Sofia. This union produced eleven children. On March 30, 1967, Sofia died in Guatemala. Less than a month later, on April 23, 1967, Antonio and private respondent contracted a marriage in Tagaytay City celebrated under Philippine laws. On March 8, 1981, Antonio died intestate leaving properties with an estimated value of P15,000,000.00. Issue: This conclusion not only presumes that children other than those who are "natural" can be legitimized in the first place, but also grants acknowledged natural children (and, consequently, natural children by legal fiction) a "right" to be legitimized when no such right exists. Legitimation is not a "right" which is demandable by a child. It is a privilege, available only to natural children proper, as defined under Art. 269. Although natural children by legal fiction have the same rights as acknowledged natural children, it is a quantum leap in the syllogism to conclude that, therefore, they likewise have the right to be legitimated, which is not necessarily so, especially, as in this case, when the legally existing marriage between the children's father and his estranged first wife effectively barred a "subsequent marriage" between their parents. Ruling: The provisions of law invoked by private respondent are couched in simple and unmistakable language, not at all subject to interpretation, and they all point to the correctness of petitioner's claim. If it should be asserted that we now trench on a gray area of law that calls for interpretation, or a lacuna that cries for filling up, then we have to pierce the shroud unintentionally created by the letter of the law and expose its spirit as evincing intent, in this case one which decidedly favors legitimacy over illegitimacy. The hierarchy of children so painstakingly erected by law and the corresponding gradation of their rights may conceivably be shattered by elevating natural children by legal fiction who are incontestably illegitimate children to the level of natural children proper, whose filiation would otherwise be legitimate had their parents blessed their union with a valid marriage. Finally, attention must be drawn to the fact that this case has been decided under the provisions of the Civil Code, not the Family Code which now recognizes only two classes of children: legitimate and illegitimate. "Natural children by legal fiction" are nothing if not pure fiction. WHEREFORE, the instant petition is hereby GRANTED. The assailed orders of the court a quo dated November 14, 1991 and January 9, 1992, are NULLIFIED and SET ASIDE. Petitioner Maria Rosario de Santos is hereby declared the SOLE LEGITIMATE CHILD of the decedent Antonio de Santos and, as such, entitled to all the rights accorded to her by law.

804

University of the Cordilleras College of Law First Year C S.Y. 2013 - 2014 Pe Lim vs Court of Appeals 270 SCRA 1 G.R. No. 112229. March 18, 1997 Full Case RAYMOND PE LIM, petitioner, vs COURT OF APPEALS, JOANNA ROSE C. PE LIM, Minor represented by her Natural Mother and Guardian, MARIBEL CRUZ y TAYAG, respondents. ROMERO, J.: All too often, immature men who allow their emotions to hold sway over their rational minds come to grief when their passions cool off, but not before inflicting irreparable psychic and spiritual damage on their victims and the fruits of their wanton acts. As they sow the proverbial "wild oats," they are heedless of the dire consequences they heap on their heads. When the inevitable confrontation explodes and they are helpless to extricate themselves from the messy situation arising from their wrongdoing, eventually they invoke the help of the courts as their final arbiter. Before us is one of those cases where a man woos a maid, succeeds in seducing and impregnating her, only to disclaim the paternity of the child when made to account for his misdeeds. DNA,[1] being a relatively new science, it has not as yet been accorded official recognition by our courts. Paternity will still have to be resolved by such conventional evidence as the relevant incriminating acts, verbal and written, by the putative father. This petition for review on certiorari sprang from a complaint filed by Maribel Cruz for child support on behalf of her daughter, private respondent Joanna Rose C. Pe Lim, against petitioner Raymond Pe Lim who, Maribel claims, is Joanna's father. Maribel's story unfolds, thus: Maribel was sixteen years old in 1978 and a part-time student. She also worked as a receptionist at Tonight's Club and Resthouse along Roxas Blvd., Manila. She met petitioner during her first night on the job. Petitioner wooed her and Maribel reciprocated his love. They soon lived together, with petitioner paying the rentals in a succession of apartments in Cubao, Quezon City, Tambo, Paraaque and Makati, Metro Manila. Maribel left for Japan in July 1981, already pregnant, and returned to Manila in October of the same year. The couple never married because petitioner claimed that he was not financially stable. On January 17, 1982, Maribel gave birth to their daughter at the Cardinal Santos Memorial Hospital. The bills for Maribel's three-day confinement at the hospital were paid for by Raymond and he also caused the registration of the name Joanna Rose C. Pe Lim on the child's birth certificate. After Joanna Rose's birth, the love affair between Maribel and petitioner continued. Towards the latter part of 1983, Maribel noted that petitioner's feelings toward her started to wane. He subsequently abandoned her and Joanna Rose. Maribel tried to support herself by accepting various jobs and with occasional help from relatives, but it was never enough. She asked petitioner for support but, despite promises to do so, it was never given. Maribel then filed a complaint against petitioner before the Regional Trial Court of Manila for support. Petitioner, on the other hand, has a different version: He claims that in 1978, he went to Tonight's Club and Resthouse along Roxas Boulevard, Manila to relax after a hard day's work. There he met Maribel, a pretty and aggressive hospitality girl. Raymond observed that while she had a pleasing personality, she seemed to be quite experienced because she started to kiss him on the cheeks and neck, whispering to him that they could go anywhere and rest. Raymond declined to take Maribel up on her offer saying that he only wanted someone to talk to. They became friends after that first meeting, and while he often saw her, there was no intimacy between them. He did admit giving Maribel sizeable tips because she confided in him that she needed money. Raymond alleged that he was not Maribel's only customer at the club. In 1980, she left for Japan to work 805

University of the Cordilleras College of Law First Year C S.Y. 2013 - 2014 as an entertainer. In 1981, she returned to Manila pregnant, and appealed to Raymond for help because she claimed that she could not face her relatives in her condition. Raymond got her an apartment and paid its rentals until she gave birth to a baby girl on January 17, 1982. Raymond admits paying the hospital bills but claims that Maribel was supposed to pay him back for it. When she failed to do so, Raymond stopped seeing her. Raymond denies being the father of Maribel's child, claiming that they were only friends and nothing more. The trial court rendered a decision on June 10, 1971, the dispositive portion of which states: "WHEREFORE, judgment is hereby rendered in favor of the plaintiff and against the defendant ordering herein defendant, Raymond Pe Lim to give support to his natural daughter, minor Joanna Rose Pe Lim in the amount of Ten Thousand Pesos (P10,000.00). Philippine Currency, per month for the support, maintenance, education and well-being of said child, the same to be paid on or before the 5th day of each month and monthly thereafter starting June, 1991, until the said minor Joanna Rose Pe Lim, shall have reached the age of majority. The defendant is further ordered to pay the plaintiff the sum of Seven Thousand Five Hundred (P7,500.00) Pesos, Philippine Currency, for attorney's fees and other litigation expenses. No costs. SO ORDERED." Petitioner then elevated his case to the Court of Appeals which affirmed the trial court's findings. Petitioner now argues before the Court that there is no clear and convincing evidence on record to show that there was actual cohabitation between him and Maribel. In fact, petitioner infers that Maribel became pregnant only when she went to Japan. In short, he denies that he is the father of Joanna Rose. He further questions the awarded support of P10,000.00 per month, saying that the same is beyond his means, considering that he has a family to support. We find no merit in this petition. In Alberto v. Court of Appeals,[2] we said: "When a putative father manifests openly through words and deeds his recognition of a child, the courts can do no less than confirm said acknowledgment. As the immortal bard Shakespeare perspicaciously said: 'Let your own discretion be your tutor; suit the action to the word, the word to the action." The evidence in the instant case shows that petitioner considered himself to be the father of Joanna Rose as shown by the hand-written letter he wrote to Maribel: "Hi Love, I wrote you this letter because I would like to erase from your mind the thought of why I can not ever [sic] you marriage right now is because I have no longer love or care for both Joanna & you. Last night when we talked things over, I was in a stage wherein everything was happening so fast that I was running out of time & works (sic) to make you understand me through this letter I would like to explain my side in a more detailed way and I hope you could understand. You know love, the main root of the problem of why marriage is impossible for us right now is not what my parents or my family circle will say about you, but the financial side of it. Okay, let say I did marry you right now disregarding my financial stability. Sooner or later they will come to know of it and I am sure that they will not consent it. I have no alternative but to leave them & to stick it up with you. This is where the financial side comes in. I can't allow myself walking away from my family making them think that I can stand on my own two feet but the truth of the matter is not and seeing both of you suffer for only one stupid mistake which is I was not yet financially ready to face the consequence. My plan is that if you could only stick it out with me until I am ready to face whatever consequence that 806

University of the Cordilleras College of Law First Year C S.Y. 2013 - 2014 might occur during our life or relation as husband and wife. You have already tried it before, why can't you stress it a little longer. In return, I promise to be a loving & caring husband & father to both of you. Love, I really don't want you to be taken away from me by anyone, whether he be single or married. This is the reason why I am still trying to convince you. But if you really have decided things up and really determined to push through with it. I guess I just have to respect your decision. Just remember I wish you the best of luck and take extra-care of yourself & Joanna. Remember, if the time comes when things get rough for you and you have no one to turn to, don't hesitate to call on me. I am very much willing to be at your side to help you. I love you very much! Miso Love, Raymond" (Underscoring supplied by Raymond himself) From the tenor of the letter and the statements petitioner made therein it is clear that, contrary to his vehement assertion that he and Maribel were just friends, they were actually lovers. In an earlier letter, this time sent to Maribel while she was in Japan, petitioner lovingly told her to take care of herself because of her "situation," obviously referring to the state of pregnancy of Maribel: "Aug. 11, 1981 Hi Love, Do you know how glad I was to receive a letter from you yesterday? At least now I'm a little bit at ease to know that everything is fine with you. Love, in your letter you seem so much concern (sic) about my situation once here. I really appreciate it, but please don't give too much thought about it because I'm physically o.k. here. The important thing is that don't think too much and have a lot of rest during your spare time especially in the situation you're in now. If you are feeling homesick just go out with your friends and try to enjoy yourself to the fullest while you are there Love, you said in your letter that you regret very much your going there & wishes (sic) that you have not left anymore. I understand your feelings to what had happened after you told me about it in the telephone. xxx xxx xxx

Love, I miss you so much that I always re-read those letters you had send me very often. At night I always think of you and the times we're together before going to sleep. xxx xxx xxx" (Underscoring supplied)

It was only after petitioner separated from Maribel that he started to deny paternity of Joanna Rose. Until he got married to another woman, he did not object to being identified as Joanna Rose's father as disclosed in the Certificate of Live Birth. The evidence on record reveals that he even got a copy of the said Certificate when Joanna Rose started schooling, as shown by a receipt in his name from the San Juan Municipal Office. His belated denial cannot outweigh the totality of the cogent evidence which establishes beyond reasonable doubt that petitioner is indeed the father of Joanna Rose.[3] Under Article 175 of the Family Code, illegitimate filiation may be established in the same way and on the same evidence as legitimate children. Article 172 of the Family Code states: "The filiation of legitimate children is established by any of the following: '(1) The record of birth appearing in the civil register or a final judgment; or

'(2) An admission of legitimate filiation in a public document or a private handwritten instrument and signed by the parent concerned.' 807

University of the Cordilleras College of Law First Year C S.Y. 2013 - 2014 "In the absence of the foregoing evidence, the legitimate filiation shall be proved by: '(1) '(2) The open and continuous possession of the status of a legitimate child; or Any other means allowed by the Rules of Court and special laws. (265a, 266a, 267a).'"

This article adopts the rule in Article 283 of the Civil Code that filiation may be proven by "any evidence or proof that the defendant is his father."[4] Petitioner has never controverted the evidence on record. His love letters to Maribel vowing to be a good father to Joanna Rose; pictures of himself on various occasions cuddling Joanna Rose and the Certificate of Live Birth say it all. Accordingly, his suit must fail. WHEREFORE, the petition is DISMISSED and the decision of the Court of Appeals is hereby AFFIRMED. Costs against petitioner. SO ORDERED. Case Digest: Pe Lim vs Court of Appeals 270 SCRA 1 G.R. No. 112229 March 18, 1997 ROMERO, J.: Facts: Before us is one of those cases where a man woos a maid, succeeds in seducing and impregnating her, only to disclaim the paternity of the child when made to account for his misdeeds. DNA, being a relatively new science, it has not as yet been accorded official recognition by our courts. Paternity will still have to be resolved by such conventional evidence as the relevant incriminating acts, verbal and written, by the putative father. Issue: Raymond denies being the father of Maribel's child, claiming that they were only friends and nothing more. Ruling: Raymond denies being the father of Maribel's child, claiming that they were only friends and nothing more. The trial court rendered a decision on June 10, 1971, the dispositive portion of which states: "WHEREFORE, judgment is hereby rendered in favor of the plaintiff and against the defendant ordering herein defendant, Raymond Pe Lim to give support to his natural daughter, minor Joanna Rose Pe Lim in the amount of Ten Thousand Pesos (P10,000.00). Philippine Currency, per month for the support, maintenance, education and well-being of said child, the same to be paid on or before the 5th day of each month and monthly thereafter starting June, 1991, until the said minor Joanna Rose Pe Lim, shall have reached the age of majority. The defendant is further ordered to pay the plaintiff the sum of Seven Thousand Five Hundred (P7,500.00) Pesos, Philippine Currency, for attorney's fees and other litigation expenses. No costs.

808

University of the Cordilleras College of Law First Year C S.Y. 2013 - 2014 Mossesgeld vs Court of Appeals 300 SCRA 464 G.R. No. 111455 December 23, 1998 Full Case MARISSA A. MOSSESGELD, petitioner, vs. COURT OF APPEALS and CIVIL REGISTRAR GENERAL, respondents. PARDO, J.: The case is an appeal via certiorari under Rule 45 of the Revised Rules of Court from the decision of the Court of Appeals. 1 affirming that of the Regional Trial Court, Pasig, Branch 69, dismissing the petition of the putative father, later subtituted by the unwed mother, to compel the local civil registrar of Mandaluyong, Metro Manila 2 to register the certificate of live birth of petitioner's illegitimate child using the surname of the presumed father. On December 2, 1989, petitioner Marissa Alfaro Mossesgeld, single, 31 years of age, gave birth to a baby boy at the Medical City General Hospital, Mandaluyong, Metro Manila. 3 It was the third time that she delivered a child. 4 The presumed father, one Eleazar Siriban Calasan, 42 years old, a lawyer, married, and a resident of 8632 San Jose St. Guadalupe Nuevo, Makati, Metro Manila, signed the birth certificate of the child as the informant, indicating therein the child's first name as Jonathan, middle name as Mossesgeld, and last name as Calasan. Both the presumed father, Eleazar S. Calasan and the mother Marissa A. Mossesgeld, accomplished the dorsal side of the certificate of live birth stating that the information contained therein were true and correct. In addition, lawyer Calasan executed an affidavit admitting paternity of the child. 5 On December 6, 1989, due to the refusal of the person in charge at the hospital to placing the presumed father's surname as the child's surname in the certificate of live birth, petitioner himself submitted the certificate to the office of the local civil registrar of Mandaluyong, for registration. On December 28, 1989, the municipal treasurer of Mandaluyong, as officer in charge of the office of the local civil registrar, rejected the registration on the basis of Circular No. 4, dated October 11, 1988, of the Civil Registrar General, providing that under Article 176 of the Family Code of the Philippines, illegitimate children born on or after August 3, 1988, shall use the surname of their mother. 6 On October 9, 1990, lawyer Eleazar S. Calasan personally went to the Local Civil Registrar of Mandaluyong to inquire about the status of the registration of his illegitimate child's certificate of birth, but was furnished with a copy of the letter dated January 17, 1990, of the Civil Registrar General denying registration of the certificate of live birth of petitioner's illegitimate child using the father's surname, for it is contrary to law. 7 On November 7, 1990, lawyer Eleazar S. Calasan filed with the Regional Trial Court, Pasig, Branch 69, a petition for mandamus to compel the Local Civil Registrar of Mandaluyong, Metro Manila, to register the certificate of live birth of his alleged illegitimate son using his surname. 8 On October 29, 1991, the lower court denied the petition, ruling that illegitimate children must use the surname of their mothers, regardless of whether or not they had been acknowledged by their fathers in the record of birth. 9 On November 21, 1991, petitioner Calasan filed a motion for reconsideration of the denial. In the meantime, on December 9, 1991, he filed a motion for leave to amend petition andto admit amended petition, substituting the child's mother Marissa A. Mossesgeld as the petitioner. 10 On February 11, 1992, the lower court granted the motion for leave to amend petition. 11 However, on June 3, 1992, the lower court denied the motion for reconsideration. In due time, petitioner interposed an appeal to the Court of Appeals. On July 23, 1993, the Court of Appeals rendered decision affirming the judgment appealed from. 12 Hence, this petition. 809

University of the Cordilleras College of Law First Year C S.Y. 2013 - 2014 The issue raised is whether mandamus lies to compel the Local Civil Registrar to register a certificate of live birth of an illegitimate child using the alleged father's surname where the latter admitted paternity . We deny the petition. Art. 176 of the Family Code of the Philippines 13 provides that "illegitimate children shall use the surname and shall be under the parental authority of their mother, and shall be entitled to support in conformity with this Code." This is the rule regardless of whether or not the father admits paternity. Consequently, the Local Civil Registrar correctly refused to register the certificate of live birth of petitioner's illegitimate child using the surname of the alleged father, even with the latter's consent. Of course, the putative father, though a much married man, may legally adopt his own illegitimate child. 14 In case of adoption, the child shall be considered a legitimate child of the adopter, entitled to use his surname. 15 The Family Code has effectively repealed the provisions of Article 366 of the Civil Code of the Philippines giving a natural child acknowledge by both parents the right to use the surname of the father. The Family Code has limited the classification of children to legitimate and illegitimate, 16 thereby eliminating the category of acknowledged natural children and natural children by legal fiction. 17 Consequently, we rule that mandamus will not lie to compel the local civil registrar to register the certificate of live birth of an illegitimate child using the father's surname, even with the consent of the latter. Mandamus does not lie to compel the performance of an act prohibited by law. WHEREFORE, the Court DENIES the petition for review on certiorari. We AFFIRM the decision of the Court of Appeals and that of the Regional Trial Court, Pasig, Branch 69, dismissing the petition for mandamus in Special Civil Action No. 60146. Costs against petitioner. SO ORDERED.

810

University of the Cordilleras College of Law First Year C S.Y. 2013 - 2014 Case Digest: Mossesgeld vs Court of Appeals 300 SCRA 464 G.R. No. 111455 December 23, 1998 PARDO, J.: Facts: On December 2, 1989, petitioner Marissa Alfaro Mossesgeld, single, 31 years of age, gave birth to a baby boy at the Medical City General Hospital, Mandaluyong, Metro Manila. 3 It was the third time that she delivered a child. 4 The presumed father, one Eleazar Siriban Calasan, 42 years old, a lawyer, married, and a resident of 8632 San Jose St. Guadalupe Nuevo, Makati, Metro Manila, signed the birth certificate of the child as the informant, indicating therein the child's first name as Jonathan, middle name as Mossesgeld, and last name as Calasan. Both the presumed father, Eleazar S. Calasan and the mother Marissa A. Mossesgeld, accomplished the dorsal side of the certificate of live birth stating that the information contained therein were true and correct. In addition, lawyer Calasan executed an affidavit admitting paternity of the child. On December 6, 1989, due to the refusal of the person in charge at the hospital to placing the presumed father's surname as the child's surname in the certificate of live birth, petitioner himself submitted the certificate to the office of the local civil registrar of Mandaluyong, for registration. On December 28, 1989, the municipal treasurer of Mandaluyong, as officer in charge of the office of the local civil registrar, rejected the registration on the basis of Circular No. 4, dated October 11, 1988, of the Civil Registrar General, providing that under Article 176 of the Family Code of the Philippines, illegitimate children born on or after August 3, 1988, shall use the surname of their mother. Issue: The issue raised is whether mandamus lies to compel the Local Civil Registrar to register a certificate of live birth of an illegitimate child using the alleged father's surname where the latter admitted paternity . We deny the petition. Ruling: The Family Code has effectively repealed the provisions of Article 366 of the Civil Code of the Philippines giving a natural child acknowledge by both parents the right to use the surname of the father. The Family Code has limited the classification of children to legitimate and illegitimate, thereby eliminating the category of acknowledged natural children and natural children by legal fiction. Consequently, we rule that mandamus will not lie to compel the local civil registrar to register the certificate of live birth of an illegitimate child using the father's surname, even with the consent of the latter. Mandamus does not lie to compel the performance of an act prohibited by law. WHEREFORE, the Court DENIES the petition for review on certiorari. We AFFIRM the decision of the Court of Appeals and that of the Regional Trial Court, Pasig, Branch 69, dismissing the petition for mandamus in Special Civil Action No. 60146. Costs against petitioner.

811

University of the Cordilleras College of Law First Year C S.Y. 2013 - 2014 Leonardo vs Court of Appeals G. R. No. 125329. September 10, 2003 Full Case ANN BRIGITT LEONARDO as represented by her parents GLORIA LEONARDO and EDDIE FERNANDEZ, petitioners, vs. COURT OF APPEALS, HON. TOMAS AFRICA, et al., respondents. CARPIO-MORALES, J.: Being assailed in the present petition for review on certiorari is the Court of Appeals Decision of March 11, 1996 and Resolution of May 27, 1996. Petitioner Ann Brigitt Leonardo was on July 14, 1993 born in Manila to common-law-spouses Eddie B. Fernandez and Gloria C. Leonardo.[1] In her birth certificate, her given surname is that of her mother, Leonardo.[2] As petitioners parents later wanted her to carry the surname of her father, the latter executed an affidavit[3] of July 29, 1994 to this effect and wrote a letter[4] of August 1, 1994 to the Local Civil Registrar of Manila requesting for the change of petitioners registered surname. The Local Civil Registrar of Manila Lucena D. Dacuan denied the request of petitioners parents on the ground that petitioner, being illegitimate, should carry her mothers surname as provided under Article 176 of the Family Code[5] which took effect on August 3, 1988.[6] Dacuan also cited Article 412 of the New Civil Code which provides that no entry in the civil register shall be changed or corrected without a judicial order. Petitioners parents appealed the denial of their request for change of petitioners surname to the Civil Registrar General, they citing, among others, the following provision of Title XIII (Use of Surnames), Book I of the New Civil Code: Article 366. A natural child acknowledged by both parents shall principally use the surname of the father. If recognized by only one of the parents, a natural child shall employ the surname of the recognizing parent. (Emphasis and underscoring supplied) Though conceding that the appeal had valid arguments, Civil Registrar General Tomas P. Africa, by letter[7] of December 26, 1994, denied the appeal on the ground that neither the Office of the Civil Registrar General nor any of the Civil Registry Offices in the country is given the power or discretion to effect an administrative change of entry in the civil register. Petitioners parents thus sought before the National Economic and Development Authority (NEDA) the review of the Civil Registrar Generals decision denying their appeal. NEDA Director-General Cielito F. Habito, by letter[8] of March 21, 1995, replied, however, that functionally, his office has no power or authority to review the decision of the Civil Registrar General on matters pertaining to a local Civil Registry. Undaunted, petitioners parents appealed to the Office of the President which, by letter[9] of May 11, 1995, upheld the decision of the Civil Registrar General and the Local Civil Registrar of Manila that the cancellation or correction of entries in the Civil Registry must be brought directly before courts of law. Petitioner, represented by her parents, thereupon filed before the Court of Appeals a Petition for Review[10] under Rule 43 of the Revised Rules of Court raising the following issues: 1. Whether or not Article 176 of the Family Code be given a mandatory application in case a child was born outside of wedlock even though the putative father acknowledges said child as his and agrees and allows his child to bear his surname [and] 2. Whether or not a judicial proceeding is required for the use of [petitioners] surname.

By Decision[11] of March 11, 1996, the Court of Appeals held that Title XIII, Book I of the New Civil 812

University of the Cordilleras College of Law First Year C S.Y. 2013 - 2014 Code on the Use of Surnames was not repealed by the Family Code, citing its repealing clause or Article 254. It held, however, that the Local Civil Registrar of Manila is not allowed to administratively correct the entry in the Civil Registry of the City by deleting and changing petitioners family name LEONARDO to FERNANDEZ upon the submission of an affidavit of her father recognizing her. It went on to declare that petitioner could change her surname by judicial action pursuant to Rule 108 of the Rules of Court. Petitioners motion for reconsideration of the appellate courts decision having been denied by Resolution[12] of May 27, 1996, the present petition raising the following issue was filed: IF PETITIONER, AS HELD IN THE 11 MARCH 1996 DECISION OF THE HONORABLE COURT OF APPEALS, MAY USE HER NATURAL FATHERS SURNAME, THE COROLLARY MATTER TO DETERMINE IN THIS CASE IS WHETHER OR NOT RESORT TO RULE 108 OF THE RULES OF COURT REQUIRING JUDICIAL PROCEEDING AND PUBLICATION, IS THE PROPER ACTION TO BE TAKEN AS DIRECTED IN THE COURT OF APPEALS DECISION TO ENABLE THE PETITIONER TO USE HER NATURAL FATHERS SURNAME.[13] Ubi jus, ibi remedium.[14] When there is a right, there is a remedy. Conversely, if there is no right, there is no remedy as every remedial right is based on a substantive right. In the case at bar, the primary issue to be resolved before determining petitioners available remedy under the facts of the case is whether an illegitimate child born after the effectivity of the Family Code has the right to use her fathers surname. This Court rules in the negative. Article 176 of the Family Code reads: Article 176. Illegitimate children shall use the surname and shall be under the parental authority of their mother, and shall be entitled to support in conformity with this Code. The legitime of each illegitimate child shall consist of one-half of the legitime of a legitimate child. (Emphasis and underscoring supplied) The rule applies even if petitioners father admits paternity. holds:[15] So Mossesgeld v. Court of Appeals

Article 176 of the Family Code of the Philippines provides that illegitimate children shall use the surname and shall be under the parental authority of their mother, and shall be entitled to support in conformity with this Code. This is the rule regardless of whether or not the father admits paternity. Consequently, the Local Civil Registrar correctly refused to register the certificate of live birth of petitioners illegitimate child using the surname of the alleged father, even with the latters consent . . . (Emphasis and underscoring supplied) Contrary to the ruling of the Court of Appeals, Article 176 of the Family Code repealed Title XIII, Book I of the New Civil Code regarding the Use of Surnames. Article 254 of the Family Code reads: Article 254. Titles III, IV, V, VI, VII, VIII, IX, XI and XV of Book I Republic Act 386, otherwise known as the Civil Code of the Philippines, as amended, and Articles 17, 18, 19, 27, 28, 29, 30, 31, 39, 40, 41 and 42 of Presidential Decree No. 603, otherwise known as the Child and Youth Welfare Code, as amended and all laws, decrees, executive orders, proclamations, rules and regulations, or parts thereof, inconsistent herewith are hereby repealed. (Emphasis and underscoring supplied) Thus this Court declared in Mossesgeld: The Family Code has effectively repealed the provisions of Article 366 of the Civil Code of the Philippines giving a natural child acknowledged by both parents the right to use the surname of the father. The Family Code has limited the classification of children to legitimate and illegitimate, thereby eliminating the category of acknowledged natural children and natural children by legal fiction. (Emphasis and underscoring supplied) Since petitioner was born an illegitimate child after the Family Code took effect, she has no right to use her fathers surname. WHEREFORE, upon the ground discussed above, the petition is hereby DENIED.

813

University of the Cordilleras College of Law First Year C S.Y. 2013 - 2014 SO ORDERED.

814

University of the Cordilleras College of Law First Year C S.Y. 2013 - 2014 Case Digest: Leonardo vs Court of Appeals 286 SCRA 495 G. R. No. 125329 September 10, 2003 CARPIO-MORALES, J.: Facts: Petitioner Ann Brigitt Leonardo was on July 14, 1993 born in Manila to common-law-spouses Eddie B. Fernandez and Gloria C. Leonardo.[1] In her birth certificate, her given surname is that of her mother, Leonardo.[2] As petitioners parents later wanted her to carry the surname of her father, the latter executed an affidavit[3] of July 29, 1994 to this effect and wrote a letter[4] of August 1, 1994 to the Local Civil Registrar of Manila requesting for the change of petitioners registered surname. The Local Civil Registrar of Manila Lucena D. Dacuan denied the request of petitioners parents on the ground that petitioner, being illegitimate, should carry her mothers surname as provided under Article 176 of the Family Code[5] which took effect on August 3, 1988.[6] Dacuan also cited Article 412 of the New Civil Code which provides that no entry in the civil register shall be changed or corrected without a judicial order. Issue: In the case at bar, the primary issue to be resolved before determining petitioners available remedy under the facts of the case is whether an illegitimate child born after the effectivity of the Family Code has the right to use her fathers surname. This Court rules in the negative. Ruling: Thus this Court declared in Mossesgeld: The Family Code has effectively repealed the provisions of Article 366 of the Civil Code of the Philippines giving a natural child acknowledged by both parents the right to use the surname of the father. The Family Code has limited the classification of children to legitimate and illegitimate, thereby eliminating the category of acknowledged natural children and natural children by legal fiction. (Emphasis and underscoring supplied) Since petitioner was born an illegitimate child after the Family Code took effect, she has no right to use her fathers surname. WHEREFORE, upon the ground discussed above, the petition is hereby DENIED.

815

University of the Cordilleras College of Law First Year C S.Y. 2013 - 2014 Jison vs Court of Appeals 286 SCRA 495 G.R. No. 124853. February 24, 1998 Full Case FRANCISCO L. JISON, petitioner, vs COURT OF APPEALS and MONINA JISON, respondent. DAVIDE, JR., J.: Decision of the Court of Appeals (CA) in CA-G.R. CV No. 32860[1] which reversed the decision of Branch 24 of the Regional Trial Court (RTC) of Iloilo City in Civil Case No. 16373.[2] The latter dismissed the complaint of private respondent Monina Jison (hereafter MONINA) for recognition as an illegitimate child of petitioner Francisco Jison (hereafter FRANCISCO).This is a petition for review under Rule 45 of the Rules of Court of the 27 April 1995 In issue is whether or not public respondent Court of Appeals committed reversible error, which, in this instance, necessitates an inquiry into the facts. While as a general rule, factual issues are not within the province of this Court, nevertheless, in light of the conflicting findings of facts of the trial court and the Court of Appeals, this case falls under an exception to this rule.[3] In her complaint[4] filed with the RTC on 13 March 1985, MONINA alleged that FRANCISCO had been married to a certain Lilia Lopez Jison since 1940. At the end of 1945 or the start of 1946, however, FRANCISCO impregnated Esperanza F. Amolar (who was then employed as the nanny of FRANCISCO's daughter, Lourdes). As a result, MONINA was born on 6 August 1946, in Dingle, Iloilo, and since childhood, had enjoyed the continuous, implied recognition as an illegitimate child of FRANCISCO by his acts and that of his family. MONINA further alleged that FRANCISCO gave her support and spent for her education, such that she obtained a Master's degree, became a certified public accountant (CPA) and eventually, a Central Bank examiner. In view of FRANCISCO's refusal to expressly recognize her, MONINA prayed for a judicial declaration of her illegitimate status and that FRANCISCO support and treat her as such. In his answer,[5] FRANCISCO alleged that he could not have had sexual relations with Esperanza Amolar during the period specified in the complaint as she had ceased to be in his employ as early as 1944, and did not know of her whereabouts since then; further, he never recognized MONINA, expressly or impliedly, as his illegitimate child. As affirmative and special defenses, FRANCISCO contended that MONINA had no right or cause of action against him and that her action was barred by estoppel, laches and/or prescription. He thus prayed for dismissal of the complaint and an award of damages due to the malicious filing of the complaint. After MONINA filed her reply,[6] pre-trial was conducted where the parties stipulated on the following issues: 1. Did Francisco Jison have any sexual relation[s] with Esperanza Am[o]lar about the end of 1945 or the start of 1946? 2. Is Monina Jison the recognized illegitimate daughter of Francisco Jison by the latter s own acts and those of his family? 3. Is Monina Jison barred from instituting or prosecuting the present action by estoppel, laches and/or prescription? 4. Damages.[7] At trial on the merits, MONINA presented a total of eleven (11) witnesses, namely: herself, Ruben Castellanes, Sr., Adela Casabuena, Arsenio Duatin, Zafiro Ledesma, Danthea Lopez, Romeo Bilbao, Rudy Tingson, Alfredo Baylosis, Dominador Zavariz and Lope Amolar. Ruben Castellanes, Sr., a 63-year old resident of Iloilo City, testified that he had worked for FRANCISCO for a total of six (6) years at Nelly Garden, FRANCISCO's Iloilo residence. Towards the end of the Japanese occupation, FRANCISCOs wife suffered a miscarriage or abortion, thereby 816

University of the Cordilleras College of Law First Year C S.Y. 2013 - 2014 depriving FRANCISCO of consortium; thereafter, FRANCISCOs wife managed a nightclub on the ground floor of Nelly Garden which operated daily from 6:00 p.m. till 3:00 a.m. of the following day, thereby allowing FRANCISCO free access to MONINAs mother, Esperanza Amolar, who was nicknamed Pansay. Adela Casabuena, a 61-year old farmer, testified that she served as the yaya (nanny) of Lourdes from July 1946 up to February 1947. Although Pansay had left Nelly Garden two (2) weeks before Adela started working for the Jisons, Pansay returned sometime in September 1946, or about one month after she gave birth to MONINA, to ask FRANCISCO for support. As a result, Pansay and Lilia Jison, FRANCISCO's wife, quarreled in the living room, and in the course thereof, Pansay claimed that FRANCISCO was the father of her baby. To which, Lilia replied: I did not tell you to make that baby so it is your fault. During the quarrel which lasted from 10:30 till 11:00 a.m., FRANCISCO was supposedly inside the house listening. Arsenio Duatin, a 77-year old retired laborer, testified that from 1947 until 1977, he worked as FRANCISCOs houseboy at the latters house on 12th Street, Capitol Subdivision, Bacolod City. Arsenio met MONINA in 1967, when Felipe Lagarto, the bookkeeper at Nelly Garden, informed Arsenio that MONINA, FRANCISCOs daughter, would arrive at Bacolod City with a letter of introduction from Lagarto. Initially, Arsenio identified seven (7) black-and-white photographs (Exhs. X-5 to X-11) of MONINA,[8] and as he paid for the telephone bills, he likewise identified six (6) telephone cards (Exhs. G to L). Arsenio then declared that when MONINA arrived in Bacolod City, she introduced herself to him as FRANCISCOs daughter. She stayed at FRANCISCOs house, but when the latter and his wife would come over, Arsenio would conceal the presence of MONINA because Mrs. Jison did not like to see her face. Once, Arsenio hid MONINA in the house of FRANCISCOs sister, Mrs. Luisa Jison Alano, in Silay City; another time, at the residence of FRANCISCOs cousin, Mr s. Concha Lopez Cuaycong. Finally, Arsenio declared that the last time he saw MONINA was when she left for Manila, after having finished her schooling at La Salle College in Bacolod City. On re-direct and upon questions by the court, Arsenio disclosed that it was FRANCISCO who instructed that MONINA be hidden whenever FRANCISCO and his wife were around; that although FRANCISCO and MONINA saw each other at the Bacolod house only once, they called each other through long distance; and that MONINA addressed FRANCISCO as Daddy during their lone meeting at the Bacolod house and were affectionate to each other. Arsenio likewise declared that MONINA stayed at FRANCISCO's Bacolod house twice: first for a month, then for about a week the second time. On both occasions, however, FRANCISCO and his wife were abroad. Finally, Arsenio recalled that FRANCISCO likewise bade Arsenio to treat MONINA like his (FRANCISCOs) other daughters. The testimony of Zafiro Ledesma, a 74-year old banker and former mayor of Iloilo City, initially touched on how he and his wife were related to FRANCISCO, FRANCISCO's wife and MONINA. Zafiro first identified Exhibit R, a diagram of the family trees of the Jison and Lopez families, which showed that former Vice-President Fernando Lopez was the first cousin of FRANCISCOs wife, then told the court that the family of Vice-President Lopez treated MONINA very well because she is considered a relative xxx by reputation, by actual perception. Zafiro likewise identified Exhibits X -13 to X-18, photographs taken at the 14 April 1985 birthday celebration of Mrs. Fernando Lopez, which showed MONINA with the former Vice-President and other members of the Lopez family. Zafiro further testified that while MONINA lived with Mrs. Cuaycong, the latter paid for some of MONINAs school needs and even asked MONINA to work in a hospital owned by Mrs. Cuaycong; and that another first cousin of FRANCISCOs wife, a certain Remedios Lopez Franco, likewise helped MONINA with her studies and problems, and even attended MONINAs graduation in 1978 when she obtained a masteral degree in Business Administration, as evidenced by another photograph (Exh. X-12). Moreover, upon Remedios recommendation, MONINA was employed as a secretary at Merchant Financing Company, which was managed by a certain Danthea Lopez, the wife of another first cousin of FRANCISCOs wife, and among whose directors were Zafiro himself, his wife and Dantheas husband. In closing, Zafiro identified MONINAs Social Security Record (Exh. W), which was signed by Danthea as employer and where MONINA designated Remedios as the beneficiary. Danthea Lopez, a 58-year old housekeeper, declared that FRANCISCO was the first cousin of her husband, Eusebio D. Lopez; and that she came to know MONINA in the latter part of 1965 when Remedios Franco recommended MONINA for employment at Merchant Financing Co., which Danthea 817

University of the Cordilleras College of Law First Year C S.Y. 2013 - 2014 managed at that time. Remedios introduced MONINA to Danthea as being reputedly the daughter of Mr. Frank Jison; and on several occasions thereafter, Remedios made Danthea and the latters husband understand that MONINA was reputedly the daughter of [FRANCISCO]. While MONINA worked at Merchant Financing, Danthea knew that MONINA lived with Remedios; however, in the latter part of 1966, as Remedios left for Manila and MONINA was still studying at San Agustin University, Danthea and her husband invited MONINA to live with them. During MONINAs 6-month stay with them, she was not charged for board and lodging and was treated as a relative, not a mere employee, all owing to what Remedios had said regarding MONINAs filiation. As Danthea understood, MONINA resigned from Merchant Financing as she was called by Mrs. Cuaycong, a first cousin of Dantheas husband who lived in Bacolod City. Romeo Bilbao, a 43-year old seaman, testified that he had worked for FRANCISCO from 1969 up to 1980 at Nelly Garden in various capacities: as a procurement officer, hacienda overseer and, later, as hacienda administrator. Sometime in May, 1971, Romeo saw and heard MONINA ask her Daddy (meaning FRANCISCO) for the money he promised to give her, but FRANCISCO answered that he did not have the money to give, then told MONINA to go see Mr. Jose Cruz in Bacolod City. Then in the middle of September that year, FRANCISCO told Romeo to pick up Mr. Cruz at the Iloilo pier and bring him to the office of Atty. Benjamin Tirol. At said office, Atty. Tirol, Mr. Cruz and MONINA entered a room while Romeo waited outside. When they came out, Atty. Tirol had papers for MONINA to sign, but she refused. Atty. Tirol said that a check would be released to MONINA if she signed the papers, so MONINA acceded, although Atty. Tirol intended not to give MONINA a copy of the document she signed. Thereafter, Mr. Cruz gave MONINA a check (Exh. Q), then MONINA grabbed a copy of the document she signed and ran outside. Romeo then brought Mr. Cruz to Nelly Garden. As to his motive for testifying, Romeo stated that he wanted to help MONINA be recognized as FRANCISCOS daughter. Rudy Tingson, a 45-year old antique dealer, testified that in 1963-1964, he was employed by FRANCISCOs wife at the Baguio Military Institute in Baguio City; then in 1965, Rudy worked at FRANCISCOs office at Nelly Garden recording hacienda expenses, typi ng vouchers and office papers, and, at times, acting as paymaster for the haciendas. From the nature of his work, Rudy knew the persons receiving money from FRANCISCOs office, and clearly remembered that in 1965, as part of his job, Rudy gave MONINA her allowance from FRANCISCO four (4) times, upon instructions of a certain Mr. Lagarto to give MONINA P15.00 a month. Rudy likewise recalled that he first met MONINA in 1965, and that she would go to Nelly Garden whenever FRANCISCOs wife was not around. On some of these occasions, MONINA would speak with and address FRANCISCO as Daddy, without objection from FRANCISCO. In fact, in 1965, Rudy saw FRANCISCO give MONINA money thrice. Rudy further declared that in April 1965, FRANCISCOs office paid P250.00 to Funeraria Bernal for the funeral expenses of MONINAs mother. Finally, as to Rudy's motives for testifying, he told the court that he simply wanted to help bring out the truth and nothing but the truth, and that MONINAs filiation was common knowledge among the people in the office at Nelly Garden. On re-direct, Rudy declared that the moneys given by FRANCISCOs office to MONINA were not reflected in the books of the office, but were kept in a separate book, as Mr. Lagarto explained that FRANCISCOs wife and children should not know [of] this. Rudy further revealed that as to the garden meetings between FRANCISCO and MONINA, Rudy saw MONINA kiss FRANCISCO on the cheek both upon arriving and before leaving, and FRANCISCOs reaction upon seeing h er was to smile and say in the Visayan dialect: Kamusta ka iha? (How are you, daughter?); and that MONINA was free to go inside the house as the household staff knew of her filiation, and that, sometimes, MONINA would join them for lunch. Alfredo Baylosis, a 62-year old retired accountant, testified that he worked for FRANCISCO at Central Santos-Lopez in Iloilo from 1951 up to 1961, then at Nelly Garden from 1961 until 1972. Alfredo first served FRANCISCO as a bookkeeper, then when Mr. Lagarto died in 1967 or 1969, Alfredo replaced Mr. Lagarto as office manager. Alfredo knew MONINA since 1961 as she used to go to Nelly Garden to claim her P15.00 monthly allowance given upon FRANCISCOs standing order. Alfredo further declared that MONINAs filiatio n was pretty well-known in the office; that he had seen MONINA and FRANCISCO go from the main building to the office, with FRANCISCOs arm on MONINAs shoulder; and that the office paid for the burial expenses of Pansay, but this was not recorded in the books in order to hide it from FRANCISCOs wife. Alfredo also disclosed that the disbursements for MONINAs allowance started in 1961 and were recorded in a separate cash book. In 1967, the allowances ceased when MONINA stopped schooling and was employed in Bacolod City with Miller, Cruz & Co., which served as FRANCISCOs accountant 818

University of the Cordilleras College of Law First Year C S.Y. 2013 - 2014 auditor. Once, when Alfredo went to the offices of Miller, Cruz & Co. to see the manager, Mr. Atienza, and arrange for the preparation of FRANCISCOs income tax return, Alfredo chanced upon MONINA. When Alfredo asked her how she came to work there, she answered that her Daddy, FRANCISCO, recommended her, a fact confirmed by Mr. Atienza. Alfredo then claimed that Mr. Jose Cruz, a partner at Miller, Cruz & Co., was the most trusted man of FRANCISCO. Dominador Savariz, a 55-year old caretaker, testified that he worked as FRANCISCOs houseboy at Nelly Garden from November 1953 up to 1965. One morning in April 1954, MONINA and her mother Pansay went to Nelly Garden and spoke with FRANCISCO for about an hour, during which time, Dominador was vacuuming the carpet about six (6) to seven (7) meters away. Due to the noise of the vacuum cleaner, FRANCISCO and MONINA spoke in loud voices, thus Dominador overheard their conversation. As FRANCISCO asked Pansay why they came, Pansay answered that they came to ask for the sustenance of his child MONINA. FRANCISCO then touched MONINA's head and asked: How are you Hija?, to which MONINA answered: Good morning, Daddy. After FRANCIS CO told Pansay and MONINA to wait, he pulled something from his wallet and said to Pansay: I am giving this for the child. In May 1954, Dominador saw MONINA at Mr. Lagartos office where Dominador was to get the days expenses, while MONINA was claiming her allowance from Mr. Diasnes. The next month, Dominador saw MONINA at Nelly Garden and heard in the office that MONINA was there to get her allowance from her Daddy. In December 1960, Dominador saw MONINA at Nelly Garden, in the room of Don Vicente (father of FRANCISCOs wife), where she asked for a Christmas gift and she was calling Don Vicente, Lolo (grandfather). At that time, FRANCISCO and his wife were not around. Then sometime in 1961, when Dominador went to Mr. Lagartos office to get the marketing expenses, Dominador saw MONINA once more claiming her allowance. Dominador further testified that in February 1966, after he had stopped working for FRANCISCO, Dominador was at Mrs. Francos residence as she recommended him for employment with her sister, Mrs. Concha Cuaycong. There, he saw MONINA, who was then about 15 years old, together with Mrs. Francos daughter and son. Mrs. Franco pointed at MONINA and asked Dominador if he knew who MONINA was. Dominador answered that MONINA was FRANCISCOs daughter with Pansay, and then Mrs. Franco remarked that MONINA was staying with her (Mrs. Franco) and that she was sending MONINA to school at the University of San Agustin. Lope Amolar, a 50-year old resident of Dingle, Iloilo, and the younger brother of Esperanza Amolar (Pansay), testified that he worked for FRANCISCO as a houseboy from March to November 1945 at Nelly Garden. Thereafter, FRANCISCO sent Lope to work at Elena Apartments in Manila. By November 1945, Pansay was also working at Elena Apartments, where she revealed to Lope that FRANCISCO impregnated her. Lope then confronted FRANCISCO, who told Lope dont get hurt and dont cause any trouble, because I am willing to support your Inday Pansay and my child. Three (3) days after this confrontation, Lope asked for and received permission from FRANCISCO to resign because he (Lope) was hurt. On 21 October 1986, MONINA herself took the witness stand. At that time, she was 40 years old and a Central Bank Examiner. She affirmed that as evidenced by certifications from the Office of the Local Civil Registrar (Exhs. E and F) and baptismal certificates (Exhs. C and D), she was born on 6 August 1946 in Barangay Tabugon, Dingle, Iloilo, to Esperanza Amolar (who passed away on 20 April 1965) and FRANCISCO.[9] MONINA first studied at Sagrado where she stayed as a boarder. While at Sagrado from 1952 until 1955 (up to Grade 4), her father, FRANCISCO, paid for her tuition fees and other school expenses. She either received the money from FRANCISCO or from Mr. Lagarto, or saw FRANCISCO give money to her mother, or Mr. Lagarto would pay Sagrado directly. After Sagrado, MONINA studied in different schools,[10] but FRANCISCO continuously answered for her schooling. For her college education, MONINA enrolled at the University of Iloilo, but she later dropped due to an accident which required a week's hospitalization. Although FRANCISCO paid for part of the hospitalization expenses, her mother shouldered most of them. In 1963, she enrolled at the University of San Agustin, where she stayed with Mrs. Franco who paid for MONINA's tuition fees. However, expenses for books, school supplies, uniforms and the like were shouldered by FRANCISCO. At the start of each semester, MONINA would show FRANCISCO that she was enrolled, then he would ask her to canvass prices, then give her the money she needed. After finishing two (2) semesters at University of San Agustin, as evidenced by her transcript of records (Exh. Z showing that FRANCISCO was listed as Parent/Guardian [Exh. Z-1]), she transferred to De Paul College, just in front of Mrs. Francos house, 819

University of the Cordilleras College of Law First Year C S.Y. 2013 - 2014 and studied there for a year. Thereafter, MONINA enrolled at Western Institute of Technology (WIT), where she obtained a bachelors degree in Commerce in April 1967. During her senior year, she stayed with Eusebio and Danthea Lopez at Hotel Kahirup, owned by said couple. She passed the CPA board exams in 1974, and took up an M.B.A. at De La Salle University as evidenced by her transcript (Exh. AA), wherein FRANCISCO was likewise listed as Guardian (Exhs. AA-1 and AA-2). MONINA enumerated the different members of the household staff at Nelly Garden, to wit: Luz, the household cook; the houseboys Silvestre and Doming; the housemaid Natang; the yaya of the adopted triplets, Deling; the yaya of Lolo Vicente, Adelina; and others. MONINA likewise enumerated the members of the office staff (Messrs. Baylosis, Lagarto, Tingson, Diasnes, Jalandoni, Supertisioso, Doroy, and others), and identified them from a photograph marked as Exhibit X-2. She then corroborated the prior testimony regarding her employment at Merchant Financing Co., and her having lived at Hotel Kahirup and at Mrs. Cuaycongs residence in Bacolod City, while working at the hospital owned by Mrs. Cuaycong. MONINA further testified that in March 1968, she went to Manila and met FRANCISCO at Elena Apartments at the corner of Romero and Salas Streets, Ermita. She told FRANCISCO that she was going for a vacation in Baguio City with Mrs. Francos mother, with whom she stayed up to June 1968. Upon her return from Baguio City, MONINA told FRANCISCO that she wanted to work, so the latter arranged for her employment at Miller & Cruz in Bacolod City. MONINA went to Bacolod City, was interviewed by Mr. Jose Cruz, a partner at Miller & Cruz, who told her she would start working first week of September, sans examination. She resigned from Miller & Cruz in 1971 and lived with Mrs. Cuaycong at her Forbes Park residence in Makati. MONINA went to see FRANCISCO, told him that she resigned and asked him for money to go to Spain, but FRANCISCO refused as she could not speak Spanish and would not be able find a job. The two quarreled and FRANCISCO ordered a helper to send MONINA out of the house. In the process, MONINA broke many glasses at the pantry and cut her hand, after which, FRANCISCO hugged her, gave her medicine, calmed her down, asked her to return to Bacolod City and promised that he would give her the money. MONINA returned to Bacolod City by plane, using a Filipinas Orient Airways plane ticket (Exh. M) which FRANCISCO gave. She called Mr. Cruz, then Atty. Tirol, as instructed by Mr. Cruz. These calls were evidenced by PLDT long distance toll cards (Exhs. G to L), with annotations at the back reading: charged and paid under the name of Frank L. Jison and were signed by Arsenio Duatin (Exhs. G-1 to L1). PLDT issued a certification as to the veracity of the contents of the toll cards (Exh. BB). Likewise introduced in evidence was a letter of introduction prepared by Mr. Cruz addressed to Atty. Tirol, on MONINA's behalf (Exh. N). MONINA also declared that Atty. Tirol then told her that she would have to go to Iloilo and sign a certain affidavit, before Mr. Cruz would turn over the money promised by FRANCISCO. She went to Atty. Tirols office in Iloilo, but after going over the draft of the affidavit, refused to sign it as it stated that she was not FRANCISCOs daughter. She explained that all she had agreed with FRANCISCO was that he would pay for her fare to go abroad, and that since she was a little girl, she knew about her illegitimacy. She started crying, begged Atty. Tirol to change the affidavit, to which Atty. Tirol responded that he was also a father and did not want this to happen to his children as they could not be blamed for being brought into the world. She then wrote a letter (Exh. O) to FRANCISCO and sent it to the latters Forbes Park residence (Bauhinia Place) by JRS courier service (Exhs. O-5 to O-7). MONINA subsequently met FRANCISCO in Bacolod City where they discussed the affidavit which she refused to sign. FRANCISCO told her that the affidavit was for his wife, that in case she heard about MONINA going abroad, the affidavit would keep her peace. MONINA then narrated that the first time she went to Atty. Tirols office, she was accompanied by one Atty. Fernando Divinagracia, who advised her that the affidavit (Exh. P)[11] would boomerang against FRANCISCO as it is contrary to law. MONINA returned to Bacolod City, then met with Atty. Tirol once more to reiterate her plea, but Atty. Tirol did not relent. Thus, on the morning of 20 or 21 September 1971, she signed the affidavit as she was jobless and needed the money to support herself and finish her studies. In exchange for signing the document, MONINA received a Bank of Asia check for P15,000.00 (Exh. Q), which was less than the P25,000.00 which FRANCISCO allegedly promised to give. As Atty. Tirol seemed hesitant to give her a copy of the affidavit after notarizing it, MONINA merely grabbed a copy and immediately left. MONINA then prepared to travel abroad, for which purpose, she procured letters of introduction (Exhs. S and T) from a cousin, Mike Alano (son of FRANCISCOs elder sister Luisa); and an uncle, Emilio Jison 820

University of the Cordilleras College of Law First Year C S.Y. 2013 - 2014 (FRANCISCOs elder brother), addressed to another cousin, Beth Jison (Emilios daughter), for Beth to assist MONINA. Exhibit S contained a statement (Exh. S-1) expressly recognizing that MONINA was FRANCISCOs daughter. Ultimately though, MONINA decided not to go abroad, opting instead to spend the proceeds of the P15,000.00 check for her CPA review, board exam and graduate studies. After finishing her graduate studies, she again planned to travel abroad, for which reason, she obtained a letter of introduction from former Vice President Fernando Lopez addressed to then United States Consul Vernon McAnnich (Exh. V). As to other acts tending to show her filiation, MONINA related that on one occasion, as FRANCISCO s wife was going to arrive at the latters Bacolod City residence, FRANCISCO called Arsenio Duatin and instructed Arsenio to hide MONINA. Thus, MONINA stayed with Mrs. Luisa Jison for the duration of the stay of FRANCISCOs wife. MONINA also claimed that she knew Vice President Fernando Lopez and his wife, Mariquit, even before starting to go to school. Thus, MONINA asked for a recommendation letter (Exh. U) from Mrs. Mariquit Lopez for possible employment with Mrs. Rosario Lopez Cooper, another second cousin of FRANCISCO. In Exhibit U, Mrs. Lopez expressly recognized MONINA as FRANCISCOs daughter. As additional proof of her close relationship with the family of Vice President Lopez, MONINA identified photographs taken at a birthday celebration on 14 April 1985. MONINA finally claimed that she knew the three (3) children of FRANCISCO by wife, namely, Lourdes, Francisco, Jr. (Junior) and Elena, but MONINA had met only Lourdes and Junior. MONINA's testimony dealt lengthily on her dealings with Junior and the two (2) occasions when she met with Lourdes. The last time MONINA saw FRANCISCO was in March 1979, when she sought his blessings to get married. In his defense, FRANCISCO offered his deposition taken before then Judge Romeo Callejo of the Regional Trial Court of Manila, Branch 48. As additional witnesses, FRANCISCO presented Nonito Jalandoni, Teodoro Zulla, Iigo Supertisioso, Lourdes Ledesma, Jose Cruz and Dolores Argenal. FRANCISCO declared that Pansays employment ceased as of October, 1944, and that while employed by him, Pansay would sleep with the other female helpers on the first floor of his residence, while he, his wife and daughter slept in a room on the second floor. At that time, his household staff was composed of three (3) female workers and two (2) male workers. After Pansay left in October 1944, she never communicated with him again, neither did he know of her whereabouts. FRANCISCO staunchly denied having had sexual relations with Pansay and disavowed any knowledge about MONINAs birth. In the same vein, he denied having paid for MONINAs tuition fees, in person or otherwise, and asserted that he never knew that Mr. Lagarto paid for these fees. Moreover, FRANCISCO could not believe that Lagarto would pay for these fees despite absence of instructions or approval from FRANCISCO. He likewise categorically denied that he told anyone, be it Danthea Lopez, Zafiro Ledesma, Concha Cuaycong or Remedios Franco, that MONINA was his daughter. FRANCISCO also disclosed that upon his return from the United States in 1971, he fired Alfredo Baylosis upon discovering that Alfredo had taken advantage of his position during the formers absence. FRANCISCO likewise fired Rudy Tingson and Romeo Bilbao, but did not give the reasons therefor. Finally, FRANCISCO denied knowledge of MONINAs long distance calls from his Bacolod residence; nevertheless, when he subsequently discovered this, he fired certain people in his office for their failure to report this anomaly. As regards the caretaker of his Bacolod residence, FRANCISCO explained that since MONINA lived at Mrs. Cuaycongs residence, the caretaker thought that he could allow people who lived at the Cuaycong residence to use the facilities at his (FRANCISCOs) house. Nonito Jalandoni, bookkeeper and paymaster at Nellys Garden from 1963 up to 1974, then from 1980 up to 1986, the assistant overseer of Hacienda Lopez, testified that he did not know MONINA; that he learned of her only in June 1988, when he was informed by FRANCISCO that MONINA had sued him; and that he never saw MONINA at Nellys Garden, neither did he know of any instructions for anyone at Nellys Garden to give money to MONINA. Teodoro Zulla, FRANCISCOs bookkeeper and paymaster from 1951 up to 1986, testified that FRANCISCO dismissed Alfredo Baylosis due to certain unspecified discrepancies; and that he never saw MONINA receive funds from either Mr. Lagarto or Mr. Baylosis. Upon questions from the trial court, however, Teodoro admitted that he prepared vouchers for only one of FRANCISCOs haciendas, and not vouchers pertaining to the latters personal expenses. Iigo Supertisioso testified that he worked for FRANCISCO at Nellys Garden from 1964 up to 1984 as a 821

University of the Cordilleras College of Law First Year C S.Y. 2013 - 2014 field inspector, paymaster, cashier and, eventually, officer-in-charge (OIC). He confirmed Alfredo Baylosis dismissal due to these unspecified irregularities, then denied that FRANCISCO ever ordered that MONINA be given her allowance. Likewise, Iigo never heard FRANCISCO mention that MONINA was his (FRANCISCOs) daughter. Lourdes Ledesma, FRANCISCOs daughter, testified that she saw (but did not know) MONINA at the Our Lady of Mercy Hospital, on the occasion of the birth of Lourdes first son, Mark. Over lunch one day, Lourdes aunt casually introduced Lourdes and MONINA to each other, but they were referred to only by their first names. Then sometime in 1983 or 1984, MONINA allegedly went to Lourdes house in Sta. Clara Subdivision requesting for a letter of introduction or referral as MONINA was then jobhunting. However, Lourdes did not comply with the request. Jose Cruz, a partner at Miller, Cruz & Co., testified that MONINA worked at Miller & Cruz from 1968 up to 1971, however, he did not personally interview her before she was accepted for employment. Moreover, MONINA underwent the usual screening procedure before being hired. Jose recalled that one of the accountants, a certain Mr. Atienza, reported that MONINA claimed to be FRANCISCOs daughter. Jose then told Mr. Atienza to speak with MONINA and see if he (Mr. Atienza) could stop her from spreading this rumor. Mr. Atienza reported that he spoke with MONINA, who told him that she planned to leave for the United States and needed P20,000.00 for that purpose, and in exchange, she would sign a document disclaiming filiation with FRANCISCO. Thus, Jose instructed Mr. Atienza to request that MONINA meet with Jose, and at that meeting, MONINA confirmed Mr. Atienzas report. Jose then informed Atty. Tirol, FRANCISCOs personal lawyer, about the matter. Atty. Tirol told Jose to send MONINA and her lawyer to his (Atty. Tirols) office in Iloilo. Jose then wrote out a letter of introduction for MONINA addressed to Atty. Tirol. Jose relayed Atty. Tirols message to MONINA through Mr. Atienza, then later, Atty. Tirol told Jose to go to Iloilo with a check for P15,000.00. Jose complied, and at Atty. Tirols office, Jose saw MONINA, Atty. Tirol and his secretary reading some documents. MONINA then expressed her willingness to sign the document, sans revisions. Jose alleged that he drew the P15,000.00 from his personal funds, subject to reimbursement from and due to an understanding with FRANCISCO. Dolores Argenal, a househelper at Nelly Garden from May 1944 up to May 1946, testified that she knew that Pansay was Lourdes nanny; that Lourdes slept in her parents room; that she had not seen FRANCISCO give special treatment to Pansay; that there was no unusual relationship between FRANCISCO and Pansay, and if there was any, Dolores would have easily detected it since she slept in the same room as Pansay. Dolores further declared that whenever FRANCISCOs wife was out of town, Pansay would bring Lourdes downstairs at nighttime, and that Pansay would not sleep in the room where FRANCISCO slept. Finally, Dolores declared that Pansay stopped working for FRANCISCO and his wife in October, 1944. The reception of evidence having been concluded, the parties filed their respective memoranda. It need be recalled that Judge Catalino Castaeda, Jr. presided over trial up to 21 October 1986, thereby hearing only the testimonies of MONINAs witnesses and about half of MONINAs testimony on direct examination. Judge Norberto E. Devera, Jr. heard the rest of MONINA's testimony and those of FRANCISCOs witnesses. In its decision of 12 November 1990[12] the trial court, through Judge Devera, dismissed the complaint with costs against MONINA. In the opening paragraph thereof, it observed: This is a complaint for recognition of an illegitimate child instituted by plaintiff Monina Jison against defendant Francisco Jison. This complaint was filed on March 13, 1985 at the time when plaintiff, reckoned from her death of birth, was already thirty-nine years old. Noteworthy also is the fact that it was instituted twenty years after the death of plaintiffs mother, Esperanza Amolar. For the years between plaintiffs birth and Esperanzas death, no action of any kind was instituted against defendant either by plaintiff, her mother Esperanza or the latters parents. Neither had plaintiff brought such an action against defendant immediately upon her mothers death on April 20, 1965, considering that she was then already nineteen years old or, within a reasonable time thereafter. Twenty years more had to supervene before this complaint was eventually instituted. The trial court then proceeded to discuss the four issues stipulated at pre-trial, without, however, summarizing the testimonies of the witnesses nor referring to the testimonies of the witnesses other than 822

University of the Cordilleras College of Law First Year C S.Y. 2013 - 2014 those mentioned in the discussion of the issues. The trial court resolved the first issue in the negative, holding that it was improbable for witness Lope Amolar to have noticed that Pansay was pregnant upon seeing her at the Elena Apartments in November 1945, since Pansay was then only in her first month of pregnancy; that there was no positive assertion that copulation did indeed take place between Francisco and Esperanza; and that MONINAs attempt to show opportunity on the part of FRANCISCO failed to consider that there was also the opportunity for copulation between Esperanza and one of the several domestic helpers admittedly also residing at Nellys Garden at that time. The RTC also ruled that the probative value of the birth and bapt ismal certificates of MONINA paled in light of jurisprudence, especially when the misspellings therein were considered. The trial court likewise resolved the second issue in the negative, finding that MONINAs evidence thereon may either be one of three categories, namely: hearsay evidence, incredulous evidence, or selfserving evidence." To the first category belonged the testimonies of Adela Casabuena and Alfredo Baylosis, whose knowledge of MONINAs filiation was based, as to the former, on utteranc es of defendants wife Lilia and Esperanza allegedly during the heat of their quarrel, while as to the latter, Alfredo's conclusion was based from the rumors going [around] that plaintiff is defendants daughter, from his personal observation of plaintiffs facial appearance which he compared with that of defendants and from the way the two (plaintiff and defendant) acted and treated each other on one occasion that he had then opportunity to closely observe them together. To the second category belonge d that of Dominador Savariz, as: At each precise time that Esperanza allegedly visited Nellys Garden and allegedly on those occasions when defendants wife, Lilia was in Manila, this witness was there and allegedly heard pieces of conversation between defendant and Esperanza related to the paternity of the latters child. xxx The RTC then placed MONINAs testimony regarding the acts of recognition accorded her by FRANCISCOs relatives under the third category, since the latter were never presented as w itnesses, for which reason the trial court excluded the letters from FRANCISCOs relatives (Exhs. S to V). As to the third issue, the trial court held that MONINA was not barred by prescription for it was of the perception that the benefits of Article 268 accorded to legitimate children may be availed of or extended to illegitimate children in the same manner as the Family Code has so provided; or by laches, which is [a] creation of equity applied only to bring equitable results, and addressed to th e sound discretion of the court [and] the circumstances [here] would show that whether plaintiff filed this case immediately upon the death of her mother Esperanza in 1965 or twenty years thereafter in 1985, xxx there seems to be no inequitable result to defendant as related to the situation of plaintiff. The RTC ruled, however, that MONINA was barred by estoppel by deed because of the affidavit (Exh. P/Exh. 2) which she signed when she was already twenty-five years, a professional and under the able guidance of counsel. Finally, the RTC denied FRANCISCOs claim for damages, finding that MONINA did not file the complaint with malice, she having been propelled by an honest belief, founded on probable cause. MONINA seasonably appealed to the Court of Appeals (CA-G.R. CV No. 32860) and sought reversal of the trial courts decision on the grounds that: I THE TRIAL COURT WAS ERRONEOUSLY PREDISPOSED TO ADJUDGE THIS CASE AGAINST APPELLANT DUE TO ITS MISPERCEPTION THAT APPELLANTS DELAY IN FILING HER COMPLAINT WAS FATAL TO HER CASE. II THE TRIAL COURT ERRED IN ITS REJECTION OF THE TESTIMONIES OF APPELLANTS WITNESSES AS TAILOR-MADE, INADEQUATE AND INCREDIBLE. III

823

University of the Cordilleras College of Law First Year C S.Y. 2013 - 2014 THE TRIAL COURT ERRED IN ITS REJECTION OF THE ADMISSIBILITY OF THE CERTIFIED COPIES OF PUBLIC DOCUMENTS PRESENTED BY APPELLANT AS PART OF HER EVIDENCE. IV THE TRIAL COURT ERRED IN ITS REQUIREMENT THAT A WITNESS TO THE ACTUAL ACT OF COPULATION BETWEEN THE APPELLEE AND APPELLANTS MOTHER SHOULD HAVE POSITIVELY TESTIFIED TO SAID EFFECT. V THE TRIAL COURT ERRED IN REJECTING THE ADMISSIBILITY OF THE DULY IDENTIFIED NOTES AND LETTER OF THE RELATIVES OF THE APPELLEE AS HEARSAY. VI THE TRIAL COURT ERRED IN CONCLUDING THAT APPELLANTS AFFIDAVIT (EXH. P) SERVED AS A BAR AGAINST HER CLAIM FOR RECOGNITION INSTEAD OF REINFORCING SAID CLAIM.[13] Expectedly, FRANCISCO refuted these alleged errors in his Appellees Brief.[14] In its decision of 27 April 1995,[15] the Court of Appeals initially declared that as no vested or acquired rights were affected, the instant case was governed by Article 175, in relation to Articles 172 and 173, of the Family Code.[16] While the Court of Appeals rejected the certifications issued by the Local Civil Registrar of Dingle, Iloilo (Exhs. E and F) as FRANCISCO did not sign them, said court focused its discussion on the other means by which illegitimate filiation could be proved, i.e., the open and continuous possession of the status of an illegitimate child or, by any other means allowed by the Rules of Court and special laws, such as the baptismal certificate of the child, a judicial admission, a family bible wherein the name of the child is entered, common reputation respecting pedigree, admission by silence, testimonies of witnesses xxx.[17] To the Court of Appeals, the bottom line issue was whether or not MONINA established her filiation as FRANCISCOs illegitimate daughter by preponderance of evidence, as to which issue said court found: [N]ot just preponderant but overwhelming evidence on record to prove that [MONINA] is the illegitimate daughter of [FRANCISCO] and that she had continuously enjoyed such status by direct acts of [FRANCISCO] and/or his relatives. In so ruling, the Court of Appeals observed that the testimonies of Lope Amolar, Adela Casabuena and Dominador Savariz were already sufficient to establish MONINAs filiation: As adverted to earlier, the trial court discredited Lope Amolars testimony by saying that Lope could not have detected Esperanzas pregnant state in November, 1945 since at that poi nt in time [sic] she was still in the initial stage of pregnancy. Apparently, the trial court paid more emphasis on the date mentioned by Lope Amolar than on the tenor and import of his testimony. As xxx Lope xxx was asked about an incident that transpired more than 41 years back, [u]nder the circumstances, it is unreasonable to expect that Lope could still be dead right on the specific month in 1945 that [he] met and confronted his sister. At any rate, what is important is not the month that they met but the essence of his testimony that his sister pointed to their employer [FRANCISCO] as the one responsible for her pregnancy, and that upon being confronted, [FRANCISCO] assured him of support for Esperanza and their child. It would appear then that in an attempt to find fault with Lopes testimony, the trial court has fallen oblivious to the fact that even [FRANCISCO], in his deposition, did not deny that he was confronted by Lope about what he had done to Esperanza, during which he unequivocally acknowledged paternity by assuring Lope of support for both Esperanza and their child. The Court of Appelas further noted that Casabuena and Savariz testified on something that they personally observed or witnessed, which matters FRANCISCO did not deny or refute. Finally, said court aptly held: Taking into account all the foregoing uncontroverted testimonies xxx let alone such circumstantial evidence as [MONINAs] Birth Certificates xxx and Baptismal Certificates which invariably bear the name of [FRANCISCO] as her father, We cannot go along with the trial courts theory that [MONINAs] 824

University of the Cordilleras College of Law First Year C S.Y. 2013 - 2014 illegitimate filiation has not been satisfactorily established. xxx Significantly, [MONINAs] testimony finds ample corroboration from [FRANCISCOs] former employees, Arsenio Duatin, Rudy Tingson and Alfredo Baylosis. xxx xxx Carefully evaluating appellants evidence on her enjoyment of the status of an illegitimate daughter of [FRANCISCO] vis-a-vis [FRANCISCOs] controversion thereof, We find more weight in the former. The positive testimonies of [MONINA] and [her] witnesses xxx all bearing on [FRANCISCOs] acts and/or conduct indubitably showing that he had continuously acknowledged [MONINA] as his illegitimate daughter have not been succeessfully [sic] refuted. In fact, [FRANCISCO] himself, in his deposition, only casually dismissed [MONINAs] exhaustive and detailed testimony as untrue, and with respect to those given by [MONINAs] witnesses, he merely explained that he had fired [them] from their employment. Needless to state, [FRANCISCOs] vague denial is grossly inadequate to overcome the probative weight of [MONINAs] testimonial evidence. Even the affidavit (Exh 2) which [FRANCISCO] had foisted on the trial court xxx does not hold sway in the face of [MONINAs] logical explanation that she at first did agree to sign the affidavit which contained untruthful statements. In fact, she promptly complained to [FRANCISCO] who, however explained to her that the affidavit was only for the consumption of his spouse xxx. Further, the testimony of Jose Cruz concerning the events that led to the execution of the affidavit xxx could not have been true, for as pointed out by [MONINA], she signed the affidavit xxx almost five months after she had resigned from the Miller, Cruz & Co. xxx At any rate, if [MONINA] were not his illegitimate daughter, it would have been uncalled for, if not absurd, for [FRANCISCO] or his lawyer to have secured [MONINAs] sworn statement xxx On the contrary, in asking [MONINA] to sign the said affidavit at the cost of P15,000, [FRANCISCO] clearly betrayed his intention to conceal or suppress his paternity of [MONINA]. xxx In fine, We hold that [MONINAs] filiation as [FRANCISCOs] illegitimate daughter has been conclusively established by the uncontroverted testimonies of Lope Amolar, Adela Casabuena and Dominador Savariz to the effect that appellee himself had admitted his paternity of the appellee, and also by the testimonies of appellant, Arsenio Duatin, Romeo Bilbao, Rudy Tingson and Alfredo Baylosis unerringly demonstrating that by his own conduct or overt acts like sending appellant to school, paying for her tuition fees, school uniforms, books, board and lodging at the Colegio del Sagrado Corazon de Jesus, defraying appellants hospitalization expenses, providing her with [a] monthly allowance, paying for the funeral expenses of appellants mother, acknowledging appellants paternal greetings and calling appellant his Hija or child, instructing his office personnel to give appellants monthl y allowance, recommending appellant for employment at the Miller, Cruz & Co., allowing appellant to use his house in Bacolod and paying for her long distance telephone calls, having appellant spend her vacation in his apartment in Manila and also at his Forbes residence, allowing appellant to use his surname in her scholastic and other records (Exhs Z, AA, AA-1 to AA-5, W & W-5), appellee had continuously recognized appellant as his illegitimate daughter. Added to these are the acts of [FRANCISCOs] relatives acknowledging or treating [MONINA] as [FRANCISCOs] daughter (Exh U) or as their relative (Exhs T & V). On this point, witness Zafiro Ledesma, former Mayor of Iloilo City, whose spouse belongs to the Lopez clan just like [FRANCISCO], testified that [MONINA] has been considered by the Lopezes as a relative. He identified pictures of the appellee in the company of the Lopezes (Exhs X-16 & X-17). Another witness, Danthea H. Lopez, whose husband Eusebio Lopez is appellees first cousin, testified that appellant was introduced to her by appellees cousin, Remedios Lopez Franco, as the daughter of appellee Francisco Jison, for which reason, she took her in as [a] secretary in the Merchants Financing Corporation of which she was the manager, and further allowed her to stay with her family free of board and lodging. Still on this aspect, Dominador Savariz declared that sometime in February, 1966 appellees relative, Ms. Remedios Lopez Franco pointed to appellant as the daughter of appellee Francisco Jison. Finally, the Certifications of the Local Civil Registrar of Dingle (Exhs E and F) as well as [MONINAs] Baptismal Certificates (Exhs C & D) which the trial ocurt admitted in evidence as part of [MONINAs] testimony, may serve as circumstantial evidence to further reinforce [MONINAs] claim that she is [FRANCISCOs] illegitimate daughter by Esperanza Amolar. 825

University of the Cordilleras College of Law First Year C S.Y. 2013 - 2014 True it is that a trial judges assessment of the credibility of witnesses is accorded great respect on appeal. But the rule admits of certain exceptions. One such exception is where the judge who rendered the judgment was not the one who heard the witnesses testify. [citations omitted] The other is where the trial court had overlooked, misunderstood or misappreciated some facts or circumstances of weight and substance which, if properly considered, might affect the result of the case. [citations omitted] In the present case, both exceptions obtain. All of [MONINAs] witnesses xxx whose testimonies were not given credence did not testify before the judge who rendered the disputed judgment. xxx The Court of Appeals then decreed: WHEREFORE, premises considered, the judgment of the trial court is SET ASIDE and another one is hereby entered for appellant Monina Jison, declaring her as the illegitimate daughter of appellee Francisco Jison, and entitled to all rights and privileges granted by law. Costs against appellee. SO ORDERED. His motion for reconsideration having been denied by the Court of Appeals in its resolution of 29 March 1996,[18] FRANCISCO filed the instant petition. He urges us to reverse the judgment of the Court of Appeals, alleging that said court committed errors of law: I. IN REVERSING THE DECISION OF THE TRIAL COURT AND DECLARING PRIVATE RESPONDENT AS THE ILLEGITIMATE CHILD OF PETITIONER, CONSIDERING [THE] IMPOSSIBILITY OF SEXUAL CONTACT BETWEEN THE PETITIONER AND THE PRIVATE RESPONDENT'S MOTHER AT THE TIME CONCEPTION WAS SUPPOSED TO HAVE OCCURRED. II. IN REVERSING THE TRIAL COURTS FINDING CONSIDERING THAT PRIVATE RESPONDENT'S TESTIMONIAL EVIDENCE OF PATERNITY AND FILIATION IS NOT CLEAR AND CONVINCING. III. IN GIVING CREDENCE TO DOCUMENTARY EVIDENCE PRESENTED BY THE PRIVATE RESPONDENT AS EVIDENCE OF FILIATION CONSIDERING THAT THE SAME ARE HEARSAY, SELF-SERVING AND CANNOT BIND THE PETITIONER UNDER THE BASIC RULES OF EVIDENCE. IV. IN INTERPRETING THE PRIVATE RESPONDENT'S SWORN STATEMENT (EXH. P/EXH. 2) IN A MANNER NOT IN CONSONANCE WITH THE RULINGS OF THE HONORABLE SUPREME COURT. V. IN NOT CONSIDERING THE LONG AND UNEXPLAINED DELAY IN THE FILING OF THE PRESENT PATERNITY SUIT AS EQUIVALENT TO LACHES. As regards the first error, FRANCISCO insists that taking into account the second paragraph of MONINAs complaint wherein she claimed that he and Pansay had sexual relations by about the end of 1945 or the start of 1946, it was physically impossible for him and Pansay to have had sexual contact which resulted in MONINAs birth, considering that: The normal period of human pregnancy is nine (9) months. If as claimed by private respondent in her complaint that her mother was impregnated by FRANCISCO at the end of 1945 or the start of 1946, 826

University of the Cordilleras College of Law First Year C S.Y. 2013 - 2014 she would have been born sometime in late September or early October and not August 6, 1946 xxx. The instant case finds factual and legal parallels in Constantino vs. Mendez,[19] thus: xxx FRANCISCO further claims that his testimony that Pansay was no longer employed by him at the time in question was unrebutted, moreover, other men had access to Pansay during the time of or even after her employment by him. As to the second error, FRANCISCO submits that MONINAs testimonial evidence is shaky, contradictory and unreliable, and proceeds to attack the credibility of her witnesses by claiming, in the main, that: (a) Lope Amolar could not have detected Pansays pregnancy in November 1945 when they met since she would have been only one (1) month pregnant then; (b) Dominador Savariz did not in fact witness the meeting between FRANCISCO, Pansay and MONINA; (c) Zafiro Ledesma had an ulterior motive in testifying for MONINA as he owned a bank in Iloilo which was then under Central Bank supervision and MONINA was the Bank Examiner assigned to Iloilo; and (d) Danthea Lopez was not related to him by blood and whatever favorable treatment MONINA received from Danthea was due to the formers employment at Merchants Financing Company and additional services rendered at Kahirup Hotel; besides, Danthea admitted that she had no personal knowledge as to the issue of paternity and filiation of the contending parties, hence Sections 39 and 40[20] of Rule 130 of the Rules of Court did not come into play. FRANCISCO likewise re-echoes the view of the trial court as regards the testimonies of Adela Casabuena and Alfredo Baylosis. FRANCISCO further asserts that MONINAs testimony that he answered for her schooling was self serving and uncorroborated by any receipt or other documentary evidence; and assuming he did, such should be interpreted as a manifestation of kindness shown towards the family of a former household helper. Anent the treatment given by his relatives to MONINA as his daughter, FRANCISCO points to the fact that Pansay was the former laundrywoman of Mrs. Franco; MONINA resided with the families of Eusebio Lopez and Concha Cuaycong because she was in their employ at Kahirup Hotel and Our Lady of Mercy Hospital, respectively; MONINA failed to present Mrs. Franco, Eusebio Lopez and Mrs. Cuaycong; and MONINAs employment at the accounting firm of Miller, Cruz & Co. was attributable to her educational attainment, there being absolutely no evidence to prove that FRANCISCO ever facilitated her employment thereat. Hence, in light of Baluyot v. Baluyot,[21] the quantum of evidence to prove paternity by clear and convincing evidence, not merely a preponderance thereof, was not met. With respect to the third assigned error, FRANCISCO argues that the Court of Appeals reliance on the certifications of the Local Civil Registrar (Exhs. E and F) and Baptismal Certificates (Exhs. C and D) as circumstantial evidence is misplaced. First, their genuineness could not be ascertained as the persons who issued them did not testify. Second, in light of Reyes v. Court of Appeals,[22] the contents of the baptismal certificates were hearsay, as the data was based only on what was told to the priest who solemnized the baptism, who likewise was not presented as a witness. Additionally, the name of the father appearing therein was Franque Jison, which was not FRANCISCOs name. Third, in both Exhibits E and F, the names of the childs parents were listed as Frank Heson and Esperanza Amador (not Amolar). FRANCISCO further points out that in Exhibit F, the status of the child is listed as legitimate, while the fathers occupation as laborer. Most importantly, there was no showing t hat FRANCISCO signed Exhibits E and F or that he was the one who reported the childs birth to the Office of the Local Civil Registrar. As to MONINAs educational records, FRANCISCO invokes Baas v. Baas[23] which recognized that school records are prepared by school authorities, not by putative parents, thus incompetent to prove paternity. And, as to the photographs presented by MONINA, FRANCISCO cites Colorado v. Court of Appeals,[24] and further asserts that MONINA did not present any of the persons with whom she is seen in the pictures to testify thereon; besides these persons were, at best, mere second cousins of FRANCISCO. He likewise assails the various notes and letters written by his relatives (Exhs. S to V) as they were not identified by the authors. Finally, he stresses that MONINA did not testify as to the telephone cards (Exhs. G to L) nor did these reveal the circumstances surrounding the calls she made from his residence. Anent the fourth assigned error, FRANCISCO contends that the Court of Appeals interpretation of MONINAs affidavit of 21 September 1971 ran counter to Dequito v. Llamas,[25] and overlooked that at the time of execution, MONINA was more than 25 years old and assisted by counsel. As to the last assigned error, FRANCISCO bewails the Court of Appeals failure to consider the long and unexplained delay in the filing of the case. 827

University of the Cordilleras College of Law First Year C S.Y. 2013 - 2014 In her comment, MONINA forcefully refuted FRANCISCOs arguments, leading FRANCISCO to file his reply thereto. On 20 November 1996, we gave due course to this petition and required the parties to submit their respective memoranda, which they subsequently did. A painstaking review of the evidence and arguments fails to support petitioner. Before addressing the merits of the controversy, we first dispose of preliminary matters relating to the applicable law and the guiding principles in paternity suits. As to the former, plainly, the Family Code of the Philippines (Executive Order No. 209) governs the present controversy. As correctly cited by the Court of Appeals, Uyguangco[26] served as a judicial confirmation of Article 256 of the Family Code[27] regarding its retroactive effect unless there be impairment of vested rights, which does not hold true here, it appearing that neither the putative parent nor the child has passed away and the former having actually resisted the latters claim below. Under Article 175 of the Family Code, illegitimate filiation, such as MONINA's, may be established in the same way and on the same evidence as that of legitimate children. Article 172 thereof provides the various forms of evidence by which legitimate filiation is established, thus: ART. 172. The filiation of legitimate children is established by any of the following: (1) The record of birth appearing in the civil register or a final judgment; or

(2) An admission of legitimate filiation in a public document or a private handwritten instrument signed by the parent concerned. In the absence of the foregoing evidence, the legitimate filiation shall be proved by: (1) (2) The open and continuous possession of the status of a legitimate child; or Any other means allowed by the Rules of Court and special laws.

This Article reproduces, with amendments, Articles 265, 266 and 267 of the Civil Code. For the success of an action to establish illegitimate filiation under the second paragraph, which MONINA relies upon given that she has none of the evidence mentioned in the first paragraph, a high standard of proof[28] is required. Specifically, to prove open and continuous possession of the status of an illegitimate child, there must be evidence of the manifestation of the permanent intention of the supposed father to consider the child as his, by continuous and clear manifestations of parental affection and care, which cannot be attributed to pure charity. Such acts must be of such a nature that they reveal not only the conviction of paternity, but also the apparent desire to have and treat the child as such in all relations in society and in life, not accidentally, but continuously.[29] By continuous is meant uninterrupted and consistent, but does not require any particular length of time.[30] The foregoing standard of proof required to establish ones filiation is founded on the principle that an order for recognition and support may create an unwholesome atmosphere or may be an irritant in the family or lives of the parties, so that it must be issued only if paternity or filiation is established by clear and convincing evidence.[31] The foregoing discussion, however, must be situated within the general rules on evidence, in light of the burden of proof in civil cases, i.e., preponderance of evidence, and the shifting of the burden of evidence in such cases. Simply put, he who alleges the affirmative of the issue has the burden of proof, and upon the plaintiff in a civil case, the burden of proof never parts. However, in the course of trial in a civil case, once plaintiff makes out a prima facie case in his favor, the duty or the burden of evidence shifts to defendant to controvert plaintiffs prima facie case, otherwise, a verdict must be returned in favor of plaintiff. Moreover, in civil cases, the party having the burden of proof must produce a preponderance of evidence thereon, with plaintiff having to rely on the strength of his own evidence and not upon the weakness of the defendants. The concept of preponderance of evidence refers to evidence which is of 828

University of the Cordilleras College of Law First Year C S.Y. 2013 - 2014 greater weight, or more convincing, that which is offered in opposition to it; at bottom, it means probability of truth.[32] With these in mind, we now proceed to resolve the merits of the instant controversy. FRANCISCOs arguments in support of his first assigned error deserve scant consideration. While it has been observed that unlawful intercourse will not be presumed merely from proof of an opportunity for such indulgence,[33] this does not favor FRANCISCO. Akin to the crime of rape where, in most instances, the only witnesses to the felony are the participants in the sexual act themselves, in deciding paternity suits, the issue of whether sexual intercourse actually occurred inevitably redounds to the victims or mothers word, as against the accuseds or putative fathers protestations. In the instant case, MONINAs mother could no longer testify as to the fact of intercourse, as she had, unfortunately, passed away long before the institution of the complaint for recognition. But this did not mean that MONINA could no longer prove her filiation. The fact of her birth and her parentage may be established by evidence other than the testimony of her mother. The paramount question then is whether MONINAs evidence is coherent, logical and natural.[34] The complaint stated that FRANCISCO had carnal knowledge of Pansay by about the end of 1945. We agree with MONINA that this was broad enough to cover the fourth quarter of said year, hence her birth on 6 August 1946 could still be attributed to sexual relations between FRANCISCO and MONINAs mother. In any event, since it was established that her mother was still in the employ of FRANCISCO at the time MONINA was conceived as determined by the date of her birth, sexual contact between FRANCISCO and MONINAs mother was not at all impossible, especially in light of the overw helming evidence, as hereafter shown, that FRANCISCO fathered MONINA, has recognized her as his daughter and that MONINA has been enjoying the open and continuous possession of the status as FRANCISCOs illegitimate daughter. We readily conclude that the testimonial evidence offered by MONINA, woven by her narration of circumstances and events that occurred through the years, concerning her relationship with FRANCISCO, coupled with the testimonies of her witnesses, overwhelmingly established the following facts: 1) FRANCISCO is MONINAs father and she was conceived at the time when her mother was in the employ of the former; 2) FRANCISCO recognized MONINA as his child through his overt acts and conduct which the Court of Appeals took pains to enumerate, thus: [L]ike sending appellant to school, paying for her tuition fees, school uniforms, books, board and lodging at the Colegio del Sagrado de Jesus, defraying appellants hospitalization expenses, providing her with [a] monthly allowance, paying for the funeral expenses of appellants mother, acknowledging appellants paternal greetings and calling appellant his Hija or child, instructing his office personnel to give appellants monthly allowance, recommending appellant for employment at the Miller, Cr uz & Co., allowing appellant to use his house in Bacolod and paying for her long distance telephone calls, having appellant spend her vacation in his apartment in Manila and also at his Forbes residence, allowing appellant to use his surname in her scholastic and other records (Exhs Z, AA, AA-1 to AA-5, W & W5) 3) Such recognition has been consistently shown and manifested throughout the years publicly,[35] spontaneously, continuously and in an uninterrupted manner.[36] Accordingly, in light of the totality of the evidence on record, the second assigned error must fail. There is some merit, however, in the third assigned error against the probative value of some of MONINAs documentary evidence. MONINAs reliance on the certification issued by the Local Civil Registrar concerning her birth (Exhs. E and F) is clearly misplaced. It is settled that a certificate of live birth purportedly identifying the putative father is not competent evidence as to the issue of paternity, when there is no showing that the putative father had a hand in the preparation of said certificates, and the Local Civil Registrar is devoid of authority to record the paternity of an illegitimate child upon the information of a third person.[37] Simply put, if the alleged father did not intervene in the birth certificate, e.g., supplying the information himself, the inscription of his name by the mother or doctor or registrar is null and void; the mere 829

University of the Cordilleras College of Law First Year C S.Y. 2013 - 2014 certificate by the registrar without the signature of the father is not proof of voluntary acknowledgment on the latters part.[38] In like manner, FRANCISCOs lack of participation in the preparation of the baptismal certificates (Exhs. C and D) and school records (Exhs. Z and AA) renders these documents incompetent to prove paternity, the former being competent merely to prove the administration of the sacrament of baptism on the date so specified.[39] However, despite the inadmissibility of the school records per se to prove paternity, they may be admitted as part of MONINAs tes timony to corroborate her claim that FRANCISCO spent for her education. We likewise disagree with the ruling of the Court of Appeals that the certificates issued by the Local Civil Registrar and the baptismal certificates may be taken as circumstantial evidence to prove MONINAs filiation. Since they are per se inadmissible in evidence as proof of such filiation, they cannot be admitted indirectly as circumstantial evidence to prove the same. As to Exhibits S, T, U and V, the various notes and letters written by FRANCISCOs relatives, namely Mike Alano, Emilio Jison, Mariquit Lopez and Fernando Lopez, respectively, allegedly attesting to MONINAs filiation, while their due execution and authenticity are not in issue,[40] as MONINA witnessed the authors signing the documents, nevertheless, under Rule 130, Section 39, the contents of these documents may not be admitted, there being no showing that the declarants-authors were dead or unable to testify, neither was the relationship between the declarants and MONINA shown by evidence other than the documents in question.[41] As to the admissibility of these documents under Rule 130, Section 40, however, this requires further elaboration. Rule 130, Section 40, provides: Section 40. Family reputation or tradition regarding pedigree. -- The reputation or tradition existing in a family previous to the controversy, in respect to the pedigree of any one of its members, may be received in evidence if the witness testifying thereon be also a member of the family, either by consanguinity or affinity. Entries in family bibles or other family books or charts, engravings on rings, family portraits and the like, may be received as evidence of pedigree. (underscoring supplied) It is evident that this provision may be divided into two (2) parts: the portion containing the first underscored clause which pertains to testimonial evidence, under which the documents in question may not be admitted as the authors thereof did not take the witness stand; and the section containing the second underscored phrase. What must then be ascertained is whether Exhibits S to V, as private documents, fall within the scope of the clause and the like as qualified by the preceding phrase [e]ntries in family bibles or other family books or charts, engravings on rights [and] family portraits. We hold that the scope of the enumeration contained in the second portion of this provision, in light of the rule of ejusdem generis, is limited to objects which are commonly known as family possessions, or those articles which represent, in effect, a familys joint statement of its belief as to the pedigree of a person.[42] These have been described as objects openly exhibited and well known to the family,[43] or those which, if preserved in a family, may be regarded as giving a family tradition.[44] Other examples of these objects which are regarded as reflective of a familys reputation or tradition regarding pedigree are inscriptions on tombstones,[45] monuments or coffin plates.[46] Plainly then, Exhibits S to V, as private documents not constituting "family possessions" as discussed above, may not be admitted on the basis of Rule 130, Section 40. Neither may these exhibits be admitted on the basis of Rule 130, Section 41 regarding common reputation,[47] it having been observed that: [T]he weight of authority appears to be in favor of the theory that it is the general repute, the common reputation in the family, and not the common reputation in community, that is a material element of evidence going to establish pedigree. xxx [Thus] matters of pedigree may be proved by reputation in the family, and not by reputation in the neighborhood or vicinity, except where the pedigree in question is marriage which may be proved by common reputation in the community.[48] Their inadmissibility notwithstanding, Exhibits S to V, inclusive, may, in like manner as MONINA's school records, properly be admitted as part of her testimony to strengthen her claim that, indeed, relatives of FRANCISCO recognized her as his daughter. We now direct our attention to MONINAs 21 September 1971 affidavit (Exh. P/Exh. 2), subject of the fourth assigned error, where she attests that FRANCISCO is not her father. MONINA contends that she signed it under duress, i.e., she was jobless, had no savings and needed the money to support herself and 830

University of the Cordilleras College of Law First Year C S.Y. 2013 - 2014 finish her studies. Moreover, she signed Exhibit P upon the advice of Atty. Divinagracia that filiation could not be waived and that FRANCISCOs ploy would boomerang upon him. On the other hand, FRANCISCO asserts that full credence should be afforded Exhibit P as MONINA was already 25 years old at the time of its execution and was advised by counsel; further, being a notarized document, its genuineness and due execution could not be questioned. He relies on the testimony of Jose Cruz, a partner at the accounting firm of Miller & Cruz, who declared that he intervened in the matter as MONINA was spreading rumors about her filiation within the firm, which might have had deleterious effects upon the relationship between the firm and FRANCISCO. On this issue, we find for MONINA and agree with the following observations of the Court of Appeals: Even the affidavit (Exh 2) which [FRANCISCO] had foisted on the trial court xxx does not hold sway in the face of [MONINAs] logical explanation that she at first did agree to sign the affidavit which contained untruthful statements. In fact, she promptly complained to [FRANCISCO] who, however explained to her that the affidavit was only for the consumption of his spouse xxx. At any rate, if [MONINA] were not his illegitimate daughter, it would have been uncalled for, if not absurd, for [FRANCISCO] or his lawyer to have secured [MONINAs] sworn statement xxx On the contrary, in asking [MONINA] to sign the said affidavit at the cost of P15,000, [FRANCISCO] clearly betrayed his intention to conceal or suppress his paternity of [MONINA]. xxx Indeed, if MONINA were truly not FRANCISCOs illegitimate daughter, it would have been unnecessary for him to have gone to such great lengths in order that MONINA denounce her filiation. For as clearly established before the trial court and properly appreciated by the Court of Appeals, MONINA had resigned from Miller & Cruz five (5) months prior to the execution of the sworn statement in question, hence negating FRANCISCOs theory of the need to quash rumors circulating within Miller & Cruz regarding the identity of MONINAs father. Hence, coupled with the assessment of the credibility of the testimonial evidence of the parties discussed above, it is evident that the standard to contradict a notarial document, i.e., clear and convincing evidence and more than merely preponderant,[49] has been met by MONINA. Plainly then, the burden of evidence fully shifted to FRANCISCO. Two (2) glaring points in FRANCISCOs defense beg to be addressed: First, that his testimony was comprised of mere denials, rife with bare, unsubstantiated responses such as That is not true, I do not believe that, or None that I know. In declining then to lend credence to FRANCISCOs testimony, we resort to a guiding principle in adjudging the credibility of a witness and the truthfulness of his statements, laid down as early as 1921: The experience of courts and the general observation of humanity teach us that the natural limitations of our inventive faculties are such that if a witness undertakes to fabricate and deliver in court a false narrative containing numerous details, he is almost certain to fall into fatal inconsistencies, to make statements which can be readily refuted, or to expose in his demeanor the falsity of his message. For this reason it will be found that perjurers usually confine themselves to the incidents immediately related to the principal fact about which they testify, and when asked about collateral facts by which their truthfulness could be tested, their answers not infrequently take the stereotyped form of such expressions as I dont know or I dont remember. xxx[50] Second, the reasons for the dismissals of Tingson, Baylosis and Savariz were unspecified or likewise unsubstantiated, hence FRANCISCOs attempt to prove ill-motive on their part to falsely testify in MONINAs favor may not succeed. As may be gleaned, the only detail which FRANCISCO could furnish as to the circumstances surrounding the dismissals of his former employees was that Baylosis allegedly took advantage of his position while FRANCISCO was in the United States. But aside from this bare claim, FRANCISCOs account is barren, hence unable to provide th e basis for a finding of bias against FRANCISCO on the part of his former employees. As to FRANCISCOs other witnesses, nothing substantial could be obtained either. Nonito Jalandoni avowed that he only came to know of MONINA in June 1988;[51] that during his employment at Nelly Garden from 1963 up to 1974, he did not recall ever having seen MONINA there, neither did he know of any instructions from FRANCISCO nor Mr. Lagarto (FRANCISCOs office manager before passing away) regarding the disbursement of MONINAs allowance.[52] Teodoro Zulla corroborated Jalandonis 831

University of the Cordilleras College of Law First Year C S.Y. 2013 - 2014 testimony regarding not having seen MONINA at Nelly Garden and MONINAs allowance; declared that Alfredo Baylosis was dismissed due to discrepancies discovered after an audit, without any further elaboration, however; but admitted that he never prepared the vouchers pertaining to FRANCISCOs personal expenses, merely those intended for one of FRANCISCOs haciendas.[53] Then, Iigo Superticioso confirmed that according to the report of a certain Mr. Atienza, Baylosis was dismissed by Mr. Jison for irregularities, while Superticioso was informed by FRANCISCO that Tingson was dismissed for loss of confidence. Superticioso likewise denied that MONINA received money from FRANCISCOs office, neither was there a standing order from FRANCISCO to release funds to her.[54] It is at once obvious that the testimonies of these witnesses for FRANCISCO are likewise insufficient to overcome MONINAs evidence. The former merely consist of denials as regards the latters having gone to Nelly Garden or having received her allowance from FRANCISCOs office, which, being in the form of negative testimony, necessarily stand infirm as against positive testimony;[55] bare assertions as regards the dismissal of Baylosis; ignorance of FRANCISCOs personal expenses incapable of evincing that FRANCISCO did not provide MONINA with an allowance; or hearsay evidence as regards the cause for the dismissals of Baylosis and Tingson. But what then serves as the coup de grce is that despite Superticiosos claim that he did not know MONINA,[56] when confronted with Exhibit H, a telephone toll ticket indicating that on 18 May 1971, MONINA called a certain Eing at FRANCISCOs office, Superticioso admitted that his nickname was Iing and that there was no other person named Iing in FRANCISCOs office.[57] All told, MONINAs evidence hurdled the high standard of proof required for the success of an action to establish ones illegitimate filiation when relying upon the provisions regarding open and continuous possession or any other means allowed by the Rules of Court and special laws; moreover, MONINA proved her filiation by more than mere preponderance of evidence. The last assigned error concerning laches likewise fails to convince. The essential elements of laches are: (1) conduct on the part of the defendant, or of one under whom he claims, giving rise to the situation of which the complaint seeks a remedy; (2) delay in asserting the complainants rights, the complainant having had knowledge or notice of the defendants conduct as having been afforded an opportunity to institute a suit; (3) lack of knowledge or notice on the part of the defendant that the complaint would assert the right in which he bases his suit; and (4) injury or prejudice to the defendant in the event relief is accorded to the complainant, or the suit is not held barred.[58] The last element is the origin of the doctrine that stale demands apply only where by reason of the lapse of time it would be inequitable to allow a party to enforce his legal rights.[59]

SO ORDERED.

832

University of the Cordilleras College of Law First Year C S.Y. 2013 - 2014 Case Digest: Jison vs Court of Appeals 286 SCRA 495 G.R. No. 124853 February 24, 1998 DAVIDE, JR., J.: Facts: Decision of the Court of Appeals (CA) in CA-G.R. CV No. 32860[1] which reversed the decision of Branch 24 of the Regional Trial Court (RTC) of Iloilo City in Civil Case No. 16373.[2] The latter dismissed the complaint of private respondent Monina Jison (hereafter MONINA) for recognition as an illegitimate child of petitioner Francisco Jison (hereafter FRANCISCO).This is a petition for review under Rule 45 of the Rules of Court of the 27 April 1995 In issue is whether or not public respondent Court of Appeals committed reversible error, which, in this instance, necessitates an inquiry into the facts. While as a general rule, factual issues are not within the province of this Court, nevertheless, in light of the conflicting findings of facts of the trial court and the Court of Appeals, this case falls under an exception to this rule.[3] Issue: On this issue, we find for MONINA and agree with the following observations of the Court of Appeals: Even the affidavit (Exh 2) which [FRANCISCO] had foisted on the trial court xxx does not hold sway in the face of [MONINAs] logical explanation that she at first did agree to sign the affidavit which contained untruthful statements. In fact, she promptly complained to [FRANCISCO] who, however explained to her that the affidavit was only for the consumption of his spouse xxx. At any rate, if [MONINA] were not his illegitimate daughter, it would have been uncalled for, if not absurd, for [FRANCISCO] or his lawyer to have secured [MONINAs] sworn statement xxx On the contrary, in asking [MONINA] to sign the said affidavit at the cost of P15,000, [FRANCISCO] clearly betrayed his intention to conceal or suppress his paternity of [MONINA]. xxx Indeed, if MONINA were truly not FRANCISCOs illegitimate daughter, it would have been unnecessary for him to have gone to such great lengths in order that MONINA denounce her filiation. For as clearly established before the trial court and properly appreciated by the Court of Appeals, MONINA had resigned from Miller & Cruz five (5) months prior to the execution of the sworn statement in question, hence negating FRANCISCOs theory of the need to quash rumors circulating within Miller & Cruz regarding the identity of MONINAs father. Hence, coupled with the assessment of the credibility of the testimonial evidence of the parties discussed above, it is evident that the standard to contradict a notarial document, i.e., clear and convincing evidence and more than merely preponderant,[49] has been met by MONINA. Plainly then, the burden of evidence fully shifted to FRANCISCO. Two (2) glaring points in FRANCISCOs defense beg to be addressed: First, that his testimony was comprised of mere denials, rife with bare, unsubstantiated responses such as That is not true, I do not believe that, or None that I know. In declining then to lend credence to FRANCISCOs testimony, we resort to a guiding principle in adjudging the credibility of a witness and the truthfulness of his statements, laid down as early as 1921: The experience of courts and the general observation of humanity teach us that the natural limitations of our inventive faculties are such that if a witness undertakes to fabricate and deliver in court a false narrative containing numerous details, he is almost certain to fall into fatal inconsistencies, to make statements which can be readily refuted, or to expose in his demeanor the falsity of his message. For this reason it will be found that perjurers usually confine themselves to the incidents immediately related to the principal fact about which they testify, and when asked about collateral facts by which their truthfulness could be tested, their answers not infrequently take the stereotyped form of such expressions as I dont know or I dont remember. xxx[50] Ruling: As FRANCISCO set up laches as an affirmative defense, it was incumbent upon him to prove the existence of its elements. However, he only succeeded in showing MONINAs delay in asserting 833

University of the Cordilleras College of Law First Year C S.Y. 2013 - 2014 her claim, but miserably failed to prove the last element. In any event, it must be stressed that laches is based upon grounds of public policy which requires, for the peace of society, the discouragement of stale claims, and is principally a question of the inequity or unfairness of permitting a right or claim to be enforced or asserted. There is no absolute rule as to what constitutes laches; each case is to be determined according to its particular circumstances. The question of laches is addressed to the sound discretion of the court, and since it is an equitable doctrine, its application is controlled by equitable considerations. It cannot be worked to defeat justice or to perpetuate fraud and injustice.[60] Since the instant case involves paternity and filiation, even if illegitimate, MONINA filed her action well within the period granted her by a positive provision of law. A denial then of her action on ground of laches would clearly be inequitable and unjust. WHEREFORE, IN VIEW OF THE FOREGOING, the petition is hereby DENIED and the challenged decision of the Court of Appeals of 27 April 1995 in CA-G.R. CV No. 32860 is AFFIRMED. Costs against petitioner.

834

University of the Cordilleras College of Law First Year C S.Y. 2013 - 2014 Estate of Rogelio G. Ong vs Diaz 540 SCRA 480 G.R. No. 171713 December 17, 2007 Full Case ESTATE OF ROGELIO G. ONG, petitioner, vs. Minor JOANNE RODJIN DIAZ, Represented by Her Mother and Guardian, Jinky C. Diaz, respondent. CHICO-NAZARIO, J.: This is a petition for Review on Certiorari under Rule 45 of the Revised Rules of Civil Procedure assailing (1) the Decision1 of the Court of Appeals dated 23 November 2005 and (2) the Resolution2 of the same court dated 1 March 2006 denying petitioners Motion for Reconsideration in CA-G.R. CV No. 70125. A Complaint3 for compulsory recognition with prayer for support pending litigation was filed by minor Joanne Rodjin Diaz (Joanne), represented by her mother and guardian, Jinky C. Diaz (Jinky), against Rogelio G. Ong (Rogelio) before the Regional Trial Court (RTC) of Tarlac City. In her Complaint, Jinky prayed that judgment be rendered: (a) Ordering defendant to recognize plaintiff Joanne Rodjin Diaz as his daughter. (b) Ordering defendant to give plaintiff monthly support of P20,000.00 pendente lite and thereafter to fix monthly support. (c) Ordering the defendant to pay plaintiff attorneys fees in the sum of P100,000.00. (d) Granting plaintiff such other measure of relief as maybe just and equitable in the premises.4 As alleged by Jinky in her Complaint in November 1993 in Tarlac City, she and Rogelio got acquainted. This developed into friendship and later blossomed into love. At this time, Jinky was already married to a Japanese national, Hasegawa Katsuo, in a civil wedding solemnized on 19 February 1993 by Municipal Trial Court Judge Panfilo V. Valdez.5 From January 1994 to September 1998, Jinky and Rogelio cohabited and lived together at Fairlane Subdivision, and later at Capitol Garden, Tarlac City. From this live-in relationship, minor Joanne Rodjin Diaz was conceived and on 25 February 1998 was born at the Central Luzon Doctors Hospital, Tarlac City. Rogelio brought Jinky to the hospital and took minor Joanne and Jinky home after delivery. Rogelio paid all the hospital bills and the baptismal expenses and provided for all of minor Joannes needs recognizing the child as his. In September 1998, Rogelio abandoned minor Joanne and Jinky, and stopped supporting minor Joanne, falsely alleging that he is not the father of the child. Rogelio, despite Jinkys remonstrance, failed and refused and continued failing and refusing to give support for the child and to acknowledge her as his daughter, thus leading to the filing of the heretofore adverted complaint. After summons had been duly served upon Rogelio, the latter failed to file any responsive pleading despite repeated motions for extension, prompting the trial court to declare him in default in its Order dated 7 April 1999. Rogelios Answer with Counterclaim and Special and Affirmative Defenses was received by the trial court only on 15 April 1999. Jinky was allowed to present her evidence ex parte on the basis of which the trial court on 23 April 1999 rendered a decision granting the reliefs prayed for in the complaint. In its Decision6 dated 23 April 1999, the RTC held: WHEREFORE, judgment is hereby rendered: 835

University of the Cordilleras College of Law First Year C S.Y. 2013 - 2014 1. Ordering defendant to recognize plaintiff as his natural child; 2. Ordering defendant to provide plaintiff with a monthly support of P10,000.00 and further 3. Ordering defendant to pay reasonable attorneys fees in the amount of P5,000.00 and the cost of the suit. On 28 April 1999, Rogelio filed a motion to lift the order of default and a motion for reconsideration seeking the courts understanding, as he was then in a quandary on what to do to find a solution to a very difficult problem of his life.7 On 29 April 1999, Rogelio filed a motion for new trial with prayer that the decision of the trial court dated 23 April 1999 be vacated and the case be considered for trial de novo pursuant to the provisions of Section 6, Rule 37 of the 1997 Rules of Civil Procedure.8 On 16 June 1999, the RTC issued an Order granting Rogelios Motion for New Trial: WHEREFORE, finding defendants motion for new trial to be impressed with merit, the same is hereby granted. The Order of this court declaring defendant in default and the decision is this court dated April 23, 1999 are hereby set aside but the evidence adduced shall remain in record, subject to cross-examination by defendant at the appropriate stage of the proceedings. In the meantime defendants answer is hereby admitted, subject to the right of plaintiff to file a reply and/or answer to defendants counterclaim within the period fixed by the Rules of Court. Acting on plaintiffs application for support pendente lite which this court finds to be warranted, defendant is hereby ordered to pay to plaintiff immediately the sum of P2,000.00 a month from January 15, 1999 to May 1999 as support pendente lite in arrears and the amount of P4,000.00 every month thereafter as regular support pendente lite during the pendency of this case.9 The RTC finally held: The only issue to be resolved is whether or not the defendant is the father of the plaintiff Joanne Rodjin Diaz. Since it was duly established that plaintiffs mother Jinky Diaz was married at the time of the birth of Joanne Rodjin Diaz, the law presumes that Joanne is a legitimate child of the spouses Hasegawa Katsuo and Jinky Diaz (Article 164, Family Code). The child is still presumed legitimate even if the mother may have declared against her legitimacy (Article 167, Ibid). The legitimacy of a child may be impugned only on the following grounds provided for in Article 166 of the same Code. Paragraph 1 of the said Article provides that there must be physical impossibility for the husband to have sexual intercourse with the wife within the first 120 days of the 300 days following the birth of the child because of a) physical incapacity of the husband to have sexual intercourse with his wife; b) husband and wife were living separately in such a way that sexual intercourse was not possible; c) serious illness of the husband which prevented sexual intercourse. It was established by evidence that the husband is a Japanese national and that he was living outside of the country (TSN, Aug. 27, 1999, page 5) and he comes home only once a year. Both evidence of the parties proved that the husband was outside the country and no evidence was shown that he ever arrived in the country in the year 1997 preceding the birth of plaintiff Joanne Rodjin Diaz. While it may also be argued that plaintiff Jinky had a relationship with another man before she met the defendant, there is no evidence that she also had sexual relations with other men on or about the conception of Joanne Rodjin. Joanne Rodjin was her second child (see Exh. "A"), so her first child, a 836

University of the Cordilleras College of Law First Year C S.Y. 2013 - 2014 certain Nicole (according to defendant) must have a different father or may be the son of Hasegawa K[u]tsuo. The defendant admitted having been the one who shouldered the hospital bills representing the expenses in connection with the birth of plaintiff. It is an evidence of admission that he is the real father of plaintiff. Defendant also admitted that even when he stopped going out with Jinky, he and Jinky used to go to motels even after 1996. Defendant also admitted that on some instances, he still used to see Jinky after the birth of Joanne Rodjin. Defendant was even the one who fetched Jinky after she gave birth to Joanne. On the strength of this evidence, the Court finds that Joanne Rodjin is the child of Jinky and defendant Rogelio Ong and it is but just that the latter should support plaintiff.10 On 15 December 2000, the RTC rendered a decision and disposed: WHEREFORE, judgment is hereby rendered declaring Joanne Rodjin Diaz to be the illegitimate child of defendant Rogelio Ong with plaintiff Jinky Diaz. The Order of this Court awarding support pendente lite dated June 15, 1999, is hereby affirmed and that the support should continue until Joanne Rodjin Diaz shall have reached majority age.11 Rogelio filed a Motion for Reconsideration, which was denied for lack of merit in an Order of the trial court dated 19 January 2001.12 From the denial of his Motion for Reconsideration, Rogelio appealed to the Court of Appeals. After all the responsive pleadings had been filed, the case was submitted for decision and ordered re-raffled to another Justice for study and report as early as 12 July 2002.13 During the pendency of the case with the Court of Appeals, Rogelios counsel filed a manifestation informing the Court that Rogelio died on 21 February 2005; hence, a Notice of Substitution was filed by said counsel praying that Rogelio be substituted in the case by the Estate of Rogelio Ong,14 which motion was accordingly granted by the Court of Appeals.15 In a Decision dated 23 November 2005, the Court of Appeals held: WHEREFORE, premises considered, the present appeal is hereby GRANTED. The appealed Decision dated December 15, 2000 of the Regional Trial Court of Tarlac, Tarlac, Branch 63 in Civil Case No. 8799 is hereby SET ASIDE. The case is hereby REMANDED to the court a quo for the issuance of an order directing the parties to make arrangements for DNA analysis for the purpose of determining the paternity of plaintiff minor Joanne Rodjin Diaz, upon consultation and in coordination with laboratories and experts on the field of DNA analysis. No pronouncement as to costs.16 Petitioner filed a Motion for Reconsideration which was denied by the Court of Appeals in a Resolution dated 1 March 2006. In disposing as it did, the Court of Appeals justified its Decision as follows: In this case, records showed that the late defendant-appellant Rogelio G. Ong, in the early stage of the proceedings volunteered and suggested that he and plaintiffs mother submit themselves to a DNA or blood testing to settle the issue of paternity, as a sign of good faith. However, the trial court did not consider resorting to this modern scientific procedure notwithstanding the repeated denials of defendant that he is the biological father of the plaintiff even as he admitted having actual sexual relations with plaintiffs mother. We believe that DNA paternity testing, as current jurisprudence affirms, would be the most reliable and effective method of settling the present paternity dispute. Considering, however, the untimely demise of defendant-appellant during the pendency of this appeal, the trial court, in consultation with out laboratories and experts on the field of DNA analysis, can possibly avail of such procedure with whatever remaining DNA samples from the deceased defendant alleged to be the putative father of plaintiff minor whose illegitimate filiations is the subject of this action for support.17 Hence, this petition which raises the following issues for resolution: I WHETHER OR NOT THE COURT OF APPEALS ERRED WHEN IT DID NOT DISMISS 837

University of the Cordilleras College of Law First Year C S.Y. 2013 - 2014 RESPONDENTS COMPLAINT FOR COMPULSORY RECOGNITION DESPITE ITS FINDING THAT THE EVIDENCE PRESENTED FAILED TO PROVE THAT ROGELIO G. ONG WAS HER FATHER. II WHETHER OR NOT THE COURT OF APPEALS ERRED WHEN IT DID NOT DECLARE RESPONDENT AS THE LEGITIMATE CHILD OF JINKY C. DIAZ AND HER JAPANESE HUSBAND, CONSIDERING THAT RESPONDENT FAILED TO REBUT THE PRESUMPTION OF HER LEGITIMACY. III WHETHER OR NOT THE COURT OF APPEALS ERRED WHEN IT REMANDED THE CASE TO THE COURT A QUO FOR DNA ANALYSIS DESPITE THE FACT THAT IT IS NO LONGER FEASIBLE DUE TO THE DEATH OF ROGELIO G. ONG.18 Petitioner prays that the present petition be given due course and the Decision of the Court of Appeals dated November 23, 2005 be modified, by setting aside the judgment remanding the case to the trial court for DNA testing analysis, by dismissing the complaint of minor Joanne for compulsory recognition, and by declaring the minor as the legitimate child of Jinky and Hasegawa Katsuo.19 From among the issues presented for our disposition, this Court finds it prudent to concentrate its attention on the third one, the propriety of the appellate courts decision remanding the case to the trial court for the conduct of DNA testing. Considering that a definitive result of the DNA testing will decisively lay to rest the issue of the filiation of minor Joanne, we see no reason to resolve the first two issues raised by the petitioner as they will be rendered moot by the result of the DNA testing. As a whole, the present petition calls for the determination of filiation of minor Joanne for purposes of support in favor of the said minor. Filiation proceedings are usually filed not just to adjudicate paternity but also to secure a legal right associated with paternity, such as citizenship, support (as in the present case), or inheritance. The burden of proving paternity is on the person who alleges that the putative father is the biological father of the child. There are four significant procedural aspects of a traditional paternity action which parties have to face: a prima facie case, affirmative defenses, presumption of legitimacy, and physical resemblance between the putative father and child.20 A child born to a husband and wife during a valid marriage is presumed legitimate.21 As a guaranty in favor of the child and to protect his status of legitimacy, Article 167 of the Family Code provides: Article 167. The children shall be considered legitimate although the mother may have declared against its legitimacy or may have been sentenced as an adulteress. The law requires that every reasonable presumption be made in favor of legitimacy. We explained the rationale of this rule in the recent case of Cabatania v. Court of Appeals22: The presumption of legitimacy does not only flow out of a declaration in the statute but is based on the broad principles of natural justice and the supposed virtue of the mother. The presumption is grounded on the policy to protect the innocent offspring from the odium of illegitimacy. The presumption of legitimacy of the child, however, is not conclusive and consequently, may be overthrown by evidence to the contrary. Hence, Article 255 of the New Civil Code23 provides: Article 255. Children born after one hundred and eighty days following the celebration of the marriage, and before three hundred days following its dissolution or the separation of the spouses shall be presumed to be legitimate. Against this presumption no evidence shall be admitted other than that of the physical impossibility of the husbands having access to his wife within the first one hundred and twenty days of the three hundred which preceded the birth of the child.

838

University of the Cordilleras College of Law First Year C S.Y. 2013 - 2014 This physical impossibility may be caused: 1) By the impotence of the husband; 2) By the fact that husband and wife were living separately in such a way that access was not possible; 3) By the serious illness of the husband.24 The relevant provisions of the Family Code provide as follows: ART. 172. The filiation of legitimate children is established by any of the following: (1) The record of birth appearing in the civil register or a final judgment; or (2) An admission of legitimate filiation in a public document or a private handwritten instrument and signed by the parent concerned. In the absence of the foregoing evidence, the legitimate filiation shall be proved by: (1) The open and continuous possession of the status of a legitimate child; or (2) Any other means allowed by the Rules of Court and special laws. ART. 175. Illegitimate children may establish their illegitimate filiation in the same way and on the same evidence as legitimate children. There had been divergent and incongruent statements and assertions bandied about by the parties to the present petition. But with the advancement in the field of genetics, and the availability of new technology, it can now be determined with reasonable certainty whether Rogelio is the biological father of the minor, through DNA testing. DNA is the fundamental building block of a persons entire genetic make-up. DNA is found in all human cells and is the same in every cell of the same person. Genetic identity is unique. Hence, a persons DNA profile can determine his identity.25 DNA analysis is a procedure in which DNA extracted from a biological sample obtained from an individual is examined. The DNA is processed to generate a pattern, or a DNA profile, for the individual from whom the sample is taken. This DNA profile is unique for each person, except for identical twins. Everyone is born with a distinct genetic blueprint called DNA (deoxyribonucleic acid). It is exclusive to an individual (except in the rare occurrence of identical twins that share a single, fertilized egg), and DNA is unchanging throughout life. Being a component of every cell in the human body, the DNA of an individuals blood is the very DNA in his or her skin cells, hair follicles, muscles, semen, samples from buccal swabs, saliva, or other body parts. The chemical structure of DNA has four bases. They are known as A (Adenine), G (guanine), C (cystosine) and T (thymine). The order in which the four bases appear in an individuals DNA determines his or her physical make up. And since DNA is a double stranded molecule, it is composed of two specific paired bases, A-T or T-A and G-C or C-G. These are called "genes." Every gene has a certain number of the above base pairs distributed in a particular sequence. This gives a person his or her genetic code. Somewhere in the DNA framework, nonetheless, are sections that differ. They are known as "polymorphic loci," which are the areas analyzed in DNA typing (profiling, tests, fingerprinting). In other words, DNA typing simply means determining the "polymorphic loci." How is DNA typing performed? From a DNA sample obtained or extracted, a molecular biologist may proceed to analyze it in several ways. There are five (5) techniques to conduct DNA typing. They are: the RFLP (restriction fragment length polymorphism); "reverse dot blot" or HLA DQ a/Pm loci which was used in 287 cases that were admitted as evidence by 37 courts in the U.S. as of November 1994; DNA process; VNTR (variable number tandem repeats); and the most recent which is known as the PCR([polymerase] chain reaction) based STR (short tandem repeats) method which, as of 1996, was availed of by most forensic laboratories in the world. PCR is the process of replicating or copying DNA in an 839

University of the Cordilleras College of Law First Year C S.Y. 2013 - 2014 evidence sample a million times through repeated cycling of a reaction involving the so-called DNA polymerize enzyme. STR, on the other hand, takes measurements in 13 separate places and can match two (2) samples with a reported theoretical error rate of less than one (1) in a trillion. Just like in fingerprint analysis, in DNA typing, "matches" are determined. To illustrate, when DNA or fingerprint tests are done to identify a suspect in a criminal case, the evidence collected from the crime scene is compared with the "known" print. If a substantial amount of the identifying features are the same, the DNA or fingerprint is deemed to be a match. But then, even if only one feature of the DNA or fingerprint is different, it is deemed not to have come from the suspect. As earlier stated, certain regions of human DNA show variations between people. In each of these regions, a person possesses two genetic types called "allele," one inherited from each parent. In [a] paternity test, the forensic scientist looks at a number of these variable regions in an individual to produce a DNA profile. Comparing next the DNA profiles of the mother and child, it is possible to determine which half of the childs DNA was inherited from the mother. The other half must have been inherited from the biological father. The alleged fathers profile is then examined to ascertain whether he has the DNA types in his profile, which match the paternal types in the child. If the mans DNA types do not match that of the child, the man is excluded as the father. If the DNA types match, then he is not excluded as the father.26 In the newly promulgated rules on DNA evidence it is provided: SEC. 3 Definition of Terms. For purposes of this Rule, the following terms shall be defined as follows: xxxx (c) "DNA evidence" constitutes the totality of the DNA profiles, results and other genetic information directly generated from DNA testing of biological samples; (d) "DNA profile" means genetic information derived from DNA testing of a biological sample obtained from a person, which biological sample is clearly identifiable as originating from that person; (e) "DNA testing" means verified and credible scientific methods which include the extraction of DNA from biological samples, the generation of DNA profiles and the comparison of the information obtained from the DNA testing of biological samples for the purpose of determining, with reasonable certainty, whether or not the DNA obtained from two or more distinct biological samples originates from the same person (direct identification) or if the biological samples originate from related persons (kinship analysis); and (f) "Probability of Parentage" means the numerical estimate for the likelihood of parentage of a putative parent compared with the probability of a random match of two unrelated individuals in a given population. Amidst the protestation of petitioner against the DNA analysis, the resolution thereof may provide the definitive key to the resolution of the issue of support for minor Joanne. Our articulation in Agustin v. Court of Appeals27 is particularly relevant, thus: Our faith in DNA testing, however, was not quite so steadfast in the previous decade. In Pe Lim v. Court of Appeals (336 Phil. 741, 270 SCRA 1), promulgated in 1997, we cautioned against the use of DNA because "DNA, being a relatively new science, (had) not as yet been accorded official recognition by our courts. Paternity (would) still have to be resolved by such conventional evidence as the relevant incriminating acts,verbal and written, by the putative father." In 2001, however, we opened the possibility of admitting DNA as evidence of parentage, as enunciated in Tijing v. Court of Appeals [G.R. No. 125901, 8 March 2001, 354 SCRA 17]: x x x Parentage will still be resolved using conventional methods unless we adopt the modern and scientific ways available. Fortunately, we have now the facility and expertise in using DNA test for identification and parentage testing. The University of the Philippines Natural Science Research Institute (UP-NSRI) DNA Analysis Laboratory has now the capability to conduct DNA typing using short tandem repeat (STR) analysis. The analysis is based on the fact that the DNA of a child/person has two (2) copies, one copy from the mother and the other from the father. The DNA from the mother, the alleged father and 840

University of the Cordilleras College of Law First Year C S.Y. 2013 - 2014 child are analyzed to establish parentage. Of course, being a novel scientific technique, the use of DNA test as evidence is still open to challenge. Eventually, as the appropriate case comes, courts should not hesitate to rule on the admissibility of DNA evidence. For it was said, that courts should apply the results of science when competently obtained in aid of situations presented, since to reject said results is to deny progress. The first real breakthrough of DNA as admissible and authoritative evidence in Philippine jurisprudence came in 2002 with out en banc decision in People v. Vallejo [G.R. No. 144656, 9 May 2002, 382 SCRA 192] where the rape and murder victims DNA samples from the bloodstained clothes of the accused were admitted in evidence. We reasoned that "the purpose of DNA testing (was) to ascertain whether an association exist(ed) between the evidence sample and the reference sample. The samples collected (were) subjected to various chemical processes to establish their profile. A year later, in People v. Janson [G.R. No. 125938, 4 April 2003, 400 SCRA 584], we acquitted the accused charged with rape for lack of evidence because "doubts persist(ed) in our mind as to who (were) the real malefactors. Yes, a complex offense (had) been perpetrated but who (were) the perpetrators? How we wish we had DNA or other scientific evidence to still our doubts." In 2004, in Tecson, et al. v. COMELEC [G.R. Nos. 161434, 161634 and 161824, 3 March 2004, 424 SCRA 277], where the Court en banc was faced with the issue of filiation of then presidential candidate Fernando Poe, Jr., we stated: In case proof of filiation or paternity would be unlikely to satisfactorily establish or would be difficult to obtain, DNA testing, which examines genetic codes obtained from body cells of the illegitimate child and any physical residue of the long dead parent could be resorted to. A positive match would clear up filiation or paternity. In Tijing v. Court of Appeals, this Court has acknowledged the strong weight of DNA testing... Moreover, in our en banc decision in People v. Yatar [G.R. No. 150224, 19 May 2004, 428 SCRA 504], we affirmed the conviction of the accused for rape with homicide, the principal evidence for which included DNA test results. x x x. Coming now to the issue of Remand of the case to the trial court, petitioner questions the appropriateness of the order by the Court of Appeals directing the remand of the case to the RTC for DNA testing given that petitioner has already died. Petitioner argues that a remand of the case to the RTC for DNA analysis is no longer feasible due to the death of Rogelio. To our mind, the alleged impossibility of complying with the order of remand for purposes of DNA testing is more ostensible than real. Petitioners argument is without basis especially as the New Rules on DNA Evidence28 allows the conduct of DNA testing, either motu proprio or upon application of any person who has a legal interest in the matter in litigation, thus: SEC. 4. Application for DNA Testing Order. The appropriate court may, at any time, either motu proprio or on application of any person who has a legal interest in the matter in litigation, order a DNA testing. Such order shall issue after due hearing and notice to the parties upon a showing of the following: (a) A biological sample exists that is relevant to the case; (b) The biological sample: (i) was not previously subjected to the type of DNA testing now requested; or (ii) was previously subjected to DNA testing, but the results may require confirmation for good reasons; (c) The DNA testing uses a scientifically valid technique; (d) The DNA testing has the scientific potential to produce new information that is relevant to the proper resolution of the case; and (e) The existence of other factors, if any, which the court may consider as potentially affecting the accuracy or integrity of the DNA testing. From the foregoing, it can be said that the death of the petitioner does not ipso facto negate the application of DNA testing for as long as there exist appropriate biological samples of his DNA. As defined above, the term "biological sample" means any organic material originating from a persons 841

University of the Cordilleras College of Law First Year C S.Y. 2013 - 2014 body, even if found in inanimate objects, that is susceptible to DNA testing. This includes blood, saliva, and other body fluids, tissues, hairs and bones.29 Thus, even if Rogelio already died, any of the biological samples as enumerated above as may be available, may be used for DNA testing. In this case, petitioner has not shown the impossibility of obtaining an appropriate biological sample that can be utilized for the conduct of DNA testing. And even the death of Rogelio cannot bar the conduct of DNA testing. In People v. Umanito,30 citing Tecson v. Commission on Elections,31 this Court held: The 2004 case of Tecson v. Commission on Elections [G.R. No. 161434, 3 March 2004, 424 SCRA 277] likewise reiterated the acceptance of DNA testing in our jurisdiction in this wise: "[i]n case proof of filiation or paternity would be unlikely to satisfactorily establish or would be difficult to obtain, DNA testing, which examines genetic codes obtained from body cells of the illegitimate child and any physical residue of the long dead parent could be resorted to." It is obvious to The Court that the determination of whether appellant is the father of AAAs child , which may be accomplished through DNA testing, is material to the fair and correct adjudication of the instant appeal. Under Section 4 of the Rules, the courts are authorized, after due hearing and notice, motu proprio to order a DNA testing. However, while this Court retains jurisdiction over the case at bar, capacitated as it is to receive and act on the matter in controversy, the Supreme Court is not a trier of facts and does not, in the course of daily routine, conduct hearings. Hence, it would be more appropriate that the case be remanded to the RTC for reception of evidence in appropriate hearings, with due notice to the parties. (Emphasis supplied.) As we have declared in the said case of Agustin v. Court of Appeals32: x x x [F]or too long, illegitimate children have been marginalized by fathers who choose to deny their existence. The growing sophistication of DNA testing technology finally provides a much needed equalizer for such ostracized and abandoned progeny. We have long believed in the merits of DNA testing and have repeatedly expressed as much in the past. This case comes at a perfect time when DNA testing has finally evolved into a dependable and authoritative form of evidence gathering. We therefore take this opportunity to forcefully reiterate our stand that DNA testing is a valid means of determining paternity. WHEREFORE, the instant petition is DENIED for lack of merit. The Decision of the Court of Appeals dated 23 November 2005 and its Resolution dated 1 March 2006 are AFFIRMED. Costs against petitioner. SO ORDERED.

842

University of the Cordilleras College of Law First Year C S.Y. 2013 - 2014 Case Digest: Estate of Rogelio G. Ong vs Diaz 540 SCRA 480 G.R. No. 171713 December 17, 2007 CHICO-NAZARIO, J.: Facts: This is a petition for Review on Certiorari under Rule 45 of the Revised Rules of Civil Procedure assailing (1) the Decision1 of the Court of Appeals dated 23 November 2005 and (2) the Resolution2 of the same court dated 1 March 2006 denying petitioners Motion for Reconsideration in CA-G.R. CV No. 70125. Issue: Remand of the case to the trial court, petitioner questions the appropriateness of the order by the Court of Appeals directing the remand of the case to the RTC for DNA testing given that petitioner has already died. Petitioner argues that a remand of the case to the RTC for DNA analysis is no longer feasible due to the death of Rogelio. To our mind, the alleged impossibility of complying with the order of remand for purposes of DNA testing is more ostensible than real. Petitioners argument is without basis especially as the New Rules on DNA Evidence28 allows the conduct of DNA testing, either motu proprio or upon application of any person who has a legal interest in the matter in litigation, thus: Ruling: The Court that the determination of whether appellant is the father of AAAs child, which may be accomplished through DNA testing, is material to the fair and correct adjudication of the instant appeal. Under Section 4 of the Rules, the courts are authorized, after due hearing and notice, motu proprio to order a DNA testing. However, while this Court retains jurisdiction over the case at bar, capacitated as it is to receive and act on the matter in controversy, the Supreme Court is not a trier of facts and does not, in the course of daily routine, conduct hearings. Hence, it would be more appropriate that the case be remanded to the RTC for reception of evidence in appropriate hearings, with due notice to the parties. (Emphasis supplied.) As we have declared in the said case of Agustin v. Court of Appeals: x x x [F]or too long, illegitimate children have been marginalized by fathers who choose to deny their existence. The growing sophistication of DNA testing technology finally provides a much needed equalizer for such ostracized and abandoned progeny. We have long believed in the merits of DNA testing and have repeatedly expressed as much in the past. This case comes at a perfect time when DNA testing has finally evolved into a dependable and authoritative form of evidence gathering. We therefore take this opportunity to forcefully reiterate our stand that DNA testing is a valid means of determining paternity. WHEREFORE, the instant petition is DENIED for lack of merit. The Decision of the Court of Appeals dated 23 November 2005 and its Resolution dated 1 March 2006 are AFFIRMED. Costs against petitioner.

843

University of the Cordilleras College of Law First Year C S.Y. 2013 - 2014

XIII. Adoption

844

University of the Cordilleras College of Law First Year C S.Y. 2013 - 2014 SSS vs Aguas 483 SCRA 383 G.R. No. 165546 February 27, 2006 Full Case SOCIAL SECURITY SYSTEM, Petitioner, vs. ROSANNA H. AGUAS, JANET H. AGUAS, and minor JEYLNN H. AGUAS, represented by her Legal Guardian, ROSANNA H. AGUAS, Respondents. CALLEJO, SR., J.: Before us is a petition for review on certiorari of the Decision of the Court of Appeals (CA) in CA-G.R. SP No. 66531 and its Resolution denying the motion for reconsideration thereof. The antecedents are as follows: Pablo Aguas, a member of the Social Security System (SSS) and a pensioner, died on December 8, 1996. Pablos surviving spouse, Rosanna H. Aguas, filed a claim with the SSS for death benefits on December 13, 1996. Rosanna indicated in her claim that Pablo was likewise survived by his minor child, Jeylnn, who was born on October 29, 1991. Her claim for monthly pension was settled on February 13, 1997. Sometime in April 1997, the SSS received a sworn letter dated April 2, 1997 from Leticia AguasMacapinlac, Pablos sister, contesting Rosannas claim for death benefits. She alleged that Rosanna abandoned the family abode approximately more than six years before, and lived with another man on whom she has been dependent for support. She further averred that Pablo had no legal children with Rosanna, but that the latter had several children with a certain Romeo dela Pea. In support of her allegation, Leticia enclosed a notarized copy of the original birth certificate of one Jefren H. dela Pea, showing that the latter was born on November 15, 1996 to Rosanna Y. Hernandez and Romeo C. dela Pea, and that the two were married on November 1, 1990. As a result, the SSS suspended the payment of Rosanna and Jeylnns monthly pension in September 1997. It also conducted an investigation to verify Leticias allegations. In a Memorandum6 dated November 18, 1997, the Social Security Officer who conducted the investigation reported that, based on an interview with Mariquita D. Dizon, Pablos first cousin and neighbor, and Jessie Gonzales (also a neighbor). She learned that the deceased had no legal children with Rosanna; Jenelyn and Jefren were Rosannas children with one Romeo C. dela Pea; and Rosanna left the deceased six years before his death and lived with Romeo while she was still pregnant with Jenelyn, who was born on October 29, 1991. Mariquita also confirmed that Pablo was not capable of having a child as he was under treatment. On the basis of the report and an alleged confirmation by a certain Dr. Manuel Macapinlac that Pablo was infertile, the SSS denied Rosannas request to resume the payment of their pensions. She was advised to refund to the SSS within 30 days the amount of P10,350.00 representing the total death benefits released to her and Jenelyn from December 1996 to August 1997 at P1,150.00 per month. Rosanna and Jeylnn, through counsel, requested for a reconsideration of the said decision. However, in its Letter dated February 6, 1998, the SSS denied the claim. This prompted Rosanna and Jeylnn to file a claim/petition for the Restoration/Payment of Pensions with the Social Security Commission (SSC) on February 20, 1998. Janet H. Aguas, who also claimed to be the child of the deceased and Rosanna, now joined them as claimant. The case was docketed as SSC Case No. 3-14769-98. The claimants appended to their petition, among others, photocopies of the following: (1) Pablo and Rosannas marriage certificate; (2) Janets certificate of live birth; (3) Jeylnns certificate of live birth; and (4) Pablos certificate of death. In its Answer, the SSS averred that, based on the sworn testimonies and documentary evidence showing the disqualification of the petitioners as primary beneficiaries, the claims were barren of factual and legal basis; as such, it was justified in denying their claims.

845

University of the Cordilleras College of Law First Year C S.Y. 2013 - 2014 In their Position Paper, the claimants averred that Jeylnn was a legitimate child of Pablo as evidenced by her birth certificate bearing Pablos signature as Jeylnns father. They asserted that Rosanna never left Pablo and that they lived together as husband and wife under one roof. In support thereof, they attached a Joint Affidavit executed by their neighbors, Vivencia Turla and Carmelita Yangu, where they declared that Rosanna and Pablo lived together as husband and wife until the latters death. In Janets birth certificate, which was registered in the Civil Registry of San Fernando, it appears that her father was Pablo and her mother was Rosanna. As to the alleged infertility of Pablo, the claimants averred that Dr. Macapinlac denied giving the opinion precisely because he was not an expert on such matters, and that he treated the deceased only for tuberculosis. The claimant likewise claimed that the information the SSS gathered from the doctor was privileged communication. In compliance with the SSCs order, the SSS secured Confirmation Reports signed by clerks from the corresponding civil registers confirming (1) the fact of marriage between Pablo and Rosanna on December 4, 1977; (2) the fact of Jefren dela Peas birth on November 15, 1996; (3) the fact of Jeylnns birth on October 29, 1991; and (4) the fact of Pablos death on December 8, 1996. The SSC decided to set the case for hearing. It also directed the SSS to verify the authenticity of Pablos signature as appearing on Jeylnns birth certificate from his claim records, particularly his SSS Form E -1 and retirement benefit application. The SSS complied with said directive and manifested to the SSC that, based on the laboratory analysis conducted, Pablos signature in the birth certificate was made by the same person who signed the members record and other similar documents submitted by Pablo. The SSC then summoned Vivencia Turla, Carmelita Yangu and Leticia Aguas-Macapinlac for clarificatory questions with regard to their respective sworn affidavits. Vivencia testified that she had known Pablo and Rosanna for more than 30 years already; the couple were married and lived in Macabacle, Dolores, San Fernando, Pampanga; she was a former neighbor of the spouses, but four years after their marriage, she (Vivencia) and her family moved to Sto. Nio Triangulo, San Fernando, Pampanga; she would often visit the two, especially during Christmas or fiestas; the spouses real child was Jeylnn; Janet was only an adopted child; the spouse later transferred residence, not far from their old house, and Janet, together with her husband and son, remained in the old house. On the other hand, Carmelita testified that she had been a neighbor of Pablo and Rosanna for 15 years and that, up to the present, Rosanna and her children, Janet, Jeylnn and Jefren, were still her neighbors; Janet and Jeylnn were the children of Pablo and Rosanna but she did not know whose child Jefren is. According to Leticia, Janet was not the real child of Pablo and Rosanna; she was just taken in by the spouses because for a long time they could not have children; however, there were no legal papers on Janets adoption. Later on, Rosanna got pregnant with Jeylnn; after the latters baptism, there was a commotion at the house because Romeo dela Pea was claiming that he was the father of the child and he got mad because the child was named after Pablo; the latter also got mad and even attempted to shoot Rosanna; he drove them away from the house; since then, Pablo and Rosanna separated; she knew about this because at that time their mother was sick, and she would often visit her at their ancestral home, where Pablo and Rosanna were also staying; Rosanna was no longer living in their ancestral home but Janet resided therein; she did not know where Rosanna was staying now but she knew that the latter and Romeo dela Pea were still living together. Subsequently, Mariquita Dizon and Jessie Gonzales were also summoned for clarificatory questions. During the hearing, Mariquita brought with her photocopies of two baptismal certificates: that of Jeylnn Aguas, child of Pablo Aguas and Rosanna Hernandez born on October 29, 1991, and that of Jenelyn H. dela Pea, child of Romeo dela Pea and Rosanna Hernandez, born on January 29, 1992. On March 14, 2001, the SSC rendered a decision denying the claims for lack of merit and ordering Rosanna to immediately refund to the SSS the amount of P10,350.00 erroneously paid to her and Jeylnn as primary beneficiaries of the deceased. The SSC likewise directed the SSS to pay the death benefit to qualified secondary beneficiaries of the deceased, and in their absence, to his legal heirs. The SSC ruled that Rosanna was no longer qualified as primary beneficiary, it appearing that she had contracted marriage with Romeo dela Pea during the subsistence of her marriage to Pablo. The SSC based its conclusion on the birth certificate of Jefren dela Pea stating that his mother, Rosanna, and father, Romeo dela Pea, were married on November 1, 1990. The SSC declared that Rosanna had a child with Romeo dela Pea while she was still married to Pablo (as evidenced by the baptismal certificate of Jenelyn H. dela Pea showing that she was the child of Rosanna Hernandez and Romeo dela Pea and 846

University of the Cordilleras College of Law First Year C S.Y. 2013 - 2014 that she was born on January 29, 1992). The SSC concluded that Rosanna was no longer entitled to support from Pablo prior to his death because of her act of adultery. As for Jeylnn, the SSC ruled that, even if her birth certificate was signed by Pablo as her father, there was more compelling evidence that Jeylnn was not his legitimate child. The SSC deduced from the records that Jeylnn and Jenelyn was one and the same person and concluded, based on the latters baptismal certificate, that she was the daughter of Rosanna and Romeo dela Pea. It also gave credence to the testimonies of Leticia and Mariquita that Jeylnn was the child of Rosanna and Romeo dela Pea. As for Janet, the SSC relied on Leticias declaration that she was only adopted by Pablo and Rosanna. The claimants filed a motion for reconsideration of the said decision but their motion was denied by the SSC for lack of merit and for having been filed out of time. The claimants then elevated the case to the CA via a petition for review under Rule 43 of the Rules of Court. On September 9, 2003, the CA rendered a decision in favor of petitioners. The fallo of the decision reads: WHEREFORE, the resolution and order appealed from are hereby REVERSED and SET ASIDE, and a new one is entered DECLARING petitioners as ENTITLED to the SSS benefits accruing from the death of Pablo Aguas. The case is hereby REMANDED to public respondent for purposes of computing the benefits that may have accrued in favor of petitioners after the same was cut and suspended in September 1997. SO ORDERED. In so ruling, the CA relied on the birth certificates of Janet and Jeylnn showing that they were the children of the deceased. According to the appellate court, for judicial purposes, these records were binding upon the parties, including the SSS. These entries made in public documents may only be challenged through adversarial proceedings in courts of law, and may not be altered by mere testimonies of witnesses to the contrary. As for Rosanna, the CA found no evidence to show that she ceased to receive support from Pablo before he died. Rosannas alleged affair with Romeo dela Pea was not properly proven. In any case, even if Rosanna married Romeo dela Pea during her marriage to Pablo, the same would have been a void marriage; it would not have ipso facto made her not dependent for support upon Pablo and negate the presumption that, as the surviving spouse, she is entitled to support from her husband. The SSS filed a motion for reconsideration of the decision, which the CA denied for lack of merit. Hence, this petition. Petitioner seeks a reversal of the decision of the appellate court, contending that it I GRAVELY ERRED IN HOLDING THAT ROSANNA AGUAS IS ACTUALLY DEPENDENT FOR SUPPORT UPON THE MEMBER DURING HIS LIFETIME TO QUALIFY AS PRIMARY BENEFICIARY WITHIN THE INTENDMENT OF SECTION 8(e), IN RELATION TO SECTION (k) OF THE SSS LAW, AS AMENDED. II ERRED IN HOLDING THAT JANET AGUAS AND JEYLNN AGUAS ARE ENTITLED TO THE PENSION BENEFIT ACCRUING FROM THE DEATH OF PABLO AGUAS. Petitioner invokes Section 8 of Republic Act No. 1161, as amended by Presidential Decree No. 735, which defines a dependent spouse as "the legitimate spouse dependent for support upon the employee." According to petitioner, Rosanna forfeited her right to be supported by Pablo when she engaged in an intimate and illicit relationship with Romeo dela Pea and married the latter during her marriage to Pablo. Such act constitutes abandonment, which divested her of the right to receive support from her husband. It asserts that her act of adultery is evident from the birth certificate of Jefren H. dela Pea showing that he was born on November 15, 1996 to Rosanna and Romeo dela Pea. Petitioner submits that Rosanna cannot be considered as a dependent spouse of Pablo; consequently, she is not a primary beneficiary. As for Janet and Jeylnn, petitioner maintains that they are not entitled to the pension because, based on the evidence on record, particularly the testimonies of the witnesses, they are not the legitimate children 847

University of the Cordilleras College of Law First Year C S.Y. 2013 - 2014 of Pablo. It argues that, in the exercise of its quasi-judicial authority under Section 5(a) of the Social Security Act, the SSC can pass upon the legitimacy of respondents relationship with the member to determine whether they are entitled to the benefits, even without correcting their birth certificates. Respondents, for their part, assert that petitioner failed to prove that Rosanna committed acts of adultery or that she married another man after the death of her husband. They contend that Janet and Jeylnns legitimacy may be impugned only on the grounds stated in Article 166 of the Family Code, none of which were proven in this case. The issue to be resolved in this case is whether Rosanna, Jeylnn and Janet are entitled to the SSS death benefits accruing from the death of Pablo. The petition is partly meritorious. The general rule is that only questions of law may be raised by the parties and passed upon by the Court in petitions for review under Rule 45 of the Rules of Court. In an appeal via certiorari, the Court may not review the factual findings of the CA. It is not the Courts function under Rule 45 to review, examine, and evaluate or weigh the probative value of the evidence presented. However, the Court may review findings of facts in some instances, such as, when the judgment is based on a misapprehension of facts, when the findings of the CA are contrary to those of the trial court or quasi-judicial agency, or when the findings of facts of the CA are premised on the absence of evidence and are contradicted by the evidence on record. The Court finds these instances present in this case. At the time of Pablos death, the prevailing law was Republic Ac t No. 1161, as amended by Presidential Decree No. 735. Section 13 of the law enumerates those who are entitled to death benefits: Sec.13. Death benefits. Effective July 1, 1975, upon the covered employees death, (a) his primary beneficiaries shall be entitled to the basic monthly pension, and his dependents to the dependents pension: Provided, That he has paid at least thirty-six monthly contributions prior to the semester of death: Provided, further, That if the foregoing condition is not satisfied, or if he has no primary beneficiaries, his secondary beneficiaries shall be entitled to a lump sum benefit equivalent to thirty times the basic monthly pension: Provided, however, That the death benefit shall not be less than the total contributions paid by him and his employer on his behalf nor less than five hundred pesos: Provided, finally, That the covered employee who dies in the month of coverage shall be entitled to the minimum benefit. Section 8(k) and (e), in turn, defines dependents and primary beneficiaries of an SSS member as follows: SECTION 8. Terms defined. For the purposes of this Act the following terms shall, unless the context indicates otherwise, have the following meanings: xxxx (e) Dependent. The legitimate, legitimated, or legally adopted child who is unmarried, not gainfully employed, and not over twenty-one years of age provided that he is congenitally incapacitated and incapable of self-support physically or mentally; the legitimate spouse dependent for support upon the employee; and the legitimate parents wholly dependent upon the covered employee for regular support. xxxx (k) Beneficiaries. The dependent spouse until he remarries and dependent children, who shall be the primary beneficiaries. In their absence, the dependent parents and, subject to the restrictions imposed on dependent children, the legitimate descendants and illegitimate children who shall be the secondary beneficiaries. In the absence of any of the foregoing, any other person designated by the covered employee as secondary beneficiary. Whoever claims entitlement to such benefits should establish his or her right thereto by substantial evidence. Substantial evidence, the quantum of evidence required to establish a fact in cases before administrative or quasi-judicial bodies, is that level of relevant evidence which a reasonable mind might accept as adequate to justify a conclusion.

848

University of the Cordilleras College of Law First Year C S.Y. 2013 - 2014 The Court has reviewed the records of the case and finds that only Jeylnn has sufficiently established her right to a monthly pension. Jeylnns claim is justified by the photocopy of her birth certificate which bears the signature of Pablo. Petitioner was able to authenticate the certification from the Civil Registry showing that she was born on October 29, 1991. The records also show that Rosanna and Pablo were married on December 4, 1977 and the marriage subsisted until the latters death on December 8, 1996. It is therefore evident that Jeylnn was born during Rosanna and Pablos marriage. It bears stressing that under Article 164 of the Family Code, children conceived or born during the marriage of the parents are legitimate. This Court, in De Jesus v. Estate of Decedent Juan Gamboa Dizon, extensively discussed this presumption There is perhaps no presumption of the law more firmly established and founded on sounder morality and more convincing reason than the presumption that children born in wedlock are legitimate. This presumption indeed becomes conclusive in the absence of proof that there is physical impossibility of access between the spouses during the first 120 days of the 300 days which immediately precedes the birth of the child due to (a) the physical incapacity of the husband to have sexual intercourse with his wife; (b) the fact that the husband and wife are living separately in such way that sexual intercourse is not possible; or (c) serious illness of the husband, which absolutely prevents sexual intercourse. Quite remarkably, upon the expiration of the periods set forth in Article 170, and in proper cases Article 171, of the Family Code (which took effect on 03 August 1988), the action to impugn the legitimacy of the child would no longer be legally feasible and the status conferred by the presumption becomes fixed and unassailable. Indeed, impugning the legitimacy of a child is a strictly personal right of the husband or, in exceptional cases, his heirs. In this case, there is no showing that Pablo challenged the legitimacy of Jeylnn during his lifetime. Hence, Jeylnns status as a legitimate child of Pablo can no longer be contested. The presumption that Jeylnn is a legitimate child is buttressed by her birth certificate bearing Pablos signature, which was verified from his specimen signature on file with petitioner. A birth certificate signed by the father is a competent evidence of paternity. The presumption of legitimacy under Article 164, however, cannot extend to Janet because her date of birth was not substantially proven. Such presumption may be availed only upon convincing proof of the factual basis therefor, i.e., that the childs parents were legally married and that his/her conception or birth occurred during the subsistence of that marriage. It should be noted that respondents likewise submitted a photocopy of Janets alleged birth certificate. However, the Court cannot give said birth certificate the same probative weight as Jeylnns because it was not verified in any way by the civil register. It stands as a mere photocopy, without probative weight. Unlike Jeylnn, there was no confirmation by the civil register of the fact of Janets birth on the date stated in the certificate. In any case, a record of birth is merely prima facie evidence of the facts contained therein. Here, the witnesses were unanimous in saying that Janet was not the real child but merely adopted by Rosanna and Pablo. Leticia also testified that Janets adoption did not undergo any legal proceedings; hence, there were no papers to prove it. Under Section 8(e) of Republic Act No. 1161, as amended, only "legally adopted" children are considered dependent children. Absent any proof that the family has legally adopted Janet, the Court cannot consider her a dependent child of Pablo, hence, not a primary beneficiary. On the claims of Rosanna, it bears stressing that for her to qualify as a primary beneficiary, she must prove that she was "the legitimate spouse dependent for support from the employee." The claimant-spouse must therefore establish two qualifying factors: (1) that she is the legitimate spouse, and (2) that she is dependent upon the member for support. In this case, Rosanna presented proof to show that she is the legitimate spouse of Pablo, that is, a copy of their marriage certificate which was verified with the civil register by petitioner. But whether or not Rosanna has sufficiently established that she was still dependent on Pablo at the time of his death remains to be resolved. Indeed, a husband and wife are obliged to support each other, but whether one is actually dependent for support upon the other is something that has to be shown; it cannot be presumed from the fact of marriage alone. In a parallel case involving a claim for benefits under the GSIS law, the Court defined a dependent as "one who derives his or her main support from another. Meaning, relying on, or subject to, someone else for support; not able to exist or sustain oneself, or to perform anything without the will, power, or aid of 849

University of the Cordilleras College of Law First Year C S.Y. 2013 - 2014 someone else." It should be noted that the GSIS law likewise defines a dependent spouse as "the legitimate spouse dependent for support upon the member or pensioner." In that case, the Court found it obvious that a wife who abandoned the family for more than 17 years until her husband died, and lived with other men, was not dependent on her husband for support, financial or otherwise, during that entire period. Hence, the Court denied her claim for death benefits. The obvious conclusion then is that a wife who is already separated de facto from her husband cannot be said to be "dependent for support" upon the husband, absent any showing to the contrary. Conversely, if it is proved that the husband and wife were still living together at the time of his death, it would be safe to presume that she was dependent on the husband for support, unless it is shown that she is capable of providing for herself. Rosanna had the burden to prove that all the statutory requirements have been complied with, particularly her dependency on her husband for support at the time of his death. Aside from her own testimony, the only evidence adduced by Rosanna to prove that she and Pablo lived together as husband and wife until his death were the affidavits of Vivencia Turla and Carmelita Yangu where they made such declaration. Still, the affidavits of Vivencia and Carmelita and their testimonies before the SSC will not prevail over the categorical and straightforward testimonies of the other witnesses who testified that Rosanna and Pablo had already separated for almost six years before the latter died. Except for the bare assertion of Carmelita that the couple never separated, there was no further statement regarding the witnesses assertion in their affidavits that the couple lived together until Pablos death. On the contrary, Leticia narrated that the two separated after Jeylnns baptism as a result of an argument regarding Romeo dela Pea. According to Leticia, there was a commotion at their ancestral house because Romeo dela Pea was grumbling why Jeylnn was named after Pablo when he was the father, and as a result, Pablo drove them away. The SSCs observation and conclusion on the two baptismal certificates of Jeylnn and Jenelyn convinces this Court to further believe Leticias testimony on why Pablo and Rosanna separated. As noted by the SSC: It appears from the records that Jeylnn Aguas and Jenelyn H. dela Pea are one and the same person. Jeylnn Aguas, born on October 29, 1991 was baptized at the Metropolitan Cathedral of San Fernando, Pampanga, on November 24, 1991 as the child of Pablo Aguas and Rosanna Hernandez. Jenelyn H dela Pea, on the other hand, was born on January 29, 1992 to spouses Rosanna Hernandez and Romeo dela Pea and baptized on February 9, 1992. It will be noted that Jenelyn dela Pea was born approximately three months after the birth of Jeylnn Aguas. It is physically impossible for Rosanna to have given birth successively to two children in so short a time. x x x The testimony of Leticia Aguas-Macapinlac that Rosanna was driven away by Pablo after the baptism of Jeylnn because of the commotion that was created by Romeo dela Pea who wanted Jeylnn to be baptized using his name explains why Jeylnn was again baptized in the Parish of Sto. Nio in San Fernando using the name Jenelyn dela Pea. They changed her date of birth also to make it appear in the record of the parish that she is another child of Rosanna.53 On the other hand, Mariquita categorically affirmed that Rosanna was no longer living at Pablos house even before he died, and that she is still living with Romeo dela Pea up to the present. Mariquita testified as follows: Hearing Officer: Nagsama ba si Rosanna at Romeo? Mrs. Dizon: Ngayon at kahit na noon. Hearing Officer: Kailan namatay si Pablo? Mrs. Dizon: 1996. Hearing Officer: Noong bago mamatay si Pablo? Mrs. Dizon: Nagsasama na sila Romeo at Rosanna noon. Hearing Officer: So, buhay pa si Pablo Mrs. Dizon: . nagsasama na sila ni Romeo. Hearing Officer: Kailan nagkahiwalay si Romeo at Rosanna? Mrs. Dizon: Hindi na sila nagkahiwalay. Hearing Officer: Hindi, ibig ko sabihin si Pablo at Rosana? Mrs. Dizon: Hindi ko alam kasi hindi ako madalas pumunta sa kanila eh, dahil namatay na yung nanay ni Kuya Pabling, yung tiyahin ko, kapatid ng nanay ko. Noon madalas ako noong buhay pa yung nanay ni Kuya Pabling dahil kami ang nag aalaga sa kanya. 850

University of the Cordilleras College of Law First Year C S.Y. 2013 - 2014 Hearing Officer: Bago namatay si Pablo, nagsasama ba sina Romeo at Rosanna? Mrs. Dizon: Oo. Hearing Officer: Sa ngayon, may alam ka pa ba kung nagsasama pa sila Romeo at Rosanna? Mrs. Dizon: Oo, nagsasama sila, may bahay sila. Hearing Officer: Saan naman? Mrs. Dizon: Doon sa malapit sa amin sa may riles ng tren. In conclusion, the Court finds that, among respondents, only Jeylnn is entitled to the SSS death benefits accruing from the death of Pablo, as it was established that she is his legitimate child. On the other hand, the records show that Janet was merely "adopted" by the spouses, but there are no legal papers to prove it; hence, she cannot qualify as a primary beneficiary. Finally, while Rosanna was the legitimate wife of Pablo, she is likewise not qualified as a primary beneficiary since she failed to present any proof to show that at the time of his death, she was still dependent on him for support even if they were already living separately. IN LIGHT OF ALL THE FOREGOING, the petition is PARTIALLY GRANTED. The Decision and Resolution of the Court of Appeals are AFFIRMED WITH MODIFICATION. Only Jeylnn H. Aguas is declared entitled to the SSS death benefits accruing from the death of Pablo Aguas. SO ORDERED.

851

University of the Cordilleras College of Law First Year C S.Y. 2013 - 2014 Case Digest: SSS vs Aguas 483 SCRA 383 G.R. No. 165546 Decided on: February 27, 2006 Ponente: CALLEJO, SR., J.: FACTS: Pablo Aguas, a member and pensioner of the SSS died. Pablos surviving spouse, Rosanna H. Aguas, filed a claim with the SSS for death benefits on indicating in her claim that Pablo was survived by his minor child, Jeylnn. Her claim for monthly pension was settled. SSS received a sworn from Leticia Aguas-Macapinlac, Pablos sister, contesting Rosannas claim for death benefits. She alleged that Rosanna abandoned the family abode approximately more than 6 years before, and lived with another man on whom she has been dependent for support. She further averred that Pablo had no legal children with Rosanna. The SSC ruled that Rosanna was no longer qualified as primary beneficiary. CA reversed the SSC decision and favored the respondents. ISSUE: Whether or not Rosanna, Jeylnn and Janet are entitled to the SSS death benefits accruing from the death of Pablo HELD: Petition is PARTIALLY GRANTED. It bears stressing that under Article 164 of the Family Code, children conceived or born during the marriage of the parents are legitimate. Jeylnns claim is justified by the photocopy of her birth certificate which bears the signature of Pablo. Petitioner was able to authenticate the certification from the Civil Registry showing that she was born on October 29, 1991. The records also show that Rosanna and Pablo were married on December 4, 1977 and the marriage subsisted until the latters death on December 8, 1996. It is therefore evident that Jeylnn was born during Rosanna and Pablos marriage. Impugning the legitimacy of a child is a strictly personal right of the husband or, in exceptional cases, his heirs. In this case, there is no showing that Pablo challenged the legitimacy of Jeylnn during his lifetime. The presumption that Jeylnn is a legitimate child is buttressed by her birth certificate bearing Pablos signature, which was verified from his specimen signature on file with petitioner. A birth certificate signed by the father is a competent evidence of paternity. For Rosanna to qualify as a primary beneficiary, she must establish 2 qualifying factors: (1) that she is the legitimate spouse, and (2) that she is dependent upon the member for support. A wife who is already separated de facto from her husband cannot be said to be "dependent for support" upon the husband, absent any showing to the contrary. If it is proved that they were still living together at the time of his death, it is presumed that she was dependent on the husband for support, unless it is shown that she is capable of providing for herself. Only Jeylnn is entitled to the SSS death benefits as it was established that she is his legitimate child. Records show that Janet was merely "adopted" by the spouses, but there are no legal papers to prove it. Rosanna was the legitimate wife of Pablo, she is likewise not qualified as a primary beneficiary since she failed to present any proof to show that at the time of his death, she was still dependent on him for support even if they were already living separately. NOTE: Legitimacy cannot be extended to other siblings.

852

University of the Cordilleras College of Law First Year C S.Y. 2013 - 2014 Cervantes vs Fajardo 169 SCRA 575 G.R. No. 79955 January 27, 1989 Full Case IN THE MATTER OF THE PETITION FOR A WRIT OF HABEAS CORPUS OF MINOR ANGELIE ANNE C. CERVANTES, NELSON L. CERVANTES and ZENAIDA CARREON CERVANTES, petitioners, vs. GINA CARREON FAJARDO and CONRADO FAJARDO, respondents. Yolanda F. Lim for petitioners. Voltaire C. Campomanes for respondents. PADILLA, J.: This is a petition for a writ of Habeas Corpus filed with this Court over the person of the minor Angelie Anne Cervantes. In a resolution, dated 5 October 1987, the Court resolved to issue the writ returnable to the Executive Judge, Regional Trial Court of Pasig at the hearing of 12 October 1987 at 8:30 a.m. Said Judge was directed to hear the case and submit his report and recommendation to the Court. On 3 December 1987, said Executive Judge, Regional Trial Court of Pasig submitted to the Court his report and recommendation, also dated 3 December 1987. It appears that the minor was born on 14 February 1987 to respondents Conrado Fajardo and Gina Carreon, who are common-law husband and wife. Respondents offered the child for adoption to Gina Carreon's sister and brother-in-law, the herein petitioners Zenaida Carreon-Cervantes and Nelson Cervantes, spouses, who took care and custody of the child when she was barely two (2) weeks old. An Affidavit of Consent to the adoption of the child by herein petitioners, was also executed by respondent Gina Carreon on 29 April 1987. The appropriate petition for adoption (Sp. Proc. No. 057-B) was filed by herein petitioners over the child before the Regional Trial Court of Rizal, Fourth Judicial District, Branch 67 which, on 20 August 1987, rendered a decision granting the petition. The child was then known as Angelie Anne Fajardo. The court ordered that the child be "freed from parental authority of her natural parents as well as from legal obligation and maintenance to them and that from now on shall be, for all legal intents and purposes, known as Angelie Anne Cervantes, a child of herein petitioners and capable of inheriting their estate ." Sometime in March or April 1987, the adoptive parents, herein petitioners Nelson and Zenaida Cervantes, received a letter from the respondents demanding to be paid the amount of P150,000.00, otherwise, they would get back their child. Petitioners refused to accede to the demand. As a result, on 11 September 1987, while petitioners were out at work, the respondent Gina Carreon took the child from her "yaya" at the petitioners' residence in Angono, Rizal, on the pretext that she was instructed to do so by her mother. Respondent Gina Carreon brought the child to her house in Paraaque. Petitioners thereupon demanded the return of the child, but Gina Carreon refused, saying that she had no desire to give up her child for adoption and that the affidavit of consent to the adoption she had executed was not fully explained to her. She sent word to the petitioners that she will, however, return the child to the petitioners if she were paid the amount of P150,000.00. Felisa Tansingco, the social worker who had conducted the case study on the adoption and submitted a report thereon to the Regional Trial Court of Rizal in the adoption case, testified on 27 October 1987 before the Executive Judge, Regional Trial Court of Pasig in connection with the present petition. She declared that she had interviewed respondent Gina Carreon on 24 June 1987 in connection with the contemplated adoption of the child. During the interview, said respondent manifested to the social worker her desire to have the child adopted by the petitioners. In all cases involving the custody, care, education and property of children, the latter's welfare is paramount. The provision that no mother shall be separated from a child under five (5) years of age, will not apply where the Court finds compelling reasons to rule otherwise. In all controversies regarding the custody of minors, the foremost consideration is the moral, physical and social welfare of the child concerned, taking into account the resources and moral as well as social standing of the contending parents. Never has this Court deviated from this criterion. 853

University of the Cordilleras College of Law First Year C S.Y. 2013 - 2014 It is undisputed that respondent Conrado Fajardo is legally married to a woman other than respondent Gina Carreon, and his relationship with the latter is a common-law husband and wife relationship. His open cohabitation with co-respondent Gina Carreon will not accord the minor that desirable atmosphere where she can grow and develop into an upright and moral-minded person. Besides, respondent Gina Carreon had previously given birth to another child by another married man with whom she lived for almost three (3) years but who eventually left her and vanished. For a minor (like Angelie Anne C. Cervantes) to grow up with a sister whose "father" is not her true father, could also affect the moral outlook and values of said minor. Upon the other hand, petitioners who are legally married appear to be morally, physically, financially, and socially capable of supporting the minor and giving her a future better than what the natural mother (herein respondent Gina Carreon), who is not only jobless but also maintains an illicit relation with a married man, can most likely give her. Besides, the minor has been legally adopted by petitioners with the full knowledge and consent of respondents. A decree of adoption has the effect, among others, of dissolving the authority vested in natural parents over the adopted child, except where the adopting parent is the spouse of the natural parent of the adopted, in which case, parental authority over the adopted shall be exercised jointly by both spouses. The adopting parents have the right to the care and custody of the adopted child and exercise parental authority and responsibility over him. ACCORDINGLY, and as recommended by the Executive Judge, Regional Trial Court of Pasig, Hon. Eutropio Migrino, the Petition is GRANTED. The custody and care of the minor Angelie Anne Cervantes are hereby granted to petitioners to whom they properly belong, and respondents are ordered (if they still have not) to deliver said minor to the petitioners immediately upon notice hereof This resolution is immediately executory. SO ORDERED.

854

University of the Cordilleras College of Law First Year C S.Y. 2013 - 2014 Case Digest: Cervantes v. Fajardo G.R. No. 79955 Decided On: January 27, 1989, Ponente: Padilla, J. Facts: The child was born on Feb. 14, 1987 to parents who are common-law husband and wife (Fajardo and Carreon). Parents offered child for adoption to wifes sister and brother-in-law (the Cervantes), spouses, who took care and custody of the child when she was barely 2 years old. Affidavit of Consentto the adoption was signed by Carreon. On March-April of the same year, adoptive parents received a letter from the natural parents demanding them to pay P150,000.00, otherwise, they would get back their child. The Cervantes refused to accede to the demand. On September, while the Cervantes spouses were at work, Carreon took the child from the adoptive parents residence. Petitioners demanded the return of the child but Carreon refused saying that she had no desire to give up the child and that she signed the affidavit without fully knowing what it meant. However, she sent word to the petitioners stating that she would give back the child if the amount requested was paid. Social worker of the adoption case testified that when she interview Carreon, Carreon manifested her desire to have the child adopted by the Cervantes. Issue: Whether or not custody of the child should revert back to natural mother. Held: No. The paramount interest of the child would not be met if custody were granted to Fajardo (who is legally married to a woman other than Carreon) and the latter. His cohabitation with Carreon will not accord the minor that desirable atmosphere where she can grow and develop into an upright and moral-minded person. Besides, Carreon has another child through another man. For the child in question to grow up with a sister whose father is not her true father, could affect the moral outlook and values of the child in question. The minor has been legally adopted by petitioners with the full knowledge and consent of respondents. A decree of adoption has the effect, among others, of dissolving the authority vested in natural parents over the adopted child, except where the adopting parent is the spouse of the natural parent of the adopted, in which case, parental authority over the adopted shall be exercised jointly by both spouses.

855

University of the Cordilleras College of Law First Year C S.Y. 2013 - 2014 Republic vs Vergara 270 SCRA 206 G.R. No. 95551 March 20, 1997 Full Case REPUBLIC OF THE PHILIPPINES, petitioner, vs. HON. CONCEPCION S. ALARCON VERGARA, in her capacity as Presiding Judge of the Regional Trial Court, Third Judicial Region, Branch 62, Angeles City and SPOUSES SAMUEL ROBERT DYE, JR. and ROSALINA D. DYE, respondents. ROMERO, J.: On June 25, 1990, the spouses Samuel R. Dye, Jr. and Rosalina Due Dye filed a petition before the Regional Trial Court of Angeles City to adopt Maricel R. Due and Alvin R. Due, ages 13 and 12 years old, respectively, younger siblings of Rosalina. Samuel R. Dye, Jr, a member of the United States Air Force, is an American citizen who resided at the Clark Air Base in Pampanga. His wife Rosalina is a former Filipino who became a naturalized American. They have two children. Both Maricel and Alvin Due, as well as their natural parents, gave their consent to the adoption. After trial, the lower court rendered its decision on September 10, 1990 granting the petition and declaring Alvin and Maricel to be the children of the spouses Dye by adoption. Respondent Regional Trial Court disregarded the sixteen-year age gap requirement of the law, the spouses being only fifteen years and three months and fifteen years and nine months older than Maricel Due, on the ground that a literal implementation of the law would defeat the very philosophy behind adoption statutes, namely, to promote the welfare of a child. The court also found that the petitioning spouses are mentally and physically fit to adopt, possess good moral character, sufficient financial capability and love and affection for the intended adoptees. The Republic filed this petition for review on a pure question of law, contending that the spouses Dye are not qualified under the law to adopt Maricel and Alvin Due. The Court finds the petition meritorious and hereby grants it. As a general rule, aliens cannot adopt Filipino citizens as this is proscribed under Article 184 of the Family Code which states: "Art. 184. The following persons may not adopt: xxx xxx xxx (3) An alien, except: (a) A former Filipino citizen who seeks to adopt a relative by consanguinity; (b) One who seeks to adopt the legitimate child of his or her Filipino spouse; or (c) One who is married to a Filipino citizen and seeks to adopt jointly with his or her spouse a relative by consanguinity of the latter. Aliens not included in the foregoing exceptions may adopt Filipino children in accordance with the rules on inter-country adoption as may be provided by law." Samuel Robert Dye, Jr. who is an American and, therefore, an alien is disqualified from adopting the minors Maricel and Alvin Due because he does not fall under any of the three aforequoted exceptions laid down by the law. He is not a former Filipino citizen who seeks to adopt a relative by consanguinity. Nor does he seek to adopt his wife's legitimate child. Although he seeks to adopt with his wife her relatives by consanguinity, he is not married to a Filipino citizen, for Rosalina was already a naturalized American at the time the petition was filed, thus excluding him from the coverage of the exception. The law here does not provide for an alien who is married to a former Filipino citizen seeking to adopt jointly with his or her spouse a relative by consanguinity, as an exception to the general rule that aliens may not adopt. On her own, Rosalina Dye cannot adopt her brother and sister for the law mandates joint adoption by husband and wife, subject to exceptions. Article 29 of Presidential Decree No. 603 (Child and Youth Welfare Code) retained the Civil Code provision that husband and wife may jointly adopt. The Family

856

University of the Cordilleras College of Law First Year C S.Y. 2013 - 2014 Code amended this rule by scrapping the optional character of joint adoption and making it now mandatory. Article 185 of the Family Code provides: "Art. 185. Husband and wife must adopt, except in the following cases: (1) When one spouse seeks to adopt his own illegitimate child; (2) When one spouse seeks to adopt the legitimate child of the other." None of the above exceptions applies to Samuel and Rosalina Dye, for they did not petition to adopt the latter's child but her brother and sister. The Court has previously recognized the ineligibility of a similarly situated alien husband with a former Filipino wife seeking to adopt the latter's nephews and niece in the case of Republic v. Court of Appeals. Although the wife in said case was qualified to adopt under Article 184, paragraph 3 (a), she being a former Filipino who seeks to adopt a relative by consanguinity, she could not jointly adopt with her husband under Article 185 because he was an alien ineligible to adopt here in the Philippines. We are not unmindful of the main purpose of adoption statutes, which is the promotion of the welfare of children. Accordingly, the law should be construed liberally, in a manner that will sustain rather than defeat said purpose. The law must also be applied with compassion, understanding and less severity in view of the fact that it is intended to provide homes, love, care and education for less fortunate children. Regrettably, the Court is not in a position to affirm the trial court's decision favoring adoption in the case at bar, for the law is clear and it cannot be modified without violating the proscription against judicial legislation. Until such time however, that the law on the matter is amended, we cannot sustain the respondent-spouses' petition for adoption. WHEREFORE, the instant petition is hereby GRANTED. The Decision of the Regional Trial Court of Angeles City in Special Proceeding No. 4203 (In the Matter of the Petition for Adoption of the minors Maricel R. Due and Alvin R. Due), dated September 10, 1990 is REVERSED AND SET ASIDE. SO ORDERED.

857

University of the Cordilleras College of Law First Year C S.Y. 2013 - 2014 Case Digest Republic vs Vergara 270 SCRA 206 G.R. No. 95551 Decided on: March 20, 1997 Ponente: ROMERO, J.: Facts: On June 25, 1990, the spouses Samuel R. Dye, Jr. and Rosalina Due Dye filed a petition before the Regional Trial Court of Angeles City to adopt Maricel R. Due and Alvin R. Due, ages 13 and 12 years old, respectively, younger siblings of Rosalina. Samuel R. Dye, Jr, a member of the United States Air Force, is an American citizen who resided at the Clark Air Base in Pampanga. His wife Rosalina is a former Filipino who became a naturalized American. Lower court rendered its decision on September 10, 1990 granting the petition and declaring Alvin and Maricel to be the children of the spouses Dye by adoption. Regional Trial Court disregarded the sixteen-year age gap requirement of the law, the spouses being only fifteen years and three months and fifteen years and nine months older than Maricel Due, on the ground that a literal implementation of the law would defeat the very philosophy behind adoption statutes, namely, to promote the welfare of a child. Issue: Whether or not the granted adoption is valid Held: Samuel Robert Dye, Jr. who is an American and, therefore, an alien is disqualified from adopting the minors Maricel and Alvin Due because he does not fall under any of the three exceptions laid down by the law. He is not a former Filipino citizen who seeks to adopt a relative by consanguinity. Nor does he seek to adopt his wife's legitimate child. Although he seeks to adopt with his wife her relatives by consanguinity, he is not married to a Filipino citizen, for Rosalina was already a naturalized American at the time the petition was filed, thus excluding him from the coverage of the exception. On her own, Rosalina Dye cannot adopt her brother and sister for the law mandates joint adoption by husband and wife, subject to exceptions. None of the exceptions applies to Samuel and Rosalina Dye, for they did not petition to adopt the latter's child but her brother and sister.

858

University of the Cordilleras College of Law First Year C S.Y. 2013 - 2014 Republic vs Court of Appeals 205 SCRA 356 G.R. No. 92326 January 24, 1992 Full Case REPUBLIC OF THE PHILIPPINES, petitioner, vs. COURT OF APPEALS and ZENAIDA C. BOBILES, respondents. The Solicitor General for petitioner. Mariano B. Miranda for private respondent. REGALADO, J.: Dissatisfied with the decision of respondent Court of Appeals promulgated on February 20, 1990 which affirmed in toto the decision of Branch 2 of the Regional Trial Court of Legaspi City granting the petition of herein private respondent to adopt the minor Jason Condat, petitioner seeks the reversal thereof in the present petition for review on certiorari. On February 2, 1988, Zenaida Corteza Bobiles filed a petition to adopt Jason Condat, then six (6) years old and who had been living with her family since he was four (4) months old, before the Regional Trial Court of Legaspi City, docketed therein as Special Proceeding No. 1386. The court a quo, finding the petition to be sufficient in form and substance, issued an order dated February 15, 1988 setting the petition for hearing on March 28, 1988. The order was duly published, with copies thereof seasonably served on the Solicitor General; Assistant Provincial Fiscal Mediavillo, Jr. of Albay; Salvador Condat, father of the child; and the social worker assigned to the court. A copy of said order was posted on the bulletin board of the court and in the other places it had required for that purpose. Nobody appeared to oppose the petition. Compliance with the jurisdictional requirements having been proved at the hearing, the testimonies of herein private respondent, together with that of her husband, Dioscoro Bobiles, and one Ma. Luz Salameno of the Department of Social Welfare and Development were taken and admitted in the proceedings. On March 20, 1988, the trial court rendered judgment disposing as follows: ACCORDINGLY, it is declared that henceforth, the minor child, JASON CONDAT, be freed from all legal obligations of obedience and maintenance with respect to his natural parents, and be, to all intents and purposes, the child of the spouses Dioscoro and Zenaida Bobiles, and the surname of the child be changed to "Bobiles" which is the surname of the petitioner. Furnish the Office of the Solicitor General, Manila, the Department of Social Welfare and Development, Regional Office, Region V, Legaspi City, and the Local Civil Registrar of Tiwi, Albay, with copies of this decision. Herein petitioner appealed to the Court of Appeals which, as earlier stated, affirmed the aforesaid decision of the court below. Hence, this present petition with the following assignment of errors: 1. The Honorable Court of Appeals erred in ruling that the Family Code cannot be applied retroactively to the petition for adoption filed by Zenaida C. Bobiles; and 2 The Honorable Court of Appeals erred in affirming the trial court's decision which granted the petition to adopt Jason Condat in favor of spouses Dioscoro Bobiles and Zenaida C. Bobiles.

The petition for adoption was filed by private respondent Zenaida C. Bobiles on February 2, 1988, when the law applicable was Presidential Decree No. 603, the Child and Youth Welfare Code. Under said code, a petition for adoption may be filed by either of the spouses or by both of them. However, after the trial court rendered its decision and while the case was pending on appeal in the Court of Appeals, Executive Order No. 209, the Family Code, took effect on August 3, 1988. Under the said new law, joint adoption by husband and wife is mandatory.

859

University of the Cordilleras College of Law First Year C S.Y. 2013 - 2014 On the foregoing consideration, petitioner contends that the petition for adoption should be dismissed outright for it was filed solely by private respondent without joining her husband, in violation of Article 185 of the Family Code which requires joint adoption by the spouses. It argues that the Family Code must be applied retroactively to the petition filed by Mrs. Bobiles, as the latter did not acquire a vested right to adopt Jason Condat by the mere filing of her petition for adoption. We are not persuaded. Preliminarily, we observe that petitioner's theory implies that the non-inclusion of Dioscoro Bobiles as a co-petitioner is a jurisdictional defect, hence its prayer for an outright dismissal on that score. It could not be taking exception only on the ground of non-joinder since petitioner must be aware that non-joinder is not a ground for the dismissal of an action or a special proceeding. We further apprehend that this objection has been raised for the first time on appeal in respondent court. Nonetheless, we shall clarify petitioner's misgivings as postulated in its aforestated assignment of errors. Article 246 of the Family Code provides for retroactive effect of appropriate relevant provisions thereof, subject to the qualification that such retrospective application will not prejudice or impair vested or acquired rights in accordance with the Civil Code or other laws. A vested right is one whose existence, effectivity and extent does not depend upon events foreign to the will of the holder. The term expresses the concept of present fixed interest which in right reason and natural justice should be protected against arbitrary State action, or an innately just and imperative right which enlightened free society, sensitive to inherent and irrefragable individual rights, cannot deny. Vested rights include not only legal or equitable title to the enforcement of a demand, but also an exemption from new obligations created after the right has vested. Under the Child and Youth Welfare Code, private respondent had the right to file a petition for adoption by herself, without joining her husband therein. When Mrs. Bobiles filed her petition, she was exercising her explicit and unconditional right under said law. Upon her filing thereof, her right to file such petition alone and to have the same proceed to final adjudication, in accordance with the law in force at the time, was already vested and cannot be prejudiced or impaired by the enactment of a new law. When private respondent filed her petition in Special Proceeding No. 1386, the trial court acquired jurisdiction thereover in accordance with the governing law. Jurisdiction being a matter of substantive law, the established rule is that the jurisdiction of the court is determined by the statute in force at the time of the commencement of the action. We do not find in the present case such facts as would constitute it as an exception to the rule. The first error assigned by petitioner warrants a review of applicable local and foreign jurisprudence. For that purpose, we start with the premise that Article 185 of the Family Code is remedial in nature. Procedural statutes are ordinarily accorded a retrospective construction in the sense that they may be applied to pending actions and proceedings, as well as to future actions. However, they will not be so applied as to defeat procedural steps completed before their enactment. Procedural matters are governed by the law in force when they arise, and procedural statutes are generally retroactive in that they apply to pending proceedings and are not confined to those begun after their enactment although, with respect to such pending proceedings, they affect only procedural steps taken after their enactment. The rule that a statutory change in matters of procedure will affect pending actions and proceedings, unless the language of the act excludes them from its operation, is not so extensive that it may be used to validate or invalidate proceedings taken before it goes into effect, since procedure must be governed by the law regulating it at the time the question of procedure arises. The jurisdictional, as distinguished from the purely procedural, aspect of a case is substantive in nature and is subject to a more stringent rule. A petition cannot be dismissed by reason of failure to comply with a law which was not yet in force and effect at the time. As long as the petition for adoption was sufficient in form and substance in accordance with the law in governance at the time it was filed, the court acquires jurisdiction and retains it until it fully disposes of the case. To repeat, the jurisdiction of the court is determined by the statute in force at the time of the commencement of the action. Such jurisdiction of a court, whether in criminal or civil cases, once it attaches cannot be ousted by subsequent happenings or events, although of a character which would have prevented jurisdiction from attaching in the first instance.

860

University of the Cordilleras College of Law First Year C S.Y. 2013 - 2014 On the second issue, petitioner argues that, even assuming that the Family Code should not apply retroactively, the Court of Appeals should have modified the trial court's decision by granting the adoption in favor of private respondent Zenaida C. Bobiles only, her husband not being a petitioner. We do not consider this as a tenable position and, accordingly, reject the same. Although Dioscoro Bobiles was not named as one of the petitioners in the petition for adoption filed by his wife, his affidavit of consent, attached to the petition as Annex "B" and expressly made an integral part thereof, shows that he himself actually joined his wife in adopting the child. The pertinent parts of his written consent read as follows: xxx xxx xxx 2. That my wife, ZENAIDA O. CORTEZA BOBILES and I mutually desire to adopt as our child, a boy named JASON CONDAT, still a minor being six (6) years old, likewise residing at 18 C. Imperial Street, Legaspi City, Albay, also in the Philippines; That we are filing the corresponding Petition for Adoption of said minor child, JASON CONDAT, before the Juvenile and Domestic Relations court, now the Regional Trial Court in Legaspi City, Albay in the Philippines; That I, Dioscoro C. Bobiles as the husband and father, am giving my lawful consent to this adoption of said minor child, JASON CONDAT; That further, my wife ZENAIDA O. CORTEZA BOBILES, and I have continuously reared and cared for this minor child, JASON CONDAT since birth; That as a result thereof, my wife and I have developed a kind of maternal and paternal love for the boy as our very own, exercising therein the care, concern and diligence of a good father toward him; That I am executing this document, an AFFIDAVIT OF CONSENT for whatever it is worth in the premises as to the matter of adoption of this minor child, JASON CONDAT, by my wife ZENAIDA O. CORTEZA BOBILES and by me, DIOSCORO C. BOBILES, in any court of justice; (Emphasis supplied.) xxx xxx xxx The foregoing declarations, and his subsequent confirmatory testimony in open court, are sufficient to make him a co-petitioner. Under the circumstances then obtaining, and by reason of his foreign residence, he must have yielded to the legal advice that an affidavit of consent on his part sufficed to make him a party to the petition. This is evident from the text of his affidavit. Punctiliousness in language and pedantry in the formal requirements should yield to and be eschewed in the higher considerations of substantial justice. The future of an innocent child must not be compromised by arbitrary insistence of rigid adherence to procedural rules on the form of pleadings. We see no reason why the following doctrines in American law should not apply to this case and, for that matter, in our jurisdiction. It is a settled rule therein that adoption statutes, as well as matters of procedure leading up to adoption, should be liberally construed to carry out the beneficent purposes of the adoption institution and to protect the adopted child in the rights and privileges coming to it as a result of the adoption. The modern tendency of the courts is to hold that there need not be more than a substantial compliance with statutory requirements to sustain the validity of the proceeding; to refuse would be to indulge in such a narrow and technical construction of the statute as to defeat its intention and beneficial results or to invalidate proceedings where every material requirement of the statute was complied with. In support of this rule it is said that it is not the duty of the courts to bring the judicial microscope to bear upon the case in order that every slight defect may be enlarged and magnified so that a reason may be found for declaring invalid an act consummated years before, but rather to approach the case with the inclination to uphold such acts if it is found that there was a substantial compliance with the statute. The technical rules of pleading should not be stringently applied to adoption proceedings, and it is deemed more important that the petition should contain facts relating to the child and its parents, which may give information to those interested, than that it should be formally correct as a pleading. Accordingly, it is

3.

4. 5. 6.

7.

861

University of the Cordilleras College of Law First Year C S.Y. 2013 - 2014 generally held that a petition will confer jurisdiction if it substantially complies with the adoption statute, alleging all facts necessary to give the court jurisdiction. In determining whether or not to set aside the decree of adoption the interests and welfare of the child are of primary and paramount consideration. The welfare of a child is of paramount consideration in proceedings involving its custody and the propriety of its adoption by another, and the courts to which the application for adoption is made is charged with the duty of protecting the child and its interests and, to bring those interests fully before it, it has authority to make rules to accomplish that end. Ordinarily, the approval of the adoption rests in the sound discretion of the court. This discretion should be exercised in accordance with the best interests of the child, as long as the natural rights of the parents over the child are not disregarded. In the absence of a showing of grave abuse, the exercise of this discretion by the approving official will not be disturbed. In the case at bar, the rights concomitant to and conferred by the decree of adoption will be for the best interests of the child. His adoption is with the consent of his natural parents. The representative of the Department of Social Welfare and Development unqualifiedly recommended the approval of the petition for adoption and the trial court dispensed with the trial custody for several commendatory reasons, especially since the child had been living with the adopting parents since infancy. Further, the said petition was with the sworn written consent of the children of the adopters. The trial court and respondent court acted correctly in granting the petition for adoption and we find no reason to disturb the same. As found and aptly stated by respondent court: "Given the facts and circumstances of the case and considered in the light of the foregoing doctrine, We are of the opinion and so hold that the decree of adoption issued by the court a quo would go a long way towards promoting the welfare of the child and the enhancement of his opportunities for a useful and happy life." Adoption statutes, being humane and salutary, hold the interests and welfare of the child to be of paramount consideration. They are designed to provide homes, parental care and education for unfortunate, needy or orphaned children and give them the protection of society and family in the person of the adopted, as well as to allow childless couples or persons to experience the joys of parenthood and give them legally a child in the person of the adopted for the manifestation of their natural parental instincts. Every reasonable intendment should be sustained to promote and fulfill these noble and compassionate objectives of the law. WHEREFORE, the instant petition is hereby DENIED. SO ORDERED.

862

University of the Cordilleras College of Law First Year C S.Y. 2013 - 2014 Case Digest: Republic vs Court of Appeals G.R. No. 92326 Decided On: January 24, 1992 Ponente: REGALADO, J.: Facts: Dissatisfied with the decision of respondent Court of Appeals which affirmed in toto the decision of the RTC of Legaspi City granting the petition of herein private respondent to adopt the minor Jason Condat, petitioner seeks the reversal thereof in the present petition for review on certiorari. Zenaida Corteza Bobiles filed a petition to adopt Jason Condat, then six years old and who had been living with her family since he was four months old. The court a quo, finding the petition to be sufficient in form and substance, issued an order setting the petition for hearing. The order was duly published, with copies thereof seasonably served. A copy of said order was posted on the bulletin board of the court and in the other places it had required for that purpose. Nobody appeared to oppose the petition. The trial court rendered judgment disposing that the minor child, Jason Condat, be freed from all legal obligations of obedience and maintenance with respect to his natural parents, and be, to all intents and purposes, the child of the spouses Dioscoro and Zenaida Bobiles, and the surname of the child be changed to "Bobiles" which is the surname of the petitioner. Issue: Whether or not CA erred in affirming the trial court's decision which granted the petition to adopt Jason Condat in favor of spouses Bobiles. Ruling: The rights concomitant to and conferred by the decree of adoption will be for the best interests of the child. His adoption is with the consent of his natural parents. The trial court and respondent court acted correctly in granting the petition for adoption and we find no reason to disturb the same. Given the facts and circumstances of the case and considered in the light of the foregoing doctrine, SC holds that the decree of adoption issued by the court a quo would go a long way towards promoting the welfare of the child and the enhancement of his opportunities for a useful and happy life.

863

University of the Cordilleras College of Law First Year C S.Y. 2013 - 2014 Republic vs Court of Appeals 209 SCRA 189 G.R. No. 97906 May 21, 1992 Full Case REPUBLIC OF THE PHILIPPINES, petitioner, vs. COURT OF APPEALS and MAXIMO WONG, respondents. Public Attorney's Office for private respondent. REGALADO, J.: Petitioner seeks to set aside the judgment of respondent Court of Appeals in affirmance of the decision of the court a quo granting the petition filed by herein private respondent Maximo Wong for the change of his name to Maximo Alcala, Jr. which was his name prior to his adoption by Hoong Wong and Concepcion Ty Wong. The facts are undisputed. Private respondent Maximo Wong is the legitimate son of Maximo Alcala, Sr. and Segundina Y. Alcala. When he was but two and a half years old and then known as Maximo Alcala, Jr., and his sister Margaret Alcala, was then nine years old, they were, with the consent of their natural parents and by order of the court in Special Case No. 593 issued on September 9, 1967, adopted by spouses Hoong Wong and Concepcion Ty Wong, both naturalized Filipinos. Hoong Wong, now deceased, was an insurance agent while Concepcion Ty Wong was a high school teacher. They decided to adopt the children as they remained childless after fifteen years of marriage. The couples showered their adopted children with parental love and reared them as their own children. Upon reaching the age of twenty-two, herein private respondent, by then married and a junior Engineering student at Notre Dame University, Cotabato City, filed a petition to change his name to Maximo Alcala, Jr. It was averred that his use of the surname Wong embarrassed and isolated him from his relatives and friends, as the same suggests a Chinese ancestry when in truth and in fact he is a Muslim Filipino residing in a Muslim community, and he wants to erase any implication whatsoever of alien nationality; that he is being ridiculed for carrying a Chinese surname, thus hampering his business and social life; and that his adoptive mother does not oppose his desire to revert to his former surname. As earlier stated, on July 2, 1986, the matter was resolved in favor of private respondent, the trial court decreeing that, the jurisdictional requirements having been fully complied with, petitioner's prayer to change his name from Maximo Wong to Maximo Alcala, Jr. was granted. On appeal to respondent court, and over the opposition of petitioner Republic through the Solicitor General, the decision of the court below was affirmed in full, hence, this petition for review on certiorari. The lone issue to be settled is whether or not the reasons given by private respondent in his petition for change of name are valid, sufficient and proper to warrant the granting of said petition. The Solicitor General contends that private respondent's allegations of ridicule and/or isolation from family and friends were unsubstantiated and cannot justify the petition for change of name. He claims that for private respondent to cast aside the name of his adoptive father is crass ingratitude to the memory of the latter and to his adoptive mother who is still alive, despite her consent to the petition for change of name. Further, the Solicitor General posits that the reversion of Maximo Wong to his old name violates Articles 341 and 365 of the Civil Code, which requires an adopted child to use the surname of the adopter, and would identify him with his parents by nature, thus giving the impression that he has severed his relationship with his adoptive parents. In refutation, private respondent argues that he did as the law required, that is, upon adoption he used the surname of the adopter. However, being already emancipated, he can now decide what is best for and by himself. It is at this time that he realized that the Chinese name he carries causes him undue ridicule and embarrassment and affects his business and social life. In fact, his adoptive mother, being aware of his predicament, gave her consent to the petition for change of name, albeit making it clear that the same shall in no way affect the legal adoption, and even underwent the rigors of trial to substantiate her sworn statement. If his adoptive mother does not take offense nor feel any resentment, abhorrence or insecurity about his desire to change his name, private respondent avers that there can be no possible prejudice on her, much less the State.

864

University of the Cordilleras College of Law First Year C S.Y. 2013 - 2014 We feel that we should preface our review of this case with a clear comprehension of the legal significance of a person's name. For all practical and legal purposes, a man's name is the designation by which he is known and called in the community in which believes and is best known. It is defined as the word or combination of words by which a person is distinguished from other individuals and, also, as the label or appellation which he bears for the convenience of the world at large addressing him, of in speaking of or dealing with him. Names are used merely as one method of indicating the identity of persons; they are descriptive of persons for identification, since, the identity is the essential thing and it has frequently been held that, when identity is certain, a variance in, or misspelling of, the name is immaterial. The names of individuals usually have two parts: the given name or proper name, and the surname or family name. The given or proper name is that which is given to the individual at birth or baptism, to distinguish him from other individuals. The name or family name is that which identifies the family to which he belongs and is continued from parent to child. The given name may be freely selected by the parents for the child; but the surname to which the child is entitled is fixed by law. A name is said to have the following characteristics: (1) It is absolute, intended to protect the individual from being confused with others. (2) It is obligatory in certain respects, for nobody can be without a name. (3) It is fixed, unchangeable, or immutable, at least at the start, and may be changed only for good cause and by judicial proceedings. (4) It is outside the commerce of man, and, therefore, inalienable and intransmissible by act inter vivos or mortis causa. (5) It is imprescriptible. Title XIII, Book I of the Civil Code, in Articles 364 to 380, provides the substantive rules which regulate the use of surnames. Considering the subject and personalities involved in this present review, particular attention must be called to Article 365 which mandates that "(a)n adopted child shall bear the surname of the adopter," in correlation with Article 341 on the effects of adoption, among which is to"(e)ntitle the adopted person to use the adopter's surname." This same entitlement of an adopted child is maintained in Article 39(3), Title II of Presidential Decree No. 603, otherwise known as the Child and Youth Welfare Code. More recently, Executive Order No. 209, as amended by Executive Order No. 227, or the Family Code, echoes the same statutory right of an adopted child to use the surname of the adopter. Clearly, from the very wordings of the law, it may be inferred that this use of the surname of the adopter by the adopted child is both an obligation and a right. Under Article 376 by the Civil Code, "(n)o person can change his name or surname without judicial authority." The application for change of name thereunder involves a special proceeding governed by and conducted under the strictures of Rule 103 of the Rules of Court and one which involves substantial changes, with the declared objective of such judicial proceedings being the prevention of fraud. The purpose of the statutory procedure authorizing a change of personal name is simply to have, wherever possible, a record of the change, and in keeping with the object of the statute, court to which application is made should normally make its decree recording such change of name. A change of name is a special proceeding to establish the status of a person involving his relation with others, that is, his legal position in, or with regard to, the rest of the community. It is a proceeding in rem and, as such, strict compliance with all jurisdictional requirements, particularly on publication, is essential in order to vest the court with jurisdiction thereover. For this purpose, the only name that may be changed is the true or official name recorded in the civil register. To digress a little for purposes of clarification, the change of name contemplated under Article 376 and reglementarily implemented by Rule 103 must not be confused with and cannot be effected through the summary proceeding proposed in Article 412 of the some Code, as procedurally regulated by Rule 108 of the Rules, which refers only to correction of clerical errors, such as those which are visible to the eye or obvious to the understanding, or an error made by a clerk or transcriber, or a mistake in copying or writing, or some harmless or innocuous change and not those which will involve substantial changes. Turning now to the case at bar, we are guided by the jurisprudential dictum that the State has an interest in the names borne by individuals and entities for the purpose of identification, and a change of name is not a matter of right but of sound judicial discretion, to be exercised in the light of reasons adduced and the consequences that will likely follow; it is a privilege which may be granted only upon a showing of a proper or reasonable cause or compelling reason therefor. We find unacceptable the assertion of the Solicitor General that private respondent's allegation of ridicule and embarrassment due to the use of his present surname is unsubstantiated. 865

University of the Cordilleras College of Law First Year C S.Y. 2013 - 2014 The testimony of private respondent in the lower court bears out the existence of valid cause in his bid for change of name: ATTY. DUMAMBA: Q Now, after adoption, when you went to school, what did you use as your surname? A "Wong," sir. Q Now, after you adopted the surname "Wong?" in your studies, what did you observe? A I observed that "Wong" as a surname embarrassed me to my friends and when I go with Chinese friends I cannot talk Chinese. I am living in Campo Muslim, a Muslim community but no one can believe that I am Muslim. I have a little business of Furniture but I have little (sic) customer because no one believes me that I am Muslim. Q You want to inform this Honorable Court that this family name you are using which is "Wong" embarrassed you from (sic) your friends and relatives and also cause(d) damage to your business? A Yes sir. xxx xxx xxx ATTY. DUMAMBA: Q Now, considering that according to you, you are embarrassed because of the family name you are using, your friends shy away from you and it is a handicap in your business, what is your desire for the Court to do in order to help you? A Change my family name. Q From "Wong" to what do you want your surname changed? A "Alcala, Jr.", sir. xxx xxx xxx COURT: Q What is your purpose in changing your family name from Maximo Wong to Maximo Alcala, Jr.? A I feel embarrassed to my friends and also to my relatives and as I said I have a little business of furniture and only a few customers buying for the fact that they don't believe I am Muslim. Cross. ATTY. SERO: With the permission of the Honorable Court. Q Your father's name is Maximo Alcala, Sr., is he still alive? A Yes, sir. Q And what does your father say to this proposed changed (sic) of your name, your family name to your real family name given to you? A Yes, sir. Q They have no objection to it? A No, sir. Q Stated before this Honorable Court, the purpose why you wanted to change your name from "Wong" to "Alcala" is so that to avoid embarrassment because you are a Muslim and your Muslim relatives think that you are Chinese. A Yes, sir. Q Not for the purpose to hide anything or what not? A No, sir. The foregoing testimony of private respondent is materially corroborated by the testimony of private respondent's adoptive mother: Q Now, what did you observe to (sic) your son Maximo Wong after you and your husband adopted him? A When I adopted him and he used the surname "Wong" I observed that some of his relatives, cousins and friends seem to shy away from him and despise him in school that is why I agreed to change his name. We uphold these observations in the decision of respondent appellate court: The purpose of the law an allowing of change of name as contemplated by the provisions of Rule 103 of the Rules of Court is to give a person an opportunity to improve his personality and to provide his best interest. (Calderon vs. Republic, 19 SCRA 721). In granting or denying the petition for change of name, the question of proper and reasonable cause is left to the discretion of the court. The evidence presented need only be satisfactory to the court and not all the best evidence available is required. (Uy vs. Republic, L-22712, Nov. 25, 1965; Nacionales vs.

866

University of the Cordilleras College of Law First Year C S.Y. 2013 - 2014 Republic, L-18067, April 29, 1966; both cases cited in 1 SCRA 843). In the present case, We believe that the court a quo had exercised its discretion judiciously when it granted the petition. From the testimony of petitioner-appellee and of his adopter mother Concepcion Ty-Wong, We discern that said appellee was prompted to file the petition for change of name because of the embarrassment and ridicule his family name "Wong" brings in his dealings with his relatives and friends, he being a Muslim Filipino and living in a Muslim community. Another cause is his desire to improve his social and business life. It has been held that in the absence of prejudice to the state or any individual, a sincere desire to adopt a Filipino name to erase signs of a former alien nationality which only hamper(s) social and business life, is a proper and reasonable cause for change of name (Uy vs. Republic, L-22712, Nov. 25, 1965, Que Liong Sian vs. Republic, L-23167, Aug. 17, 1967, 20 SCRA 1074). Justice dictates that a person should be allowed to improve his social standing as long as in doing so, he does not cause prejudice or injury to the interest of the State or other persons (Calderon vs. Republic, supra). Nothing whatsoever is shown in the record of this case that such prejudice or injury to the interest of the state or of other persons would result in the change of petitioner's name. It bears stressing at this point that to justify a request for change of name, petitioner must show not only some proper or compelling reason therefor but also that he will be prejudiced by the use of his true and official name. Among the grounds for change of name which have been held valid are: (a) When the name is ridiculous, dishonorable or extremely difficult to write or pronounce; (b) When the change results as a legal consequence, as in legitimation; (c) When the change will avoid confusion; (d) Having continuously used and been known since childhood by a Filipino name, unaware of her alien parentage; (e) A sincere desire to adopt a Filipino name to erase signs of former alienage, all in good faith and without prejudicing anybody; and (f) When the surname causes embarrassment and there is no showing that the desired change of name was for a fraudulent purpose or that the change of name would prejudice public interest. In granting or denying petitions for change of name, the question of proper and reasonable cause is left to the sound discretion of the court. The evidence presented need only be satisfactory to the court and not all the best evidence available. Summarizing, in special proceedings for change of name, what is involved is not a mere matter of allowance or disallowance of the request, but a judicious evaluation of the sufficiency and propriety of the justifications advanced in support thereof, mindful of the consequent results in the event of its grant and with the sole prerogative for making such determination being lodged in the courts. While it is true that the statutory fiat under Article 365 of the Civil Code is to the effect that an adopted child shall bear the surname of the adopter, it must nevertheless be borne in mind that the change of the surname of the adopted child is more an incident rather than the object of adoption proceedings. The act of adoption fixes a status, viz., that of parent and child. More technically, it is an act by which relations of paternity and affiliation are recognized as legally existing between persons not so related by nature. It has been defined as the taking into one's family of the child of another as son or daughter and heir and conferring on it a title to the rights and privileges of such. The purpose of an adoption proceeding is to effect this new status of relationship between the child and its adoptive parents, the change of name which frequently accompanies adoption being more an incident than the object of the proceeding. The welfare of the child is the primary consideration in the determination of an application for adoption. On this point, there is unanimous agreement. It is the usual effect of a decree of adoption to transfer from the natural parents to the adoptive parents the custody of the child's person, the duty of obedience owing by the child, and all other legal consequences and incidents of the natural relation, in the same manner as if the child had been born of such adoptive parents in lawful wedlock, subject, however, to such limitations and restrictions as may be by statute imposed. More specifically under the present state of our law, the Family Code, superseding the pertinent provisions of the Civil Code and of the Child and Youth Welfare Code on the matter, relevantly provides in this wise with regard to the issue involved in this case: Art. 189. Adoption shall have the following effects: (1) For civil purposes, the adopted shall be deemed to be the legitimate child of the adopters and both shall acquire the reciprocal rights and obligations arising from the relationship of parent and child, including the right of the adopted to use the surname of the adopters; (Emphasis supplied.)

867

University of the Cordilleras College of Law First Year C S.Y. 2013 - 2014 The Solicitor General maintains the position that to sustain the change of name would run counter to the behest of Article 365 of the Civil Code and the ruling in Manuel vs. Republic that "one should not be allowed to use a surname which otherwise he is not permitted to employ under the law," and would set a bad example to other persons who might also seek a change of their surnames on lame excuses. While we appreciate the Solicitor General's apprehensions and concern, we find the same to be unfounded. We do not believe that by reverting to his old name, private respondent would then be using a name which he is prohibited by law from using. True, the law prescribes the surname that a person may employ; but the law does not go so far as to unqualifiedly prohibit the use of any other surname, and only subjects such recourse to the obtention of the requisite judicial sanction. What the law does not prohibit, it permits. If we were to follow the argument of the Solicitor General to its conclusion, then there will never be any possibility or occasion for any person, regardless of status, to change his name, in view of the supposed subsequent violation of the legal imperative on the use of surnames in the event that the petition is granted. Rule 103 of the Rules of Court would then be rendered inutile. This could hardly have been the intendment of the law. A petition for change of name is a remedy allowed under our law only by way of exception to the mandatory provisions of the Civil Code on the use of surnames. The law fixes the surname that may be used by a person, at least inceptively, and it may be changed only upon judicial permission granted in the exercise of sound discretion. Section 1 of Rule 103, in specifying the parties who may avail of said remedy, uses the generic term "persons" to signify all natural persons regardless of status. If a legitimate person may, under certain judicially accepted exceptional circumstances, petition the court for a change of name, we do not see any legal basis or logic in discriminating against the availment of such a remedy by an adopted child. In other words, Article 365 is not an exception, much less can it bar resort, to Rule 103. We are of the view that the circumstances herein obtaining are within the ambit of the established exceptions and find merit in private respondent's submission: Rule 103 of the Rules of Court has its primordial purpose which (State) is to give a person in opportunity to improve his personality and provide his best interest (Calderon vs. Republic, 19 SCRA 721). In the instant case, the court a quo found the petition of Maximo Wong for change of name justifiable after due hearing, thus its factual findings and appreciation of testimonies count heavily and need not be disturbed unless for strong and cogent reasons because the trial court is in a better position to examine real evidence as well as to observe the demeanor of the witnesses while testifying in the case (Baliwag Transit, Inc. vs. CA, 147 SCRA 82). Moreover, the trial court could take judicial notice of other existing factors in the community where herein respondent lives which it considers material in its judicious determination of the case. . . . Additionally, herein respondent is already of age and as such he can decide what is best for him. His experience with regards (sic) his social and business dealings is personal and it is only him (sic) who can attest to the same. Finding his predicament's proper remedy is solely through legal process, herein respondent accordingly filed a petition pursuant to Rule 103 of the Rules of Court which was granted by the Court a quo. Besides, we have faith in the circumspection of our lower courts and that, in the exercise of their discretion, said courts shall consider petitions for change of name only on cogent and meritorious grounds as would justify the granting of such applications. We do not expect our trial courts to cater or give in to the whim or caprice of an applicant, aside from the fact that there is always the safeguard and corrective interdiction of appellate review. It is not fair to construe the desired reversion of private respondent to the use of the name of his parents by nature as cross ingratitude. To go by the Solicitor General's suggestion that private respondent should have his adoption revoked if he wants to use the surname of his natural father would be to exact too clear a toll for making use of an appropriate and valid remedy available under the law.

868

University of the Cordilleras College of Law First Year C S.Y. 2013 - 2014 Herein private respondent, before he filed the petition for change of name, asked for his adoptive mother's permission to do so: Q Now, in filing this petition for change of surname, you had talked with your adopted mother? A Yes, sir. Q Did you ask permission from her whether she wants you to change the surname? A Yes, sir. True enough, the above testimony of private respondent was confirmed by his adoptive mother in this manner: Q How are you related to Maximo Wong? A My adopted son. Q He is your adopted son, did your son talk to you when he filed this petition for change of his surname? A Yes, he even tried to ask me and I said, alright if you want to change. xxx xxx xxx Q Now, when you agreed to the filing of this petition for change of name, did you reduce your consent in writing? A Yes, sir, I agreed also so that his business will prosper because he is already Alcila and not Wong because Wong they said is Chinese. As proof of her assent to the filing of said petition (her husband having already passed away), Concepcion Ty Vda. de Wong executed an affidavit in Cotabato City on May 27, 1985, with these textual declarations: That I am the same and identical person, who is the surviving adapted (sic) parent of Maximo Wong. That I personally discovered it myself from the time my adapted (sic) son Maximo used the surname of my late husband Wong, his relatives and childhood friends shy away from him because he is branded as a son of a chinese which is different from them whose parents are muslim Filipinos; That I pity my son who is often rediculed (sic) by his friends and relatives because of his family name Wong, hence, in order not to humper (sic) his social and business life in the future, I am voluntarily and of my own free will without being forced, coerced, or intimidated give (sic) my consent to his desire to change his desire to change his surname without affecting however the legal adoption granted by the Court on September 9, 1967, making him as one of my legal and compulsory heir (sic). That I am executing this affidavit to attest to the truth of all the above mentioned facts and for all legal intent (sic) and purposes. There could be no other plausible reason for private respondent to first secure has adoptive mother's consent before resorting to the questioned legal recourse other than the parental respect and reverence which is owed by and to be expected of a dutiful child. If private respondent was such an ingrate, as the Solicitor General would have us believe, he would not have bothered to seek his adoptive mother's counsel. In the same breath, had his adoptive mother regarded him as an ungrateful adoptee, she would not have executed the affidavit above quoted, much less testify in his behalf at the hearing of his petition. Moreover, worthy of note is the fact that private respondent's adoptive mother emphasized that she executed the above affidavit "without affecting the legal adoption granted by the Court on September 9, 1967, making him as one of my legal and compulsory heir(s)." This is incontrovertible proof that she never entertained any misgivings or reservations with respect to her consent to his petition. This likewise dispels any possible confusion as to private respondent's legal status or adoptive paternity and his successional rights. Concordantly, we have heretofore held that a change of name does not define or effect a change in one's existing family relations or in the rights and duties flowing therefrom. It does not alter one's legal capacity, civil status or citizenship; what is altered is only the name WHEREFORE, the petition is DENIED and the decision of respondent Court of Appeals is hereby AFFIRMED in toto. SO ORDERED. 869

University of the Cordilleras College of Law First Year C S.Y. 2013 - 2014 Case Digest: Republic vs Court of Appeals 209 SCRA 189 G.R. No. 97906 Decided on: May 21, 1992 Ponente: REGALADO, J.: FACTS: Maximo Wong is the legitimate son of Maximo Alcala Sr. and Segundina Alcala. When he was two and a half years old and then known as Maximo Alcala Jr. and his sister Margaret Alcala, was then nine years old, they were, with the consent of their natural parents and order of the court, adopted by spouses Hoong Wong and Concepcion Ty Wong, both naturalized Filipinos. They decided to adopt the children as they remained childless after fifteen years if marriage. Upon reaching the age twenty-two, herein private respondent, filed a petition to change his name to Maximo Alcala Jr. It was averred that his use of the surname Wong embarrassed and isolated him from his relatives and friends, as the name suggest a Chinese ancestry when in truth and in fact he is a Muslim Filipino residing in a Muslim community, and he wants to erase any implication whatsoever of alien nationality; that he is being ridiculed for carrying a Chinese surname, thus hampering his business and social life; and that his adoptive mother does not oppose his desire to revert to his former surname. ISSUE: Whether or not the reasons given by private respondent in his petition for change of name are valid, sufficient, and proper to warrant the granting of said petition. HELD: Yes, the reasons given in his petition for change of name are valid, sufficient, and proper to warrant the granting of said petition. The change of name is justifiable because of the embarrassment and ridicule his family name "Wong" brings in his dealings with his relatives and friends, he being a Muslim Filipino and living in a Muslim community. Another justifiable cause is his desire to improve his social and business life. In granting and denying petitions for change of name, the question of proper and reasonable cause is left to the sound discretion of the court. The evidence presented need only be satisfactory to the court and not all best evidence available. Among the grounds for change of name which have been held valid are; a. When the name is ridiculous, dishonorable, or extremely difficult to write or pronounce. b. When the change results as legal consequence, as in legitimation. c. When change will avoid confusion. d. Having continuously used and been known since childhood by a Filipino name, unaware of alien parentage. e. Sincere desire to adopt a Filipino name to erase signs of former alienage, all in good faith and without prejudicing anybody. f. When the surname causes embarrassment and there is no showing that the desired change of name was for a fraudulent purpose or that change of name would prejudice public interest. Rule 103 of the Rules of Court has its primordial purpose which is to give a person an opportunity to improve his personality and provide his best interest. Concordantly, the Court held that a change of name does not define or effect a change in one's existing family relations or in the rights and duties flowing therefrom. It does not alter one's legal capacity, civil status, or citizenship; what is altered is only the name.

870

University of the Cordilleras College of Law First Year C S.Y. 2013 - 2014 Republic vs Court of Appeals 227 SCRA 401 G.R. No. 100835 October 26, 1993 Full Case REPUBLIC OF THE PHILIPPINES, petitioner, vs. THE HONORABLE COURT OF APPEALS and the SPOUSES JAMES ANTHONY HUGHES and LENITA MABUNAY HUGHES, respondents. The Solicitor General for petitioner. Westremundo y. De Guzman for private respondents. VITUG, J.: James Anthony Hughes, a natural born citizen of the United States of America, married Lenita Mabunay Hughes, a Filipino Citizen, who herself was later naturalized as a citizen of that country. On 29 June 1990, the spouses jointly filed a petition with the Regional Trial Court of Angeles City, Branch 60, to adopt Ma. Cecilia, Neil and Maria, all surnamed Mabunay, minor niece and nephews of Lenita, who had been living with the couple even prior to the filing of the petition. The minors, as well as their parents, gave consent to the adoption. On 29 November 1990, the Regional Trial Court rendered a decision granting the petition. a petition for Review on Certiorari was filed with this Court, assailing the trial court's decision. This Court referred the case to the Court of Appeals which, on 09 July 1991, affirmed the trial court's decision. Hence, the present petition. The petitioner assigned a lone error on the part of the respondent court, thus THE LOWER COURT ERRED IN GRANTING THE PETITION FOR ADOPTION OF SPOUSES JAMES ANTHONY HUGHES AND LENITA MABUNAY HUGHES BECAUSE THEY ARE NOT QUALIFIED TO ADOPT UNDER PHILIPPINE LAW. It is clear that James Anthony Hughes is not qualified to adopt. Executive Order No. 209, otherwise known as "The Family Code of the Philippines," is explicit. Art. 184. The following persons may not adopt: (1) The guardian with respect to the ward prior to the approval of the final accounts rendered upon the termination of their guardianship relation; (2) Any person who has been convicted of a crime involving moral turpitude; (3) An alien, except: (a) A former Filipino citizen who seeks to adopt a relative by consanguinity; (b) One who seeks to adopt the legitimate child of his or her Filipino spouse; or (c) One who is married to a Filipino citizen and seeks to adopt jointly with his or her Filipino spouse a relative by consanguinity of the latter. Aliens not included in the foregoing exceptions may adopt Filipino children in accordance with the rules in inter-country adoption as may be provided by law. While James Anthony unquestionably is not permitted to adopt under any of the exceptional cases enumerated in paragraph (3) of the aforequoted article, Lenita, however, can qualify pursuant to paragraph (3)(a). The problem in her case lies, instead, with Article 185 of Executive Order No. 209, expressing as follows: Art. 185. Husband and wife must jointly adopt, except in the following cases: (1) When one spouse seeks to adopt his own illegitimate child; or (2) When one spouse seeks to adopt the legitimate child of the other. Lenita may not thus adopt alone since Article 185 requires a joint adoption by the husband and the wife, a condition that must be read along together with Article 184.

871

University of the Cordilleras College of Law First Year C S.Y. 2013 - 2014 The old law on adoption, Presidential Decree No. 603 (The Child and Youth Welfare Code), exactly adopted that found in then Article 336 of the Civil Code. Article 29, Section B, Chapter I, Title II, of the said decree provided: Art. 29. Husband and wife may jointly adopt. In such case, parental authority shall be exercised as if the child were their own by nature. Observe that the law then in force used the word "may" under which regime, a joint adoption by the spouses was apparently not made obligatory. The provision was later amended, however by Executive Order No. 91, dated 17 December 1986, of President Corazon C. Aquino. The new Article 29 expressed, thus Art. 29. Husband and wife may jointly adopt. In such case, parental authority shall be exercised as if the child were their own by nature. If one of the spouses is an alien, both husband and wife shall jointly adopt. Otherwise, the adoption shall not be allowed. As amended by Executive Order 91, Presidential Decree No. 603, had thus made it mandatory for both the spouses to jointly adopt when one of them was an alien. The law was silent when both spouses were of the same nationality. The Family Code has resolved any possible uncertainty. Article 185 thereof now expresses the necessity for joint adoption by the spouses except in only two instances (1) When one spouse seeks to adopt his own legitimate child; or (2) When one spouse seeks to adopt the legitimate child of the other. It is in the foregoing cases when Article 186 of the Code, on the subject of parental authority, can aptly find governance. Article 186. In case husband and wife jointly adopt or one spouse adopts the legitimate child of the other, joint parental authority shall be exercised by the spouses in accordance with this Code. The respondent court, in affirming the grant of adoption by the lower court, has theorized that James Anthony should merely be considered a "nominal or formal party" in the proceedings. This view of the appellate court cannot be sustained. Adoption creates a status that is closely assimilated to legitimate paternity and filiation with corresponding rights and duties that necessarily flow from adoption, such as, but not necessarily confined to, the exercise of parental authority, use of surname of the adopter by the adopted, as well as support and successional rights. These are matters that obviously cannot be considered inconsequential to the parties. We are not unmindful of the possible benefits, particularly in this instance, that an adoption can bring not so much for the prospective adopting parents as for the adopted children themselves. We also realize that in proceedings of this nature, paramount consideration is given to the physical, moral, social and intellectual welfare of the adopted for whom the law on adoption has in the first place been designed. When, however, the law is clear and no other choice is given, we must obey its full mandate.

Even then, we find it difficult to conclude this opinion without having to call the attention of the appropriate agencies concerned to the urgency of addressing the issue on inter-country adoption, a matter that evidently is likewise espoused by the Family Code (Article 184, last paragraph, Family Code). WHEREFORE, the petition is GRANTED and the decision of the respondent court is REVERSED and SET ASIDE. No costs. SO ORDERED.

872

University of the Cordilleras College of Law First Year C S.Y. 2013 - 2014 Case Digest: Republic vs Court of Appeals G.R. No. 100835 Decided On: October 26, 1993 Ponente: VITUG, J.: Facts: James Hughes, a natural born citizen of the United States of America, married Lenita Mabunay, a Filipino Citizen, who herself was later naturalized as a citizen of that country. The spouses jointly filed a petition with the RTC to adopt the minor niece and nephews of Lenita, who had been living with the couple even prior to the filing of the petition. The minors, as well as their parents, gave consent to the adoption. The RTC rendered a decision granting the petition. Issue: Can the spouses adopt the minors? Held: While James Anthony unquestionably is not permitted to adopt under any of the exceptional cases enumerated in paragraph (3) of the aforequoted article, Lenita, however, can qualify pursuant to paragraph (3)(a). Lenita may not thus adopt alone since Article 185 requires a joint adoption by the husband and the wife, a condition that must be read along together with Article 184. Art 185 provides: Art. 185. Husband and wife must jointly adopt, except in the following cases: (1) When one spouse seeks to adopt his own illegitimate child; or (2) When one spouse seeks to adopt the legitimate child of the other. As amended by Executive Order 91, Presidential Decree No. 603, had thus made it mandatory for both the spouses to jointly adopt when one of them was an alien. The law was silent when both spouses were of the same nationality. The Family Code has resolved any possible uncertainty. Article 185 thereof now expresses the necessity for joint adoption by the spouses except in only two instances: (1)When one spouse seeks to adopt his own legitimate child; or (2) When one spouse seeks to adopt the legitimate child of the other. It is in the foregoing cases when Article 186 of the Code, on the subject of parental authority, can aptly find governance. Article 186. In case husband and wife jointly adoptor one spouse adopts the legitimate child of the other, joint parental authority shall be exercised by the spouses in accordance with this Code.

873

University of the Cordilleras College of Law First Year C S.Y. 2013 - 2014 Republic vs Toledano 233 SCRA 9 G.R. No. 94147 June 8, 1994 Full Case REPUBLIC OF THE PHILIPPINES, petitioner, vs. HONORABLE RODOLFO TOLEDANO, in his capacity as Presiding Judge of the Regional Trial Court, Third Judicial Region, Branch 69, Iba, Zambales and SPOUSES ALVIN A. CLOUSE and EVELYN A. CLOUSE, respondents. The Solicitor General for petitioner. R.M. Blanco for private respondents. PUNO, J.: Before us is a petition for review on certiorari of the decision of the Regional Trial Court of Iba, Zambales, Branch 69, in Special Proceeding No. RTC-140-I, entitled, "In the Matter of the Adoption of the Minor named Solomon Joseph Alcala" raising a pure question of law. The sole issue for determination concerns the right of private respondents spouses Alvin A. Clouse and Evelyn A. Clouse who are aliens to adopt under Philippine Law. There is no controversy as to the facts. On February 21, 1990, in a verified petition filed before the Regional Trial Court of Iba, Zambales, private respondents spouses Clouse sought to adopt the minor, Solomon Joseph Alcala, the younger brother of private respondent Evelyn A. Clouse. In an Order issued on March 12, 1990, the petition was set for hearing on April 18, 1990. The said Order was published in a newspaper of general circulation in the province of Zambales and City of Olongapo for three (3) consecutive weeks. The principal evidence disclose that private respondent Alvin A. Clouse is a natural born citizen of the United States of America. He married Evelyn, a Filipino on June 4, 1981 at Olongapo City. On August 19, 1988, Evelyn became a naturalized citizen of the United States of America in Guam. They are physically, mentally, morally, and financially capable of adopting Solomon, a twelve (12) year old minor. Since 1981 to 1984, then from November 2, 1989 up to the present, Solomon Joseph Alcala was and has been under the care and custody of private respondents. Solomon gave his consent to the adoption. His mother, Nery Alcala, a widow, likewise consented to the adoption due to poverty and inability to support and educate her son. Mrs. Nila Corazon Pronda, the social worker assigned to conduct the Home and Child Study, favorably recommended the granting of the petition for adoption. Finding that private respondents have all the qualifications and none of the disqualifications provided by law and that the adoption will redound to the best interest and welfare of the minor, respondent judge rendered a decision on June 20, 1990, disposing as follows: WHEREFORE, the Court grants the petition for adoption filed by Spouses Alvin A. Clouse and Evelyn A. Clouse and decrees that the said minor be considered as their child by adoption. To this effect, the Court gives the minor the rights and duties as the legitimate child of the petitioners. Henceforth, he shall be known as SOLOMON ALCALA CLOUSE. The Court dissolves parental authority bestowed upon his natural parents and vests parental authority to the herein petitioners and makes him their legal heir. Pursuant to Article 36 of P.D. 603 as amended, the decree of adoption shall be effective as of the date when the petition was filed. In accordance with Article 53 of the same decree, let this decree of adoption be recorded in the corresponding government agency, particularly the Office of the Local Civil Registrar of Merida, Leyte where the minor was born. The said office of the Local Civil Registrar is hereby directed to issue an amended certificate of live birth to the minor adopted by the petitioners. Let copies of this decision be furnished (sic) the petitioners, DSWD, Zambales Branch, Office of the Solicitor General and the Office of the Local Civil Registrar of Merida, Leyte. 874

University of the Cordilleras College of Law First Year C S.Y. 2013 - 2014 SO ORDERED. Petitioner, through the Office of the Solicitor General appealed to us for relief, contending: THE LOWER COURT ERRED IN GRANTING THE PETITION FOR ADOPTION OF ALVIN AND EVELYN CLOUSE, BECAUSE THEY ARE NOT QUALIFIED TO ADOPT UNDER PHILIPPINE LAW. We rule for petitioner. Under Articles 184 and 185 of Executive Order (E.O.) No. 209, otherwise known as "The Family Code of the Philippines", private respondents spouses Clouse are clearly barred from adopting Solomon Joseph Alcala. Article 184, paragraph (3) of Executive Order No. 209 expressly enumerates the persons who are not qualified to adopt, viz.: (3) An alien, except: (a) A former Filipino citizen who seeks to adopt a relative by consanguinity; (b) One who seeks to adopt the legitimate child of his or her Filipino spouse; or (c) One who is married to a Filipino citizen and seeks to adopt jointly with his or her spouse a relative by consanguinity of the latter. Aliens not included in the foregoing exceptions may adopt Filipino children in accordance with the rules on inter-country adoption as may be provided by law. There can be no question that private respondent Alvin A. Clouse is not qualified to adopt Solomon Joseph Alcala under any of the exceptional cases in the aforequoted provision. In the first place, he is not a former Filipino citizen but a natural born citizen of the United States of America. In the second place, Solomon Joseph Alcala is neither his relative by consanguinity nor the legitimate child of his spouse. In the third place, when private respondents spouses Clouse jointly filed the petition to adopt Solomon Joseph Alcala on February 21, 1990, private respondent Evelyn A. Clouse was no longer a Filipino citizen. She lost her Filipino citizenship when she was naturalized as a citizen of the United States in 1988. Private respondent Evelyn A. Clouse, on the other hand, may appear to qualify pursuant to paragraph 3(a) of Article 184 of E.O. 209. She was a former Filipino citizen. She sought to adopt her younger brother. Unfortunately, the petition for adoption cannot be granted in her favor alone without violating Article 185 which mandates a joint adoption by the husband and wife. It reads: Article 185. (1) (2) Husband and wife must jointly adopt, except in the following cases: When one spouse seeks to adopt his own illegitimate child; or When one spouse seeks to adopt the legitimate child of the other.

Article 185 requires a joint adoption by the husband and wife, a condition that must be read along together with Article 184. The historical evolution of this provision is clear. Presidential Decree 603 (The Child and Youth Welfare Code), provides that husband and wife "may" jointly adopt. Executive Order No. 91 issued on December 17, 1986 amended said provision of P.D. 603. It demands that both husband and wife "shall" jointly adopt if one of them is an alien. It was so crafted to protect Filipino children who are put up for adoption. The Family Code reiterated the rule by requiring that husband and wife "must" jointly adopt, except in the cases mentioned before. Under the said new law, joint adoption by husband and wife is mandatory. This is in consonance with the concept of joint parental authority over the child, which is the ideal situation. As the child to be adopted is elevated to the level of a legitimate child, it is but natural to require the spouses to adopt jointly. The rule also insures harmony between the spouses. In a distinctly similar case, we held:

875

University of the Cordilleras College of Law First Year C S.Y. 2013 - 2014 As amended by Executive Order 91, Presidential Decree No. 603, had thus made it mandatory for both the spouses to jointly adopt when one of them was an alien. The law was silent when both spouses were of the same nationality. The Family Code has resolved any possible uncertainty. Article 185 thereof expresses the necessity for a joint adoption by the spouses except in only two instances (1) When one spouse seeks to adopt his own illegitimate child; or (2) When one spouse seeks to adopt the legitimate child of the other. It is in the foregoing cases when Article 186 of the Code, on the parental authority, can aptly find governance. Article 186. In case husband and wife jointly adopt or one spouse adopts the legitimate child of the other, jointly parental authority shall be exercised by the spouses in accordance with this Code. Article 185 is all too clear and categorical and there is no room for its interpretation. There is only room for application. We are not unaware that the modern trend is to encourage adoption and every reasonable intendment should be sustained to promote that objective. Adoption is geared more towards the promotion of the welfare of the child and enhancement of his opportunities for a useful and happy life. It is not the bureaucratic technicalities but the interest of the child that should be the principal criterion in adoption cases. Executive Order 209 likewise upholds that the interest and welfare of the child to be adopted should be the paramount consideration. These considerations notwithstanding, the records of the case do not evince any fact as would justify us in allowing the adoption of the minor, Solomon Joseph Alcala, by private respondents who are aliens. WHEREFORE, the petition is GRANTED. The decision of the lower court is REVERSED and SET ASIDE. No costs. SO ORDERED.

876

University of the Cordilleras College of Law First Year C S.Y. 2013 - 2014 Case Digest: Republic vs Toledano G.R. No. 94147 Decided on: June 8, 1994 Ponente: PUNO, J.: Facts: Spouses Clouse sought to adopt the minor, Solomon Joseph Alcala, the younger brother of private respondent Evelyn A. Clouse. Alvin A. Clouse is a natural born citizen of the United States of America. He married Evelyn, a Filipino on June 4, 1981 at Olongapo City. On August 19, 1988, Evelyn became a naturalized citizen of the United States of America in Guam. Issue: Whether or not respondents spouses Alvin A. Clouse and Evelyn A. Clouse who are aliens can adopt under Philippine Law. Held: There can be no question that private respondent Alvin A. Clouse is not qualified to adopt Solomon Joseph Alcala under any of the exceptional cases in the aforequoted provision. In the first place, he is not a former Filipino citizen but a natural born citizen of the United States of America. In the second place, Solomon Joseph Alcala is neither his relative by consanguinity nor the legitimate child of his spouse. In the third place, when private respondents spouses Clouse jointly filed the petition to adopt Solomon Joseph Alcala on February 21, 1990, private respondent Evelyn A. Clouse was no longer a Filipino citizen. She lost her Filipino citizenship when she was naturalized as a citizen of the United States in 1988. Private respondent Evelyn A. Clouse, on the other hand, may appear to qualify pursuant to paragraph 3(a) of Article 184 of E.O. 209. She was a former Filipino citizen. She sought to adopt her younger brother. Unfortunately, the petition for adoption cannot be granted in her favor alone without violating Article 185 which mandates a joint adoption by the husband and wife.

877

University of the Cordilleras College of Law First Year C S.Y. 2013 - 2014

XIV. Support

878

University of the Cordilleras College of Law First Year C S.Y. 2013 - 2014 David vs Court of Appeals 250 SCRA 82 G.R. No. 111180 November 16, 1995 Full Case DAISIE T. DAVID, petitioner, vs. COURT OF APPEALS, RAMON R. VILLAR, respondents. MENDOZA, J.: Petitioner Daisie T. David worked as secretary of private respondent Ramon R. Villar, a businessman in Angeles City. Private respondent is a married man and the father of four children, all grown-up. After a while, the relationship between petitioner and private respondent developed into an intimate one, as a result of which a son, Christopher J., was born on March 9, 1985 to them. Christopher J. was followed by two more children, both girls, namely Christine, born on June 9, 1986, and Cathy Mae on April 24, 1988. The relationship became known to private respondent's wife when Daisie took Christopher J, to Villar's house at Villa Teresa in Angeles City sometime in 1986 and introduced him to Villar's legal wife. After this, the children of Daisie were freely brought by Villar to his house as they were eventually accepted by his legal family. In the summer of 1991, Villar asked Daisie to allow Christopher J., then six years of age, to go with his family to Boracay. Daisie agreed, but after the trip, Villar refused to give back the child. Villar said he had enrolled Christopher J. at the Holy Family Academy for the next school year. On July 30, 1991, Daisie filed a petition for habeas corpus on behalf of Christopher J. After hearing, the Regional Trial Court, Branch 58 at Angeles City, rendered a decision, the dispositive portion of which reads: WHEREFORE, premises considered, judgment is hereby rendered in favor of the petitioner and against the respondent: 1. the rightful custody of the minor Christopher J. T. David is hereby given to the natural mother, the herein petitioner Daisie T. David; 2. respondent is hereby ordered to give a temporary support of P3,000.00 a month to the subject minor Christopher J. T. David, Christine David and Cathy Mae David to take effect upon the finality of this decision; and 3. to pay the costs of this suit. SO ORDERED. On appeal, the Court of Appeals reversed, holding: We agree with the respondent-appellant's view that this is not proper in a habeas corpus case. Law and jurisprudence wherein the question of custody of a minor child may be decided in a habeas corpus case contemplate a situation where the parents are married to each other but are separated. This is so because under the Family Code, the father and mother have joint parental authority over their legitimate children and in case of separation of the parents there is need to determine rightful custody of their children. The same does not hold true in an adulterous relationship, as in the case at bar, the child born out of such a relationship is under the parental authority of the mother by express provision of the law. Hence, the question of custody and support should be brought in a case singularly filed for the purpose. In point of fact, this is more advisable in the case at bar because the trial court did not acquire jurisdiction over the other minor children of the petitioner-appellee and respondent-appellant and, therefore, cannot properly provide for their support. Admittedly, respondent-appellant is financially well-off, he being a very rich businessman; whereas, petitioner-appellee depends upon her sisters and parents for support. In fact, he financially supported 879

University of the Cordilleras College of Law First Year C S.Y. 2013 - 2014 petitioner-appellee and her three minor children. It is, therefore, for the best interest of Christopher J that he should temporarily remain under the custody of respondent-appellant until the issue on custody and support shall have been determined in a proper case. WHEREFORE, the decision appealed from is hereby SET ASIDE, and a NEW ONE ENTERED dismissing the petition for habeas corpus in Special Proceeding No. 4489. Daisie in turn filed this petition for review of the appellate court's decision. Rule 102, 1 of the Rules of Court provides that "the writ of habeas corpus shall extend to all cases of illegal confinement or detention by which any person is deprived of his liberty, or by which the rightful custody of any person is withheld from the person entitled thereto." It is indeed true, as the Court of Appeals observed, that the determination of the right to the custody of minor children is relevant in cases where the parents, who are married to each other, are for some reason separated from each other. It does not follow, however, that it cannot arise in any other situation. For example, in the case of Salvaa v. Gaela, it was held that the writ of habeas corpus is the proper remedy to enable parents to regain the custody of a minor daughter even though the latter be in the custody of a third person of her free will because the parents were compelling her to marry a man against her will. In the case at bar, Christopher J. is an illegitimate child since at the time of his conception, his father, private respondent Ramon R. Villar, was married to another woman other than the child's mother. As such, pursuant to Art. 176 of the Family Code, Christopher J. is under the parental authority of his mother, the herein petitioner, who, as a consequence of such authority, is entitled to have custody of him. Since, admittedly, petitioner has been deprived of her rightful custody of her child by private respondent, she is entitled to issuance of the writ of habeas corpus. Indeed, Rule 1021 1 makes no distinction between the case of a mother who is separated from her husband and is entitled to the custody of her child and that of a mother of an illegitimate child who, by law, is vested with sole parental authority, but is deprived of her rightful custody of her child. The fact that private respondent has recognized the minor child may be a ground for ordering him to give support to the latter, but not for giving him custody of the child. Under Art. 213 of the Family Code, "no child under seven years of age shall be separated from the mother unless the court finds compelling reasons to order otherwise." Nor is the fact that private respondent is well-off a reason for depriving petitioner of the custody of her children, especially considering that she has been able to rear and support them on her own since they were born. Petitioner is a market vendor earning from P2,000 to P3,000 per month in 1993 when the RTC decision was rendered. She augments her income by working as secretary at the Computer System Specialist, Inc. earning a monthly income of P4,500.00. She has an arrangement with her employer so that she can personally attend to her children. She works up to 8:00 o'clock in the evening to make up for time lost during the day. That she receives help from her parents and sister for the support of the three children is not a point against her. Cooperation, compassion, love and concern for every member of the family are characteristics of the close family ties that bind the Filipino family and have made it what it is. Daisie and her children may not be enjoying a life of affluence that private respondent promises if the child lives with him. It is enough, however, that petitioner is earning a decent living and is able to support her children according to her means. The Regional Trial Court ordered private respondent to give temporary support to petitioner in the amount of P3,000.00 a month, pending the filing of an action for support, after finding that private respondent did not give any support to his three children by Daisie, except the meager amount of P500.00 a week which he stopped giving them on June 23, 1992. He is a rich man who professes love for his children. In fact he filed a motion for the execution of the decision of the Court of Appeals, alleging that he had observed his son "to be physically weak and pale because of malnutrition and deprivation of the luxury and amenities he was accustomed to when in the former custody of the respondent." He prayed that he be given the custody of the child so that he can provide him with the "proper care and education." Although the question of support is proper in a proceeding for that purpose, the grant of support in this case is justified by the fact that private respondent has expressed willingness to support the minor child. The order for payment of allowance need not be conditioned on the grant to him of custody of the child. 880

University of the Cordilleras College of Law First Year C S.Y. 2013 - 2014 Under Art. 204 of the Family Code, a person obliged to give support can fulfill his obligation either by paying the allowance fixed by the court or by receiving and maintaining in the family dwelling the person who is entitled to support unless, in the latter case, there is "a moral or legal obstacle thereto." In the case at bar, as has already been pointed out, Christopher J., being less than seven years of age at least at the time the case was decided by the RTC, cannot be taken from the mother's custody. Even now that the child is over seven years of age, the mother's custody over him will have to be upheld because the child categorically expressed preference to live with his mother. Under Art. 213 of the Family Code, courts must respect the "choice of the child over seven years of age, unless the parent chosen is unfit" and here it has not been shown that the mother is in any way unfit to have custody of her child. Indeed, if private respondent loves his child, he should not condition the grant of support for him on the award of his custody to him (private respondent). WHEREFORE, the decision of the Court of Appeals is REVERSED and private respondent is ORDERED to deliver the minor Christopher J. T. David to the custody of his mother, the herein petitioner, and to give him temporary support in the amount of P3,000.00, pending the fixing of the amount of support in an appropriate action. SO ORDERED.

881

University of the Cordilleras College of Law First Year C S.Y. 2013 - 2014 Case Digest David vs Court of 250 SCRA 82 G.R. No. 111180 Decided on: November 16, 1995 Ponente: MENDOZA, J.: FACTS: 1. Petitioner Daisie David worked as secretary of private respondent Ramon Villar, a businessman. 2. Private respondent is a married man and the father of four children, all grown-up. 3. The relationship between Daisie and Ramon developed into an intimate one, as a result Christopher J was born to them followed by two more children, both girls. 4. Private respondents wife knew of the relationship when Daisie took Christopher J to Ramons house. 5. After this, the children of Daisie were freely brought by Ramon to his house as they were eventually accepted by his legal family. 6. In summer 1991, Ramon asked Daisie to allow Christopher J, then 6 years old to go with his family to Boracay. 7. Daisie agreed, but after the trip Ramon refused to give back the child. 8. Daisie filed a petitioner for Habeas Corpus on behalf of Christopher J. 9. The RTC rendered judgment in favor of Daisie, granting rightful custody to the natural mother. 10. The CA reversed on appeal holding that Habeas Corpus was not proper; the question of custody of a minor child may be decided in a Habeas Corpus case contemplates a situation where the parents are married to each other but are separated. 11. Hence this petition. ISSUE: Is the remedy of Habeas Corpus proper? HELD: It is indeed true, as the CA observed that the determination of the right to the custody of minor children is relevant in cases where the parents, who are married to each other, are for some reason separated from each other. It does not follow, however, that it cannot arise in any other situation. For example, in the case of SALVANA VS. GAELA (55 PHIL 680), it was held that the writ of habeas corpus is the proper remedy to enable parents to regain the custody of a minor daughter even though the latter be in the custody of a third person of her free will because the parents were compelling her to marry a man against her will. In the case at bar, Christopher J is an illegitimate child since at the time of his conception, his father, private respondent Ramon R. Villar, was married to another woman other than the childs mother. As such pursuant to Article 176 of the family Code, Christopher J is under the parental authority of his mother, the herein petitioner, who, as a consequence of such authority, is entitled to have custody of him. Since, admittedly, petitioner has been deprived of her rightful custody of her child by private respondent, she is entitled to the issuance of the writ of Habeas Corpus. The fact that private respondent has recognized the minor child may be a ground for ordering him to give support to the latter, but not for giving him custody of the child. Under Article 213 of the Family Code, no child under 7 years of age shall be separated from t he mother unless the court finds compelling reasons to order otherwise. That petitioner receives help from her parents and sister for the support of the three children is not a point against her. Cooperation, compassion, love and concern for every member of the family are characteristics of the close family ties that bind the Filipino family and have made it what it is.

882

University of the Cordilleras College of Law First Year C S.Y. 2013 - 2014 Estate of Ruiz vs Court of Appeals 252 SCRA 541 G.R. No. 118671 January 29, 1996 Full Case THE ESTATE OF HILARIO M. RUIZ, EDMOND RUIZ, Executor, petitioner, vs. THE COURT OF APPEALS (Former Special Sixth Division), MARIA PILAR RUIZ-MONTES, MARIA CATHRYN RUIZ, CANDICE ALBERTINE RUIZ, MARIA ANGELINE RUIZ and THE PRESIDING JUDGE OF THE REGIONAL TRIAL COURT OF PASIG, respondents. PUNO, J.: This petition for review on certiorari seeks to annul and set aside the decision dated November 10, 1994 and the resolution dated January 5, 1995 of the Court of Appeals in CA-G.R. SP No. 33045. The facts show that on June 27, 1987, Hilario M. Ruiz1 executed a holographic will naming as his heirs his only son, Edmond Ruiz, his adopted daughter, private respondent Maria Pilar Ruiz Montes, and his three granddaughters, private respondents Maria Cathryn, Candice Albertine and Maria Angeline, all children of Edmond Ruiz. The testator bequeathed to his heirs substantial cash, personal and real properties and named Edmond Ruiz executor of his estate.2 On April 12, 1988, Hilario Ruiz died. Immediately thereafter, the cash component of his estate was distributed among Edmond Ruiz and private respondents in accordance with the decedent's will. For unbeknown reasons, Edmond, the named executor, did not take any action for the probate of his father's holographic will. On June 29, 1992, four years after the testator's death, it was private respondent Maria Pilar Ruiz Montes who filed before the Regional Trial Court, Branch 156, Pasig, a petition for the probate and approval of Hilario Ruiz's will and for the issuance of letters testamentary to Edmond Ruiz,3 Surprisingly, Edmond opposed the petition on the ground that the will was executed under undue influence. On November 2, 1992, one of the properties of the estate the house and lot at No. 2 Oliva Street, Valle Verde IV, Pasig which the testator bequeathed to Maria Cathryn, Candice Albertine and Maria Angeline4 was leased out by Edmond Ruiz to third persons. On January 19, 1993, the probate court ordered Edmond to deposit with the Branch Clerk of Court the rental deposit and payments totalling P540,000.00 representing the one-year lease of the Valle Verde property. In compliance, on January 25, 1993, Edmond turned over the amount of P348,583.56, representing the balance of the rent after deducting P191,416.14 for repair and maintenance expenses on the estate.5 In March 1993, Edmond moved for the release of P50,000.00 to pay the real estate taxes on the real properties of the estate. The probate court approved the release of P7,722.00.6 On May 14, 1993, Edmond withdrew his opposition to the probate of the will. Consequently, the probate court, on May 18, 1993, admitted the will to probate and ordered the issuance of letters testamentary to Edmond conditioned upon the filing of a bond in the amount of P50,000.00. The letters testamentary were issued on June 23, 1993. On July 28, 1993, petitioner Testate Estate of Hilario Ruiz, with Edmond Ruiz as executor, filed an "ExParte Motion for Release of Funds." It prayed for the release of the rent payments deposited with the Branch Clerk of Court. Respondent Montes opposed the motion and concurrently filed a "Motion for Release of Funds to Certain Heirs" and "Motion for Issuance of Certificate of Allowance of Probate Will." Montes prayed for the release of the said rent payments to Maria Cathryn, Candice Albertine and Maria Angeline and for the distribution of the testator's properties, specifically the Valle Verde property and the Blue Ridge apartments, in accordance with the provisions of the holographic will. On August 26, 1993, the probate court denied petitioner's motion for release of funds but granted respondent Montes' motion in view of petitioner's lack of opposition. It thus ordered the release of the rent 883

University of the Cordilleras College of Law First Year C S.Y. 2013 - 2014 payments to the decedent's three granddaughters. It further ordered the delivery of the titles to and possession of the properties bequeathed to the three granddaughters and respondent Montes upon the filing of a bond of P50,000.00. Petitioner moved for reconsideration alleging that he actually filed his opposition to respondent Montes's motion for release of rent payments which opposition the court failed to consider. Petitioner likewise reiterated his previous motion for release of funds. On November 23, 1993, petitioner, through counsel, manifested that he was withdrawing his motion for release of funds in view of the fact that the lease contract over the Valle Verde property had been renewed for another year.7 Despite petitioner's manifestation, the probate court, on December 22, 1993, ordered the release of the funds to Edmond but only "such amount as may be necessary to cover the expenses of administration and allowances for support" of the testator's three granddaughters subject to collation and deductible from their share in the inheritance. The court, however, held in abeyance the release of the titles to respondent Montes and the three granddaughters until the lapse of six months from the date of first publication of the notice to creditors.8 The court stated thus: xxx xxx xxx

After consideration of the arguments set forth thereon by the parties the court resolves to allow Administrator Edmond M. Ruiz to take possession of the rental payments deposited with the Clerk of Court, Pasig Regional Trial Court, but only such amount as may be necessary to cover the expenses of administration and allowances for support of Maria Cathryn Veronique, Candice Albertine and Maria Angeli, which are subject to collation and deductible from the share in the inheritance of said heirs and insofar as they exceed the fruits or rents pertaining to them. As to the release of the titles bequeathed to petitioner Maria Pilar Ruiz-Montes and the abovenamed heirs, the same is hereby reconsidered and held in abeyance until the lapse of six (6) months from the date of first publication of Notice to Creditors. WHEREFORE, Administrator Edmond M. Ruiz is hereby ordered to submit an accounting of the expenses necessary for administration including provisions for the support Of Maria Cathryn Veronique Ruiz, Candice Albertine Ruiz and Maria Angeli Ruiz before the amount required can be withdrawn and cause the publication of the notice to creditors with reasonable dispatch.9 Petitioner assailed this order before the Court of Appeals. Finding no grave abuse of discretion on the part of respondent judge, the appellate court dismissed the petition and sustained the probate court's order in a decision dated November 10, 199410 and a resolution dated January 5, 1995.11 Hence, this petition. Petitioner claims that: THE PUBLIC RESPONDENT COURT OF APPEALS COMMITTED GRAVE ABUSE OF DISCRETION AMOUNTING TO LACK OR EXCESS OF JURISDICTION IN AFFIRMING AND CONFIRMING THE ORDER OF RESPONDENT REGIONAL TRIAL COURT OF PASIG, BRANCH 156, DATED DECEMBER 22, 1993, WHICH WHEN GIVEN DUE COURSE AND IS EFFECTED WOULD: (1) DISALLOW THE EXECUTOR/ADMINISTRATOR OF THE ESTATE OF THE LATE HILARIO M. RUIZ TO TAKE POSSESSION OF ALL THE REAL AND PERSONAL PROPERTIES OF THE ESTATE; (2) GRANT SUPPORT, DURING THE PENDENCY OF THE SETTLEMENT OF AN ESTATE, TO CERTAIN PERSONS NOT ENTITLED THERETO; AND (3) PREMATURELY PARTITION AND DISTRIBUTE THE ESTATE PURSUANT TO THE PROVISIONS OF THE HOLOGRAPHIC WILL EVEN BEFORE ITS INTRINSIC VALIDITY HAS BEEN DETERMINED, AND DESPITE THE EXISTENCE OF UNPAID DEBTS AND OBLIGATIONS OF THE ESTATE.12 The issue for resolution is whether the probate court, after admitting the will to probate but before payment of the estate's debts and obligations, has the authority: (1) to grant an allowance from the funds 884

University of the Cordilleras College of Law First Year C S.Y. 2013 - 2014 of the estate for the support of the testator's grandchildren; (2) to order the release of the titles to certain heirs; and (3) to grant possession of all properties of the estate to the executor of the will. On the matter of allowance, Section 3 of Rule 83 of the Revised Rules of Court provides: Sec. 3. Allowance to widow and family. The widow and minor or incapacitated children of a deceased person, during the settlement of the estate, shall receive therefrom under the direction of the court, such allowance as are provided by law. Petitioner alleges that this provision only gives the widow and the minor or incapacitated children of the deceased the right to receive allowances for support during the settlement of estate proceedings. He contends that the testator's three granddaughters do not qualify for an allowance because they are not incapacitated and are no longer minors but of legal age, married and gainfully employed. In addition, the provision expressly states "children" of the deceased which excludes the latter's grandchildren. It is settled that allowances for support under Section 3 of Rule 83 should not be limited to the "minor or incapacitated" children of the deceased. Article 18813 of the Civil Code of the Philippines, the substantive law in force at the time of the testator's death, provides that during the liquidation of the conjugal partnership, the deceased's legitimate spouse and children, regardless of their age, civil status or gainful employment, are entitled to provisional support from the funds of the estate.14 The law is rooted on the fact that the right and duty to support, especially the right to education, subsist even beyond the age of majority.15 Be that as it may, grandchildren are not entitled to provisional support from the funds of the decedent's estate. The law clearly limits the allowance to "widow and children" and does not extend it to the deceased's grandchildren, regardless of their minority or incapacity.16 It was error, therefore, for the appellate court to sustain the probate court's order granting an allowance to the grandchildren of the testator pending settlement of his estate. Respondent courts also erred when they ordered the release of the titles of the bequeathed properties to private respondents six months after the date of first publication of notice to creditors. An order releasing titles to properties of the estate amounts to an advance distribution of the estate which is allowed only under the following conditions: Sec. 2. Advance distribution in special proceedings. Nothwithstanding a pending controversy or appeal in proceedings to settle the estate of a decedent, the court may, in its discretion and upon such terms as it may deem proper and just, permit that such part of the estate as may not be affected by the controversy or appeal be distributed among the heirs or legatees, upon compliance with the conditions set forth in Rule 90 of these Rules.17 And Rule 90 provides that: Sec. 1. When order for distribution of residue made. When the debts, funeral charges, and expenses of administration the allowance to the widow, and inheritance tax if any, chargeable to the estate in accordance with law, have been paid, the court, on the application of the executor or administrator, or of a person interested in the estate, and after hearing upon notice shall assign the residue of the estate to the persons entitled to the same, naming them and the proportions or parts, to which each is entitled, and such persons may demand and recover their respective shares from the executor or administrator, or any other person having the same in his possession. If there is a controversy before the court as to who are the lawful heirs of the deceased person or as to the distributive shares to which each person is entitled under the law, the controversy shall be heard and decided as in ordinary cases. No distribution shall be allowed until the payment of the obligations above-mentioned has been made or provided for, unless the distributees, or any of them, give a bond, in a sum to be fixed by the court, conditioned for the payment of said obligations within such time as the court directs.18 In settlement of estate proceedings, the distribution of the estate properties can only be made: (1) after all the debts, funeral charges, expenses of administration, allowance to the widow, and estate tax have been paid; or (2) before payment of said obligations only if the distributees or any of them gives a bond in a 885

University of the Cordilleras College of Law First Year C S.Y. 2013 - 2014 sum fixed by the court conditioned upon the payment of said obligations within such time as the court directs, or when provision is made to meet those obligations.19 In the case at bar, the probate court ordered the release of the titles to the Valle Verde property and the Blue Ridge apartments to the private respondents after the lapse of six months from the date of first publication of the notice to creditors. The questioned order speaks of "notice" to creditors, not payment of debts and obligations. Hilario Ruiz allegedly left no debts when he died but the taxes on his estate had not hitherto been paid, much less ascertained. The estate tax is one of those obligations that must be paid before distribution of the estate. If not yet paid, the rule requires that the distributees post a bond or make such provisions as to meet the said tax obligation in proportion to their respective shares in the inheritance.20 Notably, at the time the order was issued the properties of the estate had not yet been inventoried and appraised. It was also too early in the day for the probate court to order the release of the titles six months after admitting the will to probate. The probate of a will is conclusive as to its due execution and extrinsic validity21 and settles only the question of whether the testator, being of sound mind, freely executed it in accordance with the formalities prescribed by law.22 Questions as to the intrinsic validity and efficacy of the provisions of the will, the legality of any devise or legacy may be raised even after the will has been authenticated.23 The intrinsic validity of Hilario's holographic will was controverted by petitioner before the probate court in his Reply to Montes' Opposition to his motion for release of funds24 and his motion for reconsideration of the August 26, 1993 order of the said court.25 Therein, petitioner assailed the distributive shares of the devisees and legatees inasmuch as his father's will included the estate of his mother and allegedly impaired his legitime as an intestate heir of his mother. The Rules provide that if there is a controversy as to who are the lawful heirs of the decedent and their distributive shares in his estate, the probate court shall proceed to hear and decide the same as in ordinary cases.26 Still and all, petitioner cannot correctly claim that the assailed order deprived him of his right to take possession of all the real and personal properties of the estate. The right of an executor or administrator to the possession and management of the real and personal properties of the deceased is not absolute and can only be exercised "so long as it is necessary for the payment of the debts and expenses of administration,"27 Section 3 of Rule 84 of the Revised Rules of Court explicitly provides: Sec. 3. Executor or administrator to retain whole estate to pay debts, and to administer estate not willed. An executor or administrator shall have the right to the possession and management of the real as well as the personal estate of the deceased so long as it is necessary for the payment of the debts and expenses for administration.28 When petitioner moved for further release of the funds deposited with the clerk of court, he had been previously granted by the probate court certain amounts for repair and maintenance expenses on the properties of the estate, and payment of the real estate taxes thereon. But petitioner moved again for the release of additional funds for the same reasons he previously cited. It was correct for the probate court to require him to submit an accounting of the necessary expenses for administration before releasing any further money in his favor. It was relevantly noted by the probate court that petitioner had deposited with it only a portion of the oneyear rental income from the Valle Verde property. Petitioner did not deposit its succeeding rents after renewal of the lease.29 Neither did he render an accounting of such funds. Petitioner must be reminded that his right of ownership over the properties of his father is merely inchoate as long as the estate has not been fully settled and partitioned.30 As executor, he is a mere trustee of his father's estate. The funds of the estate in his hands are trust funds and he is held to the duties and responsibilities of a trustee of the highest order.31 He cannot unilaterally assign to himself and possess all his parents' properties and the fruits thereof without first submitting an inventory and appraisal of all real and personal properties of the deceased, rendering a true account of his administration, the expenses of administration, the amount of the obligations and estate tax, all of which are subject to a determination by the court as to their veracity, propriety and justness.32 IN VIEW WHEREOF, the decision and resolution of the Court of Appeals in CA-G.R. SP No. 33045 affirming the order dated December 22, 1993 of the Regional Trial Court, Branch 156, Pasig in SP Proc. 886

University of the Cordilleras College of Law First Year C S.Y. 2013 - 2014 No. 10259 are affirmed with the modification that those portions of the order granting an allowance to the testator's grandchildren and ordering the release of the titles to the private respondents upon notice to creditors are annulled and set aside. Respondent judge is ordered to proceed with dispatch in the proceedings below. SO ORDERED.

887

University of the Cordilleras College of Law First Year C S.Y. 2013 - 2014

XV. Parental Authority

888

University of the Cordilleras College of Law First Year C S.Y. 2013 - 2014 Gamboa Hirsch vs Court of Appeals 527 SCRA 380 G.R. No. 174485 July 11, 2007 Full Case AGNES GAMBOA-HIRSCH Petitioner, vs. HON. COURT OF APPEALS and FRANKLIN HARVEY HIRSCH, Respondents. VELASCO, JR., J.: This is a petition for certiorari under Rule 65 which seeks to set aside the June 8, 2006 Decision of the Court of Appeals (CA) in CA-G.R. SP No. 94329, which granted private respondent Franklin Harvey Hirsch (Franklin) joint custody with petitioner Agnes Gamboa-Hirsch (Agnes) of their minor daughter Simone Noelle Hirsch (Simone); and the August 3, 2006 CA Resolution denying petitioners Motion for Reconsideration for lack of merit. Petitioner also prays for the issuance of a temporary restraining order/injunction preventing the execution and implementation of the assailed June 8, 2006 CA Decision. Franklin and Agnes were married on December 23, 2000 in the City of Bacolod, and established their conjugal dwelling in Diniwid, Boracay Island, Malay, Aklan. On December 21, 2002, a child was born to them and was named Simone. In 2005, the couple started to have marital problems as Agnes wanted to stay in Makati City, while Franklin insisted that they stay in Boracay Island. On March 23, 2006, Agnes came to their conjugal home in Boracay, and asked for money and for Franklins permission for her to bring their daughter to Makati City for a brief vacation. Franklin readily agreed, but soon thereafter discovered that neither Agnes nor their daughter Simone would be coming back to Boracay. Franklin then filed a petition for habeas corpus before the CA for Agnes to produce Simone in court. On May 19, 2006, the CA issued a Resolution which ordered that a writ of habeas corpus be issued ordering that Simone be brought before said court on May 26, 2006. After a series of hearings and presentation of evidence, the CA, on June 8, 2006, promulgated the assailed Decision granting Franklin joint custody with Agnes of their minor child. Agnes filed a Motion for Reconsideration of this Decision, which was denied in the CAs August 3, 2006 Resolution for lack of merit. Petitioner now comes before this Court praying that we set aside the June 8, 2006 Decision and August 3, 2006 Resolution of the CA, and that we issue a temporary restraining order/injunction on the execution and implementation of the assailed rulings of the CA based on the following grounds: (A) The Court of Appeals seriously erred and acted with grave abuse of discretion amounting to lack or excess of jurisdiction when it ruled upon, granted, and decided the matter of custody x x x during the May 26, 2006 hearing conducted on the petition for writ of habeas corpus in relation to and with custody of a minor under A.M. No. 03-03-04-SC, C.A.-GR SP. No. 94329, as no reception of evidence to support said decision was had thereon, and the honorable court merely based its decision on mere conjectures and presumptions. (B) The Court of Appeals seriously erred and acted with grave abuse of discretion amounting to lack or excess of jurisdiction when it denied the motion for reconsideration filed by [petitioner Agnes] and only made addendums thereon appertaining to the custody aspect in its Decision that the same is deemed necessary for the protection of the interest of the child and a mere temporary arrangement while the case involving the herein parties are pending before the Regional Trial Court x x x quite contrary to its

889

University of the Cordilleras College of Law First Year C S.Y. 2013 - 2014 pronouncements during the May 26, 2006 hearing when the matter of custody was insisted upon by [respondent Franklin]. (C) The Court of Appeals seriously erred and acted with grave abuse of discretion amounting to lack or excess of jurisdiction when it granted joint custody in utter disregard of the provisions of the Family Code, as to minors seven (7) years of age and below, in relation to the jurisprudence and pronouncements laid down by the Honorable Supreme Court on the matter of the said provision. Acting on the petition, this Court issued its October 2, 2006 Resolution denying petitioners prayer for the issuance of a temporary restraining order. Petitioner then filed a Motion for Reconsideration of this Resolution, and on April 11, 2007, this Court granted petitioners Motion for Reconsideration, issued a temporary restraining order, and awarded the sole custody of the minor, Simone, to petitioner. This petition has merit. The CA committed grave abuse of discretion when it granted joint custody of the minor child to both parents. The Convention on the Rights of the Child provides that "in all actions concerning children, whether undertaken by public or private social welfare institutions, courts of law, administrative authorities or legislative bodies, the best interests of the child shall be a primary consideration (emphasis supplied)." The Child and Youth Welfare Code, in the same way, unequivocally provides that in all questions regarding the care and custody, among others, of the child, his/her welfare shall be the paramount consideration. The so-called "tender-age presumption" under Article 213 of the Family Code may be overcome only by compelling evidence of the mothers unfitness. The mother is declared unsuitable to have custody of her children in one or more of the following instances: neglect, abandonment, unemployment, immorality, habitual drunkenness, drug addiction, maltreatment of the child, insanity, or affliction with a communicable disease. Here, the mother was not shown to be unsuitable or grossly incapable of caring for her minor child. All told, no compelling reason has been adduced to wrench the child from the mothers custody.1avvphi1 WHEREFORE, premises considered, the petition is GIVEN DUE COURSE. The June 8, 2006 Decision and August 3, 2006 Resolution of the CA are hereby SET ASIDE. Sole custody over Simone Noelle Hirsch is hereby AWARDED to the mother, petitioner Agnes Gamboa-Hirsch. SO ORDERED.

890

University of the Cordilleras College of Law First Year C S.Y. 2013 - 2014 Espiritu v. Court of Appeals, 242 SCRA 362 G.R. No. 115640 March 15, 1995 REYNALDO ESPIRITU and GUILLERMA LAYUG, petitioners, vs. COURT OF APPEALS and TERESITA MASAUDING, respondents. MELO, J.: This case concerns a seemingly void marriage and a relationship which went sour. The innocent victims are two children horn out of the same union. Upon this Court now falls the not too welcome task of deciding the issue of who, between the father and mother, is more suitable and better qualified in helping the children to grow into responsible, well-adjusted, and happy young adulthood. Petitioner Reynaldo Espiritu and respondent Teresita Masauding first met sometime in 1976 in Iligan City where Reynaldo was employed by the National Steel Corporation and Teresita was employed as a nurse in a local hospital. In 1977, Teresita left for Los Angeles, California to work as a nurse. She was able to acquire immigrant status sometime later. In 1984, Reynaldo was sent by his employer, the National Steel Corporation, to Pittsburgh, Pennsylvania as its liaison officer and Reynaldo and Teresita then began to maintain a common law relationship of husband and wife. On August 16, 1986, their daughter, Rosalind Therese, was born. On October 7, 1987, while they were on a brief vacation in the Philippines, Reynaldo and Teresita got married, and upon their return to the United States, their second child, a son, this time, and given the name Reginald Vince, was born on January 12, 1988. The relationship of the couple deteriorated until they decided to separate sometime in 1990. Teresita blamed Reynaldo for the break-up, stating he was always nagging her about money matters. Reynaldo, on the other hand, contended that Teresita was a spendthrift, buying expensive jewellery and antique furniture instead of attending to household expenses. Instead of giving their marriage a second chance as allegedly pleaded by Reynaldo, Teresita left Reynaldo and the children and went back to California. She claims, however, that she spent a lot of money on long distance telephone calls to keep in constant touch with her children. Reynaldo brought his children home to the Philippines, but because his assignment in Pittsburgh was not yet completed, he was sent back by his company to Pittsburgh. He had to leave his children with his sister, co-petitioner Guillerma Layug and her family. Teresita claims that she did not immediately follow her children because Reynaldo filed a criminal case for bigamy against her and she was afraid of being arrested. The judgment of conviction in the bigamy case was actually rendered only on September 29, 1994. (Per Judge Harriet O. Demetriou, Branch 70, RTC, Pasig, pp. 210-222, Rollo). Teresita, meanwhile, decided to return to the Philippines and on December 8, 1992 and filed the petition for a writ of habeas corpus against herein two petitioners to gain custody over the children, thus starting the whole proceedings now reaching this Court. On June 30, 1993, the trial court dismissed the petition for habeas corpus. It suspended Teresita's parental authority over Rosalind and Reginald and declared Reynaldo to have sole parental authority over them but with rights of visitation to be agreed upon by the parties and to be approved by the Court. On February 16, 1994, the Court of Appeals per Justice Isnani, with Justices de Pano and Ibay-Somera concurring, reversed the trial court's decision. It gave custody to Teresita and visitation rights on weekends to Reynaldo. Petitioners now come to this Court on a petition for review, in the main contending that the Court of Appeals disregarded the factual findings of the trial court; that the Court of Appeals further engaged in

891

University of the Cordilleras College of Law First Year C S.Y. 2013 - 2014 speculations and conjectures, resulting in its erroneous conclusion that custody of the children should be given to respondent Teresita. We believe that respondent court resolved the question of custody over the children through an automatic and blind application of the age proviso of Article 363 of the Civil Code which reads: Art. 363. In all questions on the care, custody, education and property of the children, the latter's welfare shall be paramount. No mother shall be separated from her child under seven years of age, unless the court finds compelling reasons for such measure. and of Article 213 of the Family Code which in turn provides: Art. 213. In case of separation of the parents parental authority shall be exercised by the parent designated by the Court. The Court shall take into account all relevant considerations, especially the choice of the child over seven years of age unless the parent chosen is unfit. The decision under review is based on the report of the Code Commission which drafted Article 213 that a child below seven years still needs the loving, tender care that only a mother can give and which, presumably, a father cannot give in equal measure. The commentaries of a member of the Code Commission, former Court of Appeals Justice Alicia Sempio-Diy, in a textbook on the Family Code, were also taken into account. Justice Diy believes that a child below seven years should still be awarded to her mother even if the latter is a prostitute or is unfaithful to her husband. This is on the theory that moral dereliction has no effect on a baby unable to understand such action. (Handbook on the Family Code of the Philippines, 1988 Ed., p. 297.) The Court of Appeals was unduly swayed by an abstract presumption of law rather than an appreciation of relevant facts and the law which should apply to those facts. The task of choosing the parent to whom custody shall be awarded is not a ministerial function to be determined by a simple determination of the age of a minor child. Whether a child is under or over seven years of age, the paramount criterion must always be the child's interests. Discretion is given to the court to decide who can best assure the welfare of the child, and award the custody on the basis of that consideration. In Unson III vs. Navarro (101 SCRA 183 [1980]), we laid down the rule that "in all controversies regarding the custody of minors, the sole and foremost consideration is the physical, education, social and moral welfare of the child concerned, taking into account the respective resources and social and moral situations of the contending parents", and in Medina vs. Makabali (27 SCRA 502 [1969]), where custody of the minor was given to a non-relative as against the mother, then the country's leading civilist, Justice J.B.L. Reyes, explained its basis in this manner: . . . While our law recognizes the right of a parent to the custody of her child, Courts must not lose sight of the basic principle that "in all questions on the care, custody, education and property of children, the latter's welfare shall be paramount" (Civil Code of the Philippines. Art. 363), and that for compelling reasons, even a child under seven may be ordered separated from the mother (do). This is as it should be, for in the continual evolution of legal institutions, the patria potestas has been transformed from the jus vitae ac necis (right of life and death) of the Roman law, under which the offspring was virtually a chattel of his parents into a radically different institution, due to the influence of Christian faith and doctrines. The obligational aspect is now supreme. As pointed out by Puig Pena, now "there is no power, but a task; no complex of rights (of parents) but a sum of duties; no sovereignty, but a sacred trust for the welfare of the minor." As a result, the right of parents to the company and custody of their children is but ancillary to the proper discharge of parental duties to provide the children with adequate support, education, moral, intellectual and civic training and development (Civil Code, Art. 356). (pp. 504-505.) 892

University of the Cordilleras College of Law First Year C S.Y. 2013 - 2014 In ascertaining the welfare and best interests of the child, courts are mandated by the Family Code to take into account all relevant considerations. If a child is under seven years of age, the law presumes that the mother is the best custodian. The presumption is strong but it is not conclusive. It can be overcome by "compelling reasons". If a child is over seven, his choice is paramount but, again, the court is not bound by that choice. In its discretion, the court may find the chosen parent unfit and award custody to the other parent, or even to a third party as it deems fit under the circumstances. In the present case, both Rosalind and Reginald are now over seven years of age. Rosalind celebrated her seventh birthday on August 16, 1993 while Reginald reached the same age on January 12, 1995. Both are studying in reputable schools and appear to be fairly intelligent children, quite capable of thoughtfully determining the parent with whom they would want to live. Once the choice has been made, the burden returns to the court to investigate if the parent thus chosen is unfit to assume parental authority and custodial responsibility. Herein lies the error of the Court of Appeals. Instead of scrutinizing the records to discover the choice of the children and rather than verifying whether that parent is fit or unfit, respondent court simply followed statutory presumptions and general propositions applicable to ordinary or common situations. The sevenyear age limit was mechanically treated as an arbitrary cut off period and not a guide based on a strong presumption. A scrutiny of the pleadings in this case indicates that Teresita, or at least, her counsel are more intent on emphasizing the "torture and agony" of a mother separated from her children and the humiliation she suffered as a result of her character being made a key issue in court rather than the feelings and future, the best interests and welfare of her children. While the bonds between a mother and her small child are special in nature, either parent, whether father or mother, is bound to suffer agony and pain if deprived of custody. One cannot say that his or her suffering is greater than that of the other parent. It is not so much the suffering, pride, and other feelings of either parent but the welfare of the child which is the paramount consideration. We are inclined to sustain the findings and conclusions of the regional trial court because it gave greater attention to the choice of Rosalind and considered in detail all the relevant factors bearing on the issue of custody. When she was a little over 5 years old, Rosalind was referred to a child psychologist, Rita Flores Macabulos, to determine the effects of uprooting her from the Assumption College where she was studying. Four different tests were administered. The results of the tests are quite revealing. The responses of Rosalind about her mother were very negative causing the psychologist to delve deeper into the child's anxiety. Among the things revealed by Rosalind was an incident where she saw her mother hugging and kissing a "bad" man who lived in their house and worked for her father. Rosalind refused to talk to her mother even on the telephone. She tended to be emotionally emblazed because of constant fears that she may have to leave school and her aunt's family to go back to the United States to live with her mother. The 5-1/2 page report deals at length with feelings of insecurity and anxiety arising from strong conflict with the mother. The child tried to compensate by having fantasy activities. All of the 8 recommendations of the child psychologist show that Rosalind chooses petitioners over the private respondent and that her welfare will be best served by staying with them (pp. 199-205, Rollo). At about the same time, a social welfare case study was conducted for the purpose of securing the travel clearance required before minors may go abroad. Social Welfare Officer Emma D. Estrada Lopez, stated that the child Rosalind refused to go back to the United States and be reunited with her mother. She felt unloved and uncared for. Rosalind was more attached to her Yaya who did everything for her and Reginald. The child was found suffering from emotional shock caused by her mother's infidelity. The application for travel clearance was recommended for denial (pp. 206-209, Rollo).

893

University of the Cordilleras College of Law First Year C S.Y. 2013 - 2014 Respondent Teresita, for her part, argues that the 7-year age reference in the law applies to the date when the petition for a writ of habeas corpus is filed, not to the date when a decision is rendered. This argument is flawed. Considerations involving the choice made by a child must be ascertained at the time that either parent is given custody over the child. The matter of custody is not permanent and unalterable. If the parent who was given custody suffers a future character change and becomes unfit, the matter of custody can always be re-examined and adjusted (Unson III v. Navarro, supra, at p. 189). To be sure, the welfare, the best interests, the benefit, and the good of the child must be determined as of the time that either parent is chosen to be the custodian. At the present time, both children are over 7 years of age and are thus perfectly capable of making a fairly intelligent choice. According to respondent Teresita, she and her children had tearful reunion in the trial court, with the children crying, grabbing, and embracing her to prevent the father from taking them away from her. We are more inclined to believe the father's contention that the children ignored Teresita in court because such an emotional display as described by Teresita in her pleadings could not have been missed by the trial court. Unlike the Justices of the Court of Appeals Fourth Division, Judge Lucas P. Bersamin personally observed the children and their mother in the courtroom. What the Judge found is diametrically opposed to the contentions of respondent Teresita. The Judge had this to say on the matter. And, lastly, the Court cannot look at petitioner [Teresita] in similar light, or with more understanding, especially as her conduct and demeanor in the courtroom (during most of the proceedings) or elsewhere (but in the presence of the undersigned presiding judge) demonstrated her ebulent temper that tended to corroborate the alleged violence of her physical punishment of the children (even if only for ordinary disciplinary purposes) and emotional instability, typified by her failure (or refusal?) to show deference and respect to the Court and the other parties (pp. 12-13, RTC Decision) Respondent Teresita also questions the competence and impartiality of the expert witnesses. Respondent court, in turn, states that the trial court should have considered the fact that Reynaldo and his sister, herein petitioner Guillerma Layug, hired the two expert witnesses. Actually, this was taken into account by the trial court which stated that the allegations of bias and unfairness made by Teresita against the psychologist and social worker were not substantiated. The trial court stated that the professional integrity and competence of the expert witnesses and the objectivity of the interviews were unshaken and unimpeached. We might add that their testimony remain uncontroverted. We also note that the examinations made by the experts were conducted in late 1991, well over a year before the filing by Teresita of the habeas corpus petition in December, 1992. Thus, the examinations were at that time not intended to support petitioners' position in litigation, because there was then not even an impending possibility of one. That they were subsequently utilized in the case a quo when it did materialize does not change the tenor in which they were first obtained. Furthermore, such examinations, when presented to the court must be construed to have been presented not to sway the court in favor of any of the parties, but to assist the court in the determination of the issue before it. The persons who effected such examinations were presented in the capacity of expert witnesses testifying on matters within their respective knowledge and expertise. On this matter, this Court had occasion to rule in the case of Sali vs. Abukakar, et al. (17 SCRA 988 [1966]). The fact that, in a particular litigation, an NBI expert examines certain contested documents, at the request, not of a public officer or agency of the Government, but of a private litigant, does not necessarily nullify the examination thus made. Its purpose, presumably, to assist the court having jurisdiction over said litigation, in the performance of its duty to settle correctly the issues relative to said documents. Even a non-expert private individual may examine the same, if there are facts within his knowledge which may help, the court in the determination of said issue. Such examination, which may properly be undertaken by a non-expert private individual, does not, certainly become null and void when the examiner is an expert and/or an officer of the NBI.

894

University of the Cordilleras College of Law First Year C S.Y. 2013 - 2014 (pp. 991-992.) In regard to testimony of expert witnesses it was held in Salomon, et al. vs. Intermediate Appellate Court, et al. (185 SCRA 352 [1990]): . . . Although courts are not ordinarily bound by expert testimonies, they may place whatever weight they choose upon such testimonies in accordance with the facts of the case. The relative weight and sufficiency of expert testimony is peculiarly within the province of the trial court to decide, considering the ability and character of the witness, his actions upon the witness stand, the weight and process of the reasoning by which he has supported his opinion, his possible bias in favor of the side for whom he testifies, the fact that he is a paid witness, the relative opportunities for study and observation of the matters about which he testifies, and any other matters which reserve to illuminate his statements. The opinion of the expert may not be arbitrarily rejected; it is to be considered by the court in view of all the facts and circumstances in the case and when common knowledge utterly fails, the expert opinion may be given controlling effect (20 Am. Jur., 1056-1058). The problem of the credibility of the expert witness and the evaluation of his testimony is left to the discretion of the trial court whose ruling thereupon is not reviewable in the absence of an abuse of that discretion. (p. 359) It was in the exercise of this discretion, coupled with the opportunity to assess the witnesses' character and to observe their respective demeanor that the trial court opted to rely on their testimony, and we believe that the trial court was correct in its action. Under direct examination an February 4, 1993, Social Worker Lopez stated that Rosalind and her aunt were about to board a plane when they were off-loaded because there was no required clearance. They were referred to her office, at which time Reginald was also brought along and interviewed. One of the regular duties of Social Worker Lopez in her job appears to be the interview of minors who leave for abroad with their parents or other persons. The interview was for purposes of foreign travel by a 5-year old child and had nothing to do with any pending litigation. On cross-examination, Social Worker Lopez stated that her assessment of the minor's hatred for her mother was based on the disclosures of the minor. It is inconceivable, much less presumable that Ms. Lopez would compromise her position, ethics, and the public trust reposed on a person of her position in the course of doing her job by falsely testifying just to support the position of any litigant. The psychologist, Ms. Macabulos, is a B.S. magna cum laude graduate in Psychology and an M.A. degree holder also in Psychology with her thesis graded "Excellent". She was a candidate for a doctoral degree at the time of the interview. Petitioner Reynaldo may have shouldered the cost of the interview but Ms. Macabulos services were secured because Assumption College wanted an examination of the child for school purposes and not because of any litigation. She may have been paid to examine the child and to render a finding based on her examination, but she was not paid to fabricate such findings in favor of the party who retained her services. In this instance it was not even petitioner Reynaldo but the school authorities who initiated the same. It cannot be presumed that a professional of her potential and stature would compromise her professional standing. Teresita questions the findings of the trial court that: 1. Her morality is questionable as shown by her marrying Reynaldo at the time she had a subsisting marriage with another man. She is guilty of grave indiscretion in carrying on a love affair with one of the Reynaldo's fellow NSC employees.

2.

895

3.

University of the Cordilleras College of Law First Year C S.Y. 2013 - 2014 She is incapable of providing the children with necessities and conveniences commensurate to their social standing because she does not even own any home in the Philippines. She is emotionally unstable with ebullient temper.

4.

It is contended that the above findings do not constitute the compelling reasons under the law which would justify depriving her of custody over the children; worse, she claims, these findings are nonexistent and have not been proved by clear and convincing evidence. Public and private respondents give undue weight to the matter of a child under 7 years of age not to be separated from the mother, without considering what the law itself denominates as compelling reasons or relevant considerations to otherwise decree. In the Unson III case, earlier mentioned, this Court stated that it found no difficulty in not awarding custody to the mother, it being in the best interest of the child "to be freed from the obviously unwholesome, not to say immoral influence, that the situation where [the mother] had placed herself . . . might create in the moral and social outlook of [the child] who was in her formative and most impressionable stage . . ." Then too, it must be noted that both Rosalind and Reginald are now over 7 years of age. They understand the difference between right and wrong, ethical behavior and deviant immorality. Their best interests would be better served in an environment characterized by emotional stability and a certain degree of material sufficiency. There is nothing in the records to show that Reynaldo is an "unfit" person under Article 213 of the Family Code. In fact, he has been trying his best to give the children the kind of attention and care which the mother is not in a position to extend. The argument that the charges against the mother are false is not supported by the records. The findings of the trial court are based on evidence. Teresita does not deny that she was legally married to Roberto Lustado on December 17, 1984 in California (p. 13, Respondent's Memorandum; p. 238, Rollo; pp. 11, RTC Decision). Less than a year later, she had already driven across the continental United States to commence living with another man, petitioner Reynaldo, in Pittsburgh. The two were married on October 7, 1987. Of course, to dilute this disadvantage on her part, this matter of her having contracted a bigamous marriage later with Reynaldo, Teresita tried to picture Reynaldo as a rapist, alleging further that she told Reynaldo about her marriage to Lustado on the occasion when she was raped by Reynaldo. Expectedly, Judge Harriet Demetriou of the Pasig RTC lent no weight to such tale. And even if this story were given credence, it adds to and not subtracts from the conviction of this Court about Teresita's values. Rape is an insidious crime against privacy. Confiding to one's potential rapist about a prior marriage is not a very convincing indication that the potential victim is averse to the act. The implication created is that the act would be acceptable if not for the prior marriage. More likely is Reynaldo's story that he learned of the prior marriage only much later. In fact, the rape incident itself is unlikely against a woman who had driven three days and three nights from California, who went straight to the house of Reynaldo in Pittsburgh and upon arriving went to bed and, who immediately thereafter started to live with him in a relationship which is marital in nature if not in fact. Judge Bersamin of the court a quo believed the testimony of the various witnesses that while married to Reynaldo, Teresita entered into an illicit relationship with Perdencio Gonzales right there in the house of petitioner Reynaldo and respondent Teresita. Perdencio had been assigned by the National Steel Corporation to assist in the project in Pittsburgh and was staying with Reynaldo, his co-employee, in the latter's house. The record shows that the daughter Rosalind suffered emotional disturbance caused by the traumatic effect of seeing her mother hugging and kissing a boarder in their house. The record also shows that it was Teresita who left the conjugal home and the children, bound for California. When Perdencio Gonzales was reassigned to the Philippines, Teresita followed him and was seen in his company in a Cebu 896

University of the Cordilleras College of Law First Year C S.Y. 2013 - 2014 hotel, staying in one room and taking breakfast together. More significant is that letters and written messages from Teresita to Perdencio were submitted in evidence (p.12, RTC Decision). The argument that moral laxity or the habit of flirting from one man to another does not fall under "compelling reasons" is neither meritorious nor applicable in this case. Not only are the children over seven years old and their clear choice is the father, but the illicit or immoral activities of the mother had already caused emotional disturbances, personality conflicts, and exposure to conflicting moral values, at least in Rosalind. This is not to mention her conviction for the crime of bigamy, which from the records appears to have become final (pp. 210-222, Rollo). Respondent court's finding that the father could not very well perform the role of a sole parent and substitute mother because his job is in the United States while the children will be left behind with their aunt in the Philippines is misplaced. The assignment of Reynaldo in Pittsburgh is or was a temporary one. He was sent there to oversee the purchase of a steel mill component and various equipment needed by the National Steel Corporation in the Philippines. Once the purchases are completed, there is nothing to keep him there anymore. In fact, in a letter dated January 30, 1995, Reynaldo informs this Court of the completion of his assignment abroad and of his permanent return to the Philippines (ff. p. 263, Rollo). The law is more than satisfied by the judgment of the trial court. The children are now both over seven years old. Their choice of the parent with whom they prefer to stay is clear from the record. From all indications, Reynaldo is a fit person, thus meeting the two requirements found in the first paragraph of Article 213 of the Family Code. The presumption under the second paragraph of said article no longer applies as the children are over seven years. Assuming that the presumption should have persuasive value for children only one or two years beyond the age of seven years mentioned in the statute, there are compelling reasons and relevant considerations not to grant custody to the mother. The children understand the unfortunate shortcomings of their mother and have been affected in their emotional growth by her behavior. WHEREFORE, the petition is hereby GRANTED. The decision of the Court of Appeals is reversed and set aside, and the decision of Branch 96 of the Regional Trial Court of the National Capital Judicial Region stationed in Quezon City and presided over by the Honorable Lucas P. Bersamin in its Civil Case No. Q-92-14206 awarding custody of the minors Rosalind and Reginald Espiritu to their father, Reynaldo Espiritu, is reinstated. No special pronouncement is made as to costs. SO ORDERED. Feliciano, Romero, Vitug and Francisco, JJ., concur.

897

University of the Cordilleras College of Law First Year C S.Y. 2013 - 2014 Santos, Sr. Vs. CA Full Case LEOUEL SANTOS, SR., petitioner-appellant, vs. COURT OF APPEALS, and SPOUSES LEOPOLDO and OFELIA BEDIA, respondents-appellees. ROMERO, J.: In this petition for review, we are asked to overturn the decision of the Court of Appeals granting custody of six-year old Leouel Santos, Jr. to his maternal grandparents and not to his father, Santos, Sr. What is sought is a decision which should definitively settle the matter of the care, custody and control of the boy. Happily, unlike King Solomon, we need not merely rely on a "wise and understanding heart," for there is man's law to guide us and that is, the Family Code. The antecedent facts giving rise to the case at bench are as follows: Petitioner Leouel Santos, Sr., an army lieutenant, and Julia Bedia a nurse by profession, were married in Iloilo City in 1986. Their union beget only one child, Leouel Santos, Jr. who was born July 18, 1987. From the time the boy was released from the hospital until sometime thereafter, he had been in the care and custody of his maternal grandparents, private respondents herein, Leopoldo and Ofelia Bedia. Petitioner and wife Julia agreed to place Leouel Jr. in the temporary custody of the latter's parents, the respondent spouses Bedia. The latter alleged that they paid for all the hospital bills, as well as the subsequent support of the boy because petitioner could not afford to do so. The boy's mother, Julia Bedia-Santos, left for the United States in May 1988 to work. Petitioner alleged that he is not aware of her whereabouts and his efforts to locate her in the United States proved futile. Private respondents claim that although abroad, their daughter Julia had been sending financial support to them for her son. On September 2, 1990, petitioner along with his two brothers, visited the Bedia household, where threeyear old Leouel Jr. was staying. Private respondents contend that through deceit and false pretensions, petitioner abducted the boy and clandestinely spirited him away to his hometown in Bacong, Negros Oriental. The spouses Bedia then filed a "Petition for Care, Custody and Control of Minor Ward Leouel Santos Jr.," before the Regional Trial Court of Iloilo City, with Santos, Sr. as respondent. After an ex-parte hearing on October 8, 1990, the trial court issued an order on the same day awarding custody of the child Leouel Santos, Jr. to his grandparents, Leopoldo and Ofelia Bedia. Petitioner appealed this Order to the Court of Appeals. In its decision dated April 30, 1992, respondent appellate court affirmed the trial court's order. His motion for reconsideration having been denied, petitioner now brings the instant petition for review for a reversal of the appellate court's decision. The Court of Appeals erred, according to petitioner, in awarding custody of the boy to his grandparents and not to himself. He contends that since private respondents have failed to show that petitioner is an unfit and unsuitable father, substitute parental authority granted to the boy's grandparents under Art. 214 of the Family Code is inappropriate. Petitioner adds that the reasons relied upon by the private respondents in having custody over the boy, are flimsy and insufficient to deprive him of his natural and legal right to have custody. 898

University of the Cordilleras College of Law First Year C S.Y. 2013 - 2014 On the other hand, private respondents aver that they can provide an air-conditioned room for the boy and that petitioner would not be in a position to take care of his son since he has to be assigned to different places. They also allege that the petitioner did not give a single centavo for the boy's support and maintenance. When the boy was about to be released from the hospital, they were the ones who paid the fees because their daughter and petitioner had no money. Besides, Julia Bedia Santos, their daughter, had entrusted the boy to them before she left for the United States. Furthermore, petitioner's use of trickery and deceit in abducting the child in 1990, after being hospitably treated by private respondents, does not speak well of his fitness and suitability as a parent. The Bedias argue that although the law recognizes the right of a parent to his child's custody, ultimately the primary consideration is what is best for the happiness and welfare of the latter. As maternal grandparents who have amply demonstrated their love and affection for the boy since his infancy, they claim to be in the best position to promote the child's welfare. The issue to be resolved here boils down to who should properly be awarded custody of the minor Leouel Santos, Jr. The right of custody accorded to parents springs from the exercise of parental authority. Parental authority orpatria potestas in Roman Law is the juridical institution whereby parents rightfully assume control and protection of their unemancipated children to the extent required by the latter' s needs. It is a mass of rights and obligations which the law grants to parents for the purpose of the children's physical preservation and development, as well as the cultivation of their intellect and the education of their heart and senses. As regards parental authority, "there is no power, but a task; no complex of rights, but a sum of duties; no sovereignty but a sacred trust for the welfare of the minor." Parental authority and responsibility are inalienable and may not be transferred or renounced except in cases authorized by law. The right attached to parental authority, being purely personal, the law allows a waiver of parental authority only in cases of adoption, guardianship and surrender to a children's home or an orphan institution. When a parent entrusts the custody of a minor to another, such as a friend or godfather, even in a document, what is given is merely temporary custody and it does not constitute a renunciation of parental authority. Even if a definite renunciation is manifest, the law still disallows the same. The father and mother, being the natural guardians of unemancipated children, are duty-bound and entitled to keep them in their custody and company. The child's welfare is always the paramount consideration in all questions concerning his care and custody. The law vests on the father and mother joint parental authority over the persons of their common children. In case of absence or death of either parent, the parent present shall continue exercising parental authority. Only in case of the parents' death, absence or unsuitability may substitute parental authority be exercised by the surviving grandparent. The situation obtaining in the case at bench is one where the mother of the minor Santos, Jr., is working in the United States while the father, petitioner Santos, Sr., is present. Not only are they physically apart but are also emotionally separated. There has been no decree of legal separation and petitioner's attempt to obtain an annulment of the marriage on the ground of psychological incapacity of his wife has failed. Petitioner assails the decisions of both the trial court and the appellate court to award custody of his minor son to his parents-in-law, the Bedia spouses on the ground that under Art. 214 of the Family Code, substitute parental authority of the grandparents is proper only when both parents are dead, absent or unsuitable. Petitioner's unfitness, according to him, has not been successfully shown by private respondents.

899

University of the Cordilleras College of Law First Year C S.Y. 2013 - 2014 The Court of Appeals held that although there is no evidence to show that petitioner (Santos Sr.) is "depraved, a habitual drunkard or poor, he may nevertheless be considered, as he is in fact so considered, to be unsuitable to be allowed to have custody of minor Leouel Santos Jr." The respondent appellate court, in affirming the trial court's order of October 8, 1990, adopted as its own the latter's observations, to wit: From the evidence adduced, this Court is of the opinion that it is to be (sic) best interest of the minor Leouel Santos, Jr. that he be placed under the care, custody, and control of his maternal grandparents the petitioners herein. The petitioners have amply demonstrated their love and devotion to their grandson while the natural father, respondent herein, has shown little interest in his welfare as reflected by his conduct in the past. Moreover the fact that petitioners are well-off financially, should be carefully considered in awarding to them the custody of the minor herein, lest the breaking of such ties with his maternal grandparents might deprive the boy of an eventual college education and other material advantages (Consaul vs. Consaul, 63 N.Y.S. 688). Respondent had never given any previous financial support to his son, while, upon the other hand, the latter receives so much bounty from his maternal grandparents and his mother as well, who is now gainfully employed in the United States. Moreover, the fact that respondent, as a military personnel who has to shuttle from one assignment to another, and, in these troubled times, may have pressing and compelling military duties which may prevent him from attending to his son at times when the latter needs him most, militates strongly against said respondent. Additionally, the child is sickly and asthmatic and needs the loving and tender care of those who can provide for it. We find the aforementioned considerations insufficient to defeat petitioner's parental authority and the concomitant right to have custody over the minor Leouel Santos, Jr., particularly since he has not been shown to be an unsuitable and unfit parent. Private respondents' demonstrated love and affection for the boy, notwithstanding, the legitimate father is still preferred over the grandparents. The latter's wealth is not a deciding factor, particularly because there is no proof that at the present time, petitioner is in no position to support the boy. The fact that he was unable to provide financial support for his minor son from birth up to over three years when he took the boy from his in-laws without permission, should not be sufficient reason to strip him of his permanent right to the child's custody. While petitioner's previous inattention is inexcusable and merits only the severest criticism, it cannot be construed as abandonment. His appeal of the unfavorable decision against him and his efforts to keep his only child in his custody may be regarded as serious efforts to rectify his past misdeeds. To award him custody would help enhance the bond between parent and son. It would also give the father a chance to prove his love for his son and for the son to experience the warmth and support which a father can give. His being a soldier is likewise no bar to allowing him custody over the boy. So many men in uniform who are assigned to different parts of the country in the service of the nation, are still the natural guardians of their children. It is not just to deprive our soldiers of authority, care and custody over their children merely because of the normal consequences of their duties and assignments, such as temporary separation from their families. Petitioner's employment of trickery in spiriting away his boy from his in-laws, though unjustifiable, is likewise not a ground to wrest custody from him. Private respondents' attachment to the young boy whom they have reared for the past three years is understandable. Still and all, the law considers the natural love of a parent to outweigh that of the grandparents, such that only when the parent present is shown to be unfit or unsuitable may the grandparents exercise substitute parental authority, a fact which has not been proven here. The strong bonds of love and affection possessed by private respondents as grandparents should not be seen as incompatible with petitioner' right to custody over the child as a father. Moreover, who is to say whether the petitioner's financial standing may improve in the future?

900

University of the Cordilleras College of Law First Year C S.Y. 2013 - 2014 WHEREFORE, the petition is GRANTED. The decision of the respondent Court of Appeals dated April 30, 1992 as well as its Resolution dated November 13, 1992 are hereby REVERSED and SET ASIDE. Custody over the minor Leouel Santos Jr. is awarded to his legitimate father, herein petitioner Leouel Santos, Sr. SO ORDERED. Feliciano, Melo, Vitug and Francisco, JJ., concur.

901

University of the Cordilleras College of Law First Year C S.Y. 2013 - 2014 Sagala-Eslao vs. CA G.R. No. 116773 Full Case January 16, 1997

TERESITA SAGALA-ESLAO, petitioner, vs. COURT OF APPEALS and MARIA PAZ CORDERO-OUYE, respondents. TORRES, JR., J.: Children begin by loving their parents. After a time they judge them. Rarely, if ever, do they forgive them. Indeed, parenthood is a riddle of no mean proportions except for its mission. Thus, a mother's concern for her child's custody is undying such is a mother's love. The right of the mother to the custody of her daughter is the issue in the case at bar. In this petition for review, Teresita Sagala-Eslao seeks the reversal of the Court of Appeals decision dated March 25, 1994, which affirmed the trial court's judgment granting the petition of Maria Paz CorderoOuye to recover the custody of her minor daughter from her mother-in-law, Teresita Sagala-Eslao. As found by the Court of Appeals, the facts of the case are as follows: From the evidence, it appears that on June 22, 1984, petitioner Maria Paz Cordero-Ouye and Reynaldo Eslao were married; 3 after their marriage, the couple stayed with respondent Teresita Eslao, mother of the husband, at 1825, Road 14, Fabie Estate, Paco, Manila; that out of their marriage, two children were begotten, namely, Leslie Eslao who was born on February 23, 1986 and Angelica Eslao who was born on April 20, 1987; in the meantime, Leslie was entrusted to the care and custody of petitioner's mother in Sta. Ana, Pampanga, while Angelica stayed with her parents at respondent's house; on August 6, 1990, petitioner's husband Reynaldo Eslao died; petitioner intended to bring Angelica with her to Pampanga but the respondent prevailed upon her to entrust the custody of Angelica to her, respondent reasoning out that her son just died and to assuage her grief therefor, she needed the company of the child to at least compensate for the loss of her late son. In the meantime, the petitioner returned to her mother's house in Pampanga where she stayed with Leslie. Subsequently, petitioner was introduced by her auntie to Dr. James Manabu-Ouye, a JapaneseAmerican, who is an orthodontist practicing in the United States; their acquaintance blossomed into a meaningful relationship where on March 18, 1992, the petitioner and Dr. James Ouye decided to get married; less than ten months thereafter, or on January 15, 1993, the petitioner migrated to San Francisco, California, USA, to join her new husband. At present, the petitioner is a trainee at the Union Bank in San Francisco, while her husband is a progressive practitioner of his profession who owns three cars, a dental clinic and earns US$5,000 a month. On June 24, 1993, the petitioner returned to the Philippines to be reunited with her children and bring them to the United States; the petitioner then informed the respondent about her desire to take informed the respondent about her desire to take custody of Angelica and explained that her present husband, Dr. James Ouye, expressed his willingness to adopt Leslie and Angelica and to provide for their support and education; however, respondent resisted the idea by way of explaining that the child was entrusted to her when she was ten days old and accused the petitioner of having abandoned Angelica. Because of the adamant attitude of the respondent, the petitioner then sought the assistance of a lawyer, Atty. Mariano de Joya, Jr., who wrote a letter to the respondent demanding for the return of the custody of Angelica to her natural mother and when the demand remain[ed] unheeded, the petitioner instituted the present action. After the trial on the merits, the lower court rendered its decision, the dispositive portion of which reads: 902

University of the Cordilleras College of Law First Year C S.Y. 2013 - 2014 WHEREFORE, finding the petition to be meritorious, the Court grants the same and let the corresponding writ issue. As a corollary, respondent Teresita Sagala-Eslao or anyone acting under her behalf is hereby directed to cause the immediate transfer of the custody of the minor Angelica Cordero Eslao, to her natural mother, petitioner Maria Paz Cordero-Ouye. No pronouncement as to costs. SO ORDERED. On appeal, the respondent court affirmed in full the decision of the trial court. Hence, the instant petition by the minor's paternal grandmother, contending that the Court of Appeals erred: I IN RULING THAT PRIVATE RESPONDENT MARIA PAZ CORDERO-OUYE, DID NOT ABANDON MINOR, ANGELICA ESLAO, TO THE CARE AND CUSTODY OF THE PETITIONER TERESITA SAGALA-ESLAO. II IN RULING THAT THERE WAS NO COMPELLING REASON TO SEPARATE MINOR, ANGELICA ESLAO, FROM PRIVATE RESPONDENT MARIA PAZ CORDERO-OUYE, IN FAVOR OF PETITIONER TERESITA SAGALA-ESLAO. III IN NOT FINDING THAT PETITIONER TERESITA SAGALA-ESLAO, IS FIT TO BE GIVEN THE CUSTODY OF MINOR, ANGELICA ESLAO. The petition is without merit. Being interrelated, the issues shall be discussed jointly. Petitioner argues that she would be deserving to take care of Angelica; that she had managed to raise 12 children of her own herself; that she has the financial means to carry out her plans for Angelica; that she maintains a store which earns a net income of about P500 a day, she gets P900 a month as pension for the death of her husband, she rents out rooms in her house which she owns, for which she earns a total of P6,000 a month, and that from her gross income of roughly P21,000, she spends about P10,000 for the maintenance of her house. Despite the foregoing, however, and petitioner's "genuine desire to remain with said child, that would qualify her to have custody of Angelica," the trial court's disquisition, in consonance with the provision that the child's welfare is always the paramount consideration in all questions concerning his care and custody convinced this Court to decide in favor of private respondent, thus: On the other hand, the side of the petitioner must also be presented here. In this case, we see a picture of a real and natural mother who is . . . legitimately, anxiously, and desperately trying to get back her child in order to fill the void in her heart and existence. She wants to make up for what she has failed to do for 903

University of the Cordilleras College of Law First Year C S.Y. 2013 - 2014 her boy during the period when she was financially unable to help him and when she could not have him in her house because of the objection of the father. Now that she has her own home and is in a better financial condition, she wants her child back, and we repeat that she has not and has never given him up definitely or with any idea of permanence. The petitioner herein is married to an Orthodontist who has lucrative practice of his profession in San Francisco, California, USA. The petitioner and her present husband have a home of their own and they have three cars. The petitioner's husband is willing to adopt the petitioner's children. If the children will be with their mother, the probability is that they will be afforded a bright future. Contrast this situation with the one prevailing in the respondent's [grandmother's] house. As admitted by the respondent, four of the rooms in her house are being rented to other persons with each room occupied by 4 and 5 persons. Added to these persons are the respondent's 2 sons, Samuel and Alfredo, and their respective families (ibid., p. 54) and one can just visualize the kind of atmosphere pervading thereat. And to aggravate the situation, the house has only 2 toilets and 3 faucets. Finally, considering that in all controversies involving the custody of minors, the foremost criterion is the physical and moral wellbeing of the child taking into account the respective resources and social and moral situations of the contending parties (Union III vs. Mariano, 101 SCRA 183), the Court is left with no other recourse but to grant the writ prayed for. Petitioner further contends that the respondent court erred in finding that there was no abandonment committed by the private respondent; that while judicial declaration of abandonment of the child in a case filed for the purpose is not her obtaining as mandated in Art. 229 of the Family Code because petitioner failed to resort to such judicial action, it does not ipso facto follow that there was in fact no abandonment committed by the private respondent. Petitioner also argues that it has been amply demonstrated during the trial that private respondent had indeed abandoned Angelica to the care and custody of the petitioner; that during all the time that Angelica stayed with petitioner, there were only three instances or occasions wherein the private respondent saw Angelica; that private respondent never visited Angelica on important occasions, such as her birthday, and neither did the former give her cards or gifts, "not even a single candy;" that while private respondent claims otherwise and that she visited Angelica "many times" and insists that she visited Angelica as often as four times a month and gave her remembrances such as candies and clothes, she would not even remember when the fourth birthday of Angelica was. We are not persuaded by such averments. In Santos, Sr. vs. Court of Appeals, 242 SCRA 407, we stated, viz: . . . [Parental authority] is a mass of rights and obligations which the law grants to parents for the purpose of the children's physical preservation and development, as well as the cultivation of their intellect and the education of their heart and senses. As regards parental authority, "there is no power, but a task; no complex of rights, but a sum of duties; no sovereignty but a sacred trust for the welfare of the minor." Parental authority and responsibility are inalienable and may not be transferred or renounced except in cases authorized by law. The right attached to parental authority, being purely personal, the law allows a waiver of parental authority only in cases of adoption, guardianship and surrender to a children's home or an orphan institution. When a parent entrusts the custody of a minor to another, such as a friend or godfather, even in a document, what is given is merely temporary custody and it does not constitute a renunciation of parental authority. Even if a definite renunciation is manifest, the law still disallows the same.

904

University of the Cordilleras College of Law First Year C S.Y. 2013 - 2014 The father and mother, being the natural guardians of unemancipated children, are duty-bound and entitled to keep them in their custody and company. Thus, in the instant petition, when private respondent entrusted the custody of her minor child to the petitioner, what she gave to the latter was merely temporary custody and it did not constitute abandonment or renunciation of parental authority. For the right attached to parental authority, being purely personal, the law allows a waiver of parental authority only in cases of adoption, guardianship and surrender to a children's home or an orphan institution which do not appear in the case at bar. Of considerable importance is the rule long accepted by the courts that "the right of parents to the custody of their minor children is one of the natural rights incident to parenthood, a right supported by law and sound public policy. The right is an inherent one, which is not created by the state or decisions of the courts, but derives from the nature of the parental relationship. IN VIEW WHEREOF, the decision appealed from dated March 25, 1994 being in accordance with law and the evidence, the same is hereby AFFIRMED and the petition DISMISSED for lack of merit. SO ORDERED. Regalado, Romero, Puno and Mendoza, JJ., concur.

905

University of the Cordilleras College of Law First Year C S.Y. 2013 - 2014 Sandejas vs Ignacio Jr. 541 SCRA 61 G.R. No. 155033 December 19, 2007 Full Case ALICE A.I. SANDEJAS, ROSITA A.I. CUSI, PATRICIA A.I. SANDEJAS and BENJAMIN A.I. ESPIRITU, vs SPS. ARTURO IGNACIO, JR. and EVELYN IGNACIO, AUSTRIA-MARTINEZ, J.: Before the Court is a Petition for Review on Certiorari under Rule 45 of the Rules of Court assailing the Decision of the Court of Appeals (CA) in CA-G.R. CV No. 62404 promulgated on August 27, 2002, which affirmed with modification the Decision of the Regional Trial Court (RTC) of Pasig City, Branch 158, in Civil Case No. 65146 dated December 18, 1998. The facts of the case, as summarized by the RTC, are as follows: It appears from the plaintiffs' [petitioners] evidence that Arturo [respondent] is the elder brother of Alice [petitioner] and Rosita [petitioner], Benjamin [petitioner] and Patricia [petitioner] are Arturo's nephew and niece. Arturo and his wife Evelyn [respondent] are residents of the United States. In October 1993, Arturo leased from Dr. Borja a condominium unit identified as Unit 28-C Gilmore Townhomes located at Granada St.,Quezon City. The lease was for the benefit of Benjamin who is the occupant of the unit. The rentals were paid by Ignacio. The term of the lease is for one (1) year and will expire on October 15, 1994. It appears that Arturo was intending to renew the lease contract. As he had to leave for theU.S., Arturo drew up a check, UCPB Check No. GRH-560239 and wrote on it the name of the payee, Dr. Manuel Borja, but left blank the date and amount. He signed the check. The check was intended as payment for the renewal of the lease. The date and the amount were left blank because Arturo does not know when it will be renewed and the new rate of the lease. The check was left with Arturo's sister-in-law, who was instructed to deliver or give it to Benjamin. The check later came to the possession of Alice who felt that Arturo cheated their sister in the amount of three million pesos (P3,000,000.00). She believed that Arturo and Rosita had a joint and/or money market placement in the amount of P3 million with the UCPB branch at Ortigas Ave., San Juan and that Ignacio preterminated the placement and ran away with it, which rightfully belonged to Rosita. Alice then inquired from UCPB Greenhills branch if Arturo still has an account with them. On getting a confirmation, she together with Rosita drew up a scheme to recover the P3 million from Arturo. Alice filled up the date of the check with March 17, 1995 and the amount with three million only. Alice got her driver, Kudera, to stand as the payee of the check, Dr. Borja. Alice and Rosita came to SBC Greenhills Branch together with a man (Kudera) who[m] they introduced as Dr. Borja to the then Assistant Cashier Luis. After introducing the said man as Dr. Borja, Rosita, Alice and the man who was later identified as Kudera opened a Joint Savings Account No. 271-410554-7. As initial deposit for the Joint Savings Account, Alice, Rosita and Kudera deposited the check. No ID card was required of Mr. Kudera because it is an internal policy of the bank that when a valued client opens an account, an identification card is no longer required (TSN, April 21, 1997, pp. 15-16). SBC also allowed the check to be deposited without the endorsement of the impostor Kudera. SBC officials stamped on the dorsal portion of the check endorsement/lack of endorsement guaranteed and sent the check for clearing to the Philippine Clearing House Corporation.

906

University of the Cordilleras College of Law First Year C S.Y. 2013 - 2014 On 21 March 1995, after the check had already been cleared by the drawer bank UCPB, Rosita withdrew P1 million from Joint Savings Account and deposited said amount to the current account of Alice with SBC Greenhills Branch. On the same date, Alice caused the transfer of P2 million from the Joint Savings Account to two (2) Investment Savings Account[s] in the names of Alice, Rosita and/or Patricia. ... On April 4, 1995, a day after Evelyn and Atty. Sanz inquired about the identity of the persons and the circumstances surrounding the deposit and withdrawal of the check, the three million pesos in the two investment savings account[s] and in the current account just opened with SBC were withdrawn by Alice and Rosita. On June 18, 1995, Arturo Ignacio, Jr. and Evelyn Ignacio (respondents) filed a verified complaint for recovery of a sum of money and damages against Security Bank and Trust Company (SBTC) and its officers, namely: Rene Colin D. Gray, Manager; and Sonia Ortiz-Luis, Cashier. The complaint also impleaded herein petitioner Benjamin A.I. Espiritu (Benjamin), a John Doe, representing himself as Manuel N. Borja; and a Jane Doe. On November 7, 1995, the complaint was amended by additionally impleading herein petitioners Alice A.I. Sandejas (Alice), Rosita A.I. Cusi (Rosita) and Patricia A.I. Sandejas (Patricia) as defendants who filed their respective answers and counterclaims. After trial, the RTC following dispositive portion: rendered judgment dated December 18, 1998 with the

WHEREFORE, in view of the foregoing, judgment is rendered in favor of plaintiffs as against defendants Security Bank and Trust Co., Rene Colin Gray, Sonia Ortiz Luis, Alice A.I. Sandejas and Rosita A.I. Cusi, ordering them to pay jointly and severally the plaintiffs the following amounts: (1) P3,000,000.00 plus legal interest on it from March 17, 1995 until the entire amount is fully paid; (2) P500,000.00 as moral damages; (3) P200,000.00 as exemplary damages; (4) P300,000.00 as attorney's fees; plus (5) the cost of suit. In turn, plaintiffs are directed to pay Benjamin A.I. Espiritu the amount of P100,000.00 as moral damages, P50,000.00 as exemplary damages and another P50,000.00 as attorney's fees. The counterclaims of Patricia A.I. Sandejas are dismissed. SO ORDERED Both parties appealed the RTC Decision to the CA. On August 14, 1999, during the pendency of the appeal with the CA, herein respondent Arturo Ignacio, Jr. (Arturo) died. On August 27, 2002, the CA promulgated the presently assailed Decision, disposing as follows: WHEREFORE, in view of the foregoing, the assailed decision of the trial court is hereby AFFIRMED with the MODIFICATION that the judgment shall read as follows:

907

University of the Cordilleras College of Law First Year C S.Y. 2013 - 2014 The defendants-appellants Security Bank and Trust Company, Rene Colin D. Gray, Sonia Ortiz-Luis, Alice A.I. Sandejas, and Rosita A.I.Cusi, are hereby ordered to jointly and severally pay the plaintiffs the following amounts: 1. P3,000,000.00 plus legal interest computed 17, 1995 until the entire amount is fully paid; 2. P200,000.00 as moral damages; 3. P100,000.00 as exemplary damages; 4. P50,000.00 as attorney's fees; plus 5. the costs of suit. from March

The award of moral damages, exemplary damages, and attorney's fees in favor of Benjamin Espiritu is DELETED. SO ORDERED. Petitioners and SBTC, together with Gray and Ortiz-Luis, filed their respective petitions for review before this Court. However, the petition filed by SBTC, Gray and Ortiz-Luis, docketed as G.R. No. 155038, was denied in a Resolution issued by this Court on November 20, 2002, for their failure to properly verify the petition, submit a valid certification of non-forum shopping, and attach to the petition the duplicate original or certified true copy of the assailed CA Decision. Said Resolution became final and executory on April 9, 2003. On the other hand, the instant petition was given due course. Petitioners enumerated the following grounds in support of their petition: I. THE COURT OF APPEALS HAD DECIDED A QUESTION OF SUBSTANCE NOT HERETOFORE DECIDED BY THIS COURT AND/OR HAD DECIDED IT IN A WAY PROBABLY NOT IN ACCORD WITH EQUITY, THE LAW AND THE APPLICABLE DECISIONS OF THIS COURT, SUCH AS: (a) IN NOT HOLDING THAT AS BETWEEN SIBLINGS, THE AGGRIEVED SIBLING HAS THE RIGHT TO TAKE MEASURES OR STEPS TO PROTECT HIS OWN INTEREST OR PROPERTY RIGHTS FROM AN ACT OF THE GUILTY SIBLING; (b) IN NOT HOLDING THAT THE ACT OF ROSITA AND ALICE IN FILLING OUT THE BLANK PORTIONS OF THE CHECK TO RECOVER WHAT ARTURO, JR. TOOK FROM AND DUE ROSITA, DID NOT GIVE RISE TO AN ACTIONABLE TORT; (c) IN NOT HOLDING THAT THE CRIMINAL ACT OF ARTURO, JR. IN SUBMITTING AN AFFIDAVIT OF LOSS OF THE CERTIFICATE OF TIME DEPOSIT FOR P3,000,000 THAT RIGHTFULLY BELONGED TO ROSITA JUST TO BE ABLE TO PRE-TERMINATE THE TIME DEPOSIT AND GET ITS FACE VALUE, WHEN HE KNEW IT WAS NOT LOST BUT IN FACT INTACT AND IN THE POSSESSION OF ROSITA, IS A DISHONEST AND REPREHENSIBLE ACT THAT JUSTIFIED ROSITA AND ALICE IN TAKING MEANS TO REGAIN THE MONEY AND TO DENY ARTURO, JR. ANY RIGHT TO RECOVER THE SAID AMOUNT AS WELL AS TO AN AWARD OF DAMAGES;

908

University of the Cordilleras College of Law First Year C S.Y. 2013 - 2014 (d) IN NOT HOLDING THAT THE CRIMINAL ACT OF ARTURO, JR. IN SUBMITTING AN AFFIDAVIT OF LOSS OF THE OWNER'S COPY OF THE TITLE IN MORAYTA AND IN TESTIFYING IN COURT AS TO SUCH, WHEN THAT IS NOT THE TRUTH AS HE KNEW THAT THE ORIGINAL OWNER'S COPY OF THE TITLE WAS WITH ROSITA, IS ANOTHER DISHONEST AND REPREHENSIBLE ACT THAT SHOULD NOT HAVE ENTITLED HIM TO ANY AWARD OF DAMAGES; AND (e) IN NOT APPLYING THE RULE ON PARI DELICTO UNDER ART. 1412 OF THE CIVIL CODE. II. THE COURT OF APPEALS HAD DEPARTED FROM THE USUAL COURSE OF JUDICIAL PROCEEDINGS WHEN IT FAILED TO RESOLVE IN THE APPEAL THE COUNTERCLAIM OF ROSITA AGAINST ARTURO, JR. FOR THE RECOVERY OF THE AMOUNTS LEGALLY HERS THAT SHOULD JUSTIFY ALICE'S BEING ABSOLVED FROM ANY LIABILITY FOR USING THE CHECK IN RECOVERING THE AMOUNT RIGHTFULLY BELONGING TO ROSITA; III. THE COURT OF APPEALS HAD DEPARTED FROM THE USUAL COURSE OF JUDICIAL PROCEEDINGS WHEN IT REVERSED THE TRIAL COURT'S FINDING THAT RESPONDENT WAS GUILTY OF BAD FAITH AND MALICE THAT ENTITLED PETITIONER BENJAMIN A.I. ESPIRITU TO THE AWARD OF DAMAGES NOTWITHSTANDING THAT THERE WAS AMPLE EVIDENCE SHOWN THAT SUCH BAD FAITH AND MALICE WAS MADE AS A LEVERAGE TO COMPEL ARTURO'S SIBLINGS TO RETURN TO HIM THE P3,000,000 WHICH WAS NOT HIS; and, IV. THE COURT OF APPEALS HAD DECIDED THE CASE NOT IN ACCORD WITH LAW WHEN IT DELETED THE AWARD OF DAMAGES TO PETITIONER ESPIRITU AND IN NOT HAVING RULED THAT HE WAS ENTITLED TO A HIGHER AWARD OF DAMAGES CONSIDERING THE CIRCUMSTANCES OF THE CASE AS WELL AS IN NOT HAVING RULED THAT PATRICIA WAS ENTITLED TO AN AWARD OF DAMAGES. Petitioners argue that the CA overlooked and ignored vital pieces of evidence showing that the encashment of the subject check was not fraudulent and, on the contrary, was justified under the circumstances; and that such encashment did not amount to an actionable tort and that it merely called for the application of the civil law rule on pari delicto. In support of these arguments, petitioners contend that the principal adversaries in the present case are full blooded siblings; that the law recognizes the solidarity of family which is why it is made a condition that earnest efforts towards a compromise be exerted before one family member can institute a suit against the other; that even if Arturo previously defrauded Rosita and deprived her of her lawful share in the sale of her property, petitioners Rosita and Alice did not precipitately file suit against him and instead took extra-legal measures to protect Rosita's property rights and at the same time preserve the solidarity of their family and save it from public embarrassment. Petitioners also aver that Rosita's and Alice's act of encashing the subject check is not fraudulent because they did not have any unlawful intent and that they merely took from Arturo what rightfully belonged to Rosita. Petitioners contend that even granting that the act of Rosita and Alice amounted to an actionable tort, they could not be adjudged liable to return the amount to respondents or to pay damages in their favor, because the civil law rule on pari delicto dictates that, when both parties are at fault, neither of them could expect positive relief from courts of justice and, instead, are left in the state where they were at the time of the filing of the case.

909

University of the Cordilleras College of Law First Year C S.Y. 2013 - 2014 Petitioners also contend that the CA erred in failing to award damages to Patricia even if the appellate court sustained the trial court's finding that she was not a party to the fraudulent acts committed by Rosita and Alice. Petitioners argue that even if Patricia did not bother to know the details of the cases against her and left everything to her mother, she did not even know the nature of the case against her, or her superiors in the bank where she worked did not know whether she was the plaintiff or defendant, these were not reasons to deny her award of damages. The fact remains that she had been maliciously dragged into the case, and that the suit had adversely affected her work and caused her mental worries and anguish, besmirched reputation, embarrassment and humiliation. As to Benjamin, petitioners aver that the CA also erred in deleting the award of damages and attorney's fees in his favor. Petitioners assert that the trial court found that Benjamin suffered mental anguish, wounded feelings and moral shock as a result of the filing of the present case. Citing the credentials and social standing of Benjamin, petitioners claim that the award of damages and attorney's fees in his favor should be increased. Lastly, petitioners contend that the award of damages and attorney's fees to respondents should be deleted for their failure to establish malice or bad faith on the part of petitioners Alice and Rosita in recovering the P3,000,000.00 which Arturo took from Rosita; and that it is Rosita who is entitled to damages and attorney's fees for Arturo's failure and refusal to give her share in the sale of her property in Morayta. In their Memorandum, respondents simply contend that the issues raised by petitioners are factual in nature and that the settled rule is that questions of fact are not subject to review by the Supreme Court in a petition for review on certiorari under Rule 45 of the Rules of Court. While there are exceptions to this rule, respondents assert that petitioners failed to show that the instant case falls under any of these exceptions. The Courts Ruling The Court finds the petition bereft of merit. There is no compelling reason for the Court to disturb the findings of facts of the lower courts. The trial court's findings are as follows: (1) Rosita failed to establish that there is an agreement between her and Arturo that the latter will give her one-third of the proceeds of the sale of the Morayta property; (2) petitioners were not able to establish by clear and sufficient evidence that the P3,000,000.00 which they took from Arturo when they encashed the subject check was part of the proceeds of the sale of the Morayta property; (3) Rosita's counterclaim is permissive and she failed to pay the full docket and filing fees for her counterclaim. Petitioners challenge the findings of the RTC and insist that they should not be held liable for encashing the subject check because Arturo defrauded Rosita and that he committed deceitful acts which deprived her of her rightful share in the sale of her building in Morayta; that the amount of P3,000,000.00 represented by the check which they encashed formed part of the proceeds of the said sale; that Alice and Rosita were merely moved by their desire to recover from Arturo, Rosita's supposed share in the sale of her property. However, the Court agrees with respondents that only questions of law are entertained in petitions for review on certiorariunder Rule 45 of the Rules of Court. The trial courts findings of fact, which the Court of Appeals affirmed, are generally binding and conclusive upon this court. There are recognized exceptions to this rule, among which are: (1) the conclusion is grounded on speculations, surmises or conjectures; (2) the inference is manifestly mistaken, absurd or impossible; (3) there is grave abuse of discretion; (4) the judgment is based on a misapprehension of facts; (5) the findings of facts are conflicting; (6) there is no citation of specific evidence on which the factual findings are based; (7) the finding of absence of facts is contradicted by the presence of evidence on record; (8) the findings of the 910

University of the Cordilleras College of Law First Year C S.Y. 2013 - 2014 CA are contrary to the findings of the trial court; (9) the CA manifestly overlooked certain relevant and undisputed facts that, if properly considered, would justify a different conclusion; (10) the findings of the CA are beyond the issues of the case; and (11) such findings are contrary to the admissions of both parties. In the instant case, petitioners failed to demonstrate that their petition falls under any one of the above exceptions. Petitioners' assignments of errors boil down to the basic issue of whether or not Alice and Rosita are justified in encashingthe subject check given the factual circumstances established in the present case. Petitioners' posture is not sanctioned by law. If they truly believe that Arturo took advantage of and violated the rights of Rosita, petitioners should have sought redress from the courts and should not have simply taken the law into their own hands. Our laws are replete with specific remedies designed to provide relief for the violation of one's rights. In the instant case, Rosita could have immediately filed an action for the nullification of the sale of the building she owns in light of petitioners' claim that the document bearing her conformity to the sale of the said building was taken by Arturo from her without her knowledge and consent. Or, in the alternative, as the CA correctly held, she could have brought a suit for the collection of a sum of money to recover her share in the sale of her property in Morayta. In a civilized society such as ours, the rule of law should always prevail. To allow otherwise would be productive of nothing but mischief, chaos and anarchy. As a lawyer, who has sworn to uphold the rule of law, Rosita should know better. She must go to court for relief. It is true that Article 151 of the Family Code requires that earnest efforts towards a compromise be made before family members can institute suits against each other. However, nothing in the law sanctions or allows the commission of or resort to any extra-legal or illegal measure or remedy in order for family members to avoid the filing of suits against another family member for the enforcement or protection of their respective rights. Petitioners invoke the rule of pari delicto to support their contention that respondents do not deserve any relief from the courts. The principle of pari delicto provides that when two parties are equally at fault, the law leaves them as they are and denies recovery by either one of them. Indeed, one who seeks equity and justice must come to court with clean hands. However, in the present case, petitioners were not able to establish that respondents are also at fault. Thus, the principle of pari delicto cannot apply. In any case, the application of the pari delicto principle is not absolute, as there are exceptions to its application. One of these exceptions is where the application of the pari delicto rule would violate wellestablished public policy. The prevention of lawlessness and the maintenance of peace and order are established public policies. In the instant case, to deny respondents relief on the ground of pari delicto would put a premium on the illegal act of petitioners in taking from respondents what the former claim to be rightfully theirs. Petitioners also question the trial court's ruling that their counterclaim is permissive. This Court has laid down the following tests to determine whether a counterclaim is compulsory or not, to wit: (1) Are the issues of fact or law raised by the claim and the counterclaim largely the same? (2) Would res judicata bar a subsequent suit on defendants claims, absent the compulsory counterclaim rule? (3) Will substantially the same evidence support or refute plaintiffs claim as well as the defendants counterclaim? and (4) Is there any logical relation between the claim and the counterclaim, such that the conduct of separate trials of the respective claims of the parties would entail a substantial duplication of effort and time by the parties and the court? Tested against the above-mentioned criteria, this Court agrees with the view of the RTC that Rosita's counterclaim for the recovery of her alleged share in the sale of the Morayta property is permissive in nature. The evidence needed to prove respondents' claim to recover the amount 911

University of the Cordilleras College of Law First Year C S.Y. 2013 - 2014 of P3,000,000.00 from petitioners is different from that required to establish Rosita's demands for the recovery of her alleged share in the sale of the subject Morayta property. The recovery of respondents' claim is not contingent or dependent upon the establishment of Rosita's counterclaim such that conducting separate trials will not result in the substantial duplication of the time and effort of the court and the parties. In Sun Insurance Office, Ltd., (SIOL) v. Asuncion, this Court laid down the rules on the payment of filing fees, to wit: 1. It is not simply the filing of the complaint or appropriate initiatory pleading, but the payment of the prescribed docket fee, that vests a trial court with jurisdiction over the subject-matter or nature of the action. Where the filing of the initiatory pleading is not accompanied by payment of the docket fee, the court may allow payment of the fee within a reasonable time but in no case beyond the applicable prescriptive or reglementary period. 2. The same rule applies to permissive counterclaims, third-party claims and similar pleadings, which shall not be considered filed until and unless the filing fee prescribed therefor is paid. The court may allow payment of said fee within a reasonable time but also in no case beyond its applicable prescriptive or reglementary period. 3. Where the trial court acquires jurisdiction over a claim by the filing of the appropriate pleading and payment of the prescribed filing fee but, subsequently, the judgment awards a claim not specified in the pleading, or if specified the same has been left for determination by the court, the additional filing fee therefor shall constitute a lien on the judgment. It shall be the responsibility of the Clerk of Court or his duly authorized deputy to enforce said lien and assess and collect the additional fee. In order for the trial court to acquire jurisdiction over her permissive counterclaim, Rosita is bound to pay the prescribed docket fees. Since it is not disputed that Rosita never paid the docket and filing fees, the RTC did not acquire jurisdiction over her permissive counterclaim. Nonetheless, the trial court ruled on the merits of Rosita's permissive counterclaim by dismissing the same on the ground that she failed to establish that there is a sharing agreement between her and Arturo with respect to the proceeds of the sale of the subject Morayta property and that the amount of P3,000,000.00 represented by the check which Rosita and Aliceencashed formed part of the proceeds of the said sale. It is settled that any decision rendered without jurisdiction is a total nullity and may be struck down at any time, even on appeal before this Court. In the present case, considering that the trial court did not acquire jurisdiction over the permissive counterclaim of Rosita, any proceeding taken up by the trial court and any ruling or judgment rendered in relation to such counterclaim is considered null and void. In effect, Rosita may file a separate action against Arturo for recovery of a sum of money. However, Rosita's claims for damages and attorney's fees are compulsory as they necessarily arise as a result of the filing by respondents of their complaint. Being compulsory in nature, payment of docket fees is not required. Nonetheless, since petitioners are found to be liable to return to respondents the amount of P3,000,000.00 as well as to pay moral and exemplary damages and attorney's fees, it necessarily follows that Rosita's counterclaim for damages and attorney's fees should be dismissed as correctly done by the RTC and affirmed by the CA. As to Patricia's entitlement to damages, this Court has held that while no proof of pecuniary loss is necessary in order that moral damages may be awarded, the amount of indemnity being left to the discretion of the court, it is nevertheless essential that the claimant should satisfactorily show the 912

University of the Cordilleras College of Law First Year C S.Y. 2013 - 2014 existence of the factual basis of damages and its causal connection to defendants acts. This is so because moral damages, though incapable of pecuniary estimation, are in the category of an award designed to compensate the claimant for actual injury suffered and not to impose a penalty on the wrongdoer. Moreover, additional facts must be pleaded and proven to warrant the grant of moral damages under the Civil Code, these being, social humiliation, wounded feelings, grave anxiety, etc. that resulted from the act being complained of. In the present case, both the RTC and the CA were not convinced that Patricia is entitled to damages. Quoting the RTC, the CA held thus: With respect to Patricia, she did not even bother to know the details of the case against her, she left everything to the hands of her mother Alice. Her attitude towards the case appears weird, she being a banker who seems so concerned of her reputation. Aside from the parties to this case, her immediate superiors in the BPI knew that she is involved in a case. They did not however know whether she is the plaintiff or the defendant in the case. Further, they did not know the nature of the case that she is involved in. It appears that Patricia has not suffered any of the injuries enumerated in Article 2217 of the Civil Code, thus, she is not entitled to moral damages and attorney's fees. This Court finds no cogent reason to depart from the above-quoted findings as Patricia failed to satisfactorily show the existence of the factual basis for granting her moral damages and the causal connection of such fact to the act of respondents in filing a complaint against her. In addition, and with respect to Benjamin, the Court agrees with the CA that in the absence of a wrongful act or omission, or of fraud or bad faith, moral damages cannot be awarded. The adverse result of an action does not per se make the action wrongful, or the party liable for it. One may err, but error alone is not a ground for granting such damages. In the absence of malice and bad faith, the mental anguish suffered by a person for having been made a party in a civil case is not the kind of anxiety which would warrant the award of moral damages. A resort to judicial processes is not, per se, evidence of ill will upon which a claim for damages may be based. In China Banking Corporation v. Court of Appeals, this Court held: Settled in our jurisprudence is the rule that moral damages cannot be recovered from a person who has filed a complaint against another in good faith, or without malice or bad faith (Philippine National Bank v. Court of Appeals, 159 SCRA 433 [1988]; R & B Surety and Insurance v. Intermediate Appellate Court, 129 SCRA 736 [1984]). If damage results from the filing of the complaint, it is damnum absque injuria (IlocosNorte Electrical Company v. Court of Appeals, 179 SCRA 5 [1989]). In the present case, the Court agrees with the RTC and the CA that petitioners failed to establish that respondents were moved by bad faith or malice in impleading Patricia and Benjamin. Hence, Patricia and Benjamin are not entitled to damages. The Court sustains the award of moral and exemplary damages as well as attorney's fees in favor of respondents. As to moral damages, Article 20 of the Civil Code provides that every person who, contrary to law, willfully or negligently causes damage to another, shall indemnify the latter for the same. In addition, Article 2219 (10) of the Civil Code provides that moral damages may be recovered in acts or actions referred to in Articles 21, 26, 27, 28, 29, 30, 32, 34 and 35 of the same Code. More particularly, Article 21 of the said Code provides that any person who willfully causes loss or injury to another in a manner 913

University of the Cordilleras College of Law First Year C S.Y. 2013 - 2014 that is contrary to morals, good customs, or public policy shall compensate the latter for the damage. In the present case, the act of Alice and Rosita in fraudulently encashing the subject check to the prejudice of respondents is certainly a violation of law as well as of the public policy that no one should put the law into his own hands. As to SBTC and its officers, their negligence is so gross as to amount to a willfull injury to respondents. The banking system has become an indispensable institution in the modern world and plays a vital role in the economic life of every civilized society. Whether as mere passive entities for the safe-keeping and saving of money or as active instruments of business and commerce, banks have attained a ubiquitous presence among the people, who have come to regard them with respect and even gratitude and most of all, confidence. For this reason, banks should guard against injury attributable to negligence or bad faith on its part. There is no hard-and-fast rule in the determination of what would be a fair amount of moral damages since each case must be governed by its own peculiar facts. The yardstick should be that it is not palpably and scandalously excessive. Moreover, the social standing of the aggrieved party is essential to the determination of the proper amount of the award. Otherwise, the goal of enabling him to obtain means, diversions, or amusements to restore him to the status quo ante would not be achieved. In the present case, the Court finds no cogent reason to modify the amount of moral damages granted by the CA. Likewise, the Court finds no compelling reason to disturb the modifications made by the CA on the award of exemplary damages and attorney's fees. Under Article 2229 of the Civil Code, exemplary or corrective damages are imposed by way of example or correction for the public good, in addition to moral, temperate, liquidated, or compensatory damages. In the instant case, the award of exemplary damages in favor of respondents is in order for the purpose of deterring those who intend to enforce their rights by taking measures or remedies which are not in accord with law and public policy. On the part of respondent bank, the public relies on a bank's sworn profession of diligence and meticulousness in giving irreproachable service. Hence, the level of meticulousness must be maintained at all times by the banking sector. In the present case the award of exemplary damages is justified by the brazen acts of petitioners Rosita and Alice in violating the law coupled with the gross negligence committed by respondent bank and its officers in allowing the subject check to be deposited which later paved the way for its encashment. As to attorney's fees, Article 2208 of the same Code provides, among others, that attorney's fees may be recovered when exemplary damages are awarded or when the defendant's act or omission has compelled the plaintiff to litigate with third persons or to incur expenses to protect his interest. WHEREFORE, the instant petition is DENIED. The Decision of the Court of Appeals dated August 27, 2002 in CA-G.R. CV No. 62404 is AFFIRMED. Costs against the petitioners. SO ORDERED.

914

University of the Cordilleras College of Law First Year C S.Y. 2013 - 2014 Sy vs. Court of Appeals G.R. No. 124518 December 27, 2007 Full Case WILSON SY, Petitioner, vs. COURT OF APPEALS, Regional Trial Court of Manila, Branch 48, and MERCEDES TAN UYSY, Respondents. TINGA, J.: In this Petition for Review on Certiorari under Rule 45 of the 1997 Rules of Civil Procedure, petitioner Wilson Sy assails the Decision dated 29 February 1996 of the Court of Appeals in C.A. G.R. SP No. 38936 and its Resolution dated 15 April 1996 denying his motion for reconsideration. The following are the antecedents: On 19 January 1994, respondent Mercedes Tan Uy-Sy filed a petition for habeas corpus against petitioner Wilson Sy before the Regional Trial Court of Manila, Branch 48, docketed as Special Proceeding No. 9469002. Respondent prayed that said writ be issued ordering petitioner to produce their minor children Vanessa and Jeremiah before the court and that after hearing, their care and custody be awarded to her as their mother. In his answer, petitioner prayed that the custody of the minors be awarded to him instead. Petitioner maintained that respondent was unfit to take custody of the minors. He adduced the following reasons: firstly, respondent abandoned her family in 1992; secondly, she is mentally unstable; and thirdly, she cannot provide proper care to the children. After trial, the trial court caused the issuance of a writ of habeas corpus and awarded custody of the children to respondent, to wit: WHEREFORE, judgment is hereby rendered maintaining to the petitioner the custody of the minors Vanessa and Jeremiah, all surnamed Uy-Sy, without, however, prejudice to the visitorial rights of the father, herein respondent, and the temporary arrangement of the custody made by the parties during pendency of this proceeding is hereby revoked, and without any further effect. The Court further orders the respondent to pay by way of monthly support for the minors, the amount of P50,000.00 payable to petitioner from [the] date of judgment for failure on the part of respondent to show by preponderance of evidence that the petitioner is unfit to the custody of the minor children who are only 6 and 4 years old. Petitioner appealed the order of the trial court to the Court of Appeals. Before the appellate court, he alleged that the trial court erred: (1) in awarding the custody of the minor children solely to respondent; and (2) in ordering him to provide respondent support in the amount of P50,000.00 per month. The Court of Appeals found no merit in the appeal and affirmed the decision of the trial court. The Court of Appeals did not find any reason to disturb the conclusions of the trial court, particularly petitioners failure to prove by preponderance of evidence that respondent was unfit to take custody over the minor children. The Court of Appeals held that petitioner was not able to substantiate his contention that respondent was unfit to have custody of the children. On respondents supposed abandonment of the family, the appellate court found instead that respondent had been driven away by petitioners family because of religious differences. Respondents stay in Taiwan likewise could hardly be called abandonment as she had gone there to earn enough money to reclaim her children. Neither could respondents act of praying outdoors in the rain be considered as evidence of insanity as it may simply be an expression of ones faith. Regarding 915

University of the Cordilleras College of Law First Year C S.Y. 2013 - 2014 the allegation that respondent was unable to provide for a decent dwelling for the minors, to the contrary, the appellate court was satisfied with respondents proof of her financial ability to provide her children with the necessities of life. As to the second assignment of error, the Court of Appeals held that questions as to care and custody of children may be properly raised in a petition for writ of habeas corpus. Moreover, petitioner was properly heard on the matter relative to the issue of support. He was questioned about his sources of income for the purpose of determining his ability to give support. As to the propriety of the amount awarded, the appellate court was unwilling to alter the trial courts conclusion for petitioner did not forthrightly testify on his actual income. Neither did he produce income tax returns or other competent evidence, although within his power to do so, to provide a fair indication of his resources. At any rate, the appellate court declared that a judgment of support is never final and petitioner is not precluded at any time from seeking a modification of the same and produce evidence of his claim. Petitioner filed a motion for reconsideration of the Court of Appeals decision but the same was denied. Hence, this appeal by certiorari wherein petitioner asserts that: (1) the Court of Appeals erred in awarding the custody of the minor children solely to respondent; (2) the Court of Appeals had no jurisdiction to award support in a habeas corpus case as: (a) support was neither alleged nor prayed for in the petition; (b) there was no express or implied consent on the part of the parties to litigate the issue; and (c) Section 6, Rule 99 of the Rules of Court does not apply because the trial court failed to consider the Civil Code provisions on support; and (3) the award of P50,000.00 as support is arbitrary, unjust, unreasonable and tantamount to a clear deprivation of property without due process of law. For her part, respondent claims that petitioner had lost his privilege to raise the first issue, having failed to raise it before the appellate court. Anent the second issue, respondent takes refuge in the appellate courts statement that the questions regarding the care and custody of children may properly be adjudicated in a habeas corpus case. Regarding the third issue, respondent maintains that the amount of support awarded is correct and proper. There is no merit in the petition regarding the question of care and custody of the children. The applicable provision is Section 213 of the Family Code which states that: Section 213. In case of separation of the parents, parental authority shall be exercised by the parent designated by the Court. The Court shall take into account all relevant considerations, especially the choice of the child over seven years of age, unless the parent is unfit. No child under seven years of age shall be separated from the mother, unless the court finds compelling reasons to order otherwise. In case of legal separation of the parents, the custody of the minor children shall be awarded to the innocent spouse, unless otherwise directed by the court in the interest of the minor children. But when the husband and wife are living separately and apart from each other, without decree of the court, the court shall award the care, custody, and control of each child as will be for his best interest, permitting the child to choose which parent he prefers to live with if he is over seven (7) years of age unless the parent so chosen be unfit to take charge of the child by reason of moral depravity, habitual drunkenness or poverty. In all controversies regarding the custody of minors, the sole and foremost consideration is the physical, educational, social and moral welfare of the child concerned, taking into account the respective resources and social and moral situations of the contending parents. However, the law favors the mother if she is a fit and proper person to have custody of her children so that they may not only receive her attention, care, supervision but also have the advantage and benefit of a mothers love and devotion for which there is no substitute. Generally, the love, solicitude and devotion 916

University of the Cordilleras College of Law First Year C S.Y. 2013 - 2014 of a mother cannot be replaced by another and are worth more to a child of tender years than all other things combined. The Civil Code Commission, in recommending the preference for the mother, explained, thus: The general rule is recommended in order to avoid many a tragedy where a mother has seen her baby torn away from her. No man can sound the deep sorrows of a mother who is deprived of her child of tender age. The exception allowed by the rule has to be for "compelling reasons" for the good of the child: those cases must indeed be rare, if the mothers heart is not to be unduly hurt. If she has erred, as in cases of adultery, the penalty of imprisonment and the (relative) divorce decree will ordinarily be sufficient punishment for her. Moreover, her moral dereliction will not have any effect upon the baby who is as yet unable to understand the situation. This preference favoring the mother over the father is even reiterated in Section 6, Rule 99 of the Rules of Court (the Rule on Adoption and Custody of Minors) underscoring its significance, to wit: SEC. 6. Proceedings as to child whose parents are separated. Appeal. When husband and wife are divorced or living separately and apart from each other, and the question as to the care, custody and control of a child or children of their marriage is brought before a Regional Trial Court by petition or as an incident to any other proceeding, the court, upon hearing the testimony as may be pertinent, shall award the care, custody and control of each such child as will be for its best interest, permitting the child to choose which parent it prefers to live with if it be over ten years of age, unless the parent so chosen be unfit to take charge of the child by reason of moral depravity, habitual drunkenness, incapacity, or poverty. If upon such hearing, it appears that both parents are improper persons to have the care, custody, and control of the child, the court may either designate the paternal or maternal grandparent of the child, or his oldest brother or sister, or some reputable and discreet person to take charge of such child, or commit it to any suitable asylum, childrens home, or benevolent society. The court may in conformity with the provisions of the Civil Code order either or both parents to support or help support said child, irrespective of who may be its custodian, and may make any order that is just and reasonable permitting the parent who is deprived of its care and custody to visit the child or have temporary custody thereof. Either parent may appeal from an order made in accordance with the provisions of this section. No child under seven years of age shall be separated from its mother, unless the court finds there are compelling reasons therefor. (Emphasis supplied) The above-quoted provision expressly acknowledges and authorizes that the matter of care and custody of the children may be raised and adjudicated as an incident to any proceeding, such as a case for habeas corpus. Evidently, absent any compelling reason to the contrary, the trial court was correct in restoring the custody of the children to the mother, herein respondent, the children being less than seven years of age, at least at the time the case was decided. Moreover, petitioners contention that respondent is unfit to have custody over the minor children has not been substantiated as found by both courts below. Thus, it is already too late for petitioner to reiterate the assertion for only questions of law may be raised before this Court. Furthermore, the determination of whether the mother is fit or unfit to have custody over the children is a matter well within the sound discretion of the trial court, and unless it is shown that said discretion has been abused the selection will not be interfered with. Consequently, the Court affirms the award of custody in respondents favor. Now, the issue of support. Article 203 of the Family Code states that the obligation to give support is demandable from the time the person who has a right to receive the same needs it for maintenance, but it shall not be paid except from the date of judicial or extrajudicial demand. The case of Jocson v. The Empire Ins. Co. and Jocson Lagniton explains the rationale for this rule: 917

University of the Cordilleras College of Law First Year C S.Y. 2013 - 2014 x x x Support does include what is necessary for the education and clothing of the person entitled thereto (Art. 290, New Civil Code). But support must be demanded and the right to it established before it becomes payable (Art. 298, New Civil Code; Marcelo v. Estacio, 70 Phil. 215). For the right to support does not arise from the mere fact of relationship, even from the relationship of parents and children, but "from imperative necessity without which it cannot be demanded, and the law presumes that such necessity does not exist unless support is demanded (Civil Code of the Philippines, Annotated, Tolentino, Vol. 1, p. 181, citing 8 Manresa 685). In the present case, it does not appear that support for the minors, be it only for their education and clothing, was ever demanded from their father and the need for it duly established. The need for support, as already stated, cannot be presumed, and especially must this be true in the present case where it appears that the minors had means of their own. As intimated earlier, the Court agrees with the courts below that Section 6, Rule 99 of the Rules of Court permits the ventilation of the question regarding the care and custody of the children as an incident to any proceeding, even a habeas corpus proceeding. Petitioner would have us believe, however, that since respondents petition did not include a prayer for support of the children in accordance with the above quoted Family Code provision, the trial court was not justified in awarding support in respondents favor. In addition, petitioner claims that he did not give consent to the trial and the threshing out of the issue as it was not raised in the pleadings. He claims that in fact, he testified on his financial status only to prove that he is financially able to provide for his children and not for the purpose of determining the amount of support. Besides, he contends that the trial court did not order the amendment of the pleadings to conform to the evidence presented pursuant to Section 5 Rule 10 of the 1997 Rules of Civil Procedure, an aspect that supports his contention that the parties never consented, expressly or impliedly, to try the issue of support. The Court is not convinced. Contrary to petitioners assertions, respondent testified during trial, without any objection on petitioners part, regarding the need for support for the childrens education and other necessities, viz: ADDL DIRECT EXAMINATION OF THE WITNESS MERCEDES TAN UY-SY Q: With the kind permission of this Honorable Court. Q: Ms. Sy, the custody of the two minors[,] of course[,] require some expenses on your part notwithstanding that you said you have savings intended for them, is it not? A: Yes, sir. Q: And what is the nature of these expenses that you expect to disburse for the children? A: For the medicine or health care. Q: What else? A: For education, for emergency expenses, for basically for food. Q: In your estimate, how much would these expenses be per month? A: Well, I think, perhaps P50,000.00, sir. Q: Which the respondent should furnish?

918

University of the Cordilleras College of Law First Year C S.Y. 2013 - 2014 A: Yes, sir. ATTY. CORTEZ That is all for the witness, Your Honor. Moreover, based on the transcript of stenographic notes, petitioner was clearly made aware that the issue of support was being deliberated upon, to wit: WITNESS: WILSON SY: will be testifying under the same oath. xxxx ATTY. ALBON: Q: In the hearing of July 23, 1994 as appearing on page 3, Mercedes Sy testified that she would be needing P50,000.00 a month expenses for her children, what can you say about that? A: That is a dillusion [sic] on her part. The trial court judge even propounded questions to petitioner regarding his sources of income for the purpose of determining the amount of support to be given to the children: COURT: I want to find out how much his income now for the purposes of giving support to the children. Please answer the question. WITNESS: A: Shares of stocks. ATTY. CORTEZ: Q: A shares [sic] of stock is the evidence of your investment in the corporation. My question is: What investment did you put in to enable you to get a share, was it money or property? A: There is no money but it was given by my father. COURT: Q: Upon the death of your father you just inherited it? A: Before. Q: After the death, did you not acquire some of the shares of your father? A: No, your Honor. Q: What happened to the shares of your father? A: It is with my mother. 919

University of the Cordilleras College of Law First Year C S.Y. 2013 - 2014 xxxx COURT: Never mind the share of the mother. What is material is his share. ATTY. CORTEZ: Q: How many shares do you have in the corporation? A: Right now I have only ten (10) shares. Q: What is the value of that [sic] shares? A: I [do not] give any importance. COURT Q: For purposes of this case, the Court is asking you how much is your share? A: I [do not ] how to appraise. Q: More or less, how much? Use the word more or less, is that one million more or less, 2 million, more or less, 10 million, more or less? Anyway, this is not a BIR proceeding, this is a Court proceeding? A: I want to speak the truth but I [do not] know. I did not even see the account. COURT: Proceed. ATTY. CORTEZ xxxx Q: At that time of your fathers death[,] you were [sic]already holding ten (10) shares or was it less? A: More. Q: More than ten (10) shares? A: Yes, sir. COURT Q: What is the par value of that one (1) share? A: I [do not] know, your Honor. xxxx COURT:

920

University of the Cordilleras College of Law First Year C S.Y. 2013 - 2014 Let it remain that he owns ten (10) shares. ATTY. CORTEZ: xxxx A: Yes, 10 shares. The other shares I already sold it. Q: How many shares did you sell? A: I only have 10 shares now. I dont know how many shares that I have left. I only know the 20 shares. Applying Section 5, Rule 10 of the 1997 Rules of Civil Procedure, since the issue of support was tried with the implied consent of the parties, it should be treated in all respects as if it had been raised in the pleadings. And since there was implied consent, even if no motion had been filed and no amendment had been ordered, the Court holds that the trial court validly rendered a judgment on the issue. Significantly, in the case of Bank of America v. American Realty Corporation, the Court stated: There have been instances where the Court has held that even without the necessary amendment, the amount proved at the trial may be validly awarded, as in Tuazon v. Bolanos (95 Phil. 106), where we said that if the facts shown entitled plaintiff to relief other than that asked for, no amendment to the complaint was necessary, especially where defendant had himself raised the point on which recovery was based. The appellate court could treat the pleading as amended to conform to the evidence although the pleadings were actually not amended. Amendment is also unnecessary when only clerical error or non substantial matters are involved, as we held in Bank of the Philippine Islands vs. Laguna (48 Phil. 5). In Co Tiamco v. Diaz (75 Phil. 672), we stressed that the rule on amendment need not be applied rigidly, particularly where no surprise or prejudice is caused the objecting party. And in the recent case of National Power Corporation v. Court of Appeals (113 SCRA 556), we held that where there is a variance in the defendants pleadings and the evidence adduced by it at the trial, the Court may treat the pleading as amended to conform with the evidence. The Court likewise affirms the award of P50,000.00 as support for the minor children. As found by both courts, petitioners representations regarding his familys wealth and his capability to provide for his family more than provided a fair indication of his financial standing even though he proved to be less than forthright on the matter. In any event, this award of support is merely provisional as the amount may be modified or altered in accordance with the increased or decreased needs of the needy party and with the means of the giver. WHEREFORE, the Decision dated 29 February 1996 of the Eleventh Division of the Court of Appeals in C.A. G.R. SP No. 38936 and its Resolution dated 15 April 1996 are AFFIRMED. Costs against petitioner. SO ORDERED.

921

University of the Cordilleras College of Law First Year C S.Y. 2013 - 2014 Perez vs. Ca G.R. No. 118870 Full Case March 29, 1996

NERISSA Z. PEREZ, petitioner, vs. THE COURT OF APPEALS (Ninth Division) and RAY C. PEREZ, respondents. ROMERO, J.:p Parties herein would have this Court duplicate the feat of King Solomon who was hailed in Biblical times for his sagacious, if, at times unorthodox, manner of resolving conflicts, the most celebrated case being that when his authority was invoked to determine the identity of the real mother as between two women claiming the same infant. Since there could only be one mother, the daunting task that confronted the king/judge was to choose the true one. In the instant case, we are faced with the challenge of deciding, as between father and mother, who should have rightful custody of a child who bears in his person both their genes. While there is a provision of law squarely in point, the two courts whose authority have been invoked to render a decision have arrived at diametrically opposite conclusions. It has fallen upon us now to likewise act as judge between the trial court, on the one hand, and the appellate, on the other. On the issue of custody over the minor Ray Perez II, respondent Court of Appeals ruled in favor of the boy's father Ray C. Perez, reversing the trial court's decision to grant custody to Nerissa Z. Perez, the child's mother. Ray Perez, private respondent, is a doctor of medicine practicing in Cebu while Nerissa, his wife who is petitioner herein, is a registered nurse. They were married in Cebu on December 6, 1986. After six miscarriages, two operations and a high-risk pregnancy, petitioner finally gave birth to Ray Perez II in New York on July 20, 1992. Petitioner who began working in the United States in October 1988, used part of her earnings to build a modest house in Mandaue City, Cebu. She also sought medical attention for her successive miscarriages in New York. She became a resident alien in February 1992. Private respondent stayed with her in the U.S. twice and took care of her when she became pregnant. Unlike his wife, however, he had only a tourist visa and was not employed. On January 17, 1993, the couple and their baby arrived in Cebu. After a few weeks, only Nerissa returned to the U.S. She alleged that they came home only for a five-week vacation and that they all had round-trip tickets. However, her husband stayed behind to take care of his sick mother and promised to follow her with the baby. According to Ray, they had agreed to reside permanently in the Philippines but once Nerissa was in New York, she changed her mind and continued working. She was supposed to come back immediately after winding up her affairs there. When Nerissa came home a few days, before Ray II's first birthday, the couple was no longer on good terms. That their love for each other was fading became apparent from their serious quarrels. Petitioner did not want to live near her in-laws and rely solely on her husband's meager income of P5,000.00. She longed to be with her only child but he was being kept away from her by her husband. Thus, she did not want to leave RJ (Ray Junior) with her husband and in-laws. She wished for her son to grow up with his mother. 922

University of the Cordilleras College of Law First Year C S.Y. 2013 - 2014 On the other hand, Ray wanted to stay here, where he could raise his son even as he practiced his profession. He maintained that it would not be difficult to live here since they have their own home and a car. They could live comfortably on his P15,000.00 monthly income as they were not burdened with having to pay any debts. Petitioner was forced to move to her parents' home on Guizo Street in Mandaue. Despite mediation by the priest who solemnized their marriage, the couple failed to reconcile. On July 26, 1993, Nerissa Z. Perez filed a petition for habeas corpus asking respondent Ray C. Perez to surrender the custody of their son, Ray Z. Perez II, to her. On August 27, 1993, the court a quo issued an Order awarding custody of the one-year old child to his mother, Nerissa Perez, citing the second paragraph of Article 213 of the Family Code which provides that no child under seven years of age shall be separated from the mother, unless the court finds compelling reasons to order otherwise. The dispositive portion of the Order reads: WHEREFORE, foregoing premises considered, Order is hereby issued ordering the respondent to turn over the custody of their child Ray Cortes Perez II, his passport and round trip ticket to herein petitioner with a warning that if he will escape together with the child for the purpose of hiding the minor child instead of complying with this Order, that warrant for his arrest will be issued. SO ORDERED. Upon appeal by Ray Perez, the Court of Appeals, on September 27, 1994, reversed the trial court's order and awarded custody of the boy to his father. Petitioner's motion for reconsideration having been denied, she filed the instant petition for review where the sole issue is the custody of Ray Perez II, now three years old. Respondent court differed in opinion from the trial court and ruled that there were enough reasons to deny Nerissa Perez custody over Ray II even if the child is under seven years old. It held that granting custody to the boy's father would be for the child's best interest and welfare. Before us is the unedifying situation of a husband and wife in marital discord, struggling for custody of their only child. It is sad that petitioner and private respondent have not found it in their hearts to understand each other and live together once again as a family. Separated in fact, they now seek the Court's assistance in the matter of custody or parental authority over the child. The wisdom and necessity for the exercise of joint parental authority need not be belabored. The father and the mother complement each other in giving nurture and providing that holistic care which takes into account the physical, emotional, psychological, mental, social and spiritual needs of the child. By precept and example, they mold his character during his crucial formative years. However, the Court's intervention is sought in order that a decision may be made as to which parent shall be given custody over the young boy. The Court's duty is to determine whether Ray Perez II will be better off with petitioner or with private respondent. We are not called upon to declare which party committed the greater fault in their domestic quarrel. When the parents of the child are separated, Article 213 of the Family Code is the applicable law. It provides:

923

University of the Cordilleras College of Law First Year C S.Y. 2013 - 2014 Art. 213. In case of separation of the parents, parental authority shall be exercised by the parent designated by the Court. The Court shall take into account all relevant considerations, especially the choice of the child over seven years of age, unless the parent chosen is unfit. No child under seven years of age shall be separated from the mother. unless the court finds compelling reasons to order otherwise. (Emphasis supplied). Since the Code does not qualify the word "separation" to mean legal separation decreed by a court, couples who are separated in fact, such as petitioner and private respondent, are covered within its terms. The Revised Rules of Court also contains a similar provision. Rule 99, section 6 (Adoption and Custody of Minors) provides: Sec. 6. Proceedings as to child whose parents are separated. Appeal. When husband and wife are divorced or living separately, and apart from each other, and the questions as to the care, custody, and control of a child or children of their marriage is brought before a Court of First Instance by petition or as an incident to any other proceeding, the court, upon hearing the testimony as may be pertinent, shall award the care, custody, and control of each such child as will be for its best interest, permitting the child to choose which parent it prefers to live with if it be over ten years of age, unless the parent chosen be unfit to take charge of the child by reason of moral depravity, habitual drunkenness, incapacity, or poverty. . . . No child under seven years of age shall be separated from its mother, unless the court finds there are compelling reasons therefor. (Emphasis supplied) The provisions of law quoted above clearly mandate that a child under seven years of age shall not be separated from his mother unless the court finds compelling reasons to order otherwise. The use of the word "shall" in Article 213 of the Family Code and Rule 99, section 6 of the Revised Rules of Court connotes a mandatory character. In the case of Lacson v. San Jose-Lacson, the Court declared: The use of the word shall in article 363 of the Civil Code, coupled with the observations made by the Code Commission in respect to the said legal provision, underscores its mandatory character. It prohibits in no uncertain terms the separation of a mother and her child below seven years, unless such separation is grounded upon compelling reasons as determined by a court. The rationale for awarding the custody of children younger than seven years of age to their mother was explained by the Code Commission: The general rule is recommended in order to avoid many a tragedy where a mother has seen her baby torn away from her. No man can sound the deep sorrows of a mother who is deprived of her child of tender age. The exception allowed by the rule has to be for "compelling reasons" for the good of the child; those cases must indeed be rare, if the mother's heart is not to be unduly hurt. If she has erred, as in cases of adultery, the penalty of imprisonment and the divorce decree (relative divorce) will ordinarily be sufficient punishment for her. Moreover, moral dereliction will not have any effect upon the baby who is as yet unable to understand her situation. (Report of the Code Commission, p. 12) The Family Code, in reverting to the provision of the Civil Code that a child below seven years old should not be separated from the mother (Article 363), has expressly repealed the earlier Article 17, paragraph three of the Child and Youth Welfare Code (Presidential Decree No. 603) which reduced the child's age to five years. The general rule that a child under seven years of age shall not be separated from his mother finds its raison d'tre in the basic need of a child for his mother's loving care. Only the most compelling of reasons shall justify the court's awarding the custody of such a child to someone other than his mother, such as her unfitness to exercise sole parental authority. In the past the following grounds have been considered ample justification to deprive a mother of custody and parental authority: neglect, abandonment, 924

University of the Cordilleras College of Law First Year C S.Y. 2013 - 2014 unemployment and immorality, habitual drunkenness, drug addiction, maltreatment of the child, insanity and being sick with a communicable disease. It has long been settled that in custody cases, the foremost consideration is always the welfare and best interest of the child. In fact, no less than an international instrument, the Convention on the Rights of the Child provides: "In all actions concerning children, whether undertaken by public or private social welfare institutions, courts of law, administrative authorities or legislative bodies, the best interests of the child shall be a primary consideration." Courts invariably look into all relevant factors presented by the contending parents, such as their material resources, social and moral situations. In the case at bench, financial capacity is not a determinative factor inasmuch as both parties have demonstrated that they have ample means. Respondent court stated that petitioner has no permanent place of work in the U.S.A. and has taken this point against her. The records, however, show that she is employed in a New York hospital and was, at the time the petition was filed, still abroad. She testified that she intends to apply for a job elsewhere, presumably to improve her work environment and augment her income, as well as for convenience. The Court takes judicial notice of the fact that a registered nurse, such as petitioner, is still very much in demand in the United States. Unlike private respondent, a doctor who by his own admission could not find employment there, petitioner immediately got a job in New York. Considering her skill and experience petitioner should find no difficulty in obtaining work elsewhere, should she desire to do so. The decision under review casts doubt on petitioner's capability to take care of the child, particularly since she works on twelve-hour shifts thrice weekly, at times, even at night. There being no one to help her look after the child, it is alleged that she cannot properly attend to him. This conclusion is as unwarranted as it is unreasonable. First, her present work schedule is not so unmanageable as to deprive her of quality time for Ray II. Quite a number of working mothers who are away from home for longer periods of time are still able to raise a family well, applying time management principles judiciously. Second, many a mother, finding herself in such a position, has invited her own mother or relative to join her abroad, providing the latter with plane tickets and liberal allowances, to look after the child until he is able to take care of himself. Others go on leave from work until such time as the child can be entrusted to day-care centers. Delegating child care temporarily to qualified persons who run day-care centers does not detract from being a good mother, as long as the latter exercises supervision, for even in our culture, children are often brought up by housemaids or "yayas" under the eagle eyes of the mother. Third, private respondent's work schedule was not presented in evidence at the trial. Although he is a general practitioner, the records merely show that he maintains a clinic, works for several companies on retainer basis and teaches parttime. Hence, respondent court's conclusion that "his work schedule is flexible (and h)e can always find time for his son" is not well-founded. Fourth, the fact that private respondent lives near his parents and sister is not crucial in this case. Fifth, petitioner's work schedule cited in the respondent court's decision is not necessarily permanent. Hospitals work in shifts and, given a mother's instinctive desire to lavish upon her child the utmost care, petitioner may be expected to arrange her schedule in such a way as to allocate time for him. Finally, it does not follow that petitioner values her career more than her family simply because she wants to work in the United States. There are any number of reasons for a person's seeking a job outside the country, e.g. to augment her income for the family's benefit and welfare, and for psychological fulfillment, to name a few. In the instant case, it has been shown that petitioner earned enough from her job to be able to construct a house for the family in Mandaue City. The record describes sketchily the relations between Ray and Nerissa Perez. The transcripts of the three hearings are inadequate to show that petitioner did not exert earnest efforts and make sacrifices to save her marriage. It is not difficult to imagine how heart-rending it is for a mother whose attempts at having a baby were frustrated several times over a period of six years to finally bear one, only for the infant to be snatched 925

University of the Cordilleras College of Law First Year C S.Y. 2013 - 2014 from her before he has even reached his first year. The mother's role in the life of her child, such as Ray II, is well-nigh irreplaceable. In prose and poetry, the depth of a mother's love has been immortalized times without number, finding as it does, its justification, not in fantasy but in reality. WHEREFORE, the petition for review is GRANTED. The decision of the Court of Appeals dated September 27, 1994 as well as its Resolution dated January 24, 1995 are hereby REVERSED and SET ASIDE. The Order of the trial court dated August 27, 1993 is hereby REINSTATED. Custody over the minor Ray Z. Perez II is awarded to his mother, herein petitioner Nerissa Z. Perez. This decision is immediately executory. SO ORDERED.

926

University of the Cordilleras College of Law First Year C S.Y. 2013 - 2014 David vs. Ca G.R. No. 111180 Full Case November 16, 1995

DAISIE T. DAVID, petitioner, vs. COURT OF APPEALS, RAMON R. VILLAR, respondents. MENDOZA, J.: Petitioner Daisie T. David worked as secretary of private respondent Ramon R. Villar, a businessman in Angeles City. Private respondent is a married man and the father of four children, all grown-up. After a while, the relationship between petitioner and private respondent developed into an intimate one, as a result of which a son, Christopher J., was born on March 9, 1985 to them. Christopher J. was followed by two more children, both girls, namely Christine, born on June 9, 1986, and Cathy Mae on April 24, 1988. The relationship became known to private respondent's wife when Daisie took Christopher J, to Villar's house at Villa Teresa in Angeles City sometime in 1986 and introduced him to Villar's legal wife. After this, the children of Daisie were freely brought by Villar to his house as they were eventually accepted by his legal family. In the summer of 1991, Villar asked Daisie to allow Christopher J., then six years of age, to go with his family to Boracay. Daisie agreed, but after the trip, Villar refused to give back the child. Villar said he had enrolled Christopher J. at the Holy Family Academy for the next school year. On July 30, 1991, Daisie filed a petition for habeas corpus on behalf of Christopher J. After hearing, the Regional Trial Court, Branch 58 at Angeles City, rendered a decision, the dispositive portion of which reads: WHEREFORE, premises considered, judgment is hereby rendered in favor of the petitioner and against the respondent: 1. the rightful custody of the minor Christopher J. T. David is hereby given to the natural mother, the herein petitioner Daisie T. David; 2. respondent is hereby ordered to give a temporary support of P3,000.00 a month to the subject minor Christopher J. T. David, Christine David and Cathy Mae David to take effect upon the finality of this decision; and 3. to pay the costs of this suit. SO ORDERED. On appeal, the Court of Appeals reversed, holding: We agree with the respondent-appellant's view that this is not proper in a habeas corpus case. Law and jurisprudence wherein the question of custody of a minor child may be decided in a habeas corpus case contemplate a situation where the parents are married to each other but are separated. This is so because under the Family Code, the father and mother have joint parental authority over their legitimate children and in case of separation of the parents there is need to determine rightful custody of their children. The same does not hold true in an adulterous relationship, as in the case at bar, the child 927

University of the Cordilleras College of Law First Year C S.Y. 2013 - 2014 born out of such a relationship is under the parental authority of the mother by express provision of the law. Hence, the question of custody and support should be brought in a case singularly filed for the purpose. In point of fact, this is more advisable in the case at bar because the trial court did not acquire jurisdiction over the other minor children of the petitioner-appellee and respondent-appellant and, therefore, cannot properly provide for their support. Admittedly, respondent-appellant is financially well-off, he being a very rich businessman; whereas, petitioner-appellee depends upon her sisters and parents for support. In fact, he financially supported petitioner-appellee and her three minor children. It is, therefore, for the best interest of Christopher J that he should temporarily remain under the custody of respondent-appellant until the issue on custody and support shall have been determined in a proper case. WHEREFORE, the decision appealed from is hereby SET ASIDE, and a NEW ONE ENTERED dismissing the petition for habeas corpus in Special Proceeding No. 4489. Daisie in turn filed this petition for review of the appellate court's decision. Rule 102, 1 of the Rules of Court provides that "the writ of habeas corpus shall extend to all cases of illegal confinement or detention by which any person is deprived of his liberty, or by which the rightful custody of any person is withheld from the person entitled thereto." It is indeed true, as the Court of Appeals observed, that the determination of the right to the custody of minor children is relevant in cases where the parents, who are married to each other, are for some reason separated from each other. It does not follow, however, that it cannot arise in any other situation. For example, in the case of Salvaa v. Gaela, it was held that the writ of habeas corpus is the proper remedy to enable parents to regain the custody of a minor daughter even though the latter be in the custody of a third person of her free will because the parents were compelling her to marry a man against her will. In the case at bar, Christopher J. is an illegitimate child since at the time of his conception, his father, private respondent Ramon R. Villar, was married to another woman other than the child's mother. As such, pursuant to Art. 176 of the Family Code, Christopher J. is under the parental authority of his mother, the herein petitioner, who, as a consequence of such authority, is entitled to have custody of him. Since, admittedly, petitioner has been deprived of her rightful custody of her child by private respondent, she is entitled to issuance of the writ of habeas corpus. Indeed, Rule 1021 1 makes no distinction between the case of a mother who is separated from her husband and is entitled to the custody of her child and that of a mother of an illegitimate child who, by law, is vested with sole parental authority, but is deprived of her rightful custody of her child. The fact that private respondent has recognized the minor child may be a ground for ordering him to give support to the latter, but not for giving him custody of the child. Under Art. 213 of the Family Code, "no child under seven years of age shall be separated from the mother unless the court finds compelling reasons to order otherwise." Nor is the fact that private respondent is well-off a reason for depriving petitioner of the custody of her children, especially considering that she has been able to rear and support them on her own since they were born. Petitioner is a market vendor earning from P2,000 to P3,000 per month in 1993 when the RTC decision was rendered. She augments her income by working as secretary at the Computer System Specialist, Inc. earning a monthly income of P4,500.00. She has an arrangement with her employer so that she can personally attend to her children. She works up to 8:00 o'clock in the evening to make up for time lost during the day. That she receives help from her parents and sister for the support of the three children is not a point against her. Cooperation, compassion, love and concern for every member of the family are characteristics of the close family ties that bind the Filipino family and have made it what it is.

928

University of the Cordilleras College of Law First Year C S.Y. 2013 - 2014 Daisie and her children may not be enjoying a life of affluence that private respondent promises if the child lives with him. It is enough, however, that petitioner is earning a decent living and is able to support her children according to her means. The Regional Trial Court ordered private respondent to give temporary support to petitioner in the amount of P3,000.00 a month, pending the filing of an action for support, after finding that private respondent did not give any support to his three children by Daisie, except the meager amount of P500.00 a week which he stopped giving them on June 23, 1992. He is a rich man who professes love for his children. In fact he filed a motion for the execution of the decision of the Court of Appeals, alleging that he had observed his son "to be physically weak and pale because of malnutrition and deprivation of the luxury and amenities he was accustomed to when in the former custody of the respondent." He prayed that he be given the custody of the child so that he can provide him with the "proper care and education." Although the question of support is proper in a proceeding for that purpose, the grant of support in this case is justified by the fact that private respondent has expressed willingness to support the minor child. The order for payment of allowance need not be conditioned on the grant to him of custody of the child. Under Art. 204 of the Family Code, a person obliged to give support can fulfill his obligation either by paying the allowance fixed by the court or by receiving and maintaining in the family dwelling the person who is entitled to support unless, in the latter case, there is "a moral or legal obstacle thereto." In the case at bar, as has already been pointed out, Christopher J., being less than seven years of age at least at the time the case was decided by the RTC, cannot be taken from the mother's custody. Even now that the child is over seven years of age, the mother's custody over him will have to be upheld because the child categorically expressed preference to live with his mother. Under Art. 213 of the Family Code, courts must respect the "choice of the child over seven years of age, unless the parent chosen is unfit" and here it has not been shown that the mother is in any way unfit to have custody of her child. Indeed, if private respondent loves his child, he should not condition the grant of support for him on the award of his custody to him (private respondent). WHEREFORE, the decision of the Court of Appeals is REVERSED and private respondent is ORDERED to deliver the minor Christopher J. T. David to the custody of his mother, the herein petitioner, and to give him temporary support in the amount of P3,000.00, pending the fixing of the amount of support in an appropriate action. SO ORDERED.

929

University of the Cordilleras College of Law First Year C S.Y. 2013 - 2014

XVI. Summary of Judicial Proceeding in the Family Law

930

University of the Cordilleras College of Law First Year C S.Y. 2013 - 2014 REPUBLIC OF THE PHILIPPINES vs COURT OF APPEALS and MAXIMO WONG SECOND DIVISION G.R. No. 97906 May 21, 1992 Full Case REPUBLIC OF THE PHILIPPINES, petitioner, vs. COURT OF APPEALS and MAXIMO WONG, respondents. Public Attorney's Office for private respondent. REGALADO, J.: Petitioner seeks to set aside the judgment of respondent Court of Appeals in affirmance of the decision of the court a quo granting the petition filed by herein private respondent Maximo Wong for the change of his name to Maximo Alcala, Jr. which was his name prior to his adoption by Hoong Wong and Concepcion Ty Wong. The facts are undisputed. Private respondent Maximo Wong is the legitimate son of Maximo Alcala, Sr. and Segundina Y. Alcala. When he was but two and a half years old and then known as Maximo Alcala, Jr., and his sister Margaret Alcala, was then nine years old, they were, with the consent of their natural parents and by order of the court in Special Case No. 593 issued on September 9, 1967, adopted by spouses Hoong Wong and Concepcion Ty Wong, both naturalized Filipinos. Hoong Wong, now deceased, was an insurance agent while Concepcion Ty Wong was a high school teacher. They decided to adopt the children as they remained childless after fifteen years of marriage. The couples showered their adopted children with parental love and reared them as their own children. Upon reaching the age of twenty-two, herein private respondent, by then married and a junior Engineering student at Notre Dame University, Cotabato City, filed a petition to change his name to Maximo Alcala, Jr. It was averred that his use of the surname Wong embarrassed and isolated him from his relatives and friends, as the same suggests a Chinese ancestry when in truth and in fact he is a Muslim Filipino residing in a Muslim community, and he wants to erase any implication whatsoever of alien nationality; that he is being ridiculed for carrying a Chinese surname, thus hampering his business and social life; and that his adoptive mother does not oppose his desire to revert to his former surname. As earlier stated, on July 2, 1986, the matter was resolved in favor of private respondent, the trial court decreeing that, the jurisdictional requirements having been fully complied with, petitioner's prayer to change his name from Maximo Wong to Maximo Alcala, Jr. was granted. On appeal to respondent court, and over the opposition of petitioner Republic through the Solicitor General, the decision of the court below was affirmed in full, hence, this petition for review on certiorari. The lone issue to be settled is whether or not the reasons given by private respondent in his petition for change of name are valid, sufficient and proper to warrant the granting of said petition. The Solicitor General contends that private respondent's allegations of ridicule and/or isolation from family and friends were unsubstantiated and cannot justify the petition for change of name. He claims that for private respondent to cast aside the name of his adoptive father is crass ingratitude to the memory of the latter and to his adoptive mother who is still alive, despite her consent to the petition for change of name. Further, the Solicitor General posits that the reversion of Maximo Wong to his old name violates Articles 341 and 365 of the Civil Code, which requires an adopted child to use the surname of the adopter, and would identify him with his parents by nature, thus giving the impression that he has severed his relationship with his adoptive parents. In refutation, private respondent argues that he did as the law required, that is, upon adoption he used the surname of the adopter. However, being already emancipated, he can now decide what is best for and by himself. It is at this time that he realized that the Chinese name he carries causes him undue ridicule and embarrassment and affects his business and social life. In fact, his adoptive mother, being aware of his predicament, gave her consent to the petition for change of name, albeit making it clear that the same shall in no way affect the legal adoption, and even underwent the rigors of trial to substantiate her sworn statement. If his adoptive mother does not take offense nor feel any resentment, abhorrence or insecurity about his desire to change his name, private respondent avers that there can be no possible prejudice on her, much less the State. 931

University of the Cordilleras College of Law First Year C S.Y. 2013 - 2014 We feel that we should preface our review of this case with a clear comprehension of the legal significance of a person's name. For all practical and legal purposes, a man's name is the designation by which he is known and called in the community in which be lives and is best known. It is defined as the word or combination of words by which a person is distinguished from other individuals and, also, as the label or appellation which he bears for the convenience of the world at large addressing him, of in speaking of or dealing with him. Names are used merely as one method of indicating the identity of persons; they are descriptive of persons for identification, since, the identity is the essential thing and it has frequently been held that, when identity is certain, a variance in, or misspelling of, the name is immaterial. The names of individuals usually have two parts: the given name or proper name, and the surname or family name. The given or proper name is that which is given to the individual at birth or baptism, to distinguish him from other individuals. The name or family name is that which identifies the family to which he belongs and is continued from parent to child. The given name may be freely selected by the parents for the child; but the surname to which the child is entitled is fixed by law. A name is said to have the following characteristics: (1) It is absolute, intended to protect the individual from being confused with others. (2) It is obligatory in certain respects, for nobody can be without a name. (3) It is fixed, unchangeable, or immutable, at least at the start, and may be changed only for good cause and by judicial proceedings. (4) It is outside the commerce of man, and, therefore, inalienable and intransmissible by act inter vivos or mortis causa. (5) It is imprescriptible. Title XIII, Book I of the Civil Code, in Articles 364 to 380, provides the substantive rules which regulate the use of surnames. Considering the subject and personalities involved in this present review, particular attention must be called to Article 365 which mandates that "(a)n adopted child shall bear the surname of the adopter," in correlation with Article 341 on the effects of adoption, among which is to"(e)ntitle the adopted person to use the adopter's surname." This same entitlement of an adopted child is maintained in Article 39(3), Title II of Presidential Decree No. 603, otherwise known as the Child and Youth Welfare Code. More recently, Executive Order No. 209, as amended by Executive Order No. 227, or the Family Code, echoes the same statutory right of an adopted child to use the surname of the adopter. Clearly, from the very wordings of the law, it may be inferred that this use of the surname of the adopter by the adopted child is both an obligation and a right. Under Article 376 by the Civil Code, "(n)o person can change his name or surname without judicial authority." The application for change of name thereunder involves a special proceeding governed by and conducted under the strictures of Rule 103 of the Rules of Court and one which involves substantial changes, with the declared objective of such judicial proceedings being the prevention of fraud. The purpose of the statutory procedure authorizing a change of personal name is simply to have, wherever possible, a record of the change, and in keeping with the object of the statute, court to which application is made should normally make its decree recording such change of name. A change of name is a special proceeding to establish the status of a person involving his relation with others, that is, his legal position in, or with regard to, the rest of the community. It is a proceeding in rem and, as such, strict compliance with all jurisdictional requirements, particularly on publication, is essential in order to vest the court with jurisdiction thereover. For this purpose, the only name that may be changed is the true or official name recorded in the civil register. To digress a little for purposes of clarification, the change of name contemplated under Article 376 and reglementarily implemented by Rule 103 must not be confused with and cannot be effected through the summary proceeding proposed in Article 412 of the some Code, as procedurally regulated by Rule 108 of the Rules, which refers only to correction of clerical errors, such as those which are visible to the eye or obvious to the understanding, or an error made by a clerk or transcriber, or a mistake in copying or writing, or some harmless or innocuous changeand not those which will involve substantial changes. Turning now to the case at bar, we are guided by the jurisprudential dictum that the State has an interest in the names borne by individuals and entities for the purpose of identification, and a change of name is not a matter of right but of sound judicial discretion, to be exercised in the light of reasons adduced and the consequences that will likely follow; it is a privilege which may be granted only upon a showing of a proper or reasonable cause or compelling reason therefor.

932

University of the Cordilleras College of Law First Year C S.Y. 2013 - 2014 We find unacceptable the assertion of the Solicitor General that private respondent's allegation of ridicule and embarrassment due to the use of his present surname is unsubstantiated. The testimony of private respondent in the lower court bears out the existence of valid cause in his bid for change of name: ATTY. DUMAMBA: Q Now, after adoption, when you went to school, what did you use as your surname? A "Wong," sir. Q Now, after you adopted the surname "Wong?" in your studies, what did you observe? A I observed that "Wong" as a surname embarrassed me to my friends and when I go with Chinese friends I cannot talk Chinese. I am living in Campo Muslim, a Muslim community but no one can believe that I am Muslim. I have a little business of Furniture but I have little (sic) customer because no one believes me that I am Muslim. Q You want to inform this Honorable Court that this family name you are using which is "Wong" embarrassed you from (sic) your friends and relatives and also cause(d) damage to your business? A Yes sir. xxx xxx xxx ATTY. DUMAMBA: Q Now, considering that according to you, you are embarrassed because of the family name you are using, your friends shy away from you and it is a handicap in your business, what is your desire for the Court to do in order to help you? A Change my family name. Q From "Wong" to what do you want your surname changed? A "Alcala, Jr.", sir. xxx xxx xxx COURT: Q What is your purpose in changing your family name from Maximo Wong to Maximo Alcala, Jr.? A I feel embarrassed to my friends and also to my relatives and as I said I have a little business of furniture and only a few customers buying for the fact that they don't believe I am Muslim. Cross. ATTY. SERO: With the permission of the Honorable Court. Q Your father's name is Maximo Alcala, Sr., is he still alive? A Yes, sir. Q And what does your father say to this proposed changed (sic) of your name, your family name to your real family name given to you? A Yes, sir. Q They have no objection to it? A No, sir. Q Stated before this Honorable Court, the purpose why you wanted to change your name from "Wong" to "Alcala" is so that to avoid embarrassment because you are a Muslim and your Muslim relatives think that you are Chinese. A Yes, sir. Q Not for the purpose to hide anything or what not? A No, sir. The foregoing testimony of private respondent is materially corroborated by the testimony of private respondent's adoptive mother: Q Now, what did you observe to (sic) your son Maximo Wong after you and your husband adopted him? A When I adopted him and he used the surname "Wong" I observed that some of his relatives, cousins and friends seem to shy away from him and despise him in school that is why I agreed to change his name. We uphold these observations in the decision of respondent appellate court: The purpose of the law an allowing of change of name as contemplated by the provisions of Rule 103 of the Rules of Court is to give a person an opportunity to improve his personality and to provide his best interest. (Calderon vs. Republic, 19 SCRA 721). In granting or denying the petition for change of name, the question of proper and reasonable cause is left to the discretion of the court. The evidence presented need only be satisfactory to the court and not all the best 933

University of the Cordilleras College of Law First Year C S.Y. 2013 - 2014 evidence available is required. (Uy vs. Republic, L-22712, Nov. 25, 1965; Nacionales vs. Republic, L-18067, April 29, 1966; both cases cited in 1 SCRA 843). In the present case, We believe that the court a quo had exercised its discretion judiciously when it granted the petition. From the testimony of petitioner-appellee and of his adopter mother Concepcion Ty-Wong, We discern that said appellee was prompted to file the petition for change of name because of the embarrassment and ridicule his family name "Wong" brings in his dealings with his relatives and friends, he being a Muslim Filipino and living in a Muslim community. Another cause is his desire to improve his social and business life. It has been held that in the absence of prejudice to the state or any individual, a sincere desire to adopt a Filipino name to erase signs of a former alien nationality which only hamper(s) social and business life, is a proper and reasonable cause for change of name (Uy vs. Republic, L-22712, Nov. 25, 1965, Que Liong Sian vs. Republic, L-23167, Aug. 17, 1967, 20 SCRA 1074). Justice dictates that a person should be allowed to improve his social standing as long as in doing so, he does not cause prejudice or injury to the interest of the State or other persons (Calderon vs. Republic, supra). Nothing whatsoever is shown in the record of this case that such prejudice or injury to the interest of the state or of other persons would result in the change of petitioner's name. It bears stressing at this point that to justify a request for change of name, petitioner must show not only some proper or compelling reason therefor but also that he will be prejudiced by the use of his true and official name. Among the grounds for change of name which have been held valid are: (a) When the name is ridiculous, dishonorable or extremely difficult to write or pronounce; (b) When the change results as a legal consequence, as in legitimation; (c) When the change will avoid confusion; (d) Having continuously used and been known since childhood by a Filipino name, unaware of her alien parentage; (e) A sincere desire to adopt a Filipino name to erase signs of former alienage, all in good faith and without prejudicing anybody; and (f) When the surname causes embarrassment and there is no showing that the desired change of name was for a fraudulent purpose or that the change of name would prejudice public interest. In granting or denying petitions for change of name, the question of proper and reasonable cause is left to the sound discretion of the court. The evidence presented need only be satisfactory to the court and not all the best evidence available. Summarizing, in special proceedings for change of name, what is involved is not a mere matter of allowance or disallowance of the request, but a judicious evaluation of the sufficiency and propriety of the justifications advanced in support thereof, mindful of the consequent results in the event of its grant and with the sole prerogative for making such determination being lodged in the courts. While it is true that the statutory fiat under Article 365 of the Civil Code is to the effect that an adopted child shall bear the surname of the adopter, it must nevertheless be borne in mind that the change of the surname of the adopted child is more an incident rather than the object of adoption proceedings. The act of adoption fixes a status, viz., that of parent and child. More technically, it is an act by which relations of paternity and affiliation are recognized as legally existing between persons not so related by nature. It has been defined as the taking into one's family of the child of another as son or daughter and heir and conferring on it a title to the rights and privileges of such. The purpose of an adoption proceeding is to effect this new status of relationship between the child and its adoptive parents, the change of name which frequently accompanies adoption being more an incident than the object of the proceeding. The welfare of the child is the primary consideration in the determination of an application for adoption. On this point, there is unanimous agreement. It is the usual effect of a decree of adoption to transfer from the natural parents to the adoptive parents the custody of the child's person, the duty of obedience owing by the child, and all other legal consequences and incidents of the natural relation, in the same manner as if the child had been born of such adoptive parents in lawful wedlock, subject, however, to such limitations and restrictions as may be by statute imposed. More specifically under the present state of our law, the Family Code, superseding the pertinent provisions of the Civil Code and of the Child and Youth Welfare Code on the matter, relevantly provides in this wise with regard to the issue involved in this case: Art. 189. Adoption shall have the following effects: (1) For civil purposes, the adopted shall be deemed to be the legitimate child of the adopters and both shall acquire the reciprocal rights and obligations arising from the relationship of parent and child, including the right of the adopted to use the surname of the adopters; (Emphasis supplied.)

934

University of the Cordilleras College of Law First Year C S.Y. 2013 - 2014 The Solicitor General maintains the position that to sustain the change of name would run counter to the behest of Article 365 of the Civil Code and the ruling in Manuel vs. Republic that "one should not be allowed to use a surname which otherwise he is not permitted to employ under the law," and would set a bad example to other persons who might also seek a change of their surnames on lame excuses. While we appreciate the Solicitor General's apprehensions and concern, we find the same to be unfounded. We do not believe that by reverting to his old name, private respondent would then be using a name which he is prohibited by law from using. True, the law prescribes the surname that a person may employ; but the law does not go so far as to unqualifiedly prohibit the use of any other surname, and only subjects such recourse to the obtention of the requisite judicial sanction. What the law does not prohibit, it permits. If we were to follow the argument of the Solicitor General to its conclusion, then there will never be any possibility or occasion for any person, regardless of status, to change his name, in view of the supposed subsequent violation of the legal imperative on the use of surnames in the event that the petition is granted. Rule 103 of the Rules of Court would then be rendered inutile. This could hardly have been the intendment of the law. A petition for change of name is a remedy allowed under our law only by way of exception to the mandatory provisions of the Civil Code on the use of surnames. The law fixes the surname that may be used by a person, at least inceptively, and it may be changed only upon judicial permission granted in the exercise of sound discretion. Section 1 of Rule 103, in specifying the parties who may avail of said remedy, uses the generic term "persons" to signify all natural persons regardless of status. If a legitimate person may, under certain judicially accepted exceptional circumstances, petition the court for a change of name, we do not see any legal basis or logic in discriminating against the availment of such a remedy by an adopted child. In other words, Article 365 is not an exception, much less can it bar resort, to Rule 103. We are of the view that the circumstances herein obtaining are within the ambit of the established exceptions and find merit in private respondent's submission: Rule 103 of the Rules of Court has its primordial purpose which (State) is to give a person in opportunity to improve his personality and provide his best interest (Calderon vs. Republic, 19 SCRA 721). In the instant case, the court a quo found the petition of Maximo Wong for change of name justifiable after due hearing, thus its factual findings and appreciation of testimonies count heavily and need not be disturbed unless for strong and cogent reasons because the trial court is in a better position to examine real evidence as well as to observe the demeanor of the witnesses while testifying in the case (Baliwag Transit, Inc. vs. CA, 147 SCRA 82). Moreover, the trial court could take judicial notice of other existing factors in the community where herein respondent lives which it considers material in its judicious determination of the case. . . . Additionally, herein respondent is already of age and as such he can decide what is best for him. His experience with regards (sic) his social and business dealings is personal and it is only him (sic) who can attest to the same. Finding his predicament's proper remedy is solely through legal process, herein respondent accordingly filed a petition pursuant to Rule 103 of the Rules of Court which was granted by the Court a quo. Besides, we have faith in the circumspection of our lower courts and that, in the exercise of their discretion, said courts shall consider petitions for change of name only on cogent and meritorious grounds as would justify the granting of such applications. We do not expect our trial courts to cater or give in to the whim or caprice of an applicant, aside from the fact that there is always the safeguard and corrective interdiction of appellate review. It is not fair to construe the desired reversion of private respondent to the use of the name of his parents by nature as cross ingratitude. To go by the Solicitor General's suggestion that private respondent should have his adoption revoked if he wants to use the surname of his natural father would be to exact too clear a toll for making use of an appropriate and valid remedy available under the law.

935

University of the Cordilleras College of Law First Year C S.Y. 2013 - 2014 Herein private respondent, before he filed the petition for change of name, asked for his adoptive mother's permission to do so: Q Now, in filing this petition for change of surname, you had talked with your adopted mother? A Yes, sir. Q Did you ask permission from her whether she wants you to change the surname? A Yes, sir. True enough, the above testimony of private respondent was confirmed by his adoptive mother in this manner: Q How are you related to Maximo Wong? A My adopted son. Q He is your adopted son, did your son talk to you when he filed this petition for change of his surname? A Yes, he even tried to ask me and I said, alright if you want to change. xxx xxx xxx Q Now, when you agreed to the filing of this petition for change of name, did you reduce your consent in writing? A Yes, sir, I agreed also so that his business will prosper because he is already Alcila and not Wong because Wong they said is Chinese. As proof of her assent to the filing of said petition (her husband having already passed away), Concepcion Ty Vda. de Wong executed an affidavit in Cotabato City on May 27, 1985, with these textual declarations: That I am the same and identical person, who is the surviving adapted (sic) parent of Maximo Wong. That I personally discovered it myself from the time my adapted (sic) son Maximo used the surname of my late husband Wong, his relatives and childhood friends shy away from him because he is branded as a son of a chinese which is different from them whose parents are muslim Filipinos; That I pity my son who is often rediculed (sic) by his friends and relatives because of his family name Wong, hence, in order not to humper (sic) his social and business life in the future, I am voluntarily and of my own free will without being forced, coerced, or intimidated give (sic) my consent to his desire to change his desire to change his surname without affecting however the legal adoption granted by the Court on September 9, 1967, making him as one of my legal and compulsory heir (sic). That I am executing this affidavit to attest to the truth of all the above mentioned facts and for all legal intent (sic) and purposes. There could be no other plausible reason for private respondent to first secure has adoptive mother's consent before resorting to the questioned legal recourse other than the parental respect and reverence which is owed by and to be expected of a dutiful child. If private respondent was such an ingrate, as the Solicitor General would have us believe, he would not have bothered to seek his adoptive mother's counsel. In the same breath, had his adoptive mother regarded him as an ungrateful adoptee, she would not have executed the affidavit above quoted, much less testify in his behalf at the hearing of his petition. Moreover, worthy of note is the fact that private respondent's adoptive mother emphasized that she executed the above affidavit "without affecting the legal adoption granted by the Court on September 9, 1967, making him as one of my legal and compulsory heir(s)." This is incontrovertible proof that she never entertained any misgivings or reservations with respect to her consent to his petition. This likewise dispels any possible confusion as to private respondent's legal status or adoptive paternity and his successional rights. Concordantly, we have heretofore held that a change of name does not define or effect a change in one's existing family relations or in the rights and duties flowing therefrom. It does not alter one's legal capacity, civil status or citizenship; what is altered is only the name WHEREFORE, the petition is DENIED and the decision of respondent Court of Appeals is hereby AFFIRMED in toto. SO ORDERED. 936

University of the Cordilleras College of Law First Year C S.Y. 2013 - 2014 REPUBLIC OF THE PHILIPPINES, vs HON. JOSE R. HERNANDEZ G.R. No. 117209 February 9, 1996 Full Case REPUBLIC OF THE PHILIPPINES, petitioner, vs. HON. JOSE R. HERNANDEZ, in his capacity as Presiding Judge, Regional Trial Court, Branch 158, Pasig City and SPOUSES VAN MUNSON y NAVARRO and REGINA MUNSON y ANDRADE, respondents. DECISION REGALADO, J.: Indeed, what's in a name, as the Bard of Avon has written, since a rose by any other name would smell as sweet? This could well be the theme of the present appeal by certiorari which challenges, on pure questions of law, the order of the Regional Trial Court, Branch 158, Pasig City, dated September 13, 1994 1 in JDRC Case No. 2964. Said court is faulted for having approved the petition for adoption of Kevin Earl Bartolome Moran and simultaneously granted the prayer therein for the change of the first name of said adoptee to Aaron Joseph, to complement the surname Munson y Andrade which he acquired consequent to his adoption. The facts are undisputed. On March 10, 1994, herein private respondent spouses, Van Munson y Navarro and Regina Munson y Andrade, filed a p petition 2 to adopt the minor Kevin Earl Bartolome Moran, duly alleging therein the jurisdictional facts required by Rule 99 of the Rules of Court for adoption, their qualifications as and fitness to be adoptive parents, as well as the circumstances under and by reason of which the adoption of the aforenamed minor was sought. In the very same petition, private respondents prayed for the change of the first name or said minor adoptee to Aaron Joseph, the same being the name with which he was baptized in keeping with religious tradition and by which he has been called by his adoptive family, relatives and friends since May 6, 1993 when he arrived at private respondents' residence. 3 At the hearing on April 18, 1994, petitioner opposed the inclusion of the relief for change of name in the same petition for adoption. In its formal opposition dated May 3, 1995, 4 petitioner reiterated its objection to the joinder of the petition for adoption and the petitions for change of name in a single proceeding, arguing that these petition should be conducted and pursued as two separate proceedings. After considering the evidence and arguments of the contending parties, the trial court ruled in favor of herein private respondents in this wise: WHEREFORE, minor child Kevin Earl Bartolome Moran is freed from all legal obligations of obedience and maintenance with respect to his natural parents, and for all legal intents and purposes shall be known as Aaron Joseph Munson y Andrade, the legally adopted child of Van Munson and Regina Munson effective upon the filing of the petition on March 10, 1994. As soon as the decree of adoption becomes final and executory, it shall be recorded in the Office of the Local Civil Registrar of Pasig, Metro Manila pursuant to Section 8, Rule 99 and Section 6, Rule 103, respectively, of the Rules of Court, and shall be annotated in the record of birth of the adopted child, which in this case is in Valenzuela, Metro Manila, where the child was born. Likewise, send a copy of this Order to the National Census and Statistics Office, Manila, for its appropriate action consisten(t) herewith. 5 At this juncture, it should be noted that no challenge has been raised by petitioner regarding the fitness of herein private respondents to be adopting parents nor the validity of the decree of adoption rendered in their favor. The records show that the latter have commendably established their qualifications under the

937

University of the Cordilleras College of Law First Year C S.Y. 2013 - 2014 law to be adopters, 6 and have amply complied with the procedural requirements for the petition for adoption, 7 with the findings of the trial court being recited thus: To comply with the jurisdictional requirements, the Order of this Court dated March 16, 1994 setting this petition for hearing (Exh. "A") was published in the March 31, April 6 and 13, 1994 issues of the Manila Chronicle, a newspaper of general circulation (Exhs. "B" to "E" and submarkings). . . . xxx xxx xxx

Petitioners apart from being financially able, have no criminal nor derogatory record (Exhs. "K" to "V"); and are physically fit to be the adoptive parents of the minor child Kevin (Exh. "W"). Their qualification to become the adoptive parents of Kevin Earl finds support also in the Social Case Study Report prepared by the DSWD through Social Worker Luz Angela Sonido, the pertinent portion of which reads: "Mr. and Mrs. Munson are very religious, responsible, mature and friendly individuals. They are found physically healthy; mentally fit, spiritually and financially capable to adopt Kevin Earl Moran aka Aaron Joseph. "Mr. and Mrs. Munson have provided AJ with all his needs. They unselfishly share their time, love and attention to him. They are ready and willing to continuously provide him a happy and secure home life. "Aaron Joseph, on the other hand, is growing normally under the care of the Munsons. He had comfortably settled in his new environment. His stay with the Munsons during the six months trial custody period has resulted to a close bond with Mr. and Mrs. Munson and vice-versa. "We highly recommend to the Honorable Court that the adoption of Kevin Earl Moran aka Aaron Joseph by Mr. and Mrs. Van Munson be legalized." 8 It has been said all too often enough that the factual findings of the lower court, when sufficiently buttressed by legal and evidential support, are accorded high respect and are binding and conclusive upon this Court. 9Accordingly, we fully uphold the propriety of that portion of the order of the court below granting the petition, for adoption. The only legal issues that need to be resolved may then be synthesized mainly as follows. (1) whether or not the court a quo erred in granting the prayer for the change of the registered proper or given name of the minor adoptee embodied in the petition for adoption; and (2) whether or not there was lawful ground for the change of name. I. It is the position of petitioner that respondent judge exceeded his jurisdiction when he additionally granted the prayer for the change of the given or proper name of the adoptee in a petition for adoption. Petitioner argues that a petition for adoption and a petition for change of name are two special proceedings which, in substance and purpose, are different from and are not related to each other, being respectively governed by distinct sets of law and rules. In order to be entitled to both reliefs, namely, a decree of adoption and an authority to change the giver or proper name of the adoptee, the respective proceedings for each must be instituted separately, and the substantive and procedural requirements therefor under Articles 183 to 193 of the Family Code in relation to Rule 99 of the Rules of Court for adoption, and Articles 364 to 380 of the Civil Code in relation to Rule 103 of the Rules of Court for change of name, must correspondingly be complied with. 10 A perusal of the records, according to petitioner, shows that only the laws and rules on adoption have been observed, but not those for a petition for change of name. 11 Petitioner further contends that what the law allows is the change of the surname of the adoptee, as a matter of right, to conform with that of the adopter and as a natural consequence of the adoption thus granted. If what is sought is the change of the registered given or proper name, and since this would involve a substantial change of one's legal name, a petition for change of name under Rule 103 should accordingly be instituted, with the substantive and adjective requisites therefor being conformably satisfied. 12

938

University of the Cordilleras College of Law First Year C S.Y. 2013 - 2014 Private respondents, on the contrary, admittedly filed the petition for adoption with a prayer for change of name predicated upon Section 5, Rule 2 which allows permissive joinder of causes of action in order to avoid multiplicity of suits and in line with the policy of discouraging protracted and vexatious litigations. It is argued that there is no prohibition in the Rules against the joinder of adoption and change of name being pleaded as two separate but related causes of action in a single petition. Further, the conditions for permissive joinder of causes of action, i.e., jurisdiction of the court, proper venue and joinder of parties, have been met. 13 Corollarily, petitioner insists on strict adherence to the rule regarding change of name in view of the natural interest of the State in maintaining a system of identification of its citizens and in the orderly administration of justice. 14 Private respondents argue otherwise and invoke a liberal construction and application of the Rules, the welfare and interest of the adoptee being the primordial concern that should be addressed in the instant proceeding. 15 On this score, the trial court adopted a liberal stance in holding that Furthermore, the change of name of the child from Kevin Earl Bartolome to Aaron Joseph should not be treated strictly, it appearing that no rights have been prejudiced by said change of name. The strict and meticulous observation of the requisites set forth by Rule 103 of the Rules of Court is indubitably for the purpose of preventing fraud, ensuring that neither State nor any third person should be prejudiced by the grant of the petition for change of name under said rule, to a petitioner of discernment. The first name sought to be changed belongs to an infant barely over a year old. Kevin Earl has not exercised full civil rights nor engaged in any contractual obligations. Neither can he nor petitioners on his behalf, be deemed to have any immoral, criminal or illicit purpose for seeking said cha(n)ge of name. It stands to reason that there is no way that the state or any person may be so prejudiced by the action for change of Kevin Earl's first name. In fact, to obviate any possible doubts on the intent of petitioners, the prayer for change of name was caused to be published together with the petition for adoption. 16 Art. 189 of the Family Code enumerates in no uncertain terms the legal effects of adoption: (1) For civil purposes, the adopted shall be deemed to be a legitimate child of the adopters and both shall acquire the reciprocal rights and obligations arising from the relationship of parent and child, including the right of the adopted to use the surname of the adopters; (2) The parental authority of the parents by nature over the adopted shall terminate and be vested in the adopters, except that if the adopter is the spouse of the parent by nature of the adopted, parental authority over the adopted shall be exercised jointly by both spouses; and (3) The adopted shall remain an intestate heir of his parents and other blood relatives. Clearly, the law allows the adoptee, as a matter of right and obligation, to bear the surname of the adopter, upon issuance of the decree of adoption. It is the change of the adoptee's surname to follow that of the adopter which is the natural and necessary consequence of a grant of adoption and must specifically be contained in the order of the court, in fact, even if not prayed for by petitioner. However, the given or proper name, also known as the first or Christian name, of the adoptee must remain as it was originally registered in the civil register. The creation of an adoptive relationship does not confer upon the adopter a license to change the adoptee's registered Christian or first name. The automatic change thereof, premised solely upon the adoption thus granted, is beyond the purview of a decree of adoption. Neither is it a mere incident in nor an adjunct of an adoption proceeding, such that a prayer therefor furtively inserted in a petition for adoption, as in this case, cannot properly be granted. The name of the adoptee as recorded in the civil register should be used in the adoption proceedings in order to vest the court with jurisdiction to hear and determine the same, 17 and shall continue to be so used until the court orders otherwise. Changing the given or proper name of a person as recorded in the civil register is a substantial change in one's official or legal name and cannot be authorized without a judicial 939

University of the Cordilleras College of Law First Year C S.Y. 2013 - 2014 order. The purpose of the statutory procedure authorizing a change of name is simply to have, wherever possible, a record of the change, and in keeping with the object of the statute, a court to which the application is made should normally make its decree recording such change. 18 The official name of a person whose birth is registered in the civil register is the name appearing therein. If a change in one's name is desired, this can only be done by filing and strictly complying with the substantive and procedural requirements for a special proceeding for change of name under Rule 103 of the Rules of Court, wherein the sufficiency of the reasons or grounds therefor can be threshed out and accordingly determined. Under Rule 103, a petition for change of name shall be filed in the regional trial court of the province where the person desiring to change his name resides. It shall be signed and verified by the person desiring his name to be changed or by some other person in his behalf and shall state that the petitioner has been a bona fide resident of the province where the petition is filed for at least three years prior to such filing, the cause for which the change of name is sought, and the name asked for. An order for the date and place of hearing shall be made and published, with the Solicitor General or the proper provincial or city prosecutor appearing for the Government at such hearing. It is only upon satisfactory proof of the veracity of the allegations in the petition and the reasonableness of the causes for the change of name that the court may adjudge that the name be changed as prayed for in the petition, and shall furnish a copy of said judgment to the civil registrar of the municipality concerned who shall forthwith enter the same in the civil register. A petition for change of name being a proceeding in rem, strict compliance with all the requirements therefor is indispensable in order to vest the court with jurisdiction for its adjudication. 19 It is an independent and discrete special proceeding, in and by itself, governed by its own set of rules. A fortiori, it cannot be granted by means of any other proceeding. To consider it as a mere incident or an offshoot of another special proceeding would be to denigrate its role and significance as the appropriate remedy available under our remedial law system. The Solicitor General correctly points out the glaring defects of the subject petition insofar as it seeks the change of name of the adoptee, 20 all of which taken together cannot but lead to the conclusion that there was no petition sufficient in form and substance for change of name as would rightfully deserve an order therefor. It would be procedurally erroneous to employ a petition for adoption to effect a change of name in the absence of the corresponding petition for the latter relief at law. Neither can the allowance of the subject petition, by any stretch of imagination and liberality, be justified under the rule allowing permissive joinder of causes of action. Moreover, the reliance by private respondents on the pronouncements in Briz vs. Brit, et al. 21 and Peyer vs. Martinez, et al. 22 is misplaced. A restatement of the rule and jurisprudence on joinder of causes of action would, therefore, appear to be called for. By a joinder of actions, or more properly, a joinder of causes of action, is meant the uniting of two or more demands or rights of action in one action; the statement of more than one cause of action in a declaration. 23 It is the union of two or more civil causes of action, each of which could be made the basis of a separate suit, in the same complaint, declaration or petition. A plaintiff may under certain circumstances join several distinct demands, controversies or rights of action in one declaration, complaint or petition. 24 As can easily be inferred from the above definitions, a party is generally not required to join in one suit several distinct causes of action. The joinder of separate causes of action, where allowable, is permissive and not mandatory in the absence of a contrary statutory provision, even though the causes of action arose from the same factual setting and might under applicable joinder rules be joined. 25 Modern statutes and rules governing joinders are intended to avoid a multiplicity of suits and to promote the efficient administration of justice wherever this may be done without prejudice to the rights of the litigants. To achieve these ends, they are liberally construed. 26

940

University of the Cordilleras College of Law First Year C S.Y. 2013 - 2014 While joinder of causes of action is largely left to the option of a party litigant, Section 5, Rule 2 of our present Rules allows causes of action to be joined in one complaint conditioned upon the following requisites: (a) it will not violate the rules on jurisdiction, venue and joinder of parties; and (b) the causes of action arise out of the same contract, transaction or relation between the parties, or are for demands for money or are of the same nature and character. The objectives of the rule or provision are to avoid a multiplicity of suits where the same parties and subject matter are to be dealt with by effecting in one action a complete determination of all matters in controversy and litigation between the parties involving one subject matter, and to expedite the disposition of litigation at minimum cost. The provision should be construed so as to avoid such multiplicity, where possible, without prejudice to the rights of the litigants. Being of a remedial nature, the provision should be liberally construed, to the end that related controversies between the same parties may be adjudicated at one time; and it should be made effectual as far as practicable, 27 with the end in view of promoting the efficient administration of justice. 28 The statutory intent behind the provisions on joinder of causes of action is to encourage joinder of actions which could reasonably be said to involve kindred rights and wrongs, although the courts have not succeeded in giving a standard definition of the terms used or in developing a rule of universal application. The dominant idea is to permit joinder of causes of action, legal or equitable, where there is some substantial unity between them. 29 While the rule allows a plaintiff to join as many separate claims as he may have, there should nevertheless be some unity in the problem presented and a common question of law and fact involved, subject always to the restriction thereon regarding jurisdiction, venue and joinder of parties. Unlimited joinder is not authorized. 30 Our rule on permissive joinder of causes of action, with the proviso subjecting it to the correlative rules on jurisdiction, venue and joinder of parties 31 and requiring a conceptual unity in the problems presented, effectively disallows unlimited joinder. 32 Turning now to the present petition, while it is true that there is no express prohibition against the joinder of a petition for adoption and for change of name, we do not believe that there is any relation between these two petitions, nor are they of the same nature or character, much less do they present any common question of fact or law, which conjointly would warrant their joinder. In short, these petitions do not rightly meet the underlying test of conceptual unity demanded to sanction their joinder under our Rules. As keenly observed and correctly pointed out by the Solicitor General A petition for adoption and a petition for change of name are two special proceedings which, in substance and purpose, are different from each other. Each action is individually governed by particular sets of laws and rules. These two proceedings involve disparate issues. In a petition for adoption, the court is called upon to evaluate the proposed adopter's fitness and qualifications to bring up and educate the adoptee properly (Prasnick vs. Republic, 99 Phil. 665). On the other hand, in a petition for change of name, no family relations are created or affected for what is looked into is the propriety and reasonableness of the grounds supporting the proposed change of name (Yu vs. Republic, 17 SCRA 253). xxx xxx xxx

. . . Hence, the individual merits of each issue must be separately assessed and determined for neither action is dependent on the other. 33 The rule on permissive joinder of: causes of action is clear. Joinder may be allowed only if the actions show a commonality of relationship and conform to the rules on jurisdiction, venue and joinder of parties (Section 5, Rule 2, Rules of Court). These conditions are wanting in the instant case. As already pointed out in our Petition (pp. 9-10), an action for adoption and an action for change of name are, in nature and purpose, not related to each other and do not arise out of the same relation between the parties. While what is cogent in an adoption proceeding is the proposed adopter's fitness and qualifications to adopt, a petition for change of first name 941

University of the Cordilleras College of Law First Year C S.Y. 2013 - 2014 may only prosper upon proof of reasonable and compelling grounds supporting the change requested. Fitness to adopt is not determinative of the sufficiency of reasons justifying a change of name. And similarly, a change of first name cannot be justified in view of a finding that the proposed adopter was found fit to adopt. There is just no way that the two actions can connect and find a common ground, thus the joinder would be improper. In contending that adoption and change of name may be similarly sought in one petition, private respondents rely upon Peyer vs. Martinez and Briz vs. Briz (p. 4, Comment) We however submit that these citations are non sequitur. In both cases, the fact of intimacy and relatedness of the issues is so pronounced. In Peyer, an application to pronounce the husband an absentee is obviously intertwined with the action to transfer the management of conjugal assets to the wife. In Briz, an action for declaration of heirship was deemed a clear condition precedent to an action to recover the land subject of partition and distribution proceeding. However, the commonality of relationship which stands out in both cases does not characterize the present action for adoption and change of name. Thus the rulings in Peyerand Briz find no place in the case at bar. Besides, it is interesting to note that although a joinder of the two actions was, in Briz, declared feasible, the Supreme Court did not indorse an automatic joinder and instead remanded the matter for further proceedings, granting leave to amend the pleadings and implead additional parties-defendants for a complete determination of the controversy (Briz vs. Briz, 43 Phil. 763, 770). Such cautionary stance all the more emphasizes that although joinders are generally accepted, they are not allowed where the conditions are not satisfactorily met. 34 It furthermore cannot be said that the proposed joinder in this instance will make for a complete determination of all matters pertaining to the coetaneous grant of adoption and change of name of the adoptee in one petition. As already stated, the subject petition was grossly insufficient in form and substance with respect to the prayer for change of name of the adoptee. The policy of avoiding multiplicity of suits which underscores the rule on permissive joinder of causes of action is addressed to suits that are intimately related and also present interwoven and dependent issues which can be most expeditiously and comprehensively settled by having just one judicial proceeding, but not to suits or actions whose subject matters or corresponding reliefs are unrelated or diverse such that they are best taken up individually. In Nabus vs. Court of Appeals, et al., 35 the Court clarified the rule on permissive joinder of causes of action: The rule is clearly permissive. It does not constitute an obligatory rule, as there is no positive provision of law or any rule of jurisprudence which compels a party to join all his causes of action and bring them at one and the same time. Under the present rules, the provision is still that the plaintiff may, and not that he must, unite several causes of action although they may be included in one of the classes specified. This, therefore, leaves it to the plaintiff's option whether the causes of action shall be joined in the same action, and no unfavorable inference may be drawn from his failure or refusal to do so. He may always file another action based on the remaining cause or causes of action within the prescriptive period therefor. (Emphasis supplied.) The situation presented in this case does not warrant exception from the Rules under the policy of liberal construction thereof in general, and for change of name in particular, as proposed by private respondents and adopted by respondent judge. Liberal construction of the Rules may be invoked in situations wherein there may be some excusable formal deficiency or error in a pleading, provided that the same does not subvert the essence of the proceeding and connotes at least a reasonable attempt at compliance with the Rules. Utter disregard of the Rules cannot justly be rationalized by harking on the policy of liberal construction. The Court is not impervious to the frustration that litigants and lawyers alike would at times encounter in procedural bureaucracy but imperative justice requires correct observance of indispensable technicalities 942

University of the Cordilleras College of Law First Year C S.Y. 2013 - 2014 precisely designed to ensure its proper dispensation. 36 It has long been recognized that strict compliance with the Rules of Court is indispensable for the prevention of needless delays and for the orderly and expeditious dispatch of judicial business. 37 Procedural rules are not to be disdained as mere technicalities that may be ignored at will to suit the convenience of a party. Adjective law is important in ensuring the effective enforcement of substantive rights through the orderly and speedy administration of justice. These rules are not intended to hamper litigants or complicate litigation but, indeed to provide for a system under which a suitor may be heard in the correct form and manner and at the prescribed time in a peaceful confrontation before a judge whose authority they acknowledge. 38 It cannot be overemphasized that procedural rules have their own wholesome rationale in the orderly administration of justice. Justice has to be administered according to the Rules in order to obviate arbitrariness, caprice, or whimsicality. 39 We have been cautioned and reminded in Limpot vs. CA, et al. that: 40 Rules of procedure are intended to ensure the orderly administration of justice and the protection of substantive rights in judicial and extrajudicial proceedings. It is a mistake to propose that substantive law and adjective law are contradictory to each other or, as has often been suggested, that enforcement of procedural rules should never be permitted if it will result in prejudice to the substantive rights of the litigants. This is not exactly true; the concept is much misunderstood. As a matter of fact, the policy of the courts is to give both kinds of law, as complementing each other, in the just and speedy resolution of the dispute between the parties. Observance of both substantive rights is equally guaranteed by due process, whatever the source of such rights, be it the Constitution itself or only a statute or a rule of court. xxx xxx xxx

. . . (T)hey are required to be followed except only when for the most persuasive of reasons they may be relaxed to relieve a litigant of an injustice not commensurate with the degree of his thoughtlessness in not complying with the procedure prescribed. . . . While it is true that a litigation is not a game of technicalities, this does not mean that the Rules of Court may be ignored at will and at random to the prejudice of the orderly presentation and assessment of the issues and their just resolution. Justice eschews anarchy. Only exceptionally in very extreme circumstances, when a rule deserts its proper office as an aid to justice and becomes its great hindrance and chief enemy such that rigid application thereof frustrates rather than promotes substantial justice, will technicalities deserve scant consideration from the court. In such situations, the courts are empowered, even obligated, to suspend the operation of the rules. 41 We do not perceive any injustice that can possibly be visited upon private respondents by following the reglementary procedure for the change in the proper or given name that they seek for their adopted child. We are hard put to descry the indispensability of a change of the first name of the adoptee to his welfare and benefit. Nor is the said change of such urgency that would justify an exemption from or a relaxation of the Rules. It is the State that stands to be prejudiced by a wanton disregard of Rule 103 in this case, considering its natural interest in the methodical administration of justice and in the efficacious maintenance of a system of identification of its citizens. The danger wrought by non-observance of the Rules is that the violation of or failure to comply with the procedure prescribed by law prevents the proper determination of the questions raised by the parties with respect to the merits of the case and makes it necessary to decide, in the first place, such questions as relate to the form of the action. The rules and procedure laid down for the trial court and the adjudication of cases are matters of public policy. 42 They are matters of public order and interest which can in no wise be changed or regulated by agreements between or stipulations by parties to an action for their singular convenience. 43 In Garcia vs. Republic, 44 we are reminded of the definiteness in the application of the Rules and the importance of seeking relief under the appropriate proceeding: 943

University of the Cordilleras College of Law First Year C S.Y. 2013 - 2014 . . . The procedure set by law should be delimited. One should not confuse or misapply one procedure for another lest we create confusion in the application of the proper remedy. Respondent judge's unmindful disregard of procedural tenets aimed at achieving stability of procedure is to be deplored. He exceeded his prerogatives by granting the prayer for change of name, his order being unsupported by both statutory and case law. The novel but unwarranted manner in which he adjudicated this case may be characterized as a regrettable abdication of the duty to uphold the teachings of remedial law and jurisprudence. II. Petitioner avers that it was error for the lower court to grant the petition for change of name without citing or proving any lawful ground. Indeed, the only justification advanced for the change of name was the fact of the adoptee's baptism under the name Aaron Joseph and by which he has been known since he came to live with private respondents. 45 Private respondents, through a rather stilted ratiocination, assert that upon the grant of adoption, the subject minor adoptee ipso facto assumed a new identification and designation, that is, Aaron Joseph which was the name given to him during the baptismal rites. Allowing the change of his first name as prayed for in the petition, so they claim, merely confirms the designation by which he is known and called in the community in which he lives. This largely echoes the opinion of the lower court that naming the child Aaron Joseph was symbolic of naming him at birth, and that they, as adoptive parents, have as much right as the natural parents to freely select the first name of their adopted child. 46 The lower court was sympathetic to herein private respondents and ruled on this point in this manner: As adoptive parents, petitioner like other parents may freely select the first name given to his/her child as it is only the surname to which the child is entitled that is fixed by law. . . . xxx xxx xxx

The given name of the minor was Kevin Earl, a name given for no other purpose than for identification purposes in a birth certificate by a woman who had all intentions of giving him away. The naming of the minor as Aaron Joseph by petitioners upon the grant of their petition for adoption is symbolic of naming the minor at birth. 47 We cannot fathom any legal or jurisprudential basis for this attenuated ruling of respondent judge and must thus set it aside. It is necessary to reiterate in this discussion that a person's name is a word or combination of words by which he is known and identified, and distinguished from others, for the convenience of the world at large in addressing him, or in speaking of or dealing with him. It is both of personal as well as public interest that every person must have a name. The name of an individual has two parts: the given or proper name and the surname or family name. The giver or proper name is that which is given to the individual at birth or at baptism, to distinguish him from other individuals. The surname or family name is that which identifies the family to which he belongs and is continued from parent to child. The given name may be freely selected by the parents for the child, but the surname to which the child is entitled is fixed by law. 48 By Article 408 of the Civil Code, a person's birth must be entered in the civil register. The official name of a person is that given him in the civil register. That is his name in the eyes of the law. 49 And once the name of a person is officially entered in the civil register, Article 376 of the same Code seals that identity with its precise mandate: no person can change his name or surname without judicial authority. This statutory restriction is premised on the interest of the State in names borne by individuals and entities for purposes of identification. 50 By reason thereof, the only way that the name of person can be changed legally is through a petition for change of name under Rule 103 of the Rules of Court. 51 For purposes of an application for change of name under Article 376 of the Civil Code and correlatively implemented by Rule 103, the only name that 944

University of the Cordilleras College of Law First Year C S.Y. 2013 - 2014 may be changed is the true or official name recorded in the civil register. As earlier mentioned, a petition for change of name being a proceeding in rem, impressed as it is with public interest, strict compliance with all the requisites therefor in order to vest the court with jurisdiction is essential, and failure therein renders the proceedings a nullity. 52 It must likewise be stressed once again that a change of name is a privilege, not a matter of right, addressed to the sound discretion of the court which has the duty to consider carefully the consequences of a change of name and to deny the same unless weighty reasons are shown. Before a person can be authorized to change his name, that is, his true or official name or that which appears in his birth certificate or is entered in the civil register, he must show proper and reasonable cause or any convincing reason which may justify such change. 53 Jurisprudence has recognized, inter alia, the following grounds as being sufficient to warrant a change of name: (a) when the name is ridiculous, dishonorable or extremely difficult to write or pronounce; (b) when the change results as a legal consequence of legitimation or adoption; (c) when the change will avoid confusion; (d) when one has continuously used and been known since childhood by a Filipino name and was unaware of alien parentage; (e) when the change is based on a sincere desire to adopt a Filipino name to erase signs of former alienage, all in good faith and without prejudice to anybody; and (f) when the surname causes embarrassment and there is no showing that the desired change of name was for a fraudulent purpose or that the change of name would prejudice public interest. 54 Contrarily, a petition for change of name grounded on the fact that one was baptized by another name, under which he has been known and which he used, has been denied inasmuch as the use of baptismal names is not sanctioned. 55 For, in truth, baptism is not a condition sine qua non to a change of name. 56 Neither does the fact that the petitioner has been using a different name and has become known by it constitute proper and reasonable cause to legally authorize a change of name. 57 A name given to a person in the church records or elsewhere or by which be is known in the community - when at variance with that entered in the civil register - is unofficial and cannot be recognized as his real name. 58 The instant petition does not sufficiently persuade us to depart from such rulings of long accepted wisdom and applicability. The only grounds offered to justify the change of name prayed for was that the adopted child had been baptized as Aaron Joseph in keeping with the religious faith of private respondents and that it was the name by which he had been called and known by his family, relatives and friends from, the time he came to live with private respondents. 59 Apart from suffusing their pleadings with sanctimonious entreaties for compassion, none of the justified grounds for a change of name has been alleged or established by private respondents. The legal bases chosen by them to bolster their cause have long been struck down as unavailing for their present purposes. For, to allow the adoptee herein to use his baptismal name, instead of his name registered in the civil register, would be to countenance or permit that which has always been frowned upon. 60 The earlier quoted posturing of respondent judge, as expressed in his assailed order that (a)s adoptive parents, petitioners like other parents may freely select the first name given to his/her child as it is only the surname to which the child is entitled that is fixed by law. . . . The given name of the minor was Kevin Earl, a name given for no other purpose than for identification purposes in a birth certificate by a woman who had all the intentions of giving him away. The naming of the minor as Aaron Joseph by petitioners upon grant of their petition for adoption is symbolic of naming the minor at birth. and supposedly based on the authority of Republic vs. Court of Appeals and Maximo Wong, supra, painfully misapplies the ruling therein enunciated. The factual backdrop of said case is not at all analogous to that of the case at bar. In the Wong case, therein petitioner Maximo Wong sought the change of his surname which he acquired by virtue of the decree of adoption granted in favor of spouses Hoong Wong and Concepcion Ty Wong. Upon reaching the age of majority, he filed a petition in court to change his surname from Wong to Alcala, which was his 945

University of the Cordilleras College of Law First Year C S.Y. 2013 - 2014 surname prior to the adoption. He adduced proof that the use of the surname Wong caused him embarrassment and isolation from friends and relatives in view of a suggested Chinese ancestry when in reality he is a Muslim Filipino residing in a Muslim community, thereby hampering his business and social life, and that his surviving adoptive mother consented to the change of name sought. This Court granted the petition and regarded the change of the surname as a mere incident in, rather than the object of, the adoption. It should be noted that in said case the change of surname, not the given name, and the legal consequences thereof in view of the adoption were at issue. That it was sought in a petition duly and precisely filed for that purpose with ample proof of the lawful grounds therefor only serves to reinforce the imperative necessity of seeking relief under and through the legally prescribed procedures. Here, the Solicitor General meritoriously explained that: Respondent Judge failed to distinguish between a situation wherein a child is being named for the first time by his natural parent, as against one wherein, a child is previously conferred a first name by his natural parent, and such name is subsequently sought to be disregarded and changed by the adoptive parents. In the first case, there is no dispute that natural parents have the right to freely select and give the child's first name for every person, including juridical persons, must have a name (Tolentino, A., Commentaries and Jurisprudence on the Civil Code, Vo. I, 1987 edition, page 721). In the second case, however, as in the case at bar, private respondents, in their capacities as adopters, cannot claim a right to name the minor adoptee after such right to name the child had already been exercised by the natural parent. Adopting parents have not been conferred such right by law, hence, the right assertes by private respondents herein remains but illusory. Renaming the adoptee cannot be claimed as a right. It is merely a privilege necessitating judicial consent upon compelling grounds. 61 The liberality with which this Court treats matters leading up to adoption insofar as it carries out the beneficent purposes of adoption and ensures to the adopted child the rights and privileges arising therefrom, ever mindful that the paramount consideration is the overall benefit and interest of the adopted child, 62 should be understood in its proper context. It should not be misconstrued or misinterpreted to extend to inferences beyond the contemplation of law and jurisprudence. The practically unrestricted freedom of the natural parent to select the proper or given name of the child presupposes that no other name for it has theretofore been entered in the civil register. Once such name is registered, regardless of the reasons for such choice and even if it be solely for the purpose of identification, the same constitutes the official name. This effectively authenticates the identity of the person and must remain unaltered save when, for the most compelling reasons shown in an appropriate proceeding, its change may merit judicial approval. While the right of a natural parent to name the child is recognized, guaranteed and protected under the law, the so-called right of an adoptive parent to re-name an adopted child by virtue or as a consequence of adoption, even for the most noble intentions and moving supplications, is unheard of in law and consequently cannot be favorably considered. To repeat, the change of the surname of the adoptee as a result of the adoption and to follow that of the adopter does not lawfully extend to or include the proper or given name. Furthermore, factual realities and legal consequences, rather than sentimentality and symbolisms, are what are of concern to the Court. Finally, it is understood that this decision does not entirely foreclose and is without prejudice to, private respondents' privilege to legally change the proper or given name of their adopted child, provided that the same is exercised, this time, via a proper petition for change of name. Of course, the grant thereof is conditioned on strict compliance with all jurisdictional requirements and satisfactory proof of the compelling reasons advanced therefor. WHEREFORE, on the foregoing premises, the assailed order of respondent judge is hereby MODIFIED. The legally adopted child of private respondents shall henceforth be officially known as Kevin Earl

946

University of the Cordilleras College of Law First Year C S.Y. 2013 - 2014 Munson y Andrade unless a change thereof is hereafter effected in accordance with law. In all other respects, the order is AFFIRMED. SO ORDERED. Republic vs Court of Appeals G.R. No. 103695 March 15, 1996 Full Case REPUBLIC OF THE PHILIPPINES, petitioner, vs. THE COURT OF APPEALS, JAIME B. CARANTO, and ZENAIDA P. CARANTO, respondents. MENDOZA, J.:p This is a petition for review on certiorari of the decision 1 of the Court of Appeals in CA-G.R. CV No. 24453 which affirmed in toto the decision of Branch XVI of the Regional Trial Court of Cavite City, granting private respondents' petition for the adoption of Midael C. Mazon with prayer for the correction of the minor's first name "Midael" to "Michael." The petition below was filed on September 21 1988 by private respondents spouses Jaime B. Caranto and Zenaida P. Caranto for the adoption of Midael C. Mazon, then fifteen years old, who had been living with private respondent Jaime B. Caranto since he was seven years old. When private respondents were married on January 19, 1986, the minor Midael C. Mazon stayed with them under their care and custody. Private respondents prayed that judgment be rendered: a) Declaring the child Michael C. Mazon the child of petitioners for all intents and purposes; b.) Dissolving the authority vested in the natural parents of the child; and c) That the surname of the child be legally changed to that of the petitioners and that the first name which was mistakenly registered as "MIDAEL" be corrected to "MICHAEL." The RTC set the case for hearing on September 21, 1988, giving notice thereof by publication in a newspaper of general circulation in the Province of Cavite and by service of the order upon the Department of Social Welfare and Development and the Office of the Solicitor General. The Solicitor General opposed the petition insofar as it sought the correction of the name of the child from "Midael" to "Michael." He argued that although the correction sought concerned only a clerical and innocuous error, it could not be granted because the petition was basically for adoption, not the correction of an entry in the civil registry under Rule 108 of the Rules of Court. Thereafter the case was heard during which private respondent Zenaida Caranto, Florentina Mazon (natural mother of the child), and the minor testified. Also presented was Carlina Perez, social worker of the Department of Social Welfare and Development, who endorsed the adoption of the minor, being of the opinion that the same was in the best interest of the child. On May 30, 1989, the RTC rendered its decision. The RTC dismissed the opposition of the Solicitor General on the ground that Rule 108 of the Rules of Court (Cancellation or Correction of Entries in the Civil Registry) applies only to the correction of entries concerning the civil status of persons. It cited Rule 108, 1, which provides that any person interested in an act, event, order or decree concerning the civil status of persons which has been recorded in the civil register, may file a verified petition for the cancellation or correction of any entry relating thereto." It held that the correction of names in the civil registry is not one of the matters enumerated in Rule 108, 2 as "entries subject to cancellation or correction." According to the trial court, the error could be corrected in the same proceeding for adoption to prevent multiplicity of actions and inconvenience to the petitioners. The dispositive portion of the RTC decision reads: 947

University of the Cordilleras College of Law First Year C S.Y. 2013 - 2014 WHEREFORE, judgment is hereby rendered granting the herein petition and declaring that: 1. Michael C. Mazon is, for all legal intents and purposes, the son by and option of petitioners Jaime B. Caranto and Zenaida P. Caranto; 2. Henceforth, the minor's name shall be Michael Caranto, in lieu of his original name of Michael Mazon, or Midael Mazon, as appearing in his record of birth; 3. The Local Civil Registrar of Cavite City, the birthplace of said minor, is hereby directed to accordingly amend (and) correct the birth certificate of said minor; and 4. This judgment shall retroact to September 2, 1988, the date of filing of the herein petition. The Solicitor General appealed to the Court of Appeals reiterating his contention that the correction of names cannot be effected in the same proceeding for adoption. As additional ground for his appeal, he argued that the RTC did not acquire jurisdiction over the case for adoption because in the notice published in the newspaper, the name given was "Michael," instead of "Midael," which is the name of the minor given in his Certificate of Live Birth. On January 23, 1992, the Court of Appeals affirmed in toto the decision of the RTC. The Court of Appeals ruled that the case of Cruz v. Republic, 2 invoked by the petitioner in support of its plea that the trial court did not acquire jurisdiction over the case, was inapplicable because that case involved a substantial error. Like the trial court, it held that to require the petitioners to file a separate petition for correction of name would entail "additional time and expenses for them as well as for the Government and the Courts." Hence this petition for review. Private respondents were required to comment. Despite opportunity given to them, however, they did not file any comment. The first issue is whether on the facts stated, the RTC acquired jurisdiction over the private respondents' petition for adoption. Petitioner's contention is that the trial court did not acquire jurisdiction over the petition for adoption because the notice by publication did not state the true name of the minor child. Petitioner invokes the ruling inCruz v. Republic. 3 There the petition for adoption and the notice published in the newspaper gave the baptismal name of the child ("Rosanna E. Cruz") instead of her name in the record of birth ("Rosanna E. Bucoy"). It was held that this was a "substantial defect in the petition and the published order of hearing." Indeed there was a question of identity involved in that case. Rosanna E. Cruz could very well be a different person from Rosanna E. Bucoy, as common experience would indicate. The present case is different. It involves an obvious clerical error in the name of the child sought to be adopted. In this case the correction involves merely the substitution of the letters "ch" for the letter "d," so that what appears as "Midael" as given name would read "Michael." Even the Solicitor General admits that the error is a plainly clerical one. Changing the name of the child from "Midael C. Mazon" to "Michael C. Mazon" cannot possibly cause any confusion, because both names "can be read and pronounced with the same rhyme (tugma) and tone (tono, tunog, himig)." The purpose of the publication requirement is to give notice so that those who have any objection to the adoption can make their objection known. That purpose has been served by publication of notice in this case. For this reason we hold that the RTC correctly granted the petition for adoption of the minor Midael C. Mazon and the Court of Appeals, in affirming the decision of the trial court, correctly did so. With regard to the second assignment of error in the petition, we hold that both the Court of Appeals and the trial court erred in granting private respondents' prayer for the correction of the name of the child in the civil registry. Contrary to what the trial court thought, Rule 108 of the Rules of Court applies to this case and because its provision was not complied with, the decision of the trial court, insofar as it ordered the correction of the name of the minor, is void and without force or effect. 948

University of the Cordilleras College of Law First Year C S.Y. 2013 - 2014 The trial court was clearly in error in holding Rule 108 to be applicable only to the correction of errors concerning the civil status of persons. Rule 108, 2 plainly states: 2. Entries subject to cancellation or correction. Upon good and valid grounds, the following entries in the civil register may be cancelled or corrected: (a) births; (b) marriages; (c) deaths; (d) legal separation; (e) judgments of annulments of marriage; (f) judgments declaring marriages void from the beginning; (g) legitimations; (h) adoptions; (i) acknowledgments of natural children; (j) naturalization; (k) election, loss or recovery of citizenship; (l) civil interdiction; (m) judicial determination of filiation (n) voluntary emancipation of a minor; and (o) changes of name. This case falls under letter "(o)," referring to "changes of name." Indeed, it has been the uniform ruling of this Court that Art. 412 of the Civil Code to implement which Rule 108 was inserted in the Rules of Court in 1964 covers "those harmless and innocuous changes, such as correction of a name that is clearly misspelled." 4 Thus, in Yu v. Republic 5 it was held that "to change "Sincio" to "Sencio" which merely involves the substitution of the first vowel "i" in the first name into the vowel "e" amounts merely to the righting of a clerical error." In Labayo-Rowe v.Republic 6 it was held that "the change of petitioner's name from Beatriz Labayo/Beatriz Labayu to Emperatriz Labayo is a mere innocuous alteration wherein a summary proceeding is appropriate." Rule 108 thus applies to the present proceeding. Now 3 of this Rule provides: 3 Parties. When cancellation or correction of an entry in the civil register is sought, the civil registrar and all persons who have or claim any interest which would be affected thereby shall be made parties to the proceeding. The local civil registrar is thus required to be made a party to the proceeding. He is an indispensable party, without whom no final determination of the case can be had. 7 As he was not impleaded in this case much less given notice of the proceeding, the decision of the trial court, insofar as it granted the prayer for the correction of entry, is void. The absence of an indispensable party in a case renders ineffectual all the proceedings subsequent to the filing of the complaint including the judgment. 8 Nor was notice of the petition for correction of entry published as required by Rule 108, 4 which reads: 4. Notice and publication. Upon filing of the petition, the court shall, by an order, fix the time and place for the hearing of the same, and cause reasonable notice thereof to be given to the persons named in the petition. The court shall also cause the order to be published once a week for three (3) consecutive weeks in a newspaper of general circulation in the province. While there was notice given by publication in this case, it was notice of the petition for adoption made in compliance with Rule 99, 4. In that notice only the prayer for adoption of the minor was stated. Nothing was mentioned that in addition the correction of his name in the civil registry was also being sought. The local civil registrar was thus deprived of notice and, consequently, of the opportunity to be heard. The necessary consequence of the failure to implead the civil registrar as an indispensable party and to give notice by publication of the petition for correction of entry was to render the proceeding of the trial court, so far as the correction of entry was concerned, null and void for lack of jurisdiction both as to party and as to the subject matter. 9 WHEREFORE, in view of the foregoing, the decision of the Court of Appeals is MODIFIED by deleting from the decision of the Regional Trial Court the order to the local civil registrar to change the name "MIDAEL" to "MICHAEL" in the birth certificate of the child. In other respects relating to the adoption of Midael C. Mazon, the decision appealed from is AFFIRMED. SO ORDERED. Regalado, Romero and Puno, JJ., concur.

949

University of the Cordilleras College of Law First Year C S.Y. 2013 - 2014 Leonor vs Court of Appeals G.R. No. 112597 April 2, 1996 Full Case VIRGINIA A. LEONOR, petitioner, vs. COURT OF APPEALS, HON. ROLINDO D. BELDIA, JR., as Presiding Judge of the Regional Trial Court of San Carlos City, Branch 57, and MAURICIO D. LEONOR, JR., respondents. PANGANIBAN, J.:p Is a judgment voiding a marriage and rendered by the regional trial court under Rule 108 of the Rules of Court valid and proper? May its validity be challenged by the wife in a petition for certiorari against the husband who abandoned her and who is now living abroad with a foreign woman? These are the two main issues that were posed before this Court in this petition for review seeking a partial reversal of the Decision 1 of the Court of Appeals 2 promulgated September 30, 1993 in CA-G.R. SP No. 30606 and its Resolution 3 promulgated November 1993, which denied petitioner's motion for partial reconsideration of the Decision. The Facts Petitioner Virginia A. Leonor was married to private respondent Mauricio D. Leonor, Jr., in San Carlos City on March 13, 1960. Out of the union, three children, Mauricio III, Ned and Don, were born. The spouses were separated for a substantial part of their married life for, while Mauricio resided in Switzerland studying and working, Virginia stayed in the Philippines working as a nurse in Laguna. Mauricio became unfaithful and lived with a certain Lynda Pond abroad. This induced petitioner to institute a civil action in Geneva, Switzerland for separation and alimony. Private respondent countersued for divorce. On February 14, 1991, the lower Cantonal Civil Court of Switzerland pronounced the divorce of the spouses Leonor but reserved the liquidation of the matrimonial partnership. The said Swiss Court denied alimony to petitioner. In a letter to the lower Cantonal Civil Court dated March 1, 1991, Mauricio, for the first time, raised the issue of the alleged non-existence of the marriage between him and Virginia. Meanwhile, Virginia learned that the solemnizing officer in the Philippines, Justice of the Peace Mabini Katalbas, failed to send a copy of their marriage contract to the Civil Registrar of San Carlos City for registration. Hence, on July 11, 1991, Virginia applied for the late registration of her marriage. The Civil Registrar, finding said application in order, granted the same. On appeal to the higher Cantonal Civil Court, Mauricio asked for the cancellation of his marriage in the Philippines. On January 17, 1992, the higher Cantonal Civil Court granted petitioner alimony, prompting Mauricio to elevate the matter on appeal to the Federal Court of Switzerland. In its decision dated July 9, 1992, the Federal Court affirmed the decision of the higher Cantonal Civil Court. 4 On May 22, 1992, Mauricio, represented by his brother Teodoro Leonor, filed a petition for the cancellation of the late registration of marriage in the civil registry of San Carlos City with the Regional Trial Court, Branch 59, San Carlos City (Special proceeding No. RTC-144). Given as grounds for the cancellation were the tardiness of the registration and the nullity of his marriage with Virginia "due to the non-observance of the legal requirements for a valid marriage." Mauricio's petition was filed pursuant to Rule 108 of the Rules of Court. After several hearings and on December 14, 1992, the trial court rendered judgment 5 declaring said marriage null and void for being sham and fictitious. The dispositive portion of said decision reads: AND IN THE LIGHT OF THE FOREGOING, this Court finds and orders that the registration of the marriage contract between Mauricio Leonor, Jr. and Virginia Amor dated March 13, 1960 must be cancelled in (sic) the Books of the Local Civil Registry of San Carlos City for being a null and void 950

University of the Cordilleras College of Law First Year C S.Y. 2013 - 2014 marriage not in accordance with a (sic) New Civil code under Articles 52, 53 and 55 now presently amended by the Family Code of the Philippines, Executive Order No. 209 as amended by Executive Order No. 227, without pronouncement as to cost. Virginia received notice of the decision on January 4, 1993, and on January 15, 1993, she filed her notice of appeal. On February 24, 1993, the trial court, on motion of Mauricio's counsel, issued an order 6 dismissing Virginia's appeal on the ground that she had failed to file a record on appeal within thirty days and had thus failed to perfect her appeal. It was the erroneous holding of the trial court that in special proceedings, a record on appeal was an indispensable requisite under Rule 19, Section 6 of the Interim Rules and Guidelines in relation to Rule 109 of the Rules of Court. Such failure, according to respondent Judge, caused the decision to become executory. On April 1, 1993, Virginia filed a petition for certiorari, prohibition and mandamus with the Court of Appeals (CA-G.R. SP NO. 30606) and sought the nullification of both the decision dated December 14, 1992 and the order dated February 24, 1993 of the trial court for having been issued in excess of jurisdiction and/or with grave abuse of discretion. The Court of Appeals dismissed the petition insofar as it sought the reversal of the decision of the trial court, saying that the remedy for said purpose was an appeal, not a special civil action. The appellate court, however, granted the petition insofar as it sought the nullification of the Order dated February 24, 1993 dismissing the appeal. Said the appellate court: Even so, this petition is an appropriate remedy against the dismissal of the petitioner's appeal, which dismissal we sense to be erroneous and issued in excess of jurisdiction. xxx xxx xxx WHEREFORE, judgment is hereby rendered setting aside the questioned order of respondent judge dated February 24, 1993, with instructions to the latter to give due course to petitioner's appeal in the subject special proceeding. Costs against private respondents. Dissatisfied with the above Decision, petitioner filed a motion for partial reconsideration asking the Court of Appeals to annul the decision of the trial court. The Court of Appeals denied the motion, stating that the central issue in the special civil action was only the validity of the trial court's order denying petitioner's right to appeal and that said issue was resolved in petitioner's favor. Further, it said that the correctness or validity of the trial court's decision should properly be resolved in the appeal. Hence, the present recourse. Issues Raised by the Parties The petition assailed the respondent Court's Decision and Order mentioned in the second paragraph of this Decision for alleged 1. Procedural Errors . . . in not finding . . . (a) that the lower court gravely abused its discretion" in recognizing the action as one for declaration of "nullity of marriage" instead of a "special proceeding for cancellation of (an) entry", in the civil registry and (b) in not finding that the "lower court had no jurisdiction (over) the issue of nullity"; and 2. Substantive errors . . . in not finding . . . (a) that the lower court gravely erred in declaring the marriage null and void . . . and (b) . . . in disregarding the presumptions in favor of the rights of children and to the administration of the conjugal property . . . and the validity of marriage . . . In her Memorandum, petitioner elucidated and spiritedly argued the above grounds. In fine, the foregoing issues could be restated as follows: 951

University of the Cordilleras College of Law First Year C S.Y. 2013 - 2014 1. Did the respondent Court err in holding that petitioner should have appealed from the trial court's decision instead of filing a petition for certiorari? 2. Did the respondent Court err in refusing to decide upon the merits of the case, that is, to declare whether or not the judgment of the trial court is null and void? Should the Supreme Court now resolve the merits of the case, i.e., decide the issue of nullity of the assailed decision of the trial court? The Court's Ruling Since these issues are intimately intertwined, we shall discuss them together. At the outset, it must be stressed that the Court of Appeals acted within its authority in simply ordering the trial court to give due course to petitioner's appeal without going into the merits of the case. In Municipality of Bian, Laguna vs. Court of Appeals, 7 we held: Respondent Court of Appeals has no jurisdiction in a certiorari proceeding involving an incident in a case to rule on the merits of the main case itself which was not on appeal before it. . . . In other words, the Court of Appeals has already done its duty by declaring that the lower court gravely abused its discretion or acted without jurisdiction in refusing to give due course to petitioner's appeal. Hence, it ordered said court to allow the appeal. Once appeal is perfected, the merits of the case, i.e. the validity/nullity of the trial court's decision, would then be resolved by said Court. Understandably, the Court of Appeals has limited itself to ruling upon the procedural question lodged before it. It cannot be seriously faulted as petitioner vehemently did for opting to navigate within the narrow banks of the placid waters of certiorari. For in doing so, it was strictly following established legal doctrines and precedents. Upon the other hand, the Supreme Court is not just a toothless promoter of procedural niceties which are understood and appreciated only by lawyers and jurists. It cannot shrink from its quintessential role as the fountain of speedy, adequate and substantial justice. If the Court, as the head and guardian of the judicial branch, must continuously merit the force of public trust and confidence which ultimately is the real source of its sovereign power, possessing neither the purse nor the sword and if it must decisively discharge its sacred duty as the last sanctuary of the oppressed and the weak, it must, in appropriate cases like the one before us, pro-actively provide weary litigants with immediate legal and equitable relief, free from the delays and legalistic contortions that oftentimes result from applying purely formal and procedural approaches to judicial dispensations. Pursuant to the foregoing principle and considering the peculiar circumstances of the present case which are patent on the basis of the admitted facts, as well as the undisputed copies of the pleadings presented by the parties, and especially the verified copy of the trial court's decision which loudly speaks for itself, the Court therefore resolved to make an exception to the normal procedures and to delve deeper into the substantive issue of the validity/nullity of the trial court's proceedings and judgment. Happily, both parties had expressed a desire to have this case resolved soonest. Upon the other hand, remanding the case back to the trial court for the perfection of the appeal and requiring the parties to re-litigate in the Court of Appeals with the use of probably the same documents and arguments ventilated in the kilometric pleadings filed here would just unnecessarily clog the courts' dockets; besides, in all likelihood the parties would eventually come before this Court anyway. Also, it must be observed that Virginia actually filed a proper Notice of Appeal which the trial court disallowed. Hence, she had no choice but to bring her petition for certiorari in the respondent Court. To constrain her to go back to said Court, this time by ordinary appeal, would be tantamount to punishing her and delaying her cause for faults not attributable to her, but rather to the manifest error of the respondent trial judge. So, too, as will be shown shortly, the trial court's decision is really a nullity for utter want of jurisdiction. Hence it could be challenged at any time. 952

University of the Cordilleras College of Law First Year C S.Y. 2013 - 2014 It is not disputed that the original petition 8 in the trial court was for "cancellation of entry in the civil registry of the "late registration of the marriage" between Leonor and Mauricio, "in consonance with Section 3, Rule 108 of the Rules of Court." Ground alleged for the nullity and cancellation of the marriage was "non-observance of the legal requirements for a valid marriage." Later, on August 22, 1992, an amended petition 9 was filed adding essentially the following allegations: (a) that there was no marriage contract, (b) that the marriage was a "sham . . . to cover-up the (alleged) shame of Virginia Amor who was then pregnant, " (c) that Virginia allegedly assured Mauricio that they "need not live together . . . and Mauricio need not give any support", (d) that the couple always had "trouble (and) quarrel," and (e) that Mauricio had been "transferring residence to avoid Virginia until he went abroad for good." The answer 10 of the Civil Registrar and the opposition 11 of Virginia, among others, disputed the propriety of the collateral attack against the marriage, under said Rule. The decision 12 of the trial court is, painfully, a sophomoric and pathetic portrayal of Virginia as allegedly an "unbecoming . . . unmarried woman (who) wormed her way to a (sic) heart of the matriarch of the Leonor Family . . . to summon the son Mauricio to come" to her rescue and as a scheming nurse who lured a "struggling young teacher . . . to this unwelcomed (sic) love affair". These matters, needless to say, border on the incredible, as they were brought up some thirty (30) years after the marriage was celebrated in 1960 and only after Virginia discovered her husband's infidelity. The said decision's crude attempt at literary sophistication is matched only by its jarring syntax and grammatical incongruencies. 13 In so far as this Court can figure out from the convoluted language of the decision, the trial court (a) declared the marriage null and void and (b) ordered the local civil registrar of San Carlos City to cancel its entry in the local civil registry, the sum total of which, coincidentally (and most conveniently) , would enable Mauricio to show to the Swiss courts that he was never married and thus, to convince said courts to reverse their order granting alimony to his abandoned wife. Such blatant abuse and mis-use of court proceedings cannot be countenanced by this Court. The ultimate legal question therefore is this: In disposing of a special proceeding under Rule 108, did the trial court have jurisdiction to declare the marriage null and void and to order the cancellation of its entry in the local civil registry? To contribute to the cause of clarity, Rule 108 of the Rules of Court is reproduced hereunder. Rule 108 CANCELLATION OR CORRECTION OF ENTRIES IN THE CIVIL REGISTRY Sec. 1. Who may file petition. Any person interested in any act, event, order or decree concerning the civil status of persons which has been recorded in the civil register, may file a verified petition for the cancellation or correction of any entry relating thereto, with the court of First Instance of the province where the corresponding civil registry is located. Sec. 2. Entries subject to cancellation or correction. Upon good and valid grounds, the following entries in the civil register may be cancelled or corrected; (a) births; (b) marriages; (c) deaths; (d) legal separations; (e) judgments of annulments of marriage; (f) judgments declaring marriages void from the beginning; (g) legitimations; (h) adoptions; (i) acknowledgments of natural children; (j) naturalization; (k) election, loss or recovery of citizenship; (l) civil interdiction; (m) judicial determination of filiation; (n) voluntary emancipation of a minor; and (o) changes of name. Sec. 3. Parties. When cancellation or correction of an entry in the civil register is sought, the civil registrar and all persons who have or claim any interest which would be affected thereby shall be made parties to the proceeding. Sec. 4. Notice and publication. Upon the filing of the petition, the court shall, by an order, fix the time and place for the hearing of the same, and cause reasonable notice thereof to be given to the persons named in the petition. The court shall also cause the order to be published once a week for three (3) consecutive weeks in a newspaper of general circulation in the province. 953

University of the Cordilleras College of Law First Year C S.Y. 2013 - 2014 Sec. 5. Opposition. The civil registrar and any person having or claiming any interest under the entry whose cancellation or correction is sought may, within fifteen (15) days from notice of the petition, or from the last date of publication of such notice, file his opposition thereto. Sec. 6. Expediting proceedings. The court in which the proceeding is brought may make orders expediting the proceedings, and may also grant preliminary injunction for the preservation of the rights of the parties pending such proceedings. Sec. 7. Order. After hearing, the court may either dismiss the petition or issue an order granting the cancellation or correction prayed for. In either case, a certified copy of the judgment shall be served upon the civil registrar concerned who shall annotate the same in his record. On its fce, the Rule would appear to authorize the cancellation of any entry regarding "marriages" in the civil registry for any reason by the mere filing of a verified petition for the purpose. However, it is not as simple as it looks. Doctrinally, the only errors that can be cancelled or corrected under this Rule are typographical or clerical errors, not material or substantial ones like the validity or nullity of a marriage 14. "A clerical error is one which is visible to the eyes or obvious to the understanding; error made by a clerk or a transcriber; a mistake in copying or writing (Black vs. Republic, L-10869, Nov. 28, 1958); or some harmless and innocuous change such as a correction of name that is clearly misspelled or of a misstatement of the occupation of the parent (Ansalada vs. Republic, No. L-10226, Feb. 14, 1958)" 15. Where the effect of a correction in a civil registry will change the civil status of petitioner and her children from legitimate to illegitimate, the same cannot be granted except only in an adversarial proceeding 16. In Vda. de Castro vs. Republic; 17 this Court held: . . . It has been the consistent ruling of this Court since the Ty Kong Tin vs. Republic, 94 Phil. 321, that substantial alterations, such as those affecting the status and citizenship of a person in the Civil Registry Records, can not be ordered by the court unless first threshed out in an "appropriate action wherein all parties who may be affected by the entries are notified or represented" (see Rule 108 of the Revised Rules of Court), and that the summary proceedings under Article 412 of the Civil Code only justify an order to correct innocuous or clerical errors, such as mis-spellings and the like, errors that are visible to the eyes or obvious to the understanding. (Baybayan vs. Republic of the Philippines, 16 SCRA 403.) Clearly and unequivocally, the summary procedure under Rule 108, and for that matter under Art. 412 of the Civil Code, cannot be used by Mauricio to change his and Virginia's civil status from married to single and of their three children from legitimate to illegitimate. Neither does the trial court, under said Rule, have any jurisdiction to declare their marriage null and void and as a result thereof, to order the local civil registrar to cancel the marriage entry in the civil registry. Further, the respondent trial judge gravely and seriously abused his discretion in unceremoniously expanding his very limited jurisdiction under such rule to hear evidence on such a controversial matter as nullity of a marriage under the Civil Code and/or Family Code, a process that is proper only in ordinary adversarial proceedings under the Rules. A void judgment for want of jurisdiction is no judgment at all. It cannot be the source of any right nor the creator of any obligation. All acts performed pursuant to it and all claims emanating from it have no legal effect. Hence, it can never become final and any writ of execution based on it is void; ". . . it may be said to be a lawless thing which can be treated as an outlaw and slain at sight, or ignored wherever and whenever it exhibits its head." 18 WHEREFORE, the petition is GRANTED. Judgment is hereby rendered DECLARING NULL and VOID the decision of the respondent judge dated February 14, 1992 in Special Proceedings No. RTC-144 and MODIFYING accordingly the Decision dated September 30, 1993 of the respondent Court of Appeals in CA-G.R. No. SP-30606. Let a copy of this Decision be spread in the records of respondent Judge in the Office of the Court Administrator. Costs against private respondent Mauricio D. Leonor, Jr. SO ORDERED.

954

University of the Cordilleras College of Law First Year C S.Y. 2013 - 2014

XVII. Latest Jurisprudence

955

University of the Cordilleras College of Law First Year C S.Y. 2013 - 2014 GERBERT R. CORPUZ vs. DAISYLYN TIROL STO. TOMAS G.R. No. 186571 August 11, 2010 Full Case GERBERT R. CORPUZ vs DAISYLYN TIROL STO. TOMAS and The SOLICITOR GENERAL BRION, J.: Before the Court is a direct appeal from the decision[1] of the Regional Trial Court (RTC) of Laoag City, Branch 11, elevated via a petition for review on certiorari[2]under Rule 45 of the Rules of Court (present petition). Petitioner Gerbert R. Corpuz was a former Filipino citizen who acquired Canadian citizenship through naturalization on November 29, 2000.[3] On January 18, 2005, Gerbert married respondent Daisylyn T. Sto. Tomas, a Filipina, in Pasig City.[4] Due to work and other professional commitments, Gerbert left for Canada soon after the wedding. He returned to the Philippines sometime in April 2005 to surprise Daisylyn, but was shocked to discover that his wife was having an affair with another man. Hurt and disappointed, Gerbert returned to Canada and filed a petition for divorce. The Superior Court of Justice, Windsor, Ontario, Canada granted Gerberts petition for
[5]

divorce

onDecember

8,

2005. The divorce decree took effect a month later, on January 8, 2006.

Two years after the divorce, Gerbert has moved on and has found another Filipina to love. Desirous of marrying his new Filipina fiance in the Philippines, Gerbert went to the Pasig City Civil Registry Office and registered the Canadian divorce decree on his and Daisylyns marriage certificate. Despite the registration of the divorce decree, an official of the National Statistics Office (NSO) informed Gerbert that the marriage between him and Daisylyn still subsists under Philippine law; to be enforceable, the foreign divorce decree must first be judicially recognized by a competent Philippine court, pursuant to NSO Circular No. 4, series of 1982.[6] Accordingly, Gerbert filed a petition for judicial recognition of foreign divorce and/or declaration of marriage as dissolved (petition) with the RTC. Although summoned, Daisylyn did not file any responsive pleading but submitted instead a notarized letter/manifestation to the trial court. She offered no opposition to Gerberts petition and, in fact, alleged her desire to file a similar case herself but was prevented by financial and personal circumstances. She, thus, requested that she be considered as a party-in-interest with a similar prayer to Gerberts. In its October 30, 2008 decision,[7] the RTC denied Gerberts petition. The RTC concluded that Gerbert was not the proper party to institute the action for judicial recognition of the foreign divorce decree as he is a naturalized Canadian citizen. It ruled that only the Filipino spouse can avail of the remedy, under the second paragraph of Article 26 of the Family Code,[8] in order for him or her to be able to remarry under Philippine law.[9] Article 26 of the Family Code reads: Art. 26. All marriages solemnized outside the Philippines, in accordance with the laws in force in the country where they were solemnized, and valid there as such, shall

956

University of the Cordilleras College of Law First Year C S.Y. 2013 - 2014 also be valid in this country, except those prohibited under Articles 35(1), (4), (5) and (6), 36, 37 and 38. Where a marriage between a Filipino citizen and a foreigner is validly celebrated and a divorce is thereafter validly obtained abroad by the alien spouse capacitating him or her to remarry, the Filipino spouse shall likewise have capacity to remarry under Philippine law. This conclusion, the RTC stated, is consistent with the legislative intent behind the enactment of the second paragraph of Article 26 of the Family Code, as determined by the Court in Republic v. Orbecido III;[10] the provision was enacted to avoid the absurd situation where the Filipino spouse remains married to the alien spouse who, after obtaining a divorce, is no longer married to the Filipino spouse.[11] THE PETITION From the RTCs ruling,[12] Gerbert filed the present petition.[13] Gerbert asserts that his petition before the RTC is essentially for declaratory relief, similar to that filed in Orbecido; he, thus, similarly asks for a determination of his rights under the second paragraph of Article 26 of the Family Code. Taking into account the rationale behind the second paragraph of Article 26 of the Family Code, he contends that the provision applies as well to the benefit of the alien spouse. He claims that the RTC ruling unduly stretched the doctrine in Orbecido by limiting the standing to file the petition only to the Filipino spouse an interpretation he claims to be contrary to the essence of the second paragraph of Article 26 of the Family Code. He considers himself as a proper party, vested with sufficient legal interest, to institute the case, as there is a possibility that he might be prosecuted for bigamy if he marries his Filipina fiance in the Philippines since two marriage certificates, involving him, would be on file with the Civil Registry Office. The Office of the Solicitor General and Daisylyn, in their respective Comments,[14]both support Gerberts position. Essentially, the petition raises the issue of whether the second paragraph of Article 26 of the Family Code extends to aliens the right to petition a court of this jurisdiction for the recognition of a foreign divorce decree. THE COURTS RULING The alien spouse can claim no right under the second paragraph of Article 26 of the Family Code as the substantive right it establishes is in favor of the Filipino spouse The resolution of the issue requires a review of the legislative history and intent behind the second paragraph of Article 26 of the Family Code. The Family Code recognizes only two types of defective marriages void[15] and voidable[16] marriages. In both cases, the basis for the judicial declaration of absolute nullity or annulment of the marriage exists before or at the time of the marriage. Divorce, on the other hand, contemplates the dissolution of the lawful union for cause arising afterthe marriage.[17] Our family laws do not recognize absolute divorce between Filipino citizens.[18]

957

University of the Cordilleras College of Law First Year C S.Y. 2013 - 2014 Recognizing the reality that divorce is a possibility in marriages between a Filipino and an alien, President Corazon C. Aquino, in the exercise of her legislative powers under the Freedom Constitution,[19] enacted Executive Order No. (EO) 227, amending Article 26 of the Family Code to its present wording, as follows: Art. 26. All marriages solemnized outside the Philippines, in accordance with the laws in force in the country where they were solemnized, and valid there as such, shall also be valid in this country, except those prohibited under Articles 35(1), (4), (5) and (6), 36, 37 and 38. Where a marriage between a Filipino citizen and a foreigner is validly celebrated and a divorce is thereafter validly obtained abroad by the alien spouse capacitating him or her to remarry, the Filipino spouse shall likewise have capacity to remarry under Philippine law. Through the second paragraph of Article 26 of the Family Code, EO 227 effectively incorporated into the law this Courts holding in Van Dorn v. Romillo, Jr.[20] and Pilapil v. Ibay-Somera.[21] In both cases, the Court refused to acknowledge the alien spouses assertion of marital rights after a foreign courts divo rce decree between the alien and the Filipino. The Court, thus, recognized that the foreign divorce had already severed the marital bond between the spouses. The Court reasoned in Van Dorn v. Romillo that: To maintain x x x that, under our laws, [the Filipino spouse] has to be considered still married to [the alien spouse] and still subject to a wife's obligations x x x cannot be just. [The Filipino spouse] should not be obliged to live together with, observe respect and fidelity, and render support to [the alien spouse]. The latter should not continue to be one of her heirs with possible rights to conjugal property. She should not be discriminated against in her own country if the ends of justice are to be served.[22]

As the RTC correctly stated, the provision was included in the law to avoid the absurd situation where the Filipino spouse remains married to the alien spouse who, after obtaining a divorce, is no longer married to the Filipino spouse.[23] The legislative intent is for the benefit of the Filipino spouse, by clarifying his or her marital status, settling the doubts created by the divorce decree. Essentially, the second paragraph of Article 26 of the Family Code provided the Filipino spouse a substantive right to have his or her marriage to the alien spouse considered as dissolved, capacitating him or her to remarry.[24] Without the second paragraph of Article 26 of the Family Code, the judicial recognition of the foreign decree of divorce, whether in a proceeding instituted precisely for that purpose or as a related issue in another proceeding, would be of no significance to the Filipino spouse since our laws do not recognize divorce as a mode of severing the marital bond;[25] Article 17 of the Civil Code provides that the policy against absolute divorces cannot be subverted by judgments promulgated in a foreign country. The inclusion of the second paragraph in Article 26 of the Family Code provides the direct exception to this rule and serves as basis for recognizing the dissolution of the marriage between the Filipino spouse and his or her alien spouse. Additionally, an action based on the second paragraph of Article 26 of the Family Code is not limited to the recognition of the foreign divorce decree. If the court finds that the decree capacitated the alien spouse to remarry, the courts can declare that the Filipino spouse is likewise capacitated to contract 958

University of the Cordilleras College of Law First Year C S.Y. 2013 - 2014 another marriage. No court in this jurisdiction, however, can make a similar declaration for the alien spouse (other than that already established by the decree), whose status and legal capacity are generally governed by his national law.[26] Given the rationale and intent behind the enactment, and the purpose of the second paragraph of Article 26 of the Family Code, the RTC was correct in limiting the applicability of the provision for the benefit of the Filipino spouse. In other words, only the Filipino spouse can invoke the second paragraph of Article 26 of the Family Code; the alien spouse can claim no right under this provision.

The foreign divorce decree is presumptive evidence of a right that clothes the party with legal interest to petition for its recognition in this jurisdiction We qualify our above conclusion i.e., that the second paragraph of Article 26 of the Family Code bestows no rights in favor of aliens with the complementary statement that this conclusion is not sufficient basis to dismiss Gerberts petition before the RTC. In other words, the unavailability of the second paragraph of Article 26 of the Family Code to aliens does not necessarily strip Gerbert of legal interest to petition the RTC for the recognition of his foreign divorce decree. The foreign divorce decree itself, after its authenticity and conformity with the aliens national law have been duly proven according to our rules of evidence, serves as a presumptive evidence of right in favor of Gerbert, pursuant to Section 48, Rule 39 of the Rules of Court which provides for the effect of foreign judgments. This Section states: SEC. 48. Effect of foreign judgments or final orders.The effect of a judgment or final order of a tribunal of a foreign country, having jurisdiction to render the judgment or final order is as follows: (a) In case of a judgment or final order upon a specific thing, the judgment or final order is conclusive upon the title of the thing; and In case of a judgment or final order against a person, the judgment or final order is presumptive evidence of a right as between the parties and their successors in interest by a subsequent title.

(b)

In either case, the judgment or final order may be repelled by evidence of a want of jurisdiction, want of notice to the party, collusion, fraud, or clear mistake of law or fact. To our mind, direct involvement or being the subject of the foreign judgment is sufficient to clothe a party with the requisite interest to institute an action before our courts for the recognition of the foreign judgment. In a divorce situation, we have declared, no less, that the divorce obtained by an alien abroad may be recognized in the Philippines, provided the divorce is valid according to his or her national law.[27] The starting point in any recognition of a foreign divorce judgment is the acknowledgment that our courts do not take judicial notice of foreign judgments and laws. Justice Herrera explained that, as a rule, no sovereign is bound to give effect within its dominion to a judgment rendered by a tribunal of another country.[28] This means that the foreign judgment and its authenticity must be proven as facts under our rules on evidence, together with the aliens applicable national law to show the effect of the 959

University of the Cordilleras College of Law First Year C S.Y. 2013 - 2014 judgment on the alien himself or herself.[29] The recognition may be made in an action instituted specifically for the purpose or in another action where a party invokes the foreign decree as an integral aspect of his claim or defense. In Gerberts case, since both the foreign divorce decree and the na tional law of the alien, recognizing his or her capacity to obtain a divorce, purport to be official acts of a sovereign authority, Section 24, Rule 132 of the Rules of Court comes into play. This Section requires proof, either by (1) official publications or (2) copies attested by the officer having legal custody of the documents. If the copies of official records are not kept in the Philippines, these must be (a) accompanied by a certificate issued by the proper diplomatic or consular officer in the Philippine foreign service stationed in the foreign country in which the record is kept and (b) authenticated by the seal of his office. The records show that Gerbert attached to his petition a copy of the divorce decree, as well as the required certificates proving its authenticity,[30] but failed to include a copy of the Canadian law on divorce.[31] Under this situation, we can, at this point, simply dismiss the petition for insufficiency of supporting evidence, unless we deem it more appropriate to remand the case to the RTC to determine whether the divorce decree is consistent with the Canadian divorce law. We deem it more appropriate to take this latter course of action, given the Article 26 interests that will be served and the Filipina wifes (Daisylyns) obvious conformity with the petition. A remand, at the same time, will allow other interested parties to oppose the foreign judgment and overcome a petitioners presumptive evidence of a right by proving want of jurisdiction, want of notice to a party, collusion, fraud, or clear mistake of law or fact. Needless to state, every precaution must be taken to ensure conformity with our laws before a recognition is made, as the foreign judgment, once recognized, shall have the effect of res judicata[32] between the parties, as provided in Section 48, Rule 39 of the Rules of Court.[33] In fact, more than the principle of comity that is served by the practice of reciprocal recognition of foreign judgments between nations, the res judicata effect of the foreign judgments of divorce serves as the deeper basis for extending judicial recognition and for considering the alien spouse bound by its terms. This same effect, as discussed above, will not obtain for the Filipino spouse were it not for the substantive rule that the second paragraph of Article 26 of the Family Code provides. Considerations beyond the recognition of the foreign divorce decree As a matter of housekeeping concern, we note that the Pasig City Civil Registry Office has already recorded the divorce decree on Gerbert and Daisylyns marriage certificate based on the mere presentation of the decree.[34] We consider the recording to be legally improper; hence, the need to draw attention of the bench and the bar to what had been done. Article 407 of the Civil Code states that [a]cts, events and judicial decrees concerning the civil status of persons shall be recorded in the civil register. The law requires the entry in the civil registry of judicial decrees that produce legal consequences touching upon a persons legal capacity and status, i.e., 960

University of the Cordilleras College of Law First Year C S.Y. 2013 - 2014 those affecting all his personal qualities and relations, more or less permanent in nature, not ordinarily terminable at his own will, such as his being legitimate or illegitimate, or his being married or not.[35] A judgment of divorce is a judicial decree, although a foreign one, affecting a persons legal capacity and status that must be recorded. In fact, Act No. 3753 or the Law on Registry of Civil Status specifically requires the registration of divorce decrees in the civil registry: Sec. 1. Civil Register. A civil register is established for recording the civil status of persons, in which shall be entered: (a) (b) (c) (d) (e) (f) (g) (h) (i) (j) births; deaths; marriages; annulments of marriages; divorces; legitimations; adoptions; acknowledgment of natural children; naturalization; and changes of name. xxxx Sec. 4. Civil Register Books. The local registrars shall keep and preserve in their offices the following books, in which they shall, respectively make the proper entries concerning the civil status of persons: (1) Birth and death register; (2) Marriage register, in which shall be entered not only the marriages solemnized but also divorces and dissolved marriages. (3) Legitimation, acknowledgment, adoption, change of name and naturalization register.

But while the law requires the entry of the divorce decree in the civil registry, the law and the submission of the decree by themselves do not ipso facto authorize the decreesregistration. The law should be read in relation with the requirement of a judicial recognition of the foreign judgment before it can be given res judicata effect. In the context of the present case, no judicial order as yet exists recognizing the foreign divorce decree. Thus, the Pasig City Civil Registry Office acted totally out of turn and without authority of law when it annotated the Canadian divorce decree on Gerbert and Daisylyns marriage certificate, on the strength alone of the foreign decree presented by Gerbert. Evidently, the Pasig City Civil Registry Office was aware of the requirement of a court recognition, as it cited NSO Circular No. 4, series of 1982,[36] and Department of Justice Opinion No. 181, series of 1982[37] both of which required a final order from a competent Philippine court before a foreign judgment, dissolving a marriage, can be registered in the civil registry, but it, nonetheless, allowed the registration of the decree. For being contrary to law, the registration of the foreign divorce decree without the requisite judicial recognition is patently void and cannot produce any legal effect.

961

University of the Cordilleras College of Law First Year C S.Y. 2013 - 2014 Another point we wish to draw attention to is that the recognition that the RTC may extend to the Canadian divorce decree does not, by itself, authorize the cancellation of the entry in the civil registry. A petition for recognition of a foreign judgment is not the proper proceeding, contemplated under the Rules of Court, for the cancellation of entries in the civil registry. Article 412 of the Civil Code declares that no entry in a civil register shall be changed or corrected, without judicial order. The Rules of Court supplements Article 412 of the Civil Code by specifically providing for a special remedial proceeding by which entries in the civil registry may be judicially cancelled or corrected. Rule 108 of the Rules of Court sets in detail the jurisdictional and procedural requirements that must be complied with before a judgment, authorizing the cancellation or correction, may be annotated in the civil registry. It also requires, among others, that the verified petition must be filed with the RTC of the province where the corresponding civil registry is located;[38] that the civil registrar and all persons who have or claim any interest must be made parties to the proceedings;[39] and that the time and place for hearing must be published in a newspaper of general circulation.[40] As these basic jurisdictional requirements have not been met in the present case, we cannot consider the petition Gerbert filed with the RTC as one filed under Rule 108 of the Rules of Court. We hasten to point out, however, that this ruling should not be construed as requiring two separate proceedings for the registration of a foreign divorce decree in the civil registry one for recognition of the foreign decree and another specifically for cancellation of the entry under Rule 108 of the Rules of Court. The recognition of the foreign divorce decree may be made in a Rule 108 proceeding itself, as the object of special proceedings (such as that in Rule 108 of the Rules of Court) is precisely to establish the status or right of a party or a particular fact. Moreover, Rule 108 of the Rules of Court can serve as the appropriate adversarial proceeding[41] by which the applicability of the foreign judgment can be measured and tested in terms of jurisdictional infirmities, want of notice to the party, collusion, fraud, or clear mistake of law or fact. WHEREFORE, we GRANT the petition for review on certiorari, and REVERSE the October 30, 2008 decision of the Regional Trial Court of Laoag City, Branch 11, as well as its February 17, 2009 order. We order the REMAND of the case to the trial court for further proceedings in accordance with our ruling above. Let a copy of this Decision be furnished the Civil Registrar General. No costs.

962

University of the Cordilleras College of Law First Year C S.Y. 2013 - 2014 Case Digest GERBERT R. CORPUZ GENERAL, G.R. No. 186571 August 11, 2010 Facts: Petitioner Gerbert R. Corpuz was a former Filipino citizen who acquired Canadian citizenship through naturalization on November 29, 2000. On January 18, 2005, Gerbert married respondent Daisylyn T. Sto. Tomas, a Filipina, in Pasig City. Due to work and other professional commitments, Gerbert left for Canada soon after the wedding. He returned to the Philippines sometime in April 2005 to surprise Daisylyn, but was shocked to discover that his wife was having an affair with another man. Hurt and disappointed, Gerbert returned to Canada and filed a petition for divorce which was granted. The divorce decree took effect a month later, on January 8, 2006. Two years after the divorce, Gerbert has moved on and has found another Filipina to love. Desirous of marrying his new Filipina fiance in the Philippines, Gerbert went to the Pasig City Civil Registry Office and registered the Canadian divorce decree on his and Daisylyns marriage certificate. Despite the registration of the divorce decree, an official of the National Statistics Office (NSO) informed Gerbert that the marriage between him and Daisylyn still subsists under Philippine law; to be enforceable, the foreign divorce decree must first be judicially recognized by a competent Philippine court, pursuant to NSO Circular No. 4, series of 1982. Accordingly, Gerbert filed a petition for judicial recognition of foreign divorce and/or declaration of marriage as dissolved (petition) with the RTC. Although summoned, Daisylyn did not file any responsive pleading but submitted instead a notarized letter/manifestation to the trial court. She offered no opposition to Gerberts petition and, in fact, alleged her desire to file a similar case herself but was prevented by financial and personal circumstances. She, thus, requested that she be considered as a party-in-interest with a similar prayer to Gerberts. Issue: Whether the second paragraph of Article 26 of the Family Code extends to aliens the right to petition a court of this jurisdiction for the recognition of a foreign divorce decree. Ruling: The alien spouse can claim no right under the second paragraph of Article 26 of the Family Code as the substantive right it establishes is in favor of the Filipino spouse The provision was included in the law to avoid the absurd situation where the Filipino spouse remains married to the alien spouse who, after obtaining a divorce, is no longer married to the Filipino spouse. The legislative intent is for the benefit of the Filipino spouse, by clarifying his or her marital status, settling the doubts created by the divorce decree. Essentially, the second paragraph of Article 26 of the Family Code provided the Filipino spouse a substantive right to have his or her marriage to the alien spouse considered as dissolved, capacitating him or her to remarry. Without the second paragraph of Article 26 of the Family Code, the judicial recognition of the foreign decree of divorce, whether in a proceeding instituted precisely for that purpose or as a related issue in another proceeding, would be of no significance to the Filipino spouse since our laws do not recognize divorce as a mode of severing the marital bond; Article 17 of the Civil Code provides that the policy against absolute divorces cannot be subverted by judgments promulgated in a foreign country. The inclusion of the second paragraph in Article 26 vs. DAISYLYN TIROL STO. TOMAS and The SOLICITOR

963

University of the Cordilleras College of Law First Year C S.Y. 2013 - 2014 of the Family Code provides the direct exception to this rule and serves as basis for recognizing the dissolution of the marriage between the Filipino spouse and his or her alien spouse. Additionally, an action based on the second paragraph of Article 26 of the Family Code is not limited to the recognition of the foreign divorce decree. If the court finds that the decree capacitated the alien spouse to remarry, the courts can declare that the Filipino spouse is likewise capacitated to contract another marriage. No court in this jurisdiction, however, can make a similar declaration for the alien spouse (other than that already established by the decree), whose status and legal capacity are generally governed by his national law.

964

University of the Cordilleras College of Law First Year C S.Y. 2013 - 2014 RICARDO P. TORING vs. TERESITA M. TORING G.R. No. 165321 August 3, 2010 Full Text RICARDO P. TORING, Petitioner, vs. TERESITA M. TORING and REPUBLIC OF THE PHILIPPINES, Respondents. BRION, J.: We resolve the appeal filed by petitioner Ricardo P. Toring from the May 31, 2004 decision1 of the Court of Appeals (CA) in CA-G.R. CV No. 71882. The CA reversed the August 10, 2001 judgment of the Regional Trial Court (RTC), Branch 106 of Quezon City in Civil Case No. Q-99-36662,2 nullifying Ricardo's marriage with respondent Teresita M. Toring on the ground of psychological incapacity. THE FACTS Ricardo was introduced to Teresita in 1978 at his aunts house in Cebu. Teresita was then his cousins teacher in Hawaiian dance and was conducting lessons at his aunts house. Despite their slight difference in age (of five years), the younger Ricardo found the dance teacher attractive and fell in love with her. He pursued Teresita and they became sweethearts after three months of courtship. They eloped soon after, hastened by the bid of another girlfriend, already pregnant, to get Ricardo to marry her. Ricardo and Teresita were married on September 4, 1978 before Hon. Remigio Zari of the City Court of Quezon City. They begot three children: Richardson, Rachel Anne, and Ric Jayson. On February 1, 1999, more than twenty years after their wedding, Ricardo filed a petition for annulment before the RTC. He claimed that Teresita was psychologically incapacitated to comply with the essential obligations of marriage prior to, at the time of, and subsequent to the celebration of their marriage. He asked the court to declare his marriage to Teresita null and void. At the trial, Ricardo offered in evidence their marriage contract; the psychological evaluation and signature of his expert witness, psychiatrist Dr. Cecilia R. Albaran, and his and Dr. Albarans respective testimonies. Teresita did not file any answer or opposition to the petition, nor did she testify to refute the allegations against her.3 Ricardo alleged in his petition and in his testimony at the trial that Teresita was an adulteress and a squanderer. He was an overseas seaman, and he regularly sent money to his wife to cover the familys living expenses and their childrens tuition. Teresita, however, was not adept in managing the funds he sent and their finances. Many times, Ricardo would come home and be welcomed by debts incurred by his wife; he had to settle these to avoid embarrassment. Aside from neglect in paying debts she incurred from other people, Teresita likewise failed to remit amounts she collected as sales agent of a plasticware and cosmetics company. She left the familys utility bills and their childrens tuition fees unpaid. She also missed paying the rent and the amortization for the house that Ricardo acquired for the family, so their children had to live in a small rented room and eventually had to be taken in by Ricardos parents. When confronted by Ricardo, Teresita would simply offer the excuse that she spent the funds Ricardo sent to buy things for the house and for their children. Ricardo likewise accused Teresita of infidelity and suspected that she was pregnant with another mans child. During one of his visits to the country, he noticed that Teresitas stomach was slightly bigger. He tried to convince her to have a medical examination but she refused. Her miscarriage five months into her pregnancy confirmed his worst suspicions. Ricardo alleged that the child could not have been his, as his three instances of sexual contact with Teresita were characterized by "withdrawals"; other than these, no other sexual contacts with his wife transpired, as he transferred and lived with his relatives after a month of living with Teresita in Cebu. Ricardo reported, too, of rumors that his wife represented herself to others as single, and went out on dates with other men when he was not around. 965

University of the Cordilleras College of Law First Year C S.Y. 2013 - 2014 Ricardo opined that his wife was a very extravagant, materialistic, controlling and demanding person, who mostly had her way in everything; had a taste for the nightlife and was very averse to the duties of a housewife; was stubborn and independent, also most unsupportive, critical and uncooperative; was unresponsive to his hard work and sacrifices for their family; and was most painfully unmindful of him.4 He believed that their marriage had broken down beyond repair and that they both have lost their mutual trust and love for one another.5 Dr. Cecilia R. Albaran testified that a major factor that contributed to the demise of the marriage was Teresitas Narcissistic Personality Disorder that rendered her psychologically incapacitated to fulfill her essential marital obligations. To quote Dr. Albaran: Teresita, the respondent[,] has [sic] shown to manifest the following pervasive pattern of behaviors: a sense of entitlement as she expected favorable treatment and automatic compliance to her wishes, being interpersonally exploitative as on several occasions she took advantage of him to achieve her own ends, lack of empathy as she was unwilling to recognize her partners [sic] feelings and needs[,] taking into consideration her own feelings and needs only, her haughty and arrogant behavior and attitude and her proneness to blame others for her failures and shortcomings. These patterns of behavior speaks [sic] of a Narcissistic Personality Disorder, which started to manifest in early adulthood. The disorder is considered to be grave and incurable based on the fact that individuals do not recognize the symptoms as it is ego syntonic and they feel there is nothing wrong in them. Because of that[,] they remain unmotivated for treatment and impervious to recovery.6 She based her diagnosis on the information she gathered from her psychological evaluation on Ricardo and Richardson (Ricardo and Teresitas eldest son). She admitted, though, that she did not personally observe and examine Teresita; she sent Teresita a personally-delivered notice for the conduct of a psychiatric evaluation, but the notice remained unanswered. In opposing the petition for annulment, the Office of the Solicitor General (OSG) contended that there was no basis to declare Teresita psychologically incapacitated. It asserted that the psychological evaluation conducted on Ricardo (and his son Richardson) only revealed a vague and general conclusion on these parties personality traits but not on Teresitas psychological makeup. The OSG also argued that the evidence adduced did not clinically identify and sufficiently prove the medical cause of the alleged psychological incapacity. Neither did the evidence indicate that the alleged psychological incapacity existed prior to or at the time of marriage, nor that the incapacity was grave and incurable. The RTC agreed with Ricardo, and annulled his marriage to Teresita. In short, the RTC believed Dr. Albarans psychological evaluation and testimony and, on the totality of Ricardos evidence, found Teresita to be psychologically incapacitated to assume the essential obligations of marriage. The OSG appealed the decision to the CA. The CA reversed the RTC decision and held that the trial courts findings did not satisfy the rules and guidelines set by this Court in Republic v. Court of Appeals and Molina.7 The RTC failed to specifically point out the root illness or defect that caused Teresitas psychological incapacity, and likewise failed to show that the incapacity already existed at the time of celebration of marriage. The CA found that the conclusions from Dr. Albarans psychological evaluation do not appear to have been drawn from well-rounded and fair sources, and dwelt mostly on hearsay statements and rumors. Likewise, the CA found that Ricardos allegations on Teresitas overspending and infidelity do not constitute adequate grounds for declaring the marriage null and void under Article 36 of the Family Code. These allegations, even if true, could only effectively serve as grounds for legal separation or a criminal charge for adultery. THE PETITION AND THE PARTIES ARGUMENTS Ricardo faults the CA for disregarding the factual findings of the trial court, particularly the expert testimony of Dr. Albaran, and submits that the trial court in declaring the nullity of the marriage fully complied with Molina. 966

University of the Cordilleras College of Law First Year C S.Y. 2013 - 2014 In its Comment,8 the OSG argued that the CA correctly reversed the RTCs decision, particularly in its conclusion that Ricardo failed to comply with this Courts guidelines for the proper interpretation and application of Article 36 of the Family Code. Reiterating its earlier arguments below, the OSG asserts that the evidence adduced before the trial court failed to show the gravity, juridical antecedence, or incurability of the psychological incapacity of Teresita, and failed as well to identify and discuss its root cause. The psychiatrist, likewise, failed to show that Teresita was completely unable to discharge her marital obligations due to her alleged Narcissistic Personality Disorder. Ricardos Reply9 reiterated that the RTC decision thoroughly discussed the root cause of Teresitas psychological incapacity and identified it as Narcissistic Personality Disorder. He claimed that sufficient proof had been adduced by the psychiatrist whose expertise on the subject cannot be doubted. Interestingly, Ricardo further argued that alleging the root cause in a petition for annulment under Article 36 of the Family Code is no longer necessary, citing Barcelona v. Court of Appeals.10 These positions were collated and reiterated in the memoranda the parties filed. THE COURTS RULING We find the petition unmeritorious, as the CA committed no reversible error when it set aside the RTCs decision for lack of legal and factual basis. In the leading case of Santos v. Court of Appeals, et al.,11 we held that psychological incapacity under Article 36 of the Family Code must be characterized by (a) gravity, (b) juridical antecedence, and (c) incurability, to be sufficient basis to annul a marriage. The psychological incapacity should refer to "no less than a mental (not physical) incapacity that causes a party to be truly incognitive of the basic marital covenants that concomitantly must be assumed and discharged by the parties to the marriage."12 We further expounded on Article 36 of the Family Code in Molina and laid down definitive guidelines in the interpretation and application of this article. These guidelines incorporate the basic requirements of gravity, juridical antecedence and incurability established in the Santos case, as follows: (1) The burden of proof to show the nullity of the marriage belongs to the plaintiff. Any doubt should be resolved in favor of the existence and continuation of the marriage and against its dissolution and nullity. This is rooted in the fact that both our Constitution and our laws cherish the validity of marriage and unity of the family. Thus, our Constitution devotes an entire Article on the Family, recognizing it "as the foundation of the nation." It decrees marriage as legally "inviolable," thereby protecting it from dissolution at the whim of the parties. Both the family and marriage are to be "protected" by the state. The Family Code echoes this constitutional edict on marriage and the family and emphasizes their permanence, inviolability and solidarity. (2) The root cause of the psychological incapacity must be (a) medically or clinically identified, (b) alleged in the complaint, (c) sufficiently proven by experts and (d) clearly explained in the decision. Article 36 of the Family Code requires that the incapacity must be psychological - not physical, although its manifestations and/or symptoms may be physical. The evidence must convince the court that the parties, or one of them, was mentally or psychically ill to such an extent that the person could not have known the obligations he was assuming, or knowing them, could not have given valid assumption thereof. Although no example of such incapacity need be given here so as not to limit the application of the provision under the principle of ejusdem generis (Salita v. Magtolis, 233 SCRA 100, 108), nevertheless such root cause must be identified as a psychological illness and its incapacitating nature fully explained. Expert evidence may be given by qualified psychiatrists and clinical psychologists.

967

University of the Cordilleras College of Law First Year C S.Y. 2013 - 2014 (3) The incapacity must be proven to be existing at "the time of the celebration" of the marriage. The evidence must show that the illness was existing when the parties exchanged their "I do's." The manifestation of the illness need not be perceivable at such time, but the illness itself must have attached at such moment, or prior thereto. (4) Such incapacity must also be shown to be medically or clinically permanent or incurable. Such incurability may be absolute or even relative only in regard to the other spouse, not necessarily absolutely against everyone of the same sex. Furthermore, such incapacity must be relevant to the assumption of marriage obligations, not necessarily to those not related to marriage, like the exercise of a profession or employment in a job. Hence, a pediatrician may be effective in diagnosing illnesses of children and prescribing medicine to cure them but may not be psychologically capacitated to procreate, bear and raise his/her own children as an essential obligation of marriage. (5) Such illness must be grave enough to bring about the disability of the party to assume the essential obligations of marriage. Thus, "mild characteriological peculiarities, mood changes, occasional emotional outbursts" cannot be accepted as root causes. The illness must be shown as downright incapacity or inability, not a refusal, neglect or difficulty, much less ill will. In other words, there is a natal or supervening disabling factor in the person, an adverse integral element in the personality structure that effectively incapacitates the person from really accepting and thereby complying with the obligations essential to marriage. (6) The essential marital obligations must be those embraced by Articles 68 up to 71 of the Family Code as regards the husband and wife as well as Articles 220, 221 and 225 of the same Code in regard to parents and their children. Such non-complied marital obligation(s) must also be stated in the petition, proven by evidence and included in the text of the decision. (7) Interpretations given by the National Appellate Matrimonial Tribunal of the Catholic Church in the Philippines, while not controlling or decisive, should be given great respect by our courts.13 Subsequent jurisprudence on psychological incapacity applied these basic guidelines to varying factual situations, thus confirming the continuing doctrinal validity of Santos. In so far as the present factual situation is concerned, what should not be lost in reading and applying our established rulings is the intent of the law to confine the application of Article 36 of the Family Code to the most serious cases of personality disorders; these are the disorders that result in the utter insensitivity or inability of the afflicted party to give meaning and significance to the marriage he or she contracted. Furthermore, the psychological illness and its root cause must have been there from the inception of the marriage. From these requirements arise the concept that Article 36 of the Family Code does not really dissolve a marriage; it simply recognizes that there never was any marriage in the first place because the affliction already then existing was so grave and permanent as to deprive the afflicted party of awareness of the duties and responsibilities of the matrimonial bond he or she was to assume or had assumed.14

In the present case and guided by these standards, we find the totality of the petitioners evidence to be insufficient to prove that Teresita was psychologically incapacitated to perform her duties as a wife. As already mentioned, the evidence presented consisted of the testimonies of Ricardo and Dr. Albaran, and the latters psychological evaluation of Ricardo and Richardson from where she derived a psychological evaluation of Teresita. a. Dr. Albarans psychological evaluation and testimony Dr. Albaran concluded in her psychological evaluation that Teresita suffers from Narcissistic Personality Disorder that rendered her psychologically incapacitated to assume essential marital obligations. To support her findings and conclusion, she banked on the statements told to her by Ricardo and Richardson,

968

University of the Cordilleras College of Law First Year C S.Y. 2013 - 2014 which she narrated in her evaluation. Apparently relying on the same basis, Dr. Albaran added that Teresitas disorder manifested during her early adulthood and is grave and incurable. To say the least, we are greatly disturbed by the kind of testimony and evaluation that, in this case, became the basis for the conclusion that no marriage really took place because of the psychological incapacity of one of the parties at the time of marriage. We are in no way convinced that a mere narration of the statements of Ricardo and Richardson, coupled with the results of the psychological tests administered only on Ricardo, without more, already constitutes sufficient basis for the conclusion that Teresita suffered from Narcissistic Personality Disorder. This Court has long been negatively critical in considering psychological evaluations, presented in evidence, derived solely from one-sided sources, particularly from the spouse seeking the nullity of the marriage. In So v. Valera,15 the Court considered the psychologists testimony and conclusions to be insufficiently in-depth and comprehensive to warrant the finding of respondents psychological incapacity because the facts, on which the conclusions were based, were all derived from the petitioners statements whose bias in favor of his cause cannot be discounted. In another case, Padilla-Rumbaua v. Rumbaua,16 the Court declared that while the various tests administered on the petitioner-wife could have been used as a fair gauge to assess her own psychological condition, this same statement could not be made with respect to the respondent-husbands psychological condition. To our mind, conclusions and generalizations about Teresitas psychological condition, based solely on information fed by Ricardo, are not any different in kind from admitting hearsay evidence as proof of the truthfulness of the content of such evidence.17 To be sure, we have recognized that the law does not require that the allegedly incapacitated spouse be personally examined by a physician or by a psychologist as a condition sine qua non for the declaration of nullity of marriage under Article 36 of the Family Code.18 This recognition, however, does not signify that the evidence, we shall favorably appreciate, should be any less than the evidence that an Article 36 case, by its nature, requires. Our recognition simply means that the requirements for nullity outlined in Santos and Molina need not necessarily come from the allegedly incapacitated spouse. In other words, it is still essential although from sources other than the respondent spouse to show his or her personality profile, or its approximation, at the time of marriage; the root cause of the inability to appreciate the essential obligations of marriage; and the gravity, permanence and incurability of the condition. Other than from the spouses, such evidence can come from persons intimately related to them, such as relatives, close friends or even family doctors or lawyers who could testify on the allegedly incapacitated spouses condition at or about the time of marriage, or to subsequent occurring events that trace their roots to the incapacity already present at the time of marriage. In the present case, the only other party outside of the spouses who was ever asked to give statements for purposes of Teresitas psychological evaluation was Richardson, the spouses eldest son who would not have been very reliable as a witness in an Article 36 case because he could not have been there when the spouses were married and could not have been expected to know what was happening between his parents until long after his birth. We confirm the validity of this observation from a reading of the summary of Richardsons interview with the pyschologist: Richardsons statement occupied a mere one paragraph (comprising eleven sentences) in the psychological evaluation and merely recited isolated instances of his parents fighting over the foreclosure of their house, his fathers alleged womanizing, and their differences in religion (Ricardo is a Catholic, while Teresita is a Mormon).19 We find nothing unusual in these recited marital incidents to indicate that Teresita suffered from some psychological disorder as far back as the time of her marriage to Ricardo, nor do we find these fights to be indicative of problems traceable to any basic psychological disorder existing at the time of marriage. For one, these points of dispute are not uncommon in a marriage and relate essentially to the usual roots of marital problems finances, fidelity and religion. The psychologist, too, never delved into the 969

University of the Cordilleras College of Law First Year C S.Y. 2013 - 2014 relationship between mother and son except to observe their estranged relationship due to a previous argument a money problem involving Ricardos financial remittances to the family. To state the obvious, the psychologists evaluation never explained how the recited incidents, made by one who was not even born at the time of the spouses marriage, showed a debilitating psychological incapacity already existing at that time.

Of more serious consequence, fatal to Ricardos cause, is the failure of Dr. Albarans psychological evaluation to fully explain the details i.e., the what, how, when, where and since when of Teresitas alleged Narcissistic Personality Disorder. It seems to us that, with hardly any supporting evidence to fall back on, Dr. Albaran simply stated out of the blue that Teresitas personality disorder manifested itself in early adulthood, presuming thereby that the incapacity should have been there when the marriage was celebrated. Dr. Albaran never explained, too, the incapacitating nature of Teresitas alleged personality disorder, and how it related to the essential marital obligations that she failed to assume. Neither did the good doctor adequately explain in her psychological evaluation how grave and incurable was Teresitas psychological disorder. Dr. Albarans testimony at the trial did not improve the evidentiary situation for Ricardo, as it still failed to provide the required insights that would have remedied the evidentiary gaps in her written psychological evaluation. In fact, Dr. Albarans cross-examination only made the evidentiary situation worse when she admitted that she had difficulty pinpointing the root cause of Teresitas personality disorder, due to the limited information she gathered from Ricardo and Richardson regarding Teresitas personal and family history. To directly quote from the records, Dr. Albaran confessed this limitation when she said that "[t]he only data that I have is that, the respondent seem [sic] to have grown from a tumultuous family and this could be perhaps the [sic] contributory to the development of the personality disorder."20 Dr. Albarans obvious uncertainty in her assessment only proves our point that a complete personality profile of the spouse, alleged to be psychologically incapacitated, could not be determined from meager information coming only from a biased source. b. Ricardos testimony Ricardo testified in court that Teresita was a squanderer and an adulteress. We do not, however, find Ricardos characterizations of his wife sufficient to constitute psychological incapacity under Article 36 of the Family Code. Article 36 contemplates downright incapacity or inability to take cognizance of and to assume basic marital obligations. Mere "difficulty," "refusal, or "neglect" in the performance of marital obligations or "ill will" on the part of the spouse is different from "incapacity" rooted on some debilitating psychological condition or illness.21 Ricardos testimony merely established that Teresita was irresponsible in managing the familys finances by not paying their rent, utility bills and other financial obligations. Teresitas spendthrift attitude, according to Ricardo, even resulted in the loss of the house and lot intended to be their family residence. This kind of irresponsibility, however, does not rise to the level of a psychological incapacity required under Article 36 of the Family Code. At most, Teresitas mismanagement of the familys finances merely constituted difficulty, refusal or neglect, during the marriage, in the handling of funds intended for the familys financial support. Teresitas alleged infidelity, even if true, likewise does not constitute psychological incapacity under Article 36 of the Family Code. In order for sexual infidelity to constitute as psychological incapacity, the respondents unfaithfulness must be established as a manifestation of a disordered personality, completely preventing the respondent from discharging the essential obligations of the marital state;22 there must be proof of a natal or supervening disabling factor that effectively incapacitated her from complying with the obligation to be faithful to her spouse.23 In our view, Ricardo utterly failed in his testimony to prove that Teresita suffered from a disordered personality of this kind. Even Ricardos added testimony, relating to rumors of Teresitas dates with other 970

University of the Cordilleras College of Law First Year C S.Y. 2013 - 2014 men and her pregnancy by another man, would not fill in the deficiencies we have observed, given the absence of an adverse integral element and link to Teresitas allegedly disordered personality. Moreover, Ricardo failed to prove that Teresitas alleged character traits already existed at the inception of their marriage. Article 36 of the Family Code requires that the psychological incapacity must exist at the time of the celebration of the marriage, even if such incapacity becomes manifest only after its solemnization.24 In the absence of this element, a marriage cannot be annulled under Article 36. Root cause of the psychological incapacity needs to be alleged in a petition for annulment under Article 36 of the Family Code Citing Barcelona,25 Ricardo defended the RTC decision, alleging that the root cause in a petition for annulment under Article 36 of the Family Code is no longer necessary. We find this argument completely at variance with Ricardos main argument against the assailed CA decision i.e., that the RTC, in its decision, discussed thoroughly the root cause of Teresitas psychological incapacity as Narcissistic Personality Disorder. These conflicting positions, notwithstanding, we see the need to address this issue to further clarify our statement in Barcelona, which Ricardo misquoted and misinterpreted to support his present petition that "since the new Rules do not require the petition to allege expert opinion on the psychological incapacity, it follows that there is also no need to allege in the petition the root cause of the psychological incapacity."26 In Barcelona, the petitioner assailed the bid for annulment for its failure to state the "root cause" of the respondents alleged psychological incapacity. The Court resolved this issue, ruling that the petition sufficiently stated a cause of action because the petitioner instead of stating a specific root cause clearly described the physical manifestations indicative of the psychological incapacity. This, the Court found to be sufficiently compliant with the first requirement in the Molina case that the "root cause" of the psychological incapacity be alleged in an Article 36 petition. Thus, contrary to Ricardos position, Barcelona does not do away with the "root cause" requirement. The ruling simply means that the statement of the root cause does not need to be in medical terms or be technical in nature, as the root causes of many psychological disorders are still unknown to science. It is enough to merely allege the physical manifestations constituting the root cause of the psychological incapacity. Section 2, paragraph (d) of the Rule on Declaration of Absolute Nullity of Void Marriages and Annulment of Voidable Marriages (Rules)27 in fact provides: SEC. 2. Petition for declaration of absolute nullity of void marriages. xxxx (d) What to allege.- A petition under Article 36 of the Family Code shall specially allege the complete facts showing that either or both parties were psychologically incapacitated from complying with the essential marital obligations of marriages at the time of the celebration of marriage even if such incapacity becomes manifest only after its celebration. The complete facts should allege the physical manifestations, if any, as are indicative of psychological incapacity at the time of the celebration of the marriage but expert opinion need not be alleged. As we explained in Barcelona, the requirement alleging the root cause in a petition for annulment under Article 36 of the Family Code was not dispensed with by the adoption of the Rules. What the Rules really eliminated was the need for an expert opinion to prove the root cause of the psychological incapacity. The Court further held that the Rules, being procedural in nature, apply only to actions pending and unresolved at the time of their adoption.1avvphi1 To sum up, Ricardo failed to discharge the burden of proof to show that Teresita suffered from psychological incapacity; thus, his petition for annulment of marriage must fail. Ricardo merely established that Teresita had been remiss in her duties as a wife for being irresponsible in taking care of 971

University of the Cordilleras College of Law First Year C S.Y. 2013 - 2014 their familys finances a fault or deficiency that does not amount to the psychological incapacity that Article 36 of the Family Code requires. We reiterate that irreconcilable differences, sexual infidelity or perversion, emotional immaturity and irresponsibility, and the like, do not by themselves warrant a finding of psychological incapacity, as the same may only be due to a persons difficulty, refusal or neglect to undertake the obligations of marriage that is not rooted in some psychological illness that Article 36 of the Family Code addresses.28 WHEREFORE, premises considered, we DENY the petition and AFFIRM the decision of the Court of Appeals in CA-G.R. CV No. 71882. Costs against the petitioner. SO ORDERED.

972

University of the Cordilleras College of Law First Year C S.Y. 2013 - 2014 Case Digest RICARDO P. TORING vs. TERESITA M. TORING and REPUBLIC OF THEPHILIPPINES, August 3, 2010 G.R. No. 165321 THE FACTS: Ricardo was introduced to Teresita in 1978 at his aunts house in Cebu. Teresita was then his cousins teacher in Hawaiian dance and was conducting lessons at his aunts house. Despite their slight difference in age (of five years), the younger Ricardo found the dance teacher attractive and fell in love with her. He pursued Teresita and they became sweethearts after three months of courtship. They eloped soon after, hastened by the bid of another girlfriend, already pregnant, to get Ricardo to marry her. Ricardo and Teresita were married on September 4, 1978 before Hon. Remigio Zari of the City Court of Quezon City. They begot three children: Richardson, Rachel Anne, and Ric Jayson. On February 1, 1999, more than twenty years after their wedding, Ricardo filed a petition for annulment before the RTC. He claimed that Teresita was psychologically incapacitated to comply with the essential obligations of marriage prior to, at the time of, and subsequent to the celebration of their marriage. He asked the court to declare his marriage to Teresita null and void. At the trial, Ricardo offered in evidence their marriage contract; the psychological evaluation and signature of his expert witness, psychiatrist Dr. Cecilia R. Albaran, and his and Dr. Albarans respective testimonies. Teresita did not file any answer or opposition to the petition, nor did she testify to refute the allegations against her.[3] Ricardo alleged in his petition and in his testimony at the trial that Teresita was an adulteress and a squanderer. He was an overseas seaman, and he regularly sent money to his wife to cover the familys living expenses and their childrens tuition. Teresita, however, was not adept in managing the funds he sent and their finances. Many times, Ricardo would come home and be welcomed by debts incurred by his wife; he had to settle these to avoid embarrassment. Aside from neglect in paying debts she incurred from other people, Teresita likewise failed to remit amounts she collected as sales agent of a plasticware and cosmetics company. She left the familys utility bills and their childrens tuition fees unpaid. She also missed paying the rent and the amortization for the house that Ricardo acquired for the family, so their children had to live in a small rented room and eventually had to be taken in by Ricardos parents. When confronted by Ricardo, Teresita would simply offer the excuse that she spent the funds Ricardo sent to buy things for the house and for their children. Ricardo likewise accused Teresita of infidelity and suspected that she was pregnant with another mans child. During one of his visits to the country, he noticed that Teresitas stomach was slightly bigger. He tried to convince her to have a medical examination but she refused. Her miscarriage five months into her pregnancy confirmed his worst suspicions. Ricardo alleged that the child could not have been his, as his three instances of sexual contact with Teresita were characterized by withdrawals; other than these, no other sexual contacts with his wife transpired, as he transferred and lived with his relatives after a month of living with Teresita 973

University of the Cordilleras College of Law First Year C S.Y. 2013 - 2014 in Cebu. Ricardo reported, too, of rumors that his wife represented herself to others as single, and went out on dates with other men when he was not around. Ricardo opined that his wife was a very extravagant, materialistic, controlling and demanding person, who mostly had her way in everything; had a taste for the nightlife and was very averse to the duties of a housewife; was stubborn and independent, also most unsupportive, critical and uncooperative; was unresponsive to his hard work and sacrifices for their family; and was most painfully unmindful of him. He believed that their marriage had broken down beyond repair and that they both have lost their mutual trust and love for one another. Dr. Cecilia R. Albaran testified that a major factor that contributed to the demise of the marriage was Teresitas Narcissistic Personality Disorder that rendered her psychologically incapacitated to fulfill her essential marital obligations. ISSUE: Does the allegations of Ricardo on Teresitas overspending and infidelity constitute adequate grounds for declaring the marriage null and void under Article 36 of the Family Code. RULING: No. We find the petition unmeritorious, as the CA committed no reversible error when it set aside the RTCs decision for lack of legal and factual basis. In the leading case of Santos v. Court of Appeals, et al.,[11] we held that psychological incapacity under Article 36 of the Family Code must be characterized by (a) gravity, (b) juridical antecedence, and (c) incurability, to be sufficient basis to annul a marriage. The psychological incapacity should refer to no less than a mental (not physical) incapacity that causes a party to be truly incognitive of the basic marital covenants that concomitantly must be assumed and discharged by the parties to the marriage.[12] In the present case and guided by these standards, we find the totality of the petitioners evidence to be insufficient to prove that Teresita was psychologically incapacitated to perform her duties as a wife. As already mentioned, the evidence presented consisted of the testimonies of Ricardo and Dr. Albaran, and the latters psychological evaluation of Ricardo and Richardson from where she derived a psychological evaluation of Teresita.

974

University of the Cordilleras College of Law First Year C S.Y. 2013 - 2014 MYRNA P. ANTONE VS. LEO R. BERONILLA, G.R. No. 183824 December 8, 2010 Full Case MYRNA P. ANTONE, VS LEO R. BERONILLA, PEREZ, J.:

Before us is a petition for review on certiorari under Rule 45 of the Rules of Court seeking to nullify and set aside the issuances of the Court of Appeals in CA-G.R. SP No. 102834, to wit: (a) the Resolution[1] dated 29 April 2008 dismissing the petition for certiorari under Rule 65, which assailed the trial courts Orders[2] dated 20 September 2007 and 6 December 2007 in Criminal Case No. 07-0907CFM for Bigamy; and (b) the Resolution[3] dated 18 July 2008 denying the motion for reconsideration of the first resolution. The trial court quashed the Information on the ground that the elements of Bigamy were rendered incomplete after herein respondent presented documents to prove a fact, which the court believed would negate the allegation in the Information that there was a first valid marriage. The evidence presented showed that respondent later obtained a judicial declaration of nullity of the first union following the celebration of a subsequent marriage. The Antecedents On 12 March 2007, herein petitioner Myrna P. Antone executed an Affidavit-Complaint[4] for Bigamy against Leo R. Beronilla before the Office of the City Prosecutor of Pasay City. She alleged that her marriage with respondent in 1978 had not yet been legally dissolved when the latter contracted a second marriage with one Cecile Maguillo in 1991. On 21 June 2007, the prosecution filed the corresponding Information[5] before the Regional Trial Court, Pasay City. The case was docketed as Criminal Case No. 07-0907-CFM and raffled to Branch 115. Pending the setting of the case for arraignment, herein respondent moved to quash the Information on the ground that the facts charged do not constitute an offense.[6] He informed the court that his marriage with petitioner was declared null and void by the Regional Trial Court, Branch 16, Naval, Biliran on 26 April 2007;[7] that the decision became final and executory on 15 May 200[7];[8] and that such decree has already been registered with the Municipal Civil Registrar on 12 June 2007. [9] He argued that since the marriage had been declared null and void from the beginning, there was actually no first marriage to speak of. Absent a first valid marriage, the facts alleged in the Information do not constitute the crime of bigamy.[10] In its comment/opposition to the motion,[11] the prosecution, through herein petitioner, maintained that the respondent committed an act which has all the essential requisites of bigamy. The prosecution 975

University of the Cordilleras College of Law First Year C S.Y. 2013 - 2014 pointed out that the marriage of petitioner and respondent on 18 November 1978 has not yet been severed when he contracted a second marriage on 16 February 1991, for which reason, bigamy has already been committed before the court declared the first marriage null and void on 27 April 2007.[12] The prosecution also invoked the rulings of the Supreme Court holding that a motion to quash is a hypothetical admission of the facts alleged in the information, and that facts contrary thereto are matters of defense which may be raised only during the presentation of evidence.[13] After a hearing on the motion,[14] the court quashed the Information.[15] Applying Morigo v. People,[16] it ruled: Hence, contrary to what was stated in the Information, accused Beronilla was actually never legally married to Myrna Antone. On this score alone, the first element appears to be missing. Furthermore, the statement in the definition of Bigamy which reads before the first marriage has been legally dissolved clearly contemplates that the first marriage must at least be annullable or voidable but definitely not void, as in this case. xxx [I]n a similar case, [the Supreme Court] had the occasion to state: The first element of bigamy as a crime requires that the accused must have been legally married. But in this case, legally speaking, the petitioner was never married to Lucia Barrete. Thus, there is no first marriage to speak of. Under the principle of retroactivity of a marriage being declared void ab initio, the two were never married from the beginning. xxx The existence and the validity of the first marriage being an essential element of the crime of bigamy, it is but logical that a conviction for said offense cannot be sustained where there is no first marriage to speak of. xxx[17] The prosecution, through herein petitioner, moved for reconsideration of the said Order[18] on the ground, among others, that the facts and the attending circumstances inMorigo are not on all fours with the case at bar. It likewise pointed out that, in Mercado v. Tan,[19] this Court has already settled that (a) declaration of the absolute nullity of a marriage is now explicitly required either as a cause of action or a ground for defense.[20] In its Order of 6 December 2007,[21] the court denied the motion for reconsideration stating that Mercado has already been superseded by Morigo. In the interim, in a Petition for Relief from Judgment[22] before the Regional Trial Court of Naval, Biliran, petitioner questioned the validity of the proceedings in the petition for the declaration of nullity of marriage in Civil Case No. B-1290 on 5 October 2007. On 24 March 2008, the court set aside its Decision of 26 April 2007 declaring the marriage of petitioner with respondent null and void, and required herein petitioner (respondent in Civil Case No. B-1290) to file her answer to the complaint.[23] On 21 July 2008, the court DISMISSED the petition for nullity of marriage for failure of herein respondent (plaintiff in Civil Case No. B-1290) to submit his pre-trial brief.[24] Respondent, however, challenged the orders issued by the court before the Court of Appeals.[25] The matter is still pending resolution thereat.[26] Meanwhile, in a petition for certiorari under Rule 65 of the Rules of Court filed on 26 March 2008 before the Court of Appeals,[27] herein petitioner alleged that the Pasay City trial court acted without 976

University of the Cordilleras College of Law First Year C S.Y. 2013 - 2014 or in excess of jurisdiction or with grave abuse of discretion amounting to lack or excess of jurisdiction when it dismissed the case of bigamy and denied her motion for reconsideration. In its Resolution of 29 April 2008, the Court of Appeals dismissed the petition stating that: The present petition xxx is fatally infirm in form and substance for the following reasons: 1. The verification is defective as it does not include the assurance that the allegations in the petition are based on authentic records. 2. Since the petition assails the trial courts dismissal of the criminal information for bigamy filed against private respondent Leo Beronilla, the petition, if at all warranted, should be filed in behalf of the People of the Philippines by the Office of the Solicitor General, being its statutory counsel in all appealed criminal cases. 3. There is a violation of the rule on double jeopardy as the dismissal of the subject criminal case is tantamount to an acquittal based on the trial courts finding that the first essential element of bigamy, which is a first valid marriage contracted by private respondent is wanting. There is no clear showing in the petition that the dismissal was tainted with arbitrariness which violated petitioners right to due process. Notably, petitioner filed her comment/opposition to private respondents motion to quash before the trial court issued its Order dated September 20, 2007 dismissing the information. Hence, if there is no denial of due process, there can be no grave abuse of discretion that would merit the application of the exception to the double jeopardy rule. [28] On 18 July 2008, the Court of Appeals denied respondents Motion for Reconsideration of the aforequoted Resolution for lack of merit. [29] Hence, this petition.[30] Our Ruling I We are convinced that this petition should be given due course despite the defect in the pleading and the question of legal standing to bring the action. The Rules of Court provides that a pleading required to be verified which lacks a proper verification shall be treated as unsigned pleading.[31] This, notwithstanding, we have, in a number of cases, opted to relax the rule in order that the ends of justice may be served.[32] The defect being merely formal and not jurisdictional, we ruled that the court may nevertheless order the correction of the pleading, or even act on the pleading if the attending circumstances are such that xxx strict compliance with the rule may be dispensed with in order that the ends of justice xxx may be served.[33] At any rate, a pleading is required to be verified only to ensure that it was prepared in good faith, and that the allegations were true and correct and not based on mere speculations.[34] There is likewise no dispute that it is the Office of the Solicitor General (OSG) which has the authority to represent the government in a judicial proceeding before the Court of Appeals. The Administrative Code specifically defined its powers and functions to read, among others:

977

University of the Cordilleras College of Law First Year C S.Y. 2013 - 2014 Sec. 35. Powers and Functions. - The Office of the Solicitor General shall represent the Government of the Philippines, its agencies and instrumentalities and its officials and agents in any litigation, proceeding, investigation or matter requiring the services of lawyers. xxx It shall have the following specific powers and functions: (1) Represent the Government in the Supreme Court and the Court of Appeals in all criminal proceedings; represent the Government and its officers in the Supreme Court, Court of Appeals, and all other courts or tribunals in all civil actions and special proceedings in which the Government or any officer thereof in his official capacity is a party.[35] As an exception to this rule, the Solicitor General is allowed to: (8) Deputize legal officers of government departments, bureaus, agencies and offices to assist the Solicitor General and appear or represent the Government in cases involving their respective offices, brought before the courts and exercise supervision and control over such legal officers with respect to such cases.[36] Thus, in Republic v. Partisala,[37] we held that the summary dismissal of an action in the name of the Republic of the Philippines, when not initiated by the Solicitor General, is in order.[38] Not even the appearance of the conformity of the public prosecutor in a petition for certiorari would suffice because the authority of the City Prosecutor or his assistant to represent the People of the Philippines is limited to the proceedings in the trial court.[39] We took exceptions, however, and gave due course to a number of actions even when the respective interests of the government were not properly represented by the Office of the Solicitor General. In Labaro v. Panay,[40] this Court dealt with a similar defect in the following manner: It must, however, be stressed that if the public prosecution is aggrieved by any order or ruling of the trial judge in a criminal case, the OSG, and not the prosecutor, must be the one to question the order or ruling before us.[41] xxx Nevertheless, since the challenged order affects the interest of the State or the plaintiff People of the Philippines, we opted not to dismiss the petition on this technical ground. Instead, we required the OSG to comment on the petition, as we had done before in some cases.[42] In light of its Comment, we rule that the OSG has ratified and adopted as its own the instant petition for the People of the Philippines. (Emphasis supplied.)

In Cooperative Development Authority v. Dolefil Agrarian Reform Beneficiaries Cooperative, Inc.,


[43]

without requiring the Office of the Solicitor General to file a comment on the petition, this Court

determined the merits of the case involving a novel issue on the nature and scope of jurisdiction of the Cooperative Development Authority to settle cooperative disputes as well as the battle between two (2) factions concerning the management of the Dolefil Agrarian Reform Beneficiaries Cooperative, Inc. (DARBCI) that inevitably threatens the very existence of one of the countrys major cooperatives.[44] And, lest we defeat the ends of justice, we opt to look into the merit of the instant petition even absent the imprimatur of the Solicitor General. After all, for justice to prevail, the scales must balance, for justice is not to be dispensed for the accused alone.[45] To borrow the words of then Justice Minita V. 978

University of the Cordilleras College of Law First Year C S.Y. 2013 - 2014 Chico-Nazario in another case where the dismissal of a criminal case pending with the trial court was sought: [T]he task of the pillars of the criminal justice system is to preserve our democratic society under the rule of law, ensuring that all those who [come or are brought to court] are afforded a fair opportunity to present their side[s]. xxx The State, like any other litigant, is entitled to its day in court, and to a reasonable opportunity to present its case.[46] II We cannot agree with the Court of Appeals that the filing of this petition is in violation of the respondents right against double jeopardy on the theory that he has already been practically acquitted when the trial court quashed the Information. Well settled is the rule that for jeopardy to attach, the following requisites must concur: (1) there is a complaint or information or other formal charge sufficient in form and substance to sustain a conviction; (2) the same is filed before a court of competent jurisdiction; (3) there is a valid arraignment or plea to the charges; and (4) the accused is convicted or acquitted or the case is otherwise dismissed or terminated without his express consent.[47] The third and fourth requisites are clearly wanting in the instant case as (a) respondent has not yet entered his plea to the charge when he filed the Motion to Quash the Information, and (2) the case was dismissed not merely with his consent but, in fact, at his instance.[48] We reiterate, time and again, that jeopardy does not attach in favor of the accused on account of an order sustaining a motion to quash.[49] More specifically, the granting of a motion to quash anchored on the ground that the facts charged do not constitute an offense is not a bar to another prosecution for the same offense.[50] Thus: It will be noted that the order sustaining the motion to quash the complaint against petitioner was based on Subsection (a) of Section 2 of Rule 117 of the Rules of Court that the facts charged in the complaint do not constitute an offense. If this is so then the dismissal of said complaint will not be a bar to another prosecution for the same offense, for it is provided in Section 8 of Rule 117 of the Rules of Court [now Section 6 of the 2000 Rules of Criminal Procedure] that an order sustaining the motion to quash is not a bar to another prosecution for the same offense unless the motion was based on the grounds specified in Section 2, Subsection[s] (f) and (h) of this rule [now substantially reproduced in Section 3, Subsections (g) and (i) of the 2000 Rules of Criminal Procedure] xxx.[51] III We now determine the merit of the petition did the trial court act without or in excess of jurisdiction or grave abuse of discretion when it sustained respondents motion to quash on the basis of a fact contrary to those alleged in the information? Petitioner maintains that the trial court did so because the motion was a hypothetical admission of the facts alleged in the information and any evidence contrary thereto can only be presented as a matter of defense during trial. Consistent with existing jurisprudence, we agree with the petitioner.

979

University of the Cordilleras College of Law First Year C S.Y. 2013 - 2014 We define a motion to quash an Information as the mode by which an accused assails the validity of a criminal complaint or Information filed against him for insufficiency on its face in point of law, or for defects which are apparent in the face of the Information.[52] This motion is a hypothetical admission of the facts alleged in the Information, [53] for which reason, the court cannot consider allegations contrary to those appearing on the face of the information.[54] As further elucidated in Cruz, Jr. v. Court of Appeals:[55] It is axiomatic that a complaint or information must state every single fact necessary to constitute the offense charged; otherwise, a motion to dismiss/quash on the ground that it charges no offense may be properly sustained. The fundamental test in considering a motion to quash on this ground is whether the facts alleged, if hypothetically admitted, will establish the essential elements of the offense as defined in the law. Contrary to the petitioners contention, a reading of the information will disclose that the essential elements of the offense charged are sufficiently alleged. It is not proper therefore to resolve the charges at the very outset, in a preliminary hearing only and without the benefit of a full-blown trial. The issues require a fuller examination. Given the circumstances of this case, we feel it would be unfair to shut off the prosecution at this stage of the proceedings and to dismiss the informations on the basis only of the petitioners evidence, such as [this].[56] As in the recent case of Los Baos v. Pedro,[57] where we found no merit in respondents allegation that the facts charged do not constitute an offense because the Information duly charged a specific offense and provide[d] the details on how the offense was committed, [58] we see no apparent defect in the allegations in the Information in the case at bar. Clearly, the facts alleged in its accusatory portion, which reads: That on or about the 16th day of February, 1991, in Pasay City, Metro Manila, Philippines and within the jurisdiction of this Honorable Court, the above-named accused, LEO R. BERONILLA, having been united in a lawful marriage with one MYRNA A. BERONILLA, which marriage is still in force and subsisting and without having been legally dissolved, did then and there willfully, unlawfully and feloniously contract a second marriage with one Cecile Maguillo, which subsequent marriage of the accused has all the essential requisites for validity.[59] sufficiently constitute an offense. It contained all the elements of the crime of Bigamy under Article 349 of the Revised Penal Code hereunder enumerated: (1) that the offender has been legally married; (2) that the first marriage has not been legally dissolved or, in case his or her spouse is absent, the absent spouse could not yet be presumed dead according to the Civil Code; (3) that he contracts a second or subsequent marriage; and (4) that the second or subsequent marriage has all the essential requisites for validity.[60]

980

University of the Cordilleras College of Law First Year C S.Y. 2013 - 2014 The documents showing that: (1) the court has decreed that the marriage of petitioner and respondent is null and void from the beginning; and (2) such judgment has already become final and executory and duly registered with the Municipal Civil Registrar of Naval, Biliran are pieces of evidence that seek to establish a fact contrary to that alleged in the Information that a first valid marriage was subsisting at the time the respondent contracted a subsequent marriage. This should not have been considered at all because matters of defense cannot be raised in a motion to quash. Neither do we find a justifiable reason for sustaining the motion to quash even after taking into consideration the established exceptions to the rule earlier recognized by this Court, among others: (1) when the new allegations are admitted by the prosecution;[61] (2) when the Rules so permit, such as upon the grounds of extinction of criminal liability and double jeopardy;[62] and (3) when facts have been established by evidence presented by both parties which destroyed the prima facie truth of the allegations in the information during the hearing on a motion to quash based on the ground that the facts charged do not constitute an offense, and it would be pure technicality for the court to close its eyes to said facts and still give due course to the prosecution of the case already shown to be weak even to support possible conviction xxx.[63] For of what significance would the document showing the belated dissolution of the first marriage offer? Would it serve to prevent the impracticability of proceeding with the trial in accordance with People v. dela Rosa thereby warranting the non-observance of the settled rule that a motion to quash is a hypothetical admission of the facts alleged in the information? We quote: [W]here in the hearing on a motion to quash predicated on the ground that the allegations of the information do not charge an offense, facts have been brought out by evidence presented by both parties which destroy the prima facie truth accorded to the allegations of the information on the hypothetical admission thereof, as is implicit in the nature of the ground of the motion to quash, it would be pure technicality for the court to close its eyes to said facts and still give due course to the prosecution of the case already shown to be weak even to support possible conviction, and hold the accused to what would clearly appear to be a merely vexatious and expensive trial, on her part, and a wasteful expense of precious time on the part of the court, as well as of the prosecution.[64] (Emphasis supplied.) We find that there is none. With the submission of the documents showing that the court has declared the first marriage void ab initio, respondent heavily relied on the rulings[65] in People v. Mendoza and Morigo declaring that: (a) a case for bigamy based on a void ab initio marriage will not prosper because there is no need for a judicial decree to establish that a voidab initio marriage is invalid;[66] and (b) a marriage declared void ab initio has retroactive legal effect such that there would be no first valid marriage to speak of after all, which renders the elements of bigamy incomplete.[67] Both principles, however, run contrary to the new provision of the Family Code, which was promulgated by the late President Corazon C. Aquino in 1987, a few years before respondents subsequent marriage was celebrated in 1991. The specific provision, which reads: 981

University of the Cordilleras College of Law First Year C S.Y. 2013 - 2014 ART. 40. The absolute nullity of a previous marriage may be invoked for purposes of remarriage on the basis solely of a final judgment declaring such marriage void. was exhaustively discussed in Mercado,[68] where this Court settled the conflicting jurisprudence on the need for a judicial declaration of nullity of the previous marriage. After establishing that Article 40 is a new provision expressly requiring a judicial declaration of nullity of a prior marriage and examining a long line of cases,[69] this Court, concluded, in essence, that under the Family Code a subsequent judicial declaration of the nullity of the first marriage is immaterial in a bigamy case because, by then, the crime had already been consummated. Otherwise stated, this Court declared that a person, who contracts a subsequent marriage absent a prior judicial declaration of nullity of a previous one, is guilty of bigamy.[70] Notably, Morigo, was indeed promulgated years after Mercado. Nevertheless, we cannot uphold the Order dated 6 December 2007 of the trial court, which maintained that Morigo has already superseded Mercado. In fact, in Morigo, this Court clearly distinguished the two (2) cases from one another, and explained: The present case is analogous to, but must be distinguished from Mercado v. Tan. In the latter case, the judicial declaration of nullity of the first marriage was likewise obtained after the second marriage was already celebrated. xxx It bears stressing though that in Mercado, the first marriage was actually solemnized xxx. Ostensibly, at least, the first marriage appeared to have transpired, although later declared void ab initio. In the instant case, however, no marriage ceremony at all was performed by a duly authorized solemnizing officer. Petitioner and Lucia Barrete merely signed a marriage contract on their own. The mere private act of signing a marriage contract bears no semblance to a valid marriage and thus, needs no judicial declaration of nullity. Such act alone, without more, cannot be deemed to constitute an ostensibly valid marriage for which petitioner might be held liable for bigamy unless he first secures a judicial declaration of nullity before he contracts a subsequent marriage.[71] The application of Mercado to the cases following Morigo even reinforces the position of this Court to give full meaning to Article 40 of the Family Code. Thus, in 2004, this Court ruled in Tenebro v. Court of Appeals:[72] Although the judicial declaration of the nullity of a marriage on the ground of psychological incapacity retroacts to the date of the celebration of the marriage insofar as the vinculum between the spouses is concerned, xxx said marriage is not without legal effects. Among these effects is that children conceived or born before the judgment of absolute nullity of the marriage shall be considered legitimate. There is therefore a recognition written into the law itself that such a marriage, although void ab initio, may still produce legal consequences. Among these legal consequences is incurring criminal liability for bigamy. xxx.[73] (Emphasis supplied.) Finally, in Re: Complaint of Mrs. Corazon S. Salvador against Spouses Noel and Amelia Serafico,[74] this Court pronounced:

982

University of the Cordilleras College of Law First Year C S.Y. 2013 - 2014 [75] In a catena of cases, the Court has consistently held that a judicial declaration of nullity is required before a valid subsequent marriage can be contracted; or else, what transpires is a bigamous marriage, reprehensible and immoral. xxx To conclude, the issue on the declaration of nullity of the marriage between petitioner and respondent only after the latter contracted the subsequent marriage is, therefore, immaterial for the purpose of establishing that the facts alleged in the information for Bigamy does not constitute an offense. Following the same rationale, neither may such defense be interposed by the respondent in his motion to quash by way of exception to the established rule that facts contrary to the allegations in the information are matters of defense which may be raised only during the presentation of evidence. All considered, we find that the trial court committed grave abuse of discretion when, in so quashing the Information in Criminal Case No. 07-0907-CFM, it considered an evidence introduced to prove a fact not alleged thereat disregarding the settled rules that a motion to quash is a hypothetical admission of the facts stated in the information; and that facts not alleged thereat may be appreciated only under exceptional circumstances, none of which is obtaining in the instant petition. WHEREFORE, the Orders dated 20 September 2007 and 6 December 2007 of the Regional Trial Court, Branch 115, Pasay City as well as the Resolutions dated 29 April 2008 and 18 July 2008 of the Court of Appeals are hereby SET ASIDE. Criminal Case No. 07-0907-CFM is REMANDED to the trial court for further proceedings.

SO ORDERED.

983

University of the Cordilleras College of Law First Year C S.Y. 2013 - 2014 Case Digest MYRNA P. ANTONE VS. LEO R. BERONILLA, G.R. No. 183824 December 8, 2010 THE FACTS: On 12 March 2007, herein petitioner Myrna P. Antone executed an AffidavitComplaint for Bigamy against Leo R. Beronilla before the Office of the City Prosecutor of Pasay City. She alleged that her marriage with respondent in 1978 had not yet been legally dissolved when the latter contracted a second marriage with one Cecile Maguillo in 1991. Herein respondent moved to quash the Information on the ground that the facts charged do not constitute an offense. He informed the court that his marriage with petitioner was declared null and void by the Regional Trial Court, Branch 16, Naval, Biliran on 26 April 2007; that the decision became final and executory on 15 May 200[7]; and that such decree has already been registered with the Municipal Civil Registrar on 12 June 2007. He argued that since the marriage had been declared null and void from the beginning, there was actually no first marriage to speak of. Absent a first valid marriage, the facts alleged in the Information do not constitute the crime of bigamy. Issue: Was there bigamy committed? Ruling: Yes. It contained all the elements of the crime of Bigamy under Article 349 of the Revised Penal Code hereunder enumerated: (1) that the offender has been legally married; (2) that the first marriage has not been legally dissolved or, in case his or her spouse is absent, the absent spouse could not yet be presumed dead according to the Civil Code; ( 3) that he contracts a second or subsequent marriage; and (4) that the second or subsequent marriage has all the essential requisites for validity.

984

University of the Cordilleras College of Law First Year C S.Y. 2013 - 2014 ENRIQUE AGRAVIADOR vs. ERLINDA AMPARO-AGRAVIADOR and REPUBLIC OF THE PHILIPPINES, G.R. No. 170729 December 8, 2010 Full Case ENRIQUE AGRAVIADOR y ALUNAN, Petitioner vs ERLINDA AMPARO-AGRAVIADOR and REPUBLIC OF THE PHILIPPINES, Respondents. BRION, J.: Enrique
[1]

Agraviador y Alunan

(petitioner)
[2]

challenges

through

his

petition

for

review

on certiorari the decision dated May 31, 2005 and the resolution dated December 6, 2005[3] of the Court of Appeals (CA) in CA-G.R. CV No. 75207.The challenged decision reversed the resolution[4] of the Regional Trial Court (RTC), Branch 276, Muntinlupa City, declaring the marriage of the petitioner and Erlinda Amparo-Agraviador (respondent) null and void on the ground of the latters psychological incapacity. The assailed resolution, on the other hand, denied the petitioners motion for reconsideration. Antecedent Facts The petitioner first met the respondent in 1971 at a beerhouse where the latter worked. The petitioner, at that time, was a 24-year old security guard of the Bureau of Customs, while the respondent was a 17-year old waitress. Their meeting led to a courtship, and they eventually became sweethearts. They often spent nights together at the respondents rented room, and soon entered into a common-law relationship. On May 23, 1973, the petitioner and the respondent contracted marriage in a ceremony officiated by Reverend Juanito Reyes at a church in Tondo, Manila. The petitioners family was apprehensive about this marriage because of the nature of the respondents work and because she came from a broken family. Out of their union, the petitioner and the respondent begot four (4) children, namely: Erisque, Emmanuel, Evelyn, and Eymarey. On March 1, 2001, the petitioner filed with the RTC a petition for the declaration of nullity of his marriage with the respondent, under Article 36 of the Family Code, as amended.[5] The case was docketed as Civil Case No. 01-081. He alleged that the respondent was psychologically incapacitated to exercise the essential obligations of marriage as she was carefree and irresponsible, and refused to do household chores like cleaning and cooking; stayed away from their house for long periods of time; had an affair with a lesbian; did not take care of their sick child; consulted a witch doctor in order to bring him bad fate; and refused to use the family name Agraviador in her activities. The petitioner likewise claimed that the respondent refused to have sex with him since 1993 because she became very close to a male tenant in their house. In fact, he discovered their love notes to each other, and caught them inside his room several times. The respondent moved to dismiss the petition on the ground that the root cause of her psychological incapacity was not medically identified and alleged in the petition. [6]The RTC denied this motion in its order dated July 2, 2001.[7] 985

University of the Cordilleras College of Law First Year C S.Y. 2013 - 2014 In her answer,[8] the respondent denied that she engaged in extramarital affairs and maintained that it was the petitioner who refused to have sex with her. She claimed that the petitioner wanted to have their marriage annulled because he wanted to marry their former household helper, Gilda Camarin. She added that she was the one who took care of their son at the hospital before he died. The RTC ordered the city prosecutor and/or the Solicitor General to investigate if collusion existed between the parties.[9] The RTC, in its Order of November 20, 2001, allowed the petitioner to present his evidence ex parte.[10] The petitioner, thus, presented testimonial and documentary evidence to substantiate his claims. In his testimony, the petitioner confirmed what he stated in his petition, i.e., that the respondent was carefree, irresponsible, immature, and whimsical; stubbornly did what she wanted; did not stay long in the conjugal dwelling; refused to do household chores; refused to take care of him and their children; and consulted a witch doctor in order to bring bad luck upon him. The petitioner further confirmed that the respondent abandoned their sick child, which led to the latters death. The petitioner further stated that the respondent became very close to a male border of their house; he discovered their love notes to each other, and caught them inside his room several times. The petitioner declared that he filed the petition for nullity because the respondent refused to change; he loves his children and does not want their children to be affected by their mothers conduct. He intimated that he might remarry if it would benefit their children. Aside from his testimony, the petitioner also presented a certified true copy of their marriage contract (Exh. B)[11] and the psychiatric evaluation report (Exh. A)[12] of Dr. Juan Cirilo L. Patac. In his Psychiatric Evaluation Report, Dr. Patac made the following findings: REMARKS AND RECOMMENDATIONS Based on the information gathered from Enrique, his son and their helper, the psychological report and the mental status examination, Enrique is found to be psychologically capable to fulfill the essential obligations of marriage. He coped with Erlindas selfish and irresponsible behavior as he dutifully performed what she failed to do for the family. He patiently tried to understand her and exerted every effort to make her realize the harm caused by her neglect to the family. Throughout their marriage, he provided emotional and material support for the family. He engaged in other business endeavors aside from his employment as he maintained to be financially productive. The same data revealed that Erlinda failed to fulfill the essential obligations of marriage. She manifested inflexible maladaptive behavior even at the time before their marriage. She is known to be stubborn and uncaring who did things her way without regard to the feelings of others. She is an irresponsible individual who selfishly ignored and neglected her role as daughter to her parents as wife to Enrique and mother to their children. Before the marriage at a young age of 17, Erlinda defied her parents as she lived alone, rented a room for herself and allowed Enrique to sleep with her. She did not care about the needs of Enrique before and after marriage and she maintained to be so with her children. She 986

University of the Cordilleras College of Law First Year C S.Y. 2013 - 2014 abandoned and relegated her duty to her family to their helper. She never stayed long in their house despite pleadings from her children and Enrique. Her irresponsible, uncaring behavior even led to the death of one of their children. Likewise, she does not show concern and ignores a daughter who is presently manifesting behavioral problem. She kept secrets as she never allowed her husband and children know where she stays when shes not at work. She falsified documents as she hid her marital status when she used her maiden surname in her present employment. She is having illicit affairs and is reported to be presently having an affair with a lesbian. Her desire to bring bad fate and death to Enrique through her consultation with a mangkukulam point out her lack of care, love, and respect to Enrique. Erlindas lack of motivation and insight greatly affected her capacity to render love, respect and support to her family. The above data shows that Erlinda is suffering from a Personality Disorder (Mixed Personality Disorder). She has been having this disorder since her adolescence. There is no definite treatment for this disorder. She is deemed psychologically incapacitated to perform the obligations of marriage. In fairness to Erlinda, she is recommended to undergo the same examination as Enrique underwent.[13] The RTC Ruling The RTC nullified the marriage of the petitioner and the respondent in its decision of April 26, 2002. It saw merit in the petitioners testimony and Dr. Patacs psychiatric evaluation report, and concluded that: Without contradiction the recitation by Petitioner and the findings of the doctor show that Respondent is indeed suffering from Mixed Personality Disorder that render her incapable of complying with her marital obligations. Respondents refusal to commit herself to the marriage, her tendencies to avoid a close relationship with Petitioner, preferring to be with her lover and finally abandoning their home for a lesbian, a disregard of social norm, show that she was never prepared for marital commitment in the first place. This incapacity is deeply rooted from her family upbringing with no hope for a cure. Therefore, for the good of society and of the parties themselves, it is best that this marriage between ENRIQUE AGRAVIADOR Y ALUNAN and ERLINDA AMPARO AGRAVIADOR be annulled as if it never took place at all. The Civil Registrar of the City of Manila and the General Civil Registrar, National Census and Statistics Office, East Avenue, Quezon City, are hereby requested to make the necessary correction of the civil record of the marriage between the parties and on their respective civil status. The children ERISQUE AGRAVIADOR, EMMANUEL AGRAVIADOR, EVELYN AGRAVIADOR and EYMAREY AGRAVIADOR will however remain as their legitimate children. It is SO ORDERED.[14] The CA Decision The Republic of the Philippines, through the Office of the Solicitor General, appealed the RTC decision to the CA. The CA, in its decision[15] dated May 31, 2005, reversed and set aside the RTC resolution, and dismissed the petition. The CA held that Dr. Patacs psychiatric evaluation report failed to establish that the respondents personality disorder was serious, grave and permanent; it likewise did not mention the root cause of her

987

University of the Cordilleras College of Law First Year C S.Y. 2013 - 2014 incapacity. The CA further ruled that Dr. Patac had no basis in concluding that the respondents disorder had no definite treatment because he did not subject her to a mental assessment. The CA added that the psychiatric remarks in the Report were nothing but a showcase of respondents character flaws and liabilities. There was no proof of a natal or supervening factor that effectively incapacitated the respondent from accepting and complying with the essential obligations of marriage. If at all, these character flaws may only give rise to a legal separation suit. The petitioner moved to reconsider this decision, but the CA denied his motion in its resolution of December 6, 2005.[16] The Petition and Issues The petitioner now comes to us via the present petition to challenge and seek the reversal of the CA ruling, based on the following arguments: I. THE EVIDENCE ADDUCED BY [HIM] WAS MORE THAN SUBSTANTIAL TO ESTABLISH THE PSYCHOLOGICAL INCAPACITY OF THE RESPONDENT[;] THE GUIDELINES SET FORTH IN REPUBLIC V. MOLINA [HAD BEEN] SATISIFIED[;] THE ADMISSIBILITY XXX OF THE PSYCHIATRIC EVALUATION REPORT XXX STILL STANDS FOR NOT HAVING BEEN CONTESTED XXX BY THE STATE AND/THE RESPONDENT[; and] THE DEGREE OF PROOF REQUIRED IN CIVIL CASES HAD BEEN SATISIFIED[.]

II.

III.

IV.

The issue in this case essentially boils down to whether there is basis to nullify the petitioners marriage to the respondent on the ground of psychological incapacity to comply with the essential marital obligations. The Courts Ruling We resolve to deny the petition for lack of merit, and hold that no sufficient basis exists to annul the marriage, pursuant to Article 36 of the Family Code and its related jurisprudence. The totality of evidence presented failed to establish the respondents psychological incapacity The petition for declaration of nullity of marriage is anchored on Article 36 of the Family Code which provides that "[a] marriage contracted by any party who, at the time of the celebration, was psychologically incapacitated to comply with the essential marital obligations of marriage, shall likewise be void even if such incapacity becomes manifest only after its solemnization." It introduced the concept of psychological incapacity as a ground for nullity of marriage, although this concept eludes exact definition.

988

University of the Cordilleras College of Law First Year C S.Y. 2013 - 2014 The initial common consensus on psychological incapacity under Article 36 of the Family Code was that it did not involve a species of vice of consent. Justices Sempio-Diy and Caguioa, both members of the Family Code revision committee that drafted the Code, conceded that the spouse may have given free and voluntary consent to a marriage but was, nonetheless, incapable of fulfilling such rights and obligations. Dr. Arturo Tolentino likewise stated in the 1990 edition of his commentaries on the Family Code that this psychological incapacity to comply with the essential marital obligations does not affect the consent to the marriage.[17] In Santos v. Court of Appeals,[18] the Court first declared that psychological incapacity must be characterized by (a) gravity; (b) juridical antecedence; and (c) incurability. It should refer to "no less than a mental (not physical) incapacity that causes a party to be truly incognitive of the basic marital covenants that concomitantly must be assumed and discharged by the parties to the marriage."[19] It must be confined to the most serious cases of personality disorders clearly demonstrative of an utter insensitivity or inability to give meaning and significance to the marriage. We laid down more definitive guidelines in the interpretation and application of Article 36 of the Family Code in Republic v. Court of Appeals[20] (the Molina case) where we said:

(1) The burden of proof to show the nullity of the marriage belongs to the plaintiff. Any doubt should be resolved in favor of the existence and continuation of the marriage and against its dissolution and nullity. This is rooted in the fact that both our Constitution and our laws cherish the validity of marriage and unity of the family. Thus, our Constitution devotes an entire Article on the Family, recognizing it as the foundation of the nation. It decrees marriage as legally inviolable, thereby protecting it from dissolution at the whim of the parties. Both the family and marriage are to be protected by the state. The Family Code echoes this constitutional edict on marriage and the family and emphasizes their permanence, inviolability and solidarity. (2) The root cause of the psychological incapacity must be (a) medically or clinically identified, (b) alleged in the complaint, (c) sufficiently proven by experts and (d) clearly explained in the decision. Article 36 of the Family Code requires that the incapacity must be psychological - not physical, although its manifestations and/or symptoms may be physical. The evidence must convince the court that the parties, or one of them, was mentally or psychically ill to such an extent that the person could not have known the obligations he was assuming, or knowing them, could not have given valid assumption thereof. Although no example of such incapacity need be given here so as not to limit the application of the provision under the principle of ejusdem generis, nevertheless such root cause must be identified as a psychological illness and its incapacitating nature fully explained. Expert evidence may be given by qualified psychiatrists and clinical psychologists. (3) The incapacity must be proven to be existing at the time of the celebration of the marriage. The evidence must show that the illness was existing when the parties exchanged their I do's. The manifestation of the illness need not be perceivable at such time, but the illness itself must have attached at such moment, or prior thereto. (4) Such incapacity must also be shown to be medically or clinically permanent or incurable. Such incurability may be absolute or even relative only in regard to the other spouse, not necessarily absolutely against everyone of the same sex. Furthermore, such incapacity must be relevant to the assumption of marriage obligations, not necessarily to those not related to marriage, like the exercise of a profession or employment in a job. x x x

989

University of the Cordilleras College of Law First Year C S.Y. 2013 - 2014 (5) Such illness must be grave enough to bring about the disability of the party to assume the essential obligations of marriage. Thus, mild characteriological peculiarities, mood changes, occasional emotional outbursts cannot be accepted as root causes. The illness must be shown as downright incapacity or inability, not a refusal, neglect or difficulty, much less ill will. In other words, there is a natal or supervening disabling factor in the person, an adverse integral element in the personality structure that effectively incapacitates the person from really accepting and thereby complying with the obligations essential to marriage. (6) The essential marital obligations must be those embraced by Articles 68 up to 71 of the Family Code as regards the husband and wife as well as Articles 220, 221 and 225 of the same Code in regard to parents and their children. Such non-complied marital obligation(s) must also be stated in the petition, proven by evidence and included in the text of the decision. (7) Interpretations given by the National Appellate Matrimonial Tribunal of the Catholic Church in the Philippines, while not controlling or decisive, should be given great respect by our courts. x x x (8) The trial court must order the prosecuting attorney or fiscal and the Solicitor General to appear as counsel for the state. No decision shall be handed down unless the Solicitor General issues a certification, which will be quoted in the decision, briefly stating therein his reasons for his agreement or opposition, as the case may be, to the petition. The Solicitor General, along with the prosecuting attorney, shall submit to the court such certification within fifteen (15) days from the date the case is deemed submitted for resolution of the court. The Solicitor General shall discharge the equivalent function of the defensor vinculi contemplated under Canon 1095. These guidelines incorporate the basic requirements we established in Santos. A later case, Marcos v. Marcos,[21] further clarified that there is no requirement that the defendant/respondent spouse should be personally examined by a physician or psychologist as a condition sine qua non for the declaration of nullity of marriage based on psychological incapacity. Accordingly, it is no longer necessary to introduce expert opinion in a petition under Article 36 of the Family Code if the totality of evidence shows that psychological incapacity exists and its gravity, juridical antecedence, and incurability can be duly established. A later case, Ngo Te v. Yu-Te,[22] declared that it may have been inappropriate for the Court to impose a rigid set of rules, as the one in Molina, in resolving all cases of psychological incapacity. We stated that instead of serving as a guideline, Molina unintentionally became a straightjacket, forcing all cases involving psychological incapacity to fit into and be bound by it, which is not only contrary to the intention of the law but unrealistic as well because, with respect to psychological incapacity, no case can be considered as on "all fours" with another. Ngo Te, therefore, put into question the applicability of timetested guidelines set forth in Molina. Ting v. Velez-Ting[23] and the fairly recent case of Suazo v. Suazo[24] squarely met the issue and laid to rest any question regarding the applicability of Molina. In these cases, we clarified that Ngo Te did not abandon Molina; far from abandoning Molina, it simply suggested the relaxation of its stringent requirements. We also explained inSuazo that Ngo Te merely stands for a more flexible approach in considering petitions for declaration of nullity of marriages based on psychological incapacity.

990

University of the Cordilleras College of Law First Year C S.Y. 2013 - 2014 Under these established guidelines, we find the totality of the petitioners evidence insufficient to prove the respondents psychological incapacity. a. Petitioners court testimony

For clarity, we reproduce the pertinent portions of the petitioners testimony that essentially confirmed what the petition alleged:

Q:

Out of your marriage with the said respondent, were you blessed with children, and how many? Yes, sir, we were blessed with four (4), two (2) boys and two (2) girls. Where are they now? All grown up with the exception of one who died of pneumonia due to the neglect and fault of my said wife who abandone[d] him at the time of his illness. Is that the reason why you file[d] the instant petition, Mr. Witness? It is only one of the several reasons, Sir. Can you cite these reasons, you mentioned? She appears to be carefree, irresponsible, immature, whimsical and used to impose what she wanted to get, she refused to do household chores, like cooking, caring for the husband and children, used to stay from the conjugal dwelling, initially for weeks, then for months and lately fully abandoned the family house and stay with a lesbian. [sic] At first, I discovered a love note while being so secretive and used to be very close to a male renter in the ground floor of their house and caught them several times alone in his room, thus explaining the reason why she refused to have sex since 1993, up to and until the present time. Lately, we discovered that she used to consult a cult mangkukulam to bring bad fate against the family and death for me.

A: Q: A:

Q: A: Q: A:

Q: A:

By the way did you give her the chance to change? I gave her but she refused to reform. xxxx

Q: A:

Can you not give a last chance for you to save your marriage? I think I cannot since she does not accept her fault and she does not want to change for the sake of our family.[25]

These exchanges during trial significantly constituted the totality of the petitioners testimony on the respondents supposed psychological or mental malady. We glean from these exchanges the petitioners theory that the respondents psychological incapacity is premised on her refusa l or unwillingness to perform certain marital obligations, and a number of unpleasant personality traits such as immaturity, irresponsibility, and unfaithfulness.

991

University of the Cordilleras College of Law First Year C S.Y. 2013 - 2014 These acts, in our view, do not rise to the level of psychological incapacity that the law requires, and should be distinguished from the difficulty, if not outright refusal or neglect, in the performance of some marital obligations that characterize some marriages.[26] The intent of the law has been to confine the meaning of psychological incapacity to the most serious cases of personality disorders existing at the time of the marriage clearly demonstrating an utter insensitivity or inability to give meaning and significance to the marriage.[27] The psychological illness that must have afflicted a party at the inception of the marriage should be a malady so grave and permanent as to deprive one of awareness of the duties and responsibilities of the matrimonial bond he or she is about to assume.[28] In the present case, the petitioners testimony failed to establish that the respondents condition is a manifestation of a disordered personality rooted on some incapacitating or debilitating psychological condition that makes her completely unable to discharge the essential marital obligations. If at all, the petitioner merely showed that the respondent had some personality defects that showed their manifestation during the marriage; his testimony sorely lacked details necessary to establish that the respondents defects existed at the inception of the marriage. In addition, the petitioner failed to discuss the gravity of the respondents condition; neither did he mention that the respondents malady wasincurable, or if it were otherwise, the cure would be beyond the respondents means to undertake. The petitioners declarations that the respondent does not accept her fault, does not want to change, and refused to reform are insufficient to establish a psychological or mental defect that is serious , grave, or incurable as contemplated by Article 36 of the Family Code. In a similar case, Bier v. Bier,[29] we ruled that it was not enough that the respondent, alleged to be psychologically incapacitated, had difficulty in complying with his marital obligations, or was unwilling to perform these obligations. Proof of a natal or supervening disabling factor an adverse integral element in the respondent's personality structure that effectively incapacitated him from complying with his essential marital obligations had to be shown. b. Dr. Patacs Psychiatric Evaluation Report

The Court finds that Dr. Patacs Psychiatric Evaluation Report fell short in proving that the respondent was psychologically incapacitated to perform the essential marital duties. We emphasize that Dr. Patac did not personally evaluate and examine the respondent; he, in fact, recommended at the end of his Report for the respondent to undergo the same examination [that the petitioner] underwent.[30] Dr. Patac relied only on the information fed by the petitioner, the parties second child, Emmanuel, and household helper. Sarah. Largely, the doctor relied on the information provided by the petitioner. Thus, while his Report can be used as a fair gauge to assess the petitioners own psychological condition (as he was, in fact, declared by Dr. Patac to be psychologically capable to fulfill the essential obligations of marriage), the same statement cannot be made with respect to the respondents condition. The methodology employed simply cannot satisfy the required depth and comprehensiveness of the examination required to evaluate a party alleged to be suffering from a psychological disorder.[31] We do not suggest that a personal examination of the party alleged to be psychologically incapacitated is mandatory. We have confirmed in Marcos v. Marcos that the person sought to be 992

University of the Cordilleras College of Law First Year C S.Y. 2013 - 2014 declared psychologically incapacitated must be personally examined by a psychologist as a condition sine qua non to arrive at such declaration.[32] If a psychological disorder can be proven by independent means, no reason exists why such independent proof cannot be admitted and given credit.[33] No such independent evidence appears on record, however, to have been gathered in this case. In his Report, Dr. Patac attempted to establish the juridical antecedence of the respondents condition by stating that the respondent manifested inflexible maladaptive behavior before marriage, pointing out how the respondent behaved before the marriage the respondent defied her parents and lived alone; rented a room for herself; and allowed the petitioner to sleep with her. These perceived behavioral flaws, to our mind, are insufficient to establish that the incapacity was rooted in the history of the respondent antedating the marriage. Dr. Patac failed to elucidate on the circumstances that led the respondent to act the way she did, for example, why she defied her parents and decided to live alone; why she neglected her obligations as a daughter; and why she often slept with the petitioner. This is an area where independent evidence, such as information from a person intimately related to the respondent, could prove useful. As earlier stated, no such independent evidence was gathered in this case. In the absence of such evidence, it is not surprising why the Psychiatric Report Evaluation failed to explain how and why the respondents so-called inflexible maladaptive behavior was already present at the time of the marriage. Dr. Patacs Psychiatric Evaluation Report likewise failed to prove the gravity or seriousness of the respondents condition. He simply made an enumeration of the respondents purported behavioral defects (as related to him by third persons), and on this basis characterized the respondent to be suffering from mixed personality disorder. In the Background History portion of his Psychiatric Evaluation Report, Dr. Patac mentioned that the respondent employed one of her siblings to do the household chores; did not help in augmenting the familys earnings; belittled the petitioners income; continued her studies despite the petitioners disapproval; seldom stayed at home; became close to a male border; had an affair with a lesbian; did not disclose the actual date of her departure to Taiwan; threatened to poison the petitioner and their children; neglected and ignored their children; used her maiden name at work; and consulted a witch doctor to bring bad fate to the petitioner. Except for the isolated and unfounded statement that Erlindas lack of motivation and insight greatly affected her capacity to render love, respect and support to the family,[34] there was no other statement regarding the degree of severity of the respondents condition, why and to what extent the disorder is grave, and how it incapacitated her to comply with the duties required in marriage. There was likewise no showing of a supervening disabling factor or debilitating psychological condition that effectively incapacitated the respondent from complying with the essential marital obligations. At any rate, the personality flaws mentioned above, even if true, could only amount to insensitivity, sexual infidelity, emotional immaturity, and irresponsibility, which do not by themselves warrant a finding of psychological incapacity under Article 36 of the Family Code. Interestingly, Dr. Patacs Psychiatric Evaluation Report highlighted only the respondents negative behavioral traits without balancing them with her other qualities. The allegations of infidelity and insinuations of promiscuity, as well as the claim that the respondent refused to engage in sexual 993

University of the Cordilleras College of Law First Year C S.Y. 2013 - 2014 intercourse since 1993, of course, came from the petitioner, but these claims were not proven. Even assuming ex gratia argumenti that these accusations were true, the Psychiatric Evaluation Report did not indicate that unfaithfulness or promiscuousness were traits that antedated or existed at the time of marriage. Likewise, the accusation that the respondent abandoned her sick child which eventually led to the latters death appears to be an exaggerated claim in the absence of any specifics and corroboration. On the other hand, the petitioners own questionable traits his flirtatious nature before marriage and his admission that he inflicted physical harm on the respondent every time he got jealous were not pursued. From this perspective, the Psychiatric Evaluation Report appears to be no more than a one-sided diagnosis against the respondent that we cannot consider a reliable basis to conclusively establish the root cause and the degree of seriousness of her condition. The Psychiatric Evaluation Report likewise failed to adequately explain how Dr. Patac came to the conclusion that the respondents personality disorder had no definite treatment. It did not discuss the concept of mixed personality disorder, i.e., its classification, cause, symptoms, and cure, and failed to show how and to what extent the respondent exhibited this disorder in order to create a necessary inference that the respondents condition had no definite treatment or is incurable. A glaring deficiency, to our mind, is the Psychiatric Evaluation Reports failure to support its findings and conclusions with any factual basis. It simply enumerated the respondents perceived behavioral defects, and then associated these traits with mixed personality disorder. We find it unfortunate that Dr. Patac himself was not called on the witness stand to expound on the findings and conclusions he made in his Psychiatric Evaluation Report. It would have aided petitioners cause had he called Dr. Patac to testify. Admittedly, the standards used by the Court in assessing the sufficiency

of psychological evaluation reports may be deemed very strict, but these are proper, in view of the principle that any doubt should be resolved in favor of the validity of the marriage and the indissolubility of the marital vinculum.[35] Marriage, an inviolable institution protected by the State, cannot be dissolved at the whim of the parties, especially where the prices of evidence presented are grossly deficient to show the juridical antecedence, gravity and incurability of the condition of the party alleged to be psychologically incapacitated to assume and perform the essential marital duties. The petitioners marriage to the respondent may have failed and appears to be without hope of reconciliation The remedy, however, is not always to have it declared voidab initio on the ground of psychological incapacity. We stress that Article 36 of the Family Code contemplates downright incapacity or inability to assume and fulfill the basic marital obligations, not a mere refusal, neglect or difficulty, much less, ill will, on the part of the errant spouse. It is not to be confused with a divorce law that cuts the marital bond at the time the grounds for divorce manifest themselves. The State, fortunately or unfortunately, has not seen it fit to decree that divorce should be available in this country. Neither should an Article 36 declaration of nullity be equated with legal separation, in which the grounds need not be rooted in psychological incapacity but on physical violence, moral pressure, moral
[36]

corruption,

civil

interdiction,

drug

addiction,

sexual

infidelity, abandonment, and the like.

Unless the evidence presented clearly reveals a situation where the

parties or one of them, by reason of a grave and incurable psychological illness existing at the time the marriage was

994

University of the Cordilleras College of Law First Year C S.Y. 2013 - 2014 celebrated, was incapacitated to fulfill the obligations of marital life (and thus could not then have validly entered into a marriage), then we are compelled to uphold the indissolubility of the marital tie. WHEREFORE, in light of all the foregoing, we DENY the petition and AFFIRM the Decision and the Resolution of the Court of Appeals dated May 31, 2005 andDecember 6, 2005, respectively, in CA-G.R. CV No. 75207. Costs against the petitioner. SO ORDERED.

995

University of the Cordilleras College of Law First Year C S.Y. 2013 - 2014 Case Digest ENRIQUE AGRAVIADOR vs. ERLINDA AMPARO-AGRAVIADOR and REPUBLIC OF THE PHILIPPINES, G.R. No. 170729 December 8, 2010 FACTS: The petitioner first met the respondent in 1971 at a beerhouse where the latter worked. The petitioner, at that time, was a 24-year old security guard of the Bureau of Customs, while the respondent was a 17-year old waitress. Their meeting led to a courtship, and they eventually became sweethearts. They often spent nights together at the respondents rented room, and soon entered into a common-law relationship. On May 23, 1973, the petitioner and the respondent contracted marriage in a ceremony officiated by Reverend Juanito Reyes at a church in Tondo, Manila. The petitioners family was apprehensive about this marriage because of the nature of the respondents work and because she came from a broken family. Out of their union, the petitioner and the respondent begot four (4) children, namely: Erisque, Emmanuel, Evelyn, and Eymarey. On March 1, 2001, the petitioner filed with the RTC a petition for the declaration of nullity of his marriage with the respondent, under Article 36 of the Family Code, as amended. The case was docketed as Civil Case No. 01-081. He alleged that the respondent was psychologically incapacitated to exercise the essential obligations of marriage as she was carefree and irresponsible, and refused to do household chores like cleaning and cooking; stayed away from their house for long periods of time; had an affair with a lesbian; did not take care of their sick child; consulted a witch doctor in order to bring him bad fate; and refused to use the family name Agraviador in her activities. The petitioner likewise claimed that the respondent refused to have sex with him since 1993 because she became very close to a male tenant in their house. In fact, he discovered their lo ve notes to each other, and caught them inside his room several times. ISSUE: Was there psychological incapacity on the part of the wife? RULING: No sufficient basis exists to annul the marriage, pursuant to Article 36 of the Family Code and its related jurisprudence. The totality of evidence presented failed to establish the respondents psychological incapacity The petition for declaration of nullity of marriage is anchored on Article 36 of the Family Code which provides that "[a] marriage contracted by any party who, at the time of the celebration, was psychologically incapacitated to comply with the essential marital obligations of marriage, shall likewise be void even if such incapacity becomes manifest only after its solemnization." It introduced the concept of psychological incapacity as a ground for nullity of marriage, although this concept eludes exact definition. The initial common consensus on psychological incapacity under Article 36 of the Family Code was that it did not involve a species of vice of consent. Justices Sempio-Diy and Caguioa, both 996

University of the Cordilleras College of Law First Year C S.Y. 2013 - 2014 members of the Family Code revision committee that drafted the Code, conceded that the spouse may have given free and voluntary consent to a marriage but was, nonetheless, incapable of fulfilling such rights and obligations. Dr. Arturo Tolentino likewise stated in the 1990 edition of his commentaries on the Family Code that this psychological incapacity to comply with the essential marital obligations does not affect the consent to the marriage. In Santos v. Court of Appeals, the Court first declared that psychological incapacity must be characterized by (a) gravity; (b) juridical antecedence; and (c) incurability. It should refer to "no less than a mental (not physical) incapacity that causes a party to be truly incognitive of the basic marital covenants that concomitantly must be assumed and discharged by the parties to the marriage." It must be confined to the most serious cases of personality disorders clearly demonstrative of an utter insensitivity or inability to give meaning and significance to the marriage.

997

University of the Cordilleras College of Law First Year C S.Y. 2013 - 2014 Marable vs Marable G.R. No. 178741 Full Case ROSALINO L. MARABLE, Petitioner, vs. MYRNA F. MARABLE, Respondent. VILLARAMA, JR., J.: On appeal is the Decision1 dated February 12, 2007 and Resolution2 dated July 4, 2007 of the Court of Appeals (CA) in CA-G.R. CV No. 86111 which reversed and set aside the Decision3 dated January 4, 2005 of the Regional Trial Court (RTC), Branch 72, Antipolo City, in Civil Case No. 01-6302. The RTC had granted petitioners prayer that his marriage to respondent be declared null and void on the ground that he is psychologically incapacitated to perform the essential obligations of marriage. The facts, as culled from the records, are as follows: Petitioner and respondent met in 1967 while studying at Arellano University. They were classmates but initially, petitioner was not interested in respondent. He only became attracted to her after they happened to sit beside each other in a passenger bus. Petitioner courted respondent and they eventually became sweethearts even though petitioner already had a girl friend. Later, respondent discovered petitioners other relationship and demanded more time and attention from petitioner. Petitioner alleged that he appreciated this gesture like a child longing for love, time and attention. On December 19, 1970, petitioner and respondent eloped and were married in civil rites at Tanay, Rizal before Mayor Antonio C. Esguerra. A church wedding followed on December 30, 1970 at the Chapel of the Muntinlupa Bilibid Prison and their marriage was blessed with five children. As the years went by, however, their marriage turned sour. Verbal and physical quarrels became common occurrences. They fought incessantly and petitioner became unhappy because of it. The frequency of their quarrels increased when their eldest daughter transferred from one school to another due to juvenile misconduct. It became worse still when their daughter had an unwanted teenage pregnancy. The exceedingly serious attention petitioner gave to his children also made things worse for them as it not only spoiled some of them, but it also became another cause for the incessant quarrelling between him and respondent. Longing for peace, love and affection, petitioner developed a relationship with another woman. Respondent learned about the affair, and petitioner promptly terminated it. But despite the end of the short-lived affair, their quarrels aggravated. Also, their business ventures failed. Any amount of respect remaining between them was further eroded by their frequent arguments and verbal abuses infront of their friends. Petitioner felt that he was unloved, unwanted and unappreciated and this made him indifferent towards respondent. When he could not bear his lot any longer, petitioner left the family home and stayed with his sister in Antipolo City. He gave up all the properties which he and respondent had accumulated during their marriage in favor of respondent and their children. Later, he converted to Islam after dating several women. On October 8, 2001, petitioner decided to sever his marital bonds. On said date, he filed a petition4 for declaration of nullity of his marriage to respondent on the ground of his psychological incapacity to perform the essential responsibilities of marital life. In his petition, petitioner averred that he came from a poor family and was already exposed to the hardships of farm life at an early age. His father, although responsible and supportive, was a compulsive gambler and womanizer. His father left their family to live with another woman with whom he had seven other children. This caused petitioners mother and siblings to suffer immensely. Thus, petitioner became obsessed with attention and worked hard to excel so he would be noticed. 998 January 17, 2011

University of the Cordilleras College of Law First Year C S.Y. 2013 - 2014 Petitioner further alleged that he supported himself through college and worked hard for the company he joined. He rose from the ranks at Advertising and Marketing Associates, Inc., and became Senior Executive Vice President and Chief Finance Officer therein. But despite his success at work, he alleged that his misery and loneliness as a child lingered as he experienced a void in his relationship with his own family. In support of his petition, petitioner presented the Psychological Report5 of Dr. Nedy L. Tayag, a clinical psychologist from the National Center for Mental Health. Dr. Tayags report stated that petitioner is suffering from "Antisocial Personality Disorder," characterized by a pervasive pattern of social deviancy, rebelliousness, impulsivity, self-centeredness, deceitfulness and lack of remorse. The report also revealed that petitioners personality disorder is rooted in deep feelings of rejection starting from the family to peers, and that his experiences have made him so self-absorbed for needed attention. It was Dr. Tayags conclusion that petitioner is psychologically incapacitated to perform his marital obligations. After trial, the RTC rendered a decision annulling petitioners marriage to respondent on the ground of petitioners psychological incapacity. Upon appeal by the Office of the Solicitor General (OSG), the CA reversed the RTC decision as follows: WHEREFORE, the foregoing considered, the appeal is GRANTED and the assailed Decision hereby REVERSED AND SET ASIDE. Accordingly, the marriage between the parties is declared valid and subsisting. No costs. SO ORDERED.6 The CA held that the circumstances related by petitioner are insufficient to establish the existence of petitioners psychological incapacity. The CA noted that Dr. Tayag did not fully explain the root cause of the disorder nor did she give a concrete explanation as to how she arrived at a conclusion as to its gravity or permanence. The appellate court emphasized that the root cause of petitioners psychological incapacity must be medically or clinically identified, sufficiently proven by experts and clearly explained in the decision. In addition, the incapacity must be proven to be existing at the time of the celebration of the marriage and shown to be medically or clinically permanent or incurable. It must also be grave enough to bring about the disability of the petitioner to assume the essential obligations of marriage. On July 4, 2007, the CA denied petitioners motion for reconsideration. Hence, this appeal. Essentially, petitioner raises the sole issue of whether the CA erred in reversing the trial courts decision. Petitioner claims that his psychological incapacity to perform his essential marital obligations was clearly proven and correctly appreciated by the trial court. Petitioner relies heavily on the psychological evaluation conducted by Dr. Tayag and quotes the latters findings: Petitioner had always been hungry for love and affection starting from his family to the present affairs that he [has]. This need had afforded him to find avenues straight or not, just to fulfill this need. He used charm, deceit, lies, violence, [and] authority just so to accom[m]odate and justify his acts. Finally, he is using religions to support his claim for a much better personal and married life which is really out of context. Rebellious and impulsive as he is, emotional instability is apparent that it would be difficult for him to harmonize with life in general and changes. Changes must come from within, it is not purely external. Clinically, petitioners self-absorbed ideals represent the grave, severe, and incurable nature of Antisocial Personality Disorder. Such disorder is characterized by a pervasive pattern of social deviancy, rebelliousness, impulsivity, self-centeredness, deceitfulness, and lack of remorse. The psychological incapacity of the petitioner is attributed by jurisdictional antecedence as it existed even before the said marital union. It is also profoundly rooted, grave and incurable. The root cause of which is deep feelings of rejection starting from family to peers. This insecure feelings had made him so selfabsorbed for needed attention. Carrying it until his marital life. Said psychological incapacity had deeply 999

University of the Cordilleras College of Law First Year C S.Y. 2013 - 2014 marred his adjustment and severed the relationship. Thus, said marriage should be declared null and void by reason of the psychological incapacity.7 According to petitioner, the uncontradicted psychological report of Dr. Tayag declared that his psychological incapacity is profoundly rooted and has the characteristics of juridical antecedence, gravity and incurability. Moreover, petitioner asserts that his psychological incapacity has been medically identified and sufficiently proven. The State, on the other hand, never presented another psychologist to rebut Dr. Tayags findings. Also, petitioner maintains that the psychological evaluation would show that the marriage failed not solely because of irreconcilable differences between the spouses, but due to petitioners personality disorder which rendered him unable to comply with his marital obligations. To t he mind of petitioner, the assailed decision compelled the parties to continue to live under a "non-existent marriage." The Republic, through the OSG, filed a Comment8 maintaining that petitioner failed to prove his psychological incapacity. The OSG points out that Dr. Tayag failed to explain specifically how she arrived at the conclusion that petitioner suffers from an anti-social personality disorder and that it is grave and incurable. In fact, contrary to his claim, it even appears that petitioner acted responsibly throughout their marriage. Despite financial difficulties, he and respondent had blissful moments together. He was a good father and provider to his children. Thus, the OSG argues that there was no reason to describe petitioner as a self-centered, remorseless, rebellious, impulsive and socially deviant person. Additionally, the OSG contends that since the burden of proof is on petitioner to establish his psychological incapacity, the State is not required to present an expert witness where the testimony of petitioners psychologist was insufficient and inconclusive. The OSG adds that petitioner was not able to substantiate his claim that his infidelity was due to some psychological disorder, as the real cause of petitioners alleged incapacity appears to be his general dissatisfaction with his marriage. At most he was able to prove infidelity on his part and the existence of "irreconcilable differences" and "conflicting personalities." These, however, do not constitute psychological incapacity. Respondent also filed her Comment9 and Memorandum10 stressing that psychological incapacity as a ground for annulment of marriage should contemplate downright incapacity or inability to take cognizance of and to assume the essential marital obligations, not a mere refusal, neglect or difficulty, much less ill will, on the part of the errant spouse. The appeal has no merit. The appellate court did not err when it reversed and set aside the findings of the RTC for lack of legal and factual bases. Article 36 of the Family Code, as amended, provides: Art. 36. A marriage contracted by any party who, at the time of the celebration, was psychologically incapacitated to comply with the essential marital obligations of marriage, shall likewise be void even if such incapacity becomes manifest only after its solemnization. The term "psychological incapacity" to be a ground for the nullity of marriage under Article 36 of the Family Code, refers to a serious psychological illness afflicting a party even before the celebration of the marriage.11 These are the disorders that result in the utter insensitivity or inability of the afflicted party to give meaning and significance to the marriage he or she has contracted.12 Psychological incapacity must refer to no less than a mental (not physical) incapacity that causes a party to be truly incognitive of the basic marital covenants that concomitantly must be assumed and discharged by the parties to the marriage.13 In Republic v. Court of Appeals,14 the Court laid down the guidelines in the interpretation and application of Article 36. The Court held,

1000

University of the Cordilleras College of Law First Year C S.Y. 2013 - 2014 (1) The burden of proof to show the nullity of the marriage belongs to the plaintiff. Any doubt should be resolved in favor of the existence and continuation of the marriage and against its dissolution and nullity. (2) The root cause of the psychological incapacity must be: (a) medically or clinically identified, (b) alleged in the complaint, (c) sufficiently proven by experts and (d) clearly explained in the decision. (3) The incapacity must be proven to be existing at "the time of the celebration" of the marriage. (4) Such incapacity must also be shown to be medically or clinically permanent or incurable. (5) Such illness must be grave enough to bring about the disability of the party to assume the essential obligations of marriage.1avvphi1 (6) The essential marital obligations must be those embraced by Articles 68 up to 71 of the Family Code as regards the husband and wife as well as Articles 220, 221 and 225 of the same Code in regard to parents and their children. (7) Interpretations given by the National Appellate Matrimonial Tribunal of the Catholic Church in the Philippines, while not controlling or decisive, should be given great respect by our courts. (8) The trial court must order the prosecuting attorney or fiscal and the Solicitor General to appear as counsel for the state. No decision shall be handed down unless the Solicitor General issues a certification, which will be quoted in the decision, briefly stating therein his reasons for his agreement or opposition, as the case may be, to the petition. In the instant case, petitioner completely relied on the psychological examination conducted by Dr. Tayag on him to establish his psychological incapacity. The result of the examination and the findings of Dr. Tayag however, are insufficient to establish petitioner's psychological incapacity. In cases of annulment of marriage based on Article 36 of the Family Code, as amended, the psychological illness and its root cause must be proven to exist from the inception of the marriage. Here, the appellate court correctly ruled that the report of Dr. Tayag failed to explain the root cause of petitioners alleged psychological incapacity. The evaluation of Dr. Tayag merely made a general conclusion that petitioner is suffering from an Anti-social Personality Disorder but there was no factual basis stated for the finding that petitioner is a socially deviant person, rebellious, impulsive, self-centered and deceitful. As held in the case of Suazo v. Suazo,15 the presentation of expert proof in cases for declaration of nullity of marriage based on psychological incapacity presupposes a thorough and an in-depth assessment of the parties by the psychologist or expert, for a conclusive diagnosis of a grave, severe and incurable presence of psychological incapacity. Here, the evaluation of Dr. Tayag falls short of the required proof which the Court can rely on as basis to declare as void petitioners marriage to respondent. In fact, we are baffled by Dr. Tayags evaluation which became the trial courts basis for concluding that petitioner was psychologically incapacitated, for the report did not clearly specify the actions of petitioner which are indicative of his alleged psychological incapacity. More importantly, there was no established link between petitioners acts to his alleged psychological incapacity. It is indispensable that the evidence must show a link, medical or the like, between the acts that manifest psychological incapacity and the psychological disorder itself.16 For sure, the spouses frequent marital squabbles17 and differences in handling finances and managing their business affairs, as well as their conflicts on how to raise their children, are not manifestations of psychological incapacity which may be a ground for declaring their marriage void. Petitioner even admitted that despite their financial difficulties, they had happy moments together. Also, the records would show that the petitioner acted responsibly during their marriage and in fact worked hard to provide for the needs of his family, most especially his children. Their personal differences do not reflect a personality disorder tantamount to psychological incapacity. Petitioner tried to make it appear that his family history of having a womanizer for a father, was one of the reasons why he engaged in extra-marital affairs during his marriage. However, it appears more likely 1001

University of the Cordilleras College of Law First Year C S.Y. 2013 - 2014 that he became unfaithful as a result of a general dissatisfaction with his marriage rather than a psychological disorder rooted in his personal history. His tendency to womanize, assuming he had such tendency, was not shown to be due to causes of a psychological nature that is grave, permanent and incurable. In fact, the records show that when respondent learned of his affair, he immediately terminated it. In short, petitioners marital infidelity does not appear to be symptomatic of a grave psychological disorder which rendered him incapable of performing his spousal obligations. It has been held in various cases that sexual infidelity, by itself, is not sufficient proof that petitioner is suffering from psychological incapacity.18 It must be shown that the acts of unfaithfulness are manifestations of a disordered personality which make petitioner completely unable to discharge the essential obligations of marriage.19 That not being the case with petitioner, his claim of psychological incapacity must fail. It bears stressing that psychological incapacity must be more than just a "difficulty," "refusal" or "neglect" in the performance of some marital obligations. Rather, it is essential that the concerned party was incapable of doing so, due to some psychological illness existing at the time of the celebration of the marriage. In Santos v. Court of Appeals,20 the intention of the law is to confine the meaning of "psychological incapacity" to the most serious cases of personality disorders clearly demonstrative of an utter insensitivity or inability to give meaning and significance to the marriage.21 All told, we find that the CA did not err in declaring the marriage of petitioner and respondent as valid and subsisting. The totality of the evidence presented is insufficient to establish petitioners psychological incapacity to fulfill his essential marital obligations. WHEREFORE, the appeal is DENIED for lack of merit. The February 12, 2007 Decision of the Court of Appeals in CA-G.R. CV No. 86111 and its Resolution dated July 4, 2007 are hereby AFFIRMED. No costs. SO ORDERED.

1002

University of the Cordilleras College of Law First Year C S.Y. 2013 - 2014 Case Digest ROSALINO L. MARABLE vs. MYRNA F. MARABLE G.R. No. 178741 January 17, 2011 FACTS: Petitioner and respondent met in 1967 while studying at Arellano University. They were classmates but initially, petitioner was not interested in respondent. He only became attracted to her after they happened to sit beside each other in a passenger bus. Petitioner courted respondent and they eventually became sweethearts even though petitioner already had a girlfriend. Later, respondent discovered petitioners other relationship and demanded more time and attention from petitioner. Petitioner alleged that he appreciated this gesture like a child longing for love, time and attention. On December 19, 1970, petitioner and respondent eloped and were married in civil rites at Tanay, Rizal before Mayor Antonio C. Esguerra. A church wedding followed on December 30, 1970 at the Chapel of the Muntinlupa Bilibid Prison and their marriage was blessed with five children. As the years went by, however, their marriage turned sour. Verbal and physical quarrels became common occurrences. They fought incessantly and petitioner became unhappy because of it. The frequency of their quarrels increased when their eldest daughter transferred from one school to another due to juvenile misconduct. It became worse still when their daughter had an unwanted teenage pregnancy. The exceedingly serious attention petitioner gave to his children also made things worse for them as it not only spoiled some of them, but it also became another cause for the incessant quarrelling between him and respondent. Longing for peace, love and affection, petitioner developed a relationship with another woman. Respondent learned about the affair, and petitioner promptly terminated it. But despite the end of the short-lived affair, their quarrels aggravated. Also, their business ventures failed. Any amount of respect remaining between them was further eroded by their frequent arguments and verbal abuses infront of their friends. Petitioner felt that he was unloved, unwanted and unappreciated and this made him indifferent towards respondent. When he could not bear his lot any longer, petitioner left the family home and stayed with his sister in Antipolo City. He gave up all the properties which he and respondent had accumulated during their marriage in favor of respondent and their children. Later, he converted to Islam after dating several women. On October 8, 2001, petitioner decided to sever his marital bonds. On said date, he filed a petition[4] for declaration of nullity of his marriage to respondent on the ground of his psychological incapacity to perform the essential responsibilities of marital life. In support of his petition, petitioner presented the Psychological Report[5] of Dr. Nedy L. Tayag, a clinical psychologist from the National Center for Mental Health. Dr. Tayags report stated that petitioner is suffering from Antisocial Personality Disorder, characterized by a pervasive pattern of social deviancy, rebelliousness, impulsivity, self-centeredness, deceitfulness and lack of remorse. The report also revealed that petitioners personality disorder is rooted in deep feelings of rejection starting from the family to peers, and that his experiences have made him so

1003

University of the Cordilleras College of Law First Year C S.Y. 2013 - 2014 self-absorbed for needed attention. It was Dr. Tayags conclusion that petitioner is psychologically incapacitated to perform his marital obligations. ISSUE: Is the husband psychologically incapacitated? RULING: No. The CA held that the circumstances related by petitioner are insufficient to establish the existence of petitioners psychological incapacity. The CA noted that Dr. Tayag did not fully explain the root cause of the disorder nor did she give a concrete explanation as to how she arrived at a conclusion as to its gravity or permanence. The appellate court emphasized that the root cause of petitioners psychological incapacity must be medically or clinically identified, sufficiently proven by experts and clearly explained in the decision. In addition, the incapacity must be proven to be existing at the time of the celebration of the marriage and shown to be medically or clinically permanent or incurable. It must also be grave enough to bring about the disability of the petitioner to assume the essential obligations of marriage.

1004

University of the Cordilleras College of Law First Year C S.Y. 2013 - 2014 ALAIN M. DIO , vs. MA. CARIDAD L. DIO G.R. No. 178044 January 19, 2011 Full Case ALAIN M. DIO , Petitioner, vs. MA. CARIDAD L. DIO, Respondent. CARPIO, J.: The Case Before the Court is a petition for review1 assailing the 18 October 2006 Decision2 and the 12 March 2007 Order3 of the Regional Trial Court of Las Pias City, Branch 254 (trial court) in Civil Case No. LP-010149. The Antecedent Facts Alain M. Dio (petitioner) and Ma. Caridad L. Dio (respondent) were childhood friends and sweethearts. They started living together in 1984 until they decided to separate in 1994. In 1996, petitioner and respondent decided to live together again. On 14 January 1998, they were married before Mayor Vergel Aguilar of Las Pias City. On 30 May 2001, petitioner filed an action for Declaration of Nullity of Marriage against respondent, citing psychological incapacity under Article 36 of the Family Code. Petitioner alleged that respondent failed in her marital obligation to give love and support to him, and had abandoned her responsibility to the family, choosing instead to go on shopping sprees and gallivanting with her friends that depleted the family assets. Petitioner further alleged that respondent was not faithful, and would at times become violent and hurt him. Extrajudicial service of summons was effected upon respondent who, at the time of the filing of the petition, was already living in the United States of America. Despite receipt of the summons, respondent did not file an answer to the petition within the reglementary period. Petitioner later learned that respondent filed a petition for divorce/dissolution of her marriage with petitioner, which was granted by the Superior Court of California on 25 May 2001. Petitioner also learned that on 5 October 2001, respondent married a certain Manuel V. Alcantara. On 30 April 2002, the Office of the Las Pias prosecutor found that there were no indicative facts of collusion between the parties and the case was set for trial on the merits. Dr. Nedy L. Tayag (Dr. Tayag), a clinical psychologist, submitted a psychological report establishing that respondent was suffering from Narcissistic Personality Disorder which was deeply ingrained in her system since her early formative years. Dr. Tayag found that respondents disorder was long-lasting and by nature, incurable. In its 18 October 2006 Decision, the trial court granted the petition on the ground that respondent was psychologically incapacited to comply with the essential marital obligations at the time of the celebration of the marriage. The Decision of the Trial Court The trial court ruled that based on the evidence presented, petitioner was able to establish respondents psychological incapacity. The trial court ruled that even without Dr. Tayags psychological report, the allegations in the complaint, substantiated in the witness stand, clearly made out a case of psychological incapacity against respondent. The trial court found that respondent committed acts which hurt and embarrassed petitioner and the rest of the family, and that respondent failed to observe mutual love,

1005

University of the Cordilleras College of Law First Year C S.Y. 2013 - 2014 respect and fidelity required of her under Article 68 of the Family Code. The trial court also ruled that respondent abandoned petitioner when she obtained a divorce abroad and married another man. The dispositive portion of the trial courts decision reads: WHEREFORE, in view of the foregoing, judgment is hereby rendered: 1. Declaring the marriage between plaintiff ALAIN M. DIO and defendant MA. CARIDAD L. DIO on January 14, 1998, and all its effects under the law, as NULL and VOID from the beginning; and 2. Dissolving the regime of absolute community of property. A DECREE OF ABSOLUTE NULLITY OF MARRIAGE shall only be issued upon compliance with Article[s] 50 and 51 of the Family Code. Let copies of this Decision be furnished the parties, the Office of the Solicitor General, Office of the City Prosecutor, Las Pias City and the Office of the Local Civil Registrar of Las Pias City, for their information and guidance. SO ORDERED. Petitioner filed a motion for partial reconsideration questioning the dissolution of the absolute community of property and the ruling that the decree of annulment shall only be issued upon compliance with Articles 50 and 51 of the Family Code. In its 12 March 2007 Order, the trial court partially granted the motion and modified its 18 October 2006 Decision as follows: WHEREFORE, in view of the foregoing, judgment is hereby rendered: 1) Declaring the marriage between plaintiff ALAIN M. DIO and defendant MA. CARIDAD L. DIO on January 14, 1998, and all its effects under the law, as NULL and VOID from the beginning; and 2) Dissolving the regime of absolute community of property. A DECREE OF ABSOLUTE NULLITY OF MARRIAGE shall be issued after liquidation, partition and distribution of the parties properties under Article 147 of the Family Code. Let copies of this Order be furnished the parties, the Office of the Solicitor General, the Office of the City Prosecutor of Las Pias City and the Local Civil Registrar of Las Pias City, for their information and guidance.5 Hence, the petition before this Court. The Issue The sole issue in this case is whether the trial court erred when it ordered that a decree of absolute nullity of marriage shall only be issued after liquidation, partition, and distribution of the parties properties under Article 147 of the Family Code.

The Ruling of this Court The petition has merit. Petitioner assails the ruling of the trial court ordering that a decree of absolute nullity of marriage shall only be issued after liquidation, partition, and distribution of the parties properties under Article 147 of the Family Code. Petitioner argues that Section 19(1) of the Rule on Declaration of Absolute Nullity of 1006

University of the Cordilleras College of Law First Year C S.Y. 2013 - 2014 Null Marriages and Annulment of Voidable Marriages6 (the Rule) does not apply to Article 147 of the Family Code. We agree with petitioner. The Court has ruled in Valdes v. RTC, Branch 102, Quezon City that in a void marriage, regardless of its cause, the property relations of the parties during the period of cohabitation is governed either by Article 147 or Article 148 of the Family Code.7 Article 147 of the Family Code applies to union of parties who are legally capacitated and not barred by any impediment to contract marriage, but whose marriage is nonetheless void,8 such as petitioner and respondent in the case before the Court. Article 147 of the Family Code provides: Article 147. When a man and a woman who are capacitated to marry each other, live exclusively with each other as husband and wife without the benefit of marriage or under a void marriage, their wages and salaries shall be owned by them in equal shares and the property acquired by both of them through their work or industry shall be governed by the rules on co-ownership. In the absence of proof to the contrary, properties acquired while they lived together shall be presumed to have been obtained by their joint efforts, work or industry, and shall be owned by them in equal shares. For purposes of this Article, a party who did not participate in the acquisition by the other party of any property shall be deemed to have contributed jointly in the acquisition thereof if the formers efforts consisted in the care and maintenance of the family and of the household. Neither party can encumber or dispose by acts inter vivos of his or her share in the property acquired during cohabitation and owned in common, without the consent of the other, until after the termination of their cohabitation. When only one of the parties to a void marriage is in good faith, the share of the party in bad faith in the co-ownership shall be forfeited in favor of their common children. In case of default of or waiver by any or all of the common children or their descendants, each vacant share shall belong to the respective surviving descendants. In the absence of descendants, such share shall belong to the innocent party. In all cases, the forfeiture shall take place upon termination of the cohabitation. For Article 147 of the Family Code to apply, the following elements must be present: 1. The man and the woman must be capacitated to marry each other; 2. They live exclusively with each other as husband and wife; and 3. Their union is without the benefit of marriage, or their marriage is void.9 All these elements are present in this case and there is no question that Article 147 of the Family Code applies to the property relations between petitioner and respondent. We agree with petitioner that the trial court erred in ordering that a decree of absolute nullity of marriage shall be issued only after liquidation, partition and distribution of the parties properties under Article 147 of the Family Code. The ruling has no basis because Section 19(1) of the Rule does not apply to cases governed under Articles 147 and 148 of the Family Code. Section 19(1) of the Rule provides: Sec. 19. Decision. - (1) If the court renders a decision granting the petition, it shall declare therein that the decree of absolute nullity or decree of annulment shall be issued by the court only after compliance with Articles 50 and 51 of the Family Code as implemented under the Rule on Liquidation, Partition and Distribution of Properties. The pertinent provisions of the Family Code cited in Section 19(1) of the Rule are:

1007

University of the Cordilleras College of Law First Year C S.Y. 2013 - 2014 Article 50. The effects provided for in paragraphs (2), (3), (4) and (5) of Article 43 and in Article 44 shall also apply in proper cases to marriages which are declared void ab initio or annulled by final judgment under Articles 40 and 45.10 The final judgment in such cases shall provide for the liquidation, partition and distribution of the properties of the spouses, the custody and support of the common children, and the delivery of their presumptive legitimes, unless such matters had been adjudicated in previous judicial proceedings. All creditors of the spouses as well as of the absolute community of the conjugal partnership shall be notified of the proceedings for liquidation. In the partition, the conjugal dwelling and the lot on which it is situated, shall be adjudicated in accordance with the provisions of Articles 102 and 129. Article 51. In said partition, the value of the presumptive legitimes of all common children, computed as of the date of the final judgment of the trial court, shall be delivered in cash, property or sound securities, unless the parties, by mutual agreement judicially approved, had already provided for such matters. The children of their guardian, or the trustee of their property, may ask for the enforcement of the judgment. The delivery of the presumptive legitimes herein prescribed shall in no way prejudice the ultimate successional rights of the children accruing upon the death of either or both of the parents; but the value of the properties already received under the decree of annulment or absolute nullity shall be considered as advances on their legitime. It is clear from Article 50 of the Family Code that Section 19(1) of the Rule applies only to marriages which are declared void ab initio or annulled by final judgment under Articles 40 and 45 of the Family Code. In short, Article 50 of the Family Code does not apply to marriages which are declared void ab initio under Article 36 of the Family Code, which should be declared void without waiting for the liquidation of the properties of the parties. Article 40 of the Family Code contemplates a situation where a second or bigamous marriage was contracted.1avvphil Under Article 40, "[t]he absolute nullity of a previous marriage may be invoked for purposes of remarriage on the basis solely of a final judgment declaring such previous marriage void." Thus we ruled: x x x where the absolute nullity of a previous marriage is sought to be invoked for purposes of contracting a second marriage, the sole basis acceptable in law, for said projected marriage to be free from legal infirmity, is a final judgment declaring a previous marriage void. Article 45 of the Family Code, on the other hand, refers to voidable marriages, meaning, marriages which are valid until they are set aside by final judgment of a competent court in an action for annulment. In both instances under Articles 40 and 45, the marriages are governed either by absolute community of property or conjugal partnership of gains unless the parties agree to a complete separation of property in a marriage settlement entered into before the marriage. Since the property relations of the parties is governed by absolute community of property or conjugal partnership of gains, there is a need to liquidate, partition and distribute the properties before a decree of annulment could be issued. That is not the case for annulment of marriage under Article 36 of the Family Code because the marriage is governed by the ordinary rules on co-ownership. In this case, petitioners marriage to respondent was declared void under Article 36 of the Family Code and not under Article 40 or 45. Thus, what governs the liquidation of properties owned in common by petitioner and respondent are the rules on co-ownership. In Valdes, the Court ruled that the property relations of parties in a void marriage during the period of cohabitation is governed either by Article 147 or Article 148 of the Family Code. The rules on co-ownership apply and the properties of the spouses should be liquidated in accordance with the Civil Code provisions on co-ownership. Under Article 496 of 1008

University of the Cordilleras College of Law First Year C S.Y. 2013 - 2014 the Civil Code, "[p]artition may be made by agreement between the parties or by judicial proceedings. x x x." It is not necessary to liquidate the properties of the spouses in the same proceeding for declaration of nullity of marriage. WHEREFORE, we AFFIRM the Decision of the trial court with the MODIFICATION that the decree of absolute nullity of the marriage shall be issued upon finality of the trial courts decision without waiting for the liquidation, partition, and distribution of the parties properties under Article 147 of the Family Code. SO ORDERED.

1009

University of the Cordilleras College of Law First Year C S.Y. 2013 - 2014 Case Digest ALAIN M. DIO , vs. MA. CARIDAD L. DIO G.R. No. 178044 January 19, 2011 FACTS: Alain M. Dio (petitioner) and Ma. Caridad L. Dio (respondent) were childhood friends and sweethearts. They started living together in 1984 until they decided to separate in 1994. In 1996, petitioner and respondent decided to live together again. On 14 January 1998, they were married before Mayor Vergel Aguilar of Las Pias City. On 30 May 2001, petitioner filed an action for Declaration of Nullity of Marriage against respondent, citing psychological incapacity under Article 36 of the Family Code. Petitioner alleged that respondent failed in her marital obligation to give love and support to him, and had abandoned her responsibility to the family, choosing instead to go on shopping sprees and gallivanting with her friends that depleted the family assets. Petitioner further alleged that respondent was not faithful, and would at times become violent and hurt him. Extrajudicial service of summons was effected upon respondent who, at the time of the filing of the petition, was already living in the United States of America. Despite receipt of the summons, respondent did not file an answer to the petition within the reglementary period. Petitioner later learned that respondent filed a petition for divorce/dissolution of her marriage with petitioner, which was granted by the Superior Court of California on 25 May 2001. Petitioner also learned that on 5 October 2001, respondent married a certain Manuel V.Alcantara. On 30 April 2002, the Office of the Las Pias prosecutor found that there were no indicative facts of collusion between the parties and the case was set for trial on the merits. Dr. Nedy L. Tayag (Dr. Tayag), a clinical psychologist, submitted a psychological report establishing that respondent was suffering from Narcissistic Personality Disorder which was deeply ingrained in her system since her early formative years. Dr. Tayag found that respondents disorder was long-lasting and by nature, incurable. ISSUE: Was there psychological incapacity on the part of the wife? RULING: Yes. The trial court ruled that based on the evidence presented, petitioner was able to establish respondents psychological incapacity. The trial court ruled that even without Dr. Tayagspsychological report, the allegations in the complaint, substantiated in the witness stand, clearly made out a case of psychological incapacity against respondent. The trial court found that respondent committed acts which hurt and embarrassed petitioner and the rest of the family, and that respondent failed to observe mutual love, respect and fidelity required of her under Article 68 of the Family Code. The trial court also ruled that respondent abandoned petitioner when she obtained a divorce abroad and married another man.

1010

University of the Cordilleras College of Law First Year C S.Y. 2013 - 2014 DANILO A. AURELIO vs. VIDA MA. CORAZON P. AURELIO G.R. No. 175367 June 6, 2011 Full Case DANILO A. AURELIO, Petitioner, vs. VIDA MA. CORAZON P. AURELIO, Respondent. PERALTA, J.: Before this Court is a petition for review on certiorari, under Rule 45 of the Rules of Court, seeking to set aside the October 6, 2005 Decision and October 26, 2006 Resolution, of the Court of Appeals (CA), in CA-G.R. SP No. 82238. The facts of the case are as follows: Petitioner Danilo A. Aurelio and respondent Vida Ma. Corazon Aurelio were married on March 23, 1988. They have two sons, namely: Danilo Miguel and Danilo Gabriel. On May 9, 2002, respondent filed with the Regional Trial Court (RTC) of Quezon City, Branch 94, a Petition for Declaration of Nullity of Marriage. In her petition, respondent alleged that both she and petitioner were psychologically incapacitated of performing and complying with their respective essential marital obligations. In addition, respondent alleged that such state of psychological incapacity was present prior and even during the time of the marriage ceremony. Hence, respondent prays that her marriage be declared null and void under Article 36 of the Family Code which provides: Article 36. A marriage contracted by any party who, at the time of the celebration, was psychologically incapacitated to comply with the essential marital obligations of marriage, shall likewise be void, even if such incapacity becomes manifest only after its solemnization. As succinctly summarized by the CA, contained in respondents petition are the following allegations, to wit: x x x The said petition alleged, inter alia, that both husband and wife are psychologically incapable of performing and complying with their essential marital obligations. Said psychological incapacity was existing prior and at the time of the marriage. Said psychological incapacity was manifested by lack of financial support from the husband; his lack of drive and incapacity to discern the plight of his working wife. The husband exhibited consistent jealousy and distrust towards his wife. His moods alternated between hostile defiance and contrition. He refused to assist in the maintenance of the family. He refused to foot the household bills and provide for his familys needs. He exhibited arrogance. He was completely insensitive to the feelings of his wife. He liked to humiliate and embarrass his wife even in the presence of their children. Vida Aurelio, on the other hand, is effusive and displays her feelings openly and freely. Her feelings change very quickly from joy to fury to misery to despair, depending on her day-to-day experiences. Her tolerance for boredom was very low. She was emotionally immature; she cannot stand frustration or disappointment. She cannot delay to gratify her needs. She gets upset when she cannot get what she wants. Self-indulgence lifts her spirits immensely. Their hostility towards each other distorted their relationship. Their incapacity to accept and fulfill the essential obligations of marital life led to the breakdown of their marriage. Private respondent manifested psychological aversion to cohabit with her husband or to take care of him. The psychological make-up of private respondent was evaluated by a psychologist, who found that the psychological incapacity of both husband and wife to perform their marital obligations is grave, incorrigible and incurable. Private respondent suffers from a Histrionic Personality Disorder with Narcissistic features; whereas petitioner suffers from passive aggressive (negativistic) personality disorder that renders him immature and irresponsible to assume the normal obligations of a marriage. 1011

University of the Cordilleras College of Law First Year C S.Y. 2013 - 2014 On November 8, 2002, petitioner filed a Motion to Dismiss6 the petition. Petitioner principally argued that the petition failed to state a cause of action and that it failed to meet the standards set by the Court for the interpretation and implementation of Article 36 of the Family Code. On January 14, 2003, the RTC issued an Order denying petitioners motion. On February 21, 2003, petitioner filed a Motion for Reconsideration, which was, however, denied by the RTC in an Order dated December 17, 2003. In denying petitioners motion, the RTC ruled that respondents petition for declaration of nullity of marriage complied with the requirements of the Molina doctrine, and whether or not the allegations are meritorious would depend upon the proofs presented by both parties during trial, to wit: A review of the petition shows that it observed the requirements in Republic vs. Court of Appeals (268 SCRA 198), otherwise known as the Molina Doctrine. There was allegation of the root cause of the psychological incapacity of both the petitioner and the respondent contained in paragraphs 12 and 13 of the petition. The manifestation of juridical antecedence was alleged in paragraphs 5 and 6 of the petition. The allegations constituting the gravity of psychological incapacity were alleged in paragraph 9 (a to l) of the petition. The incurability was alleged in paragraph 10 of the petition. Moreover, the clinical finding of incurability was quoted in paragraph 15 of the petition. There is a cause of action presented in the petition for the nullification of marriage under Article 36 of the Family Code. Whether or not the allegations are meritorious depends upon the proofs to be presented by both parties. This, in turn, will entail the presentation of evidence which can only be done in the hearing on the merits of the case. If the Court finds that there are (sic) preponderance of evidence to sustain a nullification, then the cause of the petition shall fail. Conversely, if it finds, through the evidence that will be presented during the hearing on the merits, that there are sufficient proofs to warrant nullification, the Court shall declare its nullity. On February 16, 2004, petitioner appealed the RTC decision to the CA via petition for certiorari under Rule 65 of the Rules of Court. On October 6, 2005, the CA rendered a Decision dismissing the petition, the dispositive portion of which reads: WHEREFORE, premises considered, [the] instant petition is DISMISSED. SO ORDERED. In a Resolution dated October 26, 2004, the CA dismissed petitioners motion for reconsideration. In its Decision, the CA affirmed the ruling of the RTC and held that respondents complaint for declaration of nullity of marriage when scrutinized in juxtaposition with Article 36 of the Family Code and the Molina doctrine revealed the existence of a sufficient cause of action. Hence, herein petition, with petitioner raising two issues for this Courts consideration, to wit: I. WHETHER OR NOT THE COURT OF APPEALS VIOLATED THE APPLICABLE LAW AND JURISPRUDENCE WHEN IT HELD THAT THE ALLEGATIONS CONTAINED IN THE PETITION FOR DECLARATION OF THE NULLITY OF MARRIAGE ARE SUFFICIENT FOR THE COURT TO DECLARE THE NULLITY OF THE MARRIAGE BETWEEN VIDA AND DANILO. II. WHETHER OR NOT THE COURT OF APPEALS VIOLATED THE APPLICABLE LAW AND JURISPRUDENCE WHEN IT DENIED PETITIONERS ACTION FOR CERTIORARI DESPITE THE FACT THAT THE DENIAL OF HIS MOTION TO DISMISS BY THE TRIAL COURT IS PATENTLY AND UTTERLY TAINTED WITH GRAVE ABUSE OF DISCRETION AMOUNTING TO LACK OR 1012

University of the Cordilleras College of Law First Year C S.Y. 2013 - 2014 EXCESS OF JURISDICTION; AND THAT APPEAL IN DUE COURSE IS NOT A PLAIN, ADEQUATE OR SPEEDY REMEDY UNDER THE CIRCUMSTANCES. Before anything else, it bears to point out that had respondents complaint been filed after March 15, 2003, this present petition would have been denied since Supreme Court Administrative Matter No. 0211-10 prohibits the filing of a motion to dismiss in actions for annulment of marriage. Be that as it may, after a circumspect review of the arguments raised by petitioner herein, this Court finds that the petition is not meritorious. In Republic v. Court of Appeals, this Court created the Molina guidelines to aid the courts in the disposition of cases involving psychological incapacity, to wit: (1) Burden of proof to show the nullity of the marriage belongs to the plaintiff. (2) The root cause of the psychological incapacity must be: (a) medically or clinically identified, (b) alleged in the complaint, (c) sufficiently proven by experts and (d) clearly explained in the decision. (3) The incapacity must be proven to be existing at "the time of the celebration" of the marriage. (4) Such incapacity must also be shown to be medically or clinically permanent or incurable. (5) Such illness must be grave enough to bring about the disability of the party to assume the essential obligations of marriage. (6) The essential marital obligations must be those embraced by Articles 68 up to 71 of the Family Code as regards the husband and wife, as well as Articles 220, 221 and 225 of the same Code in regard to parents and their children. Such non-complied marital obligation(s) must also be stated in the petition, proven by evidence and included in the text of the decision. (7) Interpretations given by the National Appellate Matrimonial Tribunal of the Catholic Church in the Philippines, while not controlling or decisive, should be given great respect by our courts. (8) The trial court must order the prosecuting attorney or fiscal and the Solicitor General to appear as counsel for the state. No decision shall be handed down unless the Solicitor General issues a certification, which will be quoted in the decision, briefly stating therein his reasons for his agreement or opposition, as the case may be, to the petition. This Court, pursuant to Supreme Court Administrative Matter No. 02-11-10, has modified the above pronouncements, particularly Section 2(d) thereof, stating that the certification of the Solicitor General required in the Molina case is dispensed with to avoid delay. Still, Article 48 of the Family Code mandates that the appearance of the prosecuting attorney or fiscal assigned be on behalf of the State to take steps to prevent collusion between the parties and to take care that evidence is not fabricated or suppressed. Petitioner anchors his petition on the premise that the allegations contained in respondents petition are insufficient to support a declaration of nullity of marriage based on psychological incapacity. Specifically, petitioner contends that the petition failed to comply with three of the Molina guidelines, namely: that the root cause of the psychological incapacity must be alleged in the complaint; that such illness must be grave enough to bring about the disability of the party to assume the essential obligations of marriage; and that the non-complied marital obligation must be stated in the petition. First, contrary to petitioners assertion, this Court finds that the root cause of psychological incapacity was stated and alleged in the complaint. We agree with the manifestation of respondent that the family backgrounds of both petitioner and respondent were discussed in the complaint as the root causes of their psychological incapacity. Moreover, a competent and expert psychologist clinically identified the same as the root causes.

1013

University of the Cordilleras College of Law First Year C S.Y. 2013 - 2014 Second, the petition likewise alleged that the illness of both parties was of such grave a nature as to bring about a disability for them to assume the essential obligations of marriage. The psychologist reported that respondent suffers from Histrionic Personality Disorder with Narcissistic Features. Petitioner, on the other hand, allegedly suffers from Passive Aggressive (Negativistic) Personality Disorder. The incapacity of both parties to perform their marital obligations was alleged to be grave, incorrigible and incurable. Lastly, this Court also finds that the essential marital obligations that were not complied with were alleged in the petition. As can be easily gleaned from the totality of the petition, respondents allegations fall under Article 68 of the Family Code which states that "the husband and the wife are obliged to live together, observe mutual love, respect and fidelity, and render mutual help and support." It bears to stress that whether or not petitioner and respondent are psychologically incapacitated to fulfill their marital obligations is a matter for the RTC to decide at the first instance. A perusal of the Molina guidelines would show that the same contemplate a situation wherein the parties have presented their evidence, witnesses have testified, and that a decision has been reached by the court after due hearing. Such process can be gleaned from guidelines 2, 6 and 8, which refer to a decision rendered by the RTC after trial on the merits. It would certainly be too burdensome to ask this Court to resolve at first instance whether the allegations contained in the petition are sufficient to substantiate a case for psychological incapacity. Let it be remembered that each case involving the application of Article 36 must be treated distinctly and judged not on the basis of a priori assumptions, predilections or generalizations but according to its own attendant facts. Courts should interpret the provision on a case-to-case basis, guided by experience, the findings of experts and researchers in psychological disciplines, and by decisions of church tribunals. It would thus be more prudent for this Court to remand the case to the RTC, as it would be in the best position to scrutinize the evidence as well as hear and weigh the evidentiary value of the testimonies of the ordinary witnesses and expert witnesses presented by the parties. Given the allegations in respondents petition for nullity of marriage, this Court rules that the RTC did not commit grave abuse of discretion in denying petitioners motion to dismiss. By grave abuse of discretion is meant capricious and whimsical exercise of judgment as is equivalent to lack of jurisdiction. Mere abuse of discretion is not enough. It must be grave abuse of discretion as when the power is exercised in an arbitrary or despotic manner by reason of passion or personal hostility, and must be so patent and so gross as to amount to an evasion of a positive duty or to a virtual refusal to perform the duty enjoined or to act at all in contemplation of law. Even assuming arguendo that this Court were to agree with petitioner that the allegations contained in respondents petition are insufficient and that the RTC erred in denying petitioners motion to dismiss, the same is merely an error of judgment correctible by appeal and not an abuse of discretion correctible by certiorari. Finally, the CA properly dismissed petitioners petition. As a general rule, the denial of a motion to dismiss, which is an interlocutory order, is not reviewable by certiorari. Petitioners remedy is to reiterate the grounds in his motion to dismiss, as defenses in his answer to the petition for nullity of marriage, proceed trial and, in case of an adverse decision, appeal the decision in due time. The existence of that adequate remedy removed the underpinnings of his petition for certiorari in the CA. WHEREFORE, premises considered the petition is DENIED. The October 6, 2005 Decision and October 26, 2006 Resolution of the Court of Appeals, in CA-G.R. SP No. 82238, are AFFIRMED. SO ORDERED.

1014

University of the Cordilleras College of Law First Year C S.Y. 2013 - 2014 Case Digest DANILO A. AURELIO vs. VIDA MA. CORAZON P. AURELIO G.R. No. 175367 June 6, 2011 FACTS: Petitioner Danilo A. Aurelio and respondent Vida Ma. Corazon Aurelio were married on March 23, 1988. They have two sons, namely: Danilo Miguel and Danilo Gabriel. On May 9, 2002, respondent filed with the Regional Trial Court (RTC) of Quezon City, Branch 94, a Petition for Declaration of Nullity of Marriage. In her petition, respondent alleged that both she and petitioner were psychologically incapacitated of performing and complying with their respective essential marital obligations. In addition, respondent alleged that such state of psychological incapacity was present prior and even during the time of the marriage ceremony. Hence, respondent prays that her marriage be declared null and void under Article 36 of the Family Code . The said petition alleged, inter alia, that both husband and wife are psychologically incapable of performing and complying with their essential marital obligations. Said psychological incapacity was existing prior and at the time of the marriage. Said psychological incapacity was manifested by lack of financial support from the husband; his lack of drive and incapacity to discern the plight of his working wife. The husband exhibited consistent jealousy and distrust towards his wife. His moods alternated between hostile defiance and contrition. He refused to assist in the maintenance of the family. He refused to foot the household bills and provide for his familys needs. He exhibited arrogance. He was completely insensitive to the feelings of his wife. He liked to humiliate and embarrass his wife even in the presence of their children. Vida Aurelio, on the other hand, is effusive and displays her feelings openly and freely. Her feelings change very quickly from joy to fury to misery to despair, depending on her day-to-day experiences. Her tolerance for boredom was very low. She was emotionally immature; she cannot stand frustration or disappointment. She cannot delay to gratify her needs. She gets upset when she cannot get what she wants. Self-indulgence lifts her spirits immensely. Their hostility towards each other distorted their relationship. Their incapacity to accept and fulfill the essential obligations of marital life led to the breakdown of their marriage. Private respondent manifested psychological aversion to cohabit with her husband or to take care of him. The psychological make-up of private respondent was evaluated by a psychologist, who found that the psychological incapacity of both husband and wife to perform their marital obligations is grave, incorrigible and incurable. Private respondent suffers from a Histrionic Personality Disorder with Narcissistic features; whereas petitioner suffers from passive aggressive (negativistic) personality disorder that renders him immature and irresponsible to assume the normal obligations of a marriage. ISSUE: Was there psychological incapacity on the part of the spouses? RULING: Yes. Contrary to petitioners assertion, this Court finds that the root cause of psychological incapacity was stated and alleged in the complaint. We agree with the manifestation of respondent that the family backgrounds of both petitioner and respondent were discussed in the complaint as the root causes of their psychological incapacity. Moreover, a competent and expert psychologist clinically identified the same as the root causes. Second, the petition likewise alleged that the illness of both parties was of such grave a nature as to bring about a disability for them to assume the essential obligations of marriage. The psychologist reported that respondent suffers from Histrionic Personality Disorder with Narcissistic Features. Petitioner, on the other hand, allegedly suffers from Passive Aggressive

1015

University of the Cordilleras College of Law First Year C S.Y. 2013 - 2014 (Negativistic) Personality Disorder. The incapacity of both parties to perform their marital obligations was alleged to be grave, incorrigible and incurable. Lastly, this Court also finds that the essential marital obligations that were not complied with were alleged in the petition. As can be easily gleaned from the totality of the petition, respondents allegations fall under Article 68 of the Family Code which states that the husband and the wife are obliged to live together, observe mutual love, respect and fidelity, and render mutual help and support.

1016

University of the Cordilleras College of Law First Year C S.Y. 2013 - 2014 Juanita Trinidad Ramos,et al. v.Danilo Pangilinanet al. G.R. No. 185920 Full Case JUANITA TRINIDAD RAMOS, ALMA RAMOS WORAK, MANUEL T. RAMOS, JOSEFINA R. ROTHMAN, SONIA R. POST, ELVIRA P. MUNAR, and OFELIA R. LIM, Petitioners, vs. DANILO PANGILINAN, RODOLFO SUMANG, LUCRECIO BAUTISTA and ROLANDO ANTENOR, Respondents. CARPIO MORALES, J.: Respondents filed in 2003 a complaint1 for illegal dismissal against E.M. Ramos Electric, Inc., a company owned by Ernesto M. Ramos (Ramos), the patriarch of herein petitioners. By Decision2 of April 15, 2005, the Labor Arbiter ruled in favor of respondents and ordered Ramos and the company to pay the aggregate amount ofP1,661,490.30 representing their backwages, separation pay, 13th month pay & service incentive leave pay. The Decision having become final and executory and no settlement having been forged by the parties, the Labor Arbiter issued on September 8, 2005 a writ of execution3 which the Deputy Sheriff of the National Labor Relations Commission (NLRC) implemented by levying a property in Ramos name covered by TCT No. 38978, situated in Pandacan, Manila (Pandacan property). Alleging that the Pandacan property was the family home, hence, exempt from execution to satisfy the judgment award, Ramos and the company moved to quash the writ of execution.4 Respondents, however, averred that the Pandacan property is not the Ramos family home, as it has another in Antipolo, and the Pandacan property in fact served as the companys business address as borne by the companys letterhead. Respondents added that, assuming that the Pandacan property was indeed the family home, only the value equivalent to P300,000 was exempt from execution. By Order5 of August 2, 2006, the Labor Arbiter denied the motion to quash, hence, Ramos and the company appealed to the NLRC which affirmed the Labor Arbiters Order. Ramos and the company appealed to the Court of Appeals during the pendency of which Ramos died and was substituted by herein petitioners. Petitioners also filed before the NLRC, as third-party claimants, a Manifestation questioning the Notice to Vacate issued by the Sheriff, alleging that assuming that the Pandacan property may be levied upon, the family home straddled two (2) lots, including the lot covered by TCT No. 38978, hence, they cannot be asked to vacate the house. The Labor Arbiter was later to deny, by Decision of May 7, 2009, the third-party claim, holding that Ramos death and petitioners substitution as his compulsory heirs would not nullify the sale at auction of the Pandacan property. And the NLRC6 would later affirm the Labor Arbiters ruling, noting that petitioners failed to exercise their right to redeem the Pandacan property within the one 1 year period or until January 16, 2009. The NLRC brushed aside petitioners contention that they should have been given a fresh period of 1 year from the time of Ramos death on July 29, 2008 or until July 30, 2009 to redeem the property, holding that to do so would give petitioners, as mere heirs, a better right than the Ramos. As to petitioners claim that the property was covered by the regime of conjugal partnership of gains and as such only Ramos share can be levied upon, the NLRC ruled that petitioners failed to substantiate such claim and that the phrase in the TCT indicating the registered owner as "Ernesto Ramos, married to Juanita Trinidad, Filipinos," did not mean that both owned the property, the phrase having merely described Ramos civil status. Before the appellate court, petitioners alleged that the NLRC erred in ruling that the market value of the property was P2,177,000 as assessed by the City Assessor of Manila and appearing in the documents submitted before the Labor Arbiter, claiming that at the time the Pandacan property was constituted as the July 20, 2010

1017

University of the Cordilleras College of Law First Year C S.Y. 2013 - 2014 family home in 1944, its value was way below P300,000; and that Art. 153 of the Family Code was applicable, hence, they no longer had to resort to judicial or extrajudicial constitution. In the assailed Decision7 of September 24, 2008, the appellate court, in denying petitioners appeal, held that the Pandacan property was not exempted from execution, for while "Article 153 8 of the Family Code provides that the family home is deemed constituted on a house and lot from the time it is occupied as a family residence, [it] did not mean that the article has a retroactive effect such that all existing family residences are deemed to have been constituted as family homes at the time of their occupation prior to the effectivity of the Family Code." The appellate court went on to hold that what was applicable law were Articles 224 to 251 of the Civil Code, hence, there was still a need to either judicially or extrajudicially constitute the Pandacan property as petitioners family home before it can be exempted; and as petitioners failed to comply therewith, there was no error in denying the motion to quash the writ of execution. The only question raised in the present petition for review on certiorari is the propriety of the Court of Appeals Decision holding that the levy upon the Pandacan property was valid. The petition is devoid of merit. Indeed, the general rule is that the family home is a real right which is gratuitous, inalienable and free from attachment, constituted over the dwelling place and the land on which it is situated, which confers upon a particular family the right to enjoy such properties, which must remain with the person constituting it and his heirs. It cannot be seized by creditors except in certain special cases.9 Kelley, Jr. v. Planters Products, Inc.10 lays down the rules relative to the levy on execution over the family home, viz: No doubt, a family home is generally exempt from execution provided it was duly constituted as such. There must be proof that the alleged family home was constituted jointly by the husband and wife or by an unmarried head of a family. It must be the house where they and their family actually reside and the lot on which it is situated. The family home must be part of the properties of the absolute community or the conjugal partnership, or of the exclusive properties of either spouse with the latters consent, or on the property of the unmarried head of the family. The actual value of the family home shall not exceed, at the time of its constitution, the amount of P300,000 in urban areas and P200,000 in rural areas. Under the Family Code, there is no need to constitute the family home judicially or extrajudicially. All family homes constructed after the effectivity of the Family Code (August 3, 1988) are constituted as such by operation of law. All existing family residences as of August 3, 1988 are considered family homes and are prospectively entitled to the benefits accorded to a family home under the Family Code. The exemption is effective from the time of the constitution of the family home as such and lasts as long as any of its beneficiaries actually resides therein. Moreover, the debts for which the family home is made answerable must have been incurred after August 3, 1988. Otherwise (that is, if it was incurred prior to August 3, 1988), the alleged family home must be shown to have been constituted either judicially or extrajudicially pursuant to the Civil Code. (emphasis supplied) For the family home to be exempt from execution, distinction must be made as to what law applies based on when it was constituted and what requirements must be complied with by the judgment debtor or his successors claiming such privilege. Hence, two sets of rules are applicable. If the family home was constructed before the effectivity of the Family Code or before August 3, 1988, then it must have been constituted either judicially or extra-judicially as provided under Articles 225, 229231 and 233 of the Civil Code.11 Judicial constitution of the family home requires the filing of a verified petition before the courts and the registration of the courts order with the Registry of Deeds of the area where the property is located. Meanwhile, extrajudicial constitution is governed by Articles 240 to 24212 of the Civil Code and involves the execution of a public instrument which must also be registered with the Registry of Property. Failure to comply with either one of these two modes of constitution will bar a judgment debtor from availing of the privilege.

1018

University of the Cordilleras College of Law First Year C S.Y. 2013 - 2014 On the other hand, for family homes constructed after the effectivity of the Family Code on August 3, 1988, there is no need to constitute extrajudicially or judicially, and the exemption is effective from the time it was constituted and lasts as long as any of its beneficiaries under Art. 15413 actually resides therein. Moreover, the family home should belong to the absolute community or conjugal partnership, or if exclusively by one spouse, its constitution must have been with consent of the other, and its value must not exceed certain amounts depending upon the area where it is located. Further, the debts incurred for which the exemption does not apply as provided under Art. 15514 for which the family home is made answerable must have been incurred after August 3, 1988.1avvphi1 And in both cases, whether under the Civil Code or the Family Code, it is not sufficient that the person claiming exemption merely alleges that such property is a family home. This claim for exemption must be set up and proved.15 In the present case, since petitioners claim that the family home was constituted prior to August 3, 1988, or as early as 1944, they must comply with the procedure mandated by the Civil Code. There being absolutely no proof that the Pandacan property was judicially or extrajudicially constituted as the Ramos family home, the laws protective mantle cannot be availed of by petitioners. Parenthetically, the records show that the sheriff exhausted all means to execute the judgment but failed because Ramos bank accounts16 were already closed while other properties in his or the companys name had already been transferred,17 and the only property left was the Pandacan property. WHEREFORE, the petition is DENIED. SO ORDERED.

1019

University of the Cordilleras College of Law First Year C S.Y. 2013 - 2014 Ben-Hur Nepomuceno v. Archbencel Ann Lopez, represented by hermother Araceli Lopez G.R. No. 181258 March 18, 2010 Full Case BEN-HUR NEPOMUCENO, Petitioner, vs. ARHBENCEL ANN LOPEZ, represented by her mother ARACELI LOPEZ, Respondent. CARPIO MORALES, J.: Respondent Arhbencel Ann Lopez (Arhbencel), represented by her mother Araceli Lopez (Araceli), filed a Complaint1 with the Regional Trial Court (RTC) of Caloocan City for recognition and support against Ben-Hur Nepomuceno (petitioner). Born on June 8, 1999, Arhbencel claimed to have been begotten out of an extramarital affair of petitioner with Araceli; that petitioner refused to affix his signature on her Certificate of Birth; and that, by a handwritten note dated August 7, 1999, petitioner nevertheless obligated himself to give her financial support in the amount ofP1,500 on the 15th and 30th days of each month beginning August 15, 1999. Arguing that her filiation to petitioner was established by the handwritten note, Arhbencel prayed that petitioner be ordered to: (1) recognize her as his child, (2) give her support pendente lite in the increased amount of P8,000 a month, and (3) give her adequate monthly financial support until she reaches the age of majority. Petitioner countered that Araceli had not proven that he was the father of Arhbencel; and that he was only forced to execute the handwritten note on account of threats coming from the National Peoples Army.2 By Order of July 4, 2001,3 Branch 130 of the Caloocan RTC, on the basis of petitioners handwritten note which it treated as "contractual support" since the issue of Arhbencels filiation had yet to be determined during the hearing on the merits, granted Arhbencels prayer for support pendente lite in the amount of P3,000 a month. After Arhbencel rested her case, petitioner filed a demurrer to evidence which the trial court granted by Order dated June 7, 2006,4 whereupon the case was dismissed for insufficiency of evidence. The trial court held that, among other things, Arhbencels Certificate of Birth was not prima fac ie evidence of her filiation to petitioner as it did not bear petitioners signature; that petitioners handwritten undertaking to provide support did not contain a categorical acknowledgment that Arhbencel is his child; and that there was no showing that petitioner performed any overt act of acknowledgment of Arhbencel as his illegitimate child after the execution of the note. On appeal by Arhbencel, the Court of Appeals, by Decision of July 20, 2007,5 reversed the trial courts decision, declared Arhbencel to be petitioners illegitimate daughter and accordingly ordered petitioner to give Arhbencel financial support in the increased amount of P4,000 every 15th and 30th days of the month, or a total of P8,000 a month. The appellate court found that from petitioners payment of Aracelis hospital bills when she gave birth to Arhbencel and his subsequent commitment to provide monthly financial support, the only logical conclusion to be drawn was that he was Arhbencels father; that petitioner merely acted in bad faith in omitting a statement of paternity in his handwritten undertaking to provide financial support; and that the amount of P8,000 a month was reasonable for Arhbencels subsistence and not burdensome for petitioner in view of his income. His Motion for Reconsideration having been denied by Resolution dated January 3, 2008, 6 petitioner comes before this Court through the present Petition for Review on Certiorari.7 Petitioner contends that nowhere in the documentary evidence presented by Araceli is an explicit statement made by him that he is the father of Arhbencel; that absent recognition or acknowledgment, illegitimate children are not entitled to support from the putative parent; that the supposed payment made 1020

University of the Cordilleras College of Law First Year C S.Y. 2013 - 2014 by him of Aracelis hospital bills was neither alleged in the complaint nor proven during the trial; and that Arhbencels claim of paternity and filiation was not established by clear and convincing evidence. Arhbencel avers in her Comment that petitioner raises questions of fact which the appellate court had already addressed, along with the issues raised in the present petition.8 The petition is impressed with merit. The relevant provisions of the Family Code9 that treat of the right to support are Articles 194 to 196, thus: Article 194. Support compromises everything indispensable for sustenance, dwelling, clothing, medical attendance, education and transportation, in keeping with the financial capacity of the family.1awph!1 The education of the person entitled to be supported referred to in the preceding paragraph shall include his schooling or training for some profession, trade or vocation, even beyond the age of majority. Transportation shall include expenses in going to and from school, or to and from place of work. Article 195. Subject to the provisions of the succeeding articles, the following are obliged to support each other to the whole extent set forth in the preceding article: 1. The spouses; 2. Legitimate ascendants and descendants; 3. Parents and their legitimate children and the legitimate and illegitimate children of the latter; 4. Parents and their illegitimate children and the legitimate and illegitimate children of the latter; and 5. Legitimate brothers and sisters, whether of the full or half-blood. Article 196. Brothers and sisters not legitimately related, whether of the full or half-blood, are likewise bound to support each other to the full extent set forth in Article 194, except only when the need for support of the brother or sister, being of age, is due to a cause imputable to the claimant's fault or negligence. (emphasis and underscoring supplied) Arhbencels demand for support, being based on her claim of filiation to petitioner as his illegitimate daughter, falls under Article 195(4). As such, her entitlement to support from petitioner is dependent on the determination of her filiation. Herrera v. Alba10 summarizes the laws, rules, and jurisprudence on establishing filiation, discoursing in relevant part as follows: Laws, Rules, and Jurisprudence Establishing Filiation The relevant provisions of the Family Code provide as follows: ART. 175. Illegitimate children may establish their illegitimate filiation in the same way and on the same evidence as legitimate children. xxxx ART. 172. The filiation of legitimate children is established by any of the following: (1) The record of birth appearing in the civil register or a final judgment; or

1021

University of the Cordilleras College of Law First Year C S.Y. 2013 - 2014 (2) An admission of legitimate filiation in a public document or a private handwritten instrument and signed by the parent concerned. In the absence of the foregoing evidence, the legitimate filiation shall be proved by: (1) The open and continuous possession of the status of a legitimate child; or (2) Any other means allowed by the Rules of Court and special laws. The Rules on Evidence include provisions on pedigree. The relevant sections of Rule 130 provide: SEC. 39. Act or declaration about pedigree. The act or declaration of a person deceased, or unable to testify, in respect to the pedigree of another person related to him by birth or marriage, may be received in evidence where it occurred before the controversy, and the relationship between the two persons is shown by evidence other than such act or declaration. The word "pedigree" includes relationship, family genealogy, birth, marriage, death, the dates when and the places where these facts occurred, and the names of the relatives. It embraces also facts of family history intimately connected with pedigree. SEC. 40. Family reputation or tradition regarding pedigree. The reputation or tradition existing in a family previous to the controversy, in respect to the pedigree of any one of its members, may be received in evidence if the witness testifying thereon be also a member of the family, either by consanguinity or affinity. Entries in family bibles or other family books or charts, engraving on rings, family portraits and the like, may be received as evidence of pedigree. This Court's rulings further specify what incriminating acts are acceptable as evidence to establish filiation. In Pe Lim v. CA, a case petitioner often cites, we stated that the issue of paternity still has to be resolved by suchconventional evidence as the relevant incriminating verbal and written acts by the putative father. Under Article 278 of the New Civil Code, voluntary recognition by a parent shall be made in the record of birth, a will, a statement before a court of record, or in any authentic writing. To be effective, the claim of filiation must be made by the putative father himself and the writing must be the writing of the putative father. A notarial agreement to support a child whose filiation is admitted by the putative father was considered acceptable evidence. Letters to the mother vowing to be a good father to the child and pictures of the putative father cuddling the child on various occasions, together with the certificate of live birth, proved filiation. However, a student permanent record, a written consent to a father's operation, or a marriage contract where the putative father gave consent, cannot be taken as authentic writing. Standing alone, neither a certificate of baptism nor family pictures are sufficient to establish filiation. (emphasis and underscoring supplied) In the present case, Arhbencel relies, in the main, on the handwritten note executed by petitioner which reads: Manila, Aug. 7, 1999 I, Ben-Hur C. Nepomuceno, hereby undertake to give and provide financial support in the amount of P1,500.00 every fifteen and thirtieth day of each month for a total of P3,000.00 a month starting Aug. 15, 1999, to Ahrbencel Ann Lopez, presently in the custody of her mother Araceli Lopez without the necessity of demand, subject to adjustment later depending on the needs of the child and my income. The abovequoted note does not contain any statement whatsoever about Arhbencels filiation to petitioner. It is, therefore, not within the ambit of Article 172(2) vis--vis Article 175 of the Family Code which admits as competent evidence of illegitimate filiation an admission of filiation in a private handwritten instrument signed by the parent concerned. The note cannot also be accorded the same weight as the notarial agreement to support the child referred to in Herrera. For it is not even notarized. And Herrera instructs that the notarial agreement must be accompanied by the putative fathers admission of filiation to be an acceptable evidence of filiation. Here, however, not only has petitioner not admitted filiation through contemporaneous actions. He has consistently denied it.

1022

University of the Cordilleras College of Law First Year C S.Y. 2013 - 2014 The only other documentary evidence submitted by Arhbencel, a copy of her Certificate of Birth, 11 has no probative value to establish filiation to petitioner, the latter not having signed the same. At bottom, all that Arhbencel really has is petitioners handwritten undertaking to provide financial support to her which, without more, fails to establish her claim of filiation. The Court is mindful that the best interests of the child in cases involving paternity and filiation should be advanced. It is, however, just as mindful of the disturbance that unfounded paternity suits cause to the privacy and peace of the putative fathers legitimate family. WHEREFORE, the petition is GRANTED. The Court of Appeals Decision of July 20, 2007 is SET ASIDE. The Order dated June 7, 2006 of Branch 130 of the Caloocan City RTC dismissing the complaint for insufficiency of evidence is REINSTATED. SO ORDERED

1023

University of the Cordilleras College of Law First Year C S.Y. 2013 - 2014 Ma. Virginia V. Remo vs. The Honorable Secretary of Foreign Affairs G.R. No. 169202 March 5, 2010 Full Case MARIA VIRGINIA V. REMO, Petitioner, vs. THE HONORABLE SECRETARY OF FOREIGN AFFAIRS, Respondent. CARPIO, J.: The Case Before the Court is a petition for review1 of the 27 May 2005 Decision2 and 2 August 2005 Resolution3 of the Court of Appeals in CA-G.R. SP No. 87710. The Court of Appeals affirmed the decision of the Office of the President, which in turn affirmed the decision of the Secretary of Foreign Affairs denying petitioners request to revert to the use of her maiden name in her replacement passport. The Facts Petitioner Maria Virginia V. Remo is a married Filipino citizen whose Philippine passport was then expiring on 27 October 2000. Petitioner being married to Francisco R. Rallonza, the following entries appear in her passport: "Rallonza" as her surname, "Maria Virginia" as her given name, and "Remo" as her middle name. Prior to the expiry of the validity of her passport, petitioner, whose marriage still subsists, applied for the renewal of her passport with the Department of Foreign Affairs (DFA) office in Chicago, Illinois, U.S.A., with a request to revert to her maiden name and surname in the replacement passport. Petitioners request having been denied, Atty. Manuel Joseph R. Bretana III, representing petitioner, wrote then Secretary of Foreign Affairs Domingo Siason expressing a similar request. On 28 August 2000, the DFA, through Assistant Secretary Belen F. Anota, denied the request, stating thus: This has reference to your letter dated 17 August 2000 regarding one Ms. Maria Virginia V. Remo who is applying for renewal of her passport using her maiden name. This Office is cognizant of the provision in the law that it is not obligatory for a married woman to use her husbands name. Use of maiden name is allowed in passport application only if the married name has not been used in previous application. The Implementing Rules and Regulations for Philippine Passport Act of 1996 clearly defines the conditions when a woman applicant may revert to her maiden name, that is, only in cases of annulment of marriage, divorce and death of the husband. Ms. Remos case does not meet any of these conditions.4(Emphasis supplied) Petitioners motion for reconsideration of the above-letter resolution was denied in a letter dated 13 October 2000.5 On 15 November 2000, petitioner filed an appeal with the Office of the President. On 27 July 2004, the Office of the President dismissed the appeal6 and ruled that Section 5(d) of Republic Act No. 8239 (RA 8239) or the Philippine Passport Act of 1996 "offers no leeway for any other interpretation than that only in case of divorce, annulment, or declaration [of nullity] of marriage may a married woman revert to her maiden name for passport purposes." The Office of the President further held that in case of conflict between a general and special law, the latter will control the former regardless of the respective dates of passage. Since the Civil Code is a general law, it should yield to RA 8239. On 28 October 2004, the Office of the President denied the motion for reconsideration.7 Petitioner filed with the Court of Appeals a petition for review under Rule 43 of the Rules of Civil Procedure. 1024

University of the Cordilleras College of Law First Year C S.Y. 2013 - 2014 In its Decision of 27 May 2005, the Court of Appeals denied the petition and affirmed the ruling of the Office of the President. The dispositive portion of the Court of Appeals decision reads: WHEREFORE, premises considered, the petition is DENIED, and the resolution dated July 27, 2004, and the order dated October 28, 2004 of the Office of the President in O.P. Case No. 001-A-9344 are hereby AFFIRMED. SO ORDERED.8 Petitioner moved for reconsideration which the Court of Appeals denied in its Resolution dated 2 August 2005. Hence, this petition. The Court of Appeals Ruling The Court of Appeals found no conflict between Article 370 of the Civil Code 9 and Section 5(d) of RA 8239.10 The Court of Appeals held that for passport application and issuance purposes, RA 8239 limits the instances when a married woman applicant may exercise the option to revert to the use of her maiden name such as in a case of a divorce decree, annulment or declaration of nullity of marriage. Since there was no showing that petitioner's marriage to Francisco Rallonza has been annulled, declared void or a divorce decree has been granted to them, petitioner cannot simply revert to her maiden name in the replacement passport after she had adopted her husbands surname in her old passport. Hence, according to the Court of Appeals, respondent was justified in refusing the request of petitioner to revert to her maiden name in the replacement passport.1avvphi1 The Issue The sole issue in this case is whether petitioner, who originally used her husbands surname in her expired passport, can revert to the use of her maiden name in the replacement passport, despite the subsistence of her marriage. The Ruling of the Court The petition lacks merit. Title XIII of the Civil Code governs the use of surnames. In the case of a married woman, Article 370 of the Civil Code provides: ART. 370. A married woman may use: (1) Her maiden first name and surname and add her husbands surname, or (2) Her maiden first name and her husband's surname, or (3) Her husbands full name, but prefixing a word indicating that she is his wife, such as "Mrs." We agree with petitioner that the use of the word "may" in the above provision indicates that the use of the husbands surname by the wife is permissive rather than obligatory. This has been settled in the case of Yasin v. Honorable Judge Sharia District Court.11 In Yasin,12 petitioner therein filed with the Sharia District Court a "Petition to resume the use of maiden name" in view of the dissolution of her marriage by divorce under the Code of Muslim Personal Laws of the Philippines, and after marriage of her former husband to another woman. In ruling in favor of petitioner therein, the Court explained that: When a woman marries a man, she need not apply and/or seek judicial authority to use her husbands name by prefixing the word "Mrs." before her husbands full name or by adding her husbands surname to her maiden first name. The law grants her such right (Art. 370, Civil Code). Similarly, when the marriage ties or vinculum no longer exists as in the case of death of the husband or divorce as authorized by the 1025

University of the Cordilleras College of Law First Year C S.Y. 2013 - 2014 Muslim Code, the widow or divorcee need not seek judicial confirmation of the change in her civil status in order to revert to her maiden name as use of her former husbands is optional and not obligatory for her (Tolentino, Civil Code, p. 725, 1983 ed.; Art. 373, Civil Code). When petitioner married her husband, she did not change her but only her civil status. Neither was she required to secure judicial authority to use the surname of her husband after the marriage as no law requires it. (Emphasis supplied) Clearly, a married woman has an option, but not a duty, to use the surname of the husband in any of the ways provided by Article 370 of the Civil Code.13 She is therefore allowed to use not only any of the three names provided in Article 370, but also her maiden name upon marriage. She is not prohibited from continuously using her maiden name once she is married because when a woman marries, she does not change her name but only her civil status. Further, this interpretation is in consonance with the principle that surnames indicate descent.14 In the present case, petitioner, whose marriage is still subsisting and who opted to use her husbands surname in her old passport, requested to resume her maiden name in the replacement passport arguing that no law prohibits her from using her maiden name. Petitioner cites Yasin as the applicable precedent. However, Yasin is not squarely in point with this case. Unlike in Yasin, which involved a Muslim divorcee whose former husband is already married to another woman, petitioners marriage remains subsisting. Another point, Yasin did not involve a request to resume ones maiden name in a replacement passport, but a petition to resume ones maiden name in view of the dissolution of ones marriage. The law governing passport issuance is RA 8239 and the applicable provision in this case is Section 5(d), which states: Sec. 5. Requirements for the Issuance of Passport. No passport shall be issued to an applicant unless the Secretary or his duly authorized representative is satisfied that the applicant is a Filipino citizen who has complied with the following requirements: x x x (d) In case of a woman who is married, separated, divorced or widowed or whose marriage has been annulled or declared by court as void, a copy of the certificate of marriage, court decree of separation, divorce or annulment or certificate of death of the deceased spouse duly issued and authenticated by the Office of the Civil Registrar General: Provided, That in case of a divorce decree, annulment or declaration of marriage as void, the woman applicant may revert to the use of her maiden name: Provided, further, That such divorce is recognized under existing laws of the Philippines; x x x (Emphasis supplied) The Office of the Solicitor General (OSG), on behalf of the Secretary of Foreign Affairs, argues that the highlighted proviso in Section 5(d) of RA 8239 "limits the instances when a married woman may be allowed to revert to the use of her maiden name in her passport." These instances are death of husband, divorce decree, annulment or nullity of marriage. Significantly, Section 1, Article 12 of the Implementing Rules and Regulations of RA 8239 provides: The passport can be amended only in the following cases: a) Amendment of womans name due to marriage; b) Amendment of womans name due to death of spouse, annulment of marriage or divorce initiated by a foreign spouse; or c) Change of surname of a child who is legitimated by virtue of a subsequent marriage of his parents. Since petitioners marriage to her husband subsists, placing her case outside of the purview of Section 5(d) of RA 8239 (as to the instances when a married woman may revert to the use of her maiden name), she may not resume her maiden name in the replacement passport.15 This prohibition, according to petitioner, conflicts with and, thus, operates as an implied repeal of Article 370 of the Civil Code. Petitioner is mistaken. The conflict between Article 370 of the Civil Code and Section 5(d) of RA 8239 is more imagined than real. RA 8239, including its implementing rules and regulations, does not prohibit a married woman from using her maiden name in her passport. In fact, in recognition of this right, the DFA allows a married woman who applies for a passport for the first time to use her maiden name. Such an applicant is not required to adopt her husband's surname.16 1026

University of the Cordilleras College of Law First Year C S.Y. 2013 - 2014 In the case of renewal of passport, a married woman may either ad opt her husbands surname or continuously use her maiden name. If she chooses to adopt her husbands surname in her new passport, the DFA additionally requires the submission of an authenticated copy of the marriage certificate. Otherwise, if she prefers to continue using her maiden name, she may still do so. The DFA will not prohibit her from continuously using her maiden name.17 However, once a married woman opted to adopt her husbands surname in her passport, she may not revert to the use of her maiden name, except in the cases enumerated in Section 5(d) of RA 8239. These instances are: (1) death of husband, (2) divorce, (3) annulment, or (4) nullity of marriage. Since petitioners marriage to her husband subsists, she may not resume her maiden name in the replacement passport. Otherwise stated, a married woman's reversion to the use of her maiden name must be based only on the severance of the marriage. Even assuming RA 8239 conflicts with the Civil Code, the provisions of RA 8239 which is a special law specifically dealing with passport issuance must prevail over the provisions of Title XIII of the Civil Code which is the general law on the use of surnames. A basic tenet in statutory construction is that a special law prevails over a general law,18 thus: [I]t is a familiar rule of statutory construction that to the extent of any necessary repugnancy between a general and a special law or provision, the latter will control the former without regard to the respective dates of passage.19 Moreover, petitioners theory of implied repeal must fail. Well -entrenched is the rule that an implied repeal is disfavored. T he apparently conflicting provisions of a law or two laws should be harmonized as much as possible, so that each shall be effective.20 For a law to operate to repeal another law, the two laws must actually be inconsistent. The former must be so repugnant as to be irreconcilable with the latter act.21 This petitioner failed to establish.1avvphi1 The Court notes that petitioner would not have encountered any problems in the replacement passport had she opted to continuously and consistently use her maiden name from the moment she was married and from the time she first applied for a Philippine passport. However, petitioner consciously chose to use her husbands surname before, in her previous passport application, and now desires to resume her maiden name. If we allow petitioners present request, definitely nothing prevents her in the future from requesting to revert to the use of her husbands surname. Such unjustified changes in one's name and identity in a passport, which is considered superior to all other official documents, 22 cannot be countenanced. Otherwise, undue confusion and inconsistency in the records of passport holders will arise. Thus, for passport issuance purposes, a married woman, such as petitioner, whose marriage subsists, may not change her family name at will. The acquisition of a Philippine passport is a privilege. The law recognizes the passport applicants constitutional right to travel. However, the State is also mandated to protect and maintain the integrity and credibility of the passport and travel documents proceeding from it23 as a Philippine passport remains at all times the property of the Government. The holder is merely a possessor of the passport as long as it is valid and the same may not be surrendered to any person or entity other than the government or its representative.24 As the OSG correctly pointed out: [T]he issuance of passports is impressed with public interest. A passport is an official document of identity and nationality issued to a person intending to travel or sojourn in foreign countries. It is issued by the Philippine government to its citizens requesting other governments to allow its holder to pass safely and freely, and in case of need, to give him/her aid and protection. x x x Viewed in the light of the foregoing, it is within respondents competence to regulate any amendments intended to be made therein, including the denial of unreasonable and whimsical requests for amendments such as in the instant case.25 WHEREFORE, we DENY the petition. We AFFIRM the 27 May 2005 Decision and 2 August 2005 Resolution of the Court of Appeals in CA-G.R. SP No. 87710. SO ORDERED 1027

University of the Cordilleras College of Law First Year C S.Y. 2013 - 2014 Insurance of the Philippine Islands Corp vs Spouses Gregorio G.R. No. 174104 February 14, 2011
Full Case

INSURANCE OF THE PHILIPPINE ISLANDS CORPORATION, Petitioner, vs. SPOUSES VIDAL S. GREGORIO and JULITA GREGORIO, Respondents. PERALTA, J.: Before the Court is a petition for review on certiorari under Rule 45 of the Rules of Court seeking the reversal and nullification of the Decision1 of the Court of Appeals (CA), dated June 14, 2006 and its Resolution2 dated August 10, 2006 in CA-G.R. CV No. 82303. The assailed CA Decision reversed the Decision3 of the Regional Trial Court (RTC) of Morong, Rizal, Branch 79, in Civil Case No. 748-M in favor of herein petitioner, while the questioned CA Resolution denied petitioner's motion for reconsideration. The pertinent antecedent facts of the case, as summarized by the CA, are as follows: On January 10, 1968, the spouses Vidal Gregorio and Julita Gregorio [herein respondents] obtained a loan from the Insurance of the Philippine Islands Corporation [herein petitioner] (formerly known as Pyramid Insurance Co., Inc.) in the sum of P2,200.00, payable on or before January 10, 1969, with interest thereon at the rate of 12% per annum. By way of security for the said loan, [respondents] executed a Real Estate Mortgage in favor of [petitioner] over a parcel of land known as Lot 6186 of the Morong Cadastre, then covered by Tax Declaration No. 7899 issued by the Municipal Assessor's Office of Morong, Rizal. On February 14, 1968, [respondents] again obtained another loan from [petitioner] in the sum of P2,000.00, payable on or before February 14, 1969, with 12% interest per annum. Another Real Estate Mortgage, covering a parcel of land known as Lot No. 6190 of the Morong Cadastre under Tax Declaration No. 10518, was executed by [respondents] in favor of [petitioner]. On April 10, 1968, [respondents] obtained, for the third time, another loan from [petitioner] in the amount ofP4,500.00 payable on or before April 10, 1969 with 12% interest per annum. As a security for the loan, [respondents] again executed a Real Estate Mortgage, this time covering two parcels of land: Lot 3499 under Tax Declaration No. 10631-Rizal and a lot situated in Brgy. Kay Kuliat under Tax Declaration No. 3918. [Respondents] failed to pay their loans, as a result of which the [mortgaged] properties were extrajudicially foreclosed. The extrajudicial foreclosure sale was conducted on December 11, 1969 where [petitioner] was the highest bidder. Since [respondents] failed to redeem the property, [petitioner] consolidated its ownership over the properties. The corresponding Tax Declarations were thereafter issued in the name of [petitioner].4 On February 20, 1996, petitioner filed a Complaint5 for damages against respondents alleging that in 1995, when it was in the process of gathering documents for the purpose of filing an application for the registration and confirmation of its title over the foreclosed properties, it discovered that the said lots were already registered in the names of third persons and transfer certificates of title (TCT) were issued to them. Claiming that respondents acted in a fraudulent and malevolent manner in enticing it to grant their loan applications by misrepresenting ownership of the subject properties, petitioner prayed for the grant of actual and exemplary damages as well as attorney's fees and litigation expenses. In their Amended Answer,6 respondents contended that their obligations in favor of petitioner were all settled by the foreclosure of the properties given as security therefor. In the alternative, respondents argue that petitioner's cause of action and right of action are already barred by prescription and laches.1avvphi1 In its Decision dated February 23, 2004, the RTC of Morong, Rizal, ruled in favor of petitioner, the dispositive portion of which reads: 1028

University of the Cordilleras College of Law First Year C S.Y. 2013 - 2014 WHEREFORE, premises considered, judgment is hereby rendered in favor of the plaintiff and as against the defendants, directing the latter to pay the plaintiff, jointly and severally, as follows: a. Actual damages in the amount of P1,000,000.00, representing the fair market value of the real properties subject matter of this suit; b. For defendants' deceit and bad faith, exemplary damage in the sum of P300,000.00; c. Attorney's fees and litigation expenses in the amount of P200,000.00; and d. Costs of suit. SO ORDERED.7 Aggrieved, respondents appealed the judgment of the trial court to the CA. On June 14, 2006, the CA rendered a Decision reversing and setting aside the decision of the RTC and dismissing the complaint of petitioner. It ruled that petitioner's action for damages is barred by prescription and laches. Petitioner filed a Motion for Reconsideration but the CA denied it in its Resolution of August 10, 2006. Hence, the instant petition. Petitioner's main contention is that the CA erred in ruling that petitioner's right to any relief under the law has already prescribed or is barred by laches. Petitioner argues that the prescriptive period of its action for damages should be counted from 1995, which it alleges to be the time that it discovered the fraud committed by respondents against it. On the other hand, the CA ruled that petitioner's right of action prescribed four years after the subject properties were registered with the Register of Deeds of Morong, Rizal and TCTs were subsequently issued in the names of third persons in the years 1970, 1973 and 1989. The Court finds the petition meritorious. Petitioner filed an action for damages on the ground of fraud committed against it by respondents. Under the provisions of Article 1146 of the Civil Code, actions upon an injury to the rights of the plaintiff or upon a quasi-delict must be instituted within four years from the time the cause of action accrued.8 The Court finds no error in the ruling of the CA that petitioner's cause of action accrued at the time it discovered the alleged fraud committed by respondents. It is at this point that the four-year prescriptive period should be counted. However, the Court does not agree with the CA in its ruling that the discovery of the fraud should be reckoned from the time of registration of the titles covering the subject properties. The Court notes that what has been given by respondents to petitioner as evidence of their ownership of the subject properties at the time that they mortgaged the same are not certificates of title but tax declarations, in the guise that the said properties are unregistered. On the basis of the tax declarations alone and by reason of respondent's misrepresentations, petitioner could not have been reasonably expected to acquire knowledge of the fact that the said properties were already titled. As a consequence, petitioner may not be charged with any knowledge of any subsequent entry of an encumbrance which may have been annotated on the said titles, much less any change of ownership of the properties covered thereby. As such, the Court agrees with petitioner that the reckoning period for prescription of petitioner's action should be from the time of actual discovery of the fraud in 1995. Hence, petitioner's suit for damages, filed on February 20, 1996, is well within the four-year prescriptive period. Neither may the principle of laches apply in the present case. The essence of laches or "stale demands" is the failure or neglect for an unreasonable and unexplained length of time to do that which, by exercising due diligence, could or should have been done earlier, thus, giving rise to a presumption that the party entitled to assert it either has abandoned or declined to assert 1029

University of the Cordilleras College of Law First Year C S.Y. 2013 - 2014 it.9 It is not concerned with mere lapse of time; the fact of delay, standing alone, being insufficient to constitute laches.10 In addition, it is a rule of equity and applied not to penalize neglect or sleeping on one's rights, but rather to avoid recognizing a right when to do so would result in a clearly unfair situation.11 There is no absolute rule as to what constitutes laches or staleness of demand; each case is to be determined according to its particular circumstances.12 Ultimately, the question of laches is addressed to the sound discretion of the court and, being an equitable doctrine, its application is controlled by equitable considerations.13 It cannot be used to defeat justice or perpetrate fraud and injustice.14 It is the better rule that courts, under the principle of equity, will not be guided or bound strictly by the statute of limitations or the doctrine of laches when to be so, a manifest wrong or injustice would result.15 It is significant to point out at this juncture that the overriding consideration in the instant case is that petitioner was deprived of the subject properties which it should have rightly owned were it not for the fraud committed by respondents. Hence, it would be the height of injustice if respondents would be allowed to go scot-free simply because petitioner relied in good faith on the former's false representations. Besides, as earlier discussed, even in the exercise of due diligence, petitioner could not have been expected to immediately discover respondents' fraudulent scheme. WHEREFORE, the instant petition is GRANTED. The Decision and Resolution, dated June 14, 2006 and August 10, 2006, respectively, of the Court of Appeals in CA-G.R. CV No. 82303, are REVERSED and SET ASIDE. The Decision of the Regional Trial Court of Morong, Rizal, Branch 79, dated February 23, 2004 in Civil Case No. 748-M, is REINSTATED. SO ORDERED

1030

Você também pode gostar